You are on page 1of 1137

2017-18 100 &

op kers
Class 11 T
By E ran culty
-JE Fa r
IIT enior emie .
S fP r es
o titut
Ins

PHYSICS
FOR JEE MAIN & ADVANCED
SECOND
EDITION

Exhaustive Theory
(Now Revised)

Formula Sheet
9000+ Problems
based on latest JEE pattern

2500 + 1000 (New) Problems


of previous 35 years of
AIEEE (JEE Main) and IIT-JEE (JEE Adv)

5000+Illustrations and Solved Examples


Detailed Solutions
of all problems available

Topic Covered Plancess Concepts


Tips & Tricks, Facts, Notes, Misconceptions,
Units, Dimensions Key Take Aways, Problem Solving Tactics
and Errors
PlancEssential
Questions recommended for revision
1. UNITS, DIMENSIONS
AND ERRORS

1. INTRODUCTION
Physics is an experimental science and experiments require measurement of physical quantities. Measuring a
physical quantity involves comparing the quantity with a reference standard called the unit of the quantity. Some
physical quantities are taken as base quantities and other quantities are expressed in terms of the base quantities
called derived quantities. This forms a system of base quantities and their units. Without performing proper
measurements we cannot describe the physical phenomena quantitatively.

2. UNITS
To measure a physical quantity we need some standard unit of that quantity. The measurement of the quantity
is mentioned in two parts, the first part gives magnitude and the second part gives the name of the unit. Thus,
suppose I say that length of this wire is 5 meters. The numeric part 5 says that it is 5 times of the unit of length and
the second part meter says that unit chosen here is meter.

2.1 Fundamental and Derived Quantities


The basic physical quantities, which are independent of other quantities, are known as the fundamental quantities.
For example, mass, length and time are considered to be the fundamental quantities. In the same manner, the units
which can be derived from the fundamental units are known as derived units. In mechanics, virtually all quantities
can be expressed in terms of mass, length and time. The main systems of units are given as follows:
(a) CGS or Centimetre, Gram, Second System
(b) FPS or Foot, Pound, Second System
(c) MKS or Metre, Kilogram, Second System
(d) SI system: Totally, there are seven basic or fundamental quantities in the international system of units called
the SI system which can express all physical quantities including heat, optics and electricity and magnetism.
We now provide these basic seven quantities with their units and symbols:

S. No. Physical Quantity SI Unit Symbol


1 Mass Kilogram kg
2 Length Metre m
3 Time Second s
4 Temperature Kelvin K
5 Luminous intensity Candela cd
1 . 2 | Units, Dimensions and Errors

S. No. Physical Quantity SI Unit Symbol


6 Electric current Ampere A
7 Amount of substance Mole mol.

There are also two supplementary units used as radian (rad) for plane angle and steradian (sr) for solid angle.
The above mentioned International System of Units (SI) is now extensively used in scientific measurements.
However, the following practical units of length are also conveniently used and are expressed in terms of SI system
of units.
10−6 m
(a) Micron is a small unit for measurement of length. 1 micron =1 µm =
(b) Angstrom is a unit of length in which the size of an atom is measured and is used in atomic physics.
1 Angstrom=1Å=10–10 m.
(c) Light year is a unit of distance travelled by light in 1 year free space and is used in astrophysics.
1 Light year = 3 × 108 m / s × 365 × 24 × 60 × 60
= 9.5 × 1015 meters
(d) Fermi is a unit of distance in which the size of a nucleus is measured. 1 Fermi = 10−15 m
(e) Atomic mass unit: It is a unit of mass equal to 1/12th of mass of carbon-12 atom.
1 atomic mass unit ≅ 1.67×10-27 kg
Note: There are only seven fundamental units. Apart from these, there are two supplementary units—plane angle
(radian) and solid angle (steradian). By using these units, all other units can be derived. However, we need to know
the fact that both radian and steradian have no dimensions.

3. DIMENSIONS
All the physical quantities of interest can be derived from the base quantities. Thus, when a quantity is expressed
in terms of the base quantities, it is written as a product of different powers of the base quantities. Further, the
exponent of a base quantity that enters into the expression is called the dimension of the quantity in that base. To
make it clear, consider the physical quantity ”force.” As we shall learn later, force is equal to mass times acceleration.
We know that acceleration is change in velocity divided by time interval but velocity is length divided by time
interval. Thus,
Velocity Length / Time
=Mass × Length × ( Time )
−2
Force =
Mass × Acceleration =
Mass × Mass ×
=
Time Time
Thus, the dimensions of force are 1 in mass, 1 in length and –2 in time. The dimensions in all other base quantities
are zero. Note, however, that in this type of calculation, the magnitudes are not considered. This is because only
equality of the type of quantity is what that matters. Thus, change in velocity, average velocity, or final velocity all
are equivalent in this discussion, as each one is expressed in terms of length/time.

1
Illustration 1: Validate the relation s = ut + at2 , where u is the initial velocity, a is the acceleration, t is the time
and s is the displacement.  2 (JEE MAIN)

Sol: The above relation is having units of displacement. To validate above relation dimensionally correct, we need
to match the dimensions of each quantity to the right of equality with the dimensions of displacement.
By writing the dimensions of either side of the equation, we obtain

LHS = s = displacement = M0LT0  ; RHS = ut = velocity= × time M0LT=


−1 
 T  M0LT0 
      
1 2
(acceleration) × ( time )
2 2
Further, = at = M0LT −2  =T  M0LT0 
2     
As LHS = RHS, the formula is dimensionally correct.
P hysics | 1.3

Table 1.1: SI units and dimensions of commonly used quantities

S. No. Quantity SI Units Dimensional Formula

1. Area m2 L2 
 

2. Density kg m–3 ML−3 


 

3. Velocity ms–1 LT −1 


 

4. Acceleration ms–2 LT −2 


 

5. Angular velocity rad s–1  T −1 


 

6. Frequency s–1 or hertz(Hz)  T −1 


 

7. Momentum kg ms–1 MLT −1 


 

8. Force kg ms–2 or newton (N) MLT −2 


 

9. Work, energy kg m2 s–2 or joule(J) ML2 T −2 


 

10. Power kg m2 s–3 or Js–1 orWatt ML2 T −3 


 

11. Pressure, stress Nm–2 or pascal (Pa) ML−1 T −2 


 

Coefficient of elasticity
12. Nm–2 ML−1 T −2 
 

13. Moment of inertia kg m2 ML2 


 

14. Torque Nm ML2 T −2 


 

15. Angular momentum kg m2 s–1 ML2 T −1 


 

16. Impulse Ns MLT −1 


 

17. Universal gravitational constant Nm2 kg–2 M−1L3 T −2 


 

18. Latent heat Jkg–1 L2 T −2 


 

19. Specific heat Jkg–1 K–1 L2 T −2K −1 


 
1 . 4 | Units, Dimensions and Errors

S. No. Quantity SI Units Dimensional Formula

20. Thermal conductivity Jm–1 s–1 K–1 MLT −3K −1 


 

21. Electric charge Coulomb(C)  AT 

22. Electric potential JC–1 or volt (V) ML2 T −3 A −1 


 

23. Electric resistance ML2 T −3 A −2 


VA–1 or ohm Ω ( )  

24. Electric resistivity ML3 T −3 A −2 


(Ω) m  

25. Capacitance CV–1 or farad (F) ML−1 T −2 T 4 A2 


 

26. Inductance VsA–1 or henry (H) ML2 T −2 A −2 


 

27. Electric field NC–1 or Vm–1 ML2 T −3 A −1 


 

28. Magnetic induction NA–1 m–1 or tesla(T) MT −2 A −1 


 

29. Magnetic flux Tm2 or weber (Wb) ML2 T −2 A −1 


 

30. Permittivity C2 N–1 m–2 M−1L−3 T 4 A2 


 

31. Permeability Tm A–1 or Wb A–1 m–1 MLT −2 A −2 


 

32. Plank’s constant Js ML2 T −1 


 

33. Boltzmann constant JK–1 ML2 T −2K −1 


 

Table 1.2: Table of physical quantity having same dimensional formula

S. No. Dimensional Formula Physical Quantities


1. M0 L0 T −1  (a) Frequency
 
(b) Angular frequency

(c) Angular velocity

(d) Velocity gradient


2. M0 L2 T −2  (a) Square of velocity
 
(b) Gravitational potential

(c) Latent heat


P hysics | 1.5

S. No. Dimensional Formula Physical Quantities


3. ML2 T −2  (a) Work
 
(b) Energy

(c) Torque

(d) Heat
4. ML T −2  (a) Force
 
(b) Weight

(c) Thrust

(d) Energy gradient


5. ML −1 T −2  (a) Pressure
 
(b) Stress

(c) Modulii of elasticity

(d) Energy density

4. USES OF DIMENSIONS
The major uses of dimensions are listed hereunder:
(a) Conversion from one system of units to another.
(b) To test and validate the correctness of a physical equation or formula.
(c) To derive a relationship between different physical quantities in any physical phenomenon.
(d) Conversion from one system of units to another: If we consider n1 as numerical value of a physical quantity
with dimensions a, b and c for units of mass, length and time as M1, L1, and T1, then the numerical value of the
same quantity, n2 can be calculated for different units of mass, length and time as M2, L2 and T2, respectively.
a b c
M  L   T 
n2 = n1  1   1   1 
 M2   L2   T2 
(e) To test and validate the correctness of a physical equation or formula: The principle of homogeneity
requires that the dimensions of all the terms on both sides of physical equation or formula should be equal if
the physical equation of any derived formula is correct.
(f) To derive a relationship between different physical quantities in any physical phenomenon: Suppose
that if a physical quantity depends upon a number of parameters whose dimensions are not known, then the
principle of homogeneity of dimensions can be used. As we know that the dimensions of a correct dimensional
equation are equal on both sides, it can be used to find the unknown dimensions of these parameters on
which the physical quantity depends. Further, it can be used to derive the relationships between any physical
quantity and its dependent parameters.
a
Derivation: Sn = u + (2n− 1)
2
 1  1
S= Sn + Sn−1 =  un+ an2  − (n(n− 1) + a(n− 1)2 )
n
 2  2
1  a 
Sn =
u(1) + a(1)(2n−S1)
n =u + (2n− 1) (1) (We ignore ‘1’ in formula but it carries dimension of time.)
2  2 
Where, n – dimension of time; u – dimension of velocity; s – dimension of displacement; and a – dimension
of acceleration.
1 . 6 | Units, Dimensions and Errors

PLANCESS CONCEPTS

The formula for displacement in nth second by a moving body is wrong using dimensional analysis.
NO! Actually, if we go back deeper in derivation we would very easily find that although the equation
looks dimensionally incorrect but it is precise and accurate.
Vaibhav Gupta (JEE 2009, AIR 54)

Illustration 2: A calorie is a unit of heat or energy and it equals about 4.2 J. Suppose that we employ a system of
unit in which the unit of mass equals α kg, the unit of length equals β metre, and the unit of time is γ second. Then,
show that a calorie has a magnitude 4.2 α −1β−2 γ 2 in terms of the new units. (JEE MAIN)

Sol: Here the system is expressed in one set of units. When we want to convert the units in order of magnitude
only, the conversion factor is obtained by dividing the original units by new set of units.
1 cal = 4.2 kg m2 s–2

SI New system
N1 = 4.2 N2 = ?

M1 = 1 kg M2 = α kg

L1 = 1 m L2 = β m

T1 = 1 s T2 = γ s

Dimensional formula of energy is ML2 T −2 


 

Comparing with MaLb Tc  , we find that a = 1, b = 2, and c = –2


 
a b c 1 2 −2
M   L1   T1   1 kg   1 m   1 s  −1 −2 2
Now, N2 = N1  1      = 4.2       = 4.2 α β γ
 M2   L2   T2   α kg   β m γ
   s

Illustration 3: The centripetal force F acting on a particle moving uniformly in a circle may depend upon mass (m),
velocity (v), and radius (r) of the circle. Derive the formula for F using the method of dimensions. (JEE MAIN)

Sol: To obtain the relation between force F, mass M, velocity V and radius r, we use the dimensional analysis. The
power of base units of each quantity on the right of the equality are matched with the power of same unit on left
of the equality.

Let F = K (m) ( v ) (r )
x y z
 … (i)

Here, k is a dimensionless constant of proportionality. By writing the dimensions of RHS and LHS in Eq. (i), we have
x z y
MLT −2  =
= M LT −1  L  MxLy + z T − y 
       
By equating the powers of M, L, and T of both sides, we have
x =1, y =2 and y + z =1 or z =1 − y =−1
mν2 mν2
By substituting the values in Eq. (i), we obtain F =kmν2r −1 =k ; F=
r r
(where k = 1). [The value of K cannot be calculated wring dimensional analysis].
P hysics | 1.7

PLANCESS CONCEPTS

A dimensionally correct equation may or may not be an exact equation but an exact equation must be
dimensionally correct.
Example: F = ma and F = 0.5ma, both are dimensionally correct but only one is correct w.r.t the physical
relation.
Vaibhav Krishnan (JEE 2009, AIR 22)


5. LIMITATIONS OF DIMENSIONS
(a) From a dimensionless equation, the nature of physical quantities cannot be decided, i.e., whether a given
quantity is scalar or vector.
(b) The value of proportionality constant also cannot be determined.
(c) The relationship among physical quantities having exponential, logarithmic, and trigonometric functions
cannot be established.

6. ORDER OF MAGNITUDE
In physics, we often learn quantities which vary over a wide rage. For example, we discuss regarding the size of a
mountain and the size of the tip of a pin. In the same way, we also discuss regarding the mass of our galaxy and
the mass of a hydrogen atom. Sometimes, we also discuss regarding the age of universe and the time taken by an
electron to complete a circle around the proton in a hydrogen atom. However, we observe that it is quite difficult
to get a feel of largeness or smallness of such quantities. Therefore, to express such drastically varying numbers,
we use the power of ten method.
In this method, each number is expressed as a × 10b where 1 ≤ a ≤ 10 and b is an integer. Thus, we represent the
diameter of the sun as 1.39 × 109 m and diameter of a hydrogen atom as 1.06 × 10−10 m. However, to have an
approximate idea of the number, we may round the number ‘a’ to 1 if it is less than or equal to 5 and 10 if it is
greater than 5. Thereafter, the number can be expressed approximately as 10b. Further, we then obtain the order
of magnitude of that number. Thus, now we can more clearly state that the diameter of the sun is of the order of
109m and that of a hydrogen atom is of the order of 10–10m. More precisely, we say that the exponent of 10 in such
a representation is called the order of magnitude of that quantity. Thus, now we can say that the diameter of the
sun is 19 orders of magnitude larger than the diameter of a hydrogen atom. This is due to the fact that the order
of magnitude of 109 is 9 and of 10–10 is –10. The difference is 9 –(–10) = 19.

Table 1.3: Table of SI prefixes

Power of 10 Prefix Symbol


18 exa E
15 peta P
12 tera T
9 giga g
6 mega M
3 kilo k
2 hecto h
1 deka da
1 . 8 | Units, Dimensions and Errors

–1 deci d
–2 centi c
–3 milli m
–6 micro m
–9 nano n
–12 pico p
–15 femto f
–18 atto a

Tip: The best way to remember is by memorizing from milli to atto, kilo to exa and thereafter to go the power of 3.
For example, if one asks for giga since we have already memorized from kilo to exa, then we need to go like kilo
mega giga and since it is 3 in the order shown, giga would be assigned a value of 3 * 3 = 9, i.e., 109.

7. SIGNIFICANT FIGURES
Significant figures in the measured value of a physical quantity provide information regarding the number of digits
in which we have confidence. Thus, the larger the number of significant figures obtained in a measurement, the
greater is the precision of the measurement.
“All accurately known digits in a measurement plus the first uncertain digit together form significant figures.”

7.1 Rules for Counting Significant Figures


For counting significant figures, we make use of the rules listed hereunder:
(a) All non-zero digits are significant. For example, x = 2567 has clearly four significant figures.
(b) The zeroes appearing between two non-zero digits are counted in significant figures. For example, 6.028 has
4 significant figures.
(c) The zeroes located to the left of the last non-zero digit are not significant. For example, 0.0042 has two
significant figures.
(d) In a number without decimal, zeroes located to the right of the non-zero digit are not significant. However,
when some value is assigned on the basis of actual measurement, then the zeroes to the right non-zero digit
become significant. For example, L = 20 m has two significant figures but x = 200 has only one significant
figure.
(e) In a number with decimal, zeroes located to the right of last non-zero digit are significant. For example, x =
1.400 has four significant figures.
(f) The power of ten is not counted as significant digit(s). For example, 1.4 ×10−7 has only two significant figures,
i.e., 1 and 4.
(g) Change in the units of measurement of a quantity, however, does not change the number of significant
figures. For example, suppose the distance between two stations is 4067 m. It has four significant figures. The
same distance can be expressed as 4.067 km or 4.067 × 105 cm. In all these expressions, however, the number
of significant figures continues to be four.

Table 1.4: Significant figures

Measured value Number of significant figures Rule


12376 5 1
6024.7 5 2
P hysics | 1.9

0.071 2 3
410 m 3 4
720 2 4
2.40 3 5

1.6 × 1014 2 6

7.2 Rounding Off a Digit


The rules for rounding off a measurement are listed hereunder:
(a) If the number lying to the right of cut off digit is less than 5, then the cut off digit is retained as such. However,
if it is more than 5, then the cut off digit is increased by 1.
For example, x = 6.24 is rounded off to 6.2 (two significant digits) and x = 5.328 is rounded off to 5.33 (three
significant digits).
(b) If the digit to be dropped is 5 followed by digits other than zero, then the preceding digit is increased by 1.
For example, x = 14.252 is rounded off to x = 14.3 to three significant digits.
(c) If the digit to be dropped is simply 5 or 5 followed by zeroes, then the preceding digit is left unchanged if it
is an even number. For example, x = 6.250 or x = 6.25 becomes x = 6.2 after rounding off to two significant
digits.
(d) If the digit to be dropped is 5 or 5 followed by zeroes, then the preceding digit is raised by one if it is an odd
number.
For example, x = 6.350 or 6.35 becomes x = 6.4 after rounding off to two significant digits.

Table 1.5: Significant digits

Measured value After rounding off to three Rules


significant digits
7.364 7.36 1
7.367 7.37 1
8.3251 8.33 2
9.445 9.44 3
9.4450 9.44 3
15.75 15.8 4
15.7500 15.8 4

7.3 Algebraic Operations with Significant Figures


(a) Addition and subtraction: Suppose in the measured values to be added or subtracted, the least number of
significant digits after the decimal is n. Then, in the sum or difference also, the number of significant digits
after the decimal should be n.
Example: Suppose that we have to find the sum of number 420.42 m, 420.4m and 0.402m by arithmetic
addition
420. 42

420. 4
0. 402
441.222
1 . 1 0 | Units, Dimensions and Errors

But the least precise measurement of 420.4 m is correct to only one decimal place. Therefore, the final answer
will be 441.2 m.
(b) Multiplication or division: Suppose in the measured values to be multiplied or divided, the least number of
significant digits is n; then, in the product or quotient, the number of significant digits should also be n.
Example: 1.2 × 36.72 = 44.064 ≈ 44
In the example shown, the least number of significant digits in the measured values is two. Hence, the result
when rounded off to two significant digits becomes 44. Therefore, the answer is 44.
1100ms−1
Example:= 107.8431373 ≈ 108
10.2ms−1

PLANCESS CONCEPTS

Tip: In algebraic operations with significant figures, the result shall have significant figures corresponding
to their number in the least accurate variable involved.
Nivvedan (JEE 2009, AIR 113)

Illustration 4: Round off the following number to three significant digits: (a) 15462, (b) 14.745, (c) 14.750 and (d)
14.650 ×1012 .  (JEE MAIN)

Sol: The values above when rounded off to the three significant figures, if the fourth digit of the number is greater
than or equal to 5, we increase the third digit by 1 and discard the digits after third digit. If the fourth digit is not
greater than or equal to 5, we discard the digits from fourth onwards and write the number up to third significant
figure. The power of 10 is not considered as the significant number.
(a) The third significant digit is 4. Now, this digit is to be rounded. The digit next to it is 6 which is greater than
5. The third digit should, therefore, be increased by 1. The digits to be dropped should be replaced by zeroes
because they appear to the left of the decimal point. Thus, 15462 becomes 15500 on rounding to three
significant digits.
(b) The third significant digits in 14.745 is 7. The number next to it is less than 5. Therefore, 14.745 becomes 14.7
on rounding to three significant digits.
(c) 14.750 will become 14.8 because the digit to be rounded is odd and the digit next to it is 5.
(d) 14.650 ×1012 will become 14.6 × 1012 because the digit to be rounded is even and the digit next to it is 5.

25.2 × 1374
Illustration 5: Evaluate . All the digits in this expression are significant.  (JEE MAIN)
33.3
Sol: The result of the above fraction is rounded off to the same number of significant figure as is contained by the
least precise term used in calculation, like 25.2 and 33.3.
25.2 × 1374
We have    = 1039.7838.
33.3
Out of the three numbers given in the expression, both 25.0 and 33.3 have 3 significant digits, whereas 1374 has
four. The answer, therefore, should have three significant digits. Rounding 1039.7838 to three significant digits, it
25.2 × 1374
hence becomes 1040. Thus, we write = 1040.
33.3

8. ERROR ANALYSIS
We define the uncertainty in a measurement as an ‘error’. By this we mean the difference between the measured
and the true values of a physical quantity under investigation. There are three possible ways of calculating an error
P hysi cs | 1.11

as listed hereunder:
(i) Absolute error (ii) Relative error (iii) Percentage error
Let us consider a physical quantity measured by taking repeated number of observations say x1 , x2 , x3 , x 4 ,.....
if <x> or x be the average value of the measurement, then the error in the respective measurement is

x2 − x ….; ∆x
∆x1 = x1 − x ; ∆x2 = = xexp erimental value − x true value

However, if we take the arithmetic mean of all absolute errors, then we obtain the final absolute error ∆xmean . When
arithmetic mean alone is considered, then only the magnitudes of the absolute errors are taken into account.

∆x1 + ∆x2 + ............ + ∆xn 1 n


∆xmean =
n
= ∑ ∆x
n i=1 i
It then follows clearly from the above discussion that any single measurement of x has to be such that
xmean − ∆xmean ≤ x ≤ xmean + ∆xmean
∆xmean ∆xmean
Relative error= ; percentage error= × 100
xmean xmean

9. PROPAGATION OF ERRORS

9.1 Addition and Subtraction


If x = A ± B ; then ∆x = ∆A + ∆B
i.e., for both addition and subtraction, the absolute errors are to be added up. The percentage error, then, in the
value of x is
 ∆A + ∆B 
Percentage error in the value
= of x   × 100%
 A ±B 

9.2 Multiplication and Division


A ∆y ∆A ∆B ∆y ∆A ∆B
If y = AB or y = then, = + ⇒ × 100% = × 100% + × 100%
B y A B y A B
⇒ Percentage error in Value of y = percentage error in value of A + percentage error in value of B

Illustration 6: Two resistors R1 = 100 ± 3 Ω and R2 = 200 ± 4 Ω are connected in series. Find the equivalent resistance.
 (JEE MAIN)
Sol: When resistance are added in the series, the error in the resultant combination is given by ∆R eq = ∆R1 + ∆R 2
where ∆R1 = 3 Ω and ∆R2 = 4 Ω.
The equivalent resistance R
= R1 + R=
2 (100 ± 3) Ω + ( 200 ± 4 ) Ω= 300 ± 7 Ω

Illustration 7: A capacitor of capacitance C = 2.0 ± 0.1 µF is charged to a voltage V = 20 ± 0.2 volt. What will be the
charge Q on the capacitor? Use Q = CV.  (JEE MAIN)
∆Q  ∆C ∆V  ∆C ∆V
Sol: The relative error of result of the above product is given by =
± +  where and is the relative
error in determination in C and V respectively. Q  C V  C V

If we omit all errors, then Q =CV =2.0 × 10−6 × 20 C = 40 × 10−6 C


Error in C= 0.1
= part in 2 1=
part in 20 5%
Error in V 0.2 part in 20 = 2 part in 200 = 1 part in 100 =1%; error Q = 5% + 1% = 6%
charge, Q = 40 × 10−6 ± 6% C = 40 ± 2.4 × 10−6 C
∴ Charge,

1 . 1 2 | Units, Dimensions and Errors

9.3 Power Functions

A Bm ∆y  ∆A   ∆B   ∆C 
If y = k then, =  + m  + n 
C n y  A   B   C 
 Percentage error   Percentage error   Percentage error   Percentage error 
  =   + m  + n 
 in value of y   in value of A   in value of B   in value of C 

PLANCESS CONCEPTS

•• The error in a measurement is always equal to the least count of the measuring instrument.
•• Errors never propagate particularly in case of constants.
Nitin Chandrol (JEE 2012, AIR 134)

a3b2
Illustration 8: A physical quantity P is related to four observables a, b, c and d as follows: P =
. The percentage
cd
errors of measurement in a, b, c and d are 1%, 3%, 4% and 2%, respectively. What is the percentage error in the
quantity P?  (JEE MAIN)
a3b2 ∆P  ∆N ∆D  3 2
Sol: The relative error of ratio of P = is calculated as =
± +  where N = a b and D= c d and
cd P  N D 
∆N ∆D
and are the relative error in the N and D.
N D

a3b2 ∆P ∆a ∆b 1 ∆c ∆d
P= ; =3 +2 + +
cd P a b 2 c d

∆a 1 ∆b 3 ∆c 4 ∆d 2 ∆P 1 3 1 4 2
But
= = , = , = , ∴ 3×
= + 2× + × +
a 100 b 100 c 100 d 100 P 100 100 2 100 100

% error in P = 3% + 6% + 2% + 2% = 13%.

10. LENGTH-MEASURING INSTRUMENTS


We know that length is an elementary physical quantity. The device generally used in everyday life for measurement
of length is a metre scale. This scale can be used for measurement of length with accuracy to the extent of 1 mm.
 1 
Therefore, the least count of a metre scale is 1 mm. Further, to measure length accurately up to (1 10 ) th or  
 100 
th of a millimetre, we use the following instruments.
(1) Vernier calipers (2) Micrometer (3) Screw gauge

10.1 Vernier Calipers


C D S
This instrument has three parts.  Main Scale
(i) Main scale: It consists of a strip M, graduated
E
0 1 3 5 6 7 8 9 10 M
in cm and mm at one of its edge. Also, it carries V
two fixed jaws A and C as shown in the Fig. 1.1.
P
(ii) Vernier scale: Vernier scale V slides on metallic
strip M. This scale can be fixed in any position
A B Figure 1.1: Vernier calipers
P hysi cs | 1.13

using the screw S. The side of the Vernier scale which slides over the mm sides has 10 divisions over a length
of 9 mm. Further, B and D are two movable jaws that are fixed with it. When the Vernier scale is pushed toward
A and C, then B touches A and straight side of C will touch straight side of D. In this position, however, if the
instrument is free from error, zeroes of Vernier scale will coincide with zeroes of the main scales. Further, to
measure the external diameter of an object, it is held between the jaws A and B, while the straight edges of C
and D are used for measuring the internal diameter of a hollow object.
(iii) Metallic strip: There is a thin metallic strip E attached to the back side of M and connected with Vernier
scale. When jaws A and B touch each other, the edge of E touches the edge of M. When the jaws A and B are
separated, the E moves outward. This strip E is used for measuring the depth of a vessel.

10.1.1 Principle (Theory)


In the common form, the divisions on the Vernier scale V are smaller in size than the smallest division on the main
scale M; however, in some special cases the size of the Vernier division may be larger than the main scale division.
Let n Vernier scale divisions (VSD) coincide with (n–1) main scale divisions (MSD). Then,

n−1
nV.S.D. =(n − 1) M.S.D.; 1V.S.D. =  M.S.D.; 1 M.S.D. − 1 V.S.D.
 n 
n−1 1
=1 M.S.D. −   M.S.D. = M.S.D.
 n  n

The difference between the values of one main scale division and one Vernier scale division is known as Vernier
constant (VC) or the least count (LC). This is precisely the smallest distance that can be accurately measured with
the Vernier scale. Thus,

1 Smallest division on main scale


V.C L.C=1M.S.D.-1V.S.D.
=   M.S.D.=
n Number of divisions on vernier scale

In the ordinary Vernier caliper, one main scale division is 1 mm and 10 Vernier scale divisions coincide with 9 main
scale divisions.
9
1V.S.D.= M.S.D.=0.9mm; V.C.=1M.S.D. − 1V.S.D.=1mm-0.9mm=0.1mm=0.01cm
10

10.1.2 Reading a Vernier Caliper


If we have to measure a length AB, then the end of A is coincided with 1.0cm<AB<1.1cm
the zero of the main scale. Now, suppose that the end B lies between 1.0 0 1 M 2 3
cm and 1.1 cm on the main scale. Then, let the 5th division of Vernier
scale coincides with 1.5 cm of the main scale.
A B V
Then, AB=1.0+5 × V.C = (1.0 +5 × 0.001) cm = 1.05 cm
Figure 1.2: Reading main scale
Thus, we can make use of the following formula, i.e., and vernier scale
Total reading = N+n × V.C
Here, N = main scale reading just before on the left of the zero of the Vernier scale.
n = number of Vernier division which just coincides with any of the main scale divisions.

PLANCESS CONCEPTS

The main scale reading with which the Vernier scale division coincides has no connection with reading.
Chinmay S Purandare (JEE 2012, AIR 698)
1 . 1 4 | Units, Dimensions and Errors

10.1.3 Zero Error and Zero Correction


If the zero of the Vernier scale does not coincide with the zero of the main scale when jaw B touches jaw A and the
straight edge of D touches the straight edge of C, then the instrument has an error called zero error. This zero error
is always algebraically subtracted from the measured length.
Zero correction has a magnitude that is equal to zero error but its sign is opposite to that of the zero error. Zero
correction is, as a rule of the thumb, always algebraically added to the measured length.
Zero error → algebraically subtracted
Zero correction → algebraically added

10.1.4 Positive and Negative Zero Error


If the zero of Vernier scale lies to the right of the main scale, then the zero error is positive and if it lies to the left
of the main scale then the zero error is negative (when jaws A and B are in contact)
Positive zero error= (N + x × V.C. )
Here, N = main scale reading on the left of zero of Vernier scale.
X = Vernier scale division which coincides with any main scale division.
When the Vernier zero lies before the main scale zero, then the error is said to be negative zero error. If the 5th
Vernier scale division coincides with the main scale division, then

Negative zero error=- 0.00cm + 5 × V.C.  =− 0.00cm + 5 × 0.01cm =−0.05cm



0 1 2 3 0 1

0 5 10 0 5 10
(A) Positive zero error (B) Negative zero error

Figure 1.3: Types of error of vernier scale

Illustration 9: N-divisions on the main scale of a Vernier caliper coincides with N + 1 divisions on the Vernier scale.
If each division on the main scale is of ‘a’ units, then determine the least count of the instrument. (IIT JEE 2003)

Sol: Least count of Vernier caliper is given by L. C. =1 M.S.D – 1 V.S.D.


(N+1) divisions on the Vernier scale = N divisions on the main scale
N

∴1 division on vernier scale= divisions on main scale
N+1
If each division on the main scale is of ‘a’ units
 N 
∴1 division on vernier scale= 
∴  × a unit=a′(say)
N+1
 N  a
Least count = 1 main scale division – 1 Vernier scale division = a − a' =a −  a =
N+1 N+1

Illustration 10: In the diagram provided, find the magnitude and the nature of zero error.  (JEE MAIN)

Sol: Here as the zero division of the main scale is to the left of the zero division of the Vernier scale, thus Vernier
caliper is said to have positive error. Thus the measured length of any object will be greater than actual length. This
error is to be subtracted from measured length to get actual length.
P hysi cs | 1.15

Here, zero of Vernier scale lies to the right of zero of the main scale; hence, it has positive zero error.
Further, N = 0, x = 5,  0 0.5 1
M
L.C. of, V.C. = 0.01 cm.
V
Hence, zero error = N + x × 0.01 = 0.05cm Zero correction = –0.05cm
0 5 10

∴The actual length will be 0.05 cm less than the measured length. Figure 1.4

10.2 Micrometer Screw

10.2.1 Principle of a Micrometer Screw


The least count of Vernier calipers ordinary available in the laboratory
is 0.01 cm. However, when lengths are to be measured with greater 0
accuracy, say up to 0.001 cm, then screw gauge and spherometre are 95
used which are based on the principle of micrometer screw as
discussed here under.
If an accurately cut single threaded screw is rotated in a closely fitted
nut, then in addition to the circular motion of the screw there is also a
linear motion of the screw head in the forward or backward direction, Figure 1.5: Micrometer screw gauge
along the axis of the screw. The linear distance moved by the screw,
when it is given one complete rotation, is called the pitch (p) of the screw. The pitch is basically equal to the
distance between two consecutive threads as measured along the axis of the screw. In most of the cases, it is either
1  1 
1 mm or 0.5 mm. The screw moves forward or backward by  or  of the pitch, if the circular scale (we will
100  50 
discuss later about the circular scale) is rotated through one circular division. This is exactly the minimum distance

which can be accurately measured and hence called the least count (LC) of the screw.
Pitch
Thus, Least count= . If pitch is 1 mm and there are 100 divisions on the
Number of divisions on circular scale
circular scale, then

1mm
L.C.= = 0.01mm
= 0.001cm
= 10µm
100
Since, LC is of the order of 10µm , the screw is called a micrometer screw.

10.3 Screw Gauge


Screw gauge functions based on the principle of a micrometer screw. Circular (Head)
It consists of a U-shaped metal frame M. At one end of it is fixed a scale
N H
small metal piece A. It is called stud and has a plane face. The other AB S
0E
end N of M carries a cylindrical hum, H. It is graduated in millimeters 10 K
or half millimeters based on the pitch of the screw. This scale is called R
a linear scale or a pitch scale. Linear (Pitch)
scale
A nut is threaded through both the hub and the frame N. Through
the nut moves a screw S. The front face B of the screw, however,
facing the plane face A, is also plane in nature. There is a hollow M
cylindrical cap L that is capable of rotating over the hub when screw
is rotated. As the cap is rotated, the screw either moves in or out. Figure 1.6: Main scale and circular scale of
The surface E of the cap K is divided into 50 or 100 equal parts. It is screw gauge
called the circular scale or the head scale. In an accurately adjusted
instrument when the faces A and B are just touching each other, then the zero of the circular scale should coincide
with the zero of the linear scale.
1 . 1 6 | Units, Dimensions and Errors

10.3.1 To measure diameter of a given wire using a Wire


screw gauge A B N H
S
If with the wire between plane faces A and B, the edge of the cap 0 K
lies ahead of Nth division of the linear scale, and nth division of R
circular scale lies over reference line then, Total reading = N + n
× L.C

Figure 1.7: Determination of thickness of wire


10.3.2 Zero error and zero correction
If the zero mark of the circular scale does not Circular scale Circular scale
coincide with the zero of the pitch scale when the
faces A and B are just touching each other, then 0 5 0 0
the instrument is said to possess zero error. 0 45
However, if the zero of the circular scale advances Reference Line Reference Line
beyond the reference line then the zero error is (A) Positive zero error (B) Negative zero error
negative and zero correction is positive. Further,
if it is left behind the reference line the zero error Figure 1.8: Types of errors in reading of screw gauge
is positive and zero correction is negative. For
example, if zero of the circular scale advances beyond the reference line by 5 divisions, then zero correction
=+5 × (L.C. ) and if the zero of the circular scale is left behind the reference line by 4 divisions, then zero correction
=-4 × (L.C. ) .

10.3.3 Backlash error


When the direction of rotation of the screw is suddenly changed, then the screw head may rotate, but the screw
itself may not move forward or backward. Consequently, the scale reading may not change even by the actual
movement of the screw. This is what we meant by backlash error. This error is basically due to loose fitting of
the screw. This arises mainly due to wear and tear of the threading due to prolonged use of the screw. To reduce
this error, we recommend that the screw must always be rotated in the same direction for a particular set of
observations.

PROBLEM-SOLVING TACTICS

 1. While changing units, one can visualize them as some constants multiplied to the numbers. This trick is very
helpful particularly in understanding units. Like you can cut out units from both sides of equations very easily
just like constants.
 2. Application of dimensional analysis can possibly rule out inappropriate answers in multiple choice questions
(MCQs).
Further, dimensional analysis can also be used to eliminate invalid choices in MCQ even in mathematics. It
definitely saves lot of time.
 3. Be careful in handling error approximation. Binomial theorem can only be applied in very low percentage of
errors (generally less than 5% error), else not.
 4. One must accurately know the rules for determining significant digits and be precise while dealing with 0s
and 5.
P hysi cs | 1.17

FORMULAE SHEET

1M.S.D. Smallest division on main scale


(a) V.C.
= L.C.
= = =1M.S.D.-1V.S.D.
= 1M.S.D. − 1V.S.D.
n Number of divisions on vernier scale
1
(b) In ordinary Vernier calipers, = 1mm and n=10 ; V.C. or L.C.=
= 1M.S.D. mm = 0.01cm
10
(c) Total reading = (N+n × V.C.)
(d) Zero correction=–zero error. Zero error is algebraically subtracted, whereas the zero correction is algebraically
added. If zero of Vernier scale lies to the right of zero of the main scale, then the error is positive. The actual
length in this case is less than the observed length.
(e) If zero of Vernier scale lies to the left of zero of the main scale, then the error is negative and the actual length
is more than the observed length.
(f) Positive zero error = (N + x × V.C. )
(g) Least count: The minimum measurement that can be measured accurately by an instrument is called the least
count.
Least count of Vernier caliper
{value of 1 part of main scale(s)} − {value of one part of vernier scale (V)}
or least count of Vernier calipers = 1MSD – 1VSD
where, MSD = Main scale division; VSD = Vernier scale division

Value of 1 part of main scale (s)


Least count =
Number of parts on vernier scale (n)

Pitch (p)
Least count of screw gauge =
Number of parts on circular scale (n)

(h) The maximum absolute error in x is ∆x = + ( ∆a + ∆b )


∆y
(i) Percentage error = + − × 100
ym
( j) Errors never propagate in case of constants.

∆x ∆a
(k) IfIf x a=
= then
x a

∆x ∆a + ∆b
(l) If x= a − b , then =
x a−b

a ∆x  ∆a ∆b 
(m) If x = a b or=x , then=  + 
b x  a b 

anbm ∆x  n∆a m∆b p∆z k∆y 


(n) If b = , then =  + + + 
p k
z y x  a b z y 

 n∆a m∆b p∆z k∆y 


(o) Absolute error=
∆x x  + + +  or in general ; ∆x =x (relative error )
 a b z y 
1 . 1 8 | Units, Dimensions and Errors

Solved Examples

JEE Main/Boards Or M0L0 T  = Ma+bL0 T −2b  Comparing the power of M,


   
Example 1: The value of acceleration due to gravity is L and T on both sides a + b = 0 and –2b = 1
980 cm/s2. What will be its value if the unit of length is
1 1 m
kilometer and that of time is minute? ∴ b =− and a = ; ∴T =Cm1/2 k −1/2 =C  
2 2 k
Sol: Here we have to express value of g in units of km/ This is the correct equation.
min2. Thus the conversion factor from cm/s2, to km/
min2 we need to divide the old units to the new units. Example 3: The radius of the earth is 6.37 × 106 m
Dimension of acceleration due to gravity is LT −2  . In and its mass is 5.975 × 1024 kg. Find the earth’s average
  density to appropriate significant figures.
the CGS system, let L1, T1 represent length and time
ME
measured in cm and second. The numerical value n1 = Sol: The average density of earth is given as DE = .
980 cm/sec2. Let n2 be the value of acceleration due to VE
gravity in the new system. The length L2 and time T2 are The result is rounded off to the same number of
measured in kilometer and minute respectively. Now significant figures as is contained by the least precise
2
term used in the calculation.
 L1   T1−2 
n1 L1 T1−2  n=
= L T −2  or n n1   1 −2 1  −2 Given, mass of the earth (M) = 5.975 ×1024 kg. Further,
   L2   T2  
2  2 2  2
∴ n2 = 980 radius of the earth (R) = 6.37 ×106 m and volume of
 105   60  the earth (V)
−2
 1  1  980 × 60 × 60
∴ n2 = 980  = = ⇒ n2 35.3 4 4
( )m
3
5   60 
 10    105 = × πR3m= × ( 3.142 ) × 6.37 × 106 3
3 3
980 × 60 × 60
= = ⇒ n2 35.3 Average density (D)
Example 1052: A body of mass m hung at one end of the
spring executes a simple harmonic motion. The force
Mass m 5.975 × 1024
constant of a spring is k while its period of vibration = = =
Volume v 4
( )
3
is T. Prove by dimensional method that the equation × ( 3.142 ) × 6.37 × 106
2πm 3
T= is incorrect. Derive the correct equation, = 0.005517 × 106 kgm−3 = 5.52 × 103 kgm−3
k
assuming that they are related by a lower power. (to three significant figures)
Sol: To prove the dimensional inequality of the above The density is accurate only up to three significant
relation we use the dimensional analysis. In dimensional figures which is the accuracy of the least accurate term,
analysis, the power of each basic unit to the left of the namely, the radius of the earth.
equality sign is matched with the dimensions of each
basic units of quantity on the right of the equality. Example 4: A man runs 100.5 m in 10.3 sec. Find his
2πm average speed up to appropriate significant figures.
The given equation is T=
k
Taking the dimensions of both sides, we have Sol: The average speed of man is given by

M dis tance travelled


Vavg = . The result is rounded off to
= T  = T2 Time taken
ML0 T −2 
  the same number of significant figures as is contained
As the dimensions of two sides are not equal, by the least precise term used in the calculation.

the equation is incorrect. Let the correct relation be dis tance travelled
Using average speed =
T = Cma kb where C is a constant. Time taken
100.5m
Equating the dimensions of both sides, we get
= = 9.757 ms−1
b 10.3s
a
 T  = M MT −2  ;
 
P hysi cs | 1.19

Note that, the distance 100.5 m has four significant ∆A 2∆r


figures but the time of 10.3 sec has only three. Thus, ∴ Percentage error in A = = = 2 × 0.5% =
1%
A r
we round off the final result to three significant figures.
4π 3
The average speed must be correctly expressed as (b) The volume of a sphere of radius r is V = r
3
9.76 ms–1.
∆V 3∆r
∴ Percentage error in V = = = 3 × 0.5% =
1.5%
V r
Example 5: The period of oscillation of a simple
L Example 7: In an experiment on the determination
pendulum is T = 2π . L is about 10 cm and is known
g of Young’s modulus of a wire by Searle’s method, the
following data is available:
to 1 mm accuracy. The period of oscillations is about 0.5
second. The time of 100 oscillations is measured with a Normal length of the wire = 110 cm Diameter of the
wrist watch of 1 s resolution. What is the accuracy in the wire d = 0.01 cm
determination of g?
Elongation in the wire  = 0.125 cm
Sol:The accuracy in determination of g is found in This elongation is for a tension of 50 N. The least counts
terms of minimum percentage error in calculation. The for corresponding quantities are 0.01 cm, 0.00005 cm
∆g ∆g and 0.001 cm, respectively. Calculate error in calculating
percentage error in g = ×100 % . Where is the the value of Young’s modulus (Y).
g g
relative error in determination of g.
Sol: The relative error in determination of Young’s
L L 4 π2L
T = 2π or T2= 4 π2 or g = ; ∆Y  ∆N ∆D 
g g T2 modulus is =
± +  where N = 4TL and
Y  N D 
∆g ∆L ∆T
Now, = + 2× ∆N ∆D
g L T D = πd2  and and are the relative error in the
N and D. N D
In terms of percentage,
∆g ∆L ∆T stress T / A TL 4TL
× 100= × 100 + 2 × × 100 ; Y= = ; ⇒ Y= =
g L T strain  / L A πd2 
Percentage error in L
∆Y ∆L ∆ ∆d
= +2
∆L 0.1 Y L  d
100 × = 100 × = 1%
L 10
∆T 1 4T
Percentage error in T = 100 × = 100 × = 2% Since is a constant, so it does not contribute
T 50 π
Thus percentage error in g anything to the net error:
∆g ∆Y 0.01 0.001  0.00005 
= 100 × = 1% + 2 × 2%= 5% + 2
g ⇒ = +  = 0.01809
Y 110 110  0.01 
∆Y
Example 6: The error in the measurement of the radius ⇒ × 100%
= 0.009 + 0.8 +=
1 1.809%
Y
of a sphere is 0.5%. What is the permissible percentage
error in the measurement of its (a) surface area and (b)
volume? Example 8: It has been observed that velocity of
ripple waves produced in water depends upon their
Sol: Percentage error in determination of any quantity wavelength ( λ ) , density of water ( ρ ) and surface
= Relative error in determination of quantity ×100 % . tension (T).
The relative error in area and volume of sphere are T
Prove that V 2 ∝ .
∆A 2 ∆r ∆V 3 ∆r λρ
= and = respectively.
A r V r
Sol: To prove above inequality we need to use the
∆r
Given = 0.5% dimensional analysis. The dimensions of λ, ρ and T are
r
need to be matched with dimensions of V2. The power
(a) The surface area of a sphere of radius r is A= 4 πr 2 of same base units on either side of equality sign are
1 . 2 0 | Units, Dimensions and Errors

equated to get the correct relation.


4 π2 ( 98 + 1.28 )
According to the problem we have = g = 980 cms−2 9.80ms−2
=
( 2.00 )
2

V 2 ∝ λa ρb Tc ⇒ V 2 = K λa ρb Tc  … (i)
...(1)
Error in the value of g
Dimensions of V, ρ, λ and T are [L T −1],
[M L −3], [L] and [M T-2] respectively. ∆g ∆ eff  ∆T  ∆g ∆ + ∆r  ∆T 
= + 2  ;⇒ = + 2 
g ∆ eff  T  g  +r  T 
Thus according to the equation (i),
a b c
Further, since errors can never exceed the least count of
 V 2  =λ
  ρ   T  ⇒ L2 T −2 the measuring instrument. Therefore,
       
b
= L  a ML−3  MT −2 
c
∆ =0.1cm ; ∆r =0.01cm ;
   
∆g  0.1 + 0.01   0.1 
Matching the powers of the same units we get a−3b ∆T =0.1s=
⇒   + 2 
g  98.0 + 1.28   20.0 
=2, b + c = 0 and 2c = 2
∆g ∆g
⇒c = 1, b = −1 and a = −1. ⇒ = 0.0011 + 0.01 ; ⇒ = 0.0111
g g
T T ∆g
Thus we get V 2 =K × ⇒ V2 ∝ . ⇒ Percentage error ⇒ × 100 =
1.1%
λρ λρ g
Hence proved. and absolute error =∆g =g ( 0.011 ) =0.11ms−2 ;

Example 9: In an experiment for determining the = Thus, g 9.80ms−2 ± 1.1%( )


value of acceleration due to gravity (g) using a simple
pendulum, the following observations were recorded. or,
= g ( 9.80 ± 1.1% ) ms−2

Length of the string (  ) = 98.0 cm


Example 10: A student performs an experiment to
Diameter of the bob (d) = 2.56 cm determine the Young’s modulus of a wire, exactly 2 m
Time for 10 oscillations (T) = 20.0 sec long, by Searle’s method. In a particular reading, the
student measures the extension in the length of the
Calculate the value of g with maximum permissible wire to be 0.8 mm with an uncertainty of ± 0.05 mm at
absolute error and the percentage relative error. a load of exactly 1.0 kg. The student also measures the
diameter of the wire to be 0.4 mm with an uncertainty
Sol: The absolute error in g is ∆g= gm − gi where ± 0.01 mm. Take g = 9.8 m/s2 (exact). The Young’s
I = 1,2,3……, etc. and the percentage relative error in modulus obtained from the reading is close to
∆g (A) ( 2.0 ± 0.3) 1011 n / m2
g = Relative error in g ×100 % = × 100 % .
g
(B) ( 2.0 ± 0.2 ) × 1011 n / m2
 eff
Time period for a simple pendulum is T = 2π ........(i)
g (C) ( 2.0 ± 0.1 ) × 1011 n / m2

where  eff is the effective length of the pendulum equal (D) ( 2.0 ± 0.05 ) × 1011 n / m2

 d 20.0 Sol: As relative error in the length and diameter are


to   +  and time period equals=T = 2.00s
 2  10 given, the relative error in determination of Young’s
from (i), we get ∆Y  ∆N ∆D 
modulus is calculated as = ± +  where N
Y  N D 
4 π2 (  eff )
g= ∆N ∆D
T2 = 4TL and D = πd2  and and are the relative
N D
To calculate actual value of g error in the N and D.

 d Y
=
FL
=
4FL
=
( 4 )(1.0 × 9.8 )( 2)
4 π2   + 
4 π (  eff ) 2  4π (  + r ) Al πd2 
( )( )
2 2 2
Since g =
= =  π 0.4 × 10−3 0.8 × 10−3
T2 T2 T2
= 2.0 × 10−3 N / m2
P hysi cs | 1.21

  ∆d   ∆l   the length of a line, the zero mark Vernier scale lies


∆Y  ∆d   ∆l 
Further = 2   +   ;=
∆Y 2   +   y between 10.2 cm and 10.3 cm and the third division
Y  d   l    d   l   of Vernier scale coincides with the main scale division.
 0.01 0.05  11 (a) Determine the least count of the caliper.
= 2 × +  × 2.0 × 10
 0.4 0.8  (b) Find the length of the line.
11 2 11 2
= 0.225 × 10 N/m =
0.2 × 10 N / m
Sol: The smallest count of caliper is L. C.
(by rounding off)
Samllest division on main scale
= .
Or ( Y + ∆Y )= ( 2 + 0.2) × 10
11
N/m 2 Number of divisions on vernier scale

∴ Correct option is (B). (a) Least count (L.C)

Samllest division on main scale


=
Number of divisions on vernier scale
JEE Advanced/Boards
1
= = mm 0.1mm= 0.01cm
Example 1: The pitch of a screw gauge is 1 mm and 10
there are 100 divisions on circular scale. When faces (b) L = N + n (L.C. ) = (10.2 + 3 × 0.01 ) cm = 10.23cm
A and B are just touching each other without putting
anything between the studs, 32nd division of the
circular scale coincides with the reference line. When Example 3: The pitch of a screw gauge is 1 mm
a glass plate is placed between the studs, the linear and there are 100 divisions on the circular scale. In
scale reads 4 divisions and the circular scale reads 16 measuring the diameter of a sphere, 6 divisions on the
divisions. Find the thickness of the glass plate. The zero linear scale and 40 divisions on circular scale coincide
of linear scale is not hidden from circular scale when A with the reference line. Find the diameter of the sphere.
and B touch each other.

Sol: The gauge is found to have positive error. This has Sol: The 6th division of main scale coincides with the
to be subtracted from measured value to get actual 40th division of circular scale, the diameter of sphere is
value. The error is e= number of division coinciding obtained as L.S.D + C.S.D.
with main scale (n) × leas count. 1
L.C.
= = 0.01mm
Pitch 100
Least count L.C.=
Number of divisions on circular scale Linear scale reading=6 (pitch) =6mm
1
= mm = 0.01mm Circular scale reading=n (L.C. ) =40 × 0.01 =0.4mm
100
As zero is not hidden from circular scale when A and B ∴ Total reading = ( 6 + 0.4 ) =
6.4mm
touch each other, the screw gauge has positive error.
e=
+n(L.C.) =
32 × 0.01 =
0.32mm . Example 4: Least count of Vernier calipers is 0.01 cm.
When the two jaws of the instrument touch each other,
Therefore, Linear scale reading=4 × (1mm) =
4mm the 5th division of the Vernier scale coincides with a
Circular scale reading=16 × ( 0.01mm) =
0.16mm main scale division and the zero of the scale lies to the
left of the zero of the main scale. Furthermore while
∴Measured reading= ( 4 + 0.16 ) mm =
4.16mm measuring the diameter of a sphere, the zero mark of
the Vernier scale lies between 2.4 cm and 2.5 cm and
∴Absolute reading=Measures reading-e the 6th Vernier division coincides with a main scale
= ( 4.16 − 0.32 ) mm =
3.84mm division. Calculate the diameter of the sphere.

Thickness of the glass plate is 3.84 mm. Sol: As the instrument is noted to have negative error,
the measured diameter will be less then original. Thus
Example 2: The smallest division of the main scale it has to be added to measured length to get otiginal
of a Vernier calipers is 1 mm and 10 Vernier divisions length of diameter.
coincide with 9 main scale divisions. While measuring The instrument has a negative error,
1 . 2 2 | Units, Dimensions and Errors

e =( −5 × 0.01 ) cm =−0.05cm Sol: As zero mark of circular gauge lies 6 division below
the main reference line, the gauge is noted to have
Measured reading = ( 2.4 + 6 × 0.01=
) 2.46cm True positive error. Positive error e = n × Least count . This
reading = Measured reading error has to be subtracted from the measured reading.
= 2.46 − ( −0.05
= ) 2.51cm P 1mm
L.C.
= = = 0.01mm
Therefore, diameter of the sphere is 2.51 cm. N 100
The instrument has a positive zero error,
Example 5: The pitch of a screw gauge is 1 mm and
there are 100 divisions on its circular scale. When e=+n (L.C. ) =+ ( 6 × 0.01 ) =+0.06mm
nothing is put in between its jaws, the zero of the Linear scale reading=2 × (1mm) =
2mm
circular scale lies 6 divisions below the reference line.
When a wire is placed between the jaws, 2 linear scale Circular scale reading=62 × ( 0.01mm) =
0.62mm
divisions are clearly visible while 62 divisions on circular
scale coincide with the reference line. Determine the Measured reading=2+0.62=2.62mm
diameter of the wire.
True reading
= 2.62 − 0.06 = 2.56mm

JEE Main/Boards

Exercise 1 Q.6 The error in measurement of radius of a sphere is


0.4%. What is the permissible error in the measurement
of its surface area?
Q.1 The force acting on an object of mass m, travelling
mv 2
at velocity V in a circle of radius r is given by F = Q.7 5.74 gm of a substance occupies 1.2cc. Find the
r
The measurements are recorded as density of the substance to correct significant figures.

m = (3.5 ±0.1 ) kg; v = (20 ±1 ) ms–1 and


Q.8 The diameter of a circle is 1.06 m. Calculate its area
r = (12.5 ±0.5 ) m. with regard to significant figures.
Find the maximum possible (i) relative error and
(ii) percentage error in the measurement of force. Q.9 A substance of mass 5.74 g, occupies a volume of
1.2 cm3 . Find its density with due regard to significant
Q.2 The side of a cube is measured as figures.

(75 ± 0.1 ) cm. Find the volume of the cube.


Q.10 If m1 1.2kg
= = and m2 5.42gm. Find (m1 + m2 )
4 π2L with due regard to significant figures.
Q.3 In the formula g = , (  ) has 2% uncertainty
T2
and (T) has 5% uncertainty. What is the maximum Q.11 Assuming force (F), length (L) and time (T) as
uncertainty in the value of g? fundamental units, what should be the dimensions of
mass?
Q.4 The length and breadth of a rectangular field
are measured as length,  = ( 250 ± 5 ) m; breadth Q.12 The velocity (v) of a particle depends on time (t)
according to the relation: v = At2 + Bt + C where V is
b = (150 ± 4 ) m. What is the area of the field?
in m/s and t is in s. Write the units and dimensions of
constants A, B and C.
Q.5 The initial temperature of a body is (15 ± 0.5 ) °C
0
C
and the final temperature is (17 ± 0.3) °C.
0
C What is the Q.13 A calorie is a unit of heat or energy and it equals
rise in temperature of the body? about 4.2 J, where J = 1 kg m2 s–2. Suppose we employ
a system of units in which the unit of mass equals ( α )
P hysi cs | 1.23

kilogram, the unit of length equals ( β ) meter and unit of Q.21 The coefficient of viscosity ( η) of a liquid by the
time ( γ ) seconds. Show that a calorie has a magnitude method of flow through a capillary tube is given by the
formula
4.2 α −1 β−2 γ 2 in terms of new units.
πR 4P
η=
8Q
Q.14 The centripetal force (F) acting on a particle
moving in the circumference of a circle depends upon Where R= radius of the capillary tube,
its mass (m), linear velocity (v) and radius (r) of the  =length of the tube, P= pressure difference between
circle. Use method of dimensions to find the expression its ends, and Q=volume of liquid flowing per second.
for centripetal force.
Which measurement needs to be made most accurately
and why?
Q.15 Show by method of dimensions:
(i) Joule = 107 Erg (ii) 105 dyne/cm2 = 104 N/m2 Q.22 Consider a planet of mass (m), revolving round
the sun. The time period (T) of revolution of the
Q.16 The latent head of ice is 80 cal/ gm. Express it in planet depends upon the radius of the orbit (r), mass
J/kg. of the sun (M) and the gravitational constant (G).
Using dimensional analysis, verify Kepler’s third law of
planetary motion.
Q.17 A satellite is revolving around the earth in a
circular orbit. The period of revolution (T) depends on
(i) Mass of earth (M) Exercise 2
(ii) Radius of orbit (r) and
Single Correct Choice Type
(iii) Gravitational constant (G)
 r3  Q.1 The dimensional formula for Planck’s constant is
Use the method of dimensions to prove that T∝  
 GM 
 
(A) ML2 T −1  (B) ML2 T3  (C) ML−1 T −2  (D) MLT −2 
       
Q.18 The pressure (P), volume (V) and temperature
(T) of a real gas are related through Van der Waals Q.2 Turpentine oil is flowing through a tube of length
equation:  and radius r. The pressure difference between the two
ends of the tube is P; the viscosity of the oil is given


q 
P + 2  ( V − b) =
V 
RT
by η =
(
ρ r 2 − x2) where ν is the velocity of oil at a
4ν
Find the dimensions of constants a and b and also write distance x from the axis of the tube. From this relation,
the units of a and b in the SI system. the dimensions of viscosity η are

Q.19 If the dimensions of length are expressed as (A) M0L0 T0  (B) MLT −1  (C) ML2 T −2  (D) ML−1 T −1 
       
Gx C y hz  where G, C and h are universal gravitational
 
Q.3 The time dependence of a physical quantity is
constant, speed of light in vacuum and Plank’s constant
respectively, then what are the values x, y and z?
( )
given by P=P0 exp −αt2 [Where α is a constant and
t is time]. The constant α
(A) Is dimensionless (B) Has dimensions [T-2]
Q.20 Laplace corrected Newton’s calculation for the (C) Has dimensions [T2] (D) Has dimensions of P
velocity of sound. Laplace said that speed of sound in a
solid medium depends upon the coefficient of elasticity
Q.4 which of the following quantities can be written in
of the medium under adiabatic conditions (E) and the
SI units in kg m2 A-2 s-3
density of the medium ( ρ ).
(A) Resistance (B) Inductance
E
Prove that v = k (C) Capacitance (D) Magnetic flux
ρ
1 . 2 4 | Units, Dimensions and Errors

Q.5 If L and R denote inductance and resistance Q.13 In the measurement of n from the formula
respectively, then the dimensions of L/R is 2Wg
n= , the quantity which should be measured
πr 4 θ
(A) M L T
0 0 0
(B) M L T  (C) M L T (D) MLT
0 0 2 0 2 2
        with the best care is
(A) W (B)  (C) r (D) θ
1
Q.6 The dimensions of   ∈ E2
2
Q.14 When the number 6.03587 is rounded off to the
( ∈0 : permittivity of free space; E: electric field) is
second place of decimals, it becomes
(A) MLT −1 (B) ML2 T −2 (C) ML−1 T −2 (D) ML2 T −1
(A) 6.035 (B) 6.04 (C) 6.03 (D) None

Q.7 Which of the following measurements is most


Q.15 If the velocity (V) acceleration (A) and force (F)
accurate?
are taken as fundamental quantities instead of mass
(A) 0.005 mm (B) 5.00 mm (M), length (L) and time (T), the dimension of Young’s
(C) 50.00 mm (D) 5.0 mm modulus would be
(A) FA2 V-2 (B) FA2 V-3 (C) FA2 V-4 (D) FA2 V-5
Q.8 When 97.52 is divided by 2.54, the correct result is
(A) 38.3937 (B) 38.394 Q.16 The number of particles crossing per unit area
perpendicular to x- axis in unit time is
(C) 38.39 (D) 38.4 n − n1
N = −D 2 where n1 and n2 are number of particles
x2 − x1
Q.9 The density of a cube is measured by measuring per unit volume for x1 and x2 respectively. The
its mass and the length of its sides. If the maximum dimensions of diffusion constant D are
error in the measurement of mass and length are 3%
(A) ML0 T2  (B) M0L2 T −4  (C) M0LT −3  (D) M0L2 T −1 
and 2% respectively, then the maximum error in the        
measurement of density is
(A) 9% (B) 7% (C) 5% (D) 1% Q17 If force, acceleration and time are taken as
fundamental quantities, then the dimensions of length
will be
Q.10 A physical quantity is represented by X=Ma LbT-c.
If percentage error in the measurement of M, L and T (A) FT2 (B) F-1 A2 T-1 (C) FA2 T (D) AT2
are α%, β% and γ% respectively, then total percentage
error is Q.18 In a certain system of units, 1 unit of time is 5 sec,
1 unit of mass is 20 kg and unit of length is 10m. In this
(A) ( αa − βb + γc ) % (B) ( αa + βb + γc ) %
system, one unit of power will correspond to
(C) ( αa − βb − γc ) % (D) none of the above (A) 16 watts (B) 1/16 watts
(C) 25 watts (D) None of these
Q.11 The volume of a sphere is 1.76 cm3. The volume
of 25 such spheres taking into account the significant
figures is Q.19 While measuring acceleration due to gravity by a
simple pendulum, a student makes a positive error of
(A) 0.44 × 102 cm3 (B) 44.0 cm3 1% in the length of a pendulum and a negative error
of 3% in the time period. His actual percentage error in
(C) 44 cm3 (D) 44 .00cm3 the measurement of the value of g will be:
(A) 2% (B) 4% (C) 7% (D) 10%
Q.12 The measurement of radius of a sphere is
( 4.22 ± 2% ) cm. The percentage error in volume of the
sphere is Q.20 A body is moving from height x=0.1 m to x=1.2
in 1 sec under constant acceleration of 0.5m/s2. What
(A) ( 315 ± 6% ) (B) ( 315 ± 2% ) was the initial velocity with which it started? (Correct to
C) ( 315 ± 4% ) (D) ( 315 ± 5% ) significant digits)
(A) 0.85m/s (B) 0.9 m/s (C) 1.0 m/s (D) 0.8 m/s
P hysi cs | 1.25

Q.21 A quantity y is related to another quantity x (B) Surface tension


by the equation y=kxa where k and a are constant. If
(C) Modulus of elasticity
percentage error in the measurement of x is p, then
that in y depends upon (D) Coefficient of viscosity
(A) K and a (B) x and a
Q.29 Which of the following physical quantities
(C) p and a (D) p,k and a all represents the dimensional formula M1 L −2 T −2 
 
(A) Energy/ area (B) Pressure
Q.22 Which of the following quantities has smallest
number of significant digits? (C) Force × length (D) pressure per unit length
(A) 0.00145 cm (B) 14.50 cm
Q.30 In a particular system of unit, if the unit of mass
(C) 145.00 cm (D) 145.0 ×10−6 cm become twice & that of time becomes half, then 8
joules will be written as______ units of work
3.06
Q.23 + 1.15 and express the answer in correct
1.2 (A) 16 (B) 1 (C) 4 (D) 64
significant digits
(A) 3.70 (B) 3.7 (C) 3.75 (D) 3.8 Q.31 Which of the following is not one of the seven
fundamental SI units?

Q.24 Which of the following pairs don’t have same (A) Henry (B) Ampere (C) Candela (D) Mole
dimensions?
Q.32 The dimensional formula for which of the following
(A) Solid angle and vector
pairs is not the same
(B) Potential energy and torque
(A) Impulse and momentum
(C) (Area × velocity) and rate of change of volume with
(B) Torque and work
time
(C) Stress and pressure
(D) None of these
(D) Momentum and angular momentum
Q.25 Which of the following quantities are
dimensionless? (Symbols have their usual meaning) Q.33 Dimensional formula for coefficient of viscosity

(A)
Iω2
Gp
(B) (C)
ρvr
(D)
τθ ( η) [useF= 6πη r v(r= radius ; v = velocity;
mvr T η Iω F = viscous force ]
2
[Useful relation I= mr2, F= = 6πηrv ] (A) ML-2T-1 (B) M-1L1T-1 (C) M1L1T-2 (D) ML-1T-1
5

Q.26 Suppose A=Bn Cm, where A has dimensions LT, B Q34 The time dependence of a physical quantity P is
has dimensions L2 T-1, and C has dimensions LT2. Then  −αt2 
given by p=p0 e  where

α is constant and t is time.
the exponents n and m have values:
The constant α
(A) 2/3; 1/3 (B) 2;3 (C) 4/5; -1/5 (D) 1/5; 3/5
(A) Is dimensionless (B) Has dimensions T-2
Q.27 A uniform wire of length L and mass M is stretched (C) Has dimensions T2 (D) Has dimensions of p
between two fixed points, keeping a tension F. A sound
of frequency µ is aimed on it. Then the maximum
Q.35 From the following pairs of physical quantities, in
vibrational energy is existing in the wire when µ =
which group dimensions are not same:
1 ML FL FM 1 F (A) Momentum and impulse
(A) (B) (C) 2 × (D)
2 F M L 2 ML
(B) Torque and energy

Q.28 The dimension ML-1 T-1 can correspond to (C) Energy and work

(A) Moment of a force (D) Light year and minute


1 . 2 6 | Units, Dimensions and Errors

Previous Years’ Questions (B) Emf, potential difference, Electric potential


(C) Heat, Work done, Energy
2
Q.1 In the formula X = 3Y Z , X and Z have dimensions (D) Dipole moment, Electric flux, Electric field
of capacitance and magnetic induction respectively.
What are the dimensions of Y in MKS system?  (1995)
Q.8 The circular scale of a screw gauge has 50 divisions
(A) M L T Q
−3 −1 3
4
(B) M L T Q
−3 −2 4 4 and pitch of 0.5mm. Find the diameter of sphere. Main
    scale reading is 2. (2006)

(C) M−2L−2 T 4 Q 4  (D) M−3L−2 T 4 Q  S N H


    AB E
0 5 K
1
Q.2 The dimensions of ε0E2 ( ε0 : permittivity of free R
2
space; E: electric field) is (1996)

(A) MLT −1  (B) ML2 T −1 


    M

(C) MLT −2  (D) None of these (A) 1.2 mm (B) 1.25 mm


  AB S N H
(C) 2.20 mm (D)0 2.25
25 mm
E
K
∆V
Q.3 A quantity X is given by ε0L , where ε0 is the R
∆t
Q.9 A student performs an experiment to determine the
permittivity of free space, L is a length, ∆V is a potential Young’s modulus of a wire, exactly 2m long, by Searle’s
difference and ∆t is a time interval. The dimensional method. In a particular reading, the student measures
formula for X is the same as that of (1994)
the extension in the length of the wire to be 0.8mm
M
(A) Resistance (B) Charge with an uncertainly of ± 0.05mm at a load of exactly
1.0kg. The student also measures the diameter of the
(C) Voltage (D) Current
wire to be 0.4mm with an uncertainly of ± 0.01mm .
Take g = 9.8m / s2 (exact). The young’s modulus obtained
Q.4 A cube has a side of length 1.2 x10−2 m . Calculate its from the reading is close to  (2007)
volume (1999)
(A) ( 2.0 ± 0.3) 1011 N / m2 (B) ( 2.0 ± 0.2 ) 1011 N / m2
(A) 1.7x10 m −6 3
(B) 1.73 x10 m −6 3
(C) ( 2.0 ± 0.1 ) 10 N / m (D) ( 2.0 ± 0.5 ) 1011 N / m2
11 2
(C) 1.70 x10 m (D) 1.732x10−6 m3
−6 3

a
α − Q.10 In the experiment to determine the speed of
Q.5 In the relation ρ = e 10 ρ is pressure, Z is distance,
β sound using a resonance column (2007)
k is Boltzmann constant and θ is the temperature. The (A) Prongs of the tuning fork are kept in a vertical
dimensional formula of β will be  (2007)
plane
(A) M0L2 T0  (B) ML2 T  (B) Prongs of the tuning fork are kept in a horizontal
   
plane
(C) ML0 T −1  (D) M0L−2 T −1  (C) In one of the two resonances observed, the length
   
of the resonating air column in close to the wavelength
of sound in air.
Q.6 A wire has mass ∆T radius ( 0.5 ± 0.005 ) mm and
length ( 6 ± 0.06 ) cm . The maximum percentage error in (D) In one of the two resonances observed, the length
the measurement of its density is  (2005) of the resonating air column is close to half of the
wavelength of sound in air.
(A) 1 (B) 2 (C) 3 (D) 4
Q.11 Students I, II and III perform an experiment for
Q.7 Which of the following sets have different measuring the acceleration due to gravity (g) using a
dimensions?  (2005) simple pendulum.
(A) Pressure, Young’s modulus, Stress They use different lengths of the pendulum and / or
P hysi cs | 1.27

record time for different number of oscillations. The Given that 1 division on main scale corresponds to
observations are shown in the table. 0.5 degree. Total divisions on the vernier scale is 30 and
match with 29 divisions of the main scale. The angle of
Least count for length = 0.1cm, Least count for time =
the prism from the above data  (2012)
0.1s
(A) 58.59° (B) 58.77° (C) 58.65° (D) 59°
Student Length of Number of Total time Time
pendulum oscillations for (n) period
Q.16 Let [e0] denote the dimensional formula of
(cm) (n) Oscillations (s)
the permittivity of vacuum. If M = mass, L = length,
(s)
T = time and A = electric current, then:  (2013)
I 64.0 8 128.0 16.0
(A) [e0] = [M-1L-3T4A2]
II 64.0 4 64.0 16.0
III 20.0 4 36.0 9.0 (B) [e0] = [M-1L2T-1A-2]
If EI , EII and EIII are the percentage errors is g.i.e., (B) [e0] = [M-1L2T-1A]
 ∆g  (D) [e0] = [M-1L-3T2A]
 x100  for students I, II and III respectively (2008)
 g 
Q.17 A student measured the length of a rod and wrote
(A) EI = 0 (B) EI is minimum
it as 3.50 cm. Which instrument did he use to measure
(C) EI = EII (D) EI I is max imum it? (2014)
(1) A screw gauge having 100 divisions in the circular
Q.12 A Vernier callipers has 1 mm marks on the main scale and pitch as 1 mm.
scale. It has 20 equal divisions on the vernier scale
(2) A screw gauge having 50 divisions in the circular
which match with 16 main scale divisions. For this
scale and pitch as 1 mm.
vernier callipers, the least count is (2010)
(3) A meter scale.
(A) 0.02 mm (B) 0.05 mm
(4) A vernier calliper where the 10 divisions in vernier
(C) 0.1 mm (D) 0.2 mm
scale matches with 9 division in main scale and main
scale has 10 divisions in 1 cm.
Q.13 The density of a solid ball is to be determined in
an experiment. The diameter of the ball is measured
Q.18 A student measures the time period of 100
with a screw gauge, whose pitch is 0.5mm and there are
oscillations of a simple pendulum four times. That data
50 divisions on the circular scale. The reading on the
set is 90 s, 91 s, 95 s and 92 s. If the minimum division
main scale is 2.5mm and that on the circular scale is 20
in the measuring clock is 1 s, then the reported mean
divisions. If the measured mass of the ball has a relative
time should be:  (2016)
error of 2% the relative percentage error in the density
is  (2011) (A) 92 ± 5.0 s (B) 92 ± 1.8 s
(A) 0.9% (B) 2.4% (C) 3.1% (D) 4.2% (C) 92 ± 3 s (D) 92 ± 2 s

Q.14 Resistance of a given wire is obtained by measuring Q.19 A screw gauge with a pitch of 0.5 mm and a
the current flowing in it and the voltage difference circular scale with 50 divisions is used to measure the
applied across it. If the percentage errors in the thickness of a thin sheet of Aluminium. Before starting
measurement of the current and the voltage difference the measurement, it is found that when the two jaws of
are 3% each, then error in the value of resistance of the the screw gauge are brought in contact, the 45th division
wire is  (2012) coincides with the main scale line and that the zero of
the main scale is barely visible. What is the thickness of
(A) 6% (B) zero (C) 1% (D) 3%
the sheet if the main scale reading is 0.5mm and the
25th division coincides with the main scale line? (2016)
Q.15 A spectrometer gives the following reading when
used to measure the angle of a prism. (A) 0.80 mm (B) 0.70 mm

Main scale reading: 58.5 degree (C) 0.50 mm (D) 075 mm

Vernier scale reading: 09 divisions


1 . 2 8 | Units, Dimensions and Errors

JEE Advanced/Boards

Exercise 1
density ( ρ ) , and the coefficient of viscosity ( η) of the
liquid. Obtain an expression for the critical velocity.
Q.1 A research worker takes 100 careful readings in
an experiment. If he repeats the same experiment by
taking 400 readings, then by what factor will be the Q.9 The mass m of the heaviest stone that can be moved
probable error be decreased? by the water flowing in a river varies with the speed
of water (V), density of water (d) and the acceleration
due to gravity. Prove that the heaviest mass moved is
Q.2 The length, breadth and thickness of a rectangular
proportional to the sixth power of speed. Also find the
sheet of metal are 4.234m, 1.005m and 2.01 cm
complete dependence.
respectively. Find the area and volume of the sheet to
correct significant figures.
Q.10 The frequency (f) of a stretched string of linear
mass density (m), length (  ) depends (in addition to
Q.3 The intensity of X- rays decreases exponentially
quantities specified before) on the force of stretching
according to the law I=i e−µx , where i is the initial
intensity of X-rays and I is the intensity after it penetrates k F
(F). Prove that f = where k is a dimensionless
a distance X through lead. If µ be the absorption  m
constant.
coefficient, then find the dimensional formula for µ.

Q.11 Find out the maximum percentage error while the


Q.4 Two resistors have resistance R1 = ( 24 ± 0.5 ) Ω and
following observations were taken in the determination
R 2 =( 8 ± 0.3) Ω . Calculate the absolute error and the
of the value of acceleration of the value of acceleration
percentage relative error in calculating the combination
due to gravity. Length of thread=100.2cm; radius of
of two resistors when they are in (a) Series (b) Parallel
bob=2.34cm; Time of one oscillation=2.3s. Calculate
the value of maximum percentage error up to the
Q.5 In an electrical set up, the following readings are required significant figures. Which quantity will be
obtained. measured more be measured more accurately?
Voltmeter reading (V)=6.4V
Ammeter reading (I) = 2.0A Q.12 Determine the focal length of the lens from the
following readings:
The respective least counts of the instruments used
in these measurements are 0.2V and 0.1A. Calculate Object distance,
= u 20.1 ± 0.2cm ;
the value of resistance of the wire with maximum Image distance,
= v 50.1 ± 0.5cm .
permissible absolute error and relative percentage
error.
Q.13 The specific gravity of the material of a body is
determined by weighing the body first in air and then
Q.6 The radius of a proton is 10-9 micron and that of in water. If the weight in air is 10.0 ± 0.1 gw and weight
universe is 1027m. Identify an object whose size lies in water is 50. ± 0.1 gw, then what is the maximum
approximately midway between these two extremes on possible percentage error in the specific gravity?
the logarithmic scale.
Q.14 The following observation were actually made
Q.7 If the velocity of light (c), gravitational constant during an experiment to find the radius of curvature
(G) and the plank’s constant (h) are selected as the of a concave mirror using a spherometre:  = 4.4 cm;
fundamental units, find the dimensional formulae for h=0.085 cm. The distance  between the legs of the
mass, length and time in this new system of units. spherometre was measured with a metre rod and the
least count of the spherometre was 0.001 cm. Calculate
Q.8 The critical velocity (Vc) of flow of a liquid through the maximum possible error in the radius of curvature.
a pipe depends upon the diameter (d) of the pipe,
P hysi cs | 1.29

Q.15 It has been observed that the rate of flow (V) of Q.22 As part of their introduction of the metric system
a liquid of viscosity η through a capillary tube of radius the national convention made an attempt to introduce
(r) depends upon η ,r and the pressure gradient P decimal time. In this plan, which was not successful, the
maintained across the length (  ) of the tube. Assuming day-starts at midnight into 10 decimal hours consisting
a power law dependence, prove that the rate of flow of 100 decimal minutes each. The hands of a surviving
of liquid is proportional to r4. Also find the exact decimal pocket watch are stopped at 8 decimal hours,
expression up to a constant. 22.8 decimal minutes. What time is it representing in
the usual system?
Q.16 The height h to which a liquid rises in a tube of
radius (r) depends upon the density of the liquid (d), Q.23 Figure shows a frustum of a cone
surface tension (T), and acceleration due to gravity r₁
(g). Show that it would not be possible to derive the
relation without the additional information that h is h
inversely proportional to r. Also find the relation.
r₂
Q.17 The viscosity η of gas depends upon its mass m,
Match the following dimensionally:
the effective diameter D and the mean speed v of the
molecules present in the gas. Assuming a power law, (a) Total 1/2
find dependence of η on all these quantities. (i) π (r1 + r2 ) h2 + (r − r ) 
2
circumference of the
 1 2 

flat circular faces
Q.18 The distance moved by a particle in time from
the center of a ring under the influence of its gravity is (b) Volume
given by x=a sin ωt where a and ω are constant. If ω is (ii) 2π (r1 + r2 )
found to depend on the radius of the ring (r), its mass (c) Area of the
(m) and universal gravitation constant (G), find using curved surface (
2 2
(iii) πh r1 + r1r2 + r2 )
dimensional analysis an expression for ω in terms of r,
m and G.
Q.24 Suppose that a man defines a unit of force as
mv that which acts due to gravitation between two point
Q.19 The centripetal force is given by F = . The masses each of 1 kg and 1 m apart. What would be
r
mass, velocity and radius of the circular path of an the value of ‘G’ in this new system? What would be the
object are 0.5kg, 10m/s and 0.4 m respectively. Find value of one newton in this new system?
the percentage error in the force. Given: m,v and are Given: G (in SI unit system) =6.6 ×10−11 .
measured to accuracies of 0.005 kg, 0.01m/s and 0.01
m respectively.   Gm1m2  
Use  F = 
  r 2  
Q.20 An experiment to determine the specific resistance
ρ of a metal wire provided the following observations.
Q.25 The distance between neighbouring atoms or
Resistance of R= ( 64 ± 2 ) ohm; Length  = (156 ± 0.1 ) cm; molecules, in a solid substance can be estimated by
calculating twice the radius of a sphere with volume
Radius r= ( 0.26 ± 0.02 ) cm
equal to the volume per atoms of the material.
πr 2R Calculate the distance between neighboring atoms in
If s is expressed as: ρ = Find the percentage error
in ρ .  the following: (a) iron (b) sodium
Given: The densities of iron and sodium are 7870 kg/m3
Q.21 The consumption of natural gas by a company and 1013 kg/m3 respectively, the mass of an iron atom
satisfied the empirical equation = V 1.50 t + 0.008 t2 , is 9.27 ×10−26 kg and the mass of sodium atom is 3.82
where ’V’ is the volume in millions of cubic metre and ‘t’
×10−26 kg.
is the time in months. Expressed this equation in units
of cubic metre and seconds, put the proper units on the
α
coefficients. Assume a month is of 30 days. Q.26 If force ‘F’ and density‘d’ are related as F = ,
then find out the dimensions of α & β . β+ d
1 . 3 0 | Units, Dimensions and Errors

Q.27 If the velocity of light ‘c’ Gravitational constant ‘G’ Multiple Correct Choice Type
& Plank’s constant ‘h’ be chosen as fundamental units,
find the dimensions of mass, length & time in this new Q.5 If dimension of length are expressed as Gx, cy, hz
system. where G, c and h are the universal gravitational constant,
speed of light and plank’s constant respectively, then:
a
nRT − RTV
Q.28 In the formula; ρ = e , find the (A) x=(1/2),y=(1/2) (B) x=(1/2),z=(1/2)
v −b
dimensions of ‘a’ and ‘b’ where P=pressure, n=no. of (C) y=(-3/2), z=(1/2) (D) y=(1/2), z=(3/2)
moles, T=temperature, V=volume and R=universal gas
constant. Q.6 Which of the following groups have the same
dimensions?
Q.29 A ball thrown horizontally from a height ‘H’ with (A) Velocity, speed (B) Pressure, stress
speed ‘v’ travels a total horizontal distance ‘R’. From
dimensional analysis, find a possible dependence of ‘R’ (C) Force, impulse (D) Work, energy
on H, v and g. It is known that ‘R’ is directly proportional
to ‘v’. Comprehension Type

Paragraph 1: The van-der Waals equation is


Exercise 2
 a 
RT, where P is pressure, V is molar
 P + 2  (V − b) =
Single Correct Choice Type  V 
volume and T is the temperature of the given sample of
Q.1 The percentage error in measurement of a physical gas. R is called molar gas constant, a and b are called
quantity m given by m =π tan θ is minimum when Vander walls constants.

(A) θ = 450 (B) θ =900


Q.7 The dimensional formula for b is same as that for
(C) θ =600 (D) θ =300
(A) P (B) V (C) PV2 (D) RT
(Assumed that error in θ remain constant)
Q.8 The dimensional formula for a is same as that for
Q.2 A vernier calliper having 1 main scale division =0.1
(A) V2 (B) P (C) PV2 (D) RT
cm is designed to have a least count of 0.02cm. If n be
the number of divisions on vernier scale and m be the
length of vernier scale, then Q.9 Which of the following does not possess the same
dimensional formula as that for RT
(A) n=10, m=0.5cm (B) n=9, m=0.4 cm
(A) PV (B) Pb (C) a/V2 (D) ab/V2
(C) n=10, m=0.8 cm (D) n=10, m=0.2cm

Q.10 The dimensional formula for ab/RT is


Q.3 In a vernier caliper, N divisions of vernier scale
coincides with N-1 divisions of main scale (in which (A) ML5 T −2 (B) M0L3 T0 (C) ML−1 T −2 (D) M0L6 T0
length of one division is 1 mm). The least count of the
instrument should be Q.11 The dimensional formula of RT is same as that of
(A) N mm (B) N-1 mm (A) Energy (B) Force
1 1 (C) Specific heat (D) Latent heat
(C) cm (D) mm
10 N N−1
Paragraph 2: The power of a hovering helicopter
Q.4 Choose the option whose pair doesn’t have same depends on its linear size, the density of air and
dimensions. (g × density of the helicopter) as power ∝ (linear size)
(A) (Pressure × volume) & Work done x
(density of air)y (g × density of helicopter)z where g
(B) (Force × Time) & Change in momentum is acceleration due to gravity. Given: [power] = ML2 T −3

(C) Kilocalorie & joule [Linear Size]=L; [Density]= ML-3

(D) Angle & no. of moles [g × density]= ML-2T-2


P hysi cs | 1.31

Q.12 The value of y in above expression is Q.17 Considering force (F), velocity (V) and Energy € as
1 1 3 7 fundamental quantities, match the correct dimensions
(A) (B) − (C) (D) of following quantities.
2 2 2 2
Column I Column II
Q.13 The value of x in the above expression is
(A) Mass (p) F1 V 0E1 
1 1 3 7  
(A) (B) − (C) (D)
2 2 2 2
(B) Light year (q) F1 V1E−1 
 
Q.14 The ratio of power output of engine of two
helicopters when linear size of one helicopter is one (C) (r)
fourth of linear size of other and all other parameters 1 F3 V 0E−2 
Frequency    
remaining same is T
(A) 132 (B) 16 (C) 128 (D) 4 (D) Pressure (s) F0 V −2E1 
 
Match the Columns

Q.15 Match the physical quantities in column I with Previous Years’ Questions
their dimensional formulae expressed in column II.

Column I Column II Paragraph 1: A dense collection of equal number of


electrons and positive ions is called neutral plasma.
(A) Angular momentum (p) Certain solids containing fixed positive ions surrounded
ML2 T −2
by free electrons can be treated as neutral plasma. Let
(B) Latent heat (q) ML2 T −2 A −2 N be the number density of free electrons, each of
mass m. When the electrons are subjected to an electric
(C) Torque (r) field, they are displaced relatively away from the heavy
ML2 T −1
positive ions. If the electric field becomes zero, the
(D) Capacitance (s) electrons begin to oscillate about the positive ions
ML3 T −3 A −2
with a natural angular frequency ωp' , which is called
the plasma frequency. To sustain the oscillations, a
(E) Inductance (t) M−1L−2 T 4 A2 time varying electric field needs to be applied that has
an angular frequency ω , where a part of the energy is
(F) Resistivity (u) ML2 T −2 A −1 absorbed and a part of it is reflected. As ω approaches
ωp , all the free electrons are set to resonance together,
(G) Magnetic Flux (v) and all the energy is reflected. This is the explanation of
ML−1 T −2
high reflectivity of metals  (2011)
(H) Magnetic energy (w)
L2 T −2
density Q.1 Taking the electronic charge as  and the permittivity
as ε0 , use dimensional analysis to determine the correct
expression for ωp .
Q.16 Entries in column I are representing physical
quantities whereas entries in column II are representing
Ne mε0 Ne2 mε0
dimensions. Match the columns. (A) (B) (C) (D)
mε0 Ne mε0 Ne2
Column I Column II
(A) Angle (p) Q.2 Estimate the wavelength at which plasma reflection
M1L2 T −3
will occur for a metal having the density of electronics
(B) Power (q) N = 4 × 1027 m–3. Take ε0 = 10−11 and m = 10−30 , where
M0L0 T0 these quantities are in proper SI units.
(C) Work (r) (A) 800 mm (B) 600 mm
M1L2 T −2
(C) 300 mm (D) 200 mm
(D) Unit vector (s)
M1L1 T −2
1 . 3 2 | Units, Dimensions and Errors

Paragraph 2: Phase space diagrams are useful tools Momentum Momentum


in analyzing all kinds of dynamical problems. They are
especially useful in studying the changes in motion
as initial position and momentum are changed. Here (C)
Position
(D)
Position
we consider some simple dynamical systems in one-
dimension. For such systems, phase space is a plane
in which position is plotted along horizontal axis and
Match the Columns
momentum is plotted along vertical axis. The phase
space diagram is x(t) vs p(t) curve in this plane. The Q.5 Column I gives three physical quantities. Select the
arrow on the curve indicates the time flow. For example, appropriate units for the choices given in column II.
the phase space diagram for a particle moving with Some of the physical quantities may have more than
constant velocity is a straight line as shown in the one choice. (1990)
figure. We use the sign convention in which position
or momentum upwards (or to right) is positive and Column I Column II
downwards (or to left) is negative.  (2011)
(A) Capacitance (p) Ohm-second
(B) Inductance (q) Coulomb2-Joule -1
(C) Magnetic Induction (r) Coulomb (volt)-1
Momentum

(s) Newton (ampere


metre)-1

Position (t) Volt-second


(ampere)-1
Q.3 The phase space diagram for a ball thrown vertically
up from ground is Q.6 Match the physical quantities given in column I with
Momentum Momentum dimensions expressed in terms of mass (M), length (L),
time (T), and charge (Q) given in column II and write the
correct answer against the match quantity in a tabular
(A)
Position
(B)
Position form in your answer book (1993)

Column I Column II
Momentum Momentum
(A) Angular momentum
(p) ML2 T −2 
 
(C) (D)
(B) Latent heat
(q) ML2Q −2 
Position Position
 

Q.4 Consider the spring-mass system, with the mass (C) Torque
(r) ML2 T −1 
submerged in water, as shown in the figure. The phase  
space diagram for one cycle of this system is
(D) Capacitance
(s) ML3 T −1Q2 
 

(E) Inductance
(t) M−1L−2 T2Q2 
 

(F) Resistivity
(u) L2 T −2 
 

Momentum Momentum
Q.7 Some physical quantities are given in column I
(A) (B) and some possible SI units in which these quantities
may be expressed are given in column II. Match the
physical quantities in column I with the units in
Position Position
column II.  (2007)
P hysi cs | 1.33

Column I Column II Column I Column II


(A) GMeMs (p) (volt) (A) The energy of (p) System: A
(coulomb) the system is capacitor, initially
G - Universal gravitational (metre) increased. uncharged.
constant,
Process: It is
Me - Mass of the earth, connected to a
Ms - Mass of the sun. battery.

(B) (q) (kilogram) (B) Mechanical (q) System: A gas in an


3RT energy is adiabatic container
(metre)3
M
(second)-2 provided to the fitted with an
R – Universal gas system, which is adiabatic piston.
constant, converted into
T – Absolute temperature, energy of random
motion of its
M- Molar mass. parts.
(C) (r) (metre)2 (C) Internal energy (r) System: A gas in
F2
2 2
(second)-2 of the system rigid container.
qB
is converted
F – Force, Process: The gas
into mechanical
gets cooled due to
q- Charge, energy
colder atmosphere
B – Magnetic field. surrounding it.
(D) (s) (farad) (D) Mass of the (s) System: A heavy
GMe
(volt)2 system is nucleus fission
Re decreased into two fragments
(kg)-1
G – Universal gravitational of nearly equal
constant, masses and some
Me - Mass of the earth, neutrons are
emitted.
R e - Radius of the earth.
(t) System: A resistive
wire loop.
Q.8 Column II gives certain systems undergoing a process.
Process: The loop
Column II suggests changes in some of the parameters
is placed in a time
related to the system. Match the statements in column I
varying magnetic
to the appropriate process (es) from column II.  (2009)
field perpendicular
to its plane.

Q.9 Column II shows five systems in which two objects are labelled as x and Y. Also in each case a point P is shown.
column I gives some statements about X and / or Y. Match these statements to the appropriate systems (s) from
column II.  (2009)

Column I Column II

(A) The force exerted by X on Y (p) Y Block Y of mass M left on a fixed


has a magnitude Mg. inclined plane X slides on it with a
X constant velocity.
P
1 . 3 4 | Units, Dimensions and Errors

Column I Column II

(B) The gravitational potential (q) P Two ring magnets Y and Z, each
energy of X is continuously of mass M, are kept in frictionless
vertical plastic stand so that they
Z
increasing.
Y repel each other. Y rests on the base
X X and Z hangs in air in equilibrium.
P is the topmost point of the stand
on the common axis of the two rings.
The whole system is in lift that is
going up with a constant velocity.

(C) Mechanical energy of the (r) Y


A pulley Y of mass m0 is fixed to a
system X+Y is continuously
table through a clamp X. A block of
P
decreasing. X mass M hangs from a string that goes
over the pulley and is fixed at point
P of the table. The whole system is
kept in a lift that is going down with
a constant velocity.

(D) The torque of the weight of (s) A sphere Y of mass M is put in a non-
Y about point P is zero viscous liquid X kept in a container
at rest. The sphere is released and it
moves down in the liquid
Y

X
P

(t) A sphere Y of mass M is falling with


its terminal velocity in a viscous liquid
X kept in a container.

X
P

Q.10 L, C and R represent the physical quantities Q.12 The pairs of physical quantities that have the same
inductance, capacitance and resistance respectively. dimensions is (are)  (1995)
The combinations of which have the dimensions of
(A) Reynolds number and coefficient of friction
frequency (1984)
(B) Curie and frequency of a light wave
(A) 1/RC (B) R/L (C) 1 / LC (D) C/L
(C) Latent heat and gravitational potential
Q.11 The dimensions of the quantities in one (or more) (D) Plank’s constant and torque.
of the following pairs are the same. Identify the pair are
the same.  (1986)
Q.13 Let ε0  denote the dimensional formula of
(A) Torque and work the permittivity of the vacuum and µ0  that of the
(B) Angular momentum and work permeability of the vacuum. If M = mass, L=length,
T=time and I=electric current.  (1998)
(C) Energy and Young’s modulus
M−1L−3 T −2 I  (B) ε  =
(A) ε0  =  −1 −3 4 2 
(D) Light year and wavelength    0  M L T I 
P hysi cs | 1.35

MLT −2 I −2  (D) µ  =
(C) µ0  =  2 −1  (A) 5.112 cm (B) 5.124 cm
   0  ML T I 
(C) 5.136 cm (D) 5.148 cm

Q.14 The SI unit of inductance, Henry can by written as


 (1998) Q.18 During Searle’s experiment, zero of the Vernier
scale lies between 3.20 × 10–2 m and 3.25 × 10–2 m of
(A) Weber / ampere (B) Volt-second / ampere the main scale. The 20th division of the Vernier scale
(C) Joule / (ampere)2 (D) Ohm-second exactly coincides with one of the main scale divisions.
When an additional load of 2 kg is applied to the wire,
Q.15 A student uses a simple pendulum of exactly the zero of the Vernier scale still lies between 3.20 ×
1 m length to determine g, the acceleration due to 10–2 m and 3.25 × 10–2 m of the main scale but now
gravity. He used a stop watch with the least count of the 45th division of Vernier scale coincides with one of
1s for this and recorded 40s for 20 oscillations. For this the main scale divisions. The length of the thin metallic
observation, which of the following statement (s) is / wire is 2 m and its cross-sectional area is 8 × 10–7 m2.
are true? (2004) The least count of the Vernier scale is 1.0 × 10–5 m. The
maximum percentage error in the Young’s modulus of
(A) Error ∆T in measuring T, the time period, is 0.05s.
the wire is  (2014)
(B) Error ∆T in measuring T, the time period, is 1s.
(C) Percentage error in the determination of g is 5%. Q.19 The energy of a system as a function of time t
is given as E(t) = A2exp(-at), where α = 0.2 s-1. The
(D) Percentage error in the determination of g is 2.5%.
measurement of A has an error of 1.25 %. If the error
in the measurement of time is 1.50 %, the percentage
Q.16 In the determination of Young’s modulus error in the value of E(t) at t = 5 s is  (2015)
 4MLg 
Y =  by using Searle’s method, a wire of length
 πld2  Q.20 Consider a Vernier callipers in which each 1 cm
L = 2m and diameter d = 0.5 mm is used. For a load on the main scale is divided into 8 equal divisions and
a screw gauge with 100 divisions on its circular scale.
M = 2.5 kg, an extension  = 0.25 mm in the length of
In the Vernier callipers, 5 divisions of the Vernier scale
the wire is observed. Quantities d and  are measured
coincide with 4 divisions on the main scale and in the
using a screw gauge and a micrometer, respectively.
screw gauge, one complete rotation of the circular
They have the same pitch of 0.5 mm. The number of
scale moves it by two divisions on the linear scale.
divisions on their circular scale is 100. The contributions
Then: (2015)
to the maximum probable error of the Y measurement
 (2012) (A) If the pitch of the screw gauge is twice the least
count of the Vernier callipers, the least count of the
(A) Due to the errors in the measurements of d and 
screw gauge is 0.01 mm.
are the same.
(B) If the pitch of the screw gauge is twice the least
(B) Due to the error in the measurement of d is twice
count of the Vernier callipers, the least count of the
that due to the error in the measurement of  .
screw gauge is 0.005 mm.
(C) Due to the error in the measurement of  is twice
(C) If the least count of the linear scale of the screw
that due to the error in the measurement of d.
gauge is twice the least count of the Vernier callipers,
(D) Due to the error in the measurement of d is four the least count of the screw gauge is 0.01 mm.
times that due to the error in the measurement of  .
(D) If the least count of the linear scale of the screw
gauge is twice the least count of the Vernier callipers,
Q.17 The diameter of a cylinder is measured using a the least count of the screw gauge is 0.005 mm.
Vernier callipers with no zero error. It is found that the
zero of the Vernier scale lies between 5.10 cm and 5.15
Q.21 There are two Vernier calipers both of which have
cm of the main scale. The Vernier scale has 50 divisions
1 cm divided into 10 equal divisions on the main scale.
equivalent to 2.45 cm. The 24th division of the Vernier
The Vernier scale of one of the calipers (C1) has 10 equal
scale exactly coincides with one of the main scale
divisions that correspond to 9 main scale divisions.
divisions. The diameter of the cylinder is (2013)
The Vernier scale of the other caliper (C2) has 10 equal
1 . 3 6 | Units, Dimensions and Errors

divisions that correspond to 11 main scale divisions. The readings of the two calipers are shown in the figure. The
measured values (in cm) by calipers C1 and C2 respectively, are (2016)

2 3 4

C1

0 5 10

2 3 4

C1

0 5 10

(A) 2.87 and 2.86 (B) 2.87 and 2.87 (C) 2.87 and 2.83 (D) 2.85 and 2.82

Q.22 In an experiment to determine the acceleration due to gravity g, the formula used for the time period of

7 (R − r )
a periodic motion is T = 2π . The values of R and r are measured to be (60 ± 1) mm and (10 ± 1) mm,
5g
respectively. In five successive measurements, the time period is found to be 0.52 s, 0.56 s, 0.57 s, 0.54 s and 0.59 s.
The least count of the watch used for the measurement of time period is 0.01 s. Which of the following statement(s)
is(are) true? (2016)
(A) The error in the measurement of r is 10% (B) The error in the measurement of T is 3.57%
(C) The error in the measurement of T is 2% (D) The error in the determined value of g is 11%

PlancEssential Questions
JEE Main/Boards JEE Advanced/Boards

Exercise 1 Exercise 1
Q. 1 Q. 3 Q.13 Q. 3 Q. 5 Q. 14
Q. 22

Exercise 2 Exercise 2
Q. 9 Q. 10 Q. 21 Q. 1 Q. 3 Q. 21

Q. 36 Q. 54 Q. 56
P hysi cs | 1.37

Answer Key

JEE Main/Boards
Exercise 1
Q.1 0.17,17% Q.2 (422+1.7) ×103cm3 Q.3 12%

Q.5 ( 2 ± 0.8 ) C
0
Q.4 (375 ± 0.17) × 104 m2 Q.6 0.8%

Q.7 4.8gm/cc Q.8 0.882 m2 Q.9 4.8g/cm3

Q.10 1.2 kg Q.11 FL−1 T2 


 

Q.12 m/s3, ms2, ms and [LT-3], [LT-2], [LT-1]

 mv 2 
Q.13  4.2 α −1 β−2 γ 2  Q.14 F =  Q.16 3.3 × 105 J / kg
   r   

 1 3 1
Q.18 ML5 T −2  , L3  and kg m5 s−2 ,m3 Q.19  x = ,y = − ,z = Q.21 R
     2 2 2

 2 
4π 3
Q.22 T2 =  r
 GM 
 

Exercise 2

Single Correct Choice Type

Q.1 A Q.2 D Q.3 B Q.4 A Q.5 B Q.6 C

Q.7 C Q.8 D Q.9 A Q.10 B Q.11 B Q.12 A

Q.13 C Q.14 B Q.15 C Q.16 D Q.17 D Q.18 A

Q.19 C Q.20 B Q.21 C Q.22 A Q.23 D Q.24 D

Q.25 C Q.26 D Q27 D Q.28 D Q.29 D Q.30 B

Q.31 A Q.32 D Q.33 D Q34 B Q.35 D

Previous Years’ Questions


Q.1 B Q.2 D Q.3 D Q.4 A Q.5 A Q.6 D

Q.7 D Q.8 A Q.9 B Q.10 A Q.11 B Q.12 D

Q.13 C Q.14 A Q.15 C Q.16 A Q.17 D Q.18 D

Q.19 A
1 . 3 8 | Units, Dimensions and Errors

JEE Advanced/Boards  T 
Q.16 h = k  
 rdg 
Exercise 1
kmv
Q.17 η =
Q.1 By a factor of 4 D2
Q.3 L-1 Q.19 3.7%

Q.4 R=
S (32 ± 0.8 ) Ω,R=
S (32Ω ± 2.5% ) ; Q.20 18.6%

RP= ( 6 ± 0.2) Ω,RS= ( 6Ω ± 3.33% ) Q.21 R

Q.22 7 hour 44 minute 50 second PM


Q.5 R= (3.2 ± 0.26 ) Ω,R= (3.2 ± 8% ) Ω
Q.23 (a) (ii); (c) (i)
Q.6 106 m, Earth
1

1 1 −3 1 1 −5 1 1 Q.24 1 newton=1.5 ×1010 G unit
Q.7 M
= c=
2 G 2 h2 , L c=
2 G2 h2 , T c2 G2 h2
Q.25 (a) 282 pm (b) 416 pm

Q.8 Vc =
ρd Q.26 M3/2 L−1/2 T −2 ,M1/2 L−3/2 T0
KV 6 d Q.27 h1/2 .c1/2 .G−1/2  ;
Q.9 m  
3
g
 1/2 −3/2
L  = h .c .G1/2  ;
Q.11 9.2%, Time period  
 T  = h .c
1/2 −5/2
.G 
1/2
Q.12 14.3 ± 0.4 cm  
5 −2
Q.13 3% Q.28 a ML
= = T , b  L3

Q.14 0.068
H
r 4p Q.29 R = kv
Q.15 V = K g
η

Exercise 2 Comprehension Type

Q.7 B Q.8 C Q.9 C


Single Correct Choice Type
Q.10 D Q.11 A Q.12 B
Q.1 A Q.2 C
Q.13 D Q.14 C
Q.3 C Q.4 D
Matric Match Type
Multiple Correct Choice Type
Q.15 A→r, B→ w, C→p, D→t, E→q, F→s, G→u, H→v
Q.5 B, C Q.6 A, B, D
Q.16 A→q, B→p, C→r, D→q

Q.17 A→s, B→p, C→q, D→r

Previous Years’ Questions


Q.1 C Q.2 B Q.3 D Q.4 B Q.7 A→p,q; B→r,s; C→r,s; D→r,s

Q.8 A→p,q,s,t; B→q; C→s; D→s Q.9 A→p,t; B→q,s,t; C→p,r,t; D→q Q.10 A, B, C

Q.11 A, D Q.12 A, D Q.13 C Q.14 A, B, D Q.15 A, C

Q.16 A Q.17 B Q.18 4 Q.19 4 Q.20 B, C

Q.21 C Q.22 A, B, D
P hysi cs | 1.39

Solutions

JEE Main/Boards Sol 3: g =


4 π2 
T2
Exercise 1  ∆g   ∆L   ∆T 
=    + 2 
 g   L   T 
mv 2
Sol 1: F =
r Now what we mean by 2% uncertainty in  is
∆F  ∆m   ∆v   ∆r  … (i)  ∆L 
=   + 2  +   → (1)   × 100.
F  m   v   r   L 
Generally any measured physical quantity is noted in Accordingly;
the form of (S ± ∆S) . So, here ∆S is the absolute error.
 ∆g   ∆L   ∆T 
∴ When we look at =
m 3.5 ± 0.1 ;   × 100 =   × 100 + 2   × 100
 g   L   T 
∆m 0.1 =
= and m 3.5
= 2% + 2 × 5% = 12%
Now using formula (i)

∆F  0.1   1   0.5  Sol 4:


=  (250 ± 5)m and
= b (150 ± 4)m
=  + 2  +   = 0.168
F  3.5   20   12.5 
Area of the rectangle =  × b sq.units
∆F ∴=  (1)
A Lb → … (i)
= 0.17
F
Hence the relative error is 0.17 and the percentage ∆A  ∆L   ∆b 
= +  → (2) … (ii)
would be (0.17) × 100 =17% A  L   b 

Sol 2: Volume of the cube for side of length ‘L’ is L3 Cu. Now let us first find the area,
Units. A 250 × 150m2
=
∴ V = L3 A 375 × 102 m2
=
 ∆V   ∆L  ∆A  5   4 
 =3 ⋅   → (1) … (i) Now
=  + 
 V   L  A  250   150 
Here we have to write the volume in standard form i.e.
∆A
V + ∆V = 0.046
A
V = (75)2 cm3
∆A =1750 m2
3
V = 421875cm
∴ Area = (375 × 102 + 1750)m2
3
=⇒ V 422000 cm → (1) … (ii)
Area = (375 + 0.17) × 10 4 m2
∆V  ∆L 
Now = 3.   .V
V  L  Sol 5: T initial 0
= (15 ± 0.5)°Cc
0
 0.1  = (17 ± 0.3)°C
T final c

=V 3 × 421875 
 75  Rise in temperature =
= (2 ± 0.8).

∆V =1687.5 Wondering why it’s not 0.2…..99


There we not the error value such that in the given
V 1700 → (2).
⇒ ∆= … (iii)
reading will not fluctuate above the error value. So let
0
∴ Volume of the cube = (422+1.7) ×103cm3 T1 (15 − 0.5)°C
us say = Tf (17 + 0.3)0°C
C and = C then in this
case we get DT = (2 ± 0.8)°C!
1 . 4 0 | Units, Dimensions and Errors

Now by no means we can get ∆T more then 2.80 c this


a = 1; a + b =0; −2a + c =0
is the inner meaning of error.
⇒ a = 1; b = −1; c = 2.
Sol 6: Surface area of the sphere = 4 πr 2 Sq. units ∴ M = [F L–1 T2]

∆S  ∆r   ∆S   ∆r 
= 2.   ⇒   × 100= 2   × 100 Sol 12: V = At2 + Bt + C
S  r   S   r 
Now using the concept of Dimensions; all the individual
 ∆S  terms i.e. At2 , Bt, C should have the dimension of
⇒  2(0.4%) =0.8 %
 × 100 = velocity v.
 S 
T −1 ] m / s.
∴ C [L=
=
mass And Bt = [L T −1 ]
Sol 7: Density =
volume
⇒ B[T] = [L T −1 ] = T −2 ] m / s2
⇒ B [L=
Mass = 5.74 gm → 3 significant digits
And At2 = [L T −1 ]
Volume = 1.2 cc → 2 significant digits
A[t2 ] = [L T −1 ]
5.74
∴d= = 4.78 = 4.8 gm/cc A [L T −3 ] ⇒=
= −3
A [L T = ] m / s3 .
1.2
 Final result should be in 2 significant digits. Calorie = 4.2[M2 L2 T −2 ]
Now we change the system of units to m', L', T'
Sol 8: Diameter given
m' = αm
= 1.06 m → 3 significant digits.
L' = β
π.d2
And now Area = t' = r t.
4
A = 0.88206 ⇒ A = 0.882m2  m' L'2  T' −2 
Hence=
C 4.2  ⋅ ⋅  
 α β2  γ  
 
Sol 9: Solution similar to Q.7
4.2
C= [m' L'2 T'−2 ]
Sol 10:=
m1 1.2kg → 2 Significant digits αβ2 γ −2

m2 5.42g → 3 Significant digits


= ⇒ C = 4.2 α −1 β−2 γ 2 [m' L'2 T'−2 ]
∴ (m1 + m2 ) would be of 2 significant digit … (i) Hence the magnitude in new units is 4.2 α −1 β−2 γ 2 .
3
m1 + m2 = (1.2 × 10 + 5.42)gms = 1205.42gm (*) Do the same procedure for any question on change
[using(i)]
⇒ 1200 gm[using → (1)] in the units.
∴ m1 + m2 =
1.2kg
Sol 14: This is again a very standard problem.
[This is what happens when we add 10 to a million]
F = ma v b r c
Sol 11: All the problems of this kind; can be solved by And F = [ML T −2 ]
the following method.
−2
⇒ [M L T= ] [M]a ⋅[L T −1 ]b [L]c
M = Fa Lb Tc
But we know that; ⇒ [M L T −2 ] = [Ma Lb + c T −b ]

F = [M L T −2 ] Comparing the powers;


∴ M = [ML T −2 ]a [L]b [T]c a = 1, b = 2, c = −1
a a +b −2a+ c
M = [M L T ] mv 2
∴F=
Comparing the corresponding coefficients r
P hysi cs | 1.41

2 −2
Sol 15: (i) Dimensions of energy = [M L T ] L = [M−a+ c L3a+b + 2c T −2a−b −c ]
Let M1 ,L1 , T1 represent mass in gram, length in cm and Comparing the corresponding component;
time in second. c−a =0
And M2 ,L2 , T2 represents mass in kilogram, length in 3a + b + 2c =1c
meters and time in second.
−2a − b − c =0
Now n1 [M1 L21 T1−2 ] = n2 [M2 L22 T2−2 ] Solve for a,b,c.
 M  L 2  T −2  ∗ This is a typical question from this chapter. So keep
n1 = n2  2  2   2   practicing problems of this type.
 M1  L1   T1  
 
Sol 20: V∝(k)a(E)b(ρ)c
(
⇒ n1 = n2 103 (102 )2 1 ⇒ n=
1 )
n2 × 103+ 4 
  −3 −1
k = [ML−1 T −2 ] , E = [ML2 T −2 ] , ρ =[ML ] , V = [L T ]
7
⇒ n=
1 n2 × 10
Now follow the same procedure as above to find a=1,
7
⇒ 1 Joule = 10 erg. [ n2 = 1] b = 1 / 2, c = −1 / 2

(ii) Similar method for (ii). Π R 4 pP


Sol 21: n = . .
8  qQ
Sol 16: L = 80 cal/gm. Now we have to express it in
J/kg.  ∆n   ∆R   ∆L   ∆P   ∆Q 
 = 4.  + + + 
 n   R   L   P   Q 
We know that 1 cal = 4.2 J and 1 gm = 10–3 Kg.
From this it is evident that an error in R gets magnified
 4.2 
L 80 × 
⇒= J / kg = 80 × 4.2 × 103 J / kg by four times. So we have to be careful in measuring R.
−3
 10 
L 336 × 103 J / kg
= Sol 22: T ∝ ra Mb Gc
L 3.3 × 105 J / kg
⇒= G = [M−1 L3 T −2 ]
Use the standard method followed above to derive
Sol 17: Method is explained in detail in the solution of 1/2
11. Try this yourself.  r3 
T∝ 
 GM 
[Hint: - G = [M−1L3 T −2 ] ]  
4 π2 GM
 q  Infact Keplers third law is =
2
Sol 18:  p +  (V − b) =
RT T r3
 V2  This is the real application of dimensional Analysis. One
Using the concept of dimensional analysis; can derive the body of any formula. Constant are then
q found or performing a couple of experiments.
must have dimension of P (pressure)
V2
⇒ q[L−6 ] = [m L−1 T −=
2
] ⇒ q [ML
= 5 −2
T ] kgm5 s−2 Exercise 2
And for b; it will have dimension of V,
Single Correct Choice Type
3 3
b [L=
= ] m
Sol 1: (A) For the dimension formula for Plank’s
hc
Sol 19: L = Gx Cy hz Constant, we need to know the relation E =
λ
G = [M−1L3 T −2 ] C = [LT −1 ] Where E is the Energy
C is speed of light
h = [ML2 T −1 ]
λ is the wave length
L [M−1 L3 T −2 ]a [L T −1 ]b [ML2 T − 1]c
1 . 4 2 | Units, Dimensions and Errors

1
∴ λ.E = h Sol 6: (C) Physically the term ∈ E2 equals the electric
c
2
energy per unit volume. i.e Energy/Volume.
[ML2 T −2 ][L] [M L2 T −2 ]
⇒ h= = [ML2 T −1 ] ∴ = [M L−1 T −2 ]
[L T −1 ] [L3 ]

Sol 7: (C) Writing down the significant figures of all the


p.(r 2 − x2 ) options.
Sol 2: (D) η =
4µ
[pressure] [L2 ] 0.005 − 1
For the dimension of viscosity η =
[L T −1 ] [L] 5.00 − 3
We have to know the dimensions of pressure; which in 50.00 − 4
turn is force per unit area.
5.0 − 2
∴ [pressure] = [M L−1 T −2 ]
(97.52) (2.54)
[ML−1 T −2 ] [L2 ] −1 −1
Sol 8: (D) ×
η= = [M L T ] ↓ ↓
[L T −1 ] [L] 4 3
∴ The final answer should be with 3 Significant figure.
(B) p p0 exp( − ∝ t2 )
Sol 3:= By observing the options, we can see that option (D)
To find the constant ∝ ; satisfies this condition.
Hint :- All the question inside the exp ( ) will finally end mass m
up with dimension [M0 L0 T0 ] . Sol 9: (A) density = ∴ ρ= 3
volume a
So, ∆ρ  ∆m   ∆a   ∆ρ 
=   + 3  = 3 + 3 (2) ∴   = 9%
∴ [∝] [T2 ] =
[M0 L0 T0 ] ρ  m   a   ρ 

[∝] =[M0 L0 T −2 ]
Sol 10: (B) This is just a generalization of the previous
∴ The dimension are [M0 L0 T −2 ] or [ T −2 ] which is question.
option B.
X = Ma Lb T −c

Sol 4: (A) In this, we verity each option to match the ∆x  ∆m   ∆L   ∆T 


= a.   + b  + c 
given dimension. x  m   L   T 
Capacitance: [M−1 L−2 T 4 A2 ]  ∆x 
 =  (a ∝ +βb + γc)%
 x 
Resistance: [ML2 T −3 A −2 ]
P.S:- don’t get confused with ± c .
Inductance: [ML2 T −2 A −2 ]

Mag. Flux: [ML2 T −2 A −1 ] Sol 11: (B) Here volume is an intrinsic property of each
sphere. So, it will have the same number of significant
So, the answer is resistance. i.e option (A). digit even they are measured in bulk.
Tip: - Don’t get tensed up if you don’t know these terms. ∴ The final result should be having 3 significant digit.
You will learn them later. For time being, do remember Just multiply 1.75 × 25 and then scale the result to 3
them. significant digits.

4 3
Sol 5: (B) Using the dimension mentioned in the above Sol 12: (A) V= πr
question, 3
∆v  ∆r 
We get [L / R] = [M0 L0 T ] = 3.  
v  r 
Tip: - Once check the dimension of (R*C)!!  ∆v  ∆v
  = 3(2%) ⇒ = 6%
 v  v
P hysi cs | 1.43

2WgL  1
Sol 13: (C) η = n1 = n2  
 16 
4
πr θ

∆n  ∆w   ∆g   ∆L   ∆r   ∆θ  Now=
n1 1=
n2 16 watts.
=  + +  + 4  +  
n  w   g   L   r   θ 
Sol 19: (C) T = 2π  / g
Here a small error in r gets magnified by four times in
L
the final result. So, it has to be measured with care. g= 4 π2
T2
Sol 14: (B) Refer theory. ∆g  ∆L   ∆T   ∆g 
∴=   + 2 ⇒  = 1 + 2(3) =7%
g  L   T   g 
Sol 15: (C) Let us first write dimension of Young’s (P.S:- Error is an error either it is +ve or –ve. It effect the
Module in fundamental units Y = [M L −1 −1
T ] end result)
a b c
And now let Y = v A F 1
Sol 20: (B) S1 − S2 = ut + at2
−1 a −2 b
y = [L T ] [L T ] [M L T ] −2 c 2

=s 1.1m and= a 0.5m / s2
t 1s,=
y = [Mc La+b + c T −a−2b −2c ]

But actual Y = [M L−1 T −2 ] Solving the above question;


u = 0.85m / s
∴ Comparing respectively;
But we have to around it to 1 significant digit .(why….??)
c =1; a + b + c =−1 ; − a − 2b − 2c =−2
∴ u = 0.9 m/s
Solving them gives the result.

n2 − n1 Sol 21: (C) y = k xa


Sol 16: (D) N = − D
x2 − x1 Now don’t get confused with k. It’s just a constant !
D = −N
( x2 − x1 )
(n2 − n1 ) ∆y
= a.
∆x
y x
[M0 L T0 ]
D = [M0 L0 T0 ] ∆y
[M0 L−3 T0 ] = a.p
y
D = [M0 L2 T −1 ] ∴ Depends on both a and p.

Sol 17: (D) This is very same as Q.15 Sol 22: (A) Refer theory.
Try this yourself !
Sol 23: (D)
Sol 18 : (A) Power = [M L2 T −3 ] 3.06
+ 1.15
2 −3
n1 [M1 L1 T1 ] =n2 [M2 L2 T2 ] 2 −3 1.2

m1 20kg
= = 20M2 → (1)
2.55 + 1.15
L1 10m
= = 10L2 → (2)

T=
1 5=
s 5 T2 → (3) 2.6 + 1.15
 m  L 2  T −3  ↓
Now n1 = n2  2 .  2  .  2  
 m1  L1   T1  
  3.75
 1  1 2  ↓
.   . (5 ) 
3
n1 = n2 
 20  10   3.8
 
1 . 4 4 | Units, Dimensions and Errors

Sol 24: (D) (A) Solid angel and unit vector. ∴ We get three equations;
Both are dimension. Unit vector is just unit magnitude a + b = 0→ (1) …(i)
with a direction.
a + c = 0 → (2) …(ii)
(B)Potential energy and torque
In a crude way, energy is similar to work −2a =−1 → (3) …(iii)
  
Which is F.s and torque is s × F . Hence the dimension ∴ We get a = 1 / 2, b = c = −1 / 2
would of course be the same.
F
We can also check by comparing down the dimension ∴ We have µ = λ. [ λ is any constant]
ML
of them.
(C) Area × Velocity = [L2][LT–1] = [L3T–1] Sol 28: (D)
3
∆v [L ] (A) Moment of a force = Force × Perpendicular distance
= = [L3 T −1 ] . Hence same.
∆t [T]
= M L T-2 × L = M L2 T-2

Iω2 [M L2 ][T −2 ] Force M L T −1 −2


Sol 25: (C) (A) = ∴ Using hint. (B) Surface tension = = = MT
mvr [M LT −1 L] Length L
Stress
= [ T −1 ] (C) Modulus of elasticity = = Unitless
Strain
Gρ [M−1 L3 T −2 ] [M L−3 ] Fr M L T −2 .rL
(B) = = [ T −1 ] (D) Coefficient of viscosity
= = = M L−1 T −1
T T Av L2 .LT −1
ρvr [M L−3 ] [L T −1 ] [L]
(C) = = [M0 L0 T0 ] Sol 29: (D)
η [M L−2 ] [L−1 ] [L−1 T]
Energy M L−1 T −1
(A)=
Write dimension of η using F = 6πηrv = M L−3 T −2
Area L2
(D) In an easy method,
(B) Pressure = M L−1 T −2 user
We know that T = I ∝
(C) Force × length
Iαθ αθ [T −2 ]
⇒ = = = [T −1 ] (D) pressure per unit length
Iω ω [T −1 ]

Sol 30: (B) Let M' , T' ,L' be the value of mass, time and
n m
Sol 26: (D) A = B C length respectively in the new system.
1
[L T] = [L2 T −1 ]n [L T[L2 ]m
T] = [L2n+m T −n+ 2m ] We know = that M' 2M = and T' T
2
Comparing respective exponents; ∴ n' [M1 L'2 T −2 ] = n [M L2 T −2 ]
2n + m =1…(i)
→ (1)
m'
2m − n = 1 …(ii)
→ (2) n' [M1 L' 2 T'−2 ] = 8 [ L'2 2−2 T'−2 ]
2
Give the value of n and m. 1
n1 =8 × =1
8
Sol 27: (D) Aim of the question is to use dimensional
analysis. Sol 31: (A) Refer theory.

µ =Fa Mb Lc Sol 32: (D) Momentum and angular momentum, Both


A,b,c are constants have different dimension and for momentum

[T −1 ] = [M L T −2 ]a [M]b [L]c and angular momentum mv

[T −1 ] = [Ma+b La+ c T −2a ] mvr
Hence it’s very obvious from the above.
P hysi cs | 1.45

Sol 33: (D) F= 6 π η rv. α Z  0 0 0 k θ 


Sol 5: (A)   = [M L T ] ∴ [α] = 
 F   k θ  Z 
η = 
 6πrv  α  α  k θ 
−2 Further [p] =   ∴ [=
β]  =   
[M L T ] β   p   Zp 
η=
[L] [L T −1 ]
Dimension of k θ are that of energy. Hence,
−1 −1
η =[M L T ]  ML2 T −2  0 2 0
=[β]  =  [M L T ]
−1 −2
2  LML T 
Sol 34: =
(B) p p0 exp[ −αt ]
As describe in Q3; m
Sol 6: (D) Density ρ =
−2
α = [T ] πr 2L

∆ρ  ∆m ∆r ∆L 
Sol 35: (D) Light year is the distance travelled by the ∴ × 100 =  +2 +  × 100
light in one year. ρ  m r L 
After substituting the values, we get the maximum
percentage error in density = 4 %
Previous Years’ Questions
X  Capacitance  Sol 7: (D)
Sol 1: (B)=
[Y] =   
 Z2  2
 (Magnetic induction)  Dipole moment = (charge) × (distance)
Electric flux = (electric field) × (area)
 M−1L−2Q 2 T2  −3 −2 4 4
=  [M L T Q ]
2 −2 −2
 M Q T  Sol 8: (A) Least count (LC)

Pitch 0.5
1 = = = 0.01
Sol 2: (D) ε E2 is the expression of energy density Number of divisions on circular scale 50
2 0
(Energy per unit volume) Now, diameter of ball
1   ML T  2 −2
=(2 × 0.5 mm) + (25 − 5) (0.001) =1.2 mm
E2 
 ε0=  = −1 −2
 [ML T ]
2   L
3

FL 4FL (4)(1.0 × 9.8)(2)
Sol 9: (B) Y= = =
∆q Al π d l π (0.4 ×10 −3 )2 (0.8 ×10−3 )
2
Sol 3: (D) C =
∆V
= 2.0 ×1011 N / m2
A ∆q ( ∆q)L
or ε0 = or ε0 =
L ∆V A.( ∆V) ∆Y  ∆ d   ∆l 
Further
= 2 + 
∆V ( ∆q)L ∆V Y  d   l 
X=
ε0L = L
∆t A ( ∆V) ∆t
  ∆ d   ∆ l    0.01 0.05  11
But [A] = [L ] 2 ∴ ∆ Y 2 
=  +    y = 2 × +  × 2.0 ×10
  d l
     0.4 0.8 
∆q
∴ X= = current = 0.225 ×1011 N / m=
2
0.2 ×1011 N / m2
∆t
(By rounding off)
l3 =
Sol 4: (A) V = (1.2 × 10−2 m)3 =×
1.728 10−6 m3
or (Y + ∆ Y)= (2 + 0.2) ×1011 N / m2
 Length (l) has two significant figures, the volume
(V) will also have two significant figures. Therefore, the
Sol 10: (A) Length of air column in resonance is odd
correct answer is =V 1.7 ×10−6 m3
λ
integer multiple of .
4
1 . 4 6 | Units, Dimensions and Errors

l V
Sol 11: (B) T= 2 π or Sol 14:
V (A) R =
g R= i
i
∆R ∆V ∆i
t l (4 π2 )(n2 )l ∆R⇒ ∆V ∆i =+
= 2π ∴g = ⇒ =+ R V i
n g t2 R V i

∆g  ∆l 2∆ t  ∆V
% error in g = × 100 = +  × 100 × 100 = 3
g  l t  V
∆V
 0.1 2 × 0.1  ⇒ = 0.03
EI =  +  ×100 = 0.3125% V
 64 128  ∆i
Similarly, = 0.03
 0.1 2 × 0.1  i
EII =  +  ×100 = 0.46875%
 64 128  Hence
∆R
= 0.06
 0.1 2 × 0.1  R
EIII =  +  ×100 = 1.055 %
 20 36  ∆R
So percentage error is × 100 =
6%
R
Hence EI is minimum.
Sol 15: (C)
Sol 12: (D) Least count of vernier calipers
1
LC 1MSD − 1 VSD
= L.C =
60
Smallest division on main scale 9
= Total Reading = 585 + 58.65
=
Number of divisions on vernier scale 60

20 divisions of vernier scale = 16 divisions of main


Sol 16: (A)
scale
16 1 q2
∴ 1 VSD = mm = 0.8 mm =F
20 4 πε0 r 2
∴ LC= 1MSD − 1 VSD= 1mm − 0.8 mm  A2 T2 
∴ =0.2 mm =ε0 =  M−1L−3 A2 T 4 
MLT −2L2   
 
Sol 13: (C) Least count screw gauge 1
Sol 17: (D) Least count of vernier calliper is mm
0.5 =0.1 mm = 0.01 cm 10
= = 0.01 mm = ∆r
50
0.5 90 + 91 + 95 + 92
Diameter
= r 2.5 mm + 20 × = 2.70 mm Sol
= 18: (D) tmean = 92 sec
50 4
∆ r 0.01 ∆r 1 Absolute error in each reading = 2, 1, 3, 0
= or × 100 =
r 2.70 r 2.7 2+1+3+ 0
mean
= error = 1.5 sec
m m 2
Now density =
d =
v 4  r 3 Put the least count of the measuring clock is 1 sec.
π 
3 2 So it cannot measure upto 0.5 second so we have to
Here, r is the diameter. round it off.
So mean error will be 2 second
∆d  ∆ m  ∆ r  
∴ × 100 =  + 3    ×100 So t = 92 ± 2 sec.
d  m  r  
∆m  ∆r  1  0.5   0.5 
= × 100 + 3 ×   ×100= 2% + 3 × = 3.11 % Sol 19: (A) Reading=0.5+25   +5   =0.8 mm
m  50   50 
 r  2.7
P hysi cs | 1.47

JEE Advanced/Boards Remember this formula !

d 1 1 dθ ∆θ
Exercise 1 [Hint :  =
dx  θ 
− . = !]
θ2 dx θ2

Sol 1: Increasing the number of readings reduces the  ∆R ∆R 2 


∆ R=  1+  .R 2
errors. This is because, we have more chances to get e  2 2  e ff
the mean closer to the actual value.  R1 R2 

So increasing reading from 100 to 400; reduce the  0.5 0.3  2


∆ Re 
= +  × (6)
problem error by a factor of four.  24 × 24 8 × 8 

∆ Re =
0.2
Sol 2: Length
= (L) 4.234m → 4 significant digits
Breadth
= (B) 1.005m → 4 significant digits ∆ R e = (6 ± 0.2)Ω

2.01 × 10−2 m → 3 significant digits


Thickness (H) = And for relative error;

∴ The volume = Lbh will have 3 significant digits  ∆ R e  0.2  1 


 =  =  
 Re  6  30 
−2 3
⇒ V = (4.234 × 1.005 × 2.01) × 10 m
⇒ V 8.5528 × 10−2 m3
= 1 10
% relative error = × 100
= = 3.33%
V 8.55 × 10
⇒= −2 3
m 30 3

V
Sol 3: I = I0 e−µx Sol 5: Ohm’s Law: V=IR ⇒ R =
I
Now mx should have the dimension [M0 L0 T0 ] 6.4 ∆R ∆V ∆I
R
= = 3.2Ω and = +
⇒ µ.[L] =
[M L T ] 0 0 0 2 R V I

µ =[M0 L−1 T0 ]  0.2 0.1 


R 
∆= +  3.2
 6.4 2 
Sol 4: R1 = (24 ± 0.5)Ω
∆R =0.26
R 2 =(8 ± 0.3)Ω
Resistance R =(3.2 ± 0.26) Ω
(a) Series :-
Reff. = R1 + R2 ∆R  0.26 
And relative error = = 
R  3.2 
Reff. = 32
DReff = ∆R1 + ∆R= 0.5 + 0.3 = 0.8 0.26
2 % Relative error
= × 100 =8.1%
3.2
∴ Reff = (32 ± 0.8)Ω
Now absolute error = 0.8 and Sol 6: Radius of proton
0.8 µ 10−9 × 10−6 m = 10 −15 m
= 10−9=
Relative error = × 100 = 2.5%
32
Size of universe = 1027 m
(b) Parallel:-
1 1 1 …(i) Now let us use log10 (r) as an operator.
= + → (1)
R e ff R1 R 2
log(rp ) = log10 (10−15 ) = −15
24 × 8 24 × 8
Reff
Re ff = = = 6Ω
and
24 + 8 32
27 27 + ( −15) 12
And for Reff; using (i) log(r4 ) log
= = 10 (10 ) 27 log(r)
= = = 6
2 2
∆ R e ff ∆R1 ∆R 2
= + → ⇒ r = 106 m
2
R e ff R12 R 22
1 . 4 8 | Units, Dimensions and Errors

Sol 7: Refer to the solution of Q11 (Ex – 1) and Q19 (Ex – Sol 11: Now if there is an error, the next possible value
2) and try it yourself. of L would be 100.3 or 100.4 cm.
i.e least count for r=2.34 cm, L.C=0.01 cm
Sol 8: Vc ∝ [d]a (ρ)b (η)c
Vc = [L T −1 ]
(L+r)
ρ =[M L−3 ]

d = [L]
[∴ 2.35 or 2.36]
η =[ML−1 T −1 ]
and
= for t 2.3s
= , L.C 0.1s
[L T −1 ] = [L]a [ML−3 ]b [ML−1 T −1 ]c
 ∆g   ∆L   ∆r   ∆T 
[L T −1 ] = [Mb + c La−3b −c T −c ] ∴ =   +   + 2 
 g   L   r   T 
b+c =0  ∆g   0.1   0.01   0.1 
∴ =  +  + 2.  
a − 3b − c =1  g   100.2   2.34   2.3 

−c =−1  ∆g 
  = 0.092
 g 
We get c =
1, b =
−1, and a =
−1
η  ∆g 
∴V ∝   × 100 =
9.2%
c dρ  g 
‘T’ has to be measured more accurately because each
Sol 9: m ∝ V a (d)b (g)c
error gets double magnified in calculating g.
m = [M]
1 1 1
V = [L T −1 ] Sol 12: = + ⇒ f = 14.3
f v u
d = [M L−3 ] 1 1 1
= +
f 50.1 20.1
g = [L T −2 ]
∆f ∆V ∆ 4
And then = +
[M] = [LT −1 ]a [ML−3 ]b [L T −2 ]c f2 v 2 u2
[M] = [M6 La−3b + c T −a−2c ]  ∆V ∆u  2

=f  + f
b = 1; a − 3b + c = 0; − a − 2c = 0  v 2 u2 
 0.5 (0.2) 
⇒ c=
−3 and a =
6 =∆f  +  (14.3)
2
2 2 
 (50.1) (20.1) 
∴ m ∝ V 6 . d g−3
∆f =0.4cm
kV 6 d
⇒ m= Focal length
= (14.3 ± 0.4)cm
g3
wair
Sol 10: f ∝ ma 6 Fc Sol 13: Specific gravity (s) =
w water
−1
[f] = [T ]  ∆s   ∆wair   ∆w water 
=
    +  
m ≡ [M]  s   wair   w water 
 ≡ [L]  ∆s   0.1   0.1 
=   + 
F ≡ [M L T −1 ]  s   10   5 

Now proceeding the same way as we did in Q11 − (Ex − 1)  ∆s 


 × 100  =
3%
 s 
P hysi cs | 1.49

 h   L2  [L] ≡ [Ma+b L−3a+b + c + d T −2b −2c ]


Sol 14:=
R  + 
 2   6h  ⇒ a + b = 0; − 2(b + c) = 0; − 3a + b + c + d = 1

h L2 Here we have 3 equations and four variables to solve


R= +
2 6h ∴ Not possible.

=h (0.085 ± 0.001)cm And now;

=L (4.4 ± 0.1)cm Given that h is inversely proportional to ‘r’


∴ d = –1
∆R ∆h ∆L ∆h
= +2 + Now we can solve for a, b, c
R h L h

∆R  ∆h ∆L  Sol 17: η ∝ ma Db v c
= 2 +2 
R  h L 
η ≡ [M L−1 T −1 ]
 ∆R   0.001 0.1 
=
  2 +  m ≡ [M]
 R   0.085 4.4 
 ∆R  D ≡ [L]
  = 0.068
 R  V ≡ [L T −1 ]

p
c Solve in a similarly way to Q.16
a b
Sol 15: V ∝ n r  

Sol 18: ω ∝ r a mb Gc
V-Rate of flow ⇒ Volume/time ≡ [L3 T −1 ]
[ω] ≡ [m0o LL0o TT0]o ]
η ≡ [m L−1 T −1 ]
[r] ≡ [L]
r ≡ [L]
p m ≡ [M]
≡ [m L−2 T −2 ]
p ≡ [m L−1 T −2 ] *
L
G = [M−1 L3 T −2 ]
Now solving for the constants a, b, c gives us the result.
And solve for a, b, c
[L3 T −1 ] = [M L−1 T −1 ]a [L]b [M L−2 T −2 ]c

[L3 T −1 ] = [Ma+ c L−a+b −2c T −a−2c ] mv 2


Sol 19: F =
r
a+c =0
m (0.5 ± 0.005)kg
=
−a + b − 2c =3
v (10 ± 0.01)m / s
=
−a − 2c =−1
=r (0.4 ± 0.01)m
a=
−1 c =
1 and b =
4
∆F  ∆m   ∆v   ∆r 
=  + 2 + 
Sol 16: h ∝ (ρ)a (T)b (g)c (r)d F  m   v   r 

h ≡ [L]  0.005   0.01   0.01 


=  + 2 + 
 0.5   10   0.4 
δ ≡ [ML−3 ]
 ∆F 
T ≡ [M L T −2 ]   = 0.037
 F 
g = [L T −2 ]  ∆F 
 × 100  =
3.7%
r = [L]  F 

Now proceeding; πr 2R
Sol 20: ρ =

[L] ≡ [ML−3 ]a [M L T −2 ]6 [L T −2 ]c [L]d
1 . 5 0 | Units, Dimensions and Errors

 ∆ρ   ∆r   ∆R   ∆L  G 6.6 × 10−11 N.
∴=
 =2⋅  +  + 
 ρ   r   R   L  But according to the problem, the force is 1 unit.

 ∆ρ   0.02  6.6 10−11 N ≡ 1 unit


⇒ F =×
 × 100  =
2⋅ × 100 
 ρ   0.26  1
∴1N≡ unit
6.6 × 10−11
 2   0.1 
+ × 100  +  × 100  = 18.6%
 64   156  We call this unit as Gunit.
N 1.5 × 1010 Gunit
∴ 1=
+.50t 0.008 2
Sol 21:
= V + t
↓ ↓
(2) (2) Sol 25: For each atom, we have,

Now let us examine the units of (1) and (2) for (1); unit m 9.27 × 10−26
is m2/s and dimension is [L3 T −1 ] . And for 2; unit is v
= = = 1.178 × 10−29 m3 / atom
ρ 7870
m3 / s2 and dimension is [L3 T −2 ].
4 πr 3
 m3  r 1.41 × 10−10 m
Now, = 1.178 × 10−29 m3 ⇒ =
∴ V (1.50 m3 / s1 )t +  0.008 ⋅ 1  t2
= 3
 s12 
 Hence, the distance between atoms is d = 2r= 2.82 ×
By changing the unit system; 10-10m

The value of coefficients of ‘t’ and ‘t2’ change.


α
Sol 26: F =
n1 L31 T1−1  = n2 L32 T2−1  β+ d
   
Using this we can find the values of new coefficients. Here the dimension of β and d should be same.
Hence [m1/2 L−3/2 ] = β
Sol 22: 24 hours ≡ 10 Decimal hours α
And now the dimension of should be
⇒ 1 D h = 2.4 hrs. → (1) d
and 1 Dh = 100 D mins Same as F.
α
2.4 = [MLT −2 ]
⇒ 1 D min
= → (2) [m −1/2 −3/2
L ]
100

Given time = 8 Dh and 22.8 Dmin ⇒ α =[m3/2L−1/2 T −2 ]

 2.4 
= 8(2.4) + 22.8   hours. = 19.747 hours Sol
= T −1 ] G [M−1 L3 T −2 ]
27: c [L=
 100 
h = [M L2 T −1 ]
⇒ 19 Hr, 10 minutes, 50 seconds Now for mass M = cx Gy hz
Finding the value x, y, by following the method
Sol 23: Aim of the question is to do dimensional analysis. described in Q11 (Ex1)
Circumference will have dimension [L]
Volume [L3] −a
 nRT  RTV
Area [L2] Sol 28: p =  e
 v −b
Verify the options for correct choices There we can use the ideal gas equation;
pv = nRT
Sol 24: For m
=1 m
=2 1kg and=
r 1 m, In solving question involving RT; we can replace RT by
PV and then proceed.
G.(1)2
The force
= F = GN a
1 So, now will have a dimensionally [M0 L0 T0 ]
RTV
P hysi cs | 1.51

a Minimum value of y=f(x) then that would be at a point,


= [M0 L0 T0 ]
pv 2 (x0 ) where
a ≡ [M L−1 T −2 ][L6 ] dy
= f1=
(x) 0
a = [M L5 T −2 ] dx
And that minimum/maximum value would be f (x0 )
And now for b;
now here;
nRT m=
π tan θ
should be dimension equal to P.
v −b
dm d
pv =
π (tan θ) ≡ π.sec2 θ = 0
∴ ≡p dθ dθ
v −b
⇒ v −b ≡ v sec2 θ = 0
This would give us no value of θ .!
⇒ b should be dimensionally equal to V
And now we don’t have any choice rather than to go for
∴ b = [L3 ]
an analytical method.

Sol 29: R ∝ Ha v b gc  ∆m 
First let us find  
 m 
r ≡ [L]
∆m =π sec2 θ
H ≡ [L]
∆m 1  2 
v ≡ [L T −1 ] = =   → (1)
m sin θ cos θ  sin2θ 
g ≡ [L T −2 ]
∆m
a −1 b −2 c Now for to be minimum;
[L] ≡ [L] [L T ] [L T ] m
sin2θ has to be maximum
[L] ≡ [La+b + c T −b −2c ]
sin2θ = n π / 2 (n is odd)
a + b + c = 1 → (1) … (i)
θ = n.π / 4
b + 2c =0 → (2) … (ii) Hence θ = 450 is the answer.
And also given that, R ∝ v, a = 1, b = 1
Sol 2: (C) The least count of the vernier can be measured
So a + c =0 by using the formula;
1 + 2c =
0 =L.C 1 M.S.D − 1 V.S.D
⇒ c=
−1 / 2 and a =
1/2 M.S.D → Main scale division.

H VSD → Vernier scale division.


∴R ∝ v
g Now in most cases MSD is given.
Our task would be to find VSD.
H
∴R =k v
g ∴ Let us say N division of vernier scale coincides with
N-1 division of main scale, then
N−1
1 VSD = 
Exercise 2 
 N 
N−1
Single Correct Choice Type And least count = 1 −  
 N 
Sol 1: (A) m =
π tan θ 1
L.C =  
N
In solving this problem; we shall use basic calculus. Let us
say we find to find the ∴ Here given; L.C = 0.02 cm
1 . 5 2 | Units, Dimensions and Errors

⇒ 1 USD = 0.1 – 0.02 (i) Only quantities with same physical dimension can be
added or subtracted.
1 USD = 0.08 cm
∴ b must be having a dimension of volume.
And the number of division of vernier scale =10.
∴ Length of vernier scale = 0.8
Comprehension Type
[∴ 0.8 ×10]
Sol 7: (B) b is dimension same as V.
Sol 3: (C) Explained briefly in the above question.
a
(a) 0.00145 → 3 Sol 8: (C) Now must be having dimensioned same
as P. V2
(b) 14.50 → 4
⇒ a ≡ PV 2
(c) 145.00 → 5

(d) 145.0 ×10−6 → 4 Sol 9: RT = [RT] = ML2T–2


(A) PV = [PV] = ML–1T–2L3= ML2T–2
Sol 4: (D) Option A, B, C are obvious.
(B) Pb = [pb] = ML–1T–2L3= ML2T–2
Now in option D.
a a
Angle doesn’t have any dimension. And no. of moles (C) =   = ML5T–2L6= ML1T–2
2 2
v v 
being representing the number of particles, but still we
define a dimension µ for moles. ab  ab 
(D) =   = ML5T–2L3L–6 = ML1T–2
2 2
v v 
Multiple Correct Choice Type ⟹ (C) does not match
Sol 5: (B, C) L = Gx c y hz
ab
−1 3 −2 x −1 y 2 −1 z Sol 10: (D)
L = [M L T ] [L T ] [M L T ] RT

L = [Mz − x L3x + y + 2z T −2×− y − z ] Now we know ideal gas equation PV=RT


for one mole of ideal gas.
Comparing the respective powers;
ab ab PV 2 .V
z − x = 0 …(i)
→ (1) ⇒ ≡ ≡ ≡ V 2 ≡ L6
RT PV PV
3x + y + 2z = 1…(ii)
→ (2)
Sol 11: (A) RT ≡ PV
−2x − y − z = 0 
→ (3) …(iii)
⇒ [M L−1 T −2 ].[L3 ]
1 −3 1
=x = , y = , z ⇒ [M L2 T −2 ]
2 2 2
It is equal to energy.
Sol 6: (A, B, D) (A) Velocity and speed − yes [L T −1 ]
Sol 12 – Sol 13: (B, D)
(B) Pressure and stress − yes [M L−1 T −2 ]
Power ∝ (size)x (Density of air)y
force
(C) and impulse − No (Density of Helicopter × g]z
↓ ↓
[M L T −2 ] [M L T −1 ] [M L2 T −3 ] ∝ [L]x [M L−3 ]y [M L−2 T −2 ]z

(D) Work, energy − yes [M L2 T −2 ] [M L2 T −3 ] ∝ [My + 2 Lx −3y −2z T −2z ]

 a  Comparing the corresponding exponents;


p + 2  ( v − b) =
RT
 v  y+z =
1
Now we need to understand that;
P hysi cs | 1.53

x − 3y − 2z =
2 Previous Years’ Questions
−2z =−3
Sol 1: (C) N= Number of electrons per unit volume
Solving them gives;
∴ [N] = [L−3 ], [e] =[q] =[It] =[AT]
3 1 7
λ= , y=− , x=
2 2 2 [M−1 L−3 T 4 A2 ]
[ ε0 ] =
x Substituting the dimension we can see that,
P1 L 
Sol 14: (C) =  1 
P2  L2   Ne2 
  = [T −1 ]
 m ε0 
P1  
7/2
= (4)
= 128 Angular frequency has also the dimension [T −1 ]
P2

2πc 2πc 2πc


Sol 2: (B) ω= 2 π f= ∴ λ= =
Match the Columns λ ω Ne2 / m ε0

Sol 15: A→r, B→ w, C→p, D→t, E→q, F→s, G→u, H→v Substituting the values, we get λ ≅ 600 mm

Angular momentum = MVR = MLT–1(L)=MLT–1 Sol 3: (D) Momentum is first positive but decreasing.
Latent heat = L2T–2 Displacement (or say position) is initially zero. It will
first increase. At highest point, momentum is zero and
Torque = F×R = MLT–2×L=ML2T–2
displacement is maximum. After that momentum is
Capacitance downwards (negative) and increasing but displacement
is decreasing. Only (D) option satisfies these conditions.
Charge AT
= = = M−1L−2 T 4 A2 In all the given four figures, at mean position the
Potential Difference ML T −3 A −1
2
position coordinate is zero.
Magnetic Flux At the same time mass is starting from the extreme
Inductance = = ML2T–2A–2 position in all four case. In figures (C) and (D), extreme
Current
position is more than the initial extreme position. But
Resistivity = ML3T–3A–2 due to viscosity opposite should be the case.
Magnetic Flux = ML3T–3A–2 (B =F/qv) Correct answer is (B), because mass starts from positive
extreme position (from uppermost position). Then,
Magnetic Energy Density = ML–1T–2
it will move downwards or, momentum should be
negative.
Sol 16: A→q, B→p, C→r, D→q
L
Angle and unit vector = MoLoTo Sol 4: (B) t ≡ ∴ L ≡ tR ≡ ohm-second
R
Power = W/T=M L T 1 2 –3

q2 q2
Work = F,D=M L T 1 2 –2 U≡ ∴ C≡ ≡ coulomb2 / joule
2C U
Therefore, A→q, B→p, C→r, D→q
q
q ≡ CV ∴ C ≡ ≡ coulomb / volt
V
Sol 17 A→s, B→p, C→q, D→r
−e
L≡
Force → F, Velocity → V, Energy → E di / dt

(p) [F1VoE1] ⟹ MLT–2. ML2T–2 ⟹ ML3T–4 ((B) Light year) e(dt)


∴L ≡ ≡ volt-second/ampere
(q) [F V E ] ⟹ MLT . LT .M L T ⟹ T ((C) Frequency)
1 o 1 –2 –1 –1 –2 2 –1 (di)

(r) [F1VoE1] ⟹ M3L3T–2. M–2L–4T–4 ⟹ ML–1T–2 ((D) Pressure) F=ilB


F
(s) [F1VoE1] ⟹ L–2T2. ML2T–2 ⟹ M ((A) Mass) ∴B ≡ ≡ newton/ampere-metre
il
1 . 5 4 | Units, Dimensions and Errors

Sol 5: Sol 12: Reynold’s number and coefficient of friction are


dimensionless quantities.
Column I Column II
Curie is the number of atoms decaying per unit time
Capacitance Coulomb-volt coulomb2 and frequency is the number of oscillations per unit
joule-1 time.
Latent heat and gravitational potential both have the
Inductance Ohm-sec, volt second
same dimension corresponding to energy per unit
ampere-1
mass.
Magnetic induction Newton (ampere-metre)-1
1 q1q2
Sol 13: (C) F = .
4 π ε0 r 2

Sol 6: The correct table is as under


[q1 ] [q2 ] [I T2 ]
[ ε0 ]
= = = [M−1L−3 T 4 I2 ]
2
Column I Column II [F] [r ] [MLT ] [L2 ]
−2

Angular momentum
[ML2 T −1 ] 1
Speed of light, c =
ε0 µ0
Latent heat
[L2 T −2 ]
1 1
Torque ∴ [µ 0 ] =
= = [MLT −2 I −2 ]
2
2 −2
[ML T ] [ε0 ] [c] [M L T I ] [LT −1 ]2
−1 −3 4 2

Capacitance
[M−1L−2 T2Q2 ] φ weber
Sol 14: (A, B, D) (A) L = or henry=
i ampere
Inductance
[ML2Q −2 ]
 di  e
(B) e = −L   ∴ L =−
Resistivity dt
  (di/ dt)
[ML3 T −1Q −2 ]
volt-second
or henry=
Sol 7: A → p, q; B → r, s; C → r, s; ampere
D → r, s 1 2 2
(D)
= U = L i i Rt
2
Sol 8: A → p, q, t; B → q; C → s, D → s
∴L =Rt or henry = ohm –second
Sol 9: A → p, t; B → q, s, t; C → p, r, t, D → q
40 s
Sol 15: (A, C)
= T = 2s.
Sol 10: (A, C) Resistance = ML2T‒3 A‒2 20

Inductance = L = ML2T‒2 A‒2 Further,= = 20T or ∆ t = 20 ∆T


t nT
Capacitance = C = ML‒1T‒2 A2 ∆t ∆T
∴ =or
t T
1 1
∴ = = Frequency  … (i)
LC T T  2 
∆T = .∆t =   (1) = 0.05 s
t  40 
R 1
Again = = Frequency  … (ii)
L T l
Further, T= 2 π or T ∝ g−1/2
g
Sol 11: (A, D) (A) Torque and work both have the
dimensions [ML2 T −2 ] . ∆T 1 ∆g
∴ × 100 =− × × 100
T 2 g
(D) Light year and wavelength both have the dimension
of length ie., [L] . or % error in determination of g is
P hysi cs | 1.55

∆g ∆T 200 × 0.05 1
×100 =
−200 × =
− −5%
= 1 main scale division = cm
g T 2 8
1
1 vernier scale division = cm
4MLg 10
Sol 16: (A) Y = & % ymax = %M + %L + %l + 2%d
πld2 1
So least count = cm
0.5 40
Least count of both instrument, ∆ =∆d = =5 × 10 −3
100 For screw gauge,
−3
∆ 5 × 10 pitch (p) = 2 main scale division
% × 100
= = 2%
 0.25 P
So least count p =
∆d 5 × 10 −3 100
%d × 100
= × 100
= 1%
d 0.5
Sol 21: (C)
Here we see that, contribution of  , = 2%
In first; main scale reading = 2.8 cm.
Contribution of d = 2% d = 2 × 1 = 2%
1
Hence both terms  and d contribute equally. Vernier scale reading = 7 × 0.07 cm
=
10
So reading = 2.87 cm;
Sol 17: (B)
In second; main scale reading = 2.8 cm
Main scale division (s) = .05 cm
Vernier scale reading =
49
Vernier scale division (v) = = .049 −0.1 −0.7
100 7× = = −0.07 cm
10 10
Least count = .05 – .049 = .001 cm
so reading = (2.80 + 0.10 − 0.07) cm = 2.83 cm
Diameter: 5.10+24 × .001 =5.124 cm

Sol 22: (A, B, D) Error in T


FL
Sol 18: (4) Y = since the experiment measures only
A 0.52 + 0.56 + 0.57 + 0.54 + 0.59
T=
mean = 0.556 ≈ 0.56s
change in the length of wire 5

∆Y ∆ ∆Tmean =
0.02
∴ × 100 = × 100
Y 
0.02
From the observation 1 =MSR+20(LC) ∴ error in T is given by
∴Error × 100 =
3.57%
0.56
 2 =MSR+40(LC) 1
Error in r = × 100 =10%
⇒ Change in lengths = 25(LC) 10

and the maximum permissible error in elongation is Error in g


one LC 7 (R − r )
 T = 2π
∆Y (LC) 5g
∴ × 100= × 100= 4 %
Y 25(LC)
7R −r 
T2 = 4 π2  
5 g 
Sol 19: 2(4)−αE(t) = A2 e−αt
E(t) = A e t
⇒ dE = −αA2e−αt dt + 2AdAe−αt 28π2  R − r 
⇒ dE = −αA2e−αt dt + 2AdAe−αt g=  
5  T2 
Putting the values for maximum error,
dE 4 ∆g  ∆R + ∆r  ∆T 2
⇒ = ⇒ % error = 4 % =  +2 = + 2 × 0.0357
E 100 g  R −r  T 50

∆g
Sol 20: (B, C) For vernier callipers, ∴ × 100 ≈ 11%
g
2017-18 100 &
op kers
Class 11 T
By E ran culty
-JE Fa r
IIT enior emie .
S fP r es
o titut
Ins

PHYSICS
FOR JEE MAIN & ADVANCED
SECOND
EDITION

Exhaustive Theory
(Now Revised)

Formula Sheet
9000+ Problems
based on latest JEE pattern

2500 + 1000 (New) Problems


of previous 35 years of
AIEEE (JEE Main) and IIT-JEE (JEE Adv)

5000+Illustrations and Solved Examples


Detailed Solutions
of all problems available

Topic Covered Plancess Concepts


Tips & Tricks, Facts, Notes, Misconceptions,
Motion in a Straight Line Key Take Aways, Problem Solving Tactics

PlancEssential
Questions recommended for revision
2. M OT I O N I N A
STRAIGHT LINE

1. INTRODUCTION
Mechanics is a branch of Physics which deals with motion of bodies and the cause behind it. Motion of a body that
includes position and time can be determined with respect to other bodies. Branch of physics that deals with the
motion of particles and rigid bodies irrespective of the forces responsible for their motion is known as Kinematics.
When the size of a body is too small such that its motion can be described by a point mass moving along straight
line, motion is known as rectilinear motion or one-dimensional motion.

1.1 Motion
A body is said to be in motion when it changes its position with respect to the observer while it is said to be at rest
when there is no change in its position with respect to the observer. For instance, two passengers travelling in a
moving train are at rest with respect to each other but in motion for a ground observer.

1.2 Particle
Physically, a particle is considered as analogues to a point. A body with a definite size is considered as a particle
when all of its parts have same displacement, velocity and acceleration. The motion of any such body can be
studied by the motion of any point on that body.

1.3 Basic Definitions y

1.3.1 Position and Position Vectors rA A

Position of any point can be represented by its coordinates with respect to an


x
origin in Cartesian system. For example, point A can be represented by (xA, yA, zA). O
Figure 2.1
  ∧ ∧ ∧
rA = OA = x A i + y A j + z A k y

A B
Given the co-ordinates of two points A & B, the position vector of B w.r.t. A can be
rA
determined as follows:
rB
    x
AB =rB − rA ⇒ AB =( xB − x A ) ˆi + ( yB − y A ) ˆj + ( zB − z A ) kˆ
O
Figure 2.2

∧ ∧ ∧  
Illustration 1: Find the torque (τ) exerted by force i + j + 2k at point P (2, 3, 4) w.r.t origin. Given, τ =rxF

r : is the position vector of point at which force is acting w.r.t to the given point (in this case origin)  (JEE MAIN)
2 . 2 | Motion in a Straight Line


Sol: The force F is given in Cartesian coordinates. Express the position vector of point P in Cartesian coordinates
 
and find the cross product τ =rxF
   
Here= r op= P [ P : Position vector of point P]
 ∧ ∧ ∧
⇒ r = 2 i + 3 j + 4k
 ∧ ∧ ∧ i j k
and F = i + j + 2k  ∧ ∧
  
∴ τ = r ×F 2 3 4 = i(2) − j(0) + k(
 −1) = τ = 2 i −k
∧ ∧ ∧ 1 1 2
= i + j + 2k

1.3.2. Distance and Displacement


y
A particle follows either a curve or a straight line when moving in the space. This curve
A
or line is known as its trajectory. Distance is the length of the path or trajectory covered
by the particle and displacement is the difference between the vectors of the first and rA rAB B
the last position on this path.
rB
Distance and displacement  are illustrated in the Fig. 2.3 where AB (curve length) is
 
the distance and vector ∆ rAB is the displacement ( rB − rA ) . Following points should be O x
considered about both the quantities: Figure 2.3
(a) Distance is a scalar quantity while displacement is vector.
(b) Displacement is always less than or equal to distance in magnitude.
(c) Displacement can be zero but distance cannot be zero for a moving body.

PLANCESS CONCEPTS

For a particle moving in a straight line:


•• the distance travelled is always equal to the displacement when there is no change in direction,
i.e. distance travelled = |displacement|
•• Else, distance travelled is always greater than displacement,
i.e. distance travelled ≥ |displacement|
Vaibhav Gupta (JEE 2009 AIR 54)

Illustration 2: Find the distance and displacement of a particle travelling from one point to another, say from pt. A
to B, in a given path. (JEE MAIN)

50 m

10 m
40 m
A B

55 m

Figure 2.4
P hysics | 2.3

Sol: Distance is length of the path travelled. Displacement is the vector


from initial point to final point.
Displacement
Total distance travelled = 10+50+40+55+ (40 - 10) = 185 m A B

Total Displacement = 50 + 55 =105 m


Deduction of displacement is from A (initial position) to B (final position) Figure 2.5

Illustration 3: If a particle travels a distance of 5 m in straight line and returns back to the initial point, then find
(i) Total distance travelled (ii) Total displacement (JEE ADVANCED)

Sol: Distance is length of the path travelled. Displacement is the vector from initial point to final point.
Total distance travelled = 5 + 5 = 10 m (since, initial and final 5m
position of the particle are same). A B
Displacement = 0 m (5-5=0, since the directions are opposite
and cancel with each other) Figure 2.6

1.3.3 Average Speed and Average Velocity


The total distance travelled by a particle divided by the total time taken is known as its average speed.
Total distance travelled
Average speed =
Total time taken
displacement rB − rA ∆rAB
While the average velocity is defined as v av = = or v =
Or
time elapsed tB − t A ∆t
Both average speed and average velocity are expressed in ms-1 or kmh-1, the former is a scalar quantity while the
latter is a vector.

PLANCESS CONCEPTS

For a moving body:


•• Average velocity can be zero but average speed cannot be zero.
•• The magnitude of average velocity is always less than or equal to the average speed because,
displacement ≤ dis tance

displacement distance

time elapsed time elapsed
therefore,
•• Average speed does not mean the magnitude of the average velocity vector.
Vaibhav Krishnan (JEE 2009 AIR 22)

Illustration 4: If a train moves from station A to B with a constant speed v =40 km/h and returns back to the initial
point A with a constant speed V2=30 km/h, then calculate the average speed and average velocity.  (JEE MAIN)

Sol: Average speed is distance covered divided by time taken. Distance is length of the path travelled. Average
velocity is displacement divided by time taken. Displacement is the vector from initial point to final point.
s
Let the distance AB = s, Time taken by train from A to B,   t1 =
v1
2 . 4 | Motion in a Straight Line

s Total dis tance s+s s+s


Time taken by train From B to A,t2 = ; Average =
speed = =
v2 Total time taken t1 + t2 s s
+
v1 v 2

2v1 v 2 2x40x30 240 Net displacement 0


V=
avg = = = 34.3km / h ; Average velocity
= = = 0
v1 + v 2 40 + 30 7 Total time t1 + t2

Illustration 5: Consider a train moving from station A to B with a constant speed of 40 km/h for half the time and
with constant speed of 30 km/h for the next half time of that journey. Calculate the average speed of the whole
journey. (JEE MAIN)

Sol: Average speed is distance covered divided by time taken. Here we need to assume total time of journey as T.
The speed in each half of the time T is constant. The distance covered in each half of the time can be easily written
in terms of T. Average speed is distance covered divided by total time taken.
Let AB = s and T = Total time of journey. A B
T T 40km/h 30km/h
∴ Distance travelled in first half time is, s1 = v1
2 2 Figure 2.7

T T Total dis tance


Distance travelled in second half time is, s2 = v 2 ; Average speed =
2 2 Total time

v1 (T / 2) + v 2 (T/ 2) v1 + v 2 40 + 30
Vavg = ; Vavg = = ; Vavg = 35 km / h
T 2 2
Illustration 6: A particle travels half of the journey with speed 2 m/s. For second half of the journey, the particle
travels with a speed of 3 m/s for half of remaining time, and for the other half it travel with a speed of 6 m/s. Find
its average speed.  (JEE ADVANCED)
Sol: Average speed is distance covered divided by total time taken. Here we need to assume total distance of
journey. The speed in each part of the journey is constant. The time taken to cover each part of the journey can be
calculated in terms of distance covered and speed in that part.
Let total distance = 4 s 2s 2t
Let the time taken in covering last half of journey = 2t
Speed= 2m/sec 3m/Sec 6m/Sec
For the first half of the journey
speed (v1) = 2 m/s and distance (d1) = 2s Figure 2.8

dis tance 2s
So, time taken ( t1=
) = = s
speed 2
For the first part of second half of the journey

v2 = 3 m/s ; t2 = t s; ∴ d=
2 v 2 x t=
2 3t

For the second part of second half of the journey


; d3 v=
V3 = 6 m/s ; T3 = t s= 3 x t3 6t m

Total dis tance 4s 2s


We
we know
know d2 + d3 = = = 2s ; ⇒ 3t + 6t = 2s; ⇒ t =
2 2 9
2s 2s 13s
Therefore, total time taken for journey = t1 + t2 + t3 = s + t + t =+
s + =
9 9 9
Total dis tance 4s 36
∴ Vavg = = = m/s
Time int erval 13s 13
9
P hysics | 2.5

Illustration 7: Consider the following:


Men running in a straight line with a speed of 15 m/s one behind the other at equal intervals of 20 m. Cyclists are
also riding along the straight line with a speed of 25 m/s at equal intervals of 30 m.
An observer moving in the opposite direction with a speed of v such that comes across men and cyclists at the
same time. Find the velocity v. (JEE ADVANCED)

Sol: The relative velocity between observer and men is (15 + v). The relative velocity between observer and cyclists
is (25 + v). In the reference frame of observer, the time elapsed between the passing of two men is 20/(15 + v) and
the time elapsed between the passing of two cyclists is 30/(25 + v). Both these timings should be equal so that the
men and cyclists pass the observer together.
Let us assume that a man, a cyclist and the observer are in line. Now after t time, the observer again meets with
the next man and cyclist

15m/Sec 15m/Sec
20 m Men
15t

25 m/Sec 25m/Sec
Vm/S Cyclists
30 m
25t
Observer
vt

Figure 2.9

Then the distance travelled by the observer = vt


Distance travelled by the man = 15t = 20 –vt  ... (i)
Distance travelled by the cyclist = 25t = 30 – vt  ... (ii)
Simplifying (i) and (ii) we get
15t + vt = 20 … (iii)
25t + vt = 30  … (iv)
15t + vt 20 15 + v 2
Dividing (iii) and (iv) we get = ; ⇒ = ; ⇒ 45 + 3v = 50 + 2v; ⇒ v = 5 m/sec
25t + vt 30 25 + v 3

Illustration 8: 2 Cars A and B simultaneously start with speed 20 m/sec and 30 m/sec, respectively. Both cars have
constant but different acceleration. On completing the race simultaneously, if the final velocity of A is 90 m/sec,
then find the final velocity of B (i.e. vB) (JEE ADVANCED)
Sol: Both cars travel equal distance in equal time. Initial velocities and accelerations of both the cars are different.
Problem can be best solved by equating the average velocities.
Since both cars travel equal distance at equal intervals of time, both cars have equal average speeds, i.e. average
speed of car A = average speed of car B and for
u+ v 20 + 90 30 + vB
for constant
for constant accn , avg velocity =
acceleration, ;∴ =; ⇒ VB = 80 m/s
2 2 2

1.3.4. INSTANTANEOUS SPEED AND INSTANTANEOUS VELOCITY


The average speed/velocity of a moving body with
t → 0 ) is known as instantaneous speed/velocity. Therefore
infinitesimally small time interval (i.e. ∆
2 . 6 | Motion in a Straight Line

 
 ∆r dr
Instantaneous velocity
= v lim
= .
∆t →0 ∆t dt

(a) As the time interval tends to zero (i.e. ∆t → 0 ), the displacement vector ∆ r is along the direction of motion
of the particle i.e. tangential to the path of the particle at that instant. Thus, the instantaneous velocity direction is
always tangential to the path of the the particle.
(b) Instantaneous speed and the magnitude of instantaneous velocity are always the same.
i.e. Instantaneous Speed= | Instantaneous Velocity |
A particle moving on a straight line, say along the x-axis, has an instantaneous velocity as follows
∆x
v ( t ) − lim − lim v
∆t →0 ∆t ∆t →0 av
A particle is said to move in a uniform velocity when the velocity of a particle remains constant with respect to time.
It is said to be accelerated when velocity changes with respect to time.

y
Illustration 9: The distance travelled by a particle in time t is given by s(t) = (2.5 m / s2 )t2 . V
A
Find (a) the average speed of the particle during time 0 to 5 s?
rA rAB
(b) The instantaneous speed at t = 5.0 sec  (JEE MAIN) B
rB
Sol: Average speed is distance covered divided by total time taken. Instantaneous
O x
speed is the rate of change of distance at a particular instant.
(a) The distance travelled during time 0 to 5.0 sec Figure 2.10

= s (5 sec) = 2.5 x (5)2 = 62.5 m

distance travelled 62.5m


∴ Average speed = ;=
⇒ Vav = 12.5m / s
time interval 5.0 sec− 0.0 sec
ds(t) ds
(b) s(t) = 2.5t2 ; ∴ = 5t ; ∴ instantaneous speed (v) =
= (2.5)x(2t) | = 5(5.0) = 25 m/s
dt dt t =5.0

1.3.5. Average acceleration and Instantaneous Acceleration


Rate of change of velocity is defined as acceleration. Velocity changes with change in magnitude or direction or
both.
 
Suppose the velocity of a particle at time t1 is v1 and at time t2 it is v 2 . The change produced in time interval
  
t1 to t2 is v 2 − v1. Average acceleration aav is defined as change in velocity with respect to change in time, i.e.
 
 v 2 − v1
aav =
t1 − t2
The average acceleration with infinitesimally small time interval ∆t is known as instantaneous acceleration, i.e.
 
 ∆v dv
=a lim
=
∆t dt
(a) Acceleration is a vector quantity and its SI unit is ms-2.
(b) The average acceleration vector and the change in velocity vector are in the same direction.
(c) The direction of velocity vector and the direction of acceleration vector are independent of each other.
(d) Acceleration is perpendicular to the velocity vector only when there is change in direction of velocity with time,
with its magnitude being constant.
If a body moves with uniform acceleration along a straight line, then the average acceleration and instantaneous
acceleration will always be the same.
P hysics | 2.7

PLANCESS CONCEPTS

Retardation:
•• Negative acceleration does not imply retardation.
•• Retardation refers to decrease in speed and not velocity.
Nivvedan JEE 2009 AIR 113

Illustration 10: Consider a particle moving with a speed of 5 m/s towards east. After 10 sec N
velocity of particle is 5 m/s towards north. Find the average acceleration and its direction. 
 (JEE MAIN)
∧ W E

v =5i
Sol: Average i acceleration is change in velocity divided by total time taken.
∧ ∧
 
v = 5 i ; vf = 5 j ∴ v f − fi = 5ˆj − 5iˆ S
i
∧ ∧
 ∧  
Times j v f − v=i =10
v f = 5 interval 5 jsec
−5 i ˄
J
 
  ∧ ∧ v f − vi
We
v f −know
v i = 5 jthat
− 5 i, Average acceleration = Time interval ˄
i

5 j − 5i 1 ∧ ∧ Figure 2.11
⇒ accnavg = = ( j − i )m / sec2
10 2
˄
1 ˆ ˆ 1 (-i)
∴ | accnavg =| ( j − i) = m / sec2
2 2
∧ ∧ ˄
1 45° J
Unit vector along that direction=
is ( j − i ) [ 45° due west of north]
2

Figure 2.12
PLANCESS CONCEPTS

Motion of bodies in three dimensional space:



If a body has coordinates (x, y, z) in space, its position vector r at time is given by r = xiˆ + yjˆ + zkˆ .

The velocity vector v is given by v = v X i + v y j + v z k
where vx ,vy vz are magnitudes of components of the velocity along the x-, y-, and z-axes, respectively, at
dx dy dz
time=
t. v x = , vy = , vz
dt dt dt
The acceleration a is given by

a= ax ˆi + ay ˆj + a= kˆ
where ax,ay and az are components of acceleration along x, y, and z directions respectively at time t.
Chinmay S Purandare (JEE 2012 AIR 698)

2. UNIFORMLY ACCELERATED MOTION FOR 1-D MOTION

2.1 Equations of Motion


Consider that the acceleration of a particle ‘a’ is constant.
2 . 8 | Motion in a Straight Line

First Equation: Acceleration is defined as the rate of change of velocity


dv
a= ⇒ dv = a.dt  … (i)
dt
The velocity at time 0 is u and at time t is v. Thus, at t = 0, v= u and
At t = t, v = v.
Integrating equation (i) for these limits: V = v to u = v and t = 0 to t = t
v t

∫ dv =∫ adt ; ⇒  v u = a t 0 ⇒ v − u= a ( t − 0 ) ; ⇒ v − u =at ; ⇒ v − u =at


v t

u o

Second Equation: Velocity is defined as the rate of change of displacement.

ds
v= ⇒ ds = vdt ; ⇒ ds =(u + at)dt [∴ v =u+ at]  … (ii)
dt
Suppose the position of the particle is ‘0’ at time ‘0’ and ‘s’ at time ‘t’. Hence,
at t = 0, s = 0 and
at t = t, s = s
s t
Integrating equation (ii) for these limits: ∫ ds
= ∫ (u + at)
o o
t t t
s t 1
⇒ s  = ∫ udt + ∫ at.dt ⇒ s − 0  = u t  + ∫ tdt ⇒ s = ut + at2
0
0 0
0
0
2

Third Equation: From the definition of acceleration


dv dv dx dv
a
= ⇒a . ; ⇒ a = .v ; ⇒ v.dv =
a.dx … (iii)
dt dx dt dt
If the velocity at position 0 is u and at position s is v,
at s=0, v=u and
at s=s, v=v
Integrating equation (iii) for these limits: V=u to v = v and x = 0 to x = s
v s  v2  s v 2 u2
∫ vdv = ∫ adx ; ⇒   = a[x] o
2

2

2
= a[s − 0]
u o  
v 2 − u2
= as ; ⇒ v 2 − u2 =
2as
2

Displacement In nth Second


1
Displacements Sn of a particle in n seconds and Sn-1 in (n-1) seconds are given as: s= n un + an2 ;
1 2
sn = u(n − 1) + a(n − 1)2
2
Displacement in nth second: sn – sn-1 = un + 1 an2 − u(n − 1) − 1 a(n − 1)2 ; x = a
u + (2n − 1)
n
2 2 2
Illustration 11: Consider a particle moving in straight line with constant acceleration “a” traveling 50 m in 5th
second and 100 m in 10th second. Find
(1) Initial velocity (u)
(2) Acceleration (a)
(3) Displacement till 7 s
P hysics | 2.9

(4) Velocity after 7 s


(5) Displacement between t = 6 s and t = 8 s (JEE MAIN)

Sol: We know the formula for displacement in nth second. For 5th second and 10th second we get two equations
and two variables u and a. So we solve the equations to get the values of u and a.
We know that displacement in nth second (xnth)
a
xnth =
u + (2n − 1)  … (i)
2
a
Given x5th = 50 ; ⇒ u + [2x5 − 1] =50
2
9
⇒u + a =
50  … (ii)
2
a
and X10th = 100 ; ⇒ u [2x10 − 1] =
100
2
19
⇒ u+ a =
100  … (iii)
2
 19 9 
Subtracting (ii) from (i), we get  −  a =100 − 50 ⇒ a =10m / sec2
 2 2 
9
Substituting the value of a in (i) we get, u + (10) = 50 ⇒ u = 50 − 45 = 5m / s
2
1
x ut + at2
We know =
2
1
At t= 7 sec
= x (5)(7) + (10)(7)2 =35 + 245 = 280 m
2
We know v = u + at

∴ At t = 7sec ⇒ v = (5) + (10)(7) = 75 m/sec

 1   1  1 1
= x8 − x6 =  ut + at2 
X 6 sec to 8 sec −  ut + at2  = u ( 8 − 6 ) + a  82 − 62  =
5(2) + x10(28) =
150m
 2 at t 2 2   2
= sec 
8= at t 6 sec

Illustration 12: Consider a particle moving in a straight line with constant acceleration, has a velocity ( v p ) = 7 m/s
and VQ = 17 m/s, when it crosses the point P and Q respectively. Find the speed of the particle at mid-point of PQ.
 (JEE ADVANCED)
7m/sec 17m/sec
P Q
Figure 2.13

Sol: Initial and final velocity are known for constant acceleration and a particular displacement. The final velocity
for half the displacement is to be calculated. This problem can be easily solved by using the third equation of
motion with constant acceleration.
Let the mid-point be R  VP=7m/sec V=VR VQ=17m/sec
Then PR=RQ = s (say) R
S S
From PR From RQ P Q

u=vp=7 m/s u=VR Figure 2.14

V=VR V=17 m/s


2 . 1 0 | Motion in a Straight Line

x=s x=s
Using formula v2= u2 + 2ax
We get: VR2 = 72 + 2as ... (i)
And 172 = VR2 + 2as  ... (ii)
Subtracting (i) from (ii) we get 17 - VR = VR - 7 ⇒ 2VR = 17 + 7
2 2 2 2 2 2 2

⇒ 2VR2 = 338 ∴ VR =
13m / sec

Illustration 13: A body moving with uniform acceleration covers 24 m in the 4th second and 36 m in the 6th
second. Calculate the acceleration and initial velocity.  (JEE MAIN)

Sol: We know the formula for displacement in nth second. For 4th second and 6th second we get two equations
and two variables u and a. So we solve the equations to get the values of u and a.
a
Sn =
u + (2n − 1)
2
a
∴ 24 =u + (2x4 − 1)  … (i)
2
a
u + (6x2 − 1) 
36 = … (ii)
2
12
From equation (i) and (ii) we get 12 =
2a ⇒ a == 6m / s2
2
6
From equation (i) 24 =
u + (2x4 − 1) ⇒ u = 3 ms-1
2
Observation: Motion is independent of the mass of the body and hence no equation of motion considers mass.

PLANCESS CONCEPTS

•• For uniformly accelerated motion i.e. constant acceleration:


Average velocity {Vavg } = (v + u)/ 2
Proof: Vavg =Displacement (s)/time interval (t) = s/t
= (ut + ½ at2) / t
= u + ½ at
= (2u + at)/2
= [u + (u+at)]/2
= (u + v)/2

•• If initial vector of a particle is ro , then position vector at time t can be written as

     1  
r = ro + s = ro +  ut + at2 
 2 

•• Difference between distance (d) and displacement (s)


From equations of motion
1
ut + at2 and v 2 =
s= u2 + 2as
2
P hysi cs | 2.11

s is the displacement and not the distance of the particle. The values are different when u and a are
of opposite sign or u ↑↓ a.
Case 1: When velocity u is either zero or parallel to a, then motion is simply accelerated and in this case
1
distance is equal to displacement. So, we can write, d= s= ut + at2 .
2
Case 2: When u is not parallel to a, the motion is first retarded and then accelerated in opposite direction.
Hence distance is either greater than or equal to displacement ( d ≥s).

Nitin Chandrol (JEE 2012 AIR 134)

Illustration 14: Consider an object moving with an initial velocity of 10 m/s and u = 10 m/sec
acceleration of 2 m/s2 . Find distance travelled from t = 0 to 6 s. (JEE MAIN)
a = 2m/s²
Sol: Distance covered is equal to displacement if the object moves in a straight
line and there is no change in direction of motion. If direction of motion Figure 2.15
changes, distance should be calculated separately for different parts of the
path.
t=0 t=6 sec t=5 sec
2
1
 V= u + at ; ⇒ 0= 10 + ( −2)t ; ⇒ t =5sec ; x=
5 ut + at
2
1 1
= (10)(5) + ( −2)(5)2 ; = 25m and =x6 (10)(6) + ( −2)(6)=2
24m x=0 x=24m x=25m
2 2
Figure 2.16
∴ Total distance travelled = 25 + (25-24) = 26 m.

Illustration 15: Consider a body moving with velocity 9 m/s. It is subjected to acceleration of -2 m/s2. Calculate the
distance travelled by the body in fifth second.  (JEE ADVANCED)

Sol: Distance covered is equal to displacement if the object moves in a straight line and there is no change in
direction of motion. If direction of motion changes, distance should be calculated separately for different parts of
the path.
Advice: Distance travelled in 5th sec need to be calculated and not the displacement.
Hence displacement formula cannot be used directly to calculate the distance in nth second.
According to equations of motion,  t=5
a 2
Sn =
u + (2t − 1) =
9 − (2x5 − 1) = 9 – 9 = 0 t=0 t=1 t=2 t=3 t=4 t=4.5
2 2
The value obtained is for displacement and not distance. Figure 2.17

Hence distance S in 5th sec. can be calculated as S = 2

 1   1 
(S4.5 – S=
4
) 2  9x4.5 − x2x(4.5)2  − 9x4 − x2(4)2 
 2   2 
= 2 [4.5-4.25] = 2 x 0.25 = 0.5 m

3. MOTION OF BODY UNDER GRAVITY (FREE FALL)


The force of attraction that the earth exerts on all the bodies is called force of gravity and the acceleration induced
by gravity is called acceleration due to gravity, represented by g. All bodies irrespective of their size, weight or
composition fall with the same acceleration near the surface of earth in the absence of air. Motion of a body falling
towards the earth from a small altitude (h<<R) is known as free fall (R: Radius of Earth).
2 . 1 2 | Motion in a Straight Line

3.1 Body Projected Vertically Upwards


(a) Equation of motion: Considering point of projection as origin
v=0
and direction of motion (i.e. vertically up) as positive
a = - g [as acceleration is downwards]
If a body is projected with velocity u and after time t
h
it reaches to a height h then
v = u – gt  ... (i)
h = ut – ½ gt2  … (ii)
v = u -2gh 
2 2
... (iii)
(b) For maximum height (H): v =0

u2 h=
Using equation (ii) we get 0 = u – 2gH ⇒ H = (c) Time
2 2 2g
taken to reach maximum height (t): v=0 2g

Using equation (ii) we get: 0 = u-gt; T = u/g.


(c) Time of flight (T) is the time during which the object travels. Figure 2.18
In this case, it is the time between the the maximum height
and the ground.
1 1
Thus, h = 0. Using equation (iii): = ; 0 T (u − gt)
0 uT − gt2 =
2 2
⇒ either T=0 or T=2u/g = 2 x. Time taken to reach maximum height (t)
(d) The following graphs show the displacement, velocity and acceleration with respect to time (for maximum
height) when body is thrown upwards:

s a
(u²/2g) v
+ +
(u/g)(2u/g)
O O
t t
(u/g) - g
-v -a
t

Figure 2.19

Observation:
(a) Time taken by the body to travel up is equal to the time taken by the body to fall down. Time of descent (t2)
= time of ascent (t1) = u/g
(b) The speed with which a body is projected up is equal to the speed with which it comes down. The magnitude
of velocity at any point is same whether the body is moving up or down.

Illustration 16: Consider a ball being thrown upwards with an initial speed of u. Find out u if the ball is at a height
of 80 m and the interval between two times is 6 s. (g=10 m/s2) (JEE MAIN)
Sol: Body thrown vertically upwards reaches the maximum height, stops momentarily and then starts falling
vertically downwards. So for any point at height less than the maximum height, the body will reach the point twice
during its travel, first time while ascending and the second time while descending.
u = u m/s, a=g= -10 m/s2 and s=80 m
1
s ut + at2 , we have
Substituting the value, = 80 = ut – st2 or st2 – ut + 80 = 0
2
P hysi cs | 2.13

u + u2 − 1600 u − u2 − 1600 s=80m


Or, t = and
10 10
+ve
2 2
u + u − 1600 u − u − 1600
Given that − 6
=
10 10
-ve U
u2 − 1600 2
= 6 or u − 1600
= 30
5
Or u2 – 1600 = 900; ∴ u2 =2500 ; Or u = ± 50 m/s Figure 2.20
Ignoring the negative sign, u =50 m/s.

3.2 Body Dropped From Some Height (Initial Velocity Zero)


(a) Equations of motion: Let the initial position be the origin and
direction of motion (i.e. downward direction) to be positive, then u=0
u = 0 [As body starts from rest]
a = g [As acceleration is in the direction of motion] 2h
T= g
Therefore, equations of motion are:
v= u + gt  …(i)
T= 2gh
1 H
v= ut + gt2  …(ii)
2 V²
2 2 T= 2g
v= u + 2gh  …(iii)

(b) Velocity (v) of the particle just before hitting the ground: h = H
Therefore, using equation (iii): V2 = 0 + 2gH ; v = 2gH (c) Time (t)
taken by the object to reach the ground: h = H
1 2H V
H (0) T + gT2 ; T =
Therefore, using equation (ii):=
2 g

Figure 2.21

(c) The following graphs show the distance, velocity and acceleration with respect to time (for free fall):

s v O

g
tan=g

t t t
Figure 2.22

1
( e) The distance covered
(d) covered in
in the
the nth
nth sec,
sec,h
= n
2
g(2n − 1)
Hence the ratio of the distance covered in 1st, 2nd, 3rd sec, etc. is 1:3:5 i.e. only odd integers.
These results obtained are the corollary of the Galileo’s Theorem:
For a uniform accelerating body, the distance travelled is always odd ratio, i.e. 1:3:5:7, for regular time interval.
(e) As h = (1/2) gt2, i.e. h α t2, distance covered in time t, 2t, 3t, etc., will be in the ratio of 12:22:32, i.e. square of
integers.
2 . 1 4 | Motion in a Straight Line

3.3 Body Thrown Vertically from a Height


There are two possibilities when an object is thrown. g
s=0
First, when an object is thrown in upward direction, velocity is upwards whereas acceleration acts
downwards, i.e. they are in opposite directions. Hence initially the object undergoes retardation
and rises through a certain height and then it undergoes free fall from that height. h
Second, when an object is thrown from a certain height, both the velocity and acceleration are
in the downward direction, i.e. velocity and acceleration are in the same direction. In this case,
the object undergoes acceleration. It hits the ground with a speed greater than the speed if it Figure 2.23
had gone through free fall.
Equations of motion are used in both the cases.
Assume a sign convention for the direction.
Then, note the values of displacement, velocity and acceleration with appropriate signs.
Finally, use the appropriate equations of motion.

Illustration 17: A ball is thrown upwards from 40 m high tower with a velocity of 10 m/s. Calculate the time when
it strikes the ground. (g = 10 m/s2) (JEE MAIN)

Sol: In the second equation of motion with constant acceleration, value of all the quantities need to be substituted
with proper sign. If the displacement and acceleration are in the opposite direction of initial velocity (taken as
positive) then substitute there values with negative sign. 
u = + 10 m/s, a = - 10 m/s2 
+ve u=+10m/s
a=g=-10m/s²
s = -40 m (at the point where the ball strikes the ground) s=0
1
Substituting in
Substituting s = ut + at2 ,,we
in S= wehave
have -40=10t – 5t2
2
40m
or 5t2 -10t -40 = 0 or t2-2t-8=0
Solving this, we get t = 4 s and -2 s. Considering the positive value, t=4 s.
Figure 2.24
CC
t=1s
tAB=tDE=2s
t =t =1s
t=0 B D EC CD
t=2s

t=-2s A
E t=4s
Figure 2.25

Note: The significance of t=-2 s can be understood by Fig. 2.25:

PLANCESS CONCEPTS

We have studied the formula of maximum height (H) and time taken (T) to reach the point where the
velocity of an object becomes zero under gravity.
The retardation of the object is ‘g’. H = u2/2g and T = u/g.
This can be used for an object having an initial velocity ‘u’ and retardation ‘a’.
P hysi cs | 2.15

Thus, distance at which velocity of the particle becomes zero H= u2/2a;


Distance after which the particle changes its direction;
Time taken to reach this distance = u/a
Time taken to reach its initial position = 2u/a
Anand K (JEE 2011 AIR 47)

4. NON-UNIFORMLY ACCELERATED MOTION


Equations of motion cannot be considered for particles travelling with constant or uniform acceleration. While
deriving equations of motion, we considered acceleration to be constant. So, for solving non-uniformly accelerated
  
 ds  dr  dv
motion, we will follow the two basic
= equations: (i) v = or sometimes v (ii) a =
dt dt dt
Vector quantity is not required for one-dimensional motion. Therefore the above equations can be re-written as:

(i) v = ds
dt
(ii)=
a
dv
= v
dt
dv
ds

PLANCESS CONCEPTS

dv
a= v is useful when acceleration displacement is known and velocity displacement is required.
ds
Yashwanth Sandupatla (JEE 2012, AIR 821)

Illustration 18: For a particle moving along x-axis, displacement time equation is x = 20 + t3 - 12t.
(a) Find the position and velocity of the particle at time t = 0
(b) Find out whether the motion is uniformly accelerated or not.
(c) Find out the position of particle when velocity is zero. (JEE MAIN)

Sol: Displacement is given as a function of time. Differentiating the equation of displacement w.r.t time we get the
velocity as a function of time. Differentiating the equation of velocity w.r.t time we get the acceleration as a function
of time.
(a) x = 20 + t3 - 12t  ... (i)
At t = 0, x = 20 + 0 – 0 = 20 m
dx
By differentiating Equation (i) w.r.t. time i.e. =
v = 3t2 − 12  … (ii)
dt
Velocity of particle can be obtained at time t.
At t = 0, v = 0 -12 =- 12 m/s
dv
(b) Differentiating equation (ii) w.r.t. time, we get the acceleration=
a = 6t
dt
As acceleration is a function of time, the motion is non-uniformly accelerated.
(c) Substituting v=0 in equation (ii) 0=3t2 – 12
From the above equation, t = 2 sec. Substituting it in equation (i) we have x=20 + (2)3 - 12(2) or x=4 m
2 . 1 6 | Motion in a Straight Line

Illustration 19: Acceleration of an object moving in straight line is a=v2 and initial velocity of that object is u m/sec
Find. (i) v(x) i.e. velocity as a function of displacement (ii) v(t) i.e. velocity as a function of time  (JEE ADVANCED)

Sol: Acceleration is the differentiation of velocity with respect to time. We can use the following transformation:
dv dv dx dv
a
= = = v
dt dx dt dx
v x
dv dv dv v x v
a v= v 2 ; ⇒ dx Integration ⇒ ∫
v o∫
(i)
= = = dx ⇒ nv u =x o ; nv − nu =
x ; ⇒ n x ; ∴ v= u e x
=
dx v u
u
v t
dv dv dv 1 v t 1 1 1 1 1 − ut u
(ii)=
a = v2 ; = dt Integration ⇒ ∫ ∫
= dt ; ⇒ − u =t o ⇒ −  −  =t ; ⇒ =−t + = ;∴ v =
dt v 2
uv
2
o
v  v u v u u 1 − ut

5. ANALYSIS OF MOTION THROUGH GRAPHS


Let us see some basics points of graphs before analyzing the motion of an object through graphs.

Basic Graphs: (a) A linear relationship between x and y represents a straight line. y

E.g., y=4x-2, y=5x + 3, 3x = y-2


(b) A proportionality relationship between x and y (i.e. x ∝ y or y = kx ) represents a
straight line passing through origin.
 1
(c) Inverse proportionality  x ∝  or xy=k represents a rectangular hyperbola. x
 y
Shape of a rectangular hyperbola is given in the graph: Figure 2.26

(d) A quadratic equation in x and y represents a parabola in x-y graph. E.g., y=3x2 +2,
y2=4x, x2=y-2

dy
Analysis of Graphs: (a) If z = , the value of z at any point on x-y graph can be obtained by the slope of the graph
at that point. dx

(b) If z = y(dx) or x(dy), the value of z between x1 and x2 or y1 and y2 is obtained by the area of graph between x1
and x2 or y1 and y2.

5.1 Displacement–Time Graph


With displacement of a body plotted on y-axis against time on x-axis, displacement–time curve is obtained.
(a) The slope of the tangent at any point of time gives the instantaneous velocity at any given instant.
(b) At a uniform motion the displacement–time graph is a straight line.
(i) If the graph obtained is parallel to time axis, the velocity is zero.
(ii) If the graph is an oblique line, the velocity is constant (OC and EF in Fig. 2.28).
y
y velocity constant(a=0) a v=0 b
Displacement

A B gc
a decreasing g c
decreasing
f
velocity d e
t
an
nt

v=constant
st
ta

n
ns

co
co

Retardation(a)
v=
a

constant S increasing f
a increasing ° velocity
Time(t) e x O t x
° Time
Figure 2.27 Figure 2.28
P hysi cs | 2.17

Illustration 20: Displacement–time graph of a particle moving in a straight line is shown in the Fig. 2.30. State
whether the motion is accelerated or not. Describe the motion of the particle in detail. Given s0=20 m and t0=4 s.
 V=5 m/s (JEE MAIN)

+ve
S=-20 m S=0
t=0
Figure 2.29

Sol: The velocity of the particle at any instant is the slope of the displacement time graph at S
that instant. If the slope is constant, velocity is constant.
Slope s–t is a straight line. Hence, velocity of particle is constant. At time t = 0, displacement t
of the particle from its mean position is –s0 i.e. -20 m. Velocity of particle, t
°
s0 20 -S
V slope
= = =
t0
= 5m / s
4
°
Figure 2.30
At t = 0 particle is at –20 m and has a constant velocity of 5 m/s. At t0 = 4 sec, particle will
pass through its mean position.

5.2 Velocity–Time Graph


Displacement, velocity and acceleration, specifying the entire motion, can be determined by the velocity–time
curve.
(a) Instantaneous acceleration can be obtained by the slope of the tangent at any point corresponding to a
particular time on the curve.
(b) Displacement during a time interval is obtained from the area enclosed by velocity–time graph and time axis
for a time interval.
(c) For a uniformly accelerated motion, velocity–time graph is a straight line.
(d) For constant velocity (i.e. acceleration is zero), the graph obtained is a straight line AB parallel to x-axis (time).
(e) For constant acceleration, the graph obtained is oblique.

5.3 Acceleration–Time Graph


(a) Change in velocity for a given time interval is the area enclosed between acceleration–time graph and time axis.
(b) For constant acceleration, the obtained graph is a straight line parallel to x-axis, i.e. time (t).
(c) If the acceleration is non-uniform, then the graph is oblique.

Inference: Displacement–time graph for uniformly accelerated or retarded motion is a parabola. Since, for constant
1
acceleration, then relation between displacement and time is: =s ut ± at2 which is quadratic in nature. Thus,
2
displacement–time graph will be parabolic in nature.

Illustration 21: Acceleration–time graph of a particle moving in a straight line is shown in Fig. 2.31. At time t=0,
velocity of the particle is 2 m/s. Find velocity at the end of the 4th second.  (JEE MAIN)

Sol: The area enclosed by the acceleration-time graph between t = 0 and t = 4s will give a(m/s²)
the change in velocity in this time interval.
4
dv = a dt
or change in velocity = area under a-t graph
t(s)
1 O 2 4
Hence v f −
= vi = 8m / s ; ∴ v f = v i + 8 = (2 + 8)m / s = 10m / s
(4)(4)
2 Figure 2.31
2 . 1 8 | Motion in a Straight Line

Various Position–time graphs and their interpretation

X θ = 0º, so v = 0

i.e. line parallel to time axis represents that the particle is at rest

O T
Figure 2.32

X θ = 90º,so v = ∞

i.e. line perpendicular to time axis represents that particle is changing its position
with constant time. Hence, particle possesses infinite velocity (which is not
O T possible practically).
Figure 2.33

X θ = constant, so v = constant, a=0

i.e. line with constant slope represents uniform velocity of the particle.

O T
Figure 2.34

X θ is increasing, so v is increasing and a is positive.

i.e. line bending towards position axis x represents increase in velocity of particle.
Hence, the particle possesses acceleration.
O T
Figure 2.35

X θ is decreasing, so v is decreasing and a is negative

i.e. line bending towards time axis t represents decreasing velocity of the particle.
Hence, the particle possesses retardation.
O T
Figure 2.36

X θ is constant but >90º, so v will be constant but negative

i.e. line with negative slope represents that particle returns to the point of reference
 (i.e. negative displacement).
O T
Figure 2.37

X Straight line segments of different slopes represent that velocity of the body is
different for different intervals of time.
A B C
O T
S
Figure 2.38
P hysi cs | 2.19

x At one point the particle has two positions, which is not possible.

T
O

Figure 2.39

x Particle moves towards origin initially and after that moves away from origin.

T
O

Figure 2.40

Various Velocity–time graphs and their interpretation

θ = 0º, a = 0, v = constant
Velocity

i.e. line parallel to time axis represents that the particle is moving with constant
O velocity.
Time
Figure 2.41

θ = 90º, a = ∞, v is increasing
Velocity

i.e. line perpendicular to time axis represents that particle is increasing in velocity,
O but there is no change in time. Hence the particle possesses infinite acceleration
Time (which is not possible practically).

Figure 2.42

θ =constant, so a = constant and v is increasing uniformly with time


Velocity

i.e. the slope represents uniform acceleration of the particle.

O
Time
Figure 2.43

θ is increasing, so acceleration is increasing


Velocity

i.e. line bending towards velocity axis represents increasing acceleration of the
body.
O
Time
Figure 2.44

θ is decreasing, so acceleration is decreasing


Velocity

i.e. line bending towards time axis represents the decreasing acceleration in the
body.
O
Time
Figure 2.45
2 . 2 0 | Motion in a Straight Line

θ is constant and <90º

Velocity
i.e. acceleration is constant and positive but initial velocity of the particle is
O negative.
Time
Figure 2.46

θ is constant and <90º


Velocity

i.e. acceleration is constant and positive but initial velocity of the particle is positive.
O
Time
Figure 2.47

θ is constant and >90º


Velocity

i.e. acceleration is constant and negative but initial velocity of the particle is

O positive.
Time
Figure 2.48

θ is constant and >90º


Velocity

i.e. acceleration is constant and negative but initial velocity of the particle is zero.

O
Time
Figure 2.49

θ is constant and >90º

i.e. acceleration is constant and negative but initial velocity of the particle is
Velocity

O
negative.

Time

Figure 2.50

PLANCESS CONCEPTS

The following are the lists of motions that are not possible v s
practically:
•• Slopes of v-t or s-t graphs can never be infinite at any
point, because infinite slope of v-t graph means infinite
t t
acceleration. Similarly, infinite slope of s-t graph means
infinite velocity. Hence, the graphs shown here are not
possible. v s
•• At a particular time, two values of velocities v1 and v2 v₁ s₁
or displacements S1 and S2 are not possible. Hence, the
v₂ s₂
following graphs shown here are not possible.
t t
t₀ t₀
GV Abhinav (JEE 2012 AIR 329)
                              Figure 2.51
P hysi cs | 2.21

Illustration 22: At t = 0, a particle is at rest at origin. For the first 3 s the acceleration is 2 ms-2 and for the next 3 s
acceleration is -2 ms-2. Find the acceleration versus time, velocity versus time and position versus time graphs. 
 (JEE ADVANCED)
Sol: The area enclosed by the acceleration-time graph and the time-axis Y
between t = 0 and t = t gives the change in velocity in this time interval. 2m/s²
Similarly the area enclosed by the velocity-time graph and the time-axis a(m/s²)
between t = 0 and t = t will give the change in displacement in this time
interval. O X
3 6 t(s)
Given for the first 3 s acceleration is 2 ms-2 and for next 3 s acceleration is
-2 ms-2. Hence acceleration–time graph is as shown in the Fig. 2.52. -2m/s²
The area enclosed between a–t curve and t-axis gives change in velocity for Figure 2.52
the corresponding interval. Also at t=0, v=0, hence final velocity at t=3 s
will increase to 6 ms-1. In next 3 s the velocity will decrease to zero. Hence
the velocity–time graph is as shown in figure.
Note that due to constant acceleration v-t curves are taken as straight line. 6
5
Now for x-t curve, we will use the fact that area enclosed between v-t curve v(m/s) 4
3
and time axis gives displacement for the corresponding interval. Hence 2
displacement in the first 3 s is 4.5 m and in next 3 s is 4.5 m. Also the x–t 1 1 2 3 4
curve will be of parabolic nature as the motion has a constant acceleration. 0
Therefore, x–t curve is as shown in figure. t(sec.)
Y
x(m) Figure 2.53
9

4.5

O X
3 6 t(s)

Figure 2.54

Illustration 23: The graph in the Fig. 2.55 shows the velocity of a body plotted as a function of time.
(a) Find the instantaneous acceleration at t = 3 s, 7 s, 10 s, and 13 s.
(b) Find the distance travelled by the body in the first 5 s, 9 s, and 14 s
(c) Find the total distance covered by the body during motion.
(d) Find the average velocity of the body during motion.
 (JEE ADVANCED)
C
45
40
35
30
V(m/s)

25 A B
20
15 D
10 J
5 E F G H
O
1 2 3 4 5 6 7 8 9 10 11 12 13 14 15 16
t(sec.)

Figure 2.55

Sol: The area enclosed by the acceleration-time graph and the time-axis between t = 0 and t = t gives the change
in velocity in this time interval. Similarly the area enclosed by the velocity-time graph and the time-axis between
2 . 2 2 | Motion in a Straight Line

t = 0 and t = t will give the change in displacement in this time interval.


(a) Acceleration at t = 3 s
When the particle travels from point A to B for the first 5 s, the body moves with a constant velocity. Hence, the
acceleration is zero.
Acceleration at t = 7 s.
When the particle travels from point B to C for the interval 5 to 9 s, the acceleration is uniform.
45 − 20
=a = 6.25m / s2
(9 − 5)
Hence acceleration at t=7 s is 6.25 m/s2. The acceleration at t = 10 s and 13 s are respectively -10 m/s2 and -3.75
m.s2.
(b) The distance covered in t seconds is the area enclosed by the curve in t seconds on velocity–time graph. The
distance covered by the body in 5 s = Area of rectangle ABEO = 20 x 5 = 100 m
The distance covered in first 9 s = The area of the figure ABCFO = Area ABEO + Area EBCF
1
=100 + (20 + 45) x 4 = 100 + 130 =230 m.
2
The distance covered by the body in first 14 s = Area [(ABCFO) + (CDGF) + DJHG)]
1 1
=230 + (45 + 15) x 3 + (15 + 7.5) x2 = 230 + 90 +22.5 = 342.5 m.
2 2
1
342.5 + 7.5 x2 =
(c) The distance covered by the body during the entire motion = 350 m / s.
2
350
(d) Average velocity for the =
motion = m / s 21.9m / s.
16

6. RELATIVE MOTION
(a) Motion of an object is dependent on observation.
(b) Motion is a relative term.
(c) An observation of motion is always with respect to frame of reference.

6.1 Types of Frames of Reference


(a) Inertial frames of reference: It is defined as the frame of reference with uniform velocity (both in magnitude
 
and direction). Thus, acceleration is also zero, i.e. v = cons tant and a = 0 .
(b) Non-Inertial frames of reference: It is defined as the frames of reference with non-uniform velocity (either

magnitude or direction is not constant). Thus, acceleration is also non-zero, i.e. a ≠ 0 .

6.2 Relative Velocity (Introduction to motion in 2D)


VBA
Consider two bodies A and B travelling with velocities VAO and VBO, respectively, with respect A B
to origin O, then the relative velocity of B with respect to an observer A, VBA, is given as
VAO
follows: VBO
VBA = VBO - VAO O

Similarly, VAB = VAO – VBO Figure 2.56

Thus the relative velocity of any two bodies moving from the same origin is equal to the vector difference of their
velocities.
P hysi cs | 2.23

The relative rate of change of VBA gives relative acceleration of B with respect to A and is given by aBA = aBO - aAO
and aAB = aAO - aBA
Fact: Distance between two objects with respect to is independent of the reference frame.
If ‘x’ is the minimum distance between the two objects at time ‘t’ then in any frame of reference the minimum
distance of the objects remains constant at time ‘t’.

PLANCESS CONCEPTS

To find the relative velocity of an object A (say) w.r.t to object B (say), inverse (change the direction of)
the velocity vector of object B and then add it to velocity of A.
Anurag Saraf (JEE 2011 AIR 226)

Illustration 24: A man whose velocity in still water is 5 m/s swims from point A to B (100 m down-stream of A)
and back to point A. Velocity of the river is 3 m/s. Find the time taken in going down-stream and upstream and the
average speed of the man during the motion.  (JEE MAIN)

Sol: The velocity of man in ground frame is the vector sum of the velocity of river and the velocity of man in river
frame. In going down-stream, the magnitude of velocity of river and the magnitude of velocity of man in river
frame are added to get the magnitude of velocity of man in ground frame. In going up-stream, the magnitude of
velocity of river is subtracted from the magnitude of velocity of man in river frame to get the magnitude of velocity
of man in ground frame.
  
During down-stream, velocity of the man = Vm = Vmw + Vw = 3 + 5 = 8 m / s Time taken during down-stream = 100/8
=12.5 s
  
During upstream, velocity of the man = V= m ' Vmw + Vw = - 5 + 3 = - 2 m/s.

Time taken during upstream = 100/2 = 50 s


200
Average speed = = 3.2 m/ s
62.5

Illustration 25: Yashwant started moving with constant speed 10 m/s to catch the bus. When he was 40 m away
from the bus, it started moving away from him with acceleration of 2 m/s2. Find whether Yashwant catches the bus
or not. If yes, at what time he catches the bus. If no, then find the minimum distance between the bus and him.
 (JEE MAIN)
u=0m/s
10m/sec a=2m/s²

Bus
40 m

w.r.t bus
u=10m/s u=0m/s
a=0m/s²

a=2m/s²
Bus
40 m

Figure 2.57
2 . 2 4 | Motion in a Straight Line

Sol: This problem is best solved in the reference frame of bus. In this frame the initial velocity of Yashwant is
towards the bus (assumed positive) and the acceleration of Yashwant is in the opposite direction i.e. away from the
bus (assumed negative).
Yashwant moves with the initial velocity of 10 m/s and acceleration of 2 m/s2.
Assuming Yashwant never catches the bus, let us find the distance at which his velocity becomes zero.
We know v2 = u2 + 2as
100
s
Here v = 0; u = 10 m/s; a = -2 m/s2 ; ∴ 02 = (10)2 + 2(-2) s ⇒= = 25m
4
Our assumption was right, since Yashwant travels only 25 m in the positive direction in bus reference frame.
So, minimum distance = 40 – 25 = 15 m.

6.3 Applications of relative velocity


Relative motion is widely used in two- and three-dimensional motions. The four types of problems arising based
on relative motion are as follows:
(a) Problems on minimum distance between two bodies in motion (b) River–boat problems
(c) Aircraft–wind problems (d) Rain problems

(a) Minimum distance between two bodies in motion: Minimum distance between two moving bodies or the
time taken when one body overtakes the other can be solved easily by the principle of relative motion. Here we
consider one body to be at rest and other body to be in relative motion of the other body. By combining two
problems into one, the solution becomes easy. Following examples will illustrate the statement.

Illustration 26: Car A and car B start moving simultaneously in the same direction along the line joining them.
Car A moves with a constant acceleration a = 4 m/s2, while car B moves with a constant velocity v = 1 m/s. At time
t = 0, car A is 10 m behind car B. Find the time when car A overtakes car B. (JEE MAIN)

Sol: This problem is best solved in the reference frame of any one of the two cars (say car B). In this frame the
initial velocity of car A is in the direction away from car B (assumed negative) and the acceleration of car A is in the
direction towards the car B (assumed positive).
Given: uA=0, uB=1m/s, aA = 4 m/s2 and aB = 0
Assuming car B to be at rest, we have
uAB = uA – uB = 0 – 1 =- 1m/s
aAB = aA – aB = 4 – 0 = 4m/s2
Now, the problem can be solved easily as follows:
Substituting the proper values in equation

a=4m/s² v=0m/s

A 10 m B

+ve

Figure 2.58

1 1 1 ± 1 + 80 1 ± 81
s ut + at2 , we get 10 =−t + (4)(t2 ) or, 2t2 − t − 10
= = =0 or, t =
2 2 4 4
P hysi cs | 2.25

a=4m/s² v=1m/s

A 10 m B

+ve

Figure 2.59

1±9
= = or t 2.5 s and − 2 s Ignoring the negative value, the desired time is 2.5 s.
4
B
(b) River–boat problem
y
We come across the following three terms: vbr w
 
v = absolute velocity of river x
r
v br = velocity of boatman with respect to A
 Figure 2.60
river or velocity of boatman in still water and v b = absolute velocity of
boatman
 
Here, it is important to note that v br is the velocity of boatman with which he steers and v b is the actual velocity
of boatman relative to ground.
  
Further, v= b v br + v r Now, let us derive some standard results and their special cases.

A boatman starts from point A on one bank of a river with velocity v br in the direction shown in Fig. 2.60.

River is flowing along positive x-direction with velocity v r .
Width of the river is w, then
Therefore, vbx = vrx + vbrx = vr – vbr sinθ and vby = vry + vbry = 0 + vbr cos θ
Now, time taken by the boatman to cross the river is:
w
t=
v br cos θ  … (i)

Further, displacement along x-axis when he reaches on the other bank (also called drift) is:
w
x=
v bx t =
(v r − v br sin θ)
v br cos θ
w
x=
(v r − v br sin θ)  ... (ii)
v br cos θ
Or the three special cases are:

(i) Condition when the boatman crosses the river in shortest interval of time
From Equation (i) we can see that time (t) will be minimum when θ = 0º, i.e. the boatman should steer his boat
perpendicular to the river current. 
B
w
Also, tmim
= = as cos θ 1
v br
vbr
(ii) Condition when the boatman wants to reach point B,
i.e. at a point just opposite from where he started A vr
In this case, the drift (x) should be zero. Therefore x = 0
Figure 2.61
w
Or (v r − v br sin θ) 0 or vr = vbr sin θ
=
v br cos θ
2 . 2 6 | Motion in a Straight Line

 vr   v  B
vr
 sin θ
 or = = or θ sin −1 r  
v
 v br  v br  br 
vbr
Hence, to reach point B, the boatman should row at an angle θ = sin-1 upstream 
from AB. Further, since sin θ ≤ 1.
A vr
So, if vr ≥ vbr, the boatman can never reach point B. If vr = vbr, sin θ =1 or 90º it
Figure 2.62
is quite impossible to reach B if θ =90º. Moreover it can be seen that vby = 0 if
vr = vbr and θ = 90º. Similarly, if vr ≥ vbr, sin θ > 1 i.e. no such angle exists. Practically it is not possible to reach B if
river velocity (vr) is too high.

(iii) Shortest path


Distance travelled by the boatman when he reaches the opposite shore =
is s w2 + x2
Here, w = width of river which is constant. For s to be minimum, modulus of x (drift) should be minimum. Now two
cases are possible

 v   v 
When vr < vbr: In this case x = 0, when θ = sin-1  r  or s= w at θ sin−1  r 
v mim v
 br   br 

 dx  d  w 
When vr > vbr: In this case x is minimum, where   = 0 ;  (vr − vbr sin θ) = 0
 dθ  dθ  vbr cos θ 
v 
or − v br + v r sin θ =0 or θ =sin−1  br 
 vr 
Now, at this angle we can find xmin and then smin

 vr  v 
=smin w =  at θ sin −1 br 
v 
 br   vr 

Illustration 27: A man rows a boat at 4 km/h in still water. If he is crossing a river with a 2 km/h current
(a) What will be the direction of the boat, if he wants to reach a point directly opposite the starting point on the
other bank?
(b) With these conditions, how much time it will take for him to cross the river, given the width of river is 4 km?
(c) What will be the minimum time and what direction should he head to cross the river in shortest time?
(d) If he wants to row 2 km up the stream and back to the origin, what will be the time required? (JEE MAIN)

Sol: The velocity of boat in ground frame is the vector sum of the velocity of river and the velocity of boat in river
frame. If the boat heads in the direction perpendicular to the direction of river flow in the frame of the river, it will
cross the river in shortest time. But in doing so it will get drifted in the direction of the river flow as well, and thus
will not reach the other bank directly opposite to the starting point.
(a) Given, that vbr = 4 km/h and vr = 2 km/h ; 

1  vr  −1  2  −1  1 
vbr-vr vbr+vr
∴ θ sin−=
=   sin
=   sin
= 30o0
  =30
v
 br  4
  2
  D C D C
To reach the point directly opposite to starting point, the boat
Figure 2.63(a)
should head at an angle of 30º with AB or 90º + 30º = 120º with
the river flow.
P hysi cs | 2.27

(b) Time taken to cross the river w = width of river = 4 km

4 2
= 30º ; t
vbr = 4 km/h and θ= = h
4 cos30o 3
w 4
(c) For shortest time θ = 0º and tmin= = = 1h
v br cos30 o 4

Hence, he should incline his boat perpendicular to the current for crossing the river in shortest time of 1 h.
CD DC 2 2 1 4
(d) t = tCD + tDC
= or t + = + =1 + = h
vbr − v vbr + vr 4−2 4+2 3 3
r
 
(c) Aircraft wind problem: The only difference between this and the river boat is that v br is replaced by v aw
 
(velocity of aircraft with respect to wind or velocity of aircraft in still air), v r is replaced by v w (velocity of wind)
  
and v b is replaced v a (absolute velocity of aircraft) . Further, =
v a v aw + v w. Following example will illustrate it.

Illustration 28: An aircraft flies 400 km/h in still air. If 200√2 k/h wind is blowing from the south and the pilot wants
to travel from point A to a point B, north east of A. Find the direction in which the aircraft is to be steered and time
of journey if AB = 100 km. (JEE MAIN)
Sol: The velocity of aircraft in the ground frame is the vector sum of its velocity N
B
in the wind frame and the velocity of the wind. This velocity in ground frame is
along the known direction A to B which is 45o east of north. The direction of va 45° vw=200 km/h
wind is towards north. The direction of velocity in wind frame is un-known which
C
can be found using triangle law of vector addition. 45° 
 A vaw =400 km/h
Give that v w = 200 2km / h vaw = 400 km/h. v a should be along AB or in north-
east direction.
   Figure 2.64
The direction of v aw should be such that the resultant of v w and v aw is along
AB or in north-east direction.

If v aw makes an angle α with AB as shown in Fig. 2.64, then by applying sin law in triangle ABC,

AC BC  BC  o
 200 2  1 1
= or sin α  =
=  sin 45 =  ; ∴ α = 30o
sin 45o sin α  AC   400  2 2
 
Therefore, the pilot should steer in a direction at an angle of (45o + α) or 75o from north towards east.
 
 v a  400 sin105o km
Further, = = ; | vo | × (400)
sin (180° − 45° − 30° ) sin 45° sin 45o h
 cos15o  km  0.9659  km
=  (400) =  (400) 546.47 km / h
 sin 45o  h  0.707  h
 
AB 1000
∴ The time for journey from A to B=
is t = h ; t=1.83 h
| v a | 546.47
^
Vertical (J)
(d) Rain problem: In these type of problems, we again come across three
  
terms v r , v mand v rm , Here

vr ==velocity
velocityof ofrain
rain
 ^
vm = =  vvelocity
elocity of man (itmay
man (it maybebevelocity
velocityofof cyclist
cyclist or or velocity
velocity of motorist
of motorist also )
also) Horizontal (i)

and v rm =
and =  vvelocity
elocity of rain
rain with
withrespect to man
respect to man Figure 2.65

Here, v rm is the velocity of rain which appears to the man. Now, let us take one example of this
2 . 2 8 | Motion in a Straight Line

Illustration 29: Rain appears to fall vertically to a man walking at a rate of 3km/h. At a speed of 6 km/h, it appears
to meet him at an angle of 450 of vertical. Find out the speed of rain.  (JEE MAIN)

Sol: This problem is best solved by using Cartesian coordinates. Take x-axis along the horizontal and y-axis
vertically upwards. The velocity of man is along positive x-axis. The velocity of rain has both horizontal and vertical
components. Express the velocity of man and rain in terms of unit vectors î and ˆj .
Let î and ˆj be the unit vectors in horizontal and vertical directions, respectively.
Velocity of rain
∧ ∧

v=r a i + b j  … (i)
Then the speed of rain will be

|=
vr | a2 + b2  ... (ii)


In the =
first case, v m velocity
= of man 3 i
∧ ∧
  
∴ v rm =v r − v m =(a − 3) i + b j It seems to be in vertical direction. Hence, a – 3 = 0 or a = 3


In the second case v m = 6 i
∧ ∧ ∧ ∧

∴ vm = (a − 6) i + b j =−3 i + b j
This seems to be at 450 of vertical
Hence, |b|=3

Therefore, from Eq. (ii) speed of rain is | vr |= 32 + 32 = 3 2km / h

Alternative Solution:

3kmph 6kmph

Vrm₁ Vr Vrm₁
Vr
45°

90°

Figure 2.66 Figure 2.67

Combining these two we get

6
3 3 3 3 3

Vr  Vr
45° 45° 45°
3 2 3 2

Figure 2.68

| Vr | = 3 2 km/h
P hysi cs | 2.29

PROBLEM-SOLVING TACTICS

To avoid confusion while using signs in equations of motion, the following points need to be considered:
(a) Assuming any one direction to be positive, the other automatically becomes negative.
Generally, vertically up is considered as positive and right side is taken as positive.
(b) Write down the values of velocity, displacement and acceleration according to the sign convention.
(c) On completion of sign convention, then simply use the equations of motions.

FORMULAE SHEET

Position vector of point A with respect to O: 


 ∧ ∧ ∧ r

rA = OA = x A i + y A j + z A k A
rA
  
AB= rs − rA
O X
 ∧ ∧ ∧
AB = (xB − x A ) i + (yB − y A ) j + (zB − z A )k Figure 2.69

For un-accelerated Motion: Distance = Speed x Time


Displacement = Velocity x Time  y

∆s  ∆r A B
=vVavav= Average
= speed
Averagespeed
= ;V vav
av Average
= Velocity rA
∆t ∆t
 rB
∆s ds  ∆r dr x
= v lim = and v lim
= = O
∆t →0 ∆t dt ∆t →0 ∆t dt Figure 2.70

(a) Uniform motion is due to constant Velocity


Average acceleration has the same direction as of change in velocity

 ∆v
aav =
∆t
(b) One-dimensional Uniformly Accelerated Motion
1
• v= ut + at2 and v 2 =+
u + at, s = u2 2as and v avg (v u) / 2
=+
2
u2
(i) Maximum height attained by a particle, thrown upwards from ground with initial velocity u is h =
2g
a
(c) Displacement of particle in tth second of its motion , st =
u + (2t − 1)
2
• Time taken to reach maximum height = u/g
(i) Velocity of a particle (u=0) when it touches the ground when dropped from a height h is,

v = 2gh

(ii) In (b) time of collision with ground t = 2h / g


• If acceleration of particle is not constant, basic equations of velocity and acceleration are used, i.e.
2 . 3 0 | Motion in a Straight Line

  
 ds  dr  dv    
(i) v= or v = (ii) a = (iii) ds = v dt (i v) dv = v dt
dt dt dt
• For one dimensional motion,
ds dv dv
(i)
(i) v = (ii)=
a = v
dt dt ds

(iii) ds ==
v dt and (iv) dv adt
= or vdv ads

dy y
• If z = or , the value of z at any point on x-y graph can be obtained by the slope of the graph at that
dx x
point.
 ds 
(i) slope of displacement–time graph gives velocity  as v = 
 dt 
 dv 
(ii) slope of velocity–time graph gives acceleration  as a = 
 dt 

If z = yx, y (dx), or x (dy), the value of z between x1 and x2 or y1 and y2 can be obtained by the area of
graph between x1 and x2 or y1 and y2
(iii) velocity–time graph gives displacement (as ds=v dt)
(iv) acceleration–time graph gives change in velocity (as dv= adt).
   
relative velocity of A with respect to B (written as v AB ) is v AB
= v A − vB
  
Similarly, relative acceleration of A with respect to B is aAB= aA − aB
   
v AB =− vBA or aBA = −aAB
• In case of a one dimensional motion when we can treat the vectors as scalars by assigning positive to one
direction and negative to another, the above equations can be written as v AB
= v A − vB and aAB = aA − aB

Solved Examples

JEE Main/Boards = 2 R = 2 x 40 = 80 m
Initial point to final point gives the direction of
Example 1: A person in his morning walk moves on displacement.
a semicircular track of radius 40 m. Find the distance
travelled and the displacement, when he starts from
one end of the track and reaches the other end. Example 2: A man walks 2.5 km from his house to the
market on a straight road with a speed of 5 km/h. He
Sol: Distance is length of the path travelled. instantly turns back home with a speed of 7.5 km/h
Displacement is the vector from initial point to final finding the market closed. Calculate the
point. (a) magnitude of average velocity and (b) the average
The distance covered = length of the semicircular track speed of the man over the interval of time.
(i) 0 to 30 min (ii) 0 to 50 min

40m
(iii) 0 to 40 min

Initial position Final position Sol: Average speed is distance covered divided by
time taken. Distance is length of the path travelled.
= π R = 3.14 x 40 m = 125.6 m
Average velocity is displacement divided by time taken.
Displacement = Final position - initial position Displacement is the vector from initial point to final
= diameter of semicircular track point.
P hysi cs | 2.31

Distance between market and home = 2.5 km Example 3: A particle moving with an initial velocity 2.5
m/s along the positive x direction accelerates uniformly
Speed of man from home to market = 5 km/h
at the rate 0.50 m/s2. (i) Find the distance travelled in
∴ Time taken by the man to reach the market the first 2 sec. (ii) Calculate the time taken to reach the
Dis tance 2.5 1 velocity of 7.5 m/s? (iii) Calculate the distance travelled
t1 = ;=
t1 = = h 30 min in reaching the velocity 7.5 m/s?
Speed 5 2
Speed of man during his return = 7.5 km/h Sol: This is the case of motion with uniform acceleration.
Time taken by man to return home Use the three equations of motion with uniform
acceleration.
2.5 1
t2
= = = h 20 min 1
7.5 3 (i) We have, = x ut + at2
2
Total time taken by the man returning home = 30 + 20
= 50 min 1
= (2.5m/ s)(2s) + (0.50m / s2 )(2s)2
2
(i) Over the interval 0 to 30 min:
= 5.0 m + 1.0 m 6.0 m
During this time, man goes from home to market.
Therefore, displacement s = 2.5 km. Since the particle does not turn back it is also the
distance travelled.
Displacement 2.5
Average velocity= = = 5km / h (ii) We have, v = u + at
Time 1
2 or 7.5 m/s = 2.5 m/s + (0.50 m/s2 ) t
Dis tance 2.5
Average speed = = = 5km / h =t 7.5m / s − 2.5m / s
= 10 s
Time 1
0.50m / s2
2
(iii) We have, v2= u2 + 2ax
(ii) For the time interval 0 to 50 min: In 50 min the man
goes from home to market and return back or , (7.5 m/s)2 = (2.5 m/s) 2 +2.(0.50 m/s2)x

∴ Net displacement = zero (7.5m / s)2 − (2.5m / s)2


=or, x = 50m.
Net displacement 0 2 x 0.50m / s2
∴ Average velocity
= = = 0
Total time 50
Average speed Example 4: A particle is projected vertically upwards
with velocity 40 m/s. Find the displacement and
Total
Total
disdis
tance
tance 2.52.5
+ 2.5
+ 2.5 5 5 kmkm distance travelled by the particle in
=
= = == = 6= 6
Total
Total
timetime 5050 5050 h h (i) 2s (ii) 4s (iii) 6s
6060
Take g=10 m/s2
(iii) During the time interval 0 to 4 min: During first 30
min man goes home to market converting a distance Sol: Distance covered is equal to displacement if the
2.5 km in next 10 min the man is in path from market to object moves in a straight line and there is no change
10 in direction of motion. If direction of motion changes,
home and comes a distance
= 7.5x = 1.25km
60 distance should be calculated separately for different
Displacement = 2.5 – 1.25 = 1.25 km. parts of the path.

Distance = 2.5 + 1.25 = 3.75 km Here, due to upward motion, u is positive and due to
downward motion, a is negative.
Average velocity
Velocity becomes zero at maximum height
Displacement 1.25 Time taken to reach maximum height (t0) = u/g;
= = = 1.875km / h
time 1
40x u 40
60 t=
0 = = 4s +−ve
ve
a 10
Dis tance 3.75
Averagespeed
= = = 5.625 km/h (i) t < t0. Therefore, distance and displacement are
time 1
40x 1 1
60 equal. d= s= ut + at2 = 40x2 − x10x4 = 60m
2 2
2 . 3 2 | Motion in a Straight Line

(ii) t=t0, then distance and displacement are equal. divided by time taken.
1 dx x(t2 ) − (x(t1 )
d= s= 40x4 − x10x16 = 80m Velocity, v = ; ⇒ v avg =
dt t2 − t1
(iii) t>t0. Hence, d> s;
Given, x = 8.0 + 2.0 t2
1
s=
40x6 − x10x36 =
60m
2 dx d
(i) Velocity
(a) Velocity v
= = (8.0 + 2.0 t2 )
dt dt
u2 1
While d = + | a(t − t0 )2 | = 0 + 2.0 x 2t; v = 4t
2a 2
Velocity at t = 0s, (v)t-0 = 4 x 0 ms -1
2
(40) 1 Velocity at t = 2s, (v)t-2 =4 x 2 ms-1
= + x10x(6 − 4)2= 100m
2x10 2
(ii) Average velocity

Example 5: Following information about an object’s (x)t 4=


∆x = − (x)t 2
v avg
= =
motion is given: a = t2 ∆t 4−2

Initial velocity = u (8.0 + 2t2 )t 4=


= − (8.0 + 2t2 )t 2
=
Find: (i) velocity (v) as a function of time. 2

(ii) Displacement (x) a function of time.


=
{8 + 2x(4) } − {8 + 2x(2)
2

=
} 2
40 − 16
= 12ms−1
2 2
Sol: This is the case of motion with non-uniform
acceleration. Acceleration is given as a function of
Example 7: The motion of two bodies A and B
time. Change in velocity can be found by integrating
represented by two straight lines drawn on the same
the expression for acceleration with respect to
displacement-time graph, make angles 30º and 60º with
time. Displacement can be found by integrating the
time axis, respectively. Which body possesses greater
expression for velocity.
velocity? What is the ratio of their velocities?
dv
(i)=
a t2 dt
= t2 ⇒ dv =
dt Sol: The slope of the displacement time graph at an
v t instant gives the velocity at that instant.
2
Integration we get: ⇒ ∫ dv =
∫ t dt
u o The velocity of body = slope of displacement time
graph. Therefore the line having greater slope has
t3 t3 greater velocity, i.e. the body B has greater velocity.
⇒ v =u = ⇒v= +4
3 3 Ratio of their velocities,

dx 4 + t3  4 + t3  v A tan300 1 / 3 1
(ii) and =
v = ⇒ dx =
  dt = =
dt 3  3  vB tan600 3 3
 
Integration we get
Example 8: Displacement–time graph S
x t t 3
t t4 of a particle moving in a straight line is
⇒ ∫ dx = ∫ udt + ∫ dt ; ⇒ x = ut +
3 12 as shown in Figure. State whether the
o o o
motion is accelerated or not. Describe t₀
the motion in detail. Given s0=20 m -s₀
Example 6: The position of an object moving along and t0 = 4 s.
x-axis is given by x = 8.0 + 2.0t2, where x is in meter and
t is in second. Calculate: Sol: The slope of the displacement time graph at an
(i) the velocity at t = 0 and t = 2.0 sec. instant gives the velocity at that instant. If the slope is
constant, the velocity is uniform (zero acceleration). If
(ii) average velocity between 2.0 sec and 4.0 sec. the slope changes with time the motion is accelerated.

Sol: Here the position is given as a function of time. Slope of s–t graph is constant. Hence, velocity of
Differentiate this expression w.r.t time to get velocity particle is constant. Further at time t = 0, displacement
as a function of time. Average velocity is displacement of the particle from the mean position is –s0 i.e. –20 m.
P hysi cs | 2.33

s0 20 Relative acceleration, a = (30+20)cm/s2 = 0.5m/s2


velocity of particle, =
v slope
= = = 5m / s
t0 4 If “t” is the time taken to cross each other,
v=5 m/s 1 1
s ut + at2 ; 200
= = 25 t + x 0.5 t2
+ve 2 2
s=-20 m s=0
0.5 t2 + 50 t -400 = 0
t=0
Motion of the particle is as shown in Fig. 2.72. At t = 0 is −50 ± 2500 + 4x 400 x 0.5
at –20 m and has a constant velocity of 5 m/s. At t0 = 4 t= =−50 ± 3300
2x0.5
sec particle will pass through its mean position.
As negative t is ignored,
∧ ∧
Example 9: Abhishek is moving with velocity 5 i + 3 j t=−50 + 3300 =−50 + 57.44
∧ ∧ ∧
and Amit with velocity 2 i + 4 j − 3k Find t = 7.44 sec

(i) Relative velocity of Abhishek w.r.t. Amit (v12 )

(ii) Relative velocity of Amit w.r.t. Abhishek (v 21 ) JEE Advanced/Boards
Sol: Relative velocity of body 1 with respect to body 2
Example 1: Two cars started simultaneously towards
is obtained by vector sum of the velocity of body 1 and
each other from towns A and B which are 480 km
the negative of the velocity of body 2.
apart. It took first car travelling from A to B 8 hours to
∧ ∧
 cover the distance and second car travelling from B to
Velocity of Abhishek (v1=
) 5 i+ 3 j
A 12 hours. Determine the distance (in km) from town
∧ ∧ ∧
 A where the cars meet. Assuming that both the cars
Velocity of Amit(v 2 ) = 2 i + 4 j − 3k
travelled with constant speed.
∧ ∧ ∧ ∧ ∧ ∧ ∧ ∧
 
(a)
(i) v12
= v1 − v 2= (5 i + 3 j) − ( −2 i + 4 j − 3k) = 7 i − j + 3k
∧ ∧ ∧ ∧ ∧ Sol: This problem is best solved in the reference frame
  
(b) v 21
(ii) = v 2 − v1 =−
( 2 t + 4 j − 3k) − (5 i − 3 j) of one of the cars. Find the initial relative velocity of one
car with respect to the other. The time elapsed before
∧ ∧ ∧  the cars meet is equal to the initial distance between
=−7 t + j − 3k = − v12
  the two cars divided by the relative velocity.
Note: v 21 = − v12
480
Velocity of car from=
A = 60km / hour
8
Example 10: Two trains, each of length 100 m, move
480
in opposite direction along parallel lines at speeds 60 Velocity of car from=B = 40km / hour
km/h and 30 km/h, respectively. If their accelerations 12
480
are 30 cm/s2 and 20 cm/s2, respectively, then find the ∴t
= = 4.8 hour
60 + 40
time they take to pass each other.
The distance s=vA x t=60 x 4.8 = 288 km
Sol: This problem is best solved in the reference frame of
one of the trains. Find the initial relative velocity of one Example 2: An engine driver running a train at full
train with respect to the other, the relative acceleration speed suddenly applies brakes and shuts off steam. The
of one train with respect to the other and the relative train then travels 24 m in the first second and 22 m in
displacement of one train with respect to other as they the next second. Assuming that the brakes produce a
pass each other. Use the second equation of motion constant retardation, find
with constant acceleration to find the required time.
(i) Original speed of the train
The relative displacement of the trains is
(ii) The time elapsed before it comes to rest
S=100 + 100 = 200 M.
(iii) The distance travelled during the interval
The initial velocity u of one train relative to the other
train (iv) If the length of the train is 44 m,
1000 5 find the time that the train takes to pass an observer
(60 + 30)km/ h =
= 90 x m/s =
90 x m / s = 25 m/s
3600 18 standing at a distance 100 m ahead of the train at the
time when the brake was applied.
2 . 3 4 | Motion in a Straight Line

Sol: The retardation of train is constant, so we can use Sol: (i) Find the initial velocity u in terms of t1 and t2.
the equations of motion with uniform acceleration. The Use the equations of motion with uniform acceleration.
acceleration is taken with a negative sign. The acceleration due to gravity is taken with a negative
sign.
(i) The distance covered by the body in nth second,
a C
Sn =u + (2n − 1)
2
B
a a t₂ H
S1 =24 =u − (2 − 1) =u−
2 2 t₁
a 3a
S2 =22 =u − (4 − 1) =u−
2 2 A
3a a
Subtracting, 2 = − ;a =2m / s2 Let u be initial velocity.
2 2
a Total time of flight from A to B and from B
u = 24 + = 24 + 1 = 25 m / s
2 2u g(t1 + t2 )
to C to A =t1 + t2 = ; u=
(ii) Time t taken by the train before coming to rest, g 2
1 g 1 gt t
v=u-at or 0 = 25 – 2t or t =12.5 sec. h =ut1 − gt12 = (t1 + t2 )t1 − gt12 = 1 2
2 2 2 2
(iii) If S is the distance before the train comes to rest i.e.
u2
u2 (25)2 (ii) Maximum
Maximum heightheight reached,
reached, AC
= H
=
v=0; 0 = u2 – 2aS; S = = = 156.25m. 2g
2a 2x2 g2 (t1 + t2 )2 g(t1 + t2 )2
=
(iv) The time t taken by the train to cover a distance of =
4x2g 8
1 2
100 m is given by S = ut − at , H
2 ( c )(iii)LetLetv vbebevelocity
velocityatatheight
height ,
2
1
= 25t − x 2 x t2 ; t2 -25t + 100 = 0.
100 H
2 v2 = u2 − 2g = u2 − gH
2
(t-20) (t-5) = 0.; t=20, t=5,
t = 20, is not possible as the train takes only 12.5 second g2 (t + t )2 g2 (t1 + t2 )2 g2
=1 2 − = (t + t )2
to stop. Therefore t=5 second Time t’ taken by the train 4 8 8 1 2
to cover a distance of 100 m plus length of the train, i.e,= g
v ( t1 + t2 )
1 2 2 2
44m, is given by S = 100 + 44 = ut’ − at'
2
1 2 2 Example 4: If v(s) = s2 + s where s is displacement. Find
25t'− x 2 x t' − 144 = 0 ; t' − 25t' + 144 = 0
2 acceleration when displacement is 1 m.
t'2 − 16t' + 9t + 144 =
0 ; ( t − 16)(t'− 9) =
0
Sol: Differentiate the expression for velocity with
=t 16
= s ,t' 9 s respect to time to get the expression for acceleration.
∴ Time taken by the train to pass the observer = 9 – 5 dv dv ds  dv 
= 4 second. a
We know= = =   (v)
dt ds dt  ds 

dv dv
Example 3: A particle is projected vertically upwards ⇒a=v ; ∴ = (2s) + (1)
from a point A on the ground. It takes a time t1 to reach ds ds
a point B at a height h above the ground as it continues
 dv 
to move, it takes a further time t2 to reach the ground. and
and   = s2 + s
= 3m ; v(s)
 ds s =1m
Find
(i) The height h ∴ v(1)= (1)2 + 1= 2 ; ∴a(s = 1m) = (2) x(3) =6 m / sec2

(ii) The maximum height reached


Example 5: A point mass moves along a straight line
(iii) The velocity of the particle at half the maximum
height. with a deceleration n which is equal to K v where K is
P hysi cs | 2.35

a positive constant and v is the velocity of the particle. (ii) If total distance travelled by the particle is 4250 m
The velocity of the point mass at t = 0 is equal to v 0 . then find maximum speed.
Find the distance it will travel before it stops and the
(iii) Also, find the value of acceleration and retardation.
time it will take to cover this distance.
Sol: Area under the v-t graph in the given time interval
Sol: In the expression for acceleration separate the
is equal to displacement of the particle in the given
variables and integrate to get the desired quantity.
time interval.
dv dv
Acceleration = = −K v= Kdt. ; (i) v
dt − v
Let t0 be the time which the particle takes to come to
vmax
a stop.
Integrating 
1 
t0 0 −
1 v0
− 2
v 100 sec 400 sec 550 sec
∫ Kdt ∫
− v dv =
= v 2 dv ∫ [2v1/2 ]00
= t
0 v0 0
(ii) Distance travelled = Area under v – t graph
1
2v1/2
0
=Kt0 2v
= 0 or t0
2
1  1 
k =  (100) vmax  + (400 − 100) (vmax ) +  (550 − 400) vmax 
 2   2 
dv  dv   ds  dv dv
= =   v ; ∴v −K v
= 1 1
dt  ds   dt  ds ds 4250
⇒= x100 x vmax + 300 vmax + x150 vmax
2 2
v dv = −Kds ⇒ 4250 =
vmax ⇒ vmax =
10m / s
Let s0 be the dist nce covered when the velocity decrease ∆v
from v0 to zero. (iii) a =
∆t
s0
0 10 − 0
For
= t 0 to 100 sec =
⇒a = 0.1 m /s2
Integrating, ∫ − ∫ Kds =
v dv = −Ks0
100 − 0
v0 0
10 − 10
  For t 100 to 400 =
= sec ⇒ a = 0
 v 3/2 
0
 v 3/2  2v 3/2 400 − 1000
or   = − 0  = −Ks0 ; ∴ s0 = 0 For t = 400 sec to 500 sec (retardation)
 3 / 2  v  3  3k
0
 2  0 − 10 10
⇒ a− = =− ⇒ 0.06 m/s2
550 − 400 150
Example 6: Is the variation of position, shown in Figure.
observed in nature? Example 8: A particle starts from rest at time t = 0 and
undergoes acceleration a, as shown in the Figure.
(i) Draw a neat sketch showing the velocity of the
Time (t) particle as a function of time during the interval 1 to 4
O seconds, indicating each second on the abscissa.
Position(x)
(ii) Draw a neat sketch showing the displacement of the
Sol: Time never decreases in a reference frame.
particle as a function of time during 0 to 2 second. In
No, since with increase of position x, time first increase both the cases, explain the various steps.
and then decrease, which is impossible (Time always 2
a(m/s )
increase)

+3
Example 7: A particle start from rest with constant
acceleration for 100 s, then move with constant velocity 1 4
2 3
for 5 min. Finally, particle retards uniformly and come
to rest in 150 s. t(sec.)
(i) Draw v-t graphs -3
2 . 3 6 | Motion in a Straight Line

Sol: Area under the a-t graph in the given time interval 1
is equal to the change in velocity of the particle in the distance travelled = 6 x 1 − x 3 x = 4.5m
2
given time interval. Thus v-t graph can be plotted if the
At t = 3, S3 = 6 + 4 ½ = 10.5 m
initial velocity is known. Area under the v-t graph in the
given time interval is equal to the displacement of the At t = 4, S4 = 6 + [6 x 2- ½ x 3 x 4] = 12 m
particle in the given time interval.
(i) The velocity is given by the area enclosed during the Example 9: A ball is dropped from a height of 19.6 m
time interval; and the velocity is constant from 0 to 2 above the ground. It rebounds from the ground and
sec. raises itself up to the same height. Take the starting
point as the origin and vertically downward as the
At t = 1 sec., velocity = 3 m/s positive X-axis. Draw approximate plots of x versus t, v
At t = 2 sec., velocity = 6 m/s. versus t and a versus t. Neglect the small interval during
which the ball was in contact with the ground.
At this instant the acceleration becomes negative, so
the velocity starts decreasing uniformly. Sol: From the first equation of motion for constant
acceleration, plot the v-t graph. From the second
equation of motion for constant acceleration, plot the
6 x-t graph.
5
v(m/s) 4 Since the acceleration of the ball during the contact
3
2
1 1 2 3 4 is different from ‘g’, we have to treat the downward
0 motion and the upward motion separately.
t(sec.) For the downward motion: a = g = 9.8 m/s2,
1 2
At t = 3 sec., velocity = 6 -3 = 3 m/s x =+ut at = (4.9m / s2 )t2
2
At t = 4 sec., velocity = 6 – 6 = 0 m/s. The ball reaches the ground when x = 19.6 m. This gives
(ii) The particle starts from rest, and the acceleration a t = 2 s. After that it moves up, x decreases and at t =
is constant from 0 to 2 sec. 4 s, x becomes zero, the ball reaching the initial point.
x
6 20 m

27/8 10 m

S(m) 1
3/8
1 2 3 4 t (second)
0 1/4 1 3/2 23 4
t in sec We have t = 0, x = 0
1 2 t = 1s, x=4.9 m
S= at The graph between S and t will be parabola,
2
1 1 1 1 1 31 3 t= 2s x =19.6 m
At=
= t At tsec sec
= , S1/2
= S1/2
= x 3 x=
x3x m m
2 2 2 2 4 84 8 t = 3s x = 4.9 m
1 3 t = 4s x=0
At t 1sec,=
= S1/2 = x 3 x1 m
2 2
Velocity: During the first two seconds
3 1 9 27
At t
= sec
= S1/2 = x 3x m V
2 2 4 8 20 m/s
1
At t 2=
= sec S2 x=
3 x 4 6m
2 10 m/s

Beyond t = 4s, the acceleration becomes negative, O


the curvature of the graph becomes opposite at this 1 2 3 4 t(second)
instant. -10 m/s

In the interval between t = 2s and t = 3s, -20 m/s


P hysi cs | 2.37

v = u + at = (9.8 m/s2)*t equal to the distance travelled upstream relative to the


raft. Total time of travel of boat is 2 h.
at t = 0 v=0
at t = 1 s, v = 9.8 m/s
at t = 2 s, v = 19.6 m/s Boat
T
During the next two seconds the ball goes upward,
velocity is negative, magnitude decreasing and at t =
4 s, v = 0. Thus Raft
6 km
at t = 2 s, v = -19.6 m/s
at t = 3 s, v = -9.8 m/s Vr = velocity of river w.r.t ground
at t = 4 s, v = 0. Vb = velocity of boat w.r.t ground
At t = 2 s there is an abrupt change in velocity from Let the time taken from starting to the end = T
19.6 m/s to -19.6 m/s. In fact this change in velocity
takes place over a small interval during which the ball Velocity of raft w.r.t. ground = Vr
remains in contact with the ground. Distance travelled by raft in time T = 6 km
⇒ Vr T = 6 km
-20 m/s²

Boat 1 hr

1 2 3 4 t (second)

Acceleration: The acceleration is constant 9.8 m/ s2


throughout the motion (except at t=2s).
Raft 1 hr

Example 10: Boat is moving down the stream and


crosses a raft at t = 0 sec. After 1 hour boat turns and W.r.t river, since boat travels for 1 h downstream and
again crosses the raft at a point 6 km from the initial since, velocity of boat remains constant and raft doesn’t
position of raft. Find the velocity of river assuming duty move w.r.t river so, boat will again take 1 hour to reach
of engine remain constant. the raft back.
Note: raft is an object which floats on the river i.e. it has ∴T=1+1=2h
zero velocity w.r.t. river.
On substituting value of T in (i) we get
Sol: Relative to the raft the boat is moving with constant Vr (2) = 6 km; ⇒ Vr = 3 km/h
speed. Distance travelled downstream relative to raft is

JEE Main/Boards

Exercise 1 Q.3 A body covers 12 m in 2nd second and 20 m in


4th second. Find what distance the body will cover in 4
Q.1 A body starting from rest has an acceleration of 20 seconds after the 5th second.
ms-2. Calculate the distance travelled by it in 6th second.
Q.4 A racing car moving with constant acceleration
Q.2 A train was moving at rate of 36 kmh-1. When the covers two successive kilometers in 30 s and 20 s
brakes were applied, it comes to rest in a distance of respectively. Find the acceleration of the car.
200 m. Calculate the retardation produced in the train.
Q.5 Two cars start off to race with velocities 2 m/s and
2 . 3 8 | Motion in a Straight Line

4 m/s travel in straight line with uniform acceleration Q.14 The velocity–time graph of a particle moving
2m/s2 and a m/s2 respectively. What is the length of the along a straight line is as shown in Fig. Calculate the
path if they reach the final point at the same time? distance covered between t = 0 to t = 10 second. Also
find displacement in time 0 to 10 seconds.
Q.6 Brakes are applied to a train travelling at 72 kmh- v(in ms)
1. After passing over 200 m, its velocity is raced to 36 20
kmh-1. At the same rate of retardation, how much 10
further will it go before it is brought to rest? 0
2 46 8 10 t(s)
-10
-20
Q.7 On turning a corner, a motorist rushing at 44 ms-1
finds a child on the road 100 m ahead. He instantly Q.15 The position of an object moving along x-axis is
stops the engine and applies the brakes so as to stop given by x=a+bt2, where a = 8.5 m and b = 2.5 ms-2 and
it within 1 m of the child. Calculate time required to and t is measured in second. What is the velocity at t=0
stop it. s and t=2.0 s? What is the average velocity between t =
2.0 s and t = 4.01 s?
Q.8 A body starting from rest, was observed to cover
20 m in 1 second and 40 m during the next second. Q.16 The displacement x (in m ) of a body varies with
How far had it travelled before the first observation was time t (in sec) as x =- (2/3)t2 +16 t + 2. How long does
taken? the body take to come to rest?

Q.9 An automobiles starts from rest and accelerates Q.17 The height y and the distance x along the
uniformly for 30 seconds to a speed of 72 km h-1. It then horizontal, for a body projected in the vertical plane are
moves with a uniform velocity and it is finally brought given by y = 8t – 5t2 and x = 6 t. What is initial velocity
to rest in 50 m with a constant retardation. If the total of the body?
distance travelled is 950 m, find the acceleration, the
retardation and total time taken. Q.18 The displacement of a particle along X-axis
is given by x = 3 + 8t + 7t2 . Obtain its velocity and
acceleration at t = 2s.
Q.10 From the top of a tower 100 m in height a ball
is dropped and at the same instant another ball is
Q.19 The relation between time t and distance x is t
projected vertically upwards from the ground so that it
= αx2 + βx where α and β are constants. Show that
just reaches the top of tower. At what height do the two
retardation is 2α υ, where υ is the instantaneous velocity.
balls pass one another?

Q.20 The acceleration ‘a’ in ms-2 of a particle is given


Q.11 A body falling from rest was observed to fall by a = 3t2 + 2t + 2, where t is the time. If the particle
through 78.4 m in 2 seconds. Find how long had it been starts out with a velocity υ = 2 ms-1 at t = 0, then find
falling before it was observed? the velocity at the end of 2s.

Q.12 A stone is dropped from a balloon at an altitude Q.21 A tennis ball is dropped onto the floor from a
of 300 m. How long will the stone take to reach the height of 4.0 ft. It rebounds to a height of 3.0 ft. If the
ground if (i) the balloon is ascending with a velocity of ball was in contact with the floor for 0.010 s, what was
5 ms-1. (ii) the balloon is descending with a velocity of 5 the average acceleration during contact?
ms-1. (iii) the balloon is stationary?
Q.22 A particle starts from rest with zero initial
Q.13 The velocity–time graph of a body moving in a acceleration. the acceleration increases uniformly with
straight line is shown in Fig. Find the displacement and time. Find the time average and distance average
the distance travelled by the body in 6 seconds. of velocity upto a certain instant when the velocity
becomes v.
v(in ms)
4
Q.23 A particle moves along a straight line such that
2
its displacement x from a fixed point on the line at time
-2 1 2 3 4 5 6 t(s) t is given by x2 = at2 + 2bt + c Find acceleration as a
-4 function of displacement x.
P hysi cs | 2.39

Q.24 A ball is dropped from a height of 19.6 m above Q.3 An object is tossed vertically into the air with an
the ground. It rebounds from the ground and raises initial velocity of 8 m/s. Using the sign convention
itself up to the same height. Take the starting point upwards as positive, how does the vertical component
as the origin and vertically downward as the positive of the acceleration ay of the object (after leaving the
X-axis. Draw approximate plots of x versus t, v versus t hand) vary during the flight of the object?
and a versus t. Neglect the small interval during which
(A) On the way up ay > 0, on the way down ay >0
the ball was in contact with ground.
(B) On the way up ay <0, on the way down ay < 0
Q.25 A train travelling at 72 km/h is checked by track (C) On the way up ay >0, on the way down ay < 0
repairs. It retards uniformly for 200 m covering the next
(D) on the way up ay <0, on the way down ay <0
400 m at constant speed and accelerates uniformly
to 72 km/h in a further 600 m. If the time at constant
lower speed is equal to the sum of the times taken in Q.4 If position time graph of a X
retarding and accelerating. Find the total time taken. particle is since curve as shown,
t
What will be its v-t graph?
Q.26 A point traversed half the distance with a velocity V V
v0. The remaining part of the distance was covered with (A) (B)
t t
velocity v1 for half the time, and with velocity v2 for
the other half of the time. Find the men velocity of the V V
point averaged over the whole time of motion. (C) (D)
t t

Q.27 A person sitting on the top of a tall building is Q.5 A man moves in x-y plane along the path shown.
dropping balls at regular intervals of one second. Find At what point is his average velocity vector in the same
the positions of the 3rd, 4th and 5th ball when the 6th ball direction as his instantaneous velocity vector. The man
is being dropped. starts from point P.

Q.28 A stone is dropped from a balloon going up with (A) A (B) B (C) C (D) D
a uniform velocity of 5.0 m/s. if the balloon was 50 m
high when the stone was dropped, find its height when Q.6 The greatest acceleration or deceleration that a
the stone hits the ground. train may have is a. The minimum time in which the
train reach from one station to the other separated by
Take g = 10 m/s2.
a distance D is

Exercise 2 (A)
d
(B)
2d
(C)
1 d
(D) 2
d
a a 2 a a
Single Correct Choice Type
Q.7 Acceleration versus velocity graph
Q.1 An object is moving along the x axis with position a
of a particle moving in a straight line
as a function of time given by x = x(t). Point O is at x = starting from rest is as shown in figure.
0. The object is definitely moving towards O when a a v
The corresponding velocity–time
graph would be v v
(A) dx/dt < 0 (B) dx/>dt>0 v v
(C) d(x2)/dt < 0 (D) d(x2)/dt > 0 v v v v (A) (B)
(A) (A) (B) (B) t t
v v
Q.2 A particle starts moving rectilinearly at time t t t t t
= 0 such as that its velocity ‘v’ changes with time ‘t’ v v v v (C) (D)
according to the equation v = t2 – t where t is in seconds (C) (C) (D) (D) t t
and v is in m/s. The time interval for which the particle t t t t
retards is
Q.8 Suppose a player hits several baseballs, which
(A) t < 1/2 (B) 1/2 < t <1
baseball will be in the air for the longest time?
(C) t> 1 (D) t < ½ and t>1
(A) The one with the farthest range.
2 . 4 0 | Motion in a Straight Line

(B) The one which reaches maximum height. Previous Years’ Questions
(C) The one with the greatest initial velocity.
(D) The one leaving the bat at 45º with respect to the Q.1 In the arrangement
ground. shown in the Figure. the ends A B
P and Q of an unstretchable
string move downwards with 
Q.9 A ball is thrown from a point on ground at some
angle of projection. At the same time a bird starts uniform speed U. Pulleys A P O
from a point directly above this point of projection at and B are fixed. M
a height h horizontally with speed u. Given that in its Mass M moves upwards with s speed  (1982)
flight ball just touches the bird at one point. Find the
distance on ground where ball strikes. (A) 2U cos θ (B) U/cos θ
(C) 2U/cos θ (D) U cos θ
h 2h 2h h
(A) 2u (B) u (C) 2u (D) u
g g g g
Q.2 A particle is moving eastwards with a velocity of 5
m/s. In 10 s the velocity changes to 5 m/s northwards.
Q.10 It takes one minute for a passenger standing on
The average acceleration in this time is  (1982)
an escalator to reach the top. If the escalator does not
move it takes him 3 minute to walk up. How long will it (A) Zero
take for the passenger to arrive at the top if he walks up
1
the moving escalator? (B) m / s2 towards north-east
2
(A) 30 sec   (B) 45 sec    (C) 40 sec    (D) 35 sec
1
(C) m / s2 towards north-west
Q.11 A body A is thrown vertically upwards with 2
such a velocity that it reaches a maximum height of
h. simultaneously another body B is dropped from 1
(D) m / s2 towards north
height h. It strikes the ground and does not rebound. 2
The velocity of A relative to B v/s time graph is best
represented by: (upward direction is positive) Q.3 A river is flowing from west to east at a speed of 5
m/min. A man on the south bank of the river, capable of
(A) VAB (A) VAB
(B) VAB (B) VAB swimming at 10 m/min in still water to swim across the
t t t t river in the shortest time. He should swim in a direction.
VAB VAB  (1983)
(C) VAB (C) VAB (D) (D)
t t (A) Due north (B) 30º east of north
t t
(C) 30º west of north (D) 60º east of north
Q.12 Particle A and B are moving with constant
velocities along x and y axis respectively, the graph of Q.4 A boat which has a speed of 5 km/h still crosses
separation between them with time is a river of width 1 km along the shortest possible path
y in 15 min. The velocity of the river water in km/h is
A  (1988)

x (A) 1 (B) 3 (C) 4 (D) 41
B
Q.5 A particle P is sliding down a frictionless hemis-
s s
(A) (B) pherical bowl. It passes the point A at t = 0. At this
instant of time, the horizontal component of its velocity
t t is v. A bead Q of the same mass as P is ejected from
A at t = 0 along the horizontal string mass AB, with
(C)
s
(D)
s the speed v. Friction between the bead and the string
may be neglected. Let tp and tQ be the respective times
t t
P hysi cs | 2.41

taken by P and Q to reach the point B. Then (1993) Q.9 A small block slides without friction down an
inclined plane starting from rest. Let sn be the distance
Q
A B s
travelled from t = n. Then n is 
n − 1 to t = (2004)
P sn + 1
C 2n − 1 2n + 1
(A) (B)
(A) tp < tQ (B) tp = tQ 2n 2n − 1

tp lengthof arc ACB 2n − 1 2n


(C) (D)
(C) tp > tQ (D) 2n + 1 2n + 1
tQ lengthof chord AB

Q.10 The given graph shows the variation of velocity


Q.6 In 1.0 s, a particle goes from point A to A
with displacement. Which on of the graph given below
point B, moving in a semicircle (see the
correctly represents the variation of acceleration with
Figure). The magnitude of the average 1.0
m
displacement.  (2005)
velocity is  (1999)
B v
v0
(A) 3.14 m/s (B) 2.0 m/s (C) 1.0 m/s (D) Zero

Q.7 A ball is dropped vertically from a height d above


the ground. It hits the ground and bounces up vertically x
to a height d/2. Neglecting subsequent motion and air x0
resistance, its velocity v varies with height h above the a
a a
a
ground as (2000)
x x
(A) (A) (B) (B) x x
V V
a a a a
d
(A) h (B) x x
x
(C) (C) (D) x (D)

h
d
V V
Q.11 From a tower of height H, a particle is thrown
d d vertically upwards with a speed u. The time taken by
(C) h (D) h the particle, to hit the ground, is n times that taken by
it to reach the highest point of its path. The relation
between H, u and n is: (2014)
(A) 2gH = nu2(n -2) (B) gH = (n -2)u2
Q.8 A particle starts from rest. Its acceleration (a) versus (C) 2gH = n2u2 (D) gH = (n -2)2u2
time (t) is as shown in the Figure. The maximum speed.
The maximum speed of the particle will be  (2004)
a
Q.12 Two stones are thrown up simultaneously from
the edge of a cliff 240 m high with initial speed of 10
10m/s²
m/s and 40 m/s respectively. Which of the following
graph best represents the time variation of relative
position of the second stone with respect to the first?
t(s)
11
(Assume stones do not rebound after hitting the ground
(A) 110 m/s (B) 55 m/s and neglect air resistance, take g = 10 m/s2) (The figures
are schematic and not drawn to scale) (2015)
(C) 550 m/s (D) 660 m/s
240 240
(A) (B)
2 . 4 2 | Motion in a Straight Line
t(s)
12 8 12
(A) (y2-y1)m (y2-y1)m (B) (y2-y1)m (y2-y1)m
240 240 240 (y2-y1)m 240
(y2-y1)m
(A) (B) (C) 240 (D)
240
(A) (B) t(s)
t(s) t(s)
12 8 12 8 12 8 12
(y2-y1)m t(s) (y2-y1)m t(s)
12 8 12
(C) 240 (y2-y1)m (y2-y1)m 240 (D) (y2-y1)m
(C) (D)
240 240 240
(B) (C) t(s) (D) t(s)
8 12 8 12
t(s) t(s) t(s) t(s)
8 12 8 12 8 12
(y2-y1)m
240
(D) JEE Advanced/Boards
t(s) t(s)
Exercise 1 8 12
the average velocity during the motion is 20 ms–2, then
the final value of t.
Q.1 A car moving along a straight highway with a speed

v m\s
of 126 km/h is brought to stop with in a distance of 200
m. what is the retardation of the car (assumed uniform)
O
and how long does it take for the car to stop? t 25 sec

Q.7 The figure shows the v-t graph of a particle moving


Q.2 A police van moving on a highway with a speed in the straight line. Find the time when particle returns
of 30 km/h fires a bullet at a thief’s car speeding away to the starting point.
in the same direction with a speed of 192 km/h, if the v
muzzle speed of bullet is 150 m/s, with what speed 20
does the bullet hit the thief’s car.
10
Q.3 What is the ratio of the distance travelled by a body
falling freely from rest during first, second and third 10 20 25t
second of its fall.
Q.8 A stone is dropped from a height h. simultaneously
Q.4 At a distance L = 400 m from the traffic light another stone is thrown up from the ground with such
breaks are applied to locomotive moving at a velocity a velocity that it can reach a height of 4h. Find the time
v=54 km/hr. Determine the position of the locomotive when two stones cross each other.
relative to the traffic light 1 minute after the application
of the breaks if its acceleration is –0.3m/s2 Q.9 A glass wind screen whose inclination with the
vertical can be changed is mounted on a car. The car
Q.5 An object moving with uniform acceleration has a moves horizontally with a speed of 2 m/s. At what angle
velocity of 12 cm/s in the positive x direction when its α with the vertical should the wind screen be placed so
x coordinate is 3.00 cm. If its x coordinate 2.00 s later is that the rain drops falling vertically downwards with
-5.00 cm, what is its acceleration? velocity 6 m/s strikes the wind screen perpendicularly?

Q.6 The velocity–time graph of the particle moving Q.10 Two particles are moving along two straight lines,
along the straight line is shown. The rate of acceleration in the same plane, with the same speed=20 cm/s.
and deceleration is constant and it is equal to 5ms–2, If The angle between the two lines is 60° , and their
P hysi cs | 2.43

intersection point is O. At a certain moment, the two (i) how far beyond the straight point will automobile
particles are located at distances 3m and 4m from O, overtake the truck?
and are moving towards O. Find the shortest distance
(ii) How fast will the automobile be travelling at that
between them subsequently?
instant? (It is instructive to plot qualitative graph of x
versus t for each vehicle.)
Q.11 A point mass starts moving in a straight line with a
constant acceleration a. At a time t, after the beginning
Q.18 Two bodies moves in a straight line towards
of motion, the acceleration changes sign, remaining
each other at initial velocities v1 and v2 and constant
the same magnitude. Determine the time t from the
accelerations a1 and a2 directed against the
beginning of motion in which the point mass returns to
corresponding velocities at the initial instant.
the initial position?
What must be the maximum initial separation lmax
th
1 between the bodies for which they meet during
Q.12 For   of the distance between two stations,
m th motion? Motion in which the point mass returns to the
1
a train is uniformly accelerated and for   of the initial position?
n
distance, it is uniformly retarded. It starts from rest at
Q.19 An ant runs from an ant-hill in a straight line so
one station and comes to rest at another. Find the ratio
that its velocity is inversely proportional to the distance
of its maximum velocity to its average velocity?
from the center of the ant-hill. When the ant is at point
A at a distance l1 = 1m from the center of the ant-hill,
its velocity v1 = 2cm / s , what time will it take ant to run
Q.13 The velocity of a particle moving in the positive from point A to point B, which is at a distance l2 = 2m
direction of the x axis varies as v= α v x , where α is from the center of the ant-hill?
positive constant. Assuming that at the moment t=0
the particle was located at the point x=0, find: (i) the
time dependence of the velocity and acceleration of Q.20 Distance between two points A and B is 33 m. A
the particle. particle P starts from B with velocity of 1m/s along AB
with an acceleration of 2m / s2 . Simultaneously another
(ii) the mean velocity of the particle averaged over the particle Q starts from A with a velocity of 9 m/s in the
time that the particle takes to cover the first s meter of same direction AB and has an acceleration 1m / s2 in
the path. the direction AB. Find whether Q will be able to catch P.

Q.14 A boy standing on a long railroad car throws a ball Exercise 2


straight upwards. The car is moving on the horizontal
road with an acceleration of 1 m/s2 and projection
Multiple Correct Choice Type
velocity in the vertical direction is 9.8 m/s. How far
behind the boy will the ball fall on the car? Q.1 A particle moves with constant speed v along a
regular hexagon ABCDEF in the same order. Then the
Q.15 A person walks up a stalled escalator in 90 s. magnitude of the average velocity for its motion from
When standing on the same escalator, now moving, he A to
is carried up to 60 s. How much time would it take him
(A) F is v/5 (B) D is v/3
to walk up the moving escalator?
(C) C is v 3 / 2 (D) B is v
Q.16 Two trains of lengths 180 m are moving on parallel
tracks. If they move in the same direction then they Q.2 A particle moving with a speed v changes direction
cross each other in 15 s, and if they move in opposite by an angle θ , without change in speed.
directions then they cross in 7.5 seconds, then calculate (A) The change in the magnitude of its velocity is zero.
their velocities.
(B) The change in the magnitude of its velocity is 2vsin
( θ /2).
Q.17 At the instant the traffic light turns green, an
automobile starts with a constant acceleration ax of (C) The magnitude of change in velocity is 2vsin ( θ /2).
6.0 ft/ s2 , at the same instant a truck, travelling with (D) The magnitude of change in velocity is 2v (1 - cos θ ).
a constant speed of 30 ft/s, overtakes and passes the
automobile.
2 . 4 4 | Motion in a Straight Line

Q.3 A particle has initial velocity 10 m/s. It moves due (B) The ball will never return to him.
to constant retarding force along the line of velocity
(C) The ball will return to him if the cart moves with
which produces a retardation of 5 m / s2 . Then
constant velocity.
(A) the maximum displacement in the direction of initial
(D) The ball will fall behind him if the cart moves with
velocity is 10 m.
some positive acceleration.
(B) the distance travelled in first 3 seconds is 7.5 m.
(C) the distance travelled in the first 3 seconds is 12.5 m. Assertion Reasoning Type
(D) the distance travelled in the first 3 seconds is 17.5 m. (A) Statement-I is true, statement-II, is true and
statement-II is correct explanation for statement-I
Q.4 A bead is free to slide down a smooth wire tightly (B) Statement-I is true, statement-II is true and
stretched between points A and B on a vertical circle. statement-II is NOT the correct explanation for
If the bead starts from rest at A, the highest point on statement-I.
the circle
A (C) Statement-I is true, statement-II is false.
(D) Statement-I is false, statement-II is true.

B
Q.8 Statement-I: Positive acceleration in rectilinear
motion of a body does not imply that the body is
(A) Its velocity v on arriving at B is proportional to cos θ speeding up
(B) Its velocity v on arriving at B is proportional to tan θ Statement-II: Both the acceleration and velocity are
(C) Time to arrive at B is proportional to cos θ vectors.
(D) Time to arrive at B is independent of θ .
Q.9 Statement-I: A particle having zero acceleration
must have constant speed.
Q.5 The figure shows the velocity +v0
v
(v) of a particle plotted against T Statement-II: A particle having zero acceleration must
O have zero acceleration.
time (t) t 2T
-v0
Q.10 Statement-I: A student performed an experiment
(A) The particle changes its direction of motion at some by moving a certain block in a straight line. The velocity
point. position graph cannot be as shown.
(B) The acceleration of the particle remains constant. V
(C) The displacement of the particle is zero.
(D) The initial and final speed of the particle are the
V
same.

Q.6 An observer moves with a constant speed along


the line joining two stationary objects. He will observe
Statement-II: When a particle is at its maximum in
that the two objects
rectilinear motion its its velocity must be zero.
(A) Have the same speed.
(B) Have the same velocity. Q.11 Statement-I: If the velocity time graph of a
body moving in a straight line is as shown here, the
(C) Move in the same direction.
acceleration of the body must be constant
(D) Move in opposite directions.

Q.7 A man on a rectilinearly moving cart, facing the V


direction of motion, throws a ball straight up with
respect to himself t
(A) The ball will always return to him.
P hysi cs | 2.45

Statement-II: The rate of change of quantity which is Q.17 The v-t graph of the particle is correctly shown by
constant is always zero.
V V
(A) 0 (B) 0
Comprehension Type

Paragraph 1: The figure shows a velocity–time graph


V V
of a particle moving along a straight line
(C) 0 (D) 0
T 2T t T 2T t
-1
v(ms )
10
0 Q. 18 The a-t graph of the particle is correctly shown by
2 4 6 8 t(s)

a a a a
(A) (A) T 2T
0 0 T 2T (B) (B)
0 0
-20 t t t t

Q.12 Choose the incorrect statement. The particle a a a a


comes to rest at (C) (C)
0 0
t t
(D)(D)
0 0
t t
(A) t = 0 s (B) t = 5 s
(B) t = 8 s (C) None of these Q. 19 The speed–time graph of the particle is correctly
shown by
Q.13 Identify the region in which the rate of change of
 speed speed
∆v
velocity of the particle is maximum (A) 0 T 2Tt
(B) 0
T 2Tt
∆t

(A) 0 to 2s (B) 2 to 4s speed speed


(C) 4 to 6s (D) 6 to 8 s (C) 0 (D)
T 2Tt T 2Tt

Q.14 If the particle starts from the position


x0=- 15 m, then its position at t=2s will be Match the Columns

(A) -52m (B) 5 m (C) 10 m (D) 15 m Q.20 Column I shows position versus time graph for an
object and column II shows possible graphs.
Q.15 The maximum displacement of the particle is
(A) 33.3 m (B) 23.3 m Column I Column II

(B) 18.3 m (C) Zero x


(p) A ball rolls along the floor
Q.16 The total distance travelled by the particle is (A) towards the origin
(A) 66.7 m (B) 51.6 m
(B) Zero (C) 36.6 m t
x
(q) A ball rolled towards a
Paragraph 2: The x-t graph of the particle moving (A) wall at the origin, then ball
along a straight line is shown in the figure rebounds.

t
X parabola
x
0 (r) A ball rolling away from the
T 2T (A) origin; hits a wall and bounces
straight back.
t
2 . 4 6 | Motion in a Straight Line

(i) Calculate the distance d between the towers.


Column I Column II
x (ii) Find the position where the objects hit the ground.
(s) An object rolling towards
(A) the origin and suddenly stops. Q.4 Two guns situated on the top of a hill of height 10
m fire one shot each with same speed 5 3 m/s at some
t interval of time. One gun fires horizontally and other
fires upwards at an angle of 60° with the horizontal.
(t) A book at rest on a table. The shots collide in air at point P(g= 10m / s2 ) find
(i) the time interval between the firings and
(ii) the coordinates of the point P. Take origin of the
Previous Years’ Questions coordinate system at the foot of the hill right below the
muzzle and trajectories in x-y plane.  (1996)
Q.1 Particles P and Q of mass 20 g and 40 g respectively
are simultaneously projected from points A and B on
Q.5 A cart is moving along x-direction with a velocity
the ground. The initial velocities of P and Q makes 45°
of 4 m/s. A person on the cart throws a stone with a
and 135° angles respectively with the horizontal AB as
velocity of 6 m/s relative to himself. In the frame of
shown in the Fig. Each particle has an initial speed of 49
reference of the cart the stone is thrown in y-z plane
m/s. The separation AB is 245 m.
making an angle of 30° with vertical z-axis. At the
Both particles travel in the same vertical plane and highest point of its trajectory the stone hits an object
undergo a collision. After the collision, P retraces its of equal mass hung vertically from the branch of a tree
path. Determine the position Q where it hits the ground. by means of a string of length L. A completely inelastic
How much time after collision does the particle Q take collision occurs, in which the stone gets embedded in
to reach the ground? (Take g= 9.8 m / s2 )  (1982) the object. Determine (g=9.8 m / s2 ) (1997)

P Q
(i) the speed of the combined mass immediately after
135° the collision with respect to an observer on the ground.
45°
A B (ii) the length L of the string such that the tension in
the string becomes zero when the string becomes
Q.2 A body falling freely from a given height H hits an horizontal during the subsequent motion of the
inclined plane in its path at a height h. As a result of combined mass.
this impact the direction of the velocity of the body
becomes horizontal. For what value of (h/H) the body
Q.6 A particle of mass 10−2 kg is moving along the
will take maximum time to reach the ground?  (1986)
positive x-axis under the influence of a force F(x)
= −k / 2x2 where k= 10−2 Nm2 . At time t=0 it is at x=0.1
Q.3 Two towers AB and CD are situated a distance d m and its velocity v=0.
apart as shown in Figure. AB is 20 m high and CD is
(i) Find its velocity when it reaches x=0.5 m.
30 m high from the ground. An object of mass m is
thrown from the top of AB horizontally with a velocity (ii) Find the time at which it reaches x=0.25 m.  (1998)
of 10 m/s towards CD. Simultaneously other object of
mass 2m is thrown from top of CD at an angle of 60° to Q.7 A large heavy box is sliding without friction down
the horizontal towards AB with the same magnitude of a smooth plane of inclination θ . From a point P on the
initial velocity as that of the first object. The two objects bottom of the box, a particle is projected inside the
moves in the same vertical plane, collide in mid-air and box. The initial speed of the particle with respect to box
stick to each other.  (1994) is u and the direction of projection makes an angle α
2m
60°
C with the bottom as shown in the Figure.  (1998)

m
A 
Q
P

d
B D
P hysi cs | 2.47

(i) Find the distance along the bottom of the box at the initial height. The acceleration of the train in
between the point of projection P and point Q where m/s2, is.  (2011)
the particle lands (Assume that the particle does not
hit any other surface of the box. Neglect air resistance.) Assertion Reasoning Type
(ii) If the horizontal displacement of the particle as seen Mark your answer as
by an observer on the ground is zero, find the speed of
the box with respect to the ground at the instant when (A) If Statement-I is true, statement-II is true:
the particle was projected. statement-II is the correct explanation for statement-I.
(B) If Statement-I is true, statement-II is true: statement-
Q.8 An object A is kept fixed at the point x=3 and y=1.25 II is not a correct explanation for statement-I.
m on a plank P raised above the ground. At time t=0 (C) If Statement-I is true: statement-II is false.
the plank starts moving, along the +x-direction with
an acceleration 1.5 m/s2. At the same instant a stone (D) If Statement-I is false: statement-II is true.

is projected from the origin with velocity u as shown.
A stationery person on the ground observes the stone Q.11 Statement-I For an observer looking out through
hitting the object during its downward motion at an the window of a fast moving train, the nearby objects
angle of 45o to the horizontal. All the motions are in x-y appear to move in the opposite direction to the train,

plane, Find u and the time after which the stone hits while the distant objects appear to be stationary.
the object. (Take g=10 m/s2)  (2000) Statement-IIIf the observer and the object are moving

y A at velocities v1 and v 2 respectively with reference to a
laboratory frame, the velocity
1.25m P of the object with respect


to the observer is v 2 − v1  (2008)
u
x
O 3.0m Q.12 A particle of mass m moves on the x-axis as follows:
it starts from rest at t=0 from the point x-0, and comes
Q.9 On a frictionless horizontal surface, assumed to be
to rest at t=1 at the point x=1. No other information
the x-y plane, a small trolley A is moving along a straight
is available about its motion at intermediate times
line parallel to the y-axis with a constant velocity of
(0<t<1). If α denotes the instantaneous acceleration of
( 3 − 1) m/s. At a particular instant when the line OA the particle, then  (1993)
makes an angle of 45° with the x-axis, a ball is thrown
(A) α cannot remain positive for all t in the interval
along the surface from origin O. Its velocity makes an
0≤t ≤1
angle φ with the x-axis and it hits the trolley.  (2002)
(B) α cannot exceeds 2 at any point in its path
y
A (C) α must be ≥ 4 at some point or points in its path
(D) α must change sign during the motion, but no other
assertion can be made with information given.
45°
O
Q.13 The coordinates of a particle moving in a plane
(i) The motion of the ball is observed from the frame of are given by x(t)= acos (pt) and y(t)=bsin(pt) where
the trolley. Calculate the angle θ made by the velocity a,b (<a) and p are positive constants of appropriate
vector of the ball with the x-axis in the frame. dimensions. Then

(ii) Find the speed of the ball with respect to the surface, (A) the path of the particle is an ellipse
if φ = 4θ / 3 . (B) the velocity and acceleration of the particle are
normal to each other at t= π / 2p
Q.10 A train is moving along a straight line with a
(C) the acceleration of the particle is always directed
constant acceleration a. A boy standing in the train
towards a focus.
throws a ball forward with a speed of 10 m/s, at an
angle of 60° to the horizontal. The boy has to move (D) the distance travelled by the particle in time interval
forward by 1.15 m inside the train to catch the ball back t=0 to t= π / 2p is a.
2 . 4 8 | Motion in a Straight Line

14. A rocket is moving in a gravity free space with a relative to the rocket. The time in seconds when the two
constant acceleration of 2 ms–2 along + x direction (see balls hit each other is  (2014)
figure). The length of a chamber inside the rocket is
4 m. A ball is thrown from the left end of the chamber a = 2ms
-2

in + x direction with a speed of 0.3 ms–1 relative to the -1 -1


rocket. At the same time, another ball is thrown in -x 0.3 ms 0.2 ms
x
direction with a speed of 0.2 ms–1 from its right end

4m

PlancEssential Questions
JEE Main/Boards JEE Advanced/Boards
Exercise 1 Exercise 1
Q. 9 Q.12 Q.15 Q.2 Q.5 Q.7
Q.25 Q.27 Q.9 Q.10 Q.17

Exercise 2 Exercise 2
Q.3 Q.4 Q.7 Q.4 Q.17 Q.18
Q.12 Q.19 Q20

Previous Years’ Questions Previous Years’ Questions


Q.1 Q.5 Q.1 Q.3 Q.5
Q.8

Answer Key

JEE Main/Boards Q.5 24m

Q.6 66.67m
Exercise 1
Q.7 4.5 s
Q.1 110m
Q.8 2.5 m
Q.2 0.25 ms–2
Q.9 2/3 ms–2, 4 ms–2, 65 sec
Q.3 136 m
Q.10 75 m from ground
Q.4 2/3 ms–2
Q.11 3 sec
P hysi cs | 2.49

Q.12 (i) 8.36 s (ii) 7.33 s (iii) 7.82 s Q.21 Approximately, 3000 ft/s2 (in the upward direction)

Q.13 8m, 16m Q.22 < v >time = v3v/5


/3
2
ac − b
Q.14 100m, 60m Q.23
x3
Q.15 0 ms–1; 10 ms–1; 15 ms–1 Q.25 2 min
2v 0 (v1 + v 2 )
Q.16 12 second Q.26
v1 + v 2 + 2v 0
Q.17 10 ms–2
Q.27 45 m, 20 m, 5 m
Q.18 36 ms ; 14 ms ;
–1 –1

Q.28 68.5 m
Q.20 18 ms–1

Exercise 2

Single Correct Choice Type

Q.1 C Q.2 B Q.3 D Q.4 C Q.5 C Q.6 D

Q.7 D Q.8 B Q.9 C Q.10 B Q.11 C Q.12 D

Previous Years’ Questions


Q.1 B Q.2 C Q.3 A Q.4 B Q.5 A Q.6 B

Q.7 A Q.8 B Q.9 C Q.10 A Q.11 A Q.12 B

JEE Advanced/Boards
Exercise 1 Q.11=t t1 (2 + 2)

 1 1
Q.1 11.43 sec,-3.06 ms−2 Q.12 1 + +  : 1
 m n
Q.2 105 m/s
α2 t α2  α S
Q.13=
(i) v = ,a (ii) v =
Q.3 1:3:5 2 2 2
Q.4 25m Q.14 2 m

Q.5 -16 cm / s2 Q.15 36 s

Q.6 5 s Q.16 36 m/s, 12 m/s

Q.7 36.2 sec Q.17 (i) 300 ft (ii) 60 ft/s

( v1 + v 2 )
2
Q.8 h / 8g
Q.18 lmax =
−1 2(a1 + a2 )
Q.9 tan (3)
Q.19 t=75 s
Q.10 50 3cm
Q.20 Q can not catch P
2 . 5 0 | Motion in a Straight Line

Exercise 2

Multiple Correct Choice Type

Q.1 A, C, D Q.2 A, C Q.3 A, C Q.4 A, D Q.5 A, B, C, D

Q.6 A, B, C Q.7 C, D

Assertion Reasoning Type

Q.8 A Q.9 D Q.10 A Q.11 B

Comprehension Type

Paragraph 1: Q.12 B Q.13 C Q.14 A Q.15 A Q.16 A

Paragraph 2: Q.17 B Q.18 D Q.19 C

Match the Columns

Q.20 A → p, s; B → r ; C → q ; D → s

Previous Years’ Questions


1
Q.1 Just midway between A and B, 3.53 s Q.2
2
Q.3 (i) Approximately 17.32 m (ii) 11.55 m from B Q.4 (i) 1s (ii) (5 3m,5m)

Q.5 (i) 2.5 m/s (ii) 0.32 m Q.6 (i) -0.1 m/s (ii) 1.48 s

u2 sin2α u cos(α + θ) 


Q.7 (i) (ii) (down the plane) Q.8
= u (3.75iˆ + 6.25ˆj)m/ s,1s
gcos θ cos θ

Q.9 (i) 45° (ii) 2 m/s Q.10 5

Q.11 B Q.12 A, C

Q.13 A ,B, C

Solutions

JEE Main/Boards Sol 2: We use the formula v 2 − u2 =


2as
5 −1 −1
Exercise 1 u = 36km h−1= 36 × ms = 10 ms
18
v=0
 1
Sol 1: We use the formula Sn =+
u an −  s = 200m
 2
02 − 102 =
2a.200
u = 0 (body starts from rest)
a = −0.25 ms−2
a = 20 ms−1
∴ Retardation is 0.25 ms−2
n=6
 1
Sn =
0 + 20  6 −  = 110 m
 2
P hysi cs | 2.51

 1 v= u2 + a(20)
Sol 3: Sn =+
u an −  2
 2
u=
2 v=
1 u1 + a(30)
 1 3
12 =u + a  2 −  ⇒ 12 =u + a......................(1)
 … (i) ⇒ v 2 = u1 + a(30) + a(20)= u + 50a
 2  2 1

 1 7 Substituting in (iii)

20 =u + a  4 −  ⇒ 20 =u + a......................(2) … (ii)
2 2

(u1 + 50a − u1 )(u1 + u1 + 50a) =
4as
Eq (2) – Eq (1)  7   3 
20 − 12 =  u + a  −  u + a  50a ( 2u1 + 50a) = 4as
 2   2 
 7   3  ⇒ 50(2u1 + 50a) =
4s
⇒ 20 − 12  =  u7+  a  −  u3+7−a3
20 − 12 =  u + a2 − ⇒ u=+8 a2 = 2a
  2    2 2  s = 1000m
73− 3 8
⇒= ⇒
8 =87 −= =2a 2a⇒ a= = 4ms−2
2 2 2 ⇒ 50 ( 2u1 + 50a) =
4 × 1000
a
=8 8= 4ms −2
−2
a= = 24ms in (i)
Substituting 4000
2 ⇒ 2u1 + 50a= = 80
50
3
12= u + .4 ⇒ 2u1 + 50a =
80
2
3 Dividing by 2 on both sides
u = 12
=
12 u−+ .4 = 12-6 ms−1 =6 ms−1
2
u1 + 25a =
40 ....................(4)
 … (iv)
3
u 12 − .4
=
2 st = u(t2 ) + 1 at22  − u(t1 ) + 1 at12 
   
∆s= st − v12 − u12 =
2as
2 1
 2   2 
∆=
1
s u(t2 − t1 ) + a(t22 − t12 ) ( v1 − u1 )( v1 + u1 ) =
2as
2
v1 =
u1 + 30a , s=
1000
 1   1 
∆s= st
2
− st = u(t2 ) + at22  − u(t1 ) + at12 
1
 2   2 
(u1 + 30a − u1 )(u1 + 30a + u1 ) =2a × 1000
30a(2u 1 +30a) =
2000a
1
∆=s u(t2 − t1 ) + a(t22 − t12 ) 200
2 2u1 + 30a =
t2 a,=
= t1 5 3
1 1 100
∆s = u(9 − 5) + a(92 − 52 ) = 6(9 − 5) + 4(92 − 52 ) 
u1 + 15a = ..................(5) … (v)
2 2 3
= 6(4) + 2(56)
∆s =136m Subtracting (iv)-(v)

∴ Distance covered by body in 4 seconds after 5th 100


u1 + 25a − (u1 + 15a) = 40 −
second is 136 m 3
2.0
10a =
Sol 4: v12 − u12 2as...........(1)
=  … (i) 3
2
v1 =
u1 + a(30); −30 seconds travel a=
3
v 22 − u22 =
2as...........(2)
 … (ii) 2 −2
acceleration of car is ms
v2 =
u2 + a(20); −20 seconds travel 3

Here v1 = u2 1
Sol 5: s=
1 u1 t + a1 t2 ………..for car 1
2
∴ adding (i), (ii)
1
s= u2 t + a2 t2 …….....for car 2
v12 − u12 4as
= 2
2
s1 = s2
( v 2 − u1 )( v 2 + u1 ) =
4as .................(3) … (iii)
2 . 5 2 | Motion in a Straight Line

1 1  1
∴ u1 t + a1 t2 = u2 t + at2 20 a  t1 −  .............(1)
= … (i)
2 2 2
 
−1
=u1 2ms
= a1 2ms−2
  1 1
−1 40 =
40
a =
t2a− t2 −  , Also t2 =
tt21 =
+ t11 + 1
=u2 4ms
= a2 1ms−2   2 2
1 1 (body travels 40 m in here 1 second)
⇒ 2(t) + 2t2 =4(t) + 1(t2 )
2 2
 1
1 1 ⇒ 40
= a  t1 + 1 − 
⇒ (4 − 2)t = (2 − 1)t2 ⇒ 2t = t2  2
2 2
1  1
⇒ 2= t ⇒ t = 4s ⇒=40 a  t1 +  ...................(2) … (ii)
2  2

1 1 Subtract (ii) – (i)
⇒ s=
1 u1 t + a1 t2 = 2(4) + 2(4)2 = 8 + 16 = 24m
2 2
Length of path is 24m  1  1
40 − 20= a  t1 +  − a  t1 − 
 2  2
Sol 6: We know that v 2 − u2 =
2as , Given that
 1  1
−1 −1 −1 −1 20 = a  t1 +  − a  t1 − 
=u 72kmh
= 20ms
= , v 36kmh
= 10ms  2  2

s = 200m , ⇒ 102 − 202 =


2a(200) a = 20ms−2
Substitute in (i)
100 − 400 −3 −2
=⇒ a = ms
2 × 200 4  1 1
20
= 20  t −  ⇒=
t s
v 22 − u22  2 2
s2 = (Distance travelled further)
2a 2
1 2 1 1
v2 = 0 S= at = .20.   = 2.5m
2 2 2
u2 = 10ms−1
0 − 102 −100 200 Sol 9:
=
⇒ s2 = = m  66.7m
=66.7 m
 −3  −3 3
2× 
 4  2
v
Sol 7: Since he stops 1m from child, distance travelled II
I III
is (100-1)=99m
0 t
So, we have, S = 99m, u = 44 ms−1 t₁ t₂ 25
v 2 − u2 =
2as t1 = 30 s
v=0
5
v = 72kmh−1 = 72 × = 20ms−1
−(44)2 −u2 v − u 0 − u u.2s 2s 18
=⇒ a = ⇒
= t = = =
2.100 2s a u2 u2 u
− v 20 2 −2
2s a
= = = ms
t1 30 3
2 × 99
=t = 4.5s
44 Let distance travelled in acceleration be s1

1 2 1 2 2
 1 ⇒ s1 = at = . .30 = 300m
Sol 8: s =+
u a t −  2 2 3
 2
Let distance travelled in uniform velocity and retardation
∵ body starts from rest, u=0 be s2 ,s3 respectively
 1 s3 =50m
20 =0 + a t − 
 2
P hysi cs | 2.53

Total distance travelled s=950m 1 100


= 100 − g. =100-25 = 75m
⇒ s2 =s − ( s1 + s3 ) = 950 − (300 + 50) = 600 2 2g

s2 600 ∴ Balls meet at 75 m height


t=
2 = = 30s
v 20

Retardation
retardation = r
v2
=
202
= 4ms−2
Sol 11: ∆s =
1
2
( )
g t22 − t12 t2 = 2 + t1
2s3 2 × 50
∆s is distance between travelled between t2 and t1.
v 20
⇒ t3= = = 5s 1
r 4 ⇒
= ∆s g ( t2 − t1 )( t2 + t1 )
2
1
∴ Total
∴ total timetime t = t1 + t2 + t3 = 30 + 30 + 5 = 65s = g ( t1 + 2 − t1 )( t1 + 2 + t1 )
2
2 −2
∴ acceleration
∴ Acceleration is
is ms , Retardation is 4 ms−2
3 1
= g ( 2 )( 2t1 + 2 )
2
Total time taken is 65 s.
⇒=∆s g ( 2t1 + 2 ) and we are given that ∆s =78. 4

Sol 10: ⇒ 78.4 = g ( 2t1 + 2 )


s₁
78.4 78.4
2t1 +=
2 = = 8 (g
= 9.8)
point of meeting g 9.8
2t1= 8 − 2
s₂
t1 = 3s
∴ Time travelled by ball before it was observed is 3s

For ball thrown from top, let distance travelled be s, Sol 12: (i) Given that h = 300 m. We know that
time taken t, 1
= ut + gt2
h
1 2
s1 = gt2
2 If balloon is ascending ⇒ initial velocity of stone is
−5ms−1
For ball thrown from bottom, let initial velocity be v,
given it just reaches top. 1
⇒ 300=-5(t)+ ( 9.8) t2 ⇒ 4.9 t2 -5t-300=0
=⇒v 2gs = 2
(s 100m)
2
It is a quadratic equation in t
−1
⇒ v = 2g(100) ⇒ v = 10 2g ms
−b ± b2 − 4ac
∴t = ; b=-5, a = 4.9, c=-300
Let s2 be distance travelled by ball 2, when it meets ball 2a
1,
1 2 1
s= vt −= gt 10 2g t − gt2 −( −5) ± 52 − 4(4.9)( −300) 5 ± 25 + 5880
2
2 2 ⇒ t= =
2(4.9) 9.8

s1 + s2 =
100m 5 ± 76.844
⇒ t=
9.8
1 2 1
∴ 100 = gt + 10 2gt − gt2
2 2 For solution to be real, t>0
100 10
=t = 5 + 76.844
10 2g 2g t= = 8.36s
9.8
1
s=
2 vt − gt2
2 (ii) If balloon is descending ⇒ initial velocity of stone
2 is 6 ms−1
 10  1  10 
= 10 2g   − g. v  
 2g  2  2g 
   
2 . 5 4 | Motion in a Straight Line

1 v(2)=2b(2)=4b=4(2.5) = 10 ms−1
2
⇒ 300 =5t + 9.8t2 ⇒ 4.9t + 5t − 300 =
0
2 v(2)= 10 ms−1
displacement
−5 ± 52 − 4(4.9) − (300) Average velocity =
⇒t= time
2(4.9)
x(4.01) − x(2) a + b(4.01)2 − (a + b(2)2 )
For t > 0 = =
4.01 − 2 4.01 − 2

⇒t=
−5 + 25 + 5880
= 7.33 s b  2
2.5 (16.0801 − 4 )
 ( 4.01 ) − 2  =
2
9.8 =
2.01   2.01
(iii) If balloon is stationary ⇒ initial velocity is 0 ms–1 = 15. 025 ms-1 ≅ 15 ms-1

1 300 × 2 ∴ Average velocity= 15 ms−1


2
⇒ 300 = .9.8t= ⇒t = 7.82 s
2 9.8
Sol 16:
Sol 13: Displacement travelled equals to area under v-t dx d  −2  2
graph v = =  t2 + 16t + 2  =
−2   t + 16
dt dt  3  3
∴ S =∑ vt =4(2)+(-2)(2)+2(2) = 8m
4
∴ v(t) =
− t + 16
Displacement = 8 ms−1 3
Distance travelled ∑ | vt | When body comes to rest, v(t)=0
=|4(2)|+|(-2)(2)|+|2(2)| = 8+4+4 =16m
−4 4
⇒ 0 ⇒ 16 = t ⇒ t =
t + 16 = 12
3 3
Sol 14: Displacement is area under v-t graph for
0≤t≤6 ∴ Body takes 12 seconds to come to rest.

1 1
s1 = × 6 × 20 (area of triangle = x base x height) dy d
2 2 Sol 17: v y = = (8t − 5t2 ) =−
8 10t
dt dt
=60m
1 v y (0) = 8 ms−1
s2
For 6 < t < 8,= (2)( −20) =-20m
2
dx d
For 8 < t < 10, s3 =2 × 10 =20m v=
x = = 6 ms−1
(6t)
dt dt
3
Displacement = ∑ si v= v 2x + v 2y = 82 + 62 = 10
i=1

= s1 + s2 + s3 =60-20+20 = 60 m ∴ Initial velocity is 10 ms−1


3
Distance= ∑ | si |
i=1 Sol 18: x =3 + 8t + 7t2

= | s1 | + | s2 | + | s3 | =|60|+|(-20)|+|20| dx d
Velocity
velocity(v) = = (3 + 8t + 7t2 )
dt dt
= 60+20+20 = 100m
v(t)
= 8t + 14t
∴ Distance travelled is 100 m and displacement is 60 m
v(2) = 36 ms−1
8 + 14(2) =
Sol 15: x(t)= a + bt2 dv d
a = = (8 + 14t) =14ms−2
dt dt
dx d
(
v = = a + bt2 =2bt
dt dt
) ∴ Velocity = 36 ms−1
Acceleration =14 ms−2
V(t)=2bt
Body is having a constant acceleration.
v(0)=0
P hysi cs | 2.55

Sol 19: Given that t = αx2 + βx


dt =a
v −u
=
2gh2 − − 2gh1 ( ) =
2g ( h2 + h1 )
⇒ = 2αx + β t t t
dx
g = (9.8)0.3 = 2. 94 (1 ft = 0.3 m)
dx 1 1
⇒= =
dt  dt  2αx + β
 
 dx  ∴a =
2(2.94) ( 4− 3 ) ≅ 3000ft / s 2
0.01
dv dv dx dv

= = . .(v)
dt dx dt dx Sol 22: Let a = c1 t

1 (since uniformly increasing from zero)


⇒ v=
2αx + β
v = ∫ a.dt = ∫ c1 t.dt
dv −1
=
⇒ . ( 2α ) c1 t2
dx
( 2αx + β )
2
v(t)
= + c2
c
dv −2α 2 v(0) = c2
∴ = v ⇒ =−2α(v) (v)
dt
( 2αx + β )
2
Given v(0)=0 (body starting from rest)
a =−2αv 3 c1 t2
⇒ c2 = 0 ⇒ v(t) =
Retardation = −a = −( −2αv 3 ) = 2αv 3 2
c1 t2
x = ∫ v.dt = ∫ .dt
∴ Retardation = 2αv 3 2
c1 t3
x(t) + c3 ( c3 some constant)
∫ (3t )
2 =
Sol 20: v = ∫ a.dt= + 2t + 2 .dt 6
Time average of velocity is given by
v(t) = t3 + t2 + 2t + c
t0
Given v(0) = 2 ms−1 c1 t2 c1 t3 t
v.dt ∫0
.dt + c3 0
3 2
v(0) = 0 + 0 + 2(0) + C = C v
=
∫= 2
=
6 0

−1
6
∫ dt t
∫0 dt t 0t
⇒C=
2 ms

⇒ v(t) = t3 + t2 + 2t + 2 c1 t03  c (0)3  3


+ c3 −  1 + c3  c1 t0
6  6  c1 t02
⇒ v(t) = 23 + 22 + 2(2) + 2 = 18ms−1 =  =  =6
t0 − 0 t0 6
∴ velocity at the end of 2s is 18ms−1
We have v(t0 ) = v
Sol 21: Let u be the velocity upon reaching ground c1 t02 2
= v ⇒ c1 t0 = 2v
u 2gh1
= (h1 4 ft) 2
2v v
∴ v = =
Let v be the velocity upon rebounding from ground t 6 3
v
=v 2gh2
= (h2 3 ft) ∴ v =
t 3

But v is in upward direction, u downward. Distance average of velocity is given by

So for sign convention let’s take upward positive


v =
∫ v.dx
⇒ u =− 2gh1
x
∫ dx
c1 t3 c1 t2
Time t = 0.01s x(t)
= + c3 ⇒ dx = .dt
6 2
2 . 5 6 | Motion in a Straight Line

c1 t2 c1 t2
t0 Substituting in (ii)
∫0 . .dt
∴ v
x
=2
t0 c t
2
2
a−
(ax 2
− ac + b2 )
∫0 12 .dt ac = x 2
=
ax2 − ax2 + ac − b2
x x3
c1 2
t0 4 t5 t0 t05
2 ∫0 t .dt
c1 5 0 c1 ac − b2
= 4 = . = . 5 ∴ ac =
c1 t0 2 2 t3 t 0 2 t 3 x3
2 ∫0 t .dt
3 0
0
3

3 3 Sol 24: Lets take downward as positive


= c1 t02 = .2v
10 10 a(t)=g

3 1 2
v = v h= gt
x 5 2
3 h = 19. 6 m
Distance average of velocity is v
5 g =9. 8 m

Sol 23: x2 = at2 + 2bt + c 2h


⇒ t1 = ( t1 = time of decent)
g
Differentiating by ‘t’ on both sides
dx 2(19.16)
⇒ 2x = 2at + 2b ⇒ 2xv
= 2at + 2b = = 2 sec
dt 9.8

xv= at + b.............(1) … (i) Time of ascent ( t2 ) = time of decent ( t1 )
Differentiating by ‘t’ on both sides ∴ t2 = 2 sec
dv  dx  Total time = t1 + t2 =2+2 = 4 sec
⇒ x. + v  = a
dt  dt 
a
x.ac + v(v) a
= =(ac acceleration)

a − v2
⇒ ac = ...............(2) … (ii) 9.8
x
Coming back to (i)
at + b 0 t
v= 4
x
Squaring on both sides =v gt 0≤t<2

 at + b 
2
a2 t2 + 2a + b + b2 For ascent uint = − v(2) = - g(2)
2
v =  =
 x  x2 u = 19.6 ms−1
v = u+g(t-2) = 19. 6 + g(t-2); 2 < t < 4
2 a(at2 + 2abt) + b2
v = .............(3)
 … (iii)
= v gt; 0≤t<2
x2
= 19.6 + g(t − 2); 2<t≤4
x2 = at2 + 2bt + c

at2 + 2bt =x2 − c

Substituting in (iii)
a(x2 − c) + b2
v2 =
x2
P hysi cs | 2.57

v
202 − v 2  v 2 − u2 
=200 = .............(2)  s  … (ii)
19.6 2a1  2a 
 
202 − v 2  v 2 − u2 
=200 = .............(2)  s 
2a1  2a 
0 t 
202 − v 2
600 = ..............(3) … (iii)
2a

-19.6
Divide (ii) by (iii)

1 2 202 − v 2
S gt 0≤t<2
2 200 2a1 1 a
⇒ = ⇒ = ⇒ a1 = 3a
The motion of ball is symmetric about t=2 i.e., if we 600 202 − v 2 3 a1
make t=4 as initial point, and reverse the motion of 2a
time ascent looks like decent.
Substitute in (i)
x
400  1 1 400  4
⇒ (20 − v)  +  ⇒ = (20 − v)  
=
v  3a a  v  3a 
19.6
⇒ (20 − v)(v) = 
300a............(4) … (iv)
From eq (iii)
0 t
2 4 202 − v 2
600 =
2a
Graph is symmetric about 2
1 2 (20 − v)(20 + v)
S gt 0≤t<2 a=
2 1200

1 Substitute in (iv)
g(4 − t)2 2<t≤4
2 300(20 − v)(20 + v)
(20 − v) =
1200
Sol 25: Let r = retardation = −a1
1
v
⇒= (20 + v) ⇒ 4v = 20 + v
Let a = acceleration 4
Let t1 = retardation time 20 400
⇒ v = ms−1 ⇒ t2 = = 60 s
t2 = uniform velocity time 3 20
t3 = acceleration time 3
Total time=2 t2 = 120 s = 2 min
Let v = uniform velocity

400  s Sol 26: Let v 0 velocity be for time t.


t2 = t 
v  v
Let each of half time be tn
20 − v  v −u
t1 = t  v 0 t v1 tn + v 2 tn ⇒ ( v1 + v 2 ) tn =
= v0t
a1  a 
v0t
⇒ tn =
20 − v v1 + v 2
t3 =
a total distance
mean velocity
Mean velocity ==
t2 = t1 + t3 total time
400 20 − v 20 − v total dis
Total tance =
distance = 2v 0 t
⇒ = +
v a1 a
Total
total time
time
== 2th + t
400 1 1
⇒ =(20 − v)  +  ................(1) … (i)
v  a a 
2 . 5 8 | Motion in a Straight Line

2v 0 t  2v 0   v1 + v 2 + 2v 0  ⇒ It is always pointing towards origin.


= t+ = t  1 +  = t  
v1 + v 2  v 1 + v2   v1 + v 2 
t2 − t;
Sol 2: (B) v = a= 2t − 1
Mean velocity 1
a<0 0<t<
2v o t 2v o ( v1 + v 2 ) 2
= = 1
 v + v 2 + 2v 0  v1 + v 2 + 2v 0 a>0
2
< t <1
t 1 
 v1 + v 2   Retarding means coming back to original position,

1 2 1
Sol 27: s = gt (s is distance from top) For < t < 1 , body is trying to go back to original
2 2
position.
Time of flight of 5th ball = 1 sec
Time of flight of 4th ball = 2 sec
Sol 3: (D) The acceleration on the ball is acceleration
Time of flight of 3th ball = 3 sec due to gravity which is always pointed downward,
1  2 2 2 1 towards the earth.
s= g 1 2 3 = g 1 4 9 
2   2 Hence ay < 0 always.
−2
take g = 10 ms
Sol 4: (C) Here x (t) = - sin (t)
s = 5 1 4 9  = 5 20 45
dx
∴ 3rd , 4th , 5th ball are at 45, 20 , 5 m from top v(t) = = − cos(t)
dt

Sol 28: initial velocity of stone = 5 ms−2 Sol 5: (C) Average velocity vector is along the direction
Height = 50 m of the line joining the instantaneous point and the
starting point.
1
−5(t) + 10t2
⇒ 50 = Instantaneous velocity vector is along the slope at the
2
instantaneous point.
⇒ 5t2 − 5t − 50 =
0 ⇒ t2 − t − 10 =0
Both are same for point C.
1 ± 1 + 4(10)(1) 1 ± 41
⇒t= =
2(1) 2 Sol 6: (D) Here we use the symmetry of motion. i.e.
from ending point if you go reverse in time, it looks like
1 + 41 train is accelerating.
for t > 0 t =
2
So here for minimum time, train should accelerate for
Distance travelled by balloon in this time = 5(t) half the distance and then decelerate.
Total height of balloon 50 + 5t
d 1 2 d
=∴ d 1 at= ; t d
 41  = 2
∴ 2 2 at= ; t a
=50 + 5  1 +  = 68.5 m 2 2 a
 2  d
 =T 2t=
; T 2 d
=T 2t=
; T 2 a
a
Exercise 2
Sol 7: (D) The equation of line is
Single Correct Choice Type
a + kV= C, where k is slope of line, C is y intercept.
d(x2 ) dx dv
Sol 1: (C) = 2x − <0 a=
dt dt dt
2x − v < 0 dv dv
⇒ =C − kv; =dt
 if x > 0 ⇒ v < 0 dt C − kv
if x , 0 ⇒ v > 0
P hysi cs | 2.59

Intergrade on both sides f(x) is a quadratic equation


dv f(x) > 0 ∀ x
∫ C − kv = ∫ dt
1 1 1
− log(C
n (C−−kv)
kv)=tt
= =D'(t) .f '(x); >0∀x
k 2 f(x) 2 f(x)
C − kv =e − kt D'(t) = g. f '(x)
C − e−kt g is some positive variable.
⇒ v= it corresponds to option [D]
k
Hence slope of D(t) is similar to f(x) which is quadratic
equation.
2h
Sol 8: (B) Time of flight = 2.
g So nearest graph is D.
∴T∝ h
Hence the one which reaches maximum height flies for
Previous Years’ Questions
longer time.
Sol 1: (B)
Sol 9: (C) Horizontal velocity of ball= velocity of bird=u P C Q

2h
Time of flight (t) = 2 y
g
 
2h
∴ Dis tance =
2u
g
R

M
Sol 10: (B) We may use the formula U U

1 1 1 In the right angle DPQR


= +
T t1 t2 2
= c2 + y 2

Differentiating this equation with respect to time, we


Sol 11: (C) Lets take upward as positive get
For 0 < t < T dy
d
2 = 0 + 2y
VA= V − gt dt dt
VB = −gt
 dy    d 
VAB = VA − VB = V − gt + gt Or  –  = – 
 dt  y  dt 
VAB = V
dy
After T, ball B sticks to ground Here, – = vM
dt
∴ VB =
0
 1 –d
∴ VAB =−
v gt = and =U
y cos θ dt
=∴ VA v 0<t<T
v − gt
= T<t U
Hence, vM =
cos θ
∴ Corresponding graph is C → → →
 vf – vi
∆v
Sol 2: (C) aav = =
Sol 12: (D) Distance D(t)= (A − ax t)2 + (B − ay t)2 ∆t ∆t

ax, ay are accelerations in x, y

L et (A − ax t)2 + (B − ay t)2 =
f(x)
D(t) = f(x)
2 . 6 0 | Motion in a Straight Line

N
Displacement
Sol 6: (B) |Average velocity| =
time
AB 2
W E = = = 2m/s
time 1

Sol 7: (A) (a) For uniformly accelerated/decelerated


S motion

 v2 = u2 ± 2gh
V 
vf=5m/s
i.e., v-h graph will be a parabola (because equation is
5 2 m/s quadratic).
(b) Initially velocity is downwards (–ve) and then after
collision it reverses its direction with lesser magnitude,
45° i.e., velocity is upwards (+ve). Graph (a) satisfies both

-vi=5m/s these conditions.

Note that time t = 0 corresponds to the point on the
∆ v = 5 2 m/s in north-west direction.
graph where h = d
→ 5 2 1
aav = = m/s2 (in north-west direction) at t=0, h=d
10 2
2
Sol 3: (A) To cross the river in shortest time one has to d
h
swim perpendicular to the river current. 3 1

Sol 4: (B) Shortest possible path comes when absolute Collision 2


velocity of boatman comes perpendicular to river takes place
current as shown in figure. here

vr Sol 8: (B) Area under acceleration-time graph gives
the change in velocity.

vb River 1

var Hence, vmax = × 10 × 11 = 55 m/s
current 2

ω ω
t= = Sol 9: (C) Distance travelled in tth second is,
vb 2
v br – v r2
1
s1 = u + at – a
1 1 2
∴ =
4 Given, u = 0
25 – vr2
1
sn an – a
2 2n – 1
Solving this equation, we get vr = 3 km/h ∴ = =
sn+1 1 2n + 1
a(n + 1) – a
2
Sol 5: (A) For particle P, motion between AC will be
an accelerated one while between CB a retarded one. Sol 10: (A) The v-x equation from the given graph can
But in any case horizontal component of its velocity be written as
will be greater than or equal to v. On the other hand,
in case of particle Q, it is always equal to v. Horizontal  v 
v = – 0  x + v0  … (i)
displacement for both the particles are equal. Therefore,  x
 0 
tP < tQ.
dv  v 0  dx  v 
a= =–  = – 0  v
 
dt  x0  dt  x 
 0
Substituting v from Eq (i) we get,
P hysi cs | 2.61

 v0   v 0   Velocity of bullet with respect to ground = (V1+150) ms-1


a =  –   –  x + v 0 
 x0   x0   Velocity of bullet with respect of thief
2 = Vel. w.r.t. ground – Vel. of thief =V1+150-V2
v  v 20
a=  0  x – 5
x x0 Velocity of thief (V2) = 192 km/h
= 192 × ms−1
 0  18
Thus, a-x graph is a straight line with positive slope and
∴ Velocity with which bullet will hit thief
negative intercept.
5 5
Sol 11: (A) Time to reach the maximum height = V1+150 –V2 = 150 + 36 × − 192 ×
18 18
u
t1 = 5 5
g = 150 +
18
(36 − 192) = 150 − 156 × 18 = 105 ms−1
If t2 be the time taken to hit the ground
1 1 2
−H= ut2 − gt22 Sol 3: =
S gt ⇒ S ∝ t2
2 2
Let S1=distance travelled in 1 second similarly define
But t2 = nt1 (given)
S2 , S3.
nu 1 n2u2 1 g
H u
⇒ −= − g ⇒ 2gH = nu2 (n − 2 ) =S1 = g(1)2
g 2 g2 2 2

1 1 1 g
y1 10t − gt2 and =
Sol 12: (B) = y 2 40t − gt2 =S2 = g(2)2 (4)
2 2 2 2
y2 − y1 = 30t (straight line)
1 g
but stone with 10 m/s speed will fall first and the other = S3 = g(3)2 (9)
2 2
stone is still in air. Therefore path will become parabolic g
till other stone reaches ground. Distance travelled in 1st second D1 =
2
Distance travelled in 2nd second D2 = S2 –S1
JEE Advanced/Boards g 3g
= (4 − 1)=
2 2
Exercise 1
Similarly D3 = S3 − S2 = g (9 − 4) = g (5)
Sol 1: Initial velocity
5 =126
−1 km/h 2 2
= 126of×car ms
18
5
= 126 × ms−1 = 35ms−1 g 3g 5g
18 D=
1 : D 2 : D3 =: : 1:3:5
2 2 2
= 35ms−1
Distance =200 m
∴ Ratio is 1: 3: 5
v2 35 × 352
=r2 = v 35 × 35 = 3.0625 ms–2
v =2s 2rs ⇒2
= ×r 200=
2s 2 × 200 Sol 4: Locomotive stops when V=0
= 3.0625ms−2
= 3.0625ms−2
v 35 given u = 54 km/hr=15 ms-1
E= = =v 11.43s
35
r 3.0625 E= = = 11.43s retardation (r)= 0.3 ms-2
r 3.0625
∴Retardation = 3.06 ms-2 uu 15 15
Hence =t === == 50
t= 50s s
r r 0.3
0.3
Time taken = 11.43 s
Distance travelled
30 × 5 −1 1 1
Sol 2: Velocity of police van (V1) = 30 km /h = ms ut − rt2 =
S= 15(50) − (0.3)(50)2 =
375m
18 2 2
Velocity of bullet with respect to police van =150 ms-1 Distance of traffic light (L) =400 m
2 . 6 2 | Motion in a Straight Line

Hence final distance of Locomotive= Now is you observe the graph, region I ( 0 < t < t1) is
symmetric to region II ( (t2 < t < 25)
L – S = 400-375=25 m
5 2
∴Locomotive is 25 m from traffic light. ∴ for t2< t < 25 area = t
2 1
∴ total Displacement
Sol 5: initial position = 3 cm
Final position =-5 cm 5 5
= t12 + 5t1 (25 − 2 t1 ) + t12 =5t12 + 125t1 − 10t12
2 2
Displacement = -5-(3)=-8 cm
= 125t1 − 5t12
Time =2 s
Avg velocity =20 ms-1
Initial velocity = 12 cms-1
1 125t − 5t12
= ut + at2
S ⇒ 20 = 1
2 25
1 2
⇒ 5t1 − 125t1 + 100 =
0
−=8 12(2) + a(2)2
2 ⇒ (t1 − 5)(t1 − 20) =
0
⇒ a =−16cms−2 ⇒ t= 5 , t= 20
1 1

Acceleration is −16cms−2
for t1=5 , ⇒ t2=20 or t1=20 , ⇒ t2=5
⇒ t1 < t2 ⇒ t1=5
Sol 6:
Hence the value of t is 5 sec.

Sol 7:
v

I II III
20
0 t a₂
t₁ t₂ 25

total Displacement a₁ I II
Average Velocity =
total time
total time (t) = 25 s 20 25
25
total displacement = area under v-t. graph = ∫=
v.dt
a2 =
0 − 20 
a
∆ν 

0 25 − 20 t 

V0=5(t1)
a2 = −4ms−2
Final acceleration equal declaration,
∵ Particle returns to initial position, its displacement is
V0 -5 (25 – t2)=0
zero
⇒ 5t1-5(25-t2) =0
∴ ∫ v.dt =
0
⇒ t2=25-t1
t1
For 0 < t =
< t1 area 1
∫=
5 t.dt (v at) for 0 < t < 20, area = × 20 × 20 =200m
2
0
t1
5t12 for t > 0 V = 20 - 4 ( t -20)
= ∫=
5 t.dt (v at) =
2
0
∵ v = u –at and its starts for t =20 s
5t2
For t1
= 1
< t < t2 area t t
2
t2 = ∫ [20 − 4(t − 20)].dt= ∫ (100 − 4t).dt
= ∫ 5t1 .dt= 5t1 ( t2 − t1=
) 5t1 ( 25 − 2t1 ) 20 20
t1 4
= 100(t − 20) − (t2 − 202 ) = 100t − 2t2 − 1200
2
P hysi cs | 2.63

total area = 0 3
⇒ cos θ1 =
⇒ 200 + [100t -2t2 -1200] = 0 10
1
⇒ 2t2-100t +1000 =0 ⇒ sin θ2 =
10
⇒ t2-50t+500=0
⇒ tan θ2 =3
⇒ t = 36.2 s ⇒ θ2 = tan−1 3

Sol 8: ∵the stone can reach height (H) = 4h, Hence wind stream is inclined at tan-13° with vertical.

It initial velocity
Sol 10: for particle 1
ν 2(4h)g
= v ( 2Hg ) S1= (300 - 20t) cm

Let time of flight be t for particle 2

1 2 S2 = (400- 20t) cm
Distance travelled by upper stone D1 = gt
2 Let S1 have unit vector î
Distance travelled by lower stone
1 3
D=
1
vt − gt2 ⇒ S=
2 (400 − 20t)iˆ − (400 − 20t) ˆj
2
2 2 2

D1 + D2 =
h
= (200 − 20t)iˆ − 3(200 − 10t) ˆj
1 1
⇒ gt2 − vt − gt2 = h
2 2 Separation = S1 - S2
⇒ h = vt ⇒ h = 8hg.t
= [(300 − 20t) − (200 − 2t)] ˆi + 3(200 − 10t)ˆj
h h

= t = = (100 − 10t) ˆi + 3(200 − 10t)ˆj
8hg 8g
= 10[(10 − t)iˆ + 3(20 − t)ˆj]
h
Hence time when two stone cross is . = 10 (10 − t)2 + 3(20.t)2
dist ance
8g
= 10 1300 − 140t + 4t2
d(t)
Sol 9: Let vertical direction limit vector be ˆj , for a quadratic equation, min occurs at − b = 140= 35
horizontal î 2a 2 × 8 2
Drop
2
 35   35   35 
d   = 10 1300 − 140   + 4   = 50 3cm
 2  2  2 
₂
₁ ∴ min separation is 50 3cm .

₁ + ₂ = 90
Sol 11: Final velocity (u) = at1
Let it further travel for t seconds, acceleration a.
Vcar = 2 î
Let initial displacement be S1.
Vdrops=6 ˆj
1 2
S1 = at
Vdrops w. r.t car = Vdrops - Vcar =−2iˆ + 6ˆj 2 1

ˆj.( −2iˆ + 6ˆj) Let further displacement be S2.


6 6
cos θ1
= = =
| ˆj | . | −2iˆ + 6ˆj | 2 + 62 .1
2 40 S2 =−
ut
1 2 1
at1 (t) − at2
at =
2 2

S1+S2=0  It comes back to initial position


2 . 6 4 | Motion in a Straight Line

1 1 α2
∴ at12 + at1 t − at2 = 0 dv = dt
2 2 2
⇒ t2 − 2t1 t − t12 = 0
α2
⇒v= t
2t1 ± 8t12 2
⇒t= =( ± 2 + 1)t1
2
α2 α2

= a ,v
= t
t>0 2 2
∴t1= ( 2 + 1)t1 t
t
(ii) s = ∫ v.dt
s = ∫ v.dt
Total Time = t +t1 0
0 t
t α2 2
Total Time
= ( 2 + 2)t1 s =α ∫ t.dt
s=∫ t.dt
2
0
20
2 2
Sol 12: Let max velocity = V, total distance = S 2 α t 2 s
sα t2
= ⇒= t2 s
s
= 4⇒= t α
s 4 α
V 2 = 2a ; a=acceleration s s α s
m Average velocity=s =s =α s
Average velocity= = t 2= s 2
mv 2 t 2 s 2
⇒a= α
2s α
nv 2
Similarly retardation r = α s
2s ∴ Mean velocity <v> =
2
Here we again used the principle, retardation is
acceleration in reverse time. V V
Sol
Time14:of Time
flightof=flight
2  = 2 g 
v 2s g  
∴Time of acceleration t1= =
a mv 1= 4v22 2av 2
11 at24v 2 1 .a.2av
dispalcement
dispalcement = = at2 2= .a. =2 =
v 2s 2
g2
Time of deceleration t2= = 2 2 g2 gg2
r nv 2 ×21 × (9.8)2
= 2 × 1 × (9.8)
= 2m
= = 2m
2
1 s s (9.8)2 (9.8)
Time of uniform velocity = s − − 
v m n
It falls 2m behind him.
s
Avg. Velocity =
2s 2s 1  s s Sol 15: Let distance be S
+ + s − − 
mv nv v  m n s
Vman =
V 90
=
1 1 s
1+ + Vescalator =
m n 60
Max
Max Velocity
Velocity  11 11
= 11+ +m + +n : 1 : 1
= Vman on moving escalator w. r. t ground= Vman+ Vescalator
Avg.
Avg. Velo city
Velocity  m n  1 1  s
= s + =
 90 60  36
Sol 13: (i) v = α x s s
Time= = 36
=
dv α dx α s v
= = . .v 36
dt 2 x dt 2 x
He can reach in 36 seconds.
v
= α
x Sol 16: Total distance to be covered= 180+180=360m
dv α α2 Let V1, V2 be velocities of trains
∴ = α= = a
dt 2 2 It they move in same direction, relative velocity = V1-V2
Opposite direction, relative velocity = V1+V2
P hysi cs | 2.65

360 Sol 20: Let time of travel be t


⇒ V1 − V2 = 360 = 24
⇒ V1 − V2 = 15 = 24 Distance travelled by P
15
360
⇒ V1 + V2 = 360 = 48 1 2
⇒ V1 + V2 = 7.5 = 48 S1 =+
1(t) t + t2
.2t =
7.5 2
⇒ V1 36m=
= / s , V2 12 m / s
⇒ V1 36m=
= / s , V2 12 m / s Distance travelled by Q

Sol 17: Let it overtake after time t 1 t2


S2 =9(t) + .1t2 =+
9t
2 2
Distance travelled by truck = 30 t
Hence S2 =S1 +33
1
Distance travelled by automobile = × 6t2 =3t2 t2
2 ∴ 9t + = t233 + t + t2 2
∴ 9t22+ = 33 + t + t
3t2=30t t 2
∴ 2 9t +
t −28t 2+ 33 = 33 + t + t2
⇒ t=10 s t = 0
22 − 8t + 33 = 0
t 2
2 − 8t + 33 = 0
⇒ Distance = 3(10)2= 300 ft t2 −216t + 66 = 0
⇒ t − 16t + 66 = 0
2 2
Vcar= 6t=60 ft s-1 b
t2 −−16t24ac+= 16=
66 0−24(66) =
256 − 264 =−8
b2 − 4ac = 16 − 4(66) = 256 − 264 = −8
It overtakes it at 300 ft distance. Its velocity is 60 ft s-1 . b2b− 4ac
 −24ac= < 02 − 4(66) =
16 256 − 264 =−8
 b − 4ac < 0
 b2 − 4ac < 0
Sol 18: Relative velocity (V) = V1 +V2
⇒ there is no real solution for t
Relative acceleration (a) = -(a1 +a2)
⇒ Q can’t catch P.
0 – V2=2 as
– (V1 + V2)2 = -2(a1 +a2) Imax
Exercise 2
2
(V1 + V2 )
⇒⇒dmax
Imax = Multiple Correct Choice Type
2(a1 + a2 )
Sol 1: (A, C, D) Let time to travel along each side = t
1 v∝1
Sol 19:
v∝ l Displacement A to F = vt
l
k Time to travel from A to F = st
⇒= v
k⇒=
(
= kv cons
l
(= cons tant )
ktant
)
l vt v
v −=2 2 × 10
−2 ∴ Avg. Avg vel =
∴ Velocity =
v = 2 ×10 ⇒  =1⇒  =1 st s
−2
−2 2 × 10 k
⇒ 2 ×10⇒ =k = Displacement by A to C = 3 Vt

2 ×10−2 Time for A to C = 2t


∴v =
 3vt 3
∴ Avg. Avg vel = =v
∴ Velocity
d 2t 2
v=
dt Displacement of A to B = vt
d
d = 2 ×10 2 ×10−−22 Time = t
dt = 
dt vt
−2
.d
.d== 2
2××1010−2.dt
.dt ∴ Avg. avg vel =
∴ Velocity = v
=
2 t
t
2 t −2
∫∫ dd== ∫∫ 22 ××10
10−2.. t.dt
t.dt
Sol 2: (A, C) The magnitude of velocity is same. Hence
1 1
1 1
22 2 change in magnitude is zero.
.. ||12== 2 × 10−−22.t |0tt
2 1 2 × 10 .t |0 Let initial velocity be V along: i-direction
2
1  
1 (222 − 122 ) = 2 × 10 −2
−2.t Final velocity = v cos θ i + v sin θ j
2 (2 − 1 ) =2 × 10 .t
2  
3 Change in velocity = v(cos θ − 1)i + v sin θ j
3 1022 = 75s
⇒ tt == 4 ×
⇒ × 10 = 75s
4
2 . 6 6 | Motion in a Straight Line

Magnitude of change in velocity Assertion Reasoning Type

= v (cos θ − 1)2 + sin2 θ


Sol 8: (A) Statement-II supports statement-I
θ
= v 2(1 − cos
= θ) 2v sin
2 Sol 9: (D) Constant speed means, body can have
v 10 2 2 different velocity as velocity is vector (change in
Sol 3: (A, C) Maximum displacement == = 10m direction). Hence body can accelerate.
2r 2 × 5
In the problem, the particle goes to maximum
displacement and comes back. Sol 10: (A) Statement-II supports statement-I
Displacement is minimum distance between initial and
final point Sol 11: (B) Both statements are true, but Statement-II
doesn’t explain statement-I
1
10(3) − 5.(3)2 =
d= 7.5m
2
Comprehension Type
But distance is total path covered by body i.e
D = ∑ | v | .dt Paragraph 1:
Here the body come back 2.5 m from maximum
displacement Sol 12: (B) V(t)=0 for t=0.8, 4.66 s (B)
∴ D = 10 + 2.5 = 12.5m
Sol 13: (C) Slope is maximum for 4 to 6
Sol 4: (A, D) Acceleration due to gravity along AB is ∆v
= 15
∆t
aAB= gcos θ; vB= gcos θt ∴ vB ∝ cos θ
AB 2r cos θ
=
Sol 14: (A) Position = area under V-T graph + initial
2S 2r cos θ 2r position.
=t = = 1
a gcos θ g Area under V-T graph for 0 < t < 2 = × 10 × 2 =10m
2
∴ t is independent of θ. Position =
∴∴ position =10 + ( −15) =−5m

Sol 5: (A, B, C, D) Sol 15: (A) The maximum displacement is at t = 4.66 as


•• Particle changed its direction of motion at t = T area under graph is positive.
•• Slope is constant. Hence acceleration is constant. 1 1 2
Displacement = × 2 × 10 + 10 × 2 + × × 10
2 2 3
•• Area under v-T graph is zero. So displacement is
= 10 + 20 + 3.33 = 33.33m
zero
•• Speed is magnitude of velocity. Sol 16: (A) Total distance = ∑ | v | t
Speed=|velocity| 1
= 33.33 + × ( −20) × 3.33
= 33.33 + 33.33
= 66.7m
Which is same initial and final. 2

Sol 6: (A, B, C) It’s velocity is v1 Paragraph 2:

He observes both moving with velocity –v. Sol 17: (B) For
dx
0 , t < T dx >0
Sol 7: (C, D) While throwing, the horizontal component 0, t < T > dt
0
dt dx
of the velocity of ball (with respect to earth) is equal to T < t < 2T <0
velocity of cart. dt
d2 x dv
<0∀t ⇒ <0∀t
dt2 dt
P hysi cs | 2.67

dv 20(u cos 45°) – 40(u cos 45°) = 40(v) – 20(u cos 450°)
Sol 18: (D) <0
dt \ v=0
Sol 19: (C) Speed is |v|. Apply |v| to v-t graph. i.e., particle Q comes to rest. So, particle Q will fall
vertically downwards and will strike just midway
Match the Columns between A and B.
(ii) Maximum height,
Sol 20: A → p,s; B → r; C → q; D → s
u2 sin2 θ (49)2 sin2 45°
H= = = 61.25 m
dx 2g 2 × 9.8
(A) < 0, x>0 , 0<t<T
dt Therefore, time taken by Q to reach the ground,
⇒ Ball rolling towards origin. (p)
2H 2 × 61.25
t= = = 3.53 s
dx g 9.8
= 0 t>T
dt
So it suddenly stops (s) Sol 2: Let t1 be the time from A to B and t2 the time
dx from B to C
(B) There is a sudden change in slope i.e.
dt Then
A
Hence it had bounced of something (r)
H h
(C) There is a sudden change in slope. The particle was
going to origin and went back (q) H
B
dx h
(D) < 0, x > 0 ⇒ Object going towards origin.
dt
C
dx
= 0; t>T
dt 2(H – h) 2h
t1 = and t2 =
dx g g
< 0; 0<t<T
dt
Then, the total time
⇒ Sudden change in velocity.
2
dx T = t1 + t2 = [ H–h + h ]
Also = 0 ⇒ ball stops (s) g
dt
dt
For t to be maximum =0
dh
Previous Years’ Questions
2  –1 1 
or  +  =0
g 2 H – h 2 h 
Sol 1: (i) Range of both the particles is
1 1
u2 sin2θ (49)2 sin90° or = or 2h = H
R= = h H–h
g 9.8
By symmetry we can say that they will collide at highest h 1
∴ =
point. H 2

u cos 45° ucos45° ucos45° v


Sol 3: (i) Acceleration of A and C both is 9.8 m/s2
→ ← ← →
• • • • downwards. Therefore, relative acceleration between
them is zero i.e., the relative motion between them will
20g 40g 20g 40g be uniform.
Just before collision Just after collision
Let v be the velocity of Q just after collision. Then, from
conservation of linear momentum, we have
2 . 6 8 | Motion in a Straight Line

(VCA)x Horizontal (or x) component of momentum of A. i.e.,


 C
 -V A PAx=mvAx = –10 m.
 =V C (VCA)y
st) V C A 10m X Similarly, x component of momentum of C, i.e.,
(re
A E PCx = (2m)vCx = (2m)(5) = + 10 m
d 30m Y
20m Since, PAx + PCx = 0
i.e., x-component of momentum of combined mass
B D
after collision will also be zero, i.e., the combined
mass will have momentum or velocity in vertical or
Now assuming A to be at rest, the condition of collision
y-direction only.
→ → →
will be that v CA = v C – v A = relative velocity of C VCx
w.r.t. A should be along CA. C

5m/s C
60°
A VAX
10 m/s P 2
t= s
5 3m/s 3
B F D
A 10m/s d₁ d₂

Hence, the combined mass will fall at point F just below


the point of collision P.
2
Here d1 = |(vAx)| t = (10) = 11.55 m
3
vAx = – 10 m/s ∴ d2=(d – d1) = (17.32 – 11.55) = 5.77 m

vAy = 0 d2 should also be equal to

vCx = 5 m/s  2 
|vCx| t = (5)   = 5.77 m
vCy = 5 3 m/s  3
Therefore, position from B is d1 i.e., 11.55 m and from
(v CA )yCE 10
∴ = = D is d2 or 5.77 m.
(v CA )x AE d

v Cy – v Ay Sol 4: u = 5 3 m/s
10 5 3–0 10
or = or = 1
v Cx – v Ax d 5 – (–10) d ∴ u cos60° = (5 3 )   m/s = 2.5 3 m/s
2
∴ d = 10 3 m = 17.32 m  3
and u sin 60° = (5 3 )   m/s = 7.5 m/s
 2 
AC  
(ii) Time of collision, t =

| v CA | y u 2
→ 60°u
| v CA | = (v 2CAx ) + (v CAy )2 Q
h
2 2
= {5 – (–10)} + {5 3 – 0} = 10 3 m/s 10m
x P
y=10-h
CA = (10)2 + (10 3)2 = 20 m x
0
20 2
∴t= = s Since, the horizontal displacement of both the shots
10 3 3 are equal. The second should be fired early because it’s
horizontal component of velocity u cos 60° or 2. 5 3
P hysi cs | 2.69

m/s is less than the other which is u or 5 3 m/s. Therefore, velocity of stone at highest point will be

Now let first shot takes t1 time to reach the point P and v = ( 4iˆ + 3ˆj ) m/s
the second t2.
or speed at highest point,
Then, x = (u cos 60°)t2 = ut1 →
v=|v|= (4)2 + ((3)2 m/s = 5 m/s
or x = 2.5 3 t2 = 5 3 t1  … (i)
Now, applying law of conservation of linear momentum,
or t2 = 2t1  … (ii) let v0 be the velocity of combined mass after collision.
1 2 1 Then, mv = (2m)v0
and h = |(u sin 60°)t2 – gt | = gt12
2 2 2
v 5
1 1 ∴ v0 = = m/s = 2. 5 m/s
or h = gt22 – (u sin 60°)t2 = gt12 2 2
2 2
∴ Speed of combined mass just after collision is
Taking g = 10 m/s2 2. 5 m/s
h = 5t22 – 7.5 t2 = 5t12  … (iii) (ii) Tension in the string becomes zero at horizontal
position. It implies that velocity of combined mass
Substituting t2 = 2t1 in Eq. (iii), we get also becomes zero in horizontal position. Applying
conservation of energy, we have
5(2t1)2 – 7.5(2t1) = 5t12

or 15t12 = 15t1 ⇒ t1 = 1 s T= 0
v= 0
and t2 = 2t1 = 2 s
L
x = 5 3 t1 = 5 3 m [From Eq. (i)]

and h = 5t12 = 5(1)2 =5m [From Eq. (iii)]


∴ y = 10 – h = (10 – 5) = 5 m v0
Hence,  = v ²0 -2gL
(i) Time interval between the firings θ = v 20 – 2gL
= t2 – t1 = (2 – 1) s v 20 (2.5)2
∴L= = = 0.32 m
Dt = 1 s 2g 2(9.8)
(ii) Coordinates of point
Hence, length of the string is 0.32 m
P = (x, y) = (5 3 m, 5 m)
–k
Sol 6: (i) F(x) =
2x2
Sol 5: (i) Let î , ˆj and k̂ be the unit vectors along x, y k and x2 both are positive.
and z-directions respectively.
Hence, F(x) is always negative.

Given v cart = 4iˆ m/s B A
F(x)

∴ v stone. cart =(6 sin 30°) ˆj + (6 cos 30°) k̂ x=0 x=0.5m x=1.0m
At t=0
= (3 ˆj + 3 3kˆ ) m/s
v=v
v=0
→ → →
∴ v stone = v stone. cart + v cart Applying work-energy theorem between points A and
B. Change in kinetic energy between A and B = work
= (4 î + 3 ˆj + 3 3 k̂ ) m/s done by the force between A and B

This is the absolute velocity of stone (with respect to x =0.5m 0.5 0.5
1  –k  –k dx
ground). At highest point of its trajectory, the vertical ∴
2
mv2 = ∫ F(dx) = ∫  2x2  (dx) = 2 ∫ x2
x =1.0m 1.0 1.0
component (z) of its velocity will become zero, whereas
the x and y-components will remain unchanged
2 . 7 0 | Motion in a Straight Line

0.5
k   1 1  Acceleration of particle with respect to block
k 1 – k
=   =     =
2  x 1.0 2  0.5 1.0  2 = acceleration of particle – acceleration of block
= (g sin θ î + g cos θ ˆj )–(g sin θ) î = g cos θ ˆj
k
∴v=± Now motion of particle with respect to block will be a
m
projectile as shown
Substituting the values

10 –2 Nm2
v=± = ± 1 m/s u O
10 –2 kg 
P R
Therefore, velocity of particle at x = 1.0 m is
v = – 1.0 m/s  g cos
Negative sign indicates that velocity is in negative
x-direction. The only difference is, g will be replaced by g cos θ
(ii) Applying work-energy theorem between any u2 sin2α
∴ PQ = Range (R) =
intermediate value x = x, we get gcos θ
x x
1 –k dx k 1 k 1  (ii) Horizontal displacement of particle with respect
2
mv2 = ∫ 2x 2
=   =  – 1 
2  2 1.0 2 x 
to ground is zero. This implies that initial velocity
1.0
with respect to ground is only vertical, or there is no
horizontal component of the absolute velocity of the
k 1  1 1–x
∴ v2 =  – 1 ∴v= –1 = particle.
m x  x x
u
^ 
k 10 –2 λ Nm2 j 
= u cos(+)
m ^
10 –2 kg i

 dx  1–x
but v = –   = 
 dt  x

0.25 x v cos
x x
∴ ∫ 1–x
dx = – ∫ dt or ∫ 1–x
dx = – ∫ dt 
1 0

Solving this, we get t = 1.48 s v sin




Sol 7: (i) Accelerations of particle and block are shown Let v be the velocity of the block down the plane.
in figure.
Velocity of particle
= u cos (α + θ) î + u sin(α + θ) ˆj
Velocity of block = – v cos θ î – v sinθ ˆj
g sin
g cos ∴ Velocity of particle with respect to ground

= {u cos (α + θ) – v cos q} î +
^
i ^
i {u sin (α + θ) – v sin q} ˆj
Now, as we said earlier that horizontal component of
g sin absolute velocity should be zero.

Therefore, u cos (α + θ) – v cos θ = 0
ucos(α + θ)
or v = (down the plane)
cos θ
P hysi cs | 2.71

Sol 8: Let t be the time after which the stone hits the Sol 9: (i) Let A stands for trolley and B for ball.
 
object and θ be the angle which the velocity vector u
Relative velocity of B with respect to A ( vBA ) should be
makes with horizontal. According to question, we have 
following three conditions. Vertical displacement of along OA for the ball to hit the trolley. Hence, vBA will
stone is 1.25 m make an angle of 45° with positive x-axis.

1 2 Q = 45°
1.25 = (u sin θ)t – gt
2 (ii) Let v = absolute velocity of ball.
where g = 10 m/s2 4θ 4
φ= = (45°) = 60° → with x-axis
or (u sin θ)t = 1.25 + 5t 2
… (i) 3 3

v 3v ˆ
Horizontal displacement of stone ∴ v B = (v cos θ) î + (v sin θ) ˆj = î + j
2 2
= 3 + displacement of object A →
v A = ( 3 – 1) ˆj
1 2
∴ (u cos θ)t = 3 + at , →  3v 
2 v
∴ v BA = î +  – 3 + 1  ˆj
where a = 1.5 m/s2 2  2 
 

or (u cos θ)t = 3 + 0.75 t2  … (ii) Since v BA is at 45°
Horizontal component of velocity (of stone) = vertical
v 3v
component (because velocity vector is inclined at 45° ∴ = – 3 + 1 or v = 2 m/s
2 2
with horizontal)
∴ (u cos θ) = gt – (u sin θ)  … (iii) 2usin θ 2 × 10 × sin60°
Sol 10: t = T = = = 3s
g 10
(The right hand side is written gt – u sin θ because the
stone is in its downward motion. Therefore, gt > u sin θ. 1 2
Displacement of train in time t = at
In upward motion u sin θ > gt) 2

Multiplying Eq. (iii) with t we can write Displacement of boy with respect to train = 1.15 m

or (u cos θ)t + (u sin θ)t = 10 t2  … (iv) ∴ Displacement of boy with respect to ground

Now Eqs. (iv), (ii) and (i) gives  1 


=  1.15 + at2 
4.25t – 4.25 = 0
2  2 
or t = 1 s Displacement of ball with respect to ground = (u cos
60°)t
Substituting t = 1 s in Eqs. (i) and (ii), we get
To catch the ball back at initial height,
u sin θ = 6.25 m/s
1 2
or uy = 6.25 m/s 1.15 + at = (u cos 60°)t
2
and u cos θ = 3.75 m/s 1 1
∴ 1.15 + a( 3 )2 = 10 × × 3
or ux = 3.75 m/s 2 2
→ Solving this equation, we get
therefore u = ux ˆi + uy ˆj m/s
→ a = 5 m/s2
or u = (3.75 î + 6.25 ˆj ) m/s
∴ Answer is 5
Note: Most of the problems of projectile motion are

easily solved by breaking the motion of the particle in Sol 11: (B) V₂
two suitable mutually perpendicular directions, say x
and y. Find ux, uy, ax and ay and then apply

1 2 V₁
vx = vx + ax t; sy = uy t + a t etc.
2 y


₂> ₁ ₂> ₁
2 . 7 2 | Motion in a Straight Line

θ2 > θ1 ∴ ω2 > ω1 Therefore, path of the particle is an ellipse. Hence,


option (a) is correct.
From the given equations we can
Statement-II, is formula of relative velocity. But it find
does not explain statement-I correctly. The correct
explanation of statement-I is due to visual perception dx
= vx = – ap sin pt
of motion. The object appears to be moving faster, dt
when its angular velocity is greater w.r.t. observer. d2 x
= ax = – ap2 cos pt
dt2
Sol 12: (A, C) Since, the body is at rest at x = 0 and x = 1.
dy
Hence, α cannot be positive for all time in the interval = vy = bp cos pt and
0 ≤ t ≤ 1. dt

Therefore, first the particle is accelerated and then d2 y


= ay = – bp2 sin pt
2
retarded. dt
Now, total time t = 1 s (given) π π
At time t = or pt =
2p 2
Total displacement s = 1 m (given)
s = Area under v-t graph y
2s
∴ Height or vmax= =2m/s is also fixed.
t
 t=t
1 r
[Area or s = × t × vmax] x
2 O
If height and base are fixed, area is also fixed
In case 2 : Acceleration = Retardation = 4 m/s2
In case 1 : Acceleration > 4m/s2 while
Retardation < 4 m/s2 π
ax and vy become zero (because cos = 0)
2
v(m/s)
only vx and ay are left.
or we can say that velocity is along negative x-axis and
2 acceleration along y-axis
2 π
3 Hence, at t = velocity and acceleration of the
1 2p
particle are normal to each other. So, option (b) is also
correct.

t(s) At t = t, position of the particle


0

r (t) = x î + y ˆj = a cos pt î + b sin pt ˆj
While in case 3 : Acceleration < 4 m/s and Retardation
2

> 4 m/s2 y
Hence, |a| ≥ 4 at some point or points in its path. 
t=
2p Vy
Sol 13: (A, B, C) b
a x
x
x = a cos (pt) ⇒ cos (pt) =  … (i) O ax
a
y
y = b sin (pt) ⇒ sin (pt) =  … (ii)
b
Squaring and adding Eqs. (i) and (ii), we get
x2 y2
+ =1
2
a b2
a x
O ax

P hysi cs | 2.73

t=0
y = 0 = vx = ay
x=a
vy = by and
ax = – ap2

and acceleration of the particle is



a (t) = ax î + ay ˆj

= – p2[a cos pt î + b sin pt ˆj ]



= – p [x î + y ˆj ] = – p2 r (t)
2

Therefore, acceleration of the particle is always directed


towards origin and not any of the foci.
Hence, option (C) is wrong.
π
At t = 0, particle is at (a, 0) and at t =
2p
particle is at (0, b). Therefore, the distance covered is
one-fourth of the elliptical path not a.
Hence, option (D) is wrong.

Sol 14: Maximum displacement of the left ball from the


left wall of the chamber is 2.25 cm, so the right ball has
to travel almost the whole length of the chamber (4m)
to hit the left ball. So the time taken by the right ball is
1.9 sec (approximately 2 sec)
2017-18 100 &
op kers
Class 11 T
By E ran culty
-JE Fa r
IIT enior emie .
S fP r es
o titut
Ins

PHYSICS
FOR JEE MAIN & ADVANCED
SECOND
EDITION

Exhaustive Theory
(Now Revised)

Formula Sheet
9000+ Problems
based on latest JEE pattern

2500 + 1000 (New) Problems


of previous 35 years of
AIEEE (JEE Main) and IIT-JEE (JEE Adv)

5000+Illustrations and Solved Examples


Detailed Solutions
of all problems available

Topic Covered Plancess Concepts


Tips & Tricks, Facts, Notes, Misconceptions,
Motion in a Plane Key Take Aways, Problem Solving Tactics

PlancEssential
Questions recommended for revision
3. MOTION IN A PL ANE

1. INTRODUCTION
Motion in a plane is a two dimensional motion. The analysis of this type of motion becomes easy when we consider
this motion as a combination of two straight line motions along two mutually perpendicular axes lying in the
plane of motion. In Cartesian coordinate system the two mutually perpendicular axes are the x-axis and the y-axis
respectively. The displacement, velocity and acceleration of the particle are resolved into components along the
x and y axes and motion along each axis is studied independent of the other. The net displacement, velocity or
acceleration is the vector sum of their respective components along the two axes. In this chapter we will discuss
about the motion of a projectile, the motion of a body relative to another body, the motion of a body in a river, the
motion of an airplane with respect to wind, and circular motion.

2. MOTION IN A PLANE
When a body moves in a straight line, we call it motion in a straight line or one dimension. For eg, a car going
straight on a road. When you throw a ball towards your friend, the ball follows a non-linear path. This motion is
termed as motion in two dimensions or motion in a plane.
The position of a particle that is free to move can be located by two coordinates in a plane. We choose the plane
of motion as the X-Y plane. We choose a suitable instant as t = 0 and choose the origin at the place where the
particle is situated at t = 0. Any two convenient mutually perpendicular directions in the X-Y plane are chosen as
the X and Y-axes.

3. PROJECTILE MOTION
Projectile motion is a form of motion in which an object or particle (here called a projectile) is thrown in an oblique
direction near the earth’s surface, and it moves along a curved path under the action of a continuous motive force.
The path observed during a projectile motion is called its trajectory. Projectile motion is possible only when there
is one force applied at the beginning of the trajectory, after which there is no force in operation except a constant
force.

3.1 Ground-To-Ground Projectile


In the Fig. 3.2 shown, let us consider the horizontal surface through the point O. Now, the point O here is called the
point of projection, the angle θ is called the angle of projection and the distance OB is called the horizontal range
or simply range. Further, the total time taken by the particle in describing the path OAB is called the time of flight.
3 . 2 | Motion in a Plane


u

 u
u

(a) (b) (c)



u 
u

u

(d) (e) (f)

Figure 3.1

However, we can separately discuss the motion of the projectile for both the horizontal and vertical parts. In this
regard, we begin by considering the origin as the point of projection.
Now, we have ux = ucos θ; ax = 0 ; uy =
usin θ; ay =
−g.

A
u sin 


X
O u cos  B

Figure 3.2

3.1.1 Horizontal Motion


1
As As a x = 0, we have q and x = ux t + a x t2 = ux t = ut cos q
n x = ux + a x t = ux = ucosand
2
3.1.2 Vertical Motion
In the downward direction, we know that the acceleration of the particle is g. Thus, α y =−g.
Further, the y-component of the initial velocity is uy. Thus,
1
ν y = uy − gt and y = uy t − gt2 ; also we have, ν2y = u2y − 2gy.
2

3.1.3 Time of Flight


Let us suppose that the particle is at B at time t. Therefore, the equation for horizontal motion gives OB = x = ut
cos θ .
However, the y-coordinate at the point B is zero. Thus, from the equation of vertical motion,
1 1 1
=y ut sin θ − gt2 or, =0 ut sin θ − gt2 or, t(usin θ − =gt) 0
2 2 2
2usin θ
Thus, either t = 0 or, t =
g
P hysics | 3.3

Now, t = 0 exactly corresponds to the initial position O of the particle. Hence, the time at which it reaches B is thus,
2usin θ
T=
g
This equation helps us to exactly calculate the time of flight.

3.1.4 Range
Consider the distance OB covered by a particle, which is the horizontal range. It is the distance travelled by the
2usin θ
particle in time T =
g
2u2 sin θ cos θ u2 sin2θ
By the equation of horizontal motion, =x (ucos θ) × T or, = OB =
g g
3.1.5 Maximum Height
We have, ν y = uy −=
gt u sin θ − gt

usin θ
However, at the maximum height, =
0 usin θ − gt or, =t
g
2
1  u sin θ  1  u sin θ 
The actual maximum height is H =uy t − gt2 =(u sin θ)   − g 
2  g  2  g 

u2 sin2 θ 1 u2 sin2 θ u2 sin2 θ


= − =
g 2 g 2g

4. EQUATION OF TRAJECTORY OF A PROJECTILE


x
x (ucos α )t
= t
∴=
ucos α
1
y (usin α )t − gt2 , we obtain
By substituting this value of t in =
2
gx2 gx2 2 gx2
y x tan α −
= = x tan α − sec= α x tan α − (1 + tan2 α )
2u2 cos2 α 2u2 2u2

The above are the standard equations of trajectory of any projectile. Here, we should be aware of the fact that the
equation is quadratic in x. This is why the path of a projectile is always a parabola. Further, the above equation can
 x
also be represented in terms of range (R) of the projectile as y = x  1 −  tan α
 R 

Illustration 1: Assume that a ball is thrown from a field at a speed of 12.0 m/s and at an angle of 45° with the
2
horizontal. At what distance will it hit the field again? Take g = 10.0 m/ s . (JEE MAIN)
Sol: Use the formula for the range of a projectile.

u2 sin2θ (12m / s)2 × sin(2 × 45 ) 144m2 / s2


The horizontal range == = = 14.4m
g 10m / s2 10.0m / s2

Thus, the ball hits the field exactly at 14.4 m from the point of projection.
3 . 4 | Motion in a Plane

PLANCESS CONCEPTS
(i) Range is maximum where 2= α 45 and this maximum range is:
α 1or=

u2 Y
Rmax
= = 4H
g

(ii) For given value of u, range at α and range at φ are equal although
times of flight and maximum heights may be different. Because u

u
u2 sin2(90o − α ) u2 sin(180o − 2α ) u2 sin2α
R o
= = = = Rα
90 −α g g g 30°
60°
X
As we have seen in the above derivations that ax = 0, i.e., motion of
the projectile in the horizontal direction is uniform. Hence, horizontal Figure 3.3
component of velocity u cos α does not change during its motion.
Motion in the vertical direction is first retarded and then accelerated in opposite direction. As the equa-
y ax − bx2 (equation of parabola), therefore, the path
tion of trajectory of projectile is of the form, =
followed by a projectile is a parabola.
B Rajiv Reddy (JEE 2012, AIR 11)

Illustration 2: Find the angle of a projectile for which both the horizontal range and maximum height are equal.
(JEE MAIN)
Sol: Use the formula for the range and maximum height of a projectile.
Given, R = H

u2sin2α u2sin2 α sin2 α sin α


∴ = or 2sin α cos α
= or = 4 or tan α = tan−1 (4)
= 4 α
g 2g 2 cos α

Illustration 3: The given Fig. 3.4 shows a pirate ship 560 m from a fort defending a harbor entrance. A defense
canon, located at sea level, fires balls at initial speed ν =82m / s (JEE ADVANCED)
y

63°
27° x

Figure 3.4

(a) At what angle θ from the horizontal must a ball be fired to hit the ship?

Sol: Use the formula for the range of a projectile to find the angle of projection.

(ν20 / g)sin2θ ) , which, after rearrangement, gives


We can relate the launch angle θ to the range R with Eq. (R =

1 −1 gR 1 −1 (9.8m / s2 )(560m) 1 −1
=θ sin
= sin = sin 0.816
2 ν2 2 (82m / s)2 2
0
One solution of (54.7 ) is worked out using a calculator; now, we subtract it from 180 to get the other solution
(125.3 ) . This gives =
us θ0 27 and
= θ0 63 .
P hysics | 3.5

Illustration 4: Suppose a batsman B hits a high-fly ball to the outfield, directly toward an outfielder F and with a
launch speed of ν =40m / s and a launch angle of θ0 = 35 . During the flight, a line from the outfielder to the ball
makes an angle φ with the ground. Based on the data provided, plot the elevation angle tan = θ 2cot α versus time
t, assuming that the outfielder is (a) already positioned to catch the ball, (b) is 6.0 m too close to the batsman and
(c) is 6.0 m too far away. (JEE ADVANCED)
Sol: While trying to catch a ball which has gone to a great height you can imagine that the angle of line of sight
increases as the ball moves. If we neglect air drag, then the ball is a projectile for which the vertical motion and the
horizontal motion can be analyzed individually.
90

60

 (deg)
y
Too close
30
B y  F
x Too far
x R x
R 0 1 2 3 4 5
(a) t (s)
(b)
Figure 3.5

Assuming that the ball is caught at approximately the height it is hit, the horizontal distance traveled by the ball is
the range R, given by Eq. (R =(ν20 / g)sin2θ )
The ball can be caught if the outfielder’s distance from the batsman equals the range R of the ball. Using the above
equation, we find the elevation angle φ for a ball that was hit toward an outfielder is (a) defined and (b) plotted
versus time t.
ν20 (40m / s)2 
=R sin2
= θ sin(70
= ) 153.42m
g 9.8m / s2
Fig. 3.5 (a) above shows a snapshot of the ball in flight when the ball is at height y and horizontal distance x from
the batsman (who is at the origin). The horizontal distance of the ball from the outfielder is R – x, and the elevation
angle φ of the ball in the outfielder’s view is given by tan φ = y/(R–x).

−1 (40 sin35 )t − 4.9t2


Thus, using ν0 =40 m/s and θ0 =35 , we have φ =tan
153.42 − (40 cos35 )t
By graphing this function versus t gives us the middle plot in b. We now see that the ball’s angle in the outfielder’s
view increases at an almost steady rate throughout the flight.
If the outfielder is 6.0 m close to the batsman, then we replace the distance of 153.42 m in the given equation with
153.42 m – 6.0 m = 147.42 m. Further, regraphing the function gives the “Too close” plot as in Fig. 3.5 (b).
Now, we observe that the elevation angle of the ball rapidly increases toward the end of the flight as the ball soars
over the outfielder’s head. However, if the outfielder is 6.0 m too far away from the batsman then we replace the
distance of 153.42 m in the equation with 159.42 m. The resulting plot is hence labeled “Too far” in the Fig. 3.5: The
angle first increases and thereafter rapidly decreases.
Conclude: Thus, if a ball is hit directly toward an outfielder, then the player can tell from the change in the ball’s
elevation angle φ whether to stay put, run toward the batter, or back away from the batsman.

Illustration 5: Suppose that a projectile is fired horizontally with a velocity of 98 m/s from the top of a hill that is
490 m high. Find:
(a) The time taken by the projectile to reach the ground,
(b) The distance of the point where the particle hits the ground from the foot of the hill and
(c) The velocity with which the projectile hits the ground. (take g = 9.8 m / s2 ) (JEE MAIN)
3 . 6 | Motion in a Plane

Sol: Let x-axis be along the horizontal and the y-axis be along the vertical. The projectile will have uniform velocity
along the positive x-axis and uniform acceleration along the negative y-axis.
In this problem, we cannot apply the formulae of R, H and T directly. Necessarily we have to follow the three steps
discussed in the theory. Here, however, it will be more convenient to choose x and y directions as shown in the Fig.
3.6 provided.
Here,=
ux 98m / s,a
= x 0,u
= y ay g O
0 and = u = 98 m/s

x
1 1
(a) At A, S y = 490m . Therefore, applying sy =uy t + ay t2 ∴ 490 =0 + (9.8)t2 ∴ t =10s
2 2
y
1 1
uy t + ay t2 ∴ 490 =0 + (9.8)t2 ∴ t =10s
2 2
1
(b) BA =sx =ux t + ax t2 or BA =
(98)(10) + 0 or BA =
980m A Vx
2 B


(c) ν x = ux = 980m / s; v y = uy + ay t = 0 + (9.8)(10) = 98m / s Vy

∴ ν= ν2x + ν2y = 2 2
(98) + (98) = 98 2 m / s Figure 3.6

v y 98
and tan=
β = = 1 ∴ β 45
=
v x 98

Thus, we show that the projectile hits the ground with a velocity 98 2 m / s at an angle of β = 45 with horizontal
as shown in the Fig. 3.6 provided.

5. PROJECTILE MOTION ON AN INCLINED PLANE


Projectile motion on an incline plane is one of the y y
various types of projectile motion. However, the
main distinguishing aspect is that points of
projection and return are not on the same horizontal u
plane.
We know that there are two possibilities in this
u
regard: (i) the point of return is at a higher level than
the point of projection i.e., projectile is thrown up x x
the incline and (ii) the point of return is at a lower 0,0 0,0
level than the point of projection, i.e., the projectile (a) Projection up the incline (b) Projection down the incline
is thrown down the inclined plane.
Figure 3.7

5.1 Analyzing Motion


We can make us of two different approaches of analyzing projectile motion on an inclined plane. The first approach
preferably could be to continue analyzing motion in two mutually perpendicular horizontal and vertical directions.
The second approach, therefore, could be to analyze motion by changing the reference orientation, i.e., we set up
our coordinate system along the incline and a direction along the perpendicular to the incline.

Based on the analysis, alternatives are, therefore, distinguished on the basis of coordinate system that we choose
to employ:
(a) Planar coordinates along the incline (x) and perpendicular to the incline (y)
(b) Planar coordinates in horizontal (x) and vertical (y) directions
However, we use the first approach for analyzing this kind of motion, i.e., coordinates along the incline (x) and
perpendicular to the incline (y).
P hysics | 3.7

5.2 Projectile Motion Up an Inclined Plane


Based on the details provided in the Fig. 3.8, it is clear that the g sin
g cos
angle that the velocity of projection makes with the x-axis (i.e., y
incline) is “ θ − a ”. 
R,0) x
Therefore, the components of initial velocity are
u
ux ucos(θ − α ); =
= uy usin(θ − α ) R

Hence, the components of acceleration are  
ax =
−gsin α ; ay =
−gcos α 0,0
Figure 3.8

5.2.1 Time of Flight


The time of flight (T) is calculated by analyzing motion in y-direction (which is not vertical as in the normal case).
However, the displacement in the y-direction after the projectile has returned to the incline is zero as in the normal
case. Thus,
1 1  1 
y= uy Τ + ay Τ2 = 0 ⇒ usin(θ − α )Τ + ( −gcos α )Τ2 = 0 ⇒ Τ usin(θ − α )Τ + ( −gcos α )Τ = 0; Τ= 0
2 2  2 
2usin(θ − α )
⇒Τ=
gcos α

Here, the first value represents the initial time of projection. Hence, the second expression gives us the time of
flight. However, we should note here that the expression of time of flight is as in a normal case albeit in a significant
manner.
2uy
In the generic form, we can express the formula of the time of flight as: Τ =
ay
5.2.2 Range of Flight
The first thing that we should note that we do not use the term “horizontal range” as the range on the inclined
plane is no more horizontal. Rather, we simply refer the displacement along the x-axis as “range”. Thus, we can
find range of flight by considering motion in both “x” and “y” directions. We further note that we utilize the same
approach even in the normal case. Now, let “R” be the range of the projectile motion.
1
Substituting the value of “T” as obtained earlier, we have x= ux Τ − ax Τ2
2
ucos(θ − α ) × 2usin(θ − α ) gsin α × 4u2 sin2 (θ − α )
R −
gcos α 2g2 cos2 α

u2
=⇒R
gcos 2
α
{2cos ( θ − α ) sin( θ − α ) cos α − sin α × 2sin ( θ − α )}
2

Using trigonometric relation, 2sin2 (θ − α ) = 1 − cos2(θ − α ),

u2
sin2(θ − α )cos α − sin α {1 − cos2(θ − α )}
=⇒R  
gcos2 α
u2
=⇒R {sin2(θ − α)cos α − sin α + sin α cos2(θ − α)}
gcos2 α
u2
= ⇒R
Now, we use the trigonometric relation, sin(A
{sin(2θ − 2α + α) − sin α}
B) 2sinA
= +gcos α cosB + cos AsinB
u2 u2
=⇒R {sin(2θ − 2α + α) =
− sin α} ⇒ R {sin(2θ − α) − sin α}
gcos2 α gcos2 α
u2
=⇒R {sin(2θ − α) − sin α}
gcos2 α
3 . 8 | Motion in a Plane

This is the exact expression for the range of projectile on an inclined plane. We also note that this expression
u2 sin2θ
reduces to the one for the normal case, when α =0 ⇒R =
g

5.3 Projectile Motion down the Inclined Plane


The components of initial velocity:=
ux ucos(θ + α ); =
uy usin(θ + α )
The components of acceleration: ax =
gsin α ; ay =
−gcos α
Time of flight
The expression for the time of flight differs only with respect to angle of sine function in the numerator of the
2usin(θ + α )
expression: Τ =
gcos α
Range of flight

In the same way, the expression of range of flight differs only with respect to angle of sine function:
u2
=R {sin(2θ + α) + sin α}
gcos2 α

PLANCESS CONCEPTS

It is very handy to note that expressions have changed only with respect of the sign of “ α ” for the time
of flight and the range. We only need to exchange “ α ” by”– α s”.
Vaibhav Krishan (JEE 2009, AIR 22)

Illustration 6: Assume that a projectile is thrown from the base of an incline y g sin
of angle 30° as shown in the Fig. 3.9 provided. It is thrown at an angle of 60°
u g cos
from the horizontal direction at a speed of 10 m/s. Calculate the total time of 
flight is (consider g = 10 m/s2).  (JEE MAIN) 
g
O 
Sol: The x-axis has to be assumed along the inclined.
This problem can be handled with a reoriented coordinate system as shown in
the Fig. 3.9 provided. Here, the angle of projection with respect to x-direction
is ( θ – a) and acceleration in y-direction is “g cosa”. Now, the total time of P
flight for the projectile motion, when the point of projection and return are on
the same level, is x
Figure 3.9
2usin(θ − a)
⇒Τ=
gcos α
Now,
= θ 60
= 
,a 30 , u = 10m/s. Then, by substituting these values, we finally obtain
2X10 sin(60 − 30 ) 20 sin30 2
=
⇒Τ = =
gcos30 10 cos30 3

Illustration 7: Consider that two projectiles are thrown with the same speed from point ”O” and “A” so that they hit
the incline. If t0 andt A be the time of flight in two cases, then prove which option out of those given here is true.
 (JEE MAIN)
utan θ
(A) t0 = t A (B) t0 & t A (C) t0 > t A (D) =
t0 t=
A
g
P hysics | 3.9

Sol: The x-axis has to be assumed along the inclined. Use the formula for time
of flight on an inclined. Let us first consider the projectile thrown from the point u
 A
“O”. Considering the angle the velocity vector makes with the horizontal, we
u
represent the time of flight as:
2usin(2θ − θ) 2utan θ 
=⇒ t0 = ⇒ t0 
gcos θ g
O
Further, for the projectile thrown from the point “A”, the angle with horizontal is Figure 3.10
zero. Hence, the time of flight is expressed as
2usin(2X0 − θ) 2utan θ
=⇒ tA =
gcos θ g
Thus, we observe that the times of flight in the two cases are equal. 
⇒ tA =
t0
Hence, option (A) is correct.

Illustration 8: Two inclined planes of angles 30° and 60° are placed so that X
they touch each other at the base as shown in the Fig. 3.11 provided.
Further, a projectile is projected at right angle at a speed of 10 3 m/s Y 103 m/s Q
from point “P” and hits the other incline at point “Q” normally. Then,
the time of flight is:  (JEE ADVANCED) P

(A) 1 s (B) 2 s (C) 3 s (D) 4s


30° 60°
Sol: This problem is a specific case in which the inclined planes are O
right angles with respect to each other. Therefore, we actually take Figure 3.11
advantage of this fact in assigning our coordinates along the planes, say
y-axis along first incline and x-axis against second incline.
Thus, in order to find the time of flight, we can further use the fact that projectile hits the other plane at right angle,
i.e., parallel to the y-axis. This means that the component of velocity in x-direction, i.e., along the second incline is
zero. This, in turn, suggests that we can analyze motion in x-direction to obtain the time of flight.

15

10 3 60° Vx
30°

60°
5 3 V

Vy
Figure 3.12

=Vx 10
= 3 cos 60 5 3 will remain constant
∴ Here, V cos60 = 5 3

55 33 −V−V
∴ VV = = X2 10
X2=
=10 ∴∴VyVy= =−V− Vsin30
sin30= == 5−5
−=
33 2 2

∴ VV = = uu−−gt
gt 5 5= = 1515
∴∴− − − 10t
− 10t

∴==10t
10t 20
20 =
=∴∴t t 2s2s
3 . 1 0 | Motion in a Plane

PLANCESS CONCEPTS

Given are a simple set of guidelines in a very general way:


••  nalyze motion independently along the selected coordinates for complicated problems. For simple
A
cases, try remembering derived formula and use them directly to save time.
••  ake note of information given in the question like angles, etc., which might render certain
M
components of velocity zero in certain direction.
•• If range of the projectile is given, we may try the trigonometric ratio of the incline itself to get the
answer.
•• If we use coordinate system along incline and in the direction perpendicular to it, then always
remember that component of motion along both incline and in the direction perpendicular to it are
accelerated motions. Ensure that we use appropriate components of acceleration in the equations
of motion.
••  he range is maximum for maximum value of “sin(2θ−α)”. Thus, the range is maximum for the angle
T
of projection as measured from horizontal direction, when

sin(2θ−α)=1
sin(2θ − a)= 1 ⇒⇒ sin(2θ − α )= sin π / 2
u2
⇒ 2θ − α = π / 2 ⇒ θ = π / 4 + α / 2 ⇒ Rmax = (1 − sin α )
gcos2 α

Anand K (JEE 2011, AIR 47)

6. RELATIVE MOTION
The measurements describing motion are generally subject to the state of motion of the frame of reference with
respect to which measurements are taken about. Our day-to-day perception of motion is generally based on our
earth’s view—a view common to all bodies at rest with respect to earth. However, we come across cases when there
is a subtle perceptible change in our view of earth. One such case is traveling in the city trains. We easily find that
it takes lot longer to overtake another train on a parallel track. Also, we happen to see two people talking while
driving separate cars in parallel lanes, as if they were stationary to each other! In terms of kinematics, as a matter
of fact, they are actually stationary to each other even though each of them is in motion with respect to ground.
In this topic, we study motion from a perspective other than that of our earth. The only condition that we subject
ourselves is that two references or two observers making the measurements of motion of an object, are moving at
constant velocity.
We now consider two moving observers, “A” and “B”:
   
The relative velocity of A with respect of B (written as ν AB ) is ν AB = ν A − νB
  
Similarly, the relative acceleration of A with respect to B is aAB = aA − aB Relative velocity
A B
Illustration 9: Assume that two cars, standing apart, start moving
toward each other at speeds of 1 m/s and 2 m/s along a straight road.
What could be the speed with which they approach each other?  1 m/s 2 m/s
x
 (JEE MAIN)
Sol: Let us consider that “A” denotes earth, “B” denotes the first car and
“C” denotes the second car. Therefore, the equation of relative velocity x
for this case is: νBA =
1m / s and νCA = −2m / s.
νCA = νBA + νCB ⇒ −2 = 1 + νCB ⇒ νCB = −2 − 1 = −3m / s
x
This implies that the car “C” is approaching “B” at a speed of –3 m/s along 0 m/s 3 m/s
the straight road. Further, it also means that the car ”B” is approaching
Figure 3.13
P hysi cs | 3.11

“C” at a speed of 3 m/s along the straight road. We, therefore, say that the two cars approach each other at a
relative speed of 3 m/s.
To evaluate relative velocity, we proceed as follows:
•• Apply velocity of the reference object (say object “A”) to other object(s) and hence render the reference object
at rest.
•• The resultant velocity of the other object (“B”) is therefore equal to relative velocity of “B” with respect to “A”.

PLANCESS CONCEPTS

•• The foremost thing in solving problems of relative motion is about visualizing measurement. If we
say a body “A” has relative velocity “v” with respect to another moving body “B”, then we simply
mean that we are making measurement from the moving frame (reference) of “B”.
•• It is helpful in solving problem to make reference object stationary by applying negative of its
velocity to both objects. The resultant velocity of the moving object is equal to the relative velocity
of the moving object with respect to reference object. If we interpret relative velocity in this manner,
it gives easy visualization as we are accustomed to observing motion from stationary state.

Nitin Chandrol (JEE 2012, AIR 134)

Illustration 10: Assume that a boy is riding a cycle at a speed of 5√3 m/s toward east along a straight line. It starts
raining at a speed of 15 m/s in the vertical direction. What is the direction of rainfall as observed by the boy?
(JEE MAIN)
Sol: Let us denote earth, boy and rain with symbols A, B and C, respectively.
The question here provides the velocity of B and C with respect to A (earth).
-VBA VBA
νBA = 5 3 m/s; νCA = 15 m/s x

Now, we need to determine the direction of rain (C) with respect to boy (B), VCB 
i.e., νCB .
VCA
νCA = νBA + νCB ⇒ νCB = νCA − νBA
Thus, we now draw the vector diagram to evaluate the terms on the right
side of the equation. Therefore, here, we need to evaluate “ νCA − νBA ”,
which is equivalent to “ νCA + ( −νBA ) ”. We now apply parallelogram theorem Figure 3.14
to obtain vector sum as represented in the Fig. 3.14 provided.
For the boy (B), the rain appears to fall, making an angle “ θ ” with the vertical (–y direction).
νBA 5 √ 3 1
⇒ tan
= θ = == tan30=
⇒ θ 30
νCA 15 √3

Illustration 11: Consider that a person is driving a car toward east at a speed of 80 km/hr. A train appears to move
toward north with a velocity of 80√3 km/hr to this person. Find the speed of the train as measured with respect to
earth.  (JEE ADVANCED)
Sol: The velocity of the train with respect to earth is the vector sum of its velocity with respect to car and the
velocity of car with respect to earth.
Let us first denote the car and train as “A” and “B,” respectively. Here, we are provided with the y
speed of car (“A”) with respect to earth, i.e., “ ν A ” and speed of train (“B”) with respect to “A,”
i.e., νBA . νA =80km / hr; νBA =
80 √ 3km
3 km/ /hrhr VBA
VB
Now, we are required to find the speed of train (“B”) with respect to earth, i.e., νB . From the
equation of relative motion, we have VA
x
νBA = νB − ν A ⇒ νB = νBA + ν A

Figure 3.15
3 . 1 2 | Motion in a Plane

To evaluate the right-hand side of the equation, we draw vectors “ νBA ” and “ ν A ” and use parallelogram law to find
the actual speed of the train.

⇒ νB {(ν BA + ( νA ) }
)=
2 2 
(
2
 80 3 =

) 
+ 802  160km / hr

6.1 Motion of Boat in a Stream


In this section, we consider a general situation of sailing of a boat in a y
moving stream of water. However, in order to keep our context
simplified, we consider that the stream is unidirectional in x-direction
P Q R
and the width of stream, ”d”, is constant. Let the velocities of boat (A)
and stream (B) be” ν A ” and “ νB ,” respectively with respect to ground.
The velocity of boat (A) with respect to stream (B), therefore, is VA
VAB

ν AB = ν A − νB ⇒ ν A = ν AB + νB  
x
We represent these velocities in the Fig. 3.16 provided. It is clear from O VB
the Fig. 3.16 provided that boat sails in the direction, making an angle Figure 3.16
“ θ ” with y-direction, but reaches destination in different direction.
The boat obviously is carried along the stream in x-direction. This
displacement in x-direction (x = QR) from the directly opposite position to actual position on the other side of the
stream is called the drift of the boat.

6.1.1 Resultant Velocity


We can calculate the magnitude of resultant velocity using the parallelogram theorem,

ν A= (ν 2
AB + νB2 + 2ν AB νB cos α )
where “ α ” is the angle between νB and ν AB vectors. The angle “ β ” formed by the resultant velocity with x-direction
ν AB sin α
is given as: tan β =
νB + ν AB cos α

6.1.2 Time to Cross the Stream


The boat covers a final distance equal to the width of stream ”d” in the time “t” in y-direction. Now, by applying
the concept of independence of motions in perpendicular directions, we can say that boat covers a final distance
“OQ = d” with a speed equal to the component of resultant velocity in y-direction.
Now, the resultant velocity is composed of (i) velocity of boat with y
respect to stream and (ii) velocity of stream. Here, we observe that VABy Vay
velocity of stream is perpendicular to y-direction. Therefore, it does
not have any component in y-direction. We, therefore, conclude that P Q R
the component of the resultant velocity is equal to the component
of the velocity of boat with respect to stream in y-direction. Note VA
that the two equal components shown in the Fig. 3.17 provided are VAB d
geometrically equal as they are altitudes of same parallelogram. 
Hence, ν Ay = ν ABy =ν AB cos θ
x
where ” θ ” is the angle that relative velocity of boat w.r.t stream O VB

d d Figure 3.17
makes with the vertical. =t =
ν Ay ν AB cos θ
Thus, we can use either of these two expressions to calculate time to cross the river, depending on the inputs
available.
P hysi cs | 3.13

6.1.3 Drift of the Boat


We now know that the displacement of the boat in x-direction is independent of motion in the perpendicular
direction. Hence, displacement in x-direction is achieved with the component of resultant velocity in x-direction,
x= ( ν Ax ) t = ( νB − ν ABx ) t = ( νB − ν AB sin θ ) t
d
Then, substituting for time “t”, we have: x= ( νB − ν AB sin θ ) ν
AB cos θ

6.1.4 Shortest Interval to Cross the Stream


d d
The time taken by the boat to cross the river is given by:=t =
ν Ay ν AB cos θ
Clearly, the time taken is minimum for the greatest value of denominator. The denominator is maximum for θ =0°,
d
for this value, tmin =
ν AB
This means that the boat needs to sail in the direction perpendicular to the stream to reach the opposite side in
ν d
minimum time. The drift of the boat for this condition is: x = B
ν AB

PLANCESS CONCEPTS

We have discussed motion with specific reference to boat in a water stream. However, the consideration
is general and is applicable to the motion of a body in a medium. For example, the discussion and analysis
can be extended to the motion of an aircraft, whose velocity is modified by the motion of the wind.

GV Abhinav (JEE 2012, AIR 329)

Illustration 12: An aircraft flies with velocity of 200 ( 2 )km/hr and the wind is blowing from the south. If the
relative velocity of the aircraft with respect to wind is 1000 km/hr, then find the direction in which the aircraft
should fly such that it reaches a destination in the north-east direction.  (JEE MAIN)
Sol: The vector sum of the velocity of the airplane with respect to the wind and the velocity of the wind with
respect to ground is equal to velocity of the aircraft with respect to ground. This net velocity should be in north-
east direction.
We show the velocities pertaining to this problem in the Fig. 3.18 provided. In the Fig. 3.18 provided, OP denotes
the velocity of the aircraft in still air or equivalently it represents the relative velocity of the aircraft with respect to
air in motion; PQ denotes the velocity of the wind and OQ denotes the resultant velocity of the aircraft. However, it
is clear that the aircraft should fly in the direction OP so that it is ultimately led to follow the north-east direction.
We should understand here that one of the velocities is the resultant velocity of the remaining two velocities.
Therefore, it follows that the three velocity vectors are represented by the sides of a closed triangle.
We can now demonstrate the direction of OP, if we can find the angle” θ ”. The y
easiest way to determine the angle between vectors composing a triangle is to
apply the sine law,
Q
OP PQ
= VB
sin 45 sin θ VA
P
Therefore, by substituting these values, we obtain
 VAB
 x
O

Figure 3.18
3 . 1 4 | Motion in a Plane

PQ sin 45 200 2 1


sin θ= = = = 0.2
OP 1000x 2 5
−1
θ =sin (0.2)
Hence, based on the above analysis, the aircraft should steer in the direction, making an angle with east as given
by: θ=' 45 − sin−1 (0.2)

Illustration 13: Assume that a boat, capable of sailing at 2 m/s, moves upstream in a river. The water in the stream
flows at 1 m/s. A person walks from the front to the rear end of the boat at a speed of 1 m/s along the liner
direction. What is the speed of the person (m/s) with respect to the ground?  (JEE MAIN)
Sol: First find the velocity of boat with respect to ground. The velocity y
of man with respect to boat is added to the velocity of boat with
respect to ground to get the velocity of man with respect to ground.
Let us assume that the direction of stream be in x-direction and the VAB VA VB
direction across stream be in y-direction. We further denote boat with
”A”, stream with “B”, and the person with “C”. We can now solve this
problem in two parts. In the first part, we find out the velocity of boat
(A) with respect to ground and then we calculate the velocity of the
person (C) with respect to ground. O x

Figure 3.19
Here,
velocity of boat (A) with respect to stream (B): νBA = –2 m/s y
Velocity of the stream (A) with respect to ground: νB = 1 m/s
Velocity of the person (C) with respect to boat (A): νCA = 1 m/s
Velocity of the person (C) with respect to ground: νC =? VA VCA VC

The velocity of boat with respect to ground is equal to the resultant velocity
of the boat as given by: ν A =νBA + νB ⇒ ν A =−2 + 1 =−1m / s
For the motion of person and boat, the velocity of the person with respect
to ground is equal to the resultant velocity of (i) velocity of the person (C) O x
with respect to boat (A) and (ii) velocity of the boat (A) with respect to
Figure 3.20
ground. Hence, νC =νCA + ν A ⇒ νC =1 + ( −1) =0 .

7. CIRCULAR MOTION
Circular motion  is a movement of an object/particle along the  circumference  of a  circle  or  motion  along a
circular path. However, it can be uniform or non uniform.
Familiar examples of circular motion include an artificial satellite orbiting the earth at constant height, a stone
which is tied to a rope and is being swung in circles and a car turning through a curve in a race track.
Angular displacement of a body is the angle in radians (degrees, revolutions) through which a point or line has
been turned in a specified sense about a specified axis. Angular displacement is denoted by θ .
The angular velocity  is defined as the rate of change of angular displacement. The SI  unit of angular velocity
dθ ν
is radians per second. Angular velocity is usually represented by the symbol omega ( ω ). =
ω ,=
ω where
dt r
v is linear velocity.

Angular acceleration is the rate of change of angular velocity. In SI units, it is measured in radians per second squared
(rad/ s2 ), and is usually denoted by the Greek letter alpha ( α ).

dω d2 θ aΤ
=
α = , or =
α
dt dt2 r
P hysi cs | 3.15

7.1 Uniform Circular Motion


Uniform Circular Motion, involves continuous change in the direction of velocity without any change in its
magnitude (v). A change in the direction of velocity is a change in velocity (v). This implies that UCM is associated
with acceleration and hence force. Thus, UCM signifies “presence” of force.
In other words, UCM requires a force, which is always perpendicular to the direction of velocity. Since the direction
of velocity is continuously changing, the direction of force, being perpendicular to velocity, should also change
continuously.
The direction of velocity along the circular trajectory is always tangential in nature. The perpendicular direction to
the circular trajectory is, therefore, known as the radial direction. It implies that force (and hence acceleration) in
uniform circular motion is radial. For this reason, acceleration in UCM is recognized to require center, i.e., centripetal
(seeking center).
Irrespective of whether circular motion is uniform (constant speed) or non-uniform (varying speed), the circular
motion inherently associates a radial acceleration to ensure that the direction of motion is continuously changed—
at all instants. We learn about the magnitude of radial acceleration soon, but let us be emphatic to differentiate
radial acceleration (accounting change in direction that arises from radial force) with tangential acceleration
(accounting change in the speed that arises from tangential force).
The coordinates of the particle is given by the x- and y-coordinate pair as: x = r cos θ; y = r sin θ
The angle” θ ” is measured anti-clockwise from the x-axis.
The position vector of the position of the particle, r, is represented in terms of unit vectors as:
r = xiˆ + yjˆ ⇒ r = r cos θˆi + r sin θˆj ⇒ r = r(cos θˆi + sin θˆj)
y

The magnitude of velocity of the particle (v) is constant by the definition of


UCM. In component form, however, the velocity (refer to the Fig. 3.21) is:

v
vy
v = ν x ˆi + ν y ˆj; ν x = −ν sin θ; ν y = ν cos θ
P
y x vy vx
; cos θ = ; v = − ˆi + ˆj
vx
sin θ =
r r r r r y
Acceleration: Knowing that speed, ”v” and radius of circle, “r” are constants,  x
we easily differentiate the expression of velocity with respect to time to O x A
obtain expression for centripetal acceleration as:
ν  dy dx  ν
(
a = −  ˆi − ˆj  ⇒ a = − ν y ˆi − ν x ˆj
r  dt dt  r
)
Substituting the value of component velocities in terms of angle, we obtain Figure 3.21

⇒a= −
ν
r
( )
ν cos θˆi − ν sin θˆj = ax ˆi + ay ˆj where ax =
ν2
− cos θ ; ay =
r
ν2
− sin θ
r
It is evident from the equation of acceleration that it varies as the angle with horizontal, “ θ ” change. Therefore, the
magnitude of acceleration is

a= a = ( )
ax 2 + ay 2 ⇒ a = a =
ν
r
{ (
ν2 cos2 θ + cos2 θ )} ⇒a=
ν2
r

Illustration 14: Assume that a cyclist negotiates the curvature of 20 m at a speed of 20 m/s. What is the magnitude
of his acceleration?  (JEE MAIN)
Sol: The speed of the cyclist moving along circular path is constant. So its acceleration is centripetal.
Let the speed of the cyclist be constant. Then, the acceleration of the cyclist is the centripetal acceleration that is
required to move the cyclist along a circular path, i.e., the acceleration resulting from the change in the direction
of motion along the circular path.

ν2 202
Hence, v = 20 m/s and r = 20 m ⇒ a= = = 20m / s2
r 20
3 . 1 6 | Motion in a Plane

7.2 Non-Uniform Circular Motion


We are aware of the fact that the speed of a particle under circular motion is not y
constant.
A change in speed means that unequal length of arc (s) is covered in equal time v1
intervals. It further means that the change in the velocity (v) of the particle is not
limited to change in direction as in the case of UCM. v2

Radial or centripetal acceleration. Change in direction is due to radial acceleration o x


v4
ν2
(centripetal acceleration), which is given by aR = .
r
v3
Tangential acceleration: The non-uniform circular motion basically involves a Figure 3.22
change in speed. This change is accounted by the tangential acceleration, which

results due to a tangential force and which acts along the direction of velocity. aΤ =
dt

7.3 Relation between Angular and Linear Acceleration


The relationship between angular and linear acceleration is shown hereunder.

dν d2s d2 d2 θ
aΤ = = =
dt dt2 dt2
( rθ ) = r = rα
dt2

Illustration 15: A particle, starting from the position (5 m, 0 m), is moving along a circular path about the origin in
x–y plane. The angular position of the particle is a function of time as given here, θ= t2 + 0.2t + 1 . Find (i) tangential
acceleration  (JEE MAIN)
Sol: Differentiate the expression for angular position with respect to time to get angular velocity. Tangential
acceleration is the product of angular acceleration and the radius.
From the data on initial position of the particle, it is clear that the radius of the circle is 5 m.
(i) For determining tangential acceleration, we need to have expression of linear speed in time.
ν = ωr = ( 2t + 0.2 ) × 5 = 10t + 1

We obtain tangential acceleration by differentiating the above function: a=
Τ = 10m / s2
dt

Illustration 16: At a particular instant, a particle is moving at a speed of 10 m/s on a circular path of radius 100 m.
Its speed is increasing at the rate of 1 m/s2. What is the acceleration of the particle?  (JEE MAIN)
Sol: The acceleration of the particle is the vector sum of the centripetal acceleration and the tangential acceleration.
The tangential acceleration is equal to the rate of change of speed.
The acceleration of a particle is the vector sum of mutually perpendicular radial and tangential accelerations. The
magnitude of tangential acceleration given here is 1 m/s2. Now, the radial acceleration at the particular instant is:
ν2 102
a=
R = = 1m / s2
r 100
Hence, the magnitude of the acceleration of the particle is: a = ( )
a = aΤ2 + aR 2 =12 + 12 m / s2 =2m / s2

Illustration 17: Which of the following expressions represent the magnitude of centripetal acceleration?:

d2r dν dθ
(A) (B) (C) r (D) None of these  (JEE MAIN)
dt 2 dt dt

Sol: The magnitude of centripetal acceleration depends on the square of the magnitude of velocity.
P hysi cs | 3.17

dν dθ
The expression represents the magnitude of tangential acceleration. The differential represents the
dt dt

magnitude of angular velocity. The expression r represents the magnitude of tangential velocity and the
2 dt
dr
expression is second-order differentiation of position vector (r). This is the actual expression of acceleration of
dt2
d2r
a particle under motion. Hence, the expression represents the magnitude of total or resultant acceleration.
dt2
Hence, option (d) alone is correct.

Illustration 18: A particle is executing circular motion. But the magnitude of velocity of the particle changes from
zero to (0.3i + 0.4j) m/s in a period of 1 second. The magnitude of average tangential acceleration is:

(A) 0.1 m/ s2 (B) 0.2 m/ s2 (C) 0.3 m/ s2 (D) 0.5 m/ s2 (JEE MAIN)
Sol: Tangential acceleration is equal to the rate of change of speed. Average tangential acceleration is change in
speed divided by total time.
∆ν
The magnitude of average tangential acceleration is the ratio of change in speed and time as given by: aΤ =
∆t
Now,=
∆ν (0.3 2
0.42
+= ) aΤ 0.5m / s2
0.25 0.5m / s;=
=

Hence, option (d) alone is correct.

PLANCESS CONCEPTS

Radial acceleration contributes in changing the direction of velocity of an object, but it does not affect the
magnitude of velocity. However, tangential acceleration affects the speed of the object in motion.

Vaibhav Krishan (JEE 2009, AIR 22)

FORMULAE SHEET

(a) Projectile Motion

2usin θ
Time of flight: T =
g
u2 sin2θ
Horizontal range: R =
g
y
2 2
u sin θ 
Maximum height: H = 
2g

Trajectory equation (equation of path): x


Figure 3.23
gx2  x
y x tan θ −
= = x tan θ  1 − 
2 2
2u cos θ  R

Projection on an inclined plane


3 . 1 8 | Motion in a Plane

(b) Relative Motion

v AB ( velocity of A with respect to B=


) v A − vB y
aAB ( acceleration of A with respect to B=
) aA − aB B
drift = x C
Relative motion along straight line = xBA= xB − x A

(c) Crossing River: A boat or man in a river always moves in the


direction of resultant velocity of velocity of boat (or man) and d vr
velocity of the river flow.
vmR
vm
(d) Shortest Time: Velocity along the river, VX = VR x
A
Figure 3.24
Velocity perpendicular to the river, Vf = VmR

2
The net speed is given by=
Vm VmR + VR2

(e) Shortest Path: Velocity along the river, Vx = 0 y

2
and velocity perpendicular to river
= Vy VmR − VR2 B

2
The net speed is given by=
Vm VmR − VR2

at an angle of 90° with the river direction. d


vr
velocity Vy is used only to cross the river, therefore time to cross the river,
vmR  vm

d d x
t
= = and velocity v x is zero, therefore, in A
vy 2
v mR − vR2 Figure 3.25

this case the drift should be zero.


vR
vR v mR=
= sin θ 0 or =vR vmR sin θ or θ =sin−1
v mR

 
(f) Rain Problems: vRm
= vR − v m or v=
Rm vR2 + v m
2

(g) Circular Motion

θ2 − θ1 ∆θ at
i. Average angular velocity =
ωav =
t2 − t1 ∆t a
dθ  V
ii. Instantaneous angular velocity ω = O
dt ar or a P

ω2 − ω1 ∆ω
iii. Average angular acceleration=
αav =
t2 − t1 ∆t Figure 3.26

dω dω
iv. Instantaneous angular acceleration α = =ω
dt dθ

v. Relation between speed and angular velocity v =


rω and v =
ω r
dV
vi. Tangential acceleration (rate of change of speed) at =
dt
P hysi cs | 3.19

V2
vii. Radial or normal or centripetal acceleration ar = = ω2r
r

  
( )
12
viii. Total acceleration a =at + ar ,a =a2t + ar2 W
AC
tation
Ro

ix. Angular acceleration α = (non-uniform circular motion)
dt

v 2 mv 2
x. Radius of curvature =
R = Figure 3.27
a⊥ F⊥

Solved Examples

JEE Main/Boards Then,=


y x tan
= θ
2u2 tan2 θ
g
Example 1: A particle is projected horizontally with a
speed u from the top of a plane inclined at an angle θ The distance AP = l = x2 + y 2
with the horizontal. How far from the point of projection
will the particle strike the plane? 2u2 2u2
2
= tan θ 1 + tan
= θ tan θ sec θ
Sol: Take the x-axis parallel to the horizontal. Take the g g
y-axis along the vertical. Along x-axis velocity is uniform.
Along y-axis initial velocity is zero and acceleration is Examples 2: A projectile is projected at an angle 60°
uniform.
from the horizontal with a speed of
Take, X–Y axes as shown in Figure. Suppose that the time (in seconds) after which the inclination of the
(
3 + 1 m / s . The )
particle strikes the plane at point P with coordinates (x projectile with horizontal becomes 45° is:
and y). Consider the motion between A and P.
Sol: Take the x-axis along the horizontal. Take the y-axis
u vertically upwards. Along x-axis velocity is uniform.
A X
Along y-axis initial velocity is positive and acceleration
y is uniform and negative.
P Let “u” and “v” be the speed at the two specified angles.
x 
The initial components of velocities in horizontal and
Y vertical directions are:
Motion in x-direction: initial velocity = u ux = ucos60

Acceleration = 0; X = ut … (i) uy = usin60

Motion in y-direction: initial velocity = 0


Y
1
Acceleration = g; y = gt2 … (ii)
2
Eliminating t from (i) and (ii)
vy v
2
1 x 45°
y= g Also,
= y x tan θ . vx
2 u2
u
gx2 2u2 tan θ ux
Thus, = x tan θ giving x = 0, or,
2u2 g 60°
X
O uy
2
2u tan θ
Clearly the point P corresponds to x =
g
3 . 2 0 | Motion in a Plane

Similarly, the components of velocities, when the horizontal direction. Now, considering accelerated
projectile makes an angle 45 with horizontal and vertical motion in horizontal direction, we have:
directions are:
1 1g
x= R=' ux Τ + ax Τ2 ⇒ R=' ux Τ +   Τ2 ; R=' R + H

ν x =ν cos 45 ; ν y =ν sin 45 
2 22
But we know that horizontal component of velocity
remains unaltered during motion. Hence, Example 4: An airplane has to go from a point A to
another point B, 500 km away due 30° east of north. A
ucos60 wind is blowing due north at a speed of 20 m/s. The air
ν x= ux ⇒ ν cos 45= ucos60 ⇒ ν= speed of the plane is 150 m/s. (i) Find the direction in
cos 45
which the pilot should head the plane to reach the point
Here, we know initial and final velocities in vertical B. (ii) Find the time taken by the plane to go from A to B.
direction. We can apply v = u + at in vertical direction to
know the time as required: Sol: The vector sum of the velocity of the airplane with
respect to the wind and the velocity of the wind with
ν sin 45 =u + at =usin60 − gt respect to ground is equal to velocity of the aircraft with
ucos60 − ν sin 45 respect to ground. This N
⇒ ν cos
= 45 ucos60
= 
⇒t net velocity should be C R
g
in the direction A to B. 30 20m/s
Substituting value of “v” in the above equation, we have: 30 D
In the resultant 150m/s vw = 20m/s

 cos60   direction R, the plane
W
usin60 − u   X sin 45
 cos
  
45 
cos60 reaches the point B. S
⇒t= usin60 − u   X sin 45


 cos
 g 45 

⇒t= Velocity of wind V w =20 m/s
g ( 3 +1 )  ( )
3 − 1  
⇒t
=
u
g
(
sin60 − cos60= 
)
⇒t
( 310+ 1 )  ( )
32− 1 
Velocity of aero plane V a =150 m/s

⇒t
=
u
g
(
sin60 − cos60= 
)
⇒t
10  2


y
2  
⇒ t= = 0.1s
20
2
⇒ t= = 0.1s
20
Q -VA O VA P
Example 3: A projectile is at an angle “ θ ” from the
x

horizontal at the speed “u”. If an acceleration of “g/2” VBA  VB


is applied to the projectile due to wind in horizontal
direction, then find the new time of flight, maximum
height and horizontal range. R S

Sol: Take the x-axis along the horizontal. Take the y-axis
vertically upwards. Along x-axis initial velocity is positive In ∆ACD, according to the sine formula
and acceleration is uniform and positive. Along y-axis
initial velocity is positive and acceleration is uniform 20 150 20
∴ = ⇒ sinA= sin30
and negative. sinA sin30  150
20 1 1
The acceleration due to wind affects only the motion = × = ⇒ A = sin−1 (1 / 15)
in horizontal direction. It would, therefore, not affect 150 2 5
attributes like time of flight or maximum height that
results exclusively from the consideration of motion in 20
vertical direction. The generic expressions of time of R
flight, maximum height and horizontal range of flight
with acceleration are given as under: 
150
2uy u2y gΤ2 uxuy
=
Τ ; =
H = ; =
R
g 2g 4 g
(i) The direction is sin−1 (1 / 15) east of the line AB.
The expressions above revalidate the assumption made
in the beginning. We can see that it is only the horizontal (ii) sin−1 (1 / 15) = 3 48' ⇒ 30° + 3°48'= 33°48'
range that depends on the component of motion in
P hysi cs | 3.21

We note here that WR = UQ = 4 m/s


R= 1502 + 202 + 2(150)20 cos33 48'= 167m / s
Clearly, triangles ORS and ORW are congruent as two
s 500000
Time= = = 2994 sec = 49= 50min sides and one enclosed angle are equal.
v 167
WR = RS = 4 m/s; OR = OR; ∠ORW = ∠ORS = 90°
Example 5: Rain drops appear to fall in vertical direction Hence, ∠WOR = ∠SOR = 45°
to a person, who is walking at a velocity 4 m/s in a given
direction. When the person doubles his velocity in the RS RS
In triangle ORS, sin45 = ⇒ OS = = 4 2m / s
same direction, the rain drops appear to come at an OS sin45
angle of 45° from the vertical. Find the speed of the rain
drop. Example 6: A swimmer wishes to cross a 500 m wide
Sol: Velocity of rain with respect to the person is equal river flowing at 5 km/h. His speed with respect to water
to the vector sum of the velocity of rain with respect is 3 km/h. (i) If he heads in a direction making an angle
to ground and the negative of the velocity of person θ with the flow, find the time he takes to cross the river.
with respect to ground. The direction of velocity of rain (ii) Find the shortest possible time to cross the river.
with respect to person is known in each case. Assume Sol: Time taken to cross the river will depend on
a direction of velocity of rain with respect to ground the component of velocity of swimmer which is
and draw the vector diagrams for velocity of rain with perpendicular to the river flow. For shortest time this
respect to person for both the cases. component should be maximum, i.e. θ=90o.
This is a slightly tricky question. Readers may like to (i) The vertical component 3 sin θ takes him to the
visualize the problem and solve on their own before opposite side.
going through the solution given here.
Distance = 0.5 km, velocity = 3 sin θ km/h
Let us consider the situation under two cases. Here, only
the directions of relative velocities in two conditions are Dis tance 0.5km 10
=Time = = min.
given. The Figure on the left represents initial situation. Velocity 3sin θ km / h sin θ
Here, the vector OP represents velocity of the person
( VA ); OR represents relative velocity of rain drop with
respect to person ( VA ); OS represents velocity of rain 5km/h
drop. 5km/h
3km/h
The given figure represents situation when person

starts moving with double velocity. Here, the vector OT
represents velocity of the person ( VA1 ); OW represents (ii) Here vertical component of the velocity, i.e., 3 km/hr
relative velocity of rain drop with respect to person ( takes him to the opposite side (θ=90o).
VBA1 ). We should note that velocity of rain ( VB ) drop
remains the same and as such, it is represented by OS Dis tance 0.5
Time
= = = 0.16hr
represents as before. Velocity 3
∴ 0.16hr = 60 × 0.16 = 9.6 =10minute
According to the question, we are required to know the
speed of the rain drop. It means that we need to know
the angle “ θ ” and the side OS, which is the magnitude Example 7: Two tall buildings are 200 m apart. With
of velocity of rain drop. It is intuitive from the situation what speed must a ball be thrown horizontally from
that it would help if we consider the vector diagram and a window of one building 2 km above the ground so
carry out geometric analysis to find these quantities. that it will enter a window 40 m from the ground in the
For this, we substitute the vector notations with known other?
magnitudes as shown hereunder.
Sol: The time taken by ball A B
to fall from the height of
U 4 Q 4 O 4 P 4 T
X 2000 m to the height of 40 W₁ x
m (with zero initial velocity
45° 
VB
in vertical direction)
2000 m
should be equal to the
VBA1
VBA W₂
time taken by ball to cover 40 m
W 4 R 4 S a horizontal distance
of 200 m with constant 200 m
velocity in horizontal direction.

3 . 2 2 | Motion in a Plane

Figure shows the conditions of the problem. Here, A Figure shows the conditions of the problem. The boy’s
and B are the two tall buildings having windows W1 window is at O and friend’s window is at A. Let the boy
and W2, respectively. The window W1 is 2 km (=2000 m) throw the stone with a velocity v 0 making an angle θ
above the ground while window W2 is 40 m above the with the horizontal so as to enter the window at A. The
ground. We want to throw the ball from window W1 stone will follow the parabolic path with A as the highest
with such a horizontal speed ( ν x0 ) so that it enters the point on the trajectory of stone.
window W2. Note that the horizontal range of the ball R
is R = 200 m. Let t sec be the time taken by the ball ∴ =40 or horizontal range, R = 2 × 40 = 80 m
2
to reach from window W1 to window W1. This time will
depend upon the vertical motion (downward) alone. Motion in a plane

For vertical motion: V02 sin2θ V02 2sin θ cos θ


= R =
hg = 9.8ms
2000 –−2=
40 g g
= ; ν y0= 1960
0 m; g=9.8 ms-2: vy0=0
V02 sin2 θ H tan θ
1 2 2h 2 × 1960 and
= H =
∴ or
h
∴= gt or=
t = = 20 s 2g R 4
2 g 9.8
H 10 1
R 200 tan θ = 4 × = 4 × = ∴ θ = 26.56°
NowR = ν x0 t ∴ ν x0 = = = 10ms−1 R 80 2
t 20
Maximum height attained, H = 10 m
Example 8: A particle moves in a circle of radius 20 cm. Now, the projection velocity v 0 can be found by
Its linear speed is given by v = 2t; t, is in second and substituting the value of θ in formula for H.
v in metre/second. Find the radial and tangential
acceleration at t = 3 s. v 02 sin2 θ
H=
Sol: Radial acceleration depends on the square 2g
of instantaneous speed and the radius. Tangential 2 2gH 2 × 9.8 × 10
acceleration is equal to the rate of change of ∴ v=
0 = = 980 or
sin2 θ sin2 26.56°
instantaneous speed.
v0
= 980 31.3 ms−1
=
The linear speed at t = 3 s is V = 2 t = 6 m/s.
The radial acceleration at t = 3 s is
Example 10: A body is projected with a velocity of 40 ms−1 .
2 2
36m / s After 2 s, it crosses a vertical pole of height 20.4 m.
ar = ν2 / r = = 180m / s2
0.20m Calculate the angle of projection and the horizontal
The tangential acceleration is range.

dν d(2t) Sol: Use second equation of motion with constant


at
= = = 2m / s2 acceleration in vertical direction.
dt dt
Let θ be the angle of projection. For vertical motion,
Example 9: A boy wants to throw a letter wrapped
1
over a stone to his friend across the street 40 m wide. =h ( v 0 sin θ ) t − gt2
The boy’s window is 10 m below friend’s window. How 2
1
= ( 40 sin θ ) × 2 − × 9.8 × ( 2 )
2
should he throw the ball? or 20.4
2
Sol: We assume that or 20.4
= 80 sin θ − 19.6
the boy throws the ball
20.4 + 19.6 40 1
such that the maximum A ∴ sin θ= = = ∴ θ= 30°
height attained by the 80 80 2
ball is H = 10 m. This
v 0 v 02 sin2θ
Horizontal Range, R =
implies that the range H=10 m
g
of the ball is R = 40×2
( 40 ) × sin60°
2
= 80 m. Thus from the 
=
formulae of H and R we O B 9.8
can find the values of
R
=40 m = 141.1 m
initial velocity and the
2

angle of projection.
P hysi cs | 3.23

JEE Advanced/Boards  1 
(uAx t )
2
s
= +  uAy t − gt2 
 2 
Example 1: A particle A is projected with an initial 2
velocity of 60 m/s at an angle 30° to the horizontal. At
(  1
) 
2
= 30 3 × 1.09 +  30 × 1.09 − × 10 × 1.09 × 1.09 
the same time a second particle B is projected in opposite 2
 
direction with initial speed of 50 m/s from a point at a
distance of 100 m from A. If the particles collide in air, s = 62.64m
find (i) the angle of projection α of particle B, (ii) time
when the collision takes place and (iii) the distance of P Example 2: A man running on a horizontal road at
(
from A, where collision occurs. g = 10m / s2 ) 8 km/h finds the rain falling vertically. He increases his
speed to 12 km/h and finds that the drops make an
60 m/s
50 m/s angle 30° with the vertical. Find the speed and direction
of the rain with respect to the road.
30°  Sol: Velocity of rain with respect to the man is equal to
B
A
100 m the vector sum of the velocity of rain with respect to
ground and the negative of the velocity of man with
Sol: This problem is best solved in the reference frame respect to ground. The direction of velocity of rain with
of one of the two particles, say particle B. The relative respect to man is known in each case.
acceleration between the particles is zero. So in this
We have,
reference frame, the particle A moves with uniform velocity.   
v= v rain,man + v man,road  … (i)
(i) Taking x- and y-directions as shown in the figure. rain,road

Here, The two situations given in the problem may be


represented by the following diagrams.
 ˆ a =
aA =
−gj; g gjˆ Y
Vman, road O Vman, road = 12 km/h
uAx 60 cos30
= = ° 30 3m / s 8 km/h 30°
uAy 60 sin30
= = ° 30m / s  Vrain, road 
Vrain, man

V ra
30°
uBx =
−50 cos α ; uBy =
50 sin α

in,
an

ro
,m A

ad
and Relative acceleration
ain

X
Vr

between the two is zero as


 
aA = aB . Hence, the relative uAB
motion between the two is (a) (b)
uniform. It can be assumed that B is at rest and A is
 
moving with uAB . Hence, the two particles will collide, if v rain,road is same in magnitude and direction in both the

uAB is along AB. This is possible only when uAy = uBy diagram. Taking  horizontal components in E.q. (i) for the
i.e., component of relative velocity along the y-axis first diagram, v rain,road sin α =8km / h  … (ii)
should be zero. Now, consider given figure Draw a line OA ⊥ Urain,man as
30 50sin α ∴ α = sin−1 ( 3 / 5 )
Or= shown. Taking components in Eq. (i) along the line OA,
we have
(ii) Now,
V0(12)

( )
uAB = uAx − uBx = 30 3 + 50 cos α m / s
V01
 4
=  30 3 + 50 ×  m /=
 5
s (30 )
3 + 40 m / s
0
V02


AB 100
Therefore, the time of collision =
is t = 
( )

Or t = 1.09 s uAB 30 3 + 40 v rain,road sin 30o + α =12 km / h cos30o  … (iii)

From (ii) and (iii),


(iii) Distance of point P from A where collision takes
place is
3 . 2 4 | Motion in a Plane

sin ( 30° + α ) v1 ucos θ


=
12 × 3
=
sin α 8×2 At half the greatest height during upward motion,
sin30° cos α + cos30° sin α 3 3 y=
h / 2,ay =
−g,uy =
usin θ
or, =
sin α 4
Using v 2y − u2y =
2ay y
1 3 3 3 3
or, =cot α + =or,cot α h
2 2 4 2 weget, v 2y − u2 sin2 θ= 2 ( −g)
2
3 u2
sin2
θ u2 sin2 θ
or, α =cot −1 or
= v 2y u2 sin2 θ − g × =
2 2g 2
8km / h 
From(ii) v=
rain,road = 4 7km / h u2 sin2 θ  usin θ
sin α = ∴ h =  or v y
 2g  2
Example 3: Two bodies were thrown simultaneously
from the same point; one, straight up, and the other, at Hence, the resultant velocity at half of the greatest
an angle of θ= 60° to the horizontal. The initial velocity height is
of each body is equal to v 0 = 25 ms−1 . Neglecting the
air drag, find the distance between the bodies t = 1.70 s = u2 sin2 θ
v v 2x + =
v 2y u2 cos2 θ +
later. 2
v
Sol: The relative acceleration of the bodies is zero. The Given, 1 = 2
solution of this problem becomes interesting in the v2 5
frame attached with one of the bodies. 2
v u cos2 θ
2
2 1 2
= ∴ 1 = or =
Let the body thrown straight up be 1 and the other body v22
u sin θ 5 1 + 1 tan2 θ 5
2 2
be 2, then for the body 1 in the frame of 2 from the u2 cos2 θ +
2 2
kinematical equation for constant acceleration (since
2 2
both are moving under constant acceleration) is 2 + tan θ= 5,or tan θ= 3; tan θ= 3; θ= 60°
1
r12 =r0(12) + v 0(12) t + w12 t2
2
So,r12 v 0(12) t
= = ( w12 0=
and r0(12) 0) Example 5: A cannon fires successively two shells with
velocity v 0 = 250 m / s ; the first at the angle θ1 = 60°
or,=
r12 v o(12) t But, v=
01 v=
02 v0 and the second at the angle θ2 = 45° to the horizontal,
the azimuth being the same. Neglecting the air drag,
Therefore, from properties of triangle find the time interval between firings leading to the
collision of the shells.
v 0(12
= ) v 20 + v 20 − 2v 0 v 0 cos ( π / 2 − θ0 )
Sol: At the instant of y

Hence, the sought distance is collision, the horizontal


and vertical distances
r12 v 0 t 2 (1 − sin=
θ0 ) 22m covered by both the shells
v0
= v0 (x,y)
is will be equal respectively. P
Get two equations, one for ₁
₂
Example 4: The velocity of a projectile when it is at the horizontal distance and the x
greatest height is 2 / 5 times its velocity when it is at half other for vertical distance. O

of its greatest height. Determine its angle of projection. Let the shells collide at the point P(x, y). If the first shell
Sol: The maximum height is known in terms of initial takes t seconds to collide with second and ∆ t be the
velocity and angle of projection. Horizontal component time interval between the firings, then
of velocity of projectile remains constant. Use the third
equation of motion with uniform acceleration in vertical =x v 0 cos
= θ1 t v 0 cos θ2 ( t − ∆t ) ... (1)  ... (i)
direction to find the vertical component of velocity at = x v 0 cos = θ t v cos 1 θ ( t − ∆t ) 2 ... (1)
height equal to half of the maximum height. and
= y v 0 sin θ21 ( t − ∆0 t ) − 2g ( t − ∆t ) ... (2)  ... (ii)
21 
y v sin θ ( t − ∆t ) − g ( t − ∆t )
2
Suppose the particle is projected with velocity u at an and
= ... (2)
From Eq.(1)0 t = 2 ∆t cos θ2 2 ... (3)
angle θ with the horizontal. Horizontal component of its cos∆θt2cos
− cosθ2 θ1
velocity at all height will be ucos θ . From
From Eq. (i) t =
Eq.(1) ... (3) ... (iii)
From Eqs. (2) andcos(3)θ2 − cos θ1
At the greatest height, the vertical component of
velocity is zero, so the resultant velocity is= From Eqs. (2)(and
2v 0 sin θ2 )
θ1 − (3)
From
∆t Eqs. (ii) and (iii) as ∆t ≠ 0
( cos
g2v θ2(+θ1cos
0 sin − θθ21))
= ∆t as ∆t ≠ 0
= 11 sg ((on 2 + cos θ1 ) values)
cos θsubstituting
= 11 s (on substituting values)
and
= y v 0 sin θ2 ( t − ∆t ) − g ( t − ∆t ) ... (2)
2
∆t cos θ2
From Eq.(1) t = ... (3)
cos θ2 − cos θ1 P hysi cs | 3.25

From Eqs. (2) and (3)


2v 0 sin ( θ1 − θ2 ) Tangential acceleration
∆t as ∆t ≠ 0 dv
g ( cos θ2 + cos θ1 ) =α T = =k  ... (ii)
dt
= 11 s (on substituting values) v 2 k 2 t2
Normal acceleration , αN = r = r  … (iii)

Example 6: A particle A moves or αN = 4 πnk


A
along a circle of radius R = 5
(α )
1/2
cm so that its radius vector r r ∴ α= 2
+ αN2 = k 2 + 16π2n2k 2 
R T  
relative to the point O rotates 1/2
with the constant angular O 1/2
= 0.50 1 + 16 × ( 3.14 ) × ( 0.10 ) 
2 2
= k 1 + 16π2n2 
velocity ω =0.40 rads−1 . Find the    
modulus of the velocity of the = 0.8m / s2
particle, and the modulus and
direction of its acceleration.
Example 8: Two boats, A and B move away from a
Sol: Angular velocity about point O is given. We need buoy anchored at the middle of a river along mutually
to find the angular velocity about point C, ωC. Once ωC is perpendicular straight lines: the boat A along the river,
known, velocity and acceleration can be found out from and the boat B across the river. Having moved off an
formulae of circular motion. equal distance from the buoy the boats returned. Find
Angular velocity of point A, with respect to center C of τ
the ratio of times of motion of boats A if the velocity
the circle or turning rate of line CA taking the line OCX τB
d ( 2θ )  −dθ  of each boat with respect to water is n = 1.2 times
as reference line becomes ωc =− =2   =2ω
dt  dt  greater than the stream velocity.
Because angular speed of line OA is
Sol: The velocity of boat B will be vector sum of velocity
ω = −dθ / dt . of river flow and the velocity of B with respect to river.
These three vectors form a right triangle. The velocity
The turning rate of line CA is also the turning rate of of boat B is the base, the velocity of river flow is the
velocity vector of point A, which is given by v A / R . perpendicular and the velocity of B with respect to river
is the hypotenuse.
Therefore, v A =ωCR =2 ( ω) R =4 cm / s (on substituting
the values). Let l be the distance covered by the boat A along the
The acceleration of the particle will be centripetal as its river as well as by the boat B across the river. Let v 0 be
speed is constant. the stream velocity and v′ the velocity of each boat with
respect to water. Therefore, the time taken by the boat
v 2 42 A in its journey
a
= = s2 3.2 cm / s2
cm / =
R 5 l l 2lv '
tA = + =
v '+ v 0 v '− v 0 v ' 2 − v 2
0
Example 7: A point moves along a circle with a velocity
v = kt, where k = 0.5 m / s2 . Find the total acceleration And for the boat B
of the point at the moment when it has covered the
l l 2l
nth fraction of the circle after the beginning of motion, tB = + =
v '2 − v 20 v '2 − v 02 v '2 − v 02
1
where n = .
10
tA v' η
Sol: This is the case of circular motion with constant Hence,
= =
tB v ' − v 20
2 2
η −1
tangential acceleration. Use second equation of motion
with constant acceleration and zero initial velocity to  v' 
find the time required to cover 1/10 of the circle. Total  where η = 
 v0 
acceleration is the vector sum of tangential acceleration
and centripetal acceleration. tA
On substitution, = 1.8 (approx)
tB
ds 1 1
v= = kt or ∫ s0 ds =∫
k 0t t dt ∴ s = kt2
dt 2
For completion of nth fraction of the circle, Example 9: Two particles move in a uniform gravitational
field with an acceleration ‘g’. At the initial moment the
s = (2πr)n or t2= ( 4πnr ) / k  ...(i) ... (i) particles were located at one point and moved with
velocities v1 3.0
= = ms−1 and v 2 4.0 ms1 horizontally in
3 . 2 6 | Motion in a Plane

opposite directions. Find the distance between the Both the particles are initially at the same position and
particles at the moment when their velocity vectors have same acceleration g, so
become mutually perpendicular.
r0 0, =
= w 0, and v=
0 v1 − v 2 .
Sol: The relative acceleration between the particles is
Thus, the sought distance is
zero. Initial relative distance is zero. So the final relative
distance between them is equal to product of time r
= v 0=t ( v1 + v 2 ) t (using the value of t)
and relative velocity. The time required can be found
by using the equations of final velocities in Cartesian v1 + v 2
= v1 v 2
coordinates. g
= 2.5 m, on substituting the values of v1 , v 2 and g.
v₂ v₁
Example 10: A boy whirls a stone in a horizontal circle
of radius 1.5 m and at height 2.0 m above level ground.
The string breaks, and the stone flies off horizontally and
v’₂ v’₁ strikes the ground after travelling a horizontal distance
of 10 m. What is the magnitude of the centripetal
v₂t v₁t
acceleration of the stone while in circular motion?
Let the velocities of the particles (say v '1 and v '2 ) Sol: The time of fall of the stone depends on the
become mutually perpendicular after time t. Then, their height of the stone and can be found using the second
velocities become equation of motion with constant acceleration and zero
initial velocity. The horizontal component of stone’s
v1ˆi + gt ˆj ; v 2, =
v1, = − v 2ˆi + gt ˆj
velocity remains constant is equal to the horizontal
As v1, ⊥ v 2, , so, v1, ⋅ v 2, =
0 distance covered by the stone divided by the time of
fall. The centripetal acceleration is equal to the square
( )( )
or v1ˆi + gt ˆj ⋅ v 2ˆi + gt ˆj =
0 of the horizontal velocity divided by the radius of the
2 2 horizontal circle.
or − v1 v 2 + g t =
0
v1 v 2 2h 2×2 10
=t = v = 15.63m / s
= 0.64 s ; =
Hence, t = g 9.8 t
g
In the frame attached with 2 for the particle 1 v2
a
= = 163m / s2
R
1
r =r0 + v 0 t + wt2
2

JEE Main/Boards

Exercise 1 Q.3 What is a projectile? Give its examples. Show that


the path of projectile is a parabolic path when projected
horizontally from a certain height.
Projectile Motion
Q.4 Show that there are two angles of projection for
Q.1 What do you understand by motion in two which the horizontal range is the same.
dimensions? When an object is moving with uniform
velocity in two dimensions, explain displacement,
Q.5 Find (i) time of flight, (ii) Max. height and (iii)
velocity and find the equations of motion of the object.
horizontal range of projectile projected with speed
v AB ( velocity of A with respect to B=
) v A − vB
Q.2 Find the relation for (i) velocity and time (ii)
displacement and time, when an object is moving with aAB ( acceleration of A with respect to B=
) aA − aB
uniform acceleration in two dimensions.
making an angle θ with the horizontal direction from
ground.
P hysi cs | 3.27

Q.6 Find the magnitude and direction of the velocity of


where î is a unit vector along horizontal and ˆj is unit
an object at any instant during the oblique projection
vector vertically upward. Find the Cartesian equation of
of a projectile. 2
its path. (g = 10 m s )
Q.7 Find (i) the path of projectile, (ii) time of flight, (iii)
horizontal range and (iv) maximum height, when a Q.18 Find the maximum horizontal range of a cricket
projectile is projected with velocity υ making an angle θ ball projected with a velocity of 80 m/s. If the ball is
with the vertical direction. to have a range of 100 3 m, find the least angle of
projection and the least time taken.
Q.8 What is centripetal acceleration? Find its magnitude
and direction in case of a UCM of an object. Q.19 A bullet fired from a rifle attains a maximum height
of 5m and crosses a range of 200 m. Find the angle of
Q.9 A stone is dropped from the window of a bus projection.
moving at 60 kmh−1 . If the window is 196 cm. high,
find the distance along the track which the stone moves Q.20 A target is fixed on the top of a pole 13 m high.
before striking the ground. A person standing at a distance 50 m from the pole is
capable of projecting a stone with a velocity 10 g m/s.
Q.10 A hiker stands on the edge of a cliff 490 m above If he wants to strike the target in shortest possible time,
the ground and throws a stone horizontally with an at what angle should he project the stone.
initial speed of 15 ms−1 . Neglecting air resistance, find
the time taken by the stone to reach the ground, and Q.21 A particle is projected with a velocity u so that its
the speed with which it hits the ground. Take g = 9.8 horizontal range is twice the greatest height attained.
Find the horizontal range of it.
m / s2 .

Q.22 A drunkard walking in a narrow lane takes 5 steps


Q.11 A ball is thrown horizontally from the top of a
forward and 3 steps backward, and so on. Each step
tower with a speed of 50 ms−1 . Find the velocity and is 1 m long and required 1s. Determine how long the
position at the end of 3 second g = 9.8 ms−2 . drunkard takes to fall in a pit 13 m away from the start.

Q.12 A body is projected downward at an angle of 30° Q.23 A jet airplane travelling at the speed of 500km
to the horizontal with a velocity of 9.8 m/s from the top h−1 ejects its projects of combustion at the speed of
of a tower 29.4 m high. How long will it take before
1500km h−1 relative to the jet plane. What is the speed
striking the ground?
of the later with respect to observer on the ground.

Q.13 Prove that a gun will shoot three times as high


Q.24 A car moving along a straight highway with speed
when its angle of elevation is 60° as when it is 30° , but
cover the same horizontal range. of 126 km h−1 is brought to a stop within a distance of
200 m. What is the retardation of the car (assumed
uniform) and how long does it take for the car to stop?
Q.14 Prove that the maximum horizontal range is 4
times the maximum height attained by a projectile
which is fired along the required oblique direction. Q.25 Two trains A and B of length 400 m each are moving
on two parallel tracks with a uniform speed of 72km h−1
Q.15 Two particles are projected from the ground in the same direction with A head of B. The driver of B
simultaneously with speeds of 30 m/s and 20 m/s at decides to overtake A and accelerate by 1 ms−2 . If after
angles 60° and 30° with the horizontal on the same 50 s, the guard of B just brushes past the driver of A,
direction. Find maximum distance between them on what was the original distance between them?
2
ground where they strike. g = 10 m s .
Q.26 Two towns A and B are connected by a regular bus
Q.16 A projectile has the same range when the maximum service with a bus leaving in either direction every T min.
A man cycling with a speed of 20km/h in the direction
height attained by it is either H1 or H2 . Find the relation
of A and B notices that a bus goes past him every 18
between R, H1 and H2 . min in the direction of his motion, and every 6 min in
the opposite direction. What is the period T of the bus
(
Q.17 A projectile is given an initial velocity of ˆi + 2ˆj , ) service and with what speed (assumed constant) do the
buses ply on the road?
3 . 2 8 | Motion in a Plane

Q.27 A ball is dropped from a height of 90 m on a floor. he applies brakes and reduces his speed at a constant
At each collision with the floor, the ball loses one tenth −1
rate of 0.5 ms per second. Find the magnitude of the
of its speed. Plot the speed-time graph of its motion net acceleration of the cyclist.
between t=0 to 12s. (g=10 ms−2 )
Q.38 A particle moves in a circle Y
Q.28 A man walks on a straight road from his home to of radius 4.0 cm clockwise at Q
a market 2.5 km away with a speed of 5 km/h. Finding constant speed of 2 cm s−1 . If
the market closed, he instantly turns and walks back
x̂ and ŷ are unit acceleration
with a speed of 7.5 km/h. What is the (a) magnitude of X
vectors along X-axis and O P
average velocity and (b) average speed of the man, over
Y-axis respectively, find the
the interval of time (i) 0 to 30 min. (ii) 0 to 50 min (iii) 0
acceleration of the particle at
to 40 min?
the instant half way between PQ figure.

Q.29 A dive bomber, diving at an angle of 53° with the


Q.39 Three girls skating on a Q
vertical, released a bomb at an altitude of 2400ft. the
circular ice ground of radius
bomb hits the ground 5.0 s after being released. (i) What
200m start from a point P on the
is the speed of the bomber? (ii) How far did the bomb
edge of the ground and reach a
travel horizontally during its flight? (iii) What were the A B
point Q diametrically opposite C
horizontal and vertical components of its velocity just
to P following different paths
before striking the ground?
as shown in figure. What is the
magnitude of the displacement
Circular Motion vector for each? For which girl is P
this equal to the actual length of
Q.30 Calculate the angular velocity of the minute’s hand path skated?
of a clock.
Q.40 A cyclist starts from the centre O of a circular park
Q.31 What is the angular velocity in radian per second of radius 1m reaches the edge P of the park, then cycles
of a fly wheel making 300 r.p.m.? along the circumference, and returns to the centre
along QO. If the round trip takes 10 minutes, what is the
Q.32 The wheel of an automobile is rotating with 4 (i) net displacement (ii) average velocity and (iii) average
rotations per second. Find its angular velocity. If the speed of the cyclist?
radius of the fly wheel is 50cm, find the linear velocity of
a point on its circumference. Q.41 A cyclist is riding with a speed of 36 km h−1 . As he
approaches a circular turn on the road of radius 140m,
Q.33 The angular velocity of a particle moving in a he applies break and reduces his speed at the constant
circle of radius 50 cm is increased in 5 minutes from 100
rate of 1m s−2 . What is the magnitude and direction of
revolutions per minute. Find (a) angular acceleration (b)
the net acceleration of the cyclist on the circular turn?
linear acceleration.
Q.42 A particle is moving in a
Q.34 A body is moving in a circle of radius 100 cm with
circle of radius r centres at O
a time period of 2 second. Find the acceleration.
with constant speed υ . What is O 
the change in velocity in moving
Q.35 An insect trapped in a circular groove of radius from A to B? In the figure. Given 40°
12cm moves along the grove steadily and completes 7
B
∠AOB = 40° . 
revolutions in 100s. (i) What is the angular speed, and A
the linear speed of the motion? (ii) Is the acceleration
vector a constant vector? What is its magnitude?
A
Q.43 A particle originally at rest
Q.36 Calculate the centripetal acceleration of a point on h
B
at the highest point of a smooth R
the equator of earth due to the rotation of earth due  
vertical circle of radius R, is slightly
to the rotation of earth about its own axis. Radius of 
displaced. Find the vertical distance
O mg
earth=6400 km.
below the highest point where the
particle will leave the circle.
Q.37 A cyclist is riding with a speed of 27 kmh−1 . As he
approaches a circular turn on the road of radius 80.0m,
P hysi cs | 3.29

Exercise 2 Q.7 A projectile is fired with a velocity at right angles


to the slope which is inclined at an angle θ with the
horizontal. The expression for the range R along the
Projectile Motion
incline is
Single Correct Choice Type 2v 2 2v 2
(A) sec θ (B) tan θ
g g
Q.1 A particle is projected with a certain velocity at an
angle θ above the horizontal from the foot of a given 2v 2 v2
(C) tan θ sec θ (D) tan2 θ
plane inclined at an angle of 45° to the horizontal. If the g g
particle strikes the plane normally, then equals
(A) tan−1 (1 3) (B) tan−1 (1 2) Q.8 A hunter tries to hunt a monkey with a small, very
poisonous arrow, blown from a pipe with initial speed
(C) tan−1 (1 2) (D) tan−1 3 v 0 . The monkey is hanging on a branch of a tree at
height H above the ground. The hunter is at a distance L
Q.2. Two projectiles A and B are thrown with the same from the bottom of the tree. The monkey sees the arrow
such that A makes angle θ with the horizontal and B leaving the blow pipe and immediately lose the grip
makes angle θ with the vertical, then on the tree, falling freely down with zero initial velocity.
(A) Both must have same time of fight The minimum initial speed v 0 of the arrow for hunter to
succeed while monkey is in air
(B) Both must achieve same maximum height
(C) A must have more horizontal range than B
(A)
(
g H2 + L2 ) (B)
gH2

(D) Both may have same time of flight 2H H2 + L2

Q.3 A projectile is fired with a speed u at an angle θ with


(C)
(
g H2 + L2 ) (D)
2gH2
the horizontal. Its speed when its direction of motion H H2 + L2
makes an angle ' α ' with the horizontal is
Q.9 A swimmer swims in still water at a speed=5 km/hr.
(A) usec θ cos α (B) usec θ sin α He enters a 200 m wide river, having river flow speed=4
(C) ucos θ sec α (D) usin θ sec α km/hr at point A and proceeds to swim at an angle of
127° with the river flow direction. Another point B is
Q.4 A ball is projectile from top of tower with a velocity located directly across A on the other side. The swimmer
of 5 m/s at an angle of 53° to horizontal. Its speed when lands on the other bank at a point C, from which he
it is at a height of 0.45m from the point of projection is: walks the distance CB with a speed=3 km/hr. The total
time in which he reaches from A to B is
(A) 2 m/s (B) 3 m/s
(A) 5 minutes (B) 4 minutes
(C) 4 m/s (D) Data insufficient
(C) 3 minutes (D) None

Q.5 particle is dropped from the height of 20m from Q.10 A boat having a speed of 5 km/hr. in still water,
horizontal ground. A constant force acts on the crossed a river of width 1 km along the shortest possible
particle in horizontal direction due to which horizontal path in 15 minutes. The speed of the river in Km/hr.
acceleration of the particle becomes 6 ms-2. Find the
horizontal displacement of the particle till it reaches (A) 1 (B) 3 (C) 4 (D) 41
ground.
Q.11 A motor boat is to reach at a point 30° upstream
(A) 6m (B) 10 m (C) 12 m (D) 24 m
(w.r.t. normal) on other side of a river flowing with
velocity 5m/s. Velocity of motorboat w.r.t. water is 5 3
Q.6 Find time of flight of projectile thrown horizontally m/s. The driver should steer the boat at an angle
−1
with speed 10 ms from a long inclined plane which (A) 120° w.r.t. stream direction
makes an angle of θ= 45° from horizontal.
(B) 30° w.r.t. normal to the bank
(A) 2 sec (B) 2 2 sec (C) 30° w.r.t. the line of destination from starting point.
(C) 2 sec (D) None of these (D) None of these
3 . 3 0 | Motion in a Plane

Q.12 A flag is mounted on a car moving due north with Q.18 If acceleration of A is 2 m/s2 to left and acceleration
velocity of 20 km/hr. Strong winds are blowing due East of B is 1 m/s2 to left, then acceleration C is
with velocity of 20km/hr. The flag will point in direction A B
(A) East (B) North-East
(C) South-East (D) South-West

Q.13 Three ships A, B & C are in motion. The motion of C


A as seen by B is with speed v towards north-east. The
motion of B as seen by C is with speed v towards the (A) 1 m/s2 upwards (B) 1 m/s2 downwards
north-west. Then as seen by A, C will be moving towards
(C) 2 m/s2 downwards (D) 2m/s2 upward
(A) north (B) south (C) east (D) west

Circular Motion
Q.14 Wind is blowing in the north direction at speed of
2 m/s which causes the rain to fall at some angle with Single Correct Choice Type
the vertical. With what velocity should a cyclist drive so
that the rain appears vertical to him? Q.19 Two racing cars of masses m1 and m2 are moving
(A) 2m/s south (B) 2m/s north in circles of radii r1 and r2 respectively, their speeds are
such that they each make a complete circle in the same
(C) 4m/s west (D) 4 m/s south
time t. The ratio of the angular speed of the first to the
second car is:
Q.15 When the driver of a car A sees a car B moving
towards his car and at a distance 30m, takes a left turn (A) m1 : m2 (B) r1 : r2 (C) 1:1 (D) m1r1 : m2r2
of 30° . At the same instant the driver of the car B takes
a turn to his right at an angle 60° . The two cars collides Q.20 A particle moves in a circle of radius 25 cm at two
after two seconds, then the velocity (in m/s) of the car revolution per sec. The acceleration of the particle in
A and B respectively will be : [assume both cars to be m / s2 is:
moving along same line with constant speed] (A) π2 (B) 8π2 (C) 4π2 (D) 2π2
(A) 7.5, 7.5 3 (B) 7.5, 7.5
Q.21 Two particle and Q are located at distance rP and
(C) 7.5 3 , 7.5 (D) None
rQ respectively from the centre of a rotating disc such
that rP > rQ :
Q.16 At a given instant, A is moving
with velocity of 5m/s upwards. What is (A) Both P and Q have the same acceleration
velocity of B at that time (B) Both P and Q do not have any acceleration
(A) 15 m/s ↓ (C) P has greater acceleration than Q
A
(B) 15 m/s ↑ B (D) Q has greater acceleration than P
(C) 5 m/s ↓
(D) 5 m/s ↑ Q.22 When particle moves in a circle with a uniform
speed:
Q.17 The pulleys in the diagram (A) Its velocity and acceleration are both constant
are all smooth and light. The (B) Its velocity is constant but the acceleration changes
acceleration of A us a upwards
and the acceleration of A is a (C) Its acceleration is constant but the velocity changes
upward and the acceleration A (D) Its velocity and acceleration both change
of C is f downwards. The C
acceleration is Q.23 If a particle moves in a circle described equal
(A) 1 2(f − a) up B angles in equal times, its velocity vector:

(B) 1 2(a+ f) down (A) Remains constant


(B) Changes in magnitude
(C) 1 2(a+ f) up
(C) Changes in direction
(D) 1 2(a− f) up
(D) Changes both in magnitude and direction
P hysi cs | 3.31

Q.24 If the equation for the displacement of a particle Q.2 A boat which has a speed of 5 km/h in still water
moving on a circular path is given by ( θ= crosses a river of width 1 km along the shortest possible
) 2t + 0.5 ,
3
path in 15 mi. The velocity of the river water in km/h is
where θ is in radians and t in seconds, then the angular
velocity of the particle after 2 sec from its start is: (1988)

(A) 8 rad/sec (B) 12 rad/sec Assertion Reasoning Type


(C) 24 rad/sec (D) 36 rad/sec
(A) If statement-I is true, statement-II is true:
Q.25 The second’s hand of a watch has length 6cm. statement-II is the correct explanation for statement-I
Speed of end point and magnitude of difference of (B) if statement-I is true, Statement-II is true: statement‑II
velocities at two perpendicular ll be: is not a correct explanation of statement-I
(A) 6.28 & 0 mm/s (B) 8.88 & 4.44 mm/s (C) If statement-I is true: statement-II is false
(C) 8.88 & 6.28 m/s (D) 6.28 & 8.88 mm/s (D) If Statement-I is false: statement-II is true

Q.26 A fan is making 600 revolutions per minute. If after


some time it makes 1200 revolutions per minute, then Q.3 Statement-I: For an observer looking out through
the window of a fast moving train, the nearby objects
increases in its angular velocity is:
appear to move in the opposite direction to the train,
(A) 10π rad/sec (B) 20π rad/sec while the distant objects appear to be stationary.
(C) 40π rad/sec (D) 60π rad/sec Statement-II: If the observer and the object are moving

at velocities v 1 and v 2 respectively with reference to a
Q.27 A wheel completes 2000 revolutions to cover the
laboratory frame, the
 velocity of the object with respect
9.5 km. distance, then the diameter of the wheel is: 
to the observer is v 2 − v 1 (2008)
(A) 1.5 m (B) 1.5 cm (C) 7.5 cm (D) 7.5 m
Q.4 For a particle in uniform circular motion the
Q.28 A body moves with constant angular velocity on a 
circle. Magnitude of angular acceleration is: acceleration a at a point P (R, θ ) on the circle of radius
R is (here θ is measures from the x-axis)  (2010)
2
(A) rω (B) Constant
v2 v2 v2 v2
(C) Zero (D) None of the above (A) − cos θˆi + sin θˆj (B) − sin θˆi + sin θˆj
R R R R
Q.29 For a particle in a uniformly accelerated (speed v2 v2 v2 ˆ v2 ˆ
increasing uniformly) circular motion: (C) − cos θˆi − sin θˆj (D) i+ j
R R R R
(A) Velocity is radial and acceleration is transverse only.
(B) Velocity is transverse and acceleration is radial only Q.5 A boy can throw a stone up to a maximum height
(C) Velocity is radial and acceleration has both radial of 10 m. The maximum horizontal distance that the boy
and transverse components can throw the same stone up to will be (2012)
(D) Velocity is transverse and acceleration has both
(A) 20 2 m (B) 10 m (C) 10 2 m (D) 20 m
radial and transverse components

Q.30 A particle moves in a circular orbit under the Q.6 Two cars of masses m1 and m2 are moving in circles
force proportional to the distance ‘r’. The speed of the of radii r1 and r2, respectively. Their speeds are such that
particle is: they make complete circles in the same time t. The ratio
of their centripetal acceleration is (2012)
(A) Proportional of r 2 (B) Independent of r
(A) m1r1: m2r2 (B) m1 : m2 (C) r1 : r2 (D) 1 : 1
(C) Proportional to r (D) Proportional to 1/r

Q.7 A projectile is given an initial velocity of ˆi + 2ˆj ( )


Previous Years’ Questions m/s, where î is along the ground and ˆj is along the
vertical. If g = 10 m/s2, the equation of its trajectory is:
Q.1 A river is flowing from west to east at a speed of 5 (2013)
m/min. A man on the south bank of the river, capable
of swimming at 10m/min in still water, wants to swim (A) y = 2x − 5x2 (B) 4y = 2x − 5x2
across the river in the shortest time. He should swim in (C) 4y = 2x − 25x2 (D) y = x – 5x2
a direction (1983)
3 . 3 2 | Motion in a Plane

JEE Advanced/Boards

Exercise 1 Q.7 A shell is fired from a point O at an angle of 60°


with a speed of 40 m/s & it strikes a horizontal plane
through O. at a point A. The gun is fired a second time
Projectile Motion
with the same angle of elevation but a different speed
v. If it hits the target which starts to rise 9 3 m/s at the
Q.1 A particle moves in the plane XY with constant
same instant as the shell is fired, find v. (Take g=10 m/
acceleration a directed along the negative y axis. The
equation of motion of the particle has the form y = αx − βx2 , s2 )
where α and β are positive constants. Find the velocity
of the particle at the origin of coordinates. Q.8 A cricket ball thrown from a height of 1.8 m at an
angle of 30° with the horizontal at a speed of 18 m/s is
Q.2 Two seconds after projection, a projectile is moving caught by another field’s man at a height of 0.6 m from
at 30° above the horizontal, after one more second it is the ground. How far were the two men apart?
moving horizontally. Find the magnitude and direction
of its initial velocity. (g=10 m/ s2 ) Q.9 A batsman hits the ball at a height 4.0 ft. from the
ground at projection angle of 45° and the horizontal
Q.3 A ball is projected from O with an initial velocity 700 range is 350 ft. Ball falls on left boundary line, where a
cm/sec in a direction 37° above the horizontal. A ball B, 24 ft height fence is situated at a distance of 320 ft. Will
500 cm away from O on the line of the initial velocity of the ball clear the fence? u
P

A, is released from rest at the instant A is projected. Find


Q.10 (i) A particle is projected
(i) The height through which B falls, before it is hit by A. with a velocity of 29.4 m/s at an h
(ii) The direction of the velocity A at the time of impact angle of 60° to the horizontal.
(Given g=10 m s−2 , sin 37° =0.6) Find the range on a plane

inclined at 30° to the horizontal
when projected from a point of
Q.4 On a frictionless horizontal surface, assumed to be
the plane up the plane.
the x-y plane, a small trolley A is moving along a straight
line parallel to the y-axis with a constant velocity of (ii) determine the velocity with which a stone must be
( )
3 − 1 m/s. At a particular instant, when the line OA projected horizontally from a point P, so that it may hit
the inclined plane perpendicularly. The inclination of the
makes an angle 45° with the x-axis, a ball is thrown
plane with the horizontal is θ and P is h metre above the
along the surface from the origin O. Its velocity makes
foot of the incline as shown in the Figure.
an angle φ with the x-axis and it hits the trolley. y
(i) The motion of the ball is observed from the frame of Q.11 During the
the trolley. Calculate the angle θ made by the velocity volcanic eruption
vector of the ball with the x-axis in this frame. chunks of solid rock
(ii) Find the speed of the ball with respect to the surface are blasted out of the A 37°
x
if φ = 4 θ 3. volcano.
3.3km

(i) At what initial speed B


Q.5 If R us the horizontal range and u would a volcanic object 9.4km
h, the greatest height of a projectile, h have to be ejected at
find the initial speed. g = 10m s 
2 37° to the horizontal
R from the vent A in
order to fall at B as shown in Figure.
(ii) What is the time of flight. (g=9.8 m/ s2 )
Q.6 A stone is thrown horizontally from a tower. In 0.5
second after the stone began to move, the numerical Q.12 A projectile is projected with an initial velocity of
value of its velocity was 1.5 times its initial velocity. Find
the initial velocity of stone. ( )
6iˆ + 8ˆj ms−1, ˆi =unit vector in horizontal direction and ˆj =
unit vector in vertical upward direction then calculate
its horizontal range, maximum height and time of flight.
P hysi cs | 3.33

Q.13 An aeroplane is flying at a height of 1960 metre in Q.21 A butterfly is flying with velocity y
a horizontal direction with a velocity of 100 m/s, when 10iˆ + 12ˆj m/s and wind is blowing B
it is vertically above an object M on the ground it drops along x axis with velocity u. If butterfly
a bomb. If the bomb reaches the ground at the point N, starts motion from A and after some A 37°
then calculate the time taken by the bomb to reach the time reaches point B, find the value of u. x
ground and also find the distance MN.

Q.14 A projectile is projected from the base of a hill Q.22 Rain is falling vertically with a speed of 20 m/ s−1
whose slope is that of right circular cone, whose axis is relative to air. A person is running in the rain with a
vertical. The projectile grazes the vertex and strikes the velocity of 5 m/ s−1 and a wind is also blowing with a
hill again at a point on the base. If θ be the semi-vertical
speed of 15 m/ s−1 (both towards east). Find the angle
angle of the cone, h its height u the initial velocity of
with the vertical at which the person should hold his
projection and α the angle of projection, show that
umbrella so that he may not get drenched.
gh(4 + tan2 θ)
(i) tan
= θ 2cot α (ii) u2 =
2 Circular Motion
Q.15 A person is standing on a truck moving with a
constant velocity of 14.7 m/s on a horizontal road. The Q.23 A bullet is moving horizontally with certain
man throws a ball in such a way that it returns to the velocity. It pierces two paper discs rotating co-axially
truck after the truck has moved 58.8 m. Find the speed with angular speed ω separated by a distance  . If the
and the angle of projection (i) as seen from the truck, (ii) hole made by the bullet on 2nd disc is shifted through
as seen from the road. an angle θ with respect to that in the first disc, find the
velocity of the bullet, (change of velocity in the bullet is
neglected)
Q.16 Two bodies are thrown simultaneously from the
same point. One thrown straight up and the other at an
Q.24 Position vector of a particle performing circular
angle α with the horizontal. Both the bodies have equal
velocity of v 0 Neglecting air drag, find the separation of motion is given by = r 3iˆ + 4ˆj m, velocity vector is
 
the particle at time t. v= −4iˆ + 3ˆj m/s. If acceleration is a =−7iˆ − ˆj m/ s2 find
the radial and tangential components of acceleration.
Q.17 Two particles move in a uniform gravitational field
with an acceleration g. At the initial moment the particles Q.25 An astronaut is rotating in a rotor having vertical
were located at one point and moved with velocities axis and radius 4m. If he can withstand upto acceleration
3 m/s and 4 m/s horizontally in opposite directions. Find of 10g. Then what is the maximum number of permissible
the distance between the particles at the moment when (
revolution per second? g = m s2 . )
their velocity vectors become mutually perpendicular.
Q.26 A racing-car of 1000 kg moves round a banked
Q.18 A particle is projected from O at an elevation α track at a constant speed of 108 km h−1 . Assuming the
and after t second it has an elevation β as seen from total reaction at the wheels is normal to the track and
the point of projection. Prove that its initial velocity is the horizontal radius of the track is 90 m, calculate the
gt cos β angle of inclination of the track to the horizontal and
. the reaction at the wheels.
sin(α − β)
Q.27 A particle A moves along a A
Q.19 The velocity of a particle when it is at its greatest circle of radius R=50 cm so that its r
2 radius vector r relative to the point O R
height is of its velocity when it is at half its greatest (see figure) rotates with the constant O
5 C
angular velocity ω =0.40 rad/sec.
height. Find the angle of projection of the particle. Find the modulus of the velocity
of the particle and modulus and
Q.20 A man crosses a river in a boat. If he crosses the direction of its total acceleration.
river in minimum time he takes 10 minutes with a drift
120 m. If he crosses the river taking shortest path, he Q.28 A wet open umbrella is held upright and is rotated
takes 12.5 minutes. Assuming v b r > v r find about the handle at uniform rate of 21 revolutions is 44s.
(i) Width of the river If the rim of the umbrella circle is 1 meter in diameter
(ii) Velocity of the boat with respect to water, and the height of the rim above the floor is 1.5m, find
where the drops of water spun off the rim and hit the
(iii) Speed of the current.
floor.
3 . 3 4 | Motion in a Plane

Q.29 A spaceman in training is rotated in a seat at the (A) The maximum height reached by the projectile can
end of horizontal rotating arm of length 5m. if he can be 0.25 m.
withstand acceleration up to 9 g, what is the maximum (B) The minimum velocity during its motion can be
number of revolutions per second permissible? Take
15 m/s
g = 10m s2
3
(C) The time taken for the flight can be sec.
Q.30 An insect on the axle of a wheel observes the 5
motion of a particle and ‘find’ it to take its place along (D) Minimum kinetic energy during its motion can be 6J.
the circumference of a circle of radius ‘R’ with a uniform
angular speed ω . The axle is moving with a uniform
speed ‘v’ relative to the ground. How will an observer Q.2 A particle is projected from the ground with velocity
on the ground describe the motion of the same point? u at angle θ with horizontal. The horizontal range,
maximum height and time of fight are R , H and T
Q.31 A stone is thrown horizontally with a velocity 10 u2 sin2θ u2 sin2 θ
m/s. Find the radius of curvature of its trajectory in 3 respectively. They are given
= by, R = ,H
g 2g
second after the motion began. Disregard the resistance 2usin θ
of air. and T =
g

Q.32 A table with smooth horizontal surface is fixed Now keeping u as fixed, θ is varied from 30° to 60° .
Then,
in a cabin that rotates with a uniform angular velocity
ω =220s−1 in a circular path of radius R=700m. A smooth (A) R will first increase then decrease, H will increase and
groove AB of length L=7 m is made on the surface of T will decrease
the table. The groove makes an angle θ= 30° with the (B) R will first increase then decrease while H and T both
radius OA of the circle in which the cabin rotates. A will increase
small particle is kept at the point A in the groove and is
(C) R will first decrease while H and T will increase
released to move along AB. Find the time taken by the
particle to reach the point B. (D) R will first increase while H and T will increase

B Q.3 The trajectory of particle 1 with respect to particle


O A  2 will be

R (A) A parabola (B) A straight line



(C) A vertical straight line (D) A horizontal straight line

Q.33 A smooth sphere of radius R is made to translate in Q.4 If v1 cos=


θ1 v 2 cos θ2 , then choose the incorrect
a straight line with a constant acceleration a. A particle statement.
kept on the top of the sphere is released from there at (A) One particle will remain exactly below of above the
zero velocity with respect to the sphere. Find the speed other particle
of the particle with respect to the sphere as a function
(B) The trajectory of one with respect to other will be a
of the angle θ slides.
vertical straight line
Q.34 If a particle is rotating in a circle of radius R with (C) Both will have the same range
velocity at an instant v and the tangential acceleration is
(D) None of these
a. Find the net acceleration of the particle.

Q.5 If v1 sin=
θ1 v 2 sin θ2 , then choose the incorrect
Exercise 2 statement.
(A) The time of flight of both the particle will be same
Projectile Motion
(B) The maximum height attained by the particles will
Single Correct Choice Type be same
(C) The trajectory of one with respect to another will be
Q.1 A projectile of mass 1 kg is projected with a velocity a horizontal straight line
of 20 m/s such that it strikes on the same level as the (D) None of these
point of projection at a distance of 3 m. Which of the
following options is incorrect?
P hysi cs | 3.35

Multiple Correct Choice Type (B) P will hit the roof BC, if a>g B C
(C) P will hit the wall CD if a<g
Q.6 Choose the correct alternative (s) u
(D) May be either (A), (B) or (C), P
(A) If the greatest height to which a man can throw a depending on the speed of projection A D
stone is h, then the greatest horizontal distance upto of P
which he can throw the stone is 2h.
(B) The angle of projection for a projectile motion whose Q.11 The vertical height of point P above the ground is
range R is m times the maximum height is tan−1 (4/n) twice that of point Q. A particle is projected downward
with a speed of 5 m/s from P and at the same time another
(C) The time of flight T and the horizontal range R of a particle is projected upward with the same speed from
projectile are connected by the equation= gT2 2R tan θ Q. Both particles reach the ground simultaneously, if PQ
where θ is the angle of projection. is lie on same vertical line then
(D) A ball is thrown vertically up. Another ball is thrown (A) PQ=30 m
at an angle θ with the vertical. Both of them remain in
air for the same period of time. Then the ratio of heights (B) PQ=60 m
attained by the two balls 1:1. (C) Time of flight of the stones
(D) Time of flight of the stones=1/3s
Q.7 If it is the total time of flight, h is the maximum
height & R is the range for horizontal motion, the x & y
co-ordinates of projectile motion and time t are related Q.12 Two particles A & B projected along different
as: directions from the same point P on the ground with the
same speed of 70 m/s in the same vertical plane. They
 t  t  X  X hit the ground at the same point Q such that PQ=480m.
(A) y 4th  T   1 − T  =
= (B) y 4th    1 − 
   R
  R  Then: (Use g=9.8 m/ s2 , sin−1 0.96= 74° , sin−1 0.6
= 37° )
 T  T  R  R (A) Ratio of their times of flights is 4:5
(D) y 4th    1 − 
(C) y 4th    1 −  =
=
 t  t X
  X
(B) Ratio of their maximum heights is 9:16
(C) Ratio of their minimum speeds during flight is 4:3
Q.8 A particle moves in the xy plane with a constant
acceleration ‘g’ in the negative y-direction. Its equation (D) The bisector of the angle between their directions
of motion is =y ax − bx2 , where a and b are constants. of projection makes 45° with horizontal.
Which of the following is correct?
Comprehension Type
(A) The x-components of its velocity is constant.
(B) At the origin, the y-component of its velocity is a A projectile is thrown with a velocity of 50m s−1 at an
angle of 53° with the horizontal.
g .
2b
(C) At the origin, its velocity makes an angle tan−1 (a) Q.13 Choose the incorrect statement
with the x-axis. (A) It travels vertically with a velocity of 40m s−1
(D) The particle moves exactly like a projectile.
(B) It travels horizontally with a velocity of 30m s−1
Q.9 A ball is rolled off along the edge of a horizontal (C) The minimum velocity of the projectile is 30 m s−1
table with velocity 4m/s. It hits the ground after time
0.4s. Which of the following are correct? (D) None of these

(A) The height of the table is 0.8 m. Q.14 Determine the instants at which the projectile is at
(B) It hits the ground at an angle of 60° with the vertical the same height.
(C) It covers a horizontal distance 1.6 m from the table (A) t=1s and t=7s (B) t=3s and t=5s
(D) It hits the ground with vertical velocity 4m/s (C) t=2s and t=6s (D) all the above

Q.10 A large rectangular box moves vertically downward Q.15 The equation of the trajectory is given by
with an acceleration a. A toy gun fixed at A and aimed
towards C fires a particle P. = 240x − x2
(A) 180y = x2 − 240x
(B) 180y

(A) P will hit C if a=g = 135x − x2


(C) 180y = x2 − 135x
(D) 180y
3 . 3 6 | Motion in a Plane

Two projectile are thrown simultaneously in the same Q.21 Statement-I: Three projectiles are moving in
plane from the same point. If their velocities are v1 and different paths in the air. Vertical component of relative
v 2 at angles θ1 and θ2 respectively from the horizontal, velocity between any of the pair does not change with
then answer the following questions time as long as they are in air. Neglect the effect of air
friction.
Statement-II: Relative acceleration between any of the
Match the Columns
pair of projectile is zero.
Q.16 Match the quantities is column I with possible
options from column II. Circular Motion

Particle’s Motion Trajectory Q.22 An object follows a curved path. The following
(A) Constant velocity (p) straight line quantities may remain constant during the motion-

(B) Constant speed (q) Circular (A) Speed (B) Velocity

(C) Variable acceleration (r) Parabolic (C) Acceleration (D) Magnitude of acceleration

(D) Constant acceleration (s) Elliptical Q.23 The position vector of a particle in a circular motion
about the origin sweeps out equal area in equal times-
Assertion Reasoning Type (A) Velocity remains constant

(A) Statement-I is true, statement-II is true and (B) Speed remains constant
statement‑II is correct explanation for statement-I. (C) Acceleration remains constant
(B) Statement-I is true, statement-II is true and statement‑II (D) Tangential acceleration remains constant
is NOT the correct explanation for statement-I
(C) Statement-I is true, statement-II is false.
(D) Statement-I is false, statement-II is true.
Previous Years’ Questions
Q.1 A large heavy box is sliding without friction down
Q.17 Statement-I: The speed of a projectile is minimum
a smooth plane of inclination
at the highest point.
θ . From a point P on the
Statement-II: The acceleration of projectile is constant bottom of the box, a particle 
during the entire motion. is projected inside the box. P
O

The initial speed of the particle



Q.18 Statement-I: Two stones are simultaneously with respect to the box is
projected from level ground from same point with same u and the direction makes
speeds but different angles with horizontal. Both stones an angle α with the bottom as shown in
move in same vertical plane. Then the two stones may the Figure.  (1998)
collide in mid-air. (i) Find the distance along the bottom of the box
Statement-II: For two stones projected simultaneously between the point of projection P and the point Q where
from same point with same speed at different angles the particle lands (Assume that the particle does not hit
with horizontal, their trajectories may intersect at some any other surface of the box. Neglect air resistance.)
point. (ii) If the horizontal displacement of the particle as seen
by an observer on the ground is zero, find the speed of
Q.19 Statement-I: If separation between two particles the box with respect to the ground at the instant when
does not change then their relative velocity will be zero. the particle was projected.
Statement-II: Relative velocity is the rate of change of
position of one particle with respect to another. Q.2 Airplanes A and B are flying with constant velocity
in the same vertical plane at angles 30° and 60° with
Q.20 Statement-I: The magnitude of relative velocity of respect to the horizontal respectively as shown in the
A with respect to B will be always less than VA . figure. The speed of A is 100 3 ms-1. At time t = 0 s,
Statement-II: The relative velocity of A with respect to B an observer in A finds B at a distance of 500 m. This
is given by VAB observer sees B moving with a constant velocity
= VA − VB .
perpendicular to the line of motion of A. If at t = t0, A just
P hysi cs | 3.37


escapes being hit by B, t0 in seconds is  (2014) Q.3 The distance r of the block at time t is  (2016)

A
B
(A)
4
(
R 2 ωt
)
e + e−2ωt (B)
R
2
cos2ωt


(C)
R
2
cos ωt (D)
4
(
R ωt
e + e−ωt )
o o
30 60

PlancEssential Questions
JEE Main/Boards JEE Advanced/Boards

Exercise 1 Exercise 1
Q.15 Q.20 Q.22 Q.3 Q.7 Q.10

Q.25 Q.26 Q.28 Q.15 Q.17 Q.20

Q.29 Q.35 Q.40 Q.22 Q.23 Q.32

Q.41
Exercise 2
Exercise 2 Q.9 Q.12 Q.15

Q.1 Q.5 Q.8 Q.18 Q.20 Q.21

Q.9 Q.11 Q.15

Q.17 Q.27

Answer Key

JEE Main/Boards Q.12 2.0 s


Q.15 25 3 m.
Exercise 1
Q.16 R=4 H1H2
Q.9 10.54 m Q.17 y=2x-5x2

Q.10 10s; 99.1 m s−1 Q.18 653.06 m, 7° 42’ with horizontal, 2.19 s
Q.19 5° 43’
Q.11 58 m s−1 ; 30° 27’ with horizontal; 44.1 m below
and 150 m horizontally away from the starting point Q.20 30° 58’
3 . 3 8 | Motion in a Plane

Q.21 4u2 5g Q.33 π /30 rad. s-2, 5 π /3 cm s-2


Q.22 37 seconds Q.34 987.7 cm s-2
Q.23 1000 km h-1 Q.35 (i) 0.44 rad/s; 5.3 cm s-1 (ii) Not constant, 2.3 cm s-2
Q.24 -3.06ms-2 ; 11.43 s Q.36 0.03 m/s2
Q.25 450 m Q.37 0.86 ms-2
Q.26 40 km h-1 ; 9 min. Q.38 –( xˆ + yˆ )/ 2cm /s2
Q.28 (i) (a) 5km/h, (b) 5km/h; (ii) (a) 0, (b) 6 km/h; Q.39 400 m; girl B
(iii) (a) 1.875 km/h, (b) 5.625 km/h
Q.40 (i) Zero; (ii) 0; (iii) 6×10-3 m/s
Q.29 (i) v0=667 ft/s (ii) 2667 ft (iii) vx=534 ft/s, vy=560 ft/s
5  10 
Q.41 ms-2; β =tan−1  
Circular Motion 7  7 

Q.30 π /1800 rad. s-1 Q.42 ∆v = v( −0.24iˆ + 0.64ˆj)

Q.31 31.4 rad. s-1 Q.43 h=R/3

Q.32 8π rad.s-1 ; 1257.1 cm s-1

Exercise 2

Projectile Motion
Single Correct Choice Type

Q.1 D Q.2 D Q.3 C Q.4 C Q.5 C Q.6 C Q.7 C


Q.8 A Q.9 B Q.10 B Q.11 C Q.12 C Q.13 B Q.14 B
Q.15 C Q.16 A Q.17 A Q.18 A

Circular Motion
Single Correct Choice Type

Q.19 C Q.20 C Q.21 C Q.22 D Q.23 C Q.24 C Q.25 D


Q.26 B Q.27 A Q.28 C Q.29 D Q.30 C

Previous Years’ Questions


Q.1 A Q.2 B Q.3 B Q.4 C Q.5 D Q.6 C Q.7 A

JEE Advanced/Boards Q.3 2.55m, 27o43’


Q.4 (i) 45° , (ii) 2m/sec
Exercise 1 Q.6 4.4 m/s
Q.7 50 m/s
Projectile Motion
Q.8 30.55 m

(1 + α2 )a / 2β Q.9 Yes
Q.1 v=
0
Q.10 (i) 58.8 m (ii) 2gh
Q.2 20 3 m/s, 60° 2 + cot2 θ
P hysi cs | 3.39

Q.11 (i) u=340m/s (ii) 46 s.  


Q.24 ar =−3iˆ − 4ˆjm / s2 ,aN =−4iˆ − 3ˆjm / s2
Q.12 9.8 m, 3.3 m, 1.6s.
5
Q.13 20 s, 2000 m Q.25 fmax = rev / sec

Q.15 (i) 19.6 m/s upward, (ii ) 24.5 m/s at 53° with
Q.26 45°, 2 × 10 4 N
horizontal
40 × 10−2 m s ,a =
Q.27 V = 32 × 102 m s2
Q.16 v 0 t 2(1 − sin α )
Q.17 2.47 m Q.28 0.83 metre on x-axis

Q.19 60° Q.29 0.675 rev/s


Q.20 200m, 20 m/min, 12 m/min Q.30 =
x R cos ωt + vt,=
y R sin ωt,cycloid
Q.21 6 m/s Q.31 334 m
Q.22 tan−1 (1 / 2) 2L
Q.32 2
ω R cos θ
Circular Motion 12
Q.33 2R(asin θ + g − gcos θ)
ω
Q.23 v = 2
θ  v2 
2
Q.34 a + 
R 
 
Exercise 2

Projectile Motion

Single Correct Choice Type

Q.1 D Q.2 B Q.3 B Q.4 C Q.5 D

Multiple Correct Choice Type

Q.6 A, B, C, D Q.7 A, B Q.8 A, B, C, D Q.9 A, C, D Q.10 A, B Q.11 A, C Q.12 B, C, D

Comprehension Type

Q.13 A Q.14 D Q.15 A

Match the Columns

Q.16 A → p; B → p, q, r, s; C → p, q, r, s; D → p, r

Assertion Reasoning Type

Q.17 B Q.18 D Q.19 D Q.20 D Q.21 A

Circular Motion

Multiple Correct Choice Type

Q.22 A, D Q.23 B, D

Previous Years’ Questions


u2 sin2α ucos(α + θ)
Q.1 (i) (ii) (down the plane) Q.2 5 Q.3 D
gcos θ cos θ
3 . 4 0 | Motion in a Plane

Solutions

JEE Main/Boards Let horizontal velocity = v


x
x = vt ⇒t=  ... (i)
Exercise 1 v
Let height of projection= h
Projectile Motion
1 2
Height y = h – gt
Sol 1: Consider two perpendicular unit vectors i, j. At 2
any instant, if we are able to express the position of an 1 x2
y=h– g ∴ y = a – bx2
object w.r.t to initial position as a linear combination of 2 v2
i, j, then we say object is having two dimensional motion g
a = h, b =
in the plane of î , ˆj . 2v 2
Displacement is the shortest distance between two ⇒ Parabolic path
points.
Displacement v2
Velocity = Sol 4: Range r = sin2q
time g
where ‘time’ is time taken to travel between the two It has same value for θ, 90 – θ
points.
→ → → → → 1 → 2 2v sin θ
v = u + a .t; s = u .t + at Sol 5: Time of flight, t =
2 g
→ →
v2 – u2 = 2 a s v 2 sin2 θ
more height h = 2g

Sol 2: We know that, a = ax î + ay ˆj v2


range = sin 2θ
→ g

a = dv
Sol 6: vx = v0 cosθ
dt
→ vy = v0 sinθ – gt where v0 is initial velocity
Let v = vx(t) î + vy(t) ˆj v 0 sin θ – gt
Angle of elevation = tan–1
→ v 0 cos θ

dv dv x (t) dv y (t) ˆ
a = = î + j
dt dt dt gx2
Sol 7: y = x tan θ –
dv y (t) 2v 2 cos2 θ
dv x (t)
⇒ ax= and ay = 2v sin θ
dt dt Time of flight t =
g
→ → 
x = ∫ v.dt + x0
v2
Horizontal range r = sin2θ
→ → g
⇒ x = x0 + ∫ v x (t) dt ˆi + ∫ v y (t) dt ˆj v 2 sin2 θ
More height h =
2g
Sol 3: When a body is projected at an angle in a unit
force field, then the body is called projectile Sol 8: In case of uniform circular motion, the particles
E.g.: Cannon ball shot from a cannon, bullet from a gun, will have acceleration towards the centre only and is
droplets of water coming out from a piston, ball leaving called centripetal acceleration.
a bat etc. v2
ar = = ω 2R
R
P hysi cs | 3.41

Sol 13: Let muzzle velocity be V


2h 2(1.96)
Sol 9: Time of flight t = = = 2 0.1 s Velocity along vertical direction for 60°
g 9.8
50 3
Velocity of bus = 60 kmh–1 = ms–1 v1y = v sin 60° = V
3 2
Velocity along vertical direction for 30°
50
Distance = velocity × time = ( 2 0.1 ) = 10.54 m
3 V
v2y = v sin 30° =
∴ Distance travelled is 10.54 m 2
v 2y
Max height =
2h 2.490 2g
Sol 10: Time of flight = = = 10 s
g 9.8 2
v12 
2
3 
 v1   
Vertical velocity = gt = 10 × 9.8 = 98 ms–1 2g 2
Ratio of heights = =   =   =3
v 22  v2   1 
Total velocity  
2g  2 
= (horizontal velocity)2 + (vertical velocity)2
Hence if shoots thrice as high
= (98)2 + (15)2 = 99.1 ms
–1
v
Time of height t = 2  
g
Sol 11: Vertical velocity vy = gt = 9.8×3 = 29.4 ms–1 3 v 3v
t1 = 2 . =
2 g g
1 2 1
Vertical displacement (y) = gt = × 9.8 × 32
2 2 1 v v
t2 = 2. =
= 44.1 m 2 g g
Horizontal displacements (x) = 50 × 3 = 150 m Their horizontal velocity vx = v cos q
v
v1x = vcos60° =
Final velocity = v 2x + v 2y 2
3
= v2x = v cos30° = v
(50)2 + (29.4)2 = 58 ms
–1
2
horizontal distance = vxt
Total displacement = x2 + y 2 = (150)2 + (44.1)2
v 3v 3 v2
= 156.35 m d1 = v1x . t1 = . =
2 g 2 g
44.1
Angle of depression = tan–1 = 16.38°
150 3 v 3 v2
Its final velocity is 58 ms–1 d2 = v2x . t2 = v. =
2 g 2 g
It is at a distance of 156.35 m at an angle of depression Hence their horizontal range is same.
of 16.38°

Sol 14: Maximum horizontal range occurs when it is


Sol 12: Vertical velocity = v sinq = 9.8 (sin30°)= 4.9 ms–1 fixed at angle of 45°.
1 2 Let its initial velocity be v
h = ut + gt
2
v
1 Vertical velocity vy = v sin 45° =
⇒ 29.4 = 4.9 t + × 9.8t2 ; ⇒ 29.4 = 4.9t + 4.9t2 2
2
 vy  2.v 2v
⇒ 6 = t + t ; ⇒ t2 + t – 6 = 0
2
Time of flight t = 2   = =
 g  2g g
⇒ (t – 2)(t + 3) = 0  
Since, t > 0 v 2y v  1
2
v2
Maximum height (h) = =   . =
⇒ t=2 2g  2  2g 4g
It hits the ground in 2 seconds.
3 . 4 2 | Motion in a Plane

v 2
Horizontal velocity vx = v cos 45° = 1  v2  1 2
2 = 16  sin2θ1  = R
 g  16
 
v 2v v2
Horizontal range (r) = vx.t = . = ∴ R2 = 16H1H2 ⇒ R = 4 H1H2
2 g g
2
v
r g Sol 17: Given that, v = î + 2ˆj
= 2 =4
h v Horizontal displacement x = vx . t ; x=1t
4g
x = t ... (i)
∴ Its maximum horizontal range is 4 times height.
1 2
Vertical displacement y = vyt – gt = 2t – 5t2
2
Sol 15: Horizontal range is given by
⇒ y = 2x – 5x2
v 2 sin2θ
D= where θ is angle of projection v 2 sin2θ
g Sol 18: r = ;
g
sin2(60o )
D1 = (30)2. = 90 sin 120o = 45 3 m v2 80 × 80
10 Max range = = = 640 m
g 10
sin2(30)
D2 = (20)2. = 40 sin 60° = 20 3 m 80 × 80
10 100 3 = . sin2q
10
Distance = D1 – D2 = 45 3 – 20 3 = 25 3 m
sin 2θ = 0.27

Sol 16: For a given angle of projection θ 1 –1


⇒ θ= sin 0.27 = 7.42°
2
v sin2θ 2
Horizontal Range r =
g 2v sinθ 2 × 80
t= = sinθ = 16 sin 7.42° = 2.19 s
g 10
v 2 sin2 θ
Maximum Height h =
2g
given r1 = r2 v 2 sin2 θ
Sol 19: h =
2g
v 2 sin2θ1 v 2 sin2θ2
⇒ = v 2 sin2 θ
g g ⇒5=  ... (i)
2g
⇒ sin2 q1= sin2 q2
⇒ 2q1 = 2q2 or 2q1 = 180 – 2q2 v 2 sin2θ 2v 2 sin θ cos θ
r= =
g g
 q1 ≠ q2
⇒ q1 + q2 = 90°  ... (i) 2v 2 sin θ cos θ
⇒ 200 =  ... (ii)
g
v 2 sin2 θ1
⇒ h1 = Divide (i) by (ii)
2g
v 2 sin2 θ
v 2 sin2 θ2 v 2 cos2 θ1
H2 = = (q1 + q2 = 90°) 5 2g
2g 2g =
200 2v 2 sin θ cos θ
v 2 sin2 θ1 v 2 cos2 θ1 g
H1H2 = .
2g 2g 1 1 1
= tanq; θ = tan–1
4 40 4 10
v
= . sin2q1 cos2q1
4g2 u2 sin2θ 2u2 sin θ cos θ
⇒r= =  ... (iii)
4
g g
v
= . sin22q1
16g2 u2 sin2 θ
⇒H=
2g
P hysi cs | 3.43

r tan θ r Sol 23: Velocity w.r.t ground


= Given = 2 ⇒ tanθ = 2
H 4 H = Velocity w.r.t Jet + velocity of jet
2
4 4u = 1500 km h–1 – 500 km h–1
⇒ sin2θ = ⇒r=
3 5g = 1000 km h-1

gx2
Sol 20: y = x tanθ – Sol 24: v = 126 km h–1 = 35 ms–1
2v 2 cos2 θ
g(50)2 v2 352
13 = 50 tanθ – a= = = 3.0625 ms–1
2 × (10 g)2 cos2 θ 2s 2 × 200
v v 2s 2 × 200
⇒ 26 = 100 tanθ – 25 sec2q ⇒t= = 2 = = = 11.43 s
a v v 35
⇒ 25 tan2θ – 100 tan θ + 51 = 0 (sec2θ = 1 + tan2θ) 2s
2s
100 ± (100)2 – 4 × 25 × 51 Note: - here t = can also be written as
⇒ tan θ = v
2 × 25
s v
100 ± 70 3 17 t= ; is average velocity of motion.
tanθ = = , v 2
50 5 5  
2
3
For tmin , tan θ is minimum ∴ θ = tan–1 ~
– 30.1° So in a uniform accelerated motion,
5
Distance
t=
Sol 21: Given, R=2H Avg velocity

u2 sin2θ u2 sin2 θ Try deriving the same assuming it has some final velocity.
⇒ 2
=
g 2g
⇒ 2sin θ cos θ= sin θ sin θ Sol 25: Distance to be travelled by B relative to A
⇒ tan θ =2 = 2(400) + x
2 1 x = initial separation (distance between them)
sin θ
= , cos θ
=
3 3 Relative velocity = VB – VA = 72 – 72 = 0
2 2 2 2 Relative acceleration = 1 ms–2
So, R= u sin2θ= u 2sin θ cos θ= u 2 × 2= 4u
g g g 3 3g Time = 50
1 2
∵ at = S
Sol 22: He moves 5 steps forward and 3 steps backward 2
in 8 seconds.
1
⇒ He moves 2 m in 8 second ∴ 800 + x = × 1 × (50)2
2
Lets call it drunk movement (D.M) x = 450 m
i.e. in 1 D. M = 2 m in 8 seconds Initial separation was 450 m.
∴ Distance travelled in n D.M = 2n m in 8n seconds.
Now 13m – 5m = 8 m Sol 26: Distance between two busses coming from
same direction is VT; V is speed of bus.
i.e. man will have complete D.M such that
Relative velocity of with buses coming from opposite
2n ≥ 8 for least possible n ⇒ n = 4 direction = V + VC; VC is speed of man.
⇒ he travelled 8 m in 32 seconds Relative velocity of man with buses coming from same
he falls in pit in next 5 m, 5 seconds direction as man = V – VC

∴ He falls in 32 + 5 = 37 seconds. VT VC T
18 = ⇒ 1– =  ... (i)
V – VC V 18
3 . 4 4 | Motion in a Plane

Vt VC T
6= ⇒ 1+ =  ... (ii)
V + VC V 6 Sol 28: (i) 0 to 30
equation (i) + equation (ii) 1 1
distance = V × t = 5 × (t = 30 min = Hr) = 2.5 km
2 2
1 1  distance 2.5
⇒2=T  +  speed = = = 5 km/h
 6 18  time 1/2
2 displacement 2.5
T= = 9 min velocity = = = 5 km/hr
1 1 time 1/2
+ (ii) 0 to 50
6 18
0 to 50 = 0 to 30 + 30 to 50
⇒ V = 40 km/h–1
In 30 to 50
VC = 20 km/h = velocity of cyclist 2
Distance = V × t = 7.5 × = 2.5 km
6
2
2h 2 × 90 (20 min = Hr)
Sol 27: t = = = 3 2s 6
g 10 Displacement (s) = ∑ Vt = 2.5 – 2.5 = 0 km
t = time of descent
Total distance (s) = ∑ | V | t = 2.5 + 2.5 = 5 km
2V1
t1 = s
g Velocity = =0
t
t1 = time of flight between consecutive collisions with
D 5 5
the floor Speed = = = 6 km/hr (50 min = Hr)
t 5/6 6
V1 = (0.9)V (i.e. velocity after collison)
(iii) 0 to 40
V
⇒ t1 = (1.8) ; t1 = (1.8) t 0 to 40 = 0 to 30 + 30 to 40
g
t1 + t2 = t + 1.8t = 2.8t = (2.8) 3 2 ~
– 12 s 1
= 1.25 km
In 30 to 40, distance = v × t = 7.5 ×
6
V12 V2 Displacement = 2.5 – 1.25 = 1.25 km
H1 = = (0.9)2. = (0.81)90 = 72.9 m
2g 20
Total distance = 2.5 + 1.25 = 3.75 km
H 1.25
90 m Velocity = = 1.875 km/h
4/6
72.9 m
3.75
Speed = = 5.625 km/h
4/6

V Sol 29: Vy = V cosθ (θ is angle with vertical)


O 3 2 (1.9) 3 2 12
3 1 2
Vy = V; S = Vy t + gt
Note: - Here we have a sharp change at 3 2 because 5 2
ball got rebound from surface there.
3 1
There is a sharp change at (1.9)× 3 2 because there is 2400 = V(5) + × 32.5 × (5)2 (g= 32.5 ft. /m2)
5 2
change in direction of motion and we took both sides
positive (speed) V = 664.6 ft. s–1

x = Vx. t = V
V
B 4
0.9V sinθ t = .V.5 = 2658.4 ft
5
Vx = V sinθ = 534 ft/s
V
O 3 2 (1.9) 3 2 12 Vy = V cosθ + gt = 560 ft/s
P hysi cs | 3.45

Circular Motion 1 π π
∴a= . m/s2 ;
a= m/s2
Sol 30: Minutes hand of a clock completes one 2 30 60
revolution in one hour i.e. 3600 second

1 Re v Sol 34: Time period T =
So, ω = and 1 revolution = 2π Rad ω
3600 s Given T = 2 s
2π π
ω= rad/s ⇒ ω = × 10–2 rad/s 2π
3600 18 ∴ω= = π rad/s
T
and acceleration a = Rω2 m/s2
Sol 31: A wheel making 300 rotation per minute R = 100 cm = 1 m
and one rotation = 2π rad. ∴ a = p2 m/s2
1 minute = 60 sec
300.2π Sol 35: (i) Given that insect completes 7 revolutions in
∴ω= rad/s; ω = 10π rad/s 100 seconds.
60
7.2π
Sol 32: 4 rotations per second ∴ ω = 7 Rev/100s = rad/s
100
14 π
rotations ω= rad/s
⇒ω=4 and 1 rotation = 2π rad 100
s
ω = 0.44 rad/s and v = Rω
⇒ ω = 4.(2π) rad/s; ω = 8π rad/s
R = 12 cm = 0.12 m
and the velocity of a point on its circumference
v = (0.12) (0.44) m/s2
v = Rω
1 v = 5.3 × 10–2 m/s2
R = 50 cm = m.
2 (ii) Acceleration is not constant. Because the direction of
1 the acceleration vector keeps on changing in direction.
v =   (8π) m/s Hence acceleration vector in circular motion can never
2
be a constant vector.
v = 4π m/s

| a | = Rω2 = (0.12) (0.44)2 m/s2
revolutions 2π
Sol 33: ωinitial = 100 = 100 rad/s →
minute 60
| a | = 2.3 × 10–2 m/s2
10π
ωi = rad/s
3
revolutions 2π Sol 36: Earth completes 1 rotation in 1 day
ωf = 400 = 400. rad/s
minute 60
rotation w
i.e., ω = 1.
40π day
ωf = rad/s
3

ω = 1. rad/s
∆ω ω – ωi 24 × 60 × 60
α= = f
∆t t A
π
40π 10π ω= × 10–2 rad/s
– 432
3 3 30π
α= rad/s2 = and now acceleration at point A;
5 × 60 3 × 5 × 60
a = rω2
π
α= rad/s2 r = 6400 km = 6400 × 103 m
30
and linear acceleration a = Ra r = 64 × 105 m
π2
1 ∴ a = 64 × 105 × × 10–4 m/s2
R = 50 cm = m (432)2
2 A = 0.03 m/s2
3 . 4 6 | Motion in a Plane

5   →
Sol 37: v = 27 km/h = 27 × m/s →
v 2  − 1 (iˆ +ˆj)  v2 ˆ ˆ
(i + j)
18 ⇒ Now a A = aA = –
r  2  r 2
15
v= m/s →
2 Put v = 2 cm/s and r = 4 cm to find a A

v2 (15)2
ar = = = 0.7
R 4 × 80 Sol 39: Now this tests your understanding of

1 displacement vector
Q
a t = 0.5 m/s2 = m/s2
2 → → →
→ → → Displacement vector r = rf – ri
a net = ar + a t = 2
(0.7) + (0.5) 2 →
where rf is the co-ordinates of final A B C
→ →
a net = 0.86 m/s2 position and ri is the co-ordinates of
initial position.
P
Sol 38: At point the acceleration y Now for all the three girls, final
will be centripetal acceleration position is point Q and initial destination is point P. Hence
which is radially directed towards displacement is same for all the three girls,
Q
point O. i.e. A → → → →
→ i.e. 2 r = 2( rQ – rP ) = 2 PQ = 2×200=400 m
2
v ê π/4
Physically: a = (– r ) Distance is the total length of the path travelled.
r O P x
Here for girl B; distance travelled is same as her
Remember êr and êt are the
displacement vector, since she travelled in the straight
unit vectors along radial and line connecting the points.
tangential direction respectively.
Refer to the figure.
Sol 40: Now from the argument made Q
So in this case also, êt →
→ →

2
above, displacement r = rf – ri
→ v êr
a A = r (– êr ) Here the cyclist started from the point P
A O
O and then finally reached the point O
Now, since the point is in O →

between the points P and Q, Hence r f = r i
y →
So r = zero
Hence net displacement is zero.
x Total displacement 0
π/4 And average velocity = = =0
Total time 10

total dis tance


– êr and for average speed =
total time
  π Total distance is |OP| + |PQ| + |QO|
Angle between OA and OP will be
4 |OP| = |QO| = R = 1 m
Now let us resolve (– êr ) into î and ˆj  2πR  π
And |PQ| =  4  =  2  m
π π    
(– êr ) = |– êr | cos (– î ) + |– êr | sin (– ˆj )
4 4  π
∴ Distance =  2 + 2  m.
But since êr and êt are unit vectors;  
| êr | = | êt | = 1  π
2 + 
 2
1 1 ˆ 1 Av. Speed = m/s = 6× 10–3 m/s
 (– êr ) = – î – j =– ( î + ˆj ) 10 × 60
2 2 2
P hysi cs | 3.47

Sol 41: Let us say the circular turn is B Let us say at point B, the A
of the shape AB. particle loses its contact. So N
h B
let us write the equations of
Now at the starting point of the track R cosθ
motions. At point B say the θ θ
→ → → R
i.e. C; a = ar + at particle has velocity v.
O
→ O V
→ a → 2
ar = centripetal acceleration ar mg cosθ = N +
mv

at R mg
v 2 êr C
= (– )
R mv 2
N = mg cosθ –  ... (i)
5 A R
v = 36 km/h = 36 × m/s = 10 m/s
18 Now when the particle is about to lose contact, the
R = 140 m normal reaction between the particle and the surface

(10)2 5 becomes zero.
ar = = m/s2 (– êr )
140 7 ∴ N=0
dv mv 2
and given that = 1 m/s ⇒ mg cosθ =  ... (ii)
dt R
→ dv → Now energy at point A, taking O as reference
∴ at = ( êt ); a t = 1 m/s2 ( êt )
dt 1
EA = 0 + mg R and EB = mv2 + mg R cosq
→ → → 2
Now a = a r + a t Using Energy conservation EA = EB

a = (0.7 (– êr )) + 1 êt ) m/s2 1
⇒ mg R = mv2 + mg R cosq
2
|a| = (0.7)2 + 1 = 0.49 + 1 = 1.49 m/s2 = 1.22 m/s2 ⇒ 2mg R (1 – cosθ) = mv2
 1   10  mv 2
and tan β =   ⇒ β =tan–1   2mg (1 – cosθ) =  … (iii)
 0.7   7  R

Sol 42: mv 2
Putting this value of in eqn (ii)
R
O ⇒ mg cosθ = 2mg (1 – cosθ) ⇒ 3 cos θ = 2
50° V
2 2
O V 40° 90° ⇒ cosθ = ⇒ θ = cos–1  
3 3
40° 40°  2
B and now h = R (1 – cosθ) = R  1 – 
V  3
A R
∴ h= .
→ 3
Velocity at point A, V A = v î

Velocity at point B, VB = v sin 50 î +v cos 50 ˆj Exercise 2

VB = v (0.76 î + 0.64 ˆj ) Projectile Motion
→ →
Now change in velocity DV = VB – V A Single Correct Choice Type

= v (0.76 î + 0.64 ˆj ) – v î Sol 1: (D) Lets solve the


problem taking along
DV = v (–0.24 î + 0.64 ˆj )
plane as x-axis and y x
perpendicular to plane
as y-axis (45°θ)
Sol 43: This is a very standard problem for a JEE
aspirant.
θ 45°
3 . 4 8 | Motion in a Plane

y x V = Vx if Vy(t) = 0
g sinθ
Vy2 42
Vy(t) = 0 ⇒ H = = = 0.8 m
g cosθ 45°θ 2g 2 × 10
But given height is 0.45 m
Vx = V cos (θ – 45o); Vy = V sin (θ – 45o)
∴ |V| > |Vx|
g
gy = g cosφ = ; φ = 45° The given data is sufficient to calculate Vy(t)
2
1 2
g 0.45 = 4(t) – .gt
gx = g sinφ = 2
2 From this we can get t
Vy And Vy(t) = 4 – g(t)
time of flight = 2
gy So data is sufficient.
2V cos(θ – 45) 2 2V sin(45 – θ)
= =
g g 2H
Sol 5: (C) t =
2 g
It hits perpendicularly to plane 1
Horizontal displacement (x) = a t2
⇒ Vx = 0 2 x
ax = horizontal acceleration
⇒ 0 = V cos(θ – 45) – gx(t)
2
1  2H 
2 2V sin(θ – 45)
g  
⇒ V cos(θ–45)= . x = ax  g 
2 g 2  
1 tan θ – 1 1 ax
⇒ tan(θ – 45) = ⇒ = 6
2 tan θ + 1 2 x = H. g = 20 × = 12 m
10
⇒ tanθ = 3
⇒ θ = tan–13
Sol 6: (C)
10 ms–1
Let’s solve the problem along the 45°
Sol 2: (D) Both the bodies have same horizontal range,
plane of inclined surface.
since they form complementary angles with horizontal
axis. y
It θ = 45° they have same time of flight else differs. 10 ms–1 45°
gy
45° gx
Sol 3: (C) u = u cosθ î + u sinθ ˆj x
Let final velocity be v g
gx = gsin45° =
v = v cos α î + v sin α ˆj 2
u cos θ = v cos a g
gy = gcos 45° =
{ Horizontal components of velocity are same} 2

ucos θ V = V cos θ î + V sin θ ˆj


⇒v= = u cosθ seca
cos α 10 10 ˆj
V= î +
3 4 2 2
Sol 4: (C) V = Vcos θ î + Vsin θ ˆj = 5   î + 5  5  ˆj
5  
2.
10
V = 3 î + 4 ˆ
j 2Vy 2
Time of flight = =
|V (t)| ≥ Vx gy g
2
⇒V≥3
Time of flight = 2 sec
P hysi cs | 3.49

Sol 7: (C) Let’s solve along plane of incline. Sol 9: (B) B C


V
V y
53° 127°
gy gx θ
x
2V
Time of flight t = A
gy

1 4
Horizontal range along inclined = g t2 Vy = Vsin53° = 5. = 4 km/hr
2 x 5
1 4.V 2 2V 2 .gx Vx = V cos 127° = – 3 km/hr
= gx . =
2 g2y g2y Resultant velocity along x-direction

gx = g sinq; gy = g cosq = VR + Vx = 4 – 3 = 1 km /hr


width 200
2V 2 .gsin θ 2 Time of swim = = = 180 seconds
r= ; r = 2V tan θ sec θ Vy 5
g2 cos2 θ g 4.
18
1 2  1×5 
Sol 8: (A) Height of monkey above ground = H – gt Drift = VResult × t =   × (180) = 50 m
2  18 
1 2
Height of arrow above ground = Vyt – gt Drift 50
2 Time of walk = = = 60 second
V 5
1 3×
 At point of contact both are at same height H – 18
2
1 Total time = 180 + 60 = 240 s = 4 minutes
gt2 = Vyt – gt2 ⇒ H = Vyt
2
H Sol 10: (B)
⇒t=
Vy
y
H 1 km
Now, here L = Vx . t ; ⇒ L = Vx .
Vy
x
L 
⇒ Vx = Vy  
H 1
Time = 15 min = hr
4
L2 Vy
V = Vy 1 + (V = Vx2 – Vy2 ) = H2 + L2 width 1
H2 H Vy = = = 4 km/hr
time 1
Let vertical velocity at the time of impact be Vf 4
Vf = Vy – gt For shortest path, resultant speed along x-axis = 0
Minimum value of Vt = – Vy ⇒ VR – Vx = 0 ⇒ VR = Vx
∴ – Vy = Vy – gt
Vx2 + Vy2
V=
H
2Vy = gt; t=
Vy ⇒ Vx = 3km/Hr ⇒ Vy = 3km/Hr
g.H
⇒ 2Vy =
Vy
Sol 11: (C)
gH B
⇒ Vy = y
2
V 30°
1 gH 2 2 g(H2 + L2 ) θ
⇒V= . H +L =
H 2 2H
A x
3 . 5 0 | Motion in a Plane

V = V sinθ î + V cos θ ˆj V1 3
= + V
ˆ 2 2 2
VResult = V – Vr = (V sin θ – Vr) î + Vcos θ j
They collide in 2 second
V sin θ – Vr
tan30o = V 3 
V cos θ ⇒  1+ V  (2) = 30
 2 2 2 

1 5 3 sin θ – 5
=
3 5 3 cos θ V1 + 3 V = 30
2

⇒ θ = 60° V2 = 3V
1
⇒ He should steer at 30° w.r.t the line of destination
from starting point ⇒ V1 + 3 ( 3 V ) = 30
1

⇒ V1 = 7.5 ms–1
Sol 12: (C) Car moving north ⇒ wind force acting south. V2 = 7.5 ms–1
Also normal winds are acting due east so flag will point
south-east. V1 is velocity of B
V2 is velocity of A
Sol 13: (C)  y
B C A
45° Sol 16: (A)
45°
x Length of string 1: L1
= x1 + 2x2 + x3
V V ˆ x2
VB w.r.t C = – î + j Length of string 2: L2 = x4 x3
2 2 x1
x1 = x2 + x4
V V ˆ
VA w.r.t B = î + j Differentiate on both side B
2 2 x4
dx1 = dx2 + dx4
V V ˆ V V ˆ
VA w.r.t C = VAB + VBC = î + j – î + j dx4 = 0 ∴ length of string is
2 2 2 2 constant
ˆ ⇒ dx1 = dx2 ... (i)
VAC = 2Vj
VCA = – VAC = – ˆ L1 = x1 + 2x2 + x3
2Vj
Differentiating dL1 = dx1 + 2dx2 + dx3
dl1 = 0 as length of string is constant.
Sol 14: (B) Vrain = Vx î + Vy î
dx1 + 2dx2 + dx3 = 0
Vrain = 2ms–1
⇒ 3dx1 + dx3 = 0 (dx1 = dx2)
Vrc = Vrain – Vcyclist
dx3 3dx1
Vrc = Vy ˆj ⇒ dx3 = – 3dx1 ⇒ =–
dt dt
⇒ Vcyclist = Vx î VB = – 3 VA = – 3(5) = – 15 ms –1

Sol 15: (C) Vx1 Sol 17: (A)


Vx1 = Vx2 Length of string L1= x1+2x3+x2
60°
⇒ V1 sin60° = V2 sin30° xB = x3 + x4 x1 x3 x2

3 V ... (i) dxB = dx3 + dx4


V2 = 1
dx4 = 0 (length of string
Relative velocity linearly 30°
constant) x4
= V1 cos 60 + V2 cos30 ⇒ dxB = dx3
Vx2 B
P hysi cs | 3.51

x1 = xA, x2 = xC
ω1
L1 = xA + 2x3 + xC ⇒ =1
ω2
dL1 = dxA + 2dx3 + dxC
1
dL1 = 0 Sol 20: (C) r = 25 cm = m.
4
dxA + 2dx3 + dxC = 0 And given ω = 2 rev/s
dxA + 2dxB + dxC = 0 But 1 rev = 2π rad
1  ω = 2(2π) rad/s ∴ ω = 4π rad/s
dxB = – (dxA + dxC)
2 1
Now acceleration = rω2 = (4 π)2 m / s2
2
d xB d x
2
d xC2  4
1 A a = 4p2 m/s2
⇒ =–  + 
dt 2
2  dt 2
dt2 
 
lets take upwards as positive Sol 21: (C) P ω
Now acceleration of P is rP.ω 2 rP
1 1
⇒ aB = – (a – f) ∴ aB = (f – a) towards centre of disc and
2 2 acceleration of Q is rQ.ω2
rQ
Given rp > rQ
Sol 18: (A) Length of string L = 2x1 + 2x2 + 2x3
∴ aP > aQ
x1 x2

→ →
Sol 22: (D) Velocity V = r × ω
x3 →
→ →
Acceleration = ( r × ω ) × ω
Now In uniform Circular motion, ω is constant and of
d2L  d2 x 2d2 x2 2d2 x3  course r is constant. Hence magnitude of both velocity
 1 
= 2 2 + +
dt2 dt2 dt2  and acceleration are constant. But the directions keep
 dt varying.
0 = aA + aB + aC Hence both velocity and acceleration change.
aA = 2 and aB = – 1
Sol 23: (C) Equal angles in equal time implies ω is
(B is moving away from central line) constant. Now follow the above argument
aC = – (2 – 1 ) = – 1
∴ aC = 1 ms–2 upwards (A) Sol 24: (C) θ = 2t3 + 0.5
Note: - try understanding the sign convention used here. dθ 2
Positive was towards a reference point and negative was ω= = 6t2 = 6t0
dt t to
away. o

Now here to = 2 s
Circular Motion ω = 6(2)2 = 24 rad/s

Single Correct Choice Type


Sol 25: (D)
Sol 19: (C) This is just a Kinematic problem. So nothing
A seconds hand completes 12 w→
to do with the masses of the bodies. v 1 = rwiˆ
one revolution in A
And now given that both complete a circle in time ’t’. 60 seconds w
9 3 y
B
∴ Both of them have same time period. i.e. 2π rad in 60 seconds
x
T1 = T2 = t 6

2π 2π ∴ω= →
60 v 2 = rw(– ˆj) w
and we know T1 = and T2 =
ω1 ω2
π
= rad/sec
30
3 . 5 2 | Motion in a Plane

Speed of the end point = rω Sol 29: (D)


v at
π π For uniformly accelerated motion;
= 6. cm/s = cm/s velocity will be in tangential ar
30 5
direction. And acceleration
=2π mm/s = 6.28 mm/s will have both the radial and
tangential components.
Now consider the end point at point A; velocity of the
end point would be →

v2 →
dv

ar = and a t =
R dt
v A = rω î and now when the end point is at point B;

velocity of the end point is vB = rω(– ˆj ) Sol 30: (C) F ∝ r
→ → ⇒ F = kr (k is a constant)
Now v A – v B = rω î – rω(– ˆj )
But we also know that for a particle in circular orbit;
→ →
v A – vB = rω( î + ˆj ) mv 2 mv 2
F= ∴ = kr
→ →
r r
| v A – v B | = rω( 2 ) = 2 rw
k
v= r ⇒ v ∝r
= 2 (6.28) mm/s = 8.88 mm/s m

Sol 26: (B) Initially the fan makes 600 revolutions per
minute Previous Years’ Questions
 2π 
∴ ω = 600 rev/min = 600  rad / sec  Sol 1: (A) To cross the river in shortest time one has to
 60  swim perpendicular to the river current.
∴ 1 rev = 2π rad
1 min = 60 sec Sol 2: (B) Shortest possible path comes when absolute
2π velocity of boatman comes perpendicular to river
ωi = 600 rad/sec current as shown in figure.
60
ωi = 20π rad/sec Vr

and finally the fan makes 1200 revolutions per minute



∴ ωf = 1200 rad/sec ; ωf = 40π rad/sec River
60 Vb
Vbr Courrent
Increase in angular velocity = Dω = ωf – ωi
= (40π – 20π) rad/s = 20π rad/s

ω ω 1 1
t= = ; =
Sol 27: (A) Let ‘R’ be the radius of the wheel. In one vb 2
v br – v r2 4 25 – vr2
revolution, the wheel completes a distance of 2pR.
And for 2000 revolutions, it is 2000× 2pR. Solving this equation we get vr = 3 km/h
But given the distance is 9.5 km
 2000 × 2pR = 9.5 × 103 m Sol 3: (B)

∴ R = 0.75 m θ2 > θ1 ∴ ω2 > ω1


Diameter d = 2R = 1.5 m Statement-II, is formula of relative
velocity. But it does not explain
statement-I correctly. The correct
dω explanation of statement-I is due
Sol 28: (C) Angular acceleration α =
dt to visual perception of motion.
Since ω is a constant The object appears to be moving
faster, when its angular velocity is
α = zero
greater w.r.t. observer.
P hysi cs | 3.53

u2  a 
Sol 5: (D) maximum vertical height = = 10m V= (1 + α2 )  
2g
 2β 
u2 sin2θ
Horizontal range of a projectile = Sol 2: Let Vx be velocity along Vy
2g
Range is maximum when θ = 45 0 x-axis. Let Vy(t) be velocity
along y-axis at time t.
u2
Maximum horizontal range = at t = 2, θ = 30° θ=30°
g
Vy (2)
Hence maximum horizontal distance = 20 m. ⇒ = tan30° g
Vx θ0 B
Sol 6: (C) a ∝ r Vx
1
⇒ Vy(2) = Vx
Sol 7: (A) x = t; y = 2t − 5t2 3

Equation of trajectory is y = 2x − 5x2 At t = 3, θ = 0° (given moving horizontal)


⇒ Vy (3) = 0
V – u = at
JEE Advanced/Boards
⇒ Vy (3) – Vy (2) = – g(3 – 2)

Exercise 1 ⇒0–
Vx
= – g ; ⇒ Vx = 3g
3
Projectile Motion Initial velocity of projectile

Sol 1: Let Vx Vy be velocities along x, y axis respectively. V= Vx2 + (Vy (0))2


dVy Vy(2) – Vy(0) = – g (2 – 0)
= – a (given); y = ax – bx2
dt Vx
Vy (0) = Vy(2) + 2g = + 2g = g + 2g
dy d dx dx 3
Vy = = (ax – βx2) = α – 2bx Vy(0) = 3g
dt dt dt dt
Vy = aVx – 2bxVx  ... (i) ⇒V= Vx2 + (3g)2 = ( 3g)2 + (3g)2 = g 12 = 10 12
at origin, (x, y) = (0, 0) V = 20 3 ms–1
⇒ Vy(0, 0) = aVx – 2β(0)Vx Vy (0) 3g
tanq0 = = = 3 ⇒ θ = 60°
Vy = aVx Vx 3g
V2 = Vx2 + Vy2 = Vx2 + a2 Vx2
Sol 3: V = V cosθ î + V sinθ ˆj
⇒V= 1 + α2 Vx  ... (ii) θ = 37°, V = 700 cms–1 = 7 ms–1

Coming back to equation (i) ∴ V = 7 cos37° î + 7 sin 37° ˆj = 7 (0.8) î + 7(0.6) ˆj


dVy d V = 5.6 î + 4.2 ˆj
ay = = (aVx – 2bxVx)
dt dt Distance between the balls along the line of projection
d  dx   dVx  d = 500 cm
– a = α (Vx) – 2bVx   –2bx  
dt  dt   dt  Distance between the balls along x-axis (dx)

d = d cosθ = 500 cos 37° = 500(0.8) = 400 cm = 4m


V = 0  it is having acceleration only in x-direction
dt x When the two balls hit, their x-coordinates are same
 dx  dx
∴ – a = – 2bVx   ∴ – a = – 2β(Vx)2 4
 dt  ⇒t= = s
Vx 5.6
a a
⇒ Vx = ⇒V= 1 + α2
2β 2β
3 . 5 4 | Motion in a Plane

Distance through which ball B falls is 4H 16H2


⇒ sinθ = ; sin2θ =
 4 
2 (4H)2 + R 2 16H2 + R 2
1 1
= gt2 = x 10 ×   = 2.55 m
2 2  5.6  2gH
V2 = (from (i))
sin2 θ
Vy of ball at O = 4.2 ms–1 y
2 × 10 × H 5(16H2 + R 2 )
Vy at time of collision B = =
g 16H 2 4H

cm
V – u = at

0
10H2 + R 2

50
 4 
Vy – 4.2 = – 10   37°
x 5(16H2 + R 2 )
 5.6 
O
V=
103 4H
Vy = –
35
103 Sol 6: V0 = Vx î , V(t) = Vx î + Vy ˆj , Vy = gt
Vy –
35
Angle of inclination = tan–1 = tan–1 = –27.72° 3 3
Vx 5.6 |V(t)| = V = V
2 0 2 x
Ball is directed at an angle 27.72° below x-axis. 2
3 3 
Vx 2 + Vy2 = 2
V ; ⇒ Vx + (gt)2 =  Vx 
2 x 2 
Sol 4: (i)  there is no friction and motion is taking
plane in a horizontal plane, 5 2 2gt
⇒ (gt)2 = Vx ; ⇒ Vx =
4 5
Hence acceleration = 0 in all frames of reference (except
some random accelerating frame of reference which we 1
will not be using in this problem) 2 × 10 ×
2 = 4.4 m/s
=
Vball = Vcos φ î + V sinφ ˆj 5

Vball – trolley (VbT) = Vball – Vtrolley = Vcos φ î + (V sinφ – Vtrolley) ˆj V22 sin2θ
Sol 7: OA = (V2 = 40 ms–1)
Hence motion of the ball is a straight line as observed g
by trolley.
40 × 40 sin120
In trolley’s frame of reference, O moves downward let = = 80 3 m
10
initial position of O be Oo. OoA makes 45° with x-axis.
V 3 ˆj
And the ball follows the path O0A. Hence velocity vector V = V cosθ î + V sinθ ˆj = î + V
of the ball makes 45° with the x-axis in this frame 2 2
θ = 45° OA = Vx t

4θ 4(45) V
(ii) φ = = = 60° ⇒ . t = 80 3
3 3 2
V sin φ – Vtrolley = tan θ = 1 Vt = 160 3
V cos φ 1 2 3 1
V sin60 – ( 3 – 1) y = Vyt – gt = Vt – gt2
∴ =1 2 2 2
V cos60
V 3 V = 160 3
= V – ( 3 –1) ⇒ V = 2 ms –1
t
2 2
y = a 3 t (as they meet at same point)
2 2
V sin2θ 2V 3 1
Sol 5: R = = sinθ cosq a 3t= 160 3 t – (10)t2
g g 2 2
V 2 sin2 θ ⇒ 5t2 + a 3 t – 240 = 0
H=  ... (i)
2g
16 3
H tan θ 4H t>0⇒t=
= ⇒ tanθ = 5
R 4 R
P hysi cs | 3.55

160 3 160 3 Sol 10: (i)


V= = ∴ V = 50ms–1 V
t 10 3
5
V g cos30°
Sol 8: Dy = 1.8 – 0.6 = 1.2 φ
g cos30° 60°
1 2
Dy= – ut + gt 60° 30°
2 30°
1
⇒ 1.2 = – ut + (10)t2
2 Let solve in the planes frame of reference
1 φ = 60 – 30 = 30°
u = V sin30 = 18 × = 9 ms–1
2
⇒ 1.2 = – 9t + 5 t2 V = V cos30 î + V sin30 ˆj
Note: Try to understand the sign convention, here 3V V ˆj
downward is taken positive, V= î +
2 2
1 2 V
Here y = –ut + gt ) 2.
2 2Vy V
2
5t – 9t – 1.2 = 0
2
Time of flight = = =
gy gcos30° 3
g
t~
– 1.96 t=
2V 2
D = Vxt = 18 cos 30t = 30.55 m 3g
2
 
Sol 9: y = x tanθ –
gx2 1 3V  2 V  1  2V 
x = Vxt – gxt2 = g – gsin30  
2(V cos θ)2 2 2  3  2  3g 
θ = 45°  
2
V2 V
Let 4 ft above the ground be taken as plane of referxe = –
g 3g
⇒ y1 = – 4 ft
2V 2 2 (29.4)2
x1 = 350 ft x= = . = 58.8 m
3g 3 9.8
gx12
y1 = x1 tanθ –
2V 2 cos2 θ gx2
Sol 11: (i) y = xtanθ –
gx12 2V 2 cos2 θ
⇒ = x1 tanθ – y1
2V 2 cos2 θ Let’s take A as origin
g x1 tan θ – y1
⇒ = gx2
2
2V cos θ 2
x12 y = x tan (37) –
2V 2 cos2 37
y = = – 3300 m
We have x2 = 320 ft
x = 9400 m
gx22 2 4
⇒ y2 = x2tanθ – 3 g × (94) × 10
2V 2 cos2 θ – 3300 = 94 × 102× –
4 32
2V 2 . 2
 x tan θ – y  5
= x2 tanθ – x22  1 1 
 x 2  10 4 × (94)2 g  3 
 1  = 102  94 × + 33  = 103.5
18  4 
(320)2 V2 .
= 320 tan(45) – (320tan(45°)–(–4)) 25
(350)2
⇒ y2 = 24.08 ft (10)2 × (94)2 × 9.8 25
⇒V= × =340.9 ms–1
(103.5) 18
y2 > 24 ft
∴ It will clear the fence 9400
(ii) t = = 46 s
V cos37
3 . 5 6 | Motion in a Plane

Sol 12: Angle of projection D 58.8


Vy Time of flight t = = =4s
8 Vtruck 14.7
θ = tan–1 = tan–1 = 53°
Vx 6
t 4
u = g   = 9.8 × = 19.6 ms–1 vertically upwards
V= 62 + 82 = 10 ms
–1
2
  2

V2 2 × 10 × 10 3 4 (ii) V = V + V = 19.6 ˆj + 14.7 î


ball bT truck
range = sin2θ = × × = 9.6 m
g 10 5 5
|V| = (19.6)2 + (14.7)2 = 24.5 ms–1
Vy2 (8) 2
max height = = = 3.2 m Angle of projection
2g 2 × 10
Vy
2Vy 2×8 θ = tan–1 = tan–1 19.6 = 53°
Time of height = = = 1.6 s Vx 14.7
g 10
Sol 16: V1 = V0 ˆj
2h 2 × 1960
Sol 13: E = = = 20 s V2 = V0 cos α î + V0 sin α ˆj
g 9.8
V2 = V0 (cos α) î + V0(sinα – 1) ˆj
MN = Vx × t = 100 × 20 = 2000 m
a1 = – g ˆj ; a2 = – g ˆj ; a2 = 0

1
Sol 14: (i) Let initial velocity be V Separation x(t) = V21(t) – a21t2
2
V 2 sin2 α
H= = V0(cos α)t î + V0(sinα – 1)t ˆj
2g
Vy V sin α |x(t)| = t V0 (cos α )2 + (sin α – 1)2
t= =
g g
= V0 .t 2 – 2sinα = V0 .t + 2(1 – sin α )
Horizontal distance = H tanq
Note: - body just gazes ⇒ Vy = 0 at the top. Sol 17: θ1 θ2
H tanθ = Vx .t Let the vertical components of their velocities be Vy. Let
V sin α this angle of depression be q1, q2.
H tanθ = V cosα .
g Vy
tan q1 =
2
V sin α 2
V sin α cos α 2 3
. tan θ =
2g g Vy
tan q2 =
⇒ tan θ = 2cota 4
 They both are perpendicular, q1 + q2 = 90°
u2 sin2 α
(ii) H = (v = u)
2g 1
⇒ tanq2=tan(90 – q1)=cot q1=
tanθ1
2Hg ⇒ tan q1 . tan q2 = 1
u2 =
2
sin α Vy2
u = 2Hg cosec α = 2Hg(1 + cot α)
2 2 2 ⇒ =1
12
  tan θ   gh (4 + tan θ)
2 2
⇒ Vy = 12 ms
–1
= 2Hg  1 +   =
  2   2 Vy
 Vy = gt; t=
g
Sol 15: (i) The ball returns to him
Separation = relative velocity × time = (Vx1 – Vx2) t
⇒ there is no velocity in x-direction in the truck’s frame
of reference Vy 7 × 12
= [3 – (–4)] × = = 2.43 m
⇒ Angle of projection = 90° g 10
P hysi cs | 3.57

Note: Here g is taken 10 ms–2. You may take g = 9.8 Vy


ms–2 them separation = 2.47 m. The questions takes the ⇒ Vy = 3 Vx ⇒ 3 =
Vx
value of g to intelligently manipulate the question.
Vy
angle of projection = tan–1 = tan–1 3 = 60°
Sol 18: Let initial velocity V = Vx î + Vy ˆj Vx

t = seconds V(t) = Vx î + (Vy – gt) ˆj Sol 20: If he takes minimum time ⇒ he is always
Vy Vy – gt perpendicular w.r.t water
tanα = tanβ =
Vx Vx ⇒ drift = velocity of water × time

Vy = Vx tana 120
Vw = = 0.2 ms–1 = 12 m/min
10 × 60
Vx tan α – gt
tanβ = If he takes shortest path, his resultant velocity along the
Vx flow of river is 0 ms–1
gt ⇒ i.e. Vw – Vx = 0 ; Vx = 0.2 ms–1
Vx =
tan α – tan β Lets assume his velocity is V
V= Vx2 + Vy2 = 1 + tan α . Vx
2
V(10) = Vy(12.5)

gt 4
⇒ Vy = V
= 1 + tan2 α . tan α – tan β 5
3
gt ⇒ Vx = V ( V 2 + V 2 = V)
= sec α . 5 x y
sin α cos β – cos α sin β
3
cos α cos β ⇒ 0.2 = V
5
1 gt ⇒ V = 0.33 ms–1 = 20 m/min
= . . cosα cosb
cos α sin( α – β)
width = V × 10 = 200 m
gt cos β
V=
sin(α – β) Sol 21: Velocity of wind = u î
⇒ V butterfly w.r.t earth = V + Vwind
Sol 19: Let V = Vx î + Vy ˆj = (10 + u) î + 12 ˆj
Velocity at maximum height Vh = Vx ( Vy = 0) 12
tanθ =
Vy2 10 + u
Maximum height =
2g 3 12
=
4 10 +u
Velocity at half maximum height = Vy
2
⇒ u = 6ms–1
Vy2 1 Vy2 Vy
2
= ⇒ Vy2 = Note: - The resultant velocity is directed along AB.
2g 2 29 2
2
Sol 22: Vrain/grd = – 20 ˆj 10iˆ
 Vy 
V= Vx2 +   Vm = 5 î
 2
 
Vair = 15 î 20ˆj
Now as per given information,
Vrain – Vground = – 20 ˆj θ
2
2  Vy  ⇒ Vrain = 15 î – 20 Ĵ
Vx2 +   = Vx
5  2 Vman = 5 î
 

5 2 Vy2 ⇒ Vrain/man = 15 î – 20 Ĵ – 5z = 10 î – 20 Ĵ
⇒ Vx = Vx2 +
2 2
3 . 5 8 | Motion in a Plane


10 1
So tanθ = = a r = – 3 î – 4 ˆj m/s2
20 2
→ → →
1 And also a = a r + a t
⇒ θ = tan   –1
2 →
Given a = –7 î – ˆj
Circular Motion → → →
∴ a t = a – a r = (–7 î – ˆj ) – (–3 î – 4 ˆj )
Sol 23: Let the time = t0 at the Bullet →
instant the bullet hits the first a t = – 4 î + 3 ˆj m/s2
disc and makes a hole in it. 
And time = t1 when bullet
makes hole in Disc-2. In this
D1 D2 Sol 25: Acceleration inside a rotor = Rω2
time interval Dt = t1 – t0, →
a = Rω2
An angular displacement of θ is made by point A w.r.t
the point B. →
How for a max
At t = t1 A
2
θ
B
amax = R ωmax
A is hole in Disc-I
Given amax = 10g = 100 m/s2
B is hole in Disc-II
∴ ω Dt = θ → (i) 100 10
ωmax = = rad/s = 5 rad/s
4 2
And also in the same time interval Dt;
1
Bullet travelled a distance of ‘’ we know that 1 rad = rev

∴  = v Dt → (ii) 5
∴ ωm = rev/s
Comparing eq → (i) and (ii); we get
n 2π
ω∆t θ
= Sol 26:
v∆t 
w mv 2
v= N sinθ = ... (i)
θ R
N cos θ = mg → ... (ii)
→ →
Sol 24: r = 3 î + 4 ˆj ; v = – 4 î + 3 ˆj
→ → N
| r | = 5m; | v | = 5 m/s θ
θ mv 2
v2 (5)2
We know that radial acceleration = = = 5 m/s2 R
r 5
And this acceleration will be along the negative radial
direction. θ

∴ r = 3 î + 4 ˆj mg

Unit vector in the direction of r
→ Dividing (i) and (ii)
→ r 1
Is r = = (3 î + 4 ˆj ) v2
|r | 5 ⇒ tanθ =
Rg
3 4 ˆj 5
∴ r̂ = î + ... (i) ⇒ V = 108 Km/h = 108 × m/s
5 5 18
V = 30 m/s
→ v2  3 4 
Now a r = (– r̂ ) = 5  – ˆi – ˆj  R = 90 m
5  5 5 
P hysi cs | 3.59

30.30 21 21
∴ tanθ = =1 umbrella = rev/s = .2π rad/s
90.10 44 44
π 21π
⇒ θ= ω= rad/s = 3 rad/s
4 22
Squaring (i) and (ii) and adding them;
Now for a drop on the Rim; velocity
2
 mv 2  →
N2 (sin2θ + cos2θ) =   + (mg)2
 | v | = Rω
 R 
→ 1
2 | v | =   (3) m/s
 mv 2  2
N= (mg)2 +  
 R 
  → 3
|v|= m/s
2
N = m (10)2 + (10)2
Now this is fairly a kinematics problem;
N = 10 m 2 Newton 3
v= m/s
⇒ N = 104. 2 N. 2

1.5
Sol 27: In solving this question, we will use one of the
most important theorems in circles B
Vx
A θ
A x
θ
r
R Vy
O
C 1 2
2θ 1.5 = 0. t – gt
O 2
B C
 ⇒t= 0.3 and x = vt
The figure explains us that for every θ traversed by AB ,
 3
OB traverses an angle of 2θ. x= × 0.3
2
∴ ωOB = 2 × ωAB x = 0.82 m
Hence in this case ω w.r.t C is twice that of w.r.t point C.  vy 
and tan θ =  

∴ ω = 2(0.4) = 0.8 rad/sec.  vx 
→ vy at point B is vy = 0 – gt
| v | = Rω
1 3
R = 50 cm = m vy = – 10 0.3 m/s and vx = m/s
2 2
→ 1  vy   10 0.3 
|v|= (0.8) m/s ∴ tanθ =   =  
2  vx   3/2 

   
→ → →
| v | = 0.4 m/s and a = a r + a t
 θ= 74.6°

But here a t = 0
Sol 29: Acceleration inside a rotor = Rω2
→ → v  2
∴ a = a r =   (– êr ) →
a = Rω2
R 

Now for a max
Sol 28: Angular velocity of the ω
2
amax = R ωmax

Given amax = 10g = 100 m/s2


3 . 6 0 | Motion in a Plane

cos θ = 0.3
100 10
ωmax = = rad/s = 5 rad/s
4 2 1000
R= = 334 m
1 (10)(0.3)
We know that 1 rad = rev

5 Sol 32:
∴ ωm = Rev/s
2π Fcosθ

θ
Sol 30: From the top view; The θ F=mRω2
insect looks at the particle as Insect
∴ x-co-ordinates of the Particle F sinθ
= R cosθ But θ = ωt Now mRω2 cosθ = ma
∴ x = R cos ωt; y = R sin ωt
R ∴ a = Rω2cosq

∴ r particle,insec t θ 1 2
Now s = ut + at
2
= R cos ωt î + R sin ωt ˆj
1
L=0+ Rω2 cosθ t2
Now 2
→ → →
r particle, observer = r particle,insec t + r insec t, observer 2L
t= 2
Rω cos θ

r particle, observer = vt î
Sol 33: In this case, there will mv 2

∴ rPO = (R cos ωt + vt) î + R sin ωt ˆj be a pseudo force acting on A N R
the body. Now we use Work- Rsinθ
Hence the motion will be a cycloid. Energy theorem, i.e. work ma
B
R cosθ θ
done by all the forces is equal R
to change in kinetic energy. mg
Sol 31: Now we shall follow a v We know that, work done by
standard procedure rather normal force and centripetal
than a clumsy formula to find force is zero. a
the radius of curvature. vx
θ
v Work done by pseudo force =
Let us first find vx and vy at t θ
ma.(R sin θ)
=3s vy WPF = maR sinθ
vx = v0 = 10 m/s î Work done by gravitational force = mg(R – R cos θ)

vy = 0 – gt = – 30 m/s ˆj Wmg = mgR(1– cosθ)


Net work done = maR sinθ + mgR (1–cos θ)
v 
tan θ =  y  ∴ tan θ = 3 ... (i) 1
 vx  ≡ mv2 = Rm(a sinθ + g(1 – cosθ))
  2
Now we need to resolve the gravitational force normal
to the curve at point P. v= 2R(asin θ + g(1 – cos θ))

Hence it is equal to g cos θ and


→ → →
v = v 2x + v 2y = 200 + 900 = 10 10 m/s Sol 34: a net = a radial + a tangential

Let ‘R’ be the radius of curvature,



v2
ar = . (– êr )
R
mv 2
Then; mg cos θ = →
R a t = a ( êt )
2
v
R = gcos θ ; tan θ = 3
P hysi cs | 3.61

2  2 2
→  v2  1  3  1 
| a net |=
2
a +  m/s 2 = .1 .   , [ 20 ]2
R  2  2   2  
  P.E.    = [7.5, 2.5] J

1
K.E = mV2 – PE
2
Exercise 2
1
= × 1 × ( 20 )2 – [7.5, 2.5] = [2.5, 7.5] J
2
Projectile Motion

Single Correct Choice Type Sol 2: (B) Its trivial.

Sol 1: (D) V = 20 cosθ î + 20 sinθ ˆj Sol 3: (B) V1 = V1cosq1 î + V1 sinq1 ˆj

x 3 V2 = V2 cosθ2 î + V2sinθ2 ˆj
t= =
Vx 20 cos θ V1(t) = V1 – gt ˆj
2Vy 2 20 sin θ V2(t) = V2 – gt ˆj
t = time of flight = =
g g
V12 = V1 – V2
3 2 20 sin θ V12 is independent of t.
⇒ =
20 cos θ g dV12
i.e. a12 = =0
dt
3g
sin2θ = ⇒ Trajectory of particle 1 w.r.t particle 2 is straight line
20
along the direction of V12.
3
sin2θ =
2
Sol 4: (C) V12 = V1 – V2
2θ = 120°, 60°
⇒ θ = 60°, 30° = (V1 cos θ1 – V2 cos θ2) î + (V1 sinθ1 – V2 sinθ2) ˆj

 3 1  V1 cos θ1 = V2 cos θ2
⇒ sinθ =  , 
 2 2  ⇒ V12 = (V1 sinθ1 – V2 sinθ2) ˆj

2 20  3 1 This relative velocity is along ˆj .


t=  , 
10  2 2  ⇒ Trajectory of 1 w.r.t 2 is a vertical straight line.
Note: They need not be one above the other because
 3 1 their initial x coordinates need not be same
t =  5, 5
  x = x0 + Vxt
1 3  Vx1 = Vx2
cos θ =  , 
 2 2  ∴ V1cosθ1 = V2 cosq2

1 3 ˆ 3 ˆ 1 ˆ but x θ1 ≠ x θ2 ⇒ x1 ≠ x2
ˆ
V= 20  2 i + 2 j, 2 i + 2 j
 
Sol 5: (D) Vy = V sinθ
Vx = [ 5 , 15 ] ms–1
2Vy
Vy2  
2 2 Time of flight =
H= =
1  3  ,  1   [ t
 2   2   20 ] = [0.75, 0.25]
2
2g 2g Vy2
   M a x i m u m height =
2g
Vy2 1
P.E = mgH= m × g × = m Vy2  V1 sin θ1 = V2 sin θ2
2g 2
3 . 6 2 | Motion in a Plane

⇒ Vy1 = Vy2  gT 
2

2 2  
V12 = (V1 cos θ1 – V2cos θ2) î V sin θ  2  (gT)2 gT2
h= = = =
⇒ Trajectory of one with respect to other is horizontal. 2g 2g 8g 8
gT2 = 8 h

Multiple Correct Choice Type substitute in (i)


t  t
V2 y = 4h   1 – 
Sol 6: (A, B, C, D) (a) Rmax = T  T 
g
2 x = V cosθ t
V
hmax = =H
2g R = V cosθ T
⇒ Rmax = 2H (A) t x x  x
= ⇒ y = 4h  R  1 – 
T R    R 
V2 2V 2
(b) R = sin2θ = sinθ cosq
g g
Sol 8: (A, B, C, D) The equation is same as that of a
V2 projectile equation
H= sin2q
2g ax = 0°
V2  dy 
R y = ax – bx2  Vy = 
= sin2θ dx 
h 2g 
Vy = aVx – 2bxVx
2V 2 sin θ cos θ V2
= sin2q ay = a(0) – 2b(Vx)2 – 2bVx(0)
gh 2g
dVx
4 ax = =0
tanθ = dx
h
4 ay = –2b Vx2
θ = tan–1   (B)
h
–ay
2V sin θ Vx = ay = – g
(c) T = g 2b
g
4V 2 sin2 θ ⇒ Vx =
gT2 = 2b
g
Vy = aVx – 2bxVx
V2
R= sin2θ Vy(0, 0) = aVx – 2b(0)Vx
g
Vy = aVx
4V 2 sin2 θ
20 + mθ = g
g Vy = a
2b
∴ gT2 = 2R + mq (C)
Vy
=a
Vx
2V sin θ
Sol 7: (A, B) T = tanθ = a
g
∴ θ = tan–1a
gT
Vy = V sinθ =
2
Sol 9: (A, C, D) Horizontal distance = Vx × t = 4 × 0.4
1 gT 1 = 1.6 m
y = Vyt – gt2 = t – gt2
2 2 2
Vy = gt = 10 × 0.4 = 4 ms–1
gT2  t   1 – t   Vy 
=  
T  
 ... (i) ⇒ angle of impact = tan–1   = tan–11= 45°
2 T   Vx 
 
P hysi cs | 3.63

1 2 1 h1 sin2 θ1 (3 / 5)2
H= gt = × 10 × (0.4)2 = 0.8 m = = = 9 : 16
2 2 h2 (4 / 5)2
sin2 θ2
Vmin = Vx = V cosq
Sol 10: (A, B) In box frame of
reference, a acts upwards Vmin cos θ1
1 (4 / 5)
a u g = = =4:3
i.e. Resultant acceleration is (g – Vmin cos θ2 (3 / 5)
2
a) downwards . If g = a, resultant
acceleration = 0, P will hit C. Angle bisector is 45° as θ1, θ2 are complementary angles.

For a > g it hits roof


Comprehension Type
for a < g , P will hit CD
Sol 13: (A) V = Vcos θ î + V sin θ ˆj
Sol 11: (A, C) Q Goes up and reaches same point. θ = 53°, V = 50 ms–1

2V 2.(5) ⇒ V = 50 cos(53) î + V sin(53) ˆj = 30 î + 40 ˆj


Time of flight till than t = = =1 s
g 10 ⇒ Vx = 30 ms–1
Final velocity = – 5 + 10(1) = 5 ms–1 downwards |v(t)| ≥ |Vx|
Distance to ground ⇒ min velocity = 30 ms–1
1 Vy = 40 ms–1 but Vy changes with time
h = 5(t – 1) + g(t – 1)2 = 5(t – 1) + 5(t – 1)2
2 ∴ incorrect is (A).
where t is time from starting (total time of flight)
2Vy 2 × 40
Distance travelled by P = 2H (given) Sol 14: (D) Time of flight (T) = = =8s
g 10
1 2
2H = 5t + gt Now observe the vertical motion, the body ascends
2
then it descends.
⇒ 2(5(t – 1) + 5(t – 1)2)= 5t + 5t2
Now if we observe descent in reverse time, it looks like
10t – 10 + 10t2 – 20t + 10 = 5t + 5t2 ascent,
5t2 – 15t = 0 Hence tascent = tdescent (reverse time)
⇒ t = 0, t = 3; t > 0 ⇒ t = 3 tascent = T – tdescent ⇒ tdescent = T – tascent
H = 5(3 – 1) + 5(3 – 1) = 30 m
2
T=8
⇒ td = 8 – Ta
V2
Sol 12: (B, C, D) R = sin2q ⇒ ta + td = 8 (0 < (ta, td) < 8)
g
∴ All A, B, C satisfy
(70)2 4800
480 = sin2q ⇒ sin 2θ = = 0.96
10 4900
Sol 15: (A) trajectory equation
2θ = sin–1 0.96 = 74°
θ = 37°, θ2 = 90 – θ = 53° (Complimentary angles) gx2
y = x tanθ –
2(V cos θ)2
θ , θ2 are complimentary as they have same horizontal
range. θ = 53°, V = 50 ms–1
2V sin θ 10 . x2
Time of height t = y = x tan53° –
g 2(50 cos53)2
t1 sin θ1 sin37 3
= = = 4 10x2
t2 sin θ2 sin53 4 y = x  –
3
2
 3
2  50 × 
V 2 sin2 θ  5 
Max height h =
2g ⇒ 180 y = 240 x – x2
3 . 6 4 | Motion in a Plane

Since it is single correct you may as well solve by substituting Sol 20: (D) VAB = VA – VB
any 3 points in motion. You may also eliminate option B, D
VAB > VA
as coefficient of x2 should be negative, which is common
knowledge to be known about trajectory equation. ⇒ VB < 0
Which is possible

Match the Columns Hence statement 1 false.

Sol 16: A → p; B → p, q, r, s; C → p, q, r, s; D → p, r
Sol 21: (A) Statement-II true
(A) Constant velocity ⇒ same direction ⇒ straight line
 Relative vertical acceleration is zero, relative vertical
Answer is (A) (B) velocities don’t charge.
(B) Constant speed ⇒ Constant magnitude of velocity
⇒ Variable direction of velocity ⇒ there is acceleration Circular Motion
⇒ It can follow any path.
Sol 22: (A, D) In a curved path; the direction of velocity
B → p, q, r, s →
(C) With variable acceleration, it can follow any path keeps on changing. So v cannot remain constant under

C → p, q, r, s any conditions. However | v |
(D) Consider a particle moving in circle with uniform = Speed can remain constant.
velocity u.

mu2 → dv
Magnitude of acceleration = , directed toward canter. And a = ; so it follows that acceleration also cannot
r dt

This acceleration has constant magnitude, but variable → dv
remain constant. But still | | =
a is possible
direction. dt
Hence (q) false
Sol 23: (B, D) For a circular motion
Now circle is a special case of ellipse
Sweeping equal area in equal time is
⇒ (s) is also false
only possible when ω is constant.
Straight line is a trivial example of constant acceleration.
→ → →
So p is true. ∴ Now v = r × ω
We know that trajectory of a projectile is parabola. Here So velocity is not constant
acceleration is constant g towards ground.

Hence r is true. But speed = | v | = rω = constant
D → p r. → → →
and a = ar + at
Assertion Reasoning Type → dv dω
at = = r. = zero
Sol 17: (B) Speed of projectile is minimum because dt dt
Vy = 0 → → → →
and a r = r × ( r × ω )
∴ Acceleration is not constant.
Sol 18: (D) Vx = V cosθ
 Angle of projections are different,
Vx ≠ Vx are θ1 ≠ θ2
1 2

so they do not collide

Sol 19: (D) Consider two particles in circular motion


P hysi cs | 3.65

Previous Years’ Questions Therefore, u cos (α + θ) – v cos θ = 0


ucos(α + θ)
Sol 1: (i) Accelerations of particle and block are shown or v = (down the plane)
cos θ
in figure.
Sol 2:
 
i j


 gcos  in
in gs
gs 


Acceleration of particle with respect to block The relative velocity of B with respect to A is perpendicular
to line
= (Acceleration of particle) – (acceleration of block)
of motion of A.
= (g sin θ î + g cos θ ˆj ) – (g sin θ) î = g cos θ ˆj Now
∴ AB V cos30° = VA
motion of particle with respect to block will be a
projectile as shown. ⇒ VB = 200 m/s
And time t0 = (Relative distance) / (Relative velocity)

u 500
Q = = 5sec
 VB sin30o
P R

 g cos 
Sol 3: (D)
The only difference is, g will be replaced by gcosθ.
dv
v = ω2r , where v is the velocity of the block radially
u sin2α u2 sin2α
2 dr outward.
PQ = Range (R) = PQ =
gcos θ gcos θ
v r
∫0 vdv = rdr ω2 ∫
(ii) Horizontal displacement of particle with respect R /2
to ground is zero. This implies that initial velocity
with respect to ground is only vertical, or there is no R2
horizontal component of the absolute velocity of the ⇒ v = ω r2 −
4
particle.
u sin(+) r dr r
u ∫R /2 2
= ω∫ dt
0
R
j a r2 −
 vcos 4
i
u cos(+) 
v = r
R ωt
4
e + e−ωt ( )
  v sin

Let v be the velocity of the block down the plane.


Velocity of particle

= u cos (α + θ) î + u sin (α + θ) ˆj

Velocity of block = – v cos θ î – v sin θ ˆj


∴ Velocity of particle with respect to ground

= {u cos (α + θ) – v cos θ} î + {u sin (α + θ) – v sin θ} ˆj


Now, as we said earlier that horizontal component of
absolute velocity should be zero.
2017-18 100 &
op kers
Class 11 T
By E ran culty
-JE Fa r
IIT enior emie .
S fP r es
o titut
Ins

PHYSICS
FOR JEE MAIN & ADVANCED
SECOND
EDITION

Exhaustive Theory
(Now Revised)

Formula Sheet
9000+ Problems
based on latest JEE pattern

2500 + 1000 (New) Problems


of previous 35 years of
AIEEE (JEE Main) and IIT-JEE (JEE Adv)

5000+Illustrations and Solved Examples


Detailed Solutions
of all problems available

Topic Covered Plancess Concepts


Tips & Tricks, Facts, Notes, Misconceptions,
Forces and Laws of Motion Key Take Aways, Problem Solving Tactics

PlancEssential
Questions recommended for revision
4. F ORCES AND LAWS OF
MOTION

1. INTRODUCTION
In this chapter, we study in detail the actual consequences due to motion, i.e., the concept of force, which we
specifically define as a push or pull experienced by a particular body or system. As we are of aware of the fact that
the equation(s) of motion is/are governed by the choice of reference frame made, we, therefore, also concentrate
on the same by involving different types of reference frames.

Push and pull forces

Figure: 4.1 Figure: 4.2

2. FORCE
Force, by its nature, is better understood as any influence that causes an object to undergo a certain change,
which maybe with respect to its movement, direction, or even geometrical construction. To be succinct, suffice it
to say that a force can facilitate an object with mass to change its velocity, either to accelerate or deform a flexible
object, or both. However, we can also define force using intuitive concepts such as a push or a pull. As mandated
for a vector quantity, force has both magnitude and direction. We generally measure force based on the SI unit (of
Newton) and represent the same using the symbol ‘F.’ It is imperative to understand, therefore, that in case if a body
is subjected to more than one force, then the actual net force acting on that particular body is invariably a vector
addition of all the forces in operation.

3. FREE BODY DIAGRAM


Suppose that we indicate all the operative external forces on an object, then the representation of the same is what
we call as a free body diagram (FBD) of that particular object.
(a) Weight of a body/object: Weight of a body or an object is generally regarded as the force with which earth
attracts that particular body/object toward its center. For example, if we consider ‘M’ as the mass of a body/
4 . 2 | Forces and Laws of Motion

object and ‘g’ as its acceleration due to gravity, then we can conveniently express the weight of that particular
body/object as Mg. However, we always consider that the weight of a body/object is in a direction that is
vertically downward.

(b) Normal force: To understand the concept of normal force, let us consider
a book resting on a table, as an example. The book has a specific weight,
specifically in vertically downward direction and is at rest to begin with.
Therefore, we understand that there is definitely one more force that
Normal
is operative on the block but in an opposite direction, which helps to force weight
balance its weight. The source of this force is none other than the table
Figure: 4.3
and we hence call the same as a normal force. This signifies the fact that
if in case two bodies are in contact with each other, then a contact force
arises; further, if the contact surface is smooth, then the direction of the
force is usually normal to the plane of contact. As stressed earlier, we
always mean that its direction is towards the body under consideration.

(c) Tension in a string: Let us assume that there is a block hanging from
a fixed surface by a string. The weight of this block is acting vertically Tension in
string
downward although it is not under motion; hence, its weight is adequately
balanced by a force originating from the string. We call this force as
‘tension in string.’ Thus, we define ‘tension’ as a resisting force that is Weight
operative in a stretched string. Further we understand that its direction
is along the string but away from the body/object under consideration. Figure: 4.4

Illustration 1: A cylinder of weight w is resting on groove V as shown in the


Figure 4.5. Draw the FBD of the same. (JEE MAIN)

Sol: Weight acts vertically downwards and contact force from the surface is Figure: 4.5
normal to the surface at the point of contact.
The FBD of the cylinder is as shown in the Figure 4.6.
Here, w = weight of the cylinder and represent the normal reactions between
the cylinder and the two inclined walls. N1 N₂
W

Figure: 4.6
Illustration 2: A block of mass m is attached with two strings as shown in the
Figure 4.7. Draw the FBD of the same. (JEE ADVANCED)


Sol: Weight acts vertically downwards and tension forces along the length of
the strings. The FBD of the block is as shown in the Figure 4.8.
Figure: 4.7
T₁


T₂

mg
Figure: 4.8

4. NEWTON’S FIRST LAW OF MOTION


Suppose that if a body is observed from an inertial frame i.e., a frame which is at rest or moving with uniform
velocity, then it will remain at rest or continue to move with uniform velocity unless an external force is applied
P hysics | 4.3

on it. This property due to which a body remains at rest or continues its motion with uniform velocity is called as
inertia. Force is push or pull, which disturbs or tends to disturb inertia of rest or inertia of uniform motion of a body.
Thus, Newton’s first law of motion defines inertia, force and intertial frame of reference. One example in this regard
is the straight line motion of a body in the absence of the constraining force.

Illustration 3: A heavy particle of mass 0.50 kg is hanging from a string fixed with a roof. Find the force exerted
by the string on the particle. (Take g = 9.8 m / s2 ) (JEE MAIN)

Sol: The weight of the particle is balanced by the force of tension in the string.
The forces acting on the heavy particle are
(a) Pull of the earth 0.50 kg × 9.8 m / s2 =
4.9 N , vertically downward
(b) Pull of the string, T, vertically upward.
Figure: 4.9
The heavy particle is at rest with reference to position of the earth (which we
assume to be an inertial frame). Hence, the sum of forces should be zero.
Therefore, T is 4.9 N when acting vertically upward.

Illustration 4: The given diagram shows the forces in operation on a block. Determine whether the block is under
acceleration or not. (JEE MAIN)

Sol: If the net force on the block is non-zero then the block accelerates. If the
net force on the block is zero, then acceleration is zero. 4N 4N
To check whether the particle will have any acceleration or not, let us confirm 30° 30°
if net force is zero or not by resolving the forces in both horizontal and vertical
directions.
8N
Net force in horizontal direction
= 4 cos30° − 4 cos30
= ° 0
Figure: 4.10
8 − 4 sin30° − 4 sin30° ≠ 0
Net force in vertically downward direction =
The net force is not zero. Therefore, the particle will have downward acceleration.

5. INERTIA
Inertia is the resistance of any physical object to any change in its state of motion (including a change in direction).
However, we need to understand that inertia is actually a passive property. Further, it does not permit a body to
do anything but resists active agents such as torques and forces. In other words, it is tendency of objects to keep
moving in a straight line at constant linear velocity.

5.1 Types of Inertia


There are basically three types of inertia.

5.1.1 Inertia of Rest


The inability of a body to change its state of rest by itself is known as inertia of rest. For example.
When we happen to shake the branch of a tree, we observe that the leaves or the fruits fall down. This is because
the branches comes in motion, whereas the leaves or the fruits tend to remain at rest and hence fall down.

5.1.2 Inertia of Motion


The inability of a body to change its state of uniform motion by itself its state of uniform motion is known as inertia
of motion.
Example: (i) When a moving car suddenly stops, we know that the person sitting in the car falls in the forward
direction. This is because the lower portion of the person’s body in contact with the car comes to rest, whereas the
upper portion tends to remain in motion due to inertia of motion.
4 . 4 | Forces and Laws of Motion

(ii) A person runs a certain distance before taking a long jump. This is mainly because the velocity acquired by
running prior to attempting a long jump is added at the time of jump, so that he or she can cover a long distance.

5.1.3 Inertia of Direction


The inability of a body to change by itself its direction of motion is referred to as inertia of direction.
Example: (i) When a car moves around a curve, a person sitting inside it is thrown outward. This is to ensure his or
her direction of motion.

5.2 Linear Momentum


The principle of linear momentum helps us to have a measure of an object’s translational motion. The linear
momentum p of a single particle is defined as the product of the mass m and velocity v of a particle in motion.
i.e., p = mv.
Linear momentum is a vector quantity. Its direction is in accordance with the direction of the velocity. The net
momentum of a system of particles is the sum of momenta. In a system of two particles with masses m1 and m2
p p1 =
and, having velocities v1 and v2, respectively, the total momentum, = + p2 m1 v1 + m2 v 2
In the same manner, the momenta of more than two particles can be added.

6. NEWTON’S SECOND LAW OF MOTION


Newton’s second law states that the net force on an object is equal to the rate of change of its linear momentum,
dp d (mv )
p (i.e., the derivative) in an inertial reference frame:=
F = .
dt dt

F = ma F = ma

Figure: 4.11

However, the second law can also be stated in terms of an object’s acceleration. As the law is valid only for constant-
mass systems, the mass can be considered outside of the differentiation operator by the constant factor rule in
differentiation. Thus,
dv
=F m
= ma, Where F is the net force applied, m is the mass of the body, and a is the body’s acceleration.
dt
Thus, we now know that the net force applied to a body results in a proportional acceleration. In other words, if a
body is in an accelerating mode, then there is force acting on it. Both force and acceleration are vector quantities
(as denoted by the bold type in the Figure 4.11 ). This shows that they have both a magnitude (size) and a direction
relative to some reference frame.
 
Illustration 5: Two forces F1 and F2 act on a 2 kg mass. If = 10 N and = 5 N, find the acceleration. (JEE MAIN)

Sol: Apply Newton’s second law of motion.


Acceleration, as we already know, will be in the direction of the net force and
hence will have magnitude as given by
     
∑ F = ma ; F = F1 + F2 ⇒ F = 10 10^2 2 + 25^ 2 + 2.10.5cos120 5 3N
+ 5 + 2.10.5cos120o + 5° = 3N
P hysics | 4.5

 F2=5N
2.5 3m / sec2
⇒ a =
 120°
Further, if the resultant force is at angle α with F1
2Kg
5sin120° F1=10N
tan α= ⇒ α= 30°
10 + 5cos120°
 Figure: 4.12
There fore acceleration is 2.5 3m / sec2 at an angle 30° with the direction of F1

Illustration 6: A block of mass M is pulled on a smooth horizontal table by a string


making an angle with the horizontal as shown in the Figure 4.13. If the acceleration
of the block is a, find the force applied by both the string and the table on
the block. (JEE ADVANCED)

Sol: List all the forces acting on the block. Take components of forces along the M

horizontal and the vertical. Apply Newton’s second law along the horizontal and along
the vertical. Figure: 4.13
Let us consider the block as the whole system. Therefore, the forces acting on the block are
(a) Pull of the earth, Mg, vertically downward,
N
(b) Contact force by the table, N, vertically upward, and
T
(c) Pull of the string, T, along the string.
Please observe the provided free body diagram for the block. 

The acceleration of the block is horizontal and toward the right. Now, take this direction
as the x-axis and vertically upward direction as the y-axis. Therefore, we have
Component of Mg along the x-axis = 0; component of N along the x-axis = 0 Mg
Figure: 4.14
Component of T along the Xx-axis = T cos
Hence, the total force along the x-axis = T cos.
Now, applying Newton’s second law, T cos = Ma.  ... (i)
Component of Mg along the y-axis =-Mg
Component of N along the y-axis = N
Component of T along the y-axis = T sin
The total force along the y-axis = N + T sin − Mg.
Again applying Newton’s second law, N + T sin − Mg = 0;  … (ii)

From equation (i), T = Ma . Substituting this in equation (ii) N =


Mg − Ma tan θ.
cos θ

7. BASIC FORCES IN NATURE


The various types of forces in nature can be grouped into four categories as listed hereunder:
(a) Gravitational, (b) Electromagnetic, (c) Nuclear, and (d) Weak.

7.1 Gravitational Force


Any two bodies attract each other by virtue of their masses. Now, the force of attraction between two masses is
mm
F = G 1 2 . where, m1 and m2 are the masses of the particles and r is the distance between the particles, and G is
r2
universal constant having the value 6.67 × 10−11 N − m2 / kg2
Similarly, the force of attraction exerted by the earth on any object is called gravity. The force exerted by the earth
on a small body of mass m, kept near the earth’s surface is mg in the vertically downward direction.
4 . 6 | Forces and Laws of Motion

7.2 Electromagnetic Force


Consider two particles having charges at rest with respect to the observer. Now, the force between them has
1 q1q2
magnitude F= where
= ε 8.85419 × 10−12 C2 / N− m2 is a constant
4 πε0 r 2

1 N − m2
The quantity is 9.0 × 109 .
4 πε0 C2

This is called Coulomb force and it acts along the line joining the particles. Electromagnetic force is realized in
many forms in our day-to-day life. Some examples having practical importance in this regard are listed hereunder:
(1) Forces between two surfaces in contact, (2) Tension in a string or a rope, and (3) Force due to spring.

7.3 Nuclear Force


The  nuclear force  (or  nucleon–nucleon interaction  or  residual strong force) is the actual force between two or
more nucleons.  However, its fundamental laws are unknown as of now unlike the laws of Coulomb and Newton.
This force is responsible for binding protons and neutrons in an atomic nucleus.

7.4 Weak Force


Weak force is a fundamental force of nature that underlies some forms of radioactivity. This force controls the decay
of unstable subatomic particles such as mesons, and initiates the nuclear fusion reaction that fuels the Sun. We
should know that the weak force acts upon all known fermions—i.e., elementary particles with half-integer values
of intrinsic angular momentum, or  spin. Particles are known to interact through the weak force by exchanging
force-carrier particles known as the W- and Z particles. These particles are generally heavy, with masses of about
100 times the mass of a proton. It is precisely their heavier nature that defines the extremely short-range nature of
the weak force. Understandably, therefore, this makes the weak force appear weak at the low energies associated
with radioactivity.

8. NEWTON’S THIRD LAW OF MOTION


According to this law, when two bodies interact, they apply forces to one another that are equal in magnitude but
opposite in direction.
However, for simplicity we state this law as, “To every action there is an equal and opposite reaction”. Therefore, the
third law is known also as the law of action and reaction.
But what is the meaning of action and reaction? Further, which force is “action” and which force is “reaction”? We
know that every force that acts on a body is due to the presence other bodies in environment. Suppose that a body
A experiences a force due to other body B. Then, the body B will also experience a force due to A. As per Newton’s
third
 law,
 two forces are equal in magnitude and opposite in direction. Therefore, mathematically we represent it as
FAB = −FBA Here, in this case, we can take either FAB or FBA as action force and the other will be the reaction force.
Another important thing is that these two forces always act on different bodies.

Practical examples of law of motion

(a) First Law: “Every object persists in its state of rest or uniform motion in a
straight line unless it is compelled to change that state by forces impressed on it.” Drag
Airspeed
Before release: Object is in state of rest, air speed is zero, and there is weight but
no drag. When the object is released: Object accelerates – airspeed increases.
As drag depends on airspeed – drag increases.
Weight
When drag is equal to weight: Object no longer accelerates but holds a constant
velocity – terminal velocity. Figure: 4.15
P hysics | 4.7

(b) Second Law: Differential form: Force = change of momentum with change of
d(mv)
time, i.e., F = with mass constant: Force = mass × acceleration F=ma
dt

Figure: 4.16

Force = mass × change in velocity with time


m(V1 − V0 )
F=
(t1 − t0 )

Hence, each has both magnitude and direction.

(c) Third Law: For every action, there is an equal and opposite reaction.

Exhaust Flow Pushed Backward

Engine Pushed Forward

Figure 4.17: Rocket Engine Thrust

PLANCESS CONCEPTS

Working with laws of motion


Step 1: Decide the system: The first step is to decide the system on which the laws of motion to be
applied .The system may be a single particle, a block, a combination of two blocks one kept over the
other, two blocks connected by a string, a piece of string, etc. The only restriction is that all parts of the
system should have identical acceleration.
Step 2: Identify the forces: Once the system is decided, make a list of the forces acting on the system
due to all the objects other than the system. Any force applied by the system should not be included in
the list of forces.
Step 3: Make a Free Body Diagram (FBD): Now, represent the system by a point in a separate diagram
and draw vectors representing the forces acting on the system with this point as the common origin.
Step 4: Choose the axes and Write Equations: Any three mutually perpendicular directions may be
chosen as X-Y-Z axes.
Some suggestions are given below for choosing the axes to solve the problems
4 . 8 | Forces and Laws of Motion

PLANCESS CONCEPTS

If the forces are coplanar, only two axes say X and Y, taken in the plane of forces are needed. Choose
the X-axis along the direction in which the system is known to have or is likely to have acceleration. A
direction perpendicular to it may be chosen as the Y-axis. If the system is in equilibrium, any mutually
perpendicular directions in the plane of the diagram may be chosen as the axes. Write the components
of all the forces along the X-axis and equate their sum to the product of the mass of the system and
its acceleration. This gives you an equation. Write the components of the forces along the Y-axis and
equate the sum to zero. This gives you another equation. If the forces are collinear, this second equation
is not needed.
If necessary you can go to step 1, choose another object as the system, repeat steps 2, 3 and 4 to get
more equations. These are called equations of motion. Use mathematical techniques to get unknown
quantities out of these equations. This completes the algorithm.
Note: (i) If the system is in equilibrium we will write the two equations
= as: ∑ Fx 0=
and ∑ Fy 0 (ii) If
the system is in collinear, the second equation, i.e.∑ Fy = 0 is not needed.

Nivvedan (JEE 2009, AIR 113)


9. IMPULSE
Definition:
 The impulse of a force is defined as the product of the average
 force Raindrop

F and the time interval ∆t during which the force acts: Impulse = F∆t . v0
vf 0 m/s
Impulse, hence, is a vector quantity and has the same direction as the average
force. The SI unit of impulse is Newton-second (Ns).
However, we can also define impulse as the change in the linear 
momentum of a body. Forces acting for a very short duration are called
impulsive forces. Figure: 4.18

9.1 Impulse Linear Momentum Theorem


When a net force is in operation on an object, then the impulse
of the net force is equal to the change in momentum of the object:
 
Impulse
= Change in momentum F∆= t mv f − mv 0

Illustration 7: A truck of mass travelling at 4 m/s is brought to rest in 2 s when it strikes a wall. What force (assume
constant) is exerted by the wall? (JEE MAIN)

Sol: Force on the truck is the change in momentum per unit time.
Using the relation, impulse = change in linear momentum
We have, F.t = mv f − mv 0 = m(v f − v 0 ) or F(2) = 2 × 103 0 − ( −4) or 2F = 8x103 or F = 4x103

Illustration 8: Assume that on a certain day rain comes down at a


velocity of −15 m/s and hits the roof of a car. The mass of rain per
second that strikes the roof of the car is 0.060 kg/s. Assuming that rain
comes to rest upon striking the car, find the average force exerted by the F U
+ve
rain on the roof. (JEE MAIN)
Sol: Force on the roof of the car is equal to the momentum imparted to
it per second by rain drops. Figure: 4.19
P hysics | 4.9

 
( ∑ F ) ∆=t
   
mv f − mv 0 ;
  m  
F∆t =mv f − mv 0 ⇒ F =−   v 0
 ∆t 

F= +0.90 N [Hint: Third law of motion]
−(0.060kg/ s)( −15m/ s) =

Illustration 9: A bullet of mass strikes an obstacle and moves at to its original direction. If its speed also changes
from 20 m/s to 10 m/s, then find the magnitude of impulse acting on the bullet. (JEE ADVANCED)

Sol: Find the impulse along the initial line of motion and along the perpendicular to the initial line of motion.
Mass of the bullet, m = 10-3kg

Figure: 4.20

J1 J1 10−3  −10 cos60° − ( −20) J=


Consider components parallel to = 1 15 × 10−3 N.s
Put 8 & 8.1 alter a and adjust the numbering
J2 10−3 10 sin60° − =
Similarly, parallel to J2 , we have = 0  5 3 × 10−3 N.s
The magnitude of the resultant impulse is given by

J= J12 + J22 = 10−3 (15)2 + (5 3)2 or J= 3 × 10−2 N.s

10. APPLICATION OF LAWS OF MOTION

10.1 Two Blocks in Contact


If two blocks of masses m1 and m2 are in contact on a horizontal frictionless surface, so that a force F applied
horizontally imparts an acceleration a and Fc is the contact force, which is equal and opposite for and. , then
Newton’s second law, when applied to free body diagram, gives the following equations:
m1a ;
F− Fc = Fc = m2a a
F
m₁
F
Adding a = m₂
m1 + m2

m2F a a
∴ Fc = F m₁ Fc Fc m₂
m1 + m2
Figure: 4.21
10.2 Blocks Connected by Strings
If two blocks of masses and are connected by an inextensible string so that a
F
if force F is applied to and there is an equal and opposite tension T in the T
m₂ T m₁
string and if a is acceleration of the masses, then Newton’s law gives
F− T = m1a ; T = m a Adding F ; T = m2 F Figure: 4.22
2 = F (m1 + m2 )a or a =
m1 + m2 m1 + m2
a
If three blocks of masses, , and are connected by two strings with tension T₂ T₂ T₁ T₁
and when a force F applied to imparts an acceleration a to all the blocks, m₃ m₂ m₁ F
then Newton’s law gives the following relations for these three blocks:
F− T1 = m1a ; T1 −=T2 m2a= T2 m3a Figure: 4.23
4 . 1 0 | Forces and Laws of Motion

F
Adding F = (m1 + m2 + m3 )a or a =
m1 + m2 + m3
N
(m2 + m3 )F m3 F
T1 = ; T2 = a
m1 + m2 + m3 m1 + m2 + m3

10.3 Apparent Weight in a Lift mg


If a person is standing in a stationary lift, his or her weight mg acts downward Figure: 4.24
and his or her normal reaction on the floor of the lift acts upward so that
N = mg as per Newton’s third law of motion. However, if the lift is moving
with a constant velocity, then N is equal and opposite to mg as now the net
force is zero.
N – mg = 0 or N = mg
Thus, the apparent weight is equal to true weight.
Further, if the lift is moving upward with acceleration a, N – mg = ma or N = m (g + a)
Thus, the apparent weight is greater than the actual weight.
However, if the lift is accelerating downward with a′, mg - N = ma′ or N = m (g - a′)
Therefore, in this case, the apparent weight is lesser than the actual weight.

10.4 Horse Cart Problem


To analyze this properly, it is probably best to individually consider the
cart and the horse. The cardinal rules while dealing with introductory
physics courses are first identify and isolate the body that you intend to
apply Newton’s second law to, and then identify all forces acting on that
body and add them (as vectors) to get the net force, and finally, use the
relation Fnet = ma.
Figure: 4.25
In the diagrams provided, we have used an oval or a circle to enclose the
subsystem being analyzed. The forces acting on the cart include the forward force that the horse exerts on the cart
and the backward force due to friction at the ground, acting on the wheels. However, at rest, or at constant velocity,
these two are equal in magnitude, because the acceleration of the cart is zero.
On the contrary, the forces that are acting on the horse include the backward force the cart exerts on the horse
and the forward force of the ground on its hooves. However, at rest, or
 at constant
 velocity, these two are equal in
magnitude, because the acceleration of the horse is zero. Therefore, C = −D . Similarly, for the cart, A = −B .
By Newton’s third law, the force the horse exerts on the cart is of equal size and opposite in direction
 to the force
the cart exerts on the horse. (These two forces are an action–reaction pair.) Therefore, B = −C , and this is true
whether or not anything is accelerating.

C
B

A
D

Forces on the cart, no acceleration. Forces on the horse, no acceleration.


Figure: 4.26
 
Since the horse is not accelerating,  C = −D , by Newton’s second law, and, finally we see that all the forces shown
in the diagram are of the same size.
P hysi cs | 4.11

Illustration 10: Two blocks of masses and are placed in contact with F₁ F₂
each other on a frictionless horizontal surface as shown in the Figure 4.27. M₁ M₂
Constant forces and are applied on and as shown in the Figure 4.27. Find
Figure: 4.27
the magnitude of acceleration of the system. Also, calculate the contact
force between the blocks. (JEE MAIN)

Sol: Draw the FBD of each block. Apply Newton’s first law along the vertical
and Newton’s second law along the horizontal. M₁g M₂g

In this problem, acceleration of both blocks will be the same as they are rigid
F₁ N N F₂
and in contact with each other. As the surfaces are frictionless, contact force
on any surface will be normal force only. Let us assume that the acceleration
of blocks be a and contact forces, and N as shown in free body diagrams N₁ N₂
of blocks. [FBD of M₂] [FBD of M₁]

Therefore, by applying, Newton’s second law for Figure: 4.28

F1 − N =
M1a ...... (i) and M1g− N1 =
0 .......(ii)
Applying, Newton’s second law for
N − F1 =
M2a ...... (iii) and M2g− N2 =
0 .......(iv)
F1 − F2 M2F1 + M1F2
=
By solving (i) and (iii) a = and N
M1 + M2 M1 + M2

Illustration 11: A rope of length L is pulled by a constant force F. What P


is the tension in the rope at a distance x from one end where the force is F
applied? (JEEMAIN) A B
Figure: 4.29
Sol: Acceleration of all parts of the rope will be same. Net force on a part of
rope is equal to acceleration multiplied by the mass of that part.
Let AB be a string of length L and F the constant force pulling the rope as shown in the Figure 4.29 provided.
M
Mass per unit length of rope =
L
where M is the total mass. Let P be a point at a distance x from B. If T is the tension in the rope at P then for the
part AP, the tension is toward right while for the part PB it is toward left. If a is the acceleration produced in the
rope, then for part PB
Mx . F(L − x)
F − T mass of PB × a =
= a Also for rope, F = Ma ∴T=
L L

Illustration 12: Two blocks each having mass of 20 kg rest on frictionless surfaces as are shown in the Figure 4.30.
Assume that the pulleys to be light and frictionless. Now, find:
(a) The time required for the block A to move 1 m down the plane, starting from rest;
(b) The tension in the cord connecting the blocks. sinθ = 3 / 5 (JEE ADVANCED)

T
N

MA
A
mAgsin mAgcos

B  mAg

Figure: 4.30

Sol: Draw the FBD of each block. Apply Newton’s second law along the direction of motion for each block. Solve
the equations obtained to get the values for two variables T and a.
4 . 1 2 | Forces and Laws of Motion

Blocks A and B are considered as two systems. The free body diagrams for the blocks A
and B are shown in the Figure 4.31 where T is tension in the string.
T a
mA gsin θ − T = mA a .......(i) N
mBAT
=N mA gcos θ .......(ii)
B
T = mB a ........(iii)
mng
Adding equation (i) and (iii), mA gsin=
θ (mA + mB )a
 mA  Figure: 4.31
 20  3 2
=⇒ a  =  gsin θ  =  (10)   3m / s
m
 A + mB   20 + 20  5
 
1 1
1  2s  2  1 2
(a) s = at2 ; t =   = 2×  = 0.82
2  a  3
(b) T = mB a = 20 × 3 = 60N.

11. LAW OF CONSERVATION OF LINEAR MOMENTUM


The law of conservation of linear momentum states that if no external forces act on the system of objects, then the
vector sum of the linear momentum of each body remains constant and is not affected by their mutual interaction.
By applying the principle of conservation of linear momentum
(a) Decide which objects are included in the system.
(b) Identify the internal and external forces relative to the system.
(c) Verify that the system is isolated.
(d) Set the final momentum of the system equal to its initial momentum. Remember that momentum is a vector.

Illustration 13: From a gun (mass = M) a bullet (mass = m) is fired with speed relative to barrel of the gun which
is inclined at an angle of with horizontal. The gun is placed over a smooth horizontal surface. Find the recoil speed
of the gun. (JEE ADVANCED)
vr
Sol: Apply the law of conservation of linear momentum
along the horizontal direction. 60°
vr sin 60°
Let the recoil speed of the gun is v. By taking gun + bullet
as the system, the net external force on the system in
vr cos 60°– v
horizontal direction is zero. Initially, the system was at M
rest. Therefore, applying the principle of the conservation Components of velocity
of linear momentum in horizontal direction, of bullet relative to ground

we get Mv − m(v r cos60° − v) = 0 Figure: 4.32


mv r cos60° mv r
v = or v
M+m 2(M + m)

Illustration 14: A man of mass is standing on a platform of mass kept on a smooth horizontal surface. Now,
the man starts moving on the platform with velocity relative to the platform. Based on the above, find the recoil
velocity of the platform.  (JEE MAIN)

Sol: Apply the law of conservation of linear momentum along the


horizontal direction.
vr – v
Absolute velocity of the man = – v where v = recoil velocity of the
platform. By considering together the platform and the man as a v
system, the net external force acting on the system in horizontal
direction is zero. However, the linear momentum of the system remains
Figure: 4.33
P hysi cs | 4.13

constant. Initially, both the man and the platform were at rest.
mv
Hence, =0 m1 (vr − v) − m2 v ∴v = 1 r
m1 + m2

12. VARIABLE MASS


Problems related to variable mass can be solved in the following three steps
(a) Make a list of all the forces acting on the main mass and then apply them on it.
   dm 
 (b) Apply an additional thrust force Ft on the mass, the magnitude of which is v r  ±  and direction is given
 dt 
 
by vr , in case the mass is increasing otherwise the direction of − vr , if it is decreasing.

 dv
 (c) Find the net force on the mass and then
= apply Fnet m
= (m mass at that particular instant)
dt
u v

At t=0 At t=t
v=u m=m
m = m0 v=v

Exhaust velocity = vr

Figure: 4.34

Let m0 be the mass of the rocket at time t = 0. Let m be its mass at any time t and v its velocity at that moment.
Initially, let us suppose that velocity of the rocket is u.
 dm 
Further, let  −  be the mass of gas ejected per unit time and, the exhaust velocity of the gases. Usually
 dt 
 dm 
−  and, are kept constant throughout the journey of the rocket. Now, let us write few equations which can be
 dt 

used in the problems of rocket propulsion. At time t = t,


 dm 
(a) Thrust1.force onforce
Thrust the rocket
on the rocket =
Ft v r  −  (upwards)
(upwards)
 dt 
2. of
 (b) Weight Weight of the
the rocket rocket W = mg (downwards)
(downwards)

3. Net
 (c) Net force onforce on the rocket
the rocket Fnet= Ft − W (upwards)
(upwards)

 dm 
  or or Fnet =v r  −  − mg
 dt 
F dv v r  −dm   −dm 
 (d) Net4.acceleration of theofrocket
Net acceleration
the rocket a= or
=   − g or dv v r 
=  − gdt
m dt m  dt   m 

v −dm
m t m 
v − u vr ln  0
− g∫ dt or =  − gt
∫ dv vr ∫
or=
u m0 m 0
 m 
m 
Thus, v =u − gt + v r ln  0   ...(i) ... (i)
 m 
 dm  dm
Note:
1.=Ft Vr  −  is upwards, as vr is downwards and is negative.
 dt  dt
4 . 1 4 | Forces and Laws of Motion

m 
2. If gravity is ignored and initial velocity
= eq(i) reduces to v vr ln  0
of the rocket u 0,= 
 m 

Illustration 15: (a) A rocket set for vertical firing weighs 50 kg and contains 450 kg of fuel. It can have a maximum
exhaust velocity of 2 km/s. What should be its minimum rate of fuel consumption?
(a) (i) To just lifting it off from the launching pad?
(ii) To give it an acceleration of 20 m/?
(b) What will be the speed of the rocket when the rate of consumption of the fuel is 10 kg/s after whole of the
fuel is consumed? (take g = 9.8 m/). (JEE ADVANCED)

Sol: Use the equation of motion for variable mass.


(a) (i) to just lift it off from the launching pad
 −dm   −dm  mg
Weight = thrust force or mg = v r   or  =
 dt   dt  vr
 −dm  (450 + 50)(9.8)
Substituting the value, we get  = = 2.45 kg / s
 dt  2 × 103
(ii) net acceleration a = 20m / s2
Ft v r  −dm 
∴ ma =
Ft − mg or =
a −g =
or a  −g
m m  dt 
 −dm  m(g+ a)
This gives  =
 dt  vr
 −dm  (450 + 50)(9.8 + 20)
Substituting the values, we get  = = 7.45kg / s.
 dt  2 × 103
(b) The rate of fuel consumption is 10 kg/s. So, the time for the consumption of entire fuel is t = 450/10 = 45 s
m 
Using Eq. (i), i.e., v =u − gt + vr ln  0 
 m 
Here u = 2 × 103 m / s,m0 =
0, v r = 500kg and m =
50kg
 500 
Substituting the values, we get v = 0 − (9.8)(45) + (2 × 103 )ln  
 50 
Other Example of Variable Mass System is Falling raindrop

Illustration 16: Suppose that a raindrop falls through a cloud and accumulates mass at a rate of kmv where k > 0
is a constant, m is the mass of the raindrop, and v its velocity. What is the speed of the raindrop at a given time if
it starts from rest, and what is its mass? (JEE ADVANCED)

Sol: Use the equation of motion for variable mass.


Then, the external force is its weight mg and so we have
d dv dm dv
mg = (mv) = m +v =m + kmv 2
dt dt dt dt
dv
Since we know that dm/dt = kmv. Cancelling the mass and rearranging = g − kv 2 ,
dt
t
v dv
So that, ∫0 =
g − kv 2
∫=
dt t
0

Now set V 2 = g / k and use partial fractions to get


v dv 1 v 1 1 1 V+v
t= ∫0 g − kv 2 = ∫ +
2kV 0 V + v V − v
dv =
2kV
log  
V−v
P hysi cs | 4.15

 e2kVt − 1 
so, V + v = (V − v)e2kVt ,i.e. v = V   = V tanh(Vkt), so that v = g
 e2kVt + 1  tanh( kgt )
  k
dm g
Now we may f ind the mass : we have = kmv
= km tanh( kgt
= ) m kg tanh( kgt ).
dt k
t 1 dm t
Thus, ∫ dt = ∫ kg tanh( kgt )dt
0 m dt 0

m dm t
∫m0 m
= ∫0 kg tanh( kgt )dt

logm − logm0 =
logcosh( kgt )

which gives m = m0 cosh( kgt )

13. EQUILIBRIUM
Equilibrium is the condition of a system, when net external farce is zero.

13.1 Equilibrium of Concurrent Forces


A simple mechanical body is said to be in equilibrium, if it does not experience any linear acceleration; however,
unless it is disturbed by an outside force, it will continue in that condition indefinitely. For a body facing concurrent
forces, equilibrium arises if the  vector  sum of all forces acting upon the body is zero. There are two types of
equilibrium as listed hereunder:
(a) Static equilibrium: When a body is at rest under the influence of external forces acting on the it.
(b) Dynamic equilibrium: If net external force is zero but the velocity of a body is not zero, i.e., body moves with
a constant velocity.

13.2 Constrained Motion


When a motion of a body can be controlled, then the motion is said to be a constrained motion.
For example, when a body tied with a string is lowered under the effect of gravity, then its motion
is a constrained motion. Also, motion of masses and can be controlled by choosing an appropriate
value for and. m₁ m₂

Figure: 4.35
13.2.1 Masses Connected by Pulley and Constraint Relation
Let us consider blocks of masses and connected by a string and passing over the pulley as shown in the Figure
4.36. Let be the acceleration of downward and be the acceleration of upward. Let T is the tension in the string,
so that the pulley moves clockwise. For block,
As there are three unknown parameters, we take the following steps for
writing the constraint relation and hence find the parameters:
x0
(a) Assume direction of acceleration of each body.
(b) Locate position of each block from any fixed point like, for example,
center of the pulley. x2 a2
T T
a1 x1

(c) Identify the constraint and write the equation of constraint in terms of m2 m1
distance.
m2g m1g
(d) Write the equation of constraint and hence differentiate twice to find
one of the parameters. Figure: 4.36

In this case, the string is inextensible; therefore, the constraint the length
of string remains constant. If is the length of the string passing over the
pulley, and lengths of string from the pulley to and respectively, then the
4 . 1 6 | Forces and Laws of Motion

total length L of the string remains constant.


∴ x1 + x2 + x0 =L =cons tant
dx1 dx2 d2 x1 d2 x2
Differentiate, + 0 ; Differentiate,
= + 0
=
dt dt dt2 dt2
As and have opposite directions,
0 ;
a1 − a2 = a=
1 a=
2 m1a ;
a ∴ m1g − T = T − m2g =
m2a

 m − m2  m2 (m1 − m2 )g  2m1m2 
Adding a =  1  g ; T = m2a + m2g = + m2g ; T =   g
 m1 + m2  m1 + m2  m1 + m2 
 2m1m2 
If the pulley is pulled in upward direction with an acceleration a,=
then T   (g + a)
 m1 + m2 

Illustration 17: Find the relationship between accelerations of blocks A and B based on
the Figure 4.37.  (JEE MAIN)

Sol: Measure all distances of pulley and block from a fixed point (stationary point).
The physical property that we can use here is the inextensibility of string,
B A
i.e., ab + bc + cd + de + ef = constant.
Figure: 4.37
Let at any moment A and B are distances and from the support as shown in the Fig. 4.37.
Let us take gh= and ik= and hence express the length of string in equation (i) in terms of, l1 and l2 .
We hence obtain XB − l1 + bc + (XB − l1 − l2 ) + de+ (X A − l2 ) =constant Here,, bc and de are constants.
cons tant 
∴ 2XB + X A = ..........(1) ... (i)
a
Let at time ∆t, changes to + ∆ and changes to - ∆ dk e

[therefore, B is assumed to move downward] XB XA

Then, 2(XB + ∆ XB ) + (X A − ∆ XB ) = constant 


.......(2) ... (ii) bg c
h
From (i) and (ii) 2∆ XB − ∆ X A = 0 B f
A
 2 ∆ XB   ∆ X A 
Also,   −  0 ;
 = 0 also, 2∆VB − ∆VA = 0
2VB − VA = Figure: 4.38
 ∆t   ∆t 
2∆VB ∆VA
− = 0 ∴ 2aB = aA
∆t ∆t
Hence, we prove that magnitude of acceleration of A is twice the magnitude of acceleration of B.
Let us assume that B moves by a distance x during an interval of time, and this will cause movement of pulley g by
x. Now, an extra length of 2x of string will come to the left of pulley k. This must be coming from the right side of
the pulleys. Hence, displacement of A will be 2x. On the basis of this discussion, we can say that if acceleration of
block B is a, then the acceleration of A will be 2a.

14 PSEUDO FORCE
T cos  a
4.1 Inertial and Non-inertial Frames of References T
Non-accelerated frames of reference are called inertial frames, whereas T sin 
ma
accelerated frames are called non-inertial frames. If one is travelling in a mg Observer
train which is accelerating forward, the body in the train is pushed backward
and he or she is pushed forward when the brakes are applied. This is due to
inertia of the body. Such an accelerated frame is called a non-inertial frame. Figure: 4.39
P hysi cs | 4.17

In order to make Newton’s laws applicable to such a frame, a fictitious force or pseudo force is applied on the
body. Based on the above discussion, we now understand that the magnitude of this pseudo force is equal to the
product of the mass m of the body and acceleration a of the reference frame and its direction is opposite to the
acceleration of the frame.
∴ pseudo force, F =−m × a . Thus in a non-inertial frame trolley moving with an acceleration a hanging bob of
mass m will be deflected through an angle due to a pseudo force acting in backward direction. In the non-inertial
frame of reference, this bob is in equilibrium under the action of force due to tension T, weight mg and the pseudo
force ma in a direction making an angle with the vertical.
a a
T sin θ =ma ; T cos θ =mg ; tan=θ θ tan−1  
or=
g g
Example: Motion of a block on an inclined plane is an example of accelerated frame of motion.
Motion of a Block on a Smooth Inclined Plane: Let us consider a block N
of mass m placed on a frictionless inclined plane, inclined at an angle to a
the horizontal. We observe that the normal reaction N acts perpendicular
mg sin 
to the plane and its weight is resolved into component mg sin along the mg cos q
plane which slides the block downward with acceleration a and component
mg cos perpendicular to the plane downward, which is equal and opposite m
to the normal reaction.
Figure: 4.40
∴ mgsin θ =ma mgcos θ =N or
= a gsin θ However, if the plane is
provided with a horizontal acceleration a′ in the horizontal direction as a
shown in the Figure 4.44, then the body lies in an accelerating frame of N mg cos 
reference and a pseudo force ma′ acts horizontally in a direction opposite a’
to that of a′ because an inertial force ma′ acts on it in the direction of ma’
mg sin 
a′. Thus ma′ can be resolved into a component ma′cos up the plane and

md
ma′sinperpendicular to the plane in the downward direction as shown

mg

sin

co
in the Figure 4.44. From Newton’s second law of motion, we know that: mg

s
mgsin θ − ma'cos θ = ma or=
a gsin θ − a'cos θ Figure: 4.41
=N m(gcos θ + a'sin θ) . If the body is at rest relative to the inclined plane,
then a=0 or gsin
= θ a'cos θ or =a' gtan θ

Illustration 18: A frictionless block 3 carries two other frictionless 1


blocks 1 and 2 connected by a light string passing over a weightless
and frictionless pulley as shown in the Figure 4.42. What horizontal
3 2
force must continuously be applied to block 3 so that 1 and 2 do not
move relative to 3? (JEE ADVANCED)

Sol: Analyze the motion of blocks 1 and 2 in the reference frame of block Figure: 4.42
3. As block 3 is accelerated, blocks 1 and 2 experience pseudo forces in the
frame of block 3.
Let a1, a2 and a3 be the accelerations of 1, 2, and 3, respectively. Let a1x, a2x and a3x be the absolute horizontal
acceleration of 1, 2 and 3 to the right and a1y, a2y and a3y be their downward accelerations. According to the
constraints of the problem a1y = 0, a3y = 0
Let a13x = relative acceleration of 1 w.r.t. 3. = a1x − a3x = 0 ⇒ a3x = a1x
a23y = relative acceleration of 2 w.r.t 3. = a2y − a3y = a2y − 0 ⇒ a2y = 0
a23x = relative acceleration of 2 w.r.t. 3. = a2x − a3x = 0 ⇒ a2x = a3x
N’
∴ a1x = a2x = a3x and a1y = a2y = a3y = 0 N T
N
F
Consider the free-body diagrams of blocks 1 to 3. T R 2 3
1
From the FBD, we obtain the following equations. R
T
T = m1a1x ........(i)  m₂g m₃g
 For Block I
m₁g T
N− m1g =
0 .......(ii)
Figure: 4.43
4 . 1 8 | Forces and Laws of Motion

R = m2a2x ........(iii)
 For Block II
T − m2g = 0 .......(iv) 
F −R − T = m3a3x .......(v) 
 For Block III
N'− N − T − m3g =0 ........(vi)

Adding (i), (iii) and (v)


F = m1a1x + m2a2x + m3a3x = a1x (m1 + m2 + m3 ) (∴ a1 x = a2 x = a3 x )
m m
From (i) and (iv), a1x = 2 .g ; F = (m1 + m2 + m3 ). 2 .g
m1 m1

15. FRICTION
If there are two bodies in contact with each other, then the force which
opposes the relative motion between two bodies in contact is called force
of friction. Further, the magnitude of the frictional force depends upon the fmax A
nature of two surfaces in contact. This is primarily due to surface irregularities Force of fK
at molecular levels, with the result that even a highly polished surface has Friction
irregularities. This results in producing interlocking of uneven surfaces. fS
Once there is smooth motion of the body, the friction is less than the
maximum force of static friction or limiting friction.The variation of force of O Force Applied, F
friction with the applied force is shown in the graph when any block is
moving over another surface. However, when any block is at rest, the Figure: 4.44
resultant force of static friction is equal to the force applied. Then, it reaches
to a maximum value at A, the limiting friction. Once the motion resumes, a
lesser force is required for maintaining uniform motion.

5.1 Static Friction


R
Suppose that if the force applied in the horizontal direction on a surface
is less, then the body does not have any motion because an equal and
F
opposite frictional force is present. Hence, it is clear that static friction is in
fs
operation only between surfaces that are at rest with respect to each other.
As F is increased the frictional force too increases continuously until a stage
is reached when the body is just at the point of sliding. The force of friction mg
at this stage is called a limiting friction or maximum static friction.
Figure: 4.45
15.2 Coefficient of Friction
The coefficient of friction, generally specified by the Greek letter mu (µ), is the ratio of limiting or maximum value of
fmax
force of friction to reaction R between surfaces when the body is just about to move. µ = ; fmax = flimiting = µsR
R
Thus if the body is not in motion, the static frictional force and external applied force parallel to the surface are
equal in magnitude but opposite in direction and hence F is directly proportional to . However, if external force F
exceeds, then the body slides on the surface and magnitude of frictional force decreases than, the frictional force
for sliding = = R where is the coefficient of kinetic friction. As, is less than, the coefficient of kinetic friction is less
than the coefficient of static friction.

15.3 Angle of Friction A


R
The angle between the normal reaction R and the resultant of limiting friction with
normal reaction is called the angle of friction and is denoted by λ. 
fmax
tan λ = = µ or
= λ tan−1 (µ ) Suppose that if a body of mass m is placed on an inclined fm O
R
Figure: 4.46
P hysi cs | 4.19

plane whose inclination is gradually increasing. Then the body just starts sliding down at fs
R
a certain angle of inclination θ. Now, the weight mg can be resolved into a component
mg sin due to which the body is about to slide down against maximum or limiting value
mg sin 
of friction and therefore the second component mg cos balances the normal reaction R
perpendicular to the inclined plane. mg cos 
mg
fs
; R mgcos θ ;
fs mgsin θ=
∴= ∴ = tan θ = µ as µ tan λ ;
= θ = λ = tan−1 (µ ) Figure: 4.47
R
Here, the angle θ is called angle of inclination. We now know that the angle of friction λ is that minimum angle of
inclination of the inclined plane at which a body placed at rest on the inclined plane just starts sliding down.
However, when θ < λ, then the body is in equilibrium and does not slide. On the contrary, when θ > λ, then the
body starts sliding down with an acceleration.

15.4 Motion of a Block on a Rough Inclined Plane


a N
Let us assume that a block of mass m is moving down an inclined plane N
with an acceleration a. Now, the coefficient of friction between the block
mg sin 
and inclined plane equal to µ, the force of friction µN will be acting along
the plane upward as shown in the Figure 4.48. Thus, the weight mg of the mg cos 
block is resolved into component mg cosθ opposite of normal reaction and mg
component mg sin downward opposite to µN.
Figure: 4.48
Thus, from Newton’s second law of motion, mg sin θ – µN = ma
N = mg cosθ or mg sinθ - µ mg cosθ = ma ∴ a = g(sinθ - µcos θ)
However, if the block is moving upward and its retardation is a, where the frictional force acts downward, then
ma mgsin θ + µmgcos θ
= ∴
= a gsin θ + µgcos θ.

PLANCESS CONCEPTS

Value of friction is not always equal to µN. Further, µN is the maximum value of friction. Friction does not
oppose motion; rather, it opposes relative motion between two surfaces.
Anand K (JEE 2011, AIR 47)

Illustration 19: A heavy box of mass 20 kg is pulled on a horizontal surface by applying a horizontal force. If the
coefficient of kinetic friction between the box and the horizontal surface is 0.25, then find the force of friction exerted
by the horizontal surface on the box. (JEE MAIN)
Sol: Force of friction on a body sliding on a surface is equal to the normal reaction N
multiplied by the coefficient of kinetic friction between the pair of surfaces.
The situation is shown in the Figure 4.49. In the vertical direction, there is no acceleration;
therefore, N=mg. fk F

As the box slides on the horizontal surface, the surface exerts kinetic friction on the box. mg
Therefore, the magnitude of the kinetic friction is fk =µk N = µkMg
Figure: 4.49
=0.25 × (20kg) × (9.8m/ s2 ) =49N. This force thus acts in the direction opposite to that of the pull.

Illustration 20: Two blocks, M1 and M2, connected by a massless string slide down M1
an inclined plane, having an angle of inclination of. The masses of the two blocks are
= 4 kg and = 2 kg, respectively and the coefficients of friction of and with inclined M2

plane are 0.75 and 0.25, respectively. Assuming the string to be taut, find
37°
(a) The common acceleration of the two masses and
Figure: 4.50
4 . 2 0 | Forces and Laws of Motion

(b) The tension in the string. (note: sin =0.6, cos =0.8)  (JEE ADVANCED)

Sol: Let each block is having acceleration a down the incline plane. Draw the FBD of each block and apply the
Newton’s second law of motion along the direction of motion. Solve the equations obtained to get the value of
two variables a and T.
Let a be the common acceleration of the system and T be the tension in the string µ=
1 3 4, µ=
2 1 4
Equation of motion for M1 and M2 are : M1a
Mg cos  R1
M1a M1gsin37° + T − µ1M1gcos37°
= ......(i)
Mg sin 
M2a M2gsin37° − T − µ2M2gcos37°
= .....(ii) M1 T R2 a
T

s
Now, by adding, (i) and (ii)

co
M2

g
s

1
M
(M1 + M2 )a
= (M1 + M2 )gsin37° − (µ1 M1 + µ2 M2 )gcos37° g co


Mg sin 

s
mM 2

co
M 1g

g
M2g
 3 1 

2
M
37°
∴ (4 + 2)a = (4 + 2)g× (0.6) −  4 × + × 2  g × 0.8
 4 4  Figure: 4.51
9.8 × 0.8 7.64
6a g[3.6 − 2.8]
= or =a = = 1.27m / sec2
6 6
6 1 8
From
= (ii) T M2gsin37° − µ2M2gcos37° − M2a =2 × 10 × − × 2 × 10 × − 2 × 1.27 = 12 − 4 − 2.54 = 5.46 Newtons
10 4 10

16. CIRCULAR DYNAMICS


The motion of a particle particularly along a circular path is called its circular motion and it can be uniform, with
constant angular rate (and constant speed), or non-uniform with a changing rate. In uniform circular motion, a
resultant non-zero force is in operation on the particle. This is because a particle moving in a circular path is
accelerated even if speed of the particle remains constant. This acceleration is due to change in direction of the
velocity vector. As we have already seen that in uniform circular motion tangential acceleration (at ) is zero, the
v2
acceleration of the particle is toward the center and its magnitude is . Here, v is the speed of the particle and r
r
is the radius of the circle. The direction of the resultant force F is, therefore, toward the center and its magnitude is
mv 2
F = ma or F = Or =F mrω2 (as v=r ω )
r
Here, ω denotes the angular speed of the particle. The force F is called the centripetal force. Thus, a centripetal
mv 2
force of magnitude is required to keep the particle moving in a circular path with constant speed. This force is
r
generally provided by some external sources such as friction, magnetic force, Coulomb force, gravitation, tension, etc.

Illustration 21: Assume that a small block of mass 100 g moves with uniform speed in a horizontal circular groove,
with vertical side walls, of radius 25 cm. If the block takes 2 s to complete one round, then find the normal contact
force by the side wall of the groove. (JEE MAIN)

Sol: The normal contact force provides the necessary centripetal force to the block to move in a circle.
2π × (25 cm)
The speed of the=block is v = 0.785m / s.
2.0s
v 2 (0.785m/ s)2
a
The acceleration of the block is= = = 2.5m / s2
r 0.25m
However, toward the center, the only force in this direction is the normal contact force due to the side walls. Thus,
2
from Newton’s second law, this force is =
N ma
= (0.100kg)(2.5m/ s= ) 0.25N
P hysi cs | 4.21

PLANCESS CONCEPTS

I have found students often confused over the concept of centripetal force. They
think that this force acts on a particle moving in a circle. This force does not act but
required for moving in a circle which is being provided by the other forces acting on
the particle. Let us take an example, Suppose a particle of mass ‘m’ is moving in a
vertical circle with the help of a string of length l fixed at point O.
Let v be the speed of the particle at its lowest position. When I ask the students what T
forces are acting on the particle in this position, they immediately say, three forces are
mv 2
v
acting on the particle: (i) tension, T (ii) weight, mg and (iii) centripetal force, (r=I).
I
However, they are wrong. Only the first two forces T and mg are acting on
the particle. mg
mv 2
The third force is required for circular motion which is being provided by T and
I Figure: 4.52
2
mg. Thus, the resultant of these two forces is v =
µrg or v = µrg toward O. Or we
mv 2
can write T − mg = 
I Ankit Rathore (JEE Advanced 2013, AIR 158)

17. UNIFORM CIRCULAR MOTION


If a particle moves on a circular path with constant speed, its motion is called as a uniform circular motion. In this
type of motion, angular speed of the particle is also constant. Further, linear acceleration in such motion will not
have any tangential component; therefore, the particle possesses only radial or centripetal acceleration. Therefore
in case of uniform circular motion the particle will have acceleration toward the center only and is called as
2
centripetal acceleration having magnitude v or ω2R . However, the magnitude of acceleration remains constant
R
but its direction changes with time. If a particle moving on circular path is observed from an inertial frame, then we
2
know that it has an acceleration ω2R or v acting toward the center. Therefore, from Newton’s second law of
R 2
motion, there must be a force acting on the particle toward the center of magnitude mω R or mv . This required
2

force R

mv 2
for a particle to move on a circular path is called as centripetal force. ∴ Centripetal force =
R
The term “centripetal force” merely signifies a force toward the center; however, it tells nothing about its nature or
origin. Further, the centripetal force may be a single force due to a rope, a string, the force of gravity, friction, and
so forth or it may be resultant of several forces.

Illustration 22: A ball of mass 0.5 kg is attached to the end of a cord whose length is 1.50 m. The ball is whirled in
a horizontal circular path. If the cord can withstand a maximum tension of 50.0 N, what is the maximum speed the
ball can have before the cord breaks? (JEE MAIN)

Sol: The tension force in the cord provides the necessary centripetal force to the ball to move in a circular path.
v2
Because the centripetal force in this case is the force T exerted by the cord on the ball, we have T = m ; therefore,
r
Tr
solving for v, we have v =
m
The maximum speed that the ball can have corresponds to the maximum tension. Hence, we find

Tmaxr (50.0N)(1.50m)
vmax
= = = 12.2m / s
m 0.500kg
4 . 2 2 | Forces and Laws of Motion

18. NON-UNIFORM CIRCULAR MOTION


If the speed of a particle moving in a circle is not constant, then the acceleration has both radial and tangential
v2 dv
components. These radial and tangential accelerations are given as: ar =ω2r = ; at =
r dt
2 2
 v 2   dv 
Then, the magnitude of the resultant acceleration will be: a = ar2 + a2t =   + 
 r   dt 
   
dv dt
However, if the direction of the resultant acceleration makes an angle β with the radius, where tan β =
v2 r
2 2

−1 dv dt
  v 2   dv 
then, in the direction tan  2  with radius of circle, acceleration needs a resultant force of m   +  
 v r   R   dt 
     

Illustration 23: A car moves on a horizontal circular road of radius R. The speed of the car is increasing at a rate
dv
= a . The frictional coefficient between the road and tire is µ . Find the speed at which the car will skid.
dt
(JEE ADVANCED)
Sol: The net acceleration of the car is the vector sum of the centripetal acceleration and the tangential acceleration.
By Newton’s second law the friction force on the car is (mass)×(net acceleration).
Here, at any time t, the speed of the car becomes V; therefore, the net acceleration in the plane of the road is
2
 v2 
  + a2
R 
 
( ) . This acceleration is provided by the frictional force. At the moment, the car will slide if it reaches
2
 v2 
( )
1/ 4
the speed as given by M   + a2 R 2 (µ2 g2 − a2 )
µMg ⇒ v =
=
R   
 

Illustration 24: A large mass M and a small mass m hang at the two ends of the Tcos
string that passes through a smooth tube as shown in the Figure 4.53. The small
mass m, which lies in the horizontal plane, moves around in a circular path. The T
Tsin
length of the string from the mass m to the top of the tube is l and θ is the angle
this length makes with the vertical. What should be the frequency of rotation of the
mg
small mass m so that the large mass M remains stationary? (JEE MAIN)
M
Sol: For the mass M to be stationary the tension in the string should balance the
weight of M. For mass m the horizontal component of tension in the string provides mg
the centripetal force. The vertical component of tension balances the weight of m. Figure: 4.53

The forces acting on the small mass m and the large mass M are shown in the Figure 4.56. When mass M is
stationary, then T = Mg  … (i)
where T is tension in the string.
For the smaller mass, the vertical component of tension T cos θ balances mg and the horizontal component T sin θ
supplies the necessary centripetal force.
T cos θ =mg  … (ii)
2
T sin θ= mrω  … (iii)
ω being the angular velocity and r is the radius of horizontal circular path.
2 Mgsin θ Mgsin θ Mg
From (i) and (iii), Mg sinθ = mrω
= ⇒ ω = =
mr mlsin θ ml
P hysi cs | 4.23

1 1 ω 1 Mg
= =
Frequency of rotation= ∴ Frequency =
T ' 2π / ω 2π 2π ml

19. CENTRIPETAL FORCE


An observer in a rotating system is another example of a non-inertial observer category. Suppose that a block of
mass m lying on a horizontal frictionless turntable is connected to a string as shown in the Figure 4.54. Then,

according to an inertial observer, if the block rotates uniformly it hence undergoes an acceleration of magnitude
v2
r where v is the tangential speed. The inertial observer hence concludes that this centripetal 2
acceleration is
mv
provided by the force exerted by the string T and writes as per Newton’s second law T = .
r

t
N N Noninertial observer

t
2
mv /r

mg Inertial observer mg

(a) Figure: 4.54 (b)

However, according to a non-inertial observer attached to the turntable, the block is at rest. Therefore, by applying
mv 2
Newton’s second law, this observer introduces a fictitious outward force of magnitude . According to the non-
r
mv 2
inertial observer, this outward force balances the force exerted by the string and therefore T − = 0 . In fact, the
r
centrifugal force is sufficient pseudo force only if we were analyzing the particles at rest in a uniformly rotating
frame. In contrast, if we analyze the motion of a particle that moves in the rotating frame then we may have to
assume other pseudo forces together with the centrifugal force, such forces are called Coriolis forces. The Coriolis
force, named after the 19th century French engineer-mathematician, is perpendicular not only to the velocity of the
particle but also to the axis of rotation of the frame. Once again we should be remembering that all these pseudo
forces, centrifugal or Coriolis, are needed only if the reference frame is rotating. We must know that if we work from
an inertial frame, then there is no need to apply any pseudo force. However, we should be aware of the fact that
there should not be a misconception that centrifugal force acts on a particle because the particle describes a circle.
Therefore, when we are working from a frame of reference that is rotating at a constant angular velocity ω w.r.t.
an inertial frame, then we have to obviously assume that a force m ω2 r acts radially outward on a particle of mass
m kept at the distance r from the axis of rotation. Then only we can apply Newton’s laws of motion in the rotating
frame. This radially outward pseudo force is called centrifugal force. One should be careful when using fictitious
forces to describe such physical phenomena. Remember that fictitious forces are used only in non-inertial frames
of references. Therefore, when solving problems of this nature, it is often best to use an inertial frame.

Illustration 25: A table with smooth horizontal surface is fixed


in a cabin that rotates with angular speed ω in a circular path of
O
radius R. A smooth groove AB of length L(<<R) is made on the A B
surface of table as shown in the Figure 4.55. A small particle is R
kept at the point A in the groove and is released to move. Find the
Figure: 4.55
time taken by the particle to reach the point B. (JEE MAIN)

Sol: In the reference frame of cabin the particle experiences centrifugal force in the radial direction. This force can
be assumed constant for L<<R.
Let us analyze the motion of particle with respect to the table which is moving with cabin at an angular speed of
ω . Now, along the smooth groove AB centrifugal force of magnitude mω2R will act at A on the particle which can
be treated as constant from A to B as L<<R.
4 . 2 4 | Forces and Laws of Motion

∴ Acceleration of the particle along AB with respect to the cabin a = ω2R (constant)
1 1 2L
Therefore, required time “t” is given by s= ut + at2 ⇒ L= 0 + × ω2Rt2 ⇒ t=
2 2 ω2R

19.1 Applications of Centripetal Force

19.1.1 Circular Turning of Roads


When vehicles go through turnings, we observe that they travel along a nearly circular arc. Naturally, there must be
some force which will produce the required centripetal acceleration. However, if the vehicles travel in a horizontal
circular path, then this resultant force is also horizontal. The necessary centripetal force, which we are discussing
about, is being provided to the vehicles by any one or combination of the following three ways:
(a) By friction only, (b) By banking of roads only, or (c) By both friction and banking of roads
In real life, the necessary centripetal force is provided by both friction and banking of roads. Now, let us write
equations of motion in each of these three cases separately and find out what the constraints in each case are.
(a) By Friction only: Suppose a car of mass m is moving at a speed v in a horizontal circular arc of radius r. In this
case, therefore, the necessary centripetal force to the car will be provided by force of friction, f, acting toward
mv 2
the center. Thus, f = Further, limiting value of f is µN Or fL = µ ; N = µmg (N=mg)
r
mv 2 mv 2 v2
Therefore, for a safe turn without sliding ≤ fL or ≤ µmg or µ≥ or v ≤ µrg
r r rg
Here, however, two situations may arise. If µ and r are known to us, then the speed of the vehicle should not
2
exceed and if v and r are known to us, then the coefficient of friction should be greater than v .
rg

PLANCESS CONCEPTS

You might have seen that if the speed of a car is too high, the car starts skidding outward with the radius
of the circle increased of the necessary centripetal force is reduced.
1
centripetal force ∝ Anurag Saraf (JEE 2011, AIR 71)
r

(b) By Banking of Roads only: It is a common fact that friction is not always
reliable particularly at circular turns when in high speeds and where sharp N
turns are also involved. To avoid dependence on friction, the roads are
banked at the turn in such a way that the outer part of the road is somewhat mv
2
G
lifted compared to the inner part. r
Now, by applying Newton’s second law along the radius and the first law in
mv 2 
the vertical direction, we obtain
= N sin θ = and N cos θ mg
r
Thus, from the above two equations, we obtain mg
v2 … (i) or Figure: 4.56
tan θ = =v rgtan θ … (ii)
rg

PLANCESS CONCEPTS

This is the speed at which a car does not slide down even if a track is smooth. If the track is smooth and
speed is less than rgtanθ , then the vehicle will move down so that r gets decreased and if speed is
more than this, then the vehicle will move up.
Vijay Senapathi (JEE 2011, AIR 71)
P hysi cs | 4.25

(c) By Both Friction and Banking of Road: If a vehicle is moving on a circular road which is both rough and
banked, then three forces may act on the vehicle, and out of these the first force, i.e., weight (mg) is fixed
both in magnitude and direction. The direction of the second force, i.e., normal reaction N is also fixed
(perpendicular to road) while the direction of the third force, i.e., friction f can be either inward or outward
while its magnitude can be varied up to a maximum limit ( fL = µN ). Therefore, the magnitude of normal
reaction, N and direction plus magnitude of friction, f, are so adjusted that the resultant of the three forces
2
mentioned above is mv towards the center.
r

N N
f mv
2 f
 
r
 
Nsin fcos
mg mg
 

(a) Figure: 4.57 (b)

Therefore, magnitude of N and direction plus magnitude of friction mainly depends on the speed of vehicle,
v. Thus, the situation varies on a case-to-case basis even though we can observe the following scenarios:
(i) Friction f is outward if the vehicle is at rest or v = 0. Because, in this case, the component of weight mg sin θ
is balanced by f.
(ii) Friction f is inward if v > rgtan θ

(iii) Friction f is outward if v < rgtan θ

(iv) Friction f is zero if =v rgtan θ

Let us now observe how the force of friction and normal reaction changes as speed is gradually increased.
In Figure 4.57 (a): When the car is at rest, then the force of friction is upward. However, we can resolve the forces
in any two mutually perpendicular directions. Let us resolve them in both horizontal and vertical directions.

∑ FH= 0 ∴ Nsin θ − f cos θ= 0  ... (i)

∑ F=
v θ mg 
0 ∴ Nco s θ + f sin= ... (ii)
2
In Figure 4.57 (b): Now the car is given a small speed v, so that a centripetal force mv is now required in
r
horizontal
mv 2
direction toward the center. Therefore, Eq. (i) will now become Nsin θ − f cos θ =
r
mv 2
Or we can say, in case (a) Nsinθ and f cos θ were equal while in case (b) their difference is . This can occur in
r
any of the following three ways:
(i) N increases while f remains same
(ii) N remains same while f decreases
(iii) N increases and f decreases
But only the third case is possible, i.e., N will increase but f will decrease. This is because Eq. (ii), Nco s θ + f sin θ = mg
is still has to be valid.
Therefore, to keep Nco s θ + f sin θ to be constant (=mg), N should increase but f should decrease (as θ =constant).
Now, as speed goes on increasing, force of friction first decreases But becomes zero at v= rgtanθ and then
reverses it direction. Let us show an example which illustrates this theory.
4 . 2 6 | Forces and Laws of Motion

Illustration 26: A turn of radius 20 m is banked for the vehicle of mass y


200 kg moving at a speed of 10 m/s. Find the direction and magnitude N
f
of frictional force acting on a vehicle if it moves with a speed (a) 5 m/s 
and (b) 15 m/s. Assume that friction is sufficient to prevent slipping.  x
(g = 10 m/s2) (JEE ADVANCED) 
mg
Sol: At the correct speed for which the road is banked, the self-adjusting

static friction acting on the vehicle is zero. When the speed decreases,
the vehicle has a tendency to slip downwards, thus the static friction acts Figure: 4.58
upwards. When the speed increases, the vehicle has a tendency to slip
upwards, thus static friction acts downwards.
(a) the turn is banked for speed v = 10 m/s
v2 (10)2 1
Therefore, tan=
θ = = Now, as the speed is
rg (20)(10) 2
decreased, force of friction f acts upward.
mv 2
∑ FX = ∑ Fy =0
N
Using the equations and we obtain
r 
2
mv
Nsin θ − f cos θ =  ... (i) 
r f
and Nco s θ + f sin θ = mg  … (ii) mg
1 1 
Now, by substituting, θ =tan   , v = 5 m/s, m = 200 kg and r = 20 m,

in the above equations, 2 Figure: 4.59

we obtain f = 300 5N (outward)


(b) In the second case, force of friction f will act downward.
mv 2
Using ∑ FX = r
and ∑ Fy =0 we obtain
2
mv … (iii)
Nsin θ + f cos θ =
r
O
Nco s θ − f sin θ = mg … (iv)
1
Substituting, θ =tan−1   , v = 15 m/s, m = 200 kg and r = 20 m, in the above
equations, we obtain f =2500
 5 N (downward).
h T
Tcos
19.1.2 Conical Pendulum
A conical pendulum consists of a string OA whose upper end O is fixed and N A
a bob is tied at the free end. When a horizontal push is given to the bob Tsin
by drawing aside and let it describe a horizontal circle with uniform angular
velocity ω in such way that the string makes an angle θ with the vertical, then mg
the string traces the surface of a cone of semi-vertical angle θ . It is called Figure: 4.60
a conical pendulum. Let us assume that T be the tension in string, l be the
length and r be the radius of the horizontal circle described. Now, the vertical
component of tension balances the weight, whereas the horizontal component
supplies the centripetal force.
rω2
T cos θ =mg ; T sin θ =mrω2 ∴ tan θ =
g
gtan θ 2π
ω= ; r= lsinθ ∴ ω=
r T
2π gtan θ l cos θ h
T being the period, i.e., time for one revolution ∴ = ; T=
2π 2π
= where
= h l cos θ
T lsin θ g g
P hysi cs | 4.27

PLANCESS CONCEPTS

This is similar to the case, when necessary centripetal force to vehicles is provided by the property of
banking. The only difference here is that the normal reaction is being replaced by the tension.
Yashwanth Sandupatla (JEE 2012, AIR 821)

19.1.3 Death Well or Rotor


In the case of a ‘death well’, a person drives a bicycle on a vertical r
f
surface of a large wooden well, while in the case of rotor at a certain N
angular speed of rotor a person hangs resting against the wall without f
any support from the bottom. In the death well, all walls are at rest and N r
mg
the person revolves while in case of the rotor the person is at rest but mg
the walls rotate.
In both the cases, friction balances the weight of the person while
reaction provides centripetal force for circular motion, i.e., f = mg and (A) (B)
mv 2
= mrω2
Death well Rotor
N=
r
Figure: 4.61
A cyclist on the bend of a road: In the Figure 4.62, = F N2 + f 2
When the cyclist is inclined to the center of the rounding
F
along its path, the resultant N, f and mg are directed
horizontally to the center of the circular path of the cycle. This
resultant force naturally imparts a centripetal acceleration N G G mv
2

to the cyclist. The resultants of N and f, i.e., F should pass   r


through G, the center of gravity of the cyclist (for complete 
equilibrium, rotational as well as translational). Hence,
f
f mv 2
tan θ = where f = and N = mg mg mg
N r Figure: 4.62
v2
∴ tan θ =
rg

20. CENTRIFUGAL FORCE


We know that Newton’s laws are valid only in inertial frames. This is because in non-inertial frames a pseudoforce
 
−ma has to be applied on a particle of mass m ( a =acceleration of frame of reference). After applying the pseudo
force, then one can apply Newton’s laws in their usual form. Now, suppose that a frame of reference is rotating
with constant angular velocity ω in a circle of radius ‘r’. Then, it will become a non-inertial frame of acceleration
θ toward the center. Now, if we observe an object of mass ‘m’ from this frame then obviously a pseudo force of
2
magnitude mrω will have to be applied to this object in a direction away from the center. This pseudo force is
what we call as centrifugal force. After applying this force, we can apply Newton’s laws in their usual form. The
following example will illustrate this concept clearly.

Illustration 27: A particle of mass m is placed over a horizontal circular table rotating with an angular velocity ω
about a vertical axis passing through its center. The distance of the object from the axis is r. Based on the above, find
the force of friction f between the particle and the table. (JEE ADVANCED)

Sol: The particle is stationary in the rotating reference frame rigidly fixed to the rotating table. In the list of all the
forces acting on the particle, include the centrifugal force (pseudo force) acting on the particle radially outwards.
Let us solve this problem from both the frames. The one is a frame fixed on the ground while the other is a frame
fixed on the table itself.
4 . 2 8 | Forces and Laws of Motion

 N

N = normal reaction
r f mg = weight
f = force of function
N

Figure: 4.63 mg
2
f Pseudo force =mr
From frame of reference fixed on ground (inertial)
Here, N will balance its weight and force of friction, whereas f will provide the
necessary centripetal force. mg
Figure: 4.64
Thus, f mrω2
=

From frame of reference fixed on table itself (non-inertial)


In the FBD of particle with respect to table, in addition to the above three
forces (N, mg and f) a pseudo force of magnitude mrω2 will have to be
applied in a direction away from the center. But one significant point
here is that in this frame the particle is in equilibrium, i.e., N will balance
its weight in vertical direction, whereas f will balance the pseudo force in
horizontal direction. = f mrω2
Thus, we observe that ‘f’ equals mrω2 from both the frames. Now, let us
work up few more examples of circular motion.

Illustration 28: A simple pendulum is constructed by attaching a bob of mass m to a string of length L fixed at the upper end.
The bob oscillates in a vertical circle. It is found that the speed of the bob is v when the string makes an angle α with the vertical.
Based on the above, find the tension in the string and the magnitude of net force on the bob at that instant.
(JEE MAIN)

Sol: Apply Newton’s second law on the bob in two perpendicular directions. O
One along the string and the other along the tangent to its circular path, i.e.
along the perpendicular to the string.

The forces acting on the bob are:
T
(a) Tension, T and
(b) Weight, mg

As observe that the bob moves in a circle of radius L with center at O, it is
mg sin  mg cos 
mv 2
imperative that a centripetal force of magnitude is required toward O. mg
L
However, this force will be provided by the resultant of T and mg cos α . Figure: 4.65
Thus,
mv 2  v2 
(i) T − mgcos α = or T m  gcos α +
= 
L  L 

2 4
  mv 2  2 2 v
(ii) =
Fnet 2
(mgsin α ) +   = m g sin α + 2
 L 
  L
P hysi cs | 4.29

Illustration 29: Suppose that a hemispherical bowl of radius R is rotating about its axis of symmetry which is kept
vertical. Further, a small ball kept in the bowl rotates with the bowl but without slipping on its surface. If the surface
of the bowl is smooth and the angle made by the radius through the ball with the vertical is 5gR , then find the
angular speed at which the bowl is rotating. (JEE MAIN)
Sol: The horizontal component of normal contact force acting on the ball 
will provide the necessary centripetal acceleration to move in a circular
path. The vertical component of normal contact force acting on the ball will
balance its weight. R

Let us assume that ω be the angular speed of rotation of the bowl. Now, the N
Ar
two forces acting on the ball are r
(a) Normal reaction, N and mg
(b) Weight, mg Figure: 4.66

We know that the ball is rotating in a circle of radius r (=R sinα ) with center
A at an angular speed ω .
Thus, Nsin α= mrω2= mRω2 sin α  … (i)
and Ncos α =mg  … (ii)
1 ω2R g
Thus, by dividing Eq. (i) by (ii), we obtain = ∴ ω=
cos α g R cos α

21. EFFECT OF EARTH’S ROTATION ON CIRCULAR WEIGHT


We are very familiar with the fact that the earth rotates about its own axis at an angular speed of one revolution
for every 24 hours. We also know that the line joining the north and the south poles is the axis of its rotation, and
every point on the earth moves in a circular path. Further, a point at the equator moves in a circle of radius equal to
that of the radius of the earth and the center of the circle is the same as the center of the earth. However, for any
other point on the earth, the circle of rotation is smaller than this. Now, consider a place P on the earth.
If we drop a perpendicular PC from P to the axis SN, then the place P N
rotates in a circle with the center at C, and the radius of this circle is CP. Now, the
angle between the axis SN and the radius OP through P is called the colatitude
of the place P. We have= CP OP sin θ or, = r R sin θ where R is the radius of the
earth. C P

However, if we work from the frame of reference of the earth, then we shall have O
R
to assume the existence of the pseudoforces. In particular, a centrifugal force,
mω2r has to be assumed on any particle of mass m placed at P. Here ω is the
angular speed of the earth. In contrast, if we discuss the equilibrium of bodies
at rest in the earth’s frame, then no other pseudo force is needed. Let us now S
consider a heavy particle of mass m that is suspended through a string from
the ceiling of a laboratory at colatitude θ . Observing from the earth’s frame it Figure: 4.67
appears that the particle is in equilibrium and hence the forces acting on it are
(a) Gravitational attraction, mg, toward the center of the earth, i.e., vertically downward,
(b) Centrifugal force, mω2r , toward CP, and
(c) The tension in the string, T, along the string.
As the particle is in equilibrium (in the frame of earth), the three forces on the particle therefore should add up to
zero.
The resultant of mg and

mω2r = (mg)2 + (mω2r)2 + 2(mg)(mω2r)cos(90° + θ)


4 . 3 0 | Forces and Laws of Motion

N
= m g2 + ω4R 2 sin2 θ − 2g ω2 Rsin2 θ =mg’

g'
where = g2 + ω4R 2 sin2 θ − 2g ω2 Rsin2 θ 2
C mr
Also, it is obvious that the direction of the resultant makes an angle α with  
P

the vertical OP, where O mg


mg’

mω2r sin(90° + θ) ω2 R sin θ cos θ


tan α = =
mg + mω2r cos(90° + θ) g − ω2 R sin2 θ
S
From the above, as the three forces acting on the particle must add up to zero,
we understand that the force of tension must be both equal and opposite to Figure: 4.68
the resultant of the rest two. Therefore, the magnitude of the tension in the
string must be mg′ and the direction of the string hence should make an angle α with the true vertical.
The direction of g′ is in the apparent vertical direction. This is because a plumb line stays in this direction only.
However, the walls of the buildings are constructed by making them parallel to g′ and not to g. Further, the water
surface placed at rest is perpendicular to g′.
We observe further that the magnitude of g′ is also different from g. As 2g > ω2 R , it is clear from the above
equation that g′ < g. Here, one way of measuring the weight of the body is to suspend it by a string and hence
find the tension in the string. This is because the tension itself is taken as a representative measure of the weight.
As T = mg′, the weight so observed is clearly less than the true weight, mg. We call this quantity as the apparent
weight. Similarly, if a person stands on the platform of a weighing machine, the platform then exerts a normal force
N which is equal to mg′. The reading of the weighing machine responds to the force exerted on it and hence the
weight recorded is the apparent weight, mg′ of the person. At the equator, we know that θ = 90° and hence Eq.
(7.14) gives
2 2
g'
= g2 − 2gω2R + ω4 R 2 = g − ω R Or, mg'= mg − mω R .
This value, however, can be obtained in a more straightforward way. At the equator, we know that mω2R is directly
opposite to mg and the resultant is simply mg – mω2R . Also, it is clear that this resultant is towards the center of
the earth so that at the equator the plumb line is along the true vertical.
At poles, invariably θ =0 and the first equation gives g′=g and by the next equation we know that α = 0. Thus, at
the poles there is no apparent change in g. This is basically because of the fact that the poles do not rotate and
hence the effect of earth’s rotation is negligible there.

Illustration 30: A body weighs 98 N on a spring balance at the North Pole. What will be its weight recorded on the
same scale if it is shifted to the equator? Use g=GM/R2=9.8 m/s2 and the radius of the earth R=6400 km.
(JEE MAIN)
Sol: At the equator due to the rotation of earth the body experiences centrifugal force, directed away from the
center of earth. So the apparent weight at equator is less than the actual weight at the poles.
At poles, we know that the apparent weight is the same as the true weight. Thus, 98 N = mg = m ( 9.8 m/s2)
or, m = 10 kg. At the equator, however, the apparent weight is mg′ = mg – mω2 R .
2π rad
It is given that the radius of the earth is 6400 km and hence the angular speed is=
ω = 7.27 × 10−5 rad / s
24 × 60 × 60 s
Thus, mg′= 98N – (10 kg) (7.27 × 10−5 s−1 )2 (6400 km) = 97.66 N.
P hysi cs | 4.31

PROBLEM SOLVING TACTICS

Step 1: First, construct a big schematic diagram of the physical situation. m = 30 kg


v
Then, while reading and rereading the problem statement construct
your diagram accordingly including every available information from the
P = 100 N
statement on the diagram. Thereafter, if applicable, attach appropriate
symbols to each important parameter in the problem irrespective of the  = 20° k= 0.20
fact that whether the value of the parameter is known or not. Eventually,
make straight lines straight, parallel lines parallel, perpendicular lines Figure: 4.69
perpendicular, etc., to the best of your ability in order to avoid confusion
later on.
Example at right. (Note how the following items recorded from the statement of the problem into specific items on
the drawing: “... 30 kg suitcase ... moves upward ... 20 degree incline ... applied horizontal force of 100 N ... kinetic
coefficient of friction ... is 0.20. “)
Step 2: Select a «system» to which you intend to apply Newton›s second law. In some problems, however, there
may be more than one candidate for the “system.” You may not choose the best one always the first time. That
should not be a case for worry; just choose another one and do it again.
Example: We will select the “suitcase” as our system because it is the thing to which many obvious forces are being
applied and it is the thing whose acceleration we want to find.
Step 3: Identify all the forces acting on «the system.» You can do this by drawing a dotted line around the system chosen
in step 2 and identifying all physical objects that come in contact with the system. Each of these will exert a definite force
on the system. Then, look for «field» forces—those forces that act without touching through the intermediary of a field
of some sort. We know that in introductory mechanics the only “field” force is the force of gravity. It is a force exerted
by the earth (or some other very massive body) on the system through the intermediary of the gravitational field. 
Important! It should be understood that every force on a system is exerted by some physical object exterior to the
system. If you cannot identify that object and the method of interaction (contact or field), then the force DOES NOT
EXIST! Listed here are the some commonly encountered forces and some tips on dealing with them:
(a) Ropes or strings:  These exert «tension» forces on the system in question. They are always directed  away
from the system and along the direction of extension of the rope or string used.
(b) Contacts with surfaces:  We generally split up the force due to contact with a surface into two components
called the “normal”—meaning “perpendicular”—force and the “frictional” force. The normal force is generally a
“push” type of force directed toward the system, unless the surface is sticky enabling it to exert a «pull» type of
force. In contrast, the frictional force is parallel to the surface, opposes motion or potential motion (i.e., a system on
the verge of «slipping») and is often assumed to be related to the normal force through a «coefficient of friction.»
Readers may kindly refer to the discussion on the topic, “coefficient of friction” in this regard.
(c) Hinges or Pins:  These exert forces of  arbitrary  magnitude and direction as n a (?)
required so as to ensure that the point of attachment remains stationary.
f
(d) General pushes or pulls:  If a working problem  specifies  that some object
P
is being pushed or pulled in some direction, then you may have to assume that
the force specified is being exerted by some physical object. Therefore, it is very W
important that you do not forget to include the same.
Figure: 4.70
(e) Air resistance: Air may is not visible, but it is very likely that it does establish
a physical contact with your system. Quite often we neglect air resistance because n a (?)
its effects are deemed negligible. However, if a problem specifies a certain amount
of air resistance is involved or tells you that the air resistance depends in some way +y
f
on velocity or other parameters, then do not forget to include it. P
+x
(f) Gravitational force: We are aware of the fact that the gravitational W
force—commonly called the “weight” of the system—is the only force that
acts  without  being in physical contact with the system (at least until you learn Figure: 4.71
4 . 3 2 | Forces and Laws of Motion

about electric and magnetic forces later on.) Generally, it acts in the downward
direction (by definition!) and is equal to the mass of the system times the local
gravitational field strength g—commonly, but misleadingly called “the acceleration
due to gravity.”

The example at right. Here, we find two objects in contact with the system—one Figure: 4.72
being the “surface” and other one “pusher.” Thus, we find a total of four forces—
the normal force, the frictional force (from the surface), the push (from the
pusher), and the weight (due to the only force—so far—that acts without needing
to touch—gravity.)

Step 4: Draw an “FBD.” Now, the system may be represented by a simple circle or square; however, we want to
focus our attention on the forces on and the resulting acceleration of the system. Now, draw each force with its tail
at the surface of the system extending in the proper direction. Further, include the acceleration vector as well, but
distinguish it from the force vectors by drawing it in a different way.
Example at the right. In this example, note that the normal force is directed perpendicular to the surface (not shown
in the FBD), the frictional force is directed opposite to the direction that the system slips with respect to the surface,
the push is in its given direction, and the weight is directed “down.” We also show the acceleration as a different-
looking vector that is directed upward along the plane, but we do not know this for certain; it may be directed
downward along the incline. To keep us remind ourselves of this fact, we put a “(?)” next to the acceleration vector.

Step 5: Now, pick a coordinate system and hence determine the angles that the forces and accelerations make
with the coordinate axes. It is usually “clever” and preferable to pick a coordinate system that minimizes the
number of unknown vectors that will have to be broken down into components. The answers you obtain thereafter
must  and  will  be independent of your choice of coordinate system, but clever choices will help us to arrive at
equations that are more easy to solve. However, you may need to do some geometrical work on another sheet of
paper to figure out how the angles are related to those given in the problem statement.
Example at right. In this example, we have chosen a coordinate system that requires us to determine the components
of only the weight and the push—the two forces about which we know a lot. These two forces lie at the angle theta
(given in the problem statement as 20 degrees) from one of the axis directions.

Step 6: Now, write Newton’s second law. This law is the basic physical principle you are applying; i.e., the “starting
 
point” for your calculations. Just proceed to do it! Example:  ∑ F = ma
Step 7: Thence, apply the basic equation to this problem. Now, simply write what the “sum
  of forces”
 is in this case.
If the acceleration is zero, then use that fact to simplify the equation too. Example: n + f + P + W = ma

Step 8: Now, continue by writing the component equations. This is simply a matter of recognizing that every vector
equation is shorthand for two (or, more generally, three) scalar equations. Then, simply rewrite the vector equation
for each component direction with each vector quantity rewritten as the corresponding component. Examples: 
x : nx + fx + Px + Wx =
max ;and ny + fy + Py + Wy =
may

Step 9: Now, determine what each component is in terms of the vector magnitude and trigonometric functions of
the associated angles. In this step, it is imperative that we explicitly indicate the signs of the vector components.
This is also a good time to explicitly substitute “mg” for “W” if you really happen to know the mass of the system.
Example: Notice that the normal force is purely in the +y direction, the frictional force is purely in the –x direction, the
push has a positive x-component and a negative y-component, the weight has negative x- and y-components, and the
assumed acceleration is purely in the +x direction. Thus, we have: x : 0 + ( − f) + ( + Pcos θ) + ( − mgsin θ)= m( + a)
y : ( + n) + 0 + ( − Psin θ) + ( − mgcos θ) =m(0)

Step 10: To conclude this procedure, as a final step, simplify the resulting equations and figure out where to go
from here. This is the end of “the method.” Example:  P cos θ − f − mgsin θ = ma n − P sin θ − mgcos θ = 0
P hysi cs | 4.33

FORMULAE SHEET
 
 (a) =
F1 Fcos
= θ component of F along AC F2 Fsin=
= θ component of F perpendicular to AC


F B
F₂

Figure: 4.75
F₁ A C
Figure: 4.73
dp d(mv)
F =
 (b) =
dt dt
 T
    Fnet
 (c) ∑
= F F=
net ma or=a
m

∑ Fx=
FP =ma
0 ⇒ T sin θ= ma
∑ Fy= 0 ⇒ T cos θ= mg mg
a Figure: 4.74
θ =tan−1  
g
 
 (d) Impulse = change in momentum F∆=
t mv f − mVo
fmax
 (e) µ = fmax A
R
Force of fK
Friction
fS

O Force Applied, F
A
 (f) fmax = flimiting = µsR R


fm O

Figure: 4.76

fmax
 (g) Angle of Friction: tan λ = =µ or
= λ tan−1 (µ )
R
 (h) Pseudo force: F = –ma; where m = mass of the object, a = acceleration of the reference frame

 (i) A particle in circular motion may have two types of velocities as listed hereunder.
(i) Linear velocity v and
(ii) Angular velocity ω .These two are related by the equation v = R ω (R = radius of circular path)

 ( j) Acceleration of a particle in a circular motion may have two components as listed hereunder.

(i) Tangential component ( at ) and


(ii) Normal or radial component ( an ).
As the name suggests, the tangential component is tangential to the circular path, given by at = rate of
change of speed
4 . 3 4 | Forces and Laws of Motion


dv dv dω
 (k) = = Rα where α = angular acceleration = rate of change of angular velocity =
dt dt dt
The normal or radial component, also known as centripetal acceleration is toward the center and is given by
v2
an = Rω2 =
R
 (l) Net acceleration of a particle is the resultant of two perpendicular components, an and at . Hence,
= a an2 + a2t

 (m) Tangential component at is responsible for change of speed of a particle. This can be positive, negative or
zero, depending upon the situation whether the speed of the particle is increasing, decreasing or remains
constant.


a
 (n) In general, in any curvilinear motion, direction of instantaneous velocity is
tangential to the path, while acceleration may assume any direction. If we 
resolve the acceleration in two normal directions, one parallel to velocity and


v
another perpendicular to velocity, then the first component is at while the
other is an .
  Figure: 4.77

  a.v dv d v
Thus, at = component of a along v = acos θ = = = = rate of change of speed.
v dt dt
 v2
Further, an = component of ∴ a1x = a2x = a3x perpendicular to v = a2 − a2t =
R
Here, v is the speed of the particle at that instant and R is called the radius of curvature to the curvilinear path
at that point.

 (o) In at = a cos θ , if θ is acute, at will be positive and speed increases. However, if θ is obtuse at will be negative

and speed will decrease. If θ is 90° , at is zero and speed will remain constant.

 (p) Now, depending upon the value of at , circular motion may be of three types as listed hereunder.

(i) Uniform circular motion in which speed remains constant or at = 0.


(ii) Circular motion of increasing speed, in which at is positive.

at
v v at
v

a
an a an a an

Figure: 4.78
(iii) Circular motion of decreasing speed, in which at is negative.
P hysi cs | 4.35

Solved Examples

JEE Main/Boards Sol: In the first case, the blocks A and B move together.
The friction force will be exerted on the bottom surface
Example 1: A spherical shell is of B. In the second case only block B moves. The friction
resting against the vertical wall force will be exerted both on bottom and top surface
which makes an angle 30° with of block B.

30°
the vertical as shown in the
Figure. Determine the normal Let us consider free diagrams of A and B as two separate
reaction at the wall and tension systems shown as follows:
in the string. NA NA

Sol: Draw the FBD of the sphere. m=5kg


Resolve the forces in horizontal F NA
B
and vertical directions. Apply (NA+NB)
A
Newton’s first law along the
horizontal and vertical directions. NB
(mA+mB)g mAg
T
T cos 30°

A
30°

F B
T sin 30° N

(a) =
WA m
=Ag 4 N ; WB m
= =Bg 8 N

mg NB =NA + mB g =(mA + mB )g = 4 + 8 = 12N


F=
µ × 12 =
0.25 × 12 =
3N
FBD of the sphere is provided. Since sphere is in
equilibrium, hence, (b) F =µ(NA + NB ) because B is sliding over horizontal
N – T sin30 = 0  … (i) rough surface under it and B will be sliding under A also.
= µ(mA + mB ) + µ mA g =
µ(2mA + mB )g = µ(2 WA + WB )
and ⇒ T cos30 =
mg  ...(ii) ... (ii)
= 0.25(8 + =
8) 4N
mg 5 × 10 × 2  100 

= T = =   N.
cos30 3  3 
Example 3: Two blocks each
100 1 50 having mass of 20 kg rest on
from (i) N= T sin30= × = N.
3 2 3 frictionless surfaces as shown
in the Figure. Assume that the A
Example 2: The Figure below shows blocks A and B pulleys to be light and
weighing 4N and 8N, respectively and the coefficient frictionless. Now, find (a) the B
of sliding friction between any two surfaces is 0.25. Find time required for the block A to
the force necessary to drag the block B to the left with move 1 m down the plane, starting from rest and (b) the
constant velocity in all the cases when (a) A is kept over tension in the cord connecting the blocks.
B and (b) A is held firmly over B. Sol: For block A apply Newton’s second law of motion
along the inclined and for block B apply Newton’s
second law along the horizontal.
A A T
Both the blocks A and N

F B F B B are considered as two


independent systems. The MA
mAgsin
FBDs for the blocks A and B mAgcos
are shown in the Figure and  mAg
4 . 3 6 | Forces and Laws of Motion

Figure where T is tension in the string. = A1 + A2 = 4 + 1 = 5N.sec


mA gsin θ − T = mA a  ......(i) ... (i)
Momentum at t = 3 sec =5N sec (at t = 0, P = 0)
=N mA gcos θ  ...(ii) ... (ii) 5
mv = 5 ; v= = 2.5m / sec.
2
T = mB a  ...(iii) ... (iii)
Now, by adding equations (i) and (iii), we obtain Example 5: Three blocks of masses, and are connected
by inextensible strings passing over three massless
T a pulleys as shown in the Figure. The coefficient of friction
N between the masses and horizontal surfaces is µ.
mBAT Assume that M1 and M2 are sliding. Now, find
B (a) Relation between accelerations a1, a2 and a3

mBg (b) Tension T in the strings

mA gsin=
θ (mA + mB )gsin θ x₁
a₁ a₂
x₂

 mA   20  3 2
=⇒a =
  gsin θ =  (10)   3 m / s x₃ x₃
m
 A + mB   20 + 20   5 
1
1  2s  2  1
(a)s = at2 ; t =   =  2 ×  = 0.82 M₂a₃
2  a  3
a₁ a₂
(b) T = mB a = 20 × 3 = 60N.
N₁ N₂

Example 4: A body of mass = 2 kg starts from rest T T


f M₁ M₂ f
whose force time graph is shown in the following graph. x₁ x₂
A B
(a) What is momentum of the body at t = 4 seconds? M₁g
T
M₂g
T
(b) What is velocity of the body at t = 3 seconds? x₃ x₃

Sol: Area under the force-time graph and the time axis a₃
is equal to the change in momentum. x
M₃ y
F
(N)
M₃g
2N
t =3 t =4 Sol: To find the constraint relation between the
t =2 t (sec) acceleration of the blocks, measure the distances of
blocks from the stationary pulleys. Draw the FBD for
–3N each block. For M1 and M2 apply Newton’s second law
in horizontal direction. For M3 apply Newton’s second
(a) Area under the curve from t = 0→2 sec. law in vertical direction.

A1 = 2 × 2 = 4N.sec. Area from t = 2→3 sec. (a) Forces of friction f, tension T and reaction are marked
for the blocks M1 ,M2 and M3 .
1
A2 = × 1 × 2 = 1N.sec. Now, take the horizontal line AB as the reference line,
2
i.e., x-axis and vertically downward as y-axis.
Area from t = 3→4 sec.
If x1 , x2 and x3 are the lengths of the strings, then
1
A3 =− × 1 × 3 =−1.5N.sec.
2 x1 + x2 + 2x3 =
L where L is the constant length of the
string.
Therefore, the net impulse = 4 + 1 – 1.5 = 3.5 N sec
Now, differentiating twice, a1 + a2 + 2a3 =
0
=Pƒ impulse +=
Pi 3.5 + 0 = 3.5 N.s or kg.m / s
As a3 is increasing, a1 and a2 are decreasing.
(b) Impulse from t = 0→3 sec
P hysi cs | 4.37

Thus, the constraint relation shows that (a) Let L be the length of the string. Let x1 be the length
a1 + a2 =
2a3 of the vertical string and x2 be the length of each string
in the horizontal direction. The constraint relation for the
(b) The equations of motion are given as follows string of length L is x1 + 2x2 = L Now, by differentiating
twice, a1 + 2a2 = 0
For M3 , M3g − T − T =M3a3  ...(i) ... (i)
If a1 is +ve, then a2 is –ve,
For M1 , T − µM1g =M1a1  ....(ii) ... (ii)
a a1
For M2 , T − µM2g =M2a2  ...(iii) a1 − 2a2 =0 or a2 = =
... (iii) 2 2
a
a1 + a2 =
2a3 
...(iv) ... (iv) Let a1 = a be the acceleration of M and be the
2
acceleration of 2M.
2T
Dividing (i) by M3 , g − a3
= ∴ Mg – T = Ma  … (i)
M3
T a
=2T 
2M × ...(ii) ... (ii)
Dividing (ii) by M1 , − µg = a1 2
M1
Ma
=2 T Ma
= or T
T 2
Dividing (iii) by M2 , − µg = a2
M2 Now, by substituting for T in (i)
Using (iv), a1 + a2 =
2a3
Ma 3Ma 2g
Mg − = Ma; = Mg; ∴ a=
T T 4T 2 2 3
− µg + − µg
= 2g −
M1 M2 M3 Ma 2gM Mg
(b)=
T = =
2 2×3 3
 1 1 4 
T + +  = 2µg + 2g
= 2g µ + 1 5Mg
 M1 M2 M3  (c) Force on clamp
= C (2 T)2 + (T)
= 2
5T
= .
3
2g(µ + 1) (µ + 1)g
T= = Example 7: Masses M1 ,M2 and M3 are connected by
1 1 4 1 1 2
+ + + + strings of negligible mass which pass over massless and
M1 M2 M3 2M1 2M2 M3
frictionless pulleys P1 and P2 as shown in the Figure.
The masses move such that the portion of the string
Example 6: Masses M and 2M are connected through between P1 and P2 is parallel to the incline and the
pulleys A and B with strings as shown in the Figure. portion of the string between P2 and M3 is horizontal.
Assume that both the pulleys and the strings are light The masses M2 and M3 are 0.4 kg each and the coefficient
and all the surfaces are frictionless. of kinetic friction between masses and surfaces is 0.25.
(a) Find the acceleration of the block of mass M. The inclined plane makes an angle of 37° with the
horizontal, however, the mass M1 moves with uniform
(b) Find the tension in the string.
velocity downwards. Now, find
(c) Calculate the force exerted on the clamp.
(a) The tension in the horizontal portion of the string
Sol: To find the constraint relation between accelerations
of blocks M and 2M, measure all distances from the (b) The
= mass M1 g 9.8=(
ms−2 ,sin 37° 3 / 5 . )
fixed pulley A. Apply Newton’s second law in horizontal P1
direction for block 2M and Newton’s second law in T1
M2 M
vertical direction for block M. 2 gc
 = 0.25
T1 os

a₂ T2
x₂ Mg P2
M3
2M T2 mMg
B T M1 37°
T A a₁

Clamp T x₁ Sol: Apply Newton’s first law for each of the blocks as
the velocity of each block is constant.
M
Let T1 be the tension between M1 and M2 and T2 be
Mg the tension between M2 and M3 .
4 . 3 8 | Forces and Laws of Motion

Let µ be the coefficient of kinetic friction. Then Example 9: In the adjacent Figure, masses of A, B and
(a) T2 =
µM3g =
0.25 × 4.0 × 9.8 =
9.8N C are 1 kg, 3 kg and 2 kg, respectively. Find (a) the
acceleration of the system and (b) tension in the string.
(b) T1 = M1g 2
Neglect friction ( g = 10m/ s )
T1 − T2 − µM2gcos θ − M2gsin θ = 0

4 3
M1g − 9.8 − 0.25 × 4 × 9.8 × − 4 × 9.8 × =0
5 5
60° 30°
4 12
M1 × 9.8 = 9.8 + 9.8 × + 9.8 × =0
5 5 Sol: Draw the FBD of each block and apply Newton’s
4 12 21 second law along the incline plane for each block.
∴ M =1 + + = = 4.2 kg
1 5 5 5 (a) In this case, the net pulling force

Example 8: A rod AB rests with = mA gsin60° + mB gsin60° − mC gsin60°


Y
the end A on rough horizontal
3 3 1
ground and the end B against = (1)(10) + (3)(10) − (2)(10) = 24.64N
2 2 2
smooth vertical wall. The rod is of
uniform length and of weight W. If Therefore, the total mass being pulled = 1+3+2 = 6 kg
the rod is in equilibrium in the 21.17
30° A ∴ Acceleration of the system
= a = 4.1m / s2
position shown in the Figure, then O X 6
find: (b) For the tension in the string between A and B.
(a) Frictional force at A
T2 a
(b) Normal reaction at A
(c) Normal reaction at B. Y
mCg sin 60°

Sol: For translational equilibrium, B NB FBD of A is mA gsin60° − T1 =


= (mA )(a) mA (gsin60° − a)
the vector sum of all the forces
acting on the rod is zero. Take  3 
T1 (1)  10 ×
∴= − 4.1=
 4.56N
component of forces along NA  2 
W  
horizontal (x-axis) and vertical
(y-axis) direction. Sum of O
30° A
X (b) For the tension in the string between B and C. FBD of
fA
components of forces along the x C, T2 − mc gsin30° =mc a
and y axes will be zero. For rotational equilibrium, the   1 
net torque of all the forces acting on the rod relative to T2 2 3.53 + 10  =
∴ T2= mc (a + gsin30°) ∴=   18.2N
  2 
a fixed point (say O) is zero.
Let the length of the rod be 2l. Using the three conditions
of equilibrium, the anticlockwise moment is taken as Example 10: A small smooth ring of mass m is threaded
positive. on a light inextensible string of length 8L which has its
ends fixed at points in the same vertical line at distance
(i) ∑ Fx = 0 ∴ NB=− fA 0 or
= NB fA  … (i) 4L apart. The ring describes horizontal circles at constant
(ii) ∑ Fy =∴ 0 NA − W =0 or NA =W  … (ii) speed with both parts of the string taut and with the
lower portion of the string horizontal. Find the speed
(iii) ∑ τ0 = 0
of the ring and tension in the string. The ring is then
∴NA (2lcos30°) − NB (2lsin30°) − W(lcos30°) =0 tied at the midpoint of the string and made to perform
horizontal circles at constant speed of 3 gL . Find the
3 tension in each part of the string.
or 3 NA − NB − W= 0  … (iii)
2 Sol: Apply Newton’s second law in the radial direction
Solving the above three equations, we obtain in each case.
3 3 When the string passes through the ring, the tension
(a) fA = W (b) NA = W (c) NB = W
2 2 in the string is the same in both the parts. Also from
geometry,
P hysi cs | 4.39

BP=3L and AP=5L v2 (10)2


tan=
θ = = 1 ; θ= tan−1 (1)= 45°
4 Rg (10)(10)
T cos=
θ =T mg
5
 3 8 T1
Example 12: A car moves on a horizontal circular road
T + T sin=
θ T 1 + =  T  ... (i)
 5  5 a dv
2 2 of radius R, the speed increases at a rate = a . The
mv mv  … (ii) dt
= =
BP 3L frictional coefficient between the road and the tire is µ .
v2 Now, find the speed at which the car will skid.
Dividing (ii) by (i) =2 mAg sin 60°
3Lg Sol: The net acceleration of the car is the vector sum of
mg 5 centripetal acceleration and tangential acceleration. By
v = 6Lg From (i)
= T = mg . In the second case, Newton’s second law the net force of friction acting on
4 /5 4
the car is equal to mass multiplied by net acceleration.
ABP is an equilateral triangle
Here, at any time t, the speed of the car becomes V,
o o
T1 cos60= mg + T2 cos60 2
 v2 
mg the net acceleration in the plane road is   + a2 .
T1 −=
T2 = 2mg  … (iii) R 
cos60°  

mv 2 9mgL This acceleration is provided by the frictional force. At


T1 sin60° + T2 sin60
= ° = the moment car will slide
r 4L sin60°
2
9mg  v2 
( )
1/ 4
T=
1 + T2 = 3mg  ... (iv) m   + a = µMg ⇒ v= R µ g − a 
2 2 2 2 2
4 sin2 60°
R 
By solving equations (iii) and (iv), we have

5 1
=T1 =
2
mg; T2
2
mg JEE Advanced/Boards
Example 11: A car is moving in a circular horizontal Example 1: In the system of two pulleys connected as
track of radius 10 m with a constant speed of 10 m/s. A shown in the figure, M1 = 4M2 and mass M1 is 20 cm
plumb bob is suspended from roof by a light rigid rod above the ground, whereas mass M2 is lying on the
of length 1 m. What is the angle made by the rod with ground. Find the distance covered by when the system
the track? is released. (g = 10 m / s2 ).
Sol: In the reference frame of the car the bob will
experience a centrifugal force radially outwards. The A
vector sum of the three forces acting on the bob (the
weight, the tension and the centrifugal force) will be T T
equal to zero.
a B
The different forces acting on the bob are shown in T
the Figure. Resolving the force along the length and x₂
perpendicular to the rod, we have
2a
T M₁
20 cm M₂ x₁

2
mv
R Sol: To find the constraint relation between accelerations
 of M1 and M2, measure their distances from fixed pulley
A. Apply Newton’s second law in vertical direction for
each block.
mg M1 = 4M2

mv 2 mv 2 As M1 is heavier, itwill move down with acceleration a


mgcos θ + = sin θ T; mgsin
= θ cos θ and M2 will move upward with acceleration 2a because
R R
4 . 4 0 | Forces and Laws of Motion

the strings around the pulley B will move through half Let a, b and c be the respective accelerations of masses
the distance as compared to that of A. A (8 kg), B (4 kg), and C (16 kg) such that a and b
g are downward and c is upward. Let x1 and x2 be the
∴ M1g − 2T = M1a T − M2g =M2 × 2a or a = .
4 distances of strings from axial line passing through P
Therefore, the time taken for M1 to reach the ground at and Q to the blocks A and B, respectively. Let x3 be the
1 2 1 2 length of the string from the axial line PQ to the center
20 cm distance s =+
ut at =at of the movable pulley. If L is the length of the string,
2 2
then the constraint relation gives
2s 2 × 20 × 4  20 
or
= t = s
=  x1 + x2 + x3 + x3 =L =cons tant
a 10 × 100  100 
dx1 dx2 dx3
16 4 Differentiating + +2 0
=
or= = = 0.45 dt dt dt
100 10
d2 x1 d2 x2 d2 x3
Distance travelled by M2 Differentiating again + +2 0
=
1 1 10 dt2 dt2 dt2
x1 = × (2a) × t2 = × 2 × × (0.4)2 = 0.4m
2 2 4 or a + b −=
2c 0 or a=
+ b 2c.
Velocity of M2 after 0.4 seconds =v=u+2at
As tension T is equal in all the strings as it passes over
10 smooth pulleys, equations for the strings are as follows:
= 0 + 2× × 0.4 ; v = 2ms−1
4
Distance covered by M2 with velocity 2ms-1upwards 8g − T = 8a ... (i)
before coming to rest 2T − 16 g = 16c  ... (ii)
v2 (2)2 4g − T = 4b  ... (iii)
x=
2 = = 0.2 m
2g 2 × 10 a + b = 2c  ... (iv)
From Eqs. (ii) and (iv), we obtain
Distance covered by M2 before coming to rest
=x =x1 + x2 =0.4 + 0.2 =0.6 m 2 T − 16 g =8 × 2c =8a + 8b
By substituting a and b from Eqs (i) and (iii)
Example 2: Masses 4 kg and 8 kg are attached to the
free end of an inextensible light string passing over two 2 T − 16 g = 8g − T + 8g − 2T = 16g − 3T
fixed pulleys as shown in the Figure. The movable pulley 32
carries a mass of 16 kg. Assuming frictionless motion,= 5T 32g = or T g.
5
calculate the acceleration of the three masses.
Now, from Eq (i)
32 32 g
8a = 8g − T = 8g − = ;a =
P Q 5 5 5
From Eq (iii),
a b
x3 T T T 32g −12g
x3 4b = 4g − T = 4g − =
x1 x2 5 5

 g 3g 
 − 
8kg 4kg a+b 5 5  g
c= = ∴ c =−
A c B 2 2 5
16kg ∴ 16 kg and 8 kg go downward and 4 kg go upward.
C

Sol: To find the constraint relation between accelerations Example 3: A block of mass m is pulled up by means
of blocks measure their distances from the fixed pulleys. of a thread up and inclined plane forming an angle α
Apply Newton’s second law in vertical direction for each with the horizontal. The coefficient of friction is equal
block. to µ. Find the angle β which the thread must form with
the inclined plane for the tension of the thread to be
minimum. Also, find the value of minimum tension.
P hysi cs | 4.41

T which is negative.
 ∴ For minimum T,
= β tan−1 µ
The value of Tmin can be found by writing β in terms of
µ.
 1 1
= cos β = ,sin β
2
1+µ 1 + µ2
Sol: Draw the FBD of the block. Apply Newton’s first
law along the perpendicular to the inclined plane and mg ( sin α + µ cos α )
Newton’s second law along the inclined plane for the ∴ Tmin =
cos β + µ sin β
block.
T mg ( sin α + µ cos α ) mg ( sin α + µ cos α )
N = =
T cos  1 µ2 1 + µ2
+
T sin 
 1 + µ2 1 + µ2 1 + µ2

f mg ( sin α + µ cos α )
mg sin  =

 mg cos  1 + µ2
mg

When the body is just about to move up, the force of Example 4: Find the constraint relation in the Figure.
friction f is acting downward. If N is the normal reaction,
the force of friction f is equal to µN. Further, T and mg
can be resolved into rectangular components parallel y₃
and perpendicular to the inclined plane as shown in the
Figure. x₁ y₂
y₁
a₁
∴ T cos
= β mgsin α + µN ....(i)
N+ T=
sin β mgcos α=
or N mgcos α − T sin β m₁

Now, by substituting in Eq.(i), we obtain a₂

T cos
= β mgsin α + µmgcos α − µT sin β m₂

or T cos β + µT=
sin β mg(sin α + µ cos α ) Sol: To find the constraint relation between accelerations
of blocks measure their distances from stationary points.
mg(sin α + µ cos α ) For block m1 measure the distance from fixed pulley on
T=
cos β + µ sin β the wedge. For block m2 measure the distance from the
fixed roof.
For T to be minimum, cos β + µ sin β should be
maximum. Since length of each string is constant
d x1 + (y 2 − y 3 ) + y 2 =
c1  ......(i) ... (i)
∴ (cos β + µ sin β) = 0
dβ y1 − y 2 =
c2 ⇒ 2 y 1 − 2 y 2 =
2c2 ......(ii) ... (ii)
By adding (i) and (ii), we obtain
1+²  (x1 − y 3 + 2 y1 ) = c1 + 2c2

 x1 + 2 y1 = y 3 + c1 + 2c2 = c  .....(iii) ... (iii)


1 (since y 3 = constant )
d2 d2 x1 d2 y1
and (cos β + µ sin β) is negative. Differentiating (iii) w.r.t. t +2 0
=
2
dβ dt2 dt2
sin β −a1 + 2a2 =
0 ⇒ a1 =
2a2  .....(iv) ... (iv)
∴ − sin β + µ cos=
β 0 or =
µ = tan β
cos β
d Example 5: A pendulum is hanging from the ceiling of
Also, ( − sin β + µ cos β) = − cos β + µ sin β
dβ a car having acceleration a0 with respect to the ground.
Find the angle made by the string with the vertical.
4 . 4 2 | Forces and Laws of Motion

Sol: In the reference frame of the car the pendulum A v


bob will experience a pseudo force. For the bob to be in
equilibrium, the vector sum of all the forces acting on it
in the frame of the car should be zero.
B
The situation is shown in the Figure. Suppose that the
mass of the bob is m and string makes an angle θ with
C
the vertical. We shall now proceed based on the car
frame.
 This frame is non-inertial as it has acceleration
Sol: Find the constraint relation between the acceleration
a0 with respect to an inertial frame (the road). Hence, if
of the ring and the block. Measure the distances of ring
we use Newton’s second law we shall have to include a
and the block from the fixed pulley B.
pseudo force.
A A’
Now, consider the bob as the complete system.
P
Then, the forces acting on it are:
(a) T along the string, by the string B
(b) mg downward, by the earth
C
(c) m a0 towards left (pseudo force).
C’
T
(a) Suppose in a small time interval ∆t the ring is
displaced from A to A′ and the block from C to C′. Drop a
a₀ mag perpendicular A′P from A′ to AB. For small displacement
A′B = PB. Since the length of the string is constant,

Mg we have AB + BC = A'B + BC'


(b) or, AP + PB + BC = A'B + BC
(a) or, AP = B'C− BC = CC' (as A'B = PB)
The FBD is shown in the Figure. As the bob is at rest
(remember we are discussing the motion with respect to or, AA'cos θ =CC'
car) the force in (a), (b) and (c) should add to zero. Take AA'cos θ CC'
the X-axis along the forward horizontal direction and or, =
∆t ∆t
the Y-axis along the upward vertical direction.
or, (velocity of the ring) cos θ = (velocity of the block).
The components of the forces along the X-axis give
T sin=
θ − ma0 0 or,
= T sin θ ma0  .....(i) ... (i) (b) If the initial acceleration of the ring is a, that of the
block will be a cos θ . Let T be the tension in the string at
And the components along the Y-axis give
this instant. Consider the block as the system. The forces
T cos
= θ − mg 0 or,= T cos θ mg. ......(ii) acting on the block are
... (ii)
Dividing (i) by (ii) tan θ =a0 / g. (i) Mg downward due to earth, and
Thus, the string makes an angle (ii) T upward due to string.
tan−1 (a0 / g) with the vertical
equation of motion of the block is

Example 6: A smooth ring A of mass m can slide on a Mg-T = Macosθ  ... (ii)
fixed horizontal rod. A string tied to the ring passes over Now, consider the ring as the system. The forces acting
a fixed pulley B and carries a block C of mass M(= 2m) as on the ring are
shown in the Figure. At an instant the string between the
ring and pulley makes an angle θ with the rod. (a) Show (i) Mg downward due to gravity,
that, if the ring slides with a speed v, the block descends (ii) N upward due to the rod,
with speed v cos θ . (b) With what acceleration will the
ring start moving if the system is released from rest with (iii) T along the string due to string.
θ= 30° ? Taking components along the rod, the equation of
motion of the ring is
Tcos θ =ma. 
......(ii) ... (ii)
From (i) and (ii)
P hysi cs | 4.43

ma Motion of: The acceleration is a0, in the horizontal


Mg− = Macos θ
cos θ direction. The forces on m1 are
(a) T by the string (horizontal).
Mgcos θ
or, a= (b) m1 g by the earth (vertically downward) and
m + Mcos2 θ
(c) N by the table (vertically upward).
2
Putting θ= 30° , M = 2 m and g = 9.8 m / s ; In the horizontal direction, the equation is
2
therefore, A = 6.78 m / s T = m1a 0  ... (ii)
Motion of: Acceleration is a0 − a in the downward
Example 7: Three blocks of A direction. The forces on m2 are
masses m1, m2 and m3 are m₁
connected as shown in the (a) m2 g downward by the earth and
Figure. All the surfaces are B
(b) T′ = T/2 upward by the string.
frictionless and the string
and the pulleys are light. m₂ Thus m2g − T 2= m2 (a0 − a)  .....(iii) ... (iii)
Motion of m3 : Acceleration is ( a0 + a) in the downward
m₃
Find the acceleration of m1.
Sol: Draw the FBD of all the blocks and the pulley B. direction. The forces on are
The acceleration of pulley B is same in magnitude as the (a) m3 g downward by the earth and
acceleration of m1. In the frame of pulley B blocks m2
and m3 will experience pseudo forces. (b) T′= T/2 upward by the string.

Suppose the acceleration of m1 is a0 toward the right. Thus m3g − T 2= m3 (a0 + a)  .....(iv) ... (iv)
That will also be the downward acceleration of the We want to calculate a0 , so we shall eliminate T and a
pulley B because the string connecting m1 and B is from (ii), (iii), and (iv).
constant in length. This implies that the decrease in
the separation between m2 and B equals the increase Putting T from (ii) in (iii) and (iv),
in the separation between m3 and B. Therefore, the m2g − m1 a0 2 m1a0
upward acceleration of m2 with respect to B equals the a0 − a = = g−
m2 2m2
downward acceleration of m3 with respect to B. Let this
acceleration be a. m3g − m1 a0 2 m1a0
and a0 + a = = g−
The acceleration of with respect to the ground = a0 − a m3 2m3
(downward) and the acceleration of with respect to the
ground = a0 + a (downward). ma  1 1 
2g − 1 0
Adding, 2a0 =  + 
These accelerations will be used in Newton’s laws. Let 2  m2 m3 
the tension be T in the upper string and T′ in the lower
ma  1 1 
string. Consider the motion of the pulley B. or, g− 1 0 
a0 = + 
4  m2 m3 
The forces on this light pulley are
(a) T upward by the upper string and  m  1 1 
or, a0 1 + 1  +  = g
(b) 2T′ downward by the lower string.  4  m2 m3  

As the mass of the pulley is negligible, g


or, a0 =
2T′ – T = 0 Giving T′= T/2.  ... (i)  m1  1 1 
1 +  + 
a₀  4  m2 m3  
m₁ A

T
Example 8: All the surfaces shown in the figure. are
assumed to be frictionless. The block of mass m slides
B
on the prism which in turn slides backward on the
T’ horizontal surface. Find the acceleration of the smaller
m₂ block with respect to the prism.
a ₀-a m₃
a ₀+a
4 . 4 4 | Forces and Laws of Motion

m Example 9: A block of mass 0.4 kg is attached to a


vertical rotating spindle of length 1.6 m by two springs
M each of length 1 m of equal lengths as shown in the
Figure. The period of rotation is 1.2 seconds. Find the
tension in the springs.
Sol: Draw the FBD of both the blocks. In the reference R
frame of block M, the block m will experience pseudo C T2sin 
force. Apply Newton’s second law on the block m along T₂
the inclined plane and Newton’s first law along the 0.8m 1.0m
perpendicular to the inclined plane. For block M apply
0.4 kg
Newton’s second law along the horizontal. B
A
Let the acceleration of the prism be in the backward
T₁
direction. Consider the motion of the smaller block from
the frame of the prism. mg +T1sin 
T1cos +T2cos 
The forces on the block are
Sol: The sum of the horizontal components of tensions
(i) N normal force, in the two springs will provide the necessary centripetal
(ii) mg downward (gravity), and acceleration to the block. The vector sum of the vertical
components of tensions in the two springs will balance
(iii) m a0 forward(pseudo). the weight of the block.
N
N’ Let T1 and T2 be the tension in the springs when these
ma₀ springs subtend an angle θ each with the horizontal
N
direction. Let AB = R be the radius of circular path
mg
traversed by mass B in the horizontal plane.
mg
(a) (b) R =AB = 1 − (0.8)2 = 1 − 0.64 = 0.36 =0.6
The block slides down the plane. Components of the AC 0.8
forces parallel to the incline give sin=
θ = = 0.8 m
BC 1
ma0 cos θ + mgsin θ = ma
AB 0.6
or
= a a0 cos θ + gsin θ  ......(i) ... (i) cos=
θ = = 0.6 m
BC 1
Components of the force perpendicular to the incline 2π 2π π
Angular velocity = ω = = =
give N+ ma0=
sin θ mgcos θ.  ...(ii) ... (ii) T 1.2 0.6
Now, consider the motion of the prism from the lab Resolving T1 and T2 into rectangular component
frame. No pseudo force is needed as the frame used is
T2 sin
= θ T1 sin θ + mg
inertial. The forces acting now are
(i) Mg downward, (T1 + T2 )cos θ= mω2R .....(i)

(ii) N normal to the incline (by the block), and T2cos θ= mω2R − T1cos θ .....(ii)
(iii) N′ upward (by the horizontal surface). Multiply (i) by cos θ ,
Horizontal components give, T2 sin θ cos θ= T1 sin θ cos θ + mgcos θ
Nsin θ Ma0=
= or, N Ma0 sin θ.  .....(iii) ... (iii) Multiply (ii) by sin θ ,
Ma0 T2 sin θ cos θ =mω2 R sin θ − T1 sin θ cos θ
Replacing in (ii) + Ma0=
sin θ mgcos θ
sin θ Adding, 2T2 sin θ cos θ= mω2 R sin θ + mgcos θ
mgsin θ cos θ
or, a0 =
M + msin2 θ mω2 R sin θ mgcos θ
=2T2 +
sin θ cos θ sin θ cos θ
mgsin θ cos2 θ (M+ m)gsin θ
From (i),=a0 + gsin θ = 0.4 × π2 × 0.6 0.4 × 10 0.4 × π2
M + msin θ2 ω2 R mg
M + msin2 θ = m= + = +5
+ 0.8
cos θ sin θ (0.6)2 × 0.6 (0.6)2
P hysi cs | 4.45

2T
=2 10.97 +=
5 15.97 ∴=
T2 7.99 N ≈ 8 N = cos α + µ2 cos=
α cos α(1 + µ2 )
Subtracting the above mentioned terms, 1 + µ2 1 + µ2 1 + µ2
= = = = 1 + µ2
2
mω R mg sec α 1 + tan2 α 1+µ 2
2T1
= − = 10.97 −=
5 5.97
cos θ sin θ
mgsin θ + µ mgcos θ
T1 2.99 ≈ 3 N
= ∴ Tmin =
1 + µ2
Example 10: A block of mass m is pulled by means of
T
a thread up an inclined plane forming an angle θ with Example 11: A metal ring of
the horizontal. The coefficient of friction is µ . Find the mass m and radius R is placed
inclination of the thread with the horizontal so that on a smooth horizontal table A
tension in the thread is minimum. What is the value of and is set rotating about its own
/2
the minimum tension? axis in such a way that each part O
/2 C
of the ring moves with velocity
Sol: Draw the FBD of the block. Apply Newton’s second
v. Based on the above facts, find
law along the direction of the incline and Newton’s first B
the tension in the ring.
law along the direction perpendicular to the incline.
Let the mass moves up the plane with acceleration a. Sol: Each small part of the ring T
will experience a centrifugal
Writing the equation of motion, we obtain
force radially outwards. So the ring will tend to expand,
R + T=
sin α mgcos θ i.e. the radius and circumference will tend to increase. By
=R mgcos θ − T sin α virtue of its elasticity the ring will oppose its expansion.
 … (i)
So each part of the ring will experience a force of pull or
T cos α − mgsin θ − f = ma  ... (ii) tension from the other part.
where t is the force of friction Consider a small part ACB of the ring that subtends an
f = µ(mgcos θ − Tsin α )  … (iii) angle ∆θ at the center as shown in the Figure. Let the
tension in the ring be T.
Substituting the value of f from Eq (iii)
The forces on this elementary portion ACB are:
in Eq (ii) T cos α − mgsin θ − µ mgcos θ + µ Tsin α = ma
(i) Tension T by the part of the ring left to A
T(cos α + msin α=
) ma + mgsin θ + µ mgcos θ
(ii) Tension T by the part of the ring right to B
ma + mgsin θ + µ mgcos θ  … (iv)
T= (iii) Weight ( ∆m )g
cos α + µ sin α
(iv) Normal force N by the table
For T to be minimum (cos α + µ sin α ) should be
d As the elementary portion ACB moves in a circle of
maximum (cos α + µ sin α ) = 0 radius R at constant speed v, its acceleration toward the

( ∆m) v 2
d2 centre is Resolving the forces along the radius
(cos α + µ sin α ) = − ve R
dα 2 CO
2
d  ∆θ   ∆θ  v  … (i)
(cos α + µ sin α ) = − sin α + mcos α = 0 T cos  90° −  + T cos  90° −  = ∆m R
dα  2   2 
= tan−1 (µ )
µ tan α α
=
∆θ v2
2
d 2T sin = ∆m  ... (ii)
It can be shown that is negative. 2 R
dα2 Thus the length of the part ACB = R ∆θ . The mass
T will have minimum value when a = 0 and m
per unit length of the ring is
=α tan−1 (µ ) . From Eq. (iv) 2πR
mgsin θ + µ mgcos θ ∴ Mass of this portion ACB, ∆m =R∆θm m∆θ
Tmin = =
cos α + µ sin α 2πR 2π
cos α + µ sin=
α cos α + µ(µ cos α )
4 . 4 6 | Forces and Laws of Motion

Putting the value of ∆m in (ii)  


 ∆θ 
∆θ m∆θ v 2  2  mv 2
2T sin Since  is equal to 1; T =
=   2πR
2 2π R   sin  ∆θ   
 2 
   
 
 ∆θ 
2
mv  2 
∴T = 
2πR   ∆θ   
  sin    

   2  

JEE Main/Boards

Exercise 1 Q.12 State and explain Newton’s third law of motion.


Give at least two Illustrations.
Forces and Laws of Motion
Q.13 Discuss the apparent weight of a man in a lift/
Q.1 What is meant by law of inertia? elevator.

Q.2 State the laws of motion. Q.14 Two bodies of masses 11 kg and 11.5 kg are
connected by a long light string passing over a
Q.3 A cricket player lowers his hands while catching a smooth pulley. Calculate velocity and height ascended/
ball. Why? descended by each body at the end of 4s.
Y
Q.4 An impulsive force of 100N acts on a body for 1 s. Q
What is the change in its linear momentum?
X
O P
Q.5 A force of 5N changes the velocity of a body from
10 ms-1 to 20 ms-1 in 5 sec. How much force is required
to bring about the same change in 2 sec? Q.15 A rope of mass 0.5 kg is pulling a block of mass
10 kg under the action of force of 31.5 N. If the block
Q.6 State and explain Newton’s first law of motion. is resting on a smooth horizontal surface, calculate the
force of reaction exerted by the block on the rope.
Q.7 What are the three types of inertia? Give at least
two examples of each type. Q.16 Two bodies of masses 4 kg and 3 kg respectively
are connected by a light string passing over a smooth
Q.8 State and explain Newton’s first law of motion. frictionless pulley. Calculate the acceleration of the
Hence deduce the relation F = ma, where the symbols masses and tension in the string.
have their usual meaning.
Q.17 Two bodies whose masses are m1=50 kg and
Q.9 Define absolute and gravitational units of force. m2=50 kg are tied by a light string and are placed on
What are the dimensions of force? a frictionless horizontal surface. When m1 is pulled by
a force F, an acceleration of 5 ms-2 is produced in both
Q.10 Mention some of the consequences of the the bodies. Calculate the value of F. What is the tension
Newton’s second law of motion. in the string 1?

Q.11 Explain the term ‘impulse’. Discuss some of the


applications of this concept.
P hysi cs | 4.47

Q.18 See Figure where in Q.26 A cyclist is riding with a speed of 27 kmh-1. As he
1m
a mass of 6 kg is approaches a circular turn on the road of radius 80.0 m,
T₁
suspended by a rope 50N
he applies brakes and reduces his speed at a constant
P
of length 2 m from the rate of 0.5 ms-1 per second. Find the magnitude of the
ceiling. A force of 50N 1m net acceleration of the cyclist.
T₂
in the horizontal
direction is applied at W Q.27 A particle moves in a circle of radius 4.0 cm
 
midpoint P of the rope, clockwise at constant seed of 2 cms-1. If x and y
60N
as shown. What is the are unit acceleration vectors along X-axis and Y-axis,
(a)
angle the rope makes respectively, find the acceleration of the particle at the
with the vertical in instant half-way between P and Q in the Figure.
T₂
equilibrium?
T₁
(take g = 10 ms–2) W Q.28 A cyclist is riding with a speed of 36 kmh-1. As he
P 50N
Neglect mass of the approaches a circular turn on the road of radius 140 m,
rope. T₂ he applies brakes and reduces his speed at the constant
60N rate of 1 ms−2 . What is the magnitude and direction of
(b) (c) the net acceleration of the cyclist on the circular turn?

Q.19 A body builder exerts a force of 150N against a


bull worker and compresses it by 20 cm. Calculate the Exercise 2
spring constant of the spring in the bull worker.
Forces and Laws of Motion
Q.20 A lift of mass 2000 kg is supported by thick steel Single Correct Question
ropes. If maximum upward acceleration of the lift be
1.2 m / s2 , and the breaking stress for the ropes be Q.1 A block of mass 10kg is suspended through two
2.8 × 108 Nm−2 , what would be the minimum diameter light spring balances as shown in given Figure.
of the rope?

Q.21 A car of mass one metric ton travelling at 32 m/s


dashes into rear of a truck of mass 8000 kg moving in
the same direction with the velocity of 4 m/s. After the
collision, the car bounces backward with the velocity 8
m/s. What is the velocity of the truck after the impact?

Q.22 The force on a particle of mass 10 g is (10i+5j)N. 10kg

If it starts from rest, what would be its position at time (A) Both the scales will read 10 kg
t = 5s?
(B) Both the scales will read 5 kg.
Q.23 A projectile is fired vertically from the earth’s surface (C) The upper scales will read 10 kg and the lower zero.
with an initial velocity of 10 km/s. Neglecting atmospheric
retardation, how far above the surface of the earth would (D) The readings may be anything but their sum will be
it go? Take the earth’s radius as 6400 km. 10 kg

Q.24 Two balls of mass m each are hung side by side Q.2 A block is kept on the floor of an elevator at rest.
two long threads, each of length l. If the distance The elevator starts descending with an acceleration of
between the upper end is r then find the distance r′ 12 m / s2 . Find the displacement of the block during the
2
between the centres of the ball in terms of g, r, l and m. first 0.2 s after the start. Take g=10 m / s .
(A) 10 cm (B) 20 cm (C) 30 cm (D) 40 cm
Circular Motion
Q.3 A body of mass m is kept on a rough horizontal
Q.25 Calculate the centripetal acceleration of a point on surface (friction coefficient=µ). A person is trying to pull
the equator of earth due to the rotation of earth about the body by applying a horizontal force but the body is
its own axis. not moving. The force by the surface on the body is F
Radius of earth = 6400 km. where
4 . 4 8 | Forces and Laws of Motion

(A) F=mg (B) F=µmg Q.9 Two masses m and m’ are tied with a thread passing
over a pulley. M’ is on a frictionless horizontal surface
(C) mg ≤ F ≤ mg 1 + µ2 (D) mg ≥ F ≥ mg 1 − µ2 and m is hanging freely. If acceleration due to gravity
is g, the acceleration of m’ in this arrangement will be
Q.4 Which of the following case correctly represents the (A) g (B) mg/(m + m’)
applied force on a string under tension. End of string is
represented with dot. (C) mg/m’ (D) mg/(m-m’)
F FF F
Q.10 A body of mass 60 kg is dragged with just enough
(A) (A) (B) (B) force to start moving on a rough surface with coefficients
F F of static and kinetic frictions 0.5 and 0.4 respectively.
F F
On continuing (g=9.8 m/s2) the same force what is the
acceleration:
(C) (C) (D) (D)
(A) 0.98 m/s2 (B) 9.8 m/s2
(C) 0.54 m/s2 (D) 5.292 m/s2

Q.5 A balloon is descending at a constant acceleration


Q.11 Which of the following represents 2nd law of
a. The mass of the balloon is M.
motion most correctly.
When a mass m is released from the balloon it starts   
 dv
rising with acceleration a. Assuming that volume does (A) F = ma (B) F = m
dt
not change when the mass is released, what is the value  dp  
of m? [Assume same upward buoyant force] (C) F = (D) F = mv
dt
2a a+ g
(A) M (B)  M
(a+ g)  2a  Q.12 Two objects A and B are thrown upward
2a Ma simultaneously with the same speed. The mass of A
(C) (D) is greater than the mass of B. Suppose the air exerts
(a+ g)M a+g
a constant and equal force of resistance on the two
bodies.
Q.6 A small cart with a sphere suspended from ceiling
by a string is moving up an inclined plane at a speed V. (A) The two bodies will reach the same height.
The direction of string supporting the sphere is (B) A will go higher than B
(A) Vertical (C) B will go higher than A
(B) Horizontal
(D) Any of the above three may happen depending on
(C) Perpendicular to the inclined plane the speed with which the objects are thrown.
(D) None of these
Q.13 A heavy uniform chain party lies on a horizontal
Q.7 The pulleys and strings shown in the figure are table. If the coefficient of friction between the chain and
smooth and of negligible mass. For the system of remain the table surface is 0.25, then the maximum fraction of
in equilibrium, the angle θ should be the length of the chain that can hang over edge of the
table is
(A) 20% (B) 25% (C) 33% (D) 15%.

Q.14 An insect crawls up hemispherical surface very


2m
m m slowly as shown in Figure. The coefficient of friction
between insect and surface is 1/3. If
(A) 0o
(B) 30o (C) 45o (D) 60o the line joining the centre of the

hemispherical surface to the insect
Q.8 While walking on ice, one should take small steps makes an angle α with the vertical,
to avoid slipping. This is because smaller steps ensure the max. Possible value of α is given
by
(A) Larger friction (B) Smaller friction
(A) cot α=3 (B) sec α=3 (C) cosec α=3 (D) None
(C) Larger normal force (D) Smaller normal force
P hysi cs | 4.49

Q.15 When a bird of weight W alights on a stretched Q.21 A uniform chain of length  is placed on a rough
wire, the tension T in the wire may be: table with length n  hanging over the edge (n<  ). If
(A)>W/2 (B)=W (C)<W (D) None of these. the chain just begins to slide off the table by itself from
this position, the coefficient of friction between chain
and table is
Q.16 A block of mass 3kg is at rest 3kg 1 n 1 1−n
on a rough inclined plane as shown (A) (B) (C) (D)
n 1−n n+1 1+n
in the Figure. The magnitude of net
force exerted by the surface on the 30°
block will be Circular Dynamics
(A) 26N (B) 19.5N (C) 10N (D) 30N Single Correct Question

Q.17 With what minimum acceleration can a fireman Q.22 A particle moves in a circle of radius R with a
slides down a rope whose breaking strength is two third constant speed under a centripetal force F. The work
of his weight? done F in completing a full circle is:

(A) g/2 (B) 2g/3 (C) g/3 (D) 3g/4 ( )


(A) Mv 2 / R 2πR (B) πR 2F
(C) 2πRF (D) zero
Q.18 Forces of 30N, 40N and 50N act along the sides
  
AB ,BC and CA of an equilateral triangle ABC. The triangle Q.23 When a particle is rotated in a vertical plane with
is of mass 0.5 kg and kept in a constant angular velocity magnitude of centripetal
A
vertical plane as shown in the 50N force is:
Figure. With the side AB vertical. 30N (A) Maximum at highest point
The net vertical force acting on C
W
the triangle will be (g=10 m/s2) (B) Maximum at lowest point
40N
B
(C) Same at all points
(A) 125 N upwards (B) 5 N downwards (D) Zero
(C) 10 N upwards (D) 25 N downwards
Q.24 In uniform circular motion, the quantity that
Q.19 A trolley is being pulled up an incline plane by a remains constant is:
man sitting on it (as shown in Figure). He applies a force (A) Linear velocity (B) Centripetal force
of 250N. If the combined mass of the man and trolley is
100 kg, the acceleration of trolley will be [ sin 15° =0.26 ] (C) Acceleration (D) Speed

Q.25 Two particles of equal masses are revolving in


250N
circular paths of radii r1 and r2 respectively with the
same speed. The ratio of their centripetal forces is:
5° 2 2
r2 r2 r  r 
(A) (B) (C)  1  (D)  2 
(A) 2.4 m/s2 (B) 9.4 m/s2 r1 r1 r  r 
 2  1
(C) 6.9 m/s2 (D) 4.9 m/s2
Q.26 A 500kg car takes a round turn of radius 50m with
Q.20 A body is placed on a rough inclined plane of a velocity of 36km/hr. The centripetal force is:
inclination. As the angle θ is increased from 0° to 90° , (A) 250 N (B) 750 N (C) 1000 N (D) 1200 N
the contact force between the block and plane
(A) Remains constant Q.27 A body of mass 5kg is moving in a circle of
(B) First remains constant then decreases radius 1m with an angular velocity of 2 radian/sec. The
(C) First decreases then increases centripetal force is:
(D) First increases then decreases (A) 10 N (B) 20 N (C) 30 N (D) 40 N
4 . 5 0 | Forces and Laws of Motion

Q.28 A motorcycle is going on an overbridge of radius Previous Years’ Questions


R. The driver maintains a constant speed. As the
motorcycles is ascending on the overbridge, the normal Forces and Laws of Motion
force on it:
(A) Increase Q.1 A ship of mass 3 × 107 kg initially at rest, is pulled by
a force of 5 × 10 4 N through a distance of 3 m. Assuming
(B) Decreases
that the resistance due to water is negligible, the speed
(C) Remains constant of the ship is (1980)
(D) First increases then decreases. (A) 1.5 m/s (B) 60 m/s
(C) 0.1 m/s (D) 5 m/s
Q.29 If a particle of mass m is moving in a horizontal
(
circle of radius r with a centripetal force −k / r 2 , the ) Q.2 A block of mass 2 kg rests on a rough inclined plane
total energy of the particle is:
k k 2k 4k making an angle of with the horizontal. The coefficient
(A) − (B) − (C) − (D) − of static friction between the block and the plane is 0.7.
2r r r r
The frictional force on the block is (1980)
Q.30 A person with his hands in his pocket is skating on (A)9.8N (B)0.7 × 9.8 × 3N
ice at the rate of 10m/s and describes a circle of radius (C)9.8 × 3N (D)0.7 × 9.8N
50m. What is his inclination to the vertical:
(A) tan−1 (1 / 2 ) (B) tan−1 (1 / 5 ) Q.3 A car is moving in a circular horizontal track of
(C) tan−1
(3 / 5) (D) tan (1 / 10 ) −1 radius 10 m with a constant speed of 10 m/s. A plumb
bob is suspended from the roof of the car by a light
Q.31 A ball tied to a string (in vertical plane) is swinging rigid rod. The angle made by the rod with vertical is
in a vertical circle. Which of the following remains (Take g = 10m / s2 )  (1992)
constant during the motion? (A) Zero (B)30o (C)45o (D)60o
(A) Tension in the string
Q.4 A block of mass 0.1 kg is held against a wall applying
(B) Speed of the ball
a horizontal force of 5N on the block. If the coefficient
(C) Centripetal force of friction between the block and wall is 0.5, the
(D) Gravitational force on the ball magnitude of the frictional force acting on the block is
(1994)
Q.32 A heavy particle hanging vertically from a point by (A) 2.5 N (B) 0.98 N (C) 4.9 N (D) 0.49 N
a light inextensible string of length l is started so as to
make a complete revolution in a vertical plane. The sum Q.5 A long horizontal rod has a bead which can slide
of the tension at the ends of any diameter: along its length and initially placed at a distance L from
(A) First increase then decreases one end A of the rod. The rod is set in angular motion
about A with a constant angular acceleration α. If the
(B) Is constant coefficient of friction between the rod and bead is µ,
(C) First decrease then increases and gravity is neglected, then the time after which the
bead starts slipping is (2000)
(D) Decreases continuously
µ µ 1
Q.33 In a circus, stuntman rides a motorbike in a circular (A) (B) (C) (D) infinitesimal
a a µa
track of radius R in the vertical plane. The minimum
speed at highest point of track will be: Q.6 A small block is shot into each of the four tracks as
shown below. Each of the tracks rises to the same height.
(A) 2gR (B) 2gR (C) 3gR (D) gR
The speed with which the block enters the track is the
same in all the cases. At the highest point of track, the
normal reaction is maximum in (2001)
P hysi cs | 4.51

Q.10 What is the maximum value of force F such that


the block shown in the arrangement. does not move?
(2003)
F m= 3kg
v v 60° = 1
2 3
(A) (B)

(A) 20 N (B) 10 N (C) 12 N (D) 15 N

Q.11 A block of mass m is at rest under the action of


v v
force F against a wall as shown in Figure. Which of the
(C) (D) following statement is incorrect?  (2005)

Q.7 An insect crawls up a hemispherical surface very F


slowly (see the Figure). The coefficient of friction between
the surface and insect is 1/3. If the line joining the
center of the hemispherical surface to the insect makes (A) f=mg (where f is the frictional force)
an angle α with the vertical, the maximum possible (B) F=N (where N is the normal force)
value of α is given (2001)
(C) F will not produce torque

(D) N will not produce torque

Q.12 System shown in Figure is in equilibrium


(A) cot α = 3 (B) tan α = 3
and at rest. The spring and string are massless.
(C) sec α = 3 (D) cosec α = 3 Now the string is cut. The acceleration of
mass 2m and m just after the string is cut will
be (2006)
Q.8 A string of negligible mass going 2m
over a clamped pulley of mass m (A) g/2 upwards, g downwards
m
supports a block of mass M as shown in (B) g upwards, g/2 downwards
the Figure. The force on the pulley by m
the clamp is given by (2001) (C) g upwards, 2g downwards
M (D) 2g upwards, g downwards

Q.13 A piece of wire is bent in the shape of a parabola


(A) 2Mg (B) 2mg
(y-axis vertical) with a bead of mass m on it. The bead can
slide on the wire without friction. It stays at the lowest
(C) (M+ m)2 + m2g (D)  (M+ m)2 + M2  g
  point of the parabola when the wire is at rest. The wire
is now accelerated parallel to the x-axis with a constant
acceleration a. The distance of the new equilibrium
Q.9 The pulleys and strings position of the bead, where the bead can stay at rest
shown in the Figure. are with respect to the wire, from the y-axis is (2009)
smooth and of negligible mass. a a 2a a
(A) (B) 2gk (C) (D)
For the system to remain in gk gk 4gk
equilibrium, the angle θ should
be (2001) 2m
m m
Q.14 A block of mass m is on an inclined plane of angle
θ . The coefficient of friction between the block and
plane is µ and tan θ > µ. The block is held stationary by
(A) Zero (B)30o (C)45o (D)60o applying a force P parallel to the plane. The direction of
force pointing up the plane is taken to the positive. As
P is varied from
4 . 5 2 | Forces and Laws of Motion

Fo Fo
m
P mb

v(t) v(t)
(A) (B)
=P1 mg ( sin θ − µ cos θ ) =
to P2 mg ( sin θ + µ cos θ ) the t t

frictional force f versus P graph look like (2010) Fo Fo


mb mb
f f
P₂ v(t) v(t)
(A) (B)
P₁ P P₁ P₂ P (C) (D)
t t

f f

P₁ P1 P2 Q.19 A block of mass m is placed on a surface with a


(C)
P₂ P
(D) vertical cross section given by y = x3/6. If the coefficient
P
of friction is 0.5, the maximum height above the
ground at which the block can be placed without
Q.15 A reference frame attached to the earth (1986) slipping is: (2014)

(A) Is an inertial frame by definition. 1 1 1 2


(A) m (B) m (C) m (D) m
3 2 6 3
(B) Cannot be an inertial frame because the earth is
revolving round the sun.
Q.20 Given in the figure are two blocks A and B of weight
(C) Is an inertial frame because Newton’s law are 20 N and 100 N respectively. These are being pressed
applicable in this frame. against a wall by a force F as shown. If the coefficient of
(D) Cannot be an inertial frame because the earth is friction between the blocks is 0.1 and between block B
rotating about its own axis and the wall is 0.15, the frictional force applied by the
wall on block B is (2015)
Circular Dynamics (A) 80 N (B) 120 N (C) 150 N (D) 100 N

Q.16 A car is moving in a circular horizontal track of Q.21 A point particle of mass m, moves along the
radius 10m with a constant speed of 10m/s. A plumb uniformly rough track PQR as shown in the figure. The
bob is suspended from the roof of the car by a light coefficient of friction, between the particle and the
rigid rod. The angle made by the rod with the vertical is rough track equals µ. The particle is released, from rest,
(Take g=10/s2) (1992) from the point P and it comes to rest at a point R. The
energies, lost by the ball, over the parts, PQ and QR, of
(A) Zero (B) 30
o
(C) 45o
(D) 60o
the track, are equal to each other, and no energy is lost
when particle changes direction from PQ to QR. (2016)
Q.17 A ball of mass (m) 0.5 kg is attached
to the end of a string having length (L) The values of the coefficient of friction µ and the distance
0.5m. The ball is rotated on a horizontal L x(=QR), are, respectively close to :
circular path about vertical axis. The (A) 0.2 and 3.5 m (B) 0.29 and 3.5 m
maximum tension that the string can
bear is 324 N. The maximum possible (C) 0.29 and 6.5 m (D) 0
m
value of angular velocity of ball (in rad/s)
3kg − ms−1 (2011)
(A) 9 (B) 18 (C) 27 (D) 36

Q.18 A particle of mass m is at rest at the origin at time t


= 0. It is subjected to a force F (t) = F0e–bt in the x direction.
Its speed v(t) is depicted by which of the following
curves? (2012)
P hysi cs | 4.53

JEE Advanced/Boards

Exercise 1 Q.4 The rear side of a truck is open and a box of 40 kg


mass is placed 5 m away from the open end as shown in
Forces and Laws of Motion Figure. The co-efficient of friction between the box and
the surface below it is 0.15. On a straight road, the truck
Q.1 A man of mass 70 kg stands on weighting scale in a starts from rest and accelerates with 2 ms−2 At what
lift which is moving distance from the starting point does the box fall of the
truck? (Ignore the size of the box).
(a) Upwards with a uniform speed of 10 ms−1 .
(b) Downwards with a uniform acceleration of 5 ms−2 .
(c) Upwards with a uniform acceleration of 5 ms−2 .
(d) What would be the reading if the lift mechanism
failed and it hurtled down freely under gravity?
What would be the readings on the scale in each case? Q.5 A helicopter of mass 1000 kg rises with a vertical
acceleration of 15 ms−2 . The crew and the passenger
x(m) weigh 300 kg. Give the magnitude and direction of the
(a) Force on the floor by the crew and passengers.
3 A (b) Action of the rotor of the helicopter on the
surrounding air.
0 4 t(s) (c) Force on the helicopter due to the surrounding air.
Q.2 A block of mass 25 kg is raised by a 50 kg man in two
different ways as shown in Figure. What is the action on Q.6 A block of mass 15 kg is placed on a long trolley.
the floor by the man in the two cases? If the floor yields The co-efficient of static friction between the block and
to a normal force of 700 N, which mode should man trolley is 0.18. The trolley accelerates from rest with
adopt to lift the block without the floor yielding? 0.5 ms-2 for 20 s and then moves with uniform velocity.
Discuss the motion of the block viewed by (a) a stationary
observer on the ground. (b) an observer moving with
the trolley.

Q.7 Both the springs shown in the Figure are un-


stretched. If the block is displaced by a distance x and
released, what will be the initial acceleration?
k₁ m k₂
25kg 25kg
(a) (b)
Q.8 Three equal weights of 2 kg each are hanging
Q.3 A monkey of mass 40 kg climbs on a over the frictionless pulley. Find the acceleration of the
rope which can stand a maximum system and tension of the string connecting weights A
tension of 600 N. In which of the and B. (g=10 m/s2)
following cases will the rope break: the
monkey Q.9 Find the tension in OB and AB in the given Figure.
(a) Climbs up with an acceleration of Also, calculate the tension in OB when just after the
6ms-2 string AB is burnt. O
(b) Climbs down with an acceleration of
4ms-2
(c) Climbs up with a uniform speed of 5ms-2 m
A
B
(d) Falls down the rope nearly freely under gravity.
4 . 5 4 | Forces and Laws of Motion

Q.10 A man of mass m has fallen into a ditch of width Q.15 At the moment t=0 the force F=at is applied to a
d and two of his friends are slowly pulling him out small body of mass m resting on a smooth horizontal
using a light rope and two fixed pulleys as shown in plane (a) is constant). The permanent direction of this
Figure. Show that the force (assumed equal for both the force forms as angle α with the horizontal. Find (a) The
friends) exerted by each friend on the rope increases as velocity of the body at the moment of its breaking off
the man move up. Find the force when the man is at a the plane; (b) The distance traversed by the body up to
depth h. this moment.
F

h
m

Q.16 Two identical block A and B each of mass M are


d connected through a light inextensible string. Coefficient
of friction between blocks and surfaces are µ as shown.
Q.11 The elevator shown in the
2m/s² Initially string is relaxed in its normal length. Force F is
Figure is descending with an
applied on block A as shown. Find the force of friction
acceleration of 2m/s2. The mass of A on blocks and tension in the string.
block A is 0.5 kg. What force es B
exerted by the block A on the block
Q.17 In the Figure block A is one fourth the length of
B?
the block B and there is no friction between block B and
surface on which it is placed. The coefficient of sliding
Q.12 The force of buoyancy exerted by the atmosphere
friction between A and B. Block C and block A have the
on a balloon is B in the upward direction and remains
same mass and mass of B is four times mass of A. when
constant. The force of air resistance on the balloon acts
the system is released, calculate the distance the block
opposite to the direction of velocity and is proportional
B moves when only three fourth of block A is still on the
to it. The balloon carries a mass M and is found to fall
block B.
down near the earth’s surface with a constant velocity v.
How much mass should be removed from the balloon A
so that it may rise with a constant velocity v? B

l C
Q.13 Two touching bars 1 and 2 are placed on an
inclined plane forming an angle α with the horizontal
shown in Figure. The masses of the bars are equal to Q.18 The inclined plane of forms an angle α=30o with
m1 and m2 and the coefficients of friction between the horizontal. The mass ratio. The coefficient of friction
the inclined plane and the bars are equal to k1 and k 2 between the body and inclined plane is equal to k-0.10.
respectively, with k1 > k 2 . The masses of the pulley and the threads are negligible.
Find the magnitude and the direction of acceleration of
Find (a) The force of interaction 
the body m2 when the system of masses starts moving.
of the bars in the process of 2
motion; (b) The minimum value 1 m1
of the angle α at which the bars
 m2
start sliding down.


Q.14 A small body A starts sliding down from the top Q.19 As shown in the Figure blocks of masses M/2, M
of fixed wedge (as shown in the Figure) whose base is and M/2 are connected through a light string as shown,
equal to l=2.10 m. The coefficient of friction between pulleys are light and smooth. Friction is only between
the body and the wedge surface is k=0.140. At what block C and floor. System is released from rest. Find the
value of the angle α will the time of sliding be the least? acceleration of blocks A, B and C and tension in the
string. C
A 2
B M/
 = tan
M 2
 A
l 2
M/

P hysi cs | 4.55

Q.20 On a smooth inclined plane of angle α is placed on between the bar and electric motor is equal to l. When
in such a way that the upper wedge face is horizontal. the motor is switched on, the bar, whose mass is twice
On this horizontal face is placed a block of mass m. Find as great as that of the other body, starts moving with
the resultant acceleration of the block in subsequent a constant acceleration w. How soon will the bodies
motion. collide?
m
Wedge  Q.26 Two particle of equal masses m and m are
M
connected up a light string of length 2l. A constant force
 F is applied continuously at the middle of the string,
always along the perpendicular bisector of the line
joining the two particles. Show that when the distance
Q.21 In the system shown in Figure. = mA 4m,m= 3m
B between the particles is 2x, the acceleration of approach
and mC = 8m . Friction is absent everywhere. String is
fx
light and inextensible. If the system is released from rest of particles is .
1
find the acceleration of each block.
m(2 – x2 ) 2
B
Q.27 Determine the acceleration of bodies A and B and
A
the tension in the cable due to application of the 300 N
C
force. Neglect all friction and the masses of pulleys.

Q.28 Two blocks A and B having masses m1 = 1 kg and


Q.22 Find the accelerations of m2 = 4 kg are arranged as shown in Figure. The pulleys
rod A and wedge B in the P and Q are light and frictionless. All blocks are resting
arrangement shown in the on the horizontal floor and pulleys are held such that
Figure. If the ratio of the mass strings remain just taut. At moment t=0, a force F=30N
A
of the wedge of that of the starts acting on the pulley P along vertically upward
rod equals, and the friction B direction as shown in the Figure: Determine
between all contact surfaces 
(a) The time when blocks A and B lose contact with
is negligible.
ground,
Q.23 A particle of mass m is pulled by means of a (b) The velocity of A when B loses contact with ground,
thread up an inclined plane forming an angle α with (c) The height raised by A till this instant.
the horizontal as shown in the Figure. The coefficient of
friction is equal to µ. Find the angle β which the thread
Circular Dynamics
must form with the inclined plane for the tension of the
thread to be maximum. What is it equal to? Q.29 An astronaut is rotating in a rotor having
vertical axis and radius 4m. If he can withstand upto

acceleration of 10 g. Then what is the maximum number
m
of permissible revolutions per second? 60o

 Q.30 A racing-car of 1000kg moves round a banked


track at a constant speed of 108 km ms-2. Assuming the
Q.24 A plank of mass with a block of mass m2 placed on total reaction at the wheels is normal to the track and
it lies on a smooth horizontal plane. A horizontal force the horizontal radius of inclination of the track to the
growing with time t as F=at(a is constant) is applied horizontal and the reaction at the wheels.
to the plank. Find how the accelerations of the plank
and of the bar w2 depend on t, if the coefficient of Q.31 A man whirls a stone around his lead on the end
friction between the plank and block is equal to k. Draw of a string 4metre long. If the stone has a mass of 0.4 kg
approximate plots of these dependences. and the string will break if the tension in it exceeds 8 N,
what is the smallest angle the string can make with the
Q.25 A horizontal plane with the coefficient of friction horizontal? What is the speed of the stone? 40o
k supports two bodies: a bar and an electric motor with
a battery on a block. A thread attached to the bar is
wound on the shaft of the electric motor. The distance
4 . 5 6 | Forces and Laws of Motion

Q.32 A boy whirls a stone in a horizontal circle of radius speed of the particle and the inclination of the string to
1.5m and 2m above the ground by means of a string. the vertical at the instant of the motion when the tension
The string breaks and the stone files off horizontally, in the string is equal to the weight of the particle.
striking the ground 10m away. What is the centripetal
acceleration during circular motion? Q.40 A hemispherical bowl of radius R is rotated about
its axis of symmetry which is kept vertical. A small block
Q.33 A stone is fastened to one end of a string and is is kept in the bowl at a position where the radius makes
−2
whirled in a vertical circle of radius R. Find the minimum angle ms with the vertical. The block rotates with the
speed the stone can have at the highest point of the bowl without any shipping. The frictional coefficient
circle. between the block and the bowl is β=54o28’. Find the
range of angular speed for which the block will not slip.
Q.34 A stone of mass 1kg is attached to one end of a
string of length 1m and breaking strength 500N, and Q.41 A table with smooth horizontal surface is fixed in a
is whirled in a horizontal circle on a frictionless table cabin that rotates with a uniform angular velocity m / s2
top. The other end of the string is kept fixed. Find the in a circular path of radius R=700m. A smooth groove
maximum speed the stone can attain without breaking AB of length L=7m is made on the surface of the table.
the string. (k1 + k 2 ) x
The groove makes an angle with the radius OA
m
Q.35 A circular automobile test track has a radius of
of the circle in which the cabin rotates. A small particle
200m. The track is so designed that when a car travels
if kept at the point A in the groove and is released to
at a speed of 100 kilometer per hour, the force between
move along AB. Find the time taken by the particle to
the automobile and the track is normal to the surface of
reach the point B.
track. Find the angle of the bank.

Q.42 A table with smooth horizontal surface is placed


Q.36 A block of mass M is kept on a horizontal ruler.
in a cabin which moves in a circle of a large radius
The friction coefficient between the ruler and block
R. A smooth pulley of small radius is fastened to the
mg2 cos α
is v = . The ruler is fixed at one end and the table. Two masses of m and 2m are placed on the table
2asin2 α connected through a string going over the pulley.
block is at a distance L from the fixed end. The ruler is Initially the masses were at rest. Find the magnitude of
rotated about the fixed end. Find the maximum angular the initial acceleration of the masses as seen from the
speed for which block will slip. cabin and the tension in the string.

Q.37 A motorcycle has to move with a constant speed Q.43 A particle of mass m moves along a horizontal
on an over bridge which is in the form of a circular circle of radius R such that normal acceleration of particle
are of radius R and has a total length L. Suppose the varies with time as Tab=mgtanθ, Tab=mg/cosθ, T’=mgcosθ.
motorcycle starts from the highest point. (a) What can
where K is a constant. Calculate
its maximum velocity be for which contact with road is
not broken at the highest point? (b) If the motorcycle (i) Tangential force on particle at time t
goes at speed grφ2 (r − r φ) =2lgm times the maximum (ii) Total force on particle at time t
found in part (a). Where will it lose the contact with the (iii) Power developed by total force at time t and
road? (c) What maximum uniform speed can it maintain
(iv) Average power developed by total force over first t
on the bridge if it does not lose contact anywhere on
second
the bridge?
Q.44 A smooth sphere of radius R is made to translate in
Q.38 A simple pendulum is suspended from the ceiling
a straight line with a constant acceleration a. A particle
of a car taking a turn of radius 10m at a speed of 36km/h.
kept on the top of the sphere is released from there at
Find the angle made by the string of the pendulum with
zero velocity with respect to the sphere. Find the speed
the vertical if this angle does not change during the
of the particle with respect to the sphere as a function
turn. Take kmh-2.
of the angle Mg a2 + 4h2 it slides.
4h
Q.39 A heavy particle hanging from a fixed point by
Q.45 A uniform circular ring of mass per unit length
a light inextensible string of length ms−2 is projected
  2(Mg− B)
horizontally with speed − ( x + y ) / 2cm / s2 , find the and radius R is rotating with angular velocity
g
P hysi cs | 4.57

mg2 cos α µMg µMg


f (k1 − k 2 )
= about its own axis in a gravity free (B) fA = µMg, fB =, T =
m1 + m2 2 2
space. Find the tension in the ring. (C)
(C) F ≥ (m1 + m2 + m3 )µ g

(D) F ≤ (m1 + m2 + m3 )µ g
Q.46 If a particle is rotating in a circle of radius R with
velocity at an instant v and the tangential acceleration is
a. Find the net acceleration of the particle. Q.4 Blocks A and B in Figure are connected by a bar
of negligible weight. If mass of A and M are 170 kg
each and µ A = 0.2 and µB = 0.4 , where µ A and µB are
Q.47 A metal ring of mass m and radius R is placed on
the coefficients of limiting friction between blocks and
a smooth horizontal table and is set rotating about its
own axis in such a way that each part of the ring moves (
plane, calculate the force in the bar. g = 10 m / s2 )
with speed v. Find the tension in the ring.
B
Q.48 A car goes on a horizontal circular road of
radius R the speed is increasing at a constant rate
A 8
k1 m1 + k 2 m2
cos αmin = a. The friction coefficient is 15
m1 + m2
(A) 150 N (B) 75 N (C) 200 N (D) 250 N
1  1
=α tan−1  −  . Find the speed at which the car will
2  µ Q.5 A person, standing on the floor of an elevator, drops
just skid. a coin. The coin reaches the floor of the elevator in a
time t1 . If the elevator is stationary and in time t2 if it is
Exercise 2 moving uniformly. Then
(A) t1 = t2
Forces and Laws of Motion
(B) t1 < t2
Single Correct Choice Type
(C) t1 > t2

Q.1 A chain of length L and mass M is hanging by fixing (D) t1 > t2 or t1 < t2 depending on whether the
its upper end to rigid support. The tension in the chain lift is going up or down.
at a distance x from the rigid support is

m2g3 cos α Q.6 How large must F be in the Figure shown to given
s=
6a2 sin3 α the 700 gm block an acceleration of 30 cm / s2 ? The
coefficient of friction between all surfaces is 0.15.
Q.2 A block A kept on an inclined surface just begins
to slide if the inclination is 30o. The block is replaced by 200gm
another block B and it is found that it just begins to slide
if the inclination is 40o. 700gm F
(A) Mass of A > mass of B.
(B) Mass of A < mass of B (A) 4 N (B) 2.18 N (C) 3.18 N (D) 6N

(C) Mass of A = mass of B


Q.7 If the force which acting parallel to an inclined plane
(D) All the three are possible. of angle α just sufficient to draw the weight up in n times
the force which will just let it be on the point of sliding
Q.3 The arrangement shown in the Figure, the system down, the coefficient of friction will be
of masses m1 ,m2 and m3 is being pushed by a force
(n− 1) (n+ 1)
F applied on m1 horizontally. In order to prevent the= (A) µ tan α (B) µ
= tan α
downwards slipping of m2 between m1 and m3 . If n+1 n−1
coefficient of friction between m2 and m3 is µ and all (C)
= µ ntan α (D) µ= (n + 1)tan α

the other surfaces are smooth, the minimum value of F
3
(A) fA = µMg, fB = 0, T =0
4
4 . 5 8 | Forces and Laws of Motion

Q.8 Two blocks A and B of masses m and M are placed


A P
in a platform as shown in the Figure. The friction
coefficient between A and B is µ but there is no friction B
between B and the platform. The whole arrangement is C
placed inside an elevator which is coming down with an
acceleration a(a < g). What maximum horizontal force F (A) 90 N (B) 60 N (C) 80 N (D) 100 N
can be applied to A without disturbing the equilibrium
of the system? Q.12 A block of mass rests on a rough horizontal
plane relative to which
A the coefficient of friction
F m is µ. A light string
f attached to the body
B M T passes over a light
pulley and carries at its 45°
(A) 2µmg (B) 2µm(g – a) other end a mass m2 . m2 m1
When the system just
(C) 2µm(g+a) (D) 2µma
begins to move, the
value of µ is
Q.9 A body of mass m1 is placed on a horizontal plank of
m2 m m2 m2
mass m2 which rests on a smooth horizontal table. The (A) (A) 2 (B) (B)
coefficient of friction between the mass m1 and plank 2m1 − m2 2m1 − m2 2m1 + m2 2m1 + m2
is µ. A gradually increasing force F depending on time m2 m2 m2 m2
t as F=at where a is constant is applied to the plank. (C) (C) (D) (D)
2m2 + m1 2m2 + m1 2m2 − m1 2m2 − m1
The time t0 at which the plank starts sliding under the
mass is
m1µg (m1 + m2 )µg m2µg m1m2µg
(A) (B) (C) (D) Multiple Correct Choice Type
a a a a

Q.14 In the arrangement shown in the Figure pulley


Q.10 Block A is placed on block B whose mass is greater
is smooth and massless and string is light. Friction
than that of A. There is friction between blocks while
coefficient between A and B is µ. Friction is absent
the ground is smooth. A horizontal force P increasing
between A and plane. Select the correct alternative(s)
linearly with time begins to act on A. The accelerations
a1 and a2 of A and B respectively are plotted in a graph
against time. Which of the following graphs represents
the real situation? B
a a A
a2 a2
 Fixed
a1 a2 a1
(A) (B) a1
(A) acceleration of the system is zero if
t t m − mA
a a µ≥ B tan θ and mB > mA
a1 a1 2mB
(B) Force of friction between A and B is zero if
a2 a2 a2
(C) (D) a2 mA = mB
a1 a1
t t (C) B moves upwards if mB < mA
(D) Tension in the string is mg ( sin θ − µ cos θ ) if
Q.11 Find the least horizontal force P to start motion m= m= m
A B
of any part of the system of the three blocks resting
upon one another as shown in Figure. The weight of
blocks are A=300 N, B= 100 N, C=200 N. Between A
and B, µ=0.3. Between B and C, µ=0.2. Between C and
the ground µ=0.1.
P hysi cs | 4.59

Q.15 (A, D) Two blocks A and B of Q.18 Assertion: The law of conservation of linear
30°
mass 10 kg and 20 kg respectively momentum, as applied to a single particle, is equivalent
are placed as shown in Figure. to Newton’s first law of motion.
Coefficient of friction between all
Reason: As Newton’s first law states in the absence
the surfaces is 0.2 (g=10 m/s2)
of external force state of motion of a body does not
(A) Tension in the string is 306 N change.
(B) Tension in the string is 132 N A B
(C) Acceleration of block B is 2.6m/s 2 Q.19 Assertion: The impulse of a force can be zero
(D) Acceleration of block B is 2.6m/s 2 even if force is not zero.
Reason: The impulse of a force is equal to change in
Q.16 In the arrangement shown in the Figure. all surface momentum of a body.
are smooth. Select the correct alternative(s)
Q.20 Assertion: If a book is placed on table at rest then
force exerted by table on the book and weight of the
B
A book formation reaction pairs according to Newton 3rd
law of motion
 Fixed Reason: Since both are equal in magnitude and
opposite in directions.
(A) For any value of θ acceleration of A and B are equal
Q.21 Assertion: The mass of a body can be regarded
(B) Contact force between the two blocks is zero if as a quantitative measure of the resistance of a body to
mA / m
=B tan θ acceleration by a given force.
(C) Contact force between the two is zero for any value Reason: The acceleration produced by a given force is
of mA or mB inversely proportional to mass being accelerated.
(D) Normal reactions exerted by the wedge on the block
are equal. Q.22 Assertion: While conserving the linear momentum
of the system we must specify the reference frame.
Q.17 Two blocks A and B of equal mass m are connected Reason: Like velocity, momentum also depends on the
through a massless string and arranged as shown in reference from of observer.
Figure. Friction is absent everywhere. When the system
is released from rest. Comprehension Type
(A) Tension in string is mg/2 C
Paragraph 1: A ball of mass m
(B) Tension in string is mg/4 is connected with the string AB 
(C) Acceleration in string is g/2 and BC respectively as shown in
the figure. Now string AB is cut. A
(D) Acceleration in string is 3g/2 Answer the following questions B

Assertion Reasoning Type Q.23 Tension in the string AB and BC respectively the
string AB is cut
Each of the question given below consists of two
statements, an assertion and reason. Select the number (A) mg cotB, mg cosB (B) mg tanB, mg cosB
corresponding to the appropriate alternative as follows
(C) mg tanB, mg secB (D) mg cotB, mg secB
(A) If both assertion and reason are true and reason is
the correct explanation of assertion Q.24 Tension in the string BC just after the string AB is
(B) If both assertion and reason are true but reason is cut
not the correct explanation of assertion (A) mg sinB (B) mg cosB
(C) If assertion is true but reason is false (C) mg tanB (D)mg secB
(D) If assertion is false but reason is true.
4 . 6 0 | Forces and Laws of Motion

Q.25 If string BC is cut instead of AB, what is the tension


(A) − gtan
(A)θ − gtan θ (B) + gtan
(B)θ + gtan θ
in the string AB just after gtan θ gtan θ gtan θ gtan θ
(C) − (C) − (D) + (D) +
(A) mg cosB (B) mg tanB 2 2 2 2
(C) mg sinB (D) zero
Passage 3: An arrangement designed to measure
the acceleration due to gravity at a place consist of
Q.26 If the whole system is placed in an automobile,
two blocks A and B, of mass m and 2m respectively
what is the acceleration required to be given to it so
connected to each other by means of a light inextensible
that even after cutting the string AB, it remains in the
string passing over a light frictionless pulley as shown in
same position
the Figure. A light and very rough plank, rigidly held
(A) g tanB, right ward (B) g cotB, right ward in position, supports the block A. The system, it is
(C) g tanB, left ward (D) g cotB, left ward observed does not move at all. The portion of the string
OA, is initially horizontal. Assume that the acceleration

Passage 2: A block of mass m slides down a smooth


(
due to gravity, g = 10 m / s2 )
incline of mass M and length l, solely as a result of A
O
m
the force of gravity. The incline is placed on a smooth Clamp
horizontal table as shown in Figure. Let us denote the
Plank
coordinate system relative to the table as S1 and the
coordinate system relative to the incline as Sφ
m 2m B

M
Q.31 The net force due to plank, acting on the block A,
has magnitude
(A) 2mg (B) mg (C) 3mg (D) 5mg
l

Q.27 The acceleration of m in the S’ frame is Q.32 The magnitude of the force exerted on the pulley
by the clamp is, when the system is in equilibrium
(M+ m)gsin θ
(M+ m)gsin (M+ m)gsin θ
(A) (A) 2 (B)θ (B) (M+ m)gsin θ 2 2 mg
(M+ m)gsin
(M θ 2 θθ(M
M++m)gsin + m)gsin
(M
m++m)gsin
2 θ
θMsin2 θθ (A) 4mg (B) 4mg / 3 (C) (D) 2 2mg
(A) M +(A)
msin θmsin (B) m +(B)Msin
3
2 2 2
(M
M−+m)gsin
msin θ
M−+m)gsin
(M θmsin θθ(M
m + m)gsin
Msin
m+θ
(M θ 2 θθ
Msin
m)gsin
(C) (C) 2 (D) (D) Q.33 The plank is suddenly broken by an impulsive force,
(M− m)gsin
(M θ 2 θθ(M
M−+m)gsin M++m)gsin
(M+θm)gsin
msin θ θ
(C) M +(C)
msin θmsin (D) (D) M + msin θ
M + θmsin θ M + msin
M + msin 2 2 M +θmsin θ acting downwards. The instantaneous accelerations of A
and B, just after the plank is removed, are respectively,
Q.28 The acceleration of the incline in the S frame (A) 10 m/s2 and 10 m/s2 (B) 20 m/s2 and 3.33 m/s2

 mgsin θ cos θ   mgsin θ cos θ  (C) 12 m/s2 and 6.66 m/s2 (D) None of the above
(A)   mgsin θ cos  θ (B) −   mgsin θ cos  θ
mgsin
(A) θ cos
M+mgsin
msin 2 θ
θθcos θ   mgsin
(B) − θ cos
M+mgsin
msin 2 θ
θθcos
 θ 
(A) (A) M + msin 2  θ  (B) −(B)
 −  M + msin 2
 θ 

M+M
 Mgsin msin 2
θθ 2 θ  M+ Mmsin 2
θθ 2 θ  Passage 4: A vertical gap 2.2 cm wide of infinite extent
θ
+ cos
msin  Mgsin θ cos
+ msin
(C)   Mgsin θ cos θ  (D) −   Mgsin θ cos θ contains a fluid of viscosity 2.0 NS / m2 and specific
Mgsin
(C) 2 θ
θ cos  θ  (D) Mgsin 2 θ
θ cos
− +Mgsin  θ 
(C) (C)M+Mgsin
msin θθcos
2 (D) −(D) M msin θθcos
 M + msin  θ   − M + msin 2 θ  gravity 0.9. A metallic plate 1m X 1m X 0.2cm, which
 M+M msin 2
θ 2 θ 
+ msin  M+ Mmsin 2
+ msinθ 2 θ 
is in the middle of the gap, is to be lifted up with a
constant speed 0.15 m/sec through the gap. The weight
Q.29 The force exerted by the small m on the wedge of of the plate is 48N. Assuming pulley is massless and
mass M frictionless, string is also massless. (g=10 m/s2)
Mmg Mmg
(A) mgcos θ mgcos θ
(A) (B) (B) 2
M + msinM2 θ+ msin2 θ

mg mg
(C) (C) (D) None(D) None
cos θ cos θ m-83kg

60°
Q.30 At what acceleration ax (in the S frame) must the
0.2cm
incline be accelerated to prevent m from sliding
2.2cm
P hysi cs | 4.61

Q.34 Buoyant force acting on the plate Q.42 Three identical particles are joined together by
(A) 1800 N (B) 900 N (C) 180 N (D) 18 N a thread as shown in figure. All the three particles are
moving on a smooth horizontal plane about point O.
Q.35 Net frictional force exerted by the liquid on the If the velocity of the outermost particle is v 0 , then the
plate ratio of tensions in the three sections of the string is
(A) 30 N (B) 60 N (C) 15 N (D) 120 N (A) 3:5:7 (B) 3:4:5 (C) 7:11:6 (D) 3:5:6

Q.36 Tension in the string Q.43 The kinetic energy k of a particle moving along a
circle of radius R depends on the distance covered s as
(A) 90 N (B) 108 N (C) 240 N (D) 120 N
k= as2 where a is a constant. The total force acting on
the particle is:
Q.37 For doing so the kinetic friction between the  s2 
1/2
s2 R2
inclined plane and the block should be equals to (A) 2a (B) 2as   (C) 2as (D) 2as
R  R2  s2
3 3 1 1  
(A) (B) (C) (D)
4 8 3 2 3
Multiple Correct Choice Type

Q.38 A block of mass m is placed on a plank, which is Q.44 A car of mass M is moving on a horizontal circular
pivoted at one end. The plank is slowly turned as shown path of radius r. At an instant its speed is v and is
in Figure. The friction coefficient between block and increasing at a rate a-
plank is 0.8. Angle between ground and plank friction
(A) The acceleration of the car is towards the centre of
force between block and plank at which the block starts
the path
sliding is
(B) The magnitude of the frictional force on the car is
block greater than mv 2 / R
ground
plank (C) The friction coefficient between the ground and the
car is not less than a/g
(A) 20o (B) 45o (C) 30o (D) 35o
(D) The friction coefficient between the ground and the
car is µ =tan−1 v 2 / Rg

Circular Dynamics Q.45 A circular road of radius r is banked for a speed


of v=40km/h. A car of mass m attempts to go on the
Single Correct Choice Type circular road. The friction coefficient between the tyre
and the road is negligible. Then-
Q.39 Two bodies of mass 10k and 5kg moving in (A) The car cannot make a turn without skidding
concentric orbits of radii R and r such that their periods
(B) If the car turn at a speed less than 40km/h, it will slip
are the same. Then the ratio between their centripetal
down.
acceleration is:
(C) If the car turns at the correct speed of 40km/h the
(A) R/r (B) r/R (C) R2/r2 (D) r2/R2
force by the road on the car is equal to mv 2 / r
Q.40 A string breaks if its tension exceeds 10 newton. A (D) If the car turns at the correct speed of 40 km/h, the
stone of mass 250 mg tied to this string of length 10cm force by the road on the car is greater that mg as well as
is rotated in a horizontal circle. The maximum angular greater than mv2/r
velocity of rotation can be:
Q.46 Figure shows a rod of length L pivoted near an end
(A) 20 rad/s (B) 40 rad/s (C) 100 rad/s (D) 200 rad/s
and which is made to rotate in a horizontal plane with a
constant angular speed.
Q.41 A stone of mass of 16kg is attached to a string 
A ball of mass m is
144m long and is whirled in a horizontal circle. The L
suspended by a string
maximum tension the string can withstand is 16 newton.
also of length L from the L
The maximum speed of revolution of the stone without 
other end of the rod. If θ is T m
breaking it, will be:
the angle made by sting
(A) 20ms-1 (B) 16ms-1 (C) 14ms-1 (D) 12ms-1 with the vertical, then-
4 . 6 2 | Forces and Laws of Motion

(A) T sin θ= mω2L (1 + sin θ ) (B) T cos θ =mg Q.52 Assertion: A cyclist always bends inwards
while negotiating a curve
ω2L (1 + sin θ ) Reason: By bending he lowers his centre of gravity
(C) tan θ = (D) None of above
g
Q.53 Assertion: The tendency of skidding/overturning

Q.47 A person applies a constant force F on a particle is quadrupled, when a cyclist doubles his speed of
of mass m and finds that the particle move in a circle of turning.
radius r with a uniform speed v. v2
(A) This is not possible Reason: tan θ =
rg
(B) There are other forces also on the particle
(C) The resultant of other forces is mv 2 / r towards centre Q.54 Assertion: On a banked curved track, vertical
(D) The resultant of the other forces varies in magnitude component of normal reaction provides the necessary
as well as direction centripetal force.
Reason: Centripetal force is always required for turning.
Assertion Reasoning Type
In each of the following questions, a statement of Comprehension Type Questions
Assertion (A) is given followed by a corresponding
statement of Reason (R) just below it/of the statements, Passage 1: A block of mass m moves on a horizontal
mark the correct answer as circle against the wall of a cylindrical room of radius
(A) If both assertion and reason are true and reason is R. The floor of the room on which the block moves is
the correct explanation of assertion. smooth but the friction coefficient between the block
(B) If both assertion and reason are true but reason is and the side wall is µ . The block is given initial velocity
not the correct explanation of assertion. v o . Then answer the following questions.
(C) If assertion is true but reason is false.
Q.55 What is the tangential acceleration of the block?
(D) If assertion is false but reason is true. 2
(A) µg (B) −µg (C) µv 2 / R (D) −µv / R
(E) If both assertion and reason are false.
Q.56 What is the value of velocity v as the function of
Q.48 Assertion: Centripetal and centrifugal forces time t?
cancel each other
1 1 µt 1 1 µt
Reason: This is because they are always equal and (A) = + (B) = −
opposite. v v 0 2R v v 0 2R

1 1 µt 1 1 µt
Q.49 Assertion: A cyclist bends inwards from his (C) = + (D) = −
v v0 R v v0 R
vertical position, while turning to secure the necessary
centripetal force.
Q.57 What is the value of velocity v as the function of
Reason: Friction between the tyres and road provides
distance x travelled on the circumference?
him the necessary centripetal force. 2µ µ
− − x
(A) v = v o e R (B) v = v o e R
Q.50 Assertion: The tendency of skidding or
overturning is quadrupled, when a cyclist double his  − x

=
(C) v v o 1 − e R  (D) v = v o

speed of turning.  
 

Reason: Angle of bending increases as velocity of Passage 2: In a rotor, a hollow vertical cylindrical
vehicle increases. structure rotates about its axis and a person rests
against the inner wall. At a particular speed of the rotor,
Q.51 Assertion: On banked curved track, vertical the floor below the person is removed and the person
component of normal reaction provide the necessary hangs resting against the wall without any floor. If the
centripetal force. radius of the rotor is 2m and the coefficient of static
friction between the wall and the person is 0.2. Find the
Reason: Centripetal force is always required for motion
following parameters and relations.
in curved path.
P hysi cs | 4.63

Q.58 If v is the velocity of rotation of rotor and N be the Q.62 The bob of a simple pendulum is given a velocity
reaction of wall, then- 10m/s at its lowest point. Mass of the bob is 1kg and
(A) N=mg string length is 1 m.
    Column I Column II
(B) F = F1 + F2 ⇒| F |= 102 + 52 + 2.10.5cos120o = 5 3N
(A) Minimum tension in string (in (p) 50
2 Newton)
 mv 2 
(mg)
2
(C) N
= +  (B) Magnitude of acceleration of (q) 60
 r 
  bob when the string is horizontal
(in m / s2 )
(D) None of these
(C) Minimum magnitude of accelera- (r) zero
Q.59 In order to man remain in equilibrium we must tion of bob (in m / s2 )
have (D) Tangential acceleration at the
R highest point (in m / s2 ) (s) 10 65
fs

Q.63 A car of mass 500kg is moving in a circular road


N
of radius 35 / 3 . Angle of the banking of road is 30.
mg
1
Coefficient of friction between road and tires is µ = .
2 3
Match the following:
(A) µmg =
N (B) f2 = µmg
Column I Column II
(C) µN =mg (D) None of these
(A) Maximum speed (in m/s) of car for
safe turning (p) 5 2
Q.60 The value of velocity will be given by –
(B) Minimum speed (in m/s) of car for (q) 12.50
C safe turning
(C) Speed (in m/s) at which friction
force between tires and road is zero (r) 210
D B
(D) Friction force (in 102 Newton) 350
between tires and road if speed is (s)
3
u 350
A m/s
6
rg
(A) v= µrg (B) v =
µ
Previous Years’ Questions
g µg
(C) v = (D) v =
µr r Forces and Laws of Motion
Q.1 In the Figure, the blocks A, B and C have masses 3
Match the Columns
kg, 4 kg, and 8 kg respectively. The coefficient of sliding
Q.61 A particle is suspended from a string of length ‘R’. friction between any two surfaces is 0.25. A is held at rest
by a massless rigid rod fixed to the wall, while B and C are
It is given a velocity u = 3 Rg . Match the following
connected by a light flexible cord passing around a fixed
Column I Column II frictionless pulley. Find the force F necessary to drag C
along the horizontal surface to the left at a constant
(A) Velocity at B (p) 7 mg speed. Assume that the arrangement shown in the
(B) Velocity at C Figure. i.e. B on C and A on B, is maintained throughout.
(q) 5gR
(Take g=10ms2) (1978)
(C) Tension at B (r) 7gR
(D) Tension at C (s) 4mg A

B
F
C
4 . 6 4 | Forces and Laws of Motion

Q.2 A uniform rope of length L and mass M lying on a acceleration of 0.2 m / s2 . The acceleration due to gravity
smooth table is pulled by a constant force F. What is the is g=9.8 m/s2. (1989)
S
tension in the rope at a distance l from the end where
the force is applied? (1978)

Q.3 A block of mass 2 kg slides on an inclined plane which 2.9 kg


makes an angle of with the horizontal. The coefficient
of friction between the block and the surface is 3 / 2
. What force along the plane should be applied to the
block so that it moves (a) down and (b) up without
any acceleration (Take g = 10 m / s2 ) (1978) 1.9 kg

Q.4 Two blocks connected by a massless string slides


down an inclined plane having an angle of inclination Q.8 A hemispherical bowl of radius R=0.1 m is rotating
37o. The masses of the two blocks are M1=4 kg and about its own axis (which is vertical) with an angular
M2=2 kg respectively and coefficients of friction of M1 velocity ω. A particle of mass 10−2 kg on the frictionless
and M2 with the inclined plane inner surface of the bowl is also rotating with the same
M
are 0.75 and 0.25 respectively. 1 ω. The particle is at height h from the bottom of the
Assuming the string to be bowl.
taut, find (a) the common M (a) Obtain the relation between h and ω. What is the
acceleration of two masses 2
minimum value of ω needed, in order to have a non-
and (b) the tension in the zero value of h?
37°
string. (sin37o=0.6,
(b) It is desired to measure (acceleration due to gravity)
cos37o=0.8) (Take g=9.8 m/s2) (1979)
using the setup by measuring h accurately. Assuming
that R and ω are known precisely and that the least count
Q.5 Masses M1, M2 and M3 are connected by strings in the measurement of h is 10-4 m, what is minimum
of negligible mass which passes over massless and possible error ∆g in the measured value of g?
frictionless pulleys P1 and P2 as shown in Figure. The
(1993)
masses move such that portion of the string between P1
and P2 is parallel to the inclined plane and portion of the Q.9 A smooth semicircular wire track of radius R is fixed
string between P2 and M3 is horizontal. The masses M2 in a vertical plane. One end of a massless spring of
and M3 are 4.0 kg each and coefficient of kinetic friction natural length 3R/4 is attached to the lowest point O of
between the masses and surfaces is 0.25. The inclined the wire track. A small ring of mass
plane makes an angle of 37° with the horizontal. m which can slide on the track is
P (1981) attached to the other end of the
1
spring. The ring is held stationary
at point P such that the spring
M makes an angle with the vertical. C
2
P2 The spring constant k=mg/R. P
Consider the instant when the 60°
M1
M3 ring is making an angle 60o with
37°
the vertical. The spring is released O
(a) Draw the free body diagram of
Q.6 A block of mass m rests on a horizontal floor with the ring. (b) Determine the tangential acceleration of the
which it has a coefficient of static friction µ. It is desired ring and the normal reaction?  (1996)
to make the body move by applying the minimum
possible force F. Find the magnitude of F and direction Q.10 Two blocks of mass m1 = 10 kg and m2 = 5 kg
in which it has to be applied. (1987) connected to each other by a massless inextensible
string of length 0.3 m are placed along a diameter of
Q.7 Two blocks of mass 2.9 kg and 1.9 kg are suspended turn table. The coefficient of friction between the table
from a rigid support S by two inextensible wires each of and m1 is 0.5 while there is no friction between m2 and
length 1 m. The upper wire has negligible mass and the the table. The table is rotating with an angular velocity
lower wire has a uniform mass of 0.2 kg/m. The whole of 10 rad/s about the vertical axis passing through
system of blocks, wires and support have an upward centre O. The masses are placed along the diameter of
P hysi cs | 4.65

the table on either side of center O such that the mass and gravity is neglected, then the time after which the
m1 is at a distance of 0.124 m from O. The masses are bead starts slipping is (2000)
observed to be at rest with respect to an observer on the µ µ 1
turn of table. (1997) (A) α (B)
α
(C)
µα
(D) Infinitesimal
(A) Calculate the frictional force on m1 .
(B) What should be the minimum angular speed of the Q 14. A small block is shot into each of the four track
turn table, so that the masses will slip from this position? as shown below. Each of the track rises to the same
(C) How should the masses be placed with the string height. The speed with which the block enters the
remaining taut so that there is no frictional force acting track is the same in all cases. At the highest point of
on the mass m2 ? the track, the normal reaction is maximum in (2001)

Q.11 Block A of mass m and block B of mass 2m are


placed on a fixed triangular wedge by means of a
massless, inextensible string and a frictionless pulley v v
as shown in Figure. The wedge is inclined at 45o to the (A) (B)
horizontal on both the sides. The coefficient of friction
between block A and wedge is 2/3 and that between
block B and the wedge is 1/3. If the blocks A and B
v v
released from rest, find (A) the acceleration A,
(C) (D)
(B) Tension in the string and
(C) The magnitude and direction of the force of friction
acting on A (1997) Q.15 A simple pendulum of length L and mass (bob)
M is oscillating in a plane about a vertical line between
angular limits −φ and + φ . For an angular displacement
A B ( )
θ θ < φ , the tension in the string and the velocity of
m 45° 45° 2m the bob are T and v respectively. The following relations
hold good under the above conditions. (1986)
Q.12 Two blocks A and B of equal masses are released (A) T cos θ =Mg
from an inclined plane of inclination 45o at t=0. Both Mv 2
the blocks are initially at rest. The coefficient of kinetic (B) T − Mgcos θ = L
friction between the block A and inclined plane is 0.2 (C) The magnitude of the tangential acceleration of the
while it is 0.3 for block B. Initially the block A is 2 m bob α = gsin θ
T
behind the block B. When and where their front faces
will come in a line?  (2004) =(D) T Mgcos θ

2m
A Q.16 A reference frame attached to the earth (1986)
B (A) Is an inertial frame by definition
(B) Cannot be an inertial frame because the earth is
revolving round the sun

B (C) Is an inertial frame because Newton’s laws are


45°
A applicable in this frame
(D) Cannot be an inertial frame because the earth is
rotating about its own axis
Circular Dynamics
Q.17 A point mass of 1 kg collides elastically with a
Q 13. A long horizontal rod has a bead which can slide stationary point mass of 5 kg. After their collision, the 1
along its length and is initially placed at a distance L from kg mass reverses its direction and moves with a speed of
one end A of the rod. The rod is set in angular motion 2ms-1. Which of the following statement(s) is/are correct
about A with a constant angular acceleration α .If the for the system of these two masses. (2010)
coefficient of friction between the rod and bead is µ , (A) Total momentum of the system is 3kg−ms −1
4 . 6 6 | Forces and Laws of Motion

(B) Momentum of 5 kg mass after collision is 4kg−ms −1 (C) Radially outwards initially and radially inwards later.
(C) Kinetic energy of the centre of mass is 0.75 J (D) Radially inwards initially and radially outwards later.
(D) Total kinetic energy of the system is 4J
Q.21 A block of mass m1 = 1 kg another mass m2 = 2kg,
are placed together (see figure) on an inclined plane
Q.18 A smooth semicircular wire track of radius R is
with angle of inclination θ. Various values of θ are given
fixed in a vertical plane (Figure). One end of a massless
in List I. The coefficient of friction between the block m1
spring of natural length 3R/4 is attached to the lowest
and the plane is always zero. The coefficient of static
point O of the wire track. A small ring of mass m which
and dynamic friction between the block m2 and the
can slide on the track is attached to the other end of the
plane are equal to µ = 0.3. In
spring. The ring is held stationery at point P such that
List II expressions for the
the spring makes an angle 60o
friction on the block m2 are
with the vertical. The spring
given. Match the correct
constant k=mg/R. Consider the m1
expression of the friction in List
instant when the ring is making m2
II with the angles given in List I,
an angle 60o with the vertical. The
and choose the correct option.
spring is released (a) Draw the C
P The acceleration due to gravity 
free body diagram of the ring. (b)
Determine the tangential 60° is denoted by g.
acceleration of the ring and the O [Useful information: tan (5.5°) ≈ 0.1; tan (11.5°) ≈ 0.2;
normal reaction. (1996) tan (16.5°) ≈ 0.3] (2014)

Q. 19 A small block of mass of 0.1 kg lies on a fixed List I List II


inclined plane PQ which makes an angle q with the
(P) θ = 5° (1) m2g sin θ
horizontal. A horizontal force of 1 N acts on the block
through its centre of mass as shown in the figure. The (Q θ = 10° (2) (m1+m2) g sin θ
block remains stationary if (take g = 10 m/s2) (2012) (R) θ = 15° (3) mm2g cosθ
Q (S) θ = 20° (4) µ(m1 + m2)g cosθ

1N Code:
(A) (P)→(1), (Q)→(1), (R)→(1), (S)→(1)

O P (B) (P)→(2), (Q)→(2), (R)→(2), (S)→(3)
(A) θ = 45° (C) (P)→(2), (Q)→(2), (R)→(2), (S)→(4)
(B) θ > 45° and a frictional force acts on the block (D) (P)→(2), (Q)→(2), (R)→(3), (S)→(3)
towards P.
(C) θ > 45° and a frictional force acts on the block Q.22 The net reaction of the disc on the block is
towards Q. (2016)
(D) θ < 45° and a frictional force acts on the block (A) − m ω R cos ωtjˆ − mgkˆ
2

towards Q.
1
A (B) m ω2R(e2ω t − e−2ω t )ˆj − mgkˆ
2
Q.20 A wire, which passes through
the hole in a small bead, is bent in (C) m ω2R sin ωtjˆ − mgkˆ
the form of quarter of a circle. The 1
(D) m ω2R(e2ω t − e−ω t )ˆj − mgkˆ
wire is fixed vertically on ground as
o
90 B 2
shown in the figure. The bead is
released from near the top of the
wire and it slides along the wire
without friction. As the bead moves from A to B, the
force it applies on the wire is  (2014)
(A) Always radially outwards.
(B) Always radially inwards.
P hysi cs | 4.67

PlancEssential Questions
JEE Main/Boards JEE Advanced/Boards

Exercise 1 Exercise 1
Q. 18 Q.21 Q.26 Q.1 Q.4 Q.13 Q.17
Q.27 Q.28 Q.20 Q.29 Q.38 Q.43

Exercise 2 Exercise 2
Q.5 Q.14 Q.18 Q.3 Q.6 Q.8 Q.11
Q.14 Q.15 Q.16 Q.17
Previous Years’ Questions
Q.44 Q.45
Q.51 Q.59
Previous Years’ Questions
Q.12 Q.14 Q.17

Answer Key

JEE Main/Boards Q.19 750 N/m

Q.20 1 cm
Exercise 1
Q.21 9 m/s.
Forces and Laws of Motion

Q.22 r (iˆ 12500 + ˆj 6250)m
=
Q.4 100 Ns
Q.23 2.5 × 10 4 km
Q.5 12.5 N 2
Q.24 grφ (r − r φ) =2lgm
Q.14 0.872 m/s, 1.744 m

Q.15 30N
Circular Dynamics
−2 Q.25 0.03 m/s2
Q.16 1.4 ms , 33.6 N
Q.26 0.86 ms−2
Q.17 1000 N, 750 N
 
Q.27 − ( x + y ) / 2cm / s
2
Q.18 40°
−1  10 
Q.28 1.22 m/s2; β =tan  
 7 
4 . 6 8 | Forces and Laws of Motion

Exercise 2
Forces and Laws of Motion
Single Correct Choice Type

Q.1 A Q.2 B Q.3 C Q.4 C Q.5 A Q.6 A Q.7 C

Q.8 B Q.9 B Q.10 A Q.11 C Q.12 B Q.13 A Q.14 A

Q.15 A Q.16 D Q.17 C Q.18 C Q.19 D Q.20 B Q.21 B

Circular Dynamics
Single Correct Choice Type

Q.22 D Q.23 C Q.24 D Q.25 A Q.26 C Q.27 B Q.28 A

Q.29 A Q.30 B Q.31 D Q.32 B Q.33 D

Previous Years’ Questions


Forces and Laws of Motion
Q.1 C Q.2 A Q.3 C Q.4 A Q.5 A Q.6 A Q.7 A

Q.8 D Q.9 C Q.10 A Q.11 D Q.12 A Q.13 B Q.14 A


Q.15 B, D

Circular Dynamics
Q.16 C Q.17 D Q.18 C Q.19 C Q.20 B Q.21 B


JEE Advanced/Boards
Exercise 1
Forces and Laws of Motion

Q.1 (a) 70 kg, (b) 35 kg, (c) 105 kg, (d) zero

Q.2 (a) 750 N, (b) 250 N, Mode (b) should not be adopted

Q.3 (a) T=640 N, (b) T=240 N, (c) T=400 N, (d) T=0, Rope will break in case (a).

Q.4 15 m

Q.5 (a) 7500 N downwards, (b) 32500 N downwards, (c) 32500 N upward

Q.6 (a) accelerated with acceleration 0.5 m / s2 , (b) at rest.

(k1 + k 2 ) x
Q.7
m
Q.8 g/3, 2g/3

Q.9 Tab
= mgtan θ, Tob
= mg / cos θ , =
T' mgcos θ
P hysi cs | 4.69

mg 2
Q.10 a + 4h2 acceleration w1
4h Q.24

Q.11 4N kg w2
2(Mg− B)
Q.12 g
t0 time
mg2 cos α
Q.13 (a) =
f (k1 − k 2 ) ,
m1 + m2
µ(m1 + m2 )g
When t ≤ t0 (where t0 = ) w1 =
w2 =
kg
k1 m1 + k 2 m2 a
(b) cos αmin = t > t0 w1 = at / m1 − km2g / m1 , w2 = k g
m1 + m2

1  1 2l
=
Q.14 α tan−1  −  Q.25 t =
2  µ (3 w + kg)

mg2 cos α m2g3 cos α


Q.15 (a) v = , (b) s = Q.27 aA 2.34
= = m / s2 , aB 1.558m
= / s2 , T 81.8N
2
2asin α 6a2 sin3 α
5
Q.28=
(a) t A 1sec,tB
= 2sec, (b) v A = 5m / s (c) m
3 3
Q.16 (a) fA = µMg, fB =0, T =
0
4
µMg µMg
(b) fA =
µMg, fB =, T = Circular Dynamics
2 2
13µ 5
Q.29 fmax = rev / sec
Q.17 2π
16(2 − 3µ )

g(η − sin α − cos α ) Q.30 45°, 2 × 10 4 N


Q.18 w2
= = 0.5g
η+1
Q.31 θ= 30°, v= 7.7m / s
3 Mgsin θ
Q.19 aA = aC = gsin θ,aB = gsin θ, T =
4 8 Q.32 163.3m / s2
(M+ m)gsin2 α
Q.20 f = Q.33 Rg
M + msin2 α
Q.34 22.36m / s
Q.21 Acceleration of block A is g/8 in horizontal direction
and 5g/8 in vertical direction. Acceleration of Q.35 21°29'
block B is g/2 leftwards. Acceleration of block
C is g/8 rightwards µg
Q.36
L
g g
Q.22 aA
= =
2
,aB
1 + η cot α tan α + η cot α Q.37 (a) Rg
πR
mg(sin α + µ cos α ) (b) a distance along the bridge from the
Q.23 tanB =
µ , Tmin = highest point 3
1 + µ2
(c) gR cos (L / 2R )

Q.38 45°

g
Q.39 v =
3
4 . 7 0 | Forces and Laws of Motion

1/2 1/2 1/2


 g ( sin θ − µ cos θ )   g ( sin θ + µ cos θ )  Q.44 2R ( asin θ + g − gcos θ ) 
Q.40   to    
 R sin θ ( cos θ + µ sin θ )   R sin θ ( cos θ − µ sin θ ) 
   
2 2
Q.45 λR ω
2L
Q.41 2
2
ω R cos θ  v2 
2
Q.46 a + 
R 
 
ω2R 4
Q.42 , mω2R
3 3 mv 2
Q.47
2πR
Q.43 (i) m KR (ii) m K R + Kt 4 ( )
( )
1/ 4
1 Q.48  µ2g2 − a2 R 2 
(iii)mKRt (iv) mKRt  
2

Exercise 2
Forces and Laws of Motion

Single Correct Choice Type

Q.1 C Q.2 A Q.3 A Q.4 A Q.5 A Q.6 B Q.7 A


Q.8 B Q.9 B Q.10 C Q.11 B Q.12 A

Multiple Correct Choice Type

Q.14 A, B Q.15 A, D Q.16 A, C Q.17 B, D

Assertion Reasoning Type

Q.18 A Q.19 D Q.20 D Q.21 A Q.22 A

Comprehension Type

Q.23 C Q.24 B Q.25 D Q.26 A Q.27 A Q.28 B Q.29 B


Q.30 B Q.31 D Q.32 D Q.33 C Q.34 D Q.35 B Q.36 A
Q.37 A

Match the Columns


Q38 A → q; B → r; C → r; D → q

Circular Dynamics

Single Correct Choice Type

Q.39 A Q.40 A Q.41 D Q.42 D Q.43 B


P hysi cs | 4.71

Multiple Correct Choice Type

Q.44 B, C Q.45 B, D Q.46 A, B, C Q.47 B, D

Assertion Reasoning Type


Q.48 D Q.49 C Q.50 B Q.51 E Q.52 B Q.53 A Q.54 D

Comprehension Type
Q.55 D Q.56 C Q.57 B Q.58 B Q.59 C Q.60 B

Match the Columns


Q.61 A → r; B → q; C → p; D → s Q.62 A → p; B → s; C → q; D → r
Q.63 A → r; B → p; C → s; D → q

Previous Years Questions

Forces and Laws of Motion


Q.1 37.5 N Q.2 F(1-l/L) Q.3 a=11.21 N, b=31.21N

Q.4 a= 1.5m / s2 , T=5.2 N Q.5 (a) 4.2 kg, (b) 9.8 N Q.6 mgsinθ

Q.7 (a) 20N, (b) 50N Q.8 (a) 9.89 rad/sec, (b) 9.8 × 10−3 m / s2

Q.9 (a) mg/4,


= (b) atan 5=
3g / 8,N 3mg / 8

Q.10 (a) f=36N inwards, (b) 11.67 rad/sec, (c) m2 at 0.2m and m1 at 0.1 m from O

Q.11 (a) 0, (b) T= 2 2mg / 3 (c) f= mg / 3 2 (down the plane)

Q.12 After A travel a distance of 8 2 m down the plane

Circular Dynamics
5 3 3mg
Q.13 A Q.14 A Q.15 B, C Q.16 B, D Q.17 A, C Q.18 B g,
8 8
Q.19 A, C Q.20 D Q.21 D Q.22 C
4 . 7 2 | Forces and Laws of Motion

Solutions

JEE Main/Boards ∆v 20 – 10 10
In case (i) = = = 2 m/s
∆t 5 5
Exercise 1 ∴F = ma ⇒ 5 = m(2)  … (i)

Forces and Laws of Motion Now further, we want this DV in in 2 s.


Sol 1: A body will preserve its velocity and direction as ∆V 20 – 10 10
anew = ∆t = = = 5 m/s2
long as no force acts on it in its motion. Inertia is in fact new 2 2
the resistance of any physical object to any change in
its motion. ∴Fnew = m(5)  … (ii)

Dividing equation (i) by (ii)


Sol 2: Forces are balanced
m(2)
5
⇒ =
Fnew m(5)
2
a = 0 m/s
25
Fnew = N
2
Objects at rest Objects in motion ⇒Fnew ≡ 12.5 N.
(v = 0 m/s) (v  0 m/s)
Sol 6: Conceptual. Refer to the reading manual.

Stay at rest Stay in motion


Sol 7: 1. Linear inertia: In an isolated system, a body at
(same speed and direction)
rest will remain at rest and a body moving with constant
velocity will continue to do so, unless disturbed by an
Now
external force.
Forces are Unbalanced
2. Gyroscopic Inertia: A body that is set spinning has a
tendency to keep spinning in its original orientation if
There is an acceleration no external force is applied.
3. Rotational Inertia: An object resists any change in its
Acceleration depends Acceleration depends state of rotation. If no external force is applied.
on the ‘net force’ Inversely upon the
mass of object Sol 8: Conceptual, Refer to reading manual.

Sol 9: Absolute unit of weight is Newton (N)


Sol 3: While taking a catch, a cricket player moves his
hands backwards. He has to apply retarding force to Gravitational unit is kg-weight.
stop the moving ball in his hands. If he catches the ball
1 N = 9. 8 kg. wt
abruptly, then he has to apply a large retarding force →

for a short time. So he gets hurt. On the other hand if →
dp → dv
he moves his hands backwards then the player applies Sol 10: F ∝ ; F ∝ m
dt dt
force for longer time to bring the ball at rest. In this case →
he has to apply less retarding force. F = Kma, K= 1

∴F = m a
Sol 4: Dp = FDt
Consequences
Dp = 100 . 1 Ns
1. No force is required to move a body uniformly in a
Dp = 100 Ns. straight line.

v f – vi ∆v 2. Accelerated motion is always due to an external force.


Sol 5: F = ma and a = ⇒
t ∆t
P hysi cs | 4.73

Sol 11: Impulse is defined as the product of the average ∴ N ≡ w = m (g – a)


force and change in time.
t2
Hence weight decreases.

J = Favg (t2 – t1) ; J = ∫ F dt Sol 14: writing down the equations of motion
t1
t2
dp dp
F=
dt
; J= ∫ dt dt
t1
p2

J= ∫ dp ; J = P2 – P1 = DP.
p1
T
T1 T1
Sol 12: Every action has an equal and opposite reaction. m2 11

Example (1) 11.5 m1


N Reaction
force
m1g – T = m1a  … (i)
action

mg T

m1 a
Sol 13: Lift moving uniformly

m1 g
a=0
And for second body
m T – m2g = m2a  … (ii)

Then N – mg = 0 T
∴ N ≡ w = mg.
m2 a
Lift acceleration upward
m2 g
a
Adding (i) and (ii)
m (m1 – m2)g = (m1 + m2)a
 m – m2 
a =  1  g
N = mg + ma
m  m1 + m2 
ma(pseudo force) Here m1 = 11.5 kg, m2 = 11 kg, g = 10 m/s2
mg
Now m1 will descend down by height ‘h’ and m2 moves
N ≡ w = m(g + a) up by the same height h;
1
∴ weight Increases H = ut + at2
2
Lift accelerating downwards: 1
⇒ h = 0. t + × (0. 2) (4)2 = 1. 6 m.
2
ma
And for velocity
N
a N + ma = mg v = u + at
m
m v = 0 + (0. 2) (4)
N = mg – ma
mg v = 0. 8 m/s.
4 . 7 4 | Forces and Laws of Motion

T – m2g = m2a … (ii)

Sol 15: 10 31.5 N (i) + (ii) ⇒ (m1 – m2) g = (m1 + m2)a


0.5
 m – m2   4 −3 10
a =  1  g =  = = 1.4 ms−2
 10
 m1 + m2   4 + 3  7
Let us say the whole system moves forward with an
acceleration ‘a’. And using this value, find the value of T in equation (i)
or equation (ii)
 31.5 
Then a =  10 + 0.5  m/s m1g – m1a = T ⇒ T = m1 (g – a)
 
now put m1 = 4 kg m2 = 3kg
a = 3 m/s2
to get the numerical, after putting values of m1, m2 and
Now let us consider the string. a ⇒ T = m1(g − a) = 33.6 N
a
N 31.5 N T T
m = 0.5 kg Sol 17: m2 m1 F

Now, 31.5 – N = ma
⇒ 31.5 – ma = N The total external Horizontal force applied on the
system is F.
N = 31.5 – (0. 5) (3) F
∴ Acceleration ‘a’ of the system= m/s2
N = 30 Newton. m1 + m2
Given a = 5 m/s2
F
Sol 16: Constraint Equation: ∴5 = m/s2 ∴F = 200 ×5 N
50 + 150

F = 1000 N

Now for finding the tension;

T Consider m2
T1 T N
3kg = m2
m2 a m2 T
m1 = 4 kg (pseudo force)
m2 g
am + am = 0. [ length of string is constant]
1 2
T – m 2a = 0
Let us say m1 moves down with an acceleration ‘a’, then
m2 will move up by an acceleration ‘a’. ∵ T = m2a.
T = 150 × 5
T T = 750 N.
m1 a
Sol 18:

m1 g Tcos θ
T1 θ
50 N
m1g – T = m1a  … (i) T1 sin θ P

T2
T

m2 a 60 N

m2 g
P hysi cs | 4.75

At point P In the whole process, linear momentum along the


x-direction is conserved.
For equilibrium;
∴Initial momentum = 103 × 32 + 8 × 103 × 4
T1 sin θ = 50 … (i) ^
Pi = 64 × 103 kg m/s ( i )
T1 cos θ = T2 … (ii)
Now in the final state
And for the mass; ^
Momentum of car=103 × (–8)= – 8 × 103 ( i )
T2 = 60 N … (iii)
50 Momentum of truck = 8 × 103 ( viˆ )
From (i) and (ii) tan θ = ^
50 T2 = 8v × 103 i
tan θ = ^
60 Pfinal = (– 8 + 8v) × 103 ( i )
θ = tan–1(5/6) = 40º
Pinitial = pfinal
Sol 19: F = kx. ⇒ 64 × 103 = (–8 + 8v) × 103
x = 20 cm = 0. 2 m 64 + 8
∴ v= m/s; v = 9 m/sec
8
150 = k (0. 2)
150 15 → →
k= = × 102 N/m = 7. 5 × 102 N/m. Sol 22: F = m a
0.2 2 →

F
a = ; m = 10g = 10 × 10–3 kg = 10–2 kg
m
Sol 20: T – mg = ma →
(10iˆ + 5ˆj)
T ∴a =
10 –2
m a →
a 103 ˆi + 5 × 102 ˆj
=
Now for amax, we have Tmax


1→ 2
Tmax – mg = ma r = ut+ at
2
Tmax = m (g + a) N = m (9. 8 + 1. 2) N = 2000 (11) → → 1→ 2
Since u = 0, r = at
2
= 22 × 103 N
Tmax →
103 × 25 500 × 25 ˆ
u = î + j
Now Tmax = (Breaking stress) Area 2 2

∴ 22 × 103 = (2. 8 × 108) (pR2) Sol 23: This is just an energy conservation problem on
surface of earth;
22 × 103
R= 1 Gm
28 × 107 π Ei = mv 20 + Ui ; Ui = –
2 R
R= 25 × 10 –6 m 1 Gm
∴Ei = mv 20 –
R = 5 × 10 m –3 2 R

Diameter = 2R ≡ 10 × 10–3 m ≡ 10–2 m. Now finally;


V=0
Sol 21: Before collision  Gm 
Ef = 0 +  –   … (i)
32m/s 4 m/s  R +h
1000 8000
car Truck And Ef = Ei
1 Gm Gm
After collision, ∴ mv 20 – =–
y 2 R R +h
1000 8000 u v 20 G –G
8 m/s x ⇒ – =
2 R R +h
4 . 7 6 | Forces and Laws of Motion

1 1 v
2
r – r' Gm
⇒ = – 0 ∴ =
R + h R 2G 2 (r ')2 g

1 Solving for r’, We get the value of r’.


⇒R+h=
 1 v2 
 – 0
 R 2G  Circular Motion
 
Sol 25:
1
h= –R
 1 v2  w
 – 0
 R 2G 
 

G = 6. 67 × 10–11 m3kg−1s−2 A
R = 64 × 10 m. 5

v0 = 104 m/s
After putting above values we get, h = 2.5 × 104 km

Sol 24: m1 = m2 = m Earth completes 1 rotation in 1 day


rotation
i. e. , ω = 1.
day
θ θ 2π
ω = 1. rad/s
T T 24 × 60 × 60
F π
m2 ω= × 10–2 rad/s
m1 mg 432
and now acceleration at point A;
FBD of m1;
T cos θ a = rω2
r = 6400 km = 6400 × 103 m; r = 64 × 105 m
Gm2
T sin θ F= π2
(r')2 ∴ a = 64 × 105 × × 10–4 m/s2
(432)2
mg a = 0. 03 m/s2

T sin θ = F  … (i)
5
Sol 26: v = 27 km/h = 27 × m/s
T cos θ = mg  … (ii) 18
Gm2 15
(i)/(ii) ⇒ Tan θ = v= m/s
(r ')2 mg 2
Gm

v2 (15)2
tan θ =  … (iii) ar = = = 0. 7
(r ')2 g R 4 × 80

1
r – r' a t = 0. 5 m/s2 = m/s2
tan θ = 2
2 → → →
a net = a r + a t = (0.7)2 + (0.5)2

a net = 0. 86 m/s2
θ

r – r1
2
P hysi cs | 4.77

Sol 27: y Sol 28: B



Q
A
→ O
π/4 a →

x ar
O P →
at
C

A
At point the acceleration will be centripetal acceleration
which is radially directed towards point O. i.e. Let us say the circular turn is of the shape AB.

v 2 ê Now at the starting point of the track i. e. C;
Physically: a = (– r )
r → → →
a = ar + a t
Remember êr and êt are the  êt
unit vectors along radial and
→ v2
êr ar = centripetal acceleration = (– êr )
tangential directions respectively. A R
5
Refer to the figure. O v = 36 km/h = 36 × m/s = 10 m/s
18
v 2 ê

So in this case also a A = (– r ) R = 140 m
r
Now, since the point is in between the points P and Q, → (10)2 5
ar = = m/s2 (– êr )
y 140 7
dv
and given that = 1 m/s
dt
x → dv →
π/4 ∴ at = ( ê ); a t = 1 m/s2( êt )
dt t
→ → →
Now a = a r + a t
– êr

  π a = (0. 7 (– êr )) + 1 êt ) m/s2
angle between OA and OP will be
4
|a| = (0.7)2 + 1 = 0.49 + 1 = 1.49 m/s2 = 1. 22 m/s2
Now let us resolve (– êr ) into î and ˆj .
 1   10 
π π and tan β =   ⇒ β =tan–1  
(– î ) + |– êr | sin (– ˆj )
(– êr ) = |– êr |. cos  0.7   7 
4 4
But since êr and êt are unit vectors
Exercise 2
| êr | = | êt | = 1
1 1 ˆ 1
 (– êr ) = – î – j = ( î + ˆj ) Forces and Laws of Motion
2 2 2
→ Single Correct Choice Type
v2  1 ˆ ˆ 
Now a A = − (i + j) 
r  2  Sol 1: (A) At point A;

v2 ˆ ˆ
aA = – (i + j)
r 2
→ (1)
Put v = 2 cm/s and r = 4 cm, to find a A .

After putting above values we get, aA =− ( xˆ + yˆ ) / 2 cm / s2 T1 A
T1=T2
T2 (2)

m = 10 kg
4 . 7 8 | Forces and Laws of Motion

At point B; T1 Using (i) and (iii) here,


mg=T2
0 + (mg)2 ≤ F ≤ (mg)2 + (µmg)2
mg T =10g ≡ T
2 1
mg ≤ F ≤ mg 1 + µ2
∴ Both the spring show a reading of 10 kg
Sol 4: (C) Tension will always act along the length of the
Sol 2: (B) Here acceleration of the lift is 12 m/s which
2 string and opposing the applied force.
is greater than ‘g’. In option B,

F
2
12m/s

at t = 0 Tension has to act opposite to the applied force, but


there is no string after the end point. Hence the string
The body will undergo a free fall condition. Actually the collapses.
body loses the contact with the floor of the lift.
1 1 In option C, F
∴ s = g t2 = × 10 (0. 2)2 m T
2 2

S = 20 cm.
The tension in the string acts towards the body, thus
Sol 3: (C) Here we need to understand the concept of making the string tough. Hence this is the correct
friction representation.
N
Sol 5: (A) Initially; F is upward Buyoant force
F1
a
f
mg
Mg – F = Ma
We are given that the body is not moving. Hence mg
balancing the forces in both the directions; Mg – Ma = F  … (i)

N – mg = 0  … (i) Now when the mass ‘m’ is released,

F1 – f = 0  … (ii) Balloon starts rising upwards with an acceleration ‘a’.

⇒ N = mg and f = F1. F
Now we don’t know anything about F1.
a
But we know that the force F1 must be less than maximum
static friction i.e. mmg for the body to be at rest.
∴ f = F1 ≤ mmg. And minimum F1 can be zero. (M–m)a
∴ 0 ≤ f ≤ µ mg  … (iii) F – (M – m)g = (M – m)a  … (ii)
Now we know that contact force on the body is Solving (i) and (ii); we get
F= 2
N +f 2
 2a 
m=  M
a+ g
N
Sol 6: (A) Let us assume that the string makes an angle
of ‘θ1’ with the normal of the plane.

f
P hysi cs | 4.79

B→a
m'
θ

a
θ m
A
The only external force acting on the sphere is ‘mg’ FBD of B;
which is vertically downward. Hence the string also
becomes vertical so as to balance the force mg. 
N N
N – m'g = 0 …(ii)
N – m'g = 0 …(ii)
T ∴ T ∴
Sol 7: (C) F. B. D of (1) T – m'a = 0 T – m'a = 0
…(iii) …(iii)
T  ... (i) m’g m’g
T–mg=0
m a a
⇒ T = mg …(i)

mg Using (i) and (iii)


mg = ma + m’a [(i) + (iii)]

 m 
2T cosθ a=  g
 m + m' 
T θ θ θ
T T T T T
m m m 2 Sol 10: (A) Now, the force required to just start the
m 2
motion would be the static friction (fs)
F
(1) (2) (3) f
∴ F = fs = ms mg
2T cos θ – m 2g = 0  … (ii)
i.e. after this point the body starts moving.
From (i) and (ii): 2 (mg) cos θ = m 2g
When the body is moving, kinetic friction acts on the
1 π body (i. e μk mg)
Cos θ = ⇒θ=
2 4 FBD of the body;
F F – fk = ma
Sol 8: (B) The reason for small steps is that the lateral fk
forces are decreased. Imagine taking a large step on → a ms mg – mk mg = ma
concrete. When you put your foot down well in front of
you, it will be pushing forwards on the concrete. And at ⇒ (ms – mk) mg = ma ⇒ a = (ms – mk) g
the end of that step, when that foot is well behind you,
it will be pushing backward on the concrete. The larger a = 0. 98 m/s 2

the step, the larger there forward and backward forces.


Sol 11: (C) Newton’s second law states that the net
Our shoes on Ice can only provide or sustain small force on an object is equal to the rate of change of its
forward/backward forces, before they slip. Hence we try linear momentum.
to reduce the friction. → →
dp
→ d(m v )
⇒F = =
Sol 9: (B) FBD of A; dt dt
if m is constant, then
T →

dv
a mg –T= ma  …(i) ... (i) ≡ m. ≡ ma
dt
mg
4 . 8 0 | Forces and Laws of Motion

Sol 12: (B) FBD of the body; Sol 14: (A)


f
F (Net Air Resistance); θ
N
u(speed) a
mg

mg + F = ma Mg cos α
Mg sin α mg
F
a=g+ ; which is downwards. (i. e opposite) to the
m Given that insect moves very lowly;
direction of displacement (till it reaches maximum ∴ V = 0; Acceleration of the body is also zero.
height)
f = Mg cos α
Since mA > mB; aA < aB
N = Mg sin α
i.e. Body ‘A’ has less downward acceleration when
composed to Body ‘B’. Hence A will go higher than B. Now for the maximum case;
f = fs = mN.
Sol 13: (A) Let ‘x’ be the maximum length that can hang
∴ mN = Mg cos α
hand from the table.
µ (Mg sin α) = Mg cos α
Now say fs be the static friction
1
tan α = ⇒ tan α = 3
L–x µ
But we want to express in terms of θ;

x α + θ = 90o, → α = 90o – θ
tan α = tan (90o – θ)
3 = cotq
M
fs =   . x. g  … (i)
L Sol 15: (A) T θ
T
[ Condition for Equilibrium]
And also we know that fs = mN. w
M
N = (L – x)g When the bird alights on the wire; the wire makes a
L
curve of small angle.
µM
fs = (L – x) g  … (ii) 2 T sin θ = w
L
from (i) and (ii) W
sin θ =  
 2T 
Mx M
g = µ (L – x) g
L L we know that sin θ ≤ 1
x  µ  W  W
⇒ < 1 ⇒T > 
⇒ =   2T  2 
L 1 + µ 

x   µ  1/4 Sol 16: (D) Now Balancing the forces parallel and
 × 100  =  × 100  = × 100 = 20%
L   1 + µ  5/4 perpendicular to the incline surface;
f = mg sin θ
N = mg cos θ
And Net force by surface = f 2 + N2

= (mgsin θ)2 + (mgcos θ)2 = mg = 30 N.


P hysi cs | 4.81

Sol 17: (C) While descending down; (ii) when the body just starts sliding and slides down
The fireman tries to pull the rope down and so there will For case I;
be a tension ‘T’ upwards. N
T F
mg – T = ma; mg – ma = T
2mg mg a
Now given Tmax = Mg sin θ
3 mg cos θ
mg
2mg θ
∴ amin = mg – m ∴ amin = g/3
3
As long as body doesn’t slide;
Sol 18: (C) F = mg sin θ;

40 sin 30º N = mg cos θ

40 ∴ F= f 2 + N2 = mg
50 sin 30º
50 ∴ It remains constant till a particular ‘θ’.
30º 30º For case II;
50 cos 30º
w=5 When the body is sliding down,
f = mN
30
N = mg cos θ
A  2 
50 ∴F = (µN)2 + N2 = N  µ + 1 
 
30N
 2 
C = mg cos θ  µ + 1 
w  
40 As θ increases; cos θ decreases.
B
Hence F decreases.

Fnet = 90 sin 30º – (30 + 5) = 45 – 35


Sol 21: (B) (l–n)
Fnet = 10 N upwards

Sol 19: (D)


2F nL
F
F = 250
F
2F F
m
λ (mass per unit length) =  
L m
θ = 15º Now mass of the part which is hanging= (nL)   = nm
L
And mass of the part which is on the table = (1 – n)m

3F – mg sin α = ma Now total downward force = (nm) g ≡ nmg.


This force has to be balanced by the frictional force
a =  3F – gsin α 
 
m  which is mN ≡ µ [(1 – n) mg]
250 × 3 ∴ µ (1 – n) mg = n mg
a= – 10(0.26) = 7.5 – 2.6 m/s2 =4.9m/s2
100
 n 
µ=  
Sol 20: (B) Here in the problem, two cases arises; 1 – n

(i) when the body is at rest


4 . 8 2 | Forces and Laws of Motion

Circular Dynamics mv 2
Sol 26: (C) Centripetal force =
R
Sol 22: (D) Force acting on the particle at any instant is 5
v = 36 km/hr = 36  1000 m / s  = 36
 
mRω2 towards the center. m/s
→  3600  18
ds v = 10 m/s
2
F = (500)(10)
P 50
F F = 1000 N

i.e. F = mRw2 êr [ Radial dire ction]
Sol 27: (B) Use F = mrω2
And the displacement of the particle will be ‘ds’ along
tangential direction. Sol 28: (A) N
→ P
i.e. d s = ds êt
→ → θ
Now work = F . d s A
mg
W = mrw ds ( eˆ r .eˆ t )
2
θ

W = Zero (As eˆ r ,eˆ t are perpendicular to each other) O

Hence the work done by the Centripetal force is zero.

mv 2
Sol 23: (C) Centripetal force = mRw2 N = mg cos θ –
R
C
As one goes from A to P; θ decreases, so cos θ increase.
R ∴ N increases
D B

k
A Sol 29: (A) Centripetal force = –
r2
mv 2 k
Now at any point in the circle this value remains the ⇒ =–
same. Its only that the direction keeps changing. r r2
k
⇒ mv2 = –
Sol 24: (D) In uniform circular motion, ω is constant r
Now in the options, A, B, C the quantities are constant 1 k
⇒ mv2 = –
in magnitude but keep changing in direction. 2 2r
And since they are vector Quantities, we can’t say they k
⇒ kinetic energy K = –
are constant. For speed, its only magnitude that matters. 2r
Since it’s a Scalar Quantity. And since the motion is horizontal motion; let us assume
And Speed = Rω  Constant the potential energy same as that of ground i.e. zero
k
Hence option D. ∴ total energy = K + U = – +0
k 2r
E=–
Sol 25: (A) m1 = m2 = m ; v1 = v2 = v 2r

m1 v12 mv 2 Sol 30: (B)


Now F1 = = N cosθ
r1 r1 N
m2 v 22 mv 2
F2 = = mv 2
r2 r2 N Sinθ=
R
F1  r2 
=   θ
F2  r1  mg
P hysi cs | 4.83

mv 2 mvB2
N sin θ = TB = mg cos θ +
R R
N cos θ = mg mv 2A mvB2
Now TA – TB = –
v2 R R
tan θ =
Rg m 2
1 TA – TB = ( v – vB2 )  (i)
θ = tan–1= tan–1 =tan–1   R A
5 Now using conservation of energy theorem;
1
Sol 31: (D) At point A; EA = mv 2A + UA
2
v2
1
At point B; EB = mvB2 + UB
2
B EA = EB
T
1
v1 m( v 2A – vB2 )= UB – UA
A 2
mg But we can observe that both points A and B are at
same heights from the center.
In a vertical motion, the speed of ball doesn’t remain m 2 2
∴ UA = – UB  TA – TB = . (U – UA) = (UB – UA)
constant and as we discussed earlier, centripetal force R m B R
can’t be constant in direction itself, so its ruled out.
∴ is constant
And for tension, consider two points A, B
Sol 33: (D)
→  mv 2A  ˆ → mvB2
T A =  mg +  (– j ) and T B = ( î )
 R  R mv 2
 
R
Hence tension is also not constant. Now gravitational
force on the ball is (mg) at any point on the circle.
N
mg
Sol 32: (B) At point A

mv 2A
R
mv 2
mg cosθ mg + N =
R
TA
R
v= (mg + N)
A y m
θ θ Now for minimum case; let us say he just loses contact
mg
x i.e. N = 0

B ∴v= gR . This is the minimum speed.


θ
mg
mv 2A Previous Years’ Questions
TA + mg cos θ =
R
mv 2A Forces and Laws of Motion
TA = – mg cosθ
R
and for point B, F 5 × 10 4 5
TB Sol 1: (C) a = = 7 = × 10–3 m/s2
m 3 × 10 3
Mg cosθ
mvB2 5
v= 2as = 2 × × 10 –3 × 3 = 0. 01 m/s
R 3
4 . 8 4 | Forces and Laws of Motion

Sol 2: (A) Since, mg cos θ > mg sinθ Therefore,


∴ force of friction is f = mg sinθ Ft = ma = mαL = N
∴ Limiting value of friction
Sol 3: (C)
(fr)max = mN = mmαL  … (i)
Angular velocity at time t is ω = at
θ
T ∴ Centripetal force at time t will be

θ Fc = mLw2 = mLa2t2  … (ii)


Equating equation (i) and (ii), we get
µ
mg t=
α
mv 2
FBD of bob is T sinθ = µ
R For t > , F > (fr)max i.e. , the bead starts sliding.
and T cos θ = mg α c
v2 In the figure Ft is perpendicular to the paper inwards.
(10)2
∴ tan θ = Rg =
(10)(10)
Sol 6: (A) Since, the block rises to the same heights in
tan θ = 1 or θ = 45° all the four cases, from conservation of energy, speed of
the block at highest point will be same in all four cases.
Say it is v0.
Sol 4: (A) N = 5N
(f)max = mN= (0. 5)(5) = 2. 5 N V0

5N N + mg

N Equation of motion will be


mv 20 mv 20
N + mg = or N= – mg
w = 0.98 N R R
R (The radius of curvature) in first case is minimum.
For vertical equilibrium of the block Therefore, normal reaction N will be maximum in first
F = mg = 0.98 N < (f)max case.
Note in the question it should be mentioned that all the
Sol 5: (A) Tangential force (Ft) of the bead will be given four tracks are frictionless. Otherwise, v0 will be different
by the normal reaction (N), while centripetal force (Fc) in different tracks.
is provided by friction (fr). The bead starts sliding when
the centripetal force is just equal to the limiting friction. Sol 7: (A) Equilibrium of insect give
ω N = mg cosa
mN = mg sina
L Ft
A x α
Fc µN N

α
Ft is inwards mg sinα
mg cosα
mg

From Equation (i) and (ii). We get


cotα = 1/µ = 3
P hysi cs | 4.85

Sol 8: (D) Sol 10: (A) Free body diagram (FBD) of the block (shown
by a dot) is shown in figure.
T=Mg
N Vertical

T= Mg
f
F cos 60° Horizontal
mg

Free body diagram of pulley is shown in figure. Pulley is


mg+ F sin 60°
in equilibrium under four forces. Three forces as shown
in figure and the fourth, which is equal and opposite to
the resultant of these three forces, is the force applied For vertical equilibrium of the block
by the clamp on the pulley (say F). F
N = mg + F sin 60° = 3 g + 3  ... (i)
2
Resultant R of these three forces is For no motion, force of friction
R = ( (M + m)2 + M2 )g f ≥ F cos 60°
Therefore, the force F is equal and opposite to R as or mN ≥ F cos 60°
shown in figure.
1 3F F
or ( 3g + )≥
∴ F = ( (M + m)2 + M2 )g 2 3 2 2
F
F
or g ≥ or F ≤ 2g or 20 N
Mg 2
Therefore, maximum value of F is 20 N.
R
Sol 11: (D) This is the equilibrium of coplanar forces.
Mg+mg Hence,
ΣFx = 0
Sol 9: (C) Free body diagram of m is
T ∴ F=N
∴ ΣFy = 0, f = mg
 
Stc = 0 ∴ τN + τf = 0

∴ Since, τf ≠ 0

mg ∴ τN ≠ 0

T = mg  ... (i) Sol 12: (A) Initially under equilibrium of mass m


Free body diagram of mass 2 m is T = mg

T T Now, the string is cut. Therefore, T = mg force is


θ θ decreased on mass m upwards and downwards on mass
2m.
2m mg
∴ am = = g (downwards) and
m
mg g
a2m = = (upwards)
2m 2
2 mg
2T cos θ = 2 mg  … (ii)
Dividing Eq. (ii) by Eq. (i) we get
1
cos θ = or θ = 45°
2
4 . 8 6 | Forces and Laws of Motion

Sol 13: (B) y and T cos θ = mg


v2 (10)2
∴ tan θ = =
Rg (10)(10)
N
tan θ = 1 or θ = 45°
θ

x Sol 17: (D)


mg 90°– θ

θ
N sin θ = mg
N cos θ = ma
g T
tan θ =
a θ
a dy
cot θ = = tan(90° – θ) = = 2kx
g dx C
a
∴x= mg
2kg r

R =  sinθ
Sol 14: (A) When
T cosθ component will cancel mg.
P = mg (sin θ – µ cos θ)
T sinθ component will provide necessary centripetal
F = µ mg cos θ (upwards)
force to the ball towards centre C.
when P = mg sin θ
∴ T sinθ = mrw2 = m ( sinθ) w2
f=0
or T = mlw2
and when P = mg(sin θ + µ cos θ)
T
f = mmg cos θ (downwards) ∴ ω=
m
Hence friction is first positive, then zero and then
negative. Tmax 324
or wmax = = = 36rad/s
m 0.5 × 0.5
∴ Correct option is (A).

Sol 15: (B,D) A rotating/revolving frame is acceleration −bt


and hence non-inertial. Therefore, correct options are (B) Sol 18: (C) F = F0 e
and (D). F F0 −bt
⇒ a == e
m m
Circular Dynamics
dv F0 −bt
Sol 16: (C) ⇒ = e
dt m
t
F −bt
θ
⇒ ∫ dv = ∫ me dt
T 0

F  −1   −bt 1
θ ⇒ v=   e 0
m b 

F  −bt 
mg ⇒ v= e
mb  
mv 2
FBD of bob is T sinθ =
R
P hysi cs | 4.87

=v 0=at t 0 In this case,

F N
and v→ as t → ∞
mb u = 10 m/s
man
So, velocity increases continuously and attains a a=0
F
maximum value= of v as t → ∞ mg
mb
N – mg = 0 ⇒ N = mg = 70×10 = 700 Newton.
Sol 19: (C) mg sin θ = µmg cos θ
tanθ = µ ⇒ reading by the scale = 70 kg
Case (II)
dy 1
⇒ = tan θ = µ = In the frame of the lift;
dx 2
ma
x2 1 1 N
⇒ = , x = ±1 ⇒ y= m (pseudo
2 2 6 force) man a = 5 m/s2

Sol 20: (B) Normal force on block A due to B and mg


between B and wall will be F.
Friction on A due to B = 20 N ⇒ N + ma = mg

∴ Friction on B due to wall = 100 + 20 = 120 N ⇒ N=m(g−a)


⇒ N = 70 (10 – 5)
Sol 21: (B) Since work done by friction on parts PQ and
⇒ N = 70 × 5 N
QR are equal
N = 350 Newton
3 (QR=x)
−µmg × × 4 = −µmgx ⇒ Reading by the scale = 35 kg
2


= x 2 3 m ≈ 3.5m Case (III)
N
Applying work energy theorem from P to R
m a
0 3
mg sin 30 × 4 − µmg × 4 − µmgx = 0
2
1 mg ma
⇒=µ ≈ 0.29
2 3 N = mg + ma
⇒ N = m(a + g)
⇒ N = 70 (10 + 5)
JEE Advanced/Boards
⇒ N = 70 (15)
Exercise 1 N = 1050 Newton
Forces and Laws of Motion ⇒ reading by the scale = 105 kg
Now In this case a = g downward,
Sol 1: The reading shown by the weighting scale is the
normal reaction between the man and the weighing ∴ from case (b);
scale. ⇒ N = m (g – a)
Now, in Case (I) ⇒ N = m(g – g)
N=0
i. e the man is in free fall.
4 . 8 8 | Forces and Laws of Motion

Sol 2: T – mg = ma
T m2 T = m (g + a)
Here rope tries to pull
T T = 40 (10 + 6)
the man down.
T = 640 N
T
m But Tmax = 600 N, hence the string breaks.

Case b:
N
T T
m1 T = m1g T + m2g = N a =4 m/s2

m1g T m2g mg

mg – T = ma
⇒ N = (m1 + m2)g …(i)
T = m(g – a)  … (i)
In case II;
T = 40 (10 – 4)
Now rope pulls the man up
T = 40 × 6
N T=240N
T
T
T < Tmax
T m2 m1 m1 g = T
Case c:
m1 g m2 g u = 5 m/s uniformly i.e. a = 0
T
T = mg = 40 (10)
m1
T = 400 N
T + N = m 2g
T ≤ Tmax
⇒ N = m2g – T
⇒ N = m2g – m1g Case d:

⇒ N = (m2 – m1)g In this case;

Hence normal force is less in second case. Put a = g in case (b)


We get t = m (g – a)
Sol 3: In climbing the rope, monkey tries to pull down
T = m (g – g)
the rope, and the rope pulls the monkey upwards.
T=0

Sol 4: Now with respect to the truck; forces on the mass


m =40 kg ‘m’ are
μ = 0.15
∴ On monkey; T ma (pseudo force)
a
5m

mg

Now in Case (a) N


T
m ma
a =6 f

mg mg
P hysi cs | 4.89

ma – f = ma’ ∴ Force on the floor by the crew is 7500 N downwards.


N = mg
Sol 6:
And f = mN = mmg µ= 0.18
ma – mmg = ma’
a = 0.5 m/s 0<t ≤ 20
a’=a−µg
a’ = 2 – (0.15) (10) a=0 t > 20
A’ = 2 – 1.5
For an observer on ground, this is how he depicts the
A’ = 0.5 m/s2 FBD of mass,
Now this fairly a relative motion problem
m
Box has to cover a distance of 5 m to fall off from the f
truck;
1 1 f = ma
s = 0t + at2 ⇒ 5 = (0.5)t2
2 2
Now let us check for any sliding.
T= 20 s
f ≤ fs … (i) [Condition for no sliding]
Now in the meantime, distance traveled by the truck in
fs = mmg = (0. 18) (15 × 10) = 27 N.
1
s = (2)(20) =20m and f = ma ≡ 15(0. 5) = 7. 5 N.
2
hence no sliding.
∴Distance from the starting point where the box lands
is 15m. The observer will find the body to move with acceleration
of 0. 5 m/s.
Sol 5: Now since there is no sliding, there is no relative motion
F
w. r. t. the trolley.
Helicopter
Hence observer on trolley will find the mass to be at
M 15 m/s2
rest.
m Crew
k1 k2
Sol 7: m
(M + m)g
F is the force on helicopter due to the surrounding air
∴ F – (M + m)g = (M + m)a k1 k2
m
⇒ F = (M + m)g + (M + m)a
⇒ F = (M + m) (g + a) x
⇒ F = (1300) (25) N = 32500 N upwards k1x k2x
m
Now using newton’s third law, force by helicopter on
surrounding air is F downward, i.e. 32500 downwards. k1x + k2x = ma

Now if we consider the crew, (k1 + k 2 )x


a=
N m

mg ma

⇒ N = m (a + g) = 300 (25)
N = 7500 N upwards.
4 . 9 0 | Forces and Laws of Motion

Sol 8: T2

T2sinθ A T1

T1 mg
a C T1
T2 cos θ = mg  … (i)
B T2 sin θ = T1  … (ii)
a
T2 T2=mg sec θ
T2 T1
From (ii) & (i) ⇒ tan θ =
A mg
⇒T=mgtan θ
mg Now just after the string AB is burnt,
T1 T2 = mg cos θ

a C
θ
mg
T2
T1 – mg = ma  … (i)

T1 θ
mg cos θ
a B mg

T2
Sol 10:
mg
T d T

T2 + mg – T1 = ma  … (ii) h
T
θθ T
T2

a A
d
mg θh
mg – T2 = ma  … (iii)
now, (i) + (ii) 2T cos θ = mg
gives T2 = 2ma mg
=T sec θ
2
Now using this is equation (iii)
2
g 2g 2 d
a= and T2 = h + 
3 3 2
sec θ =
h
O
Sol 9: mg h2 + (d / 2)2
∴T= .
θ 2 h
T2 cos θ
∴ We can see that when h decreases, T increases.
T2
T2 sin θ T1
m A
B
P hysi cs | 4.91

Sol 11: FBD of A; N2 = mg cos a  … (iv)


N m2g sin α – N – f2 = m2a … (v)
ma
f2 = k2N2  … (vi)
A 2 m/s2
A Now (i)/(v)
B
mg N f N f
g sin α+ – 1 = g sin α – ,– 2
m1 m1 m2 m2
mg = N + ma
Solving; f1 = k1m1 g cos a
⇒ N = m (g – a)
f2 = k2 m2g cos α
⇒ N = 0.5 (10 – 2)
1
⇒ N = (8) = 4 newton. gcos α(k1 – k 2 )m1m2
2 ∴ N=
(m1 + m2 )
Sol 12: Initially, Adding (i) + (v)
FB + FA = mg  … (i) (m1 + m2) g sin α – (f1 + f2) = (m1 + m2) a
But for just sliding case, a = 0
FB
(m1 + m2) g sin α = f1 + f2
v f1 = k1N1; f2 = k2 N2
m
FAir resistance = kv ∴ (m1 + m2 g sin α = k1m1 g cos α + k2 m2 g cos α
mg
k1m1 + k 2m2
Let us say mass ‘m’ is removed to achieve case b; finally; tan α =
m1 + m2
Fair = kv FB
Sol 14: mg sin α – f = ma
v
n f = µ mg cos α [ µ N]

(M–m)g f

(M – m )g + FA = FB  … (ii) mg sin α
From equation (i) and (ii), eliminating FA;
2(Mg – B) α
We get m =
g

mg sin a – µ mg cos α = ma
Sol 13: a N1 N
2
f1 1 ∴ a = g sin α – mg cos α
1
mg1 sin α a = g(sin α – µ cos α)
m1g cos α
Now time taken by the block to reach point O;
m1g sin α + N – f1 = m1a  … (i) 1
s= ot + at2 ∴ s =  cos α
N1 = mg cos α  … (ii) 2
1
f1 = m1N1 = k1N1  … (iii)  cos α = g (sin α – µ cos α) t2
2
N2 2 cos α
t=
f1 g(sin α – µ cos α )
N 2
m2g sin α for minimum t;
dt
m2g sin θ = 0.

4 . 9 2 | Forces and Laws of Motion

1  –1  Sol 16: First let us calculate the limiting friction on


We get α = tan–1   blocks ‘A’ and ‘B’.
2  µ 
µ B A µ
Sol 15: f cos α = ma  … (i)
f sin α + N = mg  … (ii) fsA = µ mg

F sin α fsB = µ mg
3
F Now when a force of µ mg acts on the block A; it
4
α
F cos α doesn’t cause any motion in A.
3
Hence; F = fA = µ mg
4
mg
And string is left unaltered. Hence tension is zero. And
Now at the moment, contact is lost;
hence fB = T = zero
N=0
3
(b) Now when force of µ mg is applied,
F sin α = mg 2
Body A will tend to move forward. (F ≥ fs)
at0sin α = mg
Let us assume that the whole system moves with on
 mg  acceleration ‘a’.
t0 =  
 asin α  a
dv
now F cos α = ma ≡ m
dt
dv T T
∴ at cos α = m B A F
dt
fB fA
Integrating on both sides
t0 v
On body A;
∫ (acos α)t dt = m∫ dv
0 0 a

acos α t02 N
. = vm … (i)
2 2
T F
acos α m2g2 mg2 cos α fA
⇒v= . ⇒ v =
2m a2 sin2 α 2asin2 α mg

We see that in equation (i) F – T – fA = ma  … (i)


acos α 2 mg = N  … (ii)
⇒v= t
2m fA = µ mg  … (iii)
dx acos α 2 On body B;
v= = t
dt 2m N
acos α 2 T
dx = t dt
2m fB

Integrating on both sides; mg

x t0 t0
acos α 2 acos α t3 T – fB = ma  … (iv)
⇒ ∫ dx = ∫ 2m
t dt ⇒x=
2m 3
0 0 0 fB = µ mg  … (v)
3
 acos α   mg  Adding (i) and (iv);
x=    
 6m   asin α  F – (fA + fB) = 2ma
P hysi cs | 4.93

3 µ mg – (2 µ mg) = 2 ma 13
⇒ t2 =
2 2(2 – 3µ )g
a is negative
1 2
Distance travelled by B = at
It means that our assumption that both the bodies 2 B
move is false.
1 µg 13 13µ
= × . =
T F 2 4 2(2 – 3µ )g 16(2 – 3µ )
fA

F – T – fA = 0 Sol 18: mass m2 = hm1

3 Friction force on m1 f = km1 g cos a


T = F – fA= µ mg – µ mg
2 Gravitational force on m1, f1 = m1 g sin α
mg
T= µ m2g – f – f1
2 acceleration a =
m1 + m2
Now we can see that
ηm1g – km1gcos α – m1gsin α
=
T m1 + ηm1
fB
µ mg g(η – k cos α – sin α )
T = fB = ≤ fs ≡ µ mg a=
2 (n + 1)

∴ Block B cannot move. Since they both are connected


to each other, even A can’t move. Sol 19: By constrains of string,
Acceleration of A equals to B

Sol 17: Length of block A =
4 ⇒ aA = ac = a
⇒ Distance travelled by A relative to B ⇒ (mA + mC) a = (mA + mC) g sin θ – µ (mC) g cos θ
3 1
= +   ⇒ 2 ma = 2 mg sin θ – µ mg cos θ
4 44
1 3
13 a= (2g sin θ – tan θ g cos θ) = g sin θ
l0 = 2 4
16 3
∴ aA = aC = g sin θ
Let mass of A be mA = m 4
Now for B, tensions of string cancel each other and no
mc = m; mB = 4m friction exists.
friction force = μmAg Hence the only acceleration is due to gravity
f µmA g µg
Acceleration of B aB = = = ∴ aB = g sin θ
mb mB 4

mc g – µmA g Sol 20: m will have acceleration vertically downword.


Acceleration of A aA = Let call it am.
mc + mA
mg(1 – µ ) (1 – µ ) M will have acceleration along inclined plane lets call
= = g is aM
2m 2
Relative acceleration a = aA – aB  m, M have no relative acceleration vertically
downward,
g(1 – µ ) µg g
= – = (2 – 3µ ) aM sin α = am  … (i)
2 4 4
1 2 Let normal force on block be N,
at = l
2 mg – N = m am
1 g 13
∴ (2 – 3 µ) t2 = N = m (g – am)
2 4 16
4 . 9 4 | Forces and Laws of Motion

From free body diagram of wedge N d


⇒ (cos β + µ sin β) = 0

Mg sin α + N sin α = M am
M
∴ mg sin α + m(g – am) sin α = Mam ⇒ – sin β + µ cos β = 0
mg sin α + (m (g – aM sin α ) sin α=M am M g sin α ⇒ µ = tan β
1 µ
(m + M)gsin α ⇒ β = tan–1 µ ⇒ cos β = 2 and sin β =
⇒ am = µ +1 µ2 + 1
M + msin2 α
mg(sin α + µ cos α )
(m + M)gsin2 α
⇒ am = (am = aM sin α) 1 µ2 mg(sin α + µ cos α )
M + msin2 α T= + =
2
µ +1 2
µ +1 µ2 + 1

Sol 22: Let mass of A = m


Mass of B = hm Sol 24: frictional force f = km2g

Let normal reaction between surfaces be N F–f


a1 = (F > f)
m1
Nsin α Nsin α
aB = = at − km2g
mB ηm a1 =
m1
mg – Ncos α
aA = at – km2g
m a1 = (t > t0)
m1
aA = aB tan α km2g
a2 = = kg (t > t0)
Nsin α tan α m2
N
⇒g– cos α =
m ηm for t < to , f acts as internal force as there is no sliding
at
N sin α tan α  ∴ a1 = a2 = (t < t0)
⇒g=  cos α +  m1 + m2
m η 

N g a1 a2
⇒ =
ηm η cos α + sin α tan α
N g t
aB = sin α = t t0
ηm η cot α + tan α t0
g Till time to , the bodies move together.
aA = aB tan α = 2
η cot α + 1 at t = t0, f = k m2g
km2g = m2a2
m2
Sol 23: Let tension in string be T
kg = a2 = a1 m1
Net force perpendicular to plane
ato – km2g = m1a1
N = mg cos α – T sin B m2 f = k m2g
ato = km2g + m1a1
For minimum tension acceleration is zero k(m1 + m2 )g
to = f m1 F = at
∴ mg sin α = T cos β – µ N a
mg sin α = T cos β – µ mg cos α + mT sin β
Sol 25: Let mass of motor = m
mg(sin α + µ cos α )
T= mass of bar = 2m
cos β + µ sin β
2m w = T – 2mg k
dT mg(sin α + µ cos α ) d
= (cos β + µ sin β) ⇒ T = 2 m (w + kg)
dβ –(cos β + µ sin β)2 dβ
Let acceleration or motor be am
dT
For minimum T, =0 m am = T – mg K

P hysi cs | 4.95

1 300 × 2
⇒ am = [2mw + 2m kg – mkg] ⇒ aB = = 1.558 ms-2
m 385
⇒ am = 2w + kg 3
aA = a = 2. 338 ms2
2 B
Relative accelerator a = am + w = 3w + k g
T = 81. 8 N
1 2
at = l
2
2 2 Sol 28: F = 30t N
t= = 3w + kg
a ⇒ T = 10 t
wt. of A = 10 m1 = 10 N
Sol 26:
F (a) Block A loses contact when T = weight

10t = 10
 θ t=1s
T
Similarly 2T = 10m2 when B loses contact
m
20t = 10(4)
2T cos θ = F
t = 2s
F
⇒T= (b) Net force on A FA = 10 t – 10 (t > 1)
2cos θ 1
aA = (10t – 10)
T sin θ m1
Horizontal acceleration ax =
m
aA = (10t – 10)
F sin θ F tan θ F x
= = = . dv A
2cos θ m 2m 2m ( )2 – (x)2 = 10t – 10
dt
Fx 2 2
ax = 1 vA = ∫ (10t – 10).dt = 5t2 – 10t
1
2m(2 – x2 )2 1

fx v = 5 ms–1
Acceleration of approach = 2ax = 1 t t
2
m(2 – x2 ) 2 (c) vA = ∫ (10t – 10)dt = 5t – 10t
1
1

Sol 27: Let tension in thread = T vA = 5t2 – 10t + 5


F – 3T = 35 aB dh
= 5t2 – 10t + 5
2T = 70 aA dt
2 2
⇒ F = 35 (aB + 3aA) 2 5 3 5
H = ∫ dh = ∫ (5t – 10t + 5)dt = 3
t – 5t2 + 5t =
3
m.
Constrain equation xB 1 1

xA xB Circular Dynamics

Sol 29: Acceleration inside a rotor = Rω2



xc a = Rω2

Now for a max
2(xB – xA) + (xB – xC) = Constant
2
3 amax = R wmax
⇒ 3aB – 2aA = 0 ⇒ aA = aB
2
Given amax = 10g = 100 m/s2
 11 
⇒ F = 35   aB
 2 
4 . 9 6 | Forces and Laws of Motion

Sol 31:
100 10
ωmax = = rad/s = 5 rad/s θ
4 2

1
we know that 1 rad = rev

5
∴ ωm = Rev/s

T cos θ = mg
Sol 30: 2
T sinθ = mv
N R
θ
Now the component T cosθ has to balance the weight
θ mv 2 of the body
R
∴ Tmax cosθ = mg ⇒ 8 cosθ = 0. 4 × 10
θ 1
cosθ = ⇒ θ = 60°
2
mg ∴ Angle with the horizontal is (90°– θ)= 30°
mv 2
and T sinθ =
mv 2 R
N sin θ =  ... (i)
R
3 0.4 × v 2
N cos θ = mg  ... (ii) 8. =
2 4
Dividing (i) and (ii) v= 40( 3) m/s
v2 v = 8. 3 m/s
⇒ tanθ =
Rg
5 Sol 32: Speed of the particle just before the string
⇒ v = 108 km/h = 108 × m/s
18 breaks is v. Now after the string is broken; the path of
v = 30 m/s the stone will be;
R = 90 m v
30.30
∴ tanθ = =1
90.10
π 2
⇒ θ=
4
Squaring (i) and (ii) and adding them
2 10
 mv 2 
⇒ N (sin θ + cos θ) = 
2 2
 + (mg)2
2
Writing the equations of motion;
 R 
 
1 2
2
along y : 2 = gt  ... (i)
 mv 2  2
2
⇒N= (mg) +  
 R  along x : 10=vt  ... (ii)
 
Solving for v; we get
⇒ N = m (10)2 + (10)2 = 10 2 m Newton v = 15.8 m/s
v2
and centripetal acceleration =
⇒ N = 10 . 4
2 N. R
(15.8)2
a= = 168.3 m/s2
1.5
P hysi cs | 4.97

Sol 33: Sol 35: mv 2


cosθ
B R
v N
mv 2
R
θ
mgcosθ
mg sinθ
mg mv 2
u sinθ
A R

Writing down the equation of motions at point A and B; mv 2


N= sin θ + mg cos θ
R
At B:
mv 2 mv 2
TB = – mg f= cosθ – mg sin θ
R R
mu2 Contact force is N + f
At A:- TA = + mg
R
And the angle with which the force and the surface of
Now for completing the circle; the contact lie is
Tension at the highest point has to be non-zero; or else f
tan–1  
the particle will fall down. N
So for the minimum case, T ≈ 0 But here given that the force is normal to the surface
∴ TB = 0 ⇒ Friction force = 0
2
mv mv 2
⇒ = mg
R ∴ cosθ – mg sin θ = 0
R
Rg v2  v2 
v= ⇒ tanθ = ⇒ θ = tan–1  
Rg  Rg 
 
5 250
v = 100 km/h = 100. = m/s
18 9
Sol 34:
Find θ now !

Sol 36: FBD of M;

T mv 2
R MLω2
mv 2 f
T=
R
Mg
Now for vmax
f = MLω2; N = Mg
we have Tmax = 500 N
and for static conditions;
1v 2
⇒ 500 =
1 f = mN = µMg ⇒ µMg = MLω2

v= 500 = 10 5 m/s µg
ω=
L
v = 22.36 m/s
4 . 9 8 | Forces and Laws of Motion

Sol 37: Let u be the speed at the highest point of the Let us call the point where tension is equal to the weight
bridge of the particle as ‘P’.
A
u Now at point P,
P
R θ mv 2
T= + mg cos θ  ... (i)
mg R
O Given that T = mg

mv 2
mg = + mg cos θ
R
mu2 mv 2
+ N = mg mg(1 – cos θ) =  ... (ii)
R R
mu2 Now Total energy at point O
N = mg –
R 1
0= m ( gL )2 + 0
Now for maximum speed where contact is broken; 2
N=0 mgL
2 E0 =
mu 2
∴ mg =
R 1
u= Total energy at point P = m(v2) + mgL (1 – cos θ)
Rg 2
E0 = EP

Sol 38: mgL mv 2


∴ = + mgL (1 – cos θ)
2 2
θ mgL mv 2
T – + mg L cosθ =
2 2
mv 2 mv 2
R = 2 mg cosθ – mg  ... (iii)
2
mg mv 2
Now using this value of in eqn (ii)
mv 2 L
T sin θ =  ... (i) 2 mg cos θ – mg = mg(1 – cosθ)
R
T cos θ = mg  ... (ii) 3 mg cosθ = 2 mg
2 2
cosθ = ⇒ θ = cos–1  
(i) v2 3 3
= tanθ = Considering eqn – 3
(ii) Rg
5 mv 2 2
v = 36 km/h = 36 = 10 m/s = 2mg   – mg
18 L 3
10 × 10 mv 2 mg
tan θ = ; tan θ = 1 =
10 × 10 L 3
π g
⇒ θ= v=
4 3

Sol 39: Sol 40:


θ R
N

θ r
mrω2
T
v
P r=R sinθ
mg
θ mv 2

O gL FBD of body:
R
mg
P hysi cs | 4.99

(a) For minimum ω; Sol 42:


m1
Body tends to slip down w
F1
∴ friction acts upwards F2
m2
N
f
FBD of m1;
mrω2
a
T M1 F1 F1 – T = m1a  ... (i)
mg
FBD of m2 :
N = mg cosθ + mr ω sinθ 2
a
F = mg sinθ – mr ω2 cosθ M2 F2 T – F2 = m2a  ... (ii)
T
We know that f = mN
Adding equation (i) and (ii)
⇒ mgsinθ – mrω2cosθ=m[mg cosθ + mrω2sinθ]
F1 – F2 = (m1 + m2)a
Separating all ω2 terms to one side;
F1 = mRω2 F2 = 2 mRω2
(mr sinθ+r cosθ)ω2=g sin θ – mg cosθ
∴ – mRω2 = 3m a
g(sin θ – µ cos θ)
ω= Rω2
R sin θ(µ sin θ + cos θ) a=–
3
Now for maximum limit case; Using equation (i)
Solve exactly as above N  Rω2 
mRω2 – T = m  – 
 3 
mrω2  
mRω2
f T = mRω + 2

mg
3
4
T = mRω2
Sol 41: 3

θ Sol 43: Given Normal acceleration an = Kt2


mv 2
But we know that man =
2 R
∴ v = Kt2
R
Fcosθ
v= KR t  ... (i)
θ
F=mRω 2
dv
= KR
dt
dv
F sinθ
Tangential force = m. =( KR )m = m KR
→ dt
Total force = m| a |
Now mRω2 cosθ = ma → → →
a = an + a t
∴ a = Rω cosθ
2

1 2 →
Now s = ut + at |a|= an2 + a2t = (Kt 2 )2 + ( KR )2
2
1 →
L=0+ Rω2 cosθ t2 Total force = m. | a | = m K(R + Kt 4 )
2
Now we know that work done by normal force in a
2L circular motion is zero
t=
Rω2 cos θ
4 . 1 0 0 | Forces and Laws of Motion

∴ ωN = 0 Let us consider the part OAB;


Now only work is done by tangential force T cosθ/2
T
(m. KR )ds  ... (i)
T sinθ/2
We know that
dω dω ds θ/2
Power = ≡ . mR2ω2
dt ds dt θ/2

P = v. = KR × t × m KR
ds T sinθ/2
P = mKRt T
t T cosθ/2
∫ mKRt dt m is the mass of the part OAB.
0
Avg power = t
θ
⇒ 2T sin = mR2ω2
∫ dt 2
0
1 now for small values of θ; sinθ = θ;
Pavg = mKRt
2 θ
2T.   = mRω2
mv 2 2
Sol 44: A N R
Tθ = mRw2 ... (i)
Rsinθ

B ma Now m = (λ)(Length) = (λ). Rq
R cosθ θ
R
mg ∴ Tθ = (lRθ)Rω2
T = lR2ω2

a → → →
Sol 46: a net = a radial + a tangential
In this case, there will be a pseudo force acting on the →
v2 →
body. Now we use Work-Energy theorem, i.e. work done ar = . (– êr ); a t = a( êt )
R
by all the forces is equal to change in kinetic energy. We
know that, work done by normal force and centripetal → 2
 v2 
force is zero | a net |= 2
a +  m/s2
R 
 
Work done by pseudo force = ma. (R sin θ)
WPF = maR sinθ
Sol 47: B
Work done by gravitational force = mg(R – Rcos θ)
A
Wmg = mgR(1– cosθ) θ w
Net work done = maR sinθ + mgR (1–cos θ) O
1
≡ mv2 = Rm(a sinθ + g(1 – cosθ))
2
v= 2R(asin θ + g(1 – cos θ)) Consider the part OAB;
Let the mass of this strip be ‘dm’

Sol 45: T

θ θ/2 dmv 2
O A
θ/2 R

T
P hysi cs | 4.101

θ dm.v 2 The mass of the part AB of chain has to be supported by


2T sin   = the rest of the chain.
2 R
For very small values of θ; sin θ ≈ θ m
T = (L – x)g
L
θ dm.v 2 dm.v 2
∴ 2T  2  = ; T. θ =
  R R Sol 2: (A) mg sin θ = f and N = mg cos θ
m  mθ  mθ v 2 N
Now dm = .R.θ =   ⇒ Tθ = .
2πR  2π  2π R
f
2
mv
T=
2πR
at mg sin θ mg cos θ
Sol 49: → θ
a

mg
ar For the condition of just sliding;
f = mN

→ → →
⇒ mg sin θ = µ mg cos θ
a net = ar + at ⇒ tan θ = µ ⇒ θ = tan–1(µ).
2
→ v → dv
ar = ; at = =a Hence the angle of inclination has nothing to do with
R dt the mass of the body.
2
→ v2 →  v2 
a net = (– êr ) + a ( êt ); | a net | = a2 +   Here the angles are different because of the change in
R R 
  ‘µ’ from one block to another.

f = m| a net |
Sol 3: (A) a
∴ Under static conditions

2
F m2
2
 v2  m1 m3
mmg = m a + 
R 
 

v = [(m2g2 – a2)R2]1/4
Now let us say the whole system moves with an
acceleration ‘a’.

Exercise 2 ∴ F = (m1 + m2 + m3) a … (i)


Let as consider Individual masses;
Forces and Laws of Motion
For m1 ;
Single Correct Choice Type Ng
1

m1 F – N1 = m1a; …(ii)
F N1
Ng – m1g= 0; …(iii)
1
Sol 1: (C) A x m1g
(L–x) a
B
m For m2:
λ (linear density) of chain =  
L
Now at point A; f
N1 N2 N1 – N2 = m2a; …(iv)
T m2g – f = 0; …(v)
m2 g
m
  (L – x)g
L a
4 . 1 0 2 | Forces and Laws of Motion

For m3; m1g sin θ + N – fs = m1a


1
Ng
3 N = m1a – m1g sin θ + fs
1
N2 N2 N2 = m3a; …(vi)
 35 
N = 170   – 800 + 300
Ng = f + m3g; …(vii)
3  17 
f m3 g N = – 150 N
a
i.e. force in the bar is 150 N.
We know that fmax = mN2 = mm3a
Sol 5: (A) Lift moving uniformly means lift is moving
f = m2g ≤ fmax ≡ mm3a without any acceleration.
m g Hence in both the cases; acceleration of the coin is ‘g’.
⇒ a ≥  2  … (viii)
 µm  ∴ t1 = t2.
 3 

m g
⇒ F ≥ (m1 + m2 + m3)  2  (from (viii) & (i)) T
 µm3  Sol 6: (B) m2

T 700 gm F
Sol 4: (A) µ 2 = 0.4 m1

Now if m1 moves with an acceleration ‘a’ towards right;


m2 will have an acceleration of ‘a’ towards left.

µ 1 = 0.2 [ string constraint]


FBD of m1;
θ N2 N1
f2
Here both the particles are constrained to move m1
T F
together. Hence aA = aB
f1
Now let us first find the net force down the incline;
i. e (m1 + m2)g sin θ m1g
8
Fnet = 340 × 10 ×
17 F – f1 – f2 – T = m1a  … (i)
Fnet = 1600 N.
m1g + N2 = N1  … (ii)
Now let us calculate the fs + fs
1 2
FDB of m2;
15
fs = µ1 . (m1g cos θ) = (0. 2) (170 × 10 × ) = 300 N N2
1 17

fs = µ . (m g cos θ) = 0. 4 (170 × 10 × 15 ) = 600 N T m2


2 2 2
17
f2
∴ fs1 + fs2 = 900 N.
∴ Net Acceleration of the system m2 g
 1600 – 900  700 35 N2 – m2g = 0  … (iii)
=   = m/s2 ∴a= m/s2
 170 + 170  340 17
T – f2 = m2 a  … (iv)
Now on A; N f2 = mN2 = mm2g  … (v)
a
fs
1
∴ T = m2a + mm2g
m1g sin θ
T = (a + μg) m2  … (vi)
f1 = mN1 = µ(m1g + m2g) = mg (m1 + m2)
P hysi cs | 4.103

∴In equation (i) Right.


F – mg (m1 + m2) – µ m2g – (a + mg)m2 = m1a ∴ FBD of m;
∴ F = (m1 + m2) a + 3mm2g + mm1g N ma

⇒ F = (m1 + m2)a + mg (m1 + 3m2) F m T


f
Put a = 0.3 m/s2 and m1 = 0.7 kg, m2 = 0.2 kg to get the
value of force. mg
Hence, we get F = 2.18 N
F–T–f=0
Sol 7: (A) N
N + ma = mg
m ⇒ F = f + T  … (i)
F1
N = mg – ma  … (ii)
mg cos θ
θ ∴ FBD of M;
N1 Ma
mg
f M T
As force F tends to push the mass upwards, friction will
tend to oppose it. So, it will act downwards.
∴ F = f + mg sin α Mg

f = m N = µ mg cos α
T – f = 0  … (iii)
⇒ F1 = µ mg cos α + mg sin α  … (i)
N1 + Ma = Mg  … (iv)
Now when pushing downwards, friction will be acting
From (i) and (iii);
upwards,
⇒F=f+f
N F2
⇒ F = 2f ; f = µN = µ(mg – ma)
m f
F = µm (g – a)
mg sin θ mg cos θ ⇒F = 2 mm (g – a).
θ
Sol 9: (B) a1
mg

∴ F2 + f = mg sin θ
F2 = mg sin θ – f m1 µ
f = µ mg cos θ µ= 0 m2 m F
⇒ F2 = mg sin θ – µ mg cos θ  … (ii)
Given that F1 = nF2
Let us say the whole system moves with an acceleration
∴ mmg cos θ + mg sin θ =n(mg sin θ – mmg cos θ)
a1.
n–1  F 
⇒ µ= tan θ ∴ a1 =    … (i)
n+1  m1 + m2 

Sol 8: (B) Now FBD of m1;


N
F T
m m1a1
f m1
M m f

m1 g
For maximum force, F; the friction on ‘M’ will be towards
4 . 1 0 4 | Forces and Laws of Motion

m1a1 – f= 0 And P – f = m1a ⇒ P – µ mg = m1a


⇒ m1a1 = f  … (ii) kt – µm1g
a1 =
FBD of m2 N m1
f
m2 F
m2a Sol 11: (B) Now let us check the limiting frictions
between the three surfaces
F – f – m 2a = 0
⇒ F = m2a + f  … (iii) (1) A P
µ1 = 0.3
Now when the mass m1 just tends to slide;
(2) B µ2 = 0.2
f = µ N = µm1g (3) C µ3 = 0.1
∴ m1a1 = µm1g (from (ii))
fs = µ1(mA g) = 90 N
∴ a1 = µg. 1
fs = µ2(mA + mB)g = 80 N
Now from (i) 2
fs = µ3(mA + mB + mC) g = 60 N.
F = (m1 + m2) µg 3

at = (m1 + m2) µg ∴ Now let us assume P would be greater than 60 N and


less than 80 N.
(m1 + m2 )µg
t= For this P;
a
f1 = P [ f1 < fs ≡ 90]
1
Sol 10: (C)
A p
N f1
m1 f1 = f2 = P [ f2 < fs ≡ 80]
m1 p P=kt
f A 2
f1
m2 B B
m1 g f2

p – f = m1a  … (i) Now f2 – f3 = m3a

N = m1g  … (ii) f2 [ f2 = P > 60 ≡ fs ]


C 3
f3
f
m2
∴Here f2 is greater than the maximum static friction
f = m2a  … (iii) between C and ground. Hence the block C will slide on
the ground. There by all the three blocks will slide for a
Now for f ≤ mm1g; minimum force of 60 N.
Both the block will move together;
Sol 12: (A) m2g – T = m2a1 … (i)
∴Adding (i) and (iii);
P = (m1 + m2)a.
 P   k 
a =   =   t
m
 1 + m2 
 m + m
 1 2 

Now for f = mm1g; this is the maximum frictional force; a


T
T
∴ f = m2a2 π/4
m2
µm1g = m2a2 m1

µm1g
⇒ a2 = which is constant
m2
P hysi cs | 4.105

On m1; mAg sin θ – f – T = mA aA … (iii)


Tsin θ
N2 = mAg cos θ  … (iv)
N
T cos θ m1 aA = aB (constraint equation)
f
By adding (i) and (iii)
m1g
(mA – mB) g sin θ – 2f = (mA + mB) a  … (v)
N + T sin θ = m1g
For limiting condition, a= 0.
T cos θ – f = m2a2
⇒ (mA – mB) g sin θ = 2f
Now for just initiating the motion;
Here f ≤ fs = mN1 = µ mB g cos q
a1= a2 =0
∴ m2g – T = 0  … (i) (mA – mB) g sin θ ≤ 2 µ mB g cos q

T cos θ – f = 0  … (ii) mA – mB
µ ≥ tan q
m2g cos θ = f  … (iii) 2mB

f = µN = µ(m1g – T sin θ) = µ(m1g – m2g sin θ) Now in equation (v)

⇒ m2gcos θ = µ(m1g – m2g sin θ) If m1 = m2;


Tension itself balances both the masses.
 m2 cos θ 
m =  
 m1 – m2 sin θ  So, no necessity for any friction.
C: we cannot explicitly say that. We
put θ = π/4.
need more information on µ.
D: when mA = mB;
Multiple Correct Choice Type
Put friction f = 0 in (i) and (iii)
Sol 14: (A, B)
And subtract them to get Tension ‘T’.

B Sol 15: (A, D) T sin θ


A

N1 A T cos θ
θ

FBD of B;
N1 mg f1
T

B
Acceleration of block will be zero. Since its constrained.
∴ mg + f1 = T sin θ … (i)
f mBg cos θ
mBg sin θ N1 = T cos θ  … (ii)
f1 f2
T – f – mBg sin θ = mB aB  … (i)
N1 = mBg cos θ  … (ii) N1 B N2
FBD of A;
N2
f T Mg
A
Mg – f1 – f2 = Ma  … (iii)
mAg cos θ N1 = N2 = T cos θ
mAg sin θ
f2 = µ(N2) = µ T cos θ
4 . 1 0 6 | Forces and Laws of Motion

f2 = µ N1 = µ T cos θ And adding equation (i) and (iii);


∴ From equation (i) (m1 + m2 )gsin θ
a=
mg + µ Tcos θ = T sin θ (m1 + m2 )

mg = T (sin θ – µ cos θ) a = g sin θ  … (v)

 mg  now using this we can find, N3;


100
T =  sin θ – µ cos θ  =
   3 N3 = zero, for all m1 and m2.
0.5 – 0.2  
 2 
 
Sol 17: (B, D)
T = 306 N.
Now using equation (iii) A
T
Mg – f1 – f2 = Ma B

Mg – 2 µ T cos θ = Ma
T
2µT cos θ
a=g– FBD of B;
M m2 a

a = 4. 7 m/s2 mg

N1 mg – T = ma  … (i)
Sol 16: (A, C) N3
A a
FBD of A; A
T
m1g cos θ
m1g sin θ mg sin θ

m1gsin θ + N3 = m1a1  … (i)


mg sin θ + T = ma  … (ii)
N1 = m1g cos θ  … (ii)
By adding (i) and (ii)
B mg + mg sin θ = 2 ma
A
mg
mg + = 2ma
θ fixed 2
3g
N2 ⇒ a=
N3 4
B Now using equation (i)
mg – T = ma.
m2g sinθ m2g cosθ T = mg – ma = mg –
3mg
4
m2g sin θ – N3 = m2a2  … (iii) mg
T=
4
N2 = m2g cos θ  … (iv)
now let us assume; a1 ≠ a2, then;
Assertion Reasoning Type
Both of them will lose contact
∴ N3 = 0. Sol 18: (A) Conceptual. Conservation of linear
momentum for a single particle do mean that the state
But we then find a1 = a2 = g sin θ.
of the body is conserved or constant unless an external
Hence both of them will have same acceleration. force acts on the body.
Now putting a1 = a2 = a
Sol 19: (D) Assertion: If the force is non-constant and
reverses itself over time, it can give a zero impulse.
P hysi cs | 4.107

For example: spring force would give a zero impulse


over one period of oscillation. C

α
Sol 20: (D) Here; weight of the book is because of the
Gravitational Attraction Between earth and book. There B T
will also be a gravitational force between book and ma
table, which is very small, hence always neglected.
That Gravitational force between table and book form Fnet
mg
an Action-Reaction pair.
ma
∴ = tan α ⇒ a = g tan a
Sol 21: (A) Both assertion and reason are statements of mg
Newton’s laws. Since its acting leftwards, the vehicle should move
→ rightwards.
Sol 22: (A) Momentum = m u .
We have to specify reference frame, because velocities Paragraph 2:
will vary in different frames. So, momentum which
In s’ frame;
implicitly depends on velocity might also very.
FBD of M. y
a
x
Comprehension Type
m
Paragraph 1:
M
θ
B
α
T2 cos α
a
α T2
A Nθ
T2 sin α
Ti
θ
mg
N1 Mg
T2 cos α = mg ; T2 sin α = T1
N sin θ = Ma  … (i)
⇒ T2 = mg seca; T1 = mg tana
N cos θ + Mg = N1  … (ii)
Now just after the string AB is cut;
FBD of m;
T2 = mg cos θ
N + ma sin θ
α
T2 B ma

mg sin θ + ma cos θ
mg cos θ
θ mg
mg sin θ
mg cos θ
mg N + ma sin θ = mg cos θ  … (iii)
Now when string BC is cut; mg sin θ + ma cos θ = ma’  … (iv)
Mass ‘m’ will just have force fall. Hence tension in string from (i) and (iii);
AB is zero.
N = mg cos θ – mg sin θ
Now suppose it is keep in a moving automobile;
Now in equation (i)
In automobile’s frame of reference, there is a pseudo
N sin θ = Ma
force acting on the mass.
(mg cos θ – ma sin θ) sin θ = Ma
The resultant force should be along BC.
4 . 1 0 8 | Forces and Laws of Motion

 mgsin θ cos θ  max cos θ


a=  2 
 M + msin θ 
max
Since its in –ve x direction; we add a ‘–’ sign.
 mgsin θ cos θ 
∴ a = – 
2
 M + msin θ  mg sin θ

Now using this value of a, solving equation (iv); ∴ max cos θ = mg sin θ
We get ax = g tan θ in positive x direction,
 mgsin θ cos θ 
mg sin θ + m   cos θ = ma’
2
 M + msin θ  Paragraph 3:
2
mgsin θ cos θ Given that the plank has very rough surface.
⇒ g sin θ + = a’
M + msin2 θ N
µ >> 0

Mgsin θ + mg sin3 θ + mgsin θ cos2 θ ∴FBD of A;


⇒ = a’ m T
M + msin2 θ f

Mgsin θ + mgsin θ(sin2 θ + cos2 θ)


⇒ a’ = mg
M + msin2 θ
T – f = 0 … (i) ⇔ T = f
Mgsin θ + mgsin θ (M + m)g sin θ
a’ = ∴ a’ = mg – N = 0 … (ii) ⇔ N = mg
M + msin2 θ M + m sin2 θ
FBD of B;
This is the acceleration of the block ‘m’ with respect to
T
the incline.
Force exerted by the mass ‘m’ on wedge is ‘N’. 2m
We can find this by; equation (iii)
∴ N + ma sin θ = mg cos θ 2mg

⇒ N = mg cos θ – ma sin θ T – 2mg = 0 … (iii) ⇔ T=2mg


 mg sin θ cos θ 
2
∴ f = 2mg
= mg cos θ – m  2


 M + msin θ  Net Contact force acting between block A and plank;

 Mmg  is N2 + f 2 = (mg)2 + (2mg)2


N= 
2 
 M + msin θ  F = mg 5
ax On the pulley;
N= T 2 = 2 2 mg.
m
mg sin θ T N
m
θ
T 2
T
Now in this question; the downward component of mg
sin θ has to be balanced.
Now just after this instant;
Normal reaction becomes zero.
P hysi cs | 4.109

On body A; Now for this the acceleration of the block should be


zero.
T
a1
mg sin θ = f + T
a2 m T f
120 – 90 = f
f = 30 N
mg mg sin θ
µ (mg cos θ) = 30
T = ma1 … (i) 1
µ ( 8 3 .10. ) = 30
2
mg = ma2.
3
µ= .
⇒ a2 = g 4
on body B
T Sol 38: (B)
2m a1
b µ = 0.8
2mg p
2mg – T = 2ma1  … (ii) At an angle θ;
From (i) and (ii) mg sin θ – f = 0
2g N – mg cos θ = 0 N
a1 = = 6. 66 m/s2 f
3 Now fmax = fs = mN.  m
Now aA = a1( î ) + a2 ( – ˆj )
mg sin θ mg cos θ
–2g ˆ ˆ θ
= i – gj
3 At this point the block starts sliding.

4g2 ∴ fmax = µ mg cos θ


|aA| = + g2
9 ∴ mg sin θ – µ mg cos θ = 0

13 ⇒ tan θ = µ
|aA| = g
3 ⇒ θ = tan–1(0. 8)

|aA| = 12 m/s2 θ = 40º


Now till this angle; f = mg sin θ
Paragraph 4: ∴ for θ = 30º,

Buoyant force = rgVimm = rgv f = mg/2

F = (0.9) (103).(10) (0. 2 × 1 × 1 × × 10–2) Now for θ = 45º, let us say body is not sliding mg sin
θ–f=0
FB = 18 N.
N = mg cos θ
Now T + FB = W +Fv. T FB
fs = mmg cos θ=mmg/ 2 = 0.8 (mg/ 2 )
T = W + Fv – FB A
 mg 
Fv = 60 N. f = mg sin θ =  
 2
T = 48 + 60 – 18 w Fv
But for our assumption;
T = 90 N.
f ≤ fs
 mg   mg 
⇒  ≤ (0. 8)  
 2  2
4 . 1 1 0 | Forces and Laws of Motion

which is not true. 1


∴ mv2 = as2
Hence the body would have started sliding 2

f = fs = µ N = µmg cos θ = µmg/ 2 2a


v= s
m
→ → →
Now a = a r + a t
Circular Dynamics
→ v2 2as2
 v2  ar = =
Sol 39: (A) Centripetal acceleration = rω2  or  R mR
 r 

Given that both have same periods. → dv dv ds
at = =
So ω1 = ω2 dt ds dt
a1 = Rω2 a2 = rω2 → dv
a t = v.
a1 R ds
=
a2 r
dv 2a
=
ds m
Sol 40: (A) Max Tension the string can sustain → 2as
⇒ at =
Tmax = 10 N. m
Mass of the sto → → →
1 a = ar + a t
ne = 250 gm = kg
4 2
 2as2   2as 2
Length of string = 10 cm = 0.1 m |a| = ar2 + a2t =   +
 mR   m 
T = mrω2  
T
0.1m
Tmax = 2
mr ωmax 2as s2
|a| = 1+
m R2
Tmax
ωmax = → s2
mr | F | = m |a| = 2as 1 +
R2
10
ωmax = 1 = 400 rad/s
× 0.1 Multiple Correct Choice Type
4
Sol 44: (B, C) 
wmax = 20 rad/s.
dv →
at
Given speed = v; and =a anet
dt
Sol 41: (D) Already discussed in Q. 40 So try this yourself → v2
ar = ê
r r O ar
Sol 42: (D) Let the angular speed of the thread is w. →
a t = a êt
For particle C T3 = mw 3l2 → → →
a net = ar + a t
For particle B T2 → T3 = mw22l → T2 = mw25l →
a net v2
= êr + a êt
For particle A T1 → T2 = mw2l → T1 = mw26l r
2
→  v2 
1 | a net |=   + a2
Sol 43: (B) Kinetic energy k = mv2  r 
2  

But given that k = as2 Now friction force f = m a net
P hysi cs | 4.111

2 T
 v2  θ
f=m   + a2 mrw2
 r 
 

2
 mv 2 
f=   + (ma)2 and f = mmg mg
 r 
 
Resolving into components
T cosθ = mg
Sol 45: (B, D)
T sinθ = mrω2 ; r = L + L sinθ
N cosθ ⇒ T sin θ = mω2 L (1 + sinθ)
N
T sin θ ω2L(1 + sin θ)
⇒ =
N sinθ T cos θ g
mg
θ
C
Sol 47: (B, D) F
N

Since µ ≈ 0, there would be no frictional force. D F F


B
N cosθ = mg
F
2
A
mv
N sinθ = Consider the figure, with force F on the particle at
R
different instants of time.
2
 mv 2  So it is evident that there should be some other forces
⇒N =   + (mg)2
 R  such that particle will have uniform circular motion
  → → →
2 ∴ F + F 2 =m a
mv
∴ N > mg as well as N >
R Since it’s a uniform circular motion

at = 0
Now when speed of the car is less than vc = 40 km/hr
and if we consider the frame of car; → → v2
∴ a = ar =
R
Both these forces are made equal through proper
banking. → →  mv 2 
∴ F + F 2 =  

mv 2c
cos θ  R 
mgsinθ → →
mv 2
R Now resultant of both the forces F and F 2 is which
r
Now if v < vcritical ,
mv 2 in turn keeps changing both in direction as well as
Then mg sinθ > cosθ magnitude.
R
⇒ It slips downwards. →
mv 2 ê →
∴ F2 = r–F
R

w Angle between êr and F keeps varying.
Sol 46: (A, B, C) L
θ L
Assertion Reasoning Type

So 48: (D) Concept of centrifugal force comes into


L sinθ picture only in a non-inertial frame. So, both of them
cannot co-exist in a same frame.
Free body diagram of mass m;
4 . 1 1 2 | Forces and Laws of Motion

Although it is true that they are equal and opposite they Sol 52: (B)
can’t cancel each other because of this. N

N cosθ
Sol 49: (C)
N θ

N sinθ
N cosθ = mg
θ mv 2
N sinθ =
R
So bending inwards is always essential. He does it so
2
as to get horizontal component of normal force as
N sin θ = mv
R centripetal force. Although bending lowers his center of
gravity, it’s not the reason.
N cos θ = mg
It is not the friction between the tyres that provide him Sol 53: (A)
centripetal force, but it is component of Normal force. N

Sol 50: (B) From the above solution;


mv 2
We can write N sinθ = θ
R
N cos θ = mg
m(2v)2 mv 2
Now when v is doubled, = 4. N cosθ = mg
R R
∴ Tendency is quadrupled mv 2
N sinθ =
R
 v2 
And also tan θ =   as v ↑ θ ↑ v2
 rg  tan θ =
  Rg

Sol 51: (E) when velocity is doubled


N cosθ (2v)2 V2
N tan θf = = 4.
Rg Rg
N sinθ Hence skidding tendency is quadrupled.

θ Sol 54: (D) Assertion is explained in Q. 46 and Reason is


true (It is conceptual)
mg
mv 2
N sin θ = Comprehension Type
R
Horizontal component of normal force provides the Paragraph 1:
centripetal force. Hence false.
mv 2
Reason:- R
A curved path need not always be circular path. In case
of elliptical paths, the force is not necessarily centripetal. N
P hysi cs | 4.113

At any instant, say speed is v. Normal force against wall, Paragraph 2:


2
mv
N=
R
Now frictional force, f = mN mv 2
N
R
µmv 2
F= (– êt ) [tangential]
R

And tangential acceleration say a t
Top view of the rotor
→ µmv 2
Now m a t = (– êt ) mv 2 µ.mv 2
R N= ; f = mN =
2 R R
a t = µv (– êt )

R For equilibrium;
fs
→ dv fs = mg
and also a t =
dt mN = mg
dv µv 2
=– µ.mv 2
dt R And this is = mg
R mg
dv µ
= – . dt Rg
v2 R v=
µ
Integrating both sides
v t v
dv µ 1  µt Match the Columns
∫ v 2
=–
R ∫ dt ⇒–   =–
R
v0 0  v  v0
Sol 61: mv 2c
1 1 µt R
= +
v v0 R C
mg
µv 2 T
Sol 57: (B) at = – T mvB2
R D
B R
dv dv dx dv
at = = . =v
dt dx dt dx
A u = 3 Rg
dv µv 2
⇒v =–
dx R At point B;
dv µ mvB2
=– dx TB =
v R R
Integrating both sides; And also total energy at point A;
v x
dv µ 1
E= m(u)2 + UA
∫ v
=–
R ∫ dx 2
v0 0
Now assume ground at the point A itself
 v  µx
ln   = – ∴ UA = 0
 v0  R
µx
– 1 9mRg
v = v0 e R EA = m (9Rg) =
2 2

And total energy at point B;


1
EB = m( vB2 ) + mg(R)
2
4 . 1 1 4 | Forces and Laws of Motion

According to conversation of energy 1


EB = mvB2 + mgR
EA = EB 2
1
1 9mgR EC = mv 2C + 2mgR
∴ mvB2 + mgR = 2
2 2
Now given that vA = 10
1 7
mvB2 = mgR
2 2 So; EA = EB (using conservation of energy)

vB = 7gR 1 1 2
m(10)2 = m( vB ) + mg(1). [R = 1, vA= 10, m = 1]
2 2
mvB2
and TB = = 7mg vB = 80 m/s
R
for point C; and similarly
mv 2c 1 1
Tc + mg = EA = EC ⇒ (10)2 = v 2C + g(2)
R 2 2
mv 2c vc = 60 m/s
Tc = – mg
R
∴ from (i), (ii), (iii)
Total energy at point C is TA = 10 + 100 = 110 N
1 TB = 80 N
Ec = mv 2c + mg(2R)
2
TC = 50 N
1
Ec = mv 2c + 2mgR ∴ minimum tension is 50 N
2
When string is horizontal i.e. at point B;
Ec = EA

1 9mgR
⇒ mv 2c + 2mgR = T mv 2
2 2
R
mv 2c 5mgR
= ⇒ vc = 5gR mg
2 2

mv 2c

vB2
∴ Tc = – mg = 5mg – mg = 4 mg
ar = = 80 m/s2 (– î )
R R

a t = g = 10 m/s2 (– ˆj )
Sol 62: VC C → → →
VB a net = ar + a t

D | a net | = (80)2 + (10)2
B

| a | = 10 65 m/s2

A VA At point C; tangential acceleration is zero

mv 2A vc
At point A; TA = mg +  ... (i)
R mg
mvB2
At point B; TB =  ... (ii) T
R
mv 2C
At point C; TC = – mg  ... (iii)
R
1
→ → v 2c
Energy at point A = mv 2A (point A is assumed to be ∴ a net = a r = = 60 m/s2
2 R
ground)
P hysi cs | 4.115

Sol 63: mv 2 Maximum friction between B and C = µ (mA + mB)g


N cosθ
R or f2 = 0.25 (3 + 4)(10) = 17 5 N
mv 2
R Maximum friction between C and ground
mg sinθ
f3 = µ (mA + mB + mC)g
θ mg
= 0.25(3 + 4 + 8)(10) = 37.5 N
mv 2
Block C and hence block B are moving in opposite
sinθ + mg cosθ
R directions with constant velocities and block A is at rest.
mv 2 Hence, net force no all three blocks should be zero. Free
N= sinθ + mg cosθ  ... (i) body diagrams have been shown below (Only horizontal
R
forces are shown)
Now depending on condition, friction can be upwards
f1 f2
or downwards.
B C T
For maximum speed, friction is downwards. T F
f2 f3
2
mv
∴f= cosθ – mg sinθ  ... (ii)
R For equilibrium of B
And also f = mN T = f1 + f2 = 25 N
 mv 2  mv 2 For equilibrium of C
⇒ µ sin θ + mgcos θ  = cos θ − mgsin θ
 R  R F = T + f2 + f3 = 80 N
 
 µ msin θ mcos θ  F
v2  –  = – (mg sinθ + mmg cos θ) Sol 2: Acceleration of rope a =
 R R  M
L
mg(sin θ + µ cos θ) F
v= M
M
m
(– µ sin θ + cos θ) A a B
R
a a
Rg(sin θ + µ cos θ)
vmax. = L–  
(cos θ – µ sin θ) T T
F
And for the minimum speed; A C C B
friction will be acting upwards Now to find tension at point C, a distance  from point
mv 2 B, we can write equation of motion of any one part (AC
∴ f = mg sinθ – cosθ  ... (iii)
R or CB), both moving with acceleration a.
And following the same argument
Equation of motion of part AC is
for f = 0
M F
 v2  T = (mass of AC) × (acceleration) = (L – )  
tanθ =   L M
 Rg   
  = F 1 – 
 L
v= Rgtanθ  ... (iv)
1
So 3: mg sinθ = (2)(10)   = 10 N = F1 (say)
2
Previous Years’ Questions
3
µ=
Forces and Laws of Motion 2kg 2

Sol 1: Maximum friction between A and B= mmAg


30°
or f1 = 0.25(3)(10) = 7.5 N
4 . 1 1 6 | Forces and Laws of Motion

 3  3 Equation of motion of M1
mmg cosθ =   (2) (10)   = 21.21 N = F2 (say)
 2  2  T + F1 – f1 = M1a  ... (i)
   
or T = 4a
(a) Force required to move the block down the plane
with constant velocity. Equation of motion M2

F2 F2 – T – f2 = M2a
F
v or 7.84 – T = 2a  ... (ii)
Solving eqs. (i) and (ii), we get

F1 a = 1.3 m/s2 and T = 5.2 N


30°
Sol 5: Constant velocity means net acceleration of the
F1 will be acting downwards, while F2 upwards. system is zero. Or net pulling force on the system is
zero. While calculating the pulling force, tension forces
Since F2 > F1, force required
are not taken into consideration. Therefore,
F = F2 – F1 = 11.21 N
(a) M1g = M2g sin37° + µ M2g cos37° + µ M3g
(b) Force required to move the block up the plane with
or M1 = M2 sin37° + µM2 cos37° + µ M3
constant velocity.
Substituting the values
F
v 3 4
M1 = (4)   + (0.25)(4)   +(0.25)(4) = 4.2 kg
5 5

F1+F2 (b) Since, M3 is moving with uniform velocity

30° T = µ M3g = (0.25)(4)(9.8) = 9.8 N

M3
F1 and F2 both will be acing downwards.
F = F1 + F2 = 31.21 N T
µM3g
Sol 4: Maximum force of friction between M1 and
inclined plane Sol 6: F
f1 = m1M1g cos θ = (0. 75)(4)(9. 8)(0.8) = 23.52 N θ

M1g sin θ = (4)(9.8)(0.6) = 23.52 N = F1 (say) m


µ
Maximum force of friction between M2 and inclined
plane Let F be applied at angle θ as shown in figure. Normal
f2 = m2M2g cos θ = (0. 25)(2)(9.8)(0.8) = 3.92 N reaction in this case will be
M2g sin θ = (2)(9.8)(0.6) = 11.76 N = F2 (say) N = mg – F sin θ
The limiting friction is therefore
f1
a fL = mN = µ(mg – F sin θ)
M1
T For the block to move,
T
F1
f2
F cos θ = fL = µ(mg – F sin θ)
M2 a
µmg
or F =  … (i)
F2 cos θ + µ sin θ
θ
For F to be minimum, denominator should be maximum.
Both the blocks will be moving downwards with same d
acceleration a. Different forces acting on two blocks are or (cos θ + µ sin θ) = 0

as shown in figures.
or – sin θ + µ cos θ = 0
P hysi cs | 4.117

or tan θ = µ or θ = tan–1 (µ) Therefore, minimum value of ω should be


Substituting this value of θ in Eq. (i), we get g 9.8
wmin = = rad/s
Fmin = mg sinθ R 0.1
or wmin = 9. 89 rad/s
Sol 7: (a) To find tension at mid-point of the lower g
(b) Eq. (iii) can be written as h = R –
wire we cut the string at this point. Draw the free body ω2
diagram of lower portion. If R and ω are known precisely, then
T1 ∆g
Dh = – 2 or Dg = w2 Dh (neglecting the negative
a ω
sign)
0.1kg
1.9kg (Dg)min = (wmin)2 Dh, (Dg)min = 9. 8 ×10–3 m/s2

(1.9 +0.1)g =2.0 g Sol 9: (a) CP = CO = Radius of circle (R)


∴ ∠CPO = ∠POC = 60°
The equation of motion gives
∴ ∠OCP is also 60°
T1 - 2.0g = (2.0) a
Therefore, DOCP is an equilateral triangle.
or T1=(2.0)(g + a) = (2. 0)(9.8 + 0.2) = 20 N

Sol 8: Given : R = 0 1m, m = 10–2 kg

C
R R–h
N θ
r
h 60° P
mg
O

(a) FBD of particle in ground frame of reference is shown


Hence, OP = R
in figure. Hence,
r
tan θ =
R –h

N cos θ = mg  ... (i)


C
and N sin θ = mrw2  ... (ii) N

Dividing Eq. (ii) by Eq. (i), we obtain F P


2 2
rω r rω O
tan θ = or =
g R –h g mg
g
or w2 =  ... (iii)
R –h Natural length of spring is 3R/4.
This is the desired relation between ω and h. ∴ Extension in the spring
From Eq. (iii) 3 R R
x=R– =
g 4 4
h=R–
ω2 ⇒ Spring force,
Form non-zero value of h
g
R > 2 or ω > g / R
ω
4 . 1 1 8 | Forces and Laws of Motion

m g R  mg
F = kx =     =
 4
 R  4 ω
m1 m2
The free body diagram of the ring will be as shown. r1 × r2
mg r
Here, F = kx =
4
and N = Normal reaction.
(b) Tangential acceleration aT : The ring will move Free body diagram of m1 and m2 with respect to ground
forwards the x-axis just after the release. So net force m1 m2
T+f T
along x-axis
T = m2r2w2  … (i)
Since, m2r2w2 < m1r1w2
y Therefore, m1r1w2 > T
and friction on m1 will be inward (toward centre)
N
60°
f + T = m1r1w2  … (ii)
60°
F from equations (i) and (ii), we get
O 60°
ax f = m1r1w2 – m2r2w2  … (iii)
mg = (m1r1 – m2r2)w 2

Fx = F sin 60° + mg sin60° = (10 × 0.124 – 5×0. 176) (10)2 N = 36 N


m g 3  3 Therefore, frictional force on m1 is 36 N (inwards)
=  
 + mg  
 2 
 4  2   (b) From eq. (iii)

5 3 f = (m1r1 – m2r2) w2
Fx = mg
8 Masses will start slipping when this force is greater than
Therefore, tangential acceleration of the ring, fmax or
Fx 5 3 (m1r1 – m2r2)w2 > fmax > mm1g
aT = ax = = g
m 8 ∴ Minimum value of ω is
Normal reaction N : Net force along y-axis on the ring
just after the release will be zero. µm1g 0.5 × 10 × 9.8
wmin = =
m1r1 – m2r2 10 × 0.124 – 5 × 0.176
Fy = 0
∴ N + F cos 60° = mg cos 60° wmin = 11.67 rad/s

∴ N = mg cos60° – F cos 60° (c) From Eq. (iii), frictional force f = 0


mg mg 1 mg mg r1 m 5 1
= –   = – where m1r1 = m2r2 or = 2 = =
2 4 2
  2 8 r2 m1 10 2
3mg and r = r1 + r2 = 0. 3 m
N=
8 ∴ r1 = 0.1 m and r2 = 0.2 m
i. e. , mass m2 should be placed at 0.2 m and m1 at 0.1 m
Sol 10: Given,
from the centre O.
m1 = 10 kg, m2 = 5 kg, ω = 10 rad/s
r = 0.3 m, r1 = 0.124 m Sol 11: Acceleration of block A

∴ r2 = r – r1 = 0.176 m Maximum friction force that can be obtained at A is

(a) Masses m1 and m2 are at rest with respect to rotating (fmax)A = µA(mg cos 45°)
table. 2 2mg
= (mg/ 2)=
Let f be the friction between mass m1 and table. 3 3
P hysi cs | 4.119

The FBD of the whole system will be as shown in the


figure
m 2m
A B T T
T T
mg sin45° 2mg sin45°
mg 45° 45° 2mg A fB =2mg B
= =F2 = =F1 3
2 2 mg =(fB)max
F2= fA = 2mg
2
mg F1=
Similarly, 3 2 2
(fmax)B = µB(2mg cos 45°)
Therefore, friction on A is
1 2mg
= (2mg/ 2 ) = fA = mg/3 2 (down the plane)
3 3
Therefore, maximum value of friction that can be Now for tension T in the string, we may consider either
obtained on the system is equilibrium of A or B
2 2mg Equilibrium of A gives
(fmax) = (fmax)A + (fmax)B =  ... (i)
3
mg mg 4mg 2 2mg
Net pulling force on the ststem is T = F2 + fA = + = or
2 3 2 3 2 3
2mg mg mg
F = F 1 – F2 = – =  ... (ii)
2 2 2 Similarly, equilibrium of B gives T + fB = F1
2mg 2mg 4mg
From Eqs. (i) and (ii), we can see that or T = F1 – fB = – =
2 3 3 2
Net pulling force < fmax
2 2mg
Therefore, the system will not move or the acceleration or
3
of block A will be zero. 2 2mg
Therefore, tension in the string is
(b) and (c) Tension in the string and friction at A 3
Net pulling force on the system (block A and B) Sol 12: Acceleration of A down the plane,
F = F1 – F2 = mg/ 2 aA = g sin 45° – µA g cos 45°
Therefore, total friction force on the blocks should also  1   1 
mg = (10)   – (0. 2)(10)   = 4 2 m/s
2

be equal to  2  2
2
Similarly acceleration of B down the plane,
or fA + fB = F = mg/ 2
aB = g sin 45° – µB g cos 45°
Now since the blocks will start moving from block B first
(if they move), therefore, fB will reach its limiting value  1   1 
(10)   – (0.3)(10)   = 3.5 2 m/s
2
first and if still some force is needed, it will be provided
 2  2
by fA
The front face of A and B will come in a line when,
Here, (fmax)B < F
sA = sB + 2
Therefore, fB will be in its limiting value and rest will be
provided by fA. 1 2 1 2
or a t = aBt + 2
2mg 2 A 2
Hence fB = (fmax)B =
3 1 1
× 4 2 × t2 = × 3. 5 2 × t2 + 2
mg 2 2
mg 2mg
and fA = F – fB = – =
2 3 3 2 Solving this equation, we get t = 2s
1 2 1
Further, sA = at = × 4 2 × (2)2 = 8 2 m
2 A 2
Hence, both the blocks will come in a line after A has
travelled a distance 8 2 m down the plane.
4 . 1 2 0 | Forces and Laws of Motion

Circular Dynamics (towards centre) and second along tangential. Along


2
radius net force should be equal to mv and along
Sol 13: (A) Tangential force (F1) of the bead will be given R
by the normal reaction (N), while centripetal force (Fc) is
provided by friction (fr). The bead starts sliding when
tangent it should be equal to O
the centripetal force is just equal to the limiting friction.
maT, where aT is the
ω tangential acceleration in θ
the figure. T
mv 2 v
T – Mg cos θ = and
L F1 L
A x
Mg sin θ = MaT θ
Fc mg sinθ
or aT = g sinθ mg cosθ
mg

Ft is inwards ∴ Correct options are (b) and (c).

Therefore, Ft = ma = mαL = N Sol 16: (B, D) A rotating/revolving frame is accelerating


and hence non-inertial. Therefore, correct options are (b)
∴ Limiting value of friction and (d).
(fr)max = mN = mmαL  … (i)
Sol 17: (A, C) –1 1kg
Angular velocity at time t is ω = at 2 ms
∴ Centripetal force at time t will be ⇒
Fc = mLw = mLa t
2 2 2
 … (ii) 1 kg 5 kg

µ
Equating equation (i) and (ii), we get t =  2m2 
α v1' =  m1 – m2  v +   v2
m +m  1
µ  1 2   m1 + m2 
For t > , F > (fr)max i.e. , the bead starts sliding.
α c 1 –5
–2=   v1 + 0 (as v2 = 0)
In the figure Ft is perpendicular to the paper inwards. 1 +5
∴ v1 = 3 ms–1
Sol 14: (A) Since, the block
rises to the same heights in  m2 – m1   2m1 
v0 v 2'
all the four cases, from =   v2 +   v1
conservation of energy,  m2 + m1   m1 + m2 
speed of the block at
 2×1 
highest point will be same N + mg =0+   (3) = 1 ms
–1

in all four cases. Say it is v0.  6 


Equation of motion will be PCM = Pi = (1)(3) = 3 kg-m/s
mv 20 mv 20
N + mg = or N= – mg
R R P5' = (5) (1) = 5 kg-m/s
R (the radius of curvature) in first case is minimum.
2
Therefore, normal reaction N will be maximum in first PCM 9
case. KCM = = = 0. 75 J
2MCM 2×6
Note In the question it should be mentioned that all the
1
four tracks are frictionless. Otherwise, v0 will be different Ktotal = × 1 × (3)2 = 4. 5 J
2
in different tracks.
∴ Correct options are (a) and (c).
Sol 15: (B, C) Motion of pendulum is part of a circular
motion. In circular motion it is better to resolve the Sol 18: (B) (a) CP = CO = Radius of circle (R)
forces in two perpendicular directions. First along radius ∴ ∠CPO = ?POC = 60°
P hysi cs | 4.121

∴ ∠OCP is also 60° Therefore, tangential acceleration of the ring,


Therefore, DOCP is an equilateral triangle. Fx 5 3
aT = ax = = g
m 8
Normal reaction N Net force along y-axis on the ring just
after the release will be zero
Fy = 0
C
∴ N + F cos 60° = mg cos 60°
∴ N = mg cos60° – F cos 60°
60° P
O mg mg  1  mg mg
= –   = –
2 4 2 2 8
Hence, OP = R
3mg
N=
8

Sol 19: (A, C)

C
N

F P
O

Natural length of spring is 3R/4. If θ = 45º then cos θ = sin θ hence block will be at rest.
3 R R
∴ Extension in the spring x = R – = If plane is rough & θ > 45° then sin θ > cos θ so friction
4 4
will act up the plane
m g R  mg
⇒ Spring force, F = kx =    4  = 4 If plane is rough & θ < 45 then cos θ > sin so friction will
 R   
act down the plane so (A, C) are correct
The free body diagram of the ring will be as shown.
mg
Here, F = kx = and N = Normal reaction. Sol 20: (D) Initially bead is applying radially inward
4 normal force.
(b)Tangential acceleration aT the ring will move
During motion at an instant, N = 0, after that N will act
forwards the x-axis just after the release. So, net force
radially outward.
along x-axis
Sol 21: (D) Condition for not sliding,
fmax > (m1 + m2) g sin θ
y mN > (m1 + m2) g sin θ
0.3 m2 g cos θ ≥ 30 sin θ
N
60° 6 ≥ 30 tan θ
60°
F 1/5 ≥ tan θ
O 60°
ax 0.2 ≥ tan θ
mg ∴ for P, Q
m g 3  3 f = (m1 + m2) g sin θ
Fx = F sin 60° + mg sin60° =   + mg  
 4  2  2  For R and S
 
5 3
Fx = mg F = fmax = mm2g sin θ
8
2017-18 100 &
op kers
Class 11 T
By E ran culty
-JE Fa r
IIT enior emie .
S fP r es
o titut
Ins

PHYSICS
FOR JEE MAIN & ADVANCED
SECOND
EDITION

Exhaustive Theory
(Now Revised)

Formula Sheet
9000+ Problems
based on latest JEE pattern

2500 + 1000 (New) Problems


of previous 35 years of
AIEEE (JEE Main) and IIT-JEE (JEE Adv)

5000+Illustrations and Solved Examples


Detailed Solutions
of all problems available

Topic Covered Plancess Concepts


Tips & Tricks, Facts, Notes, Misconceptions,
Work, Energy and Power Key Take Aways, Problem Solving Tactics

PlancEssential
Questions recommended for revision
5. WORK, ENERGY AND
POWER

1. INTRODUCTION
This chapter explains the concepts of work and energy and how these quantities are related to each other. The
law of conservation of energy is an important tool in physics, for the analysis of motion of a system of particles or
bodies, and in understanding various phenomena in nature. When the nature of forces involved in a process are
not exactly known, or when we want to avoid complicated calculations, then the law of conservation of energy
proves to be an indispensable tool in solving many problems. The importance of energy cannot be explained in
words. The progress of science and civilization is based on finding new ways to efficiently use the energy available
in nature in various forms. Energy is required by a person to perform his/her daily activities, as well as to run our
automobiles and machines. Depletion of natural energy resources is a major concern these days. The efficiency
of energy utilization processes and quantity of energy sources harnessed by a country determines the pace of its
economic development.

2. WORK

2.1 Work
In physics, a force is said to do work only when it acts on a body, and if there is a consequential displacement of
the point of application in the direction of the force.
For example, say if a constant force F displaces a body through displacement s then the work done, W, is given by
 
W Fscos
= = θ F.s
where s is magnitude of displacement and θ is angle between force and displacement. The SI unit of work is Joule
or Newton-metre.
Sign Convention of Work
F
0 Direction of Motion

Figure 5.1: Motion of block in directon of applied force

We now define the sign convention of work as follows:


When 0 < θ < 900 ,
then
= W Fs cos θ is positive
i.e., when the force constantly supports the motion of a body,work done by that force is said to be positive.
5 . 2 | Work, Energy and Power

F 0 Direction of Motion

Figure 5.2: Motion of block

then W = Fs cos θ =− ve
i.e., in this case force is not truly supporting the motion of the body and hence the work done by that force is said
to be negative.

2.2 Nature of Work


 
Work done is signified by the equation: F. S
Based on this equation, three possible situations are possible regarding the nature or sign of the work done as
listed here under:
(a) To begin with, the work done is said to be positive if the angle between the force and the displacement
vectors is an acute angle.
E.g., when a horse pulls a cart on a level road, the work done by the horse is positive.
(b) Second, the work done is zero if the force and the displacement vectors are perpendicular to each other.
E.g., when a body is moved along a circular path by a string, then the work done due to the string is zero.
(c) The last possible situation is that the work done is said to be negative if the angle between the force and the
displacement vectors is an obtuse angle.
E.g., when a body slides over a rough surface, the resultant work done due to the frictional force is negative. (It
is pertinent here to remember the fact that the angle between the force and the displacement is 180 degrees.)

PLANCESS CONCEPTS

Students should be able to deduce that by positive work, force is actually doing what it is meant for, i.e.
force wants to move a body in certain direction and if it moves in that direction then it’s positive work.
Anurag Saraf (JEE 2011, AIR 226)

 
( ) ( )
Illustration 1: Assume that a body is displaced from rA = 2m, 4m,− 6m to rB = 6i − 4 j + 2k m under a constant

force F = ( 2i + 3j − k ) N . Now, calculate the total work done. (JEE MAIN)
 
Sol: The work done by the constant  force
 F during displacement S of a particle is scalar product of force and
displacement and is given by W = F • S
  ∧ ∧ ∧      ∧ ∧ ∧   ∧ ∧ ∧  ∧ ∧ ∧
rA = 2 i + 4 j − 6k  mS =rB − rA = 6 i − 4 j + 2k  −  2 i + 4 j − 6k  =4 i − 8 j + 8k
     
     
   ∧ ∧ ∧  ∧ ∧ ∧ ∧
W =F.S = 2 i + 3 j − k  .  4 i − 8 j + 8k  =8 − 24 − 8 =( −24 j)
  
  

Illustration 2: A block of total mass 5 kg is being raised vertically upwards with the help of a string attached to it
and it rises with an acceleration of 2 m/s2. Find the work done due to the tension in the string if the block rises by
2.5 m. Also, calculate the work done due to the gravity and the net work done. (JEE ADVANCED)
P hysics | 5.3

Sol: The tension in the string is acting vertically upwards and the block is also moving vertically
T
upwards, so the work done by the tension will be positive. The force of gravity is acting vertically
downwards so the work done by gravity will be negative. m a
Let us first calculate the tension T. mg
From the force diagram T-mg = 5a; T = 5(9.8 + 2) = 59 N.
Figure 5.3
As it is clear that both T and displacement S are in the same direction (upwards), then work done
by the tension T is W based on which we calculate that W = Ts
= 59(2.5) = 147.5 J .
Now, work done due to gravity = –mgs = –5(9.8) (2.5) = –122.5 J
Therefore, net work done on the block = work done by T + work done by mg = 147.5+ (–122.5) = 25 J.

PLANCESS CONCEPTS

Point of application of force also plays a major role.


Zero work is done by a force in following cases: -If the point of application of force is not changed in
space but the body moves. If body doesn’t move but the point of application of force moves.
Nivvedan (JEE 2009, AIR 113)

3. WORK DONE BY A VARIABLE FORCE


We need to be aware of the fact that when the force is an arbitrary function of position, then we need the principles
of calculus to evaluate the work done by it. The Fig. 5.4 given here under shows F (x) as some function x. We now
begin our evaluation in this regard by replacing the actual variation of the force by a series of small steps. In the
Fig. 5.4 provided, the area under each segment of the curve is approximately equal to the area of a rectangle. Based
on the height of the rectangle, the amount of work done is given by the relation, ∆Wn =Fn ∆ Xn . Therefore, the
total work done is approximately given by the summation of the areas of both the recangles: W ≈ ∑ Fn ∆Xn . As the
number of the steps is reduced, the tops portions of the rectangle more closely resemble the actual curve shown
in the Fig.5.4. In limit ∆x → o , which is equivalent to letting the number of steps to be infinite, the discrete sum is
2 x
replaced by a continuous integral. W = ∫x1
F(x) dx =
area under the F − x curve and the x − axis

b
a
F(x) F(x)
F(x)

d
0 xi xf x 0 xi x xf x

Figure 5.4: Work done on particle by variable force

Illustration 3: A force=F (10 + 0.50X) is observed to act on a particle in the x direction, where F is in newton and
x in meter. Find the actual work done by this force during a displacement from x=0 to x=2.0 m. (JEE MAIN)

Sol: If a particle is being displaced under action of variable force, the work done by this force is calculated as
s  
W = ∫ 2 F • ds .
s1

As we know that the force is a variable quantity, we shall find the work done in a small displacement from x to x +
5 . 4 | Work, Energy and Power

dx and then integrate the resultant value to calcuate the total work done. The work done in this small displacement
is calculated as 2.0 2.0
   x2 
dW
= F.dx
= ∫ (10 + 0.50x ) dx
(10 + 0.50x ) dx . Thus, W = =+


10x 0.50  =
2 
21 J.
0 0

4. CONSERVATIVE AND NON-CONSERVATIVE FORCES


A force is said to be of the conservative category if the work done by it in moving a particle from one point to
another does not depend upon the path taken but depends only upon the initial and final positions. The work done
by a conservative force around a closed path calculated to be zero. Gravitational force, electric force, spring force,
etc. are some of the examples of this category. Basically, all central forces are conservative forces. In contrast, if
the work done by a force in moving a body from one point to another depends upon the path followed, then the
force is said to be of the nonconservative category. The work done by such a force around a closed path cannot be
zero. For example, both the frictional and viscous forces work in an irreversible manner and hence a definite part
of energy is lost in overcoming these frictional forces. (Mechanical energy is converted to other energy forms such
as heat, sound, etc.). Therefore, these forces are of the nonconservative category.

5. WORK DONE AGAINST FRICTION


We know that the frictional force always acts opposite to the direction of motion (and hence direction of the
displacement); therefore, the work done by the frictional force is always on the negative side. Further, the work
done by the frictional force is invariably lost in the form of heat and sound energy and thus it is a nonconservative
force.

PLANCESS CONCEPTS

The work done by the frictional force is either negative or zero, but never positive. The frictional force
always resists the attempted work done along a horizontal surface. Work done along a horizontal surface
is given by: - µmgl , where
v v

V V
f f

mg mg
l
Figure 5.5

m is the mass of the object ;


µ is the coefficient of friction
g is the acceleration due to gravity (9.8m/s2)
l is the distance traveled by the block along the rough surface
Similarly, work done along an inclined surface with an angle θ from horizontal is given by - µmglcos θ
Nitin Chandrol (JEE 2012, AIR 134)

Illustration 4: It is observed that a block of mass 4 kg slides down a plane inclined at 37o with the horizontal. The
length of the plane is calculated to be of 3 m. The value of the coefficient of sliding friction between the block and
the plane is 0.2. Based on the above, find the work done due to the gravity, the frictional force, and the normal
reaction between the block and the plane.  (JEE MAIN)
P hysics | 5.5

Sol: Normal reaction is always perpendicular to the inclined plane hence it is perpen-
dicular to the displacement and thus the work done by it is zero. Whereas the frictional
force is in opposite direction to the displacement and hence the work done by the
firctional force is negative. The work done by the component of gravitational force o
along the inclined plane will be positive.
Total force acting on the block moving on inclined plane constitutes frictional force, Figure 5.6
normal reaction due to ground and gravitational force acting on wire. The work done
on block is given as
= W Fs cos θ
As the normal reaction is perpendicular to the point of displacement, work done by
the normal reaction R = R s cos 90° = 0. The magnitude of displacement s = 3 m and s
the angle between force of gravity (mg) and displacement is equal to (90°-37°). R
Therefore, work done by gravity = mgs cos (90°-37°)
mgs sin 370 4=
== x 9.8 x 3 x 3 / 5 70.56 J mg
Work done by friction = –(µR)s = –(µ mg cos 37°)s = –0.2 × 4 × 9.8 × 4 / 5 × 3 = –18.816 J. Figure 5.7

6. POWER
Power is defined as the rate at which the actual work is done. If an amount of work ∆W is done in time ∆t , then
∆W  ∆W  dW
average power, Pn = and instantaneous= power, P Lim=   .
∆t ∆t →0  ∆t  dt
It is a well-known fact that work done by a force F on an object that has infinitesimally small displacement ds is

dW Fds  
dw=F.ds. Then, instantaneous power,
= P = = F. v .
dt dt
The S I unit of power is Watt (W) or Joule/second (J/s) and it is a scalar quantity. Dimensions of power is M1L2 T −3 .

Illustration 5: A block of mass m is allowed to slide down a fixed smooth inclined plane of angle θ and length  .
Calculate the magnitude of power developed by the gravitational force when the block reaches the bottom. 
 (JEE ADVANCED)
 
Sol: The power dlivered by the force F is the scalar product of the force and velocity i.e. P = F.v

When body reaches bottom of the inclined plane the velocity of of body= is v =
2gh 2g•  sin θ and the angle

 
between velocity and vertical will be ( 90 − θ ) . =
o
P F. V= mgsin θ 2g sin θ= 2m2g3  sin3 θ .

Illustration 6: A particle of mass m is moving in a circular path of constant radius r such that its centripetal
2 2
accelecration aC is varying with time t as ac = k rt , where k is a constant. The power delivered to the particle by
the force acting on it is (JEE MAIN)

(A) 2π mk 2 r 2 (B) mk 2 r 2 t (C)


(
mk 4r 2 t5

) (D) Zero
3
Sol: (B) As the centripetal force is perpendicular to the direction of the velocity, the work done and power delivered
by the centripetal force will be zero, whereas the tangential force is in the direction of the velocity so the power
delivered to the particle of mass m is P = Ft • v
2
Here ac= = k 2 rt2 or v k=2 2
rt or v krt
r
dv
Therefore, tangential acceleration, = at = kr or tangential force, Ft=m at =m kr
dt
However, only tangential force does work. Power = Ftv = (mkr)(krt) or Power = mk2r2t
5 . 6 | Work, Energy and Power

7. ENERGY
Generally, the energy of a body is signified by the body’s capacity to do work. It is a scalar quantity and shares
the same unit as that of work (Joule in SI unit). In mechanics, both kinetic and potential energies are involved with
dynamics of the body.

7.1 Potential Energy

7.1.1. Potential Energy


Potential energy of a body is the energy possessed by virtue of its position or due to its state. It is independent of
the way in which the body is transformed to this state. Although it is a relative parameter, it depends upon its value
at reference level. We can define the change in potential energy as the negative of work done by the conservative
force in operation in carrying a body from a reference position to the position under consideration.

7.1.2 Definition
∆U = -WAB where A is the initial state, B is the final state, and WAB is the total work done by conservative forces. We
know that potential energy depends upon the work done by conservative force only. Hence, it cannot be defined
for the nonconservative force (s). This is because of the proven fact that in this type work done depends upon the
path followed alone.

7.1.3 Gravitational Potential Energy (GPE)


Suppose if we lift a block through some height (h) from A to B, then the work is done defying the gravity. The
work done in such a case is stored normally in the form of gravitational potential energy of the block-energy
system. Therefore, we can write that work done in raising the block = (mg)h. This is exactly equal to the increase in
gravitational potential energy (GPE) of the block.
If the center of a body of mass m is raised by a height h, then increase in GPE = mgh
If the center of a body of mass m is lowered by a distance h, decrease in GPE = mgh

7.1.4 Elastic Potential Energy


Suppose when a spring is elongated (or compressed), then work is done against the restoring force of the spring.
This resultant work done is stored in the spring in the form of elastic potential energy.

7.1.5 Nature of Restoring Force


Suppose if a spring is extended or compressed by a distance x, the spring then exerts a restoring force so as to
oppose this change.

PLANCESS CONCEPTS

GPE is always thought of as only of block. But to be more specific it is the energy of block-earth system.
Potential energy never comes in context of a single particle. It is always for a configuration. In the case
of GPE, writers however generally skip writing “Earth” each time.
Chinmay S Purandare (JEE 2012, AIR 698)

7.1.6 Spring
In case of a spring, natural length of the spring is assumed to be the reference point and correspondingly is always
assigned zero potential energy (This is a universal assumption.). However, in gravity, we can choose any point as
P hysics | 5.7

our reference and hence assign it any value of potential energy.

l0 1

Figure 5.8: Energy stored in stretched spring

For Stretching
Xi l0
f  1
− ∫ kx ( −i )( dx )i ;U = kx12 
− ∫ F.dS ;Uf − 0 =
Uf − Ui =
i 0
2 l0 -x +ve
x
For Compression
f  Xi Figure 5.9: Energy stored
1
Uf − Ui =− ∫ F.dS =− ∫ kxi ( dx )( −i ) =U = kx2 in compressed spring
i 0
2

Thus, if the spring is either stretched or compressed from natural length by x the corresponding potential energy
is 1 / 2kx2

7.1.7 Relationship between Force and Potential Energy


Now, let us discuss the relationship between force and potential energy.

r
A B

U+U
Figure 5.10

Let us assume that a body is taken from A to B in such away that there is no net change in its kinetic engery. Then

⇒ Work done = − change in P.E. ; F ∆r = U − (U + ∆U) = − ∆U


 ∆U  ∆u ∂U
⇒ avg =− 
Favg  if ∆r → 0; F =− lim =−
 ∆r  ∆rr →
∆ →00 ∆r ∂r

7.2 Kinetic Energy


Kinetic energy (KE) is the energy of a body possessed by virtue of its motion alone. Therefore, a body of mass m
1
and moving with a velocity v has a kinetic energy Ek = mv 2 .
2
We already know that velocity is a relative parameter; therefore, KE is also a relative parameter.
We provide a detailed account on kinetic energy after presenting the concept of conservation of mechanical
energy.

8. EQUILIBRIUM
We have already studied in the chapter on “Laws of Motion” that a body is said to be in translatory equilibrium only
if net force acting on the body is zero, i.e., Fnet
net
==00
dU
However, if the forces are conservative, then F = − ; for equilibrium, then
dr
5 . 8 | Work, Energy and Power

dU dU
F = 0; =
Thus, − 0, or
= 0
dr dr
i.e., exactly at the equilibrium position the slope of U-r graph is zero or the potential energy is optimum (maximum
or minimum or constant). Equilibria are of three types, i.e., stable equilibrium, unstable equilibrium, and neutral
equilibrium. Further, the situations where F = 0 and dU/dr = 0 can be obtained only under three conditions as
specified hereunder.
d2U
(a) If > 0, then it is stable equilibrium;
dr 2
d2U
(b) If < 0, then it is unstable equilibrium; and
dr 2
d2U
(c) If = 0, then it is neutral equilibrium.
dr 2

PLANCESS CONCEPTS

A system always wants to minimize its energy. The above equilibriums are categorized only on this
basis. Stable indicates that if system is disturbed slightly, from these configuration, it would try to come
back to its original state (position of energy minima). For unstable equilibrium, a slight disturbance
would cause the system to find some other suitable configuration (position of energy maxima). A neutral
equilibrium is generally found when U becomes constant and each position is a state of equilibrium. A
slight disturbance has no after reactions and the new state is also an equilibrium position.
Anurag Saraf (JEE 2011, AIR 226)

Illustration 7: The potential energy of a particle of mass 5 kg, moving in xy plane, is given by U = (-7x + 24y)J
  ∧ ∧
=
where x and y being in meters. Initially (at t=0), the particle is at the origin and has velocity v  14.4 i + 4.2 j  m / s .
 
 
Then Calculate (a) the acceleration of the particle and (b) the direction of acceleration of the particle. (c) The speed
of the particle at t = 4 s.  (JEE MAIN)
dU
Sol: If particle has potential energy U then corresponding conservative force, is F = − and according to the
dr
 ay

Newton’s second law of motion F = ma . The direction of acceleration is calculated as tan θ = .
ax
(a) Acceleration,
δU δU
Fx = ,F = − ⇒ Fx =
7N, Fy =
− 24N; ⇒ ax =
7 / 5, ay =
−24 / 5
δx y δy
 ay 
(b) Direction of acceleration θ =tan−1   ;
 ax 
 
   7 24
(c) v = u + a t ; v x = 14.4 + × 4 = 20 ; v y =4.2 − × 4 =( −15 )
5 5

Illustration 8: The potential energy of a particle in a certain field has the form U= a/r2 – b/r, where a and b are
positive constants and r is the distance from the center of the field. Find the value of r0 corresponding to equilibrium
position of the particles and hence examine whether this position is stable.  (JEE ADVANCED)

dU
Sol: Conservative force acting on the particle is F = − . Under stable equilibrium particle has minimum potential
dr
P hysics | 5.9

energy while potential energy is maximum in case of unstable equilibrium.

U(r) a / r 2 − b / r
=

Force =
F=

dU
=
 −2a b 
− +  ; F= −
(br − 2a)
dr  r3 r2  r3
dU
At equilibrium, then=F = 0
dr

Hence, br – 2a = 0 at equilibrium.
Further, r = r0 = 2a/b corresponds to equilibrium.
At stable equilibrium, the potential energy of a particle is at its minimum, whereas at unstable equilibrium, it is the
maximum. From the principles of calculus, we know that for minimum value around a point r = r0, the first derivative
should be zero and the second derivative should be invariably positive.
For minimum potential energy, the applicable conditions are
dU d2U
=0 and >0 at r=
r0
dr dr 2
However, we have already used dU/dr = 0 to obtain r = r0 = 2a/b.
Now, in a similar way let us investigate the second derivative.

d2U d  dU  d  2a b  6a 2b
=   = − +  = −
dr 2 dr  dr  dr  r3 r 2  r 4 r3

d2U 6a − 2br0 2a
At r= r0= 2a / b, = = > 0.
dr 2 r04 r04
Based on our calculations, the potential energy function U(r) has a minimum value only when r0 = 2a / b . Therefore,
we conclude that the system has stable equilibrium only at the minimum potential energy state.

9. WORK ENERGY THEOREM


Suppose that a particle is acted upon by various forces and consequently undergoes a displacement. Then there is
a change in its kinetic energy by an amount equal to the total (net) work (Wnet) done on the particle by all the forces.
i.e., Wnet = ∆K 
K f − Ki = … (i)
We call the above expression as the work-energy theorem.
Expression (i) is valid irrespective of the fact that whether the forces are constant or varying and whether the path
followed by the particle is straight or curved.
We further elaborate expression (i) as follows:
∆K 
Wc + WNC + WOth = … (ii)
where Wc is the work done by conservative forces
Wsc is the work done by nonconservative forces
Woth is the work done by all other forces which are not included in the category of conservative, nonconservative,
and pseudo forces.
‘Since Wc = ∆U ’ (based on definition of potential energy), therefore, expression (ii) can be accordingly modified as
WNC + Woth = ∆K + ∆U = ∆ (K + U) = ∆ E  … (iii)
In expression (iii), the term K + U = E is known as the mechanical energy of the system.
5 . 1 0 | Work , Energy and Power

Illustration 9: Find how much will mass “m” rise if 4 m falls away. Block are at rest and in equilibrium (JEE MAIN)

Sol: Initially the block is at rest. When the block rises to the maximum height, it again comes to rest momentarily.
So, by work energy theorem the total work done on the block by force of gravity and spring force is zero.
Applying work energy theorem (WET) on a block of mass m 
Wg + Wsp = K.E.f − K.E.i k
Let the final displacement of the block from the initial equilibrium is x. Then
m
 5mg  1  25m g 2 2  1 2 1 15m g 2 2
3mg 4m
− mg  +X+ k = 0; kx2 + mgx −
 − kx = 0; x
=
 k  2  k  2 2 2k k
2
 Figure 5.11

PLANCESS CONCEPTS

Whenever there is frictional force, energy is dissipated which is equal to work done by frictional force
and the dissipated energy converts into heat. Practically, machine handlers do a lot of things to minimize
friction and reduce energy losses by applying lubricants and rollers in their parts.
Yashwanth Sandupatla (JEE 2012, AIR 821)

Illustration 10: A body of mass m was slowly hauled up the hill as shown in the Fig. 5.12 provided by a force F
which at each point was directed along a tangent to the trajectory. Find the work done due to this force if the
height of the hill is h, the length of its base is l, and the coefficient of friction is m. (JEE ADVANCED)

Sol: As block hauls slowly, the kinetic energy will not change throughout the motion.
And the sum of the work done by applied force, gravitational force, normal reaction
and frictional force will be zero as per work energy theorem.
Fi m
h
The four forces that are acting on the body are listed hereunder.
(a) Weight (mg),
(b) Normal reaction (N), l
Figure 5.12
(c) Friction (f), and
(d) The applied force (F)
According to the principle of work-energy theorem ds B
Wnet= ∆ KE or Wmg+WN+Wf+WF = 0  … (i)
F
0, because K i= 0= K f ∴ Wmg = − mgh; WN = 0
Here, ∆KE = A
f dl
(This is because the normal reaction is perpendicular to displacement at all the points.) Figure 5.13

Wf can be calculated as f =
µ mgcos θ

∴ ( dWAB )f = − f ds = − ( µ mg cos θ ) ds = − µ mg(dl) (as ds cos θ = dl)


∴ f = −µ mg ∑ dl = − µ mgl
Substituting these values in Eq. (i), we obtain the expression W
=F mgh + µ mgl .

Note: Here again, if we desire to solve this problem


 without using the concept of work-energy theorem, then  we
will first evaluate magnitude of applied force F at different locations following which we will then integrate = F.dr ( )
with proper limits.
P hysi cs | 5.11

10. KINETIC ENERGY


Now, let us attempt to develop a relationship between the work F P2
done and the change in speed of a particle. Based on the Fig. 5.14
provided, we observe that the particle moves from point P1 to P2

under the action of a net force F
  
P2  ∧ ∧ ∧ ∧ ∧ ∧ dr
W = ∫ F. dr ; F =Fx i + Fy j + Fz k ; dr = dx i + d y j + dzk
P1

P2
W= ∫ (Fx dx + FY dy + Fz dz ) P1 Figure 5.14
P2

It is very clear for us now that a particle moves along a curved path from point P1 to P2, only when acted upon by
P2 P2
mdν x dVx
a force F that varies in both magnitude and direction.=
Fx ma
= x
; ∫ Fx dx = ∫ m dx
dt P1 P1
dt
Treating now ν x as a function of position, we obtain:
P P P P2 ν
dν x dν  dx  dν dν 2 2 dν x 2 dν 1 2 x2
dt
= x   = x . ν x =ν x x ; ∴ ∫ Fx dx =
dx  dt  dx dx ∫ m
dt
dx =∫ m ν x x dx=
dx ∫ m ν x dν x= 2
1
2
(
mν x ∫= m ν2x2 − ν2x1 )
P P 1 1P 1 P1 ν x1

ν x1 = velocity in x-direction at P1; ν x2 = velocity in x-direction at P2.

We now apply the same principle for terms in y and z.

W
=
1  2
2  ( ) 
1
2
( )
M ν x2 + ν y22 + ν z22 − ν x12 + ν y12 + ν z12 = M ν22 − ν12 ; W=
1
2
1
mν22 − mν12
2

1
K
Define: = mν2 ≡ Kinetic energy of particle
2
KE: Potential of a particle to do work by virtue of its velocity.
We know that the work done on the particle by the net force equals the change in KE of the particle.
W
= ⇒ W = ∆K Work–Energy Theorem.
K 2 − K1 or
  1
For a particle P = Mv (linear momentum); ∴ K = P2
2m
Regarding KE, the following two points are very significant.
(a) Since, both m and v2 are always positive, KE is always positive and hence does not depend on the directional
parameter of motion of the body.
(b) KE depends on the frame of reference. For example, the KE of a person of mass m in a train moving with speed
1
v is zero in the frame of train, whereas in the frame of earth the KE is mv 2 for the same person.
2

PLANCESS CONCEPTS

Energy can never be negative.


No! Only kinetic energy can’t be negative. If anyone generally speaks about energy, it means the sum
of potential and kinetic energies. However, we can always choose such a reference in which this sum is
negative. Hence, total energy can be negative.
Anurag Saraf (JEE 2011, AIR 226)
5 . 1 2 | Work , Energy and Power

Illustration 11: A uniform chain of length ℓ and mass m overhangs a smooth table with its two-third parts lying on
the table. Find the kinetic energy of the chain as it completely slips off the table. (JEE MAIN)

Sol: The initial kinetic energy of the chain is zero. When chain start slipping off table the loss in its potential energy
is equal to the gain in its kinetic energy.
Let us take the potential energy at the table as zero. Now, consider a part dx of the chain at a depth x below the
m
(
surface of the table. The mass of this part is dm = dx and hence its potential energy is − m /  dx gx.

)
 /3
m
The potential energy of the one-third of the chain that overhangs is given by U=
1 ∫ −  gx dx
0
 /3
 m  x2   1
−  g  
= − mg
=
   2  18
   0

However, this is also the potential energy of the full chain in the initial position; this is because the part lying on the
table has zero potential energy. Now, we can calculate the potential energy of the chain when it completely slips
off the table as

dx

Figure 5.15


m 1
− mg The loss in potential energy is =  − 1 mg  −  − 1 mg  = 4 mg .
∫ −  gx dx =
U2 =
0
2  18   2  9

Basically, this should be equal to the gain in the KE in this case. However, the initial KE is zero. Hence, the KE of the
4
chain as it completely slips off the table is mg .
9

Illustration 12: A block of mass m is pushed against a spring of spring constant k fixed at one end to a wall. The
block can slide on a frictionless table as shown in the Fig. 5.16. The natural length of the spring is taken as L 0 and
it is compressed to half its natural length when the block is released. Now, based on the above find the velocity of
the block as a function of its distance x from the wall. (JEE ADVANCED)

Sol: The block will move under action of restoring force of spring when spring is released. The block will have
constant kinetic energy when it looses contact with the spring. In this process the energy of system will be conserved
as there are no external forces acting on the system. (Spring + block system)

m
k
v

Lo/2
x

Figure 5.16

When the block is released, naturally the spring pushes it toward right. The velocity of the block keep on inreasing
till the block loses contact with the spring and thereafter moves with constant velocity.
P hysi cs | 5.13

Initially, the compression of the spring is L0 / 2 . But when the distance of the block from the wall becomes x, where
( )
x < L 0 , the compression is L 0 − x . Applying the principle of conservation of energy

2 1/2
k  L0 2
2
1  L0  1 1
k (L0 − x ) + mv 2 . Solving this, v = − (L 0 − x ) 
2
k = 
2  2  2 2 m 4
 

k L0
Thus, when the spring acquires its natural length, then x = L 0 and v = . Thereafter, the velocity of the block
remains constant. m 2

11. MOTION IN A VERTICAL CIRCLE


Let us consider a particle of mass m attached to one end of a string and C
rotated in a vertical circle of radius r with centre O. The speed of the
particle will decrease as the particle travels from the lowest point to the
highest point but increases in the reverse direction due to acceleration h
O r
due to gravity. D E
T
Thus, if the particle is moving with velocity v at any instant at A, (where the
string is subtending an angle θ with the vertical), then the forces acting F A
on the particle are tension T in the string directed toward AO and weight
mg acting downward. B mg cos
mg
Further, the net force T–mg cos θ is directed toward the cenetr and hence mg sin
provides the centripetal force
Figure 5.17: Motion in vertical circle
mv 2  v2 
T − mg=
cos θ = ; T m  gcos θ + 
r  r 

If v0 is the speed of the particle at the highest point, then the velocity increases as the particle falls through any
height h. However, if it falls from C to A, then the vertical distance h is given by
h CF
= = CO + OF
= CO + OA cos= = r (1 + cos θ )
θ r + r cos θ;h

v 2 =v 20 + 2gh =v 02 + 2gr (1 + cos θ ) (Because there is no actual work done due to the influence of tension)

(i) At the highest point C, θ =1800


 v2   v2   ... (i)
Tension at C = m  0 + gcos(180) =
Tc = m  0 − g
r r
   

The particle will now fall because the string will slacken if Tc is negative. Therefore, the minimum velocity at the
 v2 
highest point is corresponding to the situation where Tc is just zero, i.e., when m  0 −=g 0, or=v0 rg
 r 
 v2 
(ii) At the lowest point B, θ =0 , tension TB is given by
= TB m  B + g
 r 

where vB is velocity at B. vB2 =v 02 + 4rg =rg + 4rg = (


5rg; using v 2 = )
u2 + 2gh ; vB = 5rg
 ... (ii)

 5rg 
Minimum tension at B when the particle completes the circle is given by T=
B m + g=
 6mg
 r 
0 mvE2
At the point E, when
= θ 90
= , TE
r
Where velocity at E is given by VE =Vc2 + 2rg =rg + 2rg =3rg; VE = 3rg
5 . 1 4 | Work , Energy and Power

 3rg  N
Tension at E corresponding to speed  =
VE is TE m=
  3mg 
 r 

(iii) In another case the particle of mass m is not tied to the string but is moving along a
r
circular track of radius r and has normal reaction N. However, it is moving with a velocity v
mv 2
and its radius vector is subtending an angle θ with the vertical, then mg cos θ − N = .
2 r mg
mv
At the highest point, mg − N = ; when  ... (iii) Figure 5.18
r

N 0,=
= V rg Therefore, V = rg is the minimum speed with which the particle can move at the highest point
without losing contact.
P
Condition of Looping the Loop (u ≥ 5gR )  Vmin= gR T=0

The particle will complete the circle only if the string does not slack even
at the highest point (θ=π). Thus, tension in the string should be obviously
O
greater than or equal to zero (T ≥ 0) at θ=π. In the critical case, however, by
substituting T = 0 and θ=π in Eq. (iii), we obtain R
2
mvmin 2
mg= or vmin =gR or vmin = gR (at the highest point) u Umin= 5gR
R A T= 6mg

Further, by substituting θ = π in Eq. (i), h = 2R Figure 5.19

2 2 2
Therefore, from Eq. (ii) u=
min vmin + 2gh or umin
= gR + 2g(2R) or umin = 5gR

Thus, if u ≥ 5gR , then the particle will complete the circle.

At u = 5gR , the velocity at the highest point is v = gR and the tension in the string is zero.

By substituting θ = 0° and v = 5gR in Eq. (iii), we get T = 6mg or in the critical condition tension in the string at

the lowest position is 6mg as shown in the Fig. 5.19. If u < 5gR , then the following two cases are possible.

Condition of Leaving the Circle ( 2gR < u < 5gR )

If u < 5gR , then the tension in the string will be zero before reaching the highest point. From Eq. (iii), tension in
2
−v2 2gh − u
the string is zero (T=0) where, cos θ = or cos θ =
Rg Rg
2gh − u2 h u2 + Rg
Now, by substituting, this value of cosθ in Eq. (i), we obtain = 1− = or h = h1 (say) … (iv)
Rg R 3g

v
T=0
P
v0

O  hR

u
A
Figure 5.20

Or, in other words, we can say that at height h1 tension in the string becomes zero. Further, if u < 5gR , then the
P hysi cs | 5.15

u2
0 u2 − 2gh or=
velocity of the particle becomes zero when = h = h2 (say)  ... (v)
2g
i.e., at height h2 velocity of the particle becomes zero. Now, the particle will move out from the circle if tension

alone in the string becomes zero but not the velocity or T=0 but v ≠ 0 . This is possible only when h1 < h2 or
u2 + Rg u2
< or 2u2 + 2Rg < 3u2 or u2 > 2Rg or u > 2Rg .
3g 2g
Therefore, if 2gR < u < 5gR , the particle moves out from the circle.

2
From Eq.(iv), we observe that h >R if u > 2Rg . Thus, the particle, will move out of the circle when h>R or
90° < θ < 180° . This situation is shown in the Fig. 4.75.

2gR < u < 5gR or 90° < θ < 180°

Note, however, that after leaving the circle, the particle will follow a parabolic path.

Condition of Oscillation (0 < u < 2gR )


The particle will oscillate, however, only if velocity of the particle becomes zero but not tension in the string. Or, in
other words, v = 0, but T ≠ 0 . This is possible only when h2 < h1
.

v=0
N0
u hR

Figure 5.21

u2 u2 + Rg
Or < or 3u2 < 2u2 + 2Rg or u2 < 2Rg or u < 2Rg
2g 3g

Moreover, if h1 = h2 , u = 2Rg then both tension and velocity becomes zero simultaneously.
Further, from Eq (iv), we observe that h ≤ R if u ≤ 2Rg . Thus, for 0 < u ≤ 2gR , the particle oscillates in the lower
half of the circle (0° < θ ≤ 90°) . This situation is shown in the Fig. 5.21. (0 < u < 2gR ) or (0° < θ ≤ 90°)

Note: The above three conditions have been derived for a particle that is moving only in a vertical circle and
attached to a string. The same conditions apply, however, if a particle moves inside a smooth spherical shell also of
radius R. The only difference here is that the tension is replaced by the normal reaction N.

v= gR, N=0
Condition of Looping the Loop is (u ≥ 5gR ) v

u= 5gR, N=6mg

Figure 5.22
5 . 1 6 | Work , Energy and Power

N=0
v v0
Condition of Leaving the Circle ( 2gR < u < 5gR )

hR

Figure 5.23

Condition of Oscillation (0 < u < 2gR )


O v=0
T0
R P hR
u
A

Figure 5.24

Illustration 31: A heavy particle hanging from a fixed point by a light inextensible string of length l is projected

horizontally with speed gl . Now, find the speed of the particle and the inclination of the string to the vertical at
the instant of the motion when the tension in the string is equal to the weight of the particle.  (JEE ADVANCED)

Sol: Loss in the kinetic energy of the particle is equal to the gain in the potential energy. Apply Newton’s second
law along the direction of the string.

T
 B

h mg cos 
mg sin 
A u= gl

Figure 5.25

Let T = mg at angle θ as shown in the Fig. 5.25.


H = l (1–cos θ )  … (i)
1
Applying the principle of conservation of mechanical energy between points A and B, we obtain m(u2 − v 2 ) =
mgh
2
2
Here, u = gl  ... (ii)
and v = speed of particle in position B ∴ v2 =u2 − 2gh  … (iii)
2 2
mv mv
Further, T − mgcos θ = or mg − mgcos θ = (T = mg)
l l
Or v 2 = gl(1 − cos θ)  … (iv)
Now, by substituting the values of v 2 , u2 and h from Eqs. (iv), (ii) and (i) in Eq. (iii), we obtain
2 2
gl(1 − cos θ) =gl − 2gl(1 − cos θ) or cos θ = or θ =cos−1  
3 3

2 gl
Further, by substituting cos θ = in Eq. (iv), we obtain v =
3 3
P hysi cs | 5.17

PLANCESS CONCEPTS

If a particle of mass m is connected to a light rod and whirled in a vertical circle of radius R, then

to complete the circle, the minimum velocity of the particle at the bottommost point is not 5gR .
Because, in this case, velocity of the particle at the topmost point can be zero also. Using conservation
of mechanical energy between points A and B as shown in Fig. 5.26(a) we get

B
v=0

O h=2R
R R

u
A
u  2 gR u  2 gR
dR

(a) (b)
Figure 5.26

1 1
m(u2 − v 2 ) =
mgh or mu2 = mg(2R) (as v = 0) ∴ u = 2 gR
2 2

Therefore, the minimum value of u in the case is 2 gR .


Same is the case when a particle is compelled to move inside a smooth vertical tube as shown in
Fig 5.26(b).
Anurag Saraf (JEE 2011, AIR 226)

12. A BODY MOVING INSIDE A HOLLOW TUBE

Our discussion above holds good in this case too, but instead of tension in the string

we have the normal reaction of the surface. If we take N is the normal reaction at the
O
mv12  v2 
lowest point, then N − mg
= = m  1 + g  However, at the highest point of the
; N
r  r 
 
mv 2 V1
circle, N + mg =2
r
mg
 v2 
N =m  2 − g  ; N ≥0 ⇒ Implies the condition V1 ≥ 5rg
 r  Figure 5.27: Block moving
 
inside hollow sphere
In the same way as shown above, all the other equations similarly can be obtained by
just replacing tension T by reaction N.

13. BODY MOVING ON A SPHERICAL SURFACE


Consider that the small body of mass m is placed on top of a smooth sphere whose radius is r.
Now, if the body slides down the surface, at what point does it fly off the surface?
5 . 1 8 | Work , Energy and Power

Consider the point C where the mass is, at a certain instant. Now, the acting
forces are the normal reaction R and the weight mg. Further, the radial C
component of the weight is mg cosθ acting toward the center. The
mv 2  h
centripetal force in this case is taken as mg cos θ − R =
r
where v is the velocity of the body at O. ground
 v2  Figure 5.28: Motion of body on
=R m  g cos θ −   … (i)
 r  spherical surface

Now, it is clear that the body flies off the surface at the point where R becomes zero.
mv 2
i.e..,g cos φ − R =  … (ii)
r
To find v, we apply the principle of conservation of energy
1
i.e., mv 2 = mg(BN)= mg (OB − ON)= mgr (1 − cos φ)
2
v2
v 2= 2rg (1 − cos φ); 2(1 − cos φ=
)  … (iii)
rg
From equations (ii) and (iii), we obtain
cos φ= 2 − 2cos φ; 3cos φ= 2
2 2
=cos φ ;= φ cos−1    … (iv)
3 3
This exactly denotes the angle at which the body goes off the surface. The height from the ground of that point is
 2 5
= AN = r(1 + cos φ) = r  1 +  = r
 3 3

Illustration 32: A point mass m starts from rest and slides down the surface of a frinctionless solid sphere of radius
R as shown in the Fig. 5.29 provided. At what angle will this body break off the surface of the sphere? Also, find the
velocity with which it will break off.  (JEE MAIN)
Sol: As the block slides down, the loss in potential energy is equal to gain in kinetic A
energy and at time of break off, the normal reaction from the sphere on block is N
m
zero.
R B
Applying princliple of conservation of energy (COE), at the points A and B
O v
1
mgR (1 − cos θ) = mv 2  … (i)
2 R m
2
Force equation in this equation is mg cos θ − N =mv / R  … (ii)
N = 0 for break off. Figure 5.29

∴v
= gR cos θ  … (iii)

Replacing this value in (i)


2
We get cos θ 2=
/3 Putting this in (iii) we get v gR.
3

Illustration 33: A heavy particle is suspended by a string of length  . The horizontal velocity of the particle is v 0 .
However, the string becomes slack at some angle and the particle proceeds on a parabolic path. Find the value of
v 0 if the particle passes through the point of suspension.  (JEE ADVANCED)
P hysi cs | 5.19

Sol: While particle moves in vertical circle, the tension in the string provides the necessary centripetal force. The
loss in kinetic energy is equal to the gain in potential energy. At point the string become P
slack the tension in the string is zero.

Let us suppose the string becomes slack when the particle reaches the point P. We now
assume that the string OP makes an angle θ with the upward vertical. Further, the only force O
acting on the particle at the point P is its weight mg. Further, the radial component of the
force is mg cos θ . Now, as the particle moves along the circle upto P,
 v2 
mg = cos θ m   ⇒ = v 2 g  cos θ  … (i)
  
 
Figure 5.30
where v is its speed at the point P. Now, applying the principle of conservation of energy
1 1
2
mv 20=
2
mv 2 + mg (1 + cos θ ) or v 2 = v 20 − 2g 1 + cos θ  ( ) … (ii)

From ( i) and ( ii) ,=


v 20 2g (1 + cos=
θ ) g  cos θ or v 20 = g ( 2 + 3cos θ )  … (iii)

From hereon, the particle follows a parabolic path due to acceleration due to gravity. Then as it passes through the
point of suspension O, the equations for horizontal and vertical motion give
2
  sin θ    sin θ 
 sin
= θ ( v cos θ ) t and −  cos
= θ ( v sin θ ) t − 12 gt2 ⇒ − cos
= θ ( v sin θ )  v cos θ  − 12 g  v cos θ 
   
2
1 2   sin θ    sin θ 
n θ) t −
2
gt ⇒ − cos
= θ ( v sin θ )  v cos θ  − 12 g  v cos θ 
   

1  sin2 θ 1 g sin2 θ
or, − cos2 θ= sin2 θ − g or, − cos2 θ= 1 − cos2 θ − From(i)
2 v 2cos θ 2 g cos2 θ 

1  sin2 θ 1 g sin2 θ
g or, − cos2 θ= 1 − cos2 θ − From(i)
2 v 2cos θ 2 g cos2 θ 

1 1/2
or,=1
2
tan2 θ or, = , v 0  g 2 + √ 3 
tan θ √ 2 From (iii) =
  ( )
1/2
r, =
tan θ √ 2 From (iii) =
 (
, v 0  g 2 + √ 3 
 )

14. VARIOUS FORMS OF ENERGY: THE LAW OF CONSERVATION OF ENERGY


Conservation of Energy
We observe that in many processes the sum of both the kinetic and potential energies does not remain a constant.
This may be due to the influence of dissipative forces such as friction.
(a) The more general form of law of conservation of energy was established by taking into account other forms
of energy such as thermal, electrical, chemical, nuclear, etc.
(b) The charges in all forms of energy is given by: ∆ KE + ∆U + ∆ (all other forms of energy) ≡ 0
This is what we mean by the law of conservation of energy and it is one of the most important principles of
physics.
“The total energy is neither increased nor decreased in any process. Energy can be transformed from one form
to another, and transferred from one body to another, but the total amount remains constant.”
5 . 2 0 | Work , Energy and Power

PROBLEM-SOLVING TACTICS

(a) One should always isolate the known and unknown quantities and write equations and solve them.
(b) The next step would be to find out a way from unknown to known quantities and write equations and solve
them.
(c) One should always be very careful in doing so to avoid silly mistakes such as unit change of parameter.
(d) Energy is scalar in nature. However, get a clear idea of what is being gained or lost by which entity.
(e) Physical visualization of any problem will always help in increaseaing confidence in solving equations
pertaining to the same.
(f) Further, problems involving integration would be easy to understand if you go event by event and then solve.
(g) Special cases and boundary conditions of circular motion are definitely recommended to be mastered because
many problems break down to these special cases just after few manipulations.

FORMULAE SHEET
S. NO. DESCRIPTION FORMULA

1 1  
1 Kinetic energy of the particle K(v)
= = mv 2 mv. v
2 2
 
2 Work done by force F
W = F.r (here r is total displacement)

 
3 Work done by variable force w = ∫ F.dr

dW  dr  
4 Power generated by force F acting on body P
= = F.= F. v
dt dt

5 Increase in Kinetic Energy = Decrease in Potential Energy KE = – ∆U

1 2
6 Energy conservation principle K + ∆U 0;=
∆= mv mgh
= or, v 2gh
2

7 For a Spring work done W W=


x2 1 2
(2
∫x1 −kx dx =2 k x1 − x2 )
8 Work-Energy principle Wnet =
∆KE =
K f − Ki

  
For F = F1 + F2 + .......
9 Work done by variable forces in short range    
( )
W= ∫ F.dr= ∫ F1 + F2 + ..... . dr

r  
10 For conservative forces, change in potential energy Uf − Ui =− ∫ f F.dr
ri

1 2
11 Elastic Potential Energy U= kx
2
P hysi cs | 5.21

Solved Examples

JEE Main/Boards N

Example 1: An object of mass 5 kg falls from rest


through a vertical distance of 20 m and attains a velocity
of 10 m/s. How much work is done by the resistance of kx

(
the air on the object? g = 10m / s2 )
N
Sol: According to work energy theorem, the total work
done by force of gravity and force of air resistance on mg
object is equal to the change in kinetic energy. ⇒ work done by net force for the displacement
x x
Work done by all forces = Change in KE
∫ F dx ; ⇒
⇒ W= ∆KE =− ∫ ( µ mg + kx ) dx
1
Wair + W gravity =
∆ K .E. ; Wair + mgh =mv 2 0 0
2 
 1 2 kx2 
1 2 1 ⇒  0 − mv  =−  µ mgx + 
Wair
= mv
= − mgh ; x5 x10 x10 − 5 x10 x 20  2 0  2 
2 2 

Wair = − 750 J Example 3: Two smooth balls of mass m1 and m2


connected by a light inextensible string are at the
Example 2: A block is projected horizontally on a rough opposite points of horizontal diameter of a smooth
horizontal floor. The coefficient of friction between the semicylindrical surface of radius R. if m1 is released, find
block and the floor is µ . The block strikes a light spring its speed at any angular distance θ moved by m2.
of stiffness k with a velocity v 0 . Find the maximum
compression of the spring. Sol: Loss in potential energy of system comprising
masses m1 and m2, is equal to gain in kinetic energy of
the system.
v0 Let the ball m2 moves through an angle θ , the mass m
will fall through a distance h1 = R θ .
k
 m The ball m2 rises through a height h2 as,
h2 = Rsinθ
Sol: At the instant of maximum compression the block The change in gravitational potential energy of m1 is
will come to rest momentarily. By applying work energy
theorem the sum of work done by the force of friction, ∆PE1 =
−m1gh1 =
m1gRθ
and spring force will be equal to change in kinetic
(Since m1 loses its potential energy as it falls down). The
energy. Work done by normal reaction and gravitational
change in gravitional potential energy of m2 is
force will be zero.
∆PE2 =
−m2gh2 =
m2gR sin θ

(Since m2 gains potential energy as it rises up)


v
x m

R
Since the block slides and the spring is compressed h2
m1  m2
through a distance x the net retarding force acting on it
= F = − (kx + µN) = − ( µ mg + kx )
v h1
m1
5 . 2 2 | Work , Energy and Power

⇒ The total change in gravitational potential energy Example 5: A 0.5 kg block slides from the point A on
a horizontal track with an initial speed of 3 m/s toward
= ∆PE = −m1gRθ + m2gR sin θ a weightless horizontal spring of length 1 m and force
constant 2 N/m. The part AB of the track is frictionless
= gR (m2 sin θ − m1 θ )  … (i)
and the part BC has the coefficients of static and kinetic
friction as 0.22 and 0.2, respectively. If the distances
=
1 1
∆KE =m1 v 2 + m2 v 2 =
(m1 + m2 ) v 2 … (ii) AB and BD are 2 m and 2.14 m, respectively find the
2 2 2  total distance through which the block moves before it
where v = speed of m1 and m2 at the position as comes to rest completely (Take g = 10m / s2 ) .
shown in the Fig.5.26 provided. From the principle of
conservation of energy, we obtain
= ∆KE + ∆PE = 0  … (iii)
A B D C
Using (i)–(iii), we obtain Sol: The sum of work done by force of friction and
spring force is equal to change in kinetic energy of the
1
(m + m2 ) v 2 − gR (m1θ − m2 sin θ ) = 0
2 1
block.

2gR (m1 θ − m2 sin θ ) Suppose the block comes to rest at the point E, i.e.,
⇒v= let DE = x. The kinetic energy of the block is spent
(m1 + m2 ) in overcoming friction and compressing the spring
through a distance DE = x.
Example 4: A locomotive of mass m starts moving so Kinetic energy of the block
that its velocity varies according to the law v = α s ,
1 12 2 1 1
where α is a constant and s is the distance = covered.
== mv =
mv
; ; = x 0.5
= 0.52 x32.25
xx3 2
2.25
J J … (i)
Find the total work done by all the forces acting on the 2 2 2 2
locomotive during the first second after the beginning As the part AB of the track is frictionless, work done in
of motion. moving from A to B is zero.
Let normal reaction of the block = mg.
Sol: Velocity is given as the function of distance covered
so we can find the acceleration and by second law of C oefficient of friction = µ
motion we can find the force. As force comes out to be
Force due to friction along the track
constant the work done by force is product of force and
displacement. BC = µ mg 0.2
= = x 0.5 x10 1N
Given v = α s Distance through which the block moves against the
frictional force = 2.14 + x m
Differentiating w.r.t. ‘t’, we get
dv 1 −1/2 ds α α α2
= αs = v= xα =
s
dt 2 dt 2 s 2 s 2

α2 A B D C
∴ Acceleration a = 2 2.14 xE
2
m m m
Now, force acting on the locomotive is
Work done by block against friction before it comes to
α2 rest
F ma
= = m ; Here, u = 0
2 =µ mg (2.14 + x); =(2.14 + x) J  … (ii)
1
s ut + at2 , we have
Now, using = Let the spring constant = k
2
∴ Work done by the block in compressing the spring
1 α2 2 α2 t2 through distance X
s=
0+ t =
2 2 4
1 2 1 2
Thus total work done on locomotive is when t=1 s is= 2 kx ; = 2= 2x x2 J  … (iii)

mα2 α2 t2 mα 4 Adding (ii) and (iii) and equating it to (i), we get


W= Fs = × = J
2 4 8
2.14 + x + x2 2.25;
= or= x2 + x − 0.11 0
or 100x2 + 100x − 11 = 0
or (10x + 11)(10x − 1) =
0
11 1 1
P hysi cs | 5.23

2.14 + x + x2 2.25;
= x2 + x − 0.11 0
or=
or 100x2 + 100x − 11 = 0 1 2 2h 2
h= gt =
; t = seconds
or (10x + 11)(10x − 1) =
0 2 g 5
11 1 1
∴ x=− or x = ; Since x≠− . Therefore, the horizontal distance traveled by the ball
10 10 10
1 in this time is
∴ x= = 0.1m
10 5 2
d v=
= .t x = 1m
2 5
Restoring force of the spring
= x 0.1 0.2N 
= kx 2= … (iv) Example 7: A block of mass 2 kg is pulled up on a
Static frictional force of the block smooth incline of angle 30o with horizontal. If the block
moves with an acceleration of 1m/s2, find the power
µstatic
= x10 1.1N 
mg 0.22 x 0.5= … (v) delivered by the pulling force at a time t = 4 s after
motion starts. What is the average power delivered
From (iv) and (v) it is clear that the static frictional
during these four seconds after the motion starts?
force is greater than the restoring force of the spring.
Therefore, the block will not move in the backward
N F
direction. Hence the total distance through which the
block moves before it comes to rest completely is
m
2.00 + 2.14 + 0.10 = 4.24 m

Example 6: In a spring gun having spring constant 100 o


30 mg
N/m, a small ball of mass 0.1 kg is put in its barrel by
compressing the spring through 0.05 m as shown in the Sol: Apply newton’s second law of motion along
Fig. 5.29. Find, the direction of incline to find the applied force. As
acceleration is constant and initial velocity is zero, the
final velocity at time t is v=at along incline and power
v
is P = F • v .
The forces acting on the block are shown in the Fig.
h=2m 5.30 provided.
Resolving forces parallel to incline
F − mg sin θ =ma ; =
⇒ F mgsin θ + ma
d 0
= 2 x 9.8x sin30 = + 2x1 11.8N
(a) The velocity of the ball when the spring is released. The velocity after t = 4 s is v=u+at
(b) Where a box should be placed on the ground so 0 1x4 =
=+ 4m / s
that ball falls in it, if the ball leaves the gun horizontally
2 Power delivered by force at t = 4 s
at a height of 2 m above the ground? (g = 10m / s ) .
= Force x Velocity
= 11.8 = x 4 47.2W
Sol: As the spring expands the potential energy stored
in the spring is converted in the kinetic energy of ball. The displacement during t = 4 s is given by the formula
The horizontal distance moved by the ball will depend v2=u2+2as; v2=0+2×1×S
on time taken by ball to fall vertical height h
∴S= 8m
(a) When the spring is released its elastic potential
energy is converted into kinectic energy Work done in t = 4 s is W = Force × distance
= 11.8 = x 8 94.4 J
1 1 2 5

= mv 2 kx ; ⇒ v
= m/s
2 2 2 ∴ average power
∴ average power delivered
delivered
workdone 94.4
(b) As vertical component of velocity of the ball is zero,= = = 23.6W
the time taken by the ball to reach the ground is time 4
5 . 2 4 | Work , Energy and Power

Example 8: A chain of mass m = 0.80 kg and length l 4mg


= 1.5 m rests on a rough-surfaced table so that one of Assume that lo >> . Neglect any loss of energy.
k
its ends hangs over the edge. The chain starts sliding m
off the table all by itself provided the overhanging part
equals n = 1/3 of the chain length. What will be the
total work performed by the friction forces acting on h
the chain by the moment it slides completely off the
table?
K
l0
Sol: When chain starts sliding off the table, the friction
is limiting. So from this we can find the coefficient of m
friction. As the chain slides off, the force of friction
will be variable in nature as length of chain on table is Sol: When ball falls from height h, the loss in its
decreasing. So the work done by the force potential energy is equal to the gain in its kinetic
 of friction for
infinitesimal displacement ds is dW = F.ds energy. At the point of the maximum compression of
the spring the ball comes to rest momentarily. After this
Let µ be the coefficient of friction between chain and the ball will again start moving up till the spring is again
table. elongated to the point where the lower block looses
Weight of hanging part = µ contact with ground. For minimum value of h the ball
will again come to rest at this point. So the total loss in
(weight of horizontal part) nmg =
µ (l − n)mg
gravitational potential energy will be equal to gain in
n the elastic potential energy.
µ=
l−n
The minimum force needed to lift the lower block is
Let x be the length of the hanging part at some time equal to its weight. During upward motion the spring
instant. will get elongated. If elongation in the spring for just
Frictional force f (x) =N (normal reaction) lifting the block is x0 then.
mg
= kx0 mg; = ⇒ x0  …… (i)
µ (  − x ) mg k
− The work done by the frictional force if
 From COE
the hanging part increases to (x + dx) is
) mg (l0 + x0 ) + 12 kx20 ;
mg (l0 + h=
dW = f(x)dx
1
⇒ mgh = mgx0 + kx20
(  − x ) mg
1µ µmg  x2 
i
2
∫ dW =
W= −∫
 
;w =

 
 x − 
2 (mg)
2
  1 m2g2 3mg
⇒ mgh
= + ; ⇒
= h
k 2 k 2k



(
W = −µ mg  (1 − n) − 1 − n2
2
)
During downward motion, suppose maximum
Substituting the value of µ from (I), we get compression in the spring is x. From COE
n (1 − n) mg 
w= −
2
=
1 2 mgl mgl
× ×
3 3 2
=
9
mg (l0 + h
= ) mg (l0 − x ) + 12 kx2
1
⇒ mgh = − mgx + kx2
2
JEE Advanced/Boards 3mg 1
⇒ mg = = −mgx + kx2
2k 2
Example 1: In the Fig. 5.31 shown a massless spring of
⇒ 3(mg)2 = −2mg kx + k 2 x2
stiffiness k and natural length l0 is rigidly attached to a
block of mass m and is in vertical position. A wooden
ball of mass m is released from rest to fall under gravity. ⇒ k 2 x2 − 2mgkx − 3(mg)2 =0
Having fallen a height h the ball strikes the spring and
2mgk ± 4(mgk)2 + 12k 2 (mg)2
gets stuck up in the spring at the top. What should be ⇒x =
the minimum value of h so that the lower block will just 2k 2
loose contact with the ground later on? Find also the 2mgk ± 4mgk 3mg
correspoinding maximum compression in the = spring. = ⇒x
2k 2 k
P hysi cs | 5.25

Example 2: A smooth, light horizontal rod AB can depend on time taken by disc to fall vertical height 2R.
rotate about a vertical axis passing through its end A. At point C, for minimum velocity, normal force on the
The rod is fitted with a small sleeve of mass m attached block is zero.
to the end A by a weightless spring of length  0 and
Let the block escape the point at C with a velocity V
stiffness k. What work must be performed to slowly get
horizontally. Since it hits the initial spot A after falling
this system going and reach the angular velocity ω?
( ) (
thorugh a height 2R we can write 2R = 1 / 2 gt
2
)
Sol: When system starts moving about a point A, the where t = time of its fall
spring force provides the necessary centripetal force to ⇒t=2 R/g
the sleeve of mass m to move with angular speed ω .
The work done by external agent will be equal to the ∴ the distance AB =
2v R / g
kinetic energy of the spring and elastic potential energy ⇒ d=
2v R / g  … (i)
of the spring.
Work–energy theorem applied to the motion of the
body from A to B leads
l0
∆KE =
WF
A B
1 1
m ⇒ mv 20 − mv12 =
µ mgd
2 2
The mass m rotates in a circle of radius  , which is the
extended length of the spring. Centripetal force on ⇒ v=
0 v12 + 2 µgd  … (ii)
m=k ( − 0 ) =mω  2
Energy conservation between B and C yields
0 2

=or,  = where n
1−n k
V
W = Change in KE of m + energy stored in the spring
2R
2 2 2
1 2 21 1 ω 2l 1  l 
= 1 mω2ll02 ω+ k ( 1− 0 )l=0
2 Vl
m  0+ k  0 − l0 
= 2m 2 + k −l 
2 2 1 − n
2 1 − n 2 2  1 − n 0 (1 − n)
2

( ) d A
k20  mω2 
1 k=
2  mω2 W  1  + n2  1 2 1 2
= W 0
 + n2
 2 1 − n  k ⇒ mv − mv =
mg(2R)
( )
2
2 1 − n 2  k  2 0 2 1
( ) 
⇒ v1 = v 2 + 4gR … (iii)

Example 3: A small block is projected with a speed
When the disc escapes C, its minimum speed v can be
Vo on a horizontal track which turns into a semicircle
given as
(vertical) of radius R. Find the minimum value of v 0
so that the body will hit the point A after leaving the mv 2
mg (∴ the normal contact force =
= 0)
track at its highest point. The arrangement is shown in R
the figure, given that the straight part is rough and the
curved part is smooth. The coefficient of friction is µ . ⇒ v =gR  … (iv)

By using (iii) and (iv), we obtain


C
R v1 = 5gR  … (v)

 
2R
V0 Using (i) and (iv),=
we obtain d (=
gR ) 2 
R
 g



2R … (vi)

Smooth Putting the values of v1 and d in (ii), we obtain
Rough surfacre A

Sol: While block travels on the frictional surface AB, the v 0= 5gR + 2 µg(2R) ; ⇒ v 0= (5 + 4 µ)gR
work done by the frictional force is equal to the change
v 0= The5gR
in kinetic energy of the block. + 2 µg(2R)
horizontal ;
distance ⇒ v 0= (5 + 4 µ)gR
moved by the block after leaving track at point C, will
5 . 2 6 | Work , Energy and Power

Example 4: Two bodies A and B connected by a light 1 1 1


mA ( v A ) + mB ( vB ) + mC ( v C )
2 2 2
rigid bar of 10 m long move in two frictionless guides =
2 2 2
as shown in the Figure. If B starts from rest when it is 2 2 2
vertically below A, find the velocity of B when X = 6 m. 1  dy  1  dx  1  dx 
= x 200   + x 200   + x100  
2  dt  2 dt
  2  dt 
Sol: As the blocks A and C fall vertically downwards, 2 2
 dy   dx 
the loss in its potential energy is equal to gain in
= kinetic 100   + 150  
energy of blocks A, B and C.  dt   dt 
2 2
 x dx   dx 
= 100   + 150   from (ii)
 y dt   dt 
A
2 2
 6 dx   dx 
= 100   + 150  
l
y  8 dt   dt 
2
 9   dx  3300 2
=100 x + 150    = v
0
B  16   dt  16 B
x 3300 2
C ∴ v = 9800
16 B
Assume 98 x16 2
∴ vB
= = 7 x 4= 6.9ms−1
m
= m
= 200kg 33 33
A B
.
and mc = 100kg ∴ Veloctiy of B at the required moment is = 6.9ms-1
At the instant, when the bar is as shown in the Figure
Example 5: A particle is suspended by a string of
l2 ; ∴ 2 x dx + 2y dy =
x2 + y 2 = 0 … (i) length ‘l ’. It is projected with such a velocity v along
dt dt
the horizontal such that after the string becomes slack
dx dy it flies through its initial position. Find v.
∴ x −y
=  … (ii)
dt dt
dx dy
=where velocity
= of B and velocity of A
dt dt O

Applying the law of conservation of energy, loss of


potential energy of A, if it is going down when the rod
is vertical to the position as shown in the Fig.

A
Sol: As the string becomes slack, the tension in the
string becomes zero. Apply the Newton’s second law
10m
of motion along the direction of string at the instant of
slacking. The loss in kinetic energy is equal to gain in
potential energy as the particle moves in vertical plane.
0 B
x Let the velocity be v′ at B where the string become slack
C
and the string makes angle θ with horizontal by the law
of conservation of energy.
= mA g(10 =
− 8) 2 x 200 x 9.8
mg sin
C moves down 6 m since B moves 6 m along x-axis. mg cos
mg
Total loss of potential energy
= 200 x 9.8 x 2 − 100
= x 9.8 x 6 100
= x 9.8 x10 9800 J.
This must be equal to kinetic energy gained
Kinetic energy gained
A v
P hysi cs | 5.27

1 1 Sol: While moving down the incline plane car attains


2
mv= mv '2 + mg (1 + sin θ )  … (i)
2 2 constant speed of 64 km/hr and while moving up
the incline plane it attains the constant speed of 48
or v '2 = v 2 − 2g (1 + sin θ )  … (ii) km/hr.While moving down the plane force of gravity
is balanced by force of friction. While moving up on
By the dynamics of circular motion incline plane, the force developed by the engine of
mv '2 the car is balanced by the frictional force and force of
mg sinθ = ; ⇒ v'2 = g sin θ  … (iii) gravity. The power developed by the engine is given by
  
From equations (ii) and (iii), we get PP == FF••. VV
When the motor car is moving down the plane there
∴ g sin θ= v 2 − 2g (1 + sin θ )  … (iv) is force Mg sin α down the plane. This is opposed by
the resistance, which is proportional to square of the
At B the particle becomes a projectile of velocity v′ at velocity. That is Mgsin α ∝ V 2
90 − θ with the horizontal.
1
Mg x = kV 2 where k is a cons tant.
Here ux = v' sin θ &uy = v'cos θ 20
2
ax = 0 & ay = −g ∴
1000 x g
=

k  64x
5 

20  18 
∴  cos θ − v 'sin θ t  … (v)
2
 cos θ 1000 x g  18 
∴t
= & −  (1 + sin
= θ) v ' cos θ k= x   … (i)
v 'sin θ 20  64 x5 
lcos θ 1 I2 cos2 θ When the engine is on, let the tractive force (force
− g ; ⇒ 2sin3 θ + 3sin2 θ − 1 = 0 exerted by engine) be F. This is used to overcome the
v 'sin θ 2 v '2 sin2 θ
force due to incline and the resistance offered.
1
∴ sin θ = is the acceptable solution
2 Mg
∴ F k ( 48 x5 / 18 ) +
2
= ;
1 7g 7g 20
∴ v 2= 2g + 3g x = ⇒ v= 1000 x g
F k ( 48 x5 / 18 )
2
2 2 2 = +
20
[from equation (iv)]
2 2
1000 x g  18 2  5 2 1000 x g
= F 1000 x g x  18   x  48 x5   +1000 x g
Example 6: A motorcar of = F 20 x  64 x5
 x5   x  48 x 18 + 20
F 20  64   18   20
mass 1000kg attains a speed 1000 x 9.8  9  50 x 9.8 x 2.5
of 64 km/hr when running = 2 1000 x 9.8 = 29 −1 50 x 9.8
= x 2.5 765.6N
1000 x
m = g  18   20 5 = 
  16− 1   x g =
1000 16 765.6N
=
down an inclination of 1 in F x  x
  20 48 x +
 16  16
20  64 x5   18  20
2
kv
20 with the engine shut off.
It can attain a speed of 48 = 1000 xmg 9.8  9  50 x 9.8 x 2.5
mgsing
=  − 1 = 765.6N
km/hr up the same incline 20  16  16
when the engine is switched on. Assuming that the Power developed = Force x Velocity
resistance varies as the square of the velocity, find the
= 765.6 x 48= x 5 / 18 10208 = W 10.2 W
power developed by engine.

JEE Main/Boards

Exercise 1 Q.2 Two bodies of unequal masses have same linear


momentum. Which one has greater K.E.?
Q.1 What is meant by zero work? State the conditions
under which a force does no work. Give any one Q.3 Two bodies of unequal masses have same K.E.
example. Which one has greater linear momentum?
5 . 2 8 | Work , Energy and Power

Q.4 How do potential energy and K.E. of a spring vary time t for a body of mass 2kg moving under the action
with displacement? Is this variation different from of a force is given by x = t3 / 3 , where x is the metre
variation in potential energy and K.E. of a body in free and t is in second. Calculate work done by the body in
fall? first 2 seconds.

Q.5 Explain what is meant by work. Obtain an expression Q.19 A woman pushes a trunk on a railway platform
for work done by a constant force. which has a rough surface. She applies a force of
100N over a distance of 10m. Thereafter, she gets
Q.6 Discuss the absolute and gravitational units of work progressively tired and her applied force reduces linearly
on m.k.s. and c.g.s systems. with distance to 50N. The total distance through which
trunk has been moved is 20m. Plot the force applied
by the woman and frictional force which is 50N against
Q.7 What is meant by positive work, negative work and the distance. Calculate the work done by the two forces
zero work? Illustrate your answer with two example of over 20m.
each type.

Q.20 A body of mass 50kg has a momentum of


Q.8 Obtain graphically and mathemativally work done 1000kg ms-1. Calculate its K.E.
by a variable force.

Q.21 A bullet of mass 50g moving with a velocity of


Q.9 What are conservative and non-conservative 400ms-1 strikes a wall and goes out from the other
forces, explain with examples. Mention some of their side with a velocity of 100ms−1 . Calculate work done
properties. in passing through the wall.

Q.10 What is meant by power and energy? Give their Q.22 A body dropped from a height H reaches the
units.
ground with a speed of 1.2 gH . Calculate the work
done by air-friction.
Q.11 Explain the meaning of K.E. with examples. Obtain
an expression for K.E. of a body moving uniformly?
Q.23 A bullet weighting 10g is fired with a velocity of
800ms-1. After passing thorugh a mud wall 1m thick,
Q.12 State and explain work energy principle. its velocity decreases to 100 m/s. Find the average
resistance offered by the mud wall.
Q.13 What do you mean by potential energy? Give any
two examples of potential energy other than that of the Q.24 A particle originally at rest at the highest point of
gravitational potential energy. a smooth vertical circle of radius R, is slightly displaced.
Find the vertical distance below the highest point where
Q.14 Obtain an expression for gravitational potential the particle will leave the circle.
energy of a body.

Q.15 Explain what is meant by potential energy of a


spring? Obtain an expression for it and discuss the
nature of its variation.

Q.16 Mention some of the different forms of energy


and discuss them briefly.
Exercise 2
Q.17 A particle moves along the x-axis from x=0 to x=5m Single Correct Choice Type
under the influence of a force given by f =7 − 2x + 3x2.
Calculate the work done.
  ∧ ∧
Q.1 A forceF F = −k  y i + x J  , where k is a positive
 
Q.18 The relation between the displacement x and the  
constant, acts on a particle moving in the xy plane.
P hysi cs | 5.29

Starting from the origin, the particle is taken along the vertically down over the edge of the table. If g is
positive x-axis to the point (a, 0), and the parallel to the acceleration due to gravity, then the work required to
y-axis to the point (a, a). The total work done by the pull the hanging part onto the table is:
force on the particle is:
MgL 4MgL MgL
(A) 2ka2 (B) 2ka2 (C) −ka2 (D) ka2 (A) MgL (B) (C) (D)
3 9 18

Q.2 Supposing that the earth of mass m moves around Q.8 A body is moved along a straight line by a machine
the sun in a circular orbit of radius ‘R’, the work done in delivering constant power. The distance moved by the
half revolution is: body in time t is proportional to:
mv 2 mv 2
(A) x π R (B) x 2R (A) t1/2 (B) t3/ 4 (C) t3/2 (D) t2
R R
(C) Zero (D) None of these
Q.9 An alpha particle of energy 4 MeV is scattered
through 180o by a fixed uranium nucleus. The distance
Q.3 A string of mass ‘m’ and length ‘I’ rests over a of the closest approach is of the order of
frictionless table with 1/4th of its length hanging from a 

side. The work done in bringing the hanging part back (A) 1 A (B) 10−10 cm
on the table is: (C) 10−12 cm (D) 10−15 cm
(A) mgl / 4 (B) mgl / 32
(C) mgl / 16 (D) None of these Q.10 A simple pendulum has a string of length and bob
of mass m. When the bob is at its lowest position, it is
given the minimum horizontal speed necessary for it to
Q.4 A weight mg is suspended from a spring. If the move in a circular path about the point of suspension.
elongation in the spring is xo, the elastic energy stored The tension in the string at the lowest position of the
in it is: bob is:
1 1 (A) 3mg (B) 4mg (C) 5mg (D) 6mg
(A) mgxo (B) 2mgx0 (C) mgx0 (D) mgx0
2 4
Q.11 A horse pulls a wagon with a force of 360N at an
Q.5 A ball is thrown up with a certain velocity at angle of 600 with the horizontal at a speed of 10Km/
angle θ to the horizontal. The kinetic energy KE of the hr. The power of the horse is:
horizontal. The kinetic energy KE of the ball varies with
horizontal displacement x as: (A) 1000 W (B) 2000 W
KE KE (C) 500 W (D) 750 W

(A) (B) Q.12 A man pulls a bucket of water from a well of depth
x x H. If the mass of the rope and that of the bucket full of
O O
water are m and M respectively, then the work done by
KE KE
the man is:
(D)
(C)
( ) m 
(A) m + M gh (B)  + M  gh
O
x
O
x 2 
m +M
(D)  m + M  gh
 
(C)   gh
Q.6 A body m1 is projected upwards with velocity v1  2   2
another body m2 of same mass is projected at an angle
of 45o. Both reach the same height. What is the ratio of Q13 A small block of mass m is kept on a rough inclined
their kinectic energies at the point of projection: surface of inclination θ fixed in a elevator. The elevator
goes up with a uniform velocity v and the block does
(A) 1 (B) 1/2 (C) 1/3 (D) 1/4
not slide on the wedge. The work done by the force of
friction on block in time t will be-
Q.7 A uniform chain of length L and mass M is lying on
a smooth table and one third of its length is hanging (A) Zero (B) mgvtcosθ
(C) mgvtsinθ (D) mgvtsin2θ
5 . 3 0 | Work , Energy and Power

Q.14 Two equal masses are attached to the two Previous Years Questions
ends of a spring of spring constant k. the masses are
pulled out symmetrically k. the masses are pulled out
symmetrically to stretch the spring by a length x over Q.1 Two masses of 1g and 4g are moving with equal
its natural length. The work done by the spring on each kinetic energies. The ratio of the magnitudes of their
mass is- momenta is: (1980)
1 1 1 1 (A) 4:1 (B) 2 :1 (C) 1:2 (D) 1:16
(A) kx2   (B) − kx2   (C) kx2   (D) − kx2
2 2 4 4
Q.2 A stone tied to a string of length L is whirled in a
Q.15 A particle is acted by a force F-kx, where k is a vertical circle with the other end of the string at the
+ve constant. Its potential energy at x-0 is zero. Which centre. At a certain instant of time, the stone is at its
curve correctly represents the variation of potential lowest position, and has a speed µ . The magnitude of
energy of the block with respect to x? the change in its velocity as it reaches a position, where
U U the string is horizontal, is  (1998)
U U
(A) u2 − 2gL (B) 2gL
(A) x (B) x
( )
(A) x (B) x
(C) u2 − gL (D) 2 u2 − gL

U U U U Q.3 A wind-powered generator converts wind energy


into electric energy. Assume that the generator converts
(C) (C) x (D) x (D) x x a fixed fraction of the wind energy intercepted by its
blades into electrical energy. For wind speed v, the
electrical power output will be proportional to (2000)

Q.16 If W1 , W2 and W3 represent the work done in (A) v (B) v 2 (C) v 3 (D) v 4
moving a particle from A to B along there different
paths 1, 2 and 3 respectively (as shown) in the Q.4 An ideal spring with spring constant k is hung from
gravitational field of a point mass m, find the correct the ceiling and a block of mass M is attached to its
relation between W1 , W2 and W3 . lower end. The mass is released with the spring initially
(A) W1 > W2 > W3 (B) W
= W
= W3 unstretched. Then the maximum extension in the spring
1 2
is  (2002)
(C) W1 < W2 < W3 (D) W2 > W1 > W3
2 1
M2 M1
S2 S1
Q.17 An ideal spring with spring-constant k is hung B
from the ceiling and a block of mass M is attached
to its lower end. The mass is released with the spring 2  1
initially unstretched. Then the maximum extension in M2 x M1
S2 S1
the spring is, B

(A) k =
(2 + 3 ) mg (B)
2Mg

x
3R k
Mg 4Mg 4Mg 2Mg Mg Mg
(C) (D) (A)   (B)   (C)   (D)
k 2k k k k 2k

Q.5 A block (B) is attached to two unstretched springs


Q.18 The total work done on a particle is equal to the S1 and S2 with spring constants k and 4k, respectively.
change in its kinetic energy: The other ends are attached to supports M1 and M2 not
(A) Always attached to the walls. The springs and supports have
negligible mass. There is no friction anywhere. The
(B) Only if the forces acting on it are conservative block B is displaced towards wall I by a small distance x
(C) Only if gravitational force alone acts on it and released. The block returns and moves a maximum
distance y towards wall 2. Displacement x and y are
(D) Only if elastic force alone acts on it
P hysi cs | 5.31

measured with respect to the equilibrium position of (A) Statement-I is false, Statement-II is true
the block B.
(B) Statement-I is true, Statement-II is false
y
The ratio is (2008)
x (C) Statement-I is true, Statement-II is the correct
explanation for Statement-I
1 1
(A) 4   (B) 2    (C)    (D) (D) Statement-I is true, Statement-II is true, and Statement-
2 4
II is not the correct explanation for Statement-I.
Q.6 This question has Statement-I and Statement-II. Of
the four choices given after the statements, choose the Q.7 A person trying to lose weight by burning fat
one that best describes the two statements. lifts a mass of 10 kg upto a height of 1 m 1000 times.
Assume that the potential energy lost each time he
If two springs S1 and S2 of force constants k1 and k2, lowers the mass is dissipated. How much fat will he use
respectively, are stretched by the same force, it is found up considering the work done only when the weight
that more work is done on spring S1 than on spring S2. is lifted up? Fat supplies 3.8 × 107 J of energy per kg
 (2012) which is converted to mechanical energy with a 20 %
Statement-I: If stretched by the same amount, work efficiency rate. Take g = 9.8 ms-2:  (2016)
done on S1, will be more than that on S2 (A) 6.45 × 10-3 kg (B) 9.89 × 10-3 kg
Statement-II: k1 < k2 (C) 12.89 × 10-3 kg (D) 2.45 × 10-3 kg

JEE Advanced/Boards

Exercise 1 Q.3 A 0.5kg block slides from point A on a horizontal


track with a initial speed of 3m/s towards a weightless
spring of length l m and having a force constant 2
Q.1 A small disc of mass m slides down a smooth hill of
N/m. The part AB of the track is frictionless and the
height h without initial velocity and gets onto a plank of
part BC has coefficent of static and kinetic friction as
mass M lying on the horizontal plane at the base of hill
0.22 and 0.20 respectively. If the distacnces AB and BD
as shown in the Figure. Due to friction between the disc
are 2m and 2.14m respectively, find the total distance
and the plank, disc
m through which the block moves before it comes to rest
slows down and
beginning with a certain (
completely. g = 10m / s2 )
moment, moves in one h m
piece with the plank. M 3m/s
Find out total work A B D C
performed by the frictional forces in this process.
2m 2.14m 1m

Q.2 A block of mass m starts from rest to slide along


Q.4 A particle is suspended from a fixed point by a
a smooth frictionless track of the shape shown in the
string of length 5m. It is projected horizontally from the
Figure. What should be height h so that when the mass
equilibrium position with such a speed that the string
reaches point A on the track, it pushes the track with a
slackens after the particle has reached a height of 8m
force equal to thrice it weight?
above the lowest point. Find the speed of the particle
m just before the string slackens and the height to which
A the particle will rise further.

h Q.5 Two blocks of= masses m1 2kg = and m2 5kg


are moving in the same direction along a fricionless
surface with speeds 10m/s and 3m/s respectively, m2
being ahead of m1 . An ideal spring with k=1120N/m
is attached to the back side of m2. Find the maximum
5 . 3 2 | Work , Energy and Power

compression of the spring when the blocks collide. down before coming to rest for the first time.
m1 m2
(b) How far below the initial position of m is the
m1 m2 equilibrium position of m located?

Q.9 A string, with one end fixed on a rigid wall, passing


over a fixed frictionless pulley at a distance of 2m from
Q.6 An automobile of mass m accelerates, starting from the wall, has a point mass M=2kg attached to it at a
rest, while the engine supplies constant power P; show distance of 1m from the wall. A mass m=0.5kg attached
that: at the free end is held at rest so that the string is
(a) The velocity is given as a function of time by horizontal between the wall and the pulley and vertical
beyond the pulley. Find the speed with which the mass
v = ( 2Pt / m)
1/2
M will hit the wall when the mass m is released?
(b) The position is given as a function of time by
s = ( 8P / 9m)1/2 t3/2 Q.10 A massless platform is kept
0.1 kg
on a light elastic spring, as shown
Q.7 One end of a light spring of natural length d and in the Figure. When a sand particle
spring constant k is fixed on a rigid wall and the other of mass 0.1kg is dropped on the
end is fixed to a smooth ring of mass m which can slide pan from a height of 0.2m, the
without friction on a vertical rod fixed at a distance d particle strikes the pan and sticks
from the wall. Initially the spring makes an angle of 37o of 0.2m, the particle strikes the
with the horizontal as shown in the Figure. If the system pan and sticks to it while the
is released from rest, find the speed of the ring when spring compresses by 0.01m. From
what height should be particle be dropped to cause a
the spring becomes horizontal sin370 = 3 / 5 
  compression of 0.04m?
Ring
A Q.11 Two blocks A and B each
=0
having mass of 0.32kg are
l connected by a light string S
h
passing over a smooth pully A m
o
as shown in the Figure.
37 
O The horizontal surface on
d B m
Rod which the block A slides
is smooth. The block A is
Q.8 A ring of mass m=0.3kg slides over a smooth attached to a spring of force constant 40N/m whose
vertical rod A. Attached to the ring is a light stirng other end is fixed to a support 0.40m above the
passing over a smooth fixed pulley at a distance of horizontal surface. Initially, when the system is releasesd
0.8m from the rod as shown in the figure. At the other to move, the spring is vertical and unstretched. Find the
end of the string there is a mass M=0.5kg. The ring is velocity of the block A at the instant it breaks off the
held in level with the pully and then released. surface below it. g = 10m / s2 
 

A Q.12 A block of mass m is held at rest on a smooth


0.8m
m horizontal floor. A light frictionless, small pulley is fixed
C
at a height of 6m from the floor. A light inextensible
string of length 16m, connected with A passes over the
pulley and another identical block B is hung from the
string. Initial height of B is 5m from the floor as shown
in the figure. When the system is released from rest, B
starts to move vertically downwards and A slides on the
M B
floor towards right.

(a) Determine the distance by which the mass m moves


P hysi cs | 5.33

freely and touches the surface of the table with its end
P B. At a certain moment, the end A of the chain is set
free, with what velocity will this end of the chain slip
out of the tube?
6m
B A

5m
A
h
(a) If at an instant the string makes an angle θ with the B
horizontal, calculate relation between velocity u of A
velocity v of B.
Q.17 A system consists of two identical m
(
(b) Calculate v when B strikes the floor. g = 10m / s2 ) cubes, each of mass m, linked together
by the compressed weightless spring
Q.13 Two blocks are connected by a string as shown in constant k. The cubes are also connected
the Figure. They are released from rest. Show that after by a thread which is burned through at m
a certain moment. Find:
they have moved a distance L, their common speed is (a) At what values of ∆ l the initial compression of the
2 (m2 − µml) gl spring, the lower cube will bounce up after the thread
given by , where µ is the has been burned through:
(m1 − m2 )
(b) To what height h the centre of gravity of this system
coeffiecient of friction between the floor and the blocks. will rise if the initial compression of the spring ∆l=7mg/k

m1
Q.18 A stone with weight w is thrown vertically upward
into the air with initial speed v 0 . If a constant force f
due to air drag acts on the stone throughout its flight:
m2 (a) Show that the maximum height reached by the
v 20
stone is h = .
2g 1 + ( f / w ) 
Q.14 A particle of mass m is moving in a circular path of
constant radius r such that is centripetal acceleration αc (b) Show that the speed of the stone upon impact with
1/2
is varying with time t as αc=k2rt2 when k is a constant. w−f
what is the power delivered to the particle by the forces the ground is v = v 0  
acting on it?
w+f
Q.19 One end of spring of natural length h is fixed at
Q.15 A body of mass m was slowly pulled up the hill as the ground and the other end is fitted with a smooth
shown in the Figure. by a force F which at each point ring of mass m which is allowed to slide on a horizontal
was directed along a tangent to the trajectory. rod fixed at a height h as shown in Figure. Initially, the
spring makes an angle of 37o with the vertical when the
F
m system is released from rest. Find the speed of the ring
when the spring becomes vertical.
h
m

l
h
Find the work performed by this force, if the height of o
37
the hill is h, the length of its base l, and the co-effiecient
of frinction between m and the hill is.

Q.16 A chain A B of length l is loaded in a smooth Q.20 A nail is located at a certain distance verticaly
horizontal tube so that a part of its length h hangs below the point of suspension of a simple pendulum.
5 . 3 4 | Work , Energy and Power

The pendulum bob is released from the position where spring so that the block presses the track with a force
the string makes an angle of 60o with the downward mg when it reaches the point P, where the radius of the
vertical. Find the distance of the nail from the point track is horizontal.
of suspension such that the bob will just perform a
complete revolution with the nail as centre. The length
of the pendulum is 1m. R
P

Q.21 A partical is suspended vertically O


from a point O by an inextensible
massless string of length L.A vertical L/B
line A B is at a distance of L/8 form O as
L
shown in the Figure. The particle is Q.24 A particle of mass 100g is suspended from one
given a horizontal velocity u. At some end of a weightless string of length 100cm and is
point, its motion cases to be circular allowed to swing in a vertical plane. The speed of the
and eventually the object passes
u
B mass is 200cm/s when the string makes an angle of 60o
through the line A B. At the instant of with the vertical.
crossing A B, its velocity is horizontal. Find u. Determine
(a) The tension in the string at 60o and
Q.22 A spherical ball of mass m is kept at the highest
point in the space between two fixed, concentric (b) The speed of the particle when it is at the lowest
spheres A and B. The smaller sphere A has a radius R position. (Take g=980 cm/s2)
and the space between the two spheres has a width
d. the ball has a diameter very slightly less that d. All Q.25 A smooth horizontal rod AB can rotate about a
surfaces are frictionless. The ball is given a gental push vertical axis passing thorugh its end A. The rod is fitted
(towards the right in the Figure). The angle made by with a small sleeve of mass m attached ot the end A by
the radius vector of the ball with the upwards vertical is a weightless spring of length l and spring constant k.
denoted by (see Figure). what work must be performed to slowly get this system
going and reaching to slowly get this system going and
reaching the angular velocity ω
Spbare B

Q.26 Figure shows a smooth track which consists of a


straight inclined part of length L joining smoothly with
d the circular part. A particle of mass m is projected up
O the inclined from its bottom.

R
Spbare A L

(a) Express the total normal reaction force exerted by


the spheres on the ball as a function of angle θ
(b) Let NA and NB denote the magnitudes of the normal
reaction forces on the ball exerted by the spheres A (a) Find the minimum projection speed u0 for which the
and B, respectively. Sketch the variations of NA and NB particle reaches the top of track.
as fuctions of cos θ in the range 0 ≤ θ ≤ π by drawing
(b) Assuming that the projection speed is 2 u0 and that
two separate graphs in your answer book, taking cosθ
the block does’t lose contact with the track before
on the horizontal axes.
reaching its top. Find the force acting on it when it
reaches the top.
Q.23 Figure shows a smooth track, a part of which is a
(c) Assuming that the projection speed is only slightly
circle of radius R. A block of mass m is pushed against
grater than v0, where will the block lose contact with
a spring of spring constant k fixed at the left end and
the track?
is then released. Find the initial compression of the
P hysi cs | 5.35

Q.27 A small block of mass m slides along a smooth Exercise 2


frictional track as shown in the Figure.
Single Correct Choice Type
P
Q.1 When water falls from the top of a water fall 100m
high:
(A) It freezes
5R
(B) It warms up slightly
R
(C) It evaporates
Q
R
(D) There is no change in temperature.

Q.2 A 2kg block is dropped from a height of 0.4m on


(a) If it starts from rest at P, what is the resultant force a spring of force constant 2000N/m. the maximum
acting on it at Q? compression of the spring is:
(b) At what height above the bottom of the loop should (A) 0.1m   (B) 0.2m   (C) 0.01m  (D) 0.02m
the block be released so that the force it exerts against
the track at the top of the loop equals its weight?
Q.3 A partical of mass M is moving in a horizontal circle of
radius ‘R’ under the centripetal force equal to K/R2, where
Q.28 A particle slides along a track with elevated ends K is constant. The potential energy of the particle is
and a flat central part as shown in the Figure. The flat
(A) K/2R   (B) –K/2R   (C) K/R   (D) –K/R
central part as shown in the Figure. The flat part has
a length l=3.0m. The curved portions of the track are
frictionless. For the flat part the coefficient of kinectic Q.4 A linear harmonic oscillator of force constant
friction is µk = 0.20 , the partical is released at point A
2×106N and amplitude 0.01 m has a total mechanical
which is at a height h=1.5m above the flat part of the energy of 160 J. Its
track. Where does the particle finally come to rest? (A) Maximum potential energy is 100 J
A (B) Maximum kinetic energy is 100 J
(C) Maximum potential energy is 160 J
h
(D) Maximum potential energy is zero.
n c
l Q.5 The potential energy of particle varies with position
x according to the relation
Q.29 The system of mass A and B shown in the Figure U(x) =
2 x 4 − 27x the point x =
3 / 2 is point of
is released from rest with x=0, determine (A) Unstable equilibrium (B) Stable equilibrium
4m 4m (C) Neutral equilibrium (D) None of these

Q.6 A particle of mass m is fixed to one end of a light


rigid rod of length l and rotated in a vertical circular
path about is other end. The minimum speed of the
particle at its hightest point must be
m
A
2 (A) Zero (B) g (C) 1.5g (D) 2g

Q.7 A particle of mass m is fixed to one end of a light


m B
spring of force constant k and unstretched length l. The
system is rotated about the other end of the spring
with an angular velocity ω , in gravity free space. The
(a) The velocity of mass B when x=3m.
increase in length of the spring will be:
(b) The maximum displacement of mass B.
(A) 1.5g (B) g (C) 2g (D) None
5 . 3 6 | Work , Energy and Power

Q.8 A particle of mass m is moving in a circular path of the above 1s.


constant radius r such that its centripetal acceleration
(D) The work done by the force of gravity is -20J in the
ac is varying with time as ac = k 2rt2 where k is a
above 1s.
constant. The power delivered to the particle by the
forces acting on it is:
Q.13 Consider two observers moving with respect
2 2
(A) 2πmk 2r 2 t (B) mk r t to each other at a speed v along a straight line. They
1 observe a block of mass m moving a distance  on a
(C) mk 4r 2 t5 (D) 0 rough surface. The following quantities will be same as
3
observed by the two observers.
Q.9 A simple pendulum having a bob of mass m is (A) Kinetic energy of the block at time t
suspended from the ceiling of a car used in a stunt film
(B) Work done by friction.
shooting. The car moves up along an inclined cliff at a
speed v and makes a jump to leave the cliff and lands at (C) Total work done on the block
some distance. Let R be the maximum height of the car
(D) Acceleration of the block.
from the top of the cliff. The tension in the string when
the car is in air is Multiple Correct Choice Type
mv 2 mv 2
(A) mg (B) mg (C) mg (D) Zero
R R Q.14 A particle of mass m is attached to a light string
−−
of length  , the other end of which is fixed. Initially the
Q.10 A particle, which is constrained to move along the string is kept horizontal and the particle is given an
x-axis, is subjected to a force in the same direction which upward velocity v, the particle is just able to complete
varies with the distance x of the particle from the origin a circle.
as F(x) = − kx + ax3. Here k and a are positive constants.
(A) The string becomes slack when the particle reaches
For x ≥ 0 , the functional form of the potential energy
its highest point
U ( x ) of particle is
(B) The velocity of the particle becomes zero
U(x) U(x)
(C) The kinetic energy of the ball in initial position was
1
(A) x (B) x mv 2 = mg  .
2
(D) The particle again passes through the initial position.
U(x) U(x)
Q.15 The kinetic energy of a particle continuously
increases with time.
(C) x (D) x
(A) The resultant force on the particle must be parallel
to the velocity at all instants
Q.11 A wind-powered generator converts wind (B) The resultant force on the particle must be at an
energy into electric energy. Assume that the generator angle less than 900 all the time
converts a fixed fraction of the wind energy intercepted
by its blades into electric energy. For wind speed V, the (C) Its height above the ground level must continuously
electrical power output will be proportional to decreases.

(A) V    (B) v 2    (C) v 3    (D) v 4 (D) The magnitude of its linear momentum is increasing
continuously.
Q.12 A block of mass M is hanging over a smooth and
light pulley thorugh a light string. The other end of the Q.16 One end of a light spring of constant k is fixed to
string is pulled by a constant force F. The string energy a wall and the other end is tied to block placed on a
of the block increases by 20 J in 1s. smooth horizontal surface. In a displacement, the work
1
(A) The tension in the string is Mg. done by the spring is kx2 . The possible cases are.
2
(B) The tension in the string is F.
(A) The spring was initially compressed by a distance x
(C) The work done by the tension on the block is 20J in and was finally in its natural length.
P hysi cs | 5.37

(B) It was initially stretched by a distance x and finally the correct explanation of assertion.
was in its natural length.
(C) Assertion is true but reason is false
(C) It was initially in its natural length and finally in a
(D) Assertion is false but reason is true.
compressed position.
(D) It was initially in its natural length and finally in a Q.21 Assertion: For stable equilibrium force has to be
stretched position. zero and potential energy should be minimum.
Reason: For equilibrium, it is not necessary that the
Q.17 No work is done by a force on an object if,
force is not zero.
(A) The force is always perpendicular to its velocity
(B) The force is always perpendicular to its acceleration Q.22 Assertion: The work done in pushing a block is
more than the work done in pulling the block is more
(C) The object is stationary but the point of application
than the work done in pulling the block on a rough
of the force moves on the object
surface.
(D) The object moves in such a way that the point of
Reason: In the pushing condition normal reaction is
application of the force remains fixed
more

Q.18 A particle is acted upon by a force of constant


Q.23 Assertion: Potential energy is defined for only
magnitude which is always perpendicular to the velocity ∧
of the particle. The motion of the particle takes place in  dUn
conservation forces Reason F = −
a plane. It follows that, dr
(A) Its velocity is constant Q.24 Assertion: An object of mass m is initially at rest.
(B) It acceleration is constant A constant force F acts on it. Then the velocity gained
by the object in a fixed displacement is proportional to
(C) Its kinetic energy is constant
1
(D) It moves in a circular path
m
Reason: For a given force and displacement velocity is
Q.19 A heavy stone is thrown from a cliff of height h always inversely proportional to root of mass.
in a given direction. The speed with which it hits the
ground:
Comprehension Type
(A) Must depend on the speed of projection
Paragraph 1
(B) Must be larger then the speed of projection
The work done by external forces on a body is equal
(C) Must be independent of the speed of projection
to change of kinetic energy of the body. This is true
(D) May be smaller than the speed of projection for both constant and variable force (variable in both
magnitude and direction). For a particle W = ∆k . For a
Q.20 You lift a suitcase from the floor and keep it on a system,
table. The work done by you on the suitcase does not Wnet =
Wcal + Wpseudo =
∆Kcm or
depend on:
Wext + Wnonconservative =∆K + ∆U .
(A) The path taken by the suitcase
In the absence of external and non conservative
(B) The time taken by you in doing so forces, total mechanical energy of the system remain
(C) The weight of the suitcase conserved.

(D) Your weight


Q.25 I-work done in raising a box onto a platform
Assertion Reasoning Type depends on how fast it it raised II-work is an inter
convertible form of energy.
(A) Both assertion and reason are true and reason is the
correct explanation of Assertion. (A) 1-False II-true (B) 1-False II-False

(B) Both assertion and reason are true and reason is not (C) 1-True II-False (D) 1-True II-true
5 . 3 8 | Work , Energy and Power

Q.26 Consider a case of rigid body rolling without Q.30 Relative acceleration between two beads at the
sliding over a rough horizontal surface initial moment:
(A) There will be a non-zero conservative force acting (A) g/2 vertically away from each other
on the body and work done by non-conservative force
(B) g/2 horizontally towards each other
will be positive.
(C) 2g / 3 Vertically away from each other
(B) There will be non-zero non-conservative force
acting on body and work done by non-conservative (D) 2g / 3 Horizontally towards each other
force will be negative.
(C) There will be no non-conservative force acting on Q.31 The speed of bead when spring is at normal length
the body but totoal mechanical energy will not be
conserved.
(A)
(2 − 3 ) gR (B) (2 + 3 ) gR
(D) There will be no non-conservative force acting on 3 3
the body and total mechanical energy will be conserved.
2gR
(C) (D) 3gR
Q.27 Now consider a case of rigid body rolling with 3
sliding along rough horizontal plane and Vcm is linear.
Velocity by ω =Vcm / 2R , R is radius of body at (t=0)
Q.32 Choose the correct statement
(A) There is no non-conservative force acting on body.
(A) Maximum angle made by spring after collision is
(B) There is a non-conservative force acting on body same as that at initial moment.
and direction of force is opposite to direction of velocity.
(B) If the collision is perfectly inelastic, the total energy
(C) There is a non-conservative force acting on body is conserved.
and direction of the force along the direction of velocity.
(C) If the collision is perfectly elastic, each bead
(D) None of these. undergoes SHM.

Q.28 In the above problem if W=3Vcm/R where Vcm (D) Both linear momentum and angular momentum
velocity of centre of mass at t=0 with respect to centre of smooth ring are conserved
only at the instant of collision.
(A) There is non-conservative force acting on body.
(B) There is non-conservative force acting on body the Match the Columns
direction of velocity of centre of mass.
Q.33 A single conservative force acts on a body of
(C) There is a non-conservative force acting on body
mass 1kg that moves along the x-axis. The potential
opposite to the direction of velocity
energy U(x) is given by U(x) = 20 + ( x − 2 ) where x is
2

(D) None of these the meters. At x=5.0m the particle has a kinetic energy
of 20 J then:
Paragraph 2
Column-I Column-II
Two idedtical beads are attached to free ends of two
(A) Minimum value of x in meters (p) 29

identical springs of spring constant k=


(2 + 3 ) mg . (B) Maximum value of x in meters (q) 7.38
3R
(C) Maximum potential energy in (r) 49
Initially both springs make an angle of 600 at the fixed
joules
point normal length of each spring is 2R. Where R is the
radius of smooth ring over which bead is sliding. Ring (V) Maximum kinetic Energy in (s) -3.38
is placed on vertical plane and beads are at symmetry joules
with respect to vertical line as diameter.
Q.34 A body of mass 75 kg is lifted by 15m with an
Q.29 Normal reaction on one of the bead at initial acceleration of g/10 by an ideal string. If work done
moment due to ring is by tension in string is W1, magnitude of work done by
(A) mg / 2 (B) 3mg / 2 gravitational force is W2, kinetic energy when it has
(C) mg (D) Insufficient data lifted is K and speed is W1, magnitude of work done
P hysi cs | 5.39

by gravitational force is W2, kinetic energy when it has beyond the pulley. What will be the speed with which
lifted is K and speed of mass when it has lifted is v then: the mass M will hit the wall when the mass the m is
(data in column is given in SI units) (g=10 m/s2) released? (Take g=9.8 m/s2)  (1985)

Column I Column II M
(A) W1 (p) 10800

(B) W2 (q) 1080

(C) K (r) 11880


M
(D) v (s) 5.47

Q.5 A bullet of mass M is fired with a velocity 50m/s


Previous Years’ Questions at an angle θ with the horizontal. At the highest point
of its trajectory, it collides head-on with a bob of mass
3M suspended by a massless string of length 10/3m
Q.1 The displacement x of a particle moving in one
gets embedded in the bob. After the collision the string
dimension, under the action of a constant force is
moves through an angle of 120o. Find (a) The angle θ
related to the time t by the equation= t x + 3 . Where (b) The vertical and horizontal coordinates of the initial
x is in metre and t in second, Find: (a) The displacement position of the bob with respect to the point of firing of
of the particle when its velocity is zero, and (b) The the bullet (take g = 10 m/s2) (1988)
work done by the force in the first 6s.  (1980)
Q.6 A particle is suspended vertically O
Q.2 A body of mass 2kg is being dragged with a from a point O by an inextensible L/8
A
uniform velocity of 2m/s on a rough horizontal plane. massless string of length L. A vertical
The coefficient of friction between the body and the line AB is at a distance L/8 from O as
surface is 0.20, J=4.2 J/cal and g=9.8m/s2. Calculate the shown in Figure. The object is given a
amount of heat generated in 5s. (1980) horizontal velocity u. L

At some point, its motion ceases to


Q.3 Two blocks A and B are connected to each other be circular and eventually the object
by a string and a spring; the string passes over a passes through the line AB. At the
B

frictionless pulley as shown in the Figure. Black B slides instant of crossing AB, its velocity is
over the horizontal top surface of a stationary block C horizontal. Find u.  (1999)
and the block A slides along the vertical side of C, both
with the same uniform speed. The coefficient of friction
between the surfaces of block is 0.2. Force constant Q.7 A light inextensible string that goes
of the spring is 1960 N/m. If mass of block A is 2kg. over a smooth fixed pulley as shown in
Calculate the mass of block B and the energy stored in the Figure connects two blocks of
the spring.  (1982) masses 0.36 kg and 0.72kg. taking
g = 10 ms−2 , find the work done (in
B Joule) by string on the block of mass
0.36kg during the first second after the
system is released from rest (2009)

Q.8 A block of mass 0.18kg is attached to a spring of


A force constant 2N/m. The coefficient of friction between
the block and the floor is 0.1. Initially the block is at rest
Q.4 A string, with one end fixed on a rigid wall, passing and the spring is unstretched. An impulse is given to
over a fixed frictionless pulley at a distance of 2m the block as shown in the Figure.
from the wall has a point mass M=2kg attached to it
at a distance of 1m from the wall. A mass m=0.5kg
attached at the free end is held at rest so that the string

is horizontal between the wall the pulley and vertical
K K
(A) (B)
5 . 4 0 | Work , Energy and Power
t t
The block slides a distance of 0.06m and comes to rest
for the first time. The initial velocity of the block in m/s K K
N (C) (D)
is v = . Then N is.  (2011)
10
t t
Q.9 The work done on a particle of mass m by a force
  The coefficient of friction between the block m1 and the
 x ˆ y ˆ  plane is always zero. The coefficient of static and
K i+ j (K being a constant of
3/2 
( ) ( ) dynamic friction between the block m2 and the plane
3/2
 x2 + y 2 x2 + y 2 
  are equal to µ = 0.3. In List II expressions for the friction
on the block m2 are given. Match the correct expression
appropriate dimensions, when the particle is taken
of the friction in List II with
from the point (a, 0) to the point (0, a) along a circular
the angles given in List I, and
path of radius a about the origin in the x-y plane is 
choose the correct option.
 (2013)
2Kπ Kπ Kπ The acceleration due to
(A) (B) (C) (D) 0 m1
a a 2a gravity is denoted by g. m2
[Useful information: tan (5.5°)
Q.10 A bob of mass m, suspended by a string of length
≈ 0.1; tan (11.5°) ≈ 0.2; tan
l1 is given a minimum velocity required to complete 
(16.5°) ≈ 0.3] (2014)
a full circle in the vertical plane. At the highest point,
it collides elastically with another bob of mass m List I List II
suspended by a string of length l2, which is initially at
I θ = 5° p. m2g sin θ
rest. Both the strings are mass-less and inextensible.
II θ = 10° q. (m1+m2) g sin θ
If the second bob, after collision acquires the minimum
speed required to complete a full circle in the vertical III θ = 15° r. mm2g cosθ
plane, the ratio l1/l2 is  (2013)
IV θ = 20° s. µ(m1 + m2)g cosθ

Q.11 A particle of mass 0.2 kg is moving in one Code:


dimension under a force that delivers a constant power (A) P-1, Q-1, R-1, S-3
0.5 W to the particle. If the initial speed (in m/s) of the
(B) P-2, Q-2, R-2, S-3
particle is zero, the speed (in m/s) after 5 s is  (2013)
(C) P-2, Q-2, R-2, S-4
Q.12 A tennis ball is dropped on a horizontal smooth (D) P-2, Q-2, R-3, S-3
surface. It bounces back to its original position after
hitting the surface. The force on the ball during the
Q.14 Consider two different metallic strips (1 and 2)
collision is proportional to the length of compression of
of same dimensions (lengths  , width w and thickness
the ball. Which one of the following sketches describes
d) with carrier densities n1 and n2, respectively. Strip
the variation of its kinetic energy K with time t most
1 is placed in magnetic field B1 and strip 2 is placed
appropriately? The figures are only illustrative and not
in magnetic field B2, both along positive y-directions.
to the scale.  (2014)
Then V1 and V2 are the potential differences developed
between K and M in strips 1 and 2, respectively.
Q.13 A block of mass m1 = 1 kg another mass m2 = 2kg, Assuming that the current I is the same for both the
are placed together (see figure) on an inclined plane strips, the correct option(s) is(are)  (2015)
with angle of inclination θ. Various values of θ are given
(A) If B1 = B2 and n1 = 2n2, then V2 = 2V1
in List I.
(B) If B1 = B2 and n1 = 2n2, then V2 = V1
K K (C) If B1 = 2B2 and n1 = n2, then V2 = 0.5V1
(A) (B)
(D) If B1 = 2B2 and n1 = n2, then V2 = V1
t t

K K
(C) (D)

t t
P hysi cs | 5.41

PlancEssential Questions
JEE Advanced/ Boards
JEE Main/Boards
Exercise 1
Exercise 1
Q.3 Q.5 Q.7 Q.11
Q. 18 Q.19 Q.22 Q.23
Q.13 Q.19 Q.21 Q.30

Exercise 2
Exercise 2
Q.4 Q.10 Q.12 Q.33
Q.1 Q.5 Q.10 Q.13
Q.34
Q.15 Q.16 Q.17

Previous Years’ Questions


Previous Years’ Questions
Q.3 Q.4 Q.6 Q.8
Q.3 Q.4 Q.8

Answer Key

JEE Main/Boards
Exercise 1
Q.1 Zero Q.18 16 J Q.19 1750 J; -1000 J

Q.20 10 4 J Q.21 3.75 × 103 J Q.22 -0.28 mgH

Q.23 3150N

Exercise 2
Single Correct Choice Type

Q.1 C Q.2 C Q.3 B Q.4 A Q.5 C

Q.6 B Q.7 C Q.8 C Q.9 C Q.10 D

Q.11 C Q.12 B Q.13 C Q.14 D Q.15 B

Q.16 B Q.17 B Q.18 A

Previous Years’ Questions


Q.1 C Q.2 D Q.3 C Q.4 B Q.5 C

Q.6 A Q.7 C
5 . 4 2 | Work , Energy and Power

JEE Advanced/Boards
Exercise 1

Single Correct option


 M 
Q.1 mgh   Q.2 4 R Q.3 4.24 M
M+m
1/2
1  8P 
Q.4 5.42M / S 2 ,0.97M Q.5 0.25 M Q.6 s =   t3/2
 9m 
3g k
Q.7 d + Q.8 (a) 1.5m (b) 0.6 m Q.9 3.3 m/s
2d 16m

40
Q.10 3.32 m Q.11 154 m/s Q.12 m /s
41


Q.14 mk2r2t Q.15 mgh + µmgl Q.16 2hg n  
h
1/2
3mg 8mg w−f h k
Q.17 (a) D> (b) h = Q.18 v0   Q.19
k k w+f 4 m

Q.20 0.8m Q.21 2.14 gL

−1 ( cos θ − 2 ) ; (b)for θ ≤ cos ( 2 / 3 )= mg ( 3cos θ − 2 )


−1
Q.22= θ ≤Ncos
(a) mg mg ( 3cos θ − ,N
2 )B 0,N
(b)for ( 2 / 3)=,NB 0,N = A
= A

≥ cos2 −(3cos
( 2 / 3)θ= ) =B mg ( 2 − 3cos θ)
−1
and for θ ≥ cos−1 ( 2 / 3)= NA and 0,N
= for θmg
B
NA 0,N

3mgR k2 x'(1 + x')
Q.23 Q.24 2.12 m/s Q.25
k 2(1 − x')2
 L 
Q.26 (a) 2g R (1 − cos θ ) + L sin θ  ; (b) 6mg  1 − cos θ + sin θ 
 R 
−1  2 
(c) The radius through the particle makes an angle cos   with the vertical.
3
Q.27 3 R Q.28 Third trip Q.29 8 2 m

Exercise 2

Single Correct Choice Type

Q.1 B Q.2 A Q.3 C Q.4 C Q.5 B

Q.6 A Q.7 B Q.8 B Q. 9 D Q.10 D

Q.11 C Q.12 B Q.13 D

Multiple Correct Choice Type

Q.14 A, D Q.15 B, D Q.16 A, B Q.17 A, C, D Q.18 C, D

Q.19 A, B Q.20 A, B, D
P hysi cs | 5.43

Assertion Reasoning Type

Q.21 B Q.22 A Q.23 A Q.24 B

Comprehension Type

Paragraph 1
Q.25 A Q.26 D Q.27 B Q.28 B

Paragraph 2
Q.29 C Q.30 D Q.31 C Q.32 D

Match the Columns

Q.33 A → s; B → q; C → r; D → p Q.34 A → r; B → p; C → q; D → s

Previous Years’ Questions


Q.1 0,0 Q.2 9.33 cal Q.3 0.098J
 3 3
Q.4 3.29m/s Q.5 300 , (108.25m, 31.25m) Q.6 u
= gL  2 + 
 2 

Q.7 8 J Q.8 D Q.9 D

Q.10 5 Q.11 5 Q.12 B

Q.13 D Q.14 A, C

Solutions

JEE Main/Boards Sol 2: Two bodies have same linear momentum so


m1v1 = m2v2
Exercise 1 m1 = mass of first object
 v1 = velocity of first object
Sol 1: If the vector product F. s = 0 , then work done is
zero when m2 = mass of second object

F = force acting on the body v2 = velocity of second object
 Case – I : m1 > m2
s = displacement of the body
Force does no work under given condition m1v1 = m2v2 ⇒ v1<v2
(i) Direction of force and displacement is perpendicular 1
kinetic energy KE. = m v2
2 1
(ii) Displacement is zero
1 1
Example: KE1 = m1v12 ; K2 = m v 2 ⇒ K2 > K1
2 2 2 2
m Case – II : m2 >m1

mg m1v1 = m2v2
⇒ v2 < v1 ⇒ KE1 > KE2
Work done by gravity and normal force is zero.
5 . 4 4 | Work , Energy and Power

1 W = |F||s| cos θ is negative F


Sol 3: K.E. of object 1 is KE1 = mu2
2 1 1 Examples:  V
1
K.E. of object 2 is KE2 = mu2 (1) Positive work f
2 2 2
KE1 = KE2 Work done by F is positive

1 1 Free falling object (Positive work)


⇒ m1u12 = mu2
2 2 2 2
mg
(m1u1 )2 (m2u2 )2
⇒ m1u12 = m2u22 ; = V
m1 m2
Work done by gravity is negative
Suppose m1 > m2 ⇒ m1u1 > m2u2
(2) Negative work
⇒ Linear momentum of 1st object is more than 2nd
F
object.
 V
Sol 4: Springs are generally taken as massless and f
spring transfers its potential energy to the kinetic and
energy of the body to which it is attached whereas Work done by friction is negative
during a free fall, PE of a body is converted into its KE Zero work
F
In other words, for a spring all the energy is stored as
potential energy whereas for body, potential energy is mg
converted into kinetic energy.

 N
Sol 5: work is the vector product F.s
 Work done by F is zero
w = F.s

F = force acting on the object

s = displacement of the object
If a constant force F displaces a body through F
displacements then the work done, w is given by
w = Fs cos θ
s = net displacement Work done by F is zero

θ = angle between force and displacement


Sol 8: Graphically work is the area under the force
displacement graph
Sol 6: Absolute unit of work on m.k.s is Joule (J)
Absolute unit of work on c.g.s. is erg F B
Gravitational unit of work on m.k.s. is kg-m
Gravitational unit of work on c.g.s is gm-cm A


Sol 7: Work done w = F.s
 
w = | F | | s | cos θ S1 S2 S
s2
θ = angle between force and displacement when 0 <  
θ, π/2
WA→Β = ∫ F.ds
s1

w = |F||s| cos θ is positive. Mathematically, work is the integral dot product of


when θ = π/2 ⇒ W = 0 force vector and infinitesimal displacement vector
B  
when π/2 < θ < π W = ∫ F.ds
A
P hysi cs | 5.45

Sol 9: Conservative force: It is a force with the property Example:


that the work done in moving a particle between two
(i) Energy stored in the compressed spring
points is independent of the taken path. Example
gravity (ii) When a rubber band is stretched potential energy
is stored in it.
Non-conservative force – it is a force with the property
that the work done in moving a particle between two
points is dependent on the path taken. Example friction Sol 14:
2 m
Properties of conservative force
h2
(i) Work from one point to another point on any path
h
is same
F
dFx dFy dFx dF2 dFy dF2
(ii) = ; = ; = m
dy dx dz dx dz dy
1 h1

Where F = Fx ˆi + Fy ˆj + F2kˆ Suppose a mass m is raised from point 1 to point 2
and suppose change in kinetic energy is negligible.
Properties of non-conservative force
Potential energy is negative of the work done
(i) Work done from one point to another is dependent
DU = - W
on path taken.
h2
(ii) Work cannot be recovered back W= ∫ mgds
h1
Sol 10: Power: It is defined as the rate at which the W = -mg (h2 – h1)
work is done SI unit of power is (J/s)
W = -mgh ⇒ DU = mgh
Energy: Energy of a body is the capacity of the body to
do work. SI unit of energy is J.
Sol 15: Potential energy of spring is the energy stored
in the spring when compressed or stretched relative to
Sol 11: It is the energy possessed by a body by virtue its natural length.
of its motion. A body of mass m moving with a velocity
v has a kinetic energy. Suppose a force F is acting on the spring of spring
constant k
1
Ek = mv2 To keep the spring compressed in this position, the
2
   dv 1 applied force should be same as kx
KE = w = ∫ F.ds = ∫ mads = m∫ v ds = mv2
ds 2 i.e. Fs = kx
Sol 12: When a particle is acted upon by various forces Work done in compressing by a distance x is given by
and undergoes a displacement, then its kinetic energy x
1
changes. By an amount equal to the total work done w = ∫ kx'dx' ⇒ w = kx2
2
wnet on the particle by all the forces 0
Work done is equal to change in potential energy of a
wnet = sk spring
wnet = wc + wNc + woth w=U
wC = work done by conservative force 1
U= kx2
wNc = work done by non-conservative force 2
Potential energy is zero at spring’s natural length and is
woth = work done by other forces which are not included
proportional to the square of the distance from mean
in above category
position

Sol 13: Potential energy – It is the energy of a body


Sol 16: Different forms of energy are
possessed by virtue of its position on the energy
possessed by the body due to its state (i) Kinetic Energy – It is the energy possessed by a body
5 . 4 6 | Work , Energy and Power

by virtue of its motion. So force is – 50]


(ii) Potential energy – It is the energy of a object on Work done by force f is
a system due to the position of the body and the 10 10
arrangement of the particles of the system. wf = wf1→2 + wf2→3 = ∫ F1dx + ∫ F2dx
0 0
Example: Gravitational potential energy –
10 10
 5x2 
(iii) Mechanical energy – it is the sum of potential = 100 × 10 + ∫ (100 − 5x)dx = 1000 + 100x − 
energy and kinetic energy 0  2 
0

wf = 1000 + [100(10-0) – 5/2 (10 -0)] = 1000 + 1000 – 250


2

Sol 17: Force f = 7 – 2x + 3x2  F


wf = 1750 J
Work done
5 x=0 x=5m
2 Applied force
w = ∫ F.dx = ∫ (7 − 2x + 3x )dx
100
0

5
= 7x − x2 + x3  = 7(5 – 0) – [52 – 02] + [53 – 03] 50 Friction force
  0

W = 35 – 25 + 125 = 135 N-m


10 20 x
So, work done is 135 N-m (m)

Sol 20: Mass = 50 kg = m


Sol 18: Mass = 2 kg
Momentum = 1000 kg m/s = mv
x = t3/3; dx = t2 dt
2
We need to find the force by first finding the acceleration 1 1 (mv) 1 (1000)2
K.E. = mv2 = = = 10000 J
of the body 2 2 m 2 50
dx d(t2 / 3)
v= = = t2
dt dt Sol 21: Mass = 50 g = 0.05 kg
dv Initial velocity before collision = vi = 400 m/∆
a= = 2t
dt
Final velocity after passing through the wall=vf=100m/s
F = ma = 2 × 2t = 4t
By work energy theorem, Work done by the bullet is
Work done equal to the negative change in kinetic energy of the
2 2 2
bullet
2 3
w = ∫ F.dx = ∫ 4t.t dt = ∫ 4t dt = t 4  = 24 = 16 J −1
  0 w = ∆KE = –KEf + KEi = × 0.05 [(100)2 – (400)2]
0 0 2
w = 16 J 5
w=– [1 – 16] × 100 = 3.75 × 103 J
2

Sol 19: Sol 22: By work energy theorem 


F=100N F=100-5x H
wnet = ∆KE
1 2 x 3 wgravity + wfriction = ∆KE
 10m 10m
1
mgH + wfriction = m (1. 44 gH – 0)
2
Work done by friction wf = ∫ Fdx
mgH + wfriction = 0.72 mgH
Since friction is constant all over the motion so
displacement = 20 m wfriction = (0.72 – 1) mgH = -0.28 mgH
wf = + f Dx = – 50 × 20
wf = – 1000 J Sol 23: Mass of bullet = 10 g = 0.01 kg
[∵ Direction of friction force is opposite to displacement. Initial velocity of bullet = 800 m/s
P hysi cs | 5.47

Thickness of mud wall = 1 m mg cosθ = 2mg (1 – cosθ)


Final velocity of bullet = 100 m/s 3 cos θ = 2
By work energy theorem 2 2
cosθ = ⇒ θ = cos–1  
1 3 3
Wnet = ∆KE = m [vf2 – vi2]F
2  2
And now h = R (1 – cosθ)= R  1 – 
1 100 2  3
wr = (0.01) [(100)2 – (800)2] = [1 – 82]
2 2 R
h = m.
w = –50 × 63 = –3150 N 3
So average resistance offered is 3150 N
Exercise 2
Sol 24: A
N Single Correct Choice Type
h B
R cosθ θ θ Sol 1: (C) Work done = ∫ Fx.dx + ∫ Fy.dy
R
O F = Fx î + Fy ˆj 
V a
2 3
Fx = –ky; Fy = –kx
a
mg w = w1-2 + w2-3
This is a very standard problem for a JEE aspirant. a a 1

Let us say at point B, the particle loses its contact. At ∫ Fydy + ∫ Fxdx
0 0
point B say the particle has velocity v. a a

mv 2 - ∫ K × 0 × dy + ∫ −kadx = 0 + -ka (a – 0) = –ka2


mg cosθ = N + 0 0
R
mv 2 Sol 2: (C) Work done is zero as force and
N = mg cosθ –  … (i)  displacement
R are perpendicular to each other so F.ds = 0
Now when the particle is about to lose contact, the
normal reaction between the particle and surface Sol 3: (B) By work energy theorem
becomes zero.
wnet = ∆KE[assuming negligible K.E.]
∴N=0
wgravity + wforce = 0
mv 2
⇒ mg cosθ = … (ii)
R   m  l  mgl
wforce = -  − g    =
Now energy at point A, taking O as reference;  4  8  32
EA = 0 + mg R kx0
1 Sol 4: (A) by force equilibrium 
EB = mv2 + mg R cosq
2
mg
Using Energy conservation kx0 = mg
x0
EA = EB 1 1 mg 2 mgx0
Elastic energy = kx02 = x =
1 2 2 x0 0 2
⇒ Mg R = mv2 + mg R cosq
2
v
2 mg R (1 – cosθ) = mv2 Sol 5: (C)

mv 2 u sin  u cos 
2mg (1 – cosθ) =  … (iii)
R h
u
mv 2 
Putting this value of in eqn … (iv)
R u cos 
x
5 . 4 8 | Work , Energy and Power

Vertical velocity at height h is vy Sol 9: (C) Mechanical energy conservation


vy = u sin θ – gt
KEalpa + PEi + KE = PEelectrostatic
x
t=
ucos θ 9 × 1010 × 10−19 × 100 × 10−9
4 ×106 × 10-19 =
gx r
vy = u sin θ –
ucos θ r ~ 10-14 m
1 1  g x  2 2
KE = m[vy2 + vx2] = m u2 + − 2 tan θgx  r ~ 10-12 cm
2 2 
2 2
u cos θ 
so graph (c) is correct Sol 10: (D) Minimum velocity required is v= 5gR
by Newton’s second law 
Sol 6: (B) v1 v2
mv 2 T
T – mg =
 m1 m2 o
45 R

m5gR
2 2 T = mg + = 6mg mg
u sin θ R
Maximum height H =
2g
2 Sol 11: (C) F = 360 N  36
o

 1 
2
v 22   Work done is 1 hr 60
o
v1  2 v 22
H1 = ; H2 = =
2g 2g ug is w = 360 cos 60° 10 × 100

H1 = H2
3600 × 103 cos60°
Work done per sec=
v 12 v 22 60 × 60
⇒ =
2 4 1
= 1000 = 500 w
v2 = 2v1
2 2 2

KE1 1 / 2m1 v12 v 12 1 Sol 12: (B) By work energy theorem


= = =
KE2 1 / 2m2 v 2 2
v2 2 2
wnet = ∆KE =0 
wman + wgravity = 0 H mg
Sol 7: (C) By work energy theorem
H
wnet = ∆KE [Assuming negligible K.E.] wman – MgH - mg =0
2 Hg
wgravity + wforce = 0
m 
wman =  + M  gH
 1   1  4 2 
Wnet =  − mg  −  − mg  = mg
 18   2  9
Sol 13: (C) b sin 
Sol 8: (C) P = F.V
By force equilibrium
P = ma.V
f = mg sin θ
 mdV 
P=  V Work done by friction f is equal mg mg
 dt  to f sin θ vt
t v
Pdt p v2 w = mg vt sin θ
∫ m
= ∫ vdv ⇒
m
=
2
0 0
L
1 Sol 14: (D) m m
m
2Pt +1
v= ⇒x∝ t2
m
x ∝ t3/2 m m

x/2 x/2
P hysi cs | 5.49

Potential energy of spring is ½ kx2 Sol 2: (D)


v

By work energy theorem


wnet = ∆KE
wforce + wspring = 0
−1
wspring = kx2
2
u
Since displacements of both the masses are same so
work done by spring on both masses is same. From energy conservation v2 = u2 – 2gL

−1 Now, since the rwo velocity vectors shown in figure


So work done on each mass = kx2 are mutually perpendicular, hence the magnitude of
4
change of velocity will be given by

Sol 15: (B) F = kx |∆ v | = u2 + v 2
kx2 Substituting value of v2 from eq. (i)
U = – ∫ Fdx = − ∫ kxdx ⇒ U = – +c
2 →
|∆ v | = u2 + u2 – 2gL = 2(u2 – gL)
U(x = 0) = 0 ⇒ 0 = 0 + c ⇒ c = 0
−kx2 → →
U= Sol 3: (C) Power = F . v = Fv
2
 dm   d(ρ × volume) 
Sol 16: (B) w1 = w2 = w3 as gravitational force is F = v  =v  
conservative and work done by conservative forces is  dt   dt 
independent of path taken.
 d(volume) 
= rv   rv(Av) = ρAv
2

 dt 
Sol 17: (B) By work energy theorem
wnet = ∆K ∴ power P = ρAv3 or P ∝ v3

h is the maximum extension of the spring


Sol 4: (B) Let x be the maximum
wgravity + wspring = 0 extension of the spring. From
conservation of mechanical energy
1
+mgh – kh2 = 0
2
Decrease in gravitational potential
2mg energy = increase in elastic potential v=0 M
h=
k energy
x
1
Sol 18: (A) Work energy theorem includes all the ∴ Mg x = kx2 v=0 M
forces. Conservation as well as non-conservative. This 2
theorem is always true. 2Mg
or x =
k

Previous Years’ Questions Sol 5: (C) From energy conservation,


1 2 1
Sol 1: (C) p = 2Km kx = (4k)y2
2 2
m1 1 y 1
or p ∝ m, = =
m2 4 x 2
p1 1
∴ =
p2 2 Sol 6: (A) F = k1S1 = k2 S2
5 . 5 0 | Work , Energy and Power
2
1  m 2gh 
W1 = FS1, W2 = FS2 mgh + wfriction = (M + m)  
2  M+m 
k1S12 > k2S22  
m2gh
S 1 > S2 mgh + wfriction =
M+m
k 1 < k2 m2gh
wfriction = – mgh
W∝k M+m
W 1 < W2 Mmgh
wfriction = –
(M + m)
Sol 7: (C) Let fat used be ‘x’ kg
Sol 2: A
20
⇒ Mechanical energy available = x × 3.8 × 10 × 7
100
Work done in lifting up = 10 × 9.8 × 1000
20
⇒ x × 3.8 × 107 × = 9.8 × 104
100 Drawing FBD at point A
⇒ x ≈ 12.89 × 10-3 kg. mv2 /R

JEE Advanced/Boards

Exercise 1 mg
mv 2
Sol 1: By work energy theorem for point A to point B So net upward force exerted by the mass is – mg
R
A mv 2
m Which is equal to 3 mg ⇒ - mg = 3 mg
R
mv 2
= 4 mg
R
h
v= 4gR = 2 gR
B M
Now applying work-energy theorem
wnet = ∆KE
wnet = ∆KE wgravity = Kf
wg = KEf – KEi 1
mg(h-2R) = m (4gR)
1 2
mgh = mv2
2 h – 2R = 2R
v= 2gh h = 4R
Now m, M both move together so by conservation of
linear momentum 1
Sol 3: Mass = kg
2
mv = (M + m) v’
m 2gh k
v’ = m
M+m
3m/s
v’ is the combined velocity of (m + M) system.
A B D D’ C
Applying work energy theorem for the whole process x
2m 2.14 1m
wnet = ∆KE
Let us assume that block stops at point D’ which is at
wgravity + wfriction = kEf – kEi (kEi = 0) distance x m from D.
P hysi cs | 5.51

By applying work energy theorem


wnet = ∆KE u2 sin2 θ
Maximum height attained =
wfriction + wspring= Kf – Ki  … (i) 2g

(Kf =0) 9.8 × 5 × 0.6 × (0.8)2


= = 0.97 m
wfriction = –µ mg (BD’) 2 × 9.8

wfriction = –(0.20) mg (2.14 + x)  … (ii)


Sol 5: At maximum compression, velocity of both
1
wspring = − kx2 … (iii) blocks will be equal so, let v be the final velocity of both
2 the blocks x be the compression in the spring. Applying
[ block is in motion from point B to D’ so we will take moment conservation, we get
kinetic friction]
2 × 10 + 5 × 3 = (2 + 5) V
Substituting (i), (ii) and (iii) we get
35
⇒v= = 5m/s
1 1 7
(0.2) mg (–2.14 – x) – kx2= – m (g)
2 2 Applying work energy theorem
Substituting value of m we get
wnet = ∆KE = KEf – KEi
1 9
2.14 + x + × 2x2 = + 1 1 1 1 
2 4 – kx2 = (5+2) (5)2 -  (5)(3)2 + (2)(10)2 
2 2 2 2 
x2 + x + 2.14 – 2.25 = 0
1 45 25 × 7
x2 + x – 0.11 = 0 ⇒ kx2 = 100 + –
2 2 2
(x – 0.1) (x + 1.1) = 0
x2 = 0.0625
x = 0.1 m
x = 0.25 m
So, total distance thought which block moves is 2 +
2.14 + 0.1 m ⇒ 4.24 m
Sol 6: P = Power = F.V = constant
mdv
2
Sol 4: mv
Pseudeo (a) F = ma =


R dt
os

force
gc

mdv
m

T g mg sin  P= .v
5m  dt
3
cos  =
3m 5 P
∫ vdv = ∫ m dt

v t
P
5m
∫ vdv = m ∫ dt
0 0

[ p and m are constant]

When particle is at 8m height from lowest point tension v2 pt


=
is just zero. So balancing force in the direction of string 2 m
mv 2 2pt
We get mg cos θ – =0 v=
R m
gR cos θ = v2
mvdv mvdv
v= gR cos θ (b) F = ma = ⇒P= v
dx dx
Substituting values we get v s
2 Pdx
v= 9.8 × 5 × 0.6 ⇒ 5.42 m/s  ∫ v dv = ∫ m
 O 0 0


5 . 5 2 | Work , Energy and Power

v3 ps 9 5
= h1 = × = 1.5 m
3 m 10 3
3/2 (b) Force equilibrium on M
mv 3 m  2Pt  0.8m
s= =  
3P 3P  m  T = Mg 
x
1/2 Force equilibrium on m T
 8P  T
s=   t3/2
T sin θ = mg
 9m  M Mg
Mg sin θ = mg
d 5d Mg
Sol 7: Length of the spring at point A = = M 3
cos37° 3 sin θ = =
By work energy theorem m 5

wnet = ∆KE Sol 9: M “falls” and loses potential energy. This loss
of potential energy is converted to gain in potential
1 1
mgh
 + 2 k (x -0 ) = 2 mv
2 2 2
energy of m and gain in kinetic of energy for m and M
wgravity both.
d M
[x = l – d = ] 1 1
4
5d 3 3d
h = l sin θ = . =
4 5 4 1
5 1-2
3g k h
v=d +
2d 16m
m
Sol 8: (a) When mass m comes to rest for the first time
kinetic energy of both the masses is zero.
Let the total length of the string be l. So, the length of
work energy theorem the hanging part in the beginning = l-2.
wnet = ∆KE = KEi – KEi Since, total mechanical energy is conserved.
–Mgh2 + mgh = 0 ⇒ Mh2 = mh1 Loss in M.E. = Gain in M.E.
5 1 1
⇒ h1 = h  … (i) Mg1 = mv 2 + mgh + MV 2 ........ ∗
2 2 2 2
Length of the string is constant, so h can be obtained from the conservation of the length
of the string.
BC + Ac = A’C + B’C  0.8m
C
BC – B’C = A’C – AC  A h = l − 2 − (l− 5 − 1) = 5 − 1
h2 + 0.8 = A’C h1
We want V, v can be obtained in terms of V.
(0.8)2 + h12 B’ M mA’
As M “falls”, it moves in circular path with its velocity
= (h2 + 0.8)2 h2 along the tangent. The velocity along the tangent can
B be resolved into two components, one along the length
h12 = h22 + 1. 6 h2
of the string and the other perpendicular to the length
By (i) of the string. The component along the length of the
25 2 string is same as the velocity of m as m always moves
h = h22 + 1. 6 h2 along the length of the string.
9 2
V cos θ = v
16h22
= 1.6 h2 2
9 cos θ =
5
9
h2 =
10
P hysi cs | 5.53

From, x + 0.4 = 5x
1
∗, 2 × 9.8 × 1 = × 0.5 × V 2 cos2 θ + 0.5 × 9.8 ×
2
( 5 −1 + ) 1
2
× 2V 2
x = 0.1 m
By work energy theorem
V can be obtained.
1 1
w= mv2 + mv2
Sol 10: Work energy theorem 2 2

wnet = ∆KE 1
- × 40 (0.1)2+0.32×10×0.3=0.32 v2
2
Since initial and final velocity of sand particles are zero
so ∆KE = 0 v = 1.54 m/s
wgravity = mg (0.2 + 0.01)
Sol 12:
wnet = 0 P
wgravity + wspring = 0

s 
1

co
(0.1) × (10) × (0.21) – k (0.1)2 = 0

u
2 B
 u
A V
k = 0.42 × 104 = 4200 N/m
u sin 
Now if compression is 0.04 m
wgravity + wspring = 0 (a) Velocity of A along the string is equal velocity of B
along the string
1
⇒ (0.1)×(10)(h + 0.04) – × 4200 (0.04)2 = 0
2 Length of string is constant
h + 0.04 = 2100 × 16 × 10 -4
⇒ AP + BP = constant
h + 0.04 = 0.21 × 16 Differentiate w.r.t
h = 3.36 – 0.04 = 3.32 m d(AP) d(BP)
+ =0
dt dt
Sol 11: Let the extension kx
-u cos φ + v = 0
v = u cos φ

 kx N (b) When B strikes ground length
0.4
BP = 6 cm
m T
So length of AP = 16 – 6 = 10 m
mg
6 3
sin φ = = 
10 5
m

By Newton’s second law 


10

 6m
x

4 v 5v
+0

h = 0.3 m 0.4 cos φ = ⇒u= = 8


.4

5 4 /5 4
N – mg + kx cos θ = 0
By work energy theorem
N=0
h wnet = ∆KE
kx cos θ = mg … (i) 1 1
wgravity + wstring =
mv2 + mu2 - 0
By geometry (x + 0.4) cos θ = 0.4 2 2
1 1  25v 2 
0.32 × 10 3.2 mg × 5 = mv2 + m  
By (i) x cos θ = = =0.08 2 2  16 
40 40  
0.08 25 2
(x + 0.4) = 0.4 10 g = v2 + v
x 16
5 . 5 4 | Work , Energy and Power

10g × 16 wf + wgravity + wfrictoin = 0


v2 =
41 wf – mmgl – mgh = 0
wf = µ mgl + mgh
10g × 16 40
v= = m/s
41 41
Sol 16: At any point of time, let the length of chain
remaining in tube be x
Sol 13: By work energy theorem [Velocity of both x
blocks is same] A
wnet = ∆KE
1 1 h
wfriction + wgravity = m1 u2 + m u2
2 2 2
1
–mm1 g L + m2 gL = (m1 + m2) u2
2
m
2(m2 − µm1 )gL m’ = mass of chain above ground = (h + x)
u= 
m1 + m2 Now by Newton’s second law on chain of length (x + h).
As length of chain which has fallen on ground has no
effect on the upward chain.
Sol 14: Since αc = k2rt2
F = m’a
v2 m  m(h + x)
= k 2rt2 ⇒ v 2 = k 2r 2 t2
r  h g = a
   
dv
⇒  v= krt ⇒ F = m = mkr mhg m(h + x)  vdv 
dt = − 
   dx 
Power = F.v = (mkr)×(krt) = mk2r2t
[ x is decreasing with increase in length]
0 v'
Sol 15: hg
∫ −
h+ x
dx = ∫ vdv
 −h 0

O [v’ is the velocity of the end]


mg sin  v'
mg cos  0 v2 
mg − hg log(h + x) = 
 −h 2 
0
ds
h v '2
 d – hg log   =
ds =  2
cos 
d

v’2 = 2hg log  
At any point on the path frictional force f = µ mg cos θ h
θ = angle between path and the horizontal surface at
some point 
v’ = 2hg log  
wfriction = − ∫ (µ mgcos θ) ds h

 m
d Sol 17: x
= − ∫ µ mgcos θ = – ∫ µmgd = −µ mg  m
cos θ m
mg/k
1
wgravity = – mgh
By work energy theorem h1
wnet = ∆KE
m m m m
P hysi cs | 5.55

Let the natural length of spring be l1 We will find the velocity of block B when block A will
just lift upwards
Initially there is same compression x in spring in 2 2
equilibrium 1  7mg  1  mg  8mg 1
k  – k  – mg = mv2
2  k  2  k  k 2
m
32m2g2
T=kx’ v2 =
k
Now block A and B together form a system with
mg v
acceleration – g, Vcm =
mg 2
mg = kx’ ⇒ x’ = So,
k
v2 = u2 + 2as
Now it is further compressed by Dl1 by thread
O = vcm2 – 2gs
Now if thread is burnt it will go at upward extreme
v2 = 8gs
which is x distance above natural length of spring.
Spring will just lift the lower block so by newton 2nd v2 4mg
law; T = kx = mg s= =
8g k
mg
x= Movement of centre of gravity
k
By mechanical energy conservation  8mg  m 4mg 8mg
=   + = upwards
 k  2m k k
PEspring + PEgravity = PE’gravity + PE’spring v
2
Sol 18: (a)
1  mg 
k  ∆ +  + mgh1
2  1 k  W
2
 2mg  1  mg 
= mg  + ∆1 + h1  + k
 k  2  k  By Newton’s second law
  mg 
2
mg  w
1 
k ∆12 +   +2 ∆1  (w + f) =   a
2   k  k  g
 
2
2(mg) 2
1  mg   f  vdv
= +mgDl1+ k   a = – g 1 +  =
k 2  k   w dx
2
1 2(mg) 0 s
 f 
k Dl12 =
2 k ∫ vdv = – ∫ g  1 + w  dx
v0 0
2mg
Dl1 =
k v 02  f 
To lift block of mass m 0– = – g 1 +  s
2  w 
mg
D > Dl1
k v 02
s=
3mg  f 
D > 2g  1 + 
k  w 
(b)
B Final velocity = v
mg
8mg (b) By work energy theorem
k
k wfriction + wgravity = ∆KE
7mg
k 1 −4fs
–2fs + 0 = m (v2 – v02); v2 – v02 =
A 2 m
5 . 5 6 | Work , Energy and Power

By (i) and (ii)


−4b v 02
v2 = + v02
m  f  g = 5gr
2g  1 + 
 w
1
[ −4f + 2f + 2w] [w − f] r= m
⇒ v = v0
2 2
[mg = w] = v2 = v02 5
2(w + f) [w + f] 1
x = (1 - ) m = 0.8 m
1/2 5
w−f
v = v0  
w+f Sol 21: A
D
Sol 19: Let the speed of ring is v
V
H  C’
h 5h
Length of spring = = d 
cos37° 4
L cos 
By mechanical energy conservation
PE1 + KE1 = PE2 + KE2 A
2 L sin 
1  5h  1
K  − h + 0 = 0 + mv2
2  4  2
L

1 h2 1
K. . = mv2 u
2 16 2
C
k h
v= B
m 4

Sol 20: v 2 sin2 θ


H=  
x 2g
o
60 mg
1m

By Newton’s second law


nail mv 2
mg cos θ =
r R
v2 = g cos θ  … (i)
x+r=1
Let us assume that pendulum leaves circular motion at
For velocity of pendulum when string becomes vertical
point C with velocity v making an angle θ with horizontal
is v
Applying work energy theorem from point S to C’
Work energy theorem
wnet = ∆K
wgravity = KE
1
m/g (1-cos60°) = ½ mv2 wgravity = m (v2 – a2)
2
1 1
v= 2g × -mgr (1 + cosθ) = m (v2 – u2)
2 2

v2 = u2 – 2g (1 + cos θ)  … (ii)


v= g m/s …(i)
From point C’ to D it will follow parabolic path and
For circular motion to be just completed
velocity at line AB is horizontal
2
v= 5gr  … (ii) L = v sin2θ
L sin θ - 2g … (iii)
4 
P hysi cs | 5.57

By (i) and (ii) 1 2 3mgR


kx =
u2 = g (2 + 3 cos θ) 2 2x
By (ii) and (iii)
x= 3mgR
u = 2.14 g k

Sol 22: N Sol 24:


o
 60
V
1m T
mg

o
By work energy conservation mg sin60 o
mg cos60
u
w1 = ∆KE 0.1kg mg
1
mg R (1 – cos θ) = mv2 mg
2
mv 2 (a) By newton’s second law
2mg (1 – cos θ) =
R
mv 2
By newton’s 2nd law T – mg cos60° =
R
Mg cos θ – N = 2 mg (1 – cos θ) mg m
T= + (4)= (4.9 + 4) m
N = mg (3 cos θ – 2) 2 R

For θ ≤ cos-1 (2/3) ; NB=0, = 8.9 × 0.1 = 0.89 N = 8.9 × 104 dyne

NA = mg (3 cos θ – 2) (b) By work energy theorem

For θ ≥ cos-1 (2/3) NA = 0, wnet = DkE

NB = mg (2 – 3 cosθ) NA mg (1 – cos60°) = u2 – v2


mg u2 = v2 + mg (1 – cos 60°)
 1
cos 0 = 4 + 0.1 × 9.8 × 1 × = 4 + 0.49
2/3 1
2
-1 NB
5mg u2 = 4.49
2mg
u = 2.12 m/s
cos 0
-1 2/3 1
Sol 25:
w

Sol 23: Let the initial compression be x


x

mg
+

mg
m

By Newton’s second law F = ma


mv 2 By Newton’s second law
mg =
R Kx = mω2 ( + x)
By work energy theorem mω2 
x=
wnet = ∆K k − mw2
1 mω2
wspring + wgravity = mv2 – 0 Let x’ =
2 k
1 1 x' 
kx2 – mgR = mgR x=
2 2 1 − x'
5 . 5 8 | Work , Energy and Power

By work energy theorem (c) If the projection speed is slightly greater than u0,
then speed at top most point is just than zero.
wnet = ∆KE
1
wspring + wforce = m ω2 ( + x)2
2

−1 1
kx2 + wforce = mω2 ( + x)2 N V’
2 2
mg
1 1
wforce = kx2 + mω2 ( + x)2
2 2 Particle will lose contact when normal just becomes
zero.
2
1 x'2 2 1  x'   So by Newton’s second law
= k + mω2   + 
2 (1 − x')2 2  1 − x'  mV'2
mg cos θ =
 2  mω2   R
k 2
= x +   v’2 = g R cos θ
2(1 − x')2  k 
  
By work energy theorem, wnet = ∆KE
2
k 1 1
wforce = [x’2 + x’] mg R (1–cos θ)= mv’2= mg R cos θ
2(1 − x')2 2 2
2(1 – cos θ) = cos θ
k2 x'(1 + x') mω2
= where x’ = 2 – cos θ = - cos θ
2(1 − x')2 k
2
cos θ =
3
Sol 26: (a) Minimum speed is required so in the limiting So it will lose contact when particle makes an angle
case velocity of block at highest point is zero 2
cos-1   with vertical.
By work energy theorem 3
wnet = ∆KE 
Sol 27: (a) By work energy theorem
1
wgravity = 0 - mu02 wnet = ∆KE
2
1 1
– mg [L sin θ+R (1–cos θ)]= − mu02 mg (5R – R) = mv2
2 2
v2 = 8gR
u0 = 2g[L sin θ + R(1 − cos θ)]
By Newton’s second law force exerted in horizontal
(b) Let the final velocity be v at top point mv 2 8mgR
direction = = = 8mg
1 R R  8mg
wgravity = mv [v2 – 4 u02]
2
Net force = 82 + 1 = 65 mg
1 mg
–mg [L sin θ + R (1 – cos θ)] = m[v 2 − 4u02 ]
2 By Netwons second law
v2 = 3u02 mv 2
Force = mg
By Newton’s second law
R
mv 2
2 mg + mg = ⇒ v2 = 2gR
mv 2 m(3u0 ) R
Force = ma= = mg
R R Let the height be h
3m By work energy theorem
= [2G(R(1-cosθ) + L sin q]
R 1
mg(h – 2R) = m 2gR
L 2
= 6 mg [1 – cos θ + sin q]
R h – 2R = R ⇒ h = 3R
P hysi cs | 5.59

Sol 28: (Coming to a stop) A particle can slide along


a track with elevated ends and a flat central part, as d(AP1 ) +d(P1P2 ) d(P2P3 )
shown in the figure below. The flat part has length + =0
dt dt dt
L = 40 cm. The curved portions of the track are
frictionless, but for the flat part the coefficient of kinetic –VA + VB + VB cos θ = 0
friction is μk = 0.2. The particle is released from rest at
VA = VB (cos θ + cos θ)
point A, which is at height h = L/2. How far from the
left edge of the flat part does the particle finally stop? 6
VA = v
A 5 B
h
Now, by work energy theorem

L wnet = ∆KE
wgravityA + wgravityB = ∆KE  …..(i)
The initial energy is mgh = mgL/2. On the level ground,
the particle experiences a constant friction force f = μk Initially length of string between P1 and P3 is 8m
N = μkmg. It will stop once the work W = -fs done by Finally length of string between P1 and P3 is 10 m so A
friction has dissipated all the initial energy: has moved (10 – 8) upward
L By (i)
mgL / 2 ==
fs µk mgs ⇒ s= = 100 cm
2µk
m 1 1 m 2
– g × 2 + mg × 3 = mvB2 + v
So the particle will make two full passes (one moving 2 2 2 2 A
right, one moving left) over the flat area, then stop
1  36 VB 
2
halfway across (20 cm from the left edge) on its third mg [3 - 2]= m  VB2 + 
trip. 2 25 2 
 

(3 − 2)g × 2
Sol 29: P1 VB = ≈ 4m/s
4  36 
P3 1 + 
 25 2 

3 x
V2 (b) Velocity of A and B is zero at maximum displacement
AVA

4 4

m
P2
2  h
x x2 + 42
VBcos NBcos
VB x +4
2 2

3 h= x2 + 42 - 8

By work energy theorem
m
– gh + mg x = 0
3 2
cos θ =
5 h= 2 x
4
sin θ =
5 2 x2 + h2 - 8 = 2 x
(a) Length of string is constant
x= 8 2 m
⇒ AP1 + P1P2 + P2P3 = 0
5 . 6 0 | Work , Energy and Power

Exercise 2 Sol 6: (A) Minimum speed must be zero as it is


connected to rod so it will not leave the circular motion
Single Correct Choice Type at any point in the path

Sol 1: (B) Potential of water after falling down will Sol 7: (B)
convert in heat and sound. So temperature will increase

w
slightly.

x
+
Sol 2: (A) By work energy theorem
wnet = ∆KE

m
wgravity + wspring = 0 By Newton’s second law
1 Kx = mω2 ( + x)
mg (0.4 + x) + - kx2 = 0
2
mω2 
20 × 0.4 + 20 x – 1000 x = 02 x=
k − mω2
1000 x – 20 x – 8 = 0
2

mω2
Let x’ =
20 ± 400 + 32000 k
x=
2000 x' 
x=
1 − x'
20 ± 180 200 1
x= = = = 0.1 m By work energy theorem
2000 2000 10
wnet = ∆KE
−dv
Sol 3: (C) F – 1
dR wspring + wforce = m ω2 ( + λ)2
2
k   −k  k
U = – ∫ FdR = –  dR  = –   = −1 1
R 2
 R  R kx2 + wforce = mω2 ( + x)2
2 2

Sol 4: (C) Mechanical energy is ME = KE + PE 1 1


wforce = kx2 + mω2 ( + x)2
2 2
Maximum potential energy is 160 J when
2
Kinetic energy is zero i.e. at end points. 1 x'2 2 1  x'  
= k + mω2   + 
2 (1 − x')2 2  1 − x' 
Sol 5: (B) u = 2x4 – 27 x
k 2  2  mω2  
−dU = x +  
F= 2(1 − x')2   k 
dx  

F = –[8x3 – 27] k 2
wforce = [x’2 + x’]
at x = 3/2 2(1 − x')2
Force is zero
k2 x'(1 + x') mω2
= where x’ =
 3+  2(1 − x')2 k
f   = -ve
2 
  v2
Sol 8: (B) ac = k2 rt2 =
 3−  R
F   = +ve v = krt
2 
  dv
So this is stable equilibrium at = = kr
dt
Power P = F.V= mat.v = mkr. Krt = m k2 r2 t
P hysi cs | 5.61

Sol 9: (D) Tension is zero as can and pendulum are Sol 13: (D)
falling freely under gravity 
u1 u2
Sol 10: (D)
O1 O2
f(x) = -kx + ax2
U(x) = ∫ −f(x)dx u1 + u2 = V
 kx2 ax 4  kx2 −ax 4
U(x) = -  − +  +c= +c Acceleration will be same for both as acceleration of
 2 4  2 4 both observers is zero.

It corresponds to graph (D) for c = 0 Kinetic energy will be different so by work energy
theorem work done will also depend on kinetic energy.
Sol 11: (C) Power = F.v
Force = rate of change of linear momentum of wind Multiple Correct Choice Type

dm v
= ρAV where ρ= density Sol 14: (A, D)
dt v’
A = area of blades mg

V = velocity By Newton’s second law T


2
d(m.v) dm mv
F= =v = v ρAV T + mg =
dt dt 

F = ρAV2 T = 0 at highest point

Power = ρAV3 So string becomes slack at the highest point.

P ∝ v2
Sol 15: (B, D)

Sol 12: (B)  T KE is increasing


F
⇒ Velocity is increasing
By force equilibrium
T ⇒ Resultant force must be at an angle less than 90° so
F=T that a component of force in the direction of velocity
M a
By Newton’s second law will increase its velocity
mg
T – mg = ma ⇒ Linear momentum is increasing
T = m (g + a)
Sol 16: (A, B)
By work energy theorem −1
In (C) and (D) work done by spring is kx2 but in (A)
wnet = ∆KE 2
+1
and (B) work done is kx2
wtension + wgravity = 20J 2
 
Sol 17: (A, C, D) Work done = F.ds
wtension = 120 – wgravity
(A) If force is always perpendicular to velocity then,
 
wgravity = –Mgh = – Mg v × 1 F.ds = 0
(B) If there is some initial velocity in the direction of
40 force then, work done can be non-zero
= – Mg =–g 40M
M (C), (D) Work done depends only on the displacement
1 of point of application of force.
KE = 20 J = Mv2
2
40
v=
M
5 . 6 2 | Work , Energy and Power

Sol 18: (C, D) Work done = 0 so kinetic energy is Sol 26: (D) There is no friction and non-conservative
constant force so mechanical energy is conserved.
Since in velocity and acceleration, direction is changing
so they are not constant. Sol 27: (B) As there is sliding at t =0 so friction will act
opposite to the direction of velocity
Sol 19: (A, B) By work energy theorem v cm
Sol 28: (B) ω >  3VCM
wnet = ∆KE R
R
1
mgh = m (vb2 – vi2) VCM
2
So final velocity is larger than initial and will depend on
speed of projection. So friction will act in the direction of velocity to increase
the velocity and decrease the angular acceleration
Sol 20: (A, B, D) By work energy theorem
wnet = ∆KE = 0 Sol 29: (C)

wyou + wgravity = 0

60 o
R 3R
wyou = -wgravity = + mgh
o
N
30
Assertion Reasoning Type 30
o
R/2 R
o
30 o
Sol 21: (B) Force has to be zero 30

Sol 22: (A)  In pushing In pulling kx 30 o mg


N N
F F
  Since acceleration of bead in the normal direction is
zero. So by Newton’s second law
mg mg N – kx cos 30° = mg cos 60°
N=mg + Fsin  N=mg - Fsin 
mg kx 3 mg 3 (2 + 3)mg
N= + = + (2 – 3)R
  2 2 2 2 3R
Sol 23: (A) U = - ∫ F.dr mg mg
= + = mg
Assume a closed loop in space 2 2

Since F is a conservative force, the line integral is zero.
Sol 30: (D) Newton’s second law in direction perpendicular
Thus U is a state function.
to normal
So potential energy is defined only for conservative
(i) mg cos 30° + kx cos 60° = ma
force.
mg 3 (2 + 3)mg 1
+ (2– 3 ) × = ma
Sol 24: (B) By work energy theorem wnet = ∆KE 2 3R 2
1 1 9/2 9/2
F.DS = mv2 ; v ∝ mg 3 mg
+ = ma; 
2 m 2 2 3 60
o
60
o

a a
Comprehension Type mg  1 
 3+  = ma
2  3
Sol 25: (A) Work done in raising box = –wgravity
= –(–mgh) = mgh 2g
a=
3
1 – false
a a 2g
From figure relative acceleration is + =a=
2 2 3
P hysi cs | 5.63

Sol 31: (C) By mechanical energy conservation By Newton’s second law


PEi + KEi = PEf + KEf T – mg = ma;T = m(g + a)

mgR 1 1  g  11g
+ kx2 + 0 = 0 + mv2 T = 72  R +  = × 72 = 72×11 = 792
x 2 2  10  10

mgR (2 + 3)mg Work done w1 = 792 × 15 = 11880


+ (2 – 3 )2 R2 = mv2
2 3R By work energy theorem
gR  2  wstring + wgravity = ½ mv2
v2 = +  − 1  gR
2  3  11880 – 10800 = KE

2gR KE = 1080
v=
3 KE = ½ mv2 = 1080
v = 5.47 m/s
Sol 32: (D) (A) Wrong, collision can be inelastic
(B) In perfectly inelastic collision energy is not conserved Previous Years’ Questions
(C) For SHM, θ should be small.
(D) At the instant of collision, they are at the bottom Sol 1: Given t = x +3

⇒ ΣF = 0 and ΣM = 0 or x = (t – 3)  …… (i)

⇒ Momentum conserved ∴ x = (t – 3)2 = t2 – 6t + 9 …….(ii)


Differentiating this equation with respect to time, we
get
Match the Columns
dx
Velocity v = = 2t – 6 ……. (iii)
Sol 33: A→s; B→q; C→r; D→p dt
(a) v = 0 when 2t – 6 = 0 or t = 3s
Total mechanical energy is conserved
Substituting in Eq. (i), we get
U(x) + KE = constant
x = 0 or x = 0
At x = 5
i.e., displacement of particle when velocity is zero is
U(x) = 29
also zero.
KE = 20
(b) From eq. (iii) speed of particle
Total ME at any x = 49
At t = 0 is vi = |v| = 6 m/s
maximum P.E. = 49
At t = 6 s is vf = |v| = 6 m/s
maximum K.E. = 49 – Umin = 49 – 20 = 29
From work energy theorem,
When U(x) is maximum then x will take extreme values.
Work done = change in kinetic energy
20 + (x – 2)2 = 49
1 1
(x – 2)2 = 29 = m[ v 2f – v i2 ] = m[(6)2 – (6)2] = 0
2 2
x – 2 = + 29 ; x – 2 = − 29
x = 7.38 ; x = –3.38 Sol 2: s = vt = 2 × 5 = 10 m
Q = work done against friction
Sol 34: (A→r; B-p, C-q, D-s)
T = mmgs = 0.2 × 2 × 9.8 × 10 = 39.2 J = 9.33 cal
Work done by gravity 15m
m
w = +mgh = 0.72 × 10 × 15 Sol 3: Normal reaction between blocks A and C will be
mg
w2 = 10800 zero. Therefore, there will be no friction between them.
5 . 6 4 | Work , Energy and Power

Both A and B are moving with uniform speed. Therefore


or (2)(9.8)(1) = (0.5)(9.8)( 5 – 1)
net force on them should be zero.
T = kx 2
1 1  2v 
+ × 2 × v2 + × 0.5 ×  
2 2  5
B T Solving this equation, we get v = 3.29 m/s
mBg

T Sol 5: (a) At the highest


kx point, velocity of bullet is
50cosθ. So, by conservation O
of linear momentum
A
M(50 cos θ) = 4 Mv

∴ v =  50  cos θ  … (i) 50
cos 
m Ag   v=
 4  A 4
For equilibrium of A
mAg = kx At point B. T = 0 but v ≠ 0
mA g (2)(9.8) (4M)v 2
∴x= = = 0.01 m Hence, 4 Mg cos 60° =
k 1960 
g 50
For equilibrium of B mmBg =T= kx = mAg or v2 = = … (ii)
2 3 
mA 2
mB = = = 10 kg  10 
µ 0.2 = as  = m amd g 10m / s2 
 3 
Energy stored in spring
3   10 
1 2 1 Also, v2 = u2 – 2gh = u2 – 2g    = u2 – 3(10)  
U= kx = (1960) (0.01)2 = 0.098 J 2   3 
2 2
or v = u – 100
2 2

Sol 4: Let M strikes with speed v. Then, velocity of m at or solving eqs. (i), (ii) and (iii), we get
2 cos θ = 0.86 or θ = 30°
this instant will be v cos θ or v. Further M will fall a
5
distance of 1 m while m will rise up by ( 5 – 1) m. From Range 1  u2 sin2θ 
(b) x = =  
energy conservation: decrease in potential energy of M 2 2  g 

= increase in potential energy of m+ increase in kinetic
energy of both the blocks. 50 × 50 × 3
= = 108.25 m
1m 2 × 10 × 2
u2 sin2 θ 50 × 50 × 1
2m y=H= = = 31. 25 m
2g 2 × 10 × 4
M
 Hence, the desired coordinates are (108.25 m, 31.25 m).

1m v cos 
5m Sol 6: Let the string slack s at point Q as shown in
v
figure. From P to Q path is circular and beyond Q path
is parabolic. At point C, velocity of particle becomes
( 5- 1)m horizontal therefore. QD = half the range of the
M projectile
Now, we have following equations
5
mv 2
1 (1) TQ=0. Therefore, mg sin θ =  ... (i)
L

2
P hysi cs | 5.65

C v Net pulling force 0.72g – 0.36g g


90 - 
o Sol 7: a = = =
Total mass 0.72 + 0.36 3
D Q

1 2 1 g 2 g
mg S= at =   (1) =
2 2 3 6


L L + L sin 
B
L

P u T
a

L cos  T a
0.36 kg
(2) v2 = u2 – 2gh = u2 – 2gL(1 + sinθ) … (ii)
0.72 kg
1
(3) QD = (Range) 0.72 kg
2
 L  v 2 sin2(90° – θ) v 2 sin2θ g
⇒  L cos θ –  = =  … (iii) T – 0.36 g = 0.36 a = 0.36
 g 2g 2g 3
∴ T = 0.48 g
Eq. (iii) can be written as
Now, WT = TS cos 0° (on 0.36 kg mass)
 1  v2 
 cos θ –  =   sinθ cosq g
 g  gL  = (0.48 g)   (1) = 0.08(g2) = 0.08(10)2 = 8 J
6
 v2 
Substituting value of   = sinθ from Eq. (i), we get
 gL 
  Sol 8: (D) Decrease in mechanical energy
 1
 cos θ –  = sin θ cosθ = (1 – cos θ) cosq
2 2 = work done against friction
 8 
1 1
1 ∴ mv2 – kx2 = µ mgx
or cos θ – = cosθ – cos3 q 2 2
8
2µ mgx + kx2
or v =
1 1 m
∴ cos3 θ = or cos θ = or θ = 60°
8 2
Substituting the values, we get
∴ From Eq. (i) v2 = gL sin θ = gL sin 60°
 4 
v = 0.4 m/s =   m/s
3  10 
or v =
2
gL
2 ∴ Answer is D
∴ Substituting this value of v2 in eq. (ii)
Sol 9: (D)
u2 = v2 + 2gL (1 + sin θ)
dw = F.(dx ˆi + dyjˆ )
= F.dr
3  3 3 3
= g L + 2gL 1 +  = gL + 2gL xdx ydy
2  2  2 = K∫ +
 
( ) ( )
3/2 3/2
x2 + y 2 x2 + y 2
 3 3
= gL  2 +  x2 + y 2 =
a2
 2 

0 a
K K  −a2 a2 
u=

gL  2 +
3 3

w
=
a3
∫ xdx + ∫ ydy
= 
a3  2
+ =  0
2 
 2  a 0

5 . 6 6 | Work , Energy and Power

Sol 10: (5) The initial speed of 1st bob (suspended by a 0.2 ≥ tan q
string of length l1) is 5gl1 . ∴ for P, Q
The speed of this bob at highest point will be gl1 . f = (m1 + m2) g sin q
When this bob collides with the other bob there speeds For R and S
will be interchanged.
F = fmax = mm2g sin q
l1
gl=
1 5gl2 ⇒ = 5
l2 Sol 14: (A, C)

dW
Sol 11: (5) Power = ⇒ W = 0.5 × 5 = 2.5 = KEf − KEi
dt m1g cos 

2.5
= (
M 2
v − v i2
2 f
) ⇒ vf =
5
m1
m2g cos 

m2
d(KE) dv
Sol 12: (B) = mv (m1+m2)g cos  (m1+m2)g sin 
dt dt

Sol 13: (D) Condition for not sliding,
fmax > (m1 + m2) g sin q As I1 = I2
mN > (m1 + m2) g sin q n1w1d1v1 = n2w2d2v2
0.3 m2 g cos θ ≥ 30 sin q
V2 B2 v 2 w2  B2 w2   n1 w1d1  B2n1
6 ≥ 30 tan q Now,
= =    = 
V1 B2 v1 w1  B1 w1   n2 w2d2  B1n2
1/5 ≥ tan q
2017-18 100 &
op kers
Class 11 T
By E ran culty
-JE Fa r
IIT enior emie .
S fP r es
o titut
Ins

PHYSICS
FOR JEE MAIN & ADVANCED
SECOND
EDITION

Exhaustive Theory
(Now Revised)

Formula Sheet
9000+ Problems
based on latest JEE pattern

2500 + 1000 (New) Problems


of previous 35 years of
AIEEE (JEE Main) and IIT-JEE (JEE Adv)

5000+Illustrations and Solved Examples


Detailed Solutions
of all problems available

Topic Covered Plancess Concepts


Tips & Tricks, Facts, Notes, Misconceptions,
Centre of Mass and Key Take Aways, Problem Solving Tactics
the Law of Conservation
PlancEssential
of Momentum
Questions recommended for revision
6. CENTRE OF MASS
AND THE L AW OF
C O N S E R VAT I O N O F
MOMENTUM

1. INTRODUCTION
In this chapter we will study the motion of system of particles or bodies. The individual particles or bodies comprising
the system in the general case move with different velocities and accelerations and exert forces on each other and
are influenced by external or surrounding bodies as well. We will learn the techniques to simplify the analysis of
complicated motion of such a system. We will also learn about the dynamics of extended bodies whose shape
and/or mass changes during their motion. We define the linear momentum of a system of particles and introduce
the concept of center of mass of a system. The dynamics of center of mass and the law of conservation of linear
momentum are important tools in the study of system of particles.

2 CENTER OF MASS
When we study the dynamics of the motion of a system of particles as a whole, then we need not bother about the
dynamics of individual particles of the system, but only focus on the dynamics of a unique point corresponding to
that system. The motion of this unique point is identical to the motion of a single particle whose mass is equal to
the sum of the masses of all the individual particles of the system and the resultant of all the forces exerted on all
the particles of the system, by the surrounding bodies, or due to action of a field of force, is exerted directly to that
particle. This point is called the center of mass (COM) of the system of particles. The COM behaves as if the entire
mass of the system is concentrated there. The concept of COM is very useful in analyzing complicated motion of
system of objects, in particular, when two or more objects collide or an object explodes into fragments.

2.1 Center of Mass of a System of Particles


   
For a system of n particles, having masses m1 , m2 , m3 ..... mn and position vectors r1 , r2 , r3 ,........rn respectively with

respect to the origin in a certain reference frame, the position vector of center of mass, rcm with respect to the origin
is given by
  
 m1 r1 + m2 r2 + ...... + mn rn
rcm =
m1 + m2 + ...... + mn
   
If the total mass of the system is M, then Mrcm = m1 r1 + m2 r2 + ..... + mn rn
6 . 2 | Center of Mass and the Law of Conser vation of Momentum

Co-ordinates of center of mass are

x1m1 + x2m2 + ..........xmmn y1m1 + y 2m2 + ..........ymmn z1m1 + z 2m2 + ..........zmmn


xcm = ; y cm = ; z cm =
m1 + m2 + .........mn m1 + m2 + .........mn m1 + m2 + .........mn

For a system comprising of two particles of masses m1 , m2 , positioned at co-ordinates ( x1 , y1 , z1 ) and ( x2 , y 2 , z1 ) ,


respectively, we have
m1 x1 + m2 x2 m1 y1 + m2 y 2 m1 z1 + m2 z 2
X com = ; Ycom = ; Z com =
m1 + m2 m1 + m2 m1 + m2

PLANCESS CONCEPTS

For a two-particle system, COM lies closer to the particle having more mass, which is rather obvious. If
COM’s co-ordinates are made zero, we would clearly observe that distances of individual particles are
inversely proportional to their masses.
Vaibhav Gupta (JEE 2009, AIR 54)

Illustration 1: Two particles of masses 1 kg and 2 kg are located at x = 0 and x = 3 m respectively. Find the position
of their center of mass. (JEE MAIN)

Sol: For the system of particle of masses m1 and m2, if the m1 = 1kg COM m2 = 2kg
distance of particle from the center of mass are r1 and r2
respectively then it is seen that m1r1 = m2r2. x=0 x=x x=3
Since, both the particles lie on x-axis, the COM will also lie on r1 = x r2 = (3-x)
the x-axis. Let the COM be located at x = x, then r1 = distance
of COM from the particle of mass 1 kg = x Figure 6.1

and r2 = distance of COM from the particle of mass 2 kg = (3 − x)


r1 m2 x 2
Using = or = or x = 2 m
r2 m1 3−x 1
Thus, the COM of the two particles is located at x = 2 m.

Illustration 2: Four particles A, B, C and D having masses m, 2m, 3m and 4m Y


respectively are placed in order at the corners of a square of side a. Locate the 4m 3m
center of mass.  (JEE ADVANCED) D C

Sol: The co-ordinate of center of mass of n particle system are given as

∑ mi xi ∑ mi yi m
A B 2m
X
X= i
, Y= i

∑ mi ∑ mi Figure 6.2
i i

Take the x and y axes as shown in Fig. 6.2. The coordinates of the four particles are as follows:

Particle Mass x-coordinate y-coordinate


A m 0 0 (taking A as origin)
B 2m a 0
C 3m a a
D 4m 0 a
P hysics | 6.3

Hence, the coordinates of the center of mass of the four-particle system are
m . 0 + 2ma + 3ma + 4m . 0 a m . 0 + 2m . 0 + 3ma + 4ma 7a
X = = ; Y =
m + 2m + 3m + 4m 2 m + 2m + 3m + 4m 10

 a 7a 
The center of mass is at  , .
 2 10 

2.2. Center of Mass of a Continuous Body


For continuous mass distributions, the co-ordinates of center of mass are determined by following formulae,

xcm =
∫ xdm ; y cm =
∫ y dm ; z cm =
∫ z dm
∫ dm ∫ dm ∫ dm
where x, y and z are the co-ordinates of an infinitesimal elementary mass dm taken on the continuous mass
distribution. The integration should be performed under proper limits, such that the elementary mass covers the
entire body.

PLANCESS CONCEPTS

Many people have misconception that the center of mass of a continuous body must lie inside the body.
Center of mass of a continuous body may lie outside that body also. e.g. Ring.
Vaibhav Krishnan (JEE 2009, AIR 22)

(a) Center of Mass of a Uniform Straight Rod


Center of mass of a rod of length L is at (L/2, 0, 0).
A B
X
dx
x

Figure 6.3

Y
(b) Center of Mass of a Uniform Semicircular Wire
Center of mass of a Uniform Semicircular Wire of radius R is
(0, 2R/π).
R
d

Rsin

X
O Rcos
Figure 6.4

(c) Center of Mass of a Uniform Semicircular Plate


Center of mass of a uniform semicircular plate of radius R is
(0, 4R/3π) r+dr
r
X
R
Figure 6.5
6 . 4 | Center of Mass and the Law of Conser vation of Momentum

(d) Center of mass of a uniform hollow cone


Center of mass of a uniform hollow cone of height H lies on the axis at a distance of H/3 from the center of
the bottom.

x


dx L
H

Figure 6.6

PLANCESS CONCEPTS

Student must solve the above integrations to get a better view of how we take infinitesimal segment of
a body and the corresponding limits to integrate over whole body.
Please solve the integrations for hollow cone and solid cone to note the difference.
Nivvedan (JEE 2009, AIR 113)

Illustration 3: A rod of length L is placed along the x-axis between x = 0 and x = L. The linear density (mass/length)
ρ of the rod varies with the distance x from the origin as ρ = a + bx. Here, a and b are constants. Find the position
of center of mass of this rod. (JEE MAIN)

Sol: To find C.O.M of continuous mass distributions consider a small element of distribution of mass dm. Then the
co-ordinate of C.O.M. is given as

∫ x dm
∴ xCOM = the limits of integration should be chosen such that the small elements covers entire mass
M
distribution.
Choose an infinitesimal element of the rod of length dx situated at co-ordinates (x, 0, 0) (see Fig.6.7) The linear
mass density can be assumed to be constant along the infinitesimal length dx.
Thus the mass of the element dm = rdx = (a + bx) dx P Q

As x varies from 0 to L the element covers the entire rod.


x=0 x=L
Therefore, x-coordinate of COM of the rod will be dx
L Figure 6.7
 ax2 bx3 
L L  + 

xCOM =
= 0
x dm ∫ (x) (a + bx)dx  2
0
= =
3 
0 3aL2 + 2bL3
×
2
L L L 6 2aL + bL2
∫0 dm ∫0 (a + bx)dx 
 ax +
bx2 

 2 
0
2
3aL + 2bL
xCOM = m
6a + 3bL
P hysics | 6.5

Illustration 4: Determine the center of mass of a uniform solid cone of height h and
semi angle α, as shown in Fig. 6.8 (JEE MAIN) R

Sol: To find C.O.M of continuous mass distributions consider a small element of


distribution of mass dm. Then the co-ordinate of C.O.M. is given as
r dy
∴ YCOM
∫ y dm
= the limits of integration should be chosen such that the small
H
M 
y
elements covers entire mass distribution.
We place the apex of the cone at the origin and its axis along the y-axis. As the cone
is a right circular cone then by symmetry it is clear that the center of mass will lie on Figure 6.8
its axis i.e. on the y-axis. We consider an elementary disk of radius r and infinitesimal
thickness dy whose center is on the y-axis at distance y from the origin as shown in Fig. 6.8. The volume of such a
disk is
dV = pr2 dy = π (y tan α)2 dy
The mass of this elementary disk is dm = rdV. As y varies from 0 to H, the total height of the cone, the elementary
disc covers the entire cone. The total mass M of the cone is given by,
H H3
M= ∫ dm = πρ tan2 α ∫ y 2dy = πρ tan2 α  ………(i)
0 3
The position of the center of mass is given by
1 H 1 H 3
πρ tan2 α
M ∫0 ∫0 y dy
y com= y dm
=
M
1 H4
= πρ tan2 α  ………(ii)
M 4
3H
From equations (i) and (ii), we have y com =
4

3. CENTER OF GRAVITY

Definition: Center of gravity is a point, near or within a body, at which its entire weight can be assumed to act
when considering the motion of the body under the influence of gravity. This point coincides with the center of
mass when the gravitational field is uniform.
Note: The center of mass and center of gravity for a continuous body or a system of particles will be different when
there is non- uniform gravitational field.

PLANCESS CONCEPTS

You can find the center of gravity and center of mass for a very thin cylinder extending from the surface
of earth to the height equal to radius of earth to get the difference. Just sum up all the individual weights
of infinitesimal size disks and find the position where gravity will make the same weight of body. This will
give center of gravity.
Chinmay S Purandare (JEE 2012, AIR 698)
6 . 6 | Center of Mass and the Law of Conser vation of Momentum

4. CENTER OF MASS OF THE BODY WHEN A PORTION OF THE BODY IS


TAKEN OUT
Suppose there is a body of total mass m and a mass m1 is taken out from this body. The remaining body will have
mass (m - m1) and its center of mass will be at coordinates,

mx − m1 x1 my − m1 y1 mz − m1 z1
xcm = ; y cm = ; z cm =
(m − m1 ) (m − m1 ) (m − m1 )

where (x, y, z) are coordinates of center of mass of original (whole) body and (x1, y1, z1) are coordinates of center
of mass of the portion taken out.

Illustration 5: A circular plate of uniform thickness has a diameter of 56 cm. A circular portion of diameter 42 cm
is removed from one edge of the plate as shown in Fig. 6.9. Find the center of mass of the remaining position.
 (JEE MAIN)

Sol: Let O be the center of circular plate and, O1, the center of circular portion removed
from the plate. The COM of the whole plate will lie at O and the COM of the circular
cavity will lie at O1. Let O be the origin. So OO1 = 28cm – 21cm = 7cm.
The center of mass of the remaining portion will be given as O O1
28cm
mx − m1 x1 σ(Ax − A1 x1 ) π((28)2 0 − (21)2 7) 21cm

= xcm = =
(m − m1 ) σ(A − A1 ) π((28)2 − (21)2 )

xcm = − 9 cm = − 0.09 m.
Figure 6.9
This means that center of mass of the remaining plate is at a distance 9 cm from the
center of given circular plate opposite to the removed portion i.e. in this questioon,
the new Centre of Mass will shift 9 cm left.

5. MOTION OF THE CENTER OF MASS


For a n-particle system of total mass M and individual particles having mass m1, m2, …… mn, from the definition of
center of mass we can write,
   
Mrcm = m1 r1 + m2 r2 + ........ + mn rn
   
where rcm is the position vector of the center of mass, and r1 , r2 , …… rn are the position vectors of the individual
particles relative to the same origin in a particular reference frame.
If the mass of each particle of the system remains constant with time, then, for our system of particles with fixed
mass, differentiating the above equation with respect to time, we obtain.
   
drcm dr1 dr2 drn
M = m1 + m2 + ........ + mn  ….(i)
dt dt dt dt
   
or MVcm = m1 v1 + m2 v 2 + ......... + mn vn
   
Where v1 , v 2 , …… vn are the velocities of the individual particles, and Vcm is the velocity of the center of mass.
Again differentiating with respect to time, we obtain
   
dVcm dv1 dv 2 dvn
M = m1 + m2 + ...... + mn
dt dt dt dt
   
Macm = m1 a1 + m2 a2 + ...... + mnan ….(ii)
P hysics | 6.7

   
Where a1 , a2 , …… an are the accelerations of the individual particles, and acm is the acceleration of the center
of mass. Now, from Newton’s second law, the force Fi acting on the ith particle is given by Fi = mi ai . Then, above
equation can be written as
     
Macm = F1 + F2 + ......... + Fn = Fint ernal + Fexternal  ….(iii)

Internal forces are the forces exerted by the particles of the system on each other. However, from Newton’s third


law, these internal forces occur in pairs of equal and opposite forces, so their net sum is zero. ∴ Macm =Fext
This equation states that the center of mass (C.O.M) of a system of particles behaves as if all the mass of the system
were concentrated there and the resultant of all the external forces acting on all the particles of the system was
applied to it ( at C.O.M).
Concept: Whatever may be the rearrangement of the bodies in a system, due to internal forces (such as different
parts of the system moving away or towards each other or an internal explosion taking place, breaking a body into
fragments) provided net Fext=0, we have two possibilities:
(a) If the system as a whole was originally at rest, i.e. the C.O.M was at rest, then the C.O.M. will continue to be
at rest.
(b) If before the change, the system as a whole had been moving with a constant velocity (C.O.M was moving with
a consant velocity), it will continue to move with a constant velocity.
In presence of a net external force if the C.O.M had been moving with certain acceleration at the instant of
an explosion, in a particular trajectory, the C.O.M. will continue to move in the same trajectory, with the same
acceleration, as if the system had never exploded at all.
Briefly saying, any internal changes of the body do not effect the motion of C.O.M.

Illustration 6: A man of mass m is standing on a platform of mass W


M kept on smooth ice (see Fig. 6.10). If the man starts moving on
the platform with a speed v relative to the platform, with what
velocity relative to the ice does the platform recoil ? (JEE MAIN)
V
Sol: When net external force on system is zero, the C.O.M. will either
remain at rest or continue the state of motion. i.e. Vcm = constant
 Figure 6.10
Let velocity of platform be V . If velocity of man relative to platform
   
is v then the velocity of man in reference frame of ice is w= v + V .
Center of mass of the system comprising of “man and the platform” is initially at rest and as no horizontal external
force acts on this system (ice is smooth), the center of mass will continue to remain at rest.
  
 MV + m(v + V)
Vcm= 0=
M+m
 
or (M + m)V + mv = 0

 
mv
or V= − ms-1
M+m
Negative sign shows that the platform will move in the opposite direction of relative velocity of man.

Illustration 7: Two block of masses m1 and m2 connected by a weightless spring of stiffness k rest on a smooth
horizontal plane (see Fig 6.11). Block 2 is shifted by a small distance x to the left and released. Find the velocity of
the center of mass of the system after block 1 breaks off the wall. (JEE ADVANCED)
Sol: Elastic potential energy stored in spring will get converted in kinetic energy of the blocks. If we consider the
FBD of mass m1 at the instant when it breaks off the wall, the normal reaction from the wall is zero, but normal
6 . 8 | Center of Mass and the Law of Conser vation of Momentum

reaction from the wall is equal to is equal to force exerted by spring on mass m1 so at this instant, force by spring
is also zero.
1 2
The initial potential energy of compression is = kx
2
When the block m1 breaks off from the wall, the normal reaction from the wall is zero, which in turn means that
the tension in the spring is zero. Thus the spring has its natural length at this instant and the kinetic energy of the
block m2 is given by
1 1
m v 2 = kx2
2 2 2 2
kx2
x
v 22 =
m2 m1 m2
k
v2 = x Figure 6.11
m2

Velocity of center of mass is


m1 v1 + m2 v 2
Vcm =
m1 + m2
At start v1 = 0

m2 m2 x k
∴ Vcm
= = v2
m1 + m2 m1 + m2 m2

x km2
∴ Velocity of center of mass of system Vcm = ms-1
m1 + m2

6. LINEAR MOMENTUM

The quantity momentum
 (denoted as P ) is a vector defined as the product of the mass of a particle and its velocity
 
v , i.e. P = m v ………(i)
From Newton’s second law of motion, if mass of a particle is constant
 
  dv d  dP
= F ma= m= (mv)
=
dt dt dt
Thus, for constant m, the rate of change of momentum of a body is equal to the resultant force acting on the body
and is in the direction of that force.

 a system of n particles with masses m1 , m2 ….. etc., and velocities v1 , v 2 ….. etc. respectively, the total momentum
For
P in a particular reference frame is,
      
P = P 1 + P2 + ........ + Pn = m1 v1 + m1 v 2 + ......... + mn vn ;
 
or P = MVcm
 
dP dVcm  
Also, =M = Macm = Fext
dt dt

dP 
∴ = Fext
dt

The magnitude of linear momentum may be expressed in terms of the kinetic energy as well.
p = mv
P hysics | 6.9

1 2
or =p2 m
=2 2
v 2m  mv
=  2mK
2 
p2
Thus, p = 2Km or K =
2m

6.1. Law of Conservation of Linear Momentum


“The law of conservation of linear momentum states that if no external forces act on a system of particles, then
the vector sum of the linear momenta of the particles of the system remains constant and is not affected by their
mutual interaction. In other words the total linear momentum of a closed system remains constant in an inertial
reference frame.”
Proof: For a system of fixed-mass particles having total mass m we have
  
  dv cm d(mv cm ) dP 
= Fext ma= cm m= = , where P is the total momentum of the system.
dt dt dt

 dP
Thus, Fext =
dt
In case the net external force applied to the system is zero, we have

 dP 
= = 0 or P = constant
Fext
dt
Thus for a closed system, the total linear momentum of the system remains constant in an inertial frame of reference.

PLANCESS CONCEPTS

Both linear momentum and kinetic energy are dependent on the reference frame since velocity is
inclusively dependent on the frame of reference.
Nitin Chandrol (JEE 2012, AIR 134)

Illustration 8: A gun (mass = M) fires a bullet (mass = m) with speed v r relative to barrel of the gun which is
inclined at an angle of 60° with horizontal. The gun is placed over a smooth horizontal surface. Find the recoil
speed of gun. (JEE MAIN)

Sol: When a bullet is fired, gun recoils in backward direction. Using law of conservation of linear momentum we
can find the recoil velocity of gun.
 
Let the recoil velocity of gun be v . The relative velocity of the bullet is v r at an angle of 60o with the horizontal.
Taking gun + bullet as the system the net external force on the system in horizontal direction is zero. Let x-axis
be along the horizontal and bullet be fired towards the positive direction of x-axis. Initially the system was at rest.
Therefore, applying the principle of conservation of linear momentum along x-axis, we get
o
vr sin 60

o
vr cos 60 -v
Components of velocity
of bullet relative to ground
Figure 6. 12
6 . 1 0 | Center of Mass and the Law of Conser vation of Momentum

Mv x + m(v rx + v x ) =
0
− Mv + m(v cos60° − v) = 0
r
mv r cos60°
v =
M+m
mv r
or v= ms-1
2(M + m)
-1 -1
10ms 3ms
Illustration 9: The block of mass m1 = 2kg and m2 = 5kg are
moving in the same direction along a frictionless surface with m1 m2
speeds 10 ms and 3 ms , respectively m2 being ahead of
-1 -1

m1 as shown in Fig. 6.13. An ideal spring with spring constant Figure 6.13
K = 1120 N/m is attached to the back side of m2 . Find the
maximum compression of the spring when the blocks move together after the collision. (JEE ADVANCED)

Sol: As frictional force on the blocks is zero the total momentum of blocks can be conserved during collision. At
the instant of maximum compression some part of initial total K.E. of blocks is stored as elastic P.E. in the spring.
Let v be the final velocity of the system after collision when the blocks move together.
Applying the law of conservation of momentum, we have
m1u1 + m2u2 =(m1 + m2 )v
Substituting the values,
(2 × 10) + (5 × 3) = (2 + 5)v
35
=
v = 5m/s
7
Applying the law of conservation of energy we get

1 1 1 1
m u2 + m u2 = (m + m2 )v 22 + Kx2
2 1 1 2 2 2 2 1 2
m u2 + m u2 =(m + m )v + Kx2
1 1 2 2 1 2 2


2 × (10)2 + 5 × (3)2 = [(2 + 5) × (5)2 ] + 1120 x2
70 1
x2 = = ⇒ x = 0.25m
1120 16

PLANCESS CONCEPTS

In the above questions, note that the compression would be maximum when the relative velocity
between the blocks is zero.
B Rajiv Reddy (JEE 2012, AIR 11)

7. VARIABLE MASS
 
From Newton’s second law, Fext = ma is applicable to a system whose total mass m is constant. If total mass of the
system is not constant, then this form of Newton’s second law is not applicable. If at a certain moment of time the
P hysi cs | 6.11

 
dp
total mass of a system is m and a mass dm is added (or separated) to the system, then we apply Fext = to the
system comprising “m+dm” to get dt
        
Fext .dt =dp =pfinal − pinitial =(m + dm)(v + dv) − [mv + dm(v + u)]
   
or Fext= .dt mdv − dmu ; ( dm.dv  0 )
 
dv dm 
or= Fext m − u
dt dt

dv  dm 
or m = Fext + u
dt dt
where u is velocity of adding or separating mass dm relative to the system having instantaneous mass m and
dm
instantaneous velocity v with respect to an inertial reference frame. The term can be positive or negative
dt
depending upon whether mass is added to the system or mass is separating from the system.
Problems related to variable mass can be solved in following three steps.
(a) Make a list of all the external forces acting on the main mass and draw its FBD.

(b) Apply an additional thrust force or reaction force Ft on the main mass, due to the action of the added(separated)
  dm  
mass on the main mass, the magnitude of which is u  ±  and direction is given by the direction of u in
 dt 

case the mass is being added or the direction of −u if mass is being separated.
(c) Apply the equation

dv  dm 
m = Fext + u (m = instantaneous mass)
dt dt

Illustration 10: A flat cart of mass m0 at t=0 starts moving to the left due to
a constant horizontal force F. The sand spills on the flat cart from a stationary
hopper. The rate of loading is constant and equal to µ kg/s. Find the time
dependence of the velocity and the acceleration of the flat cart in the process of
loading. The friction is negligibly small. (JEE ADVANCED)
F
Sol: The hopper is at rest in K frame, so in the frame of the cart its initial velocity
will be u=-v, where v is velocity of cart in K frame. Here we have used the
dv dm Figure 6.14
equation of motion of variable mass m = Fext + u
dt dt
dm
The rate of increase of mass of the flat car = µ kgs−1
dt
The hopper is stationary and so its relative velocity is u = 0 - v = - v
The equation of motion is given by

dv dm  dm 
m = F+ u = F − µv  µ and u =
= −v 
dt dt  dt 
At the instant t, m = m0 + µt
v t
dv dt dt dv dt

F − µv
= =
m m0 + µt
⇒ ∫ F − µv = ∫ m + µt
0 0 0

1 F − µv 1 m + µt
or, − loge = loge 0
µ F µ m0
6 . 1 2 | Center of Mass and the Law of Conser vation of Momentum

F m + µt Ft
or loge = loge 0 ⇒ v = ms−1
F − µv m0 m0 + µt
The acceleration a is given by
dv Fm0
a
= = ms−2
dt
(m0 + µt )
2

Alternative:
dv dm dm dv dm
m F+
= u=
F− v or m + v=
F
dt dt dt dt dt
mv t
d
or F ⇒ ∫ d(mv) =
(mv) = ∫ Fdt
dt 0 0
Ft
or mv= Ft ⇒ v= ; ( m= m0 + µt)
(m0 + µt)

8. ROCKET PROPULSION
The propulsion of rocket is an example of a system of variable mass. In the combustion chamber of a rocket, the
fuel is burnt in the presence of an oxidizing agent due to which a jet of gases emerges from the tail of the rocket.
Thus the mass of the rocket is continuously decreasing. This action due to emission of gases in the backward
direction produces a reaction force in the forward direction due to which the rocket moves forward.
u v

At t = 0 At t = 0
v=u m=m
m = m0 v=v

Exhaust velocity = vr

Figure 6.15: Rocket propulsion

Let m0 be the mass of the rocket and u be its velocity at time t = 0, and m be its mass and v be its velocity at any
time t. (see Fig. 6.15)
dm
The mass of the gas ejected per unit time or the rate of change of mass of the rocket is − and v r be the exhaust
dt
dm
velocity of the gases relative to the rocket. Usually − and v r are assumed constant throughout the journey of
the rocket. dt

Now using the equation of motion for a system of variable mass derived in the previous article we get,

dv   dm
m = mg + v r
dt dt
 
dv  v r dm
or = g+
dt m dt

  dm 
or
= dv v r + g.dt
m
dm
This is a vector equation and we do not assume any sign of . It is taken to be positive. After evaluating the
dt
definite integrals, when we substitute the scalar components of the vectors with proper signs we get the correct
result.
P hysi cs | 6.13


v m t
  dm 
Integrating on both sides, we get
= ∫ dv vr ∫ + g. ∫ dt

u m0
m 0

   m 
or =
v − u v r ln + g.t
m0

    m
or v =u + g.t + v r ln
m0
Now taking upwards direction as positive and downwards as negative (g and vr are downwards and u is upwards)
we get,
m
v = u − g.t + ( − v r )ln
m0

m 
Thus, v =u − gt + v r ln  0 
 m
 
dm
Now if − = µ (constant), then m = m0 - mt
dt
 m0 
Thus, v =u − gt + v r ln  
 m0 − µt 
If the initial velocity of the rocket u = 0, and the weight of the rocket is ignored as compared to the reaction force
m 
of the escaping gases, the above equation reduces to v = v r ln  0 
 m 

PLANCESS CONCEPTS

The concept of variable mass can also be physically visualized by changing the reference frame to the
instantaneous velocity of body. In that case mass is either being added by constant speed or being
removed by a constant speed. Considering the dm mass and the body as a system, and writing the
equations of conservation of momentum one can see the magic!
Anand K (JEE 2011, AIR 47)

Illustration 11: (a) A rocket set for vertical firing weighs 50 kg and contains 450 kg of fuel. It can have a maximum
exhaust velocity of 2000 m/s. What should be its minimum rate of fuel consumption?
(i) To just lift it off the launching pad?
(ii) To give it an acceleration of 20 m/s2?
(b) What will be the speed of the rocket when the rate of consumption of fuel is 10 kg/s after whole of the fuel is
consumed? (Take g = 9.8 m/s2) (JEE ADVANCED)
Sol: To lift the rocket upward against gravity, the thrust force in the upward direction due to exiting gases should
be greater than or equal to the gravitational force. During motion the mass of rocket decreases till whole of its fuel
is consumed. Final velocity of rocket is
m 
v =u − gt + vr ln  0  .
 m
(a) (i) To just lift the rocket off the launching pad
Initial weight = thrust force
6 . 1 4 | Center of Mass and the Law of Conser vation of Momentum

 dm  dm m0 g
or m=
0g vr  −  ; or − =
 dt  dt vr

dm (450 + 50)(9.8)
Substituting the values, we get − = = 2.45 kg/s
dt 2 × 103
(ii) Net acceleration a = 20 m / s2
∴ ma= Ft − mg
 dm 
or m(a + g) = Ft = v r  − 
 dt 
 dm  m(g + a)
This gives,  − =
 dt  vr

 dm  (450 + 50)(9.8 + 20)


Substituting the values, we get  − = = 7.45 kg/s
 dt  2 × 103
(b) The rate of fuel consumption is 10 kg/s. So, the time for the consumption of entire fuel is
450
=t = 45s
10
m 
The formula for speed of the rocket at time t is, v =u − gt + vr ln  0 
 m 
3
Here, u = 0, v r = 2 × 10 m / s , m0 = 500kg and m = 50 kg
 500 
Substituting the values, we get v = 0 − (9.8)(45) + (2 × 103 ) ln  
 50 
or v = – 441 + 4605.17; or v = 4164.17 m/s; or v = 4.164 km/s

9. COLLISION
An event in which two or more bodies exert forces on each other for a relatively short time is called collision.
If net external force acting on the system of bodies is zero, then according to the law of conservation of linear
momentum, the total momentum of the system of bodies before and after the collision remains constant.

9.1 Classification of Collisions


Collisions are classified into following types on the basis of the degree of conservation of kinetic energy in a
collision:
(a) Elastic Collision: If the total kinetic energy of the colliding particles is conserved before and after the collision,
the collision is said to be an elastic collision. If two bodies of masses m1 and m2 moving with velocities u1 and
u2 respectively, collide with each other so that their final velocities after collision are v1 and v 2 respectively,
then the collision will be perfectly elastic if,
1 1 1 1
m1u12 + m2u22 = m1 v12 + m2 v 22
2 2 2 2
(b) Inelastic Collision: If the total kinetic energy of the colliding particles is not conserved before and after the
collision, the collision is said to be inelastic collision. The kinetic energy is partially converted into other forms
of energy like sound, heat, deformation energy etc. u1 u2
State before collision
(c) Perfectly Inelastic Collision: The collision is said to
v1 v2
be perfectly inelastic if the collision results in “sticking State after collision
together” of the colliding particles after which they move
as a single unit with the same velocity. Figure 6.16: Collision in one dimension
P hysi cs | 6.15

Collisions can also be classified on the basis of the line of action of the forces of interaction.
(i) Head- on collisions: A collision is said to be head-on if the direction of the velocities of each of the
colliding bodies are along the line of action of the forces of interaction acting on the bodies at the
instant of collision.
(ii) Oblique collisions: A collision is Y
said to be oblique if the direction of
the velocities of the colliding bodies A v1
u1
are not along the line of action of 
X X
the forces of interaction acting on A B

the bodies at the instant of collision.
Just after collision, at least one of the B v2
colliding bodies moves in a direction
different from the initial direction of Figure 6.17: Collision in two dimensions
motion.

10. COEFFICIENT OF RESTITUTION


Coefficient of restitution is a measure of the elasticity of a u1 u2
collision between two particles. It is defined as the ratio of State before collision
relative velocity of one of the particles with respect to the other v1 v2
particle after the collision to the relative velocity of the same State after collision
particle before the collision and the ratio is negative.
Figure 6.18: Velocities before and after collision
If the velocities of two particles before the collision are u1 and
v − v2
u2 respectively and their velocities after the collision are v1 and v 2 respectively (see Fig. 6.18), then 1 = −e
u1 − u2
The coefficient of restitution is also expressed as the ratio of velocity of separation after collision to the velocity of
approach before collision.
v 2 − v1 velocity of separation
e = =
u1 − u2 velocity of approach

PLANCESS CONCEPTS

•  For perfectly inelastic collision e = 0.


•  For perfectly elastic collision e = 1.
•  For partially inelastic collision 0 < e < 1.
In elastic and inelastic collisioins, momentum is conserved whereas in inelastic collisions, kinetic energy
is not conserved.
Yashwanth Sandupatla (JEE 2012, AIR 821)

11. ELASTIC COLLISION u1 u2

Consider the collision of two small smooth spheres of masses m1 and m2 moving with m1 m1
v1 v2
velocities u1 and u2 respectively in the same direction along the line joining their centers.
Suppose m1 is following m2 with u1 > u2 i.e. m1 tries to overtake m2 but as the line of m2 m2
motion is same as the line joining the centers of the spheres, head-on ellastic collision
Figure 6.19: Head-on
takes place. Let their velocities after the elastic collision are v1 and v 2 respectively,
collision between two
with v 2 > v1 as shown in the Fig. 6.19
particles
6 . 1 6 | Center of Mass and the Law of Conser vation of Momentum

Conserving momentum before and after collision we get


m1u1 + m2u2 = m1 v1 + m2 v 2
) m2 (v 2 − u2 ) 
m1 (u1 − v1= …(i)
Conserving kinetic energy before and after collision we get
1 1 1 1
m1u12 + m2u22 = m1 v12 + m2 v 22
2 2 2 2
2 2 2 2
m1 (u1 − v1=) m2 (v 2 − u2 ) …(ii)
Dividing (ii) by (i)
u1 + v1 = v 2 + u2
So v1 =−u1 + u2 + v 2 …(iii)
Substitute v1 in equation (i)
m1 (u1 + u1 − u2 − v 2=
) m2 v 2 − m2u2
2m1u1 + (m2 − m1 )u2 = (m1 + m2 )v 2

 2m1   m2 − m1 
=v2   u1 +   u2  …(iv)
 m1 + m2   (m1 + m2 ) 

 m − m2   2m2 
Similarly, v1  1
=  u1 +   u2 …(v)
 m1 + m2   m1 + m2 
Special Cases
(i) When m1 = m2 ,
From equation (i)
u1 − v1 = v 2 − u2 or v1 + v 2 = u1 + u2 …(vi)
Equation (iii) gives
v1 − v 2 = u2 − u1 …(vii)
Solving (vi) and (vii) we get
v1 = u2 and v 2 = u1
∴ In one dimensional elastic collision of two bodies of equal masses, the bodies exchange there velocities after
collision.
(ii) When m2 is at rest i.e. u2 = 0 .

2m1u1  m − m1  2m1u1
=v2 +  2  u2 ⇒ v 2 =
m1 + m2  m2 + m1  m1 + m2

Now there are three possibilities in this case:


2mu1
(a) If m
=1 m
=2 m ; v2 = = u1 , v1 = 0 .
2m
The first body stops after collision. Both the momentum and the kinetic energy of the first body are completely
transferred to the second body.
(b) If m2 >> m1 , v1 ~ −u1 , v 2 ~ 0
Thus when a light body collides with a much heavier stationary body, the velocity of light body is reversed and
heavier body almost remains at rest.
P hysi cs | 6.17

(c) If m2 << m1 , v 2 ~ u1 and v 2 ~ 2u1


Thus when a heavy body collides with a much lighter stationary body, the velocity of heavier body remains
almost unchanged. The lighter body moves forward with approximately twice the velocity of the heavier body.

12. INELASTIC COLLISION


Consider the situation similar to previous article wherein m1 is following m2 with u1 > u2 i.e. m1 tries to overtake m2
but as the line of motion is same as the line joining the centers of the spheres, head-on collision takes place. Let their
velocities after the collision are v1 and v 2 respectively, with v 2 > v1 . Now suppose that the collision is inelastic, i.e.
kinetic energy is not conserved.
Conserving momentum we get,
m1u1 + m2u2 = m1 v1 + m2 v 2
Restitution equation gives,
v1 − v 2 =
−e(u1 − u2 )
The loss in kinetic energy ∆E in this case, is given by

1  m1m2  2 2
=∆E   (e − 1)(u1 − u2 )
2  m1 + m2 

Putting e = 0 in this equation, it is clear that the loss of kinetic energy is maximum in case of pefectly inelastic
collision.

Illustration 12: A block of mass m moving at a velcoity v collides head on with another block of mass 2m at rest.
If the coefficient of restitution is 0.5, find the velocities of the blocks after the collision. (JEE MAIN)

Sol: Solve using law of conservation of momentum, before and after collision and the equation of restitution.
Suppose after the collision the block of mass m moves at a velocity u1 and the block of mass 2m moves at a
velocity u2 . By conservation of momentum,
mv mu1 + 2mu2 
= … (i)
The velocity of sepration is u2 − u1 and the velocity of approach is v.
v
So, u2 − u1 = … (ii)
2
v
Solving (i) and (ii) we get, u1 = 0 ms−1 and u2 = ms−1.
2

Illustration 13: A ball is moving with velocity 2 m/s towards a heavy wall moving 2 m/s 1 m/s
towards the ball with speed 1 m/s as shown in Fig. 6.20. Assuming collision to be
elastic, find the velocity of ball immediately after the collision. (JEE MAIN)
Figure 6.20
Sol: The equation of conservation of momentum will not give us any fruitful
result because the mass of the wall is very large and remains at rest before and
after the collision. This problem has to be solved by using equation of restitution

2 m/s 1 m/s v 1 m/s

Before collision After collision


Figure 6.21
6 . 1 8 | Center of Mass and the Law of Conser vation of Momentum

The speed of wall will not change after the collision. So, let v be the velocity of the ball after collision in the direction
shown in Fig. 6.21. Since, collision is elastic (e = 1).
velocity of Separation=velocity of approach
or v − 1 = 2 − ( −1)
or v = 4 m/s

Illustration 14: A ball of mass m is projected vertically up from smooth horizontal floor with a speed V0. Find the
total momentum delivered by the ball to the surface, assuming e as the coefficient of restitution of impact. 
 (JEE MAIN)

Sol: By Newton’s third law the impulse delivered by the ball to the surface at each collision will be equal in
magnitude to the impulse delivered to the ball by the surface i.e. change in momentum of ball at each collision.
The total impulse will be the sum of DP due to all the collisions.
The momentum delivered by the ball at first, second, third impact etc. can be given as the corresponding change
in its momentum ( ∆P ) at each impact.

( ∆=
P)1 (mV1 )ˆj − m( −V0 )ˆj ⇒ ∆P=
1 m(V1 + V0 )

Similarly ∆P=
2 m(V1 + V2 ) , ∆P=
3 m(V2 + V3 ) , …. and so on.
⇒ The total momentum transferred ∆P = ∆P1 + ∆P2 + ∆P3 + .......

Putting the values of ∆P1 , ∆P2 etc., we obtain,


∆P m  V0 + 2(V1 + V2 + V3 + ......)
=
Putting V1 = eV0 , V2 = e2 V0 , V3 = e3 V0
We obtain,
 e  1 + e
∆P mV0 1 + 2(e + e2 + e3 + ......)
= ⇒ =
∆P mV0  1 + 2 = mV0  
   1−e 1 −e

Illustration 15: A stationary body explodes into four identical fragments such that three of them fly off mutually
perpendicular to each other, each with same K.E. Find the energy of explosion. (JEE ADVANCED)

Sol: As the body is initially at rest, the vector sum of momentum of all fragments will be zero. The energy of
explosion will appear as K.E. of fragments.
Let the three fragments move along X, Y and Z axes. Therefore their velocities can be given as
  
V1 = Viˆ , V2 = V ˆj and V3 = Vkˆ ,

where V = speed of each of the three fragments. Let the velocity of the fourth fragment be V4 Since, in explosion no
net external force is involved, the net momentum of the system remains conserved just before and after explosion.
Initially the body is a rest,
   
⇒ mV1 + mV2 + mV3 + mV4 = 0
   
(
Putting the values of V1 , V2 and V3 , we obtain, V =−V ˆi + ˆj + kˆ
4 )
Therefore, V4 = 3 V
The energy of explosion

1 2 1 2 1 2 1 2
∴E =
KEf − KEi =
 mV1 + mV2 + mV3 + mV4  − (0)
2 2 2 2 
P hysi cs | 6.19

1
Putting V=
1 V=
2 V=
3 V and setting mV 2 = E0 , we obtain, E = 6E0 .
2

13. OBLIQUE COLLISION


Let us now consider the case when the velocities of the two colliding spheres y
are not directed along the line of action of the forces of interaction or the line
of impact (line joining the centers). As already discussed this kind of impact
is said to be oblique. A B
Let us consider the collision of two spherical bodies. Since velocities v '1 and x
v '2 of the bodies after impact are unknown in direction and magnitude, their
m1v1 m2v2
determination will require the use of four independent equations.
We choose the x-axis along the line of impact, i.e. along the common normal to
the surfaces in contact, and the y-axis along their common tangent as shown Figure 6.22: Oblique collision
in Fig. 6.22. Assuming the spheres to be perfectly smooth and frictionless, of two particles
the impulses exerted on the spheres during the collision are along the line
of impact i.e., along the x-axis. So,
(i) the component of the momentum of each sphere along the y-axis, considered separately is conserved; hence
the y component of the velocity of each sphere remains unchanged. Thus we can write
(v1 )y = (v '1 )y ….(i)
(v 2 )y = (v '2 )y  ….(ii)
(ii) the component of total momentum of the two spheres along the x-axis is conserved. Thus we can write
m1 (v1 )x + m2 (v 2 )x = m1 (v '1 )x + m2 (v '2 )x  ….(iii)
(iii) The component along the x-axis of the relative velocity of the two spheres after impact i.e. the velocity of
separation along x-axis is obtained by multiplying the x-component of their velocity of approach before
impact by the coefficient of restitution. Thus we can write
( v '2 ) − ( v '1 ) = e ( v1 ) − ( v 2 )  ….(iv)
x x  x x
Now the four equations obtained above can be solved to find the velocities of the spheres after collision.

PLANCESS CONCEPTS

It is not advised to break the components of velocity in any other direction even though they are still
valid. The only problem will be in using the coefficient of restitution.
Definition of coefficient of restitution can be applied in the normal direction in the case of oblique
collision.
G.V. Abhinav (JEE 2012, AIR 329)

Illustration 16: After perfectly inelastic collision between two identical particles moving with same speed in
different directions, the speed of the particles becomes half the initial speed. Find the angle between the two
before collision. (JEE MAIN)
v
Sol: In case of an oblique collision, the momentum of individual particles are added m 2m
v
vectorially in the equation of conservation of linear momentum. 
m 2
Let θ be the desired angle. Linear momentum of the system will remain conserved.
v
Figure 6.23
6 . 2 0 | Center of Mass and the Law of Conser vation of Momentum

Hence P2 = P12 + P22 + 2P1P2 cos θ


2
  v   2 2
or 2m    = (mv) + (mv) + 2(mv)(mv)cos θ
  2  
1
or 1 = 1 + 1 + 2cos θ or cos θ = −
2
∴ =
θ 120°

Illustration 17: A ball of mass m hits the floor with a speed v making an angle of incidence θ with the normal. The
coefficient of restitution is e. Find the speed of the reflected ball and the angle of reflection of the ball.
 (JEE MAIN)
Sol: In case of an oblique collision with fixed surface the component of velocity of
colliding particle parallel to surface doesn’t change. The impulse will act along the
v
normal to the surface so use the equation of restitution along the normal. v’

See Fig. 6.24. Let the angle of reflection is θ ' and the speed after the collision is v ' . ’
The impulse on the ball is along the normal to the floor during the collision. There
is no impulse parallel to the floor. Thus, the component of the velocity of the ball
parallel to the surface remains unchanged before and after the collision. This gives Figure 6.24

θ ' v sin θ …(i)


v 'sin=
As the floor is stationary before and after the collision, the equation of conservation of momentum in the direction
normal to the floor will not give any result. We have to use the formula for coefficient of restitution along the
direction normal to the floor.
The velocity of separation along the normal= v 'cos θ '
The velocity of approach along the normal = v cos θ
Hence, v 'cos
= θ ' ev cos θ  …(ii)
From (i) and (ii),
tan θ
=v ' v sin2 θ + e2 cos2 θ and tan θ ' =
e
For elastic collision, e = 1 so that θ ' =θ and v ' = v

PLANCESS CONCEPTS

Inelastic collision doesn’t always mean that bodies will stick, which is very clear from the concept of
oblique collision. Only the velocities along n-axis become same and may be different in t-direction.
Anurag Saraf JEE 2011, AIR 226

14. CENTER OF MASS FRAME


We can rigidly fix a frame of reference to the center of mass of a system. This frame is called the C-frame of
reference and in general is a non-inertial reference frame. Relative to this frame, the center of mass is at rest
  
(Vcom,C = 0) and according to equation P = MVcom the total momentum of a system of particle in the C-frame of
reference is always zero.
 
P =ΣPi =0 in the C-frame of reference.
P hysi cs | 6.21

Note : When the net external force acting on the system is zero, the C-frame becomes an inertial frame.

14.1 A System of Two Particles


Suppose the masses of the particles are equal to m1 and m2 and their velocities in the given reference frame K
 
be v1 and v 2 respectively. Let us find the expressions defining their momentum and total kinetic energy in the

C-frame. The velocity of C-frame relative to K-frame is v c .
    
The momentum of the first particle in the C-frame is= P1/c m= 1 v1/c m1 (v1 − v c ) where v c is the velocity of the

center of mass of the system in the K frame. Substituting the expression for v c
 
 m v + m2 v 2
vc = 1 1
m1 + m2
  
we get P=1/c µ(v1 − v 2 )
m1m2
where µ is the reduced mass of the system, given by µ =
m1 + m2
  
Similarly, the momentum of the second particle in the C-frame is P=2/c µ(v 2 − v1 )
Thus, the momenta of the two particles in the C-frame are equal in magnitude and opposite in direction; the
modulus of the momentum of each particle is
P1/c = µv rel
 
where v=
rel | v1 − v 2 | is the modulus of velocity of one particle relative to another.

Finally, let us consider total kinetic energy. The total kinetic energy of the two particles in the C-frame is
1 2 1 2
P1/c2 P1/c2
K sys/c = K1/c + K 2/c = m1 v1/c + m2 v 2/c = +
2 2 2m1 2m2
m1m2 1 1 1
Now, µ= or + =
m1 + m2 m2 m2 µ

P1/c2 µv rel
2
Then K=
sys/c =
2µ 2

The total kinetic energy of the partices of the system in the K-frame is related to the total kinetic energy in C-frame.
The velocity of the ith particle of the system in K-frame can be expressed as:
  
= v i v i/c + v c

1 1   2 1   v 2c
So we can write K=
sys
2
∑ i i 2 ∑ i i/c c 2 ∑ i i/c c ∑ i i/c 2
m=v 2
m (v + v=) m v 2
+ v m v + ∑ mi
 
In the C-frame, the summation ∑
= mi v i/c MV
= com,C 0.

1 v2 v 2c
So we get K sys =∑ mi v i/c
2
+ c ∑ mi =
K sys/c + ∑ mi
2 2 2

For a two-particle system, we get


2
µvrel Mv c2
=
K sys + (where =
M m1 + m2 )
2 2
6 . 2 2 | Center of Mass and the Law of Conser vation of Momentum

Illustration 18: Two blocks of mass m1 and m2 connected by an ideal spring of v0


spring constant k are kept on a smooth horizontal surface. Find maximum extension m1 m2
of the spring when the block m2 is given an initial velocity of v 0 towards right as
shown in Fig. 6.25.  (JEE ADVANCED) Figure 6.25

Sol: In absence of frictional forces on block, the total mechanical energy of the system comprising the blocks and
spring will be conserved. At the time of maximum expansion of spring, the mechanical energy in C frame will be
totally stored as elastic P.E. of the spring
This problem can be best solved in the C-frame or the reference frame rigidly fixed to the center of mass of the
system of two blocks.
Initially at t=0 when the block m2 is given velocity v 0 , the total kinetic energy of the blocks in C-frame is related
to the total kinetic energy in the given frame K by the relation,
2
µvrel (m + m2 )v c2 (m + m2 )v c2  m 1 m2 m2 v 0 
K sys = + 1 K sys/c (0) + 1
= =
 µ ; vrel v 0=
= ; vc 
2 2 2  m1 + m2 m1 + m2 

where the first term on the right hand side of this relation is the total kinetic energy in C-frame at t=0, Ksys/c(0), and
the second term is the kinetic energy associated with the motion of the system of blocks as a whole in the K-frame.
As there are no dissipative external forces acting on the system, the total mechanical energy will remain constant,
both in the C-frame and the K-frame. In the C-frame the blocks will oscillate under the action of spring force and
the kinetic energy in the C-frame will get converted into the elastic potential energy of the spring and vice-versa,
the total mechanical energy remaining constant at each instant, equal to the total kinetic energy in C-frame at t=0,
Ksys/c(0).
Initially at t=0 when the block m2 is given velocity v 0 , the mechanical energy in C frame will be totally kinetic
(Ksys/c(0)), and at the instant of maximum extension of the spring, the mechanical energy in C-frame will be totally
converted into elastic potential energy of the spring. So we have,
1 2 1 m1m2 2 1 2
K sys/c (0) = kxmax ⇒ v = kx
2 2 m1 + m2 0 2 max

m1m2
Thus, maximum extension is xmax = v 0
k(m1 + m2 )

15. IMPULSE AND MOMENTUM


When two bodies collide during a very short time period, large impulsive forces are exerted between the bodies
along the line of impact. Common examples are a hammer striking a nail or a bat striking a ball. The line of impact
is a line through the common normal to the surfaces of the colliding bodies at the point of contact.
When two bodies collide, the momentum of each body is changed due to the force on it exerted by the other. On
an ordinary scale, the time duration of the collision is very small and yet the change in momentum is sizeable. This
means that the magnitude of the force of interaction must be very large on an ordinary scale. Such large forces
acting for a very short duration are called impulsive forces. The force may not be uniform during the interaction.

 dP   F
We know that the force is related to momentum as F = ⇒ Fdt = dP
dt

We can find the change in momentum of the body during a collision (from Pi to Area=Pf-Pi

Pf ) by integrating over the time of collision and assuming that the force during
  P  t 
collision has a constant direction, Pf −=Pi ∫ f dP
= ∫ f Fdt ;
Pi ti
Here the subscripts i (= initial) and f (= final) refer to the times before and after ti tf t
the collision. The integral of a force over the time interval during which the force
Figure 6.26: Impulse imparted to
acts is called impulse.
the particle
P hysi cs | 6.23

tf  
Thus the quantity ∫ti F . dt is the impulse of the force F during the time interval ti and t f and is equal to the

change in the momentum of the body in which it acts.


tf

The magnitude of impulse ∫ Fdt is the area under the force-time curve as shown in Fig. 6.26
ti

Illustration 19: A block of mass m and a pan of equal mass are connected by a string going over a smooth light
pulley as shown in Fig. 6.27. Initially the system is at rest when a particle of mass m falls on the pan and sticks to
it. If the particle strikes the pan with a speed v find the speed with which the system moves just after the collision.
 (JEE MAIN)

Sol: By Newton’s third law, the impulse imparted to the particle in upward direction will be equal in magnitude to
the total impulse imparted to the system of block and the pan.
Let N be the contact force between the particle and the pan during the collision.
Consider the impulse imparted to the particle. The force N will be in upward direction and the
impulse imparted to it will be ∫ N dt in the upward direction. This should be equal to the change
in momentum imparted to it in the upward direction.
Thus, ∫ N dt =Pf − Pi =−mV − ( −mv) =mv − mV ….(i) m

Similarly considering the impulse imparted to the pan. The forces acting on it are tension T upwards
and contact force N downwards. The impulse imparted to it in the downward direction will be, m m
∫ (N − T)dt= mV − 0= mV  ….(ii)
Figure 6.27
Impulse imparted to the block by the tension T will be upwards,

∫ T dt= mV − 0= mV  ….(iii)
Adding (ii) and (iii) we get, ∫ Ndt = 2mV  ….(iv)
v
2mV or
Comparing (i) and (iv) we get, mv − mV = V= ms−1
3

PROBLEM-SOLVING TACTICS
Applying the principle of Conservation of Linear Momentum
(a) Decide which objects are included in the system.
(b) Relative to the system, identify the internal and external forces.
(c) Verify that the system is isolated.
(d) Set the final momentum of the system equal to its initial momentum. Remember that momentum is a vector.
(e) Always check whether kinetic energy is conserved or not. If it is conserved, it gives you an extra equation.
Otherwise use work-energy theorem, carefully.
(f) Try to involve yourself physically in the question, imagine various events. This would help in some problems
where some parameters get excluded by conditions. This will also help in checking your answer.

Impulse
(g) Ignore any finite-value forces, while dealing with impulses.
(h) Write impulse equations carefully, because integration which we are unable to calculate will always cancel out.

Collisions
(i) Remembering special cases of collisions would be nice.
6 . 2 4 | Center of Mass and the Law of Conser vation of Momentum

FORMULAE SHEET


∑ m i ri
Position of center of mass of a system: rcom = i
M

rCOM = xCOMˆi + y COMˆj + z COMkˆ

m1 x1 + m2 x2 + ..... + mn xn ∑ mi xi
xCOM = = i
m1 + m2 + ..... + mn ∑ mi
i

For continuous bodies xCOM =


∫ x dm =
∫ x dm
∫ dm M

For a two-particle system, we have


m1=1kg COM m2=2kg
 m2   m1 
r1 =   d and r2 =   d x=0 x=x x=3
 m2 + m1   m1 + m2  r1=x r2=(3-x)
where d is the separation between the particles.
Figure 6.28
L 
Center of Mass of a Uniform Rod  ,0,0 
 2 

COM COM

R R
R 3R
2 8
O O
Hemispherical shell Solid hemispherical

COM COM

R R
R 3R
2 8
O O
Hemispherical shell Solid hemispherical

Figure 6.29

If some mass or area is removed from a rigid body, then the position of center of mass of the remaining portion is
obtained from the following formula:
 
 m r − m2 r2
rCOM = 1 1
m1 − m2
Where m1 is the mass of the body after filling all cavities with same density and m2 is the mass filled in the cavity.
Cavity mass is assumed negative.


  
m1 v1 + m2 v 2 + ..... + mn v n
∑ mi vi
Velocity of COM v COM = i
=
m1 + m2 + ..... + mn ∑ mi
i
P hysi cs | 6.25

    
Total momentum of a n-particle system PCOM = P1 + P2 + ...... + Pn = Mv COM

   ∑ mi ai
 m a + m2 a2 + ...... + mnan i
Acceleration of COM aCOM = 1 1
m1 + m2 + ...... + mn ∑ mi
    i
Net force acting on the system FCOM = F1 + F2 + ...... + Fn
 
Net external force on center of mass is Macm = Fext
        
If net force on the system F = F1 + F2 + F3 + ...... + Fn = 0 then, P1 + P2 + P3 + ...... + Pn = constant

Equation of motion of a body with variable mass is:



 dv    dm  
m  = F +  u
 dt   dt 

Where
 u is the velocity of the mass being added(separated) relative to the given body of instantaneous mass m
and F is the external force due to surrounding bodies or due to field of force.
dm
In case of reducing mass of a system = µ kgs−1
dt

 dv    dm  
For a rocket we have, m  =  mg +   vr
dt
   dt 

Where v r is the velocity of the ejecting gases relative to the rocket.
In scalar form we can write

 dv   dm 
m   = −mg + v r  − 
 dt   dt 
dm  dm 
Here − = rate at which mass is ejecting and v r  −  =Thrust force.
dt  dt 
m 
Final velocity of rocket v =u − gt + v r ln  0 
 m 
    
Impulse of a force: J =∫ F dt =∆p =pf − pi

Collision
(a) In the absence of any external force on the system the linear momentum of the system will remain conserved
before, during and after collision, i.e.,
m1 v1 + m2 v 2 =(m1 + m2 )v =m1 v '1 + m2 v '2 …(i)

(b) In the absence of any dissipative forces, the mechanical energy of the system will also remain conserved, i.e.
1 1 1 1 2 1 1
m1 v12 + m2 v 22 = (m1 + m2 )v 2 + kxm = m1 v1' 2 + m2 v '22  …(ii)
2 2 2 2 2 2

Head on Elastic Collision

 m − m2   2m2 

= v '1  1  v1 +   v 2
 m1 + m2   m1 + m2 

 m − m1   2m1 

= v '2  2  v 2 +   v1
 m1 + m2   m1 + m2 
6 . 2 6 | Center of Mass and the Law of Conser vation of Momentum

separation speed after collision


=e
approach speed before collision

 m − em2   m2 + em2 

= v '1  1  v1 +   v 2
 m1 + m2   m1 + m2 

 m − em1   m1 + em1 

= v '2  2  v 2 +   v1
 m1 + m2   m1 + m2 

The C-frame: Total kinetic energy of system in K-frame is related to total kinetic energy in C-frame as:
Mv 2c
K sys =
K sys/c +
2
;M= ∑ mi
  mm  
−P2/c = 1 2 ( v1 − v 2 )
For a two-particle system: P1/c =
m1 + m2
2 2
mm   µv rel P1/c
Or P1/c = µv rel = 1 2 v1 − v 2 and K=
P2/c = sys/c =
m1 + m2 2 2

Solved Examples

JEE Main/Boards
∫ x dm
Example 1: The linear mass density of rod of a length Center of mass of rod X cm =
l=2 m varies from A as (2+x) kg/m. What is the position ∫ dm
of center of mass from end A. l
l
x3
∫ x(2 + x)dx (x2 + )
Sol: To find C.O.M of continuous mass distributions 3
0 0
= X cm =
consider a small element of distribution of mass dm. l
2
l
x
Then the co-ordinate of C.O.M. is given as ∫ (2 + x)dx 2x +
2
0
0
y
l3
l2 + 2
X cm =
= 3 6l + 2l
l2 12 + 3l
2l +
2
6 × 2 + 2 × 4 20 10
x For l = 2 m, X cm
= = = m
(0,0) A 12 + 3 × 2 18 9
x dx
10
So center of mass is at a distance m from A.
∫ x dm
∴ xCOM = the limits of integration should be
9
M
chosen such that the small elements covers entire mass Example 2: One fourth of the mass of square lamina is
distribution. cut off (see figure). Where does the center of mass of
the remaining part of the square shift.
Take an element of the rod of infinitesimal length dx at
distance x from point A. The mass of the element will be Sol: To find the C.O.M. of a body having a cavity we first
dm = λ dx = (2 + x)dx As x varies from o to l the element fill the cavity with the same density as body and find
covers the entire rod. the C.O.M. (x,y,z) of the whole body. Then we consider
the cavity as second body having negative mass and
P hysi cs | 6.27

find the C.O.M (x1,y1,z1) of the cavity. The C.O.M. of the So v =


'Ay v= v A sin 30
= o
15 ms−1  …(i)
Ay
body with cavity is o
v=
'By v=
By vB sin 60
= 3 34.6ms−1
20= ...(ii)

y y

A x

vA vB
o
30
o
60
x
(0,0)

mx − m1 x1 my − m1 y1 mz − m1 z1 Along the x axis we conserve the momentum to get


xcm = ; y cm = ; z cm =
(m − m1 ) (m − m1 ) (m − m1 )
mA v Ax + mB vBx = mA v 'Ax + mB v 'Bx
Part of the lamina cut-off is taken as negative mass.
Coordinates of center of mass of whole lamina are m v A cos 30 + m( − vB cos 60o=
) mv 'Ax + mv 'Bx
a a
 ,  and the coordinates of center of mass of cut-off v 'Ax + v 'Bx = 15 3 − 20  … (iii)
2 2
 3a 3a  Velocity of separation = e (velocity of approach)
part are  ,  . So the center of mass of remaining
 4 4 ⇒ v 'Bx − v 'Ax = e(v Ax − vBx )
part is given as,
a m 3a a 3a 'Ax 0.9 (15 3 + 20)
v 'Bx − v=  … (iv)
m − . −
X cm
= 2 =4 4 2= 16 5a m
m 3 12 Solving (iii) and (iv) are get
m−
4 4
v 'Ax = −17.7 ms−1
a m 3a
m − . v 'Bx = 23.68 ms−1
2 4 4 5a
=Ycm = m
m 12
m− ⇒ v 'A= v 'Ax 2 + v 'Ay 2= 23.2 ms−1
4

Example 3: The magnitude and direction of the and v 'B = v 'Bx 2 + v 'By 2 = 41.92 ms−1
velocities of two identical frictionless balls before they
strike each other are as shown in figure. Assuming v’A v’B
e=0.90, determine the magnitude and direction of the
velocity of each ball after the impact. v A = 30 ms−1 ,
vB = 40 ms−1
o
40.3 55.6
o

Sol: In case of an oblique collision, the momentum of


individual particles are added vectorially in the equation
of conservation of linear momentum. The equation of Example 4: The mass of a rocket is 2.8 x 106 kg at
restitution is used along line of impact launch time of this 2x106kg is fuel. The exhaust speed
The component of velocity of each ball along the is 2500m/s and the fuel is ejected at the rate of
common tangent at the point of impact will remain the 1.4x104kg/sec.
same before and after the collisions. Let x and y axes (a) Find thrust on the rocket.
be along the common normal and common tangent
(b) What is initial acceleration at launch time?
respectively.
Ignore air resistance.
6 . 2 8 | Center of Mass and the Law of Conser vation of Momentum

Sol: To lift the rocket upward against gravity, the thrust R


force in the upward direction due to exiting gases point x1 = .
2
should be greater than or equal to the gravitational
force. The equation of motion of the rocket can be 2u2 sin θ cos θ 2 × 10 4 × 0.6 × 0.8
X cm= R= =
dv  dm  g 10
written in terms of force as m = Fg + v r  
dt  dt  m 960 3m
. + .x
Thrust force X= 960
= 4 2 4 2
cm
m
 dm  −1 4 −1
Fth vr  =
=  2500 ms × 1.4 × 10 kgs 960 3
 dt  ⇒ 960 = + x2
8 4
F=
th 3.5 × 107 N
4 7
Equation of motion of rocket is ⇒ x2 = × 960 × = 160 × 7 = 1120m
3 8
dv  dm   dm 
m =−W + vr  =−mg + v r  
dt  dt   dt  Example 6: A bullet of mass m strikes a block of mass M
connected to a light spring of stiffness k, with a speed
dv Fth 3.5 × 107 N v0 and gets embedded into mass M. Find the loss of K.E.
⇒ =−g + =−9.8 +
dt t =0
m0 2.8 × 106 kg of the system just after impact

 350 
⇒ a0=  − 9.8  ms−2 m k
 28  M
v0
⇒ a0 = 12.5 − 9.8 = 2.7 ms−2

Example 5: A projectile is fired at a speed of 100 m/s


at an angle of 37° above horizontal (see figure) At the Sol: During collision as there is no net external force
highest point the projectile breaks into two parts of acting on the bullet-block system, hence the momentum
mass ratio 1:3. Find the distance from the launching of the system can be conserved. As the bullet hits block
point to the point where the heavier piece lands. The in-elastically, some of its initial K.E. is lost during the
smaller mass has zero velocity with respect to the earth collision.
immediately after explosion. As the bullet of mass m hits the block of mass M, and
gets embedded into it, we can write the equation
Sol: The range of center of mass of the system during of conservation of linear momentum at the instant
u2 sin2θ of collision, assuming the force due to spring to be
projectile motion is given by X CM= R= . This
g negligible, as at this instant the block M has just started
range is not effected by any internal changes in the moving and the compression in the spring is negligible
system. (see figure)
-1
u=100 ms mV0
3m/4 mV0 = (m + M)V ⇒ V =
m m+M
4
37o V
O x1 R x2
The C.O.M of the projectile will hit the horizontal plane M+m
k
at the same point where it would have hit without any
explosion i.e. the range of COM will not change. Both the where V is the velocity of block just after collision.
smaller and larger mass will reach the ground together
because the vertical components of their velocity are Loss in kinetic energy of the system of bullet and block
equal to zero after the explosion. (Explosion took place is,
at the highest point of the trajectory and the smaller 1 1
mass comes to rest just after the explosion). At highest ∆K
= mv 20 − (M+ m) V 2
2 2
P hysi cs | 6.29

1 2 m2 v 20  1 2 1
= mv 0 − (M + m)  K sys
= m vB= +0 mv 2
2 2 2 2
 (M + m) 
2mv 2cm
Now =
K sys K sys/c +
mv 20  m  2
= 1 −  1 v2
2  M + m
⇒ mv 2 =K sys/c + m
2 4
mMv 20
∆K = J 
2(M + m) 1 µv 2 m v2 
⇒ K sys/c = mv 2 K sys/c = rel = . 
4 2 2 2
 
Example 7: Two blocks B and C of mass m each
connected by a spring of natural length l and spring Initially the potential energy of spring is zero and when
constant k rest on an absolutely smooth horizontal the compression is maximum the energy in C-frame
surface as shown in figure A third block A of same mass will be entirely converted into potential energy of the
collides elastically to block B with velocity v. Calculate the spring, thus we can write
velocities of blocks, when the spring is compressed as
mv 2 1
much as possible and also the maximum compression. 2
= k xmax
4 2
m
v ⇒ xmax =
v
2k
A B C

Sol: In absence of frictional forces on block, the total JEE Advanced/Boards


mechanical energy of the system comprising the blocks
B and C and spring will be conserved. At the time of Example 1: A body of mass M with a small disc of
maximum compression of spring, the mechanical mass m placed on it rests on a smooth horizontal
energy of this system in C-frame will be totally stored plane as shown in figure. The disc is set in motion in
as elastic potential energy of the spring. the horizontal direction with velocity v. To what height
Block A collides with block B elastically. So conserving relative to the initial level will the disc rise after breaking
momentum between A and B we get, (spring force is off the body M? The friction is assumed to be absent.
negligible at the instant of collision)
mv mv
= = A mv A + mvB
v v A + vB 
or = …(i) m
v vB − v A 
= …(ii) (restitution equation) v
M
Solving (i) and (ii), we get
= and vB v 
v A 0= …(iii)
For system comprising blocks B and C, the velocity of Sol: As there are no external forces acting on the system
center of mass after collision is, comprising m and M in the horizontal direction, the
mvB v momentum is conservative in the horizontal direction.
v cm
= = ms-1  …(iv)
m+m 2 At the instant the disk m breaks – off from block M, it
has a component of velocity u in vertical direction and
As there are no dissipative forces in the horizontal
the disk m and block M have a common velocity V in
direction the velocity of COM will remain constant. Let
the horizontal direction.
us consider the motion of B and C in the C-frame. At
the instant of maximum compression the blocks B and In horizontal direction we can write,
C will come to rest in the C-frame. So there velocity in mv
K- frame will become equal to the velocity of COM. mv = (m + M) V ⇒ V = ms-1 … (i)
m+M
v Once the disc breaks–off the block, then its horizontal
∴ vB = v C = ms-1  …(v)
2 velocity will not change and at the highest point of
Kinetic energy of system ‘’B + C’’ just after collision in its trajectory, the vertical component of its velocity
K frame is, becomes zero.
6 . 3 0 | Center of Mass and the Law of Conser vation of Momentum

Using law of conservation of energy, we get v −u u m + M2


⇒ 1− = 1−
=
1 1 1 v v m + M1 + M2
mv 2 = mV 2 + mgH + MV 2  … (ii)
2 2 2
∆v M1
⇒ × 100%
= × 100%
where H is the height raised by the disc. v m + M1 + M2

1 1 Putting the values of m, M1 and M2 we get


⇒ mgH
= m(v 2 ) − (m + M)V 2
2 2 1
1 1 m2 v 2 =% loss × 100%
⇒ mgH = mv 2 − (using (1))
(using (i)) 0.020 + 1 + 2.98
2 2 (m + M)
% loss = 25%
v2  m 
H
⇒= 1 − 
2g  m + M Example 3: Two bodies A and B of masses m and
2m respectively are placed on a smooth floor. They
v2  M  are connected by a spring. A third body C of mass m
H= .  m moves with a velocity v0 along the line joining A and
2g  m + M 
B and collides elastically with A, as shown in figure.
At a certain time t0, it is found that the instantaneous
Example 2: A 20 gm bullet pierces through a plate of velocities of A and B are the same. Further, at this
mass M1=1 kg and then comes to rest inside a second instant the compression of the spring is found to be x0.
plate of mass M2=2.98 kg as shown in the figure. It is
Find: (a) The common velocity of A and B at the time t0.
found that the two plates, initially at rest, now move
(b) The spring constant.
with equal velocities. Find the percentage loss in the
initial velocity of the bullet when it is between M1 and
Sol: The collision between the blocks A and C is elastic.
M2. Neglect any loss of material of the plates, due to
In C-frame at time of maximum compression of spring,
action of bullet.
the total mechanical energy will be stored as elastic
potential energy of spring.
M1 M2
Masses of bodies C and A are same and C

C v0 A B
m m 2m

collides elastically with body A initially at rest. So after


collision C will come to rest and A will take up the
velocity of C (spring force during collision is negligible.)
Sol: As the net external force on the system during
collision is zero, the momentum of system can be The velocity of center of mass (COM) of the system
conservative. comprising blocks A and B just after collision is,
Conserve momentum for collision of bullet with fist m.v 0 v 0
v cm
= = ms-1
plate, mv = mu + M1V  ….(i) 3m 3
Conserve momentum for collision of bullet with second As there are no external forces acting in horizontal
plate, mu = (m + M2)V  ….(ii) direction, the velocity of COM will be constant.
Here the two plates move with equal velocity V after In the C-frame when the compression in the spring is
the collision. maximum the blocks will come to rest momentarily.
Thus there velocity in K – frame will be equal to the
Eliminate V from equations (i) and (ii) to get
velocity of COM.
mu
mv
= mu + M1 v
(m + M2 ) ⇒ v A =vB =v cm = 0 ms-1
3
 M1  v(m + M2 ) Just after collision the total kinetic energy of blocks A
u 1 + =v; ⇒u=
 m + M2  m + M1 + M2 and B in C – frame is,
P hysi cs | 6.31

1 2 (m)(2m) 2 mv 20 Now ∆ x1= (R − r) + ∆ x2


K sys/c = µ vrel = .v 0 ⇒ K sys/c = J
2 2 × 3m 3 ⇒ m(R − r + ∆x2 ) + M∆x2 =0
This energy will get converted into the elastic potential m
energy of the spring at the instant of maximum ⇒ ∆x2 = − (R − r)  …(ii)
M+m
compression,
Now (R – r) is towards right, so ∆x2 will be towards left.
mv 20 1 2 1 2mv 2
= = k xmax k x02 ; ⇒ k = 0 J (b) Now as there are no dissipative forces acting on the
3 2 2 3 x20
system, total energy of system is conserved. i.e.
Example 4: A block of mass M with a semi-circular track 1 1
mg(R −
= r) mv12 + Mv 22  ….(iii)
of radius R rests on a horizontal frictionless surface. A 2 2
uniform cylinder of radius r and mass m is released From (i) and (iii) eliminate v1 to get
from rest at the point A as shown in the figure. The
2 2
cylinder slips on the semicircular frictionless track. 1 M v2 1
mg(R
= − r) m + Mv 22
2 m2 2
A r C
m
1 M 
R ⇒ mg(R
= − r) Mv 22  + 1
2 m 

M(M + m)v 22
⇒ mg(R − r) =
B M 2m

2g(R − r)
⇒ v2 =
m ms-1
M(M + m)
(a) How far has the block moved when the cylinder
reaches the bottom point B of the track?
Example 5: Two balls of masses m and 2m are
(b) How fast is the block moving when the cylinder suspended by two threads of same length l from the
reaches the bottom of the track? same point on the ceiling. The ball m is pulled aside
through an angle α and released after imparting to it a
Sol: As there are no frictional forces acting on the tangential velocity v0 towards the other stationery ball.
system comprises cylinder and block, the gravitational To what heights will the balls rise after collision, if the
potential energy of cylinder is converted into the kinetic collision is perfectly elastic?
energy of cylinder and block.
Sol: In case of perfectly elastic collision, the kinetic
energy of the system is conserved. At the maximum
(a) There are no external forces acting on the system
vertical displacement of the ball the total kinetic energy
comprising cylinder and the block in the horizontal
is converted in to gravitational potential energy.
direction. So we can conserve momentum in the
horizontal direction, so when cylinder reaches point B
on the block, let its velocity in K-frame be v1 towards
right and velocity of block in K-frame be v2 towards left.
So we get 
0= Mv 2 ….(i)
mv1 − Mv 2 or mv1 =
Also the COM of the system was initially at rest and
will continue to remain at rest in absence of horizontal
external forces. When m moves towards right a distance
of (R – r) relative to block M. m
h = l(1-cos )
We can write, 2m
v0
mx1 + Mx2 m ∆x1 + M ∆x2
X cm = = ⇒ ∆X cm = 0
m+M m+M Ball of mass m will collide the ball of mass 2m, which is
initially at rest.
6 . 3 2 | Center of Mass and the Law of Conser vation of Momentum

The velocity of impact of m be v, then by conserving Suppose the velocity of the shot relative to the gun be
energy of m we get u and its velocity relative to the ground be V. The gun
1 1 recoils with a speed v. As the system comprising gun
m v 20 + mgl(1 − cos α ) = mv 2 and the shot rests on a smooth horizontal plane, the
2 2
net horizontal external force will be zero, so conserving
2 2  …(i)
v + 2gl(1 − cos α ) =v
0 momentum in the horizontal direction, taken as the
Conserve momentum of balls before and after collision x – axis, we get
to get,
(nm) v x + mVx =0 ⇒ nm v x + m(ux + v x ) =0
v v1 + 2v 2 
mv mv1 + 2mv 2 or =
= …(ii)
⇒ (nm)( − v) + m(u cos 45 − v) =
0
Equation for coefficient of restitution gives
mu
v v 2 − v1 
= …(iii) ⇒ − (n + 1)mv + =0 …(i)
2
Add (ii) and (iii) to get 2v = 3v2
Again, Vx =−n v x =−n ( − v) ; ⇒ Vx =
nv  …(ii)
2 v −1 v
or v 2 = ms and v1 = − ms−1  …(iv)
3 3 Now the component of the velocity of the gun along
Conserve energy for ‘m’ as it reaches maximum height, the vertical i.e. along the y – axis is zero, so the velocity
2 of the shot along the y – axis will be given by
1 v
m   = mg h1 u
2 3 =Vy u sin 45 + 0 ; Vy =  …(iii)
2
v2 v 2 + 2gl(1 − cos α ) Vy u / 2
or=
h1 = 0 [using (i)] ⇒ tan =
θ = (using (ii) & (iii))
18g 18 g Vx nv
u
Conserve energy for ‘2m’ as it reaches maximum height, ⇒ tan θ =  ….(iv)
2 2nv
1  2v 
2m   = 2mgh2 u
2  3  From (i) we get v =
2(n + 1)
1 4v 2 u
⇒ h2 =. ⇒ = 2 (n + 1)  ….(v)
2g 9 v
2 From (iv) and (v) we get
h2
⇒= [v 02 + 2gl(1 − cos α )] [using (i)]
9g
n+1 n+1
tan
= θ ⇒
= θ tan−1  
Example 6: A gun is mounted on a gun carriage n  n 
movable on a smooth horizontal plane and the gun is
elevated at an angle 45° to the horizontal. A shot is
fired and leaves the gun inclined at an angle θ to the
horizontal. If the mass of gun and carriage is n times
that of the shot, find the value of θ.

Sol: As the frictional force on the cart is zero, the


momentum of cart comprising cart and bullet is
conserved in horizontal direction.

m

o
45
y
nm
x

Let the mass of the shot be m and the mass of the gun
carriage be nm.
P hysi cs | 6.33

JEE Main/Boards

Exercise 1 Q.13 Two particles of masses 1 kg and 3 kg are located


at (2iˆ + 5ˆj + 13k)
ˆ and ( −6iˆ + 4ˆj − 2k)
ˆ meter respectively.
Q.1 Show that center of mass of an isolated system Find the position of their center of mass.
moves with a uniform velocity along a straight line path.
Q.14 Four particles of masses m1 = 1kg , m2 = 2kg ,
Q.2 Locate the center of mass of uniform triangular m3 = 3kg and m4 = 4kg are located at the corners of
lamina and a uniform cone. a rectangle as shown in figure. locate the position of
center of mass.
Q.3 Explain what is meant by center of gravity.
Y
Q.4 Obtain an expression for the position vector of m4 m3
center of mass of a two particle system.

Q.5 Obtain an expression for the position vector of the


center of mass of a system of n particle.
m1
X
O a m2
Q.6 Prove that center of mass of an isolated system
moves with a uniform velocity along a straight line path. Q.15 Find the center of mass of uniform L shaped (a
thin flat plate) with dimensions as shown in figure. The
Q.7 Find the center of mass of three particles at the mass of the lamina is 3 kg.
vertices of an equilateral triangle. The masses of the
particles are 0.10 kg, 0.15 kg and 0.20 kg respectively. Y
Each side of the quilateral triangle is 0.5 m long.
E(1,2)
F(0,2)
Q.8 Find the center of mass of a triangular lamina. C3
D(1,1)
(0,1) B(2,1)
Q.9 Two bodies of masses 0.5 kg and 1 kg are lying in
XY plane at ( –1, 2) and (3, 4) respetively. What are the C1 C2
co-ordinates of the center of mass ? X
O(0,0) (1,0) A(2,0)

Q.10 Three point masses of 1 kg, 2 kg and 3 kg lie


at (1, 2), (0, –1) and (2, –3) respectively. Calculate the
co-ordinates of the center of mass of the system.
Exercise 2
Q.11 Two particles of mass 2 kg and 1 kg are moving
along the same straight line with speeds 2 ms-1 and Single Correct Choice Type
5 ms-1 respectively. What is the speed of the center of
mass of the system if both the particles are moving (a) Q.1 A bullet of mass m moving with a velocity v strikes
in same direction (b) in opposite direction ? a vertically suspended wooden block of mass M and
embedded in it. If the block rises to a height h, the
Q.12 Consider a two-particle system with the particles initial velocity of the bullet will be
having masses m1 and m2 . If the first particle is pushed M+m
towards the center of mass through a distance d, by (A) 2hg (B)   2hg
what distance should the second particle be moved so  m 
as to keep the center of a mass at the same position?  m  M+m
(C)   2hg (D)   hg
 M + m   m 
6 . 3 4 | Center of Mass and the Law of Conser vation of Momentum

Q.2 A body of mass 1 kg, which was initially at rest, Q.8 A body of mass 2kg moving with a velocity of 3
explodes and breaks into three fragments of masses in ms-1 collides head-on with a body of mass 1 kg moving
the ratio of 1 : 1 : 3 . with a velocity of 4 ms-1. After collision the two bodies
stick together and move with a common velocity which
Both the pieces of equal masses fly off perpendicular to
in the units m/s is equal to
each other with a speed of 30 m/s each. The velocity of
the heavier fragment is 1 1 2 3
(A) (B) (C) (D)
10 4 3 3 4
(A) ms-1 (B) 10 2 ms-1
2
Q.9 Two particles of masses M and 2M are at a distance
(C) 20 ms-1 (D) 20 2 ms-1 D apart. Under the mutual gravitational force they start
moving towards each other. The acceleration of their
center of mass when they are D/2 apart is:
Q.3 If the linear momentum of a body is increased by
50%, its kinetic energy will increase by (A) 2GM / D2 (B) 4 GM / D2

(A) 50% (B) 100% (C) 125% (D) 150% (C) 8 GM / D2 (D) Zero

Q.4 Two perfectly elastic particles A and B of equal


Previous Year’s Questions
masses travelling along the line joining them with velcity
25 ms-1and 20 ms-1 respectively collide. Their velocities
after the elastic collision will be (in ms-1) respectively. Q.1 Two particles A and B initially at rest, move towards
each other by mutual force of attraction. At the instant
(A) 0 and 45 (B) 5 and 45
when the speed of A is v and the speed of B is 2v, the
(C) 20 and 25 (D) 25 and 20 speed of the center of mass of the system is  (1982)
(A) 3v (B) v (C) 1.5v (D) zero
Q.5 A body of mass 2.9 kg is suspended from a string
of length 2.5 m and is at rest. A bullet of mass 0.1 kg,
Q.2 A shell is fired from a cannon with a velocity v
moving horizontally with a speed of 150 ms-1 strikes
(ms-1) at an angle θ with the horizontal direction. At
and sticks to it. What is the maximum angle made by
the highest point in its path it explodes into two pieces
the string with the vertical after the impact?
of equal mass. One of the pieces retraces its path to
(g =10 ms-2) the cannon and the speed (ms-1) of the other piece
(A) 30° (B) 45° (C) 60° (D) 90° immediately after the explosion is (1986)
(A) 3v cos θ (B) 2v cos θ
Q.6 An isolated particle of mass m is moving in a
3 3
horizontal plane (x-y), along the x-axis, at a certain (C) v cos θ (D) v cos θ
height above the ground. It suddenly explodes into two 2 2
m 3m
fragments of mass and . An instant later, the Q. 3 Two particles of masses m1 and m2 in projectile
4 4  
smaller fragment is at y = +15 cm. The larger fragment motion have velocities v1 and v 2 respectively at time

at this instant is at t = 0. They collide at time t0 .Their velocities become v1′

(A) y = -5 cm (B) y = +20 cm and v 2′ at time 2t0 while still moving in air. The value of
   
| (m1 v1′ + m2 v 2′ ) − (m1 v1 + m2 v 2 ) | is  (2001)
(C) y = +5 cm (D) y = -20cm
(A) Zero (B) (m1 + m2 )gt0
Q.7 A ball collides elastically with another ball of the 1
same mass. The collision is oblique and initially one (C) 2(m1 + m2 )g t0 (D) (m + m2 )g t0
2 1
of the ball was at rest. After the collision, the two
balls move with same speeds. What will be the angle
between the velocity of the balls after the collision? Q.4 Two blocks of masses 10 kg and 4 kg are
connected by a spring of negligible mass and placed
(A) 30° (B) 45° (C) 60° (D) 90° on a frictionless horizontal surface. An impulse gives a
velocity of 14 ms-1 to the heavier block in the direction
P hysi cs | 6.35

of the lighter block. The velocity of the center of mass of 20 m and the bullet at the distance of 100 m from
is  (2002) the foot of the post. The initial velocity v of the bullet
is  (2011)
(A) 30 ms-1 (B) 20 ms-1
v m/s
(C) 10 ms-1 (D) 5 ms-1

Q.5 Look at the drawing given in the figure, which has


been drawn with ink of uniform line-thickness. The
mass of ink used to draw each of the two inner circles, 0 20 100
and each of the two line segments is m. The mass of the
ink used to draw the outer circle is 6m. The coordinates (A) 250 ms-1 (B) 250 2 ms-1
of the center of the different parts are: outer circle (C) 400 ms-1 (D) 500 ms-1
(0, 0), left inner circle (-a, a), right inner circle (a,
a), vertical line (0, 0) and horizontal line (0, -a). The
Paragraph: Q.8 - Q.10
y-coordinate of the center of mass of the ink in this
drawing is  (2009) A small block of mass M moves on a frictionless surface
y of an inclined plane, as shown in figure. The angle of
the incline suddenly changes from 60° to 30° at point B.
the block is initially at rest at A. Assume that collisions
between the block and the incline are total inelastic
(g = 10 ms−2 )
A
x M

v
o
60 B
a a a a
(A) (B) (C) (D)
10 8 12 3
o
30 C
Q.6 Two small particles of equal masses start moving
in opposite directions from a point A in a horizontal 3 3 3m
circular orbit. Their tangential velocities are v and 2v
respectively, as shown in the figure. Between collisions, Q.8 The speed of the block at point B immediately after
the particles move with constant speed. After making it strikes the second incline is  (2008)
how many elastic collisions, other than that at A, these
(A) 60 ms-1 (B) 45 ms-1
two particles will again reach the point A?  (2009)

v A (C) 30 ms-1 (D) 15 ms-1

2v
Q.9 The speed of the block at point C, immediately
before it leaves the second incline is  (2008)

(A) 120 ms-1 (B) 105 ms-1

(C) 90 ms-1 (D) 75 ms-1

(A) 4 (B) 3 (C) 2 (D) 1 Q.10 If collision between the block and the incline
is completely elastic, then the vertical (upward)
Q.7 A ball of mass 0.2 kg rests on a vertical post of component of the velocity of the block at point B,
height 5 m. a bullet of mass 0.01 kg, travelling with a immediately after it strikes the second incline is ( 2008)
velocity v ms-1 in a horizontal direction, hits the center
(A) 30 ms-1 (B) 15 ms-1
of the ball. After the collision, the ball and bullet travel
independently. The ball hits the ground at a distance (C) Zero (D) − 15 ms-1
6 . 3 6 | Center of Mass and the Law of Conser vation of Momentum

Q.11 This question has Statement-I and Statement-II. (D) Statement-I is true, statement-II is true, statement-II
Of the four choices given after the Statements, choose is a correct explanation of statement-I.
the one that best describes the two Statements.
Statement-I: A point particle of mass m moving with Q.12 Distance of the centre of mass of a solid uniform
speed v collides with stationary point particle of mass cone from its vertex is z 0 . If the radius of its base is R
M. If the maximum energy loss possible is given as and its height is h then z 0 is equal to: (2015)
1   m  3h 5h 3h2 h2
f  mv 2  then f =  . (A) (B) (C) (D)
 2  M+m 4 8 8R 4R
Statement-II: Maximum energy loss occurs when the
particles get stuck together as a result of the collision. Q.13 A particle of mass m moving in the x direction
 (2013) with speed 2v is hit by another particle of mass 2m
moving in the y direction with speed v. If the collision
(A) Statement-I is true, statement-II is true, statement-II is perfectly inelastic, the percentage loss in the energy
is not a correct explanation of statement-I. during the collision is close to (2015)
(B) Statement-I is true, statement-II is false.
(A) 50% (B) 56% (C) 62% (D) 44%
(C) Statement-I is false, statement-II is true

JEE Advanced/Boards

Exercise 1 45° and 135° respectively with line AB. Each particle has
an initial speed of 49 ms-1. The separation AB is 245
Q.1 A block of mass 10 kg is suspended from a 3 m m. Both particles travel in the same vertical plane and
long weightless string. A bullet of mass 0.2 kg is fired undergo a collision. After the collision, P retraces its
into the block of horizontally with a speed of 20 ms-1 path. Taking g = 9.8 ms-2, determine
and it gets embedded in the block. Calculate (a) The position of Q when it hits the ground.
(a) The speed acquired by the block (b) How much time, after the collision, does Q take to
(b) The maximum displacement of the block reach the ground.

(c) The energy converted to heat in the collision.


P
Q.2 A projectile of mass 50 kg shot vertically upwards o o O
45 45 o
135
with an initial velocity of 100 ms-1. After 5 s it explodes
B
into two fragments, one of which having mass 20 kg
travels vertically up with a velocity of 150 metres/sec. Q.4 A shell of mass 500 kg travelling horizontally at a
if g = 9.8 ms-2. speed of 100 ms-1 explodes into just three parts. The
first part of mass 200 kg travels vertically upwards
(a) What is the velcoity of the other fragment at that at a speed of 150 ms-1 and the second part of mass
instant ? 150 kg travels horizontal with a speed of 60 ms-1, but in
(b) Calculate the sum of the momenta of the two a direction opposite to that of the original shell. What
fragments 3 s after the explosion. What would have is the velocity fo the third part? What is the path of the
been the momentum of the projectile at this instant if center of mass of the fragments after the explosion?
there had been no explosion?
Q.5 A small sphere of mass 10 g is attached to a point
Q.3 Particle P and Q of mass 20 g and 40 g respectively of smooth vertical wall by a light string of length 1 m.
are simultaneously projected from points A and B on The sphere is pulled out in vertical plane perpendicular
the ground. The initial velocities of P and Q make angle to the wall so that the string makes an angle of 60°
P hysi cs | 6.37

with the wall and is then released. It is found that after Q.10 Prove that in case of oblique elastic collision of
the first rebound, the string makes a maximum angle of two particles of equal mass out of which one is at rest,
30° with the wall. Calculate the coefficient of restitution the recoiling particles always move off at right angles
and the loss of kinetic energy due to impact. If all the to each other.
energy is converted into heat, find the heat produced
by the impact. Q.11 A uniform thin rod of mass M and length L
is standing vertically along the y-axis on a smooth
Q.6 A small ball A slides down the quadrant of a circle hroizontal surface, with its lower end at the origin (0,0).
as shown in the figure and hits the ball B of equal mass A slight disturbance at t = 0 causes the lower end to slip
which is initially at rest. Find the velocities of both the on the smooth surface along the positive x-axis, and
balls after collision. Neglect the effect of friction and the rod starts falling.
assume the collision to be elastic.
(a) What is the path followed by the center of mass of
A the rod during its fall?
10 cm
(b) Find the equation of trajectory of a point on the rod
located at a distance r from the lower end.
10 cm

Q.12 Two blocks of masses m1 and m2 are connected


B
by a light inextensible string passing over a smooth
fixed pulley of negligible mass. Find the acceleration
of the center of mass of the system when blocks move
under gravity.
Q.7 Two balls A and B of mass 0.10 kg and 0.25 kg
respectively are connected by a stretched spring of Q.13 A block of mass m is resting on the top of a
negligible mass and spring constant 2 Nm-1. Unstretched smooth prism of mass M which is resting on a smooth
length of the spring is 0.6 m and placed on a smooth table. Calculate the distance moved by the prism when
table. When the balls are released simultaneously the the block reaches the bottom.
initial acceleration of ball B is 50 cm s-2 west-ward.
m
(a) What is the magnitude and direction of the initial
acceleration of the ball A?
(b) What is the initial compression of the spring.
M
(c) What is the maximum distance between balls A and B. 

Q.8 Find the center of mass of a uniform disc of radius Q.14 A shell is fired from a cannon with a velocity v
a from which a circular section of radius b has been m/s at an angle θ with the horizontal direction. At the
removed. The center of the hole is at a distance c from, highest point of its path is explodes into two pieces
the center of the disc. of equal masses. What is the speed of other piece
immediately after explosion, if one of the piece retraces
its path to the cannon?
Q.9 A man of mass m climbs a rope of length L
suspended below a balloon of mass M. The ballon is
stationary with respect to ground, Q.15 A particle of mass 4m which is at rest explodes into
three fragments. Two of the fragments each of mass
(a) If the man begins to climb up the rope at a speed vrel
m are found to move with a speed v each is mutually
(relative to rope) in what direction and with what speed
perpendicular directions. Calculate the energy released
(relative to ground) will the balloon move?
in the process of explosion.
(b) How much has the balloon by climbing the rope.
(c) What is the state of motion after the man stops Q.16 A moving particle of mass m makes a head on
climbing? elastic collision with a particle of mass 2m which is
initially at rest. Show that the colliding particle losses
(8/9)th of its energy after collision.
6 . 3 8 | Center of Mass and the Law of Conser vation of Momentum

Q.17 A ball is dropped on the ground from a height h. Q.21 A 60 kg man and a 50 kg woman are standing on
If the coefficient of restitution is e, then find the total opposite ends of a platform of mass 20 kg. The platform
distance travelled by the ball before coming to rest and is placed on a smooth horizontal ground. The man and
the total time elapsed. the woman begin to approach each other. Find the
displacement of the platform when the two meet in
Q.18 A block of mass m1 = 150 kg is at rest on a very terms of the displacement x0 of the man relative to the
long frictionless table, one end of which is terminated platform. The length of the platform is 6m.
in a wall. Another block of mass m2 is placed between
the first block and the wall, and set in motion towards Q.22 A rope thrown over a pulley has a ladder with a
m1 with constant speed u2 . man A on one of its ends and a counter balancing mass
M on it other end. The man whose mass is m, climb

upwards by ∆ r relative to the ladder and the stops.
150 kg m1 2 Ignoring the masses of the pulley and the rope, as well
m2
as the friction in the pulley axis, find the displacement
of the center of mass of this system.
Assuming that all collisions are completely elastic, find
3a
the value of m2 for which both blocks move with the Q.23 A drinking straw of length and mass 2m is
2
same velocity after m2 has collided once with m1 and
once with the wall. The wall has effectively infinite mass. placed on a square table of side ‘a’ parallel to one of its
sides such that one third of its length extends beyond
m
Q.19 A simple pendulum is suspended from a peg on a the table. An insect of mass lands on the inner end
2
vertical wall. The pendulum is pulled away from the wall
to a horizontal position and released. The ball hits the of the straw (i.e., the end which lies on the table) and
walks along the straw until it reaches the outer end.
 2  It does not topple even when another insect lands on
wall, the coefficient of restitution being  .
 5 top of the first one. Find the largest mass of the second
What is the minimum number of collisions after which insect that can have without toppling the straw. Neglect
the amplitude of oscillation becomes less than 60°? friction.

Q.20 A block A of mass 2m is placed on another block Q.24 A boy throws a ball with initial speed 2 ag at an
b of mass 4 m which in turn in placed on a fixed table. angle θ to the horizontal. It strikes a smooth vertical
The two blocks have the same length 4d and they are wall and returns to his hand. Show that if the boy is
placed as shown in the figure. standing at a distance ‘a’ from the wall, the coefficient
of restitution between the ball and the wall equals
1
A 2m . Also show that θ cannot be less than 15°.
(4 sin2θ − 1)
m v B 4m
2d Q.25 A ball is projected from a point A on a smooth
d CM
P inclined plane which makes an angle α to the
4d horizontal. The velocity of projection makes an angle
θ with the plane upwards. If on the second bounce
The coefficient of friction (both static and kinetic)
the ball is moving perpendicular to the plane, find e in
between the block B and the table is µ. There is no
terms of α and θ . Here e is the coefficient of restitution
friction between the two blocks. A small object of mass
between the ball and the plane.
m moving horizontally along a line passing through the
center of mass of the block B and perpendicular to its
face with a speed v collides elastically with the block B Q.26 Two identical smooth balls are projected toward
at a height d above the table. each ther from points A and B on the horizontal ground
with same speed of projection. The angle of projection.
(a) What is the minimum value of v (call it v0), required
to make the block A topple? The angle of projection in each case is 30°. The distance
between A and B is 100 m. The balls collide in air
(b) If v = 2v0 find the distance (from the point P) at and return to their respective points of projection. If
which the mass m falls on the table after collision. coefficient of restitution is e = 0.7, find
P hysi cs | 6.39

(a) The speed of projection of either ball. Q.30 Ball B is hanging from an inextensible cord BC.
An identical ball A is released from rest when it is just
(b) Coordinates of point with respect to A where the
touching the cord and acquires a velocity v 0 before
balls collide.(Take g = 10 ms-2)
striking ball B. Assuming perfectly elastic impact
(e = 1) and no friction, determine the velocity of each
Q.27 Three identical particles A, B and C lie on a smooth ball immediately after impact.
horizontal table. (see figure) Light inextensible strings
which are just taut connect AB and BC and ∠ABC is
Q.31 A particle whose initial mass is m0 is projected
135°. An impulse J is applied to the particle C in the
vertically upwards at time t = 0 with speed gT, where
direction BC.
T is a constant. The particle gradually acquires mass
Find the initial speed of each particle. The mass of each on its way up and at time t the mass of the particle
particle in m. has increased to m0 edT . If the added mass is at rest
J relative to the particle when it is acquired, find the time
when the particle is at highest point and its mass at
135
o C that instant.

A B Q.32 Two blocks of mass 2kg and M are at rest on an


Q.28 A 2 kg sphere A is connected to a fixed point O inclined plane and are separated by a distance of 6.0
by an inextensible cord of length 1.2 m (see figure). The m as shown. The coefficient of friction between each
sphere is resting on a frictionless horizontal surface at a block and the inclined plane is 0.25. the 2 kg block is
distance of 0.5 m from O when it is given a velocity v 0 in given a velocity of 10.0 ms-1 up the inclined plane.
a direction perpendicular to the line OA. It moves freely
until it reaches position A’ when the cord becomes taut. M
A’ m
6.0
g
2m

2k
1.

v0

O A 
Determine
It collides with M, comes back and has a velocity of
(a) The maximum allowable velocity v 0 if the impulse 1.0 m/s when it reaches its initial position. The other
of the force exerted on the cord is not to exceed 3 Ns. blocks M after the collision moves 0.5 m up and comes
(b) The loss of energy as the cord becomes taut, if the to rest. Calculate the coefficient of restitution between
sphere is given the maximum allowable velocity v 0 . the blocks and the mass of the block M.
(Take sin θ ≈ tan θ =0.05 and g = 10 ms-2)
Q.29 An open car of mass 1000 kg is running at 25 m/s
holds three men each of mass 75 kg. Each man runs Exercise 2
with a speed of 5 ms–1 relative to the car and jumps off
from the back end. Find the speed of the car if the three Single Correct Choice Type
men jump off.
Q.1 A bullet of mass m moving with a velocity v strikes
C
a vertically suspended wooden block of mass M and
A embedded in it. If the block rises to a height h, the
initial velocity of the bullet will be
v0
M+m
(A) 2hg (B)   2hg
B  m 
(a) In succession
 m  M+m
(C)   2hg (D)   hg
(b) All together. M+m  m 
Neglect friction between the car and the ground.
6 . 4 0 | Center of Mass and the Law of Conser vation of Momentum

Q.2 Two identical billiard balls A and B of equal mass Q.7 A body of mass m1 and speed v1 makes a head-on,
and radius are in contact on a horizontal table. A elastic collision with a body of mass m2, initially at rest.
similar third ball C strikes these balls symmetrically in The velocity of m1 after the collision is
the middle and remains at rest after the impact, the
m1 + m2 m1 − m2
coefficient of restitution of the balls is (A) v1 (B) v1
m1m2 m1 + m2

A 2m1 v1 2m2 v1
(C) (D)
C m1 + m2 m1 + m2
B
Q.8 In the above example, the velocity of mass m2 after
1 1 2 3 the collision is
(A) (B) (C) (D)
6 3 3 2 m1 + m2 m1 − m2
(A) v1 (B) v1
m1m2 m1 + m2
Q.3 A sphere of mass m moving with a constant velocity
u hits another stationary sphere of the same mass. If e 2m1 v1 2m2 v1
is the coefficient of the restitution, then the ratio of the (C) (D)
m1 + m2 m1 + m2
velocities of the two spheres after collision will be

1 −e 1 + e e+1  e −1  Q.9 A ball of mass m approaches a moving wall of


(A)   (B)   (C)   (D)   infinite mass with speed v along the normal to the wall.
1 + e 1 −e  e −1  e+1
The speed of the wall is u towards the ball. The speed of
the ball after an elastic collision with the wall is
Q.4 A cannon ball is fired with a velocity of 200 ms-1
(A) u + v away from the wall
at an angle of 60° with the horizontal. At the highest
point it explodes into three equal fragments. One (B) 2u + v away from the wall
goes vertically upwards with a velocity of 100 ms-1, the
(C) u – v away from the wall
second one falls vertically downwards with a velocity of
100 ms-1. The third one moves with a velcoity of (D) v – 2u away from the wall.
(A) 100 ms-1 horizontally
Q.10 A neutron is moving with velocity u. It collides
(B) 300 ms-1 horizontally
head on and elastically with an atom of mass number
(C) 200 ms-1 at 60° with the horizontal A. If the initial K.E. of the neutron is E, how much K.E. is
retained by neutron after collision?
(D) 300 ms-1at 60° with the horizontal
(A) [A / (A + 1)]2 E B) [A / (A + 1)2 ]E
Q.5 A bullet of mass 0.01 kg, travelling at a speed of 500
m/s, strikes a block of mass 2kg, which is suspended by (C) [(1 − A) / (A + 1)2 ]E (D) [(A − 1) / (A + 1)2 ]E
a string of length 5 m, and emerges out. The block rises
by a vertical distance of 0.1 m. The speed of the bullet Q.11 A ball is dropped from a height h on the ground.
after it emerges from the block is If the coefficient of restitution is e, the height to which
(A) 55 ms-1 (B) 110 ms-1 the ball goes up after it rebounds for the nth time is

(C) 220 ms-1 (D) 440 ms-1 e2n h


(A) he2n (B) he2 (C) (D)
h e2n
Q.6 A 1 kg ball, moving at 12 ms-1collides head-on with
a 2 kg ball moving in the opposite direction at 24 m/s. If Q.12 Two equal spheres A and B lie on a smooth
horizontal circular groove at opposite ends of diameter.
2
the coefficient of restitution is , then the energy lost A is projected along the groove and at the end of time
in the collision is 3
t impinges on B. If e is coefficient of restitution, the
(A) 60 J (B) 120 J second impact will occur after a time
2t t πt 2πt
(C) 240 J (D) 480 J (A) (B) (C) (D)
e e e e
P hysi cs | 6.41

Q.13 The center of mass of triangle shown in the figure. (B) If both A and R are true but R is not the correct
has co-ordinates. explanation of A.
y (C) If A is true but R is false
(D) If A is flase but R is true

h Q.16 Assertion: When two bodies of different masses


are just released from different position above the
ground, then acceleration of their center of mass is
x
b zero.
h b b h Reason: When bodies move, their center may change
(A)
= x = ;y (B)=x = ;y
2 2 2 2 position but is not accelerated.

b h h b
(C)
= x = ;y (D)
= x = ;y Q.17 Assertion: The center of mass of a proton and
3 3 3 3 an electron, released from their respective positions
remains at rest.
Q.14 A cart of mass M is tied to one end of a massless
rope of length 10 m. The other end of the rope is in the Reason: The proton and electron attract and move
hands of a man of mass M, the entire system is on a towards each other. No external force is applied,
smooth horizontal surface. The man is at x = 0 and the therefore, their center of mass remains at rest.
cart at x = 10 m. if the man pulls the cart by a rope, the
man and the cart will meet at the point : Q.18 Assertion: The center of mass of a body may lie
where there is no mass.
(A) x = 0 (B) x = 5 m
Reason: Center of mass of a body is a point, where the
(C) x = 10 m (D) They will nevemeet
whole mass of the body is supposed to be concentrated.

Multiple Correct Choice Type


Q.19 Assertion: When a body dropped from a height
Q.15 Which one of the following statements does explodes in mid air, the pieces fly in such a way that their
not hold god when two balls of masses m1 and m2 center of mass keeps moving vertically downwards.
undergo elastic collision? Reason: Explosion occurs under internal forces only.
(A) when m1 < m2 and m2 at rest, there will be External force = 0.
maximum transfer of momentum.
Q.20 Assertion: The center of mass of a circular disc
(B) when m1 > m2 and m2 at rest, after collision the
lies always at the center of the disc.
ball of mass m2 moves with four times the velocity of
m1 Reason: Circular disc is a symmetrical body.
(C) when m1 = m2 and m2 at rest, there will be
maximum transfer of K.E. Q.21 Assertion: At the center of earth, a body has
center of mass, but no center of gravity.
(D) when collision is oblique and m2 at rest with
m1 = m2 , after collision the ball moves in opposite Reason:This is because g = 0 at the center of earth.
directions.
Q.22 Assertion: The center of mass of a body may lie
Assertion Reasoning Type where there is no mass.
Reason: The center of mass has nothing to do with the
Each of the questions given below consists of two
mass.
statements, an assertion (A) and reason (R). Select the
number corresponding to the appropriate alternative
as follows.
(A) If both A and R are true and R is the correct
explanation of A.
6 . 4 2 | Center of Mass and the Law of Conser vation of Momentum

Comprehension Type (C) The total momentum of the particles after collision
is less than that before collision.
In physics, we come across many examples of collisions.
The molecules of a gas collide with one another and (D) Kinetic energy and momentum are both conserved
with the container. The collisions of a neutron with an in the collision.
atom is well known. In a nuclear reactor, fast neutrons
produced in the fission of uranium atom have to be Q.25 In elastic collision
slowed down. They are, therefore, made to collide with
hydrogen atom. The term collision does not necessarily (A) Only energy is conserved.
mean that a particle or a body must actually strike (B) Only momentum is conserved.
another. In fact, two particles may not even touch each
other and yet they are said to collide if one particle (C) Neither energy nor momentum is conserved.
influences the motion of the other. When two bodies (D) Both energy and momentum are conserved.
collide, each body exerts an equal and opposite force
on the other. The fundamental conservation law of
physics are used to determine the velocities of the Previous Years’ Questions
bodies after the collision. Collision may be elastic or
inelastic. Thus a collision may be defined as an event in
which two or more bodies exert relatively strong forces Q.1 A body of mass m moving with a velocity v in
on each other for a relatively short time. The forces the x-direction collides with another body of mass
that the bodies exert on each other are internal to the M moving in the y-direction with a velocity V. They
system. coalesce into one body during collsion. Find

Almost all the knowledge about the sub-atomic (a) The direction and magnitude of the momentum of
particles such as electrons, protons, neutrons, muons, the composite body.
quarks, etc. is obtained from the experiments involving (b) The fraction of the initial kinetic energy transformed
collisions. into heat during the collision. (1978)
There are certain collisions called nuclear reactions in
which new particles are formed. For example, when a Q.2 A 20 g bullet pierces through a plate of mass
slow neutron collides with a U235 nucleus, new nuclei M1 = 1kg and then comes to rest inside a second plate
barium-141 and Kr92 are formed. This collisioin is called of mass M2 = 2.98kg as shown in the figure. It is found
nuclear fission. In nuclear fusion, two nuclei deuterium that the two plates initially at rest, now move with equal
and tritium collide (or fuse) to form a helium nucleus velocities. Find the percentage loss in the initial velocity
with the emission of a neutron. of the bullet when it is between M1 and M2 . Neglect
any loss of material of the plates due to the action of
Q.23 Which one of the following collisions is not elastic? bullet. Both plates are lying on smooth table. (1979)

(A) A hard steel ball dropped on a hard concrete floor


and rebounding to its original height.
(B) Two balls moving in the same direction collide and
stick to each other
(C) Collision between molecules of an ideal gas.
(D) Collisions of fast neutrons with hydrogen atoms in
a fission reactor.

M1 M2
Q.24 Which one of the following statemnts is true
about inelastic collision?
Q.3 A circular plate of uniform thickness has a diameter
(A)The total kinetic energy of the particles after collision of 56 cm. A circular portion of diameter 42 cm is
is equal to that before collision. removed from one edge of the plate as shown in figure.
Find the position of the center of mass of the remaining
(B) The total kinetic energy of the particle after collision
portion.  (1980)
is less than that before collision.
P hysi cs | 6.43

Q.7 A uniform thin rod of mass M and length L


is standing vertically along the y-axis on a smoth
horizontal surface, with its lower end at the origin
(0, 0). A slight disturbance at t = 0 causes the lower end
to slip on the smooth surface along the positive x-axis,
and the rod starts falling.
(a) What is the path followed by the center of mass of
42cm the rod during its fall?
56cm (b) Find the equation of the trajectory of a point on the
rod located at a distance r from the lower end. What is
Q.4 Three particles A, B and C of equal mass move
the shape of the path of this point?  (1993)
with equal speed v along the medians of an equilateral
triangle as shown in figure. They collide at the centroid
G of the triangle. After the collision, A comes to rest, B Q.8 A wedge of mass m and triangular cross-section
retraces its path with speed v. What is the velocity of C? (AB= BC = CA= 2R) is moving with a constant velocity
 (1982) ˆ towards a sphere of radius R fixed on a smooth
( − vi)
A horizontal table as shown in the figure. The wedge
makes an elastic collision with the fixed sphere and
returns along the same path without any rotation.
Neglect all friction and suppose that the wedge
remains in contact with the sphere for a very short time

G ∆t during which the sphere exerts a constant force F
B C on the wedge. (1998)

Q.5 Two bodies A and B of masses m and 2m respectively y A


are placed on a smooth floor. They are connected by a R v
spring. A third body C of mass m moves with velocity
v 0 along the line joining A and B and collides elastically
with A as shown in figure. At a certain instant of time x B C
t0 after collision, it is found that the instantancous  
velocities of A and B are the same. Further at this (a) Find the force F and also the normal force F exerted
instant the compression of the spring is found to be x0 by the table on the wedge during the time ∆t .
. Determine (a) the common velocity of A and B at time
(b) Let h denote the perpendicular distance between
t0 and (b) the spring constant.  (1984)
the center of mass of the wedge and the line of action
A of force. Find the magnitude of the torque due to the
C 
normal force N about the center of the wedge during
the interval Dt.

Q.6 A simple pendulum is suspended from a peg on a


Q.9 Three objects A, B and C are kept in a straight line
vertical wall. The pendulum is pulled away from the wall
on a frictionless horizontal surface (see figure). These
to a horizontal position (see figure) and released. The
have masses m, 2m and m, respectively. The object A
2
ball hits the wall, the coefficient of restitution being . moves towards B with a speed 9ms−1 and makes an
5 elastic collision with it. Thereafter, B makes completely
What is the minimum number of collisions after which inelastic collision with C. All motions occur on the same
the amplitude of oscillations becomes less than 60 straight line. Find the final speed (in ms−1 ) of the object
degrees? (1987) C. (2009)

l
6 . 4 4 | Center of Mass and the Law of Conser vation of Momentum

Q.10 A particle of mass m is projected from the Q.11 A bob of mass m , suspended by a string of length
ground with an initial speed u0 at an angle α with the l1, is given a minimum velocity required to complete
horizontal. a full circle in the vertical plane. At the highest point,
At the highest point of its trajectory, it makes a it collides elastically with another bob of mass m
completely inelastic collision with another identical suspended by a string of length l2, which is initially at
particle, which was thrown vertically upward from rest. Both the strings are mass-less and inextensible. If
the ground with the same initial speed u0 . The angle the second bob, after collision acquires the minimum
that the composite system makes with the horizontal speed required to complete a full circle in the vertical
immediately after the collision is  (2013, 14, 15, 16) plane, the ratio l1 / l2 is  (2013)

π π π π
(A) (B) + α (C) − α (D)
4 4 4 2

PlancEssential Questions
JEE Main/Boards JEE Advanced/Boards
Exercise 1 Exercise 1
Q. 7 Q.9 Q.16 Q.3 Q.6 Q.7
Q.10 Q.20 Q.21
Q.28 Q.32

Exercise 2 Exercise 2
Q.1 Q.9 Q.1 Q.2 Q.3
Q.4 Q.9

Previous Years' Questions Previous Years' Questions


Q.2 Q.3 Q.5 Q.2 Q.5 Q.8
Q.8 Q.9 Q.10
P hysi cs | 6.45

Answer Key

JEE Main/Boards

Exercise 1

5 10 7 3
Q.9 , Q.10 ,−
3 3 6 2
1
Q.11 (a) 3 ms-1 (b) ms-1 in the direction of motion of 1 kg
3
m1 17 ˆ 7 ˆ 5 5
Q.12 d Q.13 − ˆi + j+ k Q.14 0.5aiˆ + 0.7b ˆj Q.15 m; m
m2 4 4 6 6

Exercise 2

Single Correct Choice Type


Q.1 B Q.2 B Q.3 C Q.4 C Q.5 C Q.6 A
Q.7 D Q.8 C Q.9 D

Previous Years Questions


Q.1 D Q.2 A Q.3 C Q.4 C Q.5 A Q.6 C
Q.7 D Q.8 B Q.9 B Q.10 C Q.11 C Q.12 A
Q.13 B

JEE Advanced/Boards
Exercise 1
Q.1 (a) 0.39 m/s (b) 0.220 m (c) 39.32 J Q.2 15 m/s, 1080 kg ms-1

Q.3 (a) 122.5 m (b) 5 2 second. Q.4 441.25 m/s, −27°

Q.5 0.518, 0.0359 J, 0.0085 cals Q.6 vA = 0, vB = 1.4 m/s

Q.7 (a) 1.25 cm/s2 (eastwards) (b) 6.25 cm (c) 66.25 cm

cb2
Q.8 At a distance from O on the other side of the hole.
a2 − b2
 m
Q.9 (a) −mv rel / (M + m) (b) L (c) system is stationary
M+m

x2 y2
Q.11 (a) Straight line (b) + =1
[L / 2 − r]2 r2
2
 m − m2  mhcot θ
Q.12  1  g Q.13 Q.14 3v cos θ
 m1 + m2  M + m
6 . 4 6 | Center of Mass and the Law of Conser vation of Momentum

3 h(1 + e2 ) 2h  1 + e 
Q.15 mv 2 Q.17 ,   Q.18 50 kg
2 1−e 2 g 1 − e 

5 30 − 11x0
Q.19 4 Q.20 (a) 6µgd (b) = −6d 3µ Q.21
2 13

m  m cot θ cot α


Q.22 ∆r Q. 23 m'− m = Q.25 −1
2M 2 2

2 10 3J
Q.27 , , Q.28 (a) 1.65 m/s (b) 2.25 J Q.29 (a) 25.97 m/s (b) 25.92 m/s
7m 7m 7m

Q.30 | v 'B | = 0.721v 0 , | v 'A | = 0.693v 0

16.1o

Q.31 T ln (2), 2m0 Q.32 0.84, 15.011 kg


Exercise 2

Single Correct Choice Type


Q.1 B Q.2 C Q.3 A Q.4 B Q.5 C Q.6 C
Q.7 B Q.8 C Q.9 B Q.10 C Q.11 A Q.12 A
Q.13 C Q.14 B

Multiple Correct Choice Type


Q.15 C, D

Assertion Reasoning Type


Q.16 D Q.17 A Q.18 B Q.19 A Q.20 D Q.21 A
Q.22 B

Comprehension Type
Q.23 B Q.24 B Q.25 D

Previous Years’ Questions

MV ∆K Mm(v 2 + V 2 )
Q.1 (a) θ =tan−1 = ,P m2 v 2 + M2 V 2 (b) = Q.2 25%
mv K i (M + m)(mv 2 + MV 2 )
2mv 20
Q.3 9 cm Q.4 Opposite to velocity of B Q.5 (a) v0/3 (b)
3x20
2 2
x y
Q.6 4 Q.7 (a) a straight line (b) + 1
=
2
L  r2
 −r
 2 

2mv ˆ 2mv ˆ  2mv  4 mvh


Q.8=
(a) F i− =k, N  + mg  kˆ (b) ⇒ | τN | =
∆t 3∆t  3∆t  3∆t

Q.9 4 ms-1 Q.10 A Q.11 5


P hysi cs | 6.47

Solutions

JEE Main/Boards

Exercise 1 4  3a3 a3  4 3a3


= ×2×  −  = ×
3a2  16 8  3a2 16
Sol 1: Isolated system, so external force = 0 a
= (from bottom)
Fext = 0, therefore acceleration of centre of mass = 0. 2 3
So the centre of mass moves with a constant velocity (ii)
along a straight line path (Ist law of motion).
 (h–x)
Sol 2: (i) Lamina: mass per unit area r h
M dx
= = ρ (say) h
3a2 x
r
4 3
a
2
(ρ.dA).x r
then XCOM = ∫ tan θ =
0
M h
3M mass
Let ρ = =
πr h 2 volume
a 30º
a then dm = rdV, where dV = volume of shaded region
dx
dV = π. r2. dx
x
π.(h − x)2 .r 2
a = π × [(h – x). tanq] 2 dx = . dx
h
h2
where dA = area of the strip of thickness dx (shaded
reg.)
∫ x dm h
x.ρ.dV x.dV
h
so, xCOM = 0
M
= ∫ M
= ρ. ∫
M
 3  0 0
= 2 a − x  . tan 30º dx
 2  h h
  3M 3 π.x.(h − x)2 .r 2
a x 
=
πr 2h.M 0
∫ x.dV = πr 2h 0
∫ h2
dx
dA = 2  −  . dx
2 3 h
3 2 3 h4 h
3a a x 
=
h3
∫ x.(h − x) dx = .
h3 12
; xCOM =
4
ρ×2× −  .x.dx 0
2
2
3
so, XCOM = ∫
0
M Sol 3: Centre of gravity is the point at which all the
force of gravity is assumed to be applied i.e., there is a
3a
2 
force of gravity on each point of the body and hence
ρ ax x2  the complications are reduced by finding a point where
= ×2× ∫  −  dx
M  2 3  all the force is assumed to be applied, this point is
0 
centre of gravity.
3a
4  ax2 x2  2
= ×2×  −  Sol 4: Now we have
3a2  4 3  0
Mtot a = m1 a + m2 a
COM 1 2
4 a 3 1 3 3a3 
= × 2 ×  × a2 − ×  where m = mass
3a2  4 4 3 3 8 
6 . 4 8 | Center of Mass and the Law of Conser vation of Momentum

a = acceleration 25 + 180 125


   = =
∂VCOM ∂V1 ∂V2 450 450
so MTot. = m1 + m2 
dt dt dt xCOM = 0. 277 î
x dt, on integrating w. r. t dt, we get   
    m y + m2 y 2 + m3 y 3
MTot. V = m1 V1 + m2 V2 y COM = 1 1
COM MTot.
  
∂x ∂x1 ∂x2 150 × 0 + 200 × (0) + 100 × 0.433 43.3
⇒MTot. COM = m1 + m2 = =
dt dt dt 450 450

⇒ On multiplying by dt, and integrating y COM = 0. 096 ˆj
 
MTot x = m1 x1 + m2 x2 
So rCOM = (0.277 ˆi + 0.096 ˆj)
COM
 
 m x + m2 x2
So xCOM = 1 1 Sol 8:
Mtot. A

 a  a
Sol 5: MTot. . aCOM = m1 a + m2 a + …. mn a
1 2 n F dx
(Now, just like above question, question-4, we can find
that
 x
   B b C
 m x + m2 x2 + ....mn xn
xCOM = 1 1
Mtot. b 4a2 − b2
sinθ = , cosθ = ,
2a 2a
Sol 6: We have
 b
Mtot. . aCOM =m1 a + m2 a + …. mn a tanθ =
1 2 n
 4a2 − b
Now, aCOM = 0, so we have
M M
   Now, ρ = =
dV1 dV2 dVn 1 2 2
a sin θ cos θ
0 = m1 + m2 + ....mn a sin2θ
dt dt dn 2
EF 
x dt, and integrating, we get
    = 2(acos
 θ − x).
 tan θ
C = m1 V1 + m2 V2 + .....mn Vn =MTot. VCOM AE
(AD −DE)
C
So VCOM = = constant So, dA = . dx
MTot.
Hence proved. So, dm = ρ. dA = ρ. . dx
acos θ

Sol 7: 
r3 =(0.025,0.433) 
∫ x.dx
100 So xCOM = 0
y M
0.5 acos θ
0.5 ρ
x =
M
. ∫ x × 2x(acos θ − x). tan θ.dx
0
150 200
 0.5  acos θ
r1 =(0,0) r2 =(0.5,0) 1×2
=
a2 sin θ.cos θ
. ∫ x(asin θ − x tan θ)dx
0
  
 m x + m2 x2 + m3 x3 acos θ
So xCOM = 1 1 2  x2 x3 tan θ 
MTot. = .  asin θ − 
a2 sin θ.cos θ  2 3 
0
150 × (10) + 200 × (0.5) + 100 × (0.25)
=  a3 sin θ.cos2θ a3 sin θ.cos2 θ 
450 2
= . − 
a2 sin θ.cos θ  2 3 
P hysi cs | 6.49

Take origin at xCOM for simplicity.


a3 sin θ.cos2 θ acos θ
= =
3a2 sin θ.cos θ 3 Assuming the xCOM at origin, we have

4a2 − b2 m1(–a + d) + m2(b – d2) = 0


=
6 ⇒ m2(b – d2) = m1(a – d)
  ⇒ m2b – m2d2 = m1a – m1d…(2)
 m r + m2 r2
Sol 9: rCOM = 1 1
MTot. from (1), m2b = m1a, putting this in (2)
0.5(–1,2) + 1(3, 4) m1a – m2d2 = m1a – m1d
=
0.5 + 1
m1d
(–0.5, 1) + (3, 4) (2.5, 5) ⇒ = d2
= = m2
1.5 1.5
 
  5 10   m1 r1 + m2 r2
rCOM =  ,  Sol 13: rCOM =
3 3  m1 + m2

 2 × 1 + 3(–6) −18 + 2
xCOM = = =–1
Sol 10: Same as question (9) 1+3 4
    5×1 + 4 ×3 12 + 5 17
 m1 x1 + m2 x2 + m3 x3 y COM = = =
xCOM = and 1+3 4 4
m1 + m2 + m3
    13 × 1 + 3 × ( −2) 13 − 6 7
 m1 y1 + m2 y 2 + m3 y 3 z COM = = =
y COM = 1+3 1+3 4
m1 + m2 + m3
  17 7 
So, rCOM =  −1, , 
 4 4
Sol 11: (a) 2 1
2m/s 5m/s
    m1 x1 + m2 x2 + m3 x3 + m4 x 4
 m v + m2 v 2 Sol 14: xCOM =
v COM = 1 1 m1 + m2 + m3 + m4
m1 + m2
1× 0 + 2× a + 3× a + 4 × 0 5a a
2× 2 + 5×1 9 = = = î
= = = 3 m/s 1+2+3+ 4 10 2
2+1 3
m1 y1 + m2 y 2 + m3 y 3 + m4 y 4
(b) y yCOM =
2 1 m1 + m2 + m2 + m4
x 2m/s –5m/s
 2× 2 −5×1 1× 0 + 2× 0 + 3×b + 4 ×b 7b ˆ
v COM = = –1/3 m/s = = j
2+1 10 10

(–ve direction ⇒ velocity is negative direction)  a 7b ˆ


So rCOM = ˆi + j
2 10
Sol 12:
Sol 15: Divide the lamina in 3 equal parts with centre of
m1 xCOM m2 masses as C1, C2, C3 respectively.
(–a,0) (0,0) (b,0)
So, the centre of mass of the whole plate can be found
d2 using the centre of mass of these three plates.
m1
xCOM Now, from symmetry, we can say that the centre of
(–a+d) (b–d2) m2 mass of square plate lies at its centre.

and m2b – m1a = 0 So, rC = (0.5, 0.5)
1
 
⇒ m2b = m1a  …(i) rC = (1.5, 0.5) and rC = (0. 5, 1.5)
2 3
6 . 5 0 | Center of Mass and the Law of Conser vation of Momentum

mass of each plate = 1 kg 9 1  1


So increase = ×  mv 2  – mv2
0.5 × 1 + 1.5 × 1 + 0.5 × 1 2.5 4 2  2
So xCOM = =
(1 + 1 + 1) 3
0.5 × 1 + 0.5 × 1 + 1.5 × 1 2.5 5 1  5
yCOM = = = ×  mv 2  = of initial K. E.
3 3 4 2  4

  2.5 2.5  5 5
So rCM =  ,  =  ,  Sol 4: (C)
 3 3  6 6

A B
Exercise 2 25 m/s 20 m/s

Single Correct Choice Type


A B
vA vB
Sol 1: (B)
25 − 20 5
h M+m e= = =1
vB − v A vB − v A
v
M ⇒ vB – vA = 5
m
Momentum conservation ⇒
velocity of system after collision = 2gh
m. (25) + m. (20) = m. (vA) + m(vB)
so using momentum constant
⇒ vA + vB = 45
(M + m) 2gh = mv ⇒ vB = 25 m/s

(M + m) vA = 20 m/s
⇒v= 2gh
m
Sol 5: (C)
Sol 2: (B) m1 = 200g, m2 = 200 g, m3 = 600 g 2.5m

30 m/s
1.25m
15m/s
2.9 g
m m=0.1kg
30 m/s
m
3m Using momentum conservation
vx
1.50 × 0.1 = (2.9 + 0.1) v
vy
⇒ 15 = 3 × v ⇒ v = 5 m/s
from momentum conservation 1 v2
so mv2 = mgh ⇒h =
x ⇒ 30 × m = 3m × vx = 10 m/s 2 2g
25 5
y ⇒ 30 × m = 3m × vy = 10 m/s ⇒h = = = 1.25 m
2 × 10 4
⇒v= 102 + 102 = 10 2 m/s So L. (1 – cosθ) = 1.25 m
1
⇒cosθ = ⇒ θ = 60º
Sol 3: (C) Momentum = mv (mass = constant) 2

3v
so new vn = Sol 6: (A) The center of mass will be on x-axis, so yCOM
2
= 0.
1 1 9
New, K. E. = ×m×vn2= ×mv2×   m 3m
2 2 4 ⇒15 × +y× =0
4 4
P hysi cs | 6.51

⇒y = –5 cm m1 v1 + m2 v 2
Sol 4: (C) v CM =
m1 + m2
Sol 7: (D) Elastic collision ⇒ Energy is conserved.
10 × 14 + 4 × 0 140
1 1 1 = = = 10 m/s
× mv12 = ×m×v22 + m×v22 10 + 4 14
2 2 2
v1=14 m/s v2=0
⇒v12 = 2v22
⇒v1 = 2 v'2

v2 m1=10 kg m2=4 kg


v1 
m m
m  Sol 5: (A) y CM

v2 m1 y1 + m2 y 2 + m3 y 3 + m4 y 4 + m5 y5
=
Using momentum conservation m1 + m2 + m3 + m4 + m5
⇒2mv ⇒2cosθ = mv1
(6m)(0) + (m)(a) + m(a) + m(0) + m( −a)
1v1 =
⇒cosθ = × 6m + m + m + m + m
v′2 2
a
=
1 10
cos θ = ⇒ θ = 45º ⇒ 90º
2
Sol 6: (C) At first collision one particle having speed
 4π 
Sol 8: (C) Momentum conservation 2v will rotate 240°  or  while other particle having
 3 
⇒ 2 × 3 – 1 × 4 = (2 + 1) v
 2π 
2 speed v will rotate 120°  or  . At first collision they
⇒v= m/s  3 
3
will exchange their velocities. Now as shown in figure,
after two collisions they will again reach at point A.
Sol 9: (D) No external force ⇒ aCOM = 0

v A 2v
Previous Years’ Questions
2v 2/3 2/3 v
Sol 1: (D) Net force on centre of mass is zero. Therefore,
centre of mass always remains at rest. 2/3
st nd
1 collision 2 collision
Sol 2: (A) Let v’ be the velcoity of second fragment. v 2v
From conservation of linear momentum,
2m v cos θ m m
v cos θ v' 2h
Sol 7: (D) R = u
Just before explosion Just after explosion g

2m(v cos θ) = mv '− m(v cos θ)


2×5 2×5
⇒ 20 = v1 and 100 = v 2
∴ v’ = 3v cos θ 10 10

    ⇒ v1 = 20 m/s, v 2 = 100 m/s


Sol 3: (C) | (m1 v1 + m2 v 2 ) − (m1 v1 + m2 v 2 ) |
Applying momentum conservation just before and just
= |Change in momentum of the two particle|
after the collision.
= |External force on the system| × time interval
(0.01)(v) = (0.2) (20) + (0.01)(100)
= (m1 + m2 )g (2t0 ) = 2 (m1 + m2 )gt0
v = 500 m/s
6 . 5 2 | Center of Mass and the Law of Conser vation of Momentum

Sol 8: (B) Between A and B, height fallen by block  3


= v || v1 cos30° = ( 60)  

= h1 3 tan60° = 3m.  2 
 
∴ Speed of block just before striking the second incline, −1
= 45 ms
v1 = 2gh1 = 2 × 10 × 3 = 60 ms−1 1
=v ⊥ v1 sin30° = ( 60)  
In perfectly inelastic collision, component of v1 2
−1
perpendicular to BC will become zero, while component = 15 ms
of v1 parallel to BC will remain unchanged. Now vertical component of velocity of block
∴ Speed of block B immediately after it strikes the
= v v ⊥ cos30° − v || cos60°
second inline is,
 3 1
= ( 15)   −( 45)   = 0
 2  2
 
B 30 o

o
30 o 30
o
Sol 11: (C) Loss of energy is maximum when collision
30
C is inelastic as in an inelastic collision there will be
v1 maximum deformation.

v 2 = component of v1 along BC 1  Mm  2
KE in COM frame is  V
2  M + n  rel
 3
= v1 cos30° = ( 60)   1  Mm  2
 2 
  KEi =
2
  V= ( Vrel 0 )
KEf 0=
M+m
=( 45) ms−1
1  Mm  2
Hence loss in energy is  V
2 M+m
Sol 9: (B) Height fallen by the block from B to C M
⇒f=
M+m
h2 = 3 3 tan 30° = 3 m
Let v 3 be the speed of block, at point C, just before it h 3h
leaves the second incline, then: Sol 12: (A) z 0 =h − =
4 4
v3 = v 22 + 2gh2
1 1
m ( 2v ) + 2m ( v ) = 3mv 2
2 2
Sol 13: (B) Einitial =
= 45 + 2 × 10 × 3 = 105 ms −1 2 2

1 4 4 2 4
Sol 10: (C) In elastic collision, component of v1 E=
final 3m  v 2 + v= mv 2
2  9 9  3
parallel to BC will remain unchanged, while component
perpendicular to BC will remain unchanged in 4
3−
magnitude but its direction will be reversed. ∴ Fractional loss= 3= 5= 56%
3 9
v
B
60°
 B
30°
60° v||
30° C
30°
C
Just before Just after
P hysi cs | 6.53

JEE Advanced/Boards 1 1
× mTot × vTot2 = ×10.2×(0. 392)2 = 0. 8 J
2 2
Exercise 1 So energy lost = 40 J – 0.8 J = 39.2 J

Sol 1: (a) Sol 2:

20 100 m/s
30 v
0.2kg 100 m/s g=9.8m/s2
10kg
20m/s 50
From the conservation of momentum
(a) Now, from equation of motion
we have,
v = u + at = 100 – 9. 8 × (5) = 51 m/s
m1v1 + m2v2 = mTot × v
so v = 51 m/s
0. 2 × 20 + 10 × 0 = (10 + 0. 2) × v
Now at this velocity, particle exploded, Dt is very small
⇒ 4 = 10.2 × v and hence momentum can be conserved.
4
v= m/s = 0. 392 m/s So 50 × 51 = 20 × 150 + 30 × v′
10.2
⇒2550 = 3000 + 30 × v′
(b) From conservation of energy (Force by string is
perpendicular to displacement, hence no work done by −450
string) ⇒=v′ = –15m / s
30
1
× m × v2 = mghvert. (b) When no explosion:
2
v 2
(0.392)2 v = u + at
⇒h= = = 0. 0078 m
2g 2 × 9.81 ⇒ v = 100 – 9. 8 × 8 = 21.6 m/s
= 7. 8 mm ≈ 0. 008 m so momentum = m × v
In horizontal direction: = 50 × 21. 6= 1080 kg. m/s
When explosion:

For 20 kg v = u + at
= 150 – 9. 8 × (s) = 120.6 m/s
For 30 kg v = u + at
R(1 – cosθ) = 0. 008 m
= – 15 – 9. 8 × 3 = –44.4 m/s
1 − 0.08
⇒ cosθ = = 1 – 0.0027 = 0.9973 So total momentum
R
= 20 × (120.6) – (44.4)×(30)
So R sinθ = 3 × (1 − cos2 θ)
= 2412 – 1332 = 1080 kg m/s
2
=3× 1 − (0.9937) = 0. 220 m
Sol 3: (a) The particle are meet at the mid-point of the
So total displacement = 0. 220 m trajectory (i. e. vertical velocity = 0)
(c) Initial energy: u = 49 m/s
1 1 usin θ
×mB×vB2 = ×(0. 2)×(20)2 = 40 J So t =
2 2 g
Final energy: Thus, vx = horizontal velocity = ucosθ
6 . 5 4 | Center of Mass and the Law of Conser vation of Momentum

Now to retrace the path velocity of P must be ucosθ in Similarly in y-direction


the (–)ve direction, so now using momentum balance
0 = 150 × 200 + 150 × (–vy)
m1v1 + m2v2 = m1v'1 + m2v'2
⇒vy = 200 m/s
⇒20×(u cosθ) + 40 × (–u cosθ)
So vIII = 393. 33 î – 200 ˆj
= 20×(ucosθ) + 40 × v'2
|| vIII || = 441. 26 m/s
⇒ 40 × v'2 = 0 ⇒v'2 = 0 −200
and θ = tan–1 393.33 = –27º
so the horizontal velocity of particle Q after collision
would be 0.
so position of Q would be just below the point of Sol 5:
collision

P Q 60º 
P 30º

45º O 135º
45º
A B x Velocity of ball just before impact
position of Q
= 2gL(1 − cos θ) = 2 × 9.8 × 1 / 2
ucos 45º ×usin 45º = 9.8 = 3. 13 m/s
So position of Q =
g
1
v of ball after impact ⇒ mg(Dh) = mv2
u2 sin90º u2 (49)2 × 10 2
= = = = 122. 5 m
2g 2g 2 × 9.8 (energy conservation)
From position A in the (+)ve x-direction
⇒ 2gL.(1 − cos30º ) = vf
(b) Time take would be same as the vertical component
has not changed, so  3
⇒ 2 × 9.8 ×  1 −  = vf
usin θ 49 × sin 45º  2 
t= = = 3. 54 sec 
g 9.8
⇒ vf = 2.626
Sol 4: ⇒ vf = 1. 62 m/s
150 m/s 1.62
so coefficient of rest. = =0. 517
3.13
200 kg Loss of kinetic energy = heat produced
1
y
100 m/s = × m × (vi2 – vf2)
500 kg 60 m/s
150 kg vx 2
150 kg
x 1
= ×(0.01) × [(3. 13)2 – (1. 62)2]
2
0.0717
vy = = 0. 036 J
2
Now, since there is no external force = 0. 0085 cal.
Using momentum conservation in x-direction
Sol 6: No friction ⇒ no torque, so its pure translational
500 × 100 = 150 × vx + 150 × (–60)
motion
⇒150 × vx = 5 × 104 + 9000
Now, conservation of energy
150 × vx = 59 × 103
1
⇒ × mv2 = mgh
⇒vx = 393.33 m/s 2
P hysi cs | 6.55

Sol 8:
⇒v= 2gh

= 2 × 9.8 × 0.1 y
v = 1. 4 m/s a b
Now, as collision is elastic c x
 
 B  m r + m2 r2
A
1.4 m/s
B A
Now rCOM = 1 1
m vA vB m1 + m2
Take the disks as these two bodies and treat m2 as
so vB – vA = 1. 4 m/s …(i)
negative
and conservation of momentum gives:
Given body =
mAvA + mBvB = mA × 1. 4 m/s
⇒vA + vB = 1. 4 m/s …(ii)
on solving (i) and (ii) a b
vB = 1. 4 m/s
vA = 0 m/s
ρ.πa2 .(0) − ρ.πb2 × c
So xCOM =
ρ.( πa2 − πb2 )
Sol 7:
m
0.6-x where ρ =
W E area
−b2c
A B xCOM =
0.6m (a2 − b2 )

(a) Initial acceleration of B = 0.5 m / s2 W Sol 9: (a) No external force, hence vCOM = 0
So, kx = (0.5) (0.25)
As the magnitude of force would be the same for A, vB
Initial acceleration of A =
kx
=
( 0.5) ( 0.25)
0.1 0.1
2
vmax.
= 1.25 m/ s E m
= 1.25 cm/ s2 E thus,

(=
0.5 ) ( 0.25 ) ( 0.5 ) ( 0.25 ) m. vmax. – M. vB = 0
=(b) x
k 2 ⇒ mvmax. = M. vB …(i)
and vrel. = vmax. + vB …(ii)
= 0.0625 m
= 6.25 cm M 
vrel. =  + 1  vB
(c) Max distance would be when spring is fully
m 
elongated. And, symmetry of conservation of energy vrel .m
⇒vB =
implies that expansion would be equal to companion. (m + M)
So, Maximum distance between m.vrel
⇒vB = in (–)ve direction
A and B = 60 cm + 6.25 cm (m + M)
= 66.25 cm (b) Drel. = L
Let xCOM = 0
then m. x1 – M. x2 = 0
6 . 5 6 | Center of Mass and the Law of Conser vation of Momentum

(a) Now as there is no horizontal force on rod, there


would be no displacement of COM of the rod. Thus, the
x2
path followed will be a straight line.
L
x1. (b) x comp. of r = cosθ – rcosθ
2
L 
x1 ≠ x2 = distance covered by man and balloon in x =  − r  cosθ
ground frame, so 2 
and y comp. of r = r sinθ = y
x1 + x2 = L
x2 y2
M  ⇒ + =1
⇒  + 1  x2 = L (L / 2 − r)2 r2
 m 
 mL  Sol 12:
⇒x2 =  
m +M
in the downward direction
T
(c) No external force: initial velocity = final velocity = 0 T
a1
m1
Sol 10: (+) m2 a
1
v1 (–)
v1  y
m  So, T – m1g = m1a1
m  x
v2 m2g – T = m2a1
No generation of momentum would be there in the ⇒ (m2 – m1)g = (m1 + m2)a
y-direction
(m2 − m1 )g
⇒a =
Energy conservation ⇒ m1 + m2
1 1 1 m1a1 + m2a2
mv2 = mv12 + mv22
2 2 2 Now aCOM =
m1 + m2
⇒v2 = v12 + v22 …(i)
 −(m2 − m1 )g   m2 − m1 
Also, mv1sinθ = mv2sinθ m2 .   + m1 .  g
 (m1 + m2 )   m1 + m2 
⇒v1 = v2  … (ii) =
m1 + m2
Now, using (i) and (ii), we get
−m22g + m1m2g + m1 .m2g − m12g
v = 2v1
2 2 =
(m1 + m2 )2
⇒v = 2 v1 −(m1 − m2 )2
= g ⇒ in the (–)ve direction.
and using momentum conservation in x-direction: (m1 + m2 )2
mv = 2mv1cosθ
Sol 13: There is no external force in horizontal direction,
1
⇒cosθ = so xCOM is same after this relative horizontal distance
2
= h cotθ
⇒θ = 45º so angle between them = 90º
x2
Hence proved. m

Sol 11: h M x1

Mass
L 
r Let x1 and x2 be the distance travelled by the block and
M
prism, respectively in ground frame.

P hysi cs | 6.57

so, x1 + x2 = h cosθ …(i) Sol 16:


and m1x1 – Mx2 = 0
m 2m  m 2m
v
[taking xCOM as origin] v1
v2
v2=0
⇒m1x1 = Mx2 v 2 − v1
Elastic collision ⇒ =1
M v −0
⇒x1 = x , putting this in (i)
m 2 ⇒ v2 – v1 = v  …(i)
M  and using momentum conservation
 + 1  x2 = h cotθ
m 
mv = mv1 + 2mv2
m.hcot θ
⇒ x2 = ⇒mv = m(v2 – v1) + 2mv2
(m + M)
⇒2mv = 3mv2
Sol 14: 2v −v
⇒v2 = and thus v1 =
vcos vcos v’ 3 3
mm
2m so kinetic energy before collision:
1
= mv2
2
If the particle retraces its path, its velocity must be same as kinetic energy after collision
before i.e. vcosθ in the opposite direction (independent 1 v2 mv 2
of mass), so using the momentum conservation: = ×m× =
2 9 18
2m. (vcosθ) = m. (–vcosθ) + m. v′ 1 mv 2 8mv 2 8 1
loss = mv2 – = = × mv 2
⇒mv′ = 3mvcosθ 2 18 18 9 2
8
⇒ v′ = 3vcosθ = ×(initial K. E. )
9

Sol 15:
Sol 17:
v
y
m h
vx x
2m m v

v at first impact = 2gh

vy 2h
and time at first impact =
Momentum conservation in x-direction g

m. v + 2m. (–vx) = 0 (eq. of motion)


v Now, un = velocity after nth impact
⇒vx =
2 = en. 2hg
v
Similarly, vy =
2
∞ u2
so total distance = h + ∑ 2gn × 2
so total energy n=1
un2 u2
1 1 1 [ ×2 = ⇒ distance between two impacts]
= ×m×v2 + ×m×v2 + ×(2m)(vx2 + vy2) 2g g
2 2 2
 v2 v2  = h + 2e2. h + 2e4. h …. .
3mv 2
= mv + m.  +  =
2

 4 4  2 = h. [1 + 2e2(1 + e2 + e4 …. . )]
6 . 5 8 | Center of Mass and the Law of Conser vation of Momentum

 ⇒ v2 < gL
2e2  h.[1 + e2 ]
= h. 1 + =
2
 1 − e  [1 − e2 ] v after n collisions = en. 2gL
Similarly total time ⇒ e2n. (2gL) < gL
 2 × un  1 4 1
n
2h ∞ 2un  = time between ⇒ e2n < ⇒  <
= +∑ 8 2 5 2
g n=1 g    
 two impacts 
⇒ n = 4 is the largest value satisfying.
2h 2h 2h 2
= + .2e + .2e +…
g g g Sol 20: (a)

2h m
= [1 + 2e(1 + e + e2…. )]
g v 2d
d
2h  2e  2h  1 + e 
= 1 + = . 
g  1 − e g 1 − e  4d
v=0
v1
Sol 18: If the block m2 is moving with same velocity  v2
after wall collision ⇒ the velocity of block m1 and m2
m 4m
has same magnitude.
v1 –v1 Now, using momentum conservation ⇒ 4mv2 + mv1 = mv

u2 ⇒ 4v2 + v1 = v  … (i)
m1 m2  m1 m2 v 2 − v1
Elasticity =1
u2 v
Now =1
v1 − ( − v1 ) ⇒ v2 – v1 = v  …(ii)
2v
⇒ u2 = 2v1  …. (i) (i) + (ii) ⇒5v2 = 2v ⇒ v2 = and
5
and using momentum conservation
−3v
v1 =
u2m2 = m1v1 – m2v1 5
⇒2m2v1 = m1v1 – m2v1 ⇒3m2v1 = m1v1 To topple, the distance by 4m block should be 2d. (No
horizontal force on 2m block, and hence that block
⇒3m2 = m1
remains stationary)
⇒ m2 = 50 kg
N2

Sol 19:
N2 4mg
L
N1
2mg f=uN2
60º
N1 + 4mg = N2,
Now N1 = 2mg
h
⇒N2 = 6 mg
For the amplitude to be less than 60º. So f = 6μmg
1 thus acceleration (or deceleration)
mv2 < mg(Dh)
2
f 6µmg 3µg
⇒v2 < 2g (Dh) a= = =
4m 4m 2
v2< 2gL (1 – cos60º) So now the velocity (or v2) required
P hysi cs | 6.59

⇒v2 = u2 + 2as and distance by man w.r.t. platform


 −3µg  x0 = x1 – x3 …(iii)
⇒0 = v22 + 2   ×(2d) ⇒v2 = 6µgd
 2  and displacement of woman w. r. t platform = x2 – x3
2v
so, v2 = from above, also x1 – x3 – (x2 – x3) = 6
5
5 5 ⇒x1 – x2 = 6 …(ii)
⇒ v= v2 = 6µgd
2 2 x2 = x1 – 6 and x1 = x0 + x3
10 so thus putting these in (i)
(b) Now v = 2v0 = 6µgd
2
6. (x0 + x3) + 5(x1 – 6) + 2x3 = 0
v = 5 6µgd m/s
6x0 + 6x3 + 5(x0 + x3)–30+2x3 = 0
Now
11x0 + 13x3 – 30 = 0
m v 4m  m v1 4m v1
30 − 11x0
⇒x3 =
so v2 – v1 = v (elasticity) 13
and m × v = m × v1 + 4m × v2 30 − 11x0
so displacement = in (+)ve direction.
[Momentum conservation] 13
⇒v = v1 + 4v2 …(ii)
Sol 22:
⇒2v = 5v2
2v −3v
⇒v2 = and v1 =
5 5
xB
So v1 = –3 6µgd
m m
2d xm xL
Now time ⇒
g
M–m
so distance= v1 × t
Now, we have
2d
= –3 × 6µgd × = –6d. 3µ xB = xL(constraint)
g 
Now xm – (–xL) = ∆ r
Now displacement of centre of mass
Sol 21:
 M.xB + m.xm − (M − m)xL
⇒ ∆S =
m1=60kg m2=50kg M+M−m+m

M.xB + m.xm − (M − m)xL


=
 system 2M
µ=0 mP=20kg
M(xB − xL ) + m(xm + xL )
x0 (6–x0) =
2M
[(xB – xL) = 0]
(–)

x m.∆ r
(+) =
2M
Now no external force thus xCOM=constant ≈ 0.
x1, x2, x3 ⇒ displacement of man, woman and platform Sol 23: Let the total mass of both insects be ‘m’
respectively w.r.t. ground frame then,
For the limiting case of toppling, normal reaction would
60 × x1 + 50 × x2 + 20 × x3 = 0 pass through the corner.
⇒6x1 + 5x2 + 2x3 = 0 …(i)
6 . 6 0 | Center of Mass and the Law of Conser vation of Momentum

N Sol 25:
a a 
4 2



os


gc
in


gs
a 2mg m’g
 g
2µsinθ
Time for first collision, T1 =
a gcos α
 2µsinθ 
N = 2mg+m’g Time for second collision, T2 = e  
 gcos α 
a a As velocity becomes perpendicular to the surface, its
m’g   = 2mg  
2 4 horizontal component (along the surface) must go to
‘O’.
⇒ m' =
m
m O = µ cos θ − gsin α(T1 + T2 )
As the mass of first insect is , the second insect would
also have the same mass. 2
 2µ sin θ 
m ⇒ µ cos
= θ gsin α   (1 + e )
Hence, mass of the other insect = m'− m =  gcos α 
2
cot θ cot α
⇒1 + e =
Sol 24: 2
cot θ cot α
⇒e
= −1
2

Sol 26:

d u
gcos
Horizontal velocity: 2 ag . cosθ
gsin 

2 ag cos θ
after collision: 2usin θ
(4 sin2θ − 1) here time of flight =
gcos α
so time (total) total time for second bounce
d d(4 sin2θ − 1) d.(4 sin2θ) 2usin θ 2uesin θ
= + = = +
2 ag cos θ 2 ag cos θ 2 ag cos θ gcos α gcos α
ucos θ
Now this time must be equal to time of flight Also equation of motion along incline =
gsin α
2usin θ 2.2 ag d(4 sin2θ) ucos θ 2usin θ 2uesin θ
= ⇒ = so = +
g g 2 ag cos θ gsin α gcos α gcos α

a.g.sin2θ cos θ 2sin θ 2esin θ


⇒ = d. sin2θ ⇒ d = a ⇒ − =
g sin α cos α cos α
1 (cot θ.cot α − 2)
also, e < 1 ⇒ <1 ⇒ =e
4 sin2θ − 1 2
1 1
⇒sin2θ > ⇒ 2θ > 30o⇒ θ > 15o ⇒e= cotα . cotθ – 1
2 2
P hysi cs | 6.61

Sol 27: So mv0 cos(90º – θ) < 3 NS


C ⇒2 × v0 × sin θ < 3
J
3
⇒v0 <
2sinθ
C
3 × 1.2
A B ⇒ v0 <
J 2 × (1.2)2 − (0.5)2
J ⇒v0 < 1. 65 m/s
vB J
vB A (b) The energy remaining
J B vA
1 1
45º = ×m(v0cosθ)2 = mv02cos2θ
2 2
1 1
Now we have, so loss = mv02– mv02cos2θ
2 2
J – J′ = mvc  … (i) (moment eqn)
1 1
vc = vB||  …(ii) (constraint eqn) = mv02 (1 – cos2θ) = mv02. sin2θ
2 2
J′ – J′′ cos45º = mvB||  …(iii)
1 (1.2)2 − (0.5)2
= ×2×(1. 65)2× = 2. 25 Joules.
(impulse momentum eqn) 2 (1.2)2
J′′ sin45º = mvB⊥ …(iv)
(impulse momentum eqn) Sol 29:

J′′ = mvA …(v)


(1000+225) kg 25 m/s
(impulse momentum eqn)
1
(vB|| – vB⊥) = vA …(vi) (a) (i)
2
(constraint equation) v1
so we have 6 variables (J′, J′′, vB||, vB⊥, vA and vc) and six
v2
equations,
on solving, we get So v2 – v1 = 5⇒v1 = v2 – 5

3J and
vc = ,
7m (1225) × 25 = 1150 × v2 + 75 × v1

10 1225×25 = 1150×v2 + 75 × (v2 – 5)


vB = ,
7m 1240 × 25 = 1225 v2
2 1240 × 25
vA = ⇒v2 = = 25. 306 m/s
7m 1225
(ii)
Sol 28: v1

A’ v2
1150

90 -
o 1150 × 25. 306 = 1075×v2 + 75×v1
2m

⇒ 1150×25. 306 = 1075v2+75(v2–5)


1.

v0
 1150 × 25.306 + 75 × 5
O ⇒ = v2
0.5m (m) 1075

(a) The comp. ⊥ to the string will only be there and the ⇒v2 = 25. 63 m/s
momentum along the thread will be lost.
6 . 6 2 | Center of Mass and the Law of Conser vation of Momentum

(iii)  2v 
2

v1 ⇒ v0 = 2vA +  v 0 − A
2 2

 3 
1075 v2 2 3v 0
⇒ vA = ⇒ 0. 693 v0
1075 × 25.63 = 1000 × v2 + 75 × v1 3
⇒ 1075×25.63 = 1000v2 + 75(v2–5) So, vBx = 0. 693 v0 and

1075 × 25.63 + 75 × 5 2v A 4 v
⇒ = v2 vBy = v0 – = v0 – v 0 = 0 = 0. 2 v0
1075 3 5 5
⇒v2 = 25.97 m/s
So, vB = (0.693)2 + (0.2)2 .v0
(b)All together
⇒ 1225 ×25 = 1000 × v2 + 225 × v1 ⇒ vB = 0. 721 v0

⇒ 1225 × 25 = 1000v2 + 225(v2 – 5)


dP d(mu) udm m.du
1225 × 25 + 225 × 5 Sol 31: =F= = +
⇒ = v2 = 25.92 m/s dt dt dt dt
1225
u du
–mg = .m0 et/T + m.
T dt
Sol 30:
u du
v –g = +
T dt
 u, t t
R d(u.et/T ) = t/T
vBX ⇒ ∫ ∫ −g.e .dt
u0 ,0 0
R
 v0 u. et/T – u0 = –gT. (et/T – 1)
R ⇒ u(t) = u0. e–t/T – gT. (1 – e–t/T)
vA so here, u(t) = 0
R 1 ⇒ gT. e–t/T – gT(1 – e–t/T) = 0
sin θ = = ⇒ θ = 30º
2R 2 1
⇒ e–t/T =
Now, using elasticity, (along line of impact) 2
⇒ et/T = 2 ⇒ t = T ln 2
3 1
v A cos60º + vBy . + vBx . So m = m0. et/T
2 2
=1
3 m = 2m0
v0 .
2

vA 3vBy vBx v0 3 Sol 32:


⇒ + + = y
2 2 2 2 x
motion
⇒ vA + vBy. 3 + vBx = v 0 3 …(i) N
using energy balance,
1 1 1
mv02 = mvA2 + m(vBx2 + vBy2) Mgcos
2 2 2 mgsin
 f=μN
⇒ v02 = vA2 + vBx2 + vBy2 …(ii)
Momentum along x-direction N = Mg cosα

⇒ vA – vBx = 0 ⇒ vA = vBx  …(iii) So f = μN = μMg cosα

2v A so acceleration of 2 kg block
so on, solving v0 – = vBy
3 µmgcos α + mgsin α
= = μgcosα + gsinα
Putting this in (ii) m
P hysi cs | 6.63

So for a distance of 6m, v just before impact would be: v of system after collision = 2gh
v2 = u2 – 2as so using momentum conservation.
⇒v2 = (10)2 – 2 × 6 × [0. 25 × 10
(M + m) 2gh = mv
× 0. 998 + 10×0. 05]
= 100 – 12 × [3] (M + m)
⇒v = 2gh
m
v2 = 64
vin. = 8 m/s Sol 2: (C)
Now, after collision: vA
N
motion A
f=uN 30º

mgsin mgcos C
B
so acc. = μg cosα – g sinα vB
= 2. 5 – 0. 5 = 2 m/s2 Let the initial velocity of
So v = u – 2as
2 2
C=v
1 =u –2×2×6
2 2
then momentum conservation in y gives
⇒u2 = 25 ⇒u = 5 m/s vA = vB
so after collision v of 2 kg block = 5 m/s and using momentum conservation in x,
For M block: mv = 2mvA cos30º
acc = same as in 1 part (ind. of mass,)
st
v
⇒v = 3 vA ⇒vA =
= μgcosα + gsinα = 3m/s2 (in (–)ve direction) 3
So v = u + 2as
2 2 so coefficient of restitution

0 = u2 + 2 × (–3) × (0. 5) final relative velocity v/ 3 2


= = =
initial relative velocity v 3 /2 3
⇒u = 1. 732 m/s
so coefficient of restitution Sol 3: (A)
5 + 1.732 6.732 u
= = = 0. 84 m m  m v1 m v2
8 8
Using momentum conservation, just before impact v 2 − v1
e= ⇒v2 – v1 = ue  …(i)
m × vi = m × vf + M × vm u
and momentum conservation ⇒
2 × 8 = 2 × (–5) + M × 1. 732
26 mu = mv1 + mv2
⇒ = M = 15.011 kg
1.732 ⇒v1 + v2 = u  …(ii)
So 2v2 = u(e + 1)
Exercise 2 ⇒v2 =
u(e + 1)
and thus,
2
Single Correct Choice Type (1 − e)u
v1 =
2
Sol 1: (B)
M+m
h
v
M
m
6 . 6 4 | Center of Mass and the Law of Conser vation of Momentum

Sol 4: (B) ⇒ v2 = – 4m/s and v1 = –28 m/s


100m/s
1 1
m
vy Einitial = ×(1)×122 + ×2×242
m vx 2 2
m
= 72 + 576 = 648 J
100m/s
60º 1 1
Efinal = ×1×(28)2 + ×2×42
We have 2 2
= 16 + 392 = 408 J
Momentum cons.: in y-direction
So ∆E = 240 J
m × 100 + m(–100) + mvy = 0
⇒vy = 0 Sol 7: (B)
x-direction: 3m. u cos60º = mvx v1 v1 v2
m1 m2  m1 m2
1
⇒vx = 3 × 200 × = 300 m/s
2 v′2 − v′1
e=1 = ⇒ v'2 – v'1 = v1  …(i)
v1
Sol 5: (C) and m1v1 = m1v'1 + m2v'2
m2
5m ⇒v1= v'1 + . v'2  …(ii)
m1
500 m/s  m 
0.01 kg 2kg v so (1) + (2) ⇒  1 + 2  v'2 = 2v1
 m1 
v of block after the bullet emerges can be found using
energy conservation 2v1 .m1
⇒v'2 =
(m1 + m2 )
1
mgh = mv2
2 2v1 .m1
and v'1 = v'2 – v1 = – v1
(m1 + m2 )
⇒v = 2gh = 2 × 9.8 × 0.1 = 1. 4 m/s
m1 v1 − m2 v1  m − m2 
so now using momentum conservation = =  1  v1
m1 + m2  m1 + m2 
500 × 0. 01 = 2 × 1. 4 + 0. 01 × v′
⇒5 – 2. 8 = 0. 01 × v′ ⇒2. 2 = 0. 01 × v′ 2v1m1
Sol 8: (C) v'2 =
v′ = 220 m/s (m1 + m2 )

Sol 6: (C) Sol 9: (B)


12 m/s 24 m/s m v  vx
1kg 2kg
u
u
v1 v2 v x − ( −u)
1kg 2kg e=1=
−v − u
v 2 − v1 2 vx = –u – v – u = –v – 2u
So e = =
36 3
2u + v ⇒ away from wall.
⇒v2 – v1 = 24  …(i)
Using momentum conservation Sol 10: (C)
12 × 1 – 2 × (24) = 1 × v1 + 2×v2 m u Am
E
⇒ –36 = v1 + 2v2  …(ii)
v1 v2
⇒3v2 = –12
v 2 − v1
e= =1 ⇒ v2 – v1 = u  …(i)
u
P hysi cs | 6.65

Momentum conservation M 2M x.dm


ρ= = So xCOM = ∫
mu = Amv2 + mu1 hb / 2 hb M
 x
⇒ Av2 + v1 = u b x.ρ.h.  1 −  dx b
 b 2  x 2 x3  b
2u xCOM = ∫ =  −  =
⇒ (A + 1) v2 = 2u or v2 = 0
M b  2 3b  0 3
(A + 1)
h
2u (1 – A)u Similarly, yCOM =
v1 = v2 – u = –u= 3
(A + 1) (1 + A)
Sol 14: (B)
1 (1 − A)2 u2 E.(1 – A)2
E= ×m× = y x x2
2 (1 + A)2 (1 + A)2
x1
10cm u=0
M M
Sol 11: (A) v at first imp. = 2gh
man cart
v at after 1 imp. = e 2gh
st
At x = 5 m
 
v at after nth imp. = en 2gh M1 x1 + M2 x2 = 0
   
v2 2gh ⇒ x1 + x2 = 0 and x1 − x2 = 10
h= = e2n . = e2n. h  
2g 2g ⇒ x1 = 5 m & x2 = –5m

Sol 12: (A)


Multiple Correct Choice Type

A v1 Sol 15: (C, D)


A R B Elastic collision ⇒ 100 % energy transfer
v2
The relative velocity along tangent is zero but in
oblique collision the tangent direction is not the one
πR perpendicular to the line joining centres.
vA = ,
t
v2 – v1 = evA(elasticity) Assertion Reasoning Type
and v2 + v1 = vA(mom. cons. ) Sol 16: (D) aCOM ≠ 0 as Fext ≠ 0. (in COM frame it is zero)
(e + 1)v A (1 − e)v A
⇒v2 = and v1 =
2 2 Sol 17: (A) Proper exp.
Drel 2πr
so total time = =
v rel  e + 1  1 − e  Sol 18: (B) It is average and it may be outside body.
 −  v A
 2  2 
2πr × t 2t Sol.19: (A) In explosion only internal forces are
= =
e.πr e involved.

Sol 20: (D) Disk may be non-uniform.


Sol 13: (C)
y
Sol 21: (A) (A) true, (R) true ⇒ correct reason

h
Sol 22: (B) A → true, R → true. But R not explanation
of A.
b x
y x
Equation ⇒ + = 1,
h b
6 . 6 6 | Center of Mass and the Law of Conser vation of Momentum

Comprehension Type
m V2
Sol 23: (B) Obvious (inelastic) m V  V1 +  V1

Sol 24: (B) Inelastic collision leads to loss of energy.


M1 M1 M2 m +M2

Sol 25: (D) Basic concept. Solving Eqs. (i) and (ii), we get
v2 M2 + m
=
v M1 + M2 + m
Previous Years’ Questions
Substituting the values of m : M1 and M2 we get,
percentage of velocity retained by bullet.
Sol 1: (a) From conservation of linear momentum,
momentum of composite body v2  2.98 + 0.02 
× 100 =   × 100 = 75%
=
  
p (pi )1 + (pi )2 = (mv)iˆ + (MV)ˆj
v  1 + 2.98 + 0.02 

 ∴ %loss = 25%
|p|
∴= (mv)2 + (MV)2

Let it makes an angle α with positive x-axis, then Sol 3: Suppose r1 be the distance of centre of mass of
the remaining portion from centre of the bigger circle,
 py   MV  then
α =tan−1   = tan−1  
 px   mv  A1r1 = A2r2
 
(b) Fraction of initial kinetic energy transformed into A 
heat during collision r1 =  2  r2
 A1 
K f − Ki K
= = f -1
Kf Ki π(42)2
r1 = × 7 = 9 cm
p2 / 2(M + m) π[(56)2 − (42)2 ]
= −1
1 2 1 2
mv + MV
2 2 Sol 4: Before collision net momentum of the system
was zero. No external force is acting on the system.
(mv)2 + (MV)2
= -1 Hence, momentum after collision should also be zero.
(M + m)(mv 2 + MV 2 ) A has come to rest. Therefore, B and C should have
equal and opposite momenta or velocity of C should
Mm(v 2 + V 2 )
= be V in opposite direction of velocity of B.
(M + m)(mv 2 + MV 2 )
Sol 5: Collision between A and C is elastic and mass of
Sol 2: Applying conservation of linear momentum
both the blocks is same. Therefore, they will exchange
twice. We have
their velocities i.e., C will come to rest and A will be
moving will velocity v 0 . Let v be the common velocity of
A and B, then from conservation of linear momentum,
we have
v0
(A) C A B
At rest
v0
(B) C A B
M1 M2
v v
mv M1 v1 + mv 2 ….(i)
= (C) C A B
mv
= 2 (M2 + m)v1 ….(ii)
mA=
v 0 (mA + mB )v
P hysi cs | 6.67

1
v0 For θn to be less than 60°, i.e., cos θn is greater than
or mv=
0 (m + 2m)v or v = 2
3 1
, i.e., (1 − cos θn ) is less than , we have
(b) From conservation of energy, we have 2
n
4 1

1 1 1
m v 2 = (m + mB )v 2 + kx20   < 
2 A 0 2 A 2 5 2
2 The condition is satisfied for n = 4.
1 1 v  1
or
= mv 20 (3m)  0  + kx20
2 2 3 2 ∴ Required number of collisions = 4.
 
1 1 2mv 20
or kx20 = mv 20 or k = Sol 7: (a) Since, only two forces are acting on the rod,
2 3 3x20 its weight Mg (vertically downwards) and a normal
reaction N at point of contact B (vertically upwards).
Sol 6: As shown in figure initially when the bob is at A, No horizontal force is acting on the rod
its potential energy is mgl . When the bob is released
and it strikes the wall at B, its potential energy mgl is (surface is smooth).
converted into its kinetic energy. If v be the velocity
with which the bob strikes the wall, then A A
O
A
C C
n

B O M B
C x
h (A) (B)
B
Therefore, CM will fall vertically downwards towards
1 negative y-axis i.e., the path of CM is a straight line.
mgl = mv 2 or v = (2gl )  …..(i)
2 (b) Refer figure (B). We have to find the trajectory of a
Speed of the bob after rebounding (first time) point P(x, y) at a distance r from end B.
v1 = e (2gl )  …..(ii) CB = L/2
∴ OB = (L/2) cos θ
The speed after second rebound is v 2 = e2 (2gl )
MB = r cos θ
In general after n rebounds, the speed of the bob is
∴ x = OB – MB = cos θ {(L/2 – r)}
v n = en (2gl ) …..(iii)
x
or cos θ = ….(i)
Let the bob rises to a height h after n rebounds. {(L / 2) − r}
Applying the law of conservatioin of energy, we have
Similarly, y = r sin θ
1
mv n2 = mgh y
2 or sin θ = ….(ii)
r
v n2 e2n . 2gl Squaring and adding Eqs. (i) and (ii), we get
∴ h= = = e2n .l
2g 2g
x2 y2
sin2 θ + cos2 θ = +
 2 
2n
4
n
{(L / 2) − r}2 r2
=
  . l =   l  …..(iv)
 5 5 x2 y2
or + 1 ….(iii)
=
If θn be the angle after n collisions, then {(L / 2) − r}2 r2
This is an equation of an ellipse. Hence, path of point P
h = l − l cos θn = l (1 − cos θn )
is an ellipse whose equation is given by (iii).
From Eqs. (iv) and (v), we have

4
n
4
n Sol 8: (a) Since, the collision is elastic, the wedge will
  l = l (1 − cos θn ) or   = (1 − cos θn ) return with velocity viˆ
5 5
6 . 6 8 | Center of Mass and the Law of Conser vation of Momentum

(i)  vˆi vˆi ∴ Magnitude of torque of N about CM = magnitude of


z

y torque of F about CM
=F.h
x
  4mv 
F cos 30° | τN |= h
 3 ∆t 
 
////////////////////////////////
Fixed
Sol 9: After elastic collision,
F sin 30° F  m − 2m  2(2m)
v 'A =   (9) + (0) = −3ms−1
 m + 2m  m + 2m
Now, linear impulse in x-direction Now from conservation of linear momentum after all
= change in momentum in x-direction. collisions are complete,

∴ (Fcos30°) ∆=
t mv − ( −mv)
= 2mv m( +9ms−1 ) = m( −3 ms−1 ) + 3m (v C )
or v C = 4ms−1
2mv 4mv
∴ F= =
∆t cos30° 3 ∆t Sol 10: (A) Velocity of particle performing projectile
4mv motion at highest point
F=
3 ∆t = v=
1 v 0 cos α

F (Fcos30°)iˆ − (Fsin30°)kˆ
∴= Velocity of particle thrown vertically upwards at the
position of collision
  2mv   2mv 
or F 
=  ˆi −   kˆ
 ∆t   3 ∆t 
(ii) Taking the equilibrium of wedge in vertical mv0
z-direction during collision. 
N mu0cos
2 2
u sin α
C v 22 =
= u20 − 2g v 0 cos α
=
2g
mg So, from conservation of momentum
mv 0 cos α
tan θ
= = 1
F sin 30
o mu0 cos α

N=
mg + Fsin30° ⇒ θ = π/4

2mv
N mg +
= Sol 11: The initial speed of 1st bob (suspended by a
3 ∆t
string of length l1) is 5gl1 .
or in vector form

  2mv  ˆ The speed of this bob at highest point will be gl1 .


N  mg +
= k
 3 ∆t 
 When this bob collides with the other bob there speeds
will be interchanged.
(b) For rotational equilibrium of wedge [about (CM]
anticlockwise torque of F = clockwise torque due to N. l
gl1= 5gl1 ⇒ 1= 5 .
l2
N

C
h

mg
2017-18 100 &
op kers
Class 11 T
By E ran culty
-JE Fa r
IIT enior emie .
S fP r es
o titut
Ins

PHYSICS
FOR JEE MAIN & ADVANCED
SECOND
EDITION

Exhaustive Theory
(Now Revised)

Formula Sheet
9000+ Problems
based on latest JEE pattern

2500 + 1000 (New) Problems


of previous 35 years of
AIEEE (JEE Main) and IIT-JEE (JEE Adv)

5000+Illustrations and Solved Examples


Detailed Solutions
of all problems available

Topic Covered Plancess Concepts


Tips & Tricks, Facts, Notes, Misconceptions,
Rotational Mechanics Key Take Aways, Problem Solving Tactics

PlancEssential
Questions recommended for revision
7. R OTAT I O N A L
MECHANICS

1. INTRODUCTION
In this chapter we will be studying the kinematics and dynamics of a solid body in two kinds of motion. The first
kind of motion of a solid body is rotation about a stationary axis, also called pure rotation. The second kind of
motion of a solid is the plane motion wherein the center of mass of the solid body moves in a certain stationary
plane while the angular velocity of the body remains permanently perpendicular to that plane. Here the body
executes pure rotation about an axis passing through the center of mass and the center of mass itself translates
in a stationary plane in the given reference frame. The axis through the center of mass is always perpendicular to
the stationary plane. We will also learn about the inertia property in rotational motion, and the quantities torque
and angular momentum which are rotational analogue of force and linear momentum respectively. The law of
conservation of angular momentum is an important tool in the study of motion of solid bodies.

2. BASIC CONCEPT OF A RIGID BODY


A solid is considered to have structural rigidity and resists 
change in shape, size and density. A rigid body is a solid body
B
which has no deformation, i.e. the shape and size of the body
remains constant during its motion and interaction with other A
bodies. This means that the separation between any two points
of a rigid body remains constant in time regardless of the kind of
motion it executes and the forces exerted on it by surrounding 
v
bodies or a field of force.
A metal cylinder rolling on a surface is an example of a rigid
body as shown in Fig. 7.1. 

Let velocities of points P and Q of a rigid body with respect to a Figure 7.1: Metal cylinder rolling on a surface is a
reference frame be VP and VQ as shown in the Fig. 7.2. rigid body system. Relative distance between points
A and B do not change.
As the body is rigid, the length PQ should not change during
the motion of the body, i.e. the relative velocity between P and
Q along the line joining P and Q should be zero i.e. velocity of approach or separation is zero. Let x-axis be along
PQ, then

VQP = relative velocity of Q with respect to P

VQP = ( VQ cosθ2 î + VQ sinθ2 ˆj ) – ( VP cosθ1 î – VP sinθ1 ˆj )

VQP = ( VQ cosθ2 – VP cosθ1) î + ( VP sinθ1 + VQ sinθ2) ˆj
7 . 2 | Rotational Mechanics

Now VP cosθ1 = VQ cosθ2


VPsin1
(Since velocity of separation is 0) P
 P
VPcos1
VQP = ( VP sinθ1 + VQ sinθ2) ˆj (which is VP 1

perpendicular to line PQ). Q


Q VQsin2
2 VQ VQcos2
Hence, we can conclude that for each and VQP
every pair of particles in a rigid body, relative
motion between the two points in the pair will
be perpendicular to the line joining the two
points.
Figure 7.2: Relative velocity between
two points of a rigid body

PLANCESS CONCEPTS

A A 1
B B
 
1
C C

(a) (b)

Figure 7.3: (a) Angular velocity of A and B w.r.t. C is ω1 (b) Angular velocity of A and C w.r.t. B is ω1

Suppose A, B, C are points of a rigid system hence during any motion the lengths of sides AB, BC, and CA
will not change, and thus the angle between them will not change, and so they all must rotate through
the same angle. Hence all the sides rotate by the same rate. Or we can say that each point is having the
same angular velocity with respect to any other point on the rigid body.
Neeraj Toshniwal (JEE 2009 AIR 21)

3. MOTION OF A RIGID BODY 

We will study the dynamics of three kinds of motion of a rigid body. r1


m1
(a) Pure Translational motion r2
m2
(b) Pure Rotational Motion
rn mn
(c) Combined Translational and Rotational motion
Let us briefly discuss the characteristics of these three types of motion of a
rigid body. axis of rotation

3.1 Pure Translational motion Figure 7.4: Body in pure


rotational motion.
A rigid body is said to be in pure translational motion if any straight
P hysics | 7.3

line fixed to it remains parallel to its initial orientation all the time. E.g. a car
moving along a straight horizontal stretch of a road. In this kind of motion, the
displacement of each and every particle of the rigid body is the same during
any time interval. All the points of the rigid body have the same velocity and m6
acceleration at any instant. Thus to study the translational motion of a rigid m3
body, it is enough to study the motion of an individual point belonging to that m8
rigid body i.e. the dynamics of a point. m2
m5

3.2 Pure Rotational Motion


m7
m1
m4
Suppose a rigid body of any arbitrary shape rotates about an axis which is
stationary in a given reference frame. In this kind of motion every point of the
body moves in a circle whose center lies on the axis of rotation at the foot of
the perpendicular from the particle to this axis, and radius of the circle is equal
to the perpendicular distance of the point from this axis. Every point of the rigid
body moves through the same angle during a particular time interval. Such a
motion is called pure rotational motion. Each particle has same instantaneous v v
angular velocity (since the body is rigid) and different particles move in circles of
m6
different radii, the planes of all these circles are parallel to each other. Particles v
m8
moving in smaller circles have less linear velocity and those moving in bigger m3 v
circles have large linear velocity at the same instant. v
m2 m5
In the Fig. 7.4 particles of mass m1, m2, m3….. have linear velocities v1, v2, v3…. v
v m7
If ω is the instantaneous angular velocity of the rigid body, then v
m1 m4
v1 = ωr1 , v 2 = ωr2 , v3 = ωr3 ...... , vn = ωrn

Figure 7.5: Body in pure


3.3 Combined Translational and Rotational Motion translational motion.
A rigid body is said to be in combined translational and rotational motion if the body performs pure rotation about
an axis and at the same time the axis translates with respect to a reference frame. In other words there is a reference
frame K’ which is rigidly fixed to the axis of rotation, such that the body performs pure rotation in the K’ frame. The
K’ frame in turn is in pure translational motion with respect to a reference frame K. So to describe the motion of
the rigid body in the K frame, the translational motion of K’ frame is super-imposed on the pure rotational motion
of the body in the K’ frame.

Illustration 1: A body is moving down into a well through a rope passing over a fixed pulley of radius 10 cm.
Assume that there is no slipping between rope and pulley. Calculate the angular velocity and angular acceleration
of the pulley at an instant when the body is going down at a speed of 20 cm s-1 and has an acceleration of 4.0 m
s-2. (JEE MAIN)

Sol: Since the rope does not slip on the pulley, the linear speed and linear acceleration of the rim of the pulley will
be equal to the speed and acceleration of the body respectively.
Therefore, the angular velocity of the pulley is
linear velocity of rim 20 cm s-1
ω= = = 2 rad s-1
radius of rim 10 cm
And the angular acceleration of the pulley is

linear acceleration of rim 4.0 ms-2


α= = = 40 rad s-2
radius of rim 10cm
7 . 4 | Rotational Mechanics

4. ROTATIONAL KINEMATICS
Suppose a rigid body performing pure rotational motion about an axis of rotation rotates by an angle ∆θ in a time
interval ∆t. The instantaneous angular velocity ω, is defined as,
∆θ dθ
ω = Lim = …(i)
∆t→0 ∆t dt
Similarly, the instantaneous angular acceleration α is defined as,
dω d2 θ
α= = 2 …(ii)
dt dt
The relations between linear distance s, linear velocity v and linear acceleration a, and the corresponding angular
variables describing circular motion θ, ω, and α respectively are given as:
s = rθ ; v = rω ; at = rα  …(iii)
Here the subscript t along with a in the expression for acceleration signifies that this is the tangential component
of linear acceleration.
If a body rotates with uniform angular acceleration,
1 2
ω = ω0 + αt ; θ = ω0 t + αt ; ω2 = ω20 + 2αθ …(iv)
2
where ω0 is initial angular velocity.
The equations for angular displacement, angular velocity and angular acceleration are similar to the corresponding
equations of linear motion.

Illustration 2: A disc starts rotating with constant angular acceleration of π / 2 rad s-2 about a fixed axis
perpendicular to its plane and through its center. Calculate
(a) The angular velocity of the disc after 4 s
(b) The angular displacement of the disc after 4s and
(c) Number of turns accomplished by the disc in 4 s.  (JEE MAIN)

Sol:Use the first and second equations of angular motion with constant angular acceleration.
π
=
Here  α rad s−2 ;       ω0 = 0  ;       t = 4 s ;
2
π 
(a) ω(4 s) = 0 +  rad s-2  × 4 s = 2π rad s-1
2 
1π 2 2
(b) θ  rad s  × (16s ) = 4π rad
(4s) = 0 + 2 2 
(c) ⇒ n × 2π rad = 4π rad ⇒ n = 2.

PLANCESS CONCEPTS

For variable angular acceleration we should proceed with differential equation = α
dt
Akshat Kharaya (JEE 2009 AIR 235)
P hysics | 7.5

5. MOMENT OF INERTIA
Before discussing the dynamics of rigid body motion let us study about an important property of a rigid body
called Moment of Inertia which is indispensable in understanding its dynamics.
Physical Significance of Moment of Inertia: As the name suggests, moment of inertia is the measure of the
rotational inertia property of a rigid body, the rotational analog of mass in translational motion. “It is the property
of the rigid body by virtue of which it opposes any change in its state of uniform rotational motion.” The moment
of inertia of a rigid body depends on its mass, on the location and orientation of the axis of rotation and on the
shape and size of the body or in other words on the distribution of the mass of the body with respect to the axis
of rotation. SI units of moment of inertia is Kg-m2. Moment of inertia about a particular axis of rotation is a scalar
positive quantity.
Definition: Moment of inertia of a system of n particles about an axis is defined as:
n
I = m1r12 + m2r22 + .................+ mnrn2 i.e. I = ∑ mi ri2  …(i)
i=1
where, ri is the perpendicular distance of ith particle of mass mi from the axis of rotation.
For a continuous rigid body, the moment of inertia can be calculated as:
I = ∫ r2 (dm) …(ii)
where dm is the mass of an infinitesimal element of the body at a perpendicular distance r from the axis of rotation.
Moment of inertia depends on:
(a) Mass of the rigid body.
(b) Shape and size of the rigid body.
(c) Location and orientation of the axis of rotation.

PLANCESS CONCEPTS
Moment of inertia does not change if the mass:
(i) Is shifted parallel to the axis of rotation because ri does not change.
(ii) Is rotated about the axis of rotation in a circular path because ri does not change.
Chinmay S Purandare (JEE 2012 AIR 698)

Illustration 3: Two particles having masses m1 & m2 are situated in a plane A


perpendicular to line AB at a distance of r1 and r2 respectively as shown.
(i) Find the moment of inertia of the system about axis AB?
(ii) Find the moment of inertia of the system about an axis passing though m1
and perpendicular to the line joining m1 and m2 . r2 r1

(iii) Find the moment of inertia of the system about an axis passing through m2 m1
m1 and m2 .
(iv) Find moment of inertia about an axis passing though center of mass and
perpendicular to line joining m1 and m2 . (JEE MAIN)

Sol: Use the formula for moment of inertia of a system of n particles. Find the B
distance of center of mass from m1.
Figure 7.6
(i) Moment of inertia of particle on right is I1 = m1r12
7 . 6 | Rotational Mechanics

Moment of inertia of particle on left is I2 = m2r22


Moment of inertia of the system about AB is I = I1 + I2 = m1r12 + m2r22
(ii) Moment of inertia of particle on right is I1 = 0
Moment of inertia of particle on left is I2 = m2 (r1 +r2 )2
Moment of inertia of the system about axis is I = I1 + I2 = 0 + m2 (r1 +r2 )2
(iii) Moment of inertia of particle on right is I1 = 0
Moment of inertia of particle of left is I2 = 0
Moment of inertia of the system about axis is I = I1 + I2= 0 + 0
 r +r 
(iv) of system rcm = m2  1 2  = Distance of center mass from mass m1
 m1 + m2 
 r +r 
Distance of center of mass from mass m2 = m1  1 2 
 m1 + m2 
2 2
 r +r   r +r 
So moment of inertia about center of mass = Icm = m1  m2 1 2  + m2  m1 1 2 
 m1 + m2   m1 + m2 

m1m2
Icm = (r + r2 )2 .
m1 + m2 1

Illustration 4: Three particles each of mass m, are situated at the vertices of an equilateral x
triangle PQR of side a as shown in the Fig 7.7. Calculate the moment of inertia of the R
system about
(i) The line PX perpendicular to PQ in the plane of PQR.
(ii) One of the sides of the triangle PQR
(iii) About an axis passing through the centroid and perpendicular to plane of the triangle P a Q
PQR.  (JEE MAIN)
Figure 7.7
Sol: Use the formula for moment of inertia of a system of n particles.
(i) Perpendicular distance of P from PX = 0; perpendicular distance of Q from PX = a perpendicular distance of R
from PX = a/2. Thus, the moment of inertia of the particle at P is 0, that of particle Q is ma2 , and of the particle at
R is m(a/2)2 .
2
The moment of inertia of the three particle system about PX is I = 0 + ma2 + m(a/2)2 = 5ma
4
Note that the particles on the axis do not contribute to the moment of inertia.

(ii) Moment of inertia about the side PR = mass of particle Q × square of perpendicular distance of Q from side PR,
2
 3  3ma2
IPR = m  a =
 2  4
 
a
(iii) Distance of centroid from each of the particles is , so moment of inertia about an axis passing through the
3 2
 a 
centroid and perpendicular to the plane of triangle PQR = IC = 3m 
 3 
= ma2
 
P hysics | 7.7

Table 7.1: Formulae of MOI of symmetric bodies

S. No. Body, mass M Axis Figure I K(Radius of Gyration)


1. Ring or loop of Through its R
MR 2
radius R center and
perpendicular
to its plane
2. Disc, radius R Perpendicular
to its plane MR 2 R
through its 2 2
center

3. Hollow cylinder, Axis of cylinder


MR 2 R
radius R

4. Solid cylinder, Axis of cylinder


radius R MR 2 R
2 2

5. Thick walled Axis of cylinder


cylinder,
(
M R12 + R 22 ) R12 + R 22
2 2
R1
R2

6. Solid sphere, Diameter


radius R 2 2
MR 2 R
5 5

7. Spherical shell Diameter


radius, R 2MR 2 2
R
3 3

8. Thin rod, length L Perpendicular


to rod at ML2 L
middle point 12 2 3
7 . 8 | Rotational Mechanics

9. Thin rod, length L Perpendicular


to rod at one ML2 L
end 3 3

10. Solid cylinder, Through


length l center and MR 2 Ml2 R 2 l2
R + +
perpendicular 4 12 4 12
to length
l

11. Rectangular sheet, Through


length l and center and M(l2 + b2 ) l2 + b2
breadth b perpendicular 12 12
to plane
b

l l

PLANCESS CONCEPTS

While deriving the MOI of any rigid body the element chosen should be such that:
Either perpendicular distance of axis from each point of the element is same or the moment of inertia of
the element about the axis of rotation is known.
Nitin Chandrol (JEE 2012 AIR 134)

5.1 Theorems on Moment of Inertia


1. Theorem of Parallel Axes: This theorem is very useful in cases when the moment Z ZC
of inertia about an axis zC passing through the center of mass (C.O.M) of the rigid body
is known, and we sought to find the moment of inertia about any other axis z which is
parallel to the axis zC as shown in Fig. 7.8. The moment of inertia of the rigid body about
axis z is equal to the sum of the moment of inertia about axis zC and the product of the
mass m of the body by the square of perpendicular distance between the two axes. If the d
COM
moment of inertia of the rigid body about axis zC is IC , then the moment of inertia I of
this body about any parallel axis z , is given by I = IC + Md2 …(i)
where d is the perpendicular distance between the two axes.

Illustration 5: Find the moment of inertia of a uniform sphere of mass m and radius R
about a tangent if the sphere is (i) solid (ii) hollow  (JEE MAIN) Figure 7.8: Parallel axes

Sol: We know the formula for moment of inertia of sphere about an axis passing through its center. Use the parallel
axes theorem to find the moment of inertia about the tangent.
(i) Using parallel axis theorem
P hysics | 7.9

I = IC + md2 R R
For solid sphere
2 7
IC = mR 2 , d = R ; I= mR 2
5 5
(ii) Using parallel axis theorem Solid sphere Hollow sphere
2
I = IC + md
For hollow sphere
Figure 7.9
2 5
IC = mR 2 , d = R ; I = mR 2
3 3


Illustration 6: Find the moment of inertia of the two uniform joint roads having mass m each
P
X
about point P as shown in Fig 7.10. Use parallel axis theorem.  (JEE MAIN)

Sol: We know the formulae for moment of inertia of rod about the axes passing through its
center and through one of its ends and perpendicular to it. Use the parallel axes theorem to
find the moment of inertia about the point P.
Figure 7.10
m2
Moment of inertia of rod 1 about axis P, I1 = P  1
3
2 /2
m2   5/2
Moment of inertia of rod 2 about axis p, I2 = + m 5  COM
12  2

5m2 2
So moment of inertia of a system about axis p; I =
3 Figure 7.11

lz

2. Theorem of Perpendicular Axes: This theorem is applicable only in case of


two dimensional rigid body or planar lamina as shown in Fig. 7.12. Let the lamina
lie in the x-y plane and Ix and I y be the moment of inertia of the lamina about lx
x and y axes respectively then the moment of inertia about z-axis perpendicular
to the plane of the lamina and passing through the point of intersection of x and
y axes is given as:
ly
Iz = Ix + I y …(ii)
Figure 7.12: Perpendicular axes

Illustration 7: Find the moment of inertia of a half-disc about an axis perpendicular to


the plane and passing through its center of mass. Mass of this disc is M and radius is R.
 (JEE MAIN) C.M 4R
 3
Sol: We know the formula for the moment of inertia of the half disc about a perpendicular
lA
axis through the center A. Use the parallel axes theorem to find the moment of inertia
about a perpendicular axis through the center of mass.
The COM of half disc will be at distance 4R/3π from the center A. Let moment of inertia
of half disc about a perpendicular axis passing through A be IA.
Figure 7.13
First we fill the remaining half with same density to get a full disc of mass 2M.
The moment of inertia about center A of full disc will be 2IA,
2 2
2MR 2 MR 2  4R  MR 2  4R 
So, IA = = ; I A = ICM + M ×   ; ICM = − M×  
2×2 2  3π  2  3π 
7 . 1 0 | Rotational Mechanics

Illustration 8: Calculate the moment of inertia of a uniform disc of mass M and B


radius R about a diameter.
 (JEE MAIN) Z

Sol: For a uniform disc all diameters are equivalent, i.e. moment of inertia about any
diameter will be equal to that about any other diameter. We know the formula for C 0 D
moment of inertia of disc about axis perpendicular to its plane and passing through
its center. Use the perpendicular axes theorem to find the moment of inertia about
a diameter.
A
Let AB and CD be two mutually perpendicular diameters of the disc. Take them as x
and y axes and the line perpendicular to the plane of the disc through the center as Figure 7.14
1
the Z – axis. The moment of inertia of the ring about the Z – axis is I = MR 2 . As the
disc is uniform, all of its diameters are equivalent and so I = I 2
x y
I MR 2
From perpendicular an axis theorem Iz = Ix + I y ; hence Ix = z =
2 4

1
Illustration 9: In the Fig 7.15 shown find the moment of inertia of square plate having
I 2
mass m and sides a about axis 2 passing through point C (center of mass) and in the
plane of plate. (JEE MAIN) C
3
Sol: For uniform square plate axes 2 and 4 along diagonals are equivalent and axes 1
I
a
and 3 are equivalent. Suppose IC is the moment of inertia about the axis perpendicular
to the plane of plate and passing through the center C. Use perpendicular axes theorem 4
to prove that the axes 1 and 2 are also equivalent. I a
Using perpendicular axes theorem IC = I 4 + I2 = I' + I'=2I' …. (i)

Figure 7.15
Using perpendicular axes theorem IC = I3 + I1 = I + I = 2I  ….(ii)
From (i) and (ii) we get I' = I
ma2 ma2
IC = 2I = ⇒ I' =
6 12

5.2 Radius of Gyration


The radius of gyration of a rigid body about an axis z is equal to the radius of a ring whose mass is equal to
the mass of the rigid body, and the moment of inertia of the ring about an axis passing through its center and
perpendicular to its plane is equal to the moment of inertia of the rigid body about the axis z. Radius of gyration
can also be defined as the perpendicular distance from the axis of rotation where all mass of the rigid body can be
assumed to be concentrated when the rigid body is performing pure rotation to get the equation of motion of the
body. Thus, the radius of gyration is the ‘’equivalent distance’’ of the rigid body from the axis of rotation.
I = MK 2
I = Moment of inertia of the rigid body about an axis
M = Mass of the rigid body
K = Radius of gyration about the same axis
I
or K= …(iii)
M
Length K is the property of the rigid body which depends upon the shape and size of the body and on the
orientation and location of the axis of rotation. S.I. Unit of K is meter.
P hysi cs | 7.11

Illustration 10: Find the radius of gyration of a hollow uniform sphere of radius R about its tangent. (JEE MAIN)

Sol: Use the formula for radius of gyration.


5
Moment of inertia of a hollow sphere about a tangent = MR 2
3
5 5
MK 2 = MR 2 ⇒ K = R
3 3

5.3 Moment of Inertia of a Body Having a Cavity


If we know the moments of inertia of different parts of a rigid body about the same axis, then the moment of inertia
of the entire body can be calculated by simply adding the moments of inertia of the different parts (about the same
axis) i.e. moment of inertia is an additive quantity. This principle can be used to calculate the moment of inertia of
a body having hollow spaces by first assuming the hollow spaces to be filled with same density as that of the body
and evaluating the moment of inertia of the whole body about the given axis and then add the moments on inertia
of the hollow spaces about the same axis considering them to have negative mass.

Illustration 11: A uniform disc of radius R has a round disc of radius R/3 cut as shown in Fig 7.16. The mass of the
disc equals M. Find the moment of inertia of such a disc relative to the axis passing through geometrical center of
original disc and perpendicular to the plane of the disc.  (JEE ADVANCED)

Sol: Consider the whole disc without the cavity. The cavity can be thought of as a negative mass of same density as
disc. We know the formula for moment of inertia of uniform disc about axis perpendicular to its plane and passing
through its center. Find the moment of inertia of cavity (negative mass) about the perpendicular axis passing
through center of whole disc. The moment of inertia of disc with cavity is the sum of the moment of inertia of whole
disc and the moment of inertia of cavity (negative).
Let the mass per unit area of the material of disc be σ . Now the empty space can be considered as having density
–σ
Now I0 = Iσ + I −σ

Iσ = (σπR 2 )R 2 /2 = MI of σ about O = MR 2 /2

−σπ(R/3)2 (R/3)2
I-σ = + [ −σπ(R / 3)2 ](2R/ 3)2 O
2 R/3
2
R
= M.I of − σ About O = -MR /18
I0 = MR 2 /2 − MR 2 /18
4
I0 = MR 2
9
Figure 7.16
6 TORQUE
6.1 Torque About a Point
Torque of force F relative to a point O is defined as Line of action
   of force
τ = r × F …(x)
  P
where F = force applied to a point on a body

r = position vector of the point of application of force relative to the point O in r
a chosen reference frame about which we want to determine the torque (see Fig. r sin
7.17). O
Torque is a vector quantity and its direction is given by the right hand rule for cross Figure 7.17: Torque of a force
product of vectors.
7 . 1 2 | Rotational Mechanics


Magnitude of torque | τ | = r F sin θ = r⊥F = rF⊥
 
where θ is the angle between the force F and the position vector r of point of application.
r⊥ = r sin θ = perpendicular distance of line of action of force from point O.
 
F⊥ = F sin θ = component of F perpendicular to r
SI unit of torque is N-m.

Illustration 12: Find the torque about point O and A. (JEE MAIN) y
o
30
Sol: Express the position vector of A relative to O in terms of unit vectors î and (1, 1)
o
ˆj . Force is given in terms of unit vectors î and ˆj . 60
    
Torque about point O, τ = r0 × F, r0 = ˆi + ˆj , F = 5 3 ˆi + 5jˆ

τ = (iˆ + ˆj) × (5 3iˆ + 5j)
ˆ = 5(1 − 3)kˆ
     x
Torque about point A, τ = ra × F, ra = ˆj , F = 5 3 ˆi + 5jˆ O A B

τ = ˆj × (5 3iˆ + 5j)
ˆ = -5 3kˆ
Figure 7.18

Illustration 13: A particle of mass m is released in vertical plane from a point on the x0
x – axis, it falls vertically along the y – axis. Find the torque τ about origin?  O P
 (JEE MAIN) 
r
Sol: Torque is produced by the force of gravity. This will be equal to the product of
force of gravity and the perpendicular distance between the line of action of force of

gravity and the origin O.
 
τ = rF sin θ kˆ Or τ =r⊥F = x0mg mg

x0 Figure 7.19
 = r mg = mgx0kˆ
r

6.2 Torque About An Axis


  A
The torque of a force F about an axis AB is the component of the torque of F
about point A along the axis AB. F P

Alternatively to find torque of force F about axis AB we choose any point O on r
    O
the axis AB and find the torque of F about O as τ0 = r × F . Then we calculate
 
the component of τ0 along AB to get τAB (see Fig. 7.20).
B
There are a few special cases of torque of a force about an axis:
  Figure 7.20: Torque about
Case I: Applied force is parallel to the axis of rotation, i.e. F || AB an axis
  
Therefore torque r × F about any point on the axis will be perpendicular to F and hence F A
    P
perpendicular to AB . Therefore the component of r × F along AB will be zero.
r
Case II: The line of action of the applied force intersects the axis of rotation O
 
( F intersects AB )
 
If we choose the point of intersection of line of action of F and AB as the origin O then
 
the position vector r and applied force F will be collinear (see Fig. 7.21). Therefore the B
 
torque about O is r × F = 0 and thus the component of this torque along line AB will Figure 7.21: Force
also be zero. intersects axis
P hysi cs | 7.13

  
Case III: Line of action of F and axis AB are skew and F ⊥ AB A

Let O be the origin on the axis AB and P be the point of application



of force F such that OP is perpendicular to the axis AB (see Fig. 7.22).
   P  
Then torque =τ OP × F will be parallel to axis AB and the component
 r
of τ along AB will be equal to its magnitude i.e. O
F
τAB = F×(OP)sinθ = F×l
where  = (OP)sinθ is the length of the common perpendicular to
the line of action of force and the axis called the lever arm or moment B
arm of this force.
Figure 7.22: Force and axis are skew

Illustration 14: Find the torque of


weight about the axis passing through r
point P.  (JEE MAIN)
m
Sol: Required torque is equal to the r
product of force of gravity and the X
perpendicular distance between the P
line of action of force of gravity and
the point P. 
  F
F = mg Downwards
   Figure 7.23
τ = F × r= F.r sin θ

Illustration 15: A bob of mass m is suspended at point O by string of length  . Bob is moving in a horizontal circle
find out. (i) Torque of gravity and tension about point O and O’ (ii) Net torque about axis OO’. (JEE ADVANCED)

Sol: Torque of a force about an origin is equal to the product of force and the perpendicular distance between the
line of action of force and the origin.
Z
(i) Torque about point O
O
Torque of tension (T), τnet = 0 (tension is passing through point O)
Torque of gravity τmg = mg  sin θ (along negative ˆj )



Torque about point O’ y


o
τmg = mgr = mg  sin θ (along negative ˆj )
x
Torque of gravity m r
v
Torque of tension τT = Tr sin (90 + θ) (T cos θ= mg)
mg Figure 7.24
τT
= Tr cos
= θ ( sin θ)cos
= θ mg sin θ (along positive ˆj )
cos θ
(ii) Torque about axis OO’ F=T

Torque of gravity about axis OO’ τmg = 0 (force mg is parallel to axis OO’) 
Torque of tension about axis OO’ τT = 0 (force T intersects the axis OO’) 90+
O
Net torque about axis OO’ τnet = 0 
r

6.3 Force Couple Figure 7.25

A pair of forces each of same magnitude and acting in opposite directions is called a force couple.
7 . 1 4 | Rotational Mechanics

Torque due to couple = magnitude of one force x distance F


between their lines of action. d

Magnitude of torque = τ = F (d)


A couple does not exert a net force on an object even though Rigid body y2
it exerts a torque.
Net torque due to a force couple is the same about any point
(see Fig. 7.26). y1 B

Total torque about A = x1F + x2F = F(x1 + x2 ) = Fd


F
Total torque about B = y1F - y 2F = F(y1 − y 2 ) = Fd x1 x2
Figure 7.26: Force couple

6.4 Torque on a Rigid Body Executing Pure Rotation


Suppose I is the moment of inertia of a rigid body about the axis of rotation
which is stationary in a given reference frame. The body is executing pure
rotational motion about this fixed axis.
text = resultant torque about the axis of rotation due to all the external forces
acting on the body
α = instantaneous angular acceleration of the body.
ω = instantaneous angular velocity of the body.
Fixed axis of
Consider one particle of the body say ith particle of mass mi at perpendicular Rotation
distance ri from the axis.
Figure 7.27: Rigid body executing
Radial force on the particle Fr = mω2r towards the center of its circular path. pure rotation
Tangential force on the particle Ft = miat = miαri
Torque of the radial force about the axis of rotation is zero as it intersects the axis. Torque of tangential force about
the axis will be,
2
τi = rF
i t = miri α

To find the total torque on the rigid body about the axis we take summation of torques acting on all the particles
of the body. The total torque comes out to be equal to the resultant torque due to external forces only as the
torques due to internal forces cancel each other in pairs when summation is taken on all the particles of the body
(By Newton’s third law of motion internal forces form pairs of equal and opposite collinear forces. So the lever arms
of the forces of a pair with respect to the axis will be equal so their torques will have equal magnitude but opposite
directions and cancel each other in the summation). So

τext= ∑ τi= (∑ miri2 ) α = I α …(i)


i i
Remember: This formula is applicable only for pure rotational motion of a rigid body about a fixed axis.

7. KINETIC ENERGY OF BODY IN PURE ROTATION


When a rigid body performs pure rotational motion about a given axis, all of its constituent particles move in
circular paths with centers on the axis and radii r1 , r2 …… and rn (say), and with linear velocities v1 = ω r1 ,
v 2 = ω r2 ,……….. and vn = ω rn . If m1 , m2 ,….. and mn are the masses of the particles then the total kinetic
energy of the rigid body is given by
1 1 1
K= m v2 + m v 2 + ......... + m v2 …(i)
2 1 1 2 2 2 2 n n
P hysi cs | 7.15

1 1 1
= m1 ω2r12 + m2 ω2r22 + .........+ mnω2rn2
2 2 2
1
= (m1r12 + m2r22 + .......... + mnrn2 )ω2
2
Now as we have learnt the term m1r12 + m2r22 + .......... + mnrn2 is the moment of inertia of the rigid body.

Hence, the rotational kinetic energy of a body is given by


1 2
K= Iω  …(ii)
2

PLANCESS CONCEPTS

Most of the problems involving incline and a rigid body, can be solved by using the conservation of
energy. Care has to be taken in writing down the total Kinetic energy. Rotational Kinetic Energy term has
to be taken into consideration along with translational kinetic energy. And while writing the rotational
energy, the axis about which the moment of inertia is taken should be carefully chosen.
The point about which the conservation is done should be inertial to avoid calculating the work done
by pseudo forces or the point itself should be the COM so that the work done by the torque of pseudo
forces would be 0.
Shrikant Nagori (JEE 2009 AIR 30)

Illustration 16: A uniform circular disc has radius R and mass m. A particle, A
also of mass m, is fixed at a point A on the edge of the disc as shown in Fig
7.28. The disc can freely rotate about a fixed horizontal chord PQ that is at a R
distance R/4 from the center C of the disc. The line CA is ⊥ to PQ. Initially the
disc is held vertical with point A at its highest point. It is then allowed to fall C
so that it starts rotating about PQ. Find the linear speed of the particle as it R/4
reaches lowest point.  (JEE ADVANCED) P Q

Sol: Find the moment of inertia of circular disc and the particle at point A
about the chord PQ. The loss in potential energy of the system comprising the Figure 7.28
disc and the particle will be equal to the gain in its rotational kinetic energy.
2 2
1 mR 2 R   5R  15mR 2 A
I= × + m   + m  =
2 2 4  4  8
Energy equation C
O
5R mgR 1 5R mgR
mg + = Iω2 − mg −
4 4 2 4 4 C
g
ω=4
5R
5Rω A’
V= = 5gR
4 Figure 7.29

Illustrations 17: A pulley having radius r and moment of inertia I about its axis is fixed at the top of an inclined
plane of inclination θ as shown in Fig 7.30. A string is wrapped round the pulley and its free end supports a block
7 . 1 6 | Rotational Mechanics

of mass m which can slide on the plane initially. The pulley is rotated at a
speed ω0 in a direction such that the block slides up the plane. Calculate the
distance moved by the block before stopping?  (JEE ADVANCED)

Sol: Apply Newton’s second law of motion for block M along the inclined M
plane. Find the torque (about its axis) of force of tension acting on pulley.
This will be equal to the product of moment of inertia I and the angular
acceleration of pulley. 
Suppose the deceleration of the block is a. The linear deceleration of the rim
of the pulley is also a. The angular deceleration of the pulley is α = a/r. If the Figure 7.30
tension in the string is T, the equations of motion are as follows:
ma   and Tr = Iα  = Ia / r.         
mg sinθ  – T =
Eliminating T from these equations,
a mgr 2 sin θ
mg sinθ – I ma; Giving, a =
=
r2 I + mr 2
The initial velocity of the block up the incline is v = ω0r Thus, the distance moved by the block before stopping is
2
v2 (I + mr )ω20
x= =
2a 2mg sin θ

Illustration 18: A uniform rod of mass m and length  can rotate in vertical plane 
about a smooth horizontal axis hinged at point H. H X
(i) Find angular acceleration α of the rod just after it is released from initial /2 mg
A
horizontal position from rest?
Figure 7.31
(ii) Calculate the acceleration (tangential and radial) of point A at this moment.
(iii) Calculate net hinge force acting at this moment.
(iv) Find α and ω when rod becomes vertical.
(v) Find hinge force when rod become vertical. (JEE ADVANCED)
Sol: The axis of rotation passing through H is fixed. So the torque of force of gravity about axis through H is equal
to the product of moment of inertia about axis through H and angular acceleration of rod. Angular acceleration at
an instant can be found if the torque of force of gravity at the instant is known.
(i) τH = IH α

 m2 3g 
mg. = α ⇒ α=
2 3 2 X
3g 3g H A
(ii) atA = α = . =
2 2 Figure 7.32
2
aCA = ω r = 0. = 0 (∵ ω = 0 just after release)
N1
(iii) Suppose hinge exerts normal reaction in component form as shown
In vertical direction
Fext = maCM
3g N2
⇒ mg − N1 = m.
4
(We get the value of aCM from previous example) Figure 7.33

mg
⇒ N1 =
4
P hysi cs | 7.17

In horizontal direction
Fext = maCM ⇒ N2 = 0 (∵ aCM in horizontal = 0 as ω = 0 just after release)

(iv) Torque = 0 when rod becomes vertical so α = 0

Using energy conservation mg = 1 Iω2


 m2 
 I= 
2 2 
 3 

(Work done by gravity when COM moves down by (½)  = change in K.E.)
3g
ω=

(v) When rod becomes vertical

3g
α = 0, ω = (Using Fnet = MaCM )

mω2 
FH - mg = ( aCM = centripetal acceleration of COM)
2
5mg
Ans. FH =
2

Bar
Illustration 19: A bar of mass m is held as shown between 4 disks each of mass m’ and radius
r = 75 mm. Determine the acceleration of the bar immediately after it has been released
from rest, knowing that the normal forces exerted on the disks are sufficient to prevent any
slipping and assuming that. (a) m = 5 kg and m’ = 2 kg.
A
(b) The mass m’ of the disks is negligible.
(c) The mass m of the bar is negligible (JEE ADVANCED)
B
Sol: Apply Newton’s second law of motion in vertical direction for the motion of center
of mass of bar. Write the equation of torque due to force of friction acting on disc, for
rotational motion about fixed axis through center of disk. Acceleration of rod will be equal to Figure 7.34
the tangential acceleration of the disc in the case of no slipping.
(a) Equation of center of mass of rod,
mg – 4f = ma ….(i)
(where f is frictional force from one disk)
Torque acting on each disk due to frictional force is
m'r 2 a
fr =  ….(ii)
2 r
From (i) and (ii) we get
mg – 2m’ a = ma  ….(iii)
5g
5g = (5 + 2 × 2)a; a=
9
(b) Putting m’=0 in eqn. (iii) we get a = g
(c) Putting m = 0 in eqn. (iii) we get a = 0
5g
(a) ↓
9
(b) g ↓ c) 0
7 . 1 8 | Rotational Mechanics

7.1 Work Done and Power Delivered by Torque


If a torque τ rotates a body through an angle dθ, the work, dW done by it is given by
dW = τ.dθ
The total work done W in rotating a body from the initial angle θ1 to the final angle θ2 , is
θ2 ω2 ω2
dω 1 2 1 2
W= ∫ τ.dθ = ∫ I
dt
ωdt = ∫ I ω dω = Iω − Iω
2 2 2 1
θ1 ω1 ω1

So the work done by torque is equal to the change in the rotational kinetic energy.
1 
W = ∆Krot = ∆  Iω2  …(i)
2 
This is called the Work-Energy Theorem for rotation of rigid body.
The rate at which work is done is called power P, given by
dW dθ
P == τ = τω …(ii)
dt dt
Also, the power P delivered by the torque on the rigid body is equal to the rate of change of kinetic energy
1 2 dK d 1 2 
K= Iω ∴ P = =  Iω 
2 dt dt  2 
1 dω dω
∴ P = × I × 2ω= I ω=τω
2 dt dt

8. EQUILIBRIUM OF RIGID BODIES


A rigid body can be in linear equilibrium as well as in rotational equilibrium. If a rigid body is in linear equilibrium,
then the vector sum of all the forces acting on it should be zero.

i.e. Σ Fext = 0
Taking scalar components along the three axes x, y and z we get ΣFx = 0, ΣFy = 0, ΣFz = 0
If a rigid body is in rotational equilibrium then the vector sum of all the external torques acting on it with respect
to an axis in a given reference frame must be zero.
Στext = 0 ⇒ Στx = 0, Στy = 0, Στz = 0

Illustrations 20: Two boys weighing 20 kg and 25 kg are trying to balance a seesaw of total length 4 m, with the
fulcrum at the center. If one of the boys is sitting at an end, where should the other sit?  (JEE MAIN)

Figure 7.35

Sol: For rotational equilibrium, the net torque about the fulcrum of all the forces acting on the boys and the seesaw
should be zero.
It is clear that the 20 kg kid should sit at the end and the 25 kg kid should sit closer to the center. Suppose his
P hysi cs | 7.19

distance from the center is x. As the boys are in equilibrium, the normal force between a boy and the seesaw equals
the weight of that boy. Considering the rotational equilibrium of the seesaw, the torque of the forces acting on it
should add to zero. The forces are
(a) (25kg) g downward by the 25 kg boy
(b) (20kg) g downward by the 20 kg boy
(c) Weight of the seesaw and
(d) The normal force by the fulcrum.
Taking torques about the fulcrum.
(25 kg) g x = (20 kg) g (2 m) or x = 1.6 m

9. ANGULAR MOMENTUM
9.1 Angular Momentum of a Particle About a Point

If p is the linear momentum of a particle in a given reference frame, then angular momentum of the particle about
an origin O in this reference frame is defined as
  
L = r × p …(i)

where r is the position vector of the particle with respect to origin O (see Fig. 7.36).
Magnitude of angular momentum =
is L r p sinθ Pcos
or L = r⊥ p or L = p⊥ r  
  P
θ = angle between vectors r and p Psin


 r
r⊥ = component of position vector r perpendicular to vector p .
 
p⊥ = component of vector p perpendicular to position vector r . o
2
SI unit angular momentum is kg m s . -1
Figure 7.36: Angular
momentum about a point
Relation between Torque and Angular Momentum
  
∵ L = r × p
Differentiating with respect to time we get
  
dL dr   dp       
= × p+r × = v × (mv) + r × F = 0 + r × F = τ
dt dt dt

dL 
⇒ = τ  …(ii)
dt
For a single particle moving in a circle of radius r with angular velocity ω we have
v = ωr and p = mωr
So angular momentum comes out to be L = r p= mr2ω

Y
Illustration 21: A particle of mass m is projected at time t = 0 from a
point O with a speed u at an angle of 45° to the horizontal. Calculate the v
magnitude and direction of the angular momentum of the particle about
the point O at time t = u/g.  (JEE ADVANCED) r
X
Sol: Express the position and velocity of particle in Cartesian coordinates O
in terms of unit vectors î and ˆj and then calculate the cross product in Figure 7.37
7 . 2 0 | Rotational Mechanics

Cartesian coordinates.
Let us take the origin at O, X –horizontal axis and
Y – Axis along the vertical upward direction as shown in Fig 7.37 for horizontal during the time 0 to t.
u u u2
v x = u cos45°= u/ 2 and x = v x t = . =
2 g 2g
For vertical motion,
u (1 − 2 )
v y = u sin 45° - gt = − u= u
2 2
1 u2 u2 u2
and y = (u sin 45 º) t − gt2 = − = ( 2 − 1)
2 2g 2g 2g
The angular momentum of the particle at time t about the origin is
    
L r × P = m r × v = m(ix ˆ + ˆjy) × (iv
ˆ + ˆjv ) = m(kxv ˆ ˆ
= x y y − kyv x )

 u2  u u2 u2  mu3
= m kˆ   (1 − 2) − ( 2 − 1)  = − kˆ
 2g  2 2g 2  2 2g
 
mu3
Thus, the angular momentum of the particle is in the negative z – direction i.e., perpendicular to the plane
of motion, going into the plane. 2 2g

Illustration 22: A cylinder is given angular velocity ω0 and kept on a horizontal rough surface the initial velocity is
zero. Find out distance travelled by the cylinder before it performs pure rolling and work by frictional force. 
 (JEE ADVANCED)
Sol: Due to backward slipping force of friction will act forwards. The cylinder is accelerated forwards. The torque
due to friction and hence the angular acceleration is opposite to the initial angular velocity. So the angular velocity
will decrease and the linear velocity of center of mass of cylinder will increase in the forward direction, till the
slipping stops and pure rolling starts. The work done by frictional force is equal to change in the kinetic energy of
the cylinder. The kinetic energy includes both rotational kinetic energy and translational kinetic energy.
MR 2 α
µMg R =
2
2µg 0 v
α=
R  … (i) R

Initial velocity u = 0 a v
R
2 2
v = u + 2as
fk
v 2 = 2as  … (ii)
fK = ma; µMg = Ma ;
S
a = µg … (iii)
Figure 7.38
ω = ω0 − αt
2µg
From equation (i) ω = ω0 − t ; V = u + at
R
From equation (iii) v = µ g t
2v ω
ω = ω0 − ; ω = ω0 − 2ω ; ω = 0
R 3
From equation (ii)
2
 ω0R   ω 2R 2 
  = (2as) = 2µ gs ; S=  0 
 3   18µg 
 
P hysi cs | 7.21

Work done by the frictional force

µmg × ω20R 2
W = ( − fkRdθ + fk ∆s) = − µmgR∆θ + ;
18µg
  ω0R  1 2µg  ω0R  
2
 ω2R ω2R 
1 2
×
 2ω20R
∆θ = ω0 × t − αt =  0 
ω  ×    =  0 − 0  =
2   3µg  2 R  3µg    3µg 9µg  9µg
  
 2ω20R   ω2R 2   mω20R 2
W =  −µmg × R 
 +  µmg × 0   = −
 9µg 



18µg  

6

Illustration 23: A hollow sphere is projected horizontally along a rough surface with speed v and angular velocity
v
ω0 . Find out the ratio , so that the sphere stops moving after some time.  (JEE ADVANCED)
ω0
Sol: For the sphere to stop after sometime, the acceleration should be opposite to velocity, i.e. the force of friction
should be backwards (forward slipping). Also, the torque due to friction should be opposite to angular velocity, i.e.
if the torque due to friction is clockwise (see Fig. 7.39), then the initial angular velocity should be anti-clockwise.
Torque about lowest point of sphere
2 3 ωg
fK × R = Iα ; µmg × R = mR 2 α ; α= (Angular acceleration in opposite direction of angular velocity)
3 2R
0
ω = ω0 − αt (Final angular velocity ω = 0)
a=0
3 ωg ω0 × 2R a v v=0
ω0 = × t ; t=
2R 3ì g
Acceleration a = μg ft v
v t = v – at       (Final velocity v t = 0); 0R
v Figure 7.39
v = µg × t ; t =
µg
To stop the sphere, time at which v and ω are zero, should be same.

v 2ω0R v 2R
= ; ⇒ =
µg 3ì g ω0 3

Illustration 24: A rod AB of mass 2m and length  is lying m m v1=0 A


v0 A
on a horizontal frictionless surface. A particle of mass m
travelling along the surface hits the end of the rod with a
velocity v 0 in a direction perpendicular to AB. The collision /2 /2
is elastic. After the collision the particle comes to rest. Find
out after collision
2m COM COM v
(a) Velocity of center of mass of rod 
(b) Angular velocity.  (JEE ADVANCED) /2 /2

Sol: Conserve linear momentum and angular momentum of


the system constituting “the rod and the particle” before and B B
after collision. Here the linear and angular momentum of the Before collision After collision
rod before collision is zero. Angular momenta of the rod and
Figure 7.40
particle are calculated about the center of the rod.
7 . 2 2 | Rotational Mechanics

(a) Let just after collision the speed of COM of rod is v and angular velocity about COM is ω .
External force on the system (rod + mass) in horizontal plane is zero.
Apply conservation of linear momentum in x direction;
mv 0 = 2mv  ….. (i)
(b) Net torque on the system about any point is zero
Apply conservation of angular momentum about COM of rod.
  2m2
mv 0 = Iω ⇒ mv 0 = ω  …... (ii)
2 2 12
v0
From equation (i) velocity of center of mass v =
2
3v 0
From equation (ii) angular velocity ω = .

9.2 Angular Momentum of a Rigid Body Rotating About Fixed Axis


For a system of particles the total angular momentum about an origin is the sum of the angular momenta of all the
particles calculated about the same origin.
  A
L =∑ Li 
i
Differentiating with respect to time we get, r1
  m1
dL dLi     
dt
= ∑ dt = ∑ (∑ τik + τiext ) = ∑∑ τik + ∑ τiext = 0 + τext r2
i i k i k i m2
The double summation term corresponds to the sum of torques due to rn
internal forces and as explained earlier, according to Newton’s third law mn
of motion these internal torques cancel out in pairs.
So for a system of particles
 Axis of rotation
dL  B
= τext …(xvii)
dt
 
Impulse of a torque is defined as J = ∫ dL= ∫ τext dt Figure 7.41: Angular momentum
of rigid body
Angular momentum of a rigid body rotating about a fixed axis can be
calculated as below:
Angular momenta of its individual particles about the axis are

L1 = m1r12 ω, L2 = m2r22 ω, L3 = m3r32 ω, .... Ln = mnrn2 ω where ω is the instantaneous angular velocity of the
rigid body
Total angular momentum of the body
L = m1r12 ω + m2r22 ω + m3r32 ω ............+ mnrn2 ω
L= ∑ mi (ri )2ω = I ω
i

So L = I ω
Remember: This formula is applicable only for rotation of the rigid body about a fixed axis.
Again differentiating this relation with respect to time we get,
dL dω
=I = I α = τext
dt dt
P hysi cs | 7.23

Illustration 25: Two small balls of mass m each are attached to a light rod of length  , one at its center and the
other at its free end. The rod is fixed at the other end and is rotated in horizontal plane at an angular speed ω .
Calculate the angular momentum of the ball at the end with respect to the ball at the center.  (JEE MAIN)

Sol:Both the balls A and B have same angular velocity but different linear velocities.

The situation is shown in Fig 7.42. The velocity of the ball A with respect to
the fixed end O is v A = ω ( /2) and that of B with respect to O is vB = ω A
. Hence the velocity of B with respect to A is vB – v A = ω( /2) . The angular B
O 
momentum of B with respect to A is, therefore,
 1 Figure 7.42
L = mvr = mω   = mω2
22 4
along the direction perpendicular to the plane of rotation.

9.3 Conservation of Angular Momentum


In the previous article we have proved the relation
 
dL   ext
= τext where L and τ are evaluated about the same origin.
dt 

From the above equation we see that if τext =
0 then L of the system of particles remains constant.
In some situations the component of external torque about an axis is zero even if the net external torque is not
zero. So in these cases the component of the total angular momentum, about the particular axis, remains constant.

Illustration 26: A uniform rod of mass m and length  can rotate freely on a smooth horizontal plane about a
vertical axis hinged at point H. A point mass having same mass m coming with an initial speed u perpendicular to
the rod strikes the rod in-elastically at its free end. Find out the angular velocity of the rod just after collision? 
 (JEE MAIN)

Sol: After collision the rod and the particle execute pure rotational motion about vertical axis ml
through fixed point H. +
H
Angular momentum is conserved about H because no external force is present in horizontal u
plane which is producing torque about H.
 m2  3u m
mul =  + m2  ω ⇒ ω=
 3  4
  Figure 7.43

Illustration 27: A uniform rod of mass m1 and length  lies on a frictionless horizontal plane. A particle of mass
m2 moving at a speed v 0 perpendicular to the length of the rod strikes it at a distance  /3 from the center and
stops after the collision. Calculate (a) the velocity of the center of the rod and (b) the angular velocity of the rod
about its center just after the collision. (JEE ADVANCED)
w
Sol: Conserve the linear momentum of the system comprising
“the rod and the particle” before and after the collision. Conserve R

the angular momentum, about the center of the rod, of the system A  v
A
comprising “the rod and the particle” before and after the collision. /3
The situation is shown in the Fig 7.44. Consider the rod and the
particle together as the system. As there is no external resultant force,
the linear momentum of the system will remain constant. Also there (A) (B)
is no resultant external torque on the system and so the resultant
Figure 7.44
external torque on the system and the angular momentum of the
7 . 2 4 | Rotational Mechanics

system about the line will remains constant. Suppose the velocity of the center of the rod is V and the angular
velocity about the center is ω .
(a) The linear momentum before the collision is m2v0 and that after the collision is M1V.
m 
Thus m2 v 0 = m1 v, or V =  2  v 0
m 
 1
(b) Let A be the center of the rod when at rest. Let AB be the line perpendicular to the plane of the Fig 7.44.
Consider the angular momentum of N “the rod plus the particle” system about AB.
Initially the rod is at rest. The angular momentum of the particle about AB is
L = m2 v 0 ( /3)
After collision the particle comes to rest. The angular momentum of the rod about a is
   
L = LCM + m1 r0 × v
     
As r0 || v , r0 × v = 0 thus, L = LCM

Hence the angular momentum of the rod about AB is


m1 2 m2 v m1 2 4m2 v 0
L L==Iùlw= ω Thus, = ω Or ω=
12 3 12 m1 

10. RIGID BODY IN COMBINED TRANSLATIONAL AND ROTATIONAL


MOTION
As discussed earlier, in this type of motion the rigid body is performing pure rotational motion about an axis and
the axis itself is performing pure translational motion relative to a given reference frame.
Consider a car moving over a straight horizontal road with some instantaneous velocity v with respect to a reference
frame K fixed to the road. Now let us observe the motion of a wheel of the car from the K frame. This motion of the
wheel in K frame is an example of combined translational and rotational motion. Let us suppose a reference frame
K’ which is translating with respect to frame K with same instantaneous velocity v. In other words frame K’ is rigidly
fixed to the body of the car. In this frame the wheel of the car performs pure rotational motion. The body of the car
itself is performing pure translational motion.
Take another example of motion of a fan fixed inside the car.
If the fan is switched off while the car is moving on the road, the motion of fan is pure translational with respect
to K frame.
If the fan is switched on while the car is at rest, the motion of fan is pure rotational about its axis, as the axis is at
rest in the K frame.
If the fan is switched on while the car is moving on the road, the motion of the fan with respect to K frame is neither
pure translational nor pure rotational but a combination of both. Now if an observer A is sitting inside the car, as
the car moves, the motion of fan will appear to him as pure rotational while the motion of the observer A with
respect to K frame is pure translational. Hence in this case we can see that the motion of the fan can be resolved
into two components, pure rotational motion relative to observer A and pure translational motion of observer A
relative to K frame.
Such a resolution of motion of a rigid body into components of pure rotational and pure translational motion is an
important tool used in the study of their dynamics.

10.1 Kinematics of a General Rigid Body Motion


For a rigid body the value of angular displacement θ, angular velocity ω, and angular acceleration α is same for
all points on the rigid body. Also, if we choose any point of the rigid body as origin O and any other point, say A,
P hysi cs | 7.25


of the body has a position vector r relative to O, and during any 
vA

time interval the vector r rotates by an angle θ relative to its initial 
direction, then position vector of any other point, say B, relative to
  
vB = vA + vBA
any other origin, say O’, inside the rigid body will also rotate by the B
same angle θ. This means the angular variables θ, ω, and α do not
depend on the choice of origin in the rigid body.
  
vBA =  x rBA

The above concept is very important as it enables us to calculate 


vA
the velocity of each point of the rigid body if we know the velocity
A
of any one point (say A) in the rigid body with respect to a reference rigid body
frame K and angular velocity of any point in the rigid body relative
to any other point in the rigid body.
Suppose we want to calculate thevelocity of a point B in the rigid
Figure 7.45: Kinematics of rigid body
body which has a position vector rBA relative to A (see Fig. 7.45).

The velocity of point A is v A , so we have velocity of B as
     
vB = v A + vBA = v A + ω × rBA

Direction of ù is given by right hand thumb rule. If we curl the fingers of the right hand in the direction of rotation

 ω .
of the body, thumb gives the direction of  
Similarly the acceleration of point B is: aB = aA + α × rBA

Illustration 28: Consider the general motion of a wheel (radius r) which can be viewed as pure translation of its
center O (with the velocity v) and pure rotation about O (with angular velocity ù )
       
Find out v AO , vBO , v CO , vDO and v A , vB , v C ,vD  (JEE MAIN)

Sol: Express the angular velocity, linear velocity of point O and C j


position vectors of points A, B and C relative to O in Cartesian =(-k)
coordinates.
   
v AO = (ω × r AO ) = ω( −k) ( ˆ × OA
) D

V B
i

= ω( −k)

( 
)
ˆ × r( −ˆj) = −ωri

O
ˆ ; vDO = ωr(ˆj)
Similarly vBO = ωr ( −ˆj) ; v CO = ωr(i)
  
v A = v O + v AO = v ˆi − ωriˆ ; A
   Figure 7.46
vB = v O + vBO = v ˆi + ωrjˆ
     
v C = v O + v CO = v ˆi + ωr ˆi ; vD = v O + vDO = v ˆi + ωrjˆ

10.2 Dynamics of a General Rigid Body Motion


Combined rotation and translation of a rigid body is considered as combination of pure rotation in C frame about
an axis passing through the center of mass and translation of center of mass in a reference frame K. Dynamics of
combined rotational and translational motion of a rigid body in K frame is defined by two vector equations. One
of them describes the dynamics of the center of mass of the rigid body in the K frame, and the other the equation
of dynamics of pure rotation of the body about center of mass in the C frame.

So if the total mass of the rigid body is M and the net external force acting on it is Fext then we have in the K frame,

dVC 
M = Fext  …(i)
dt

If IC is the moment of inertia of the rigid body about the axis passing through center of mass and τC is the net
torque of all external forces about the axis passing through the center of mass, then we have in the C frame,

 dω 
τC = IC = IC α …(ii)
dt
7 . 2 6 | Rotational Mechanics

 
If Ptotal is the total linear momentum
 of the rigid body in the K frame, L C is angular momentum of the body in C
frame about center of mass and r C is the position vector of center of mass relative to some origin in K frame, then
we have,
 
Ptotal = MVC
Total Kinetic energy
1 1
K = MVC2 + IC ω2  …(iii)
2 2
 
LC = IC ω  …(iv)
 
Angular momentum in K frame = LC about C.O.M + L of the C.O.M about some origin in K frame
    
L = IC ω + r C × MVC …(v)

10.3 Pure Rolling (Rolling Without Slipping)


Pure rolling is a special case of combined translational and rotational motion of a rigid body with circular cross
section (e.g. wheel, disc, ring, cylinder, sphere etc.) moving on a surface. Here, 
there is no slipping between the rolling body and the surface at the point of R
contact.
V
Suppose a sphere rolls on a stationary surface and the point of contact between
the sphere and the surface is A (see Fig. 7.47). Let the velocity of the center of
sphere be v, radius be R and its angular velocity be ω. For pure rolling the relative A
velocity between the point A of the sphere and the surface must be zero. As the
surface is at rest, the velocity of point A is also zero. Figure 7.47: Sphere rolling on a
stationary surface.
∴ v A = v - ωR = 0
∴ v = ωR
If sphere is rolling on a plank moving velocity v0, then for pure rolling, v A = v - ωR = v 0 (see Fig. 7.48)
Same is true for the tangential acceleration of the point of contact in case 
of pure rolling.
Now let’s discuss the case where a rolling cylinder of mass m moves forward
V
on a rough plate of same mass with acceleration “a” and the rough plate
moves forward with an acceleration “a0” under action of force F on a smooth
surface. A
As the cylinder accelerates in the forward direction, so by Newton’s second V0
law, the friction on the cylinder at the point of contact will be in forward
direction and on the plate in backward direction by Newton’s third law (see Figure 7.48: Sphere rolling on a
Fig. 7.50). moving surface.

Equation of torque about center of cylinder:


mR 2
τ=
C fR
= α  a
2 m Rough
2f
⇒ α=  ……. (i)
mR Smooth
a0
m F
Equation of motion of center of cylinder:
f = ma  …… (ii)
From (i) and (ii) we get Figure 7.49: Cylinder rolling on an accelerating plate.
αR
a=
2
P hysi cs | 7.27

At contact point a0
3αR f
a0 = a + aR = = 3a …… (iii) a
2
 m m F
Equation of motion of plate: f
F – f = ma0
Figure 7.50: (a) FBD of Cylinder. (b) FBD of Plate.
F = m(a + a0)
F 3F
F = 4ma ; a = ; a0 =
4m 4m

Illustration 29: A wheel of radius r rolls (rolling without slipping) on a level B 


road as shown in fig 7.51.
r
Find out velocity of point A and B.  (JEE MAIN) v

Sol: Linear velocity of any point on the rim of the wheel has magnitude ωr in A
the reference frame of center of wheel (C-frame). Velocity in ground frame is
the vector sum of velocity in C-frame and the velocity of center of wheel.
Figure 7.51
Contact point at surface is in rest for pure rolling
Velocity of point is A zero.
So v = ω r
Velocity of point B = v + ω r = 2v

Illustration 30: A uniform sphere of mass 200 g rolls without slipping on a plane surface so that its center moves
at a speed of 2.00 cm s-1. Find its kinetic energy. (JEE MAIN)
Sol: The kinetic energy of sphere is the sum of the translational kinetic energy and the rotational kinetic energy.
As the sphere rolls without slipping on the plane surface its angular speed about center is

v 1 1 1 2 1
ω = cm . The kinetic energy is K = Icmω2 + Mv cm2 = . Mr 2 ω2 + Mv cm2
r 2 2 2 5 2

1 1 7 7
= Mv cm2 + Mv cm2 = Mv cm2 = (0.200 kg)(0.02 m s-1 )2 = 5.6 × 10−5 J
5 2 10 10

Illustration 31: A constant force F acts tangentially at the highest point of a uniform disc of mass m kept on a
rough horizontal surface as shown in Fig 7.52. If the disc rolls without slipping, calculate the acceleration of the
Center C and point A and B of the disc.  (JEE ADVANCED)

Sol: Apply Newton’s second law for the motion of center of mass of the disc. B
F
Find the torque of the force F and the force of friction acting on the disc at point r
A about the center of mass of the disc and thus obtain the equation relating C
the angular acceleration in the C-frame to the torques of all the external forces.
The situation is shown in Fig 7.52. As the force F rotates the disc, the point of
contact has a tendency to slip towards left so that the static friction on the A rough surface
disc will act towards right. Let r be the radius of the disc and be the linear
acceleration of the center of the disc. The angular acceleration about the center Figure 7.52
of the disc is
α = a/r, as there is no slipping.
7 . 2 8 | Rotational Mechanics

For the linear motion of the center,


F + f = ma  …….. (i)
And for the rotation motion about the center,

1  a  1
Fr – f r = I α = Fr – f r = I =  mr 2    or F – f = ma …….. (ii)
 2  r  2 
From (i) and (ii),
3 4F
2F = ma or a =
2 3m
Acceleration of point A is zero

 4F   8F 
Acceleration of point B is 2a = 2   =   .
 3m   3m 

Illustration 32: A circular rigid body of mass m, radius R and radius of gyration (k) rolls without slipping on an
inclined plane of an inclination θ. Find the linear acceleration of the rigid body and force of friction on it. What must
be the minimum value of coefficient of friction so that rigid body may roll without sliding?
 (JEE ADVANCED)

Sol: Apply Newton’s second law for the motion of center of mass of the rigid a/R
body. Find the torque of the force F and the force of friction acting on the rigid
body about the center of mass of the disc and thus obtain the equation relating
the angular acceleration in the C-frame to the torques of all the external forces. f
If a is the acceleration of the center of mass of the rigid body and f the force of mgcos
friction between sphere and the plane, the equation of translational and rotational
mg mgsin
motion of the rigid body will be
ma (Translational motion)
Mg sin θ – f =

fR = I α (Rotational motion)
Figure 7.53

f= I = m k 2 , due to pure rolling a = α R
R

Iα Iα mk 2 α R2 + k2 
mg sin θ − = m αR = mαR +=  ma + = a 
R R R  R
2


gsinθ gsin θ Iα mk 2a mg k 2 sin θ


a =  = ; f = = ⇒
 R2 + k2   k2  R R2 R2 + k2
 2   1 + 2 
 R   R 

mk 2
f ≤ mN; a ≤ µ ≤ mg cos θ
R2
k2 g sin θ tanθ tanθ
R2 × ≤ µg cos θ ; µ ≥ ; µmin ≥
R 2
(
k2 + R2 )  R 
1+ 2 
2  R2 
1+ 2 
 k   k 
P hysi cs | 7.29

PLANCESS CONCEPTS

•• From above example if rigid bodies are solid cylinder, hollow cylinder, solid sphere and hollow sphere
(having radius ‘r’ and mass ‘m’)
•• Increasing order of acceleration

a >a > a >a


solid sphere hollow sphere solid cylinder hollow cylinder

•• Increasing order of required friction force for pure rolling

f >f >f >f


hollow cylinder hollow sphere solid cylinder solid sphere
•• Increasing order of required minimum friction coefficient for pure rolling

µ >µ >µ >µ


hollow cylinder hollow sphere solid cylinder solid sphere

•• I would advise you to derive these, verify and remember!


Anand K (JEE 2011 AIR 47)

10.4 Instantaneous Axis of Rotation


The combined translational and rotational motion of a rigid body can be reduced to a purely rotational motion.
When we know the velocity VC of the center of mass and the instantaneous angular velocity ω of the body then
we can find a point whose velocity comes out to be zero at a given moment of time. The axis passing through this
point at the given moment is called instantaneous axis of rotation and the rigid body performs pure rotation about
this axis with same angular velocity at that moment.
The position of the instantaneous axis of rotation changes with time. E.g. in pure rolling the point of contact with
the surface is the instantaneous axis of rotation (see Fig. 7.54).

 (l.A.R.)
V+r=2v
V2
rig
id

v
ro
d

2v
P Instantaneous axis
V1

Figure 7.54: IAR (a) pure rolling; (b) Rod slipping down a wall

Geometrical construction of instantaneous axis of rotation (I.A.R). If we know the velocity vectors of any two
points in the rigid body then the I.A.R. is the axis passing through the point of intersection of the perpendiculars
drawn to the velocity vectors at those points.
Once location of I.A.R is known, we find the moment of inertia of the body about this axis, and then the equations
of rotation about fixed axis can be used for this axis.
0

Illustration 33: Prove that kinetic energy = 1/2 IP ω2  (JEE MAIN)


v
Sol: Kinetic energy is the sum of the translational kinetic energy and the rotational kinetic v=R
energy.
P
Figure 7.55
7 . 3 0 | Rotational Mechanics

1
K.E. =
1
2
1
2
1
Icmω2 + Mv cm2 = Icmω2 =
2
1
+ Mω2R 2
2 2
(Icm + MR 2 )ω2 =
1
I
2 contact point
ω2 ( )
Notice that in pure rolling of uniform object, equation of torque can also be applied about the contact point.

Illustration 34: A uniform bar of length  and mass m stands vertically


touching a vertical wall (y – axis). When slightly displaced, its lower end
begins to slide along the floor (x – axis). Obtain an expression for the angular
velocity ( ω ) of the bar as a function of θ . Neglect friction everywhere.
 (JEE ADVANCED)

Sol: As the rod falls, it executes pure rotational motion about the instantaneous
axis of rotation. The loss in gravitational potential energy is equal to the gain
in the rotational kinetic energy.

The position of instantaneous axis of rotation (IAOR) is shown in Fig 7.57.
Figure 7.56
   
C =  cosθ, sinθ  ; r= = half of the diagonal
2 2  2

All surfaces are smooth, therefore, mechanical energy will remain conserved.
∴ Decrease in gravitational potential energy of bar = increase in rotational
kinetic energy of bar about IAOR.
(lAOR)
 1
mg (1 – sinθ) = Iω2  ... (i) A

2 2 r
2
m
Here, I = +mr 2 (about IAOR) or I
12 C

2 2 2 O
m m m B
= + = Substituting in Eq. (i)
12 4 3 Figure 7.57

 1  m2  2 3g(1 − sin θ)


We have mg (1 – sinθ) =   ω or ω =
2 2  3  

PLANCESS CONCEPTS

Nature of friction for rigid bodies:


•• A rigid body rolling with a speed of v and angular velocity of ω at an instant. Then it falls under one of the
following cases.

Cases Rough/Smooth Diagram Inference


V<rω Rough Surface 1. There is relative motion at point of contact.
 With respect to the body the surface moves
v slower than itself. So the surface tries to
decrease its angular velocity by a frictional force
fk in forward direction. And this friction is kinetic
Rough surface
friction.

2. It increases v and decreases ω So, after


sometime, v =r ω and pure rolling will resume.
P hysi cs | 7.31

PLANCESS CONCEPTS

Cases Rough/Smooth Diagram Inference


Smooth Surface No friction is possible and it is not pure rolling.

v

Smooth surface
Rough surface With respect to the COM of the cylinder, the
surface moves at a higher speed than itself. So
v > rω 
v the surface tries to increase its angular velocity
by exerting a frictional force in backward
Rough surface direction. And this friction would be kinetic
friction.

2. The friction tries to reduce V and increase ω


V>rω Smooth Surface No friction and no pure rolling.

v

Smooth surface

V=rω Rough Surface This is pure rolling. However there might be


 static friction acting on the body.
v

Smooth surface

Smooth Surface No friction is possible and it is pure rolling



v

Smooth surface

Rohit Kumar (JEE 2012 AIR 79)

Illustration 35: A rigid body of mass m and radius r rolls without slipping on a surface. A force is acting on the rigid
body at x distance from the center as shown in Fig 7.58. Find the value of x so that static friction is zero. 
 (JEE MAIN)
Sol: For static friction to be zero, the linear and angular accelerations a and α
caused by the force F should be related as a = α R, for rolling without slipping.

P F
x
Torque about center of mass Fx = Icm α  … (i)
a
o
a=R
F = ma  … (ii) f
Rough surface
From equation (i) & (ii) max = Icmα (a = α R) ;
I Figure 7.58
x = cm
mR
7 . 3 2 | Rotational Mechanics

Illustration 36: There are two cylinders of radii R₁ and R2 having moments of
inertia I₁ and I2 about their respective axes as shown in Fig 7.59. Initially, the 1 2
cylinders rotate about their axes with angular speed w1 and w2 as shown in the R1
11 R2 12
Fig 7.59. The cylinders are moved close to touch each other keeping the axes
parallel. The cylinders first slip over each other at the contact but the slipping
finally ceases due to the friction between them. Calculate the angular speeds Figure 7.59
of the cylinders after the slipping ceases.  (JEE ADVANCED)

Sol: The force of friction acting on the cylinder moving faster will be such that its angular velocity decreases. The
force of friction acting on the cylinder moving slower will be such that its angular velocity increases. When slipping
ceases, the linear speeds of the points of contact of the two cylinders will be equal.
If w'1 and w'2 be the respective angular speeds at the instant slipping ceases, we have

w'1 R1 = w'2 R2  …(i)

The change in the angular speed is brought about by the frictional force which acts as long as the slipping exists. If
this force f acts for a time t. the torque on the first cylinder is fR1 and that on the second is fR2. Assuming w1 > w2.
The corresponding angular impulses are – fR1 t and fR2 t,
We therefore, have

– f R1 t = I1 (ω`1 −ω1 ) and f R 2 t = I2 (ω`2 −ω2 )

I1 I2
or − (ω`1 −ω1 ) = (ω`2 −ω2 )  …(ii)
R1 R2

I ω R + I2 ω2R1 I ω R + I2 ω2R1
Solving (i) and (ii) ω '1 =1 1 2 R 2 and ω '2 =1 1 2 R1 .
2 2
I2R1 + I1R 2 I2R12 + I1R 22

10.5 Energy Method in Solving Problems of Rolling Body


We can conserve energy in case of pure rolling of a rigid body because the point of
contact between the surfaces remains at rest and so the frictional forces acting at
the point of contact do not do any work. Thus only conservative force do work on
the body. S
h
Thus Potential energy + total K.E. = constant
As shown in the Fig. 7.60, a disc is rolling down on an inclined plane. Then we can 
conserve total mechanical energy. If the disc falls a height h then loss in potential
energy is equal to gain in kinetic energy. Figure 7.60

1 1
Mgh = Iω2 + MV 2  … (i)
2 2 C
1  K2 
1 1 2  1+  
Its total kinetic energy = MV 2 + Iω2 = MVC …(ii)
2 C 2 2  R2 
 

where K is the radius of gyration of the disc and VC the velocity of center of mass.

1  K2  2gh
So MVC2  1 + 2 Mgh;
 = VC2 =
2  R   K2 
 1 + 2 
 R 

Thus the velocity of center of mass of a body rolling down an inclined plane is given by
P hysi cs | 7.33

2gh
VC = 1/2
 K2 
1 + 2 
 R 

If aC is linear acceleration of center of mass down this plane, and distance covered on the plane is s, then if the body
starts from rest we have

2 VC2 2 gh g sin θ
VC = 2aCs ∴ aC = = or aC =
2s  K 
2
h K2
 1 + 2  × 2 × 1+ 2
 R  sin θ R

PLANCESS CONCEPTS

Rather than going in a conventional way, using this method greatly simplifies our effort. But take care
while writing the kinetic energy!
Nitin Chandrol (JEE 2012 AIR 134)

Illustration 37: A solid sphere is released from rest from the top of an incline of inclination θ and length  . If the
sphere rolls without slipping. What will be its speed when it reaches the bottom?  (JEE MAIN)
Sol: The loss in the gravitational potential energy of the solid sphere is equal to the gain in the kinetic energy.
The kinetic energy of the sphere comprises the rotational kinetic energy as well as the translational kinetic energy.
Let the mass of the sphere be m and its radius be r. Suppose the linear speed of the sphere when it reaches the
bottom is v. As the sphere rolls without slipping, its angular speed ω about its axis is v/r. The kinetic energy at the
bottom will be
1 2 1 12  1 1 1 7
K= Iω + mv 2 =  mr 2  ω2+ mv 2 = mv 2 + mv 2 = mv 2
2 2 25  2 5 2 10
This should be equal to the loss of potential energy mg  sinθ. Thus

7 10
mv 2 = mg sinθ Or v= g sinθ .
10 7

11. TOPPLING
When an external force is applied to the upper edge of a body Q R
with a flat base to cause it to slide along a surface, the body may Ftopple
topple before sliding starts. Toppling is more likely to happen
when the width of the base of the body is small.
Toppling occurs due to the turning effect of torques of applied
force at the upper edge and frictional force at the base. mg N
Let the surface be quite rough and the force F is applied at f P S
h
height h above the base of the block as shown in Fig. 7.61. Width
of the base is b. The static friction at the base is f = F. The normal Rough surface
reaction is N = mg. The couple of forces F and f try to topple
b
the block about point S. To cancel the effect of this unbalanced
torque the normal reaction N shifts towards S by a distance x so
that torque of N counter balances torques of F and f. Figure 7.61: Block toppling on rough surface
7 . 3 4 | Rotational Mechanics

Fh
Fh = (mg)x or x =
mg
If F or h or both increase, distance x also increases, but it cannot go beyond the maximum value of xmax = b/2 i.e in
extreme case N passes through edge S. If F is further increased block will topple.
mgb
So, Ftopple =
2h

Here we assumed that the surface is sufficiently rough so that sliding starts only when
b
F = fmax = µmg > Ftopple or µ > (toppling before sliding)
2h
If surface is not sufficiently rough, the body slides before F is increased to Ftopple i.e. the body will slide before
toppling. This is the case when
b
F = fmax = µmg < Ftopple or µ <
2h

Illustration 38: A uniform cube of side ‘a’ and mass m rests on a rough horizontal table. A horizontal force F is
a
applied normal to one of the faces at a point directly below the center of the face, at a height above the base.
4
(i) What is the minimum value of F for which the cube begins to tip about an edge?
(ii) What is the minimum value of µs so that toppling occurs?

(iii) If f1= µmin , find minimum force for toppling.
(iv) Minimum µs so that Fmin can cause toppling.  (JEE ADVANCED)

Sol: For part (i) we consider toppling before sliding. The normal reaction will pass through the edge. In part (ii) it
is not mentioned whether the toppling occurs before sliding or sliding occurs before toppling. So the toppling will
occur for any value of µs , sliding or no sliding. Part (iii) is same as part (i). Part (iv) is the case of toppling before
sliding.
(i) In the limiting case normal reaction will pass through O. The cube will tip about O if torque of F about O
exceeds the torque of mg.
a a
Hence, F   > mg   or F > 2 mg
4 2
Therefore, minimum value of F is 2 mg.
(ii) In this case since it is not acting at COM, toppling can occur even after body started sliding even if there is no
friction by increasing the torque of F about COM. Hence µmin = 0.
(iii) Now body is sliding before toppling. O is not I.A.R., torque equation cannot be applied across it. It can be
applied about COM.
a a
F× =N×  … (i) N
4 2
N = mg  ... (ii)
From (i) and (ii) -> F = 2 mg
F

(iv) F > 2 mg ………………… (i) (From sol. (i)) a/2 a/4


fr
N = mg  … (ii)
O
F = µs N = µs mg  … (iii)
mg
From (i) and (iii) µs = 2
Figure 7.62
P hysi cs | 7.35

Illustration 39: Find minimum value of  so that truck can avoid the dead end, without toppling the block kept
on it. (JEE ADVANCED)

b
v
h m
Sufficiently

rough surface
power breaks

Figure 7.63

Sol: The block kept on truck will experience pseudo force in forward direction and friction
b
force due to the floor of the truck in backward direction. We assume the case of toppling
before sliding. In extreme case the normal reaction N = mg will pass through the edge.
h ma
h b b
ma ≤ mg ⇒ a ≤ g
2 2 h
f
N
b
Final velocity of truck is zero. So that 0 = v 2 − 2( g) mg
h
Figure 7.64
h v2
=
2b g

PROBLEM-SOLVING TACTICS

•• Most of the problems involving incline and a rigid body can be solved by using conservation of energy during
pure rolling. In case of non-conservative forces, work done by them also has to be taken into consideration in
the equation. Care has to be taken in writing down the Kinetic energy. Rotational Kinetic Energy term has to be
taken into consideration. And while writing the rotational energy, the axis about which the moment of inertia
is taken should pass through the COM.
•• The motion of a body in pure rolling can be viewed as pure rotation about the bottommost point of the body
or the point of contact with the ground. Hence an axis passing through the point of contact and tangential
to the point would be the Instantaneous axis of rotation. So problems on pure rolling can be solved easily by
using the concept of instantaneous axis of rotation.
•• Problems on toppling can be easily solved by writing the moments on the body and visualizing them as forces
acting on the body. If the net moment is tending to stabilize the body, then the body doesn’t topple. For any
condition else it may get toppled.
•• Problems which include the concept of sliding and rolling can be solved easily by using the concept of
conservation of angular momentum. But care has to be taken in selecting the proper axis so that net moment
about that axis vanishes.
7 . 3 6 | Rotational Mechanics

FORMULAE SHEET

S. Term Description Linear Motion Rotational motion &


No relation
1 Displacement Displacement (linear or angular) is the physical s
θ
change in the position of the body when a body
moves linearly or angular in position. (s = rθ)
(a) The linear displacement ∆s is difference
between final and initial position measured in
linear direction.

S.I. unit: meter m

(b) The angular displacement of the body while


rotating about a fixed axis is the displacement
∆θ it swept out with respect to its initial
position in sense of rotation. It can be positive
(anti clockwise) or negative (clockwise)

S.I. unit: radians rad,


2 Velocity Velocity of any moving object is the time
rate of change of position. The velocity is the ds dθ
vector quantity. Linear velocity is in the plane v= ω= (v = rω)
dt dt
of motion. Angular velocity can be positive or
negative & its direction is perpendicular to the
plane of rotation

Linear velocity is categorized as

- Average velocity= ∆s / ∆t

- Instantaneous velocity= ds/dt.

S.I. unit: m/s

Angular velocity is categorized as

- Average angular velocity ∆θ / ∆t

- Instantaneous angular velocity ω = dθ / dt

S.I. unit: rad/s


3 Acceleration Acceleration is the time rate change of
velocity of a body. It’s a vector quantity. Linear dv dω
acceleration can be positive or negative and a= α= (a = rα )
dt dt
related to direction of motion.

Linear acceleration is categorized as

- Average acceleration= ∆v / ∆t

- Instantaneous acceleration = dv/dt.


P hysi cs | 7.37

S. Term Description Linear Motion Rotational motion &


No relation
S.I. unit: m/s–2

Angular acceleration is categorized as

- Average angular acceleration ∆ω / ∆t

- Instantaneous angular acceleration


α = dω / dt
S.I. unit: rad/s–2
4 Mass Mass is the basic entity of any body by virtue of M I ( I = ∑mr2)
which the body gains weight.

In linear kinematics the mass of whole body is


constant. S.I. unit: kilogram kg

In angular kinematics mass of body is


distributed among various tiny rigid points
so mass is measured about inertia of rotating
body- moment of inertia I
5 Momentum Momentum of body is product of mass and its p = mv

L =I
velocity of motion. It’s a vector quantity.
  
Linear momentum= mv
L= r × p

S.I. unit: kg m/s

Angular momentum of body is a vector in


direction perpendicular to plane of rotation

given by L

S.I. unit: kg m2/s


6 Impulse Impulse is the product of force and time period

And it is categorized as ∫ F dt ∫ τdt


-Linear impulse

-Angular impulse
  
7 Force From the newton second law of motion, force is F = ma τ= r×F = I×α
time rate of change of momentum. It’s a vector 
(Newton’s If = 0 the body is in
quantity. dL
second law of equilibrium with its =
dp
motion) Linear force F= = ma surrounding dt
dt
If = 0 the body is in
S.I. unit: Newton N
equilibrium with its
 
Angular force τ = I × α surrounding

Laws of conservation of momentum

- Linear momentum is said to be conserved if


dp
= 0, than P remains constant
dt
- Angular momentum is said to be conserved if

dL
= 0 than L remains constant
dt
7 . 3 8 | Rotational Mechanics

S. Term Description Linear Motion Rotational motion &


No relation
8 Work Work is the product of displacement of body
under action of external applied force. W= ∫ F ds W= ∫ τdθ
9 Power Power is the time rate change of work done P =F P=τω

10 Kinetic energy The phenomenon associated with the moving


1 1 2
bodies K.E. tran = mv 2 K.E. rot = Iω
2 2

11 Kinematics of Kinematical equation are the interrelation of v = u + at ω = ω0 + αt


Motion displacement, velocity, acceleration and time 1
s = ut + at2
and are categorized as follows: 2 1 2
θ = ω0 t + αt
v 2 = u2 + 2as 2
-Linear kinematical equation

-Angular kinematical equation ω2 = ω20 + 2αθ

12 Parallel Axis
I xx = Icc +Md2 where Icc is the moment of
Theorem
inertia about the center of mass

13 Perpendicular I xx + I yy = I zz It is valid for plane laminas


Axis Theorem
only.

14 Work energy Work energy principle is used to determine the


1 1 1 1
principle change in the kinetic energy of moving body W= mv 2 − mu2 W= Iω2 − Iω20
2 2 2 2

Solved Examples

JEE Main/Boards
X v
R
The first five Examples discussed below show us the cm
strategy to tackle down any problem in the rigid body L
motion. Hence follow them up properly! They may be
lengthy but are very learner friendly!!
Sol: The frictional forces acting on the feet of man will
Example 1: A person of mass M is standing on a railroad provide the necessary centripetal acceleration to move
car, which is rounding an unbanked turn of radius at in a circular path. Apply the Newton’s second law of
speed v. His center of mass is at a height of L above motion at the center of mass of the man to get the
the car midway between his feet, which are separated equation of motion along the circular path. In the
by a distance of d. The man is facing the direction of vertical plane the man is in rotational and translational
motion. What is the magnitude of the normal force on equilibrium under the action of its weight acting
each foot? vertically downwards and the normal reactions at its
feet acting vertically upwards. Get one equation each
P hysi cs | 7.39

for rotational and translational equilibrium in vertical d


τcm2 = − N2 + Lf2 (clockwise)
plane. 2

X v NL
R
N1 N2 L Fcm
f1 f2
f

d
We draw the free body diagram of the man, as shown 2
in figure.
 
Static friction f1 and a normal reaction N1 is acting on Both these torques about the center of mass must add

the inner foot. Static friction f 2 and normal reaction up to zero.

N2 is acting on the outer foot. We do not assume the Therefore
limiting value of frictional forces. The weight of the man
d d
acts at its center of mass. ( N1 + Lf1 )+ ( − N2 + Lf2 )=0
2 2
As the man is moving in a circular path with speed, by
d
Newton’s Second Law the forces of friction should act (N − N2 ) + L(f1 + f2 )= 0  ….(iii)
2 1
towards the center of the circular path.
Putting (i) in (iii) we get,
cm
d v2
(N1 − N2 ) + Lm = 0
N1 2 R
Fcm 2Lmv 2
or
L
N2 − N1 =  ….(iv)
Rd
F1 Solving (ii) and (iv) we get
d
2 1 2Lmv 2 
N1=  mg −   ….(v)
 2 R d 
v2
f1 + f2 =m ….(i)
R  1  2Lmv 2 

= N2  + mg   …. (vi)
For vertical equilibrium we should have 2  R d 

N1 + N2 − mg =
0
or mg 
N1 + N2 = ….(ii) Example 2: A Yo-Yo of mass m has an axle of radius b
and a spool of radius R. Moment of inertia about the
For rotational equilibrium of the man about its center center can be taken to be Icm = (1/2) MR² and the
 total thickness of the string can be neglected. The Yo-Yo is
of mass we have τcm 0
=
released from rest. You will need to assume that the
The gravitational force does not contribute to the center of mass of the Yo-Yo descends vertically, and
torque about center of mass because it is acting at the that the string is vertical as it unwinds.
center of mass itself. We draw a torque diagram in the
figure showing the line of action of the forces at the
inner foot.
The torque on the inner foot about COM is given by
d
τcm1 = N + Lf1 (clockwise) b
2 1
We draw a similar torque diagram for the forces at the
outer foot. R

The torque on the outer foot about COM is given by


7 . 4 0 | Rotational Mechanics

(a) What is the tension in the cord as the Yo-Yo or


descends? Mg Mg Mg
T= = = ….(v)
(b) What is the magnitude of the angular acceleration  Mb2   Mb2   2b2 
as the Yo-Yo descends and the magnitude of the linear  1+   1+   1+ 2 
 Icm   (1 / 2)MR 
2  R 
acceleration?    

(c) Find the angular velocity of the Yo-Yo when it (b) Substitute Eq. (v) into Eq. (i) to determine the angular
reaches the bottom of the string when a length L of the acceleration
string has unwound. bT 2bg
α= = 2  ….(vi)
Icm (R +2b2 )
Sol: Apply the Newton’s second law of motion at the
center of mass of Yo-Yo to get the equation of motion From (iii) and (vi) we get
along the vertical direction. Get the relation between
net torque, of all the external forces acting on Yo- 2b2g g
acm = bα = =  ….(vii)
2 2
Yo, and its moment of inertia, both these quantities (R + 2b ) 1 + (R / 2b2 )
2

calculated about the axis passing through the center of For a typical Yo-Yo, the acceleration is much less than
mass of Yo-Yo. As the Yo-Yo descends, the loss in the that of an object in free fall.
gravitational potential energy is equal to the gain in the
(c) Use conservation of energy to determine the angular
translational and rotational kinetic energy.
velocity of the Yo-Yo when it reaches the bottom of
the string (Tension force does not perform any work
>

k
because point of contact between string and Yo-Yo is
>

X i always at rest).
Loss in gravitational potential energy = Gain in kinetic
>

j
T energy MgL
b
1 1 1 1
= MgL
= Mv 2cm + = Icmω2 2
M( v cm + R 2 ω2 ) ….(viii)
R 2 2 2 2
Linear velocity of COM and angular velocity are related
mg by the constraint condition,
(a) The torque of tension in the cord about the center of v cm - bω= 0 ⇒ v cm= bω ….(ix)
mass of the Yo-Yo is in the clockwise direction. So as the
Yo-Yo descends with linear acceleration acm, it rotates in Solving (viii) and (ix) for ω, we get ω =
4gL
the clockwise direction with angular acceleration α. (2b2 + R 2 )
τcm = b T = Icmα (clockwise)  ….(i)
Example 3: A uniform cylinder of radius R and mass
Applying Newton’s Second Law for the motion of COM
M with moment of inertia about the center of mass
in the vertical direction,
Icm = (1/2) MR² starts rolling due to the mass of the
Mg - T = Macm  ….(ii) cylinder, and has dropped a vertical distance h when
As the string is stationary, and the Yo-Yo does not slip it reaches the bottom of the incline. Let g denote the
on the string, the angular acceleration and the linear gravitational constant. The coefficient of static friction
acceleration of COM are related by the constraint between the cylinder and the surface is µ . The cylinder
condition, rolls without slipping down the incline. The goal of this
problem is to find the magnitude of the velocity of
acm - bα = 0 ⇒ acm= bα  ….(iii) the center of mass of the cylinder when it reaches the
From (ii) and (iii) we get, bottom of the incline.

Mg – T = M bα  ….(iv)
Eliminating α from (i) and (iv) we get
Mb2 T
Mg – T =
Icm
P hysi cs | 7.41


R
cm

>
cm Fs X k
h
fs
mg
N

 

Sol: This problem can be solved either by applying law


Then the only force exerting a torque about the center of
of conservation of mechanical energy, or by applying
mass is the friction force, and so we have fsR = Icmα z
Newton’s laws of motion.
…(iii)
We shall solve this problem in three different ways.
Use Icm = (1/2) M R² and the kinematic constraint
1. Applying the torque equation about the center of for the no-slipping condition α z = ax /R in Eq. (xxxiv)
mass and the force equation for the center of mass to solve for the magnitude of the static friction force
motion. yielding
2. Applying the energy equation. fx =(1/2)Max …(iv)
3. Using torque about a fixed point that lies along the Substituting Eq. (iv) into Eq. (v)
line of contact between the cylinder and the surface.
Mg sin θ = (1/2) M ax =M ax  …(v)
Applying the torque equation about the center of mass
and the force equation for the center of mass motion. Which we can solve for the acceleration

We will find the acceleration and hence the speed at 2


ax = sin β  …(vi)
the bottom of the incline using kinematics. A figure 3
showing the force is shown below.
The displacement of the cylinder is x = h/ sin β in
f
the time it takes the bottom, t . The x – component
e f
of the velocity v x at the bottom is v x.f = ax.f . The
displacement in the time interval t satisfies x = (1/2)
mg f f
 ax t2f . After eliminating tf , we have x f = v x .f 2 /2 ax ,
fs
so the magnitude of the velocity when the cylinder
N j i reaches the bottom of the inclined plane is

v x .f = 1ax xf
 
=          2((2 / 3)g sin=
β)(h / sinβ) (4 / 3)gh
Choose x = 0 as the point where the cylinder just …(viii)
starts to roll. With the unit vectors shown in the figure
above, Newton’s second Law, applied in the x – and y –
Note that if we substitute Eq. (vi) into Eq. (iv) the
directions in turn, yields
magnitude of the friction force is
Mg sin β − fs = Max  …(i)
fs = (1/3) Mg sinb  …(ix)
−N + Mg cos β =0  …(ii)
In order for the cylinder to roll without slipping.
Choose the center of the cylinder to compute the
torque about (see figure below). fs ≤ µsMg cos β  …(x)

So combining Eq. (ix) and Eq. (x) we have the condition


that

(1/3) Mg sin β ≤ µs Mg cosβ  …(xi)


7 . 4 2 | Rotational Mechanics

Thus in order to roll without slipping the coefficient of 


static friction must satisfy

>
1 P1 X k
µs ≥ tanβ  …(xii) mg
3
P
Applying the energy equation mg
We shall use the fact that the energy of the cylinder- f i
N
earth system is constant since the static frictional j
d2
force does no work. Choose a zero reference point
for potential energy at the center of mass when the 
cylinder reaches the bottom of the incline plane. 
The gravitational force M g = Mg sin β ˆj acts at the
Then the initial potential energy is Ut = Mgh …(xiii) center of mass. The vector from the point P to the
Mg – N = 0 center of mass is given by rp.mg = dp ˆi − Rjˆ , so the
torque due to the gravitational force about the point
For the given moment of inertia, the final kinetic energy
P is given by
is
  
1 1 τp.mg= rp.mg × Mg= (dP ˆi − Rj)
ˆ × (Mg sin β ˆi + Mg cos βˆj)
Kf = Mv x.f 2 + Icmωz.f 2
2 2
1 1            = (dpMgcos β + RMgsin β)kˆ  …(xvii)
= Mv x.f 2 + (1 / 2)MR 2 (v x.f /R)2  … (xiv)
2 2
The normal force acts at the point of contact between
3
= Mv x.f 2 the cylinder and the surface and is given by
4 
N = −Njˆ . The vector from the point P to the point
 of
Setting the final kinetic energy equal to the initial
contact between the cylinder and the surface is τP.N =
gravitational potential energy leads to
dpˆi . So the torque due to the normal force about the
point P is given by
  
vxy=0 τp.N = τp.N × N =(dpˆi) × ( −Nj)
ˆ =−d kˆ 
p …(xviii)
h
Substituting Eq. (xxxiii) for the normal force into Eq.
v=0 (xviii) yields
x
vxy 
 τp.N = −dPMg cosβ kˆ  …(xix)

3 Therefore the sum of the torques about the point P is


Mgh= Mv x.f 2  …(xv)
4   
τp = τp.Mg + τp.N = ( Mg cos β + RMg sinβ) k̂ – dP
The magnitude of the velocity of the center of mass of Mg cosβ k̂ = Rmg sinβ k̂  …(xx)
the cylinder when it reaches the bottom of the incline
is v x.f = (4/3)gh  …(xvi) The angular momentum about the point P is given by
   
LP = Lcm + rp.cm × MVcm
In agreement with Eq. (viii)
= Icmωzkˆ +(dpˆi − Rj)
ˆ × (Mv )iˆ
x
Using torque about a fixed point that lies along the line
of contact between the cylinder and the surface = (Icmωz + RMv x )kˆ  …(xxi)
Choose a fixed point that lies along the line of contact The time derivative of the angular momentum about
between the cylinder and the surface. Then the torque the point P is then
diagram, is shown below. 
dLP
= Icmα z +RMaxkˆ  …(xxii)
dt
Therefore the torque equation

 dLP
τ=  …(xxiii)
dt
P hysi cs | 7.43

Becomes RMg sin βk̂ = (Icmα z +RMax )kˆ  …(xxiv) in direction and have same line of action and will also
pass through the point of contact S. (Point of contact is
Using the fact that Icm = (1 / 2 ) MR² and α x = vertically below the COM of the ball).
ax / R, we can conclude that RM ax = ( 3 / 2 ) MR ax  Thus we choose the initial point of contact S as the
…(xxv) origin and the net torque of all the forces about the
origin S comes out to be zero at all times. So we can
We can now solve Eq. (xxv) for the x – component of conserve the angular momentum of the ball about the
the acceleration initial point of contact (origin S).

ax  = ( 2 / 3) g sin β  …(xxvi) The initial angular momentum about the origin S is


only due to the translation of the center of mass.
In agreement with Eq. (vi). Li= mv0R
The final angular momentum about the origin S has
Example 4: A bowling ball of mass m and radius R is
both translational and rotational contribution.
initially thrown down an alley with an initial speed v 0
and it slides without rolling but due to friction it begins Lf= m vcm,f R + Icmω
to roll. The moment of inertia of the ball about its Finally the ball rolls without slipping, so we have vcm,f=
center of mass is Icm = (2/5) mR². Using conservation Rω
of angular momentum about a point (you need to find
that point), find the speed v f of the bowling ball when Now Icm = (2/5)mR²
it just start to roll without slipping? Therefore the final angular momentum about the origin
S is

VC WC VF


rs,cm,f mvcm,f

S W2,f
Sol: The angular momentum of any rigid body about
a fixed point in ground reference frame is the sum of L f =(mR + (2/5)mR)v cm,f = (7/5)mRv cm,f
the angular momentum in the C-frame and the angular Now equating Li = Lf we get
momentum corresponding to the translation of the
center of mass relative to the fixed point in ground mR v0 = (7/5) m R v cm,f
frame. or v cm,f = (5/7) v 0

 Example 5: A long narrow uniform stick of length 


and mass m lies motionless on ice (assume the ice
>

X k
cm provides a frictionless surface). The center of mass of
the stick is the same as the geometric center (at the
mg i midpoint of the stick). The moment of inertia of the
stick about its center of mass is Icm . A puck (with
fk N j putty on one side) has same mass m as the stick. The
puck slides without spinning on the ice with a speed
of v 0 towards the stick, hits one end of the stick, and
At t = 0, when the ball is released v cm,i = v 0 towards attaches to it. You may assume that the radius of the
right and ωi =0 , so the ball slips towards right on the puck is much less than the length of the stick so that
surface and hence the frictional force on the ball, will moment of inertia of the puck about its center of mass
be towards left. is negligible compared to Icm .
The frictional force will pass through the point of
contact S with the surface.
The weight of the ball as well as the normal reaction
from the surface are equal in magnitude and opposite
7 . 4 4 | Rotational Mechanics

m (c) As the collision is perfectly inelastic the mechanical


energy of the system is not conserved.
(d) Choose the center of mass of the stick-puck
combination, as found in part (a) as the point about
which we find the angular momentum before and after
OVC the collision. This choice is advantageous as there will
m be no angular momentum due to the translation of
before after the center of mass just after the collision about the
(a) How far from the midpoint of the stick is the center of center of mass itself. Before the collision, the angular
mass of the stick–puck combination after the collision? momentum was entirely due to the motion of the puck,

(b) What is the linear velocity of the stick plus puck after L0 = ( / 4)(mv 0 )
the collision? After the collision, the angular momentum is L= Icmωf
f
(c) Is mechanical energy conserved during the collision? where Icm is the moment of inertia about the center of
Explain your reasoning. mass of the stick-puck combination.
(d) What is the angular velocity of the stick plus puck This moment of inertia of the stick about the new center
after the collision? of mass is found from the parallel axis theorem, and the
(e) How far does the stick’s center of mass move during moment of inertia of the puck is m ( / 4)2 , and so
one rotation of the stick?
Icm = m(2 / 12) + m( / 4)2 + m( / 4)2 = 5m2 / 24

Sol: Apply the law of conservation of linear momentum 2


and law of conservation of angular momentum before Equating L0 = Lf we get ( / 4)(mv
= 0 ) 5m ωf / 24
and after collision. The angular momentum is to be
6v 0
calculated about the center of mass of “stick-puck This gives ωf =
system”. 5
(e) Time taken by stick-puck system for one rotation is
(a) From the midpoint of the stick the center of mass
T = 2π/ωf
of the stick–puck combination after the collision is at a
distance dcm. Distance travelled by COM during this time is

mstick dstick + mpuck dpuck m × 0 + m( / 2)  2π 2π


(b) dcm
= = = xcm = v cmT = v cm = (v / 2)
mstick + mpuck m+m 4 ωf 6v 0 / 5 0
5π
This gives xcm =
There are no external forces acting on this system 6
comprising “stick and puck” so the momentum of the
system before and after the collision is conserved. Example 6: Two small kids of masses 10 kg and 15 kg
are trying to balance a seesaw of total length 5.0 m with
>

m j the fulcrum at the center. If one of the kids is sitting at


>

i
ends, where should the other sit?
>

cm Wf Sol: For rotational equilibrium, the net torque about


the fulcrum of all the forces acting on the boys and the
Vcm
OV0
m seesaw should be zero.
before after
It is clear that the 10 kg kid should sit at the end and
After the collision, suppose the center of mass of the the 15 kg kid should sit closer to the center. Suppose
system is moving with speed v f this distance from the center is X. As the kids are in
equilibrium, the normal force between a kid and the
Equating initial and final linear momentum we get, see-saw equals the weight of that kid. Considering the
v0 rotational equilibrium of the seesaw, the torques of the
mv= (2m)v f ⇒ =
vf
0
2 forces acting on it should add to zero. The forces are.
The direction of the velocity is the same as the initial
direction of the puck’s velocity.
P hysi cs | 7.45

2 2
T= N = (392N) × = 450 N.
3 3

Example 8: A solid cylinder of mass m and radius r


starts rolling down an inclined plane of inclination θ .
Friction is enough to prevent slipping. Find the speed
of its center of mass when, its center of mass has fallen
a height h.
(a) (15kg) g downward by the 15kg kid,
(b) (10 kg) g downward by the 10kg kid,
(c) Weight of the seesaw and
(d) The normal force by the fulcrum. 
Taking torques about by the fulcrum
Sol: Loss in the gravitational potential energy of
(15kg) g x = (10 kg) g (2.5m) or x = 1.7 m. the cylinder is equal to the gain in rotational plus
translational kinetic energy.
Example 7: The ladder shown in figure has negligible Consider the two shown positions of the cylinder. As it
mass and rests on a frictionless floor. The crossbar does not slip, total mechanical energy will be conserved.
connects the two legs of the ladder at the middle. The
angle between the two legs is 60°.
The fat person sitting on the ladder has a mass of 80 kg.
Find the contact force exerted by the floor on each leg
and the tension in the crossbar. h

Sol: The forces of normal reaction at the feet of the 


ladders balance the weight of the person. For rotational
equilibrium of ladder the toque due to normal reaction
at the foot is balanced by the torque due to tension in Energy at position 1 is E1 = mgh
the crossbar. Both the torques are calculated about the 1 1
Energy at position 2 is E2 = mv 2 + I ω2
upper end of the ladder. 2 cm 2 cm
The forces acting on the different parts are shown in v mr 2 3
 cm = ω and Ic.m = ⇒ E2 =mv 2c.m
figure. Consider the vertical equilibrium of “the ladder r 2 4
plus the person” system. The forces acting on this 4
system are its weight (80kg)g and the contact force N + From COE, E1 = E2 ; v c.m = gh
3
N = 2N due to the floor. Thus
2N = (80kg) g or N = (40 kg) (9.8m/s²) = 392 N Example 9: A uniform disc of radius R and mass M is
rotated to an angular speed ω0 in its own plane about
its center and then placed on a rough horizontal surface
such that plane of the disc is parallel to the horizontal
w surface. If co-efficient of friction between the disc and
1m the surface is µ then how long will it take for the disc
0 to come to stop.
60
N N
1m T T
300
dx

Next consider the equilibrium of the left leg of the C x


ladder. Taking torque of the forces acting on it about
the upper end.
N (2 m) tan 30°= T (1 m) or R
7 . 4 6 | Rotational Mechanics

Sol: The disc can be thought of made-up of elementary The slipping will stop when the condition of pure rolling
rings of infinitesimal thickness. The torque about the is satisfied.
center of disk due to friction force on each ring will be v
Velocity about the center = 0 . Thus v 0 > ω0r . The
different from the other rings in the disc as the radii 2r
of rings are different, varying from 0 to R. So use the sphere slips forward and thus the friction by the plane
method of integration to find the torque on the entire on the sphere will act backward. As the friction is kinetic
disc. its value of N is given by µN = µMg and sphere will be
Consider a differential circular strip of the disc of radius decelerated by acm = f/M. Hence.
x and thickness dx. Mass of this strip is dm = 2ρ πx dx , This friction will also have a torque T = fr about the
M center. This torque is clockwise and in the direction of
where ρ = . Frictional force on this strip is along
πR 2 ω0 . Hence the angular acceleration about the center
the tangent and is equal to dF = 2µρπgx dx will be
r 5f
Torque on the strip due to frictional force is equal to α=f =
(2 / 5)Mr 2 2Mr
dt = mρg2px2dx
The disc is supposed to be the combination of Number and the clockwise angular velocity at time t will be
of such strips hence torque on the disc is given by v 0 5f
5f
ω(t) = ω0 + t= + t
R 2Mr 2r 2Mr
2 R3
τ= ∫ dτ = µρ g2πx ∫ x dx = µρg2π 3
Pure rolling starts when
0
⇒ τ = µMg(2/3)R Eliminating t from (i) and (ii)
5 5 v
2µMgR 4 µg v(t) + v(t) = v 0 + 0 Or
⇒ α= = 2 2 2
 MR 2  3 R
3 Thus the sphere rolls with translational velocity 6 v 0 /7
 2  in the forward direction.
 
The α is opposite to the ù

 ω(t) − ω0 + αt ⇒ 0 = ω0
4µg
t
JEE Advanced/Boards
3R
3ω0R Example 1: A carpet of mass M made of inextensible
⇒t= . material is rolled along its length in the form of a
4µg
cylinder of radius R and is kept on a rough floor. The
carpet starts unrolling without sliding on the floor
Example 10: A sphere of mass M and radius r shown
when a negligibly small push is given to it. Calculate
in figure slips on a rough horizontal plane. At some
the horizontal velocity of the axis of the cylindrical part
instant it has translational velocity v 0 and rotational
of the carpet when its radius decreases to (R/2).
v
velocity about the center is 0 . Find the translational
2r 50N
velocity after the sphere starts pure rolling. P

30o
=V0/2r
r V0
Sol: As the carpet unrolls, the radius and mass of
A cylindrical part decreases and center of mass descends.
F Thus loss in the gravitational potential energy is equal
to gain in rotational plus translational kinetic energy.
Sol: Due to forward slipping the friction will act
If ρ is the density of material of the carpet, initial mass
backwards. So the sphere will decelerate. The torque
of the carpet (cylinder) M will be πR 2Lρ . When its radius
due to friction will be in the direction of initial angular
becomes half, the mass of cylindrical part will be
velocity. So the angular velocity will increase.
π(R / 2)2 Lρ = M/4
MF =
P hysi cs | 7.47

So initial PE of the carpet is MgR while final energy. At the edge the COM moves in circular arc during
the time interval when the vertical radius through the
(M/4) g(R/2) = MgR/8
point of contact turns by angle α to become normal to
So loss in potential energy when due to unrolling radius the inclined plane. During this interval normal reaction
changes from R to R/2 from edge should always be greater than zero.
MgR (1–(1/8)) = (7/8)MgR  … (i)
1 1
Initial energy E1= mv 20 + Ic.mω2+mgR
2 2
v0
For rolling =ω
R


R R/2
(A) (B) 

This loss in potential energy is equal to increase in 1 1 1 v


⇒ E1= mv 20 + . mR 2 0 + mgR
rotation KE which is 2 2 2 R2
1 1 3
K = K T +KR = Mv 2 + Iω2 = mv 02 +mgR
2 2 4
If v is the velocity when half the carpet has unrolled, 1 1
E2= mv 2 + Ic.mω`2 + mgR cosα
then as 2 2
3
R M 1 M  R 
2 = mv 2 + mgR cosα
v = ω, M → and I =     4
2 4 2  4  2
From COE
2
1  M  2 1  MR 2   2v 
K =  v +    3 3
24 2  32   R  mv 2 + mgR cosα = mv 20 + mgR
4 4
1 1 3 4
i. e., K = Mv 2+ Mv 2= Mv 2  … (i) mv 2 = mv 20 +
3
mgR(1 − cos α )  …. (i)
8 16 16
F.B.D. of the cylinder when it is at the edge.
So from equation (i) and (ii) Center of mass of the cylinder describes circular motion
 3  2 7 about P.
  Mv =   MvR
 16  8 Hence mg cos α - N = mv 2 /R
i.e., v = (14gR) / 3 ⇒ N = mgcos α − mv 2 /R
mv 20 4 4
= mg cos α - − mg + mgcos α
Example 2: A uniform solid cylinder of radius R = 15 R 3 3
cm rolls over a horizontal plane passing into an inclined For no jumping, N ≥ 0
plane forming an angle α = 30° with the horizontal. 7 4 mv 0 2

Find the maximum value of the velocity v 0 which still ⇒ mgcos α − mg − ≥0


3 3 R
permits the cylinder to roll onto the inclined plane
section without a jump. The sliding is assumed to be 7gR 4
absent.
⇒ v0 ≤ cos α − g
3 3

V0 Example 3: Two thin circular disc of the mass 2 kg each


R and radius 10 cm each are joined by a rigid massless
 rod of length 20 cm. The axis of the rod is perpendicular
to the plane of the disc through their centers as
Sol: As the cylinder rolls into the inclined plane section, shown in the figure. The object is kept at the center
its center of mass descends, thus loss in gravitational as shown in the figure. The object is kept on a truck in
potential energy will be equal to increase in kinetic such a way that the axis of the object is horizontal and
7 . 4 8 | Rotational Mechanics

perpendicular to the direction of motion of the truck. Here a’ = acceleration of the center of mass of the
Its friction with the floor of the truck is large enough object.
to prevent slipping. If the truck has an acceleration of
For rotational motion
9 m/s² calculate.
fR = Ia
mR 2 a'
= 2. . for no slipping α = a/R
2 R
f
⇒ a' =  ….. (ii)
m
From (i) and (ii) we get
(a) The force of friction on each disc.
2
(b) The magnitude and direction of the frictional torque F= ma î ⇒ Force of friction on each disc is
3
acting on each disc about the center of mass ‘O’ of the
f ma ˆ
object. Take x-axis along the direction of the motion of = i = 6iˆ N
the truck, and z-axis along vertically upwards direction. 2 3
1 2
Express the torque in the vector form in terms of unit 2
vectors î , ˆj and k̂ in the x, y and z directions. z

(c) Find the minimum value of the co- efficient of r1 O


friction µ between the object and the floor of the truck r2
y
which makes rolling of the object possible.
 ma ˆ
f1 = i
Sol: This problem is best solved in the reference frame 3
of truck. Each disc will experience pseudo force as well 
as frictional force. Get two equations, one by applying r1 =  ˆj − R kˆ
Newton’s second law at the center of mass of the object,
  
and the other relating the torques of forces about the τ f = r1 × f 1 = ˆ × ma ˆi
−(ˆj + Rk)
center of mass and the moment of inertia about axis 1 3
passing through the center of mass. maR ˆ ma ˆ
= − j+ k
F.B.D. of the object with respect to truck. 3 3
= 6 × 0.1 î + 6 × 0.1 k̂
N
= –0.6 ˆj + 0.6 k̂

z τ(f ) = –0.6 ˆj – 0.6 k̂
2ma 2
(c) Maximum value of frictional force is 2 µ mg
f x 2 a
mg Side view ⇒ ma ≤ 2µmg ⇒ µ> .
3 3g
In the reference frame of truck it experiences a pseudo
Example 4: A uniform disc of mass m and radius r
force F = -2ma î is projected horizontal with velocity v 0 on a rough
where a = acceleration of the truck. horizontal floor so that it starts of with a purely sliding
motion at t = 0. At t = t0 second it acquires a purely
rolling motion.
V0 V (a) Calculate the velocity of the center of mass of the
disc at t = t0 .

Pseudo force does not provide torque about the center


of the disc. Because of this force object has tendency
to slide along – Ve x –axis, hence frictional force will act
along + Ve x – axis. For translational motion. P
2ma – f = ma’  ……… (i) (b) Assuming coefficient of friction to be µ calculate t0 .
P hysi cs | 7.49

(c) The work done by the frictional force as a function 2(v 0 − v)


of time. ⇒ω=  … (iii)
r
(d) Total work done by the friction over a time t much (Using (i)) Vc.m = ωr
longer then t0 .
⇒ v = 2 ( v 0 – v) (using (iii))
Sol: This problem can be solved either by applying
Newton’s laws of motion or by law of conservation of 2
⇒ v= v
angular momentum about the point of contact of the 3 0
disc with the floor. The force of friction will act opposite
to the direction of motion, and the work done by friction Alternative method: Since frictional force passes
will be equal to loss in kinetic energy. The friction will through the point of contact, hence about this point no
stop doing any work once pure rolling starts. external torque is acting.
F.B.D. of the disc. Therefore angular momentum of the disc about point
of contact does not change.
N
Initial angular momentum about p is given by

L=1 0 + mv 0 r (Using LP = L c.m + r × Pc.m )
When it starts pure rolling its angular momentum
about P is given by
L2 = Lc.m + ω + mvr
f
For rolling v = ωr
mg
mr 2 v 3
⇒ L2 = + mvr = mvr
When the disc is projected it starts sliding and hence 2 r 2
there is a relative motion between the points of From COAM
contact. Therefore frictional force acts on the disc in
2
the direction opposite to the motion. L1 = L2 ⇒ v = v 0
3
(a) Now for translational motion
(b) Putting the value of v in equation (i)
f v
ac.m = We get t0 = 0
m 3µg
f = µN (as it slides) = µmg
(c) Work done by the frictional force is equal to change
⇒ ac.m =
−µg , negative sign indicates that in K.E.
ac.m is opposite to v c.m ⇒ Wfriction
2
v 0 − µgt0
⇒ v c.m(t) = 1 1  mr 2   2µgt  1
= m(v 0 − µgt)2 +   2
 − mv 0
2 2  2   r  2
(v 0 − v)
⇒ t0 =  …(i) 3 
µg = m  µ2g2 t2 − v 0 µgt  , For t ≤ t0
 2 
where v c.m(t ) = v (d) For time t ≥ t0 work done by the friction is zero.
0
For rotational motion about center For longer time total work done is same as that in part

mr 2  2
v 0 
τf +τmg = Ic.mα ⇒ µmgr = α  3 2 2 2  v0 
2 (c) ⇒ W = m µ g t   − v 0 µgt .
2  3µg  3µg 
2µg  
⇒ α= …(ii) mv 20
r = −
2µg 6
Therefore ω(t ) = 0 + t0
0 r

Using ωt = ω0 + αt
7 . 5 0 | Rotational Mechanics

Example 5: In the shown figure a mass m slides down  ML2 


a frictionless surface from height h and collides with Where I=  + mL2 
 3 
a uniform vertical rod of length L and mass M. After  
collision the mass m sticks to the rod. The rod is free Gain in potential energy of the system
to rotate in a vertical plane about fixed axis through O.
find the maximum angular deflection of the rod from  From conservation of energy
its initial position.
1 2  M
Iω =  m +  gL(1 − cosθ)
o 2  2

m 1  ML2  m2 v 20
⇒  + mL2  ×
2  3 
 M
2
 2
 + m L
h
3 
 M
=  m +  gL(1 − cos θ)
 2
Sol: During the collision between rod and the mass,
the linear momentum is not conserved because of the 1 m v0
2 2
 M
reaction force acting on the rod due to the hinge at =  m +  gL(1 − cos θ)
2 M   2
the fixed point O. The torque of reaction force at the  + m
hinge will be zero about the hinge itself, i.e. about point 3 
O. So we can conserve the angular momentum of the 1 m v0 2 2

“rod and mass system” before and after collision. As the cos θ
cos è= .
2 M  M 
rod rotates, the gain in gravitational potential energy is 3 + m 2 + m  gL
  
equal to the loss in the kinetic energy.
Just before collision, velocity of the mass m is along the Example 6: A billiard ball, initially at rest, is given a
horizontal and is equal to v 0 = 2gh . In the process of sharp impulse by a cue. The cue is held horizontally
a distance h above the center line as shown in figure.
collision only angular momentum of the system will be
The ball leaves the cue with a speed v 0 and because
conserved about the point O.
of its forward rotation (backward slipping) eventually
If L1 and L2 are the angular momentum of the system 9
just before and just after the collision then L1 = mv 0L acquires a final speed v 0 show that.
7
 ML2 
And L2 = Iω =  + mL2  ω F
 3 
  h
From Conservation of Angular Momentum

o
4
(L/2)(1-cos) m h= R where R is the radius of the ball.
5
h
Sol: Initial linear and angular velocity of ball is found
L(1-cos)

by calculating the linear and angular impulse delivered


M  by the cue. The angular momentum of the ball about
 + m  L2 ω = mv 0L
3  the point of contact with ground surface, during its
combined translational and rotational motion remains
mv 0 conserved.
⇒ω=
M 
 + m L
hF
 3  h 9
V= V0
7
Let the rod deflect through an angle θ.
1 2
Initial energy of rod and mass system = Iω fmax
2
P hysi cs | 7.51

Let ω0 be the angular speed of the ball just after it be limiting. Write the equations of Newton’s second
leaves the cue. The maximum friction acts in forward law and torque about center of mass for the sphere
direction till the slipping continues. Let v be linear and the equation of Newton’s second law for the plank.
speed and ω the speed when slipping is ceased. The free body diagram of the sphere and the plank are
v as shown below:
∴ v = Rω or ω =
R Writing equation of motion:
9 µsMg
Given, v = v 0  ….. (i)
7 For sphere: Linear acceleration a1 = =µsg  ….(i)
M
9 v0
∴ ω=  ….. (ii) (µSMg)R
7 R 5 µS g
Angular acceleration: α = =  ….(ii)
Applying Linear impulse = change in linear momentum 2 2 R
MR 2
5
For plank: Linear acceleration
F dt = V0  …. (iii)
Applying Angular impulse = change in angular 
momentum
2 B1
or =
Fh dt mR 2 ω0  …. (iv)
5 s Mg
Angular momentum about bottommost point will
remain conserved.
i.e., Li = L f s Mg a2
or Iω0 + mRv 0 = Iω + mRv F
2
∴ mR 2 ω0 + mRv 0
5
F − µSMg
2  9 v0  9 α2 =
= mR 2   + mRv 0 …... (v) m
5 7 R  7
For no slipping α2 = a1 + Rα
4
Solving Eqs. (iii), (iv) and (v), we get h = R Solving the above four equations, we get
5
 7 
F = µS g  M + m 
Example 7: Determine the maximum horizontal force F  2 
that may be applied to the plank of mass m for which  7 
Thus, maximum value of F can be µS g  M + m 
the solid does not slip as it begins to roll on the plank.  2 
The sphere has a mass M and radius R (see figure). The
coefficient of static and kinetic friction between the Example 8: A uniform disc of radius r0 lies on a
sphere and the plank are µs and µk respectively. smooth horizontal plane. A similar disc spinning with
the angular velocity ω0 is carefully lowered onto the
M first disc. How soon do both discs spin with the same
angular velocity if the friction coefficient between them
is equal to µ ?

m F Sol: The initial angular momentum about its center of


the disc being lowered will be equal to the combined
angular momentum of both the discs about their
centers once they start rotating together. Each disc
Sol: As the plank moves forward, the sphere, due to its can be thought of made-up of elementary rings of
inertia, has a tendency to slip backwards relative to the infinitesimal thickness. The torque about the center of
plank. So the force of friction acts on the sphere in the disk due to friction force on each ring will be different
forward direction. For maximum force F, the friction will from that on the other rings in the disc as the radii
7 . 5 2 | Rotational Mechanics

of rings are different, varying from 0 to r0. So use the Let m be the mass of the sphere.
method of integration to find the torque on the entire
Since, it is a case of backward slipping, force of friction
disc.
is in forward direction. Limiting friction will act in this
From the law of conservation of angular momentum. case.
Iω0 = 2Iω
Here, I = moment of inertia of each disc relative to
common rotation axis
ω0 fmax
∴ ω= = steady state angular velocity
2
The angular velocity of each disc varies due to the
torque τ of the frictional forces. To calculate τ , let us f µmg
Linear acceleration
= a = =µg
take an elementary ring with inner and outer radii r and m m
r + dr. The torque of the friction acting on the given is τ f.r 5 µg
equal to. Angular retardation α= = =
I 2 2 2 r
mr
 mg   2µmg  5
dτ = µr   2πr dr =   r 2 dr
 πr 2   r2  Slipping ceases when v = rω
 0   0 
Or (at) = r (ω0 − αt)
where m is the mass each disc. Integrating this respect
to r between 0 and r0 , we get  5 µg 
Or µgt = r  ω0 − 
2  2 r 
τ= µmgr0
3
7 2 rω0
The angular velocity of the lower disc increases by d ω r 0; t =
µgt =ω
2 7 µg
over the time interval
2
 3r  v= at =µgt =rω0
=  0  dω 7
 4µg  v 2
And ω= = rω
Integrating this equation with respect to ω between 0 r 7 0
ω0 3r0 ω0
and , we find the desired time t = Alternative solution: Net torque on the sphere about
2 8µg
the bottommost point is zero. Therefore, angular
momentum of the sphere will remain conserved about
Example 9: A solid sphere of radius r is gently placed
the bottommost point.
on a rough horizontal ground with an initial angular
speed ω0 and no linear velocity. If the coefficient of Lt = L f
friction is µ , find the time when the slipping stops. In
∴ Iω0 = Iω + mrv
addition, state the linear velocity v and angular velocity
ω at the end of slipping. 2 2 2 2
Or mr= ω0 mr ω + mr(ωr)
5 5
2
∴ ω = ω0
7
2
And v = rω = rω0
7

Example 10: A thin massless thread is wound on reel


Sol: Due to backward slipping the force of friction of mass 3 kg and moment of inertia 0.6 kg-m². The hub
will act forwards and torque due to friction will be radius is R = 10 cm and peripheral radius is 2R = 20 cm.
anti-clockwise. This problem can be solved either by The reel is placed on a rough table and the friction is
Newton’s second law and torque about center of mass enough to prevent slipping. Find the acceleration of the
method or by applying the law of conservation of center of reel and of hanging mass of 1 kg (see figure).
angular momentum about the point of contact of the
sphere with the ground.
P hysi cs | 7.53

Sol: Apply Newton’s second law at the center of mass τ f(2R) − T.R 0.2f − 0.1T
of reel in horizontal direction. Find relation between net α = τ = f(2R) − T.R = 0.2f − 0.1T
α= I = I = 0.6
torque about center of mass of reel and moment of I I 0.6
f T
inertia about axis passing through the center of mass. = f − T…(ii)
…….(ii)
= 3 − 6 …….(ii)
Apply Newton’s second law for hanging mass in vertical 3 6
direction.
T

2R
R
a2

10N

Free body diagram of mass is


Let, a1 = acceleration of center of mass of reel Equation of motion is,
a2 = acceleration of 1 kg block 10 − T = a2 .....(iii) …(iii)

α = angular acceleration of reel (clockwise) For no slipping condition,
T = tension in the string a1 = 2Rα or a1 = 0.2α  …(iv)
and f = force of friction And a2 = a1 − Rα or a2 = a1 − 0.1 α .....(v)
Free body diagram of reel is as shown below; (only Solving the above five equations, we get
horizontal forces are shown.).
a1 = 0.27 m/s²; a2 = 0.135 m/s²
Equations of motion are: T – f = 3a1  … (i)

a1
T

JEE Main/Boards

Exercise 1 Q.4 What are two theorems of moment of inertia?

Q.5 What is moment of inertia of a solid sphere about


Q.1 What are the units and dimensions of moment of
its diameter?
inertia? Is it a vector?

Q.6 What is moment of inertia of a hollow sphere about


Q.2 What is rotational analogue of force?
an axis passing through its center.

Q.3 What is rotational analogue of mass of a body?


7 . 5 4 | Rotational Mechanics

Q.7 What are the factors on which moment of inertia of Q.20 A uniform circular disc and a uniform circular ring
a body depends? each has mass 10kg and diameter 1m. Calculate their
moment of inertia about a transverse axis through their
Q.8 Is radius of gyration of a body a constant quantity? center.

Q.9 There are two spheres of same mass and same Q.21 Calculate moment of inertia of earth about its
radius, one is solid and other is hollow. Which of them diameter, taking it to be a sphere of radius 6400 km
has a larger moment of inertia about its diameter? and mass 6 ×1024 kg.

Q.10 Two circular discs A and B of the same mass and Q.22 Calculate moment of inertia of a uniform circular
same thickness are made of two different metals whose disc of mass 700 g and diameter 20 cm about
densities are dA and dB ( dA > dB ). Their moments of (i) An axis through the center of disc and perpendicular
inertia about the axes passing through their centers of to its plane,.
gravity and perpendicular to their planes are I A and IB .
(ii) A diameter of disc
Which is greater, I A or IB ?
(iii) A tangent in the plane of the disc,
Q.11 The moments of inertia of two rotating bodies (iv) A tangent perpendicular to the plane of the disc.
A and B are I A and I B ( I A > I B ) and their angular
moments are equal. Which one has a greater kinetic
Q.23 Three particles, each of mass m, are situated at
energy?
the vertices of an equilateral triangle ABC of side L. Find
the moment of inertia of the system about the line AX
Q.12 Explain the physical significance of moment of perpendicular to AB in the plane of ABC, in the given figure.
inertia and radius of gyration.

x C
Q.13 Obtain expression of K.E. for rolling motion.

Q.14 State the laws of rotational motion.

Q.15 Establish a relation between torque and moment


of inertia of a rigid body. A B
L

Q.16 State and explain the principle of conservation of


Q.24 Calculate K.E. of rotation of a circular disc of mass
angular momentum. Give at least two examples.
1 kg and radius 0.2 m rotating about an axis passing
through its center and perpendicular to its plane. The
Q.17 Derive an expression for moment of inertia of disc is making 30/π rpm.
a thin circular ring about an axis passing through its
center and perpendicular to the plane of the ring.
Q.25 A circular disc of mass M and radius r is set into
pure rolling on a table. If w be its angular velocity,
Q.18 The moment of inertia of a circular ring about an show that it’s total K.E. is given by (3/4) Mv², where
axis passing through the center and perpendicular to v is its linear velocity. M.I. of circular disc = (1/2)
its plane is 200 g cm². If radius of ring is 5 cm, calculate mass × (radius)².
the mass of the ring.
Q.26 The sun rotates around itself once in 27 days. If
Q.19 Calculate moment of inertia of a circular disc it were to expand to twice its present diameter, what
about a transverse axis through the center of the disc. would be its new period of revolution?
Given, diameter of disc is 40 cm, thickness = 7 cm and
density of material of disc = 9 g cm−3 Q.27 A 40 kg flywheel in the form of a uniform circular
disc 1 meter in radius is making 120 r.p..m. Calculate
its angular momentum about transverse axis passing
through center of fly wheel.
P hysi cs | 7.55

Q.28 A body is seated in a revolving chair revolving at Q.5 A rigid body can be hinged about any point on the
an angular speed of 120 r.p.m. By some arrangement, x-axis. When it is hinged such that the hinge is at x, the
the body decreases the moment of inertia of the system moment of inertia is given by I = 2x² - 12x + 27
from 6 kg m² to 2 kg m². What will be the new angular
The x-coordinate of center of mass is
speed?
(A) x = 2 (B) x = 0 (C) x = 1 (D) x = 3

Q6. A weightless rod is acted upon by upward parallel


Exercise 2 forces of 2N and 4N at ends A and B respectively. The
total length of the rod AB=3m. To keep the rod in
Single Correct Choice Type equilibrium, a force of 6N should act in the following
manner:
Q.1 Thee bodies have equal masses m. Body A is solid (A) Downwards at any point between A and B.
cylinder of radius R, body B is square lamina of side
R, and body C is a solid sphere of radius R. Which (B) Downwards at mid-point of AB
body has the smallest moment of inertia about an axis (C) Downwards at a point C such that AC=1m
passing through their center of mass and perpendicular
to the plane (in case of lamina) (D) Downwards at a point D such that BD= 1m.

(A) A (B) B (C) C (D) A and C both


Q.7 A heavy rod of length L and weight W is suspended
horizontally by two vertical ropes as shown. The first
Q.2 For the same total mass which of the following rope is attached to the left end of rod while the second
will have the largest moment of inertia about an axis rope is attached a distance L/4 from right end. The
passing through its center of mass and perpendicular tension in the second rope is:
to the plane of the body.
(A) A disc of radius a 1 2
(B) A ring of radius a L/4
(C) A square lamina of side 2a
(D) Four rods forming a square of side 2a L

Q.3 A thin uniform rod of mass M and length L has its (A) (1/2) W (B) (1/4) W
moment of inertia I1 about its perpendicular bisector. (C) (1/3) W (D) (2/3) W (E) W
The rod is bend in the form of semicircular arc. Now its
moment of inertia perpendicular to its plane is I2 . The
ratio of I1 : I2 will be Q.8 A right triangular plate ABC of mass m is free to
rotate in the vertical plane about a fixed horizontal
(A) < 1 (B) >1 (C) =1 (D) Can’t be said axis though A. It is supported by a string such that the
side AB is horizontal. The reaction at the support A in
Q.4 Moment of inertia of a thin semicircular disc (mass equilibrium is:
= M & radius = R) about an axis through point O and
perpendicular to plane of disc, is given by:

O l
A B
R
l

1 1 1
(A) MR 2 (B) MR 2 (C) MR 2 (D) MR 2 C
4 2 8
mg 2mg mg
(A) (B) (C) (D) mg
3 3 2
7 . 5 6 | Rotational Mechanics

Q.9 A rod is hinged at its center and rotated by applying


(I + mR 2 )ω2 − mv 2
a constant torque from rest. The power developed by B)
the external torque as a function of time is: I

Pext Pext Iω − mvR


(C)
(A) (B) I
time time
(I + mR 2 )ω − mvR
(D)
Pext Pext I
(C) (D)
time time Q.13 A uniform rod of length l and mass M is rotating
about a fixed vertical axis on a smooth horizontal table.
It elastically strikes a particle placed at a distance l/3
Q.10 Two uniform spheres of mass M have radii R and from its axis and stops. Mass of the particle is
2R. Each sphere is rotating about a fixed axis through its
diameter. The rotational kinetic energies of the spheres
are identical. What is the ratio of the angular moments
L2 2R l
of these sphere? That is, = l/3
LR
(A) 4 (B) 2 2 (C) 2 (D) 2 (E) 1
3M 3M 4M
(A) 3M (B) (C) (D)
Q.11 A spinning ice skater can increase his rate of 4 2 3
rotation by bringing his arms and free leg closer to
his body. How does this procedure affect the skater’s Q.14 a disc of radius R is rolling purely on a flat
momentum and kinetic energy? horizontal surface, with a constant angular velocity. The
angle between the velocity and acceleration vectors of
(A) Angular momentum remains the same while kinetic
point P is
energy increases.
(B) Angular momentum remains the same while kinetic
energy decreases
C
(C) Both angular momentum and kinetic energy remains P
the same.
(D) Both angular momentum and kinetic energy
increase.
(A) Zero (B) 45°
Q.12 A child with mass m is standing at the edge of (C) 135° (D) tan−1 (1 / 2)
a disc with moment of inertia I, radius R, and initial
angular velocity w. See figure given below. The child
Q.15 A particle starts from the point (0m, 8m) and
jumps off the edge of the disc with tangential velocity
moves with uniform velocity of 3m/s. After 5 second,
v with respect to the ground. The new angular velocity
the angular velocity of the particle about the origin will
of the disc is
be:
V y
3 m/s

8m

0 x

8 3
(A) rad/s (B) rad/s
2 2 289 8
(A) Iω − mv
I 24 8
(C) rad/s (D) rad/s
289 17
P hysi cs | 7.57

Q.16 Two points of a rigid body are moving as shown.


The angular velocity of the body is:

B
V

A
R (a) (b)
9 10
(A) 1 (B) (C) (D) None
10 9
v v 2v 2v
(A) (B) (C) (D)
2R R R 3R Q.20 A body kept on a smooth horizontal surface is
pulled by a constant horizontal force applied at the
Q.17 A yo-yo is released from hand with the string top point of the body. If the body rolls purely on the
wrapped around your finger. If you hold your hand still, surface, its shape can be:
the acceleration of the yo-yo is (A) Thin pipe (B) Uniform cylinder
(A) Downward, much greater than g (C) Uniform sphere (D) Thin spherical shell
(B) Downward much greater than g
(C) Upward, much less than g Q.21 A uniform rod AB of mass m and length l is at rest
on a smooth horizontal surface. An impulse j is applied
(D) Upward, much greater than g to the end B, perpendicular to the rod in the horizontal
(E) Downward, at g l
direction. Speed of point P at A distance from the
6
Q.18 Inner and outer radii of N a spool are r and R πml
center towards a of the rod after time t = is
respectively. A thread is wound over its inner surface 12J
J J
and placed over a rough horizontal surface. Thread is (A) 2 (B)
pulled by a force F as shown in figure. Then in case of m 2m
pure rolling. J J
(C) (D) 2
m m

Q.22 The moment of inertia of a solid cylinder about its


axis is given by (1/2) MR². If this cylinder rolls without
F slipping, the ratio of its rotational kinetic energy to its
translational kinetic energy is
(A) Thread unwinds, spool rotates anticlockwise and (A) 1: 1 (B) 2: 2 (C) 1: 2 (D) 1: 3
friction acts leftwards
(B) Thread unwinds, spool rotates clockwise and friction Q.23 A force F is applied to a dumbbell for a time
acts leftwards interval t, first as in (i) and then as in (ii). In which case
(C) Thread wind, spool moves to the right and friction does the dumbbell acquire the greater center-of-mass
acts rightwards. speed?

(D) Thread winds, spool moves to the right and friction m m


does not come into existence. F
F
Q.19 A sphere is placed rotating with its center initially
at rest in a corner as shown in figure (a) & figure (b). m m
Coefficient of friction between all surfaces and the (i) (ii)
1 f
sphere is . Find the ratio of the frictional force a by (A) (i)
3 fb
ground in situations (a) & (b). (B) (ii)
7 . 5 8 | Rotational Mechanics

(C) There is no difference Q.27 Choose the correct statement(s)


(D) The answer depends on the rotation inertia of the (A) The momentum of the ring is conserved
dumbbell
(B) The angular momentum of the ring is conserved
about its center of mass
Q.24 A hoop and a solid cylinder have the same
(C) The angular momentum of the ring is conserved
mass and radius. They both roll, without slipping on a
about a point on the horizontal surface.
horizontal surface. If their kinetic energies are equal
(D) The mechanical energy of the ring is conserved.
(A) The hoop has a greater translational speed then the
cylinder
(B) The cylinder has a greater translational speed then
Previous Years’ Questions
the hoop
(C) The hoop and the cylinder have the same Q.1 Let I be moment of inertia of a uniform square plate
translational speed about an axis AB that passes through its center and is
parallel to two of its sides. CD is a line in the plane of
(D) The hoop has a greater rotational speed then the the plate that passes through the center of the plate
cylinder. and makes an angle è with AB. The moment of inertia
of the plate about the axis CD is then equal to. (1998)
Q.25 A ball rolls down an inclined plane, as shown in (A) I (B) I sin2 θ
figure. The ball is first released from rest from P and
then later from Q. Which of the following statement (C) I cos2 θ (D) I cos2 (θ / 2)
is /are correct?
Q.2 A smooth sphere is moving on a frictionless
horizontal plane with angular velocity ω and center
of mass velocity v. It collides elastically and head on
P with an identical sphere B at rest, Neglect friction
2h everywhere. After the collision their angular speeds are
ωA and ωB respectively. Then, (1999)
h
(A) ωA < ωB (B) ωA = ωB
O
(C) ωA = ω (D) ωB = ω
(i) The ball takes twice as much time to roll from Q to O
as it does to roll from P to O.
Q.3 A particle of mass m is projected with a velocity
(ii) The acceleration of the ball at Q is twice as large as v making an angle of 45 ° with the horizontal. The
the acceleration at P. magnitude of the angular momentum of the projectile
(iii) The ball has twice as much K.E.at O when rolling about the point of projection when the particle is at its
from Q as it does when rolling from P. maximum height h is  (1990)

(A) i, ii only (B) ii, iii only (A) Zero (B) mv 3 /(4 2g)

(C) i only (D) iii only (C) mv 3 /( 2g) (D) m 2gh3

Q.26 If a person is sitting on a rotating stool with his Q.4 Consider a body, shown in figure, consisting of two
hands outstretched, suddenly lowers his hands, then identical balls, each of mass M connected by a light rigid
his rod. If an impulse J = Mv is imparted to the body at one
(A) Kinetic energy will decrease of its end, what would be its angular velocity? (2003)

(B) Moment of inertia will decrease


L
(C) Angular momentum will increase M M

(D) Angular velocity will remain constant


J=Mv

(A) v/L (B) 2v/L (C) v/3L (D) v/4L


P hysi cs | 7.59

Q.5 A tube of length L is filled completely with an Q.9 A cubical block of side L rests on a rough horizontal
incompressible liquid of mass M and closed at both the surface with coefficient of friction µ . A horizontal force
ends. The tube is then rotated in a horizontal plane about F is applied sufficient high, so that the block does not
one of its ends with a uniform angular velocity ω . The slide before toppling, the minimum force required to
force exerted by the liquid at the other end is (1992) topple the block is (2000)
N
Mω2L Mω2L Mω2L2
(A) (B) Mω2L (C) (D)
2 4 2
F
Q.6 A cylinder rolls up an inclined plane, reaches some
height and then rolls down (without slipping throughout G
these motions.) The directions of the frictional force L
acting on the cylinder are  (2002) L
2
(A) Up the incline while ascending and down the incline f1
while descending.
mg
(B) Up the incline while ascending as well as descending
(A) Infinitesimal (B) mg/4
(C) Down the incline while ascending and up the inline
while descending. (C) mg/2 (D) mg (1 – µ )

(D) Down the incline while ascending as well as


descending. Q.10 An equilateral triangle ABC formed from a uniform
wire has two small identical beads initially located at A.
The triangle is set rotating about the vertical axis AO.
Q.7 Two point masses of 0.3 kg and 0.7 kg fixed at the Then the beads are released from rest simultaneously
ends of a rod of length 1.4 m and of negligible mass. and allowed to slide down, one along AB and other
The rod is set rotating about an axis perpendicular to along AC as shown. Neglecting frictional effects, the
its length with a uniform angular speed. The point on quantities that are conserved as beads slide down are
the rod through which rotation of the rod is minimum,  (2000)
is located at a distance of  (1995)
A
(A) 0.42 m from mass of 0.3kg
g
(B) 0.70 m from mass of 0.7 kg
(C) 0.98 m form mass of 0.3 kg
(D) 0.98 m form mass of 0.7 kg
B C
O
Q.8 A disc of mass M and radius R is rolling with
angular speed ω on a horizontal plane as shown. The
(A) Angular velocity and total energy (kinetic and
magnitude of angular momentum of the about the
potential)
origin O is  (1999)
y (B) Total angular momentum and total energy
(C) Angular velocity and moment of inertia about the
axis of rotation
(D) Total angular momentum and moment of inertia
about the axis of rotation.


M
Q.11 One quarter section is cut from a O uniform
x circular disc of radius R. This section has a mass M. It is
O
made to rotate about a line perpendicular to its plane
1 and passing through the center of the original disc. Its
(A)   MR 2 ω (B) MR 2 ω
2 moment of inertia about the axis of rotation is  (2001)

3
(C)   MR 2 ω (D) 2MR 2 ω
2
7 . 6 0 | Rotational Mechanics

a
v
M

O
(A) 3v / 4a (B) 3v / 2a

1 1 1 (C) 3 / 2a (D) Zero


(A) MR 2 (B) MR 2 (C) MR 2 (D) 2MR 2
2 4 8
Q.15 A thin wire of length L and uniform linear mass
Q.12 A circular platform is free to rotate in a horizontal density ρ is bent into a circular loop with center at O
plane about a vertical axis passing through its center. A as shown. The moment of inertia of the loop about the
tortoise is sitting at the edge of the platform. Now the axis XX’ is (2000)
platform is given an angular velocity ω0
x x’
When the tortoise moves along a chord of the platform o
90
with a constant velocity (with respect to the platform).
The angular velocity of the platform ω (t) will vary with
time t as  (2002) O

(t) (t)

(A) 0 (B) 0
ρL3 ρL3 5ρL3 3ρL3
(A) (B) (C) (D)
8π2 16π2 16π2 8π2
t t
Q.16 A diatomic molecule is made of two masses m1
and m2 which are separated by a distance r. If we
calculate its rotational energy by applying Bohr’ s rule
(t) (t)

of angular momentum quantization, its energy will be


(C) 0 (D) 0 given by (n is an integer) (2012)

(m1 + m2 )
2
n2 2 n2 2
(A) (B)
2 (m1 + m2 ) r 2
t t
2m12 m22 r 2
(A)

2n2 2 (m1 + m2 ) n2 2
Q.13 A thin circular ring of mass M and radius r is (C) (D)
rotating about its axis with a constant angular velocity
(m1 + m2 ) r2 2m1 m2 r 2
ω . Two objects, each of mass m2 are attached gently
to the opposite ends of a diameter of the ring. The Q.17 A hoop of radius r and mass m rotating with an
wheel now rotates with an angular velocity  (2006) angular velocity ω0 is placed on a rough horizontal
(A) wM/(M + m) surface. The initial velocity of the centre of the hoop is
zero. What will be the velocity of the centre of thehoop
(B) w(M – 2m)/(M + 2m) when it ceases to slip? (2013)
(C) wM/(M + 2m)
r ω0 r ω0 r ω0
(D) w(M + 2m)/M (A) (B) (C) r ω0 (D)
3 2 4

Q.14 A cubical block of side a moving with velocity v on Q.18 A bob of mass m attached to an inextensible
a horizontal smooth plane as shown. It hits at point O. string of length  is suspended from a vertical support.
The angular speed of the block after it hits O is  (1999) The bob rotates in a horizontal circle with an angular
speed w rad/s about the vertical. About the point of
suspension: (2014)
P hysi cs | 7.61

(A) Angular momentum changes in direction but not in Which of the following
 statements is false for the
magnitude. angular momentum L about the origin? (2016)
(B) Angular momentum changes both in direction and  R 
magnitude. (A) L mv 
= − a kˆ when the particle is moving from
 2 
(C) Angular momentum is conserved. C to D.
(D) Angular momentum changes in magnitude but not  R 
(B) L mv 
= + a kˆ when the particle is moving from
in direction.
 2 
B to C.
Q.19 The current voltage relation of diode is given  mv
R kˆ when the particle is moving from D to A.
by I e
= (
1000 V /T
)
− 1 mA , where the applied voltage V
(C) L =
2
is in volts and the temperature T is in degree Kelvin.  mv ˆ
If a student makes an error measuring ± 0.01 V while (D) L = − R k when the particle is moving from
2
measuring the current of 5 mA at 300 K, what will be A to B.
the error in the value of current in mA? (2014)
(A) 0.5 mA (B) 0.05 mA Q.22 A roller is made by joining together two cones
(C) 0.2 mA (D) 0.02 mA at their vertices O. It is kept on two rails AB and CD
which are placed asymmetrically (see figure), with its
axis perpendicular to CD and its centre O at the centre
Q.20 From a solid sphere of mass M and radius R a
of line joining AB and CD (see figure). It is given a light
cube of maximum possible volume is cut. Moment of
push so that it starts rolling with its centre O moving
inertia of cube about an axis passing through its center
parallel to CD in the direction shown. As it moves, the
and perpendicular to one of its faces is: (2015)
roller will tend to: (2016)
MR 2 4MR 2 4MR 2 MR 2
(A) (B) (C) (D) B D
16 2 π 9 3π 3 3π 32 2 π

Q.21 A particle of mass m is moving along the side of


O
a square of side ‘a’, with a uniform speed v in the x-y
plane as shown in the figure:
A C
y a
D
v
C (A) Turn right
a v v a (B) Go straight
A v B
(C) Turn left and right alternately
a
o
R (D) Turn left
45
O x
7 . 6 2 | Rotational Mechanics

JEE Advanced/Boards

Exercise 1
0.3m C
Q.1 A thin uniform rod of mass M and length L is
0.1m
hinged at its upper end, and released from rest in a String
horizontal position. Find the tension at a point located B
at a distance L/3 from the hinge point, when the rod A
becomes vertical.

Q.6 A uniform wood door has mass m, height h, and


Q.2 A rigid body in shape of a triangle has VA = 5 m/s width w. It is hanging from two hinges attached to one
downwords, VB = 10 m/s downwords. Find velocity of side; the hinges are located h/3 and 2h/3 from the
point C. bottom of the door.
C

W
a

A a B
com
Hinges h
Q.3 A rigid horizontal smooth rod AB of mass 0.75 kg
and length 40 cm can rotate freely about a fixed vertical
axis through its mid-point O. Two rings each of mass 1
kg are initially at rest at a distance of 10 cm from O on
either side of the rod. The rod is set in rotation with an
Suppose that m = 20.0 kg, h = 2.20m, and W = 1.00 m
angular velocity of 30 rad per second. Find the velocity
and the bottom smooth hinge is not screwed into the
of each ring along the length of the rod in m/s when
door frame, find the forces acting on the door.
they reach the ends of the rod.

Q.7 A thin rod AB of length a has variable mass per unit



 x
length ρ0  1 +  where x is the distance measured
A C D B  a 
O from a and ρ0 is a constant
(a) Find the mass M of the rod.
(b) Find the position of center of mass of the rod.
Q.4 A straight rod AB of mass M and length L is placed
(c) Find moment of inertia of the rod about an axis
on a frictionless horizontal surface. A horizontal force
passing through A and perpendicular to AB. Rod is
having constant magnitude F and a fixed direction starts
freely pivoted at A and is hanging in equilibrium when
acting at the end A. The rod is initially perpendicular to
it is struck by a horizontal impulse of magnitude P at
the force. Find the initial acceleration of end B.
the point B.
(d) Find the angular velocity with which the rod begins
Q.5 A wheel is made to roll without slipping, towards
to rotate.
right, by pulling a string wrapped around a coaxial
spool as shown in figure. With what velocity the string (e) Find minimum value of impulse P if B passes through
should be pulled so that the center of wheel moves a point vertically above A.
with a velocity of 3 m/s?
P hysi cs | 7.63

Q.8 Two separate cylinders of masses m (=1 kg) and 4m


r
and radii R (=10cm) and 2R are rotating in clockwise
direction with ω1 = 100rad/sec and ω2 = 200 rad/
sec. Now they are held in contact with each other as
in figure Determine their angular velocity after the
slipping between the cylinders stops.

Q.12 A thin wire of length L and uniform linear mass


density ρ is bent into a circular loop with center at O as
Q.9 A spool of inner radius R and outer radius 3R shown in the figure. The moment of inertia of the loop
has a moment of inertia = MR2 about an axis passing about the axis XX` is
through its geometric center, where M is the mass of
the spool. A thread wound on the inner surface of the X X'
spool is pulled horizontally with a constant force = O
Mg. Find the acceleration of the point on the thread
which is being pulled assuming that the spool rolls
purely on the floor.

Q.13 A block X of mass 0.5 kg is held by a long massless


string on a frictionless inclined plane of inclination 30°
to the horizontal. The string is wound on a uniform
solid cylindrical drum Y of mass 2 kg and of radius
0.2 m as shown in the figure. The drum is given an initial
angular velocity such that the block X stands moving
up the plane. (g = 9.8 m/s²)

Q.10 A sphere of mass m and radius r is pushed onto


Y
a fixed horizontal surface such that it rolls without
slipping from the beginning. Determine the minimum
X
speed v of its mass center at the bottom so that it rolls
completely around the loop of radius (R + r) without
leaving the track in between.

(i) Find the tension in the string during the motion.


(R+r) (ii) At a certain instant of time the magnitude of the
Sphere angular velocity of Y is 10rad/sec. Calculate the distance
travelled by X form that instant of time unit it comes to
rest.

Q.11 Two uniform cylinders each of mass m = 10 kg Q.14 A uniform rod AB of length L and mass M is lying
and radius r = 150 mm, are connected by a rough belt on a smooth table. A small particle of mass m strikes
as shown. If the system is released from rest, determine the rod with a velocity v 0 at point at distance from the
center O. The particle comes to rest after collision. Find
(a) The tension in the portion of the belt connecting the
the value of x, so that of the rod remains stationary just
two cylinder.
after collision.
(b) The velocity of the center of cylinder a after it has
moved through 1.2 m.
7 . 6 4 | Rotational Mechanics

B
m V0 m V0

R R
m v0
C
x (i) Calculate the velocity of the center of mass of the
O disk at t0
(ii) Assuming the coefficient of friction to be µ calculate
A t0 . Also calculate the work done by the frictional force
as a function of time and the total work done by it over
Q.15 A uniform plate of mass m is suspended in each of a time t much longer then t0 .
the ways shown. For each case determine immediately
after the connection at B has been released:
Q.18 A circular disc of mass 300 gm and radius 20 cm
can rotate freely about a vertical axis passing through
Pin support its center of mass o. A small insect of mass 100 gm
is initially at a point A on the disc (which is initially
stationary). The insect starts walking from rest along the
A B
rim of the disc with such a time varying relative velocity
that the disc rotates in the opposite direction with a
(1/2)C constant angular acceleration = 2 π rad/s². After some
time T, the insect is back at the point A. By what angle
has the disc rotated till now, as seen by a stationary
C earth observer? Also find the time T.
(I) y

Q.19 A uniform disc of mass m and radius R rotates


x about a fixed vertical axis passing through its center
with angular velocity ω . A particle of same mass m and
(a) The angular acceleration of the plate. having velocity 2 ω R towards center of the disc collides
(b) The acceleration of its mass center. with the disc moving horizontally and stick to its rim.
Find

Q.16 A carpet of mass ‘M’ made of inextensible (a) The angular velocity of the disc
material is rolled along its length in the form of cylinder (b) The impulse on the particle due to disc.
of radius ‘R’ and is kept on a rough floor. The carpet
starts unrolling without standing on the floor when (c) The impulse on the disc due to hinge.
a negligibly small push is given to it. The horizontal
velocity of the axis of the cylindrical parts of the carpet Q.20 The door of an automobile is open and
when its radius decreases to R/2 will be: perpendicular to the body. The automobile starts with
an acceleration of 2 ft/sec², and the width of the door
is 30 inches. Treat the door as a uniform rectangle, and
neglect friction to find the speed of its outside edge as
seen by the driver when the door closes.

R/2 Q.21 A 20 kg cabinet is mounted on small casters that


R allow it to move freely ( µ = 0) on the floor. If a 100 N
force is applied as shown, determine.
Q.17 A uniform disk of mass m and radius R is projected
horizontally with velocity v 0 on a rough horizontal floor
so that it starts off with a purely sliding motion at t = 0.
After t0 seconds it acquires a purely rolling motion as
shown in figure.
P hysi cs | 7.65

Exercise 2
G Single Correct Choice Type
F
0.9m Q.1 Let I1 and I2 be the moment of inertia of a uniform
h square plate about axes APC and OPO` respectively as
I
shown in the figure. P is center of square. The ratio 1
0.6m of moment of inertia is I2
(a) The acceleration of the cabinet,
O
(b) The range of values of h for which the cabinet will A B
not tip.
P
Q.22 Two thin circular disks of mass 2 kg and radius
10 cm each are joined by a rigid massless rod of length
20 cm. The axis of the rod is along the perpendicular to D C
the planes of the disk through their center. The object O’
is kept on a truck in such a way that the axis of the
1 1
object is horizontal and perpendicular to the direction (A) (B) 2 (C) (D) 1
of motion of the truck. Its friction with the floor of the 2 2
truck is large enough so that the object can roll on
the truck without slipping. Take x-axis as the vertically Q.2 Moment of inertia of a rectangular plate about an
upwards direction. If the truck has an acceleration of axis passing through P and perpendicular to the plate
9m/s² calculate. is I. Then moment of PQR about an axis perpendicular
to the plane.

P Q
O

20cm
S R
(a) The force of friction on each disk.
(A) About P = I/2 (B) About R = I/2
(b) The magnitude and the direction of the frictional
torque acting on each disk about the center of mass O (C) About P > I/2 (D) About R > I/2
of the object. Express the torque in the vector form of
unit vectors in the x-y and z direction. Q.3 Find the moment of inertia of a plate cut in shape
of a right angled triangle of mass M, AC=BC=a about
Q.23 Three particles A, B, C of mass m each are joined an axis perpendicular to plane, side the plane of the
to each other by mass less rigid rods to form an plate and passing through the mid-point of side AB.
equilateral triangle of side a. Another particle of mass
m hits B with a velocity v 0 directed along BC as shown. A
The colliding particle stops immediately after impact.
O
(i) Calculate the time required by the triangle ABC to
complete half-revolution in its subsequent motion. (ii)
What is the net displacement of point B during this
interval? C B

Ma2 Ma2 Ma2 2Ma2


(A) (B) (C) (D)
12 6 3 3

Q.4 Let I be the moment of inertia of a uniform square


plate about an axis AB that passes through its center
7 . 6 6 | Rotational Mechanics

and is parallel to two of its sides. CD is a line in the


plane of the plate that passes through the center of the P
plate and makes an angle θ with AB. The moment of L
inertia of the plate about the axis CD is then equal to
(A) I (B) I sin² θ 

(C) I cos² θ (D) I cos² ( θ /2)


(A)  (2/3) gcos  θ (B) (2 / 3)g

Q.5 A heavy seesaw (i.e., not mass less) is out of balance. 3
(C) g (D)   gcos θ
A light girl sits on the end that is tilted downward, and 2
a heavy body sits on the other side so that the seesaw
now balances. If they both move forward so that they (E) (3 / 2)g
are one-half of their original distance from the pivot
point (the fulcrum) what will happen to the seesaw?
Q.8 A mass m is moving at speed v perpendicular to a
(A) The side the body is sitting on will tilt downward rod of length d and mass M = 6m which pivots around
(B) The side the girl is sitting on will once again tilt a frictionless axle running through its center. It strikes
downward and sticks to the end of the rod. The moment of inertia
of the rod about its center is Md² /12. Then the angular
(C) Nothing; the seesaw will still be balanced speed of the system right after the collision is.
(D) It is impossible to say without knowing the masses (A) 2v / d (B) 2v / (3d)
and the distances.
(C) v / d (D) 3v / (2d)
Q.6 A pulley is hinged at the center and a mass less
thread is wrapped around it. The thread is pulled with Q.9 A sphere of mass M and radius R is attached by a
a constant force F starting from rest,. As the time light rod of length l to a point P. The sphere rolls without
increases, slipping on a circular track as shown. It is released from
the horizontal position. The angular momentum of the
F system about P when the rod becomes vertical is:

l
P

(A) Its angular velocity increases, but force on hinge


remains constant
(B) Its angular velocity remains same, but force on
hinge increases
10 10 2
(C) Its angular velocity increases and force of hinge (A) M gl[l + R] (B) M gl[l + R]
7 7 5
increases
(D) Its angular velocity remains same and force on 10 7
(C) M gl[l + R] (D) None of the above
hinge is constant 7 5

Q.7 A uniform flag pole of length L and mass M is


Q.10 A ladder of length L is slipping with its ends
pivoted on the ground with a frictionless hinge. The
against a vertical wall and a horizontal floor. At a
flag pole makes an angle θ with the horizontal. The
certain moment, the speed of the end in contact with
moment of inertia of the flag pole about one end is
the horizontal floor is v and the ladder makes an angle
(1/3) ML². If it starts falling from the position shown in
α = 30 ° with the horizontal. Then the speed of ladder’s
the accompanying figure, the linear acceleration of the
center must be
free end of the flag pole – labeled P – would be:
(A) 2v 3 (B) v/2 (C) v (D) None
P hysi cs | 7.67

Q.11 In the previous question, if dv/dt = 0, then the (C) x  = vt +  0.5b sin ( 3vt / b ) ,  y =
 0.5c =  0.5b cos ( 3vt / b )
angular acceleration of the ladder when α = 45 ° is
 0.5vt +  0.5b sin ( 3vt / b ) ,  y =
(D) x  =  0.5b cos ( 3vt / b )
(A) 2v 2 /L2 (B) v 2 /2L2
Q.14 A yo-yo is resting on a perfectly rough horizontal
(C) 2[v 2 L2 ] (D) None table. Forces F1 , F2 and F3 are applied separately as
shown. The correct statement is
Q.12 A uniform circular disc placed on a rough horizontal
surface has initially a velocity v 0 and an angular velocity F3 F2
ω0 as shown in the figure. The disc comes to rest after
moving some distance in the direction of motion.
v0
Then is
rω0
F1
0

(A) When F3 is applied the center of mass will move to


V0 the right.
(B) When F2 is applied the center of mass will move to
the right.
(A) 1/2 (B) 1 (C) 3/2 (D) 2 (C) When F1 is applied the center of mass will move to
the right.
Q.13 An ice skater of mass m moves with speed 2v to
(D) When F2 is applied the center of mass will move to
the right, while another of the same mass m moves
the right.
with speed v toward the left, as shown in figure I. Their
paths are separated by a distance b. At t = 0, when they
are both at x = 0, they grasp a pole of length Multiple Correct Choice Type

y Q.15 A rod of weight w is supported by two parallel


knife edges and B and is in equilibrium in a horizontal
m 2v position. The knives are at a distance d from each other.
The center of mass of the rod is at a distance x from A.
x wx
b (A) The normal reaction at a is
d
m w(d − x)
t<0 v (B) The normal reaction at a is
d
wx
(C) The normal reaction at B is
b and negligible mass. For r > 0 consider the system d
as a rigid body of two masses m separated by distance w(d − x)
(D) The normal reaction at B is
b, as shown in figure II. Which of the following is the d
correct formula for motion after t = 0 of the skater
initially at y = b/2? Q.16 A block with a square base measuring and height
h, is placed on an inclined plane. The coefficient of
y
friction is µ . The angle of inclination (α) of the plane is
gradually increased. The block will
b/2 a
>x (A) Topple before sliding if µ >
t=0 h
a
(A) x   2=
= vt,  y   b / 2  (B) Topple before sliding if µ <
h
vt +  0.5 b sin ( 3vt / b ) ,  y =
(B) x  =  0.5b cos ( 3vt / b )
7 . 6 8 | Rotational Mechanics

a Q.20 A hollow sphere of radius R and mass m is fully


(C) Slide before toppling if µ >
h filled with non-viscous liquid of mass m. It is rolled
a down a horizontal plane such that its center of mass
(D) Slide before toppling if µ < moves with a velocity v. If it purely rolls
h
5
(A) Kinetic energy of the sphere is mv 2
Q.17 A particle falls freely near the surface of the earth. 6
Consider a fixed point O (not vertically below the 4
particle) on the ground. (B) Kinetic energy of the sphere is mv 2
3
(A) Angular momentum of the particle about O is
(C) Angular momentum of the sphere about a fixed
increasing.
8
(B) Torque of the gravitational force on the particle point on ground is mvR
3
about O is decreasing.
(C) The moment of inertia of the particle about O is (D) Angular momentum of the sphere about a fixed
decreasing. 14
point on ground is mvR
5
(D) The angular velocity of the particle about O is
increasing.
Q.21 In the figure shown, the plank is being pulled to
the right with a constant speed v. If the cylinder does
Q.18 The torque τ on a body about a given point is not slip then:
found to be equal to a × L where a constant vector is
and L is the angular momentum of the body about that
point. From this it follows that
(A) dL/dt is perpendicular to L at all instants of time R

(B) The components of l in the direction of a does not


change with time. V
(C) The magnitude of l does not change with time.
(D) L does not change with time.
(A) The speed of the center of mass of the cylinder is 2v.
Q.19 In the given figure. a ball strikes a uniform rod of (B) The speed of the center of mass of the cylinder is
same mass elastically and rod is hinged at point A. Then zero.
which of the statement (S) is /are correct?
(C) The angular velocity of the cylinder is v/R.
A
(D) The angular velocity of the cylinder is zero.

Q.22 A disc of circumference s is at rest at a point A on


u a horizontal surface when a constant horizontal force
begins to act on its center. Between A and B there is
sufficient friction to prevent slipping and the surface is
(A) Linear momentum of system (ball + rod) is smooth to the right of B. AB = s. The disc moves from A
conserved. to B in time T. To the right of B,

(B) Angular momentum of system (ball + rod) about


the hinged point A is conserved. Force
(C) Kinetic energy of system (ball + rod) before the
collision is equal to kinetic energy of system just after A B
the collision
(A) The angular acceleration of disc will disappear,
(D) Linear momentum of ball is conserved. linear acceleration will remain unchanged
(B) Linear acceleration of the disc will increase
P hysi cs | 7.69

(C) The disc will make one rotation in time T/2. Q.27 A man spinning in free space changes the shape
of his body, eg. By spreading his arms or curling up. By
(D) The disc will cover a distance greater then s in
doing this, he can change his
further time T.
(A) Moment of inertia
Q.23 A rigid object is rotating in a counterclockwise (B) Angular momentum
sense around a fixed axis. If the rigid object rotates
(C) Angular velocity
though more than180 ° but less than 360 ° , which of
the following pairs of quantities can represent an initial (D) Rotational kinetic energy
angular position and a final angular position of the
rigid object. Q.28 A ring rolls without slipping on the ground. Its
(A) 3 rad, 6 rad (B) –1 rad, 1 rad center C moves with a constant speed u. P is any point
on the ring. The speed of P with respect to the ground
(C) 1 rad, 5 rad (D –1rad, 2.5 rad
is v.
(A) 0 ≤ v ≤ 2u
Q.24 ABCD is a square plate with center O. The moments
of inertia of the plate about the perpendicular axis (B) v = u, if CP is horizontal
through O is I and about the axes 1, 2, 3 & 4 are I1 , I2 ,
(C) v = u is CP makes an angle of 30 ° with the horizontal
I3 , & I 4 respectively. If follows that:
and P is below the horizontal level of c.
1 (D) v = 2 u, if CP is horizontal
A 2
B
Q.29 A small ball of mass m suspended from the ceiling
at a point O by a thread of length  moves along a
3
O horizontal circle with a constant angular velocity ω .

C
D 4
(A) I2 = I3 (B) I = I1 + I 4

(C) I = I2 + I 4 (D) I1 = I3

Q.25 A body is in equilibrium under the influence of m C


a number of forces. Each force has a different line of
action. The minimum number of forces required is
(A) Angular momentum is constant about O
(A) 2, if their lines of action pass through the center of
(B) Angular momentum is constant about C
mass of the body.
(C) Vertical component of angular momentum about O
(B) 3, if their lines of action are not parallel.
is constant
(C) 3, if their lines of action are parallel.
(D) Magnitude of angular momentum about O is
(D) 4, if their lines of action are parallel and all the forces constant.
have the same magnitude.
Q. 30 If a cylinder is rolling down the incline with sliding.
Q.26 A block of mass m moves on a horizontal rough
(A) After some time it may start pure rolling
surface with initial velocity v. The height of the center
of mass of the block is h from the surface. Consider a (B) After sometime it will start pure rolling
point a on the surface in line with the center of mass. (C) It may be possible that it will never start pure rolling
(A) Angular momentum about a is mvh initially (D) None of these.
(B) The velocity of the block decreases as time passes.
(C) Torque of the forces acting on block is zero about a. Q.31 Which of the following statements are correct.

(D) Angular momentum is not conserved about A. (A) Friction acting on a cylinder without sliding on an
7 . 7 0 | Rotational Mechanics

inclined surface is always upward along the incline Q. 34 A wheel of radius r is rolling on a straight line,
irrespective of any external force acting on it the velocity of its center being v. At a certain instant
the point of contact of the wheel with the grounds is M
(B) Friction acting on a cylinder without sliding on an
and N is the highest point on the wheel (diametrically
inclined surface may be upward may be downwards
opposite to M). The incorrect statement is:
depending on the external force acting on it
(A) The velocity of any point P of the wheel is
(C) Friction acting on a cylinder rolling without sliding
proportional to MP
may be zero depending on the external force acting
on it (B) Points of the wheel moving with velocity greater
than v form a larger area of the wheel than points
(D) Nothing can be said exactly about it as it depends
moving with velocity less than v
on the frictional coefficient on inclined plane.
(C) The point of contact M is instantaneously at rest
Q. 32 A plank with a uniform sphere placed on it rests (D) The velocities of any two parts of the wheel which
on a smooth horizontal plane. Plank is pulled to right are equidistant from center are equal.
by a constant force F. If sphere does not slip over the
plank. Which of the following is correct?
Q.35 A ring of mass M and radius R sliding with a
velocity v 0 suddenly enters into a rough surface where
e the coefficient of friction is µ , as shown in figure.

> V0
(A) Acceleration of the center of sphere is less than that
of the plank Rough ()

(B) Work done by friction acting on the sphere is equal Choose the correct statement(s)
to its total kinetic energy
(A) As the ring enters on the rough surface, the limiting
(C) Total kinetic energy of the system is equal to work frictional force acts on it
done by the force F
(B) The direction of friction is opposite to the direction
(D) None of the above. of motion.
(C) The frictional force accelerates the ring in the
Q. 33 a uniform disc is rolling on a horizontal surface. clockwise sense about its center of mass
At a certain instant B is the point of contact and A is at
(D) As the ring enters on the rough surface it starts
height 2R from ground, where R is radius of disc.
rolling.

Q.36 Choose the correct statement (s)


(A) The ring starts its rolling motion when the center of
mass is stationary
B
(B) The ring starts rolling motion when the point of
contact becomes stationary
(A) The magnitude of the angular momentum of the v0
disc about B is thrice that about A (C) The time after which the ring starts rolling is
2µg
(B) The angular momentum of the disc about A is v0
anticlockwise (D) The rolling velocity is
2
(C) The angular momentum of the disc about B is
clockwise Q.37 Choose the correct alternative (s)
(D) The angular momentum of the disc about A is equal
(A) The linear distance moved by the center of mass
to that about B.
3v 20
before the ring starts rolling is
8µg
P hysi cs | 7.71

3
(B) The net work done by friction force is − mv 20
8 A
mv 20
(C) The loss is kinetic energy of the ring is
4
X Y
mv 20
(D) The gain in rotational kinetic energy is +
8

Q.38 A tightrope walker in a circus holds a long flexible


pole to help stay balanced on the rope. Holding the
pole horizontally and perpendicular to the rope helps Statement-II: About any point in the plane, the torque
the performer. of gravity force and normal contact force by ground
balance each other

Q.41 Statement-I: The angular velocity of all the points


on the laminar rigid body lying in the plane of a body
as seen from any other point on it is the same.
Statement-II: The distance between any 2 points on
the rigid body remains constant.

Q.42 Consider the following statements:-


Statement-I: The moment of inertia of a rigid body
(A) By lowering the overall center-of- gravity
reduces to its minimum value as compared to any other
(B) By increasing the rotation inertia parallel axis when the axis of rotation passes through its
center of mass.
(C) In the ability to adjust the center- of -gravity to be
over the rope. Statement-II: The weight of a rigid body always acts
through its center of mass in uniform gravitational field.
(D) In achieving the center of gravity to be under the
rope.
Q.43 Statement-I: The moment of inertia of any rigid
body is minimum about axis which passes through its
Assertion Reasoning Type
center of mass as compared to any other parallel axis.
(A) Statement-I is true, statement-II is true and
Statement-II: The entire mass of a body can be
statement-II is correct explanation for statement-I.
assumed to be concentrated at its center of mass for
(B) Statement-I is true, statement-II is true and statement- applying Newton’s force Law.
II is NOT the correct explanation for statement-I.
(C) Statement-I is true, statement-II is false. Q.44 A uniform thin rod of length L is hinged about
one of its ends and is free to rotate about the hinge
(D) Statement-I is false, statement-II is true.
without friction, Neglect the effect of gravity. A force
F is applied at a distance x from the hinge on the rod
Q.39 Consider the following statements such that force is always perpendicular to rod. As the
Statement-I: a cyclist always bends inwards while value of x is increased from zero to L,
negotiating a curve Statement-I: The component of reaction force by hinge
Statement-II: By bending he lowers his center of on the rod perpendicular to length of rod increases.
gravity of these statements. Statement-II: The angular acceleration of rod increases.

Q.40 Statement-I A disc A moves on a smooth Q.45 Statement-I: For a round shape body of radius R
horizontal plane and rebounds elastically from a rolling on a fixed ground, the magnitude of velocity of
smooth vertical wall (Top view is shown in Fig 7.166), its center is given by ωR , where ω is its angular speed.
in this case about any point on line XY the angular
momentum of the disc remains conserved. Statement-II: When distribution of mass is symmetrical
then center of round shape body is its center of mass.
7 . 7 2 | Rotational Mechanics

Paragraph 2:
F
A uniform rod is fixed to a rotating turntable so that its
x lower end is on the axis of the turntable and it makes
an angle of N20°to the vertical. (The rod is thus rotating
with uniform angular velocity about a vertical axis
Q.46 Statement-I: a body cannot roll on a smooth passing through one end.) If the turntable is rotating
horizontal Surface. clockwise as seen from above.
Statement-II: when a body rolls purely, the point of
contact should be at rest with respect to surface. Q.51 What is the direction of the rod’s angular
momentum vector (calculated about its lower end)?
Comprehension Type (A) Vertically downwards
Paragraph 1: (B) Down at 20°to the horizontal
The figure shows an isosceles triangular plate of mass (C) Up at 20° to the horizontal
M and base L. The angle at the apex is 90°. The apex lies (D) Vertically upwards
at the origin and base is parallel to X – axis

 Q.52 Is there torque acting on it, and if so in what


direction?
M
(A) Yes, vertically
(B) Yes, horizontal
x
(C) Yes at 20°to the horizontal

Q.47 The moment of inertia of the plate about the z – (D) No


axis is
ML2 ML2 ML2 Paragraph 3:
(A) (B) (C) (D) None of these
12 24 6 In the following problems, indicate the correct direction
of friction force acting on the cylinder, which is pulled
Q.48 The moment of inertia of the plate about the x on a rough surface by a constant force F.
axis is
ML2 ML2 ML2 ML2 Q.53 A cylinder of mass M and radius R is pulled
(A) (B) (C) (D)
8 32 24 6 horizontal by a force F. The frictional force can be given
by which of the following diagrams
Q.49 The moment of inertia of the plate about its base
parallel to the x – axis is C F

ML2 ML2 ML2


(A) (B) (C) (D) None of these
18 36 24 FF FF
(A)
(A) C
C (B) C
(B) C
 
Q.50 The moment of inertia of the plate about the y –
axis is
20
o
(C) C FF (D)
(D) Cannot
Cannot be
be interpret
interpret
(C) C
=0
=0
Q.54 A cylinder is pulled horizontally by a force F acting
at a point below the center of mass of the cylinder, as
shown in figure. The frictional force can be given by
ML2
which of the following diagrams?
ML2 ML2
(A) (B) (C) (D) None of these
6 8 24
P hysi cs | 7.73

F Previous Years’ Questions


C

Q.1 A thin uniform angular disc (See figure) of mass


M has outer radius 4R and inner radius 3R. The work
(A) C
F (B) C F required to take a unit mass from point P on its axis to
  infinity is  (2010)
P
F (D) Cannot be interpret 4R
(C) C
=0 3R 4R

Q.55 A cylinder is pulled horizontally by a force F acting


at a point above the center of mass of the cylinder, as 2GM 2GM
(A) (4 2 − 5) (B) − (4 2 − 5)
shown in figure. The frictional force can be given by 7R 7R
which of the following diagrams 2GM
GM
(C) (D) ( 2 − 1)
4R 5R
F
C
Q.2 A solid sphere of radius R has moment of inertia
I about its geometrical axis. If it’s moment of inertia
F F about the tangential axis (which is perpendicular to
(A) C (B) C
plane of the disc), is also equal to I, then the value of r

=0 is equal to  (2006)
l
(C) F (D) Cannot be interpret
C

r

Q.56 A cylinder is placed on a rough plank which in


turn is placed on a smooth surface. The plank is pulled
with a constant force F. The frictional force can be given 2 2 3 3
(A) R (B) R (C) R (D) R
by which of the following diagrams. 15 5 15 15

Q.3 A block of base 10 cm × 10 cm and height 15 cm


C
F is kept on an inclined plane. The coefficient of friction
between them is 3 . The inclination θ of this inclined
plane from the horizontal plane is gradually increased
(A) (A)
C C (B) C(B) C (C) C(C) C from 0°. Then,  (2009)
(A) At θ = 30°, the block will start sliding down the plane
   
=0 =0
(B) The block will remains at rest on the plane up to
C (C) C (D) Cannot be interpreted.
 certain θ and then it will topple
(C) At θ = 60°, the block will start sliding down the plane
=0
and continue to do so at higher angles
(D) At θ = 60°, the block will start sliding down the plane
and on further increasing θ, it will topple at certain θ.
7 . 7 4 | Rotational Mechanics

Q.4 From a circular disc of radius R and mass 9M, a (C) hA = hC ; KB = K C


small disc of radius R/3 is removed from the disc. The
(D) hA < hC ; KB > K C
moment of inertia of the remaining disc about axis
perpendicular to the plane of the disc and passing
through O is  (2010) Q.8 A child is standing with folded hands at the center
of platform rotating about its central axis. The kinetic
energy of the system is K. The child now stretches
R/3
his arms so that the moment of inertia of the system
doubles. The kinetic energy of system now is  (2004)
K K
(A) 2K (B) (C) (D) 4 K
2 4

O Q.9 Consider a body, shown in figure, consisting of two


R
identical balls, each of mass M connected by a light rigid
rod. If an impulse J = Mv is imparted to the body at one
of its end, what would be its angular velocity?  (2003)
L
40 37
(A) 4 MR² (B) MR 2 (C) 10MR² (D) MR 2 M
9 9

Q.5 Let I be moment of inertia of a uniform square plate J=Mv


about an axis AB that passes through its center and is
(A) v / L (B) 2v / L (C) v / 3L (D) v / 4L
parallel to two of its sides. CD is a line in the plane and
makes an angle θ with AB. The moment of inertia of the
plate about the axis CD is then equal to  (1998) Q.10 A disc is rolling (without slipping)
on a horizontal surface. C is its center Q
(A) I (B) I sin² θ
and Q and P are two points equidistant C
(C) I cos² θ (D) I cos² (θ/2) from C. Let vP , v Q and v C be the P
magnitude of velocity of points P, Q,
Q.6 A solid sphere is in pure rolling motion on an
and C respectively, then  (2004)
inclined surface having inclination θ (2006)
(A) Frictional force acting on sphere is f = µ (A) v Q > v C> vP

(B) f Is dissipative force (B) v Q < v C< vP


1
(C) Friction will increase its angular velocity and (C) v Q = vP , v C = v
2 P
decrease its linear velocity (D) v Q < v C > vP
(D) If θ decreases, friction will decrease
Paragraph 1: Two discs A and B are mounted coaxially
Q.7 A ball moves over a fixed track as shown in figure. on a vertical axle. The discs have moments of inertia I
From A to B the ball rolls without slipping. If surface BC and 2I respectively about the common axis. Disc A is
is frictionless and K A , KB and K C are kinetic energies imparted an initial angular velocity 2 ω using the entire
of the ball at A, B and C respectively, then  (2006) potential energy of a spring compressed by a distance
x1. Disc B is imparted an angular velocity ω by a spring
C having the same spring constant and compressed by
a distance x2. Both the discs rotate in the clockwise
hA r direction.  (2007)

x1
B Q.11 The ratio is
x2
(A) hA > hC ; KB > K C
1 1
(B) hA > hC ; K C > K A (A) 2 (B) (C) 2 (D)
2 2
P hysi cs | 7.75

Q.12 When disc B is brought in contact with disc A, Q.16 The center of mass of the disk undergoes simple
they acquire a common angular velocity in time t. The harmonic motion with angular frequency ω equal to
average frictional torque on one disc by the other
during this period is. (A)
k
(B)
2k
(C)
2k
(D)
4k
M M 3M 3M
2Iω 9Iω 9Iω 3Iω
(A) (B) (C) (D)
3t 2t 4t 2t
Q.17 The maximum value of v 0 for which the disk will
Q.13 The loss of kinetic energy during the above roll without slipping is
process is
Iω2 Iω2 Iω2 Iω2 M M 3M 5M
(A) (B) (C) (D) (A) µg (B) µg (C) µg (D) µg
2 3 4 6 k 2k k 2k

Q.14 A small object of uniform Q.18 A thin uniform rod, pivoted at O, is rotating in
density rolls up a curved the horizontal plane with constant angular speed ω , as
surface with an initial velocity shown in the figure. At time t = 0, a small insect starts
V
v. If reaches up to a maximum from O and moves with constant speed v, with respect
height of 3v /4g with respect
2
to the rod towards the other end. It reaches the end
to the initial position. The object is of the rod at t = T and stops. The angular speed of
(A) Ring (B) Solid sphere the system remains ω throughout. The magnitude of

(C) Hollow sphere (D) Disc ( )
the torque τ about O, as a function of time is best

represented by which plot?  (2012)


Paragraph 2: A uniform thin cylindrical disk of mass
M and radius R is attached to two identical mass less
springs of spring constant k which are fixed to the wall
as shown in the figure. The springs are attached to the (A) l l (B) l l
disk diametrically on either side at a distance d from its
center. The axle is mass less and both the springs and the r r
axle are in a horizontal plane. The un-stretched length O T O T
of each spring is L. The disk is initially at its equilibrium
position with its center of mass (CM) at a distance L from
the wall. The disk rolls without slipping with velocity
 (C) l l (D) l l
v 0 = v 0 î . The coefficient of friction is µ . (2008)

r r
O T O T

y
Q.19 A small mass m is attached to a massless string
whose other end is fixed at P as shown in the figure.
The mass is undergoing circular motion in the x-y
2d

planewith centre at O and constant angular speed ω .


If the angular momentum of the system, calculated
 
about O and P are denoted by LO and LP respectively,
then  (2012)
R z
v0

P
x

Q.15 The net external force acting on the disk when its O m
center of mass is at displacement x with respect to its
equilibrium position is

2kx 4kx  
(A) −kx (B) −2kx (C) − (D) − (A) LO and LP do not vary with time.
3 3
7 . 7 6 | Rotational Mechanics

 
(B) LO varies with time while LP remains constant. Q.22 Consider a disc rotating in the horizontal plane
with a constant angular speed ω about its centre O. The
 
(C) LO remains constant while LP varies with time. disc has a shaded region on one side of the diameter
  and an unshaded region on the other side as shown in
(D) LO and LP both vary with time. the figure. When the disc is in the orientation as shown,
two pebbles P and Q are simultaneously projected at an
angle towards R. The velocity of projection is in the y-z
Q.20 A lamina is made by removing a small disc of
plane and is same for both pebbles with respect to the
diameter 2R from a bigger disc of uniform mass density
disc. Assume that (i) they land back on the disc before
and radius 2R, as shown in the figure. The moment of
inertia of this lamina about axes passing though O 1
the disc has completed rotation (ii) their range is less
and P is I0 and IP respectively. Both these axes are 8
perpendicular to the plane of the lamina. The ratio than half the disc radius and (iii) ω remains constant
IP / IO to the the nearest integer is  (2012) throughout. Then  (2012)

R 
y
2R
x Q
2R P O
O

(A) P lands in the shaded region and Q in the unshaded


Q.21 Two identical discs of same radius R are rotating region.
abouttheir axes in opposite directions with the same
(B) P lands in the unshaded region and Q in the shaded
constant angular speed ω . The discs are in the same
region.
horizontal plane. At time t = 0, the points P and
(C) Both P and Q land in the unshaded region.

  (D) Both P and Q land in the shaded region.

P Q Paragraph for Questions 23 and 24


R R
The general motion of a rigid body can be considered
to be a combination of (i) a motion of its centre of mass
about an axis, and (ii) its motion about an instantaneous
Q are facing each other as shown in the figure. The
axis passing through the centre of mass. These axes
relative speed between the two points P and Q is vr
need not be stationary. Consider, for example, a thin
In one time period (T) of rotation of the discs, vr as a
uniform disc welded (rigidly fixed) horizontally at its
function of time is best represented by - (2012)
rim to a massless stick, as shown in the figure. When
vr vr
the disc-stick system is rotated about the origin on a
horizontal frictionless plane with angular speed ω , the
(A)
(A) (B)
(B)
motion at any instant can be taken as a combination of
O T
t
O T
t
(i) a rotation of the centre of mass of the disc about
vr vr the z-axis, and (ii) a rotation of the disc through an
(C)
(C) (D)
(D) instantaneous vertical axis passing through its centre of
mass (as is seen from the changed orientation of points
O T
t
O T
t P and Q). Both these motions have the same angular
speed ω in this chase.
P hysi cs | 7.77

z with the horizontal. Then with respect to the horizontal


 surface, (2012)
z

Q P P Q
y 

x /2
Now consider two similar systems as shown in the P
3R
figure: Case (a) the disc with its face vertical and parallel R
30o x
to x-y plane; Case (b) the disc with its face making an O
angle of 45° with x-y plane and its horizontal diameter
2R
parallel to x-axis. In both the cases, the disc is welded
at point P, and the systems are rotated with constant
angular speed ω about the z-axis.
z z
Q
(A) The point O has a linear velocity 3Rωˆi
 
Q

45o 11 ˆ 3
(B) The point P has a linear velocity Rω kˆ
y y
P P Rω i +
4 4
x x
13 ˆ 3
Case (a) Case (b) (C) The point P has a linear velocity Rω i − Rω kˆ
4 4
Q.23 Which of the following statements about the  3 1
instantaneous axis (passing through the centre of mass) (D) The point P has a linear velocity  3 −  Rω ˆi + Rω kˆ
 4  4
is correct?  (2012)  

(A) It is vertical for both the cases (a) and (b) (B) It is
Q.26 Two solid cylinders P and Q of same mass and
vertical for case (a); and is at 45° to the x-z plane and
same radius start rolling down a flixed inclined plane
lies in the plane of the disc for case (b)
from the same height at the same time. Cylinder P has
(C) It is horizontal for case (a); and is at 45° to the x-z most of its mass concentrated near its surface, while Q
plane and is normal to the plane of the disc for case (b) has most of its mass concentrated near the axis. Which
statement(s) is (are) correct? (2012)
(D) It is vertical for case (a); and is at 45° to the x-z
plane and is normal to the plane of the disc for case (b) (A) Both cylinders P and Q reach the ground at the
same time.
Q.24 Which of the following statements regarding the (B) Cylinder P has larger linear acceleration than
angular speed about the instantaneous axis (passing cylinder Q.
through the centre of mass) is correct? (2012)
(C) Both cylinders reach the ground with same
(A) It is 2 ω for both the cases translational kinetic energy
ω (D) Cylinder Q reaches the ground with larger angular
(B) It is ω for case (a); and for case (b)
2 speed

(C) It is ω for case (a); and 2 ω for case (b)


Q.27 A uniform circular disc of mass 50 kg and radius
(D) It is ω for both the cases 0.4 m is rotating with an angular velocity of 10 rad
s-1 about its own axis, which is vertical. Two uniform
circular rings, each of mass 6.25 kg and radius 0.2 m,
Q.25 The figure shows a system consisting of (i) a ring
are gently placed symmetrically on the disc in such a
of outer radius 3R rolling clockwise without slipping
manner that they are touching each other along the
on a horizontal surface with angular speed ω and (ii)
axis of the disc and are horizontal. Assume that the
an inner disc of radius 2R rotating anti-clockwise with
friction is large enough such that the rings are at rest
angular speed ω / 2 . The ring and disc are separated
relative to the disc and the system rotates about the
by frictionless ball bearings. The system is in the x-z
original axis. The new angular velocity (in rad s-1) of the
plane. The point P on the inner disc is at a distance
system is (2013)
R from the origin, where OP makes an angle of 30°
7 . 7 8 | Rotational Mechanics

Q.28 In the figure, a ladder of mass m is shown leaning


against a wall. It is in static equilibrium making an angle
θ with the horizontal floor. The coefficient of friction
between the wall and the ladder is µ1 and that between
the floor and the ladder is µ2 . The normal reaction of
the wall on the ladder is N1 and that of the floor is N2 .
If the ladder is about to slip, then  (2014)
Q.31 Two identical uniform discs roll without slipping on
1 two different surfaces AB and CD (see figure) starting at
A and C with linear speeds v1 and v 2 , respectively ,and
always remain in contact with the surfaces. If they reach
B and D with the same linear speed and v1 = 3 m / s ,

then v 2 in m/s is (g = 10 m/s2)  (2015)
2

mg v1=3m/s v2
(A) µ1 = 0 µ2 ≠ 0 and N2 tan θ = A C
2 30m 27m

mg
B D
(B) µ1 ≠ 0 µ2 =0 and N1 tan θ =
2
mg
(C) µ1 ≠ 0 µ2 ≠ 0 and N2 =
1 + µ1 µ2
Q.32 A ring of mass M and radius R is rotating with
mg angular speed ω about a fixed vertical axis passing
(D) µ1 = 0 µ2 ≠ 0 and N1 tan θ =
2 through its centre O with two point masses each of
M
Q.29 A uniform circular disc of mass 1.5 kg and radius 0.5 mass at rest at O. These masses can move radially
8
m is initially at rest on a horizontal frictionless surface. outwards along two massless rods fixed on the ring
Three forces of equal magnitude F = 0.5 N are applied
simultaneously along the three sides of an equilateral as shown in the figure. At some instant the angular
triangle XYZ with its vertices on the perimeter of the 8
disc (see figure). One second after applying the forces, speed of the system is ω and one of the masses is at
9
the angular speed of the disc in rad s-1 is  (2014) 3
a distance of R from O. At this instant the distance of
5
F the other mass from O is (2015)
X


O
Y E
Z

F
O
Q.30 A horizontal circular platform of radius 0.5 m
and mass 0.45 kg is free to rotate about its axis. Two
massless spring toy -guns, each carrying a steel ball of
mass 0.05 kg are attached to the platform at a distance 2 1 3 4
(A) R (B) R (C) R (D) R
0.25 m from the centre on its either sides along its 3 3 5 5
diameter (see figure).Each gun simultaneously fires the
balls horizontally and perpendicular to the diameter
in opposite directions. After leaving the platform, the Q.33 The densities of two solid spheres A and B of the
balls have horizontal speed of 9ms-1 with with respect r
to the ground. The rotational speed of the platform in same radii R vary with radial distance r as ρA (r ) =
k 
r
5 R 
rad s-1 after the balls leave the platform is (2014)
and ρB (r ) =k   , respectively, where k is a constant.
R 
The moments of inertia of the individual spheres about
P hysi cs | 7.79

 
axes passing through their centres are I A and IB ,
I n
(C) The force F is given by F= (ˆi + 2 ˆj) N
respectively. If B = , the value of n is  (2015)
I A 10 
(D) The torque τ = − ( 20 / 3) kˆ N m
Q.34 A uniform wooden stick of mass 1.6 kg and length
 rests in an inclined manner on a smooth, vertical
Q.36 Two thin circular discs of mass m and 4m, having
wall of height h ( <  ) such that a small portion of the
radii of a and 2a, respectively, are rigidly fixed by a
stick extends beyond the wall. The reaction force of the
wall on the stick is perpendicular to the stick. The stick massless, rigid rod of length  = 24 a through their
makes an angle of 30° with the wall and the bottom of centers. This assembly is laid on a firm and flat surface,
the stick is on a rough floor. The reaction of the wall on and set rolling without slipping on the surface so that
the stick is equal in magnitude to the reaction of the the angular speed about the axis of the rod is ω . The
floor on the stick. The ratio h /  and the frictional force angular momentum of the entire assembly about the
(
fat the bottom of the stick are g = 10 ms−2 (2016)) 
point ‘O’ is L (see the figure). Which of the following
statement(s) is(are) true?  (2016)
h 3 16 3
(A)
= = ,f N
 16 3 4m

h 3 16 3 m
(B)
= = ,f N
 16 3 z
 
h 3 3 8 3 2a
(C)
= 2 = ,f N 
 16 3 O
a

h 3 3 16 3
(D)
= = ,f N
 16 3

(A) The magnitude of angular momentum of the


 assembly about its center of mass is 17 ma2 ω / 2
Q.35 The position vector r of a particle of mass m is
 
given by the following equation r ( t ) = α t3 ˆi + β t2 ˆj, where (B) The magnitude of the z-component of L is 55 ma2 ω
= / 3 ms−3 , β 5 ms−2 and m = 0.1 kg . At t = 1 s,
α 10 = (C) The magnitude of angular momentum of center of
mass of the assembly about the point O is 81 ma2 ω
which of the following statement(s) is(are) true about
(D) The center of mass of the assembly rotates about
the particle?  (2016)
the z-axis with an angular speed of ω / 5

(A) The velocity v is given
=

v (10 ˆi + 10ˆj) ms −1


(B) The angular momentum L with respect to the origin

is given by L = − (5 / 3) kˆ Nms
7 . 8 0 | Rotational Mechanics

PlancEssential Questions
JEE Main/Boards JEE Advanced/Boards
Exercise 1 Exercise 1
Q.19 Q.22 Q.23 Q.5 Q.7 Q.8
Q.27 Q.28 Q.10 Q.11 Q.21
Q.24

Exercise 2 Exercise 2
Q.12 Q.25 Q.7 Q.9 Q.21
Q.22 Q.28 Q.41
Q.54
Previous Years’ Questions
Q.4 Q.7 Q.9 Previous Years’ Questions
Q.12 Q.1 Q.3

Answer Key

JEE Main/Boards
Q.11 KB> K A
Exercise 1 Q.18 8 g
Q.19 1.584 × 10⁷ g cm²
Q.1 kg m2, [M1L2T0], No
Q.20 1.25 kg m²;
Q.2 Torque Q.21 9.83 × 10³⁷ kg m²
Q.3 Inertia
Q.22 3.5 × 10⁴ g cm² ; 1.75 × 10⁴ g cm² ; 8.75 × 10⁴ g
Q.4 Theorem of parallel axes and theorem of cm² ; 10.5× 104 g cm²
perpendicular axes
5 2
2 Q.23 mL
Q.5 I = MR 2 , M = mass & R = radius 4
5
2 Q.24 0.01J
Q.6 I = MR 2 , M = mass & R = radius
3 Q.26 108 days
Q.8 No
Q.27 80π kg m2 s−1
Q.9 Hollow sphere
Q.10 IB> IA Q.28 360 r.p.m
P hysi cs | 7.81

Exercise 2

Single Correct Choice Type


Q.1 B Q.2 B Q.3 A Q.4 B Q.5 D Q.6 D
Q.7 D Q.8 B Q.9 B Q.10 C Q.11 A Q.12 D
Q.13 B Q.14 B Q.15 C Q.16 B Q.17 B Q.18 B
Q.19 B Q.20 A Q.21 D Q.22 C Q.23 C Q.24 B
Q.25 D Q.26 B Q.27 C

Previous Years’ Questions


Q.1. A Q.2 C Q.3 B Q.4 A Q.5 A Q.6 B
Q.7 C Q.8 C Q.9 C Q.10 B Q.11 A Q.12 C
Q.13 C Q.14 A Q.15 D Q.16 D Q.17 B Q.18 A
Q.19 C Q.20 B Q.21 A, C Q.22 D

JEE Advanced/Boards 3L2ρ


Q.12
8π2
Exercise 1
Q.13 1.65 N, 1.224 m
Q.1 2Mg
Q.14 L/6
Q.2 5 5 m/s
1.2g 1.2g  ˆ
Q.3 3 Q.15
(i)(a) (i)
(i)(a)
(a) (clockwise)  
(clockwise)  (b) −i  + 
(b) − 0.3g( 0.3g(i +  2 ˆj) 
2 j)      
c c
2.4g
Q.4 2F/M (ii) (a)
(ii)(a) (clockwise) (b) 0.5g
c
Q.5 2m/s
14gR
→ → Q.16 V =
Q.6 FA = (–133.64 î + 196 ˆj )N and FB = 133.64 î 3

2v 0 v0
3aρ0 5a 7ρ0 a3 12 Q.17 (i) v = (ii) t =
Q.7 (a) ; (b) ; (c) ; (d) ; 3 3µg
2 9 12 7ρ0 a2
1
w = 1 [3µ22mg22 t22 − 2µ mg t v 0 ](t < t0 ),
(e)
7 2 3
ρ ga w = 2 [3µ mg t − 2µ mg t v 0 ](t < t0 ),
4 0 2 1
w = − 1 mv 202 (t > t0 )
w = − 6 mv 0 (t > t0 )
6
Q.8 300 rad/sec, 150 rad/sec
Q.18 t = 2 5 sec, q = 4π/5 rad
Q.9 16 m/s²
37 37
Q.19 (a) ω / 3 , (b) m ω R, (c) mωR
27 3 3
Q.10 v = gR
7
Q.20 15 ft/sec
200 3
Q.11 (a) N; (b) 4 m/s Q.21 (a) 5 m/s2, (b) 0.3 < h < 1.5 m
7 7
7 . 8 2 | Rotational Mechanics

6aπ a
Q.22 6N, −0.6jˆ ± 0.6kˆ Q.23=
(i) t ;=
(ii) s 1 + (2π + 3)2
3v 0 3

Exercise 2

Single Correct Choice Type


Q.1 D Q.2 C Q.3 B Q.4 A Q.5 B Q.6 A
Q.7 D Q.8 B Q.9 D Q.10 C Q.11 A Q.12 A
Q.13 C Q.14 C

Multiple Correct Choice Type


Q.15. B, C Q.16 A, D Q.17 A, C, D Q.18 A, B, C Q.19 B, C Q.20 B, C
Q.21 B, C Q.22 B, C, D Q.23 C, D Q.24 A, B, C, D Q.25 B, C Q.26 A, B, D
Q.27 A, C, D Q.28 A, C, D Q.29 B, C, D Q.30 A, C Q.31 B, C Q.32 A, C
Q.33 A, B, C Q.34 A, B, C Q.35 A, B, C Q.36 B, C, D Q.37 A, B, C, D Q.38 B, C

Assertion Reasoning Type


Q.39 B Q.40 B Q.41 A Q.42 B Q.43 B Q.44 D
Q.45 B Q.46 D

Comprehension Type
Q.47 C Q.48 A Q.49 C Q.50 C Q.51 B Q.52 B
Q.53 A Q.54 A Q.55 D Q.56 B

Previous Years’ Questions


Q.1 A Q.2 A Q.3 B Q.4 A Q.5 A Q.6 D
Q.7 A Q.8 B Q.9 A Q.10 A Q.11 C Q.12 A
Q.13 B Q.14 D Q.15 D Q.16 D Q.17 C Q.18 B
Q.19 C Q.20 C Q.21 A Q.22 C, D Q.23 A Q.24 D
Q.25 A, B Q.26 D Q.27 8 Q.28 C, D Q.29 2 Q.30 4
Q.31 7 Q.32 D Q.33 6 Q.34 D Q.35 A, B, D Q.36 A, D
P hysi cs | 7.83

Solutions

JEE Main/Boards Sol 5: Moment of inertia of a solid sphere about its


diameter is
Exercise 1 I=
2
MR2
5
Sol 1: Moment of inertia of any body is given by where M = mass of the sphere
I = ∫ r 2dm where ‘dm’ is mass of a small element under R = Radius of the sphere
consideration and ‘r’ is the distance between the axis of
Derivation:
rotation and the element. Then,
S.I. units of moment of inertia will be m2 kg or kg.m2
R
DIMENSIONS – dm
[M0LT0]2[M1L0T0] = [M1L2T0] r
dr
It is not a vector quantity since direction is nowhere
considered.

Sol 2: Torque is the rotational analogue of force. Considering, the solid sphere as a large group of hollow
spheres whose radii range from 0 to R with a thickness
Sol 3: Moment of inertia is the rotational analogue of of dr. Integrating moment of inertia of these dements
mass of a body. gives the required value.
dm = ρ. 4pr2.dr (dm = ρ. dv) and I = ∫ dI
Sol 4: The theorem of parallel axes R
2 ∴
I = Icm + m.d 2 I= ∫ 3 .r
2
ρ. 4pr2dr ( I = ∫ r 2dm ) and
0
where Icm = Moment of Inertia about an axis passing 2
through center of mass and parallel to the considered (Ihollow sphere = MR2)
3
axis R
2 2  r5 
m = mass of the body ⇒ dI = (dm)r2 = . 4pρ  
3 3  5  0
d = distance between the axis (about which the value
of I is required) and the axis passing through center of  M 
R4 2
mass and parallel to the considered axis. = 4pρ . = MR  ρ = 4 π 3 
2 
5 5 R
The theorem of perpendicular axes  3 
Iz = Ix + Iy
z Sol 6: The moment of inertia of a hollow sphere about
2
an axis passing through its center is I = MR2
3
where M = mass of sphere
R = Radius of sphere
x
Derivation:

y Considering the hollow sphere, as a large group of


rings with thickness dr and radii ranging from 0 to R.
This theorem is applicable for planar bodies only and Integrating the moment of inertia of these rings we will
Ix and Iy should be about two perpendicular axes lying get the required moment of inertia.
in plane.
7 . 8 4 | Rotational Mechanics

∴ Ihollow sphere > Isolid sphere


r dm = . 2rRd
Sol 10: Given,
 
d R d Iz
m m

Ix

I= ∫ dI = ∫ r 2dm = ∫ r. ρ 2πrRdθ
Iy

[ Iring = MR ] 2
Circular discs A and B of same mass and same thickness
but different densities dA and dB (dA > dB)
⇒ dI = dmr2 ⇒I= ∫ 2πρR (R
3
sin3θ)dq
π MA = MB
3
I = 2prR4 ∫ sin θdθ
0 ⇒ πR 2A . dA tA = πRB2 . dB .tB
π 1/2
MR 2 (3sin θ – sin3θ) RA d 
⇒I=
2 ∫ 4
. dq ⇒ =  B 
0 RB  dA 

MR 2
π
 3cos θ cos3θ  MAR 2A
⇒I= (IA)x = = (IA)y
– +  4
2 0
4 12 
MAR 2A
MR 2
 3 1  3 1  2 ⇒ (IA)z = (IA)x + (IA)y =
⇒I= –  – +   ⇒ I = MR2 2
 –
2  4 12  4 12   3 [By perpendicular axes theorem]
also
Sol 7: Factors on which moment of inertia depend
MBRB2
→ Mass of the body (IB)z =
2
→ Mass distribution of the body 2
IA R  d
→ Size of the body =  A  = B ⇒ IA < IB since dB < dA
IB  RB  dA
→ Axis about which moment of inertia is required
Sol 11: Given.
Sol 8: No, it is not a constant
Moment of inertia of two rotating bodies A and B as
It depends on the axis about which moment of inertia
IA and IB (IA > IB)
is calculated
and
Iaxis
Since, K = Angular moment (LA and LB) are equal
M
⇒ LA = LB
Iaxis = moment of inertia about a given axis
then kinetic energies
M = mass of the body (constant)
(K.E.)A and (K.E.)B will be
Sol 9: Given 1 2 1  1 2 1 1 L2 
L A / I A and L2B / IB  K.E.= Iω = Lω= 
A solid sphere and hollow sphere have same mass and 2 2  2 2 2 I 
same radius
2 ⇒ (K.E.)A < (K.E.)B since IB < IA
Isolid sphere = MR2
5
2
Ihollow sphere = MR2 Sol 12: Moment of inertia is the measure of tendency
3
P hysi cs | 7.85

of a body to resist rotational motion if it was at rest or  2π 


1 1 2
 ∫ cos θ dθ = 0 
resist being stopped it was rotating. = I ω2 + MVcm
2 cm 2  0 
Radius of Gyration is the radius of the circle is which
a zero sized particle of mass M (which is equal to the 1 1
∴ K.E. = Icm ω2 + MVcm
2
mass of a body considered) such that the moment of 2 2
inertia of both the particle and the body about any axis
are equal
Sol 14: Laws of rotational motion
axis First law: Every body has tendency to be in rest or is
state of rotation unless acted upon by a torque

M Second law: The torque applied on a body is moment of


inertia times the angular acceleration of the body J – Iα
M K
MK2
Third law: Every action (torque) has an equal and
opposite reaction (torque) on the body which gave the
I I= MK2 action.

I
⇒K= Sol 15: Newton’s second law of linear motion is
M
F = ma
Sol 13: Kinetic energy of rolling motion where F = force acting on the body
m = mass of body
a = acceleration of body
w Vcm Now, consider ‘dm’ part of as body acted upon by a force
F and the body is rotating with an angular acceleration
of ‘α’
Consider a body rolling with angular velocity ω and
linear center of mass velocity vcm
Velocity of a particle at any point is given by dm
(V + rω cosθ) î + (rω sinθ) ˆj
cm F r

 Vcm+r cos 
 
 dT = r × F
r
= r. dm (rα)
r sin 

= r2 . dm . α
K.E. = ∫ d(K.E.)
∫ dT = ∫ r 2dm.α
∫ 2 dm ((Vcm + rω cos θ) )
1 2
= + (rω sin θ)2
⇒ Taxis = Iaixs aaxis
M M
1 2 2 1 2
=
2 ∫r ω dm +
2 ∫ Vcm dm + Sol 16: Principle of conservation of angular momentum.
0 0
In absence of a torque, the angular momentum of a
M
1 body is always conserved (constant)
2 ∫ 2Vcm r ω cos θ dm
0 dL
m T= =0
1 1 2 dt
= Icm ω2 + Vcm
2 2
+ Vcm ω ∫ r cos θ dm ⇒ L = constant ⇒ Iω = constant
0
R 2π
1 1 or I1ω1 = I2ω2
= Icm ω2 + MVcm
2
+ Vcm ω ∫ ∫ r cos θ .dθ .dr
2 2 0 0
7 . 8 6 | Rotational Mechanics

Example: By stretching hands a ballet decreases the angular Sol 21: Given,
speed of the body by increasing the moment of inertia
Radius of earth assuming it as a sphere as 6400 km
Example: A person sitting is a chair which can rotate mass of earth is 6 × 1024 kg
holds a rotating wheel in hands and when he flips the
Moment of inertia of the earth is
wheel, the person along with chair rotates conserving
angular momentum. 2 ∴ 2
I= MR2 ( I = MR2 for a sphere (solid))
5 5
Sol 17: Consider a circular ring of radius R and mass M 2
⇒I= × 6 × 1024 × (6400 × 103)2
5
⇒ I = 9.8304 × 1037 kgm2

R
dm Sol 22: Given,
A Uniform circular disc of mass 700 gms and diameter
2 2 20 cm
I= ∫ dI = ∫ R .dm = MR
(i) Moment of inertia about the transverse axis through
center of disc is
Sol 18: Given,
MR 2
I = 200 g cm2 I=
2
r = 5 cm 700 × 10 × 10
⇒ I= = 3.5 × 104 gcm2
We know that for a thin ring the moment of inertia 2
about an axis passing through center is Mr2 (ii) Moment of inertia about the diameter of disc is
T MR 2
⇒M= = 8 grams I=
2
r 4

[ Ix = Iy and Ix + Iy = Iz
Sol 19: Given,
(perpendicular axis theorem)]
diameter of disc = 40 cm
z
thickness of disc = 7 cm
density of disc = 9 gm cm–3
x
πD2
mass of the disc = ρ.V = ρ. .t
4
y
= 9 × π × 400 × 7 = 79168.13 grams
700 × (10 × 10)
moment of inertia of disc about a transverse axis ⇒ I= =1.75×104gcm2
through the center of the disc is 4

MR 2
I=
2
79168.13 × 20 × 20 R
⇒I= ⇒ I = 1.584 × 107 g cm2 (iii)
2

Sol 20: Given,


A uniform circular disc and A Uniform circular ring of
Moment of inertia about a tangent is plane is
same mass of 10 kg and diameter of 1 m
I = Id + MR2 (parllel axis theorem)
Moment of inertia of disc about a transverse axis
1 5MR 2
through center of the disc is I = MR2 ⇒I=
2 4
2
1 1 5
⇒ I = × 10 ×   = 1.25 kg m2 ⇒I= × (700) × (10 × 10) = 8.75 × 104 gcm2
2 2 4
P hysi cs | 7.87

A circular disc of mass 1 kg and radius 0.2 m rotating


(iv) about transverse axis passing through its center.
R Moment of inertia about the given axis as
MR 2 1
I= ⇒ I = × (0.2)2 = 0.02 kg–m2
2 2
30
Given, disc makes rotations per minute then,
Ic I π

Moment of inertia about a tangent perpendicular to 30 1


Angular velocity = 2π × × rad/s
the plane is 11 60
= 1 rad/s
I = IC + MR2
1 2
3 kinetic energy = Iw
⇒I= MR2 2
2
1
3 = × 0.02 × (1)2 = 0.01 joules
⇒ I= ×700 ×10 ×10 = 10.5 ×104 gcm2 2
2

Sol 25: Given,


Sol 23: x

C
D w V
1 60°
2
L L
A circular dice of mass M and radius r is set rolling on
table the kinetic energy of the disc is given by
60°
A B
1 2 1
L K.E = Iω + mv2(refer Q. 14)
2 2
Given, particles of masses m are placed at A, B and C 1 mr 2 2 1
⇒ K.E. = ω + mv2
and side of triangle ABC is L 2 2 2
Then, mv 2 mv 2
⇒ K.T. = +
4 2
AC AB L
CD = AC cos60° = = =
2 2 2 [For pure rolling v = rω]
moment of inertia of system is
3
I = I A + IB + IC ⇒ K.E. = mv2
4
2
L 
⇒ I = m(0)2 + m(L)2 + m  
2 Sol 26: Given,

5mL2 Sun rotates around itself once in 27 days angular


⇒I=
4 2π
velocity of sum = rad/days
27
Sol 24: Given, If it expands to truce its present diameter, then moment
of inertia becomes 4
2
∴I= MR2 (for sphere)
5

I2 R 22
=
I1 R12

⇒ I2 = 4I1
7 . 8 8 | Rotational Mechanics

I 1 ω1 = I 2 ω2 Exercise 2
(By principle of conservation of angular momentum)
Single Correct Choice Type
ω1 ω1
⇒ ω2 = = rad/days
4 108
Sol 1: (B)
Then, the new period of revolution
A

= = 108 days B MR 2
(2π / 108) I=
12
∴ π MR 2
T =  I=
 ω 12

Sol 27: Given, MR 2


MR 2 I=
I= 6
A 40 kg of flywheel is form of a uniform circular disc 1 2
meter in radius is making 120 r.p.m. C
1
Angular velocity = 120 × 2π × rad/s
60
= 4π rad/s sphere
Moment of inertia about transverse axis passing
through center is
MR 2 2MR 2
I= I=
2 5
40 × (1)2 B< C < A
⇒I= = 20 kg–m2
2
angular momentum = I.ω Sol 2: (B)
= 80 π kgm2/s = 251.33 kgm2/s

Sol 28: Given,


Angular velocity of the system = 120 r.p.m Ma2 I=Ma2
I=
Moment of inertia of the system initially = 6 kgm2 2
Moment of inertia of the system finally = 2 kgm2 I=
M(2a)2
12
I 1ω1 = I 2 ω2
M(2a)2
(By principle of conservation of angular momentum) I=
12
I1
⇒ ω2 = × ω1 = 360 r.p.m.
I2
2Ma2
∴ Final angular velocity = 360 r.p.m I=
3

D C

2a

A B

M
Mass of each rod =
4
P hysi cs | 7.89

Moment of inertia of one rod about the given axis is Sol 5: (D) Given,
 M(2a)2  Ma2 4Ma2 Ma2 I’ as a function of x of a rigid body
I=   + = =
 4 × 12  4 4×3 3 I = 2x2 – 12x + 27
 
Ma2 The value of moment of inertia is minimum at center
Total moment of inertia = 4I =
3 of mass point
To calculate min value of I, differentiate w.r.t. x and equate
Sol 3: (A) Given, it to 0

I1 is the moment of inertia about perpendicular bisector dI


=0
of rod dx
I1 ⇒ 4x – 12 = 0
⇒x=3
2
ML
I1 =
12 To check whether it is minimum,

d2 I
>0
Now rod is bent into semi-circular arc dx2

⇒4>0
∴ x = 3 is the x-coordinate of center of mass.

Sol 6: (D)

R 3m
Length of arc = L = pR A B
Moment of inertia of the arc = MR2
2N 4N
ML2
= = I2``
π2 To keep the body in equilibrium a 6 N acts at point x
from A
I2 > I1 (since p2 < 12)

x 6N
1 1
Sol 4: (B) So,=
2I
2
(M + M) R2 ⇒ 2I =
2
( 2M) R2 A B
P
2N 4N
Imaginary
R semi-disc Taking moment of torques about A
of same
specific list ∑ MA = 0 (In equilibrium)

⇒6×x–4×3=0
Mars=M
Radius=R ⇒ x = 2m

1 Sol 7: (D)
⇒ I = M R2 1 2
2 T1 T2
L/4
A B
L C
L/2
w
7 . 9 0 | Rotational Mechanics

Let T1 and T2 be tensions is the strings considering force Sol 10: (C) Given,
equilibrium
Two spheres of same mass M and radii R and 2R and
T1 + T2 = W also have equal rotational kinetic energies
Also Considering moment equilibrium about A 1 1 L2 L2
⇒ I1 ω12 = I2 ω22 ⇒ 1 = 2
2 2 I1 I2
∑ MA = 0 ∴
[ Iω= L (Angular momentum)]
3L L
⇒ T2 × –W× =0
4 2 L2 I2 R 22 R2
⇒ = = = =2
2W W L1 I1 R12 R1
⇒ T2 = ⇒ T1 =
3 3
Sol 11: (A) Angular momentum remains constant since,
Sol 8: (B) RA T no torque is acting on the skater.

A B While kinetic energy increases, since,


1
m K.E. = L.ω.
2
and as L = constant and ω increase K.E. increases
3
Sol 12: (D) Energy is not conserved in this case because
C W=mg the disc is fixed at its center and a force is acting on it
Center of mass of the triangular plate is at a distance when the child jumps.
 But Angular momentum can be conserved since No
of from AC.
3 torque is present in the boy-disc system
Considering force equilibrium ∴ Initial angular momentum = final angular momentum
RA + T = W = mg (I + mR2)ω = Iω’ + MRv
Considering torque or moment equilibrium about B (I + mR 2 )ω – mRv
⇒ ω’ =
 2  I
⇒ RA(  ) = W  
 3
2mg
⇒ RA = Sol 13: (B)
3

Sol 9: (B) Given, A constant torque is applied on a rod


which hinged the angular acceleration in rod will be 
/3
T
∝=
I
t
Tt
Angular velocity ω = ∫ α dt =
I
+ ω0
M2
0 Moment of inertia of the rod a bout pivot =
12
 Tt 
Power developed = T.ω = T.  + ω0  Since, the collision is elastic and the rod stops, velocity
 I 

If ω0 = 0 of the particle is v = .ω
3
T2 By principle of Angular Momentum,
P= (t)
I 
Pext Iω = m.v.
3
M2   3M
⇒ .ω = m. ω. ⇒ m =
Time 12 3 3 4
P hysi cs | 7.91

v 2  viˆ

Sol 14: (B)


C R
p v
2R
Sol 16: (B)
w v1  vjˆ R

The point P experiences centripetal acceleration


towards centre

∴ a = a î relative velocity
Angular velocity =
AB
the velocity of point P is
| viˆ – vjˆ | v
→ = =
v = v î + Rω ˆj 2R R


⇒ v = v î + v ˆj
Sol 17: (B)
T

[ v = Rω pure rolling]

⇒ v = v( î + ˆj )
→ →
a.v 1
cosθ = =
→ →
2
| a || v | mg
⇒ θ = 45°
mg – T T
Acceleration of yo-yo = =g–
m m
Sol 15: (C)
y
Sol 18: (B) In case of pure rolling v = Rω
3m/s (Bottom–most point has zero velocity)
8m

x
O
v
y
f
after 5 seconds w
15m
v
 f
8m  as ω is in clockwise direction, thread winds also friction
acts leftwards to increase w.
v cos

O

v cos θ v8 Sol 19: (B)


Angular velocity = =
 2
24 24 N1
= = rad/s w mg
82 + 152 289 f1

fa
Na

(a)
7 . 9 2 | Rotational Mechanics

6J
⇒ω=
m
f1=0
N1=0 B
mg w
L
.w
6 w B
fb
Nb vcm
vcm /2
1
f1 + Na = mg Nb = mg .w
2
N1 mg
+ Na = mg fb =
3 3
π m
fa ∴ After time t =
+ Na = mg [ N1 = fa] 12J
3
π m 6J π
The angle rotated by rod θ = ωt = . =
9mg  Na  3mg 12 J m 2
Na =  fa =  ⇒ fa =
10  3  10
2
 ω  2
fa 9 Velcoity of point P =   + (v cm )
=  6
fb 10
2 2
 J  J J
=   +  = 2
F m m m
Sol 20: (A)

Sol 22: (C) Solid cylinder rolls without slipping

⇒ vcm = Rω
F = Ma (By Newton’s second law), also T = Ta 1 2
(K.E.)rotational Iω
Ia ∴ a = 2
⇒ FR = ( α = for pvre rolling) (K.E.)translational 1
R R mv 2
2
⇒ I = MR2
(K.E.)r mR 2 ω2 1
This is satisfied for thin pipe ⇒ = =
(K.E.)t 2mv 2 2

Sol 21: (D)


T B Sol 23: (C) There will be no diff in velocity of centre of
mass
⇒ F = macm

C ∴ acm = same in both cases


/6
P

Sol 24: (B)


A
By conservation of linear momentum,
ˆj A hoop & solid cylinder of same mass
mvcm = J ⇒ vcm =
m ∴ Ihoop > Icylinder as mass is distributed away from center

By conservation of angular momentum, Iω = J. ∴ Since gain in potential energy in both case is same
2
P hysi cs | 7.93

Let P for hoop Previous Years’ Questions


2
1 1
P= mv2 + Ihoop  V  Sol 1: (A) A`B` ⊥ AB and C`D` ⊥ CD
2 2  
R 
2
From symmetry I AB = I A`B` and ICD = IC`D`
1 1
and P = mv2 + Icy  V  From theorem of perpendicular axes,
2 2 R 
 
A’
∴ Vhoop < Vcylinder C’ D

Sol 25: (D) (i) Upward acceleration = a = same for both


case A

B
1 4h
⇒ 2h = 0 × tQ + a × t2Q ⇒ tQ =
2 g
1 C D’
h = 0×tp + a × tp2 B’
2
2h IZZ = IAB + IA’B’ = ICD + IC’D’ = 2IAB = 2ICD
⇒ tp =
g
Alternate:
∴ tQ ≠ 2tp
The relation between I AB and ICD should be true for
(ii) The acceleration of both the balls is same = g sin q all value of θ

(iii) ∆KE = DPE At θ = 0, ICD = I AB

∴ ∆KEQ = 2mgH Similarly, at è = π /2 , ICD = I AB (by symmetry)

∆KEP = mgH Keeping these things in mind, only option (A) is correct.

∴ ∆KEQ = 2∆KEP
Sol 2: (C) Since, it is head on elastic collision between
two identical spheres, they will exchange their linear
Sol 26: (B) Moment of inertia decreases since mass is velocities i.e., A comes to rest and B starts moving
closer to axis while. with linear velocity v, As there is no friction anywhere,
torque on both the spheres about their center of mass
Angular momentum remains constant which implies
is zero and their angular velocity remains unchanged.
angular velocity increases and which intern implies
Therefore,
increase is kinetic energy.
∴ 1
( L = Iω and K.E = Lω) A=
2  v

A B A B
Sol 27: (C) Angular momentum is conserved on any Before collision After collision
point on the ground since the only force present passes
through that point making torque zero. v
Sol 3: (B) L = m r⊥
2

v 2 sin2 45° v2
Here, r⊥ = h = =
2g 4g

 v   v2  mv 3
∴L = m    =
 2   4g  4 2g
7 . 9 4 | Rotational Mechanics

Sol 4: (A) Let ω be the angular velocity of the rod. For work to be minimum, the moment of inertia (I)
Applying, angular impulse = change in angular should be minimum or
momentum about center of mass of the system dI
=0
L dx
J. = Ic ω
2
or 2(0.3) – 2 (0.7)(1.4 – x) = 0 or (0.3)x = (0.7)(1.4 – x)
L   ML2 
∴ (Mv)
=   (2)  ω (0.7)(1.4)
2 
 4  ⇒ x= = 0.98m
0.3 + 0.7
v
∴ ω=
L
Sol 8: (C) From the theorem
    
L0 = LCM + M (r × v)  … (i)
M M
We may write
J = Mv Angular momentum about O = Angular momentum
about CM + Angular momentum of CM about origin

∴ L0 = Iω + MRv
M
Sol 5: (A) Mass of the element dx is m = dx. 1 3
L = MR 2 ω + MR(Rω) = MR 2 ω
2 2
This element needs centripetal force for rotation.
y

y

v=R
F F+dF M  v
x=L x x
O B O
x dx (a) (b)
x=0 Note that is this case both the terms in
  
M  Eq. (i) i.e., LCM and M (r × v)
∴ dF = mx ω2 =  xω2dx 
L  Have the same direction. That is why we have used
L m L 2
Mω L L0 = Iω ~ MRv if they are in opposite direction as
∫0 dF = L .ω ∫0 xdx =
2
∴ F= shown in figure (b).
2
This is the force exerted by the liquid at the other end.
Sol 9: (C) At the critical condition, normal reaction N
will pass through point P. In this condition.
Sol 6: (B) mg sin θ component is always down the
τN = 0 = τfr (About P)
plane whether it is rolling up or rolling down. Therefore,
for no slipping, sense of angular acceleration should N
also be same in both the cases.
Therefore, force of friction f always act upwards. F

Sol 7: (C) Work done W= 1 Iω2 G


L
2
If x is the distance of mass 0.3 kg from the center of L
2
mass, we will have
f1
I = (0.3) x2 + (0.7)(1.4 – x2 )
mg
P hysi cs | 7.95

The block will topple when


Here, r 2 = a2 + [ R 2 − a2 − vt]2
L
τF > τmg or FL > (mg) Form conservation of angular momentum
2
mg ω0 I1 =
ω(t)I2
∴ F>
2
Substituting the values we can see that variation of ω
Therefore, the minimum force required to topple the (t) is non-linear.
block is
mg Sol 13: (C)∴ I1ω1 = I2ω2
F=
2
I1  Mr 2  M 
Sol 10: (B) Net external torque on the system is zero. ω2 = ω = ω =
2   ω
I2 2
 Mr + 2mr   M + 2m 
Therefore, angular momentum is conserved. Force
acting on the system are only conservative. Therefore,
total mechanical energy of the system is also conserved. a a2
Sol
= 14: (A) r 2 or r 2
=
2 2
Sol 11: (A) Mass of the whole disc = 4M a
Moment of inertia of the disc about the given axis M v M
a C r
1
= (4M)R 2 = 2MR 2

2 O O
∴ Moment of inertia of quarter section of the disc
Net torque about O is zero. Therefore, angular
1 1 momentum (L) about O will be conserved,
= (2MR 2 ) = MR 2
4 2 Or Li = L f
Note: These type of questions are often asked in a
objective. Students generally error in taking mass of the Mv  = I0 ω = (ICM + Mr 2 )ω
whole disc. They take if M instead of 4 M. 2
 Ma2   a2   2 2
=   + M =  ω Ma ω
Sol 12: (C) Since, there is no external torque, angular  6   2  3
   
momentum will remains conserved. The moment of
inertia will first decrease till the tortoise moves from 3v
ω=
A to Can then increase as it moves from C and D. 4a
Therefore, ω will initially increase and then decrease.
Let R be the radius of platform, m the mass of disc and Sol 15: (D) Mass of the ring M = ρL
M is the mass of platform.
Let R be the radius of the ring, then
Moment of inertia when the tortoise is at a
1
L=
2 π R or R =
MR 2 2π
I1 = mR 2+
2 Moment of inertia about an axis passing through O and
And moment of inertia when the tortoise is at B. parallel to XX` will be
1
I0 = MR 2
2
O Therefore, moment of inertia about XX` (from parallel
axis theorem) will be given by
r a
1 3
A D IXX' = MR 2 + MR 2 = MR 2
BC 2 2
vt Substituting values of m and R
2
MR  L2  3ρL3
I2 = mr 2+ 3
2 IXX' = (ρL)  =
2  4π2  8π2
 
7 . 9 6 | Rotational Mechanics

m2 r m1 r a2
Sol=
16: (D) r1 = ;r Moment of inertia about the required axis = I = ρa3 ,
m1 + m2 2 m1 + m2 6
nh M
(l1 + l)=
ω = n

where ρ =
4 3
πR
3
1 n  (m1 + m2 )
2 2
K.E.
=
2
( l1 + l2 ) ω=
2
 2R 
5
3M 1 32R5 4MR 2 4MR 2
2m1 m2 r 2 3M 1
=I =  = =
4 π R3 6  3 4 π R3 6 9 3 9 3 π 9 3 π
Sol 17: (B) From conservation of angular momentum
about any fix point on the surface  R
mr 2 ω
=0 2mr 2 ω Sol 21: (A,=
C) L0 mv
2
− kˆ ( ) [D to A]

ω0  R 
∴ω = =L0 mv  + a kˆ [C to D]
2  2 
ω0 r
∴ VCM =
2 Sol 22: (D) From normal reactions of roller, we can
conclude it moves towards left.
Sol 18: (A)

JEE Advanced/Boards

L

Exercise 1

Sol 1: Given,
V
A thin uniform rod of mass M and length L is hinged at
its upper end, and is released from rest in a horizontal

L changes in direction not in magnitude position.
O
L/3 COM
V
1000 L/2
Sol 19: =
(C) 5 e T −1 h1
L COM
V
⇒ e1000 6 ….(i)
=
T h2
V
1000
Again,
= I e T −1
Let angular velocity of the rod about hinge ‘O’ when it
1000 V is vertical be ‘ω’
dI 1000
= e T
dV T Moment of inertia of rod about o is
1000
2
1000 V L 
dI = e T dV I = Icom+ M   (parallel axis theorem)
T 2
Using (i) ML2 ML2 ML2
⇒I= + =
12 4 3
1000 60 60
∆ I= × 6 × 0.01= = = 0.2mA
T T 300 By using principle of conservation of energy
∆K.E = – DP.E
Sol 20: (B) For maximum possible volume of cube 1 2
⇒ Iω – 0 = – Mg(h2 – h1)com
2
2R = 3 a, a is side of the cube.
P hysi cs | 7.97

1 ML2 2 L 3g A is stationary
⇒ .ω = Mg ⇒ ω =
2 3 2 L Since, angular velocity of system would be same through
L VB 5
The tension in the rod at a point from hinge would ω= = rad/s
3 a a
be due to weight below that point and centrifugal force and
of that part.
VC = 2 a.ω = 5 2 m/s perpendicular to AC in vector
form.
VC = +5 î + (– 5) ˆj if co-ordinate system is along AB
r T and BC
Velocity of ‘C’ in original frame
dm 2L → → →
V c = V c + V a =+ 5iˆ – 5ˆj – 5ˆj ∴ (VA =– 5ˆj )
3

Mg ⇒ V c = 5iˆ – 10ˆj

2L 2M  M | Vc | = 52 + 102 = 5 5 m/s
m = ρ.A. =  ρ = 
3 3  A.L 

L Sol 3: Initial
2
Centrifugal force = ∫ rω .dm ω
L/3

L L
 r2 
= ω ∫ r.ρ.Adr = ω ρ.A  
2 2
/4 /4
L/3  2 L/3

3g M 1  2  8  4Mg
= .   [L ]   = final
L L 2
  9
  3
ω’
Tension at the point is T = mg + FC
2Mg 4Mg
= + = 2Mg
3 3 /2 /2

Sol 2: C Angular momentum of the system is conserved, since


no torque is applied
a Li = Lf
⇒ Ii ω = If ω1
A a B
 m 2 
2 
5 m/s ⇒  r + m  × 2  ω
 12 4 
 
Given,
10 m/s
 m 2 
2 
VA = 5m/s VB = 10 m/s =  r
+ m   × 2  ω1
 12 2 
In the frame of A
C  
0.03
⇒ × 30 = ω1
2 a 0.09
⇒ ω1 = 10 rad/s
A a B
By energy conservation,

5 m/s
7 . 9 8 | Rotational Mechanics

1 2 1 1 2
Sol 6: Given,
I ω = If(ω1)2 +  mv  × 2
2 1 2 2  A uniform wood door of mass m, height h and width w.
L 0.03 × 30 × (20) h 2h
⇒ (ω – ω1) = V2 ⇒ = V2 Location of hinges are and from the bottom of
2m 0.2 the door. 3 3
⇒ v = 3 m/s Let the hinges be named A and B.
∴ Velocity of ring along rod = 3m/s
w

Sol 4: Given,
(FA)
A (FA)x
A straight rod AB of mass M and length L, a horizontal
force F starts on A h/3 COM h
A F
mg
B
h/3 (FB)x
acm
Given, hinge A is screwed while B is not, So, the upward
component of force by hinge B is absent.
By equilibrium equations,
B acm
L
∑ Fx

2 = 0 ⇒ (FA)x = (FB)x
F = Macm (by newton’s second law)
∑ Fy = 0 ⇒ (F ) A y
= mg
T = Ia
L ML2
∑ MCOM =0
⇒ F. = ×a
2 12 (Moment about center of mass)
6F
⇒α=
ML ⇒ (FA)y  w  +(FA)x  h  + (FB)x  h  = 0
     
1 2 6 6
Acceleration of end B= acm î – x î
2
ω 3mgω
2F  F ⇒ mg . + (FB)x  h  = 0 ⇒ (FB)x = –
=– î  acm =  2   2h
M  M 3
→ →
2F and FA = (FA)x î + (FB) ˆj , FB = FB(– î )
∴ Magnitude of acceleration of end B =
M

3mgω →
3mgω
FA =– î + mg ˆj , FB = î
2h 2h
Given m = 20 kg, h = 2.2 m, ω = 1m
vcm=3m/s → →
Sol 5: 0.3 0.1 ⇒ FA = (–133.64 î + 196 ˆj )N and FB = 133.64 î
B String
w
Sol 7: Given, A thin rod of length ‘a’ with variable mass
A
Given, the wheel is rolling without slipping per unit length ρ = r0  1 + x  where x is distance from A.
 
∴  a
rAω = Vcm( VA = 0)
x
(pure rolling)   0 1 
x  a
The velocity of the string should be

VB = Vcm – rBω = Vcm  1 – rB  = 2m/s A B


  dx
 rA 
P hysi cs | 7.99

(a) Mass of the elemental part is dm = ρ.dx (c) Given, to find the moment of inertia about axis
perpendicular to rod and passing through A.
 x
⇒ dm = ρ  1 +  .dx
0
 a
dx
Mass of the rod
M a A B
 x
m= ∫ dm = ∫ ρ0  1 + a  .dx
0 0
a
 x2   3a 
⇒ m = ρ x +  m = ρ  
0
 2a  0
2 M
0
2 ∴
3aρ0
I= ∫ x dm ( dI = x2 dm)
∴ Mass of rod = 0
2
 x
(b) Center of mass is situated at distance of C from A but dm = ρ  1 +  .dx
where
0
 a 
M
a 
∫ x.dm 2 x3 
C= 0
⇒I= ∫ ρ0  x +  .dx
a 
M 0 
∫ dm  x3 x 4  7ρ0 a3
a
0 ⇒I= ρ  +  ⇒I=
M a
 x
0
 3 4a  0 12
Value of ∫ x.dm = ∫ x(ρ0 )  1 + a  dx
0 0
(d) We know that,
a
 x 2 x3   5a2 
= ρ  +  ⇒ ρ   Angular momentum L = I.ω
0  6 
 2 3a  0
0
 
7ρ0 a2
P.a = .ω
12
12
ω=
7ρ0 a2
a/2
(e) Given, an impulse of ‘P’ is applied at point B, then
Angular impulse about the axis will be
a/2
L = P.a
⇒ (Iω).ω = 2mga
P Pa.12.P
⇒ = 2 mga
7ρ0 a2
for minimum value of P, the angular velocity rod in the
final position should be zero 7 3  7 2 3
⇒ P2 = ρ ga2  2 P0 a  = ρ ga
by applying conservation of energy 6 0   4 0

K.E = – DP.E.
Sol 8: Given, two cylinders of mass 1 kg and 4 kg with
⇒ K.Ef – K.Ei = – mg(hf – hi)
radii 10 cm and 20 cm respectively.
1 2
0– Iω = – mg (a) also initial angular velocities as
2
5a2 ω1 = 100 rad/s and ω2 = 200 rad/s
5a
⇒C= 6 ⇒C= final angular velocities will be such that there is no slip
3a2 9 at point of contact
2
7 . 1 0 0 | Rotational Mechanics

r1 w21 Since, the body is in pure rolling

1
(Vcm = 3Rω) ⇒ (acm = 3Rα)
w2
w1
1 Solving (i) and (ii) we get,
6
acm = .g
5
r1 w11 Acceleration of the point where force is applied B
 3 8
a = acm î + R(α) î = acm î = g î
⇒ Vcontact = 0 ⇒ r1 ω11 – r2 ω12 =0 4 5

r1 ω11 | a | ≅ 16 m/s2
⇒ ω12 =
r2
Angular impulse on one cylinder due to other is Sol 10:
I1( ω11 – ω1) = (DP)(R1)
R+r
where DP = linear impulse while for the other sphere
v

r
R1 P R2 Given, A sphere of mass m and radius r and radius of
P loop as R + r the velocity of the sphere at the top most
point should be such that the centrifugal force balances
I2( ω12 –ω2) = (DP)(R2) the weight of sphere
1 mv 2f
I1 (ω1 – ω1 ) R1 ⇒ = mg
⇒ = R
I2 (ω1 – ω ) R2 ∴
2 2
( Center of mass makes circle of radius R)
⇒ 8( ω12 – ω2) = ω11 – ω1
⇒ vf = Rg
ω1 – 8ω2
⇒ ω11 = – 300 rad/s Since, the sphere is in pure rolling at every point of time,
5
ω1 vcm = rω
while ω12 = – = 150 rad/s
2 By principle of conservation of energy

Sol 9: K . E1 + P.E.i = K.E.f + P.E.f



1 1
Rf Mg ⇒ mv2 + Iω2 + mg(r)
2 2
3R a cm 1 1
= mv 2f + Iω2f + mg(2R + r)
2 2
I=MR2
1 1 2 1
⇒ mv2 +   mv2 = mv 2f +
f 2 2 5 2

Let, 1 2 2 2


    mv + mg(2R)
acm be acceleration of center of mass α be angular 2 5 5 f
acceleration 10
⇒ v2 = v 2f + g(2R)×
by Newton’s second law, 7

Mg – f = Macm  .... (i) 20Rg 27Rg


⇒v= Rg + =
and by considering torque 7 7

Mg × R + f × 3R = I . α .... (ii) 27Rg


∴v=
7
P hysi cs | 7.101

Sol 11: r.T2 = I2 .a2; Ma


r
⇒T=–
a2 2
a2 =
r2 2 Ma
– – mg sinα = ma
2
mg – T1 – T2 = ma1; T2
 M
a1 ⇒ – mg sin α =  m +  a
a1 = T1
 2
r1
(a) (T1 – T2).r = I1α1 also, mgsin α
1 ⇒a=–
r  M
a2 = 2a1 by constraint relations. m + 
 2
2g
⇒ a1 = Ma M ( + mgsin α )
7 So, T = – = ×
2x 2 2  M
2mg 200 m + 
⇒ T2 = = N  2
7 7
(b) And also velocity ‘v’ of the M.m.gsin α 2 × (1 / 2) × (9.8)(1 / 2) 9.8
T1 = = = = 1.65 N
body after traveling 1.2 m (2m + M) (2 × 0.5 + 2) 6

3 x x
(b) ω = 10 rad/s
v= 2× a× s = 4 m/s
7
mgsin α mg (0.5)(9.8)
⇒ α= a/R = – =– =–
 M R(2m + M) 0.2(1 + 2)
R m + 
Sol 12: The moment of inertia of a thin hoop about it’s  2 
diameter is
5
=–– . 9.8 rad/s2
1 6
M R2
2 =– 8.166 rad/s2
Here M = Lρ Now, ω2 = ω20 + 2αq
Also we have 2πR = L ⇒ 0 = (10)2 + 2(– 8.166) × q
1
⇒R = 100
2π ⇒θ= = 6.123 rad,
2 × 8.166
2
1 1  L  L3ρ So distance = θR = 6.123 × 0.2 = 1.224 m
So we have, I =M R 2 =L ρ   =
2 2  2π  8π2
Now using parallel axis theorem we have Sol 14:
2 2 v =0
L3ρ 3L ρ v0
 L 
I xx=
' Icm 2
+ M R= + L ρ  = m m
8π2
 2π  8π2
x Vcom

Sol 13:
A
T T×R
a
ω
ωL
 vA = Vcom – ; so for vA = 0
2
mg sin
ωL
⇒ Vcom =
(a) Now T – mg sina = ma  ... (i) 2
Now, mv0 = M.vcom (moment cons.)
MR 2 MRα
TxR = – Iα = – ×α=– ... (ii) mv 0
2 2  ⇒ Vcom =
M
Rα= a  ... (iii)
7 . 1 0 2 | Rotational Mechanics

and mv0x = Iω c


Torque about A, T = (mg)   = Ia
[Angular momentum conservation about O] 2
c
ML2 mg
So mv0x = ×ω ⇒α= 2 = 6g = 1.2g
12 5c c
5mc2
12mv 0 x 12
⇒ω=
ML2 5c  1.2g 
Acceleration of the center is a = ×  
4  c 
So Vcom = ωL/2’
= 0.3( 5g )
mv 0 12mv 0 x L or
⇒ = × ⇒ x = L/6
M ML 2 2 →
a = – 0.3( î + 2 ˆj )

Sol 15:
A B

C
2
mg

C
Before connection B is released
TA + TB = mg (By force equilibrium)
After connection B is released
and mg
TA is still , while TB = 0
TA = TB (for torque equilibrium) 2
Mg  mg 
TA = TB = g  mg – 
2 linear acceleration a = 2 
2  m 
⇒ Just after B is released  

TA is mg but TB = 0 angular acceleration = 0.5 g


⇒ FBD
TA Sol 16: By energy conservation,

A
C 5
2 4 R R/2
C
4 C
2 K.Ei + P.Ei = K.Ef + P.Ef
⇒ 0 + mgR
mg
1 m 2  1  m R 
=   (v )  1 +  +   (g)  
Moment of inertia about 24  2  4  2
2
m  2  C  
2
C 5  14gR
A= C +  + m .  ⇒v=
12   2  2 4 3
   
14gR
mc2  5  5 5 ∴ Velocity of the axis of cylinder =
⇒I=   + mc2 ⇒ I = mc2 3
12  4  16 12
P hysi cs | 7.103

Sol 17: FBD of disc is


v0 v

Angular momentum about any point on ground is 


conserved
f
⇒ mv0R = Iω + mvR
mR 2 ω MR 2 2f
mv0R = + mvR f.R = Iα ⇒ f. R = α⇒α=
2 2 MR
2 ∴ FBD of ant, in frame of disc
⇒v= v0 [ v = Rω]
3
a1
Work done by frictional for time to

1 1 1  m(R)
W=– mv 20 +  mv 2 + Iω2 
2  2 2 
mv 20 3 4 
f
W=– +  m. v 20  mRα
2 4 9  ⇒ mRα + f = ma1 ⇒ mRα + = a1
2
1 5Rα
W=– mv 20 ⇒ a1 = ∴ M = 3m
6 2
Also for t > to No frictional force exists Given, after time T, the ant reaches same point
1 1  5Rα  2 2
⇒W=– mv 20 for t ≥ to ⇒   T = 2π .R ⇒ T = seconds
6 2 2  5
Also ma = – mmg ⇒ a = – mg 1 2 4π
Also the angle moved by disc = aT = radians
and v = v0 – mgt 2 5

v0
to = Sol 19:
3µg
w
T f.R 2µg
α= = =
I MR 2 R R
m
2
Work done by friction for t < to = K.Ef – K.Ei m v=2wR
1 1  2µgt    1
2
2
⇒ W =  m(v 0 – µgt)2 + I   –  mv 0 
2 2  R   2 
 
1 1  Angular momentum of the system is conserved
⇒ W =  mµ2g2 t2 + m(2µ2g2 t2 ) – mv 0 µgt 
2 2  Li = Lf

1 mR 2 ω  mR 2 
⇒W= (3mm2g2t2 – 2mv0mgt) ⇒ =  + mR 2  ω1
2 2  2 
 
ω
⇒ ω1 =
Sol 18: 3
300cm
Impulse in the direction of velocity
20cm 100cm
 = m (vf – vi) = – mv = – 2mωR
7 . 1 0 4 | Rotational Mechanics

Impulse perpendicular to direction of velocity Sol 21:


= m(vf – mi) = m(Rω1)
mRω
=
3 F
mRω
Net Impulse = – 2mωR ˆj + î
3 h ma 0.9 m

37
| ∆p | = mRω
3
0.6 m
Impulse on particle due to disc =
F = 100 N
Impulse on hinge due to disc F = ma ⇒ a = 5 m/s2

37 The cabinet will tip when
⇒ | ∆P |disc due to hinge = mRω
3 F. h > mg(0.3) + ma(0.9)
20 × 10 × 0.3 20 × 5 × 0.9
Sol 20: h> +
30 inches 100 100
⇒ h > 1.5 m

View and also when


from the 2ft/s2 F.h > m = (0.9) – mg (0.3)
⇒ h > 0.3 m
car

∴ 0.3 < h < 1.5 m is the range of values of h for which


 cabinet will not tip.

Sol 22: In the frame of truck,



am/s2
2ft/s2
2cos ft/s2

2cos θ
Angular acceleration of door = ft/s2
w f f
 
w is the width of the door 2
z
We know that,

dω dω dθ
=α⇒ . =α y
dt dθ dt
x
⇒ ω.dω = α . dq
π /2 2M(a) – 2f = 2Ma1 (force equation)
ω2 4

2
–0=
w ∫ cos θ .dq (2f)R = (2T).α (Moment equation)
0
a1 = Rα (pure rolling)
ω2 4
⇒ = [1]
2 w Iα Ma1
f= =
⇒ω=2 R 2

Velocity of the outer edge = (ω)(w) = 8w 3a1 2a


⇒a= ⇒ a1 = = 6 m/s2
2 3
P hysi cs | 7.105

f=
Ma1
= 6N
Exercise 2
2
Single Correct Choice Type
Frictional torque magnitude about rod is
f.R = 0.6 Nm
Sol 1: (D) A
O
B
Friction torque about O is
I2
I1
 0.2 
= ± 0.6   ( k̂ ) – 0.6(0.1) ˆj = – 0.6 ( ˆj ± k̂ )
P
 2 
O’
D C
Sol 23: A a

We know the moment of inertia of a square plate along


Ma2
OO' as
12
v cm
Ma2
1 3a
.
3 2 ∴ I2 =
12
mv0 B
Also IZ = IX + Iy
C

Conserving linear momentum, we get (Perpendicular axis theorem)


(3m)(vcm) = mv0 ⇒ IZ = 2IX
v0 ∴
( IX = Iy fn square)
⇒ vcm =
3
Ma2
Conserving angular momentum about COM we get ⇒ IZ =
6
a (IZ is about transverse axis through p) to find IAPC or I1,
mv0 = = I.ω
2 3 we take x and y axis as diagonals of square and apply
mv 0 a  a 
2 perpendicular axis theorem again
⇒ = 3m   .ω z
2 3  3
y
v0
⇒ω=
2 3a
π
Time taken to complete one revolution =
ω x

3πa
= Iz Ma2
v0 ⇒ IX = Iy = ⇒ I1 = IX =
2 12
Displacement of point B will be I1
=1
 2a 3 ˆ 2a 1 ˆ  I2
vcm t î +  × i+ . j
 3 2 3 2 

Sol 2: (C) Given, moment of inertia of rectangular plate
 2π  a ˆ about transverse axis through P as I then the moment
⇒  + 1  (a) î + j
 3  3 I
of inertia of PQR about P will be greater that since
2
mass is distributed away from P unlike in PSR. Since, I
depends on distance ‘r’, the farther the mass, the more
the moment of inertia. The moment of inertia of PQR
7 . 1 0 6 | Rotational Mechanics

I then about any axis as shown below will be


will be less than about R since mass is distributed
closes to R 2 I
A B

P Q

D C
R I1
Consider, this axis as x-axis and y-axis perpendicular to
a line to it
Sol 3: (B) Given, A triangle ABC such AB = BC = a and
∠ACB = 90° of mass M (I1)x-axis = (I1)y-axis |By symmetry|

A
I1x
I1y
O 
a


C B
a

O is the midpoint
Iz = I1x + I1y = 2 I1x
Consider a counterpart with same mass such a square
is formed Iz
I1x =
2
A C1
Ma2
We know that Iz is independent θ since Iz =
O 6

⇒ I1x is also independent of q


C B
⇒ I1x (θ) = I1x (0) = I
we know the moment of inertia of a square about
Ma2
transverse axis through center as Sol 5: (B) Given, see-saw is out of balance.
6
⇒ Centre of mass is not at the center of see-saw.
But, here m = 2M (total mass)
Ma2 wH
⇒ Isquare =
3 L/2
Since both triangles are symmetric about axis through O
x
wG
they have equal moment of inertia about axis through O. COM
Isquare Ma2 ws
⇒ Irequired = =
2 6
Let ‘x’ be the distance of COM from center. By moment
equilibrium at center
Sol 4: (A) Given, I is moment of inertia of a uniform
L (wH – wG ) L
square plate about axis parallel to two of its sides and [wH – wG] = wS . x ⇒ x =
passing through center 2 wS 2
I Now if the girl and body move to half of the original
A B wH

L/2
x
wG
D C COM
ws
P hysi cs | 7.107

L Moment of inertia of rod about point ICOM


moment due to heavy body and girl is (wH – wG)
4 Md2
(opposite in direction to wS.x) =
12
while Angular momentum is conserved as no torque is acting
L on the system. (while energy is not since a force acts on
wS . x = (wH – wG)
2 rod at point)
L ∴ Initial angular momentum = final angular momentum
∴ wS . x > (wH – wG)
4 d
I 1ω1 + m v 1 = I 2ω2
The side the girl is sitting on will once again tilt 2
downward
Md2 dv  Md2 Md2 
⇒ (0) + m =  +  ω1
3 2  12 4 
 
Sol 6: (A) F
md 3
⇒= v md2 ω1
2 4
F 2v
⇒ ω1 =
3d
The force on the hinge is same as the force on the
thread this can be found by using force equilibrium
conditions. Sol 9: (D)
l
Since, there is a torque always about hinge on pulley,
angular velocity increases. P

Sol 7: (D)
P


w
 By the principle of conservation of energy
O
torque acting on the pole due to weight about point K.Ei + P.Ei = K.Ef + P.E.f
O is 1 1

⇒ 0 + mg(l) = mv2 + Iω2 + 0
→ L 2 2
T = r × F = W cosq
2 1 2 2 1 2 2 2
⇒ mgl = mr w + . mr ω
T = Ia 2 2 5
L
W cos θ
3mgcos θ 3gcos θ 10 gl
⇒α= 2 = = ⇒ω=
ML 2 2mL 2L 7 r2
3
3gcos θ Angular momentum of the sphere about P is
Acceleration of point P=L.α =
2 L = Iω + m.l.v
2 2
⇒L= mr ω + m. l. r ω
Sol 8: (B) 5

10 2 
⇒ L = m. gl .  r + l
7  5 
6m d

m v
7 . 1 0 8 | Rotational Mechanics

Sol 10: (C) when α = 45°


vA = – v and y =
2
A
dv A 2v 2 . 2
⇒ =–
y dt 
vA
Angular acceleration =
30° v
C B
x 45°
x2 + y2 = l2 aA
aA
aA
L = length of ladder = constant 2
2
dx dy
⇒ x. + y. =0 a=0
dt dt
⇒ x. v + y.vA = 0 aA

α= 2
x
⇒ vA = – .v = – 3v (L)
y
2v 2
O α=
L2
L/2

3v L ( L sin30°) OC= L
Sol 12: (A)
3 60° 2 2
L /2 Initial Final
ω0
30°
v

Angular velocity of rod


v0
ω=0 v=0
| – 3vjˆ + viˆ | 2v
= =
L L
f f
L
Velocity of center = .ω = v Frictional forces acts to reduce the velocity of bottom
2
most point.
dv
Sol 11: (A) If =0 f = – macm (By Newton’s second law)
dt
T = Ia
x2 + y2 = l2 MR 2 MRα
⇒ f.R = .a⇒f=
dx dy 2 2
⇒ x. + y. =0
dt dt After time t = t, angular velocity and lineal velocity
⇒ x. v + y vA = 0 becomes zero.

dx dv dy dv ⇒ 0 –v0 = – acmt and 0 – ω0 = – at


⇒ .v + x. + . v + y. A = 0
dt dt dt A dt
f
dv A v0 Macm
⇒ v2 + v 2A + y. ⇒ = =
dt w0 α 2f
MR
dv A 0 – (v 2 + v 2A )
⇒ = v0 R v 1
dt y ⇒ = ⇒ 0 =
ω0 2 Rω0 2
P hysi cs | 7.109

Sol 13: (C) Conserving linear momentum Sol 14: (C) When F1 is applied, the body moves right
and angular acceleration is developed accordingly by
2mv – mv = 2m × vcm
friction
v
vcm = when F3 is applied, the angular acceleration developed it
2
the body move left.
Initial angular momentum
When F2 is applied the body can move either left of
b b 3vbm right depending on angle of inclination.
= m × 2v × + mv × =
2 2 2
Final angular momentum Multiple Correct Choice Type
 b 2
b
2
mb2 ω
= m   + m    × ω = Sol 15: (B, C) x
 2  2   2

COM
3mvb mb2 ω A B
⇒ =
2 2
FBD of rod
3v
∴ ω2 =
b x
For skater at x = b/2

RA w RB

wb RA + RB = w (force equilibrium)

2 RB.d = w.x (torque equilibrium)

wx w(d – x)
⇒ RB = and RA =
d d
ωb
vx = v + cosq
2
Sol 16: (A, D) mg cos
ωb
vy = – sinq
2
 mgsin
θ = ωt
mg cos
v
∴ vx at t =

2
mg
3v  3vt  Sliding condition = mg sinθ > mmgcosθ
vx = v + cos  
2  b 
⇒ tanθ > m
3V  3vt  h a
∴ x = ∫ v x dt = ∫ v + cos   Toppling condition = mgsinθ. > mgcosθ.
2  b  2 2
a
⇒ tanθ >
3V  3vt  h
= vt + × b × sin  
2 × 3V  b  a
If µ >
h
b  3vt 
= vt + sin   a
2  b  tanθ > is met earlier than tanθ > m
h
ωb ∴ Topples before sliding
y= ∫ v y dt = ∫ – 2
sin (ωt)
a
If µ <
+ωb b  3vt  h
= cos (ωt) = cos  
2×ω 2  b 
It will slide before toppling
7 . 1 1 0 | Rotational Mechanics

Sol 17: (A, C, D) Kinetic energy of system before collision is equal to


a=g kinetic energy of system just after collision since, the
mg h collision is elastic

d mR
Sol 20: (B, C)
Angular momentum = mvd = mgtd m
dL
Torque of gravitational force = = mgd
dt
Kinetic energy of the body
Moment of inertia = m(d2 + h2)
1 1 1
1 = mv2 + mv2 + Iω2
where h = H0 – gt2 2 2 2
2
v gt  1 2 4 ∴
Angular velocity = = =  1 + .  mv2 = mv2 ( v = Rω)
d d  2 3 3

→ → → ∴ 2
Sol 18: (A, B, C) T = A × L ( I= mR2 only hollow sphere
3
→ ∴
Non-viscous liquid )
dL → →
⇒ = A×L
dt Angular momentum about any point on ground
→ 2 8
dL → = 2mRv + mR2ω = mRv
∴ ⊥ L 3 3
dt
→ →
Components of L on A remain unchanged because Sol 21: (B, C) A
→ R
if L component changes the L.H.S changes while w
R.H.S remains unchanged which is a contradiction. If  
magnitude of L changes with time, thin L.H.S and R.H.S
vary differently with time which is a contradiction. B v


L
Suppose L = (x.t) Since the cylinder does not slip

|L| At point B velocity = 0
→ →
→ → → → 
dL xL → →
A× L ⇒ − V + V cm + Rω = 0
Then = while A × L = xt
dt → → →
|L| |L| ⇒ V cm = – (v – Rw) î

At point A, velocity = 0
Sol 19: (B, C) A
⇒ v = Rω

⇒ V cm = 0

x
Sol 22: (B, C, D) To the right of B, angular acceleration
will disappear but linear acceleration will increase since
no friction is present angular velocity attained by disc
m
after time T is
Linear momentum is not conserved because of hinge
force angular momentum about A is conserved since ω = aT
torque at A is zero. 1
and 2π = aT2
2
P hysi cs | 7.111

Time to complete one rotation (c)


2π = ωt F/2
Ix F
2π 2π.T T
⇒t= = = Ix
αT 2π.2 2 F/2

Sol 23: (C, D) Given,


180° < Qf – Qi > 360°
(d) While D is possible
π rad < Qf – Qi < 2π rad
F
Sol 24: (A, B, C, D)
1
2 F F

3
Sol 26: (A, B, D)

v
4 h w
I
A f = w = mg
I = I1 + I3 (Perpendicular axes theorem)
L = mvh (Angular momentum)
Also I1 = I3 (by symmetry)
µmg
I acceleration = – = – mg
⇒ = I1 = I3 m
2
Vt = v – mgt
I = I2 + I4 (perpendicular axis theorem)
Lt = m(v – mgt)h
Also I2 = I4 (by symmetry)
dL t
I = T = – mmgh
⇒ = I2 = I4 dt
2
Sol 27: (A, C, D) If Re spreads or curls up his hands,
Sol 25: (B, C) Option A is incorrect, since the statement moment of inertia changes, accordingly angular
indicates a force body system as below. velocity changes too.
If I ω = Constant, it cant keep
1 2
F1 F2 I ω the same, rotational kinetic energy would also
2
change.

Which is not in equilibrium while B, C are possible Sol 28: (A, C, D)

F3 F1
u
w

u = Rω (for pure rolling)


F2
→ → →
(b) F1 + F2 + F3 = 0 The velocity of bottom most point is u – Rω = 0
7 . 1 1 2 | Rotational Mechanics

the velocity of topmost point is u + Rω = 2u Sol 32: (A, C)


∴ 0 ≤ v ≤ 2u
a1

fpseudo=ma
Rw 30°
120° f

P
u In frame of plank,
Fpseudo – f = ma1
Velocity of P is
f
a1 = a – where a is acceleration of plank
→  u 3u ˆ  m
V = u î +  – ˆi + j
 2 2  F–f
 a=
M

u 3u ˆ
⇒ V = î + j Total K.E. of system = work done by force F
2 2 ∴
( no other external forces is doing work)
⇒V=u
Work done on sphere = work done by friction + work
If CR is horizontal done by pseudo force = change in K.E.

V = uiˆ + ujˆ
Sol 33: (A, B, C)
V= 2u A
2v

Sol 29: (B, C, D) Angular moment about O is not


constant because a component of weight causes v
torque at point ‘O’.
Angular moment about C is zero since weight is parallel
axis B

About O, VA = V + Rω = 2V
→ → → VB = V – Rω = 0
L = m( r × v ) gives angular momentum in direction
perpendicular to length of thread and velocity. The 1 3mRV
L about B = mvR + mR2ω = clockwise
vertical component never changes direction. 2 2
1 mRV
L about A = – mvR + mR2ω = anti-clockwise
Sol 30: (A, C) 2 2

Sol 34: (A, B, C)


w
v
N

>v P

r v
o o
30 30
A cylinder rolling down with incline may or may not
w
<v
attain pure rolling. It depends on length of the incline

Sol 31: (B, C) Friction on cylinder under pure rolling B


depends on the external forces
P hysi cs | 7.113

N Sol 38: (B, C) By holding a pole horizontally, the


moment of inertia is increased leading to slower
>v angular acceleration due to undesired torques.
C
v Also, adjusts center of gravity to be vertically over rope
to eliminate torque.
<v R
R M R Assertion Reasoning Type

VP = (V + rω cosθ) î + (rω sinθ) ˆj
→ Sol 39: (B) A cyclist always bends inwards to reduce the
| VP |2 = V2 + r2ω2 + 2rω cosqV centrifugal force.
Also, he lowers the center of gravity.
= (R2 + r2 + 2r R cosθ)ω2
→ But the reason does not explain the assertion
| VP |2 = M2P2ω2

⇒ | VP | = ( MP ).ω
Using the above relation we draw a circle with radius R Li
to get point with velocities R.ω = v
θi
Sol 35: (A, B, C) As the ring enters frictional force Sol 40: (B)
x θf
(limiting) acts on sphere increases angular acceleration
in clockwise direction and slows down the linear motion.

Sol 36: (B, C, D) Rolling motion starts when the Lf


point of contact has zero velocity. Conserving angular
momentum about point of ground gives Statement-I is true because the initial angular moment
mvR = Iω + mv’R about any point on xy plane is Li=mvisinqix and final
∴ angular momentum Lf = mvfxsin qf
⇒ mvR = mR2ω1 + mR2ω1 ( v1 = Rω1)
for elastic collision vf = vi, qi = qf
v
⇒ ω1 = ⇒ Li = Lf
2R
v Statement-II is also correct since the disc is in equilibrium
v1 =
2 But II is explanation of I
Time taken to achieve pure rolling is
v v Sol 41: (A)
v – u = at ⇒ – v = –mgt ⇒ t =
2 2µg
Q
P
Sol 37: (A, B, C, D) Distance moved by ring
W
v 2 – u2 3v 20 r1
= = r2
2a 8µg
3v 20
Work done by friction = – mmg. In frame of O
8µg
3mv 20 ωP = ωQ = ω
= –
8 vP = rPωP vQ = rQωQ
1 v 20 mv 20 In frame of P
Gain in rotational K.E. = mR2. =
2 4R 2 8
v0 = – rPωP vQ = rQωQ –rPωP
 3mv 20 1  mv 20 v0
Loss in K.E. = –  – + mv 20  = w0 = – = – ωP = ω
 8 8  4 rP
 
7 . 1 1 4 | Rotational Mechanics

(rQ – rP )(ω) Taking an elemental point dy


w1Q = =ω
(rQ – wP )

∴ Statement-I is true and statement-II is correct


explanation of I I dy
y
Sol 42: (B) Statement-I true by parallel axis theorem.
Statement-II is true but doesn’t explain parallel axis x
theorem.
dm = ρ.(2x)dy

Sol 43: (B) See above = ρ.(2y)dy [ y = x]

M 4M
Sol 44: (D) fy ρ= =
1 2 2

fx 4
dm(2x)2
dI2 = + dm(y)2
12
L
f
 m2 
 for rod, parallel axis theorem
 12 
 /2  
y2 ∴
⇒ ∫ dI2 = ∫  + y 2  (2y)ρ dy ( y = x)
Fx = F (force equilibrium in x-direction )  3 
0  
∴ Assertion is false  /2  /2
8ρ 3 8ρ  y 4 
While reason is true since ⇒ ∫ dI1 = I2 =
3 ∫ y .dy = .  
3  4 
0 0
F
T = Iα = F.x ⇒ α = .x  4 
I 2ρ ML2
=   =
3  16  6
Sol 45: (B) Statement-I is true, which is the condition
for pure rolling
Sol 48: (A) dIx = (dm)(y2)
Statement-II is also correct by the definition of center of
 /2
mass but II is not correct explanation of I. 3
⇒ ∫ dIx = ∫ 2ρy dy
0
Sol 46: (D) Statement-I is false, because a body can roll if  /2
 y4  8M  4 M2
we throw it with property determined linear and angular ⇒ Ix = 2ρ   = . =
velocities.  4  0 2 64 8
Statement-II is true by definition of pure rolling
Sol 49: (C) Moment of inertia about base is
Comprehension Type 2
L 
dI = dm  – y 
Sol 47: (C) 2 
y L /2 2
 L 
45° 45°
⇒ ∫ dI = ∫  – y  (2ρy) dy
2 
0

M L/2
L 
⇒ I = 2ρ ∫ y 2  – y  . dy
0 2 
45°
x
P hysi cs | 7.115

 L y3 y 4 
L /2 So 55: (D)
⇒ I = 2ρ  . – 
 2 3 4 
0 F

1 1   4 – 3  ML2 C
⇒ I = 2ρL4  48 – 64  = 8ML2  64 × 3  =
    24

Frictional force in this depends on distance between


Sol 50: (C) Iz = Ix + Iy (perpendicular axis theorem) application of force and center

ML2 ML2 ML2


⇒ Iy = – = f
6 8 24 Sol 56: (B) F

→ → →
Sol 51: (B) d L = dm ( r × v )
pseudo
force = Fs
dm 1
= ma f
y
 By basic FBD’s we can understand that friction acts in
r  forward direction
v x
∴ Option B is correct

L
z And the torque is acting horizontally, since the
horizontal component of angular momentum is only
dr changing.

 Previous Years’ Questions


r 
L
 Sol 1: (A) W =∆U =Uf − Ui =U∞ − UP
v
= −UP = −VP (as m = 1)
−mVP =
(z direction)
Potential at point P will be obtained by integration as
∴ Angular momentum is down at 20° to horizontal given below.

Sol 52: (B) There is a torque since angular momentum


is always changing direction. 2 2
4R 16R +r
Sol 53: (A)

r
dr
F

f
Let dM be the mass small ring as shown
Friction reduces linear acceleration and increases
angular velocity M 2Mrdr
= dM = (2πr)dr
π(4R)2 − π(3R)2 7R 2
Sol 54: (A) Same as previous
G dM
dVP = −
16R 2 + r 2
7 . 1 1 6 | Rotational Mechanics

4R I zz = IAB + IA`B` = ICD + IC`D` = 2I AB = 2ICD


2GM r 2GM
= − ∫
7R 2 3R 16R 2 + r 2
dr = −
7R
(4 2 − 5)
I AB = ICD

2GM
∴W =+ (4 2 − 5) Alternate The relation between I AB and ICD should
7R be true for all values of Nθ
At θ = 0, ICD = I AB
2 1 2
Sol 2: (A) =MR 2 Mr + Mr 2 Similarly, at θ = π /2, ICD = I AB
5 2
(By symmetry)
2 3
Or MR 2 = Mr 2 Keeping these things in mind, only option (a) is correct.
5 2
2
∴r = R Sol 6: (D) In case of pure rolling,
15
mgsin θ
f= (Upwards)
Sol 3: (B) Condition of sliding is mR 2
1+
ms sin θ > µ mg cos θ or tan θ > µ I
or tan θ > 3  … (i) ∴ f ∝ sin θ
Condition of toppling is Therefore, as θ decreases force of friction will also
decrease.
N
f Sol 7: (A) On smooth part BC, due to zero torque,
angular velocity and hence the rotational kinetic
O
mg energy remains constant. While moving from B to C
sin
mg cos   translational kinetic energy converts into gravitational
potential energy.


Torque of mg sin θ about O>torque of mg about


Sol 8: (B) From conservation of angular momentum
 15   10  (I ω = constant), angular velocity will remains half. As
∴ (mg sin θ )   > (mg cos θ )  
 2   2  1 2
K= Iω
2 2
or tan θ >  … (ii)
3 The rotational kinetic energy will become half. Hence,
With increase in value of θ , condition of sliding is the correct option is (b).
satisfied first.
Sol 9: (A) Let ω be the angular velocity of the rod.
Sol 4: (A) I Applying angular impulse = change in angular
remaining= Iwhole − Iremoved momentum about center of mass of the system
 2 2
1 2 1  R  1  2R   J.
L
= IC ω
or I (9M)(R) − m   + m    ….(i) 2
9 2  3  2  3  
   ML2 
L  v
2 ∴ (Mv)
=   (2)   ω ∴ ω =
2  
Here,=
m
9M R 
× π = M  4  L
πR 2
3
Substituting in Eq. (i), we have I = 4MR 2 Sol 10: (A) In case of pure rolling bottom most point is
the instantaneous center of zero velocity.

Sol 5: (A) A’B’ ⊥ AB and C’D’ ⊥ CD Velocity of any point on the disc,, where r is the distance
of point from O.
From symmetry IAB = IA`B` and ICD = IC`D` From
theorem of perpendicular axes, rQ > rC > rP
P hysi cs | 7.117

Paragraph 2

C Q Sol 15: (D) a= Rα

 
P 2kx − f  fR 
∴ =R  
M  1 MR 2 
O   2 
Solving this equation, we get
∴ v Q > v C> v P
a

Paragraph 1

1 1
Sol 11: (C) I(2ω)2 =kx12  …(i) 2kx
2 2
1 1
(2I)(ω)2 =kx22 ….. (ii)
2 2  f
x1 2kx
From Eqs. (i) and (ii), we have = 2 f=
x2 3
2kx 4kx
Sol 12: (A) Let ω' be the common velocity. Then from ∴ | Fnet=| 2kx − f= 2kx − =
3 3
conservation of angular momentum, we have
This is opposite to displacement.
(I + 2I)ω'= I(2ω) + 2I(ω)
4kx
∴ Fnet = −
4 3
ω' = ω
3
From the equation,  4kx 
Sol 16: (D) Fnet = −  x
Angular impulse = change in angular momentum, for  3 
any of the disc, we have F  4k 
∴ a = net = −  2
 x = −ω x
 4  2Iω M  3M 
τ.t = I(2ω) − I  ω  =
3  3
4k
2Iω ∴ ω=
∴ τ= 3M
3t
Sol 17: (C) In case of pure rolling mechanical energy
Sol 13: (B) Loss of kinetic energy = Ki − K f will remains conserved.
2 2
1 1  1 4  1 1 11  v  1 2 
=  I(2ω)2 + (2I)(ω)2  − (3I)  ω  = Iω2 ∴ Mv 20 +  MR 2   0  =
2  kxmax 
2 2  2 3  3 2 22  R  2 

2
 3v 2  3M
1 2 1 v 1 ∴ xmax = v
Sol 14: (D) mv + I   =
mg   ∴ I =mR 2 4k 0
2 2 R   4g  2
 
2kx
∴ Body is disc. As f =
3
2kx 2k 3M
µMg = max =
∴ Fmax = v
3 3 4k 0

3M
∴ v 0 = µg
k
7 . 1 1 8 | Rotational Mechanics

Sol 18: (B) Angular momentum about rotational axis  m 2R 2 


IP=  ( ) 2  m R
+ m ( 2R ) − 
2
m 
+ 5R 2 
  2   4 (2) 4 
  

 mR 2 5mR 2 
v ⇒ 2mR 2 + 4mR 2  −  + 
x    8 4 

x=vt
11 37
⇒ 6mR 2 − mR 2 ⇒ mR 2
I + m vt 2  ω
L( t ) =
 ( )  8 8
IP 37 8
dL t = × ≈3
= 2mv 2 t ω IO 8 13
dt

Torque
= τ (2mv ω) t
2
Sol 21: (A)

Sol 19: (C)

P  

P Q  = t
 = t

 
R sin    R sin 
L‘P LP
 
R R
R cos  R cos 
  
L0= r0 × p 2 ωR sin ( ω t )
So, v r =
 At t T=
= / 2, v r 0
L0 is always directed along the axis & its magnitude is
constant. So two half cycles will take place.

Sol 20: (C) Sol 22: (C, D)

O P
2R
t R sin t
Let mass of original disc = m
m m
The mass of disc removed = × πR 2 =
π 4R( ) 2 4 Rt
According to problem particle is to land on disc.
So M.O.I of remaining section about axis passing
If one consider a time ‘t’ then x component of disc is
m ( 2R )
2
 m R2 m 
through “O” IO = − + R2  Rωt
2  4 ( 2 ) 4  R sin ωt < R ω t

 mR 2 + 2mR 2   3 13 This particle ‘P’ land on unshaded region. For “Q”


⇒ 2mR 2 −  2
 ⇒ 2 −  mR ⇒ mR 2 x-component is very small and y-component equal to
 8   8 8
P it will also land in unshaded region.
MOI of remaining section about “P” Now repeat same thing when right part is shaded then
correct answer is “C” or “D”
P hysi cs | 7.119

Sol 23: (A) In both the cases, the instantaneous axis  50 0.4 2 
50 ( 0.4 ) ( )
2
will be along z-axis i.e. along vertical direction. + 4 ( 6.25 ) ( 0.2 ) ω
 2
× 10
=
2  2 
 
Sol 24: (D) w.r.t. centre of mass only pure rotation of
disc will be seen. So in both the cases, angular speed ω =8rad / sec
about instantaneous axis will be " ω " .

Sol 28: (C, D) Condition of translational equilibrium


Sol 25: (A, B)
N1 = µ2 N2
VP = Rωˆi +
ω ˆ
2
( ) (
− j × R cos 30° ˆi + R sin 30° kˆ ) N2 + µ1 N1 =Mg

ω ω 11 ˆ 3 mg
= 3Rω ˆi + 3 Rkˆ − Riˆ= Rω i + Rω kˆ Solving N2 =
4 4 4 4 1 + µ1 µ2
µ2 mg
N1 =
Sol 26: (D) 1 + µ1 µ2

N Applying torque equation about corner (left) point on


f the floor

mg cos θ N1  sin θ + µ1 N1  cos θ
=
mg 2
1 − µ1 µ2
Solving tan θ =
Translation motion: 2 µ2
mg sin θ − f = macm  …(i)
Sol 29: (2)
Rotational motion
R Icmα 
f= …(ii) F

Rolling without slipping


αR =acm  …(iii)
From (ii) & (iii) o
R 30
Icm acm
f=
R2
Rsin30o
Put this in (i)
Icm acm F
mg sin θ − = macm
2
R
mg sin θ
acm = F
 Icm 
 2 + m 
R  τ = Iα
As IP > IQ 3FR sin30° = I α

MR 2
Sol 27: (8) Conservation of angular momentum about I=
2
vertical axis of disc
α =2
ω = ω0 + α t
ω =2 rad / s

Sol 30: (4) Since net torque about centre of rotation


is zero, so we can apply conservation of angular
momentum of the system about center of disc
7 . 1 2 0 | Rotational Mechanics

Li = L f 3
N = mg
2
0 = I ω+ 2mv (r / 2 ) ; comparing magnitude

 0.45 × 0.5 × 0.5  0.5 2


∴ =ω 0.05 × 9 × ×2 N= mg
 2  2 3

∴ω =4 mg 16 16 3
fr
= = =
3 3 3
Sol 31: (7) Kinetic energy of a pure rolling disc having
Torque balance (about A)
1 1  mR 2  v 2 3
velocity of centre of mass v =+mv 2   = mv 2 h L
2 2  2  R 2 4 N× mg × sin30°
=
cos 30° 2
So,
3 2 2h L
(30 ) 34 m ( v 2 ) + mg ( 27 )=
m ( 3 ) + mg= mg × = mg ×
2 2
∴ v2 7 m / s 3 4
4 3

h 3 3
Sol 32: (D) Using conservation of angular momentum =
L 16
mR 2 ω
 8 ω   m 9R 2 8 ω   m 2 8 ω 
=  mR 2 × + × × + ×x ×  Sol 35: (A, B, D)
 9   8 25 9   8 9 

 d r (t)
4R 
⇒x =  dv
v= = 3 α t2 ˆi + 2 β t ˆj ,a = = 6 α t ˆi + 2 β ˆj
5 dt dt

2
Sol 33: I = ∫ ρ 4 πr 2 r 2 dr
At t = 1,=

v (10 ˆi + 10 ˆj) ms −1

3 
a 20 ˆi + 10 ˆj ms−2
=
∫ (r ) (r
)(r ) dr
2 2
IA ∝
 10 ˆ ˆ
=r i +5 j m
IB ∝ ∫ (r )(r )(r ) dr
5 2 2 3
    5 ˆ
IB 6 L0 =
r × mv =
− k N m s
∴ =  3 
I A 10
 
dv
(
F m = 2 ˆi + ˆj N
=
dt
)
Sol 34: (D) Force balance
      20 ˆ 
τ = r × F = r × ma =  − k N m
 3 
o
N sin 30

N cos 30o
ωa
Sol 36: (A, D) ωz = cos θ = ω /5
30 o

B
30o c
N

mg A
fr

N + N sin 30° =mg


2017-18 100 &
op kers
Class 11 T
By E ran culty
-JE Fa r
IIT enior emie .
S fP r es
o titut
Ins

PHYSICS
FOR JEE MAIN & ADVANCED
SECOND
EDITION

Exhaustive Theory
(Now Revised)

Formula Sheet
9000+ Problems
based on latest JEE pattern

2500 + 1000 (New) Problems


of previous 35 years of
AIEEE (JEE Main) and IIT-JEE (JEE Adv)

5000+Illustrations and Solved Examples


Detailed Solutions
of all problems available

Topic Covered Plancess Concepts


Tips & Tricks, Facts, Notes, Misconceptions,
Simple Harmonic Motion Key Take Aways, Problem Solving Tactics
and Elasticity
PlancEssential
Questions recommended for revision
8. SIMPLE HARMONIC
MOTION AND
ELASTICITY
SIMPLE HARMONIC MOTION

1. INTRODUCTION
There are so many examples of oscillatory or vibrational motion in our world. E.g. the vibrations of strings in a
guitar or a sitar, the vibrations in the speakers of a music system, the to and fro motion of a pendulum, vibration
in a suspension bridge as a vehicle passes on it, the oscillations in a tall building during an earthquake etc. Simple
harmonic motion (SHM) is a type of oscillatory or vibrational motion. Every kind of oscillation or vibration of
a particle or a system is not necessarily simple harmonic. The particle executing SHM like any other oscillatory
motion has a variable acceleration, but this variation is different in different kinds of oscillations. The study of
SHM is very useful and forms an important tool in understanding the characteristics of sound and light waves and
alternating currents. Any oscillatory motion which is not simple harmonic can be expressed as a superposition of
several simple harmonic motions of different frequencies.

2. PERIODIC AND OSCILLATORY MOTION


Periodic Motion: A motion which repeats itself after equal intervals of time is called periodic motion.
Oscillatory Motion: A body is said to possess oscillatory or vibratory motion if it moves back and forth repeatedly
about a mean position. For an oscillatory motion, a restoring force is required.
Examples of Periodic and Oscillatory motion are revolution of earth around sun and motion of bob of a simple
pendulum respectively.

PLANCESS CONCEPTS

All Oscillatory motions are periodic but all Periodic motions need not be oscillatory.
A body experiencing force F = −k(x − a)n is in Oscillatory motion only if n is odd and its mean position is
x=a. As, if n is odd only then we would have restoring force.
Vaibhav Krishan (JEE 2009, AIR 22)

2.1 Periodic Functions


A function is said to be periodic if it repeats itself after time period T i.e. the same function is obtained when the
variable t is changed to t + T. Consider the following periodic functions:
8 . 2 | Simple Harmonic Motion and Elasticity

2π 2π
f(t) = sin t and g(t) = cos t
T T
Here T is the time period of the periodic motion. We shall see that if the variable t is changed to t + T, the same
function results.

 2π   2πt   2πt 
f(t
= + T) sin  (t=+ T) sin  2π  sin 
+=  ∴ f(t + T) =
f(t)
T   T   T 
Similarly, g(t + T) =
g(t)
It can be easily verified that: f(t + nT) =
f(t) and g(t + nT) =
g(t)
where n = 1,2,3,……..

PLANCESS CONCEPTS

These functions could be used to represent periodic motion i.e. Periodic functions represent periodic
motion
T is the period of the above function.
To find periodicity of summation of two or more periodic functions the periodicity would be the L.C.M of
the periodicities of the each function
Vaibhav Gupta (JEE 2009, AIR 54)

Illustration 1: Find the period of the function, y= sin ωt + sin2ωt + sin3ωt  (JEE MAIN)

Sol: The function with least angular frequency will have highest time period.
The given function can be written as, y = y1 + y 2 + y 3
2π 2π π
Here y=
1 sin ωt , T1 = =
y 2 sin2ωt , =
T2 = , and =
y 3 sin3ωt
ω 2ω ω

T3 = ∴ T1 = 2T2 and T1 = 3T3


So, the time period of the given function is T1 or .
ω

Because in time T = , first function completes one oscillation, the second function two oscillations and the third,
three. ω

3. SIMPLE HARMONIC MOTION


Simple Harmonic Motion is a periodic motion in which a body moves to and fro about its mean position such that
its restoring force or its acceleration is directly proportional to the displacement from its mean position and is
d2 x
directed towards its mean position. It can be expressed mathematically as, F = m = −kx. Where m is the mass
2 dt2
d x
on which a restoring force F acts to impart an acceleration along x-axis such that the restoring force F or
dt2
acceleration is directly proportional to the displacement x along x-axis and k is a constant. The negative sign shows
that the restoring force or acceleration is directed towards the mean position.
P hysics | 8.3

d2 x K  d2 x
The differential equation of a simple harmonic motion is given by, 0 or
+  x = + ω2 x = 0
dt2 m
  dt 2

K acceleration
Where=
ω =
m displacement

The time period T, to complete one complete cycle by a body undergoing simple harmonic motion is given by

2π acceleration m
T= 2π
= 2π
=
ω displacement K

3.1 Types of SHM


Two Types of Simple Harmonic Motion
(a) Linear SHM (b) Angular SHM
Important among all oscillatory motion is the simple harmonic motion. A particle executing linear simple harmonic
motion oscillates in straight line periodically in such a way that the acceleration is proportional to its displacement
from a fixed point (called equilibrium), and is always directed towards that point.
If a body describes rotational motion in such a way that the direction of its angular velocity changes periodically
and the torque acting on is always directed opposite to the angular displacement and magnitude of the torque is
directly proportional to the angular displacement, then its motion is called angular SHM.

4. REPRESENTATION OF SIMPLE HARMONIC MOTION


If a point mass m is moving with uniform speed along a circular path of radius a, it’s projection on the diameter of
the circle along y-axis represents its simple harmonic motion (see Fig. 8.1). =
y asin ωt
Where ω is the uniform angular velocity of the body of mass m along a circular path Y
of radius a than ωt is angle covered by the radius in time t from the initial position
A at t = 0 to the position B. As ∠AOB = ∠OBC = wt, the foot of perpendicular from C B
t a
B to the diameter YOY‘ gives the projection at the point C such that y = OC is the y
projection of this body on the diameter and represents the displacement of the body O t
A
executing SHM along y-axis. If the body does not start its motion from the point A 
but at a point A’ so that ∠AOA’ is the phase angle φ ,
then=y asin ( ωt ± φ )  ... (i) A’
Where φ is the phase angle which may be positive or negative. The phase angle Y’
represents the fraction of the angle by which the motion of the body is out of step
Figure 8.1: Particle moving
between the initial position of the body and the mean position of simple harmonic
in a circle with angular
motion. The phase difference is the fraction of angle 2π or time period T of SHM by
speed w in X-Y plane
which the body is out of step initially from the mean position of the body.
dy
Differentiating equation (i), = v = a ω cos ( ωt ± φ )
dt
y y2 a2 − y 2
As sin ( ωt ± φ=
) ,cos ( ωt ± φ=
) 1 − sin2 ( ωt ± φ=
) 1− =
a a2 a

∴ v = ω a2 − y 2  … (ii)

d2 y
Differentiating (ii), the acceleration = = −aω2 sin ( ωt ± φ )
2
dt
d2 y 2
∴ = −ω y
dt2
8 . 4 | Simple Harmonic Motion and Elasticity

 d2 y  acceleration
It represents the equation of simple harmonic motion=
where ω  2 y
=
 dt  displacement
 

displacement
Time period, T = 2π ω = 2π
acceleration

4.1 Alternative Method for Finding Velocity and Acceleration in SHM


Let v be the velocity of the reference particle at P. Resolve velocity V into two Y V vcos
rectangular components V cos θ parallel to YOY’ and V sinθ perpendicular

vsin
to YOY’ (see Fig. 8.2). The velocity v of the projection N is clearly V cos θ . N
P

y
2 
∴ v = V cos θ = Aω cos ωt or v =Aω 1 − sin ωt X’
O
X

y2 A2 − y 2 
or v =Aω 1 − or v = Aω or v = ω A2 − y 2
A2 A2 Y’
2 Figure 8.2: Relation between v and w.
V
The centripetal acceleration of the particle at P can be resolved
A
2
Y V cos
A
V2
into two rectangular components − cos θ Perpendicular to YOY’ 2
A N

P V sin
A
V2 V2 y
and sin θ anti-parallel to YOY’ Acceleration of N =
− sin θ 
A A X’
O
X

V2
2

( A sin θ ) = −ω2 ( A sin ωt )


V cos
or Acceleration =
− A
2
A

or Acceleration = −ω2 y Y’
Figure 8.3: Direction of centripetal
acceleration of particle
4.2 Time Period or Periodic Time of SHM
It is the smallest interval of time at which the details of motion repeat. It is generally represented by T.

 t+T   2πt 
' x'at (=
t + T ) A cos  2π + φ0  A cos 
= + 2π + φ0   ... (i)
 T   T 

It is clear from here that the details of motion repeat after time T. Time period may also be defined as the time
taken by the oscillating particle to complete one oscillation. It is equal to the time taken by the reference particle to
complete one revolution. In one revolution, the angle traversed by the reference particle is 2π radian and T is the
2π 2π
time taken. If ω be the uniform angular velocity of the reference particle, then ω = or T =
T ω
4.3 Frequency
It is the number of oscillations (or vibrations) completed per unit time. It is denoted by f. In time T second, one
vibration is completed.
1 1
In 1 second, vibrations are completed or f = or fT = 1
T T
2π 1
Also, ω = = 2π × = 2πf So, equation (i) may also be written as under
T T
=x A cos ( 2πft + φ0 )  … (ii)
P hysics | 8.5

The unit of f is s‒1 or hertz or ‘cycles per second’ (cps). ∴ φs−1 =


φHz =
φcps.

4.4 Angular Frequency



It is frequency f multiplied by a numerical quantity 2π . It is denoted by ω so that ω = 2πf = . Equation (vi) may
be written =as x A cos ( ωt + φ ) T

4.5 Phase
Phase of a vibrating particle at any instant is the state of the vibrating particle regarding its displacement and
direction of vibration at that particular instant.
The argument of the cosine in equation
= x A cos ( ωt + φ0 ) gives the phase of oscillation at time t.
t
It is denoted by φ . ∴ φ = 2π + φ0 or φ = ωt + φ0
T
It is clear that phase φ is a function of time t. The phase of a vibrating particle can be expressed in terms of fraction
of the time period that has elapsed since the vibrating particle left its initial position in the positive direction. Again,
2πt 2πt
φ − φ0 = ωt = . So, the phase change in time t is . The phase change in T second will be 2π which actually
T T
means a ‘no change in phase’. Thus, time period may also be defined as the time interval during which the phase
of the vibrating particle changes by 2π .

PLANCESS CONCEPTS

x
The phase difference between acceleration and
displacement is 180°. In SHM phase difference between +A
velocity and acceleration is π 2 and velocity and
displacement is π 2 . t

Fig. 8.4 (a) displacement, (b) velocity and (c) acceleration -A


x = A sin t
vs. time in SHM.
(a)
v = ±ω A2 − y 2 . Graphical variation of v with y is an V

ellipse.
+A
Max velocity at y = 0 i.e. at mean position and t
2
Vmax = Aω ; a = −ω y -A
dx
V= A cos t
Graph between acceleration and displacement of a dt

particle executing SHM is straight line. a (b)


Max acceleration at y = A i.e. at extreme position and 2
amax = Aω2 + A

2 dx
- A V= 2
dt - A sin t

(c)
Figure 8.4

Nivvedan (JEE 2009, AIR 113)


8 . 6 | Simple Harmonic Motion and Elasticity

Illustration 2: A particle executes simple harmonic motion about the point x = 0 . At time t = 0 it has displacement
x = 2 cm and zero velocity. If the frequency of motion is 0.25 s−1 , find (a) the period, (b) angular frequency, (c) the
amplitude, (d) maximum speed, (e) the displacement at t = 3s and (f) the velocity at t = 3s . (JEE MAIN)

is x A sin ( ωt + φ ) . When velocity is zero, the particle is at


Sol: The standard equation for displacement in SHM =
maximum displacement.
1 1
(a) Period T= = = 4s
f 0.25s−1
2π 2π π
(b) Angular frequency =
ω = = rad /=
s 1.57rad/ s
T 4 2
(c) Amplitude is the maximum displacement from mean position. Hence, A = 2 − 0 = 2cm
π
(d) Maximum speed v max = Aω = 2. = π cm / s = 3.14 cm / s
2
(e) The displacement is given by
= x A sin ( ωt + φ )
Initially at t=0; x = 2cm , then =2 2sin φ or sin φ = 1 = sin90 or φ =90

π π
Now, at t = 3s
= x 2sin  × 3 +=  0
 2 2 
(f) Velocity at x = 0 is vmax i.e.,3.14 cm/s.

Illustration 3: Two particles move parallel to x-axis about the origin with the same amplitude and frequency. At a
A
certain instant, they are found at distance from the origin on opposite sides but their velocities are found to be
3
in the same direction. What is the phase difference between the two? (JEE ADVANCED)

is x A sin ( ωt + φ ) . Displacement on opposite sides of the


Sol: The standard equation for displacement in SHM =
mean position has opposite signs. Equation for velocity is v = Aω cos ( ωt + φ ) . Velocities in same direction have
same sign.
Let equations of two SHM be=
x1 A sin ωt ... (i)

x2 A sin ( ωt + φ ) 
= … (ii)
A A 1
Give that= A sin ωt and =
− A sin ( ωt + φ ) Which gives sin ωt =  … (iii)
3 3 3
1
sin ( ωt + φ ) = −  … (iv)
3

1 1 1 1
From Eq.(iv), sin ωt cos φ + cos ωt sin φ = − ; cos φ + 1 − sin φ = −
3 3 9 3

7 7
Solving this equation, we get or cos φ = −1, ; φ = π or cos−1  
9 9
Differentiating Eqs. (i) and (ii), we obtain; v1 =Aω cos ωt and v 2 = Aω cos ( ωt + φ )
If we put φ = π , we find v1 and v 2 are of opposite signs. Hence, φ = π is not acceptable.

7
φ =cos−1  
9
P hysics | 8.7

Illustration 4: With the assumption of no slipping, determine the


mass m of the block which must be placed on the top of a 6 kg cart in 600N/m
S
order that the system period is 0.75s. What is the minimum coefficient
of static fraction µs for which the block will not slip relative to the
cart if the cart is displaced 50mm from the equilibrium position and

( )
released? Take g = 9.8m / s2 . (JEE ADVANCED)
Figure 8.5

k
Sol: ω = where M is the total mass attached to the spring. The maximum restoring force on the blocks will be
M
at the extreme position. The limiting friction on mass m should be greater than or equal to the maximum restoring
force required for mass m.

( 0.75) × 600 − 6
2
m+6  m m+6
(a) T = 2π  T = 2π  ∴ 0.75= 2π= ;m = 2.55kg
600  k  600
( 2π )
2

(b) Maximum acceleration of SHM is amax = ω2 A ( A = amplitude )

i.e., maximum force on mass ‘m’ is mω2 A which is being provided by the force of friction between the mass
2 2
ω2 A  2π  A  2π   0.05 
and the cart. Therefore, µsmg ≥ mω A or µs ≥
2
or µs ≥   or µs ≥     ( A = 50mm)
g  T  g  0.75   9.8 
or µs ≥ 0.358 . Thus, the minimum value of µs should be 0.358.

5. ENERGY IN SHM
The displacement and the velocity of a particle executing a simple harmonic motion are given by

=x A sin ( ωt + δ ) and v = Aω cos ( ωt + δ ) . The potential energy at time t is, therefore,

1 2 1 1
U= kx and k= mω2 Therefore=
U mω2 x2 = mω2 A2 sin2 ( ωt + δ ) , and the kinetic energy at time t
2 2 2
1 1
is K = mv 2 = mA2 ω2 cos2 (ωt + δ)
2 2

The total mechanical energy time t is E= U + K

=
1
2  ( )  1 mω2 A2
mω2 A2 sin2 ( ωt + δ ) + cos2 ( ωt + δ )=
 2
Average value of P.E. and K.E

By equation (i) P.E. at distance x is given by

1 1
U
=
2
mω2 x=
2
2
mω2 A2 sin2 ( ωt + φ ) {sinceat time=
t, x A sin ( ωt + φ ) }
The average value of P.E. of complete vibration is given by
T T T
1 1 1 mω2 A2 1
Uaverage
=
T0∫
= Udt ∫
T02
mω2 2
A sin2
( ω t + =
φ ) 4T 0 ∫ 2sin2 ( ωt + φ ) dt
=
4
mω2 A2

Because the average value of sine square or cosine square function for the complete cycle is 0.
2 2
1  dx  1 d
Now, KE at x is given
= by K.E. m 
=
2  dt 
m
2  dt
{
A sin ( ωt + φ ) }= 1
2
mω2 A2 cos2 ( ωt + φ )
8 . 8 | Simple Harmonic Motion and Elasticity

T
1 1 2 2
mω A cos2 ( ωt + φ ) dt
T ∫0 2
The average value of K.E. for complete cycle K.E.average
=

T
mω2 A2 mω2 A2 1
=
4T ∫ {1 + cos2 ( ωt + φ )} dt
=
4T
.=
T
4
mω2 A2
0

PLANCESS CONCEPTS

Thus average values of K.E. and P.E. of harmonic oscillator are equal to half of the total energy.
The total mechanical energy is constant but the kinetic energy and potential energy of the particle are
oscillating

E,U,K
E
U= 1 kx
2
E= 1 kA
2
2 2

Energy
k(t)+U(t)
u(t)

K= 1 k(x -x )
2 2
k(t)
2 t
O (1/2) T T
x Variation of energy w.r.t time
-A +A

Figure 8.6

Graph for Energy of SHM: Figure 8.6 shows the variation of total energy (E), Potential energy (U) and
kinetic energy (K) with Displacement (x).
Chinmay S Purandare (JEE 2012, AIR 698)

At a glance

S.No. Name of the equation Expression of the Equation Remarks


1. Displacement-time X varies between +A and –A
=x A cos ( ωt + φ )

2.
 dx  v = −Aω sin ( ωt + φ ) v varies between + Aω and − Aω
Velocity-time  V = 
 dt 
3.

Acceleration-time  a =
dv  a = −Aω2 cos ( ωt + φ ) a varies between + Aω2 and − Aω2

 dt 
4.
 1  1 1
Kinetic energy-time  K = mv 2  =K mA2 ω2 sin2 ( ωt + φ ) K varies between 0 and mA2 ω2
 2  2 2
5.
 1  1 1
Potential energy-time =
U mω2 x2  K
= mω2 A2 cos2 ( ωt + φ ) U varies between mA2 ω2 and 0
 2  2 2
6. E is constant
1
(
Total energy-time E= K + U ) E
=
2
mω2 A2
P hysics | 8.9

S.No. Name of the equation Expression of the Equation Remarks


7. Velocity-displacement
ω A2 − X 2
v= v = 0 at x = ± A and at x = 0
v =± Aω
8. Acceleration-displacement
a = −ω2 x a = 0 at x = 0
a = ±ω2 A at x =  A
9. Kinetic energy-displacement
1
(
K = mω2 A2 − X 2
2
) K = 0 at x =  A
1
K
= mω2 A2 at x = 0
2
10. Potential energy-displacement
1 1
U
= mω2 x2 U = 0 at x = 0=
U mω2 A2 at
2 x = ±A 2

11. Total energy-displacement E is constant


1
E
= mω2 A2
2

PLANCESS CONCEPTS

At mean position → K is the maximum and U is the minimum (it may be zero also, but it is not necessarily
zero). At extreme positions → K is zero and U is the maximum.
U(J) U(J) U(J)

x(m) x(m) x(m)


3 -3

Figure 8.7

B Rajiv Reddy (JEE 2012, AIR 11)

Illustration 5: The potential energy of a particle oscillating on x-axis is given as U = 20 + ( x − 2 )


2

Here, U is in joules and x in meters. Total mechanical energy of the particle is 36J. (JEE MAIN)
(a) State whether the motion of the particle is simple harmonic or not.
(b) Find the mean position.
(c) Find the maximum kinetic energy of the particle.

Sol: At the mean position the kinetic energy is the maximum and potential energy is the minimum. The sum of
kinetic energy and potential energy is constant throughout the SHM, equal to the total mechanical energy.
dU
(a) F =
− −2 ( x − 2 )
=
dx
8 . 1 0 | Simple Harmonic Motion and Elasticity

By assuming x − 2 =X , we have F = −2x , Since, F ∝ −X


The motion of the particle is simple harmonic
(b) The mean position of the particle is X = 0 or x − 2 =0 , which gives x = 2 m
(c) Maximum kinetic energy of the particle is, Kmax = 36 − 20 =16J
E − Umin =
Note Umin is 20J at mean position or at x = 2m.

Illustration 6: A block with mass M attached to a horizontal spring with force constant k is moving with simple
harmonic motion having amplitude A1 . At the instant when the block passes through its equilibrium position a
lump of putty with mass m is dropped vertically on the block from a very small height and sticks to it.
 (JEE ADVANCED)
(a) Find the new amplitude and period.
(b) Repeat part (a) for the case in which the putty is dropped on the block when it is at one end of its path.

Sol: Sticking of putty constitutes an inelastic collision. Kinetic energy at equilibrium position converts into potential
1 1
energy at extreme position, mv 2 = kA2 .
2 2
1
(a) Before the lump of putty is dropped the total mechanical energy of the block and spring is E1 = kA12 . Since,
2
the block is at the equilibrium position, U = 0 , and the energy is purely kinetic. Let v1 be the speed of the block at
k
the equilibrium position, we have v1 = A
M 1
During the process momentum of the system in horizontal direction is conserved. Let v 2 be the speed of the
M
combined mass, then (M + m ) v 2 =
Mv1 ; v 2 =
M+m 1
v

1 2 1
Now, let A2 be the amplitude afterwards. Then,=
E2 kA
=
2 2 2
(M + m) v 22

M
Substituting the proper values, we have A2 = A1
M+m
M+m
Note: E2 < E1 , as some energy is lost into heating up the block and putty. Further, T2 = 2π
k
(b) When the putty drops on the block, the block is instantaneously at rest. All the mechanical energy is stored in
the spring as potential energy. Again the momentum in horizontal direction is conserved during the process, but
now it is zero just before and after putty is dropped. So, in this case, adding the extra mass of the putty has no
effect on the mechanical energy, i.e.,
1 2 M+m
E=2 E=
1 kA and the amplitude is still A1 . Thus, A2 = A1 and T2 = 2π
2 1 k

6. ANGULAR SIMPLE HARMONIC MOTION


When a particle executes SHM on a curve path, then it is said to be angular SHM. E.g. - Simple pendulum. In this
case, to find out the time period, we find out restoring torque and hence angular acceleration.
i.e. τ = −kθ Where k is a constant ⇒ Iα = −kθ where I is moment of inertia  ... (i)
−k
⇒ α= θ  ... (ii)
I
Also, the equation of SHM for angular SHM, is α = −ω2 θ . Comparing (i) and (ii), we get ω , hence the time period.
P hysi cs | 8.11

Problem solving strategy


Step 1: Find the stable equilibrium position which usually is also known as the mean position. Net force or torque
on the particle in this position is zero. Potential energy is the minimum.
Step 2: Displace the particle from its mean position by a small displacement x (in case of a linear SHM) or θ (in
case of an angular SHM).
Step 3: Find net force or torque in this displaced position.
Step 4: Show that this force or torque has a tendency to bring the particle back to its mean position and magnitude
of force or torque is proportional to displacement, i.e.,
F ∝ −x or F = −kx …(i); τ ∝ −θ or τ = −kθ  …(ii)
This force or torque is also known as restoring force or restoring torque.
Step 5: Find linear acceleration by dividing Eq.(i) by mass m or angular acceleration by dividing Eq.(ii) by moment
of inertia I.
k k
Hence, a= − .x = −ω2 x or α = − θ = −ω2 θ
m I

a α 2π a α
Step 6: Finally, ω = or or = or
x θ T x θ

x θ
∴ T = 2π or 2π
a α

Energy Method: Repeat step 1 and step 2 as in method 1. Find the total mechanical energy (E) in the displaced
dE
position. Since, mechanical energy in SHM remains constant. = 0 By differentiating the energy equation with
dt
dx dθ dv dω
respect to time and substituting = v, = ω, = a , and = α we come to step 5. The remaining procedure
dt dt dt dt
is same.

Note: (i) E usually consists of following terms:


(a) Gravitational PE (b) Elastic PE (c) Electrostatic PE (d) Rotational KE and (e) Translational KE
(ii) For gravitational PE, choose the reference point (h=0) at mean position.

Illustration 7: Calculate the angular frequency of the system shown in


Fig 8.8. Friction is absent everywhere and the threads, spring and pulleys are
massless. Given, that m= A m=B m.  (JEE ADVANCED)

Sol: This problem can be solved either by restoring force method or by k


the energy method. The gain in kinetic energy is at the cost of decrease in
gravitational and/or elastic potential energy.
Let x0 be the extension in the spring in equilibrium. Then equilibrium of A Figure 8. 8
and B give T =kx0 + mgsin θ  … (i)
and 2T = mg  ... (ii)
Here, T is the tension in the string. Now, suppose A is further displaced by a distance x from its mean position and
x v
v be its speed at this moment. Then B lowers by and speed of B at this instant will be . Total energy of the
2 2
system in this position will be,
8 . 1 2 | Simple Harmonic Motion and Elasticity

2
1 1 1 V
k ( x + x0 ) + mA v 2 + mB   + mA ghA − mB ghB
2
E=
2 2 2 2
1 1 1 x
k ( x + x0 ) + mv 2 + mv 2 + mgx sin θ − mg
2
or
= E
2 2 8 2
1 5 x
k ( x + x0 ) + mv 2 + mgx sin θ − mg
2
or
= E
2 8 2
dE dx 5  dv   dx  mg  dx 
Since, E is constant, = 0 or 0 =k ( x + x0 ) + mv   + mg(sin θ)   −  
dt dt 4  dt   dt  2  dt 
dx dv mg
Substituting, = v ; = a and kx0 + mgsin θ = [From Eqs. (i) and (ii)]
dt dt 2
5
We get, ma = −kx Since, a ∝ −x
4
x 5m 2π 4k
Motion is simple harmonic, time period of which is, T =
2π 2π
= ∴ =
ω =
a 4k T 5m

7. SIMPLE PENDULUM
It is an example of angular simple harmonic motion. Let’s calculate its time period. Let us 
suppose that a bob of mass m is executing SHM (see Fig. 8.9). The length of the pendulum 
is  , which is the distant between the point of oscillation and the center of mass of the bob.
Torque acting on the bob about the point O.
=Γ mg  sin θ (And for small θ , sin θ  θ )
mg mg
Γ mgθ ⇒ Iα = −mgθ ⇒ α = −
⇒= θ
I
mg g Figure 8.9:
where α is angular acceleration =− θ ; α=− θ  ... (i) Oscillations of
2 
m
The equation of SHM is α = −ω2 θ  ... (ii) simple Pendulum

g g 2π g 
Comparing (i) and (ii), we get ω2 = ; ω = ⇒ = ; T = 2π
  T  g

PLANCESS CONCEPTS

The period is independent of the mass of the suspended particle.


Nitin Chandrol (JEE 2012, AIR 134)

Various scenarios: Time period


Pendulum in a lift descending with acceleration “a”, T = 2π
( g − a)

Pendulum in a lift ascending with acceleration “a”, T = 2π
( g + a)

Pendulum suspended in a train accelerated with ‘’a” uniformly in horizontal direction T = 2π
1

(a
2
+ g2 ) 2
P hysi cs | 8.13


Pendulum suspended in car taking turn with velocity v in a circular path of radius r, T = 2π 1
  2 2 2
  v  + g2 
  r  
  
Note: If the pendulum is suspended in vacuum, then the time period of the pendulum decreases.

Illustration 8: A simple pendulum consists of a small sphere of mass m suspended by a thread of length . The
sphere carries a positive charge q. The pendulum is placed in a uniform electric field of strength E directed vertically
upwards. With what period will pendulum oscillate if the electrostatic force acting on the sphere is less than the
gravitational force? (JEE MAIN)
Sol: The electrostatic force is acting opposite to the weight of the block. So the effective value of acceleration due
to gravity will be less than the actual value of g.
Fe=qE
The two forces acting on the bob are shown in Fig 8.10.
w − Fe mg − qE qE
geff in this case will be or geff= = g−
m m m

  w=mg
∴ T = 2π = 2π
geff qE
g− Figure 8.10
m

PLANCESS CONCEPTS

In case of a pendulum clock, time is lost if T increase and gained if T decreases. Time lost or gained in
time t is given by.

∆Τ
∆t = t e.g., if Τ= 2s, Τ=' 3s, then ∆Τ= 1s
Τ'
1
∴ Time lost by the clock in 1 hr. ∆t = × 3600 = 1200s
3
Second pendulum is a with its time period precisely 2 seconds
Vaibhav Gupta (JEE 2009, AIR 54)

Illustration 9: A simple pendulum of length l is suspended from the ceiling of a cart which is sliding without friction
on an inclined plane of inclination θ . What will be the time period of the pendulum?  (JEE MAIN)
Sol: The cart accelerates down the plane with acceleration a = g sinθ.
 
geff= g − a= g2 + 2g2 sin θ cos(90o + θ) + g2sin2 θ= gcos θ
→ →
Here, point of suspension has acceleration.
= a gsin θ (down the Plane). Further, g can be resolved into two
components g sin θ (along the plane) and g cos θ (perpendicular to plane)
→ → →
g − a = g cos θ
∴ g eff =
in os
(perpendicular to plane) sin
 gs gc
g
a=
 
∴ Τ = 2π = 2π
→ gcos θ
| g eff | Figure 8.11
8 . 1 4 | Simple Harmonic Motion and Elasticity


Note: If θ = 0° , Τ = 2π which is quiet obvious.
g

8. PHYSICAL PENDULUM
Any rigid body mounted so that it can swing in a vertical plane about some P
axis passing through it is called a physical pendulum (see Fig. 8.12).
A body of irregular shape is pivoted about a horizontal frictionless axis
through P and displaced from the equilibrium position by an angle θ . (The d d
equilibrium position is that in which the center of mass C of the body lies C
vertically below P).

The distance from the pivot to the center of mass is d. The moment of inertia
of the body about an axis through the pivot is I and the mass of the body is
M. The restoring torque about the point P, mg

τ = Mgdθ (if θ be very small, sin θ = θ)


Mgd
τ = Mgdθ; Iα = −Mgdθ; α=− θ ......(i) Figure 8.12: Rotation of
I
Physical Pendulum
Comparing with the equation of SHM

Mgd Mgd Mgd I


ω2 = ; ω= ; 2π / Τ = ; Τ = 2π
I I I Mgd

PLANCESS CONCEPTS

It may be necessary to use parallel axis theorem to find Moment of Inertia about the pivoted axis
=I IG + ml2
Yashwanth Sandupatla (JEE 2012, AIR 821)

9. TORSIONAL PENDULUM
In torsional pendulum, an extended body is suspended by a light thread
or a wire (see Fig. 8.13). The body is rotated through an angle about the
wire as the axis of rotation. The wire remains vertical during this motion
but a twist is produced in the wire. The lower end of the wire is rotated
through an angle with the body but the upper end remains fixed with the
support. Thus, a twist θ is produced. The twisted wire exerts a restoring
torque on the body to bring it back to its original position in which the 
twist θ in the wire is zero. This torque has a magnitude proportional to
the angle of twist which is equal to the angle rotated by the body. The
proportionality constant is called the torsional constant of the wire. Thus, Figure 8.13: Torsional pendulum
if the torsional constant of the wire is κ and the body is rotated through
an angle θ , the torque produced is Γ = −κθ . If I be the moment of inertia
Γ κ κ
of the body about the vertical axis, the angular acceleration is α= = − θ = −ω2 θ where ω =
Ι Ι Ι
2π Ι
Thus, the motion of the body is simple harmonic and the time period is Τ = = 2π
ω κ
P hysi cs | 8.15

Illustration 10: A ring of radius r is suspended from a point on its circumference.


Determine its angular frequency of small oscillations. (JEE ADVANCED)

Sol: This is an example of a physical pendulum. Find moment of inertia about point of
suspension and the distance of the point of suspension from the center of mass.

Ι
It is physical pendulum, the time period of which is, Τ = 2π
mgl

Here, I = moments of inertia of the ring about point of suspension


=mr 2 + mr 2 =2mr 2
Figure 8.14
and l = distance of point of suspension from centre of mass = r

2mr 2 2r 2π g
∴ Τ = 2π = 2π ; ∴ Angular frequency
= ω or ω
=
mgr g t 2r

Illustration 11: Find the period of small oscillations of a uniform rod with length l, pivoted at one O
end. (JEE MAIN)

Sol: This is an example of a physical pendulum. Find moment of inertia about point of suspension
and the distance of the point of suspension from the center of gravity.

I° 1 2 l
T=
2π Here, I° = ml and OG = G
mg(OG) 3 2

1 2 
 ml 
3 2l
∴ 2π 
T=  or T =

l 3g OG = l
(m)(g)   2
2
Figure 8.15

Illustration 12: A uniform disc of radius 5.0 cm and mass 200 g is fixed at its center to a metal wire, the other end
of which is fixed with a clamp. The hanging disc is rotated about the wire through angle and is released. If the disc
makes torsional oscillations with time period 0.20 s, find the torsional constant of the wire. (JEE MAIN)

Sol: This is an example of a torsional pendulum. Find moment of inertia about the axis passing
through the wire.
The Situation is shown in Fig 8.16. The moment of inertia of the disc about the wire is

( 0.200kg) (5.0 × 10−2 m)


2
2
mr
I
= = = 2.5 × 10−4 kg.m2
2 2
The time period is given by

Τ = 2π
I
; K=
4 π2 I
=
(
4 π2 2.5 × 10−4 kg − m2 )= 0.25
kg − m2
K Figure 8.16
Τ2 ( 0.20s )
2
s2
8 . 1 6 | Simple Harmonic Motion and Elasticity

10. SPRING - MASS SYSTEM


As shown in the Fig. 8.17 a mass m is attached to a massless spring. It is k
displaced from its mean position to a distance x. The restoring force is m x m
given by

F = −kx where
wherek isk,is
thethe
force constant.
force cons tant. Figure 8.17: Block of mass m
k attached to spring
=> ma = −kx ; a =−x ...(i)
m
=> a ∝ −x, ∴ Motion is SHM
k k m
=> ω2 = or ω= ; T = 2π
m m k

10.1 Series and Parallel Combination of Springs k1 k2 k3


m
10.1.1. Serial Combination of Springs
If springs are connected in series, having force constants k1 ,k 2 ,k 3
then the equivalent force Figure 8.18: Series combination of springs

1 1 1 1
constant is = + +
k eff k1 k 2 k 3

10.1.2 Parallel Combination of Springs


If springs are connected in parallel, then the effective force constant is given by k1 k2 k3 k4
k eff = k1 + k 2 + k 3 + ....................
The force constant of a spring is inversely proportional to its length. If a spring of spring
constant k is cut into two equal parts, the spring constant of each part becomes 2k. In m
general, if a spring of spring constant k is divided into n equal parts, the spring constant
Figure 8.19: Parallel
of each part is nk.
combination of springs

PLANCESS CONCEPTS

Time Period for various scenarios at glance

k1
k k1 k1
m
m
m
k1

m m m
T=
2π T=
2π T=

k k1 + k 2 k1 + k 2
P hysi cs | 8.17

PLANCESS CONCEPTS

k1 k1 k2
k2 m m

m m1m2
T=
2π T=

k1 + k 2 k(m1 + m2 )

k1 k2 k
m m m

m m(k1 + k 2 )
T=
2π T=

(k1 + k 2 ) k1k 2

k1
k1 k1 k
m m

m
k1

m(k1 + k 2 ) m
T=
2π T=

k1k 2 k

Figure 8.20

GV Abhinav (JEE 2012, AIR 329)

Illustration 13: For the arrangement shown in Fig 8.21, the spring is initially compressed by 3 cm. When the spring
is released the block collides with the wall and rebounds to compress the spring again. (JEE ADVANCED)
1
(a) If the coefficient of restitution is , find the maximum
2 m=1 kg
compression in the spring after collision. 4
k=10 N/m
Sol: Conserve energy to find the velocity of the block. Use
equation of restitution for collision of block with the wall.
(a) Velocity of the block just before
4 cm
collision,
1
2
1 2 1 2
mv 20 + =
2
kx kx
2 0
or =
ν0 (
k 2
x − x2
m 0
) Figure 8.21
8 . 1 8 | Simple Harmonic Motion and Elasticity

Here, x=
0 0.03m, x 0.01m,
= k= 10 4 Ν / m, m
= 1kg ∴ ν=
0 2 2m / s
1
After collision, ν = eν0 = 2 2 = 2m / s
2
Maximum compression in the spring

1 (2)
2
1 2 1 2 1 m
( 0.01)
2
kxm= kx + mν2 or xm= x + ν2=
2
+ m = 2.23cm
2 2 2 κ 10 4

Illustration 14: Figure 8.22 shows a system consisting of a massless pulley, a spring of force constant k and a block
of mass m. If the block is slightly displaced vertically down from its equilibrium position and released, find the
period of its vertical oscillation in case (a), (b) and (c).  (JEE ADVANCED)

Sol: The restoring force on the block will depend on the


elongation of the spring. For a small displacement of
block find the elongation in the spring.
k k
(a) In equilibrium, kx0 = mg ...(i) m
k
When further depressed by an amount x, net restoring
force (upwards) is,
m m
F= {
− k ( x + x0 ) − mg =}
−kx (as(askx
kx0 0= =
mg)
mg)

k x m (a) (b) (c)


a= − x ∴ T=
2π or T = 2π
m a k Figure 8.22
(b) In this case if the mass m moves down a distance x from its
kx
x
equilibrium position, then pulley will move down by . So, the extra 2
2
x kx
force in spring will be k . Now, as the pulley is massless, this force
2 2 m x0 kx
k
kx 4
is equal to extra 2 T or T = . This is also the restoring force of the m
4 x
m
mass. Hence,
kx
kx k x 4m 4
F=
− ; a=
− x or T=
2π or T=

4 4m a k
Figure 8.23 Figure 8.24
(c) In this situation if the mass m moves down a distance x from its
equilibrium position, the pulley will also move by x and so the spring
will stretch by 2x. Therefore, the spring force will be 2kx. The restoring
force on the block will be 4kx. 2kx 2kx
4k
Hence, F=
−4kx or a=
− ,x
m 4Kx

x m
∴ T=
2π or T =

a 4k

Figure 8.25

Illustration 15: A Spring mass system is hanging from the ceiling of an elevator in equilibrium. The elevator
suddenly starts accelerating upwards with acceleration ‘a’ Find: (a) The frequency and (b) The amplitude of the
resulting SHM.  (JEE MAIN)
P hysi cs | 8.19

Sol: The time period of spring mass system does not depend on g or acceleration of elevator.
m
(a) Frequency = 2π (Frequency is independent of g in spring) k
k
mg
(b) Extension in spring in equilibrium in initial = m
k
m ( g + a)
Extension in spring in equilibrium in accelerating lift = Figure 8.26
k
m(g + a) mg ma
∴ Amplitude
= − =
k k k

11. BODY DROPPED IN A TUNNEL ALONG EARTH DIAMETER


Assume earth to be a sphere of radius R and center O. Let a tunnel be dug along m
the diameter of the earth as shown in Fig. 8.27. If a body of mass m is dropped at
one end of the tunnel, the body executes SHM about the center of the earth. Let, at
any instant body in the tunnel is at a distance y from the center O of the earth. Only y R
the inner sphere of radius y will exert gravitational force F on the body as the body
is inside the earth. The force F serves as the restoring force that tends to bring the O
body to the equilibrium position O.

∴ Restoring force, F = −G
( 4 3 πy ρ) m
3

y2
Where ρ is the density of the earth. The negative sign is assigned because the force Figure 8.27: Body moving
is of attraction. along diameter of earth
F 4 
Acceleration of the body, a = =−  πGρ  y  … (i)
m  3 
Now the quantity (4 / 3)πGρ is constant so that: a ∝ − y
Thus the acceleration of the body is directly proportional to the displacement y and its direction is opposite to the
displacement. Therefore, the motion of the body is simple harmonic.

3 3π 3π … (ii)
∴ Time period, Τ = 2π = or Τ = 
4 πGρ Gρ Gρ

12. DAMPED AND UNDAMPED OSCILLATIONS


Damped oscillations is shown in the Fig 8.28 (a) given below. In such a case, during each oscillation, some energy
is lost. The amplitude of the oscillation will be reduced to zero as no compensating arrangement for the less is
provided. The only parameters that will remain unchanged are the frequency or time period. They will change only
according to the circuit parameters.
As shown in Fig 8.28 (b), undamped oscillations have constant have amplitude oscillations.
dx
Damping Force, Fd = −bV = −b where b is a constant giving the strength of damping. We can write
dt
Newton’s law, now including damping force along with the restoring force. For a spring-mass system, we have,
d2 x dx d2 x dx
m =−kx − b or m +b kx =
+= 0 ; x ae−bt/2m cos(ωt + φ) … (i) E.q. (i) describes sinusoidal motion
dt 2 dt dt 2 dt
whose amplitude (a) decreases exponentially with time. How fast the amplitude drops depends on the damping
2
1 k  b 
constants b and m. The frequency of this damped motion is given by:
= f − 
2π m  2m 
8 . 2 0 | Simple Harmonic Motion and Elasticity

+ Maximum value + Maximum value

- Maximum value - Maximum value

(a) Damped oscillation (b) Undamped or Sustained osclliation

Figure 8.28: Damped and undamped oscillation

1 k
If the frictional forces are absent, b=0 so that: f = (undamped oscillations)
2π m

13. FREE, FORCED AND RESONANT OSCILLATIONS


(a) Free oscillations are executed by an oscillating body that vibrates with its own frequency.
For example, when a simple pendulum is displaced from its mean position and then left free, it executes
free oscillations. The natural frequency of the simple pendulum depends upon its length and is given by;
1 g
f=
2π 
(b) Forced oscillations - When a body is maintained in a state of oscillations by an external periodic force of
frequency other than the natural frequency of the body, it executes forced oscillations.
The frequency of forced oscillations is equal to the frequency of the periodic A0
force. The external applied force on the body is called the driver and the body
F0
set into oscillations is called driven oscillator.
b
Examples. (a) When the stem of a vibrating tuning fork is held in hand, only a
feeble sound is heard. However, if the stem is pressed against a table top, the
sound becomes louder. It is because the tuning fork forces the table to vibrate
with fork’s frequency. Since the table has a large vibrating area than the tuning
fork, these forced oscillations produce a more intense sound.
Fig 8.29 shown the graph of forced oscillations as a function of ω . 
0
 f 
At ω = ωa , the value of A0 is  0  Figure 8.29: Forced oscillation
 bω 
 

PLANCESS CONCEPTS

Notice that amplitude of motion A0 is directly proportional to the amplitude of driving force.
GV Abhinav (JEE 2012, AIR 329)

Mathematical analysis: Most of the oscillations that occur in systems (e.g. machinery) are forced oscillations;
oscillations that are produced and sustained by an external force. The simplest driving force is one that oscillates
P hysi cs | 8.21

as a sine or a cosine. Suppose such an external force Fext is applied to an oscillator that moves along x axis such as
a block connected to a spring. We can represent the external forces as:= Fext F0 cos ωt Where F0 is the maximum
magnitude of the force and ω( = 2πf) is the angular frequency of the force. Then the equation of motion (with
damping) is ma = −kx − bV + F0 cos ωt . This equation can be written as

d2 x dx d2 x dx
m =−kx − b + F cos ωt or m +b + kx =F0 cos ωt  … (i)
dt 2 dt 0 dt 2 dt

F0 m
The solution of eq. (i)=
is x A0 cos(ωt + φ) Where =
A0  … (ii)
2

(ω )  bω 
2
2
− ω20 + 
m
and ω0 = k / m is the frequency of undamped (b=0) oscillator i.e., natural frequency.
(iii) Resonant oscillations: When a body is maintained in a state of oscillations by a periodic force having the same
frequency as the natural frequency of the body, the oscillations are called resonant oscillations. The phenomenon
of producing resonant oscillations is called resonance.
(b) The amplitude of motion (A0) depends on the difference between the applied frequency ( ω) and natural
frequency ( ω0 ) . The amplitude is the maximum when the frequency of the driving force equals the natural
frequency i.e., when ω = ω0 . It is because the denominator in eq. (ii) is the minimum when ω = ω0 . This condition is
called resonance. When the frequency of the driving force equals ω0 , the oscillator is said to be in resonance with
the driving force.

F0 m F0 / m F0
A0 = At resonance, ω = =
ω0 and A0 =
( bω / m ) bω
2 2

( )  bω 
ω2 − ω20 + 
m

PROBLEM-SOLVING TACTICS
d2 x
To verify SHM see whether force is directly proportional to y or see if + ω2 x = 0 in cases when the equation is
2
dt
directly given compare with general equation to find the time period and other required answers

FORMULAE SHEET

1. Simple Harmonic Motion (SHM):

F = –kxn
n is even - Motion of particle is not oscillatory
n is odd - Motion of particle is oscillatory.
If n = 1, F = -kx or F ∝ -x. The motion is simple harmonic.
x = 0 is called the mean position or the equilibrium position.

d2 x
Condition for SHM ∝ −x
dt2
8 . 2 2 | Simple Harmonic Motion and Elasticity

F k
Acceleration, a = = − x = −ω2 x -A O A X
m m
Figure 8.30
Displacement
=  x A cos ( ωt + φ ) (A is Amplitude)
  
phase angle

2π m
Time period of SHM T = = 2π
ω k

1 ω 1 k
Frequency ν of SHM ν= = =
T 2π 2π m

dx
Velocity of particle v = = −ωA sin ( ωt + φ )
dt
d2 x
Acceleration of particle a = = −ω2 A cos ( ωt + φ ) = −ω2 x
dt2
2. Energy in SHM:


1
Kinetic energy of particle = mω2 A2=
2
− x2
1
2
(k A2 − x2) ( )
1 2 1
Potential energy=
U kx
= mω2 x2
2 2
1 1 2
Total energy E= P.E + K.E = mω2 A2 = kA
2 2
E is constant throughout the SHM.


3. Simple pendulum: Time period T = 2π
geff
    
Here,  is length of simple pendulum and geff= g − a where g is acceleration due to gravity and a is
acceleration of the box or cabin etc. containing the simple pendulum.

m
4. Spring-block system: Time period T = 2π
k

I
5. Physical pendulum: Time period T = 2π
mg

Here I is the moment of inertia about axis of rotation and  is the distance of center of gravity from the point
of suspension.
6. Torsional Pendulum:

I
T = 2π
k

I is the moment of Inertia about axis passing through wire, k is torsional constant of wire.
P hysi cs | 8.23

7. Springs in series and parallel


k1 k1 k2 k1 k
m1 m2

k2
k1

Figure 8.31

1 1 1
Series combination = +
k k1 k 2
Parallel combination =
k k1 + k 2
8. For two blocks of masses m1 and m2 connected by a spring of constant k:

µ
Time period T = 2π
k
m 1m2
where µ = is reduced mass of the two-block system.
m 1 + m2

Solved Examples

JEE Main/Boards 2 + 12 ( 0.25 )


2
2 + 12d2
T=
2π 2π
= 153 s.
=
Example 1: What is the period of pendulum 12gd 12x9.8x0.25
formed by pivoting a meter stick so that it is O
free to rotate about a horizontal axis passing d Example 2: A particle executes SHM.
through 75 cm mark? C
(a) What fraction of total energy is kinetic and what
Sol: This is an example of a physical pendulum. fraction is potential when displacement is one half of
Find moment of inertia about point of suspension and the amplitude?
the distance of the point of suspension from the center (b) At what value of displacement are the kinetic and
of gravity. potential energies equal?
Let m be the mass and  be the length of the stick.
 = 100cm The distance of the point of suspension Sol: The sum of kinetic energy and potential energy
from center of gravity is d = 25cm is the total mechanical energy which is constant
throughout the SHM.
Moment of inertia about a horizontal axis through O is
1
m2 We know that Etotal
= mω2 A2
=I Ic + md
=2
+ md2 2
12
m2
+ md2
1
2
(
KE = mω2 A2 − X 2 ) and U
=
1
2
mω2 x2
I 12
T = 2π ; T = 2π A 1 3A2 KE 3
mgd mgd (a) When x = , KE = mω2 ⇒ =
2 2 4 Etotal 4
8 . 2 4 | Simple Harmonic Motion and Elasticity

return back. It is accelerated due to the spring force till


2 the spring acquires its natural length. The contact of
A 1 A PE 1
At x = U mω2
,= ⇒ = the block with the spring is now broken. At this instant
2 2 4 Etotal 4
it has regained its speed v (towards left) as the spring
is not stretched and no potential energy is stored. This
(b) Since, K = U ,
1
2
(1
mω2 A2 − x2 = mω2 x2 ;
2
) process takes half the period of oscillation, i.e. π m / k
. The block strikes the left wall after a time L / v and as
2 A the collision is elastic, it rebounds with the same speed
2x
= A2 or=x = 0.707 A
2 v. After a time L / v, it again reaches the spring and the
process is repeated. The block thus undergoes periodic
Example 3: Show that the period of oscillation of simple 2L
pendulum at depth h below earth’s surface is inversely motion with time period π m / k + .
v
proportional to R − h, where R is the radius of earth.
Find out the time period of a second pendulum at a Example 5: A particle is subjected to two simple
depth R / 2 from the earth’s surface? harmonic motions in the same direction having equal
amplitudes and equal frequency. If the resultant
Sol: As we go at a depth below the earth surface, the amplitude is equal to the amplitude of the individual
acceleration due to gravity decreases. The value of g motions, find the phase difference between the
inside the surface of earth is directly proportional to individual motions.
the radial distance from the center of the earth.
l/2 l/2
At earth’s surface the value of time period is given by

L 1 x
T=
2π or T ∝
g g T   T

 h m
g' g  1 − 
At a depth h below the surface,=
 R
Sol: The amplitude in case of combination of two or
T'
g 1 R R more SHMs in same direction and same frequency
∴ = = = ∴ T' = T is obtained by vector addition of the amplitudes of
T g'  h R −h R −h
1 −  individual SHMs. The angle of each of the individual
 R amplitude with the x-axis is equal to the phase constant
1 of the respective SHM.
or T' ∝ Hence Proved.
R −h Let the amplitudes of the individual motions be A each.
The resultant amplitude is also A. If the phase difference
R between the two motion is δ,
Further,
= TR /2 2= 2 2 s
R −R / 2
A= A2 + A2 + 2A.A.cos δ
Example 4: Describe the motion of the mass m shown
in figure. The walls and the block are elastic. A A 2 (1 + cos=
or = δ ) A cos δ 2

V δ 1
or cos = or δ = 2π / 3
k 2 2
m

Example 6: The figure shown below a block collides


L
in-elastically with the right block and sticks to it. Find
Sol: As the collision of the block with the wall is elastic, the amplitude of the resulting simple harmonic motion.
there will not be any loss in the kinetic energy and block
will execute periodic motion of constant time period. v
m m
The block reaches the spring with a speed ‘v’. It now k
compresses the spring. The block is decelerated due to
1 I
the spring force, comes to rest when mv 2 = kx2 and
2 2
P hysi cs | 8.25

Sol: Conserve momentum before and after collision.


The kinetic energy of blocks after collision is converted
= T 3.14
= x 
(
 4x10−2 (1.0 ) 
)
 0.2s
into elastic potential energy of the spring at the instant 
 10 

of maximum compression. Maximum compression is  
equal to amplitude of resulting SHM.
Example 8: If a tunnel is dug through the earth from
Assuming the collision to last for a small interval
one side to the other side along a diameter. Show that
only, we can apply the principle of conservation of
the motion of a particle dropped into the tunnel is
momentum. The common velocity after the collision
simple harmonic motion. Find the time period. Neglect
2
v 1 v 1 all the frictional forces and assume that the earth has a
is . The kinetic
2
= energy
2
=( 2m)   mv 2 . This is
uniform density.
2 4
also the total energy of vibration as the spring is un-
stretched at this moment. If the amplitude is A, the
1
total energy can also be written as kA2 .
2 M
1 1 m y
Thus, kA2
= = mv 2 , giving A v.
2 4 2k

Example 7: Find the time period of small oscillations m


in a horizontal plane performed by a ball of mass 40 g
fixed at the middle of a horizontally stretched string 1.0
m in length. The tension of the string is assumed to be G = 6.67 x10−11 Nm2kg−2 ; Density of earth
constant and equal to 10 N. = 5.51 x10 kgm 3 −3

Sol: Use the restoring force method to find the angular Sol: Use the restoring force method to find the angular
frequency. frequency.

Consider a ball of mass m placed at the middle of a Consider a tunnel dug along the diameter of the earth.
string of length l and tension T. The components of A particle of mass m is placed at a distance y from
tension T towards mean position is T cos θ . the center of the earth. There will be a gravitational
attraction of the earth experienced by this particle due
The force acting on the=
ball 2T cos θ to the mass of matter contained in a sphere of radius y.
Force acting on particle at distance y from center
2Tx
∴ ma =

((l / 4) + x )
2 2
F=
GM
R3
.y
x
= T F and
= cos θ GMm
((l / 4) + x )
2 2 ⇒ ma =

R3
.y

4 3
G× d× πR
As x is small, x2 can be neglected in the denominator. GM 3 4 πG
⇒ a =− .y = − y =− .d. y
R 3
R 3 3
2Tx  4T  2
∴a =− =−   x =− ω x
m (l / 2 )  ml  As the force is directly proportional to the displacement
and is directed towards the mean position, the motion
The acceleration is directly proportional to negative
is simple harmonic.
displacement x and is directed towards the mean
position. Hence the motion is SHM 4  3 
⇒ ω2 = πdG. and T = 2π  
2π 2π  ml  3  4 πdG 
T= = =π  
ω ( 4T / ml)  T   3π   3x3.14 
= =
   −11 
Substituting the given values, we get  dG  3
 5.51x10 x6.67x10 
= 5062s
= 84.4min
8 . 2 6 | Simple Harmonic Motion and Elasticity

Example 9: The pulley shown in figure below has a 4k


or a = − ω2 x, where ω2 =
moment of inertia I about its axis and mass m. find the  I 
time period of vertical oscillation of its center of mass.  2 + m
 r 
The spring has spring constant k and the string does
not slip over the pulley. Thus, the center of mass of the pulley executes a simple
harmonic motion with time period

 I 
2π  + m  / ( 4k ) .
T=
k 2
r 

Example 10: Two light springs of force constant k1


I and k 2 and a block of mass m are in one line AB on
a smooth horizontal table such that one end of each
spring is fixed on rigid supports and the other end is
Sol: For a small displacement of the pulley find the free as shown in figure.
extension in the spring. Use the energy method to find
the angular frequency.
A 60m B
Let us first find the equilibrium position. For rotational m
equilibrium of the pulley, the tensions in the two strings k1
C D
k2
V
should be equal. Only then the torque on the pulley
will be zero. Let this tension be T. The extension of the
spring will be y = T / k, as the tension in the spring will
The distance CD between the free ends of the springs
be the same as the tension in the string. For translational
60 cm. If the block moves along AB with a velocity
equilibrium of the pulley,
120 cm/sec in between the springs, calculate the period
mg of oscillation of the block
=2 T mg =
or, 2ky mg
= or, y .
2k
mg
= (k1 1.8N= / m, k 2 3.2N
= / m,m 200 gm)
The spring is extended by a distance when the
2k If initially block is mid-way of CD.
pulley is in equilibrium.
Sol: As there are no dissipative forces the motion of the
Now suppose, the center of the pulley goes down
block is oscillatory with constant time period. Add the
further by a distance x. The total increase in the length
time of motion of different segments to get the time
of the string plus the spring is 2x (x on the left of the
period.
pulley and x on the right). As the string has a constant
length, the extension of the spring is 2x. The energy of If initially block is mid-way of CD their the time period
the system is T is equal to sum of time to travel 30 cm to right, time
2 in contact with spring k2, time to travel 60 cm to left,
1 1 1  mg  time in contact with spring k1 and time to travel 30 cm
U = Iω2 + mv 2 − mgx + k  + 2x 
2 2 2  2k  to right.
1 I  2 m2g2 30 1   m   60 1   m   30
=  2 + m v + + 2kx2 . +  2π
2r  8k ∴ T=    + +  2π    +
120 2   k 2   120 2   k1   120

dU
As the system is conservative, = 0,
dt  0.2   0.2 
= 0.25 + π   + 0.5 + π   + 0.25
 3.2   1.8 
 I  dv
giving 0 =+
 2 m v + 4kxv
r  dt = 0.25 + π / 4 + 0.5 + π / 3 + 0.25 = 2.83 s.

dv 4kx
Or, =− Example 11: The moment of inertia of the disc used
dt  I 
 2 + m in torsional pendulum about the suspension wire is
r  0.2 kg − m2 . It oscillates with a period of 2s. Another
disc is played over the first one and the time period of
P hysi cs | 8.27

the system becomes 2.5 s. Fine the moment of inertia 1


= mv 2 mgl ( cos θ − cos θ0 )…(i)
. ...............(i)
of the second disc about the wire. 2
As the bob moves in a circular path, the force towards
the center should be equal to mv2 / l. Thus,
T − mgcos θ =m v 2 / l.

Using (i),
T − mgcos
= θ 2mg ( cos θ − cos θ0 )
Sol: As another disc is placed on the first disc moment
or
= T 3mgcos θ − 2mgcos θ0 .
of inertia about the axis passing through the wire
increases and thus time period increases. Now cos θ is maximum at θ =0 and decreases as | θ |
(
increases for | θ | < 90o .)
Let the torsional constant of the wire be k. The moment Thus, the tension is maximum when θ =0, i.e., at the
of inertia of the first disc about the wire is 0.2 kg − m2 . mean position and is minimum when θ = ± θ0 , i.e., at
hence, the time period is extreme positions.

I 0.2kg − m2
2s =
2π 2π
= ... (i) …(i)
K k  JEE Advanced/Boards
When the second disc having moment of inertia I1
Example 1: A simple pendulum is suspended from the
about. The wire is added, the time period is
ceiling of a car accelerating uniformly on a horizontal
0.2kg − m2 + I1 road. If the acceleration is a0 and the length of the
2.5s= 2π ...(ii) …(ii) pendulum is l, find the time period of small oscillations
0.2kg − m2
 about the mean position.
2
6.25 0.2kgm − m + I1 Sol: The car accelerates with acceleration a. In the
From (i) and (ii), = .
4 0.2kg − m2 reference frame of car the effective value of acceleration
due to gravity is
This gives
= I1 0.11kg − m2 .
 
geff = g − a = g2 + a2
Example 12: A simple pendulum having a bob of
mass m undergoes small oscillations with amplitude
θ0 . Find the tension in the string as a function of the
angle made by the string with the vertical. When is this l
tension maximum, and when is it minimum? T 
ma0 l
a0
0 +
mg x


T We shall work in the car frame. As it is accelerated with


respect to the road, we shall have to apply a pseudo
force m a0 on the bob of mass m.
For mean position, the acceleration of the bob with
mg respect to the car should be zero. If θ be the angle
made by the string with the vertical, then tension,
Sol: The forces acting on the bob are tension due to weight and the pseudo force will add to zero in this
string and weight mg. The bob moves in a circular position.
path. The acceleration of the bob has both radial and
Suppose at some instant during oscillation, the
tangential component.
string is further deflected by an angle a so that the
Suppose the speed of the bob at angle θ is υ. Using displacement of the bob is x. Taking the components
conservation of energy between the extreme position perpendicular to the string, component of T = 0,
and the position with angle θ,
8 . 2 8 | Simple Harmonic Motion and Elasticity

mg mgsin ( α + θ ) and component of


component of= Example 2: A long uniform rod of length L and mass
M is free to rotate in a vertical plane about a horizontal
ma0 = − ma0 cos ( α + θ ) . Thus, the resultant component axis through its one end ‘O’. A spring of force constant
=F m gsin ( α + θ ) − a0 cos ( α + θ )  . k is connected vertically between one end of the rod
and ground. When the rod is in equilibrium it is parallel
Expanding the sine and cosine and putting cos α ≈ l, to the ground.
sin a ≈ x/l, we get,
 x
F m gsin θ − a0 cos θ + ( gcos θ + a0 sin θ ) 
= ...(i)
L
 l
A
At x = 0, the force F on the bob should be zero, as this
is the mean position. Thus by (i), O

=0 m gsin θ − a0 cos θ   …(ii)

a0
Giving tan θ = (a) What is the period of small oscillation that result
g
when the rod is rotated slightly and released?
a0
Thus, sin θ =  …(iii) (b) What will be the maximum speed of the displaced
a20 + g2 end of the rod, if the amplitude of motion is θ0 ?

g Sol: The rod executes angular SHM. Use restoring


cos θ = …(iv)

a20 +g 2 torque method to find angular frequency of SHM.
(a) Restoring torque about ‘O’ due to elastic force of
Putting (ii), (iii) and (iv) in
the spring

x g2 + a20 τ = −FL = −kyL (F = ky)


(i), F m g2 + a20
= or, F= mω2 x, where ω2 = .
l l
τ = −kL2 θ (as y = Lθ)
This is an equation of simple harmonic motion with
1 2 d2 θ
2π l τ = Iα = ML
time period=
t = 2x . 3 dt2
(g )
ω 1/ 4
2
+ a20
1 2 d2 θ d2 θ 3k
ML =−kL2 θ ; =− θ
As easy working rule may be found out as follows. In 3 dt 2
dt 2 M
the mean position, the tension, the weight and the
pseudo force balance. From figure, the tension is 3k M
ω= ⇒ T = 2π
M 3k

(b) In angular SHM, maximum angular velocity
T
ma0
 dθ  3k  dθ 
  = θ0 , ω = θ0 , vυ==r  
mg  dt max M  dt 
 dθ  3k
=T (ma0 ) + (mg)
2 2
So, v max = L   = Lθ0
 dt max M
T
or, = a20 + g2 .
m Example 3: A block with mass of 2 kg hangs without
vibrating at the end of a spring of spring constant
This plays the role of effective ‘g’. Thus the time period 500 N/m, which is attached to the ceiling of an elevator.
g
The elevator is moving upwards with acceleration . At
I l time t=0, the acceleration suddenly ceases. 3
is t =
2π 2π
= .
T /m g2 + a2 
1/ 4
(a) What is the angular frequency of oscillation of the
 0
block after the acceleration ceases?
P hysi cs | 8.29

(b) By what amount is the spring stretched during the


(ii) At time t = 0, block is at x = −A. Therefore,
time when the elevator is accelerating?
substituting x = −A and t = 0 in equation,
(c) What is the amplitude of oscillation and initial phase

angle observed by a rider in the elevator? Take the = x A sin(ω t + φ) We get initial phase φ =
upward direction to be positive. Take g = 10.0m / s2 . 2

Example 4: A solid sphere (radius = R) rolls without


kx
slipping in a cylindrical through (radius = 5R). Find the
a=
g
time period of small oscillations.
3
mg

Sol: The angular frequency of the spring block r x x-axis


system in vertical oscillations does not depend on o

the acceleration due to gravity or the acceleration of
c
the elevator. The equilibrium position depends on the
acceleration due to gravity and the elevator. When the
acceleration of the elevator ceases the block moves to
the new equilibrium position. Sol: The sphere executes pure rolling in the cylinder.
The mean position is at the lowest point in the cylinder.
(a) Angular frequency kx0 Find the acceleration for small displacement from the
mean position and compare with standard equation of
k 500 SHM to find angular frequency.
=ω or ω
=
m 2
or ω =15.81 rad / s mg 5R
(b) Equation of motion of the block (while elevator is
accelerating) is, R

g
kx − mg =ma =m
3 For pure rolling to take place, v = Rw

x
∴=
4mg
=
( 4 )( 2)(10=) 0.053m ω' =Angular velocity of COM of sphere C about O
3k (3)(500 ) v Rω ω
= = =
or x = 5.3cm 4R 4R 4 ’
O
5R

(c) (i) In equilibrium when the elevator has zero dω' 1 dω α 
∴ = or α' =
acceleration, the equation of motion is dt 4 dt 4
c

kx0 = mg or x0
=
mg
=
( 2)(10
=
) 0.04m
a
α = for pure rolling;
v
k 500 R
= 4cm gsin θ 5gsin θ
Where,
= a =
I + mR 2 7
x= +A
x0
2 5gsin θ
As, I = mR 2 ∴ α' =
5 28R
Mean position For small θ, sin θ = θ, being restoring in nature,

5g θ 28R
α' =− θ ∴ T =2π =2π
x= -A 28R α ' 5g

∴ Amplitude A=
x − x0
Example 5: Consider the earth as a uniform sphere of
= 5.3 − 4.0 mass M and radius R. Imagine a straight smooth tunnel
made through the earth which connects any two points
= 1.3 cm
on its surface. Show that the motion of a particle of
8 . 3 0 | Simple Harmonic Motion and Elasticity

mass in along this tunnel under the action of gravitation (a) Calculate the frequency of oscillation of ball B.
would be simple harmonic. Hence, determine the time
(b) Find the speed of the ball A when A and B are at the
that a particle would take to go from one end to the
two ends of diameter PQ
other through the tunnel.
(c) What is the total energy of the system
Sol: Use the restoring force method to find the angular
frequency. Sol: Here the two balls connected by the springs are
Suppose at some instant the particle is at radial free to oscillate along the length of the springs, so the
distance r from center of earth O. Since, the particle time period will depend on the reduced mass of the
is constrained to move along the tunnel, we define two-ball system.
its position as distance x from C. Hence, equation of (a) Restoring force on A or B = k∆x + k∆x= 2k∆x.
motion of the particle is, max = Fx
Where ∆x is compression in the spring at one end?
The gravitational force on mass m at distance r is, Effective force constant = 2k
GMmr 1 2k
F= (Towards O) Frequency v =
R3 2π µ
GMmr  x 
Therefore, Fx = −Fsin θ = −   Where µ is reduced mass of system.
R3  r 
Since Fx ∝ −x , motion is simple harmonic in nature.
mm m
Further, reducedmass.
= µ =
m+m 2
GMm GM
max =
− .x or ax =
− .x
R 3
R3 1 2k 1 0.1 1
=v = = s
2π m / 2 3.14 0.1 3.14
∴ Time period of oscillation is,
(b) P and Q are equilibrium position. Balls A and B at
x R3 P and Q have only kinetic energy and it is equal the
T=
2π 2π
=
ax GM potential energy at extreme positions.

The time taken by particle to go from one end to the Potential energy at extreme position
T 1 1
other is = k(2∆x)2 + k(2∆x)2 = 4k( ∆x)2
2 2 2
T R3
∴ t= = π π
2 GM Where ∆x =Rx
6
π2kR 2 (3.14)2 x0.1x(0.06)2
Example 6: Two identical balls A and B, each of mass ⇒ P.E.= = ≈ 3.94x10 −4 J
36 36
0.1 kg are attached to two identical massless springs.
The spring mass system is constrained to move inside a When the balls A and B are at points P and Q respectively.
rigid smooth pipe bent in the form of a circle as shown
in figure. The pipe is fixed in a horizontal plane. The KE(A) + KE(B) = PE. ; 2KE(A) = P.E.
centers of the balls can move in a circle of radius 0.06 m.
Each spring has a natural length 0.06 π m and spring 1
2x mv 2 = 3.94 x10−4
constant 0.1 N/m. Initially both the balls are displaced 2
by angle π /6 radian with respect to the diameter PQ of 1
the circle and released from rest.  3.94  2
⇒v 
= = x10−2 6.28x10
= −2
0.0628ms−1
 0.1 

A B (c) Total potential and kinetic energy of the system


is equal to total potential energy at the extreme
/6 /6 position=3.94x10-4J.
P Q
O
P hysi cs | 8.31

JEE Main/Boards

Exercise 1
(b) Potential energy of the oscillating mass.
Q.1 A simple harmonic motion is represented by
y(t)=10 sin (20t+0.5). Write down its amplitude, angular Q.8 A trolley of mass 3.0 kg is connected to two
frequency, time period and initial phase, if displacement identical springs each of force constant 600 Nm−1
is measured in meters and time in seconds. as shown in figure. If the trolley is displaced from its
equilibrium position by 5.0 cm and released, what is the
Q.2 A particle executing SHM along a straight line has total energy stored?
a velocity of 4 ms-1, when at a distance of 3 m from its
mean position and 3 ms−1 , when at a distance of 4 m 3.0 kg
from it. Find the time it takes to travel 2.5 m from the
-1 -1
600 Nm 600 Nm
positive extremity of its oscillation.

Q.3 A simple harmonic oscillation is represented by the


equation. Q.9 A pendulum clock normally shows correct time. On
Y=0.4sin (440t+0.61) an extremely cold day, its length decreases by 0.2%.
Compute the error in time per day.
Here y and t are in m and s respectively. What are
the values of (i) amplitude (ii) angular frequency (iii)
frequency of oscillation (iv) time period of oscillation Q.10 Two particles execute SHM of same amplitude
and (v) initial phase? and frequency on parallel lines. They pass one another
when moving in opposite directions and at that time
their displacement is one third their amplitude. What is
Q.4 A particle executing SHM of amplitude 25 cm and
the phase difference between them?
time period 3 s. What is the minimum time required for
the particle to move between two points 12.5 cm on
either side of the mean position? Q.11 What is the frequency of a second pendulum in an
elevator moving up with an accelerating of g/2?
Q.5 A particle executes SHM of amplitude a. At what
distance from the mean position is its K.E. equal to its Q.12 Explain periodic motion and oscillatory motion
P.E? with illustration.

Q.6 An 8 kg body performs SHM of amplitude a. At Q.13 What is a simple pendulum? Find an expression for
what distance from the mean position is its K.E. equal the time period and frequency of a simple pendulum.
to its P.E?
Q.14 Explain the oscillations of a loaded spring and find
−1
Q.7 A spring of force constant 1200 Nm is mounted the relations for the time period and frequency in case
on a horizontal table as shown in figure. A mass of of (i) horizontal spring (ii) vertical spring
3.0 kg is attached to the free end of the spring. Pulled
sideways to a distance of 2cm and released, what is Q.15 What is a spring factor? Find its value in case of
two springs connected in (i) series and (ii) parallel.
m
Q.16 Explain phase and phase difference, angular
frequency, displacement in periodic motion with
illustrations.
(a) The speed of the mass when the spring is compressed
by 1.0 cm? Q.17 Explain displacement, velocity, acceleration and
time period in SHMs. Find the relation between them.
8 . 3 2 | Simple Harmonic Motion and Elasticity

Q.18 From the figure (a) and (b). Obtain the equation Q.26 A system of springs with their spring constants are
of simple harmonic motion of the y-projection of the as shown in figure . What is the frequency of oscillations
radius vector of the revolving particle P in each case. of the mass m?
y

P(t=0) P(t=0)
T=3s
o
k1
o
60 90
x (a) x (b)
2cm 3cm
T=1min
k2

Q.19 Two particles execute SHM of the same amplitude


and frequency along does parallel lines. They pass k3 k4
each other moving in opposite directions, each time
their displacement in half their amplitude. What is their
phase difference? m

Q.20 A body oscillates with SHM according to the


equation, X=6 cos (3π t + π / 3) metres. What is (a) Exercise 2
amplitude and (b) the velocity at t=2s.
Single Correct Choice Type
Q.21 A bob of simple pendulum executes SHM of
period 20 s. Its velocity is 5 ms-1, two seconds after it Q.1 A simple harmonic motion having an amplitude
has passed through its mean position. Determine the A and time period T is represented by the equation:
amplitude of SHM. y= 5sin π (t + 4)m
Then the values of A (in m) and T (in s) are:
Q.22 A particle is moving in a straight line with SHM Its
velocity has values 3 ms-1 and 2 ms-1 when its distance (A) A = 5; T = 2 (B) A = 10; T = 1
from the mean positions are 1 m and 2 m respectively. (C) A = 5; T = 1 (D) A = 10; T = 2
Find the period of its motion and length of its path.

Q.2 The maximum acceleration of a particle in SHM is


Q.23 A particle executes SHM with an amplitude 4 cm. made two times keeping the maximum speed to be
Locate the position of point where its speed is half its constant. It is possible when
speed is half its maximum speed. At what displacement
is potential energy equal to kinetic energy? (A) Amplitude of oscillation is doubled while frequency
remains constant
Q.24 A block whose mass is 1 kg is fastened to a spring. (B) Amplitude is doubled while frequency is halved
The spring has a spring constant 50 N m-1. The block (C) Frequency is doubled while amplitude is halved
is pulled to a distance x=10 cm from its equilibrium
position at x=0 on a frictionless surface at t=0. Calculate (D) Frequency is doubled while amplitude remains
the kinetic, potential and total energies of the block constant
when it is 5 cm away from the mean position.
Q.3 A stone is swinging in a horizontal circle 0.8 m in
Q.25 Two point masses of 3.0 kg and 1.0 kg are diameter at 30 rev/min. A distant horizontal light beam
attached to opposite ends of a horizontal spring whose causes a shadow of the stone to be formed on a nearly
spring constant in 300 Nm-1 as shown in figure. Find the vertical wall. The amplitude and period of the simple
natural frequency of vibration of the system. harmonic motion for the shadow of the stone are
(A) 0.4 m, 4 s (B) 0.2 m, 2 s
1kg 3kg (C) 0.4 m, 2 s (D) 0.8 m, 2 s
P hysi cs | 8.33

Q.4 A small mass executes linear SHM about O with


1 mv 2 mv 2
amplitude a and period T. Its displacement from O at (C) (D) 2
time T/8 after passing through O is: 2 k k

(A) a/8 (B) a/2 2 (C) a/2 (D) a/ 2 Q.12 In the above question, the find amplitude of
oscillation of the block in the reference frame of point
Q.5 The displacement of a body executing SHM is given A of the spring.
by x A sin(2πt + π / 3). The first time from t=0 when
=
1 mv 2 1 mv 2
the velocity is maximum is (A) (B)
4 k 2 k
(A) 0.33 s (B) 0.16 s (C) 0.25 s (D) 0.5 s
mv 2 mv 2
(C) (D) 2
Q.6 A particle executes SHM of period 1.2 s. and k k
amplitude 8 cm. Find the time it takes to travel 3cm
from the positive extremely of its oscillation. Q.13 For a particle acceleration is defined as

(A) 0.28 s (B) 0.32 s (C) 0.17 s (D) 0.42 s  −5xi 
a= for x ≠ 0 and a =0 for x =0.
x
Q.7 A particle moves along the x-axis according to If the particle is initially at rest (a, 0) what is period of
: x = A[I + sin ωt] . What distance does it travel between? motion of the particle.
t = 0 andt = 2.5π / ω ?
(A) 4A (B) 6A (C) 5A (D) None (A) 4 2a / 5 sec. (B) 8 2a / 5 sec.

(C) 2 2a / 5 sec. (D) Cannot be determined


Q.8 Find the ratio of time periods of two identical
springs if they are first joined in series & then in parallel
Q.14 A mass m, which is attached to a spring with
& a mass m is suspended from them:
spring constant k, oscillates on a horizontal table, with
(A) 4 (B) 2 (C) 1 (D) 3 amplitude A. At an instant when the spring is stretched
by 3A / 2 , a second mass m is dropped vertically onto
Q.9 The amplitude of the vibrating particle due to
superposition of two SHMs, the original mass and immediately sticks to it. What is
the amplitude of the resulting motion?
 π
y1= sin  ωt +  and y 2= sin ωtis: 3 7
 3 (A) A (B) A
2 8
(A) 1 (B) 2 (C) 3 (D) 2
13 2
(C) A (D) A
16 3
Q.10 Two simple harmonic motions y1=A sin ωt
are superimposed on a particle of mass m. The total
mechanical energy of the particle is: Q.15 A particle is executing SHM of amplitude A, about
the mean position x=0. Which of the following cannot
1
(A) mω2 A2 (B) mω2 A2 be a possible phase difference between the positions of
2
1 the particle at x=+ A/2 and x = −A / 2
(C) mω2 A2 (D) Zero
4 (A) 75° (B) 165° (C) 135° (D) 195°

Q.11 A block of mass ‘m’ is attached to a spring in


natural length of spring constant ‘k’. The other end A
of the spring is moved with a constant velocity v away
from the block. Find the maximum extension in the
spring.

1 mv 2 mv 2
(A) (B)
4 k k
8 . 3 4 | Simple Harmonic Motion and Elasticity

Previous Years’ Questions amplitude of oscillation is a then its time period T is


 (1998)

Q.1 A particle executes simple harmonic motion with (A) Proportional to 1 / a


a frequency ƒ. The frequency with which its kinetic (B) Independent of a
energy oscillates is (1987)
(C) Proportional to a
(A) ƒ/2 (B) ƒ (C) 2 ƒ (D) 4 ƒ
(D) Proportional to a 3/2

Q. 2 Two bodies M and N of equal masses are suspended


Q.6 A spring of force constant k is cut into two pieces
from two separate massless springs of spring constants
k1 and k 2 respectively. If the two bodies oscillate such that one piece is double the length of the other
the long piece will have a force constant of  (1999)
vertically such that their maximum velocities are equal,
the ratio of the one amplitude of vibration of M to that (A) 2/3 k (B) 3/2 k (C) 3k (D) 6k
of N is (1988)

(A) k1 / k 2 (B) k 2 / k1 Q.7 A particle free to move along the x – axis has porential
energy by U(x)= k[1 − exp( −x2 )] for − ∞ ≤ x ≤ + ∞ Where
(C) k 2 / k1 (D) k1 / k 2 k is a positive constant of appropriate dimensions. Then
 (1999)

Q.3 A highly rigid cubical block A of small mass M and (A) At points away from the origin, the particle is in
side L is fixed rigidly on to another cubical block B of unstable equilibrium
the same dimensions and of low modulus of rigidity (B) For any finite non-zero value of x, there is a force
η such that the lower face of A completely covers
directed away from the origin
the upper face of B. The lower face of B is rigidly held
on a horizontal surface. A small force F is applied (C) If its total mechanical energy is k/2, it has its
perpendicular to one of the side faces of A. After the minimum kinetic energy at the origin
force is withdrawn. Block A executes small oscillations. (D) For small displacements from x=0, the motion is
The time period of which is given by (1992) simple harmonic


(A) 2π MηL (B) 2π Q.8 The period of oscillation of simple pendulum of
L
length L suspended from the roof of the vehicle which
moves without friction, down an inclined plane of
ML M inclination α, is given by (2000)
(C) 2π (D) 2π
η ηL
L L
(A) 2π (B) 2π
gcos α gsin α
Q.4 One end of a long metallic wire of length L is tied
to the ceiling. The other end is tied to a massless spring
of spring constant k. A mass m hangs freely from the L L
(C) 2π (D) 2π
free end of the spring. The area of cross-section and the g gtan α
Young’s modulus of the wire are A and Y respectively.
If the mass is slightly pulled down and released, it will
Q.9 A particle executes simple harmonic motion
oscillate with a time period T equal to  (1993)
between x= -A and x= + A. The time taken for it to go
from O to A/2 is T1 and to go from A/2 to A is T2, then
m(YA + kL
(A) 2π (m / k)1/2 (B) 2π  (2001)
YAk

(C) 2π [(mYA / kL)1/2 (D) 2π [(mL / YA)1/2 (A) T1 < T2 (B) T1 > T2

(C) T1 = T2 (D) T1 = 2T2


Q.5 A particle of mass m is executing oscillation
about the origin on the x-axis. Its potential energy
is U(x) = k x , Where k is a positive constant. If the
3
P hysi cs | 8.35

Q.10 For a particle executing SHM the displacement


π2 3 2
x is given by x=A cos ωt . Identify the graph which (C) cms−2 (D) − π cms−2
represents the variation of potential energy (PE) as a 32 32
function of time t and displacement. (2003)
Q.14 A uniform rod of length L and mass M is pivoted
PE
l ll
PE at the center. Its two ends are attached to two springs
lll of equal spring constants k. The spring are fixed to rigid
supports as shown in the Fig, and rod is free to oscillate
lV
t x in the horizontal plane. The rod is gently pushed
through a small angle θ in one direction and released.
(A) I, III (B) II, IV (C) II, III (D) I, IV The frequency of oscillation is  (2009)

Q.11 A block P of mass m is placed on a horizontal


frictionless plane. A second block of same mass m is
placed on it and is connected to a spring of spring
constant k, the two blocks are pulled by a distance
A. Block Q oscillates without slipping. What is the
maximum value of frictional force between the two
blocks? (2004)

1 2k 1 k
(A) (B)
k 2π M 2π M
Q a
1 6k 1 24k
P (C) (D)
2π M 2π M

(A) kA (B) kA
Q.15 The mass M shown in the figure oscillates in simple
(C) µs mg (D) Zero
harmonic motion with amplitude A. The amplitude of
the point P is (2009)
Q.12 A simple pendulum has time period T1 . The point
of suspension is now moved upward according to the
k1 P k2
relation ,(k 1m / s2 ) where y is the vertical
2 m
= y kt=
displacement.
k1 A k2A
The
The time
time period
period now
now beomes T22..
beomes T (A) (B)
k2 k1
T12
The ratio of is (Take g = 10m / s2 )  (2005) k1 A k2A
T2 2
(C) (D)
k1 + k 2 k1 + k 2
(A) 6/5 (B) 5/6 (C) 1 (D) 4/5
Q.16 A wooden block performs SHM on a frictionless
Q.13 The x-t graph of a particle undergoing simple surface with frequency v 0 . The block carries a charge

harmonic motion is shown below. The acceleration of +Q on its surface. If now a uniform electric field E is
the particle at t=4/3 s is  (2009) switched-on as shown, then the SHM of the block will
x be (2011)
(cm)

1 E

0 +Q
4 8 12 t(s)
-1

(A) Of the same frequency and with shifted mean


3 2 −π2
(A) π cms−2 (B) cms−2 position
32 32
8 . 3 6 | Simple Harmonic Motion and Elasticity

(B) Of the same frequency and with the same mean Q.19 The amplitude of a damped oscillator decreases
position to0.9 times its original magnitude is 5s. In another 10s it
will decrease to α times its original magnitude, where
(C) Of changed frequency and with shifted mean
α equals.  (2013)
position
(A) 0.81 (B) 0.729 (C) 0.6 (D) 0.7
(D) Of changed frequency and with the same mean
position
Q.20 For a simple pendulum, a graph is plotted
between its kinetic energy (KE) and potential energy
Q.17 A point mass is subjected to two simultaneous
(PE) against its displacement d. Which one of the
sinusoidal 3π displacements in x-direction
2A, following represents these correctly?
4
(Graphs are schematic and not drawn to scale) (2015)
 2π  .
x=
1 (t) A sin ω t and x=
2 (t) A sin  ωt + 
 3 
Adding a third sinusoidal displacement
(t) B sin(ωt + φ) brings the mass to a complete rest.
x3=
The values of B and φ are  (2011)

6π 4π
(A) A, (B) A,
3 3

5π π
(C) 3A, (D) A,
6 3

Q.18 If a simple pendulum has significant amplitude (up


to a factor of 1/e of original) only in the period between Q.21 A particle performs simple harmonic motion with
t = Os to t = τs , then τ may be called the average amplitude A. Its speed is trebled at the instant that it is
life of the pendulum. When the spherical bob of the
2A
pendulum suffers a retardation (due to viscous drag) at a distance from equilibrium position. The new
3
proportional to its velocity, with ‘ b’ as the constant of
proportionality, the average life time of the pendulum amplitude of the motion is:  (2016)
is (assuming damping is small) in seconds: (2012)
(A) 3 A (B) A 3

7A A
 (C) (D) 41
3 3
mbv


v
mg
0.693
(A) (B) b
b
1 2
(C) (D)
b b
P hysi cs | 8.37

JEE Advanced/Boards

Exercise 1 (i) New amplitude of oscillations.


(ii) New equation for position of the combined body.
Q.1 A body is in SHM with period T when oscillated (iii) Loss of energy during collision. Neglect friction.
from a freely suspended spring. If this spring is cut in
two parts of length ratio 1:3 & again oscillated from the
Q.7 A mass M is in static equilibrium on a massless
two parts separately, then the periods are T1 & T2 then
vertical spring as shown in the figure. A ball of mass m
find T1/T2.
dropped from certain height sticks to the mass M after
colliding with it. The oscillations they perform reach to
Q.2 A body undergoing SHM about the origin has its height ‘a’ above the original level of scales & depth ‘b’
equation is given= by x 0.2cos5π t. Find its average below it.
speed from
= t 0= to t 0.7 sec.

Q.3 Two particles A and B execute SHM along the same


line with the same amplitude a, same frequency and M a
same equilibrium position O. If the phase difference
b
between them is φ =2sin−1 (0.9) , then find the
maximum distance between the two.

Q.4 The acceleration-displacement (a − x) graph of a


particle executing simple harmonic motion is shown in (a) Find the constant of force of the spring;
the figure. Find the frequency of oscillation.
(b) Find the oscillation frequency.

a (c) What is the height above the initial level from which
 the mass m was dropped?

- O x Q.8 Two identical balls A and B each of mass 0.1 kg
- are attached to two identical massless springs. The
spring mass system is constrained to move inside a
Q.5 A point particle of mass 0.1kg is executing SHM rigid smooth pipe in the form of a circle as in figure.
with amplitude of 0.1m. When the particle passes The pipe is fixed in a horizontal plane. The centers
through the mean position, its K.E. is 8 × 10−3 J. Obtain of the ball can move in a circle of radius 0.06m. Each
the equation of motion of this particle if the initial spring has a natural length 0.06π m and force constant
phase of oscillation is 45° . 0.1N/m. Initially both the balls are displaced by an
angle of θ = π / 6 radian with respect to diameter PQ of
the circle and released from rest
Q.6 One end of an ideal spring is fixed to a wall at origin
O and the axis of spring is parallel to x-axis. A block
of mass m=1 kg is attached to free end of the spring
and it is performing SHM. Equation of position of block A B
in coordinate system shown is = x 10 + 3sin10t , is in
second and x in cm. Another block of mass M=3kg, /6 /6
moving towards the origin with velocity 30cm/s collides P
O
Q
with the block performing SHM at t=0 and gets stuck
to it, calculate:

1kg 3kg
8 . 3 8 | Simple Harmonic Motion and Elasticity

(a) Calculate the frequency of oscillation of the ball B. Q.14 The resulting amplitude A’ and the phase of the
vibrations δ
(b) What is the total energy of the system?
A  π A
(c) Find the speed of the ball A when A and B are at the =S A cos(ωt) + cos  ωt +  + cos ( ωt + π )
two ends of the diameter PQ. 2  2 4
A  3π 
+ cos  ωt +=  A'cos ( ωt + δ )
Q.9 Two blocks A(2kg) and B(3kg) rest up on a smooth 8  2 
horizontal surface are connected by a spring of stiffness
120 N/m. Initially the spring is unreformed. A is imparted are ____________ and ___________ respectively.
a velocity of 2m/s along the line of the spring away
from B. Find the displacement of A, t seconds later.
Q.15 A spring block (force constant k=1000N/m and
mass m=4kg) system is suspended from the ceiling
3kg 2kg
of an elevator such that block is initially at rest. The
2m/s
B A elevator begins to move upwards at t=0. Acceleration
time graph of the elevator is shown in the figure. Draw
the displacement x (from its initial position taking
Q.10 A force= F 10 x + 2 acts on a particle of mass upwards as positive) vs time graph of the block with
0.1 kg, where ‘k’ is in m and F in newton. If it is released respect to the elevator starting from t=0 to t=1 sec.
from rest at x = 0.2m , find : Take π2 = 10.
(a) Amplitude; (b) time period; (c) equation of motion.
a
(m/s ) 5
2
Q.11 Potential Energy (U) of a body of unit mass moving k k=100 n/m
in one-dimension conservative force field is given by, 0.6 a m m=4kg
U = (x2 − 4x + 3) . All units are in S.I. t(sec)
(i) Find the equilibrium position of the body.
(ii) Show that oscillations of the body about this Q.16 A particle of mass m moves in the potential energy
equilibrium position are simple harmonic motion & U shown below. Find the period of the motion when the
find its time period. particle has total energy E.
(iii) Find the amplitude of oscillations if speed of the
body at equilibrium position is 2√6 m/s.
U(X)

Q.12 A body is executing SHM under the action of 1 2


force whose maximum magnitude is 50N. Find the U= 2 kx ,x<0
magnitude of force acting on the particle at the time U=mgx, x>0
when its energy is half kinetic and half potential.
X
Q.13 The system shown in the figure can move on a
smooth surface. The spring is initially compressed by
Q.17 The motion of a particle is described by x=30
6cm and then released. Find
sin( πt + π / 6) , where x is in cm and t in sec. Potential
energy of the particle is twice of kinetic energy for
the first time after t=0 when the particle is at position
k=800N/m
3kg 6kg ___________ after ________ time.

Q.18 Two blocks A (5kg) and B (2kg) attached to the


(a) Time period ends of a spring constant 1120N/m are placed on a
(b) Amplitude of 3kg block smooth horizontal plane with the spring undeformed.
Simultaneously velocities of 3m/s and 10m/s along the
(c) Maximum momentum of 6kg block line of the spring in the same direction are imparted to
A and B then
P hysi cs | 8.39

3m/s 10m/s Q.24 A spring mass system is hanging from the ceiling
of an elevator in equilibrium Elongation of spring is l .
A 5 2 B The elevator suddenly starts accelerating downwards
with accelerating g / 3 find
(a) Find the maximum extension of the spring. (a) The frequency and
(b) When does the first maximum compression occurs (b) The amplitude of the resulting SHM.
after start.

Q.19 Two identical rods each of mass m and


length L, are rigidly joined and then suspended
A Exercise 2
in a vertical plane so as to oscillate freely about
an axis normal to the plane of paper passing Single Correct Choice Type
k
through ‘S’ (point of suspension). Find the time
period of such small oscillations. Q.1 A particle executes SHM on a straight line path. The
B amplitude of oscillation is 2 cm. When the displacement
of the particle from the mean position is 1 cm, the
numerical value of magnitude of acceleration is equal
to the numerical value of magnitude of velocity. The
Q.20 (a) Find the time period of oscillations of a
frequency of SHM (in second−1 ) is:
torsional pendulum, if the torsional constant of the wire
is K= 10π 2 J / rad . The moment of inertia of rigid body 2π 3 1
is 10kg- m2 about the axis of rotation. (A) 2π 3 (B) (C) (D)
3 2π 2π 3
(b) A simple pendulum of length l = 0.5m is hanging
from ceiling of a car. The car is kept Q.2 A particle executed SHM with time period T and
on a horizontal plane The car starts amplitude A. The maximum possible average velocity
accelerating on the horizontal road
S
T
with acceleration of 5m / s2 . Find in time is
4
the time period of oscillations of the
pendulum for small amplitudes about 2A 4A 8A 4 2A
(A) (B) (C) (D)
the mean position. T T T T

Q.21 An object of mass 0.2kg executes SHM along Q.3 A particle performs SHM with a period T and
the x-axis with frequency of (25 / π)Hz . At the point amplitude a. The mean velocity of the particle over the
x = 0.04m the object has KE 0.5 J and PE 0.4 J. The time interval during which it travels a distance a/2 from
amplitude of oscillation is ____________. the extreme position is
(A) a/T (B) 2a/T (C) 3a/T (D) a/2T
Q.22 A body of mass 1kg is suspended from a
weightless spring having force constant 600N/m. Q.4 Two particles are in SHM on same straight line with
Another body of mass 0.5 kg moving vertically upwards amplitude A and 2A and with same angular frequency
hits the suspended body with a velocity of 3.0m/s and ω . It is observed that when first particle is at a distance
get embedded in it. Find the frequency of oscillations
A / 2 from origin and going toward mean position,
and amplitude of motion.
other particle is at extreme position on other side of
mean position. Find phase difference between the two
Q.23 A body A of mass m1 = 1kg and a body B of mass particles
m2 = 4kg are attached to the ends of a spring. The
body a performs vertical simple harmonic oscillations (A) 45° (B) 90° (C) 135° (D) 180°
of amplitude a=1.6 cm and angular frequency ω =25
rad/s. Neglecting the mass of the spring determine Q.5 A body performs simple harmonic oscillations
the maximum and minimum values of force the along the straight line ABCDE with C as the midpoint of
system exerts on the surface on which it rests. [Take AE. Its kinetic energies at B and D are each one fourth
g = 10m / s 2 ] of its maximum value. If AE=2R, the distance between
B and D is
8 . 4 0 | Simple Harmonic Motion and Elasticity

Wall
A B C D E

3R R
(A) (B)
2 2
m
(C) 3R (D) 2R

Q.6 In an elevator, a spring clock of time period Ts (mass 2π m m


(A) (B) 2π
attached to a spring) and a pendulum clock of time 3 k k
period Tp are kept. If the elevator accelerates upwards
π m π m
(A) Ts well as Tp increases (C) (D)
3 k 6 k
(B) Ts remain same, Tp increases
(C) Ts remains same, Tp decreases Q.10 A 2 kg block moving with 10 m/s strikes a spring
(D) Ts as well as Tp decreases of constant π2 N/m attached to 2 Kg block at rest kept
on a smooth floor. The time for which rear moving
block remain in contact with spring will be
Q.7 Two bodies P & Q of equal mass are suspended
from two separate massless springs of force constants 1
(A) 2 sec (B) sec
k1 and k2 respectively. If the maximum velocities of them 2
are equal during their motion, the ratio of amplitude of 1
P to Q is: (C) 1sec (D) sec
2
k1 k2
(A) (B) Q.11 In the above question, the velocity of the rear 2 kg
k2 k1
block after it separates from the spring will be:

k2 k1 (A) 0 m/s (B) 5 m/s


(C) (D)
k1 k2 (C) 10 m/s (D) 7.5 m/s

Q.12 A rod whose ends are A & B of length 25 cm is


Q.8 The spring in figure. A and B are identical but
hanged in vertical plane. When hanged from point A
length in A is three times each of that in B. the ratio of
and point B the time periods calculated are 3 sec &
period TA/TB is
4 sec respectively. Given the moment of inertia of rod
A B about axis perpendicular to the rod is in ratio 9:4 at
points A and B. Find the distance of the center of mass
from point A.
(A) 9 cm (B) 5 cm (C) 25 cm (D) 20 cm
m
m
Q. 13 A circular disc has a tiny hole in it, at a distance
z from its center. Its mass is M and radius R (R > z). A
(A)√3 (B) 1/3 (C) 3 (D) 1/√3
horizontal shaft is passed through the hole and held
fixed so that the disc can freely swing in the vertical
Q.9 In the figure the block of mass m, attached to the plane. For small disturbance, the disc performs SHM
spring of stiffness k is in contact with the completely whose time period is the minimum for z =
elastic wall, and the compression in the spring is ‘e’. The
spring is compressed further by ‘e’ by displacing the (A) R/2 (B) R/3
block towards left and is then released. If the collision (C) R / 2 (D) R / 3
between the block and the wall is completely elastic
then the time period of oscillations of the block will be:
P hysi cs | 8.41

Multiple Correct Choice Type 1


(D) Mean velocity is of maximum velocity.
2
Q.14 The displacement-time graph of a particle
Q.18 A spring has natural length 40 cm and spring
executing SHM is shown which of the following
constant 500 N/m. A block of mass 1 kg is attached
statement is/are true?
at one end of the spring and other end of the spring
y is attached to ceiling. The block released from the
position, where the spring has length 45cm.
(A) The block will performs SHM of amplitude 5 cm.

O T/4 T/2 3/4 T t (B) The block will have maximum velocity 30 5 cm / sec .
(C) The block will have maximum acceleration 15m / s2
(D) The minimum potential energy of the spring will be
zero.
(A) The velocity is maximum at t=T/2
(B) The acceleration is maximum at t=T Q.19 The figure shows a graph between velocity
(C) The force is zero at t= 3T/4 and displacement (from mean position) of a particle
performing SHM:
(D) The potential energy equals the oscillation energy
at t=T/2. v(in cm/s)
10
Q.15 The amplitude of a particle executing SHM about 2.5
O is 10 cm. Then: x
(in cm)
(A) When the K.E. is 0.64 of its max. K.E. its displacement
is 6cm from O. (A) The time period of the particle is 1.57s
(B) When the displacement is 5cm from O its K.E.is 0.75 (B) The maximum acceleration will be 40cm / s2
of its max. P.E.
(C) The velocity of particle is 2 21cm / s when it is at a
(C) Its total energy at any point is equal to its maximum distance 1 cm from the mean position.
K.E.
(D) None of these
(D) Its velocity is half the maximum velocity when its
displacement is half the maximum displacement.
Q.20 Two blocks of masses 3 kg and 6 kg rest on a
horizontal smooth surface. The 3 kg block is attached
Q.16 A particle of mass m performs SHM along a to A Spring with a force constant
straight line with frequency f and amplitude A.
m
(A) The average kinetic energy of the particle is zero.
(B) The average potential energy is mπ2f2A2 .
(C) The frequency of oscillation of kinetic energy is 2f. 2m
3kg 6kg
(D) Velocity function leads acceleration by π / 2

Q.17 A system is oscillating with undamped simple Equilibrium


harmonic motion. Then the position
k = 900Nm−1 Which is compressed 2 m from beyond
(A) Average total energy per cycle of the motion is its
the equilibrium position. The 6 kg mass is at rest at 1m
maximum kinetic energy.
1 from mean position 3kg mass strikes the 6kg mass and
(B) Average total energy per cycle of the motion is the two stick together.
times its maximum kinetic energy. 2
(A) Velocity of the combined masses immediately after
1 the collision is 10ms−1
(C) Root means square velocity times its maximum
velocity. 2
8 . 4 2 | Simple Harmonic Motion and Elasticity

(B) Velocity of the combined masses immediately after Q.25 A mass of 0.2 kg is attached to the lower end of a
the collision is 5ms−1 massless spring of force-constant 200 N/m, the upper
end of which is fixed to a rigid support. Which of the
(C) Amplitude of the resulting oscillations is 2m following statements is/are true?
5 (A) In equilibrium, the spring will be stretched by 1cm.
(D) Amplitude of the resulting oscillation is m.
2
(B) If the mass is raised till the spring is in not stretched
state and then released, it will go down by 2 cm before
Q.21 A particle is executing SHM with amplitude A. moving upwards.
time period T, maximum acceleration a0 and maximum
velocity v 0. . Its starts from mean position at t-0 and at (C) The frequency of oscillation will be nearly 5 Hz.
time t, it has the displacement A/2, acceleration a and (D) If the system is taken to moon, the frequency of
velocity v then oscillation will be the same as on the earth.
(A) t=T/12 (B) a = a0 / 2
(C) v = v 0 / 2 (D) t=T/8 Q.26 The potential energy of particle of mass 0.1kg,
moving along x-axis, is given by U=5x(x-4)J where x is
in meters. It can be concluded that
Q.22 For a particle executing SHM, x=displacement
from equilibrium position, v= velocity at any instant (A) The particle is acted upon by a constant force.
and a = acceleration at any instant, then (B) The speed of the particle is maximum at x=2m
(A) v-x graph is a circle (C) The particle executes simple harmonic motion
(B) v-x graph is an ellipse (D) The period of oscillation of the particle is π /5 s
(C) a-x graph is a straight line
(D) a-v graph is an ellipse Q.27 The displacement of a particle varies according
to the relation x=3 sin 100t + cos2 50t. Which of the
following is/are correct about this motion.
Q.23 A particle starts from a point P at a distance of
A/2 from the mean position O & travels towards left as (A) The motion of the particle is not SHM
shown in the figure. If the time period of SHM, executed (B) The amplitude of the SHM of the particle is 5 units
about O is T and amplitude A then the equation of
motion of particle is: (C) The amplitude of the resultant SHM is 73 units.

A/2 (D) The maximum displacement of the particle from the


origin is 9 units.

O P
Q.28 The equation of motion for an oscillating particle
is given by x=3sin (4πt) + 4cos (4πt), where x is in mm
A
and t is in second
 2π π   2π 5π 
(A) x A sin  t + =
=  (B) x A sin  t +  (A) The motion is simple harmonic
 T 6  T 6 
(B) The period of oscillation is 0.5 s
 2π π   2π π  (C) The amplitude of oscillation is 5 mm
=(C) x A cos  t +=  (D) x A cos  t + 
 T 6  T 3
(D) The particle starts its motion from the equilibrium

Q.24 Two particles execute SHM with amplitude A and


Q.29 A linear harmonic oscillator of force constant
2A and angular frequency ω and 2ω respectively. At
2 x106 Nm−1 and amplitude 0.01 m has a total
t=0 they starts with some initial phase difference. At,
mechanical energy of 160 J. Its

t= difference is: . They are in same phase. Their (A) Maximum potential energy is 100 J
initial phase 3ω
(B) Maximum kinetic energy is 100 J
π 2π 4π
(A) (B) (C) (D) π (C) Maximum potential energy is 160
3 3 3
(D) Minimum potential energy is zero.
P hysi cs | 8.43

Q.30 The two blocks shown here rest on a frictionless (B) Statement-I is true, statement-II is true and
surface. If they are pulled apart by a small distance and statement-II is NOT the correct explanation for
released at t=0, the time when statement-I

K=24N/m (C) Statement-I is true, statement-II is false.


2kg
1kg (D) Statement-I is false, statement-II is true

1 kg block comes to rest can be


Q.34 Statement-I: A particle, simultaneously subjected
2π to two simple harmonic motions of same frequency
(A) sec (B) π sec.
3 and same amplitude, will perform SHM only if two
π SHM’s are in the same direction
π
(C) sec (D) sec
2 9 Statement-II: A particle, simultaneously subjected
to two simple harmonic motions of same frequency
and same amplitude, perpendicular to each other the
Assertion Reasoning Type particle can be in uniform circular motion.

Q.31 Statement-I: A particle is moving along x-axis. (A) Statement-I is true, statement-II is true and
The resultant force F acting on it at position x is given statement-II is correct explanation for statement-I
by F=-ax-b. Where a and b are both positive constants. (B) Statement-I is true, statement-II is true and
The motion of this particle is not SHM. statement-II is NOT the correct explanation for
Statement-II: In SHM restoring force must be statement-I.
proportional to the displacement from mean position. (C) Statement-I is true, statement-II is false.
(A) Statement-I is true, statement-II is true and (D) Statement-I is false, statement-II is true.
statement-II is correct explanation for statement-I
(B) Statement-I is true, statement-II is true and statement- Q.35 Statement-I: In case of oscillatory motion the
II is NOT the correct explanation for statement-I average speed for any time interval is always greater
(C) Statement-I is true, statement-II is false. than or equal to its average velocity.

(D) Statement-I is false, statement-II is true. Statement-II: Distance travelled by a particle cannot
be less than its displacement.

Q.32 Statement-I: For a particle performing SHM, (A) Statement-I is true, statement-II is true and
its speed decreases as it goes away from the mean statement-II is correct explanation for statement-I
position. (B) Statement-I is true. statement-II is true and statement-
Statement-II: In SHM, the acceleration is always II is NOT the correct explanation for statement-I.
opposite to the velocity of the particle. (C) Statement-I is true, statement-II is false.
(A) Statement-I is true, statement-II is true and (D) Statement-I is false, statement-II is true.
statement-II is correct explanation for statement-I.
(B) Statement-I is true, statement-II is true and Statement- Comprehension Type
II is NOT the correct explanation for statement-I
Paragraph 1: When force acting on the particle is of
(C) Statement-I is true, statement-II is false.
nature F = −kx , motion of particle is SHM, Velocity at
(D) Statement-I is false, statement-II is true. extreme is zero while at mean position it is maximum. In
case of acceleration situation is just reverse. Maximum
Q.33 Statement-I: Motion of a ball bouncing elastically displacement of particle from mean position on both
in vertical direction on a smooth horizontal floor is a sides is same and is known as amplitude. Refer to figure
periodic motion but not an SHM. One kg block performs vertical harmonic oscillations
with amplitude 1.6 cm and frequency 25 rad s−1 .
Statement-II: Motion is SHM when restoring force is
proportional to displacement from mean position.
(A) Statement-I is true, statement-II is true and
statement-II is correct explanation for statement-I
8 . 4 4 | Simple Harmonic Motion and Elasticity

1 Kg Match the Columns

Q.41 The graph plotted between phase angle ( φ ) and


displacement of a particle from equilibrium position
(y) is a sinusoidal curve as shown below. Then the best
matching is
4.10 kg

Q.36 The maximum value of the force that the system


 3/2 2
exerts on the surface is
O /2 P t
(A) 20 N (B) 30 N (C) 40 N D) 60 N

Q.37 The minimum force is


Column A Column B
(A) 20 N (B) 30 N (C) 40 N (D) 60 N
(a) K.E. versus
phase angle
Paragraph 2: The graphs in figure show that a quantity (i)
curve
y varies with displacement d in a system undergoing 2
simple harmonic motion. O /2  3/2 t

(b) P.E. versus


phase angle
y y
curve (ii)
2
(A) (B) O /2  3/2 t

(c) T.E. versus


phase angle
O d O d

curve (iii)
y y
O t

(d) Velocity
(C) (D)

versus phase
angle curve
O d O d  /2
(iv) t
/2 3/2
Which graphs best represents the relationship obtained
when Y is
(A) (a)-(i), (b)-(ii), (c)-(iii) & (d)-(iv)
Q. 38 The total energy of the system
(B) (a)-(ii), (b)-(i), (c)-(iii) & (d)-(iv)
(A) I (B) II (C) III (D) IV
(C) (a)-(ii), (b)-(i), (c)-(iv) & (d) – (iii)

Q.39 The time (D) (a)-(ii), (b)-(iii), (c)-(iv) & (d)-(i)

(A) I (B) II (C) III (D) IV


Q.42 Column I is a list of possible set of parameters
measured in some experiments. The variations of the
Q.40 The unbalanced force acting on the system parameters in the form of graphs are shown in Column
(A) I (B) II (C) III (D) None II. Match the set of parameters given in Column I with
the graphs given in Column II. Indicate your answer by
darkening the appropriate bubbles of the 4 x 4 matrix
given in the ORS.
P hysi cs | 8.45

Q.1 If the total energy of the particle is E, it will perform


Column I Column II
periodic motion only if
(A) Potential energy of y
a simple pendulum (y
(A) E< 0 (B) E>0
axis) as a function of (p) (C) V0 >E>0 (D) E> V0
displacement (x axis)
O x Q.2 For periodic motion of small amplitude A, the time
period t of this particle is proportional to
(B) Displacement (y axis) y
as a function of time (x m 1 m
axis) for a one dimensional (A) A (B)
(q)
α A α
motion at zero or constant
acceleration when the
α 1 α
body is moving along the O x (C) A (D)
positive x-direction.
m A m

(C) Range of projectile y


(y axis) as a function of Q.3 The acceleration of this particle for x > X 0 is
its velocity (x axis) when (A) Proportional to V0
projected at a fixed angle. (r)
V0
x (B) Proportional to
O mX 0
(D) The square of the time y
period (y axis) of a simple V0
(C) Proportional to
pendulum as a function of mX 0
its length (x axis) (s)
(D) Zero
O x

Q.4 A small block is connected to one end of a massless


spring of un-stretched length 4.9 m. The other end of
Previous Years’ Questions the spring (see the figure) is fixed. The system lies on a
horizontal frictionless surface. The block is stretched by
0.2 m and released from rest at t = 0. It then executes
Paragraph 1: When a particle of mass m moves on the simple harmonic motion with angular frequency
x-axis in a potential of the form V ( x ) = kx2 , it performs π
simple harmonic motion. The corresponding time ω = rad/s. Simultaneously at t = 0, a small pebble
3
m is projected with speed v form point P at an angle of
period is proportional to , as can be seen easily 45° as shown in the figure. Point P is at a horizontal
k
using dimensional analysis. However, the motion of a distance of 10 m from O. If the pebble hits the block at
t = 1 s, the value of v is (take g = 10 m/s2) (2012)
particle can be periodic even when its potential energy
increases on both sides of x = 0 in a way different z
from kx2 and its total energy is such that the particle
does not escape to infinity. Consider a particle of
mass m moving on the x-axis. Its potential energy is v
v ( x ) = αx ( α > 0 ) for x near the origin and becomes
2
45
o

x
a constant equal to V0 for x ≥ X 0 (see figure below) O
P
 (2010) 10m
V(x)
(A) 50 m / s (B) 51 m / s

V0 (C) 52 m / s (D) 53 m / s

Xn
8 . 4 6 | Simple Harmonic Motion and Elasticity

Q.5 A particle of mass m is attached to one end (A) The amplitude of oscillation in the first case changes
of a mass-less spring of force constant k, lying on a
M
frictionless horizontal plane. The other end of the by a factor of , whereas in the second case it
spring is fixed. The particle starts moving horizontally m+M
from its equilibrium position at time t = 0 with an initial remains unchanged
velocity u0 . When the speed of the particle is 0.5 u0 . It
(B) The final time period of oscillation in both the cases
collides elastically with a rigid wall. After this collision,
is same
 (2013)
(C) The total energy decreases in both the cases
(A) The speed of the particle when it returns to its
equilibrium position is u0 (D) The instantaneous speed at x0 of the combined
masses decreases in both the cases
(B) The time at which the particle passes through the
m Q.8 Column I describes some situations in which a small
equilibrium position for the first time is t = π .
k object moves. Column II describes some characteristics
of these motions. Match the situations in column I with
(C) The time at which the maximum compression of the the characteristics in column II. (2007)
4π m
spring occurs is t =
3 k
Column I Column II
(D) The time at which the particle passes through the (A) The object moves on the (p) The object
x-axis under a conservative executes a simple
5π m
equilibrium position for the second time is t = force in such a way that its harmonic motion.
3 k
speed and position satisfy
Q.6 Two independent harmonic oscillators of equal = v c1 c2 − x2 , where c1 and
mass are oscillating about the origin with angular
c2 are positive constants.
frequencies ω1 and ω2 and have total energies E1
and E2 , respectively. The variations of their momenta p (B) The object moves on the (q) The object does
a x-axis in such a way that its not change its
with positions x are shown in the figures. If = n2 and velocity and its displacement direction.
b
a from the origin satisfy v = −kx ,
= n , then the correct equation(s) is(are) (2015)
R where k is a positive constant.
P P (C) The object is attached to (r) The kinetic energy
Energy=E1 Energy=E2 one end of a mass-less spring of the object keeps on
of a given spring constant. decreasing.
b
x x The other end of the spring is
a R attached to the ceiling of an
elevator. Initially everything
is at rest. The elevator starts
ω2 going upwards with a constant
(A) E1 ω1 = E2 ω2 (B) = n2
ω1 acceleration α . The motion of
E1 E2 the object is observed from the
(C) ω1 ω2 =n2 (D) =
ω1 ω2 elevator during the period it
maintain this acceleration.
Q.7 A block with mass M is connected by a massless (D) The object is projected from (s) The object can
spring with stiffness constant k to a rigid wall and moves the earth’s surface vertically change its direction
without friction on a horizontal surface. The block only once.
GMe
oscillates with small amplitude A about an equilibrium upwards with a speed 2 ,
Re
position x0 . Consider two cases: (i) when the block is at
x0 ; and (ii) when the block is at =
x x0 + A . In both the where Me is the mass of the
cases, a particle with mass m (< M) is softly placed on earth and R e is the radius of
the block after which they stick to each other. Which of the earth. Neglect forces from
the following statement(s) is (are) true about the motion objects other than the earth.
after the mass m is placed on the mass M? (2016)
P hysi cs | 8.47

Q.9 A linear harmonic oscillator or force constant (C) If A=B; C=0


2 × 106 N/m and amplitude 0.01m has a total
mechanical energy of 160 J. Its  (1989) (D) If A=B; C=2B, amplitude= B
(A) Maximum potential energy is 100 J
Q.12 A metal rod of length L and mass m is pivoted
(B) Maximum kinetic energy is 100J
at one end. A thin disk of mass M and radius R ( < L )
(C) Maximum potential energy is 160J is attached at its center to the free end of the rod.
(D) Maximum potential energy is zero Consider two ways the disc is free to rotate about its
center. The rod-disc system performs SHM in vertical
Q.10 Three simple harmonic motions in the same plane after being released from the same displaced
direction having the same amplitude and same period position. Which of the following statement(s) is/are
are superposed. If each differ in phase from the next by true?  (2011)
45° , then  (1999)

(
(A) The resultant amplitude is 1 + 2 a)
(B) The phase of the resultant motion relative to the
first is 90°
(C) The energy associated with the resulting motion is

(3 + 2 2 ) times the energy associated with any single (A) Restoring torque in case A=Restoring torque in
case B
motion
(B) Restoring torque in case A<Restoring torque in
(D) The resulting motion is not simple harmonic case B
(C) Angular frequency for case A>Angular frequency
Q.11 Function x = A sin2 wt + B cos2 wt + C sin wtcos wt for case B
represent SHM (2006)
(D) Angular frequency for case A<, angular frequency
(A) For any value of A, B and C (except C=0) for case B

(B) If A=-B, C=2B, amplitude= B 2

PlancEssential Questions
JEE Main/Boards JEE Advanced/Boards
Exercise 1 Exercise 1
Q. 7 Q.8 Q.20 Q.4 Q.6 Q.8
Q.24 Q.25 Q.18 Q.23 Q.24

Exercise 2 Exercise 2
Q.3 Q.9 Q.15 Q.1 Q.2 Q.4
Q.5 Q.9 Q.14
Previous Years' Questions
Q.15 Q.20 Q.25
Q.9 Q.10 Q.11
Q.29 Q.30 Q.42
Q.14 Q.15 Q.16
8 . 4 8 | Simple Harmonic Motion and Elasticity

Answer Key

JEE Main/Boards
Exercise 1
Q.1 0.5 rad Q.2 1.048 s Q.3 0.61 rad

Q.4 0.5s Q.5 0.71 a Q.6 7.56J

Q.7 0.35 ms−1 , 0.06 J Q.8 1.5J Q.9 86.4s

Q.10 141°.4 ’ Q.11 0.612 s−1

 2πt π   π 
Q.18 (a) y= 2sin  +  (b)y= 3cos  t  Q.19 2π / 3rad
 3 3   30 

Q.20 (a) 6m (b) −48.99ms−1 Q.21 19.67m Q.22 4.86s; 5.06m

Q.23 2 3cm;2 2cm Q.24 0.1875 J; 0.0625 J, 0.25J Q.25 3.2 Hz

1/2
1  k1k 2 (k 3 + k 4 ) 
Q.26 
{ }
( 2π )  (k1 + k 2 ) × (k3 + k 4 ) + k1k 2 m 

Exercise 2

Single Correct Choice Type

Q.1 A Q.2 C Q.3 C Q.4 D Q.5 A Q.6 C

Q.7 C Q.8 B Q.9 C Q.10 B Q.11 B Q.12 C

Q.13 A Q.14 B Q.15 C

Previous Years’ Questions


Q.1 C Q.2 B Q.3 D Q.4 B Q.5 A Q.6 B
Q.7 D Q.8 A Q.9 A Q.10 A Q.11 A Q.12 A
Q.13 D Q.14 C Q.15 D Q.16 A Q.17 B Q.18 D
Q.19 B Q.20 C Q.21 C

JEE Advanced/Boards

Exercise 1

Q.1 1 / 3 Q.2 2 m/s Q.3 1.8a

1 β
Q.4 Q.5 x = 0.1sin(4t + p / 4)
2π α
P hysi cs | 8.49

2mg2mg ab ab1 1 2mg2mg


Q.6 3cm, x = 10 − 3sin5t; ∆E = 0.135J Q.7 (a) kK = K = : ( c ) : ( c ) , (b) ,
b − ab − a b − ab −2πa 2(πb − (ab)(−Ma)(
+M m)+ m)
1
Q.8 f = ;E = 4 π2 × 10−5 J; v = 2π × 10−2 m / s Q.9 0.8t + 0.12 sin10t
π
Q.10 (a) 0.4
0.4m, (b ) 5π sec., ((c)
m, (b) c )=
xx =0.2
0.2−‒0.4
0.4cos
coswt Q.11
ωι (i) x=
0 2m;(ii)T
= 2π sec.;(iii)2 3 m

π
Q.12 25 2N Q.13 (a) sec, (b ) 6cm (c)2.40kgm / s.
10

1
x
−1  0.04
3 5A
Q.14 tan  2  Q.15 0.2 0.4 0.6 1.D
8 0.8 t(sec)
-0.04

1  2 1
Q.16 π m / k + 2 2E / mg2 Q.17 10 6cm, sin−1   − sec
π  3 6
 
17L
Q.18 (a) 25cm, (b) 3p/56 seconds Q.19 2π
18g

2
Q.20 (a)
( a)2sec,
2sec, ((b)
b) T = sec Q.21 0.06m
1/ 4
5
gg
Q.22 10πHz,
5 37
cm Q.23 60N, 40N ((aa))1T1T ==212π1π LL, ,((b)
Q.24 (a) (bb))L3L3
6

Exercise 2

Single Correct Choice Type

Q.1 C Q.2 D Q.3 C Q.4 C Q.5 C Q.6 C


Q.7 B Q.8 C Q.9 A Q.10 C Q.11 A Q.12 D
Q.13 C

Multiple Correct Choice Type

Q.14 B, C, D Q.15 A, B, C Q.16 B, C Q.17 A, C Q.18 B, C, D Q.19 A, B, C

Q.20 A, C Q.21 A, B Q.22 B, C, D Q.23 B, D Q.24 B, C Q.25 A, B, C, D

Q.26 B, C, D Q.27 B, D Q.28 A, B, C Q.29 B, C Q.30 A, B, C

Assertion Reasoning Type

Q.31 D Q.32 C Q.33 A Q.34 D Q.35 A

Comprehension Type

Paragraph 1: Q.36 D Q.37 C

Paragraph 2: Q.38 A Q.39 D Q.40 D


8 . 5 0 | Simple Harmonic Motion and Elasticity

Match the Columns


Q.41 B Q.42 A → p, s; B → q, r, s; C → s; D → q

Previous Years’ Questions


Q.1 C Q.2 B Q.3 D Q.4 A

Q.5 A, D Q.6 B, D Q.7 A, B, D Q.8 A → p; B → q, r; C → p; D → r, q

Q.9 A Q.10 A, C Q.11 A, B, D Q.12 A, D

Solutions

JEE Main/Boards t=
θ
×T=
60
× 2π sec
360° 360°
Exercise 1 π
t= sec
3
Sol 1: y(t) = 10 sin (20t + 0.5)
Sol 3: y = 0.4 sin (440 t + 0.61)
A = 10 m
(i) Amplitude = 0.4 m
ω = 20 rad./sec
(ii) ω = 440 rad.sec
φ = 0.5 radians
w 20 10 ω 220
f= = = hz (iii) f = , hz
2π 2π π 2π π
1 π 1 2π π
T= = sec (iv) T = = = sec
f 10 f ω 220
y (v) Initial phase = 0.61 radians

5m
=60
o
Sol 4: A = 25 cm, T = 3s

2.5m y
v

o
=30
25 12.5
o
30
o
V
12.5 30
Sol 2: V = ω A2 − y 2

4 = ω A2 − 9

60° 1
⇒t= × = sec.
3 = ω A2 − 16 360° 2


T= = 2π secω = 1 sec
ω
P hysi cs | 8.51

Sol 5: Amplitude = 0 Sol 8:


1
Total energy = Ka2 3.0 kg
2
-1
1 600 Nm -1
Potential energy = Kx2 600 N
2
1 1 1 a
Kx2 = × Ka2 ⇒ x =
2 2 2 2

Sol 6: m = 8 kg
a = 30 cm Kx Kx
60
k × 0.3 = 60 ⇒ k = = 200 n/m
0.3
x
m 8 2π
T = 2π ⇒ T = 2π = = 0.4 p keq = 2k = 1200 Nm-1
k 200 5
1
(a) T = 0.4 π sec. Total energy stored = kA2
2
−k 2
(b) a = x 1 1
m = × 1200 ×   = 1.5 Joules.
2 2
−200
a= × 0.12 = 3m/sec2
84 
Sol 9: T = 2π ; T ∝  1/2
1 2 1 g
P.E. = kx = × 200 × (0.12)2 = 1.44 J
2 2  → 0.998 l
1
K.E. = k(A2 – x2)  → (0.998)1/2 T
2
T → 0.999 T
= 100 x (0.09 – 0.0144) = 7.56 J
Error in a day = 0.001 × (60 × 60 × 24) = 86.4 sec
Sol 7: k = 1200 Nm -1

2.0 cm
Sol 10:

1/3 A y
m=3.0 kg


v
k 1200
(a) w = = = 20 rad/s.
m 3

4 −1 3
v = ω A2 − x2 = 20 =
100 5 Phase Difference = θ = 2 cos-1 (1/3) = 141.05°

v = 0.35 m/s
2
Sol 11: Elevator moving up
1 2 1  1 
(b) P.E. = kx = × 1200 ×   Frequency of seconds pendulum = f0= 0.5 Hz
2 2  100 
g 3
1 geff = g + = g
= 600 × 2 2
100 × 100
1 geff 3 3
P.E. = 0.06 J f= = f = × 0.5 Hz; f = 0.61 Hz
2π  2 0 2
8 . 5 2 | Simple Harmonic Motion and Elasticity

Sol 12: Periodic motion: A motion which repeats itself Service:-parallel :


after equal intervals of time is called periodic motion K1

eq; motion of a pendulum


K1 K2 K2

k1k 2
k= k = k1 + k2
k1 + k 2

Sol 16: Phase: Phase of a vibrating particle at any


instant is the state of the vibrating particle regarding
it’s displacement and direction of vibration at that
Oscillatory motion: A body is said to possess oscillatory particular instant. It is denoted by φ.
or vibratory motion if it moves back and forth Phase difference is the difference in phases of two
repeatedly about a mean position. For an oscillatory vibrating particles at a given time.
motion, a restoring force is required.
y
2

 1
V

Sol 13: Simple Pendulum: A simple pendulum is a Particle 1 lags in phase by θ.


weight suspended from a pivot so that it can swing
freely. i.e. f2 – f1 = θ
Angular frequency:- It is frequency f multiplied by a
numerical quantity ω. It is denoted by ω.

ω = 2πf =
T
f → frequency
T → Time period
Displacement in periodic motion: It is the displacement
from the mean/equilibrium position.

Time period = 2π d2 x
g Sol 17: a = − = -w2x
dt2
 → length of pendulum

a = –w2xω =
g → acceleration due to gravity T
1 g 4 π2 x
Frequency = f = a=– x → displacement
2π  T2
T → time period
Sol 14: Refer spring mass system and ex.3
v=ω A2 − x2 v → velocity

Sol 15: Spring factory: It is a mesure of the stiffness of A → Amplitude


a spring
x → Displacement
P hysi cs | 8.53

π x = 6 sin (3xt + 5π/6)


Sol 18: Figure (a) Initial phase = φ = 60° =
3
x = 6 cos 60° = 6 sin 30°
y = 2 sin (wt + φ) x = 3cm
 2π π
y = 2 sin  t +  V
 3 3
2π 2π
ω= =
T 3
o
60
X
π
Figure (b) initial phase = φ =
2
A = 3 cm
2π π v=ω A2 − x2 = 3π 36 − 9
ω= =
60 30
y = A sin (wt + φ) v= 6 3π
 πt π  v = –48.97 ms-1
y = 3 sin  +  cm
 3 2 X

Sol 19:
y 5/6 V
pa

A
2
r
tic

cle
le

rti 2
1

pa 3
V

Sol 21: T = 20 s V = 5 ms-1


2π 2
φ= ⇒θ= × 360° ⇒ θ = 36°
3 20
5 = Aω cos 36°
Sol 20: x = 6 cos (3pt + π/3) metres
2π π
(a) A = 6m ω= =
20 ω
(b) V = ω A2 − x2 π
5=A cos 36°
10
w = 3π 50
A= = 19.68 m
2π π cos36°
T= = 2/3 sec.
ω
Amplifide of SHM = 19.68 m
At t = 2s particle will complete 3 orcillations
So the position will be same as at t = 0 s. Sol 22: x = 1m v = 3 ms–1

6m x = 2mv = 2 ms–1

v= 3 = ω A2 − 1
o
60
3=ω A2 − 1

2=ω A2 − 4
8 . 5 4 | Simple Harmonic Motion and Elasticity

9 A2 − 1 x
⇒ = 2 ⇒ 9A2 – 36 = 4 A2 – 4 10 cm
4 A −4
5 cm
32 o
⇒ 5A2 = 32 ⇒ A = = 6.4 30 v
5

⇒ A = 2.53 m

3=ω 6.4 − 1
v = -Aω cos 30°
3 10 3 3
w= = 1.29 rad/s v= ×5 2 × ; v =
5.4 100 2 2
2π v = 0.61 ms-1
Period of motion: T = = 4.86 s
ω
1 1 3
K.E. = × m × v2 = ×1× = 0.1875 J
Length of path = 2A = 5.06 m 2 2 8
1
P.E. = × 50 × (0.05)2 = 0.0625 J
Sol 23: A = 4 cm 2

vmax = Aω 1
Total energy = × 50 × (0.1)2 = 0.25 J
2
vmax Aω
v= = = ω A2 − ω2
2 2 Sol 25:
A2
A –x =
2 2 -1
4 300 Nm
1 kg 3 kg
3
x= A
2
For two mass system.
3
x= × 4 = 2 3 = 3.464 cm We take effective mass instead of mass to calculate
2
frequency.
P.E. = K.E.
m1m2 1×3 3
1 1 µ= = = kg
⇒ P.E. = × kA2 m1 + m2 1+3 4
2 2
k 300
1 2 1 w2 = = = 400
kx = kA2 µ 3/4
2 4
ω = 20 rad/sec.
x = 2 2 = 2.828 m
10
f= Hz ≅ 3.2 Hz
Sol 24: k = 50 Nm -1 π

m = 1kg
Sol 26: k34 = k3 + k4
x=0.1m
1 1 1 1
= + +
k1234 k1 k2 k 34

1 1 1
= + +
k1 k2 (k 3 + k 4 )

k 50 k 2 (k 3 + k 4 ) + k1 (k 3 + k 4 ) + k1k 2
w2 = = = 50 =
m 1 k1k 2 (k 3 + k 4 )
ω = 5 2 rad/s
1 (k1 + k 2 )(k 3 + k 4 ) + k1k 2
=
k1234 k1k 2 (k 3 + k 4 )
P hysi cs | 8.55

θ
1/2 t= × 1.2; t = 0.17 sec
k  360
ω =  1234 
 m 
1/2 Sol 7: (C) x = A + a sin wt
1  k1234 
f=   5
2π  m  t= T
4
1/2
1  k1k 2 (k 3 + k 4 )  x
f=  
2π  (k1 + k 2 )(k 3 + k 4 ) + (k1k 2 )m 

Exercise 2
v
Single Correct Choice Type

Sol 1: (A) y = 5 sin (π t + 4π) Distance in one rev. = 4A

2π 2π Total distance covered = 4A + A = 5A


A=5T= = = 2 sec
ω π k ×k k
Sol 8: (B) k1 = =
A = 5; T = 2 sec k +k 2
k2 = k + k = 2k
Sol 2: (C) amax. = Aw 2

T1 k2
vmax. = Aω t ∝ k-1/2 = =2
T2 k1
Double ω; half the amplitude

0.8  π
Sol 3: (C) A = = 0.4 m Sol 9: (C) y1 = sin  ωt +  y2 = sin wt
2  3 
30 1  π π
f= = hz T = 2 sec y1 + y2 2 sin  ωt +  cos
60 2  6  6

 π
Sol 4: (D) = 3 sin  ωt + 
x  6
a
Sol 10: (B) y = A sin wt + A cos wt
a o
2 45 π 
V
= 2A (sin wt + sin  + ωt  )
 2 
 π π  π
= 2A sin  ωt +  sin   = 2 A sin  ωt + 
 4  4  4
π
Sol 5: (A) 2pt + =π 1
( 2A )
2
3 T.E. = × mw2 ×
2π ' 2
2pt =
3
T.E. = mw2A2
t = 1/3 sec
1 1 mv 2
Sol 11: (B) kA2 = mv2 ⇒ A =
Sol 6: (C) T = 1.2 sec 2 2 k
A = 8 cm
5 Sol 12: (C) Amplitude dose not depend on frame of
θ = cos-1 ; θ = 51.31°
8 reference.
8 . 5 6 | Simple Harmonic Motion and Elasticity

1 2 1 Sol 3: (D)
Sol 13: (A) s = ut + at ⇒ –a = 0 + × (–5) × t2 x
2 2
2a
t=
5
L
2a 
T = 4t = 4
5

1  3 1 Modulus of rigidity, η = F/Aq


Sol 14: (B) kA2  1 −  = mv2
2  4  2 x
Here, A = L2 and θ =
1/2 L
k A
⇒v=   Therefore, restoring force force is
m 2
1/2 F = – ηAθ = – ηLx
k A ω
⇒v =   1
, w2 = 1 F ηL
Or acceleration, a = =– x
m 4 2 M M
3 2m  k  A2 7 Since, a ∝ – x, oscillations are simple harmonic in nature,
T.E. = kA2 +   = kA2
8 2  m  16 16 time period of which is given by

7 1 displacement x
⇒ kA2 = kA’2 T = 2π = 2π
16 2 acceleration a
7 M
A’ = A = 2π
8 ηL

Sol 15: (C) YA


k1k 2 L YAk
Sol 4: (B) Keq = = =
3 2 k1 + k 2 YA YA + Lk
+k
30
o
L
o
45
4 1
Y.A.L

q24 = 195° k1 =
YA
L
q12 = 75°
q31 = 165° k

m
k2=k
Previous Years’ Questions

Sol 1: (C) In SHM frequency with which kinetic energy


m
oscillation is two times the frequency of oscillation of
displacement.
m m(YA + Lk)
∴ T = 2π = 2π
Sol 2: (B) (vM)max = (vN)max k eq YAk
∴ ωMAM = wNAN

AM ωN k2  k 
or = =  ω = 
AN ωM k1  m 
P hysi cs | 8.57

Note Equivalent fore constant for a wire is given by k U


AY  AY  K
= . Because in case of a wire. F =   ∆L and in
L  L 
case of spring F = k.Dx. Comparing these two, we find
AY
k of wire = U
L
–dU
F= = – (slope of U-x graph) = 0.
Sol 5: (A) U(x) = k|x| 3 dx
Therefore, origin is the stable equilibrium position.
[U] [ML2 T –2 ]
∴ [k] = = = [ML–1T–2] Hence, particle will oscillate simple harmonically about
[x3 ] [L3 ] x = 0 for small displacement. Therefore, correct option
Now, time period may depend on is (d).
T ∝ (mass)x(amplitude)y(k)2 (a), (b) and (c) options are wrong due to following
reasons.
[M0L0T]=[M]x[L]y[ML–1T–2]z=[Mx+zLy–2T–2z]
–dU
(a) At equilibrium position F = = 0 i.e., slope of U-x
Equating the powers, we get dx
– 2z = 1or z = –1/2 graph should be zero and from the graph we can see
that slope is zero at x = 0 and x = ±∞
y – z = 0 or y = z = – ½
Now among these equilibriums stable equilibrium position
Hence, T ∝ (amplitude)–1/2 ∝ (a)–1/2 is that where U is minimum (Here x=0). Unstable equilibrium
1 position is that where U is maximum (Here none).
or T ∝
a Neutral equilibrium position is that where U is constant
(Here x = ±∞ )
Sol 6: (B)
Therefore, option (a) is wrong.
(b) For any finite non-zero value of x, force is directed
towards the origin because origin is in stable equilibrium
position. Therefore, option (b) is incorrect.
1
 2=
k1 1=
2
 3 (c) At origin, potential energy is minimum, hence kinetic
k
3 energy will be maximum. Therefore, option (c) is also wrong.

Sol 8: (A) Free body diagram of bob of the pendulum


with respect to the accelerating frame of reference is as
follows
l1 = 2l2 
sin
g
2 M
∴ l1 = l
3
1
Force constant k ∝
length of spring

3
∴ k1 = k
2
mg
–x2
Sol 7: (D) U(x) = k(1 – e ) 
sin
It is an exponentially increasing graph of potential M
g
energy (U) with x2. Therefore, U versus x graph will be
as shown. At origin.
Potential energy U is minimum (therefore, kinetic energy 
sin
will be maximum) and force acting on the particle is zero M
g  Mg cos
because.
8 . 5 8 | Simple Harmonic Motion and Elasticity

∴ Net force on the bob is Fnet = mg cosa At time t = 0, x = A. Hence, PE should be maximum.
Further in graph III, PE is minimum at x = 0 Hence, this
or Net acceleration of the bob is geff = g cosa
is also correct.
L
T = 2π
geff Sol 11: (A) Angular frequency of the system,
L k k
or T = 2π ω= =
gcos α m+m 2m

Note: Whenever point of suspension is accelerating Maximum acceleration of the system will be, w2 A or
kA
in
gs 2m
This acceleration to the lower block is provided by
friction.
90°+
Hence, fmax = mamax

 kA  kA
= mw2A = m   =
 2m  2
g
L
Take T = 2π Sol 12: (A) y = kt2
geff
d2 y
→ → → = 2k or ay = 2m/s2(as k = 1 m/s2)
Where g eff = g – a dt2
 
→ T1 = 2π and T2 = 2π
a = acceleration of point of suspension. g g + ay
→ T12 g + ay 10 + 2 6
In this question a = g sin α (down the plane) ∴ = = =
T22 g 10 5
→ →
∴ | g – a | = geff
2π π
Sol 13: (D) T = 8s, ω = =   rads–1
T 4
= g2 + (gsin α )2 + 2(g)(gsin α )cos(90° + α )
x = A sin wt
= g cosa  π2  π 
∴ a = – w2x = –   sin  t 
 16  4 
Sol 9: (A) In SHM, velocity of particle also oscillates  
simple harmonically. Speed is more near the mean 4
Substituting t = s, we get
position and less near the extreme positions. Therefore, 3
the time taken for the particle to go from O to A/2 will  3 2
be less than the time taken to go it from A/2 to A, or a=–  π  cms–2
 32 
T1 < T2  

Note From the equation of SHM we can show that Sol 14: (C)
t1 = T0–A/2 = T/12
x
and t2 = TA/2– A = T/6

So, that t1 = t2 = T0–A = T/4


Sol 10: (A) Potential energy is minimum (in this case
zero) at mean position (x = 0) and maximum at extreme x
positions (x = ± A). L
x= q
2
P hysi cs | 8.59

Restoring torque = – (2kx). L Then by applying simple mathematics we can prove


2 that

kL(L / 2θ)  kL2 / 2   6k  B = A and φ = .
α=– = –  θ = –  θ 3
I 2
 ML / 12  M
Sol 18: (D) As retardation = bv
1 α 1 6k ∴ Retarding force = mbv
∴f= =
2π θ 2π M
∴ Net restoring torque when angular displacement is
θ is given by
Sol 15: (D) x1 + x2 = A and k1x1 = k2x2
= −mg  sin θ + mbv 
x1 k2
or = ∴ I α = − mg  sin θ + mbv 
x2 k1
Solving these equations, we get Where, I = m 2

 k2  d2 θ g bv
x1 =  ∴ =α=− sin θ +
 A dt 2  
 k1 + k 2 
for small damping, the solution of the above differential
Sol 16: (A) Frequency or time period of SHM depends equation will be
on variable forces. It does not depend on constant −
bt

external force. Constant external force can only change ∴ θ =θ0 e 2 sin ( wt + φ )
the mean position. For example, in the given question
−bt
mean position is at natural length of spring in the
∴ Angular amplitude will be = θ.e 2
absence of electric field. Whereas in the presence of
electric field mean position will be obtained after a According to question, in τ time (average life–time),
compression of x0. Where x0 is given by
1
Kx0 = QE Angular amplitude drops to value of its original
value ( θ ) e
QE
or x0 =
K 6τ
θ0 −
∴ θ0 e
= 2 ⇒ 6τ = 1
Sol 17: (B) e 2

A 2
∴τ =
A2 = A b

Sol 19: (B) A = A0 e−kt


2/3
4/3 A1– A A0 e−5k
⇒ 0.9 A0 =
A0 e−15k
and α A0 =
Solving ⇒ α =0.729
A3 = A At mean position, K.E. is maximum where as P.E. is
π minimum.
Resultant amplitude of x1 and x2 is A at angle  
3
from A1. To make resultant of x1, x2 and x3 to be zero. 2 2
 2A   2A 
4π Sol 20: (C) 3 ω A2 −  ω A12 − 
 = 
A3 should be equal to A at angle φ = as shown in  3   3 
3
figure. 7A
∴ A1 =
3
Alternate solution: It we substitute, x1 + x2 + x3 = 0

 2π 
or A sin wt +A sin  ωt +  + B sin(ωt + φ)
 3 
8 . 6 0 | Simple Harmonic Motion and Elasticity

2 Sol 3: φ = 2 sin-1 (0.9)


 2A 
ω A2 − 
Sol 21: (C) v =  x
 3  a
P1 Condition
Aω maximum
v= 5
3 -1
sin 0.9
distance
-1
sin 0.9 V
v new
= 3v
= 5 Aω

So the new amplitude is given by P2


2
 2A 
ω Anew 2 − x2 ⇒ 5 Aω =
Vnew = ω Anew 2 −   P1P2 | | y-axis
 3 
Max. Distance = 1.8 a
7A
Anew =
3
Sol 4:

JEE Advanced/Boards 

Exercise 1 -

x

1 m
Sol 1: T ∝ ; T = 2π -
k 1/2 k

4k
k1 = 4k ; k2 = a = –w2 x
3
β
By k1l1 = k2l2 = kl – w2 = = slope of a-x graph
α
T T 3
T1 = ; T2 = β
2 2 ω=
α
T1 1
= ω 1 β
T2 3 Frequency = =
2π 2π α

Sol 2: x = 0.2 cos 5pt Sol 5: m = 0.1 kg


dx A = 0.1 m
velocity = = -π sin 5pt
dt
1
× mvmax2 = 8 × 10–3 J
speed = π |sin 5pt| 2
0.7 0.1× vmax2 = 16 × 10–3 ⇒ vmax = 0.4 m/s
π∫ | sin5πt | dt
vavg = 0
Aω = 0.4
0.7
0.1× ω = 0.4 ⇒ ω = 4
0.1
π 10π 0.1
=
0.7
×7× ∫ sin5πt dt =

 − cos5πt 
0
x = A sin (wt + φ)
0
x = 0.1 sin (4t + π/4)
vavg = 2 m/s
Sol 6: (i)

1kg 3kg
30cm/sec
30cm/sec
P hysi cs | 8.61

x = 10 + 3 sin 10 t 1  Mg 
2
= k − a  + (M + m) ga
At t = 0 s block 1 is at equilibrium position. 2  k 
v1 = Aω = 3 × 10 = 30 cm/s  2mg 
k  (b + a) + b2 − a2  = 2 (M + m)g (a + b)
v2 = 30 cm/s  k 
Conservation of momentum 2mg
2Mg (b + a) + k (b2 – a2) = 2 (M + m) g (a + b) k =
m1 v1 + m2 v2 = (m1 + m2) v b−a

– 1 × 30 + 3 × 30 = 4 × v 2mg
Constant of force of spring =
v = 15 cm/s b−a

Final velocity is in opposite direction of initial velocity k 2mg


of block 1. This causes a phase change of π. (b) ω = =
(M + m) (M + m)(b − a)
ω ∝ m–1/2
1 2mg
ω‘= 5 rad/s f=
2π (M + m)(b − a)
A’ω’ = 15; A’ = 3 cm  Mg   Mg  
2 2
1 
New amplitude = 3 cm (c) mgh = k  − a −   
2  k   k  
 
(ii) New equation
+ (M + m) ga
x = 10 + 3 sin (5t + π)
(iii) Loss of energy   2Mg 
1
mgh = – k a ×  − a 
1 1 1  2   k 
=  × 1 × 302 + × 3 × 302 − × 4 × 152  × 10-4 J
 2 2 2  + (M + m) ga
1
= (900+2700 – 900)×10-4 J = 1350 × 10-4 J −ka  2mg 
2 mgh =  − a  + (M + m) ga
2  k 
∆Eloss = 0.1350 J
ka2
mgh = –Mga + + (M + m) ga
2
Sol 7:
2mga2
mgh = mga +
h
O (b − a)2
a
a2 ab
b h=a+ =
(b − a) b−a

Sol 8:

2
1  Mg 
(a) mgh + k  
2  k 
2
1  Mg  /6 /6
= k  + b  - (M + m) gb
2  k 
2
1  Mg 
= k  − a  + (M + m) ga
2  k 
Equalising energies in 3 states
2
1  Mg  (a) Frequency
k  + b  - (M + m) gb
2  k  Displace by dθ
8 . 6 2 | Simple Harmonic Motion and Elasticity

Dx = 2Rdθ Sol 10: (a) m = 0.1 kg

k 2Rdθ F = 10 x + 2
dα = –2 ×
m R Only variable force causes SHM
dα = –w dθ 2
(a) F(x) = 10 x + 2
4k k a(x) = 100 x + 20
w2 = ω= 2 =2
m m v(x) = 50 x2 + 20 x + c
2 1 v(0.2) = 0
f= =
2π π
2
50 × 0.04 + 20 × 0.2 + c = 0
1  π
(b) Total energy = 2 × k R  c = −6
2  3 x=0.2
1  0.06 × π 
2 v = 50 x2 + 20 x − 6
=2× × 0.1 ×   x=–0.6
2  3 
0.2 − ( −0.6)
= 3.94 × 10-4 J/4p2 × 10-5 J A= = 0.4 m
2
1
(c) 2 × mv2 = 4p2 × 10-5 Amplitude = 0.4 m
2
4 π2 × 10−5 10
v2 = (b) ω = = 10 rad/sec
0.1 0.1

v2 = 4p2 × 10-4 2π π
T= = sec.
ω 5
v = 2π × 10-2 = 0.02 π m/sec
(c) x = 0.2 – A cos wt

Sol 9: 5t
x = 0.2 – 0.4 cos
π
3kg 2kg
k=12N/m 2m/s
B A Sol 11: u = (x2 – 4x + 3)
dU
2× 2 + 3× 0 (i) F = –
Vcom = = 0.8 m/s dx
5
F = -2x + 4
xA = vcom t + A sin wt At equilibrium F = 0
At maximum expansion –2x + 4 = 0 ⇒ x = 2 m
1 1 1 (ii) dF = –2dx similar to dF = -w2dx as in SHM
× 5 × (0.8)2 + kx2 = × 2 × 22
2 2 2 ω2
2= = w2
kx2 = 8 – 3.2 = 4.8 m
x = 0.2 ω= 2
3 3 2π
A= x= × 0.2 = 0.12 T= = 2 π sec
5 5 ω
3× 2 (iii) Aω = 2 6
µ= = 1.2
3+2
2 6
A= ⇒ A= 2 3 m
k 120 2
ω= = = 10
µ 1.2
xA = 0.8 t + 0.12 sin 10 t Sol 12: Fmax = mw2A
1
P.E. = K.E.
2
P hysi cs | 8.63

1 2 1 1
⇒ kx = × kA2 m
2 2 2 Sol 15: T = 2π = 0.4 sec
k
A
⇒x= ω = 5π
2
A Fmax For 0 < t < 0.6 sec
F = mw2 =
2 2 mg mg π
x=– + sin (5pt + )
2k 2k 2
F = 25 2 N

µ
Sol 13: (a) T = 2π
k o

3× 6 18 -0.02
µ= = = 2 kg
3+6 9
-0.04
1 π
T = 2π = sec
400 10 mg 4 × 10
= = 0.02 m
(b) A = 6 cm 2k 2 × 1000

−1 for 0 < t < 0.6 sec


(c) vcm = 0; vB = v
2 A x = –0.02 + 0.02 sin (5pt + π/2)
1 1 1 for 0.6 < t 1 sec
× k × A2 = × 3 × (-2vB)2 + × 6 × vB2
2 2 2
x = –0.04 + 0.04 sin (5pt)
800 × (0.06)2 = 12 vB2 + 6vB2
x
8 × 0.36
vB =
2
0.04
18
1.0
2 × 0.6
vB = 0.2 0.6 0.8
3 0.4 t(sec)
VBmax. = 0.4 m/s -0.04
PBmax = 0.4 × 6
PBmax = 2.4 kg ms–1 Sol 16:

 A A A
Sol 14: s =  A −  cos wt -  −  sin wt U(X)
 4  2 8
1 2
3A 3A U= 2 kx ,x<0
s= cos wt - sin wt
4 8 U=mgx, x>0

3A
s= (2 cos wt – sin wt)



8 SHM Body under gravity


3 5  2 1 
s= A  cos ωt − sin ωt 
8 1 m 2v
 5 5  T= × 2π +
2 k g
3 5   1 
s= A cos  ωt + sin−1    ∴
8  1 m 2 2E
  5  E= mv2; T= π +
2 k g m
3 5  1 
A’ = A; δ = sin-1   ∴
8 2E m 2 2 E
 5 v= ; T= π +
m k g m
8 . 6 4 | Simple Harmonic Motion and Elasticity

 π Maximum extension = 0.25 m


Sol 17: x = 30 sin  πt + 
 6 3
(b) t = T
2π 4
T= =2
π µ
T = 2π
k
30

15
/6

5×2 10
P.E. = 2 K.E. µ= =
7 7
2 10 2π π
P.E. = T.E. T = 2π = =
3 7 × 1120 28 14
2 Time for first maximum compression
x= A
3
3 π 3π
= × = sec
2 4 14 56
Position: x = × 30
3
I
x = 10 6 cm Sol 19: T = 2π
mgx
 2 π
 sin−1 −  m2  m2 
 3 6  I= +  + m2 
=t  × 2 sec  12 
3  

1
mx + mx
1  2 π 2 3 4m2 13 2
t=  sin−1 −  sec x= = = + m
π  3 6  2m 4 12 12

17
I= m 2
Sol 18: (a) 12

3m/s 10m/s 17 2
m
12 17
T = 2π ;T = 2π
A 5 2 B 3 18g
2mg
4
5 × 3 + 10 × 2
vcm = = 5ms-1
7 Sol 20: (a) Iα = - kθ
1 1 k
5 × 33 + × 2 × 102 α=– θ
2 2 I
1 1 2 k k
= 7 × 52 + kx w2 = ω= =π
2 2 2 I
45 + 200 = 175 + kx2 2π 2π
T= = ;T = 2 sec
kx = 70
2
ω π
70 1
x2 = x= m
1120 4
P hysi cs | 8.65

(b) 1  1 
= kh2 – 1.5 × 10 × h − 
2  60 

1 1 1.5
× 600 × +
2 60 2 2
2 1  1 
5m/S = × 600 × h2 – 15 × h − 
2  60 
2
10m/S 60 h2 – 3h – 7/60 = 0

1.5 37
geff. = 102 + 52 = 125 h= + m
60 120
geff. = 5 5 1.5 37
A=h- = m
 0.5 60 120
T = 2π = 2π
g 5 5 × 10 37 5 37
A= × 100 cm = cm
2 120 60
T= sec
1/ 4
5
Sol 23: m1 = 1kg; m2 = 4kg
25 a = 1.6 cm
Sol 21: m = 0.2 kg f = Hz
π
kx = m1 g
4
P.E. = T.E.
9
1 2 4 1 2 x
kx = × kA2; x = A A
2 9 2 3
a
3 3
A= x= × 0.04
2 2 k

A = 0.06 m B

Sol 22: 0.5 × 3 = 1.5 × v k = w2 m1 = 252 × 1 = 625 N/m

v = 1 m/s Nmax = m2 g + k(x + a)

kx N
k=600N/m
eqdb
A B
x=1/60m
k(x+a)
m1=1kg
m1g m2g

3m/s 625 × 1.6


= (m1 + m2)g + ka = 50 +
m2=0.5 kg 100
Nmax = 60 N
600
ω= = 400 = 20 rad./sec Nmin = (M1 + M2) g – ka
1.5
Nmin = 40N
20 10
f= = Hz
2π π

1 2 1.5 × 12
kx +
2 2
8 . 6 6 | Simple Harmonic Motion and Elasticity

a/2 3a
k 9 Sol 3: (C) vmean = =
Sol 24: k = mg; ω= = T/6 T
m 
1/2
1 g Sol 4: (C)
f=  
2π   
2g
A =  /3 ; m × =k×x P1
3
2 mg 2
x= = l 135
o

3 k 3

g/3
P2
Natural length
New equilibrium
Sol 5: (C)

3

A B C D E
1
VB2 = V2
4 A
Exercise 2
1 2 2
R w = w2 (R2 – x2) ⇒ R2 = (R2 – x2) 4
Single Correct Choice Type 4
3
x= R
Sol 1: (C) 2

2cm dBD = 2x = 3R

m
Sol 6: (C) TS = 2π T doesn’t depend on g.
k s

Tp = 2π ; Tp ∝ g-1/2
ω 4 −1 = w × 1 2 g
∴ Tp decreases
ω= 3
3 A k
F= Hz Sol 7: (B) vmax = Aω =
2π m
A1 k1 A2 k 2
Sol 2: (D) =
x m m
A1 k2
=
A2 k1

4 v

Sol 8: (C) kA = k/3; kB = 3k
4
TA
TA ∝ k–1/2 ; =3
TB
displacement 2×A
vavg = = m 2π / 3
time T/4 Sol 9: (A) T =2π ×
k 2π
4 2A
vavg = 2π m
T T=
3 k
P hysi cs | 8.67

Sol 13: (C)

e m
2
e 3 R z

I mR 2 + 2mz 2
T = 2π ⇒ I=
mgz 2
T
Sol 10: (C) t =
4
2π R2
π µ T= m + 2mz
t= g 2
2 k

1 1 mR 2
t=
π
; t= sec = 2mz for minimum T
2 2 2
π2
R
z=
2

Multiple Correct Choice Type

Sol 14: (B, C, D) v = 0 at t = T/2


a is maximum at extremes
3T
F = 0 at t =
Sol 11: (A) Both block have speed same as 4
vcm = 5m/s K.E. = 0 at t = T/2

Sol 12: (D) Sol 15: (A, B, C) K.E. = 0.64 KEmax.


v = 0.8 vmax.
A B
∴ x = 0.6 A = 6 cm
COM
A PEmax 3
x= P.E. = KE = PEmax
I 2 4 4
T = 2π
mg KEmax = TE at mean position
TA 3 9 B A 3v max
= = × x= v=
TB 4 4 A 2 2
3 3 B
=
4 2 A Sol 16: (B, C) (A) KEavg is never zero in SHM
1
B 1 (B) PEavg = TE = mp2 t2 A2
= 2
A 4
(C) Frequency of occurrence of mean position =2f
4
A = × 25 = 20 cm (D) Acceleration leads
5
T
2
∫v dt
v
Sol 17: (A, C) vrms = 0
=
T 2
8 . 6 8 | Simple Harmonic Motion and Elasticity

T
A
∫ v dt 8
vmean = 0
= V A
π
T π 2 6

Sol 18: (B, C, D) A = 3cm


1 1
mvm2 = × 500 × 9
2 2
vm = 3 × 10 5 cm/s ; ω = 500 = 10 5 Sol 22: (B, C, D) v2 = w2 (A2 – x2)
amax. = ω vm a = -w2 x

= 10 5 × 30 5 cm/s2  a2 
v2 = w2  A2 − 
 ω4 
= 15 m/s2 

PEmin = 0 at mean position  2πt 5π   2πt π 


Sol 23: (B, D) x = A sin  +  = A cos  + 
 T 6   T 3
Sol 19: (A, B, C) A = 2.5
vmax 2π π 5π
ω= = 4; T = = = 1.57 s
A 4 2
6
vmax = 16 × 2.5= 40 cm/s2

v=ω A2 − x2 = 4 2.52 − 12

= 4 5.25 cm/s = 2 21 cm/s


Sol 24: (B, C)
Sol 20: (A, C) Energy conservation:
1 1 1
× 900 × 22 = × 900 × 12 + × 3 v12
2 2 2
2700 = 3 × v12
v12 = 900
v1 = 30 m/s
2π 4 π
Conservation of momentum:- Initial phase difference = 0, ,
3 3
3 × 30 + 6 × 0 = 9 × v
mg 0.2 × 10
v = 10 ms-1 Sol 25: (A, B, C, D) x = = = 1 cm
k 200
Energy conservation:- Amplitude = 1 cm
1 1 1
× 900 × 12 + × 9 × 102 = × 900 × A2 k 200
2 2 2 ω= = = 10 10
m 0.2
A2 = 2; A= 2 m
10 10 5 10
f= = ≅ 5Hz
2π π
π/6 T
Sol 21: (A, B) t = ×T⇒t= Amplitude changes, frequency remains the same.
2π 12
3 Sol 26: (B, C, D) m = 0.1 kg
v= V0 ⇒ a ∝ x ⇒ a = a0/2
2
U = 5x(x – 4)
dU
F=– = 20 – 10 x
dx
P hysi cs | 8.69

P.E. minimum at x = 2 m Sol 34: (D) Phase remains same and SHMs are
Force is linear function of x with negative slope. perpendicular.

10
w2 = Sol 35: (A) Statement-II is the correct explanation.
m
10
ω= = 10 rad/s Comprehension Type
0.1
2π π Paragraph 1:
T= = sec
10 5
Sol 36: (D) ω = 25 rad/s
Sol 27: (B, D) x = 3 sin 100 t + 8 cos2 50 t k = mw2 = 1 × 625 = 625 Nm-1
= 3 sin 100 t + 4 cos 100 t + 4 16
Fmax =1×9.8+625× + 4.1×9.8
100 × 10
x = 5 sin (100 t + sin-1 4/5) + 4
=59.98 N ≅ 60 N
Sol 28: (A, B, C) x = 5 sin (4pt + sin-1 4/5) mm Fmin = 5.1 × 9.8 – 10 ≅ 40 N

T= = 0.5 s Sol 37: (C) Minimum force on the surface = (50 – 10)

N = 40 N
A = 5 mm
φ = sin-1 (4/5) Sol 38: (A) TE of system is constant

Sol 29: (B, C) k = 2 × 106 Nm-1 Sol 39: (D) d = A sin (wt + φ)
A = 0.01 m
T.E. = 160 J Sol 40: (D) F = –kx + c

PEmax = 160 J k>0

when KE = 0 J
Match the Columns
i.e. at equilibrium
1 Sol 41: (B) (a) y = A sin (t)
KEmax = × 2 × 106 × 10-4 = 100 J
2 v = A cos (t)
PEmin = 60 J KE = c × cos2 (t)

T (a) → (ii)
Sol 30: (A, B, C) t = n
2 (b) → (i) PE + KE = const.
m 2 π PE = c × sin2 t
T = 2π = 2π =
k 3 × 24 3 (c) → (iii) TE constant always
(d) → (iv) v = A cos t
Assertion Reasoning Type

a Sol 42: (A) PE ∝ x2 (A) → p, s


Sol 31: (D) The motion is SHM with ω =
m 1 2
(B) s = ut + at
If the force is linear w.r.t. x and slope is negative. The 2
motion is always SHM. q, r when a = 0 ; S when a ≠ 0

Sol 32: (C) When particle moves from extreme to mean v 2 sin2θ
(C) Range =
position velocity and acceleration have same direction. g
2
4 π 
(D) T2 =
g
Sol 33: (A) Statement-II is the correct explanation.
8 . 7 0 | Simple Harmonic Motion and Elasticity

Previous Year’s Questions Value of x from eq. (i)


3 1 1 2
Sol 1: (C) If E > VB, particle will escape. But simultaneously × mu20 = kx
4 2 2
for oscillations, E > 0
3u0 m
Hence, the correct answer is V0 > E > 0 x=
2 k
Or the correct option is (c)
t2
 ML2 T –2 
o
 PE  3 A 30 A
Sol 2: (B) [a] =   =   = [ML–2T–2]
4 4
x   L 
o
2 60
t=0
m  1 m
∴   = [L2T2]; ∴   = [T]
α  A α 
As dimensions of amplitude A is [L]
2π m
Time to reach eq. position first time ⇒
3 k
Sol 3: (D) For |x| > x0, potential energy is constant.
Hence, kinetic energy, speed or velocity will also remain Second time it will reach at time ⇒
constant.
2π m T 2π m 2π m 5π m
∴ Acceleration will be zero + ⇒ + ⇒ as (D)
3 k 2 3 k k ×2 3 k

For max. compression time is t2


2v sin 45°
Sol 4: (A) =1
g 2π m T
=t2 +
∴ v =50 m / s 3 k 4

2π m 2π m 7π m
= + =
Sol 5: (A, D) 3 k k k 6 k

eq. position
Sol 6: (B, D)

1 b2 a 1
E1 = mω12a2 = = = n2  … (i)
Rigid wall 2 2m b mω1
m
1 R2 … (ii)
E2= mω22 R 2= mω2= 1 
2 2m

ω2
From (i) and (ii) = n2
0.5 uo ω1

2
x E1 ω  a 1 ω1 2 ⇒ E1 =
E2
=  2  2   = ⋅ ⋅n
E2  ω1   R  2
n ω2 ω1 ω2
1 2 1 2 1
mu=
0 kx + × m0.25 u20 ….(i)
2 2 2
After elastic collision Sol 7: (A, B, D)

Block speed is 0.5 u0 K   K 


(A) ωi   =
= and ωf  
So when it will come back to equilibrium point its M  (M + m) 
speed will be u0 as (A)
Mv i 1 1 2
1 1 Case I: v f
= = Mv 2 KA
Amplitude mu20 = kA2 M+m 2 2 i
u 2 2
A= 0
k
P hysi cs | 8.71

M 2 1 1 2 Sol 11: (A, B, D)


⇒ Ai2
= v1 and =(M + m)v 2f KA
K 2 2 i A B C
x= (1 − cos 2ω t) + (1 + cos 2ω t) + sin 2ωt
Mv2
M A M 2 2 2
⇒ A2f = ⋅ ⇒ f =
K M +m Ai M+m For A = 0, B = 0

M C
(B) Tf = 2π for both =x sin 2ωt
M+m 2

A = -B and C = 2B
1 1  M 
(C) TEcase I = (M + m)v 2f = Mv 2  
2 2 M m X = B cos 2wt + B sin 2wt

1 2 1 2 Amplitude = B 2
TE=
case II =KA KA
2 f 2 i For A = B; C = 0
(D) VEP = Afwf : Decreases in both cases. X = A,
Hence this is not correct option.
Sol 8: A → p; B → q, r; C → p; D → r, q
For A = B, C = 2B

Sol 9: (A) The total mechanical energy = 160 J X = B + B sin 2wt


It is also represent SHM.
The maximum PE will be 160 J at the instant when KE = 0

Sol 10: (A, C) By principle of superposition y = y1 + y2 + y3 Sol 12: (A, D) Restoring torque is same in both cases
= asin (ω t + 45°) + asin ω t + asin(ω t − 45°)
T
α= = − ω2 θ
= asin (ω t + 45°) + asin(ω t − 45°) + asin ω t I
= 2asin ω t cos 45° + asin ω t
In case A the moment of inertia is more as compared
= (1 + 2 ) asin ω t
2 asin ω t + asin ω t =
to B, so wB > wA

∴ Amplitude of resultant motion = (1 + 2 ) a …(i)

(b) The option is incorrect as the phase of the resultant


motion relative to the first is 45°.

(c) Energy is SHM is proportional to (amplitude)2

(1 + 2 ) a= (1 + 2 + 2 2 )
2
2
E E
∴ R
= ∴ R
ES a2 ES 1

or E=
R (3 + 2 2 )E S

(d) Resultant motion is y =


1 + 2 asin ωt( )
Ii is SHM.
8 . 7 2 | Simple Harmonic Motion and Elasticity

ELASTICITY
1. INTRODUCTION
We have learnt that the shape and size of a rigid body does not change but this is an ideal concept. Actually a rigid
solid does experience some kind of deformation under the action of external forces and if the magnitude of forces
cross a certain limit, the deformation is so severe that the material of the solid loses its rigidity. We say that the
material has broken-down or failure has happened. In this chapter we learn about the properties of solid bodies
by virtue of which they resist the deformation in their shape and size. These properties constitute the strength of
a material and the knowledge of these is very essential in constructing small and large structures like houses, tall
buildings, bridges, railway tracks etc.

2. MOLECULAR STRUCTURE OF A MATERIAL


Matter is made up of atoms and molecules. An atom is made up of a nucleus and electrons. Nucleus contains
protons and neutrons (collectively known as “nucleons”). Nuclear forces are responsible for the structure of nucleus.
Likewise, forces between different atoms and molecules are responsible for the structure of a material.

2.1 Interatomic and Intermolecular Forces


The forces that are responsible for holding the atoms/molecules in place
in a solid or liquid are called interatomic and intermolecular forces. The
interaction between any isolated pair of atoms and molecules may be
represented by a curve that shows how the potential energy varies with U(R)
R0 R
the separation between them as shown in the Fig. 8.32 O

We see that as the distance R decreases, the attractive force first increases
and then decreases to zero at a separation R0 where the potential energy
is the minimum. For smaller distance, force is repulsive. U0
The above picture of interatomic or intermolecular force is an over
Figure 8.32: Potential energy versus
simplification on the actual situation. However, it provides a reasonable
separation
visualisation.
The force between the atoms can be found from the potential energy
using the relation,
dU
F(R) = - F(R)
dR O
R
The resulting force curve is shown in Fig. 8.33. R0
Force is along the line joining the atoms or molecules, and is shown
negative for attraction & positive for repulsion.

2.2 Classification of Matter


Figure 8.33: Graph of force versus
Matter can be classified into three states:- solids, liquids and gases. separation

Solids: A solid is that state of matter whose atoms and molecules are strongly bound so as to preserve their
original shape and volume. Solids are of two types-crystalline & amorphous.
(a) Crystalline solid: A crystalline solid is one which has regular & periodic arrangement of atoms or molecules
in three dimensions. Examples of crystalline solids are diamond, rock salt, mica, sugar etc.
(b) Amorphous solids or glassy solids: The word ‘amorphous’ literally means ‘without any form’. There is no
‘order’ in arrangement of atoms in such a solid. Example - glass.
P hysi cs | 8.73

In solids, the intermolecular forces are so strong that there is no change in shape and size easily.

Liquids: The intermolecular forces are comparably less than that in solids, so the shape can easily be changed.
But volume of a given mass of a liquid is not easy to change. It needs quite an effort to change the density of
liquids.
Liquids are not able to produce reaction forces to applied forces in arbitrary directions.

Gases: This is the third state of matter which cannot support compressive, tensile, or sharing forces. Densities of
gases change very rapidly with increase in temperature.

Liquids and gases are together classified as fluids: The word ‘’fluid’’ comes from a Latin word meaning ‘’to flow’’.
On an average, the atoms or molecules in a gas are far apart, typically about ten atomic diameters at room
temperature and pressure. They collide much less frequently than those in a liquid. Gases in general are compressible.

3. INTRODUCTION TO ELASTICITY
When external forces are applied to a body which is fixed to a rigid support, there is a change in its length, volume
or shape. When the external forces are removed, the body tends to regain its original shape and size. Such a
property of a body by virtue of which a body tends to regain its original shape or size, when the external forces are
removed, is called elasticity.
If a body completely regains its shape and size, it is called perfectly elastic. If it does not regain its shape and
size completely, it is called inelastic material. Those materials which hardly regain their shape are called plastic
material.
An elastic body is one that returns to its original shape after a deformation. Eg- golf ball, rubber band, soccer
ball.
An inelastic body is one that does not return to its original shape after a deformation. Eg – dough or bread, clay,
inelastic ball.

PLANCESS CONCEPTS

Microscopic reason of elasticity


Each molecule in a solid body is acted upon by forces due to neighboring molecules. When all molecules
are in a state of stable equilibrium, the solid takes a particular shape. When the body is deformed,
molecules are displaced from their stable equilibrium positions. The intermolecular distances change
and restoring forces start acting which drives the molecules to come back to its original shape.
Vaibhav Krishnan (JEE 2009, AIR 22)

One can compare this situation to a spring-mass system. Consider a particle connected to several particles through
spring. If this particle is displaced a little, the spring exerts a resultant force which tries to bring the particle towards
its natural position. In fact, the particle will oscillate about this position. In due course, the oscillations will be
damped out and the particle will regain its original position.

3.1 Stress and Strain

Stress: Elastic bodies regain their original shape due to internal restoring forces. This internal restoring force, acting
per unit area of a deformed body is called a stress.
Restoring force
i.e. Stress =
Area
8 . 7 4 | Simple Harmonic Motion and Elasticity

SI unit of stress is N/m2 and Dimensional formula of stress is [ML–1T–2]


An object can be deformed in different ways.

PLANCESS CONCEPTS

Misconception: People often get confused between pressure and stress.


Difference between pressure v/s stress:

S. No. Pressure Stress

1 Pressure is always normal to the area Stress can be normal or tangential

2 Always compressive in nature May be compressive or tensile in nature

Nivvedan (JEE 2009, AIR 113)

3.1.1 Types of Stress


There are 2 types of stresses – NORMAL stress and SHEAR stress
Normal stress – When the force applied is perpendicular to the area of application of force, it is called normal
stress. Normal stress usually leads to a change in length (longitudinal stress) or a change in volume.
Normal stress can be of two types – tensile stress and compressive stress.
(a) Tensile Stress: Pulling force per unit area. It is applied parallel to the length.

A F
F

Figure 8.34: Tensile stress

It causes increase in length or volume.


(b) Compressive Stress: Pushing force per unit area. It is applied parallel to the length.

A F
F

Figure 8.35: Compressive stress

It causes decreases in length or volume.


If the force is applied tangentially to one face of a rectangular body keeping the opposite face fixed, the stress is
called tangential or shearing stress.
Stress is measured in units of 1N/m2. 1N/m2 = 10 dynes/cm2.
P hysi cs | 8.75

Strain: The fractional or relative change in shape, size or dimensions of body is called the strain.
change in dimension
Strain =
original dimension
There are three types of strains:
∆
(i) Longitudinal strain: It is the ratio of the change in length, ∆ , to the original length,  i.e. .

∆V
(ii) Volume strain: It is the ratio of change in volume, DV, to the original volume V i.e.
V
(iii) Shearing strain: The angular deformation, θ, in radians of a face of a F X
rectangular body is called shearing strain.
If a tangential force F is used to displace upper face of rectangular body

through a small angle θ such that the upper face is displaced through distance 

∆x
Dx where  is height of the body, then shearing strain = θ ≈ tan θ =

Strain is a ratio of two similar quantities and does not have any units.
Figure 8.36: Shearing strain
Illustration 1: A 4.0 m long copper wire of cross sectional area 1.2 cm is 2

stretched by a force of 4.8 × 103 N. Stress will be- (JEE MAIN)


(A) 4.0 × 107 N/mm2 (B) 4.0 × 107 kN/m2 (C) 4.0 × 107 N/m2 (D) None

Sol: (C) Stress is restoring force per unit area of cross-section.


F 4.8 ×103 N
Stress = = = 4.0 × 107 N/m2
A 1.2 ×10−4 m2

Illustration 2: A copper rod 2m long is stretched by 1mm. Strain will be (JEE MAIN)
(A) 10–4, volumetric (B) 5 × 10–4, volumetric (C) 5 × 10–4, longitudinal (D) 5 × 10–3, volumetric

Sol: Longitudinal strain is equal to change in length per unit length.


∆ 1 ×10 −3
(C) Strain = = = 5 × 10–4, Longitudinal
 2

Illustration 3: A lead of 4.0 kg is suspended from a ceiling through a steel wire of radius 2.0 mm. Find the tensile
stress developed in the wire when equilibrium is achieved. Take g = 3.1π ms–2. (JEE MAIN)

Sol: Stress is restoring force per unit area of cross-section.


Tension in the wire is F = 4.0 × 3.1 π N.
The area of cross section is A = pr2 = π × (2.0 × 10–3 m)2 = 4.0 π × 10–6 m2.
F 4.0 × 3.1 π
Thus, the tensile stress developed = N/m2 = 3.1 × 106 N/m2.
A 4.0 π×10−6

Illustration 4: Find the stress on a bone (1 cm in radius and 50 cm long) that supports a mass of 100kg. Find the
strain on the bone if it is compressed 0.15 mm by this weight. Find the proportionality constant C for this bone.
 (JEE MAIN)
8 . 7 6 | Simple Harmonic Motion and Elasticity

Sol: Stress is restoring force per unit area of cross-section. Strain is equal to change in length per unit length. Strain
∝ Stress
Stress = F/A = (100kg) (9.8 m/s2) / π × (0.01 m)2 = 3.1 × 106 N/m2
Strain = ∆L/L0 = (0.15 × 10–3m) / (0.5m) = 3.0 × 10–4
Since strain = C × stress, C = strain / stress = 0.96 × 10–10 m2/N.

4. HOOKE’S LAW AND MODULI OF ELASTICITY

Hooke’s Law: It states that for small deformations, stress is directly proportional to strain within elastic limits and
the ratio is a constant called modulus of elasticity.
Stress
= modulus of Elasticity = E
Strain

4.1 Young’s Modulus


Young’s modulus is a measure of the resistance of a solid to a change in its L0
length when a force is applied perpendicular to its surface. Consider a rod
A
with an unstressed length L0 and cross-sectional area A, as shown in the Fig.
8.37. When it is subjected to equal and opposite forces Fn along its axis and
Fn Fn
perpendicular to the end faces, its length changes by ∆L. These forces tend
F L
to stretch the rod. The tensile stress on the rod is defined as σ = n
A Fn L
Forces acting in the opposite direction, as shown in Fig. 8.37, would produce
Fn
a compressive stress. The resulting strain is defined as the dimensionless
∆L Figure 8.37: Variation in length of rod
ratio, ε = Young’s modulus Y for the material of the rod is defined as
L0
the ratio of tensile stress to tensile strain.
Tensile stress σ Fn / A FnL0
So Young’s Modulus = ;=
Y= =
Tensile strain ε ∆L / L0 A∆L
A force applied normal to the end face of a rod cause a change in length.

PLANCESS CONCEPTS
FL
(a) For loaded wire: ∆L =
πr 2 γ
For rigid body ∆L = 0 so Y = ∞ i.e. Elasticity of rigid body is infinite.
L
(b) If same stretching force is applied to different wires of same material, ∆L ∝ [As F and Y are
const.] r2

Greater the value ∆L, greater will be elongation.


Following conclusions can be drawn from γ=stress/strain:
(i) E ∝ stress (for same strain), i.e. if we want the equal amount of strain in two different materials,
the one which needs more stress is having more E.
P hysi cs | 8.77

PLANCESS CONCEPTS

1
(ii) E ∝ (for same stress), i.e., if the same amount of stress is applied on two different
strain
materials, the one having less strain is having more Elasticity. Rather we can say that, the one
which offers more resistance to the external forces is having greater value of E. So, we can see
that modulus of elasticity of steel is more than that of rubber or Esteel > Erubber
 ∆x 
(iii) E = stress for unit strain  = 1 or ∆=x x  , i.e. suppose the length of a wire is 2m, then the
 x 
Young’s modulus of elasticity (Y) is the stress applied on the wire to stretch the wire by the

same amount of 2m.


Chinmay S Purandare (JEE 2012, AIR 698)

Illustration 5: Two wires of equal cross section but one made of steel and the other of copper, are joined end to
end. When the combination is kept under tension, the elongations in the two wires are found to be equal. Find the
ratio of the lengths of the two wires. Young’s modulus of steel = 2.0 × 1011 Nm–2. (JEE ADVANCED)

Sol: The wires joined together have same stress and same elongation. Ratio of stress and young’s modulus is strain.
As young’s modulus for steel and copper is different, strains of the wires will be different.
As the cross sections of the wires are equal and same tension exists in both, the stresses developed are equal. Let
the original lengths of the steel wire and the copper wire be LS and LC respectively and the elongation in each wire
be  .
 stress
=  … (i)
LS 2.0 ×1011 Nm−2
 stress
And =  … (ii)
LC 1.1 ×1011 Nm−2

Dividing (ii) by (i), LS/LC = 2·0 / 1·1 = 20:11.

Illustration 6: A solid cylindrical steel column is 4.0 m long and 9.0 cm in diameter. What will be decrease in length
when carrying a load of 80000 kg? Y = 1.9 × 1011 Nm–2. (JEE MAIN)

Sol: The stress will be equal to load per unit cross section. Strain is the ratio of stress and young’s modulus.
Let us first calculate the cross-sectional area of column = pr2 = π(0.045m)2 = 6.36 × 10–3 m2
F/A FL [(8 ×10 4 )(9.8N)](4.0m)
Then, from Y = we have ∆L = = = 2.6 × 10–3 m.
∆L / L AY −3 2
(6.36 ×10 m )(1.9 ×10 Nm )11 −2

Illustration 7: A load of 4.0 kg is suspended from a ceiling through a steel wire of length 20 m and radius 2.0 mm.
It is found that the length of the wire increases by 0.031 mm as equilibrium is achieved. Find Young’s modulus of
steel. Take g = 3.1 π m/s2.  (JEE MAIN)
Sol: The stress will be equal to load per unit cross section. Strain is the change in length per unit length. Young’s
modulus is the ratio of stress and strain.
(4.0kg)(3.1 πms−2 )
The longitudinal stress = = 3.1 × 106 N/m2
π (2.0 ×10−3 m)2
8 . 7 8 | Simple Harmonic Motion and Elasticity

0.031 ×10−3 m
The longitudinal strain = = 0.0155 × 10–3
2.0m

3.1 ×106 Nm−2


Thus Y = −3
= 2.0 × 1011 N/m2.
0.0155 ×10

Illustration 8: A bar of mass m and length  is hanging from point A as shown in Fig. 8.38. Find A
the increase in its length due to its own weight. The Young’s modulus of elasticity of the wire is
Y and area of cross-section of the wire is A.  (JEE ADVANCED)

Sol: Find the elongation for an elementary length dx of the wire due to tension in the wire at
the location of the element. B
Consider a small section dx of the bar at a distance x from B. The weight of the bar for a length Figure 8.38
x is,
 mg  A
W=  x
  
W  mg 
Elongation in section dx will=
be d =  dx   x dx dx
 AY   AY 
x
Total elongation in the bar can be obtained by integrating this expression for x = 0 to x =  . B
x = Figure 8.39
 mg   mg
\ =
∆ ∫ =d    ∫ x dx or
 AY  0
∆ =
2AY
x =0

Illustration 9: One end of a metal wire is fixed to a ceiling and a load of 2 kg hangs from the other
end. A similar wire is attached to the bottom of the load and another load of 1 kg hangs from this T2
lower wire. Find the longitudinal strain in both the wires. Area of cross section of each wire is 0.005
2kg
cm2 and Young modulus of the metal is 2.0 × 1011 N m–2. Take g = 10 ms–2. (JEE ADVANCED)
T1
Sol: Find the tension in each wire. Stress is tension per unit area of cross section. Strain is the ratio
1kg
of stress and Young’s modulus.
The situation is described in Fig. 8.40. As the 1kg mass is in equilibrium, the tension in the lower Figure 8.40
wire equals the weight of the load.
Thus T1 = 10N; Stress = 10N/0.005 cm2 = 2 × 107 N/m2
stress 2 ×107 N / m2
Longitudinal strain
= = = 10−4
Y 2 ×1011 N / m2
Considering the equilibrium of the upper block, we can write, T2 = 20N + T1 or T2 = 30N
Stress = 30 N/0.005 cm2 = 6 × 107 N/m2
6 ×107 N / m2
Longitudinal strain = = 3 × 10–4 .
2 ×1011 N / m2

A B
Illustration 10: Each of the three blocks P, Q and R shown in Figure Q
P
has a mass of 3 kg. Each of the wires A and B has cross-sectional
area 0.005 cm2 and Young modulus 2 × 1011 N/m2. Neglect friction.
Find the longitudinal strain developed in each of the wires. Take
g = 10 m/s2.  (JEE ADVANCED) R

Figure 8.41
P hysi cs | 8.79

Sol: Find the tension in each wire. Stress is tension per unit area of cross section. Strain is the ratio of stress and
Young’s modulus.
The block R will descend vertically and the blocks P and Q will move on the frictionless horizontal table. Let the
common magnitude of the acceleration be a. Let the tensions in the wires A and B be TA and TB respectively.
Writing the equations of motion of the blocks P, Q and R, we get,

TA = (3kg) a  …. (i)

TB – TA = (3kg) a  …. (ii)

And (3kg)g – TB = (3kg)a  …. (iii)


By (i) and (ii), TB = 2TA ; By (i) and (iii), TA + TB = (3kg) g = 30 N
or 3TA = 30N or, TA = 10N and TB = 20 N.

Longitudinal stress
Longitudinal strain =
Young modulus

10N / 0.005 cm2 20N / 0.005 cm2


Strain in wire A = = 10–4 ; And strain in wire B = = 2 × 10–4.
2 ×1011 N / m2 2 ×1011 N / m2

PLANCESS CONCEPTS

In practical life, we often hear something like elastic band is usually referred to a rubber band because it
is easily stretchable and a steel rod is not.
However, here elasticity has some different meaning. Being more elastic means, the material will resist
more to any external force which tries to change its configuration.
That is why Esteel > Erubber.
Nitin Chandrol (JEE 2012, AIR 134)

4.2 Shear Modulus


The shear modulus of a solid measures its resistance to a shearing force, which is a A
 x
force applied tangentially to a surface, as shown in the Fig. 8.42. (Since the bottom of
the solid is assumed to be at rest, there is an equal and opposite force on the lower
surface). The top surface is displaced by x relative to the bottom surface. y
Tangential force F
The shear stress is defined as, Shear stress = = τ = t where A is the 
area of the surface. Area A F
Figure 8.42: Shearing
x
The shear strain is defined as Shear strain = stress
y
where y is the separation between the top and the bottom surfaces.
The shear modulus G is defined as
Shear Stress Ft / A F y
Shear modulus = ; =G =
Shear Strain x / y Ax

8 . 8 0 | Simple Harmonic Motion and Elasticity

Illustration 11: A box shaped piece of gelatin dessert has a top area of 15 cm2 and a height of 3cm. When a
shearing force of 0.50 N is applied to the upper surface, the upper surface displaces 4 mm relative to the bottom
surface. What are the shearing stress, the shearing strain and the shear modulus for the gelatin? (JEE MAIN)

Sol: Shearing stress is tangential force per unit area of surface. Shearing Strain is the ratio of displacement of
the surface to the distance of the surface from the fixed surface. Shear modulus is the ratio of shearing stress to
shearing strain.

tangential force 0.50N


Shear stress = = = 333 N/m2
area of face −4 2
15 ×10 m

Displacement 0.4 cm
Shear stress = = = 0.133
height 3cm

stress 333
Shear modulus G = =
strain 0.133

= 2.5 × 103 N/m2 (1 Pa = 1 N/m2)

4.3 Bulk Modulus


The bulk modulus of a solid or a fluid indicates its resistance to a change in volume. Fn
Consider a cube of some material, solid or fluid, as shown in the Fig. 8.43. We
assume that all faces experience the same force Fn normal to each face. (One way to
accomplish this is to immerse the body in a fluid-as long as the change in pressure
over the vertical height of the cube is negligible). The pressure on the cube is fn
Fn
F Fn Fn
defined as the normal force per unit area p = n
A
The SI unit of pressure is N/m2 and is given the name Pascal (Pa).
Fn
The change in pressure ∆P is called the volume stress and the fractional change
in volume ∆V / V called the volume strain. The bulk modulus B of the material is Figure 8.43: Determination
defined as of bulk modulus of an
object
Volumestress −∆P
Bulk modulus = or B=
Volume strain ∆V / V

The negative sign is included to make B a positive number since an increase in pressure ( ∆p > 0 ) leads to a
decrease in volume ( ∆V < 0 ) .
1
The inverse of B is called the compressibility factor k =
B
Elastic properties of matter
Sate Shear Modulus Bulk Modulus
Solid Large Large
Liquid Zero Large
Gas Zero Small
P hysi cs | 8.81

PLANCESS CONCEPTS

Bulk Modulus has very important applications in case of fluids. Actually, it has various applications in
adiabatic expansion of gases. Also, while calculating speed of sound through air, one would find that
it would come out to be directly proportional to square root of bulk modulus of air. (In general, speed
of sound depends of elastic properties of matter. A more general statement is that mechanical waves’
speed depends on elastic properties of matter)
B Rajiv Reddy (JEE 2012, AIR 11)

Illustration 12: Find the decrease in the volume of a sample of water from the following data. Initial volume = 1000
cm3, initial pressure = 105 Nm–2, final pressure = 106 Nm–2, compressibility of water = 50 × 10–11 m2N–1. (JEE MAIN)
Sol: Using the formula for bulk modulus deduce the value for decrease in volume.
The change in pressure = ∆P = 106 Nm–2 – 105 Nm–2 = 9 × 105 Nm–2.
1 ∆V / V
Compressibility = = − or,
Bulk modulus ∆P
∆V
50 × 10–11 m2N–1 = –
(10 m ) × (9 ×105 Nm−2 )
−3 3

or, ∆V = – 50 × 10–11 × 10–3 × 9 × 105 m3 = – 4.5 × 10–7 m3 = – 0.45 cm3.


Thus the decrease in volume is 0.45 cm3.

PLANCESS CONCEPTS

A solid will have all the three moduli of elasticity Y, B and η. But in case of a liquid or a gas, only B can
be defined because a liquid or a gas cannot be framed into a wire or no shear force can be applied on
 −dP 
them. For a liquid or a gas, B =  
 dV / V 
So, instead of P, we are more interested in change in pressure dP.
In case of a gas, B = XP
Anand K (JEE 2011, AIR 47)

Elastic
5. THE STRESS-STRAIN CURVE zone Plastic zone

The stress-strain graph of a ductile metal is shown in Fig. 8.44 B U


10
Initially, the strain graph is linear and it obeys the Hooke’s Law up
B
to the point P called the proportional limit. After the proportional P
Y
limit, the σ − ε graph is non-linear but it still remains elastic up Stress()
to the yield point Y where the slope of the curve is zero. At -2
(Nm )
the yield point, the material starts deforming under constant
stress and it behaves like a viscous liquid. The yield point is the
beginning of the plastic zone. After the yield point, the material
starts gaining strength due to excessive deformation and this
phenomenon is called strain hardening. The point U shows the 0.01 0.2 Strain()

Figure 8.44
8 . 8 2 | Simple Harmonic Motion and Elasticity

ultimate strength of the material. It is the maximum stress that the material can sustain without failure. After the
point U the curve goes down towards the breaking point B because the calculation of the stress is based on the
original cross-sectional area whereas the cross-sectional area of the sample actually decreases.

PLANCESS CONCEPTS

It is generally thought that strain results from stress, or many say that Hooke’s law states wrong statement
that stress is directly proportional to strain.
However, we must not worry because Hooke’s law is correct. Going deeper to a microscopic level will
help us understand better. It appears that external force cause strain in the body on which it is applied.
However, stress is defined as restoring force (at equilibrium) per unit area. There can be no restoring
force if there is no strain. Hence, strain is the cause and not stress. The only glitch here is that restoring
force is equal to the force applied because (again not to forget) body is in equilibrium. So, it creates
confusion but we must not take it for granted and understand the minute concepts.
Yashwanth Sandupatla (JEE 2012, AIR 821)

6. RELATION BETWEEN LONGITUDINAL STRESS AND STRAIN


For small deformations, longitudinal stress is directly
proportional to the longitudinal strain. What if the d
deformation is large? The stress-strain relation gets more b c
complicated in that case and depends on the material a Plastic behavior
under study. Let’s take a metal wire and a rubber piece as
example and study the same. Stress

Metal Wire: The Fig. 8.45 shows the relation between Elastic behavior
stress and strain as the deformation gradually decreases
in a stretched wire.
Up to a strain < 0.01, Hooke’s law is valid and Young’s


Strain
modulus is defined. Point a represents proportional limit 0.01 0.30
up to which stress is proportional to strain. Permanent set
Point b is called the yield point or elastic limit up to which a=Proportional limit
stress is not proportional to strain (a to b) but elasticity still b=Elastic limit
holds true. d=Fracture point
The wire shows plastic behavior after point b where there Figure 8.45: Graph of Stress versus Strain
is a permanent deformation in the wire and it does not
return back to its original dimensions.
The wire breaks at d which is the fracture point if stretched beyond point c. The corresponding stress is called
breaking stress.

PLANCESS CONCEPTS

If large deformation takes place between the elastic limit and the fractured point, the material is called
ductile. If it breaks soon after the elastic limit is crossed, it is called brittle.
Yashwanth Sandupatla (JEE 2012, AIR 821)
P hysi cs | 8.83

Rubber: Vulcanized rubber shows a very different stress-strain behavior. It


remains elastic even if it is stretched to 8 times its original length. There are
2 important phenomena to note from the above Fig 8.46. Firstly, stress is
nowhere proportional to strain during deformation. Secondly, when external Strain
forces are removed, body comes back to original dimensions but it follows
a different retracing path.
The work done by the material in returning to its original shape is less than
the work done by the deforming force when it was deformed. A particular 2 4 6 8 Strain
amount of energy is, thus, absorbed by the material in the cycle which Figure 8.46: Stress versus Strain curve
appears as heat. This phenomenon is called elastic hysteresis. for rubber
Elastic hysteresis has an important application in shock absorbers.

PLANCESS CONCEPTS

The material which has smaller value of Y is more ductile, i.e., it offers less resistance in framing it into
a wire. Similarly, the material having the smaller value of B is more malleable. Thus, for making wire, we
choose a material having less value of Y.
GV Abhinav (JEE 2012, AIR 329)

7. POISSON’S RATIO
When a longitudinal force is applied on a wire, its length increases but its radius decreases. Thus two strains are
produced by a single force.
∆l ∆R
(a) Longitudinal strain = and (b) Lateral strain =
l R
The ratio of these two strains is called the Poisson’s ratio.

Lateral strain ∆R / R
Thus, the Poisson’s ratio σ = =−
Longitudinal strain ∆l / l

Negative sign in σ indicates that radius of the wire decreases as the length increases.

PLANCESS CONCEPTS

Relation between Y, B, η and σ : Following are some relations between the four
Y Y 3B − 2η 9 1 3
(a) B = (b) η = (c) σ = (d) = +
3(1 − 3σ) 2(1 + σ) 2η + 6B Y B η
Anurag Saraf (JEE 2011, AIR 226)

8. ELASTIC POTENTIAL ENERGY OF A STRAINED BODY


When a body is in its natural shape, potential energy due to molecular forces is minimum and assumed to be zero.
When deformed, internal forces come into existence and work is done against these forces. Thus potential energy
of the body increases. This is called elastic potential energy.
8 . 8 4 | Simple Harmonic Motion and Elasticity

8.1 Work Done in Stretching a Wire


If a force F is applied along the length of a wire of length l, area of cross-section A and Young’s modulus Y, such
Fl YAx
that the wire is extended through a small length x, then Y = or F =
Ax l
The work done, W, in extending the wire through length Dl is given by
∆l ∆l
YA YA( ∆l )2 1  Y∆l   ∆l  1
= ∫ Fdx    ( Al ) = × stress × strain × volume
l ∫0
W= = xdx = 
0
2l 2  l  l  2

1  Y A∆l  1
Also W =   ∆l = × force × extension
2 l  2

This work is stored in the wire as elastic potential energy.

1  Y∆l  ∆l 1 1
Y × ( strain)

Work done per unit volume ==  × × = x stress x strain .
2 l  l 2 2

Illustration 13: Spring is stretched by 3 cm when a load of 5.4 × 106 dyne is suspended from it. Work done will be
(A) 8.1 × 106 J (B) 8 × 106 J (C) 8.0 × 106 ergs (D) 8.1 × 106 ergs  (JEE MAIN)

Sol: Work done in stretching the spring is equal to the elastic potential energy stored in the spring.
1
(D) W = × load × elongation W = 8.1 × 106 ergs =0.81 J
2

Illustration 14: A steel wire of length 2.0 m is stretched through 2.0 mm. The cross-sectional area of the wire is
4.0 mm2. Calculate the elastic potential energy stored in the wire in the stretched condition. Young modulus of steel
= 2.0 × 1011 N/m2. (JEE MAIN)

Sol: We know the formula to find the elastic potential energy stored per unit volume of the wire. Calculate the
volume of the wire and find the energy stored in the entire wire.
∆l 2.0mm
The strain in the wire = = 10–3.
l 2.0m
The stress in the wire = Y × strain = 2.0 × 1011 N m–2 × 10–3 = 2.0 × 108 N/m2.
The volume of the wire = (4 × 10–6 m2) × (2.0 m) = 8.0 × 10–6 m3.
1
The elastic potential energy stored = × stress × strain × volume
2
1
= × 2.0 × 108 Nm–2 × 10–3 × 8.0 × 10–6 m3 = 0.8 J
2

PLANCESS CONCEPTS

This energy can also be thought of as elastic potential energy of a spring. You just need to calculate
spring constant.
A simple way would be considering ∆l=x and rearranging terms of Hooke’s law in the form of F=-kx.
1
Remember F here is restoring force. Now energy is simply kx2
2
Vijay Senapathi (JEE 2011, AIR 71)
P hysi cs | 8.85

9. THERMAL STRESS AND STRAIN


A body expands or contracts whenever there is an increase or decrease in
l,  , Y, A
temperature. No stress is induced when the body is allowed to expand and
contract freely. But when deformation is obstructed, stresses are induced.
A B
Such stresses are called thermal/ temperature stresses. The corresponding
strains are called thermal/temperature strains.
l,  , Y, A
Consider a rod AB fixed at two supports as shown in Fig. 8.47.
B
Let l = Length of rod
l
A = Area of cross-section of the rod Figure 8.47: Thermal expansion
Y = Young’s modulus of elasticity of the rod of a rod

And α = Thermal coefficient of linear expansion of the rod


Let the temperature of the rod is increased by an amount t. The length of the rod would had increased by an
amount Dl, if it were not fixed at two supports. Hence Dl = lat
But since the rod is fixed at the supports, a compressive strain will be produced in the rod. Because at the increased
temperature, the natural length of the rod is l + Dl, while being fixed at two supports its actual length is l.
∆l l α t
Hence, thermal strain ε = = = αt or ε = at
l l
F F F F
Therefore, thermal stress σ = Yε (stress = Y × strain)
or σ = Yat or force on the supports, F = σA = YAat
This force F is in the direction shown: Figure 8.48: Thermal stress on
a rod

Illustration 16: A wire of cross sectional area 3 mm2 is just stretched between two fixed points at a temperature of
20°C. Determine the tension when the temperature falls to 20°C. Coefficient of linear expansion α = 10–5 / °C and
Y = 2 × 1011 N/m2.  (JEE MAIN)
(A) 120 kN (B) 20 N (C) 120 N (D) 12 N

Sol: Thermal stress is equal to product of young’s modulus and thermal strain. Tension is product of area of cross-
section and stress.
(C) F = Y A α Dt = 2 × 1011 × 3 × 10–6 × 10–5 × 20; F = 120 N

10. DETERMINATION OF YOUNG’S MODULUS IN LABORATORY


The given Fig. 8.49 shows an experimental set up of a simple method to determine
Young’s modulus in laboratory. A 2-3 metres long wire is suspended from a fixed  
support. It carries a graduated scale and below it a heavy fixed load. This load
keeps the wire straight. Wire A is the reference wire whereas wire B serves as the
experimental wire. A Vernier scale is placed at the end of the experimental wire. A B

Now the stress due to the weight Mg at the end is


Mg l MgL
Stress = and strain = ; Thus, Y=
πr 2 L π r2 l
All the quantities on the right-hand side are known and hence Young’s modulus Y
may be calculated.

Figure 8.49: Searle's method for


determination of young's modulus
8 . 8 6 | Simple Harmonic Motion and Elasticity

PROBLEM-SOLVING TACTICS

•• Be careful while using the Hooke’s law of elasticity. Always remember that this law is not valid for an elastic
material when it is stretched beyond its elastic limit. Stress is proportional to strain only when the material is
stretched up to a certain limit.
•• Always keep the stress-strain graph in mind while solving elasticity problems.
•• The extent of ductility of a material can be calculated using the strain formulae. Greater the elongation, greater
the ductility of the material. This concept can be used in questions where one is asked to arrange the elastic
material in the order of increasing brittleness or ductility.
•• Conservation of energy principle can be used to solved many problems where elastic potential energy gets
converted to other forms of energy in the given problem system.
•• Elongation and compression can be thought as analogous to a spring (refer to Plancess concept to how to do
it) in appropriate limits.
•• Direct questions may be asked on relation between Poisson’s ratio and modulus of elasticity, so it would be
nice if you learn them.

FORMULAE SHEET

Elasticity:

Restoring force
Stress: Stress (σ) =
Area
SI units = N/m2
Fn
Normal/ longitudinal stress sn =
A Ft
Fn is the normal force
Fn
A is the cross-sectional area
Ft
Tangential / shearing stress st =
A
Figure 8.50
Ft is the tangential force
F
Volume stress sV =
A
Note: This is the stress developed when body is immersed in a liquid.
∆l
Strain: Longitudinal strain ε= F x
l A B A A’ B B’
∆V
Volumetric strain ε=
V x
F F
D and DV are change in length and volume 
respectively.
D C D C
∆X F
Shearing strain ε=
X Figure 8.51
P hysi cs | 8.87

Hooke’s Law: stress ∝ strain


stress = (E) (strain) (E is modulus of elasticity)
E is constant for a particular type of strain for a particular material. SI unit of E is N/m2.

longitudinal stress (Fn / A)


Young’s modulus of
= elasticity (Y) Y =
Longitudinal strain ( ∆l / l )

−Volumestress −F / A −P
Bulk modulus of elasticity (B) B= = =
Volume strain ∆V / V ∆V / V

dp
For a liquid or gas B= −
(dV / V)

1
Compressibility =  
β
shearing stress F /A F /A
Modulus of rigidity (η) η = = t = t (See Fig. 8.52)
shearing strain ( ∆x / x) θ
l,  , y, A
Elastic potential energy stored per unit volume in a stretched wire A B
1
u = (stress x strain)
2 Figure 8.52
∆l
Thermal stress and strain ∈= = α∆T
l
σ F F
Y= = =
∈ A ∈ Aα∆T

α is thermal coefficient of linear expansion of rod. DT is change in temperature of the rod.


Variation of density with pressure: As pressure on a body increases, its density also increases. When pressure
ρ
increases by dp, the new density ρ’ in terms of the previous density ρ is ρ ' = where B is the Bulk modulus.
dp
1−
B
Poisson’s ratio: As the length of a wire of circular cross-section increases, its radius decreases.
lateral strain ∆R / R
Poisson’s ratio is defined as σ= = −
longitudinal strain ∆l / l
Relation between Y, B, η and σ
Y 3B − 2η Y 9 1 3
B= ; σ= ; η= ; = +
3(1 − 2σ) 2η + 6B 2(1 + σ) Y B η
8 . 8 8 | Simple Harmonic Motion and Elasticity

Solved Examples

JEE Main/Boards Find


(a) The energy density of wire,
Example 1: A steel wire of length 4 m and diameter
(b) The elastic potential energy stored in the wire.
5 mm is stretched by 5 kg-wt. Find the increase
in its length, if the Young’s modulus of steel is
Sol: Find the stress and strain and use the formula for
2.4 × 1012 dyne/cm2.
energy density. Product of energy density and volume
is energy stored in entire wire.
Sol: From the formula for Young’s modulus deduce the
change in length. Here, l = 4.0 m, Dl = 2 × 10–3 m,
Here, l = 4 m = 400 cm, 2r = 5 mm A = 2.0 × 10–6 m2, Y = 2.0 × 1011 N/m2
or r = 2.5 mm = 5mm (a) The energy density of stretched wire
F = 5 kg-wt = 5000 g-wt = 5000 × 980 dyne 1 1
U= × stress × strain = × Y × (strain)2
Dl = ?, Y = 2.4 × 1012 dyne/cm2 2 2
2
1  (2 ×10−3 
F l = × 2.0 × 1011 ×  
As=
Y × 2  4 
πr 2 ∆l  
= 0.25 × 105 = 2.5 × 104 J/m3.
(5000 × 980) × 400
∆l = = 0.0041 cm
(22 / 7) × (0.25)2 × 2.4 ×1012 (b) Elastic potential energy = energy density × volume=
2.5 × 104 × (2.0 × 10–6) × 4.0 J
∆l = 4.1 × 10-5 m = 20 × 10–2 = 0.20 J

Example 2: One end of a wire 2 m long and 0.2 cm2 in Example 4: The bulk modulus of water is
cross section is fixed in a ceiling and a load of 4.8 kg is
attached to the free end. Find the extension of the wire. 2.3 × 109 N/m2.
Young’s modulus of steel = 2.0 × 1011 N/m2. (a) Find its compressibility.
Take g = 10 m/s . 2
(b) How much pressure in atmosphere is needed to
compress a sample of water by 0.1%?
Sol: From the formula for Young’s modulus deduce the
extension in wire. Sol: Compressibility is inverse of bulk modulus. From
stress T / A the formula for bulk modulus deduce the change
We have
= Y = in pressure required to produce the given change in
strain l / L
volume.
With symbols having their usual meanings. The
TL Here, B = 2.3 × 109 N/m2
extension is l =
AY 2.3 ×109
= atm = 2.27 × 104 atm
As the load is in equilibrium after the extension, the 1.01 ×105

tension in the wire is equal to the weight of the load 1
= 4.8 kg × 10 ms–2 = 48 N (a) Compressibility =
B
(48N)(2m) 1
Thus, l = = = 4.4 × 10–5 atm–1
−4 2 11 −2
(0.2 ×10 m ) × (2.0 ×10 Nm ) 2.27 ×10 4
∆V
= 2.4 × 10–5 m. (b) Here, = −0.1% = – 0.001
V

Example 3: A steel wire 4.0 m in length is stretched Required increase in pressure,


through 2.0 mm. The cross-sectional area of the wire is
2.0 mm2. If Young’s modulus of steel is 2.0 × 1011 N/m2.
P hysi cs | 8.89

 ∆V  = – 0.87 × 10–3.
DP = B ×  − 
 V  The negative sign indicates that the radius decreases.
= 2.27 × 104 × 0.001= 22.7 atm
Example 7: Find the greatest length of steel wire that
can hang vertically without breaking. Breaking stress of
Example 5: One end of a nylon rope of length 4.5
steel =8.0 × 108 N/m2.
m and diameter 6 mm is fixed to a tree-limb. A
monkey weighing 100 N jumps to catch the free end Density of steel = 8.0 × 103 kg/m3.
and stays there. Find the elongation of the rope and Take g = 10 m/s2.
the corresponding change in the diameter. Young’s
modulus of nylon = 4.8 × 1011 Nm–2 and Poisson ratio Sol: Breaking stress gives the maximum weight per unit
of nylon = 0.2. area of cross-section that the wire can withstand.

Sol: From the formula for Young’s modulus deduce Let l be the length of the wire that can hang vertically
the change in length of the rope. From the formula for without breaking. Then the stretching force on it is
Poisson ratio deduce the change in diameter. equal to its own weight. If therefore, A is the area of
cross-section and ρ is the density, then
As the monkey stays in equilibrium, the tension in the
rope equals the weight of the monkey. Hence, weight
Maximum stress (sm) =
A
stress T / A TL
Y= = or l =
strain l / L AY  force  (Alρ)g
 stress =  or σm =
or, elongation  area  A
(100 N) × (4.5 m) σm
l= \ l= Substituting the values
−6 2 11 −2
( π× 9 ×10 m ) × (4.8 ×10 Nm ) ρg
= 3.32 × 10–5 m 8.0 ×108
l= = 104 m
(8.0 ×103 )(10)
∆d / d ( ∆d)L
Again, Poisson ratio = =
l /L ld
Example 8: A copper wire of negligible mass, length 1
∆d × 4.5 m m and cross-sectional area 10–6 m2 is kept on a smooth
or, 0.2 =
(3.32 × 10−5 m) × (6 × 10−3 m) horizontal table with one end fixed. A ball of mass 1
kg is attached to the other end. The wire and the ball
0.2 × 6 × 3.32 ×10−8 m are rotating with an angular velocity of 20 rad/s. If the
or, Dd = = 8.8 × 10–9 m
4.5 elongation in the wire is 10–3 m, obtain the Young’s
modulus of copper. If on increasing the angular velocity
Example 6: A solid lead sphere of volume 0.5 m3 to 100 rad/s, the wire breaks down, obtain the breaking
is taken in the ocean to a depth where the water stress.
pressure is 2 × 107 N/m2. If the bulk modulus of lead is
7.7 × 109 N/m2. Find the fractional change in the radius Sol: The stress developed in the wire will be due to
of the sphere. the centrifugal force. Ratio of stress and strain is the
Young’s modulus. The breaking stress will be due to the
Sol: From the formula for bulk modulus deduce the centrifugal force at increased angular velocity.
change in volume for the given increase in pressure.
The stretching force developed in the wire due to
4 3 ∆r 1 ∆V rotation of the ball is
V= πr ⇒ =
3 r 3 V
F = mrw2 = 1 × 1 × (20)2 = 400 N
∆P
Bulk modulus K = – F 400
( ∆V / V) Stress in the wire = = N/m2 Strain in the wire
A 10−6
∆V ∆P
or =– 10−3
V K = = 10–3
1
∆r 1 ∆P 1 2 ×107
or = − =– × Stress 400
r 3 K 3 7.7 ×109 Y= = = 4 × 1011 N/m2
Strain 10 ×10−3
−6
8 . 9 0 | Simple Harmonic Motion and Elasticity

1 × 1 × (100)2 C
Breaking stress = = 1010 N/m2. A B
−6
10

T T
Example 9: (a) A wire 4 m long and 0.3 mm in diameter  
is stretched by a force of 100 N. If extension in the wire D
is 0.3 mm, calculate the potential energy stored in the m
wire.
(b) Find the work done in stretching a wire of cross- mg
section 1 mm2 and length 2 m through 0.1 mm, Young’s ∴ AD2 = AC2 + CD2 = (50)2 + (1)2
modulus for the material of wire is 2.0×1011 N/m2.
or AD = 50.01 cm
Sol: Work done in stretching the wire is equal to the Increase in length = 0.01 cm
elastic potential energy stored in the wire. (a) Energy
stored 0.01
Strain = = 2 ×10−4
50
1
U= (stress)(strain)(volume) Stress = 2 × 1012 × 2 × 10–4
2
1  F   ∆l  1 ∴ Stress = 4 × 108 dynes/cm2
or U =     (Al ) = F ⋅∆l
2  A  l  2 Tension T = Stress × Area of cross-section
= 4 × 108 × π × (0.08)2
=
1
2
( ( )
100 ) 0.3 ×10−3 = 0.015 J
Since the mass m is in equilibrium
(b) Work done = potential energy stored 2T cos θ
mg = 2T cos θ or m =
g
1 1  YA   YA  2 × 4 ×108 × π(0.08)2 × (1 / 50.01)
 ( ∆l )
2
= k( ∆l )2 =   as k =  = = 82 gm.
2 2 l   l  980
Substituting the values, we have

1 (2.0 ×1011 )(10−6 )


JEE Advanced/Boards
W= (0.1 × 10–3)3
2 (2)
Example 1: A light of rod of length 200 cm is suspended
= 5.0 × 10 J –4 from the ceiling horizontally by means of two vertical
wires of equal length tied to its ends. One of the wires
is made of steel and is of cross-section 0.1 sq cm
Example 10: A steel wire of diameter 0.8 mm and
and the other is of brass of cross-section 0.2 sq. cm.
length 1 m is clamped firmly at two points A and B
Find the position along the rod at which a weight may
which are 1 m apart and in the same horizontal plane.
be hung to produce (a) equal stresses in both wires and
A body is hung from the middle point of the wire such
(b) equal strains in both wires.
that the middle point sags 1 cm lower from the original
position. Calculate the mass of the body. Given Young’s (Ybrass = 10 × 1011 dynes/cm2.
modulus of the material of wire = 2 × 1012 dynes/cm2.
Ysteel = 20 × 1011 dynes/cm2).
Sol: Tension in the wire is the product of stress and
Sol: Net torque of the tensions in the wires about the
area of cross-section. Stress is the product of Young’s
point of suspension of the weight on the rod must be
modulus and strain. The vertical components of
zero.
tensions in the two parts of the wire will balance the
weight of the body hung from the wire. Steel Brass
wire wire
Let the body be hung from the middle point C so that it
sags through 1cm to the point D as shown in the figure. T1 T2

C
A B
x (200 x)
P hysi cs | 8.91

Let AB be the rod and let C be the point at which the A A’


weight is hung.
l1=0.1m
T1
(a) Stress in steel wire = B
0.1 B’
/A1
l2=0.2m
T2
Stress in brass wire =
0.2
C
As the two stresses are equal,
l3=0.15m l1+l2
T1 T2 T1
= or = 0.5  … (i) C’
0.1 0.2 T2 D

Taking moments about C,


l1+l2+l3
T1 200 − x
T1 = T2 (200 – x) or =  … (ii)
T2 x D’
Equations (i) and (ii) give
200 − x
= 0.5 For segment BC:
x
10 × 9.8 × 0.2
Or x = 133.3 cm = 1.33 m ∆l2 = = 4.9 × 10–6 m
10−4 × 4.0 ×1010
Stress T
(b) Strain = = Displacement of C: = Dl1 + Dl2
Y AY
As the strain in both wires are equal, = 4.9 × 10–6 m
T1 T2 T1 A1 Y1 0.1 × 20 ×1011 For segment CD:
= or = =
A1 Y1 A2 Y2 T2 A2 Y2 0.2 × 10 ×1011
10 × 9.8 × 0.15
∆l3 = =14.7 × 10–6 m
∴ T1 = T2 10−4 ×1.0 ×1010

Now, T1x = T2 (200 – x) ⇒ x = 200 – x Displacement of D = Dl1 + Dl2 + Dl3


or x = 100 cm =1 m. = 23.52 × 10–6 m.

Example 2: A rod AD, consisting of three segments AB, Example 3: A steel rod of length 6.0 m and diameter
BC and CD joined together, is hanging vertically from 20 mm is fixed between two rigid supports. Determine
a fixed support at A. The lengths of the segments are the stress in the rod, when the temperature increases
respectively 0.1 m, 0.2 m and 0.15 m. The cross-section by 80° C if
of the rod is uniformly equal to 10–4 m2. A weight of 10
kg is hung from D. Calculate the displacements of the
points B, C and D using the data on Young’s moduli
given below (neglect the weight of the rod). 6.0m
YAB = 2.5 × 10 N/m, 2
(a) The ends do not yield
YBC = 4.0 × 10 N/m and 2
(b) The ends yield by 1 mm.
YCD = 1.0 × 10 N/m 2
Take Y = 2.0 × 106 kg/cm2

Sol: From the formula for Young’s modulus deduce the And α = 12 × 10–6 per °C.
elongation in each segment of the wire.
Sol: Rise in temperature causes thermal strain and
We know that thermal stress. Use the formula for coefficient of
mgl 10 × 9.8 × 0.1 thermal expansion to obtain thermal strain. Thermal

=l = = 3.92 × 10–6 m stress is the product of Young’s modulus and thermal
AY 10 −4 × 2.5 ×1010
strain.
This is the displacement of B.
Given, length of the rod l = 6 m
8 . 9 2 | Simple Harmonic Motion and Elasticity

Diameter of the rod d = 20 mm = 2 cm Sol: Find the tension in the metal wire due to the masses
connected to it. The stress due to tension should not
Increase in temperature t = 80°C
exceed the breaking stress.
Young’s modulus Y = 2.0 × 106 kg/cm2
The stress in the wire
And thermal coefficient of linear expansion
Tension
=
α = 12 × 10–6 per °C Area of crosssec tion
(a) When the ends do not yield
To avoid breaking, this stress should not exceed the
Let, s1 = stress in the rod breaking stress.
Using the relation σ = atY Let the tension in the wire be T. The equations of motion
of the two blocks are,
∴ s1 = (12 × 10–6) (80) (2 × 106)
T – 10 N = (1kg) a and 20 N – T = (2kg) a
= 1920 kg/cm2 =19.2 × 106 N
Eliminating a from these equations,
(b) When the ends yield by 1 mm
T = (40/3) N
Increase in length due to increase in temperature Dl =
lat (40 / 3) N
The stress =
Of this 1mm or 0.1 cm is allowed to expand. Therefore, πr 2
net compression in the rod If the minimum radius needed to avoid breaking is r,
Dlnet = (lt – 0.1) N (40 / 3) N
2 × 109 =
2
or compressive strain in the rod, m πr 2

Solving this, r = 4.6 × 10–5 m.


∆lnet  0.1 
ε= =  αt − 
l  l 
Example 5: A steel rod of cross-sectional area 16 cm2
 0.1  and two brass rods each of cross-sectional area 10 cm2
\ Stress s2 = Yε = Y  αt −  together support a load of 5000 kg as shown in figure
 l 
Find the stress in the rods. Take Y for steel = 2.0 × 106
Substituting the values, kg/cm2 and for brass = 1.0 × 106 kg/cm

 0.1 
s2 = 2 × 106  12 ×10−6 × 80 − 
5000kg
 600 
Brass
= 1587 kg/cm2 = 15.8 × 106 N Brass Steel 30m 20cm

Example 4: Two blocks of masses 1 kg and 2 kg are


connected by a metal wire going over a smooth pulley
as shown in figure The breaking stress of the metal is
2 × 109 Nm–2. What should be the minimum radius of
the wire used if it is not to break?
Take g = 10 ms–2.
Sol: Compression in the length of steel and brass rods
is equal. From the formula for Young’s modulus deduce
the compression in length of each rod and equate them
to get the relation between respective stresses.
Given area of steel rod AS = 16 cm2
Area of two brass rods
1 kg
AB = 2 × 10 = 20 cm2

2 kg Load, F = 5000 kg
Y for steel YS = 2.0×106 kg/cm2 Y
P hysi cs | 8.93

for brass YB = 1.0×106 kg/cm2 l + Dl + 2r = 5.22


Length of steel rod lS = 30 cm or Dl = 5.22 – l – 2r
Length of steel rod lB = 20 cm 5.22 – 5 – 2 × 0.1 = 0.02 m
Let sS = stress in steel
Let T be the tension in the wire at mean position during
and σB = stress in brass T/A
oscillations, Y =
Decrease in length of steel rod = decrease in length of ∆l / l
brass rod YA∆l Yπr 2 ∆l
∴ T= =
σS σB l l
or × lS = × lB
YS YB Substituting the values, we have
YS lB (1.994 ×1011 ) × π× (0.5 × 10−3 )2 × 0.02
or sS = × ×σB T=
YB lS 5
= 626.43 N
2.0 ×106 20
= × ×σB
1.0 ×106 30 The equation of motion at mean position is,
mv 2
4 T – mg =
\ sS = σ  …. (i) R
3 B
Hence, R = 5.22 – r = 5.22 – 0.1 = 5.12 m
Now, using the relation,
and m = 8π kg = 25.13 kg
F = ssAs + σBAB or
Substituting the proper values in Eq. (i), we have
5000 = sS × 16 + σB × 20  …. (ii)
(25.13)v 2
Solving eq. (i) and (ii), we get (626.43) – (25.13 × 9.8) =
5.12
σB =120.9 kg/cm2 and sS =161.2 kg/cm2 Solving this equation, we get V = 8.8 m/s

Example 6: A sphere of radius 0.1 m and mass 8π kg Example 7: A thin ring of radius R is made of a material
is attached to the lower end of a steel wire length 5.0 of density ρ and Young’s modulus Y. If the ring is rotated
m and diameter 10–3 m. The wire is suspended from about its center in its own plane with angular velocity
5.22 m high ceiling of a room. When sphere is made ω, find the small increase in its radius.
to swing as a simple pendulum, it just grazes the floor
at its lowest point. Calculate velocity of the sphere at Sol: As the ring rotates each element of the ring of
the lowest position. Young’s modulus of steel is 1.994 infinitesimal length experiences a centrifugal force, due
× 1011 N/m2. to which the ring slightly expands, thus increasing its
radius. The longitudinal strain in the ring produces a
Sol: The elongation in the wire is known, thus the tensile stress or tension in the ring.
corresponding stress can be calculated. The stress in
turn gives the tension in the wire. At the lowest point Consider an element PQ of length dl. Let T be the
the net acceleration of the sphere is centripetal, i.e. tension and A the area of cross-section of the wire.
directed vertically upwards. Apply Newton’s second law Mass of element dm = volume × density = A (dl)ρ
at the lowest point to find the speed of the sphere.
The component of T, towards the center provides the
Let Dl be the extension of wire when the sphere is at necessary centripetal force
mean position. Then, we have
θ 2
\   (dm)Rω 
2T sin= … (i)
2
θ θ (dl / R)
For small angles sin ≈ =
2 2 2
5.22m T Substituting in eq. (i), we have
dl
T. = A(dl )ρRω2 or T = Arw2R2
R
8 . 9 4 | Simple Harmonic Motion and Elasticity

Let DR be the increase in radius, Area of part AB, A2 = 25 cm2


Longitudinal strain Area of part CD, A3 = 12. 5 cm2
∆l ∆(2πR) ∆R Length of part AB, l1 = 120 cm
= =
l 2πR R Length of part BC, l2 = 60 cm
Length of part CD, l3 = 90 cm
Tcos(/2) Tcos(/2)
(/2) (/2) Young’s modulus of elasticity
Y = 2.1 × 106 kg/cm2
F
 Magnitude of the force F2 for equilibrium

O The magnitude of force F2 may be found by equating


T T the forces acting towards right to those acting towards
left,
F2 + F4 = F1 + F3
T/A F2 + 13000 = 4500 + 45000
Now, Y =
∆R / R \ F2 = 36500 kg
TR (Aρω2R 2 )R ρω2R3
∴ ∆R= = or DR =
AY AY Y Total Elongation of the member
For the sake of simplicity, the force of 36500 kg (acting
Example 8: A member ABCD is subjected to point at B) may be split up into two forces of 4500 kg and
loads F1, F2, F3 and F4 as shown in figure Calculate the 32000 kg. The force of 45000 kg acting at C may be split
force F2 for equilibrium if F1 = 4500 kg, into two forces of 32000 kg and 13000 kg. Now, it will
F3 = 45000 kg and F4 =13000 kg. be seen that the part AB of the member is subjected
to a tension of 4500 kg, part BC is subjected to a
Determine the total elongation of the member, compression of 32000 kg and part CD is subjected to a
assuming modulus of elasticity to be 2.1 × 106 kg/cm2. tension of 13,000 kg. Using the relation.
2
25cm 1 F l F l F l 
∆l =  1 1 − 2 2 + 3 3 
Y  A1 A2 A3 
2
6.25cm B C
A D
F1 F2 F3 F4 With usual notation
2 1
12.5cm ∆l = x
2.1 ×106
120cm 60cm 90cm  4500 ×120 32000 × 60 13000 × 90 
 − +  cm
Sol: Find the tension in each segment of the member  6.25 25 12.5 
ABCD. From the formula of Young’s modulus, find the = 0.049 cm or Dl = 0.49 mm
elongation in each segment.
Given
Area of part AB, A1 = 6.25 cm2
P hysi cs | 8.95

JEE Main/Boards

Exercise 1 Q.9 A metal cube of side 10 cm is subjected to a


shearing stress of 104 Nm–2. Calculate the modulus of
Q.1 A wire is replaced by another wire of same length rigidity if the top of the cube is displaced by 0.05 cm
and material but of twice diameter. with respect to its bottom.

(i) What will be the effect on the increase in its length


under a given load? Q.10 Calculate the increase in energy of a brass bar
of length 0.2 m and cross sectional area 1cm2 when
(ii) What will be the effect on the maximum load which combined with a load of 5kg weight along its length.
it can bear? Young’s modulus of brass = 1.0 × 1011 Nm–2 and g =
9.8 ms–2.
Q.2 Two wires are made of same metal. The length of the
first wire is half that of the second wire and its diameter Q.11 A wire 30m long and of 2 mm2 cross-section is
is double that of the second wire. If equal loads are stretched due to a 5kg-wt by 0.49 cm. Find
applied on both wires, find the ratio of increase in their
lengths. (i) The longitudinal strain
(ii) The longitudinal stress and
Q.3 The breaking force for a wire is F. What will be the (iii) Young’s modulus of the material of the wire.
breaking forces for
(i) Two parallel wires of this size and
Exercise 2
(ii) For a single wire of double thickness?
Single Correct Choice Type
Q.4 What force is required to stretch a steel wire 1 sq.
cm in cross section to double its length? Ysteel = 2 × 1011
Q.1 A wire of length 1m is stretched by a force of 10N.
Nm–2.
The area of cross-section of the wire is 2 × 10–6 m2 & γ
is 2 × 1011 N/m2. Increase in length of the wire will be-
Q.5 A structural steel rod has a radius of 10 mm and
length of 1.0 m. A 100 kN force stretches it along its (A) 2.5 × 10–5 cm (B) 2.5 × 10–5 mm
length. Calculate (a) stress, (b) elongation, and (c)% (C) 2.5 × 10–5 m (D) None
strain in the rod. Young’s modulus of structural steel is
2.0 × 1011 Nm–2.
Q.2 A uniform steel wire of density 7800 kg/m3 is
2.5 m long and weighs 15.6 × 10–3 kg. It extends by
Q.6 Find the maximum length of a steel wire that can 1.25 mm when loaded by 8kg. Calculate the value of
hang without breaking. young’s modulus for steel.
Breaking stress = 7.9×1012 dyne/cm2. (A) 1.96 × 1011 N/m2 (B) 19.6 × 1011 N/m2
Density of steel = 7.9 g/cc. (C) 196 × 1011 N/m2 (D) None

Q.7 A spherical ball contracts in volume by 0.01%, Q.3 The work done in increasing the length of a one
when subjected to a normal uniform pressure of 100 meter long wire of cross-sectional area 1mm2 through
atmosphere. Calculate the bulk modulus of the material. 1mm will be (Y = 2 × 1011 N/m2)
(A) 250 J (B) 10 J (C) 5 J (D) 0.1 J
Q.8 A sphere contracts in volume by 0.02% when taken
to the bottom of sea 1 km deep. Find bulk modulus
of the material of sphere. Density of sea water is
1000 kg/m3.
8 . 9 6 | Simple Harmonic Motion and Elasticity

Q.4 The lengths and radii of two wires of same material (C) Length=200cm, diameter= 2 mm
are respectively L, 2L, and 2R, R. Equal weights are (D) Length=300 cm, diameter=3 mm
applied on them. If the elongations produced in them
are l1 and l2 respectively, then their ratio will be
Q.2 A given quantity of an ideal gas is at pressure p and
(A) 2 : 1 (B) 4: 1 (C) 8 : 1 (D) 1 : 8 absolute temperature T. The isothermal bulk modulus
of the gas  (1998)
Q.5 What is the density of lead under a pressure of 2 3
(A) p (B) p (C) p (D) 2p
2.0×108 N/m2, if the bulk modulus of lead is 8.0×109 N/m2 3 2
and initially the density of lead is 11.4g/cm3 ?
Q.3 The pressure of a medium is changed from 1.01 ×
(A) 11.69g/cm3 (B) 11.92g/cm3
105 Pa to 1.165 ×105 Pa and change in volume is 10%
(C) 11.55g/cm3 (D) 11.862g/cm3 keeping temperature constant. The bulk modulus of
the medium is (2005)

Q.6 A rubber rod of density 1.3 ×103 kg / m3 and (A) 204.8 ×105 Pa (B) 102.4 ×105 Pa
Young’s modulus 6 ×106 N / m2 hangs from the ceiling (C) 51.2 ×105 Pa (D) 1.55 ×105 Pa
of a room. Calculate the deviation in the value of its
length from the original value 10m.
Q.4 A pendulum made of a uniform wire of cross
(A) 10.9 cm (B) 5.8 cm (C) 9.3 cm (D) 10.6 cm sectional area A has time period T. When an additional
mass M is added to its bob, the time period changes to
Q.7 A metal rod is trapped horizontally between two TM . If the Young’s modulus of the material of the wire
vertical walls. The coefficient of linear expansion of the 1
is Y then is equal to: (g = gravitational acceleration
rod is equal to 1.2 ×10−5 /  C and its Young’s modulus Y
2 ×1011 N / m2 . If the temperature of the rod is increased  (2015)
by 5°C, calculate the stress developed in it.   T 2    T 2 
Mg A
(A) 2.2×10 N/m
7 2
(B) 3.1 × 10 N/m
7 2 (A)  M  − 1 (B) 1 −  M  
 T   A   T   Mg
(C) 1.2 × 107 N/m2 (D) 1.2 × 104 N/m2    
  2    T 2 
T A A
(C) 1 −    (D)  M  − 1
  TM   Mg  T   Mg
Previous Years’ Questions    

Q.1 The following four wires are made of the same


material. Which of these will have the largest extension
when the same tension is applied? (1981)
(A) Length=50 cm, diameter=0.5 mm
(B) Length=100 cm, diameter = 1 mm

JEE Advanced/Boards

Exercise 1 Q.2 Calculate the pressure required to stop the increase


in volume of a copper block when it is heated from 50oC
to 70°C. Coefficient of linear expansion of copper = 8.0 ×
Q.1 A rubber cord has a cross-sectional area 1mm2 10‒6/°C and the bulk modulus of elasticity = 1011 N/m2.
and total unstretched length 10.0 cm. It is stretched to
12.0 cm and then released to project a missile of Q.3 Calculate the increase in energy of a brass bar of
mass 5.0g. Taking Young’s modulus Y for rubber as length 0.2m and cross-sectional area 1.0 cm2, when
5.0 ×108 N / m2 . Calculate the velocity of projection. compressed with a load of 5kg-weight along its
= 1.0 ×1011 N / m2 and
length. Young’s modulus of brass
g = 9.8 m/s2.
P hysi cs | 8.97

Exercise 2
4

Q.1 A steel wire of uniform cross-section of 2mm2 is 3


heated upto 50° and clamped rigidly at two ends. If the
temperature of wire falls to 30° then change in tension 2
in the wire will be, if coefficient of linear expansion of
steel is 1.1 × 10–5 /°C and young’s modulus of elasticity 1
of steel is 2 × 1011 N/m2.
W(N)
(A) 44 N (B) 88 N (C) 132 N (D) 22 N 20 40 60 80

Q.2 A metallic wire is suspended by suspending weight (A) 2 × 1011 N/m2 (B) 2 × 10–11 N/m2
to it. If S is longitudinal strain and Y its young’s modulus
(C) 2 × 1012 N/m2 (D) 2 × 1013 N/m2
of elasticity. Potential energy per unit volume will be
1 2 2 1 2 1 2 Q.3 In Searle’s experiment, which is used to find Young’s
(A) Y S (B) Y S (C) YS (D) 2YS2
2 2 2 modulus of elasticity, the diameter of experimental
wire is D = 0.05 cm (measured by a scale of least count
Q.3 The compressibility of water is 5 ×10−10 m2 / N . 0.001 cm) and length is L = 110 cm (measured by a
Find the decrease in volume of 100 ml of water when scale of least count 0.1 cm). A weight of 50N causes an
subjected to a pressure of 15 mPa. extension of l = 0.125 cm (measured by a micrometer
(A) 0.75 ml (B) 0.75 mm of least count 0.001 cm). Find maximum possible error
in the values of Young’s modulus. Screw gauge and
(C) 0.75 mm (D) 7.5 mm meter scale are free from error.  (2004)

Q.4 The upper end of a wire 1 meter long and 2mm Q.13 In plotting stress versus strain curves for two
radius is clamped. The lower end is twisted through an materials P and Q, a student by mistake puts strain
angle of 45º. The angle of shear is on the y-axis and stress on the x-axis as shown in the
(A) 0.09° (B) 0.9° (C) 9° (D) 90° figure. Then the correct statement(s) is(are)  (2015)

Previous Years’ Questions


Strain P
Q.1 Two rods of different materials having coefficient Q
of thermal expansion a1, a2 and Young’s moduli Y1, Y2
respectively are fixed between two rigid massive walls.
The rods are heated such that they undergo the same Stress
increase in temperature. There is no bending of the (A) P has more tensile strength than Q
rods. If a1 : a2 = 2:3, the thermal stresses developed in
the two roads are equal provided Y1 : Y2 is equal to (B) P is more ductile than Q
 (1989) (C) P is more brittle than Q
(A) 2 : 3 (B) 1 : 1 (C) 3 : 1 (D) 4 : 9 (D) The Young’s modulus of P is more than that of Q

Q.2 The adjacent graph shows extension (Dl) of a


wire of length 1m suspended from the top of a roof
at one end and with a load W connected to the
other end. If the cross-sectional area of the wire is
10–6m2, calculate from the graph the Young’s modulus
of the material of the wire. (2003)
8 . 9 8 | Simple Harmonic Motion and Elasticity

PlancEssential Questions
JEE Main/Boards JEE Advanced/Boards
Exercise 1 Exercise 2
Q. 1 Q.3 Q.7 Q.1 Q.4
Q.10

Exercise 2 Previous Years’ Questions


Q.4 Q.6 Q.7 Q.1 Q.2 Q.3

Previous Years’ Questions


Q.9 Q.11

Answer Key

JEE Main/ Boards


Exercise 1
Q.2 1: 8 Q.3 (i) 2F (ii) 4F Q.4 2 × 107 N

Q.5 (a) 3.18 × 108 N m–2 (b) 1.59 mm (c) 0.16% Q.6 1.02 × 109 cm

Q.7 1.013 × 1011 Q.8 4.9 × 1010 Q. 9 2 × 106 Nm–2

Q.10 2.4 × 10–5 J Q.11 1.5 × 1011 N/m2.

Exercise 2

Single Correct Choice Type

Q.1 C Q.2 A Q.3 D Q.4 D Q.5 A

Q.6 D Q.7 C

Previous Years’ Questions


Q.1 A Q.2 B Q.3 D Q.4 D
P hysi cs | 8.99

JEE Advanced/Boards
Exercise 1
Q.1 20 m/s Q.2 1.728 × 108 N/m2 Q3 2.4 × 10–5 J

Exercise 2
Q.1 B Q.2 C Q.3 A Q.4 A

Previous Years’ Questions


Q.1 C Q.2 A Q.3 1.09 × 1010 N/m2 Q.4 A, B

Solutions

JEE Main/Boards Sol 4: F = y


∆L
A
L
Exercise 1 ∆L = 2L – L = L
L
fL F = 2 × 1011 10–4 = 2 × 107 N
Sol 1: ∆L = L
Ay
If diameter is increased to twice then Sol 5: r = 10 × 10–3 m
(i) ∆L will decrease to ¼ value R = 10–2 m
∆L L=1m
(ii) F = A
L
F 100 × 103 105 109
Maximum load capacity will decrease to ¼ of initial (a) Stress = = = =
A π(10−2 )2 π × 10−4 π
value.
= 3.18 × 10 N/m
8 2

L2 stress × length
Sol 2: L1 = (b) Elongation = ∆L =
2 y
d1 = 2d2 ; A1 = 4A2 3.18 × 108 × 1
= = 1.59 × 10–3 m = 1.59 mm
F 1 = F2 2 × 1011
FL ∆L 1.59 × 10−3 × 100
∆L = (c) % Strain = × 100 = = 0.159%
Ay L 1
∆L1 L A 1 1 1
= 1 2 = =
∆L2 A1L2 2 4 8

Sol 3: Breaking force for two parallel wires of this size


(i) F’ = F1 + F2 = F + F = 2F
Sol 6: Stress=7.9 × 107 N/cm2 = 7.9 × 1011 N/m2
(ii) If thickness is double that means area is 4 times.
F ρAg
∆L ∆L Stress = = = ρlg = 7.9 × 1011
F= YA ⇒ F’ = Y 4A = 4F A A
L L
7.9 × 1011
= = 107 m = 109 cm
7900 × 10
8 . 1 0 0 | Simple Harmonic Motion and Elasticity

∆V stress
Sol 7: = –10–4 (iii) Young’s modulus =
V strain
P = 100 × 105 N/m2 2.45 ×107
= 1.5 × 1011 N/m2
−P 1.013 × 107 1.633 ×10−4
B= = = 1.013 × 1011 Nm–2
∆V / V 10 − 4

Sol 8: Pressure at 1 km depth = P


Exercise 2
= P0 + 1000 × 98 × 1000
= 105 + 98 × 107 = 99 × 105 N/m2 Sol 1: (C) Stress = F/A = 10/(2 × 10–6)
5
−P 99 × 10
Bulk modulus = = = 5 × 106 N/m2
∆V / V 2 × 10−4
Stress 5 ×106
= 4.9 × 1010 Pa Strain = =
Y 2 ×1011
∆X 0.05 = 2.5 × 10–5
Sol 9: Stream = = = 5 × 10–3
L 10 l = L × strain = 1 × 2.5 × 10–5
stress 10 4
Modulus = = l = 2.5 × 10–5 m
strain 5 × 10 −3
10
= × 106 = 2 × 106 N/m2 Sol 2: (A) Volume = Mass / density
5
Area of cross-section = Volume/length
5 × 10 5 × 10
Sol 10: Strain = = = 5 × 10 –6
mass 15.6 ×10−3
10−4 × 1011 107 = = = 8 × 10–7 m2
density × length 7800 × 2.5
Increase in energy = work done
1 50 Fl 8 × 9.8 × 2.5
× × 5 × 10–6 × 10–4 × 0.2 Y= =
2 10 −4 A∆L (8 ×10−7 ) ×1.25 ×10−3
250 × 10−11 Y = 1.96 × 1011 N/m2
= = 250×10–7=2.5× 10–5 J
10 −4
Sol 3: (D) Work done on the wire
Sol 11: (i) The initial length of the wire
1
W
= F ×l
= L = 30m 2
The increase in length of the wire, 1
= × stress × volume × strain
l = 0.44 × 10–2 2
1
Longitudinal stress W= × Y × strain2 × volume
2
/ L 1.633 ×10−4 .
= l=
1 ∆lL22 YA∆L2
(ii) The tension applied to the wire= Mg = 5 × 9.8 N W= ×Y× × AL =
2 L22 2L
Area of cross section of the wire,
A = 2 mm2 = 2 × 10–6 m2 2 ×1011 ×10−6 × 10−6
=W = 0.1 J
2×1
∴ Longitudinal stress
5 × 9.8 l1 L1r22
= Mg
= /A Sol 4: (D) =
2 ×10−6 l2 L2r12
= 2.45 ×107 N / m2
L1 = L, L2 = 2L, r1 = 2R., r2 = R

l L R2 1
∴ 1
= =
l2 2L 4R 2 8
P hysi cs | 8.101

ρ  dp  p
Sol 5: (A) The changed density, ρ ' = ∴ pdV + Vdp = 0 or   = − 
dp
1−  dV  V
B
Substituting the value, we have ∴ Bulk modulus,

11.4  dp   dp 
ρ' = B=–   = − V
2.0 ×10 8
 dV / V   dV 
1−
8.0 ×109  p  
∴ B =−  −  V  =p
ρ ' =11.69 g / cm ≈ 11.7g/cm
3 3
 V  
∴B=p
AL
Sol 6: (D) Mass of the rod = if A is its cross sectional Note: Adiabatic bulk modulus is given by B = gp.
area ρ

Weight acts at the mid-point Sol 3: (D) From the definition of bulk modulus,
mg (L / 2) −dp
Y
∴= × B=
A ∆L (dV / V)
If L is the original length Substituting the values, we have

mgL gρL2 (1.165 − 1.01) ×105


⇒ ∆L = = B
= = 1.55 ×105 Pa
2AY 2Y ( 10 / 100 )
9.8 ×1.3
= = 10.6 cm
120 Sol 4: (D)


Sol 7: (C) If L = initial length of the rod, increase in Time period, T= 2 π
g
length caused by temperature increase
When additional mass M is added to its bob
=Lαθ
 + ∆
If this expansion is prevented by a compressive force, TM= 2 π
then g
Mg
Lαθ +
Strain = αθ = 6 ×10−5 Mg AY
L ∆ = ⇒ TM =2π
AY g
∴ Stress developed in the rod
2
 TM  Mg
= Y × strain = 12 × 106 N/m2   = 1 +
 T  AY
= 1.2 × 107 N/m2
 2 
1 A  TM  
=   − 1
Y Mg  T  
 

Previous Years’ Questions


JEE Advanced/Boards
 
 
Fl  Fl  1 Exercise 1
Sol 1: (A) ∆l = =  or ( ∆l ) ∝ 2
AY   π d  
2
d
  4  Y  Sol 1: Equivalent force constant of rubber cord.
  
1 YA (5.0 ×108 )(1.0 ×10−6 )
Now, is maximum in option (A). k
= = = 5.0 ×103 N / m
d2 l (0.1)
Now, from conservation of mechanical energy, elastic
Sol 2: (B) In isothermal process potential energy of cord
pV = constant
8 . 1 0 2 | Simple Harmonic Motion and Elasticity

= Kinetic energy of missile Where A is the area of cross-section of the bar and L is
the initial length
1 1
∴ k( ∆l )2 =mv 2
2 2 FL
∴ l=
 k   5.0 ×103  AY
∴ v 
=  ∆l =   (12.0 − 10.0 ) ×10−2
 Hence from equation (i), we have
 m  5.0 ×10−3



F2L
= 20 m/s W=
2AY
Note: Following assumptions have been made in this
Here F = 5kg, wt=5 × 9.8 N, L=0.2 m
problem:
A = 1.0 cm2 = 1.0 × 10–6 m2 and
(i) k has been assumed constant, even though it
depends on the length (l). Y = 1.0 × 10–5 N/m2
(ii) The whole of the elastic potential energy is (5 × 9.8)2 × 0.2
converting into kinetic energy of missile. ∴ W=
2 × (1.0 ×10−4 ) × (1.0 ×1011 )

Sol 2: Let the initial volume of the block be V and v the = 2.4 ×10−5 J
increase in volume when it is heated t1 to t2. Then This is the increase in energy of the bar.
v= V × γ × (t2 − t1 )
Where γ is the coefficient of volume expansion. The Exercise 2
volume strain is therefore,
v
=γ(t2 − t1 ) Sol 1: (B)
V
F = Y α DtA; A = 2 × 10–6 m2
The bulk modulus is
change in pressure Y = 2 × 11 N/m2 ; α = 1.1 × 10–5
B=
volume strain T = 50 – 30 = 20°C
P F = 2 × 1011×1.1×10–5×20 × 2 × 10–6 = 88 N
B=
γ(t2 − t1 )
Sol 2: (C) Potential energy per unit volume = u
P =Bγ(t2 − t1 )
1 stress
= × stress × strain ; But Y =
Given B = 3.6 × 10 N/m
11 2 2 strain
γ = 3α = 3 × 8.0 × 10–6 ∴ stress = Y × strain = Y × S

= 24 × 10–6 /°C ∴ Potential energy per unit volume = u

(t2 – t1) = 70 – 50 = 20°C 1 1 2


= × (YS)S = YS
2 2
∴ P (3.6 × 1011) × (24 × 10–6) × 20
= 1.728 × 108 N/m2
Sol 3: (A)
1 ∆V
Sol 3: Work done in compressing the bar is given by  Compressibility= =
K V × ∆P
1
W = Fl 1
2 DV = (V × DP) ×
K
Where F is the force applied on the bar and l is the
DV = (100 × 15 × 106) × 5 × 10–10
compression in the length of the bar. By Hooke’s law, the
Young’s modulus of the material of the bar is given by DV = 0.75 ml
F / A FL
=Y = rφ (2 / 1000)45

l / L Al Sol 4: (A) θ = = = 0.09°
L 1
P hysi cs | 8.103

Previous Years’ Questions Sol 4: (A B)


stress
Y=
Sol 1: (C) Thermal stress σ = Y α Dθ Given, σ 1 = σ 2 strain
∴ Y1 a1Dθ = Y2 a2Dθ 1 strain 1 1
⇒= ⇒ > ⇒ YP < YQ
Y stress YP Yθ
Y1 α2 3
or = =
Y2 α1 2

 l 
Sol 2: (A) ∆l =  .W
 YA 
i.e., graph is a straight line passing through origin (as
l
shown in question also), the slope of which is .
YA
 l 
\Slope =  
 YA 
 l  1 
∴Y =   
 YA   slope 

 1.0  (80 − 20)


= 
 10−6  (4 − 1) ×10−4
= 2.0 × 1011 N/m2

Sol 3: Young’s modulus of elasticity is given by

Stress F / A FL FL
Y
= = = =
Strain l / L l A  π d2 
l
 4 
 
Substituting the values, we get

50 ×1.1 × 4
Y=
(1.25 ×10−3 ) × π× (5.0 ×10−4 )2

= 2.24 ×1011 N / m2

∆Y ∆L ∆l ∆d
Now, = + + 2
Y L l d
 0.1   0.001   0.001 
= +  + 2  = 0.0489
 110   0.125   0.05 
DY = (0.0489) Y
= (0.0489) × (2.24 × 1011) N/m2 = 1.09 × 1010 N/m2
2017-18 100 &
op kers
Class 11 T
By E ran culty
-JE Fa r
IIT enior emie .
S fP r es
o titut
Ins

PHYSICS
FOR JEE MAIN & ADVANCED
SECOND
EDITION

Exhaustive Theory
(Now Revised)

Formula Sheet
9000+ Problems
based on latest JEE pattern

2500 + 1000 (New) Problems


of previous 35 years of
AIEEE (JEE Main) and IIT-JEE (JEE Adv)

5000+Illustrations and Solved Examples


Detailed Solutions
of all problems available

Topic Covered Plancess Concepts


Tips & Tricks, Facts, Notes, Misconceptions,
Fluid Mechanics Key Take Aways, Problem Solving Tactics

PlancEssential
Questions recommended for revision
9. FLUID MECHANICS

1. INTRODUCTION
Fluid is a collective term for liquid and gas. A fluid cannot sustain shear stress when at rest. We will study the
dynamics of non-viscous, incompressible fluid. We will be learning about pressure variation, Archemides principle,
equation of continity, Bernoulli’s Theorem and its applications and surface tension, Stoke’s Law and Terminal
velocity of a spherical body.

2. DEFINITION OF A FLUID
A fluid is a substance that deforms continuously under the application of a shear (tangential) stress no matter how
small the shear stress may be.

F F

(a) Solid (b) Fluid


Figure 9.1: Behavior of a solid and a fluid, under the action of a constant shear force.

3. FLUID STATICS
It refers to the state when there is no relative velocity between fluid elements. In this section we will learn some of
the properties of fluid statics.

3.1 Density
The density ρ of a substance is defined as the mass per unit volume of a sample of the substance. If a small mass
∆m
element ∆m occupies a volume ∆V, the density is given by ρ =
∆V
In general, the density of an object depends on position, so that ρ =f(x, y, z)
9 . 2 | Fluid Mechanics

If the object is homogeneous, its physical parameters do not change with position throughout its volume. Thus for
M
a homogeneous object of mass M and volume V, the density is defined as ρ =
V
Thus SI units of density are kg m–3.

PLANCESS CONCEPTS

Note: As pressure is increased, volume decreases and hence density will increase.
As the temperature of a liquid is increased, mass remains the same while the volume is increased.
Vaibhav Krishnan (JEE 2009, AIR 22)

3.2 Specific Gravity


The specific gravity of a substance is the ratio of its density to that of water at 4ºC, which is 1000 kg/m3. Specific
gravity is a dimensionless quantity numerically equal to the density quoted in g/cm3. For example, the specific
Density of substance
gravity of mercury is 13.6, and the specific gravity of water at 100ºC is 0.999. RD=
Density of water at 4º C

Illustration 1: Find the density and specific gravity of gasoline if 51 g occupies 75 cm3?  (JEE MAIN)

Sol: Density is mass per unit volume, and specific gravity is the ratio of density of substance and density of water.
mass 0.051kg
Density = = = 680 kg/m3
volume 75 × 10−6 m3

density of gasoline 680kg / m3 mass of 75 cm3gasoline


Sp. gr = = =0.68 or Sp. gravity =
density of water 1000kg / m3 mass of 75 cm3 water
51g
= =0.68
75g

Illustration 2: The mass of a liter of milk is 1.032 kg. The butterfat that it contains has a density of 865 kg/m3 when
pure, and it constitutes 4 percent of the milk by volume. What is the density of the fat-free skimmed milk?
 (JEE MAIN)
Sol: Find the mass of butterfat present in the milk. Subtract this from total mass to get mass of fat-free milk. The
density of fat-free milk is equal to its mass divided by its volume.
Volume of fat in 1000 cm3 of milk = 4% × 1000 cm3 = 40 cm3
Mass of 40 cm3 fat = Vρ = (40 × 10–6 m3)(865 kg/m3) = 0.0346 kg
mass (1.032 − 0.0346)kg
Density of skimmed milk = =
volume (1000 − 40) × 10−6 m3

3.3 Pressure
The pressure exerted by a fluid is defined as the force per unit area at a point within the F
fluid. Consider an element of area ∆A as shown in the figure and an external force ∆F
is acting normal to the surface. The average pressure in the fluid at the position of the
∆F
element is given by Pav = [A normal force ∆F acts on a small cylindrical element of A
∆A
cross-section area ∆A.] Figure 9.2
P hysics | 9.3

As ∆A → 0, the element reduces to a point, and thus, pressure at a point is defined as


∆F dF
p = lim =
∆A →0 ∆A dA
F
When the force is constant over the surface A, the above equation reduces to p =
A
The SI unit of pressure is Nm-2 and is also called Pascal (Pa). The other common pressure units are atmosphere and
bar.
1 atm = 1.01325 × 105 Pa; 1 bar = 1.00000 × 105 Pa; 1 atm = 1.01325 bar

3.3.1 Pressure Is Isotropic


Imagine a static fluid and consider a small cubic element of the fluid deep within
the fluid as shown in the figure. Since this fluid element is in equilibrium therefore,
forces acting on each lateral face of this element must also be equal in magnitude.
Because the areas of each face are equal, therefore, the pressure on each face is equal
in magnitude. Therefore the pressure on each of the lateral faces must also be the
same. In the limit as the cube element to a point, the forces on top and bottom
surfaces also become equal. Thus, the pressure exerted by a fluid at a point is the same
in all directions – pressure is isotropic.
Note: Since the fluid cannot support a shear stress, the force exerted by a fluid pressure
Figure 9.3: A small cubical
must also be perpendicular to the surface of the container that holds it.
element is in equilibrium
inside a fluid
3.3.2 Atmospheric Pressure (P0)
It is pressure of the earth’s atmosphere. This changes with weather and elevation. Normal atmospheric pressure at
sea level (an average value) is 1.013 × 105 Pa. Thus,
1 atm = 1.013 × 105 Pa=1.013 Bar

3.3.3 Absolute Pressure and Gauge Pressure


The excess pressure above atmospheric pressure is usually called gauge pressure and the total pressure is called
absolute pressure. Thus, Gauge pressure = absolute pressure – atmospheric pressure. Aboslute pressure is always
greater than or equal to zero. While gauge pressure can be negative also.

Illustration 3: Atmospheric pressure is about 1.01 × 105 Pa. How large a force does the atmosphere exert on a 2
cm2 area on the top of your head? (JEE MAIN)

Sol: Force = Pressure × Area


Because p = F/A, where F is perpendicular to A, we have F = pA. Assuming that 2 cm2 of your head is flat (nearly
correct) and that the force due to the atmosphere is perpendicular to the surface (as it is), we have F = pA = (1.01
× 105 N/m2) (2 × 10–4 m2) ≈ 20N

3.3.4 Variation of Pressure with Depth


Weight of a fluid element of mass Dm, DW = (Dm)g . The force acting on the lower face of the element is pA
and that on the upper face is (p + Dp)A. The figure (b) shows the free body diagram of the element. Applying the
condition of equilibrium, we get, pA – (p + Dp) A – (Dm)g = 0
if ρ is the density of the fluid at the position of the element, then Dm = ρA(Dy)
9 . 4 | Fluid Mechanics

and pA – (p + Dp) A – rgA(Dy) = 0 (p+ p)A


∆p
or = - rg m ( m)g
∆y y
pA
∆p
In the limit ∆y approaches to zero, becomes y
∆y
dp
= −ρg . The above equation indicates that the
dy
(a) (b)
slope of p versus y is negative. That is, the pressure p
Figure 9.4
decreases with height y from the bottom of the fluid. In
dp
other words, the pressure p increases with depth h, i.e., = ρg
dh

3.4 The Incompressible Fluid Model


For an incompressible fluid, the density ρ of the fluid remains constant
throughout its volume. It is a good assumption for liquids. To find pressure at
the point A in a fluid column as shown in the figure, is obtained by integrating
p
the following equation: h
A
p h
dp = rgdh or ∫ dp = ρg∫ dh or p – p0 = rgh or p = p0 + rgh  …(xvi) y
p0 0

where ρ is the density of the fluid, and p0 is the atmospheric pressure at the
free surface of the liquid. Figure 9.5: A point A is located in
Note: Further, the pressure is the same at any two points at the same level in a fluid at a height from the bottom
the fluid. The shape of the container does not matter. and at a deth h from the free
surface
P0 P0

h
A B

PA= PB= P0+pgh

Illustration 4: Find the absolute pressure and gauge pressure at point A, B and C as shown in the Fig. 9.6 (1 atm =
105 Pa) (JEE MAIN)

Sol: Gauge Pressure = rgh, Absolute Pressure is sum of gauge pressure


and atmospheric pressure. 1m Kerosene
A
Patm = 10 Pa.
5
2m
Absolute Pressure A -> PA + Patm= r1ghA = (800)(10)1 = 8 kPa
3
p1=800 kg/m
p′A =pA + patm =108 kPa Water
1.5m
Gauge Pressure = 8 kPa. 2m
B 3
p2=1000 kg/m
B -> pB = ρ1g(2) + ρ2g(1.5)
C Mercury 0.5m
′ pB + patm = 131 kPa = (800)(10)(2) + (103)(10)(1.5) = 131 kPa
p=
B
p3=13.6x10 3 kg/m3
Gauge Pressure = 31 kPa.
Figure 9.6
P hysics | 9.5

C->
= pc p1g(2) + ρ2g(2) + ρ3g(0.5)
ρC′ = pC + patm = 204kPa
= (800)(10)(2) + (10)3(10)(2) + 1(13.6 × 103)(10)(0.5) = 204 kPa
Gauge Pressure = 104 kPa.

Illustration 5: A glass full of water of a height of 10 cm has a bottom of area 10 cm2, top of area 30 cm2 and volume
1 litre. (JEE ADVANCED)
(a) Find the force exerted by the water on the bottom.
(b) Find the resultant force exerted by the side of the glass on the water.
(c) If the glass is covered by a jar and the air inside the jar is completely pumped out, what will be the answer to
parts (a) and (b).
(d) If a glass of different shape is used provided the height, the bottom area and the volume are unchanged, will
the answers to parts (a) and (b) change.
Take g = 10m/s2, density of water = 103 kg/m3 and atmospheric pressure = 1.01 × 105 N/m2.

Sol: Pressure at the bottom depends on the height of water in the container. Force = Pressure × Area. The force
on water surface due to atmospheric pressure plus the weight of water are balanced by the force on water by the
container bottom and its walls.
(a) Force exerted by the water on the bottom F1 = (P0 + rgh)A1  … (i)
Here, P0 = atmospheric pressure = 1.01 × 105 N/m2; ρ = density of water = 103 kg/m3
g = 10 m/s2, h = 10 cm = 0.1 m and A1 = area of base 10 cm2 = 10–3 m2 . Substituting in Eq. (i), we get F1=
(1.01 × 10 + 10 × 10 × 0.1) ×10 or F1 = 102 N (downwards)
5 3 –3

(b) Force exerted by atmosphere on water F2 = (P0)A2


Here, A2 = area of top = 30 cm2 = 3 × 10–3 m2 ; F2 = (1.01 × 105)(3 × 10–3) = 303 N (downwards)
Force exerted by bottom on the water F3 = – F1 or F3= 102 N (upwards)
Weight of water W = (volume)(density)(g) = (10–3)(103)(10) = 10 N (downwards)
Let F be the force exerted by side walls on the water (upwards). Then, from equilibrium of water
Net upward force = net downward force or F + F3 = F2 + W
F – F2 + W – F3 = 303 + 10 – 102 or F = 211 N (upwards)
(c) If the air inside of the Jar is completely pumped out,
F1 = (rgh)A1 (as P0 = 0) = (103)(10)(0.1)(10–3) = 1 N (downwards). In this case F2 = 0 and F3 = 1 N (upwards)
∴ F = F2 + W – F3 = 0 + 10 – 1 = 9 N (upwards)
(d) No, the answer will remain the same. Because the answers depend upon P0, ρ, g, h , A1 and A2.

Illustration 6: Two vessels have the same base area but different shapes. The first vessel takes twice the volume
of water that the second vessel requires to fill up to a particular common height. Is the force exerted by water on
the base of the vessel the same in the two cases? If so, why do the vessels filled with water to the same height give
different reading on a weighing scale? (JEE MAIN)

Sol: Force on the base of the vessel depends on the pressure on it, and pressure depends on the height of the
liquid in the vessel. On the other hand the normal reaction from the surface on which the vessel is kept, depends
on both the pressure at the base as well as the weight of the liquid in the vessel.
9 . 6 | Fluid Mechanics

Pressure (and therefore force) on the two equal base areas are identical. But force is exerted by water on the sides
of the vessels also, which has non-zero vertical component when the sides of the vessel are not perfectly normal to
the base. This net vertical component of force by water on the side of the vessel is greater for the first vessel than
the second. Hence, the vessels weigh different when the force on the base is the same in the two cases.

3.4.1 Pascal’s Laws F2

According to the equation p = p0 + rgh . Pressure at any depth h in a


F1

fluid may be increased by increasing the pressure p0 at the surface.
Pascal recognized a consequence of this fact that we now call Pascal’s
Law. A pressure applied to a confined fluid at rest is transmitted equally A1 A2
undiminished to every part of the fluid and the walls of the container.
This principle is used in a hydraulic jack or lift, as shown in the figure.
The pressure due to a small force F1 applied to a piston of area A1 is
transmitted to the large piston of area A2. The pressure at the two pistons
is the same because they are at the same level.
A hydraulic jack
Figure 9.7
F1 F2 A 
p =
= Or F2 =  2  F1 . Consequently, the force on the larger piston is large.
A1 A2  A1 
Thus, a small force F1 acting on a small area A1 results in a larger force F2 acting on a larger area A2.

PLANCESS CONCEPTS

Since energy is always conserved, F1x1 = F2x2 where x1 and x2 are the distances moved by the pistons.
Nitin Chandrol (JEE 2012, AIR 134)

Illustration 7: Find the pressure in the air column


at which the piston remains in equilibrium. Assume
Air
the pistons to be massless and frictionless. Piston
 (JEE MAIN)
5m 1.73m
Kerosene
Sol: Apply Pascal’s law at two points at equal 60° S=0.8
height from a common datum. Datum
A B
Water
Let pa be the air pressure above the piston.
Applying Pascal’s law at point A and B. Figure 9.8

3
Patm + r wg(5) =pa + rkg(1.73) ; Pa = 138 kPa
2

Illustration 8: A weighted piston confines a fluid of density ρ in a closed


container, as shown in the figure. The combined weight of piston and h
container is W = 200 N, and the cross-sectional area of the piston is B
A = 8 cm2. Find the total pressure at point B if the fluid is mercury and
h = 25 cm (pm = 13600 kgm-3). What would be an ordinary pressure gauge
reading at B? (JEE ADVANCED)

Sol: Pressure difference between two points at different heights is equal to


ρgh, where h is difference in heights of two points. Apply Pascal’s law at two
points at different heights from a common datum. Figure 9.9
P hysics | 9.7

Pascal’s principle tells us about the pressure applied to the fluid by the piston and atmosphere. This added pressure
is applied at all points within the fluid. Therefore the total pressure at B is composed of three parts: Pressure of
atmosphere = 1.0 × 105 Pa
W 200N
Pressure due to piston and weight = = = 2.5 × 105 Pa
A −4 2
8 × 10 m
Pressure due to height h of fluid = hrg = 0.33 × 105 Pa
In this case, the pressure of the fluid itself is relatively small. We have
Total pressure at B = 3.8 × 105 Pa = 383 kPa. The gauge pressure does not include atmospheric pressure. Therefore,
Gauge pressure at B = 280 kPa

Illustration 9: For the system shown in the figure, the cylinder on the left, at L,
has a mass of 600 kg and a cross-sectional area of 800 cm2. The piston on the F
right at S, has cross-sectional area 25 cm2 and negligible weight. If the apparatus
is filled with oil (ρ=0.78 g/cm3), find the force F required to hold the system in S
equilibrium as shown in figure. (JEE ADVANCED)
600 kg 8m
Sol: Apply Pascal’s law at two points at different heights from a common datum. L
H1 H2
The pressures at point H1 and H2 are equal because they are at the same level in
the single connected fluid. Therefore, Pressure at H1 = pressure at H2 = (pressure
due to F plus pressure due to liquid column above H2)

(600)(9.8)N F Figure 9.10


= + (8m)(780 kg/m–3)(9.8)
2
0.08m 25 × 10−4 m2
After solving, we get, F = 31 N

Illustration 10: As shown in the figure, as column of water 40 cm high supports 31 cm of an unknown fluid. What
is the density of the unknown fluid? (JEE MAIN)

Sol: Find the hydrostatic pressure at the bottom most point A due
to both the water column and the unknown fluid column.
The pressure at point A due to the two fluids must be equal (or
the one with the higher pressure would push lower pressure fluid
away). Therefore, pressure due to water = pressure due to known 40 cm
h 40
fluid; h1r1g = h2r2g, from which r2 = 1 p1 = (1000 kg/m2) = 1290 31 cm
h 31
kg/m3 2

For gases, the constant density assumed in the compressible


model is often not adequate. However, an alternative simplifying A
assumption can be made that the density is proportional to the Figure 9.11
pressure, i.e., ρ = kp. Let r0 be the density of air at the earth’s surface
ρ0
where the pressure is atmospheric po, then r0 = kp0 ; After eliminating k, we get ρ = p
p0
ρ 
Putting the value of ρ in equation dp = –rgdy or dp = −  00 p  gdy
p 
 
p
dp ρ h
On rearranging, we get ∫ p
= − 0 g∫ dy where p is the pressure at a height y = h above the earth’s surface.
p0 0
p0
9 . 8 | Fluid Mechanics

−p0
ρ gh
p p
After integrating, we get ln = – 0 gh or p = p0 0
p0 p0

Note: Instead of a linear decrease in pressure with increasing height as in the case of an incompressible fluid, in
this case pressure decreases exponentially.

4. PRESSURE MEASURING DEVICES

4.1 Manometer P0
A manometer is a tube open at both ends and bent into the
shape of a “U” and is partially filled with mercury. When one
end of the tube is subjected to an unknown pressure p, the
mercury level drops on that side of the tube and rises on the h
other so that the difference in mercury level is h as shown in P =?
the figure. P0 h0
When we move down in a fluid, pressure increases with B
A
depth and when we move up the pressure decreases with
height. When we move horizontally in a fluid, pressure
remains constant. Therefore, p + r0gh0 – rmgh = p0 where p0 is
atmospheric pressure, and rm is the density of the fluid inside
the vessel. Figure 9.12: An U-shaped manometer tube
connected to a vessel

Po=
4.2 The Mercury Barometer
O

Pm
It is a straight glass tube (closed at one end) completely filled with mercury
and inserted into a dish which is also filled with mercury as shown in the Po Po
h
figure. Atmospheric pressure supports the column of mercury in the tube to A B
a height h. The pressure between the closed end of the tube and the column
of mercury is zero, p = 0. Therefore, pressure at points A and B are equal and
thus p0 = 0 + rmgh. Hence, p0 = (13.6 × 103)(9.8)(0.76) = 1.01 × 105 Nm-2 for Pa. A mercury barometer
Figure 9.13

Illustration 11: What must be the length of a barometer tube used to measure atmospheric pressure if we are to
use water instead of mercury? (JEE MAIN)
Sol: The length of the barometer tube will be inversely proportional to the density of fluid used in it.
We know that p0 = rmghm = r wghw where r w and hw are the density and height of the water column supporting the
atmospheric pressure p0.
ρm ρm
\ hw = hw ; Since = 13.6 ; hw = 0.76 m = (13.6)(0.76) = 10.33 m.
ρw ρw

5. PRESSURE DIFFERENCE IN ACCELERATING FLUIDS


Consider a beaker filled with some liquid of density p accelerating upwards with an acceleration ay along positive
y-direction. Let us draw the free body diagram of a small element of fluid of area A and length dy as shown in figure.
Equation of motion for this fluid element is, PA – W – (P + dP)A = (mass)(ay) or –W – (dP) A =(Aρ dy)(ay)
dP
or (Arg dy) – (dP)A = (Aρ dy)(ay) or = −ρ(g + ay )
dy
P hysics | 9.9

P + dP (P + dP)A

A dy A ay

P
x PA

Figure 9.14

Similarly, if the beaker moves along positive x-direction with acceleration ax, the equation of motion for the fluid
element shown in the figure is, PA – (P + dP) A
= (mass)(ax) y

dP
or (dP)A = (Aρ dx)ax Or = −ρax
dx
But suppose the beaker is accelerated and it (P + dP)A
has components of acceleration ax and ay in x ax PA
P + dP
and y directions respectively, then the pressure P A
decreases along both x and y directions. The A
above equation ax
dx
in that case reduces to, x

dP dP Figure 9.15
= −ρax and = −ρ(g + ay )  ….. (i)
dx dy
For surface of a Liquid Accelerated in Horizontal Direction.
Consider a liquid placed in a beaker which is accelerating horizontally with an y
acceleration ‘a’. Let A and B be two points in the liquid at a separation x in the
same horizontal line. As we have seen in this case.
dP
= −ρa or dP = -ra dx. Integrating this with proper limits, we get
dx h1 a
PA – PB = pax ….. (ii)
h2
A B
Further, PA = P0 + rgh1 And PB = P0 + rgh2
h1 − h2
Substituting in Eq. (ii), we get pg(h1 – h2) = pax \ x
x
x
a a Figure 9.16
= = tan θ \ tan θ =
g g

Note: When ay is not equal to zero then the angle of inclination is given by

 
 
dy  (dp)  ax
tan=
θ = =
dx   dp   g + ay
   
  dy  
9 . 1 0 | Fluid Mechanics

Illustration 12: A liquid of density ρ is in a bucket that spins with angular velocity ω as shown is
the figure. Show that the pressure at a radial distance r from the axis is
ρω2r 2
P
= P0 + where P0 is the atmospheric pressure. (JEE ADVANCED)
2

Sol: The net force on the liquid surface in equilibrium is always perpendicular to it as the liquid
surface cannot sustain shear stress.
Consider a fluid particle P of mass m at coordinates (x, y). From a non-inertial rotating frame of
reference, two forces are acting on it.
Figure 9.17
(i) Pseudo force (mxω2 ) y
(ii) Weight (mg) in the direction shown in figure.
Net force on it should be perpendicular to the free
P mx²
surface (in equilibrium). Hence,
P
mxω2 xω2 dy xω2 x
tan θ
= = or = mg F net
mg g dx g

y x
xω2 x2 ω2
∫ dy
∴= ∫ g ⋅ dx ∴ y =
2g P(x,y)
0 0
Figure 9.18

This is the equation of the free surface of the liquid, which is a parabola.
P y
r 2 ω2 ρω2r 2
As x = r, y = ∴ P(r)= P0 + ρgy or P(r)
= P0 +
2g 2 x=r P(r)
Figure 9.19
Hence proved.

a Front
Rear
Illustration 13: An open rectangular tank 5 m × 4 m × 3 m high containing
water up to a height of 2 m is accelerated horizontally along the longer side. 3m
Water 2m
(a) Determine the maximum acceleration that can be given without spilling
the water.
(b) Calculate the percentage of water split over, if this acceleration is 5m
increased by 20%. Figure 9.20
(c) If initially, the tank is closed at the top and is accelerated horizontally
by 9 m/s2, find the gauge pressure at the bottom of the front and rear 0
walls of the tank. (Take g = 10 m/s2) (JEE MAIN)
3m a0
Sol: As the water column is accelerated towards right in horizontal direction, 2m
the free surface will not be horizontal but will be inclined at an angle with y0
the θ horizontal, such that the left edge of the surface is at a higher level
than the right edge. This is because the pressure at the left of water column 5m
will be more than the pressure at the right of it. Figure 9.21
(a) Volume of water inside the tank remains constant

 3 + y0  3 −1
  5 × 4 = 5 × 2 × 4 or y0 = 1m \tan q0 = 5 = 0.4 3m
 2 
a v
Since, tan q0 = 0 , therefore a0 = 0.4 g = 4 m/s2
g
5m
(b) When acceleration is increased by 20% Figure 9.22
P hysi cs | 9.11

a x
a = 1.2 a0 = 0.48 g ∴ tan θ = = 0.48
g Air
Now, y = 3 – 5 tan θ = 3 – 5 (0.48) = 0.6 m y

(3 + 0.6) 3m
4 × 2×5 − ×5× 4
Fraction of water split over = 2 = 0.1 Water W
2×5× 4

Percentage of water split over = 10%


5m
a'
(c) a’ = 0.9 g; tan θ’ = = 0.9 Figure 9.23
g
1
Volume of air remains constant -> 4 × yx = (5)(1) × 4 ⇒ Pressure does not change in the air.
2
1 2
Since y = x tan θ’∴ x tan θ ' =5 or x = 3.33 m; y = 3.0m
2
Gauge pressure at the bottom of the
(i) Front wall pf = zero
(ii) Rear wall pr = (5 tan θ’)rwg = 5(0.9)(103)(10) = 4.5 × 104 Pa 60 rpm

Illustration 14: A vertical U-tube with the two limbs 0.75 m apart with water and rotated
about a vertical axis 0.5 m from the left limb, as shown in the figure. Determine the difference
in elevation of the water levels in the two limbs, when the speed of rotation is 60 rpm.
 (JEE MAIN)
0.5m
Sol: Each element of water in the tube is accelerated towards the axis. Along the horizontal 0.75m

part of the tube, the pressure will increase gradually as one moves radially away Figure 9.24
from the axis. The extra pressure provides the required centripetal acceleation.
Consider a small element of length dr at a distance r from the axis of rotation.
Considering the equilibrium of this element.
(p + dp) – p = rw2 r dr or dp = rw2 r dr
On integrating between 1 and 2
r1
2 ρω2 2 2
p1 – p2 = ρω ∫ r dr = (r − r )
r2
2 1 2

ω2 2 2 (2π)2
or h1 – h2 = [r1 − r2 ] = [(0.5)2 – (0.25)2] = 0.37 m.
2g 2(10)
Figure 9.25

6. BUOYANCY
If a body is partially or wholly immersed in a fluid, it experiences an upward force due to the fluid surrounding it.
The phenomenon of force exerted by fluid on the body is called buoyancy and the force is called buoyant force. A
body experiences buoyant force whether it floats or sinks, under its own weight or due to other forces applied on it.
Note: The buoyant force is due to the fact that the hydrostatic pressure at different depths is not the same.
Buoyant force is independent of:
(a) Total volume and shape of the body.
(b) Density of the body.
9 . 1 2 | Fluid Mechanics

6.1 Archimedes Principle


A body immersed in a fluid experiences an upward buoyant force equivalent to the weight of the fluid displaced by
it. The proof of this principle is very simple. Imagine a body of arbitrary shape completely immersed in a liquid of
density ρ. A body is being acted upon by the forces from all directions. Let us consider a vertical element of height
h and cross-sectional area dA.The force acting on the upper surface of the element is F1 (downward) and that on
the lower surface is F2 (upward). Since F2> F1, therefore, the net upward force acting on the element is dF = F2 – F1.
It can be easily seen that
F1 = (rgh1)dA and F2= (rgh2)dA. So dF = rg(h) dA
Also, h2 – h1 = h and h(dA) = dV \ The net upward force is F = ∫ ρgdV =
ρVg
Hence, for the entire body, the buoyant force is the weight of the volume of the fluid displaced.
Note: Buoyant force acts on the centre of gravity of the displacement liquid. This point is called centre of Buoyancy.

PLANCESS CONCEPTS

The fluid exerts force on the immersed part of the body from all directions.
The net force experienced by every vertical element of the body is in the upward direction.
A uniform body floats in a liquid if density of the body is less than or equal to the density of the liquid
and sinks if density of the uniform body is greater than that of the liquid.
B Rajiv Reddy (JEE 2012, AIR 11)

6.1.1 Detailed Explanation


An object floats on water if it can displace a volume of water whose weight is greater than that of the object. If the
density of the material is less than that of the liquid, it will float even if the material is a uniform solid, such as a
block of wood floats on water surface. If the density of the material is greater than that of water, such as iron, the
object can be made to float provided it is not a uniform solid. An iron built ship is an example to this case
Apparent weight of a body immersed in a liquid = w – w0, where ‘w’ is the true weight of the body and w0 is the
apparent loss in weight of the body, when immersed in the liquid.

6.1.2 Buoyant Force in Accelerating Fluids



Suppose a body is dipped inside a liquid of density ρL placed in an elevator moving with acceleration a . The
buoyant force F in this case becomes, F = VρL geff ;
 
Here, geff = | g − a |

Illustration 15: An iceberg with a density of 920 kgm-3 floats on an ocean V


of density 1025 kgm-3. What fraction of the iceberg is visible?(JEE MAIN) Above water
Sol: The buoyant force on the iceberg will be equal to its weight. The w(V0-V)
buoyant force is equal to the weight of water displaced by the iceberg, i.e.
the weight of volume of water equal to the volume of iceberg immersed. wV0
Let V be the volume of the iceberg above the water surface, then the
volume under inside is V0 – V. Under floating conditions, the weight (ρIV0g) V0-V
of the iceberg is balanced by the buoyant force rw(V0 – V)g. Under water
Thus, ρIV0g = rw(V0 – V)g Figure 9.26
or rwV = (rw – ρI)V0
P hysi cs | 9.13

V ρ w − ρI
or =
V0 ρw
V 1025 − 920
Since, r w = 1025 kg m-3 and ri = 920 kg m3, therefore, = = 0.10
V0 1025
Hence 10% of the total volume is visible.

Illustration 16: When a 2.5 kg crown is immersed in water, it has an apparent weight of 22 N. What is the density
of the crown? (JEE MAIN)
Sol: Apply Archemides principle.
Let W = actual weight of the crown and W’ = apparent weight of the crown
ρ = density of crown, r0 = density of water. The buoyant force is given by FE = W – W’ or
W
r0Vg = W – W’. Since W = rVg, therefore, V = . Eliminating V from the above equation, we get
ρg
ρ0 W (10)3 (25)
ρ= . Here W = 25 N; W’ = 22 N; r0 = 103 kg m-3 ; ρ = = 9.3 × 103 kg m-3.
W − W' 25 − 22

Illustration 17: The tension in a string holding a solid block below the surface of a liquid (of
density greater than that of solid) as shown in figure is T0 when the system is at rest. What
will be the tension in the string if the system has an upward acceleration a? (JEE MAIN)
Sol: The weight and tension force on the block are balanced by the buoyant force on it.
When the system is accelerated upwards, the effective value of g is increased.
Let m be the mass of block.
Initially for the equilibrium of block, F = T0 + mg ….(i)
Figure 9.27
Here, F is the up thrust on the block.

When the lift is accelerated upwards, geff becomes g + a instead of g.

g+a
Hence F' = F   ...(ii)
 g 
From Newton’s second law, F’ – T – mg = ma  ...(iii)
 a
Solving equations (i), (ii) and (iii), we get
= T T0  1 + 
 g
Figure 9.28

Illustration 18: An ice cube of side 1 cm is floating at the interface of kerosene


and water in beaker of base area 10 cm2. The level of kerosene is just covering Kerosine
the top surface of the ice cube. S=0.8
(a) Find the depth of submergence in the kerosene and that in the water.
(b) Find the change in the total level of the liquid when the whole ice melts into
water. (JEE ADVANCED)
Figure 9.29
Sol: Apply Archemedes principle. Sum of the buoyant forces by kerosene and
water will be equal to the weight of the ice cube.
(a) Condition of floating 0.8 rwghk + rwghw = 0.9 rwgh
or 0.8 hk + hw = (0.9)h  … (i)
Where hk and hw are the submerged depths of the ice in the kerosene and water, respectively.
9 . 1 4 | Fluid Mechanics

Also hk + hw = h ... (ii)


Here it is given that h = 1 cm
Solving equations (i) and (ii), we get
hk = 0.5 cm, hw = 0.5 cm
m heat
→ 0.9 cm3
(b) 1 cm3 
Ice (water)
0.5 0.9 − 0.5 0.4
Fall in the level of kerosene Dhk = ; Rise in the level of water Dhw = =
A A A
0.1 0.1
Net fall in the overall level Dh = = = 0.01 cm = 0.1 mm.
A 10

6.2 Stability of a Floating Body


The stability of a floating body depends on the effective point of application of the buoyant force. The weight of
the body acts at its centre of gravity. The buoyant force acts at the centre of gravity of the displaced liquid. This is
called the centre of buoyancy. Under equilibrium condition, the centre of gravity G and the centre of buoyancy B
lie along the vertical axis of the body as shown in the figure(s).

Fa
M
Fb
W
W
W

(a) (b)

Figure 9.30

(a) The buoyant force acts at the centre of gravity of the displaced fluid.
(b) When the boat tilts, the line of action of the buoyant force intersects the axis of the boat at the metacentre M. In
a stable boat, M is above the centre of gravity of the boat. When the body tilts to one side, the centre of buoyancy
shifts relative to the centre of gravity as shown in the figure (b). The two forces act along different vertical lines.
As a result, the buoyant force exerts a torque about the centre of gravity. The line of action of the buoyant force
crosses the axis of the body at the point M, called metacentre. If G is below M, the torque will tend to restore the
body to its equilibrium position. If G is above M, the torque will tend to rotate the body away from its equilibrium
position and the body will be unstable.

Illustration 19: A wooden plank of length 1 m and uniform cross section is hinged at
one end to the bottom of a tank as shown in the figure. The tank is filled with water
up to a height of 0.5 m. The specific gravity of the plank is 0.5. Find the angle θ that mg
the plank makes with the vertical in the equilibrium position. (Exclude the case θ = 0)
 (JEE ADVANCED) Figure 9.31

Sol: The net torque about the hinge due the weight of the plank and due to the buoyant force acting on the plank
should be zero.
The forces acting on the plank are shown in the figure. The height of water level is 0.5m. The length of the plank
is 1.0 = 2  . We have OB =  . The buoyant force F acts through the mid-point of the dipped part OC of the plank.
OC 
We have OA = = ; Let the mass per unit length of the plank be ρ.
2 2cos θ
P hysi cs | 9.15

  
Its weight mg = 2  rg; The mass of the part OC of the plank =  ρ .
 cos θ 
1 1 2ρ 2ρg
The mass of water displaced = ρ= ; The buoyant force F is, therefore, F = .
0.5 cos θ cos θ cos θ

Now, for equilibrium, the torque of mg about O should balance the torque of F about O.

 2ρ     1 1
So, mg (OB) sin θ = F(OA) sin θ or (2  ρ)  =    or cos θ = or cos θ =
2
, or θ = 45º
 cos θ  2cos θ  2 2

6.3 Forces on Fluid Boundaries


Whenever a fluid comes in contact with solid boundaries, it exerts a force
on it. Consider a rectangular vessel of base size l × b filled with water to a
height H as shown in figure The force acting at the base of the container is
given by Fb = p × (area of the base)
Pressure is same everywhere at the base and is equal to rgH. Therefore, Fb
Fb = rgH(lb) = ρ glb H Since, lbH = V (volume of the liquid) .Thus,
Fb = rgV = weight of the liquid inside the vessel.
A fluid contained in a vessel exerts forces on the boundaries. Unlike the base, l
the pressure on the vertical wall of the vessel is not uniform but increases Figure 9.32
linearly with depth from the free surface. Therefore, we have to perform the
integration to calculate the total force on the wall. Consider a small rectangular element of width b and thickness dh
at depth h from the free surface. The liquid pressure at this position is given by p = rgh. The force at the element is
dF = p(dbh) = rgbh dh;
H
1 F 1
The total force is F = rgb ∫ h dh= ρgbH2 . The total force acting per unit width of the critical walls is = ρgH2
O
2 b 2
H
1
F ∫0
The point of application (the centre of force) of the total force from the free surface is given by hc = h dF

H
Where ∫ h dF is the moment of force about the free surface.
0
H H H
1
Here ∫ h dF = ρgb ∫ h2dh =
∫ h(ρ gbh dh) = ρgH3 ;
0 0 0
3
1 2
Since F = rgbH2 , therefore, hc = H
2 3

Illustration 20: Find the force acting per unit width on a plane wall
inclined at an angle θ with the horizontal as shown in the figure.
 (JEE MAIN) h=y sin
dF y
Sol: The pressure at each point on the wall will be different, H
depending on the height. Find pressure on a small element, and use dy
the method of integration.
Consider a small element of thickness dy at a distance y measured Figure 9.33
along the wall from the free surface. There pressure at the position
of the element is p = rgh = rgy sin θ. The force given by dF = p(b dy) = rgb(y dy) sin q

H/sin θ H/sin θ
F  y2 
The total force per unit width b is given by =ρgsin θ . ∫ y dy =
ρg sin θ  
b 0  2  0
9 . 1 6 | Fluid Mechanics

F 1 H2
Or = ρg
b 2 sin θ
1
Note: That the above formula reduces to rgH2 for a vertical wall (θ = 90º)
2

6.4 Oscillations of a Fluid Column


The initial level of liquid in both the columns is the same. The area of cross-section of
the tube is uniform. If the liquid is depressed by x in one limb, it will rise by x along x
the length of the tube is the other limb. Here, the restoring force is provided by the x
hydrostatic pressure difference.
∴ F = − ( ∆P ) A = − (h1 + h2 ) ρgA = − ρgA ( sin θ1 + sin θ2 ) x
1 2

suppose, m is the mass of the liquid in the tube. Then, ma = −ρgA ( sin θ1 + sin θ2 ) x Figure 9.34

Since, F or a is proportional to –x, the motion of the liquid column is simple harmonic
in nature, time period of which is given by,

x m
Τ = 2π or Τ = 2π
a ρgA ( sin θ1 + sin θ2 )

6.5 Oscillations of a Floating Cylinder


Consider a wooden cylinder of mass m and cross-sectional area A floating in a
liquid of density ρ . At equilibrium, the cylinder is floating with a depth h submerged
[See Fig. 8.35]. If the cylinder is pushed downwards by a small distance y and then
released, it will move up and down with SHM. It is desired to find the time period
h h+y
and the frequency of oscillations.
According to the principle of flotation, the weight of the liquid displaced by
the immersed part of the body is equal to the weight of the body. Therefore, at
equilibrium,
Figure 9.35
Weight of cylinder = Weight of liquid displaced by the immersed part of cylinder
or mg= ( ρ Ah) g ∴ Mass of cylinder, m= ρ Ah
When the cylinder is pushed down to an additional distance y, the restoring force F (upward) equal to the weight
of additional liquid displaced acts on the cylinder.
∴ Restoring force, F= - (weight of additional liquid displaced) or F =− ( ρ A y ) g
The negative sign indicates that the restoring force acts opposite to the direction of the displacement.

F − (ρ A y ) g g
Acceleration a of the cylinder is given by a = = = −  y ...(i) … (i)
m ρ Ah h 
Since g/h is constant, a α − y Thus the acceleration a of the body (wooden cylinder) is directly proportional to
the displacement y and its direction is opposite to the displacement. Therefore, motion of the cylinder is simple
harmonic.

h
∴ Time period T = 2π … (ii)
g

1 1 g
∴ Frequency =
f =  … (iii)
T 2π h
These very interesting results show that time period and frequency have the same form as that of simple pendulum.
The submerged depth at equilibrium takes the place of the length of the pendulum.
P hysi cs | 9.17

7. FLUID DYNAMICS
In the order to describe the motion of a fluid, in principle, one might apply Newton’s laws to a particle (a small
volume element of fluid) and follow its progress in time. This is a difficult approach. Instead, we consider the
properties of the fluid, such as velocity, pressure, at fixed points in space. In order to simplify the discussion we
take several assumptions:
(i) The fluid is non viscous (ii) The flow is steady
(iii) The flow is non rotational (iv) The fluid is incompressible.

7.1 Equation of Continuity


B
It states that for streamlined motion of the liquid, the volume of liquid
flowing per unit time is constant through different cross-sections of A
the container of the liquid. Thus, if v1 and v2 are velocities of fluid at
respective points A and B of areas of cross-sections a1 and a2 and r1
and r2 be the densities respectively. Then the equation of continuity is a1
given by r1a1v1 = r2a2v2  ... (i) v1 
a2
v2 
If the same liquid is flowing, then ρ1 =ρ2 ; then the equation (i) can
be written Figure 9.36
As a1v1 = a2v2  ...(ii)
⇒ av = constant ⇒ v ∝ 1/a

PLANCESS CONCEPTS

Equation of continunity repersents the law of conservation of mass of moving fluids.


a1v1r1 = a2v2r2 (General equation of continuity)
This equation is applicable to actual liquids or to other fluids which are not incompressible.
Yashwanth Sandupatla (JEE 2012, AIR 821)

Illustration 21: Water is flowing through a horizontal tube of non-uniform cross-section. At a place, the radius of
the tube is 1.0 cm and the velocity of water is 2 m/s. What will be the velocity of water, where the radius of the pipe
is 2.0 cm? (JEE MAIN)
Sol: Apply the equation of continuity. Where area of cross-section is larger, the velocity of water is lesser and vice-
versa.
2
A   πr 2  r 
Using equation of continuity, A1v1 = A2v2 ; v 2 =  1  v1=
or v 2 = 1 
v1  1  v1
 A2   πr 2   r2 
 2 
 1.0 × 10−2 
Substituting the value, we get v 2 =   or v2 = 0.5 m/s
 2.0 × 10−2 
 

Illustration 22: Figure shows a liquid


being pushed out of a tube by pressing
a piston. The area of cross-section of the
piston is 1.0 cm2 and that of the tube
at the outlet is 20 mm2. If the piston is
pushed at a speed of 2 cm-s-1, what is the Figure 9.37
speed of the outgoing liquid?
9 . 1 8 | Fluid Mechanics

Sol: Apply the equation of continuity. Where area of cross-section is larger, the velocity of liquid is lesser and vice-versa.
From the equation of continuity A1v1 = A2v2
or (1.0 cm2) (2 cm s-1) = (20 mm2) v2
1.0 cm2
or v2 = × 2 cm s−1
20 mm2
100 mm2
= × 2 cm s−1 =
10 cm s−1
20 mm2

SHM of fluids in tubes:


Tubes form angles θ1 and θ2 with the horizontal.
x
m
T = 2π x
ρgA ( sin θ1 + sin θ2 )
1 2
m is total mass of fluid in tubes, A is area of cross – section ρ is density of fluid.
Figure 9.38

8. BERNOULLI’S THEOREM
When a non-viscous and an incompressible fluid flows in a streamlined motion from one place to another in a
container, then the total energy of the fluid per unit volume is constant at every point of its path. Total energy =
pressure energy + Kinetic energy + Potential energy
1
= PV + Mv2 + Mgh
2
Where P is pressure, V is volume, M is mass and h is height from a
reference level.
1
∴ The total energy per unit volume = P + rv2 + rgh
2
Where ρ is density. Thus if a liquid of density ρ, pressure P1 at a height
h1 which flows with velocity v1 to another point in streamline motion Figure 9.39
where the liquid has pressure P2, at height h2 which flows with velocity
v 2,
1 1
then P1 + ρv12 + ρgh1= P2 + ρv 22 + ρgh2
2 2

8.1 Derivations

8.1.1 Pressure Energy


If P is the pressure on the area A of a fluid, and the liquid moves through a distance due to this pressure, then
Pressure energy of liquid = work done = force × displacement = PAl
The volume of the liquid is Al.
PAl
∴ Pressure energy per unit volume of liquid = =P
Al

8.1.2 Kinetic Energy


1
If a liquid of mass m and volume V is flowing with velocity v, then the kinetic energy is mv2.
2
P hysi cs | 9.19

1m 2 1 2
∴ Kinetic energy per unit volume of liquid. =  v = ρv . Here, ρ is the density of liquid.
2 V  2

8.1.3 Potential energy


If a liquid of mass m is at a height h from the reference line (h = 0), then its potential energy is mgh. ∴ Potential
m
energy per unit volume of the liquid =   gh = rgh
v
1
Thus, the Bernoulli’s equation P + rv2 + rgh = constant
2
This can also be written as: Sum of total energy per unit volume (pressure + kinetic + potential) is constant for an
ideal fluid.

PLANCESS CONCEPTS

P v2
is called the ‘pressure head’, the velocity head and h the gravitational head.
ρg 2g
GV Abhinav JEE 2012, AIR 329

Intresting takeaway is the SI unit of each of these is meter (m).

Illustration 23: Calculate the rate of flow of glycerin of density 1.25×103 kg/m3 through the conical section of a
pipe, if the radii of its ends are 0.1 m and 0.04 m and the pressure drop across its length is 10 N/m2.  (JEE MAIN)

Sol: Apply the equation of continuity. Where area of cross-section is larger, the velocity
of fluid is lesser and vice-versa.
From continuity equation, A1v1 = A2v2
2
v1 A2 πr22  r2   0.04  4
or = = =  r =
  =   ... (i)
v 2 A1 πr 2 1  0.1  25
1 Figure 9.40
1 1
From Bernoulli’s equation , P1 + ρv12 = P2 + ρv 22
2 2
2 × 10
or v 22 − v12 = = 1.6 × 10−2 m2 / s2  ... (ii)
3
1.25 × 10
Solving equations (i) and (ii), we get v2 = 0.128 m/s
\ Rate of volume flow through the tube
Q = A2v2 = (pr22) v2= π (0.04)2(0.128) = 6.43 × 10–4 m3/s

Illustration 24: Figure shows a liquid of density 1200 kg m–3 flowing steadily in
a tube of varying cross section. The cross section at a point A is 1.0 cm2 and that
at B is 20 mm2, the points A and B are in the same horizontal plane. The speed
of the liquid at A is 10 cm s-1. Calculate the difference in pressure at A and B. 
 (JEE ADVANCED) Figure 9.41

Sol: Apply the equation of continuity. Where area of cross-section is larger, the velocity of fluid is lesser and vice-
versa.
9 . 2 0 | Fluid Mechanics

From equation of continuity. The speed v2 at B is given by, A1v1 = A2v2

1.0cm2
or (1.0 cm2) (10 cm s-1) = (20 mm2)v2 or v2 = ×10cm s−1 =50 cm s−1
20mm2
1 1
By Bernoulli equation, P1 + ρgh1 + ρv12= P2 + ρgh2 + ρv 22
2 2
1 2 1 2 1
Here h1 = h2. Thus P1 – P2 = ρv − ρv = × (1200 kg m−2 )(2500 cm2 s−2 − 100 cm2 s−2 )
2 2 2 1 2
= 600 kg m-3 × 2400 cm2 s-2 = 144 Pa

8.2 Application Based on Bernoulli’s Equation

8.2.1 Venturimeter
Figure shows a venturimeter used to measure
flow speed in a pipe of non-uniform cross-
section. We apply Bernoulli’s equation to the h
wide (point 1) and narrow (point 2) parts of
the pipe, with h1 = h2
1 1
P1 + ρv12 = P2 + ρv 22 p1
2 2
p2
A1 v1
From the continuity equation v 2 = v1 v2
A2
A2

Substituting and rearranging, H A1

Venturimeter
1  A2 
we get P1 − P2 = ρv12  1 − 1  …(i) Figure 9.42
2  A2 
 2 
The pressure difference is also equal to rgh, where h is the difference in liquid level in the two tubes.

2gh
Substituting in equation (i), we get v1 =
2
 A1 
  − 1
 A2 

PLANCESS CONCEPTS

Because A1 is greater than A2, v2 is greater than v1 and hence the pressure P2 is less than P1.

dV 2gh
The discharge or volume flow rate can be obtained as, = A=
1 v1 A1
dt  A1 
2

 A  − 1
 2
Anurag Saraf (JEE 2011, AIR 226)
P hysi cs | 9.21

9. TORRICELLI’S THEOREM
It states that the velocity of efflux of a liquid through an orifice is
equal to that velocity which a body would attain in falling from a
height from the free surface of a liquid to the orifice. If h is the height
of the orifice below the free surface of a liquid and g is acceleration
due to gravity, the velocity of efflux of liquid = v= 2gh . Total energy
per unit volume of the liquid at the surface = KE + PE + Pressure
energy = 0 + rgh + P0 ...(i)

and total energy per unit volume at the orifice = KE + PE + Pressure


1 Figure 9.43
energy = ρv 2 + 0 + P0
2
Since total energy of the liquid must remain constant in steady flow, in accordance with Bernoulli’s equation,
1 2
we have rgh + P0 = ρv + P0 or v= 2gh
2
Range = velocity × time ; R = Vx × time = 2gh × 1

1 2 2(H − h)
Now, H–h= gt ⇒ t = . From equation (i),
2 g

2(H − h)
R= 2gh × = 2h × 2(H − h) × h(H − h)2
g

\ =R 2 h(H − h)

dR H − 2h H
Range is max. if = 0 ⇒2× 0 ⇒ H – 2h = 0 ⇒ h =
=
dh 2 h(H − h) 2

PLANCESS CONCEPTS

R h = RH – h

Rh = 2 h(H − h)

RH–h = 2 h(H − h)
i.e. Range would be the same when the hole is at a
height h
or at a height H – h from the ground or from the top
of the beaker.
H Figure 9.44
R is maximum at h = and Rmax =H.
2
Vijay Senapathi (JEE 2011, AIR 71)
9 . 2 2 | Fluid Mechanics

9.1 An Expression for the Force Experienced by the Vessel


The force experienced by the vessel from which liquid is coming out.
dp d d
F= (Rate of change of momentum) = (mv) = (ρAvtv)
dt dt dt

F = ρAv 2 Where ρ = It is the density of the liquid.

A = It is the area of hole through which liquid is coming out.

9.2 Time taken to Empty a Tank


Consider a tank filled with a liquid of density ρ up to a height H. A small hole of area of cross section a is made at
the bottom of the tank. The area of cross-section of the tank is A.
Let at some instant of time the level of liquid in the tank be y. Velocity of efflux at this instant of time would be,
v = 2gy .

 dV 
At this instant volume of liquid coming out of the hole per second is  1  .
 dt 
 dV 
Volume of liquid coming down in the tank per second is  2  .
 dt 
t 0
dV1 dV2  dy   dy  A −1/2
dt
=
dt
; \ av
 dt 
= A  −  Or
= A  −  ∴ a 2gy
 dt 
∫ dt = − ∫y
a 2g H
dy
0

2A A 2H
∴t = [ y ]H0 =
a 2g a g

Illustration 25: A tank is filled with a liquid up to a height H. A small hole is made at the bottom of this tank. Let t1
be the time taken to empty first half of the tank and t2 the time taken to empty rest half of the tank.
t1
Then find . (JEE MAIN)
t2

Sol: This problem needs to be solved by method of integration.


Substituting the proper limit in equation (i), derived in the theory, we have
t1
A H/2 −1/2 2A 2A  H
∫ dt = −
a

2g H
y dy Or t1 =
a 2g
y ]H
[= H/2
Or  H−
a 2g 

2 
0

A H
Or t1
= ( 2 − 1)  …(ii)
a g

t2 A 0 A H
Similarly ∫0 dt = − ∫ y
−1/2
dy Or t2 =  ... (iii)
a 2g H/2 a g

t1 t1
From equations (ii) and (iii), we get = 2 − 1 Or = 0.414
t2 t2
P hysi cs | 9.23

PLANCESS CONCEPTS

From here we see that t1< t2. This is because inititally the pressure is high and the liquid comes out with
greater speed.
Ankit Rathore (JEE Advanced 2013, AIR 158)

10. VISCOSITY
When a liquid moves slowly and steadily on a horizontal surface, its layer in contact with the fixed surface is
stationary and the velocity of the layers increase with the distance from the fixed surface.
Consider two layers CD and MN of a liquid at distances x and x + dx from the fixed surface AB having velocities v
 dv 
and v + dv respectively as shown in the figure. Here   denotes the rate of change of velocity with distance and
 dx 
is known as velocity gradient. The tendency of the upper layer is to accelerate the motion and the lower layer tries
to retard the motion of upper layer. The two layers together tend to destroy their relative motion as if there is some
backward dragging force acting tangentially on the layers. To maintain the motion, an external force is applied to
overcome this backward drag.
Hence the property of a liquid virtue of which it opposes the relative motion between its different layers is
known as viscosity.
dv
The viscous force is given by F = −ηA
dx
Where η is a constant, called the coefficient of viscosity.
The SI unit of η is N-s/m2. It is also called decapoise or Pascal second. Thus,
1 decapoise = N-s/m2 = 1 Pa-s = 10 poise.
Dimensions of h are [ML-1T-1]
Figure 9.45

PLANCESS CONCEPTS

The negative sign in the above equation shows that the direction of viscous force F is opposite to the
direction of relative velocity of the layer.
Viscous force depends upon the velocity gradient whereas the mechanical frictional force is independent
of the velocity gradient.
Vaibhav Gupta (JEE 2009, AIR 54)

10.1 Effect of Temperature


In case of liquids, coefficient of viscosity decreases with increase of temperature as the cohesive forces decrease
with increase of temperature.

Illustration 26: A plate of area 2 m2 is made to move horizontally with a speed of 2 m/s by applying a horizontal
tangential force over the free surface of a liquid. The depth of the liquid is 1 m and the liquid in contact with the
bed is stationary. Coefficient of viscosity of liquid is 0.01 poise. Find the tangential force needed to move the plate.
 (JEE MAIN)
9 . 2 4 | Fluid Mechanics

Sol: Apply the Newton’s formula for the frictional force between two layers v=2 m/s
of a liquid. F
∆v 2−0 m/s
Velocity gradient = = =2 1m
∆y 1−0 m
From Newton’s law of viscous force,
∆v
|F| = ηA = (0.01 × 10-1)(2)(2) = 4 × 10-3 N. Figure 9.46
∆y
So, to keep the plate moving, a force of 4 × 10-3 N must be applied.

10.2 Stokes’ Law and Terminal Velocity


Stokes established that the resistive force or F, due
to the viscous drag, for a spherical body of radius r,
moving with velocity V, in a medium of coefficient of
viscosity η is given by
F = 6 pη rV

Mg

Figure 9.47

10.3.1 An Experiment for Terminal Velocity


Consider an established spherical body of radius r and density ρ falling freely from rest under gravity through a
fluid of density σ and coefficient of viscosity η. When the body acquires the terminal velocity V
W = Ft+ 6πηrV ;

4 2 r 2 (ρ − σ)g
6πηrV= πpr3 (ρ − σ)g ⇒ V =
3 9 η

Note: From the above expression we can see that terminal velocity of a spherical body is directly proportional to
the densities of the body and the fluid (ρ – σ). If the density of the fluid is greater than that of the body (.i.e. σ>ρ),
the terminal velocity is negative. This means that the body instead of falling, moves upward. This is why air bubbles
rise up in water.

Illustration 28: Two spherical raindrops of equal size are falling vertically through air with a terminal velocity of
1 m/s. What would be the terminal speed if these two drops were to coalesce to form a large spherical drop?

 (JEE MAIN)
Sol: Use the formula for terminal velocity for spherical body.
vT ∝ r2. Let r be the radius of small rain drops and R the radius of large drop.

4 4 
Equating the volume, we have πR 2 = 2  πr 3 
3  3 
2
R vT 'R 
\ R = (2) . r
1/3
or = (2)1/3 \ =   = (2)2/3
r vT  r 

\ vT’ = (2)2/3 vT = (2)2/3 (1.0) m/s = 1.587 m/s.


P hysi cs | 9.25

Illustration 29: An air bubble of diameter 2 mm rises steadily through a solution of density 1750 kg m-3 at the rate
of 0.35 cm s-1. Calculate the coefficient of viscosity of the solution. The density of air is negligible. (JEE MAIN)
Sol: As the air bubble rises with constant velocity, the net force on it is zero.
4 3
The force of buoyancy B is equal to the weight of the displaced liquid. Thus B = pr sg.
3
This force is upward. The viscous force acting downward is F = 6 π hrv.
The weight of the air bubble may be neglected as the density of air is small. For uniform velocity

4 3 2r 2 σg 2 × (1 × 10−3 m)2 × (1750 kg m−3 )(9.8 ms−2 )


F = B or, 6 phrv = pr σg or, η = = ≈ 11 poise .
3 9v 9 × (0.35 × 10−2 ms−1 )
This appears to be a highly viscous liquid.

10.3 Stream Line Flow


When liquid flows in such a way that the velocity at a particular point is
the same in magnitude as well as in direction. As shown in figure every
molecule should have the same velocity at A, if it crossed from that point.
Notice that the velocity at the point B will be different from that of A.
But every molecule which reaches at the point B, gets the velocity of the
point B.
Figure 9.48

10.4 Turbulent Flow


When the motion of a particle at any point varies rapidly in magnitude and direction, the flow is said to be
turbulent or beyond critical velocity. If the paths and velocities of particles change continuously and haphazardly,
then the flow is called turbulent flow.

10.5 Critical Velocity and Reynolds Number


When a fluid flows in a tube with small velocity, the flow is steady. As the velocity is gradually increased, at one
stage the flow becomes turbulent. The largest velocity which allows a steady flow is called the critical velocity.
Whether the flow will be steady or turbulent mainly depends on the density, velocity and the coefficient of viscosity
ρvD
of the fluid as well as the diameter of the tube through which the fluid is flowing. The quantity N = is called
η
the Reynolds number and plays a key role in determining the nature of flow. It is found that if the Reynolds number
is less than 2000, the flow is steady. If it is greater than 3000, the flow is turbulent. If it is between 2000 and 3000,
the flow is unstable.

11. SURFACE TENSION


The properties of a surface are quite often marked different from the properties of
the bulk material. A molecule well inside a body is surrounded by similar particles
from all sides. But a molecule on the surface has particles of one type on one side
and of a different type on the other side. Figure shows an example: A molecule of
water well inside the bulk experiences force from water molecules from all sides,
but a molecule at the surface interacts with air molecules from above and water
molecules from below. This asymmetric force distribution is responsible for surface
tension.
A surface layer is approximately 10-15 molecular diameters. The force between two
molecules decreases as the separation between them increases. The force becomes Figure 9.49
9 . 2 6 | Fluid Mechanics

negligible if the separation exceeds 10-15 molecular diameters. Thus, if we go 10-15


molecular diameters deep, a molecule finds equal forces from all directions.
Imagine a line AB drawn on the surface of a liquid (figure). The line divides the surface
in two parts, surface on one side and the surface on the other side of the line. Let us
call them surface to the left of the line and surface to the right of the line. It is found
that the two parts of the surface pull each other with a force proportional to the
length of the line AB. These forces of pull are perpendicular to the line separating the
two parts and are tangential to the surface. In this respect the surface of the liquid
behave like a stretched rubber sheet. The rubber sheet which is stretched from all
sides is in the state of tension. Any part of the sheet pulls the adjacent part towards
itself.
Figure 9.50
Let F be the common magnitude of the forces exerted on each other by the two parts
of the surface across a line of length  . We define the surface tension T of the liquid as T = F/ 
The SI unit of surface tension is N/m.
Note: The surface tension of a particular liquid usually decreases as temperature increases. To wash clothing
thoroughly, water must be forced through the tiny spaces between the fibers. This requires increasing the surface
area of the water, which is difficult to do because of surface tension. Hence, hot water and soapy water is better
for washing.

PLANCESS CONCEPTS

Surface tension acts over the free surface of a liquid only and not within the interior of the liquid.
Due to surface tension the insects can walk on liquid surface.
Vaibhav Krishnan (JEE 2009, AIR 22)

Illustration 30: Calculate the force required to take away a flat circular plate of radius 4 cm from the surface of
water, surface tension of water being 75 dyne cm-1.  (JEE MAIN)

Sol: Force = Surface tension×length of the surface


Length of the surface = circumference of the circular plate = 2pr = (8π) cm
Required force = T × L = 72 × 8π = 1810 dyne.

12. SURFACE ENERGY


When the surface area of a liquid is increased, the molecules from the interior rise to
the surface. This requires work against force of attraction of the molecules just below
the surface. This work is stored in the form of potential energy. Thus, the molecules in
the surface have some additional energy due to their position. This additional energy
per unit area of the surface is called ‘surface energy’. The surface energy is related to the
surface tension as discussed below:
Let a liquid film be formed on a wire frame and a straight wire of length  can slide on
this wire frame as shown in figure. The film has two surfaces and both the surfaces are in Figure 9.51
contact with the sliding wire and hence, exert forces of surface tension on it. If T be the
surface tension of the solution, each surface will pull the wire parallel to itself with a force T  . Thus, net force on
the wire due to both the surfaces is 2T  . One has to apply an external force F equal and opposite to it to keep the
wire in equilibrium. Thus, F = 2T 
P hysi cs | 9.27

Now, suppose the wire is moved through a small distance dx, the work done by the force is,
dW = F dx = (2T  )dx
But (2  )(dx) is the total increase in the area of both the surfaces of the film. Let it be dA. Then,
dW
dW = T da or T =
dA
Thus, the surface tension T can also be defined as the work done in increasing the surface area by unity. Further,
since there is no change in kinetic energy, the work done by the external force is stored as the potential energy of
the new surface.
dU
∴T= (as dW = dU)
dA
Thus, the surface tension of a liquid is equal to the surface energy per unit surface area.

Illustration 31: How much work will be done in increasing the diameter of a soap bubble from 2 cm to 5 cm?
Surface tension of soap solution is 3.0 × 10-2 N/m. (JEE MAIN)
Sol: Work done will be equal to the increase in the surface porential energy, which is surface tension multiplied by
increase in area of surface of liquid.
Soap bubble has two surfaces. Hence, W = T ∆A
Here, ∆A = 2[4p{(2.5×10–2)2 – (1.0×10–2)2}] = 1.32 × 10-2 m2
W = (3.0×10–2)(1.32×10–2)J = 3.96×10–4J

Illustration 32: Calculate the energy released when 1000 small water drops each of same radius 10–7m coalesce to
form one large drop. The surface tension of water is 7.0×10-2 N/m.  (JEE MAIN)

Sol: Energy released will be equal to the loss in surface potential energy.
Let r be the radius of smaller drops and R of bigger one.
4 3 4 
Equating the initial and final volumes, we have πR = (1000)  πr 3 
3 3 
R = 10r = (10)(10–7) m = 10-6 m. Further, the water drops have only one free surface. Therefore,
∆A = 4pR2 – (1000)(4pr2) = 4p[(10–6)2 – (103)(10–7)2] = –36π(10–12)m2
Here, negative sign implies that surface area is decreasing. Hence, energy is released in the process.
U = T[∆A] = (7×10–2)(36p×10–12)J = 7.9×10–12J

13. EXCESS PRESSURE


The pressure inside a liquid drop or a soap bubble must be in excess of the pressure outside the bubble drop
because without such pressure difference, a drop or a bubble cannot be in stable equilibrium. Due to the surface
tension, the drop or bubble has got the tendency to contract and disappear altogether. To balance this, there must
be excess of pressure inside the bubble.
9 . 2 8 | Fluid Mechanics

13.1 Excess Pressure Inside a Drop

Figure 9.52

To obtain a relation between the excess of pressure and the surface tension, consider a water drop of radius r and
surface tension T. Divide the drop into two halves by a horizontal passing through its centre as shown in figure and
consider the equilibrium of one-half, say, the upper half. The force acting on it are:
(a) Force due to surface tension distributed along the circumference of the section.
(b) Outward thrust on elementary areas of it due to excess pressure.
Obviously, both the types of forces are distributed. The first type of distributed forces combine into a force of
magnitude 2pr×T. To find the resultant of the other type of distributed forces, consider an elementary area DS of
the surface. The outward thrust on DS = pDS where p is the excess of the pressure inside the bubble. If this thrust
makes an angle θ with the vertical, then it is equivalent to DSp cos θ along the vertical and DSp sin θ along the
horizontal. The resolved component DSp sin θ is infective as it is perpendicular to the resultant force due to surface
tension. The resolved component DSp cos θ is equal to balancing the force due to surface tension
The resultant outward thrust = ΣDSp cos θ = pΣDS cos θ = pΣDS cos q = pΣDS’
where DS’ = DS cos θ = area of the projection of DS on the horizontal dividing plane
= p × pr2 (  DS’ = pr2)
2T
For equilibrium of the bubble we have pr2 p = 2pr T or p =
r

PLANCESS CONCEPTS

If we have an air bubble inside a liquid, a single surface is formed.


There is air on the concave side and liquid on the convex side.
The pressure in the concave side (that is in the air) is greater than P2
P1
2T
the pressure in the convex side (that is in the liquid) by an amount .
R
2T
∴ P2 − P1 = Figure 9.53
R

Nivvedan (JEE 2009, AIR 113)

13.2 Excess Pressure Inside Soap Bubble


A soap bubble consists of two spherical surface films with a thin layer of liquid between them. P'− P1 =
2S / R where
R is the radius of the bubble.
P hysi cs | 9.29

As the thickness of the bubble is small on a macroscopic scale, the difference in the radii P1
of the two surfaces will be negligible.
P’
Similarly, looking at the inner surface, the air is on the concave side of the surface, hence
P2 − P' = 2S / R . Adding the two equations, P2 − P1 = 4S / R P2

Illustration 33: What should be the pressure inside a small air bubble of 0.1 mm radius
situated just below the water surface? Surface tension of water = 7.2×10 –2 N/m and
atmospheric pressure = 1.013 ×105 N/m2. 
 (JEE MAIN) Figure 9.54

Sol: Pressure inside the air bubble is larger than that outside it by amount 2T/R, where T is surface tension and R
is its radius.
Surface tension of water T = 7.2×10–2 N/m; Radius of air bubble R = 0.1 mm = 10–4 m
2T
The excess pressure inside the air bubble is given by, P2 – P1 =
R
2T
∴ Pressure inside the air bubble, P2 = P1 + ; Substituting the values, we have,
R
(2 × 7.2 × 10−2 )
P2 = (1.013×105) + = 1.027 × 105 N/m2
10−4
Illustration 34: A 0.02 cm liquid column balances the excess pressure inside a soap bubble of radius 7.5 mm.
Determine the density of the liquid. Surface tension of soap solution = 0.03 Nm–1. (JEE MAIN)
Sol: Pressure inside the soap bubble is larger than that outside it by amount 4T/R, where T is surface tension and
R is its radius. Gauge pressure of liquid column is ρgh where symbols have the usual meaning.
4 × 0.03Nm−1
The excess pressure inside a soap bubble is DP = 4S/R = = 16 Nm–2
7.5 × 10−3 m
The pressure due to 0.02 cm of the liquid column is P = hrg = (0.02 × 10–2 m) ρ (9.8 ms–2)
Thus, 16 N m–2 = (0.02 × 10–2 m) ρ (9.8 ms–2); ρ = 9.2 × 103 kg m–3.

14. CAPILLARY ACTION


When a glass tube of very fine bore called a capillary tube is dipped in a liquid
(like water), the liquid immediately rises into it due to the surface tension. The
phenomenon of rise of a liquid in a narrow tube is known as capillarity.
Suppose that a capillary tube of radius r is dipped vertically in a liquid. The liquid
surface meets the wall of the tube at some inclination θ called the angle of contact.
Due to surface tension, a force, ∆  T acts on an element ∆  of the circle of contact
along which the liquid surface meets the solid surface and it is tangential to the
liquid surface at inclination θ to the wall of the tube. (The liquid on the wall of
the tube exerts this force. The tube also exerts the same force on the liquid in
the opposite direction.) Resolving this latter force along and perpendicular to the
wall of the tube, we have ∆  Tcosθ along the tube vertically upwards and ∆  Tsinθ Figure 9.55
perpendicular to the wall. The latter component is ineffective. It simply comes the
liquid against the wall of the tube. The vertical component ∆  Tcos θ pulls the liquid up the tube.
The total vertical upward force = Σ∆  T cos θ = T cos qΣD  = T cosθ.2pr (  ΣD  = 2pr). Because of this upward
pull liquid rises up in the capillary tube till it is balanced by the downward gravitational pull. If h is the height of the
liquid column in the tube up to the bottom, the gravitational pull, i.e. weight of the liquid inside the tube is (pr2 h
+ V)rg, where V is the volume of the liquid in meniscus. For equilibrium of the liquid column in the tube 2prT cos
θ = (pr2h + V)rg
9 . 3 0 | Fluid Mechanics

2T cos θ
If value of the liquid in meniscus is negligible then, 2prT cos θ = (pr2h )rg; h =
rρg
The small volume of the liquid above the horizontal plane through the lowest point of the meniscus can be
calculated if θ is given or known. For pure water and glass θ = 0º and hence the meniscus is hemispherical.
\ V = volume of the cylinder of height r – volume of hemisphere.
1 4π 3 2 1
πr3 −
= r = πr3 − πr3 = πr3
2 3 3 3
3
 πr 
\ For water and glass 2πrT = πr 2h +  ρg
 3

 r 2T r
2T = r  h +  ρg ⇒ = h −
 3  rρg 3
For a given liquid and solid at a given place as ρ, T, θ and g are constant, \ hr = constant
i.e. lesser t the radius of capillary greater will be the rise and vice-versa.

Illustration 36: A capillary tube of radius 0.20 mm is dipped vertically in water. Find the height of the water
column raised in the tube. Surface tension of water = 0.075 N m –1 and density of water = 1000 kg m–3. Take
g = 10 m s–2. (JEE MAIN)
Sol: Use the formula for height of the liquid in the capillary.
2S cos θ 2 × 0.075 N m−1 × 1
We have, h= = = 0.075 m = 7.5 cm.
rρg (0.20 × 10−3 m) × (1000 kg m−3 )(10 m s−2 )

PROBLEM SOLVING TACTICS

(a) Suppose two liquids of densities r1 and r2 having masses m1 and m2 are mixed together.

(m1 + m2 )
Then the density of the mixture will be =
 m1 m2 
 + 
 ρ1 ρ2 

If two liquids of densities r1 and r2 having volume V1 and V2 are mixed, then the density of the mixture will be

ρ1 V1 + ρ2 V2
.
V1 + V2

(b) When solving questions on Bernoulli’s always assume a reference level and calculate the heights from the
reference level.
P hysi cs | 9.31

FORMULAE SHEET
Fluid Statics:
mass
 1. Density = , S.I. units: kg/m3
volume
Ratio of its density
2. Specific gravity / Relative density / Specific density = ,
Ratio of density of water at 4º C
S.I. units: No units

3. If two liquids of volume V1 and V2 and densities d1 and d2 respectively are mixed then the density d of the
V d + V2d2 d + d2
mixture is d = 1 1 ; If V1 = V2 then d = 1
V1 + V2 2

4. If two liquids of densities d1 and d2 and masses m1 and m2 respectively are mixed together,
m1 + m2 2d1d2
then the density d of the mixture is d = ; if m1 = m2 then d =
m1 m2 d1 + d2
+
d1 d2
Normal component of force f
5. Pressure = = , S.I. units: N/m2, Pa
Area on which force acts A

6. Pressure P acting at the bottom of an open fluid column of height h and density d is

= 1.013 × 105 Pa = 1.013 × 105 Pa = 1.013 × 106 dynes/cm2= 76 cm of Hg = 760 torr = 1.013 bars.

Pa P = Pa + hdg dp
=sg
dh
(As h increases
Pv h Absolute Atmospheric
P increases)
pressure pressure

Figure 9.56

P – Pa = hdg
gauge pressure = absolute – atmospheric pressure.

7.

Figure 9.57

Area of smaller piston, a; area of larger piston, A, f is applied on the smaller piston
F f
Force F developed on the larger piston =
A a
9 . 3 2 | Fluid Mechanics

fA
\ F=
a
8. Beaker is accelerated in horizontal direction

Figure 9.58

9. Beaker is accelerated and it has components of acceleration ax, and ay in x and y directions respectively.

ay

y ay

Figure 9.59

dP
P increases with depth → = p(g + ay)
dy
P is the density of the fluid.
dP
ρ is the density of the fluid. = −pax
dx
 
10. Buoyant force F = V1ρ1 (g − a)

V1 = immersed volume of liquid


r1 = density of liquid
g = acceleration due to gravity
a = acceleration of body dipped inside liquid.

11. Body floats when Buoyant force balances the weight of the body.

Viρ2g = Vb ρb g
 Buoyant   Weight 
 force   of body 
   

Figure 9.60

Vb, rb = volume and density of body.


Vi = Volume of the immersed part of body.
P hysi cs | 9.33

r2 = density of liquid.
Vi ρb
Fraction of volume immersed =
ρ2 Vb
Vi ρ
% of volume immersed × 100 = b ×100.
Vb ρ2

12. Apparent weight of a body inside a fluid is Wapp = Wact – Upthrust


   Wapp = Vbg (rb – r2)
Vb, db = volume and density of body.
Vi = Volume of the immersed part of body.
r2 = density of liquid.
13. General equation of continuity
ρ2 A2 V2 Generally r1 = r2 i.e., density is uniform.
ρ1 A1 V1 =

A1& A2 are area of cross-section at point P and Q.
V1& V2 are velocities of the fluid at point P and Q.
14. Bernoulli’s Equation Figure 9.61
1 1
P1 + ρgh1 + ρV12= P2 + ρgh2 + ρV22
2 2

i.e.,

at that point from the reference level

dV
15. Volumetric flow Q = Aν = A – Area of cross section; ν – Velocity; V– Volume
dt
m3
S.I. unit =
S

16. Torricelli Theorem:

V = 2gh
height
velocity of
efflux

Figure 9.62
9 . 3 4 | Fluid Mechanics

Range
= R 2 h(H − h)

H
Range is maximum at h = and Rmax = H
2
Ab – Area of orifice
A – Area of cross-section of the container.
A 2
Time taken to fall from H1 to H2 = t ×
A0 g
dv
17. Viscous Force F = ηA
dy

coefficient of viscosity
L – Length of pipe
P1 and P2 are pressure at two ends of pipe.
R – Radius of pipe.
When liquid is flowing through a tube, velocity of flow of a liquid at distance from the axis.

=V
4
P
η L
( )
r 2 − x2 . Velocity distribution curve is a parabola.

18. Stoke’s Law: Formula for the viscous force on a sphere

F = 6phrv (η – coefficient of viscosity)


(r – radius of sphere)
(v – velocity of sphere)
2 2 (ρ − σ)g
VT = r (ρ – density of sphere)
9 η
(∝ – density of fluid)
19. Surface Tension

Figure 9.63

20. Surface Energy: dW = TdA


dV Surface energy
Surface Tension=
T =
dA Area

21. Pressure inside the soap bubble is P, then

4T
P – P0 =
R
P hysi cs | 9.35

22. Air Bubble Inside a Liquid

Figure 9.64

R – radius of bubble
T – surface tension force

23. Capillary Rise


2T cos θ
h= r = is the radius of capillary tube
rρg

θ = angle of contact

Solved Examples

JEE Main/Boards
Example 1: For the arrangement shown in the figure. Sol: Level of liquid will rise or fall depending on the
What is the density of oil? density of the solid.
Let M = Mass of the floating solid.
C r1 = density of liquid formed by the melting of the solid.
d=12.3 mm
d=12.3mm
Oil r2= density of the liquid in which the solid is floating.
The mass of liquid displaced by the solid is M. Hence,
M
the volume of liquid displaced is . When the solid
 = 135 mm ρ2
M
B melts, the volume occupied by it is . Hence, the level
ρ1
Water of liquid in container will rise or fall according as
M M
− is less than or greater than zero.
ρ2 ρ1
Sol: Pressure will be same at all points at the same
⇒ rises for ρ1 < ρ2
height in the same liquid.
⇒ falls for ρ1 > ρ2
P0 + rw gl = P0 + roil (  + d)g
There will be no change in the level if the level if r1 =
ρw  1000.(135) r2. In case of ice floating in water r1 = r2 and hence, the
⇒ ρoil
= = = 916 kg / m3
 + d (135 + 12.3) level of water remains unchanged when ice melts.

Example 2: A solid floats in a liquid of different material. Example 3: An iron casting containing a number of
Carry out an analysis to see whether the level of liquid cavities weighs 6000 N in air and 4000 N in water. What
in the container will rise or fall when the solid melts.
9 . 3 6 | Fluid Mechanics

is the volume of the cavities in the casting? Density of Sol: Use the formula for height of the liquid in the
iron is 7.87 g/cm3. capillary.
Take g = 9.8 m/s2 and density of water = 103 kg/m3. Let r1 and r2 be radii of upper and lower ends of the
conical capillary tube. The radius r at the meniscus is
Sol: Apply Archemides principal. The volume of iron given by
without the cavity is easily found. The total volume is
found from the upthrust. The difference in volumes is A
the volume of cavity.
t h
Let ν be the volume of cavities and V the volume of
solid iron. Then, B
r2
mass  6000 / 9.8  3
=V =  =  0.078m
density  7.87 × 103 
Further, decrease in weight = upthrust
∴ (6000 – 4000) = (V + ν)ρ wg
or 2000 = (0.078 + ν) × 103 × 9.8   −h
r =r1 + (r2 − r1 )  
or 0.078 + ν ≈ 0.2   
\ v = 0.12 m3
 0.1 − 0.08 
= (2.5 × 10−4 ) + (2.5 × 10−4 )  
 0.1 
Example 4: A boat floating in a water tank is carrying
a number of stones. If the stones were unloaded into = 3.0×10–4 m
water, what will happen to the water level?
The surface tension at 0ºC is given by
Sol: When the stones are in boat they will displace rhρg
more water as compared to the case when they are out T0 =
2
of the boat and inside water.
Let weight of boat = W and weight of stone = w. (3.0 × 10−4 )(8 × 10−2 )(1 / 4 × 10 4 ) × 9.8
= =0.084
2
Assuming density of water = 1 g/cc
T0 h0 6 × 10−2 12
Volume of water displaced initially= (w + W)/ρw For tube B, N/m = = =
T50 h50 5.5 × 10 −2 11
 W w
Later, Volume displaced =  + 
11 11
 ρw ρ  ⇒ T0 = × T0 = × 0.084 = 0.077 N/m
(ρ = density of stones) 12 12

⇒ Water level comes down. Considering the change in the surface tension as linear,
the change in surface tension with temp is given by

Example 5: A conical glass capillary tube A of length 0.1 T50 − T0 0.077 − 0.084 1
m has diameters 10–3m and 5×10–4m at the ends. When α= = =− k.
T0 − T50 0.084 × 0.077 60
it is just immersed in a liquid at 0ºC with larger radius
in constant contact with it, the liquid rises to 8×10–2m Negative sign shows that with rise in temp surface
in the tube. In another cylindrical glass capillary tube tension decreases.
B, when immersed in the same liquid at 0ºC, the liquid
rises to 6×10–2m height. The rise of liquid in tube B is
only 5.5×10–2m when the liquid is at 50ºC. Find the rate Example 6: A piece of copper having an internal cavity
at which the surface tension changes with temperature weighs 264 gm in air and 221 gm when it is completely
considering the change to be linear. The density of immersed in water. Find the volume of the cavity. The
liquid is (1/4) × 104 kg/m3 and the angle of contact is density of copper is 9.8 gm/cc.
zero. Effect of temp on the density of liquid and glass
is negligible. Sol: Apply Archemides principal. The volume of copper
without the cavity is easily found. The total volume is
found from the upthrust. The difference in volumes is
P hysi cs | 9.37

the volume of cavity. ∴ (10–  1) × 100 × 13.6 × g = 7800 g


Mass of copper in air = 264 gm 7800
10 –  1 = = 5.74
Mass of copper in water = 221 gm 100 × 13.6
Apparent loss of mass in water ∴ length of block above mercury surface

= 264 – 221 = 43 gm = 10 – 5.74 = 4.26 cm

∴ Mass of water displaced by copper piece when (b) Let  2 be the height of water column above mercury
completely immersed in water is equal to 43 gm. surface so that water just covers the top of the steel
block. The upward thrust due to mercury and water
mass of displaced
Volume of water displaced= displaced is equal to the weight of the body
density of water
∴ weight of block = wt. of water displaced + wt. of
43
= = 43.0 cc mercury displaced
1.0
∴ Volume of copper piece including volume of cavity = ∴ 7800 g =  2 × 1000 × 1 × g
43.0 cc. Volume of copper block only + (10 –  2) × 100 × 13.6 × g
mass 264 7800 = 100  2 + 13600 – 1360  2
= = = 30.0cc
density 8.8
1260  2 = 13600 – 7800 = 5800
Volume of cavity = 43.0 – 30.0 = 13.0 cc
∴ Height of water column above mercury=
Example 7: A cubical block of each side equal to 10 5800
l2 = = 4.6 cm
cm is made of steel of density 7.8 gm/cm3. It floats on 1260
mercury surface in a vessel with its sides vertical. The
density of mercury is 13.6 gm/cm3. Example 8: A cubical block of wood of each side 10 cm
(a) Find the length of the block above mercury surface. long floats at the interface between oil and water with
its lower surface 2 cm below the interface. The height
(b) If water is poured on the surface of mercury, find the of oil and water column is 10 cm each. The density of
height of the water column when water just covers the oil is 0.8 g cm–3.
top of the steel block.
(a) What is the mass of the block?
Sol: Apply Archemides principal. The weight of the (b) What is the pressure at the lower side of surface of
block will be equal to the weight of the liquid displaced. block?
(a) Volume of steel block
= (10)3 = 1000 cm3 Sol: Apply Archemides principle. The weight of the
block will be equal to the weight of the liquid displaced.
Mass of steel block = 1000 × 7.8 = 7800 gm
(a) Buoyant force = (mass of liquid displaced) × g
Let  1 be the height of steel block above the surface
= [10 × 10 × 8 × 0.8 + 10 × 10 × 1]g= 840 g
of mercury. Height of block under mercury = 10 –  1.
Weight of mercury displaced by block If m is mass of block
=(10 –  1) × 100 × 13.6 × g gm mg = 840 g or m = 840 gm
Archimedes’ principle shows that upward thrust is
equal to the weight of mercury displaced by block is Oil
equal to the weight of the block. 10 cm
8 cm

10 cm Water

(b) Pressure at the lower surface of block


9 . 3 8 | Fluid Mechanics

= pressure at any point on the same level. aρ 2


(b) Equation (i) gives
= F (v − v12 ) =
2 2
10 × 0.8 × g + 2 × 1 × g
= 10g = 10 × 9.81 = 99.1 Newton/meter2 π × (4)2 × 1000  1
× 64 − 
2  4
Example 9: A massless smooth piston forces water 1 22 255
with a velocity of 8 m/s out of a tube shaped container = × × 16 × 1000 × 160.3N
=
2 7 4
with radii 4.0 cm and 1.0 cm respectively as shown in
the figure. Assume that the water leaving the container
Example 10: A horizontal tube has different cross-
enters air at 1 atmospheric pressure. Find
sections at two points A and B. The diameter at A is
(a) The velocity of the piston 4.0 cm and that at B is 2 cm/ The two manometer arms
(b) Force F applied to the piston. are fixed at A and B. When a liquid of density 800 kg/
m3 flows through the tube, the difference of pressure
Sol: Apply Bernoulli’s Theorem at two points, one near between the arms of two manometers is 8 cm. Calculate
the piston and the other at the end of the tube. the rate of flow of tube liquid.

(a) Let F be the force applied horizontally such that v1 Sol: Apply Bernoulli’s Theorem and equation of
is the velocity of water in tube A of radius 4.0 cm and continuity.
v2 equal to 8 m/s is the velocity of water out of tube B
of radius 1.0 cm. From Bernoulli’s principle:

Let p0 be atmospheric pressure. 1 1


p1 + ρ v12 = p2 + ρ v 22
2 2
From the equation of continuity: A1v = A2v2
4 cm
F
1 cm pressure difference: p1 – p2 = hrg

2gh
B p0 These equations give v1 = A2
(A12 − A22 )
A
Rate of flow of volume
F
∴ At A, v1 = ?, r1 = 4.0 cm, p1 = p0 + Where a is area 2gh
a V = A1v1 = A1A2
of cross-section of piston or tube A. At B, v2 = 8 m/s, r2 (A12 − A22 )
= 1.0 cm, p2 = p0
2 × 9.8 × 8 × 10−2
Bernoulli’s theorem sat A and B gives, = π2 (4 × 10 −4 )(1 × 10 −2 )
(4 π × 10−4 ) − ( π × 10 4 )2
1 1
p1 + ρv1 + hρg= p2 + ρv 22 + hρga
2 2 = 4.06 × 10–4 m3/s
where ρ is density of water and h is height of axis of
both tubes from ground level
1 1
JEE Advanced/Boards
∴ p1 + ρv12 = p2 + ρv 22
2 2
Example 1: Under isothermal condition two soap
bubbles of radii a and b coalesce to form a single
F 1 bubble of radius c. If the external pressure is p0
+ p0 = p0 + ρ(v 22 − v12 )
a 2
F ρ 2 p0 (c3 − a3 − b3 )
= (v − v12 )  ...(i) show that surface tension T =
a 2 2 4(a2 + b2 + c2 )
Equation of continuity at A and B gives
Sol: Pressure inside the soap bubble is larger than that
v1a1 = v2a2 outside it by amount 4T/R, where T is surface tension
2 and R is its radius.
πr22 1
or v1 =v2 × 8   =
=× 0.5 m / s
2
πr1 4
P hysi cs | 9.39

As we know that for a soap bubble, the excess pressure Total P.E. when they are not connected
4T
is = . External pressure is p0 h  h   h2 h2 
r Ah1ρg  1  + Ah2ρg  2  = Aρg  1 + 2 
∴ p= p0 +
4T
∴ p= p0 +
4T
and 2  2   2 2

a b
a b
When the levels are equal, the potential energy is given
4T as
p=
c p0 +
c
4 4 4 3  h + h2   h1 + h2   h1 + h2   h1 + h2 
and v a =πa3 , v b =πb3 & v c= πc  ….(i) = A  1  ρg 
  + A 
  ρg  
3 3 3  2   4   2   4 
Applying conservation of mass
(h1 + h2 )2 (h1 + h2 )2
na + nb =
nc = 2Aρg = Aρg
2× 4 4
pa v a pb v b pc v c Pv The change in potential energy
⇒ + = [ pv= nRT ⇒ n= ]
RTa RTb RTc RT
Aρg  (h1 + h2 ) 
2

Since, temp is constant =


2 

2
(
− h12 − h22 )
 
i.e.Ta = Tb = Tc, so the expression reduces to
Aρg  h1 + h2 − 2h1 − 2h2 + 2h1h2 
2 2 2 2
pava + pbvb = pcvc =  
2  2 
with the help of equation (i), we have 

Aρg  −(h1 + h2 − 2h1h2 ) 


2 2
 4T   4 3   4T  4 3  =  
 p0 +   πa  +  p0 +  πb  2 

2 
 a  3   b  3 
2
 h − h2 
 4T  4 3  Work done due to gravity = −Aρg  1 
=  p0 +  πc   2 
 c  3 
The negative sign shows that the work is done by the
⇒ 4T(a2 + b2 – c2) = p0(c3 – a3 – b3)
gravitational field on the liquid.
p0 (c3 − a3 − b3 )
⇒T =
4(a2 + b3 − c3 ) Example 3: A container of large uniform cross-section
area A resting on a horizontal surface holds two
immiscible, non-viscous and incompressible liquids
Example 2: Two identical cylindrical vessels with their of densities d and 2d, each of height H/2 as shown
bases at the same level contain a liquid of density ρ. in figure. The lower density liquid is open to the
The height of liquid in one vessel is h1 and that in the atmosphere having pressure P0.
other vessel is h2. The areas of either base is A. What is
the work done by gravity in equalizing the levels when (a) A homogeneous solid cylinder of length L (L < H/2)
the two vessels are connected. and cross-section area A/5 is immersed such that, it
floats with its axis vertical at the liquid-liquid interface
Sol: Work done by gravity is equal to the loss in the with length L/4 in the dense liquid.
gravitational potential energy.
The center of gravity of liquid column would be at
height h1 and h2 respectively. A is area of cross-section.

h1
h2
9 . 4 0 | Fluid Mechanics

Determine: 1 H  H   3H 
=  Adg   + A(2d)g    = dg  
(i) The density D of the solid. A 2  2   2 
(ii) The total pressure at the bottom of the container.
Pressure due to buoyancy reaction
(b) The cylinder is removed and the original arrangement
is restored. A tiny hole of area S(S<<A) is punched on Buoyancy reaction force  A  LDg
the vertical side of the container at a height h(h<H/2). = = 
area 5 A
Determine:
A 5d l Lgd
= ×L × × × g =
(i) The initial speed of efflux of liquid at the hole. 5 4 A 4
(ii) The horizontal distance x travelled by the liquid  3H  Ldg
initially. ∴ Total pressure = P0 + dg   +
 2  4
(iii) The height hm at which the hole should be punched
so that the liquid travels the maximum distance xm  3H L
P0 + dg  +
= 
initially. Also calculate xm: (Neglect the air-resistance in 2 4
these calculations)
(b) (i) Let vA and vB be velocity of fluids at points A and
Sol: Apply the principles of hydrostatic pressure, B.
Archemedes and Bernoulli’s Theorem.
(A) (i) As per Archimedes’ principle, the buoyant force
on a body is equal to the weight of the fluid displaced
by the body.

AvA = svB

∴ v =  s  v  0 (  A>>H/2)
A   B
A
1
Bernoulli’s Equation: p + ρv 2 + ρgh = constant
2
A
Weight of solid cylinder =
×D × g = F ↓L×
5 1 H H
At A, P0 + dv 2A + dg + 2d(g)   =
constant or
F ↑= Buoyant force = weight of liquid displaced 2 2 2
L Α 3L A
= × × 2dg + × × d × g 3
4 5 4 5 P0 + dgH = constant ( VA = 0)
2
A
Equating: L × ×D × g At point B,
5 1 1
P0 + dv 2A + (2d)vB2 + 2dgh =
cosntant
L A 3L A 2 2
= × × 2dg + × × d × g
4 5 4 5
or P0 + dvB2 + 2dgh =
constant
d 3d 2d + 3d 5d
D= + = = 3
2 4 4 4 Equating: P0 + dvB2 + 2dgh =P0 + dgH
2
(ii) Pressure at the bottom of the cylinder
3
= Patmosphere + Pdense liquid + Plight liquid dvB2 + 2dgh = dgH
2

Force 3  3 
Pressure due to liquid = vB2 g  H − 2h
= =  ; vB g  H − 2h 
Area 2  2 
P hysi cs | 9.41

(ii) Time t taken by liquid to fall through height h under Sol: The cross section of tube can be thought of made-
2h up of elementary rings of infinitesimal thickeness. Find
g with zero initial velocity. t = the volume flow rate and kinetic energy of one ring.
g
Use the method of integration to find the flow rate and
Horizontal distance
energy for the tube.
2h 3 
x = vB t = × g  H − 2h 
g  2  (a) Let us consider a cylindrical section at a distance of
r and having thickness dr. The volume of fluid flowing
3H  r2 
h(3H − 4h) = 2 × h × −h through this section per second. dv = (2πrdr)v 0  1 − 
4  R 2 

dx So, the volume of fluid flowing across the section of the
(iii) To find height h at which x is max, =0.
dh tube in unit time.

d  1/2 d 1/2
3Hh − 4h2  0;
= h(3H − 4h) 0
=
dh   dh 

d  3H 
2 × h − h =0.
dh  4 
1/2 −1/2
1  3H  1  3H 
2×  − h +2 h×  − h (–1)=0 R  r2 
R 
r2 
2h  4  2 4  ∫
v= (2 πrdr)v 
0 1 − 
R 2 
= 2 πv 0∫ 
r  1 −  dr
R 2 
0  0 
1/2
1  3H  h R
 − h =  r2 r4   R2 
h 4   3H 
1/2
= 2πv 0  −  = 2πv 0  
 4 
 2 4R  0
2
 4 − h  
 
3H 3 (b) The kinetic energy of the fluid within the volume
or −h =h or h = H element of thickness dr
4 8
2
1 1  r2 
3H  3H 3H 
1/2
(dm)v 2 = (2πrdr )ρv 20  1 − 
\ xm =
2× − 2 2  R2 
   
8  4 8 
So, the K.E. of fluid within the tube
3H 3H 3H 2
=
2× × = 1
R
r2 
8 8 4 = (2π )ρv 20 ∫  1 −  rdr
2  R2 
0 

Example 4: A tube of length  and radius R carries a Integrating, we get


steady flow of liquid whose density is ρ and viscosity η.
 R2 
 r2  K.E. = pr   v 20   
The velocity v of flow is given by
= V V0  1 −  , where  6 
 R 2   

r is the distance of flowing fluid from the axis. Find (c) The viscous drag exerts a force on the tube
(a) Volume of fluid, flowing across the section of the  dv 
F = ηA  
tube, in unit time.  dx r =R
(b) Kinetic energy of the fluid within the volume of the
tube.  dv   2r  −2v 0
Hence   = v 0  −  =
 dr r =R
2
 R r =R R
(c) The frictional force exerted on the tube by the fluid,
and ∴ F = –η(2pR  )(–2v0  R) = 4phhlv0
(d) The difference of pressures at the ends of the tube. (d) The pressure difference DP is given by
DP = P2 – P1 = P
9 . 4 2 | Fluid Mechanics

Where P1 = O and P2 = P Example 6: The U-tube acts as a water siphon. The bend
Force(F) in the tube is 1m above the water surface. The tube outlet
As we know that P= is 7 m below the water surface. The water issues from
area of section of tube
the bottom of the siphon as a free jet at atmospheric
F 4 πηv 0 pressure. Determine the speed of the free jet and the
P= =
πR 2
πR 2 minimum absolute pressure of the water in the bend.
Given atmospheric pressure = 1.01 × 105 N/m2.
4 η v 0
P= g = 9.8 m/s2 and density of water = 103 kg/m3.
R2

Example 5: A fresh water reservoir is 10 m deep. A


horizontal pipe 4.0 cm in diameter passes through the
reservoir 6.0 m below the water surface as shown in
figure. A plug secures the pipe opening.
(a) Find the friction force between the plug and pipe
wall.
(b) The plug is removed. What volume of water flows
out of the pipe in 1 h? Assume area of reservoir to be
too large.
Sol: Apply Bernoulli’s Theorem at points 1, A and 2.
(a) Applying Bernoulli’s equation between point (1)
and (2)
1 1
P1 + ρv12 + ρh1 + P2 + ρv 22 + ρgh2
2 2
Since, area of reservoir >> area of pipe
v1 ≈ 0, also P1 = P2 = atmospheric pressure
Sol: Force of friction will balance the force due to
pressure difference on the plug. Use the formula for So, v1 = 2g(h1 − h2 ) = 2 × 9.8 × 7 = 11.7 m / s
velocity of efflux for part (b)
(b) The minimum pressure in the bend will be at A.
(a) Force of friction Therefore, applying Bernoulli’s equation between (1)
= pressure difference on the sides of the plug × area of and (A)
cross section of the plug 1 1
P1 + ρv12 + ρgh1 = PA + ρv 2A + ρghA
= (rgh) A= (10) (9.8)(6.0)(π)(2 × 10 )
3 –2 2 2 2

= 73.9 N Again, v1 ≈ 0 and from conservation of mass v A = v 2 ;


1
(b) Assuming the area of the reservoir to be too large. PA= P1 + ρg(h1 − hA ) − ρv 22
2
Velocity of efflux
= v 2gh cons tant
=
Therefore, substituting the values, we have
\ v= 2 × 9.8 × 6 = 10.84m / s PA = (1.01 × 105) + (1000)(9.8)(–1)
1
Volume of water coming out per sec, – × (1000)(11.7)2 2 =2.27 × 104 N/m2
2
dV
= 1.36 × 10−2 m3 / s
= Av =π(2 × 10−2 )2 (10.84)
dt Example 7: Two separate air bubbles (radii 0.004 m and
∴ The volume of water flowing through the pipe in 1 h. 0.002 m) formed of the same liquid (surface tension 0.07
N/m) come together to form a double bubble. Find the
 dV  −2 radius and the sense of curvature of the internal film
V= =  t (1.36 × 10 )(3600) = 49.96 m
3

 dt  surface common to both the bubbles.


P hysi cs | 9.43

Sol: Pressure inside the soap bubble is larger than that


outside it by amount 4T/R, where T is surface tension
and R is its radius.

r2

P
P2

P1
r1

For the two bubbles,


4T i.e., α = av or α =a 2ghmax
P=
1 P0 + ;
r1
α2
4T ⇒ hmax =
P=
2 P0 + ,r<r 2ga2
r2 2 1
(b) Let at time t, the level of water be h. Then,
\ P2> P1
 dh  h dh t dt
i.e. pressure inside the smaller bubble will be more. The A   = α − a 2gh or ∫0 α − a = ∫0 A
 dt  2gh
excess pressure
Solving this, we get
r −r 
P = P2 − P1 = 4T  1 2   …(i)
 r1r2  A  α  α − a 2gh  
= t  ln   − 2gh 
ag  a  α 
This excess pressure acts from concave to convex side,    
the interface will be concave towards smaller bubble
and convex towards larger bubble. Let R be the radius Example 9: Under isothermal condition, two soap
of interface then, bubbles of radii r1 and r2 coalesce to form a single
bubble of radius r. The external pressure is P0. Find
4T
P=  ….(ii) the surface tension of the soap in terms of the given
R parameters.
From equations (i) and (ii)
Sol: Pressure inside the soap bubble is larger than
r1r2 (0.004)(0.002) that outside it by amount 4T/R, where T is surface
=R = = 0.004 m
r1 − r2 (0.004 − 0.002) tension and R is its radius. Use ideal gas equation and
the condition that he total number of moles of air is
conserved.
Example 8: A cylindrical tank of base area A has a small
hole of area ‘a’ at the bottom. At time t = 0, a tap starts As mass of the air is conserved,
to supply water into the tank at a constant rate α m3/s. \ n1 + n2 = n (as PV = nRT)

(a) What is the maximum level of water hmax in the tank? P1 V1 P2 V2


PV
\ + =
RT1 RT2 RT
(b) Find the time when level of water becomes h(< hmax).

Sol: The height of water level will increase till the rate of
P
inflow is greater than the rate of outflow. Use method P1 P2
of integration to find the time taken by water level to
+ =
reach height h.
r1 r2 r
(a) Level will be maximum level when
Rate of inflow of water = rate of outflow of water
Although not given in the question, but we will have to
assume that temperature of A and B are the same.
9 . 4 4 | Fluid Mechanics

L L
Restoring torque
= τ Fx sin θ =
SL(d2 − d1 )g sin θ
2 2
sin θ ≈ θ (θ is small)

1 2
=
∴τ SL (d2 − d1 )g θ
nB pB VB / RT pB VB 2
= =
nA pA VA / RT pA VA  ML2  d2 θ  SLd1 xL2  d2 θ
τ = Iα =   2 =  
 3  dt  3  dt2
(p + 4s / rA ) × 4 / 3π(rA )3    
=
(p + 4s / rB ) × 4 / 3π(rB )3 d2 θ 3 1
=
∴ x SL2 (d2 − d1 )gθ
dt SL d1 2
3
(s = surface tension)
nB d2 θ 3g  d2 − d1 
Substituting the values, we get =6
= or   θ ;so motion is S.H.M;
nA dt 2L  d1 
comparing with differential equation of S.H.M.
Example 10: A thin rod of length L and area of cross
section S is pivoted at its lowest point P inside a
d2 θ 3g  d2 − d1 
stationary, homogeneous and non-viscous liquid as + ω2 θ= 0; ω =  ;
shown in the figure. The rod is free to rotate in a vertical dt 2 2L  d1 
plane about a horizontal axis passing through P. The
density d1 of the material of the rod is smaller than the 2π 2Ld1
Timeperiod, T = = 2π
density d2 of the liquid. The rod is displaced by a small ω 3g(d2 − d1 )
angle. From its equilibrium position and then released,
show that the motion of the rod is simple harmonic and
determine its angular frequency in terms of the given Example 11: Two non-viscous, incompressible and
parameters. immiscible liquids of density ρ and 1.5 ρ are poured
into two limbs of a circular tube of radius R and small
cross-section kept fixed in a vertical plane as shown in
the figure.
d1
Each liquid occupies one fourth the circumference of
the tube.
d2
(a) Find the angle that the radius vector to the interface
makes with the vertical in the equilibrium position.
P
(b) If the whole liquid is given a small displacement
Sol: Use the restoring torque method to find the from its equilibrium position, show that the resulting
angular frequency. oscillations are simple harmonic. Find the time period
of these oscillations.
Consider the rod be displaced through an angle θ . The
different forces on the rod are shown in the figure. Sol:Use the restoring torque method to find the angular
Weight of rod acting downward=S L d1 g=mg frequency.

Buoyant force acting upwards =S L d2 g (a) Density of liquid column BC = 1.5 ρ ;


Density of liquid column CD = ρ
Net thrust acting on the rod upwards; F=S L (d2-d1)g
Pressure at A due to liquid column BA= ρ AB
S
F

G D


G L O
R
B  
F C
mg E
P
y
A
P hysi cs | 9.45

= AFx1.5ρxg= (AO − OF)1.5 ρ gxg ρ y d2 θ


θ= or y = Rθ, Angular acceleration = ,
=(R − R sin θ)1.5g ρ R dt2
Pressure at A due to liquid column AD = ρ AD d2 θ 2.5πR 3ap  d2 θ 
Torque about
= A I=  2
= AEx1.5ρ xg + EG ρg dt2 2  dt 
 
∴ ρAD − (AO − OE)1.5 ρg + (EO + OG) ρg Restoring torque due to displaced liquid.
−(R − R cos θ)1.5 ρg + R(cos θ + sin θ)ρg
− ay x1.5pg + ay pg xR cos θ
τrest =
InequilibriumPAB = PAD
= −2.5ay pgxR cos θ = −2.5apgR 2 cos θ.θ
R(1 − sin θ)1.5 ρg= R(1 − cos θ)1.5 ρg + R(cos θ + sin θ) ρg
[R cos θ is perpendicular distance of gravitational force
0.5 1 1
tan=
θ = . or tan−1   from axis of rotation]
2.5 5 5
 2.5πR 3ap  d2 θ
(b)If a is area of cross − sec tion,
2πR πR
Equating 

 2  dt

(
 2 =− 2.5ap gR 2 cos θ θ )
length of each column= = 2
4 2 dθ  2gcos θ  2
= −  θ = −ω θ
πRa dt 2
 π R 
Volume of each column =
2 2gcos θ
πRa As Acceleration is proportional to angular
Mass of column
= BC x1.5 ρ πR
2 displacement and is directed towards mean position,
πRa the liquid undergoes SHM
Mass of column CD = xp
2
 πRap  2π πR
M.I. of whole liquid=
about O   (1.5 + 1)R
2 T= = 2π x
 2  ω 2gcos θ
2.5πR3ap 1 5
or I = As tan
= θ .cos
= θ .
2 5 26
3
Let y be small displacement toward left and θ be the πR R
T=
2π 2( π) 2
=
angular displacement, 5  10g 
2 x gx  
26  26 

JEE Main/Boards

Exercise 1

Q.1 If water in one flask and castor oil in other are Q.4 How does the viscosity of gases depend upon
violently shaken and kept on a table, then which one will temperature?
come to rest earlier?
Q.5 Explain the effect of (i) density (ii) temperature and
Q.2 What is the acceleration of a body falling through a (iii) pressure on the viscosity of liquids and gases.
viscous medium after terminal velocity is reached?
Q.6 Two equal drops of water falling through air with
Q.3 The liquid is flowing steadily through a tube of a steady velocity ν. If the drops coalesced, what will be
varying diameter. How are the velocity of liquid flow (V) the new steady velocity?
in any portion and the diameter (D) of the tube in that
portion related?
9 . 4 6 | Fluid Mechanics

Q.7 What is the viscous force on a drop of liquid of Q.17 Calculate the force required to take away a flat
radius 0.2 mm moving with a constant velocity 4 cm s–1 plate of radius 5 cm from the surface of water. Given
through a medium of viscosity 1.8×10–1 Nm–2 s. surface tension of water = 72 × 10–3 Nm–1.

Q.8 Eight rain drops of radius 1 mm each falling Q.18 A square wire frame of side 10 cm is dipped in a
downwards with a terminal velocity of 5 cm s–1 coalesce liquid of surface tension 28×10–3 Nm–1. On taking out,
to form a bigger drop. Find the terminal velocity of a membrane is formed. What is the force acting on the
bigger drop. surface of wire frame?

Q.9 The flow rate of water from a tap of diameter 1.25 Q.19 The air pressure inside a soap bubble of diameter
cm is 0.48 L/min. The coefficient of viscosity of water is 3.5 mm is 8 mm of water above the atmosphere.
10–3 Pa-s. After sometime, the flow rate is increased to Calculate the surface tension of soap solution.
3 L/min. The coefficient of viscosity of water is 10–3 Pa-s.
Characterize the flow. Q.20 What should be the radius of the capillary tube
so that water will rise to a height of 8 cm in it? Surface
Q.10 A block of wood is floating in a lake? What is tension of water 70 × 10–3 Nm–1.
apparent weight of the floating block?
Exercise 2
Q.11 A block of wood is floating in a lake. What is apparent
weight of the floating block? Single Correct Choice Type

Q.12 A body floats in a liquid contained in a beaker. Q.1 The area of cross-section of the wider tube shown
The whole system shown in the figure falls freely under in figure is 800 cm2. If a mass of 12 kg is placed on the
gravity. What is the up thrust on the body due to the massless piston, the difference in heights h in the level
liquid? of water in the two tubes is:

(A) 10 cm (B) 6 cm (C) 15 cm (D) 2 cm

Q.2 Two cubes of size 1.0 m side, one of relative


Q.13 A force of 60 N is applied on a nail, where tip
density 0.60 and another of relative density = 1.15 are
has an area of cross-section of 0.0001 cm2. Find the
connected by weightless wire and placed in a large tank
pressure on the tip.
of water. Under equilibrium the lighter cube will project
above the water surface to a height of:
Q.14 If the water pressure gauge shows the pressure at
(A) 50 cm (B) 25 cm (C) 10 cm (D) Zero
ground floor to be 270 kPa, how high would water rise in
the pipes of a building?
Q.3 A cuboidal piece of wood has dimensions a, b and
c. Its relative density is d. It is floating in a large body of
Q.15 A metal cube is 5 cm side and relative density 9,
water such that side a is vertical. It is pushed down a bit
suspended by a thread is completely immersed in a liquid
and released. The time period of SHM executed by it is:
of density 1.2×103 kg m–3. Find the tension in the thread.
abc h
(A) 2π (B) 2π
Q.16 A boat having a length of 3 m and breadth 2 m g da
is floating on a lake. The boat sinks by one cm, when a
man gets on it. What is the mass of the man? bc da
(C) 2π (D) 2π
dg g
P hysi cs | 9.47

Q.4 The frequency of a sonometer wire is f, but when Q.9 Fountains usually seen in gardens are generated
the weights producing the tensions are completely by a wide pipe with an enclosure at one end having
immersed in water the frequency becomes f/2 and on many small holes. Consider one such fountain which is
immersing the weights in a certain liquid the frequency produced by a pipe of internal diameter 2 cm in which
becomes f/3. The specific gravity of the liquid is: water flows at a rate 3 ms–1. The enclosure has 100 holes
4 16 15 32
each of diameter 0.05 cm. The velocity of water coming
(A) (B) (C) (D) out of the holes is (in ms–1):
3 9 12 27
(A) 0.48 (B) 96 (C) 24 (D) 48
Q.5 A small ball of relative density 0.8 falls into water
from a height of 2m. The depth to which the ball will Q.10 A vertical tank open at the top, is filled with a
sink is (neglect viscous forces): liquid and rests on a smooth horizontal surface. A small
(A) 8 m (B) 2 m (C) 6 m (D) 4 m hole is opened at the centre of one side of the tank.
The area of cross-section of the tank is N times the area
of the hole, where N is a large number. Neglect mass
Q.6 A hollow sphere of mass M and radius r is immersed of the tank itself. The initial acceleration of the tank is:
in a tank of water (density rw). The sphere would float
if it were set free. The sphere is tied to the bottom of g g
(A) (B)
the tank by two wires which makes angle 45º with the 2N 2N
horizontal as shown in figure. The tension T1 in the wire g g
(C) (D)
is: N 2 N

Q.11 Two water pipes P and Q having diameters 2×10–2m


and 4×10–2m, respectively, are joined in series with the
main supply line of water. The velocity of water flowing
in pipe P is:
(A) 4 times that of Q (B) 2 times that of Q
(C) 1/2 times that of Q (D)1/4times that of Q

Q.12 A rectangular tank is placed on a horizontal


4 3 ground and is filled with water to a height H above the
πR ρw g − Mg
3 2 3
(A) (B) πR ρw g − Mg base. A small hole is made on one vertical side at a
2 3 depth D below the level of the water in the tank. The
4 3 distance x from the bottom of the tank at which the
πR ρw g − Mg
4 3 water jet from the tank will hit the ground is:
(C) 3 (D) πR ρw g − Mg
2 3
(A) 2 D(H − D) (B) 2 DH

Q.7 A large tank is filled with water to a height H. A 1


small hole is made at the base of the tank. It takes T1 (C) 2 D(H + D) (D) DH
2
times to decrease the height of water to H η, (η> 1) and
it takes T2 time to take out the rest of water. If T1 = T2,
Q.13 A horizontal pipe line carries water in a streamline
then the value of η is:
flow. At a point along the tube where the cross-
(A) 2 (B) 3 (C) 4 (D) 2.2 sectional area is 10–2 m2, the water velocity is 2ms-1
and the pressure is 8000 Pa . The pressure of water
Q.8 In the case of a fluid, Bernoulli’s theorem exes the at another point where the cross-sectional area is
application of the principle of conservation of: 0.5 ×10–2m2 is:

(A) Linear momentum (B) Energy (A) 4000 Pa (B) 1000 Pa

(C) Mass (D) Angular momentum (C) 2000 Pa (D) 3000 Pa


9 . 4 8 | Fluid Mechanics

Q.14 Which of the following is not an assumption for (A) Siphon works when h3> 0
an ideal fluid flow for which Bernoulli’s principle is valid:
(B) Pressure at point 2 is P2 = p0 – rgh3
(A) Steady flow (B) Incompressible
(C) Pressure at point 3 is P0
(C) Viscous (D) Irrotational
(D) None of the above

Q.15 A solid metallic sphere of radius r is allowed to fall


Q.19 A steady flow of water passes along horizontal
freely through air. If the frictional resistance due to air
tube from a wide section X to the narrower section Y,
is proportional to the cross-sectional area and to the
see figure. Manometers are placed at P and Q of the
square of the velocity, then the terminal velocity of the
sections. Which of the statements A,B,C,D is most
sphere is proportional to which of the following?
correct?
(A) r2 (B) r (C) r3/2 (D) r1/2

Q.16 If two soap bubbles of different radii are connected


by a tube.
(A) Air flows from the bigger bubble to the smaller
bubble till the sizes become equal
(A) water velocity at X is greater than at Y
(B) Air flows from bigger bubble to the smaller bubble
till the sizes are interchanged (B) the manometer at P shows lower pressure than at Q
(C) Air flows from the smaller bubble to the bigger (C) kinetic energy per m3 of water at X = kinetic energy
per m3 at Y
(D) There is no flow of air
(D) the manometer at P shows greater pressure than
Q.17 A long capillary of radius r is initially just vertically at Y
completely immerged inside a liquid of angle of contact
0º. If the tube is slowly raised, then relation between Previous Years’ Questions
radius of curvature of meniscus inside the capillary tube
and displacement (h) of tube can be represented by:
Q.1 A metal ball immersed in alcohol weighs W1 at 0º
C and W2 at 50ºC. The coefficient of cubical expansion
of the metal is less than that of the alcohol. Assuming
that the density of the metal is large compared to that
of alcohol, it can be shown that: (1980)
(A) W1> W2 (B) W1 = W2
(C) W1< W2 (D) All of these

Q.2 A vessel containing water is given a constant


acceleration a towards the right along a straight
horizontal path. Which of the following diagrams
represent the surface of the liquid?  (1981)

Q.18 Figure shows a siphon. Choose the wrong


statement:
P hysi cs | 9.49

Q.3 A body floats in a liquid contained in a beaker. Q.7 A large open tank has two holes in the wall. One is
The whole system as shown in figure falls freely under a square hole of side L at a depth y from the top and
gravity. The upthrust on the body due to the liquid is: the other is a circular hole of radius R at a depth 4y
 (1982) from the top. When the tank is completely filled with
water the quantities of water flowing out per second
from both the holes are the same. Then R is equal to
 (2000)
(A) L / 2π (B) 2πL
(A) Zero (C) L (D) L/2p
(B) Equal to the weight of the liquid displaced
(C) Equal to the weight of the body in air Q.8 A wooden block, with a coin placed on its top,
floats in water as shown in fig. The distance l and h
(D) Equal to the weight of the immersed position of the are shown there. After some time the coin falls into the
body water. Then: (2002)

Q.4 A U-tube of uniform cross-section is partially filled


with a liquid I. Another liquid II which does not mix
with liquid I is poured into one side. It is found that the l
liquid levels of the two sides of the tube are the same,
while the level of liquid I has risen by 2 cm. If specific h
gravity of liquid I is 1.1, the specific gravity of liquid I is
1.1, the specific gravity of liquid II must be:  (1983)
(A) 1.12 (B) 1.1 (C) 1.05 (D) 1.0 (A) l Decreases and h increases
(B) Increases and h decreases
Q.5 A homogeneous solid cylinder of length L. Cross-
sectional area A/5 is immersed such that it floats with (C) Both l and h increase
its axis vertical at the liquid-liquid interface with length (D) Both l and h decrease
L/4 in the denser liquid as shown in the fig. The lower
density liquid is open to atmosphere having pressure
p0. Then density D of solid is given by: (1995) Q.9 Water is filled in a cylindrical container to a height
of 3 m. The ratio of the cross-sectional area of the
orifice and the beaker is 0.1. The square of the speed of
the liquid coming out from the orifice is (g = 10 m/s2)
 (2005)

5 4 d
(A) d (B) d (C) 4d (D)
4 5 5
(A) 50 m2/s2 (B) 50.5 m2/s2
Q.6 Water from a tap emerges vertically downwards (C) 51 m2/s2 (D) 52 m2/s2
with an initial speed of 1.0 m/s. The cross-section
area of the tap is 10–4m2. Assume that the pressure is
constant throughout the steam of water and that the Q.10 A glass tube of uniform internal radius (r) has
flow is steady, the cross-sectional area of stream 0.15 m a valve separating the two identical ends. Initially,
below the tap is:  (1998) the valve is in a tightly closed position. End 1 has a
hemispherical soap bubble of radius r. End 2 has sub-
(A) 5.0×10-4 m2 (B) 1.0×10-4 m2 hemispherical soap bubble as shown in figure.  (2008)
(C) 5.0×10-5 m2 (D) 2.0×10-4 m2
9 . 5 0 | Fluid Mechanics

Q.13 A uniform cylinder of length L and mass M having


cross-sectional area A is suspended, with its length
vertical, from a fixed point by a massless spring, such
that it is half submerged in a liquid of density σ at
equilibrium position. The extension x0 of the spring
when it is in equilibrium is:  (2013)

Mg  LAσ  Mg  LAσ 
Just after opening the valve: (A) 1 −  (B) 1 − 
k  M  k  2M 
(A) air from end 1 flow towards end 2. No change in the
volume of the soap bubbles. Mg  LAσ  Mg
(C) 1 +  (D)
(B) air from end 1 flows towards end 2. Volume of the k  M  k
soap bubble at end 1 decreases
(Here k is spring constant)
(C) no change occurs
(D) air from end 2 flows towards end 1. Volume of the Q.14 Assume that a drop of liquid evaporates by decrease
soap bubble at end 1 increases in its surface energy, so that its temperatureremains
unchanged. What should be the minimum radius of
Q.11 A uniform cylinder of length L and mass M thedrop for this to be possible? The surface tensionis
having cross-sectional area A is suspended, with its T, density of liquid is ρ and L is its latent heat of
length vertical, from a fixed point by a massless spring, vaporization.  (2013)
such that it is half-submerged in a liquid of density p
(A) T / ρL (B) T / ρL
at equilibrium position. When the cylinder is given a
small downward push and released it starts oscillating
(C) 2 T / ρL (D) ρL / T
vertically with a small amplitude. If the force constant
of the spring is k1the frequency of oscillation of the
cylinder is  (1990) Q.15 An open glass tube is immersed in mercury in
such a way that a length of 8 cm extends above the
1 −2 1/2
1  k − Aρg  1  k + Aρg  mercury level. The open end of the tube is then closed
(A)   (B)  
2π  M  2π  M  and sealed and the tube is raised vertically up by
additional 46cm. What will be length of the air column
1/2 1/2 above mercury in the tube now? (Atmospheric pressure
1  k + ρgL2  (D) 1  k + Aρg 
(C)   = 76 cm of Hg)  (2014)
2π  M 

2π  Aρg 
(A) 38 cm (B) 6 cm (C) 16 cm (D) 22 cm
Q.12 A thin liquid film formed between a U-shaped
wire and a light slider supports a weight of 1.5 × 10−2 N Q.16 On heating water, bubbles being formed at the
(see figure). The length of the slider is 30 cm and its bottom of the vessel detach and rise. Take the bubbles
weight negligible. The surface tension of the liquid film to be spheres of radius R and making a circular contact
is (2012) of radius r with the bottom of the vessel. If r < < R, and
the surface tension of water is T, value of r just before
bubbles detach is: (density of water is ρw )  (2014)

Film

W 2r

(A) 0.0125 Nm-1 (B) 0.1 Nm-1 ρw g 3 ρw g


(A) R 2 (B) R 2
(C) 0.05 Nm-1 (D) 0.025 Nm-1 T T

ρw g
(C) R 2 (D) None of these
3T
P hysi cs | 9.51

JEE Advanced/Boards

Exercise 1 Q.5 A test tube of thin walls has lead shots in it at its
bottom and the system floats vertically in water, sinking
by a length l = 10 cm. A liquid of density less than that
Q.1 A piston of mass M = 3 kg and radius R=4 cm of water, is poured into the tube till the levels inside
has a hole into which a thin pipe of radius r = 1 cm and outside the tube are even. If the tube now sinks to
is inserted. The piston can enter a cylinder tightly and a length l = 40 cm, the specific gravity of the liquid is
without friction, and initially it is at the bottom of the …………..
cylinder. 750 gm of water is now poured into the pipe
so that the piston and pipe are lifted up as shown. Find
Q.6 A large tank is filled with two liquids of specific
the height H of water in the cylinder and height h of
gravities 2σ and σ. Two holes are made on the wall of
water in pipe.
the tank as shown. Find the ratio of distances from O
of the points on the ground where the jets from holes
A and B strike.

Q.2 A solid ball of density half of that of water falls freely


under gravity from a height of 19.6m and then enters
Q.7 A jet of water having velocity = 10 m/s and stream
the water. Upto what depth will the ball go? How much
cross-section = 2 cm2 hits a plate perpendicularly, with
time will it take to come again to the water surface?
the water splashing out parallel to plate. Find the force
Neglect air resistance & velocity effects in water.
that the plate experiences.

8m Q.8 A laminar stream is flowing vertically down from


L
600kg a tap of cross-section area 1 cm2. At a distance 10 cm
below the tap, the cross-section area of the stream has
reduced to 1/2m2. Find the volumetric flow rate of water
from the tap.

Q.3 For the system shown in the figure, the cylinder


Q.9 A cylindrical vessel open at the top is 20 cm high
on left at L has a mass of 600 kg and a cross sectional
and 10 cm in diameter. A circular hole whose cross-
area of 800 cm2. The piston on the right, at S, has cross
sectional area is 1 cm2 is cut at the centre of the bottom
sectional area 25 cm2 and negligible weight. If the
of the vessel. Water flows from a tube above it into the
apparatus is filled with oil (ρ = 0.75 gm/cm3). Find the
vessel at the rate 100 cm3 s–1. Find the height of water
force F required to hold the system in equilibrium.
in the vessel under steady state.

Q.4 (a) A spherical tank of 1.2 m radius is half filled with


Q.10 Calculate the rate of flow of glycerin of density
oil of relative density 0.9. If the tank is given a horizontal
1.25×103 kg/m3 through the conical section of a 0.1m
acceleration of 10 m/s2, calculate the inclination of the
and 0.04m and the pressure drop across its length is
oil surface to horizontal and maximum gauge pressure
10N/m2.
on the tank.
(b) The volume of an air bubble is doubled as it rises from Q.11 A ball is given velocity v0 (greater than the
the bottom of a lake to its surface. If the atmospheric terminal velocity vT) in downward direction inside a
pressure is H m of mercury & the density of mercury highly viscous liquid placed inside a large container. The
is n times that of lake water, find the depth of the lake. height of liquid in the container is H. The ball attains
9 . 5 2 | Fluid Mechanics

the terminal velocity just before striking at the bottom rises to its rim. What is the density of the material of
of the container. Draw graph between velocity of the which the beaker is made?
ball and distance moved by the ball before getting
terminal velocity. Q.17 A level controller is shown in the figure. It
consists of a thin circular plug of diameter 10 cm and a
cylindrical float of diameter 20 cm tied together with a
light rigid rod of length 10 cm. The plug fits in snugly in
a drain hole at the bottom of the tank which opens into
the atmosphere. As water fills up and the level reaches
height h, the plug opens. Find h. Determine the level
of water in the tank when the plug closes again. The
float has a mass 3kg and the plug may be assumed as
massless.
Q.12 A spherical ball of radius 1 × 10–4m and density
104 kg/m3 falls freely under gravity through a distance
h before entering a tank of water. If after entering the
water the velocity of the ball does not change, find h.
The viscosity of water is 9.8×10–6 N-s/m2.

Q.13 Two arms of a U-tube have unequal diameters


d1 = 10 mm and d2 = 1.0cm. If water (surface tension
7×10–2N/m) is poured into the tube held in the vertical
ρ
position, find the difference of level of water in the Q.18 A cylindrical rod of length l=2m and density
U-tube. Assume the angle of contact to be zero. 2
floats vertically in a liquid of density ρ as shown in fig.
(a)

B B

A A
(a) (b)
Q.14 A soap bubble has radius R and thickness d(<<R) (a) Show that it performs SHM when pulled slightly up
as shown. It collapses into a spherical drop. Find the & released & find its time period. Neglect change in
ratio of excess pressure in the drop to the excess liquid level.
pressure inside the bubble. (b) Find the time taken by the rod to completely
immerse when released from position shown in figure
(b). Assume that it remains vertical throughout its
motion.
(take g = p2m/s2)

Q.19 A thin rod of length L and area of cross-section


S is pivoted at its lowest point P inside a stationary,
Q.15 Two soap bubbles with radii r1 and r2 (r1>r2) homogeneous & non-viscous liquid (Figure). The rod
come in contact. Their common surface has a radius is free to rotate in a vertical plane about a horizontal
curvature r. axis passing through P. the density d1 of the material of
the rod is smaller than the entity d2 of the liquid. The
Q.16 Place a glass beaker, partially filled with water, in rod is displaced by a small angle θ from its equilibrium
a sink. The beaker has mass 390 gm and an interior position and then released. Show that the motion of
volume of 500 cm3. You now start to fill the sink with the rod is simple harmonic and determine its angular
water and you find, by experiment, that if the beaker is frequency in terms of the given parameters.
less than half full, it will float; but if it is more than half
full, it remains on the bottom of the sink as the water
P hysi cs | 9.53

A1
d1 60° 60°
30°

d2
A A
P

Q.20 A hollow cone floats with its axis vertical up to Q.25 A siphon has a uniform circular base of diameter
one-third liquid of its height in a liquid of relative 8
density ρ is filled in it up to one-third of its height, the cm with its crest A 1.8 m above water level as in
π
cone floats up to half its vertical height. The height of
figure. Find
the cone is 0.10 m and the radius of the circular base is
0.05m. Find the specific gravity ρ. (a) Velocity of flow.
(b) Discharge rate of the flow in m3/sec.
Q.21 In the figure shown, the heavy cylinder (radius
(c) Absolute pressure at the crest level A.
R) resting on a smooth surface separates two liquids
of densities 2ρ and 3ρ. Find the height ‘h’ for the [Use P0=105 N/m2& g=10m/s2]
equilibrium of cylinder.

R 3
2
h R

Q.22 The vertical limbs of a U shaped tube are


filled with a liquid of density ρ up to a height h on Q.26 Two very large open tanks A and F both contain
each side. The horizontal portion of the U tube the same liquid. A horizontal pipe BCD, having a
having length 2h contains a liquid of density 2ρ. The constriction at C leads out of the bottom of tank A, and
U tube is moved horizontally with an acceleration g/2 a vertical pipe E opens into the constriction at C and
parallel to the horizontal arm. Find the difference in dips into the liquid in tank F. Assume streamline flow
heights in liquid levels in the two vertical limbs, at and no viscosity. If the cross section at C is one half that
steady state. at D and if D is at a distance h1 below the level of liquid
in A, to what height h2 (in terms of h1) will liquid rise in
Q.23 A wooden stick of length l and radius R and density pipe E?
ρ has a small metal piece of mass m (of negligible
volume) attached to its one end. Find the minimum
value for the mass m (in terms of given parameters)
that would make the stick float vertically in equilibrium
in a liquid of density σ(>ρ).

Q.24 A vertical cylindrical container of base area A and


upper cross-section area A1 making angle 30º with the Q.27 A cube with mass ‘m’ completely wet by water
horizontal placed in an open rainy field as shown near floats on the surface of water. Each side of the cube is
another cylindrical container having same base area A. ‘a’. What is the distance h between the lower face of
Find the ratio of rates of collection of water in the two cube and the surface of the water as rw. Take angle of
containers. contact as zero.
9 . 5 4 | Fluid Mechanics

Exercise 2 the top apex of the cone has a small hole. Neglecting
atmospheric pressure, the total upward force exerted
Single Correct Choice Type by water on the cone is:
(A) (2/3)pR2hrg (B) (1/3)pR2hrg
Q.1 A bucket contains water filled up to a height = (C) pR2hrg (D) None
15cm. The bucket is tied to a rope which is passed on
a frictionless light pulley and the other end of the rope
Q.6 A slender homogeneous rod of length 2L floats
is tied to a weight of mass which is half of that of the
partly immersed in water, being supported by a string
(bucket + water). The water pressure above atmosphere
fastened to one of its ends, as shown. The specific
at the bottom is:
gravity of the rod is 0.75. The length of rod that extends
(A) 0.5 kPa (B) 1 kPa out of water is:
(C) 5 kPa (D) None of these

Q.2 A cone of radius R and height H, is hanging inside a


liquid of density ρ by means of a string as shown in the
figure. The force, due to the liquid acting on the slant
surface of the cone is (Neglect atmosphere pressure)

(A) L2 (B) L2/2


(C) L2/4 (D) 3L2/4

Q.7 A dumbbell is placed in water of density ρ. It is


observed that by attaching a mass m to the rod, the
dumbbell floats with the rod horizontal on the surface
(A) prgHR2 (B) pρHR2 of water and each sphere exactly half submerged
(C) 4/3 prgHR2 (D) 2/3 prgHR2 as shown in the figure. The volume of the mass m is
negligible. The value of length  is:

Q.3 An open cubical tank was initially fully filled with


water. When the tank was accelerated on a horizontal
plane along one of its side, it was found that one third
of volume of water spilled out. The acceleration was:
(A) g/3 (B) 2g/3
(C) 3g/2 (D) None

Q.4 Some liquid is filled in a cylindrical vessel of radius d(Vρ − 3M) d(Vρ − 2M)
R. Let F1be the force applied by the liquid on the bottom (A) (B)
2(Vρ − 2M) 2(Vρ − 3M)
of the cylinder. Now the same liquid is poured into a
vessel of uniform square cross-section of side R. Let F2
d(Vρ + 2M) d(Vρ − 2M)
be the force applied by the liquid on the bottom of this (C) (D)
new vessel. (Neglect atmosphere pressure). Then: 2(Vρ − 3M) 2(Vρ + 3M)

(A) F1 = πF2 (B) F1 = F2/p


Q.8 A small wooden ball of density ρ is immersed in
(C) F1 = πF2 (D) F1 = F2 water of density σ to depth h and then released. The
height H above the surface of water up to which the
Q.5 A heavy hollow cone of radius R and height h is ball will jump out of water is:
placed on a horizontal table surface, with its base on
σh σ 
the table. The whole volume inside the cone is filled (A) (B)  − 1  h
with water of density ρ. The circular rim of the cone’s
ρ ρ 
base has a water tight seal with the table’s surface and (C) h (D) Zero
P hysi cs | 9.55

Q.9 A sphere of radius R and made of material of Q.13 A cubical block of side ‘a’ and density ‘ρ’ slides
relative density σ has a concentric cavity of radius r. It over a fixed inclined plane with constant velocity
just floats when placed in a tank full of water. The value ‘v’. There is a thin film of viscous fluid of thickness ‘t’
of the ratio R/r will be: between the plane and the block. Then the coefficient
1/3 1/3 of viscosity of the thin film will be:
 σ   σ −1
(A)   (B)  
 σ −1  σ 
1/3 1/3
 σ +1  σ −1 
(C)   (D)  
 σ   σ +1

Q.10 A fire hydrant delivers water of density ρ at a


volume rate L. The water travels vertically upward 3ρ a g t 4ρ a g t
(A) (B)
through the hydrant and then does 90º turn to emerge 5v 5v
horizontally at speed V. The pipe and nozzle have
ρagt
uniform cross-section throughout. The force exerted (C) (D) None of these
5v
by the water on the corner of the hydrant is:

Q.14 Which of the following graphs best represent the


motion of a raindrop?

(A) rVL (B) Zero

(C) 2rVL (D) 2 rVL

Q.11 A cylindrical vessel filled with water up to height


of H stands on a horizontal plane. The side wall of the
vessel has a plugged circular hole touching the bottom.
The coefficient of friction between the bottom of vessel Q.15 Which of the following is the incorrect graph
and plane is µ and total mass of water plus vessel is M. for a sphere falling in a viscous liquid? (Given at t = 0,
What should be the minimum diameter of the hole so velocity v = 0 and displacement x = 0)
that the vessel begins to move on the floor if plug is
v v
removed (here density of water is ρ)
(A) (B)
2µM µM t
(A) (B) t
πρH 2πρH

µM v v
(C) (D) None
ρH (C) (D)
t t
Q.12 A Newtonian fluid fills the clearance between a
shaft and a sleeve. When a force of 800N is applied to Q.16 A container, whose bottom has round holes with
shift, parallel to the sleeve, the shaft attains of 1.5 cm/ diameter 0.1 mm is filled with water. The maximum
sec. If a force of 2.4 kN is applied instead, the shaft height in cm up to which water can be filled without
would move with a speed of leakage will be what?
(A) 1.5 cm/sec (B) 13.5 cm/sec Surface tension=75×10–3N/m and g=10 m/s2:
(C) 4.5 cm/sec (D) None (A) 20 cm (B) 40 cm (C) 30 cm (D) 60 cm
9 . 5 6 | Fluid Mechanics

Q.17 A liquid is filled in a spherical container of radius (B) 5 cm of it will be under water.
R till a height h. At this position the liquid surface at the
(C) 2 cm of it will be above the common surface of oil
edges is also horizontal. The contact angle is:
and water.
(D) 8 cm of it will be under water.

Q.20 Water coming out of a horizontal tube at a speed


v strikes normally a vertically wall close to the mouth of
h the tube and falls down vertically after impact. When is
the speed of water increased to 2v.
(A) the thrust exerted by the water on the wall will be
R −h
(A) 0 (B) cos−1   doubled.
 R 
(B) the thrust exerted by the water on the wall will be
h−R  R −h four times
(C) cos−1   (D) sin−1  
 R   R  (C) the energy lost per second by water striking the wall
will also be four times

Q.18 The vessel shown in the figure has two sections. (D) the energy lost per second by water striking the wall
The lower part is a rectangular vessel with area of cross- be increased eight times.
section A and height h. The upper part is a conical
vessel of height h with base area ‘A’ and top area ‘a’ and Q.21 A beaker filled with water is accelerated a m/
the walls of the vessel are inclined at an angle 30º with s2 in +x direction. The surface of water shall make on
the vertical. A liquid of density ρ fills both the sections angle:
up to a height 2h. Neglecting atmospheric pressure,
(A) tan–1(a/g) backwards
a (B) tan–s draw of (g/a)1
h
30° (C) cot–1(g/a) backwards
(D) cot–1 (a/g) backwards

h
Q.22 The spring balance A read 2 kg with a block m
suspended from it. A balance B reads 5 kg when a
beaker with liquid is put on the pan of the balance. The
A two balances are now so arranged that the hanging
mass is inside the liquid in the beaker as shown in the
(A) The force F exerted by the liquid on the base of the
figure in this situation:
(A + a)
vessel is 2hrg
2 A
(B) The pressure P at the base of the vessel is 2hrg
a
(C) The weight of the liquid W is greater than the force
exerted by the liquid on the base.
(D) The walls of the vessel exert a downward force (F-
W) on the liquid.

Multiple Correct Choice Type

Q.19 A cubical block of wood of edge 10 cm and mass


0.92 kg floats on a tank of water with oil of relative
density 0.6 to a depth of 4 cm above water. When the
block attains equilibrium with four of its side edges (A) The balance A will read more than 2kg
vertical, (B) The balance B will read more than 5 kg
(A) 1 cm of it will be above the force of oil.
P hysi cs | 9.57

(C) The balance A will read less than 2 kg and B will read
more than 5 kg.
(D) The balance A and B will read 2 kg and 5 kg
respectively

Q.23 When an air bubble rises from the bottom of a


deep lake to a point just below the water surface, the
Q.27 Statement-I: Submarine sailors are advised that
pressure of air inside the bubble:
they should not be allowed to rest on floor of the ocean.
(A) Is greater than the pressure outside it
Statement-II: The force exerted by a liquid on a
(B) Is less than the pressure outside it submerged body may be downwards.
(C) Increases as the bubble moves up
Q.28 Statement-I: When a body floats such that it’s
(D) Decreases as the bubble moves up
parts are immersed into two immiscible liquids then
force exerted by liquid-1 is of magnitude r1v1g.
Q.24 A tank is filled up to a height h with a liquid and is
placed on a platform of height h at a distance of y from Statement-II: Total Bouyant force r1v1g + r2v2g.
the free surface of the liquid. Then

Q.29 Statement-I: When temperature rises the


coefficient of viscosity of gases decreases.
Statement-II: Gases behave more like ideal gases at
higher temperature.

(A) xm = 2h (B) xm = 1.5 h


Q.30 Statement-I: The free surface of a liquid at rest
(C) y = h (D) y = 0.75 h with respect to stationary container is always normal

to the geff .
Assertion Reasoning Type Statement-II: Liquids at rest cannot have shear stress.
(A) Statement-I is true, statement-II is true and
Statement-II is the correct explanation for statement-I
(B) Statement-I is true, statement-II is true and statement- Previous Years’ Questions
II is NOT the correct explanation for statement-I
(C) Statement-I is true, statement-II is false. Q.1 A hemispherical
portion of radius R
(D) Statement-I is false, statement-II is true is removed from the
bottom of a cylinder of
h
Q.25 Statement-I: A helium filled balloon does not rise radius R. The volume of
indefinitely in air but halts after a certain height. the remaining cylinder
is V and mass M. It is 
Statement-II: Viscosity opposes the motion of balloon.
suspended by a string in a
liquid of density ρ, where
Q.26 Statement-I: A partly filled test tube is floating it stays vertical. The upper 2R
in a liquid as shown. The tube will remain as if its surface of the cylinder is
atmosphere pressure changes. at a depth h below the
Statement-II: The buoyant force on a submerged liquid surface. The force on the bottom of the cylinder
object is independent of atmospheric pressure. by the liquid is  (2001)
9 . 5 8 | Fluid Mechanics

(A) Mg (B) Mg – Vrg Q.4 Now level of the liquid starts decreasing slowly.
When the level of liquid is at a height h1 above the
(C) Mg + rR2hrR (D) rg(V + pR2h)
cylinder the block starts moving up. At what value of h1,
will the block rise?  (2005)
Q.2 When a block of iron floats in mercury at 0°C,
fraction k1 of its volume is submerged, while at the (A) 4h/9 (B) 5h/9
temperature 60°C, a fraction k2 is seen to be submerged. (C) 5h/3 (D) Remains same
If the coefficient of volume expansion of iron is γFe and
that of mercury is γHg , then the ratio k1 / k 2 can be
Q.5 The block in the above question is maintained at
expressed as  (2001)
the position by external means and the level of liquid is
1 + 60 γ Fe 1 − 60 γ Fe lowered. The height h2 when this external force reduces
(A) (B) to zero is: (2006)
1 + 60 γ Hg 1 + 60 γ Hg

1 + 60 γ Fe 1 + 60 γHg
(C) (D)
1 − 60 γ Hg 1 + 60 γ Fe

Q.3 Water is filled up to a height h in a beaker of radius


R as shown in the figure. The density of water is ρ,
the surface tension of water is T and the atmospheric 4h 5h 2h
pressure is p0. Consider a vertical section ABCD of the (A) (B) (C) Remains same (D)
9 9 3
water column through a diameter of the beaker. The
force on water on one side of this section by water on
Q.6 If height h2 of water level is further decreased, then:
the other side of this section has magnitude (2007)
 (2006)
(A) cylinder will not move up and remains at its original
position
(B) for h2=h/3, cylinder again starts moving up
(C) for h2=h/4, cylinder again starts moving up
(D) h2=h/5, cylinder again starts moving up

Paragraph 2: (Q.7 - Q.9)


(A) | 2p0Rh + πR 2ρgh − 2RT |
When a liquid medicine of density ρ is to be put in the
(B) | 2p0Rh + Rρgh2 − 2RT |
eye, it is done with the help of a dropper. As the bulb
(C) | p0 πR 2 + Rρgh2 − 2RT | on top of the dropper is pressed, a drop forms at the
opening of the dropper. We wish to estimate the size
(D) | p0 πR 2 + Rρgh2 − 2RT | of the drop.
We first assume that the drop formed at the opening
Paragraph 1: (Q.4 - Q.6) is spherical because that requires minimum increase in
its surface energy. To determine the size, we calculate
A wooden cylinder of diameter 4r, height h and density
the net vertical force due to the surface tension T when
ρ/3 is kept on a hole of diameter 2r of a tank, filled with
the radius of the drop is R. When this force becomes
liquid of density ρ as shown in the figure.
smaller than the weight of the drop, the drop gets
detached from the dropper. (2010)

Q.7 If the radius of the opening of the dropper is r, the


vertical force due to the surface tension on the drop of
radius R (assuming r<<R) is
2πr 2 T 2πR 2 T
(A) 2prT (B) 2pRT (C) (D)
R r
P hysi cs | 9.59

Q.8 If r = 5×10–4m, r = 103 kg m–3, g= 10ms–2, T =0.11 Q.12 A thin uniform cylindrical shell, closed at both
Nm–1, the radius of the drop when it detaches from the ends, is partially filled with water. It is floating vertically
dropper is approximately: in water in half-submerged state. If ρc is the relative
density of the material of the shell with respect to water,
(A) 1.4×10–3 m (B) 3.3×10–3 m
then the correct statement is that the shell is –  (2012)
(C) 2.0×10–3 m (D) 4.1×10–3 m
(A) More than half-filled if ρc is les sthan 0.5

Q.9 After the drop detaches, its surface energy is: (B) More than half-filled if ρc is less than 0.5

(A) 1.4×10–6 J (B) 2.7×10–6 J (C) Half-filled if ρc is more than 0.5

(C) 5.4×10–6 J (D) 9.1×10–9 J (D) Less than half – filled if ρc is less than 0.5

Q.10 The spring A reads 2 kg with a block m suspended Q.13 A solid sphere of radius R and density ρ is attached
from it. A balance reads 5 kg when a beaker with liquid to one end of a mass-less spring of force constant k.
is put on the pan of the balance. The two balances The other end of the spring is connected to another
are now so arranged that the hanging mass is inside solid sphere of radius R and density 3 ρ . The complete
the liquid in the beaker as shown in the figure. In this arrangement is placed in a liquid of density 2ρ and is
situation:  (1985) allowed to reach equilibrium. The correct statement(s)
is (are) (2013)
4 π R3 ρ g
(A) the net elongation of the spring is
3k
8 π R3 ρ g
(B) the net elongation of the spring is
3k
(C) the light sphere is partially submerged.
(D) the light sphere is completely submerged.

Paragraph for Questions 14 and 15


A spray gun is shown in the figure where a piston
pushes air out of a nozzle. A thin tube of uniform cross
(A) The balance A will read more than 2 kg section is connected to the nozzle. The other end of the
(B) The balance A will read more than 5 kg tube is in a small liquid container. As the piston pushes
air through the nozzle, the liquid from the container
(C) The balance A will read less than 2 kg and B will read rises into the nozzle and is sprayed out. For the spray
more than 5 kg gun shown, the radii of the piston and the nozzle are
(D) The balances A and B will read 2 kg and 5 kg 20 mm and 1 mm respectively. The upper end of the
respectively. container is open to the atmosphere.

Q.11 Two solid spheres A and B


of equal volumes but of different
densities dA and dB are connected by
a string. They are fully immersed in a Q.14 If the piston is pushed at a speed of 5 mms-1 , the
fluid of density dF. They get arranged air comes out of the nozzle with a speed of (2014)
into the equilibrium state as shown in
the figure with a tension in the string. (A) 0.1 ms-1 (B) 1 ms-1
The arrangement is possible only if (2011) (C) 2 ms-1 (D) 8 ms-1
(A) dA< dF (B) dB > dF
(C) dA> dF (D) dA + dB= 2dF
9 . 6 0 | Fluid Mechanics

Q.15 If the density of air is ρa and that of the liquid ρ , Q.17 Two spheres P and Q of equal radii have densities
for a given piston speed the rate (volume per unit time) ρ1 and ρ2 , respectively. The spheres are connected
at which the liquid is sprayed will be proportional to by a massless string and placed in liquids L1 and L2
 (2014) of densities σ1 and σ2 and viscosities η1 and η2 ,
respectively. They float in equilibrium with the sphere
ρa P in L1 and L2 has terminal
(A) (B) ρa ρ  velocity VP and Q alone in
ρ L1 has terminal velocity VQ , then  (2015)

ρ
(C) (D) ρ L1
ρa P

Q.16 A person in a lift is holding a water jar, which has L2


a small hole at the lower end of its side. When the lift Q
is at rest, the water jet coming out of the hole hits the
floor of the lift at a distance d of 1.2 m from the person.
In the following, state of the lift’s motion is given in List  
I and the distance where the water jethits the floor of VP η VP η
(A)  = 1 (B)  = 2
the lift is given in List II. Match the statements from List VQ η2 VQ η1
I with those in List II and select the correct answer using
the code given below the lists.  (2014)    
(C) VP . VQ > 0 (D) VP . VQ < 0

List I List II Q.18 A spherical body of radius R consists of a fluid of


1. Lift is accelerating (p) d=1.2 m constant density and is in equilibrium under its own
vertically up. gravity. If P(r) is the pressure at r(r < R), then the correct
option(s) is(are)  (2015)
2. Lift is accelerating (q) d > 1.2 m
vertically down with an P (r = 3R / 4 ) 63
acceleration less than the (A) P (= ) 0
r 0= (B) =
P (r = 2R / 3) 80
gravitational acceleration.
3. Lift is moving vertically up (r) d < 1.2 m P (r = 3R / 5 ) 16 P (r = R / 2 ) 20
(C) = (D) =
with constant speed. P (r = 2R / 5 ) 21 P (r = R / 3) 27
4. Lift is falling freely. (s) No water leaks
out of the jar Q.19 Consider two solid spheres P and Q each of density
Code: 8 gm cm-3 and diameters 1cm and 0.5cm, respectively.
Sphere P is dropped into a liquid of density 0.8 gm cm-3
(A) 1 - q, 2 - r, 3 - q, 4-s and viscosity η =3 poiseulles. Sphere Q is dropped
(B) 1 - q, 2 - r, 3 - p, 4 - s into a liquid of density 1.6 gm cm-3 and viscosity η =2
poiseulles. The ratio of the terminal velocities of P and
(C) 1 - p, 2 - p, 3 - p, 4 - s Q is (2016)
(D) 1 - q, 2 - r, 3 - p, 4 - p
P hysi cs | 9.61

PlancEssential Questions
JEE Main/Boards JEE Advanced/Boards
Exercise 1 Exercise 1
Q. 7 Q.9 Q.15 Q.3 Q.6 Q.9
Q.16 Q.20 Q.17

Exercise 2 Exercise 2
Q. 1 Q.7 Q.9 Q.1 Q.4 Q.10
Q.13 Q.17 Q.11 Q.19 Q.22

Previous Years’ Questions Previous Years’ Questions


Q.8 Q.9 Q.10 Q.7 Q.8 Q.9

Answer Key

JEE Main/Boards
Exercise 1
Q.2 Zero Q.6 (2)2/3 vT Q.7 2.714×10–9 m/s
Q.9 Streamline, turbulent Q.10 Turbulent Q.13 60×108 Pa
Q.14 27.6 m Q.15 9.56 N Q.16 60 kg
Q.17 72π × 10−4 N Q.18 0.0224 Q.19 3.5 × 10−2 Nm−1
Q.20 1.785×10–4 m

Exercise 2

Single Correct Choice Type


Q.1 C Q.2 B Q.3 D Q.4 D Q.5 A Q.6 A
Q.7 C Q.8 B Q.9 D Q.10 C Q.11 A Q.12A
Q.13 C Q.14 C Q.15 D Q.16 C Q.17 B Q.18 D
Q.19 D
9 . 6 2 | Fluid Mechanics

Previous Years’ Questions


Q.1 C Q.2 A Q.3 A Q.4 B Q.5 A Q.6 C
Q.7 A Q.8 D Q.9 A Q.10 B Q.11 B Q.12 D
Q.13 B Q.14 C Q.15 C Q.16 D

JEE Advanced/Boards

Exercise 1

2m 11
Q.1
= h = ,H m Q.2 19.6m, 4 sec. Q.3 37.5 N
π 32π

Q.4 (a) 9600 2 , (b) nH Q.5 0.75 Q.6 3: 2

Q.7 20N Q.8 4.9 litre/min Q.9 5 cm

Q.10 6.43×10–4 m3/s Q.11

1
 R 3
Q.12 20 m Q.13 2.5 cm Q.14  
 24d 
r1r2
Q.15 r = Q.16 2.79 gm/cc
r1 − r2

2(3 + π) 3+π
Q.17
= h1 = 0.26; h1 = 0.195
= Q.18 2 sec., 1 sec
15π 10π

3g  d2 − d1  3
Q.19 w =   Q.20 1.9 Q.21 R
2L  d1  2

8h
Q.22
7

Q.23 mmin = πr 2 
buoyancy.
( )
ρσ − ρ ; if tilted then it’s axis should become vertical, C.M. should be lower than centre of

Q.24 2 : 1 Q.25 (a) 6 2 m/s, (b) 9.6 2 × 10−3 M3 / sec, (c) 4.6×104

mg + 4Sa
Q.26 h2 = 3h1 Q.27 h =
ρw a2g

Exercise 2

Single Correct Choice Type

Q.1 B Q.2 D Q.3 B Q.4 D Q.5 A Q.6 A


Q.7 B Q.8 B Q.9 A Q.10 D Q.11 A Q.12 C
Q.13 A Q.14 C Q.15 C Q.16 C Q.17 B Q.18 D
P hysi cs | 9.63

Multiple Correct Choice Type


Q.19 C, D Q.20 B, D Q.21 A, C Q.22 B, C Q.23 A, D Q.24 A, C

Assertion Reasoning Type


Q.25 B Q.26 D Q.27 A Q.28 D Q.29 D Q.30 A

Previous Years’ Questions


Q.1 D Q.2 A Q.3 B Q.4 C Q.5 A Q.6 A
Q.7 C Q.8 A Q.9 B Q.10 B, C Q.11 A, B, D Q.12 A
Q.13 A, D Q.14 C Q.15 A Q.16 C Q.17 A, D Q.18 B, C

Solutions

JEE Main/Boards Sol 6: Volume remains same so


2 × 4/3 pr3 = 4/3 π R3
Exercise 1 R = (2)1/3 r
VT ∝ r2
Sol 1: Castor oil will come to rest first because its
viscosity is greater than water ⇒ V’ T = k 21/3 R2 = (2)2/3 VT

Sol 2: Acceleration is zero as velocity is constant Sol 7: r = 0.2 mm = 2 × 10–4 m


v = 4cm/s = 4 × 10–2 m/s
Sol 3: Flow rate is equal in any part of the body so F = 6π η rv
A1 V1 = constant = 6π × 1.8 × 10–5 × 2 × 10–4 × 4 × 10–2
2
D  = 6π × 14.4 × 10–11 = 2.714 × 10–9 m/s
π   V = constant
2
Sol 8: Refer Q-6 Exercise –I JEE Main
Sol 4: Viscosity of gas increases with increase in kη
temperature Sol 9: Critical velocity = Vc =
ρr
k = Reynolds’s number = 1000
Sol 5: For gas, viscosity of gases are independent of
density and pressure but viscosity of gas increases with 1000 × 10−3 1
increase in temperature VC = = = 0.16 m/s
1.25 6.25
1000 ×
For liquids:– Viscosity decreases with increase in 200
temperature. Viscosity increase with increase in density 0.8
Q = flow rate = 0.48L/min = L/sec = 8 × 10–6 m3/
viscosity of liquid is normally independent of pressure, 100
sec
but liquid under extreme pressure after experience an
increase in viscosity  1.25 
2
Area = π   × 10 = 1.227 × 10
–4 –4

 2 
9 . 6 4 | Fluid Mechanics

Q 8 × 10−6 6
Velocity = = m = 1000 × = 60 kg
A 1.222 × 10−4 100

= 6.5 × 10–2 m/s Sol 17: Force = 2pr S


V1 < VC ⇒ Streamline 5
= 2π × × 72 × 10-3
When flow rate is 3L/min 100
3 1 = π × 72 × 10–4
Q’ = 3L/min = L/sec = × 10–3 m3/sec
60 20
A = 1.227 × 10–4 Sol 18: F = 2 × perimeter × S

Q' 1 × 10−3 1
V2 = = =2×4× × 28 × 10–3
A 20 × 1.227 × 10−4 10
= 8 × 28 × 10–4
1
= = 0.40 m/s = 0.0224
2 × 1.227
V2 > Vc ⇒ turbulent flow
Sol 19: Pressure inside above atmospheric pressure
Sol 10: Refer Q – 9 Exercise–I JEE Main 4T
ρgh =
r
Sol 11: Apparent weight of the floating block is zero. 4T × 2
104 × 8 × 10–3 =
3.5 × 10−3
Sol 12: Up thrust will be zero as body is not exerting T = 3.5 × 10–2 Nm–1
any force on water during free fall and there is no
buoyant force 2T
Sol 20: h =
rρg
F 60
Sol 13: Pressure = = = 60 × 108 Pa
A 10−8 2 × 70 × 10−3
r=
8 × 10−2 × 10 4
Sol 14: 370×103 = ρgh + 105
70
= × 10–5 = 1.785 × 10–4 m
ρgh = (3.7–1) × 105 4
2.7 × 105
h= = 27.6 m
9.8 × 103
Exercise 2
Sol 15:
Single Correct Choice Type

Sol 1: (C) Pressure due to difference in heights will be


balanced by pressure due to 12 kg block

120
⇒ ρgh =
T = mg – ρwvg 800 × 10−4

= 9000 ×125×10–6×9.8 – 1200 × 125 × 10–6 × 9.8 120


104 h = × 10 4
800
= 7800 × 125 × 10–6 × 9.8
= 7.8 × 125 × 10–3 × 9.8= 9.56 N 12 3
h= = m = 15 cm
80 20
Sol 16: Change in depth corresponds to mass of man
1
ρ×3×2× × 10 = m × 10
100
P hysi cs | 9.65

Sol 2: (B) ⇒ Tension become 1/9 of the initial

3
T
s1=600kg/m

h SVg
3
s=1000kg/m mg
2
v1=1 .h=h 3mg
For water ρVg =
3 4
s2=1150kg/m
8
for liquid dρVg = mg
9
8 32
Downward force on the cubes = (m1 + m2) g ⇒d= ×4=
27 27
= ρ1 Vg + ρ2 Vg
(1750) × 10 Sol 5: (A) By work energy theorem
Upward force on the cubes = ρ(V1 + V) g Wwater + Wgravity = ∆KE = 0
= 1000 (h + 1) × 10 Wwater = (ρVg) h
Since cubes are in equilibrium
Wgravity = –(0.8 ρ Vg) (h + 2)
So 17500 = 10000 (h + 1)
⇒ ρVgh – 0.8 ρVg (h + 2) = 0
1.75–1 = h
5h
⇒ h = 0.75 m h - 0.8 (h + 2) = 0 ⇒ =h+2
4
h
Sol 3: (D) = 2 ⇒ h = 8m
4


Sol 6: (A) The vertical component of tension balances
out the net of weight & buoyancy.

In equilibrium Pushed down Sol 7: (C) We know that time taken for the vessel to
by y distance
2H
empty is to = , H = height of water
Initially in equilibrium g
When pushed down by y distance, an extra upward H
Time taken to empty vessel of height is t2
force will act on the cube η
ρ (ybc) g = d ρ abc A 2H
=
[A = acceleration of the cube] gη

da g g g t1 = t0 – t2 and t1 = t2
y= A⇒A= y ⇒ ω2 = ⇒ω= ⇒T=
g da da da
2H 2H 2H 2H 2H
2π da ⇒ – = ⇒ = 2 ⇒η=4
= 2π g gη gη g gη
w g

Sol 8: (B) Bernoulli’s theorem is derived by the


Sol 4: (D) f ∝ √T , T = tension in the wire
conservation of energy.
in water frequency becomes f/2
⇒ Tension becomes ¼ of the initial Sol 9: (D) Volume flow rate is same
in liquid frequency becomes f/3 So π (1 × 10–2)2 × 3
9 . 6 6 | Fluid Mechanics

2 Q = A1 v1 = A2 v2
 0.05 × 10−2 
= 100 × π   ×V
 2  Q 2 × 10−2
  v2 = = = 4 m/s
A2 1 / 2 × 10−2
π × 10–4 × 3
By Bernoulli equation
= 100 × π × ¼ × 25 × 10–8 × V
4×3 P1 + ½ ρv12 = P2 + ½ ρv22
V= × 100 = 48
25 8000 + ½ × 1000 × 4 = P2 + ½ × 1000 × 16
10000 = P2 + 8000
Sol 10: (C) We know that force exerted by fluid coming
out on the container is ρAv2 P2 = 2000 Pa

v = velocity of fluid
Sol 14: (C) Viscosity is not an assumption
H
v= 2g
2 Sol 15: (D) Frictional resistance f ∝ Av2
A = area of the hole f = kAv2 = kpr2v2
2
ρAv k = constant
Acceleration of the tank =
ρ(NAH)
When ball acquires terminal velocity
ρ(AgH) g
= = f = mg
ρNAH N
kπ r2 v2 = mg
Sol 11: (A) A1V1 = A2 V2 k π r2 v2 = (4/3 π r3) ρ g
π (10–2)2 VP = π (2 × 10–2)2 VQ v2 ∝ r ⇒ v ∝ r1/2
VP = 4VQ
Sol 16: (C)
Sol 12: (A)

D
P2=P0+4T/R
h P1=P0+4/R
H-D as P1 > P2 so air will flow out of the small bubble.

Sol 17: (B)


R
Velocity of water = 2Dg

Time taken by water to come to the ground

2(H − D) h
t=
g

Distance where water hit the surface = vt


2T
h=
2(H − D) Rρg
2Dg . = 2 D(H − d)
g R = radius of curvature
2T
hR = = constant
Sol 13: (C) A = volume flow rate ρg
= 10–2 × 2 = 2 × 10–2 m3/s So graph of R vs. h will be hyperbola
P hysi cs | 9.67

Sol 18: (D) By Bernoulli equation Force on a water particle P on the free surfaces have
been shown in the figure. In the figure ma is the pseudo
P0 + ρgh3 = P0 + ½ ρv2
force.
½ ρv2 = ρgh3
P0 = P2 + ½ ρv2 Sol 3: (A) In a freely falling system geff = 0 and since,
Upthrust = V1ρLgeff
P2 = P0 – ρgh3
(V1 = immersed volume, ρL = density of liquid)
Sol 19: (D) By continuity Upthrust = 0.
Ax Vx = Ay Vy
Sol 4: (B)
Ax > Ay
⇒ vx < vy
By Bernoulli equation
Px + ½ ρvx2 = Py + ½ ρvy2 h

vx < vy 1 2
⇒ Px > Py
KE per m3 of water = ½ ρv2
KEx= = ½ ρvx2
KEy = ½ ρvy2 p1 = p2 ⇒ p0 + ρIgh = p0 + ρIIgh

KEx < KEy \ρI = ρII

Sol 5: (A)

Previous Years’ Questions


d
D 3L/4

Sol 1: (C) Wapp = Wactual – Upthrust


L/4
Upthrust F = VsρLg
2d
Here, Vs = volume of solid,
Considering vertical equilibrium of cylinder
rL= density of liquid.
Weight of cylinder = Upthrust due to upper
At higher temperature F’ = V’sρ’Lg
liquid+ upthrust due
F' V's ρ 'L (1 + γ s ∆θ)
=
∴ = . to lower liquid.
F Vs ρL (1 + γL ∆θ)
∴ (A/5) (L) Dg = (A/5) (3L/4) (d)g
Since gs < γL (given)
+ (A/5)(L/4)(2d)(g)
∴F’ < F or W’app > Wapp
3 1
∴ D =   d +   (2d)
Sol 2: (A) Net force on the free surface of the liquid 4
  4
in equilibrium (from accelerate frame) should be
5
perpendicular to it. D= d
4

ma P Sol 6: (C) From conservation of energy


90o
2
F v=
2 v12 + 2gh  …..(i)
Fnet mg [can also be found by applying Bernoulli’s theorem]
From continuity equation
9 . 6 8 | Fluid Mechanics

A1 v1 = A2 v 2 1
A 
h
v 2 =  1  v1  …..(ii)
 A2 
2
Substituting value of v2 from Eq. (ii) in Eq. (i)
A12 2
.v= v12 + 2gh
A22
1 Solving eq. (i) and (ii), we have

A12 v12 2gh


or A22 = . v 22 =
v12 + 2gh A22
1−
A1 v1 A12
∴ A2 =
Substituting the values, we have
v12 + 2gh
2 × 10 × 2.475
Substituting the given value =v 22 = 50m2 / s2
1 − (0.1)2
(10−4 )(1.0)
A2 =
(1.0)2 + 2(10)(0.15) 4T 4T
Sol 10: (B) Dp1 = and Dp2 =
A2 = 5.0 × 10 m –5 2 r1 r2
r1 < r2
Sol 7: (A) Velocity of efflux at a depth h is given by v \ Dp1 > Dp2
= 2gh . Volume of water flowing out per second from ∴ Air will flow from 1 to 2 and volumes of bubble at
both the holes are equal. end 1 will decrease.

\ a1v1 = a2v2 Therefore, correct option is (B).

or (L2 ) 2g(y) = pR2 2g(4y) Sol 11: (B) When cylinder is displaced by an amount x
from its mean position, spring force and upthrust both
L
or R= will increase. Hence, Net restoring fore = extra spring
2π force + extra upthrust

Sol 8: (D) l will decreases because the block moves up  k + ρAg 


or F = – (kx + Ax rg) or a = –  x
and h will decrease because the coin will displace the  M 
volume of water (V1) equal to its own volume when it
is in the water whereas when it is on the block it will 1 a 1 k + ρAg
displace the volume of water (V2) whose weight is equal Now, f = =
2π x 2π M
to weight of coin and since density of coin is greater
than the density of water, V1 < V2.

Sol 9: (A) Applying continuity equation at 1 and 2, we K


have
A1v1 = A2v2 …..(i)
Further applying Bernoulli’s equation at these two
points, we have
1 1
p0 +rgh + rv12 = p0 + 0 + rv22 ….(ii)
2 2

Sol 12: (D) The force of surface tension acting on the


slider balances the force due to the weight.
F 2T =
⇒=  w
P hysi cs | 9.69

⇒ 2T ( 0.3) =
4
1.5 × 10−2 2 R ρw g
 r2 =
⇒ T = 2.5 × 10−2 N / m
F = 2Tl 3 T

2 ρw g
r = R2
3T
w
Sol 13: (B) 
kx0 Buoyant force
At equilibrium ∑ F =0

 AL  
kx0 +  σ g  − Mg =
0
 2 

 LAσ  Mg
x0 Mg 1 −
= 
 2M 
JEE Advanced/Boards
Sol 14: (C) Exercise 1
= T4 π R 2 − (R − ∆R ) 
2 2
ρ4 πR ∆RL
  Mg
Sol 1: Pressure at A = P0 + = P0 + ρgh
A
ρR 2 ∆
=RL T R 2 − R 2 + 2R ∆R − ∆R 2 
  A = π (0.04)2 – (0.01)2 = π × 15 × 10–4
R
ρR 2 ∆RL = T2R ∆R ( ∆ R is very small) M
= ρh
A
2T M 3 2
R=  h= = = m
ρL Aρ −4 π
π × 15 × 10 × 1000

Sol 15: (C) mass of water = 750 gm = 0.75 kg

( 76 ) ( 8 ) =(54 − x ) ( 76 − x ) mass of water below piston

x = 38 cm = 0.75 - (1000) (π × (0.01)2) × h

Length of air column = 54 – 38 = 16 cm. = (1000) × π × (0.04)2 × H


π 2 16π
0.75 = × + ×H
10 π 10
16π 5.5 11
Air 0.55 = H⇒H= = m
10 16π 32π

46+8=54 Sol 2: Net force on the ball in water


x ρw Vg ρw Vg
FB = ρwVg - = = mg
2 2
Air 8 Air 8

19.6m

h
Sol 16: (D)
4
( 2 π r T ) sin θ= 3
π R3 ρw .g Let us assume that ball will go up to depth h in water.
By work energy theorem
r 4
T × × 2 πr = πR3 ρw g -mg (19.6) + mg h = 0 ⇒ h = 19.6 m
R 3
9 . 7 0 | Fluid Mechanics

Upward force F = mg in water


= 800 × 10 2 × 1.2 = 9600 2 N/m2
Acceleration = g
(b) h = depth of lake
Time required to come on surface
When bubble is at bottom pressure inside
2s 2 × 19.6
= = = 2 sec = P0 + ρgh
g 9.8
When bubble is at surface pressure is = P0
Time required to go inside surface is also 2 sec
T0 = surface tension
So total time required = 4 sec
2T T
P0 = =
2r r
Sol 3:
2T
P0 + ρgh =
r
A
T 2T
+ ρgh =
r r
T
B ρgh = = P0 = ρm g hm
r
ρm = nρ
F h = nH
Pressure at point A = P0 + = PA
A1
mg Sol 5: Upward force on test tube initially = ρsA×0.1 g
Pressure at point B = P0 + = PB
A2 Upward force after adding liquid = ρwA×0.4g
Difference in pressure = ρg × 8 = PB – PA Weight of the fluid = ρwA × 0.4g – ρwA × 0.1g = ρ’ A ×
0.4 g
F mg
⇒ P0 + – P0 – = –ρg × 8 3ρw
A1 A2 ⇒ ρ’ =
4
F 6000 15
= – 750 × 10 × 8 = × 104 – 6 × 104
A1 800 × 10 −4 2 Sol 6: At point A by Bernoulli equation
F 3
= × 10–4 h 1 gh
A1 2 P0 + σ g = σ v2 + P0 ⇒ v =
4 2 2
F = 1.5 × 104 × 25 × 10–4
2(3h / 4) 3h
F = 37.5 N Time take = =
g 2g

Sol 4: Distance travelled = vt

gh 3g
= ×
2 2g

h
Distance = 3
 2
At point B
h
a P0 = σg = P0 + (+2σ g (–h/4)) + ½ 2σ v’2
(a) tan θ = =1 2
g
gh gh
θ = 45° =- + v’2
2 2
Maximum gang pressure = ρ a2 + g2 r
V’ = gh
P hysi cs | 9.71

Sol 9:
2(h / 4) h
Time t’ = =
g 2g
3
2=100cm /S

h h
Distance travelled = vt’ = gh =
2g 2 h

200cn
h
h 3 3
Ratio of distance travelled = =
h 2
2
2 2=100cm /S
3

Sol 7: Force exerted is change in momentum per sec


By Bernoulli’s equation
d(mv) dm P0 + ρgh = P0 + ½ ρv2
= = v
dt dt
= v ρ Av v= 2gh

= ρAv2 = 1000 × 2 × 10–4 × 100 Q = 100 cm3/s


= 20 N A = 1 cm2
v = 100 cm/s = 1 m/s
Sol 8: By Bernoulli’s equation
P0 + ρgh + ½ ρv12 = P0 + ½ ρv22 v= 2 × 10 × h = 1
1
By continuity equation h= m = 5 cm
20
A1v1 = A2v2
v2 Sol 10:
v1 =
2 Q
2 r1=0.1m
v  1 2
gh + ½  2  = v
2
  2 2

v 22
gh = [1–1/4] = 3/8 v22
2 r2=0.04m
Q
8gh 2gh 2
v2 = ⇒ v1 = =
3 3 3 By Bernoulli’s equation
2 P1 + ½ ρV12 = P2 + ½ ρV22
Volume flow rate = × 10–4 m3/s
3 – P2 + P1 = 10 N/m2

2 10 + ½ × 1250 v12 = ½×1250 v22  ….(i)


= 60 × 10–4 m3/min
3 Continuity equation

2 1 A1v1 = A2v2
= 60 × litre/min
3 10 π (0.1)2 v1 = π (0.04)2 v2

= 4.9 litre/min 1 4×4


v1 = v
100 10000 2
v1 = 0.16 v2  …..(ii)
By (i) and (ii)
10 + 625 (0.16 v2)2 = 625 (v2)2
625 (0.9744) v22 = 10
9 . 7 2 | Fluid Mechanics

v2 = 0.128 m/s Sol 14: Surface energy of bubble = 4pr2 T1


Q = A2v2 Surface energy of drop = 4pr22 T2
= π (0.04)2 × 0.128 Volume of bubble and drop is same so
= π × 16 × 10–4 × 0.128 4πR2d = (4/3) πr23
= 6.44 × 10–4 m3/s r23 = 3R2d
r22 = (3R2d) 2/3
Sol 11: Since velocity is greater than terminal velocity,
so it will decrease until velocity reaches terminal velocity P1 4T 2r 24d
= r2 = 2 = 3
P2 R2T R R

V0 P2 R
= 3
P1 24d
Velocity

4T 4T 4T
Sol 15: P0 + + = P0 +
VT r1 r r2

Dist. moved H r1r2


⇒r=
r1 − r2
2 2 (ρ − σ ) g
Sol 12: Terminal velocity = V = r η
9 Sol 16: Let the volume of bearer be V
2 (10 4 − 103 ) × 9.8 Then balancing force on beaker
= 10–8
9 9.8 × 10−6 ρw V.g = (0.39)g + ρw × 250 g
v = 20 m/s 103 × V = 0.39 + 0.25
Distance required to reach terminal velocity is V = 640 × 10–6 m3

v2 (20)2 400 V = 60 cm3


h= = = = 20 m
2g 2 × 10 20 Volume of glass = 640 – 500 = 140 cm3
m 390
Density = = = 2.785 gm/cc
Sol 13: v 140

A
Sol 17: Plug will open when float is lifted upwards due
h1 B to buoyant force

2T
PA = P0 – h’
r1

2T Balancing force we get


PB = P0 –
r2 ρ h’ × π (0.1)2 × g = 3g
1 1  3 3
PB – PA = ρgh = 2T  −  = 2T [2000 – 200] h’ = =
 r1 r2  1 10π
1000 × π ×
ρgh = 2 × 2 × 7 × 9 = 252 100
3 1 3+π
252 height h = h’ + 10 cm = + =
h= = 2.5 cm 10π 10 10π
10 × 103
P hysi cs | 9.73

Sol 18: d1 α
=
( d1 − d2 ) gθ
3 2
y 3/2x2Ag
3(d1 − d2 )g
α= θ
2d1 

3(d1 − d2 )g
(SxAg) ω=
2d1 
(a) By Newton’s second law
a = Upward acceleration Sol 20:
−ρ
× 2 Ag + ρ(1 – y) Ag = (ρ/2) 2Aa 0.05m
2
–ρAg + ρ(1–y) Ag = ρAa
r r
–g + g – gy = a =
3h
h/3 0.05 h/2
a = –gy h/3

acceleration is density proportional to the displacement h = 0.1m h = 0.1m


so it will perform SHM
h = 0.1m
a = –w2 y
By force balance
w2 = g
 1  0.05 2  h  
w= g = π2 = π mg = 0.8  π      …..(i)
 3  3   3 
2π  
Time period = = 2 sec
ω When liquid is added
(b) Time taken for rod to go from 1 extreme position to  1  0.05 2  h  
other is half of the time period Mg + ρ  π    
 3  3   3 
 
So time taken = 2/2 = 1 sec
 1  0.05 2  h  
=0.8  π       ......(ii)
Sol 19:  3  2   2 
 
By (i) and (ii)
d’
 1 (0.05)2 h   1 (0.05)2 h 
d2 (0.8)  π  +ρ  π 
3 33  3 33 
  
  1 (0.05)2 h 
d, 5L = 0.8  π 
3 23 

Torque on rod = (d1  Sg – d2  sg) l/2 sin θ 0.8 + ρ 0.8
= = 0.1 ⇒ 0.8 + ρ = 2.7
for small θ 3 3 8

τ = (d – d2) (  2/2) s g θ ρ = 1.9

Net torque = Iα
Sol 21:
Iα = (d1 – d2)  s g θ
2

(d1s ) 2
I= 2S-h 3-S
3 R
d1s3 2
α = (d1 – d2) ρgθ
3 2 Balancing force on both sides
9 . 7 4 | Fluid Mechanics

Horizontal force acting on the cylinder can be assumed Sol 23:


to be acting on the cross–sectional area in the vertical
direction
h 3ρgR.R
2ρgh. =
2 2
3 2
h2 = R
2

3
h= R
2 h
h/2
Sol 22:

For the rod to be in equilibrium centre of mass of (rod


+ mass m) system should be below centre of gravity of
S
the volume displaced by the rod.

S For minimum m should coincide so.


h2 h1 Suppose h length of rod is below water then, by force
balance
σ(pR2h) g – ρlpR2 g – mg = 0
2S h-h1
(pR2) (σh – ρ  ) = m  …..(i)
h = h2 – (h – h1) Reaction of centre of mass should be at h/2 distance
h = h2 – h + h1 from bottom

h1 + h2 ρπR 2 × 
h= 2
2 ⇒ h/2 =  …..(ii)
m + ρπR 2
P0 + ρgh1 + ρa (h – h1) + 2ρa (h + h1) – 2ρg (h – h1) –
ρgh = P0
ρ2 ρ
⇒ h2 = ⇒h=   …. (iii)
gh1 + ah – ah1 + 2ah + 2ah1 – 2gh + 2gh1 – gh = 0 σ 6
h1 (g – a + 2a + 2g) + h (a + 2a – 2g – g) = 0
By (i) and (iii)
3(g − a)h
h1 =
3g + a m = ( σ − ρ) pR2 l

a = g/2
Sol 24: Volume of water collected = A.V
3h
A = cross sectional area perpendicular to the rain.
3h
⇒ h1 = 2 =
1 7 v = velocity of rain
3+
2 in 1st beaker A2 = A1 cos 30°
h2 = h + h – h1 in 2nd beaker A’3 = A cos 60°
h2 = 2h – h1
>

n
Difference in height = h2 – h1
8h o
30
= 2h – h1 – h1 = 2(h – h1) =
o
60
7

o o
30 30
P hysi cs | 9.75

A Sol 27: By force equilibrium on the cube


A1 =
cos30° Fgravity + Fbuoyant + Fsurface tension = 0
A
⇒ A2 = cos 30° = A –mg + ρw a2 hg – S × 4a = 0
cos30°
A mg + 4Sa
⇒ A3 = h=
2 ρw a2g
Q2 A2 V
So = =2:1
Q3 A3 V

Sol 25: (a) By Bernoulli’s equation Exercise 2


P0 + ρg(3.6) = P0 + ½ ρv2
Sol 1: (B) By Newton’s second law
v= 2g(3.6) = 72

v = 6 2 m/s
(b) Discharge rate = pr2 v
T
16 a
=π× × 10–4 × 6 2 m3/s
π T m/2

= 9.6 2 × 10–3 m3/s m/2g


m a
(c) By Bernoulli equation
PA + ½ ρv2 + ρg (5.4) = P0 + ½ ρV2 mg
PA = P0 – ρ g × 5.4 mg ma
T– =
= 10 × 10 – 5.4 × 10
4 4 2 2
m
= 4.6 × 10–4 Pa T= (g + a)  .…. (i)
2
–T + mg = me  ….. (ii)
Sol 26: Pressure at C = Pc
By (i) and (ii)
By Bernoulli’s equation
−m
P0 + ρgh1 = P0 + ½ ρVD2 (g + a) + mg = ma
2
½ ρVD2 = ρgh1
−g −a
P0 + ρgh1 = Pc + ½ ρVC2 +g=a
2 2
AC VC = AD VD
A g 3a g
= ;a=
AC = D 2 2 3
2
⇒ vC = 2vD  g
Effective acceleration of the bucket is
g − 
P0 + ρgh1 = PC + 2ρVD2  3
downwards water pressure at the bottom above
⇒ Pc =P0+ρgh1 – 4ρgh1 = P0 – 3 ρgh1  ….(i)
atmospheric pressure is
Pressure at C can also be written as
2g 2 15
P= hρ = 1000 × × 10 × = 1 kPa
PC + ρgh2 = P0 3 3 100
PC = P0 – ρgh2  …… (ii)
Sol 2: (D) Buoyant force = sum of all forces acting on
By (i) and (ii) the body
ρgh2 = 3 ρgh1 = force acting on the slant surface
h2 = 3h1 + force acting on the bottom surface
9 . 7 6 | Fluid Mechanics

FB = Fs + Fb Sol 6: (A) A
FB = (1/3) pR2 Hρg
Fb = pR2 ρgH l1
2L-L1
⇒ Fs = (–2/3) pR2 ρgH
2
Sol 3: (B) mg

a -L 1
Ltan 2L

L L Let the length of rod that extends out of water is l1
since the rod is in equilibrium
So balancing net torque about point A
A = area of the base
A = area of the base  2L + L1 
we get (ρA(2L–L1)g)   cos q
tan θ = a/g  2 
Finally 1/3 rd of the water spilled out = 0.75 ρAL g L cos θ
So volume of water spilled out finally 4L2 − L21 3 2
3 = L
2 tan θ × A L tan θ 2 4
= Vf = =
2 2
this is 1/3 volume of L3 Sol 7: (B) 
tan θ 1
⇒ = ⇒ tan θ = 2/3 = a/g d d/2
2 3
a = 2g/3 By force equilibrium we get
ρVg ρVg
–Mg – 2Mg – mg + + =0
Sol 4: (D) Force applied by the liquid will be same on 2 2
both the vessels as the mass of liquid is same in both ⇒ m = ρv – 3M  …..(i)
the vessels
By torque equilibrium about mass M we get
Sol 5: (A) Total force exerted on the base by water and dρVg
–mg (d –  ) – 2 Mgd + =0
cane’s slant surface = mg 2
= 1/3 pR2 Hρg downwards ρV
m  – d (m + 2M – )=0
2
Force exerted by the water =
 ρv 
d  2M + m − 
(ρgH) (pR2) downwards 2 
 =   …(ii)
So force exerted by the slant surface = m
2/3 ρgH pR2 upwards d(ρV − 2M)
By (i) and (ii) we get  =
2(ρV − 3M)
So force exerted by water on slant surface = 2/3 ρgH
pR2
Sol 8: (B) By work energy theorem
Wwater + Wgravity = ∆KE = 0
(σ v g h) – ρ v g (h + H) = 0
σ h = ρ (h + H)

(σ − ρ)h σ 
H= =  − 1 h
ρ  ρ 
P hysi cs | 9.77

Sol 9: (A) Buoyant force = ρw × 4/3 πR3 g Sol 14: (C) Graph (c) best represents the motion of
raindrop because velocity of rain approaches the
Gravitational force = (σρw) (4/3 π(R3 – r3)) g
terminal velocity.
Sphere is in equilibrium so
Sol 15: (C) Graph (D) incorrect because at t = 0; x = 0
ρw 4/3 π R3 g = (σρw) (4/3 π(R3 – r3) g)
and graph will not be straight time
R3 = σ(R3 – r3)
Sol 16: (C)
1 r3
=1– P0
σ R3
r3 1 σ −1
=1– =
R 3 σ σ h
1/3
R  σ 
=  
r  σ −1

Sol 10: (D) Force exerted = change in momentum per P = P0 + 2T/r


sec
per sec P0 + 2T/r – ρgh = P0
mv x
2T
= ρgh
mv y r

2T 2 × 75 × 10−3
h= = = 0.30 m
rρg 10−4
mvjˆ − mviˆ × 1000 × 10
= = ρLvjˆ − ρLviˆ = 2 ρvL 2
t
h = 30 cm
Sol 11: (A) Force exerted by water = ρAV 2

A = area of hole Sol 17: (B)

V = velocity of water through hole


Friction force = μ Mg
for the vessel to just move R
R-h
ρAV = μ Mg
2

90-
πD2 2µM h
ρ× × 2 g H = µMg ⇒ D =
4 πρH

R −h
Sol 12: (C) We know that force applied is proportional cos (90 – θ) = sin θ =
R
to velocity of shaft. So if the force is increased three
times, velocity will also increase three times. R −h
Angle of contact = 90 – θ = cos–1  
 R 
dv
Sol 13: (A) Viscous force F = – η A
dx Sol 18: (D) Force exerted by liquid = ρg (2h).A = F
v weight of liquid is W
F = –η A
t
Force exerted by liquid on walls = F – W
3mg
F = mg sin 37° = (upwards)
5
So force exerted by the walls on the liquid
3mgt 3ρa3gt 3ρagt
η= = =
5AV 2
5a V 5V = (F – W) downwards
9 . 7 8 | Fluid Mechanics

Multiple Correct Choice Type Sol 21: (A, C)


A
Sol 19: (C, D)

a
0.1-h1-h2 h
h2 
10cm B
0.92kg C
Oil L
h1
Water
PA = P0
PB = P0
PB = PA + ρgh – ρa  = P0
Balancing net force on the block we get
gh = a 
–0.92×10 + (1000)×h1×(0.01)×10
h a
+ (600)h2×(0.01) × 10 = 0 tan θ = =
L g
10 h1 + 6h2 = 0.92
Sol 22: (B, C)
if h2 = 4 cm
then 10h1 + 6 × 0.04 = 0.92
10h1 = 0.68
0.68 5av
h1 = = 0.068 m = 6.8 cm
10
mg
h1 + h2 should be less than 10 cm so
Balance A will read less than 2 kg as an upward buoyant
h2 < 4 cm
force is acting on the block. Balance B will read more
and h1 + h2 = 10 cm than 5 kg as downward reaction of the block due to
buoyant force is acting on beaker.
⇒ 10 h1 + 6 (0.1 – h1) = 0.92
4h1 + 0.6 = 0.92
Sol 23: (A, D)
4h1 = 0.32
h1 = 0.08 m
⇒ h1 = 8 cm
h
h2 = 2 cm

Bubble at
Sol 20: (B, D) Thrust exerted by the water is ρAV2 if
velocity is doubled then thrust will increase 4 times. the bottom

dm 2 Pressure inside the bubble at the bottom is


Energy lost per second = ½ v
dt 2T
P1 = P0 + ρgh +
= ½ ρAv. v2 = ½ ρAv3 r1
Pressure inside the bubble near the surface is
If velocity is doubled then energy lost per second will
be 8 times 2T
P2 = P0 +
r2
Pressure inside the bubble near the surface is
2T
P2 = P0 +
r2
So pressure will decrease as we move upwards.
P hysi cs | 9.79

Sol 24: (A, C) Sol 28: (D)


h
Velocity of fluid coming out of the hole =
1 v1 s1
v= 2gy
2 s2
time taken by the fluid to collide with surface = v2

2(h + h − y)
t=
g
range = vt Force exerted by liquid – 1 = (ρgH + P0) A downwards
2(h + h − y) So statement-I is false
= 2gy .
g
dR Sol 29: (D) Coefficient of viscosity of gases increase
R= 4y(2h − y) . For maximum R, =0
dy with increasing temperature
1
⇒ (2h – 2y) = 0
4y(2h − y) Sol 30: (A) Free surface is always perpendicular to the
geff. Liquids at rest can have only normal forces.
⇒y=h

Rmax = 4h2 = 2h
Previous Years’ Questions
Assertion Reasoning Type
Sol 1: (D) F2 – F1 = upthrust
Sol 25: (B) Pressure of air decreases with increase in F1
height so when pressure outside the balloon is equal to
balloon pressure, it will not size up.
Upthrust
h
Sol 26: (D)

Pressure=6 

h 2R

∴ F2 = F1 + upthrust
F2= (p0 + rgh) pR2 + Vrg
Pressure inside the tube is P = P0 + ρgh
=p0pR2+rg(pR2h+V)
When pressure changes height will also change.
∴ Most appropriate option is (D).
So Statement-I is true.
ρ  ρ 
Buoyant force is independent of atmospheric pressure. Sol 2: (A) k1 =  Fe  and k 2 =  Fe  .
 ρHg   ρHg 
  0° C  60°C
Sol 27: (A) Suppose submarine is resting on the floor,
then water is exerting only net downward force on Here, ρ = Density
the submarine as lower surface is not available for the
upward force. k1 (ρ )  ρHg  (1 + 60 γFe )
∴ = Fe 0°C ×   =
k 2 (ρHg )0°C   (1 + 60 γHg )
 ρFe 60°C

Note: In this problem two concepts are used:


9 . 8 0 | Fluid Mechanics

(i) When a solid floats in a liquid, then Equating the net downward forces and net upward
ρsolid 5
Fraction of volume submerged (k) = forces, we get, h1 = h .
ρliquid 3
This result comes from the fact that Sol 5: (A) Again equating the forces, we get
Weight = Upthrust
h2=4h/9
Vρsolid
= g Vsubmerged ρliquidg ρ0A1

Vsubmerged ρsolid ρ/3 ghA1


\ =
V ρliquid
ρθ°C 1
(ii) =
ρ0°C 1+ γ⋅θ ρ0A2 (ρ0+ρgh2)A3
1
This is because ρ ∝ (mass remaining constant)
Volume
4h
ρθ°C V0°C V0°C Sol 6: (A) For h2 < , buoyant force will further
∴ = = 9
ρ0°C Vθ°C V0°C + ∆V decrease. Hence, the cylinder remains at its original
position.
V0°C 1
= =
V0°C + V0°C γθ 1 + γθ
Sol 7: (C) Vertical force due to surface tension.
Fv =F sin q
Sol 3: (B) Force from right hand side liquid on left hand
side liquid.
F
(i) Due to surface tension force
θ
=2RT (towards right) r
θ
(ii) Due to liquid pressure force R
x =h

= ∫x =0 (p0 + ρgh)(2R.x)dx
= (2p0Rh + Rρgh2 )(towards left)

= (T2pr)(r/R)
∴ Net force is |2p0Rh + Rrgh2 – 2RT|
2πr 2 T
=
Sol 4: (C) Let R

A1 = Area of cross-section of cylinder = 4pr2 2πr 2 T 4


Sol 8: (A) =mg = pR3. ρ.g
A2 = Area of base of cylinder in air = pr2 R 3
and A3 = Area of base of cylinder in water 3r 2 T 3 × (5 × 10−4 )2 (0.11)
∴ R4 = =
= A1 – A2 = 3pr2 2ρg 2 × 103 × 10
Drawing free body diagram of cylinder = 4.125 × 10–12 m4
∴ R = 1.425 × 10–3 m
(p0+h1ρg)A1
≈ 1.4 × 10−3 m

(ρ/3) ghA1 Sol 9: (B) Surface energy,


E = (4pR2)T
= (4π) (1.4×10–3)2 (0.11)
p0A2 {p0+ρg(h1+h)}A3
= 2.7×10–6 J
P hysi cs | 9.81

Sol 10: (B, C) Liquid will apply an upthrust on m. An V 1 


equal force will be exerted (from Newton’s third law) on Vw= + Vc  − ρc 
2 2 
the liquid. Hence, A will read less than 2 kg and B more
than 5 kg. Therefore, the correct options are (B) and (C). 1 V
F If ρc < ;V >
2 w 2
Sol 11: (A, B, D)
Sol 13: (A, D) At equilibrium,
A WA
4 4
πR 3 2 ρ g = πR 3ρg + T
T 3 3
4
T T= π R3 ρ g
3
B F 4
∴ ∆=
 π R3 ρ g
WB 3k
3
(4/3)R 2g
F = upthrust = VdFg
Equilibrium of A 3
(4/3)R 2g
VdFg = T + WA
= T + VdAg ….(i)
T
Equilibrium of B T
T + VdFg = VdBg ….(ii) 3
(4/3)R 2g
Adding eqns. (i) and (ii), we get
2df = dA + dB
3
∴ Option (D) is correct. (4/3)R 2g

From Eq. (i) we can see that


For equilibrium of the complete system, net force of
dF > dA (as T>0) buoyancy must be equal to the total weight of the
sphere which holds true in the given problem. So both
∴ Option (A) is correct.
the spheres are completely submerged.
From equation (ii) we can see that,
dB > dF Sol 14: (C) By A1 V1 = A2 V2
∴ Option (B) is correct.
⇒ π ( 20 ) × 5 =π (1 ) V2 ⇒ V2 = 2 m / s2
2 2

∴ Correct options are (A), (B) and (D).

1 1
Sol 12: (A) Sol 15: (A) ρa Va2 = ρ V2
2 2
Inside volume = V For given Va
Container material
ρa
volume = VC V ∝
ρ
Vw

Sol 16: (C) In P, Q, R no horizontal velocity is imparted


to falling water, so d remains same.
mc g + mwh =
FB
In S, since its free fall, aeff = 0
V V  ∴ Liquid won’t fall with respect to lift.
ρc Vc g + 1Vw g = 1  + c  g
2 2 
9 . 8 2 | Fluid Mechanics

Sol 17: (A, D) From the given conditions,


ρ1 < σ1 < σ2 < ρ2

2  ρ1 − σ2  2  ρ2 − σ1 
VP =   g and VQ  g
9  η2  9  η1 

VP η  
So,  = 1 and VP . VQ < 0
VQ η2

 2 
(r ) K  1 − r 2 
Sol 18: (B, C) P=
 R 

2 r2
Sol 19: Terminal velocity=
vT
9 η
( ρ − σ ) g , where ρ
is the density of the solid sphere and σ is the density

of the liquid
2

vP
( 8 − 0.8 ) ×  12  × 2
=∴ =   3
vQ 1
2

( 8 − 1.6 ) ×  4  × 3
 
2017-18 100 &
op kers
Class 11 T
By E ran culty
-JE Fa r
IIT enior emie .
S fP r es
o titut
Ins

PHYSICS
FOR JEE MAIN & ADVANCED
SECOND
EDITION

Exhaustive Theory
(Now Revised)

Formula Sheet
9000+ Problems
based on latest JEE pattern

2500 + 1000 (New) Problems


of previous 35 years of
AIEEE (JEE Main) and IIT-JEE (JEE Adv)

5000+Illustrations and Solved Examples


Detailed Solutions
of all problems available

Topic Covered Plancess Concepts


Tips & Tricks, Facts, Notes, Misconceptions,
Gravitation Key Take Aways, Problem Solving Tactics

PlancEssential
Questions recommended for revision
10. G R AV I TAT I O N

1. INTRODUCTION
Have you ever wondered whether we would still be studying about with Gravitation if a
stone had fallen on Newton’s head instead of an apple? Anyways, the real question is,
why does an apple fall down rather than go upward?

2. NEWTON’S LAW OF UNIVERSAL GRAVITATION


“Every particle of matter in the universe attracts every other particle with a force equal
to the product of masses of particles and inversely proportional to the square of the
distance between them”
If m1 and m2 are two point masses separated by a distance r, the gravitational force of Figure 10.1
attraction F is given by
m1m2 r
F ∝
r2
Gm1m2 m1 m1
F =
2
r Figure 10.2
Where G is a constant and is called the Universal gravitational
constant.
Magnitude (and unit) of G : 6.67 × 10−11newton. 22
Newton.mm kg2 2
//kg
Dimension of G : M-1L3T-2

PLANCESS CONCEPTS
The direction of force F is independent of the medium, not affected by the presence of the other bodies
and acts along the line joining the two particles.
If two persons come very close to each other such that the distance between them is almost 0, the two
persons should experience a high force of attraction. Observe keenly the value of G. It’s of order -11.
The Universal gravitational constant G is an experimental value calculated by Cavendish 71 years after
the law was formulated.
Always remember Gravitational Force is conservative in nature i.e. work done doesn’t depend on the
path taken and depends only on the end points.
Vaibhav Gupta (JEE 2009, AIR 54)
1 0 . 2 | Gravitation

Illustration 1: Two particles of masses 1.0 kg and 2.0 kg are placed at a separation of 50 cm. Assuming that the
only forces acting on the particles are their mutual gravitation, find the initial accelerations of the two particles.
 (JEE MAIN)
Gm1m2
Sol: The force of mutual gravitation acting on particles is F = . As the particle are accelerating under the
r2
force of gravitation, the acceleration is obtained using Newton’s laws of motion.
The force of gravitation exerted by one particle on the other is
N − m2
6.67 × 10−11 × (1.0kg) × (2.0kg)
Gm1m2 kg2
F= = = 5.3 × 10−10 N.
r2 (0.5m) 2

F 5.3 × 10−10 N
The acceleration of 1.0 kg particle is a=
1 = = 5.3 × 10−10 ms−2
m1 1.0kg

This acceleration is towards the 2.0 kg particles. The acceleration of the 2.0 kg particle is

F 5.3 × 10−10 N
a2
= = = 2.65 × 10−10 ms−2
m2 2.0kg

This acceleration is towards the 1.0 kg particle.

Illustration 2: Spheres of the same material and same radius r are touching each other.
Show that gravitational force between them is directly proportional to r 4 . (JEE MAIN)

Sol: The force of gravitation is directly proportional to the masses of the spheres. As the
spheres are having the same masses, and mass m ∝ V ⇒ m ∝ r 3 thus the proportionality
between the force and distance is easily established.
As the spheres are made of same material, and density so the mass of each sphere is
2r
m1=m2= (volume) (destiny) Figure 10.3

4 3
=  3 πr  ρ
 

4  4 
G  πr 3   πr 3  ρ2
Gm1m2 3  3 
=F = or F ∝ r4
(2r)2 4r 2

Illustration 3: Three particles each of mass m, are located at the vertices of an equilateral a/2
Am
triangle of side a. At what speed will they move if they all revolve under the influence of
their gravitational force of attraction in a circular orbit circumscribing the triangle while
30 o

m
still preserving the equilateral triangle?  (JEE MAIN)
Sol: The net force of gravitation on any one particle is due to other two particles. This r
O
gravitational force provides the necessary centripetal force to the particles to move in the
Cm Bm
circular orbit around the equilateral triangle.
    GM2   GM2 
F=A FAB + FAC
= 2  cos30=
°  3 Figure 10.4
2 2
 a   a 

a mv 2 mv 2 3 GM2 GM
r= , Now = F ; Or = 3;
= ∴v
3 r a a2 a
P hysi cs | 10.3

3. GRAVITATIONAL FIELD
How would a particle interact with the surrounding or with other particles?
Every particle creates a field and when the other particle comes in to this particle’s field, there would be an
interaction between the particles.
The intensity of the field i.e. how intensely
 would it attract another particle in its field is called Gravitational field
intensity or Gravitational field strength E . It is defined as the force experienced by a unit mass placed at a distance
 F
r due to mass M, i.e. E =
M

PLANCESS CONCEPTS
Always remember, it is a vector quantity and should be added vectorially when calculating Gravitational
field intensity at a point by one or more masses.
Vaibhav Krishnan (JEE 2009, AIR 22)

4. GRAVITATIONAL FIELD INTENSITY


(a) Due to a point mass M: M
r
O P
GMm F GM GM 
F
= ; = E
= ; =E E
r 2 m r 2
r2
(b) Due to uniform ring of Mass M and radius a on its axis. Figure 10.5

Consider any particle of mass dm on the ring, say at point A. The distance of this particle from P is
 Gdm
AP= z= a2 + r 2 .The gravitational field at P is dm is along PA and its magnitude is dE =
z2
Gdm
The component along PO is dEcos = α cos α
z2 A
2 2
The net gravitational field at P due to the ring is Z = a +r
a

Gdm Gcos α GMcos α GMr
=
E ∫ z2= cos α
z2
= ∫ dm =
z2 2 2 2
3
O r P
(a + r )
The field is directed towards the center of the ring.
Figure 10.6
(c) Due to uniform disc of mass M and radius a on its axis.
Let us draw a circle of radius x with the center at O. We draw
another concentric circle of radius x+dx. The part of the disc a
enclosed between these two circles can be treated as a uniform
ring of radius x. The point P is on its axis at a distance r from

the center. The area of this ring is 2πxdx .The area of the whole o P
r
disc is πa2 . As the disc is uniform, the mass of this ring is x

M 2Mxdx
dm= 2πxdx = dx
2
πa a2
The gravitational field at P due to the ring is, by equation, Figure 10.7
 2Mxdx 
G r
 a2  2GMr xdx
= dE =
( ) )
32 2
r 2 + x2 a  r 2 + x2 32


1 0 . 4 | Gravitation

As x varies from 0 to a, the rings cover up the whole disc. The field due to each of these is in the same direction
PO. Thus, the net field due to the whole disc is along PO and its magnitude is
a a
2GMr xdx 2GMr xdx
= E ∫= 2 2 2 32 ∫
a 0 (r + x2 )3 2
2 2
...(i) …(i)
o a (r + x )

2 2 2
z then 2x dx=2z dz and
Let r + x =

xdx zdz 1 1 1
∫ (r2 + x2 )3 2 =∫ =∫ dz =− =−
z 3
z 2 z r − x2
2

a
2GMr  1  2GMr 1 1 
From
= (i) E  − =   − 
2
a  r + x  0
2 2 a  r
2
r + a2
2 

Equation may be expressed in terms of the angle θ subtended by a radius of the disc at P as,
2GM
E
= (1 − cos θ). ad
a2
(d) Due to uniform thin spherical shell of mass M A
B z
and radius a from the triangle OAP, d
 
z 2 = a2 + r 2 − 2ar cos θ or
0 P
a sin 
2z=
dz 2ar sin θdθ
zdz
or sin θdθ = .  …(ii)
ar
Figure 10.8
Also from the triangle OAP,
z 2 + r 2 − a2
a2 = z 2 + r 2 − 2zr cos α or cos α = .  … (iii)
2zr

GM  a2 − r 2  GM  a2 − r 2 
Putting from (ii) and (iii) in (i),
= dE 
4ar 2 
1− 
z 2 
dz or
= ∫ dE 
4ar 2 
z +
z 

Case I: P is outside the shell (r > a)


In this case, z varies from r – a to r + a. The field due to the whole shell is
r+a
GM  a2 − r 2  GM
E= z +  =
4ar 2  z r − a r
2

We see that the shell may be treated as a point particle of the same mass placed at its center to calculate the
gravitational field at an external point.
Case II: P is inside the shell
a+r
GM  a2 −r 2 
In this case, z varies from a - r to a + r. The field at P due to the whole shell is E = z +  0
=
4ar 2  z  a − r
Hence the field inside a uniform spherical shell is zero.
(e) Due to uniform solid sphere of mass M and radius a
(i) At an external point r (>a): Let us divide the sphere into thin spherical
shells each centered at O. Let the mass of one such shell be dm. To
o r
calculate the gravitational field at P, we can replace the shell by a single P
particle of mass dm placed at the shell that is at O. a
Gdm
The field at P due to this shell is then dE =
r2
Figure 10.9
P hysi cs | 10.5

Towards PO. The field due to the whole sphere may be obtained by summing the fields of all the shells
making the solid sphere.
Gdm G GM
∫=
dE ∫
2 ∫
Thus, =E = dm
=
2
r r r2
Thus, a uniform sphere may be treated as a single particle of equal mass placed at its center for calculating
the gravitational field at an external point.
(ii) At an internal point r (<a):
r
Suppose the point P is inside the solid sphere (See Fig 10.10). In this case r<a. a
The sphere may be divided into thin spherical shells all centered at O.
o A
Suppose the mass of such a shell is dm. If the radius of the shell is less than r, the P
Gdm
point is outside the shell. The field due to the shell is dE = along PO.
r2
If the radius of the shell considered is greater than r, the point P is internal and Figure 10.10
the field due to such a shell is zero. The total field due to the whole sphere is
obtained by summing the fields due to all the shells. As all these fields are along
the same direction, the net field is

GdM G
=E ∫=
dE ∫ =
r2 r2
∫ dm  ...(i) … (i)

Only the masses of the shells with radii less than r should be added to get
= z a2 + r 2 . These shells form
4 3 4
πr .The volume of the whole sphere is πa3 . As
a solid sphere of radius r. The volume of this sphere is
3 3
M  4 3  Mr3
the given sphere is uniform, the mass of the sphere of radius r is  πr  =3
4 33  a
πa
Mr 3 G Mr3 GM 3
Thus, ∫ dm = 3 and by =(i) E = r.
a r 2 a3 a3

The gravitational field due to a uniform sphere at an internal point is proportional to the distance of the
point from the center of the sphere.

PLANCESS CONCEPTS

One could assume the whole mass is concentrated at the center of mass (now assume it as point mass)
for calculating the gravitation field at an external point for spherical shell, sphere nevertheless of mass
distribution (uniformly/non-uniformly)
Mass distribution should be a function of radial distance only.
Remember the Gauss theorem in Electricity?
 
Equivalent Gauss theorem for gravitational field is ∫ E.dS =−4πG(m) , m=enclosed mass I guess now you
could deduce the note above. Can you?
Nivvedan (JEE 2009, AIR 113)

Illustration 4: Three concentric shells of homogenous mass distribution of masses M1 , M2 and M3 having radii a,
b and c respectively are situated as shown in Fig. 10.11. Find the force on a particle of mass m  (JEE MAIN)
(a) When the particle is located at Q.
(b) When the particle is located at P.
1 0 . 6 | Gravitation

Sol: For a particle of mass m, lying at a distance r from the center of the spherical M3
 GMm 
shell of mass M and radius r, the gravitational force of attraction is   . If
M2
2
 r 
M1
the particle is lying inside the spherical shell then the force of gravitation on it x
is zero. C
y
 GMm 
Attraction at an external point due to spherical shell of mass M is   while
 r2 
at an internal point is zero.
(a) Point is external to shell M1 , M2 and M3 , Figure 10.11

GM1m GM2m GM3m Gm


So, force at Q will be Fq
= + + = (M1 + M2 + M3 )
2 2
y y y2 y2
GM1m GM2m Gm
(b) Force at P will be Fp
= + +0
= (M1 + M2 )
2 2
x x x2

Illustration 5: A uniform ring of mass m and radius a is placed directly above a uniform sphere of mass M and of
equal radius. The center of the ring is at a distance 3a from the center of the sphere. Find the gravitational force
exerted by the sphere on the ring. (JEE ADVANCED)
Sol: The field due to ring at the center of the sphere can be found easily, as the center of m a
the sphere is lying at the axis of the ring. From Newton’s third law of motion the force on
the sphere due to the ring will be equal in magnitude to the force exerted by the sphere on
the ring. 3a
The gravitational field at any point on the ring due to the sphere is equal to the field due to a
a single particle of mass M placed at the center of the sphere. Thus, the force on the ring due
M
to the sphere is also equal to the force on it by a particle of mass M placed at this point. By
Newton’s third law, it is equal to the force on the particle by the ring.
Figure 10.12
Now the gravitational field due to the ring at a distance d = 3a on its axis is
G md 3 Gm
=E =
(a + d2 )3 2
2
8 a2
3 GMm
The force on a particle of mass M placed here is F=ME = . Thus we have used the formula for field due
to a ring. 8 a2

This is also the force due to the sphere on the ring.

5. EARTH’S GRAVITATIONAL FIELD


We have seen what gravitational field is and how an object would interact with other objects. Earth is no different
as it creates a gravitational field and interacts with us.
g = F m (g should be written as g bar and F as F bar. Take care of that)

6. VARIATION IN THE VALUE OF ACCELERATION DUE TO GRAVITY (g)

Variation in the value of g: The value of g varies from place to place on the surface of earth. It also varies as we
go above or below the surface of the earth. Thus, value of g depends on the following factors:-
P hysi cs | 10.7

(a) Shape of the earth: The earth is not a perfect sphere. It is somewhat flat at the two poles. The equatorial
radius is approximately 21 km more than the polar radius. And since
GM 1
g = Or g∝
2
R R2
The value of g is minimum at the equator and maximum at the poles.

(b) Height above the surface of the earth: The gravitational force on mass m due to Earth of mass M at height
h above the surface of earth is
m
GMm
F= h
(R + h )
2

F GM
So the acceleration due to gravity is g=
′ =
m (R + h)2
GM g GM R
This can also be written as, g′ = Or g′ = as =g
 h
2
 h
2
R2 M
R 2 1 +  1 + 
 R  R Figure 10.13
Thus, g′ < g i.e., the value of acceleration due to gravity g goes on decreasing as we go
above the surface of earth. Further,
−2
 h  2h 
g′ g 1 + 
= or g′ ≈ g 1 −  if h<<R
 R  R 

So on going above the surface of the earth, acceleration due to gravity decreases. Note that mass is always
constant.

(c) Depth below the surface of the earth: Let an object of mass m is situated at a
h
depth h below the earth’s surface. Its distance from the center of earth is (R - h). m
This mass is situated at the surface of the inner solid sphere and lies inside the outer
spherical shell. The gravitational force of attraction on a mass inside a spherical
shell is always zero. Therefore, the object experiences gravitational attraction only O
due to inner solid sphere.
R
 M 4 (R − h)3
The mass of this sphere
= is M′   π(R − h)3 or M′ = M
 4 3 πR 3  3 R3
  Figure 10.14
GM′ m GMm(R − h) F

= F = and g′ =
(R − h)2
R 3 m

 h
Substituting the values, we get =
g′ g 1 −  i.e., g′ < g
 R
(d) Axial rotation of the earth: Let us consider a particle P at rest on the surface of y
the earth, in latitude φ . Then the pseudo force acting on the particles is mrω2
in outward direction. The true acceleration g is acting towards the center O of r P
the earth. Thus, the effective accelerating g′ is the resultant of g and rω2 or
 g’
g′
= g2 + (rω2 )2 + 2g(rω2 )cos(180 − φ) O X
R

or ′
g= g2 + r 2 ω4 − 2grω2 cos φ  ... (i)
Figure 10.15
2 4 2π 2π
Here, the term r ω comes out to be too small as =
ω =
T 24 × 3600
1 0 . 8 | Gravitation

rad/s is small. Hence, this term can be ignored. Also,


= r R cos φ . Therefore, Eq. (i) can be written as
g′ = (g2 − 2gRω2 cos2 φ)1 2

 2Rω2 cos2 φ   Rω2 cos2 φ 


= g 1 − 2 g 1 −
1= 


 g  g
   

Thus, g′ = g − Rω2 cos2 φ Rω2 is almost 0.03 m / s2

PLANCESS CONCEPTS

They is always a decrease in the value of acceleration due to gravity from that of g at the surface
irrespective of the condition.
If earth were to rotate faster ‘g’ would decrease at all points except at the poles. Guessed it? φ is 90 at
poles. Also remember φ is 0 at equator.
Chinmay S Purandare (JEE 2012, AIR 698)

Illustration 6: Suppose the earth increases its speed of rotation. At what new time period will the weight of a body
on the equator become zero? Take g = 10 m/s2 and radius of earth R = 6400km.  (JEE MAIN)
Sol: When rotational speed of earth is increased, the centrifugal force acting on the particle at rest at equator also
increases. At the equator, the centrifugal force is opposite to the force of gravity. Thus the apparent value of g is
g 2π
g' = g − Rω2 . For mass of body to be zero at the equator, g’ = 0 i.e. ω = . The time period of rotation is T = .
R ω
The weight will become zero, when g′ = 0 or g − Rω2 =0 (on the equator g′ = g − Rω2 )

g 2π g R
or
= ω ; ∴
= or T=2π
R T R g

6400 × 103

Substituting the values, T= 10 h or T=1.4 h
3600

Thus, the new time period should be 1.4 h instead of 24 h for the weight of a body to be zero on the equator.

Illustration 7: A simple pendulum has a time period exactly 2 s when used in a laboratory at North Pole. What will
be the time period if the same pendulum is used in a laboratory at equator? Account for the earth’s rotation only.
GM
Take= g = 9.8m / s2 and radius of earth=6400 km. (JEE ADVANCED)
R2

Sol: The time period of simple pendulum is given by t = 2π where  is the length of pendulum. At the equator
g
value of acceleration due to gravity ‘g’ is different than at the pole. The apparent value of g is g' = g − Rω2 . Thus
the time periods will be different.
Consider the pendulum in its mean position at the North Pole. As the pole is on the axis of rotation, the bob is
GMm
in equilibrium. Hence in the mean position, the tension T is balanced by earth’s attraction. Thus,
= T = mg.
R2
 
The time period t is t =2π =2π  ... (i) … (i)
T /m g
P hysi cs | 10.9

2π radian
At equator, the lab and the pendulum rotate with the earth at angular velocity ω = in a circle of radius
equal to 6400 km. Using Newton’s second law, 24 hour

GMm
− T′ =ω2R or, =
T′ m(g − ω2R)
2
R
Where T′ is the tension in the string.
The time period will be

l l
t′ =
2π 2π
=  ...(ii) … (ii)
(T′ / m) g − ω2R
By (i) and (ii)
−1 2
t′ g  ω 2R   ω2R 
= = 1 −  or, =
t′ t 1 + 
t g − ω 2R  g   2g 

Putting the values, t ' = 2.004 seconds.

7. GRAVITATIONAL POTENTIAL ENERGY


Suppose I would like to move a particle form another particle’s field, work is either done against the gravitational
field or extracted from it. This negative work is called as Gravitational Potential energy.
f  
Gravitational force is a conservative in nature. Work done by gravitational field= Uf − Ui =− ∫ F.dr.
i
Let a particle of mass m 1 be kept fixed at a point A (See r
Fig 10.16) and another particle of mass m 2 is taken from
a point B to a point C. Initially, the distance between the r1 dr
particles is AB = r1 and finally it becomes AC = r2 . We have
A
to calculate the change in potential energy of the system m1 B D E C
of the two particles as the distance changes from r1 to r2 . r2
Consider a small displacement when the distance between
Figure 10.16
the particles changes from r to r + dr. In the Fig 10.16, this
corresponds to the second particle going from D to E.
Gm1m2 
The force on the second particle is F = along DA
r2
Gm1m2
The work done by the gravitational force in the displacement is dW = − dr.
r2
Gm m
The change in potential energy of the two-particle system during this displacement is dU =
−dW = 1 2 dr.
r2
The change in potential energy as the distance between the particles from r1 to r2 is
r2 r2 r
Gm1m2 1  1 2 1 1
U(r2 ) − U(r1 ) = ∫ dU = ∫ dr = Gm1m2 ∫ dr = Gm1m2  −  = Gm1m2  − 
r2  r r1  r1 r2 
2
r1 r1 r

This is the change in potential energy of the particles when moved from B to C.
Suppose the same particles which are of mass m 1 and m 2 are very far from each other and we need to calculate
the change in potential energy when the distance between them becomes r. Then using above formulae,

 1 1 Gm1m2
we get U(r) − U(∞ ) =Gm1m2  −  =−
∞ r  r
1 0 . 1 0 | Gravitation

We make a standard assumption that the potential energy of the two-particle system to be zero when the distance
between them is infinity. This means that we choose U(∞ ) =0.
Note: Just as one assumed current to be in opposite direction with the flow of electrons, the potential at infinity is
assumed to be zero.

8. GRAVITATIONAL POTENTIAL
The potential at a point may also be defined as the work done per unit mass by an external agent in bringing a
particle slowly from the reference point to the given point. Generally the reference point is chosen at infinity so
that the potential at infinity is zero.

PLANCESS CONCEPTS

By slowly I mean, the particle is moved in such a way that there is no increase in Kinetic energy.
Since the Kinetic energy of the particle is zero, from the work energy theorem, the total work done
is change in potential energy. So, what is the difference between the Potential and Potential energy?
Observe it is the work done per unit mass.
Nitin Chandrol (JEE 2012, AIR 134)

UB − UA
We define the “change in potential” VB − VA between the two points as VB − VA =
m
Calculation of some Gravitational potentials:
(a) Potential due to point mass M at a point P which is at a distance r
(b) (ii) Potential due to Uniform ring of radius “a” and mass M at a point P on its axis.
r
U(r) − U( ∞ )
(c) V(r) =
m M
A P
GMm GM
But U (r) – U (∞) = − so that V= − Figure 10.17
r r
GM
The gravitational potential due to a point mass M at a distance r is −
r
(d) Consider any small part of the ring of mass dm. The point P is at a distance=z a2 + r 2 from dm.

GdM Gdm
dV =
− =
− ; a z
r a2 + r 2

O r P
Gdm G GM

V = ∫ dV = ∫− = − ∫ dm = −
a2 + r 2 a 2+ r 2 a2 + r 2

Figure 10.18

PLANCESS CONCEPTS

Remember that potential is a scalar quantity and one can directly add the contributions due to each of
the point masses.
Potential due to Uniform Thin spherical shell and due to Uniform sphere can be derived similarly and
here is the table of all the results.
P hysi cs | 10.11

PLANCESS CONCEPTS

Potential Gravitational Field

Point Mass at a distance r


−GM −GM 
er
r r2

GMr

(a )
32
2
−GM + r2
Uniform Ring at a point on its axis
a2 + r 2
towards center of ring

−GM 0 (inside)
(inside)
a
Uniform Thin spherical shell
−GM GM
(outside) (outside)
r r2

GMr
−GMr 2 (inside)
(Inside) a3
a3
Uniform Solid Sphere
GM
(outside)
−GM
2a3
(3a 2
−r 2
) (outside) r2

Only the magnitudes of gravitational field are written. As the gravitational force is attractive in nature,
the direction could be easily found out.
Gravitational force, potential and potential energy all are taken with negative sign because the
gravitational force is always attractive in nature.
∂V ∂V ∂V
Ex = − , Ey = − and Ez = −
∂x ∂y ∂z

  r2  

( ) ()
Potential using the field for various cases V r2 − V r1 =
− ∫ E .dr.

r1

B Rajiv Reddy (JEE 2012, AIR 11)

Illustration 8: A particle of mass 1 kg is kept on the surface of a uniform sphere of mass 20 kg and radius 1.0 m.
Find the work to be done against the gravitational force between them to take the particle away from the sphere.
 (JEE MAIN)
Sol: The work done in moving a particle away from the sphere will be equal to the change in gravitational potential
energy of the particle in the gravitational field of the sphere.
GM (6.67 × 10−11 )(20)
Potential at the surface of sphere, V=
− =
− −1.334 × 10−9 J kg
J kg =
R 1
1 0 . 1 2 | Gravitation

i.e., 1.334 × 10−9 J work is obtained to bring a mass of 1 kg from infinity to the surface of sphere. Hence, the same
amount of work will have to be done to take the particle away from the surface of sphere. Thus,= W 1.334 × 10−9 J

Illustration 9: A particle is fired vertically upward with a speed of 9.8 km/s. Find the maximum height attained by
the particle. Radius of earth = 6400 km and g at the surface=9.8 m/s2. Consider only earth’s gravitation.
 (JEE MAIN)
Sol: Particle initially moves with kinetic energy only in upwards direction opposite to the gravitation pull of earth.
The loss in its kinetic energy is equal to the gain in the potential energy. At the highest point of its vertical motion,
kinetic energy is converted completely into potential energy.

GMm
At the surface of the earth, the potential energy of the earth-particle system is − with usual symbols. The
R
1 2
kinetic energy is mv 0 where v0 = 9.8 km/s. At the maximum height the kinetic energy is zero. If the maximum
2
GMm
height reached is H, the potential energy of the earth-particle system at this instant is − . Using conservation
R +H
GMm 1 2 GMm
of energy, − + mv 0 = −
R 2 R +H
v 2 −gR 2 R2 v2 R2
Writing GM = gR 2 and dividing by m, −gR + 0 = or = R − 0 or R + H = Putting the
2 R +H R +H 2g v 20
values of R, v 0 and g on the right side, R−
2g

( 6400km=
) ( 6400km )
2 2

R +H
= = 27300km or H =(27300 − 6400)km =20900km.
(9.8kms ) 1500km
2
−1

6400km −
2 × 9.8ms−2

Illustration 10: Two particles of equal masses go round a circle of radius R under the action of their mutual
gravitational attraction. Find the speed of each particle. (JEE MAIN)
Sol: As the particles go around the circle they always remain diametrically opposite to each other. To sustain their
respective circular motion the necessary centripetal acceleration is provided by the gravitation force of attraction
between them.
The particles will always remain diametrically opposite so that the force on each particle will be directed along the
Gm2
radius. Consider the motion of one of the particles. The force on the particle is F = . If Thus, by Newton’s law,
4R 2
Gm2 mv 2 Gm
= or v=
4R 2 R 4R

9. BINDING ENERGY m
It is the energy due to which a system is bound. Suppose the mass m is placed on the surface

of earth. The radius of the earth is R and its mass M. Then, the kinetic energy of the particle K=0 R
M
GMm
and potential energy of the particle is UU == − .
R
GMm GMm Figure 10.19
Therefore, the total mechanical energy of the particle is, E = K + U = 0 − or E = −
R R
It is due to this energy, the particle is attached to the earth. If this amount of energy is supplied
to the particle in any form (normally kinetic), the particle no longer remains bound to the earth. It goes out of the
gravitational field of earth.
P hysi cs | 10.13

Illustration 11: Assuming the earth to be a sphere of uniform mass destiny, calculate the energy needed to
completely disassemble it against the gravitational pull amongst its constituent particles. Given the product of
2
2.5 × 1031 kgm, g =.
mass and radius of the earth = 10m s  (JEE MAIN)

Sol: The work done to completely disassemble the earth will be equal to change in potential energy of the earth.
Initial potential energy is negative and final will be zero.
3M
If M and R are the mass and radius of the earth, then the density ρ of the earth is ρ =
4 πR 3

The earth may be supposed to be made up of a large number of thin concentric spherical shells. It can be
disassembled by removing such shells one by one. When a sphere of radius x is left, the energy needed to remove
Gm1m2
a shell of thickness lying between x and x + dx is dU =
x
4 3
Where m 1 = mass of the sphere of radius x= πx p,
3
4 x2dxρ
and m 2 = mass of the spherical shell of radius x and thickness dx =π
4
3

(
G  πx3ρ  4 πx2dxρ )
16 2 2 4
∴ dw = dU =   = Gπ ρ x dx
x 3
2
16Gπ2ρ2 R5 16 2  
R
16Gπ2ρ2 4 M 3 GM2
 R5 =

Total energy required U = dU =
3 ∫ x dx = =
3 5 15

 ( 4 3 ) πR 3  5 R
0  
3 3
= gMR = × 10 × 2.5 × 1031 =1.5 × 1032 J.
5 5

10. ESCAPE VELOCITY


The minimum velocity needed to take a particle infinitely away from the earth is called the escape velocity. On the
surface of earth its value 11.2 km/s.
GMm
As we discussed the binding energy of a particle on the surface of earth kept at rest is . If this much energy
R
in the form of kinetic energy is supplied to the particle, it leaves the gravitational field of the earth. So, if v e is the
escape velocity of the particle, then

1 GMm 2GM GM
mv 2e = or ve = or v e = 2gR as g =
2 R R R2

PLANCESS CONCEPTS

Escape velocity is independent of angle of projection.


Anand K (JEE 2011, AIR 47)

Illustration 12: Calculate the escape velocity from the surface of moon. The mass of the moon is 7.4 × 1022 kg and
= 1.74 × 106 m 
radius (JEE MAIN)

2GMm
Sol: Escape velocity of any object placed on moon is given by v e =
Rm
1 0 . 1 4 | Gravitation

2GMm
Escape velocity from the surface of moon is v e =
Rm

2 × 6.67 × 10−11 × 7.4 × 1022


Substituting the values, we have ve = = 2.4 × 103 m s or2.4 km/s
1.74 × 106

11. SATELLITES
Satellites are generally of two types:
Natural Satellites: Moon is a natural satellite of the earth.
Artificial Satellite: These are launched in to space by humans and they help us in weather forecasting,
telecommunications etc. The path of these satellites is elliptical with the center of earth at a focus.

Orbital Speed: The necessary centripetal force to the satellite is being


provided by the gravitational force exerted by the earth on the satellite.
Thus,
VO
GM 1
∴ vo = or v o ∝ M r
r r M
R Satellite
Hence, the orbital speed ( v o ) of the satellite decreases as the orbital
Earth
radius (r ) of the satellite increases. Further, the orbital speed of a satellite
GM ve
close to the earth’s surface (r ≈ R ) is,=
vo = gR
= ;
R 2 Figure 10.20
Substituting v e = 11.2km / s ; v o = 7.9km / s

2πr 2πr r3
Period of Revolution: The period of revolution (T) is given by T = or T = or T = 2π
vo GM GM
r
r3
Or T = 2π (as GM= gR 2 )
gR 2

GMm
Energy of Satellite: The potential energy of the system is U= −
r

1 1  GM 
The kinetic energy of the satellite=
is, K = mv 20 m 
2 2  r 
1 GMm K
or K =
2 r
GMm GMm r
The total energy is, E =K +U =− or E= − E
2r 2r E=K+U
U
This energy is constant and negative, i.e., the system is closed. The farther the
satellite from the earth the greater its total energy.
Figure 10.21
P hysi cs | 10.15

PLANCESS CONCEPTS

The velocity of a satellite is independent of its mass. It only depends upon the mass of the planet around
which it revolves.
What if the time period of rotation of satellite is exactly 24 hours just as the time period of rotation of
earth? Its position w.r.t earth is fixed right! Try calculating the distance from the earth’s surface. By the
way, these satellites are called Geo-stationary (stationary w.r.t earth) satellites.
Yashwanth Sandupatla (JEE 2012, AIR 821)

Illustration 13: Consider an earth’s satellite so positioned that it appears stationary to an observer on earth and
serves the purpose of a fixed relay station for international transmission of TV and other communications. What
would be the height at which the satellite should be positioned and what would be the direction of its motion?
Given that the radius of the earth is 6400 km and acceleration due to gravity on the surface of the earth is
9.8 m/s2.  (JEE ADVANCED)

Sol: For any artificial satellite to appear stationary with respect to a point on earth, it must rotate with the same
angular speed as that of the earth and in the direction of motion as of the earth. The angular velocity of the satellite
at height h above earth surface is given by ω = GM / r 3 where r=R+h.
For a satellite to remain above a given point on the earth’s surface, it must rotate with the same angular velocity
as the point on earth’s surface. Therefore the satellite must rotate in the equatorial plane from west to east with a
time period of 24 hours.
Now as for a satellite orbital velocity is v 0 = GM / r
 1 
2πr r r 2
 2 3 
T == 2πr 2πr
= (as g = GM R ) or r =gR  =4.23 × 107m =42300km
vo GM gR 2  4 π2 
 
So the height of the satellite above the surface of earth, h =r − R = 42300 − 6400 ≈ 36000km

[The speed of a geostationary satellite v o = R g r = rω = 3.1 km s ]

Illustration 14: Two satellites S1 and S2 revolve round a planet in coplanar circular orbits in the same sense. Their
periods of revolution are 1 h and 8 h respectively. The radius of the orbit of S1 is 10 4 km. When S2 is closest to S1
find (a) the speed of S2 relative to S1 and (b) the angular speed of S2 as observed by an astronaut in S1. 
 (JEE ADVANCED)
Sol: According to Kepler’s laws of planetary motion, T2 ∝ R 3 . The orbital velocity of V1
2πR
the satellite v 0 = = Rω where ω is the angular velocity of revolution of satellite. R2
T
Let the mass of the planet be M, that of S1 be m1 and of S2 be m2 .
Let the radius of the orbit of S1 be R1( = 10 4 km) and so S2 be R 2 . R1 S1 S2
Let v1 and v 2 be the linear speeds of S1 and S2 with respect to the planet. The given
Fig 10.22 shows the situation.
As the square of the time period is proportional to the cube of the radius,
Figure 10.22
3 2 2
 R2   T2   8h  R2
=  =  =  64 or = 4 or R 2= 4R1= 4 × 10 4 km
 R1  T
 1 1h
  R1
1 0 . 1 6 | Gravitation

Now the time period of S1 is 1 h.


2πR1 2πR1
So, = 1h or v1 = = 2π × 10 4 kmh−1
v1 1h
2πR 2
Similarly, v2 = = π × 10 4 kmh−1
8h
(a) At the closest separation, they are moving in the same direction. Hence the speed of S2 with respect to S1 is
| v 2 − v1 |= π × 10 4 kmh−1

(b) As seen from S1 , the satellite S2 is at a distance R 2 − R1 =3 × 10 4 km at the closest separation. Also, it is moving
at π × 10 4 kmh−1 in a direction perpendicular to the line joining them.
π × 10 4 kmh−1 π
Thus, the angular speed of S2 as observed=
by S1 is ω = radh−1
4
3 × 10 km 3

Illustration 15: A spaceship is launched into a circular orbit close to the earth’s surface. What additional
velocity is now to be added to the spaceship in the orbit to overcome the gravitational pull? Radius of earth
= 6400
= km, g 9.8 m s2 . (JEE MAIN)

Sol: The potential energy of the spaceship close to the earth is negative (- mgR). The orbital speed close to the
earth is v = gR , so the kinetic energy is mgR/2. The total energy is - mgR/2. We need to provide the additional
kinetic energy = mgR/2 such that the spaceship escapes the gravitational pull of the earth.
mv 2 mgR
The extra kinetic energy to be given is = , so that the extra velocity given is v ' = gR .
2 2

The velocity is v ' = 7.91× 103 m / s =


9.8 × 6400000 = 7.91 km / s

Illustration 16: An artificial satellite is moving in a circular orbit around the earth with a speed equal to one fourth
the magnitude of escape velocity from the earth.
(i) Determine the height of the satellite above the earth’s surface.
(ii) If the satellite is stopped suddenly in its orbit and allowed to fall freely towards the earth, find the speed with
which it hits the surface of the earth. (JEE MAIN)
Sol: For satellite the escape velocity is v e = 2Rg . According to given data the satellite is moving in the orbit with
one fourth the magnitude of this velocity. When satellite stops revolving, it falls freely under action of gravity from
the height h above the surface of the earth. The loss in the gravitational potential energy in falling height h is equal
to gain in the kinetic energy of the satellite.
(i) Let M and R be the mass and radius of the earth respectively. Let m be the mass of satellite. Here escape velocity
from earth v e = (2Rg)
ve
Velocity of satellite v=
g = (2Rg) / 4 ...(i) …(i)
4 
 GM   R 2g  R 2g
Further
= vc =
    ∴ v 2g = ...(ii) …(ii)
 r   
R +h R +h 
From equation (i) and (ii), we get H=7R=44800km
(ii) Now, the total energy at height h=total energy on earth’s surface (principle of conservation of energy). Let it
reach earth’s surface with velocity v.
m 1 m 1 2 GMm GMm
∴ 0 − GM =mv 2 − GM Or =mv − ( h = 7R )
R +h 2 R 2 R 7R
P hysi cs | 10.17

Solving we get v = 12Rg / 7 ∴ =


v (1.714 × 6400 × 103
× 9.8)
= 10.368km sec

12. PLANETS AND THEIR MOTION

12.1 Law of Orbits


All the planets move in elliptical orbits with the sun as one of its focii.

12.2 Law of Areas


The radius vector from the sun at the focus of elliptical orbit to the planet sweeps out equal areas in equal intervals
of time.
If the radius vector R sweeps an angle dθ in time dt, area ASB
A Rd
1
swept by radius vector in time dt = dA = × R × Rdθ B
2
1 2 dθ 1 2
∴ dA= R dt= ωR dt d
2 dt 2 R
S
area dA 1 2
Areal velocity= = = ωR F1
time dt 2
So ωR 2 is constant for area SAB and area SCD. It shows that the
C
angular momentum mR 2 ω is conserved for planetary motion.
When R decreases, ω increases so that ωR 2 is constant. D
Figure 10.23
12.3 Laws of Periods
The square of the time period of revolution of a planet is proportional to the cube of the mean distance of the
planet from the sun.
If a is the mean distance of sun from the planet, T2 is proportional to a3 or T2 = Ka3 where K is a constant.
If a1 and a2 are semi-major axis of the orbits of two planets around the sun with respective time periods T1 and T2 ,
T12 a13
then =
T22 a32

PLANCESS CONCEPTS

Observe the time period of rotation of satellite. Got it? (It follows Kepler’s third law too)
When the planet is farthest from Sun, it is said to be at the Apogee of Aphelion.
When the planet is at nearest to the Sun, it is said to be at Perigee or Perhilion.
GV Abhinav (JEE 2012, AIR 329)

Illustration 17: The minimum and maximum distance of a satellite from the center of the earth are 2R and 4R
respectively, where R is the radius of earth and M is the mass of the earth. Find:
(a) Its minimum and maximum speeds,
(b) Radius of curvature at the point of minimum distance. (JEE ADVANCED)
1 0 . 1 8 | Gravitation

Sol: The speed of the satellite is minimum when is at the maximum distance from the earth and vice versa. At the
point of minimum or maximum distance from earth the velocity vector is perpendicular to the radius vector from
the earth. Apply law of conservation of angular momentum and energy at the two points.
(a) Applying conservation of angular momentum

mv1(2R) = mv 2 (4R) v1 = 2v 2 …(i)


...(i) V2
R 3R
From conservation of energy A B
R
1 2 GMm 1 2 GMm V1
mv − = mv 2 − …(ii)
...(ii)
2 1 2R 2 4R
Figure 10.24
Solving Eqs. (i) and (ii), we get

GM 2GM
v2 = , v1 =
6R 3R

(b) If r is the radius of curvature at point A

mv12 GMm 4v12R 2 8R


= = ; r = (Putting value of v1 )
r (2R 2 ) GM 3

Illustration 18: The planet Neptune travels around the Sun with a period of 165 year. Show that the radius of its
orbit is approximately thirty times that of Earth’s orbit, both being considered as circular. (JEE ADVANCED)
Sol: According to the Kepler’s laws of planetary motion T2 ∝ R 3 where T is the time period of revolution and R is
the radius of the orbit of revolution of planet. Taking the ratio of time periods of revolution of Earth and Neptune,
we get the ratio of radius of their orbits.
T1 = TEarth =1 year;T2 = TNeptune =165 year=165 T1

Let R1 and R 2 be the radii of the circular orbits of Earth and Neptune respectively.

T12 R 13 R13 T22 R13 × 1652


= ∴ R 32 = or R 32 =
T22 R 32 T12 12

∴ R 32 = 1652 R13 or R 2 ≈ 30R1

13. MOTION ABOUT THE CENTRE OF MASS



As shown in the Fig 10.25, for the case of circular orbits, two objects are
moving about their common center of mass. If we consider the motion of the m
smaller body, R

GMm r M
= mω2r
(r + R )
2

The revised law of periods in
Two bodies moving in circular
 4 π2  3  R 2 orbits under the influences of each
=T2  r 1+ 
 GM   r other’s gravitational attraction
 
Figure 10.25
P hysi cs | 10.19

Illustration 19: A pair of stars rotate about their common center of mass. One of them has mass m and the other
2m. Their centers are a distance d apart, d being large compared to the size of either star.
(a) Derive an expression for the period or rotation of the stars about their common center of mass in terms of d,
m and G
(b) Compare the angular moments of the two stars about their common center of mass.
(c) Compare the kinetic energies of the two stars.  (JEE MAIN)

Sol: The gravitational pull between two stars provides the necessary centripetal acceleration to make them revolve

in a circular orbit. The time period of revolution of each star is T = . The angular momentum of the revolving
ω 2

body is given by L=I ω = m r2 ω. And the kinetic energy is given by E = .
2
The center of mass O is at a distance 2d/3 from the star of mass m and d/3 from the star of mass 2m. Both the stars
rotate with the same angular velocity ω .
(a) Since the gravitational force provides the centripetal force, then

 2d  2 Gm.2m 2π 
m = ω ⇒ ω = 3Gm d3 or T = = 2π d3 3Gm
 3  d2 ω
O
m 2m
(b) Ratio of angular momenta
2d/3 d/3

Lsmall m ( 2d 3) ω
2

= = 2
Llarge 2m d 3 2 ω
( )
(c) Ratio of kinetic energies Figure 10.26
1 2
Esmall 2 ismallω
= = 2
Elarge 1
ibigω2
2

PROBLEM-SOLVING TACTICS

 1. Most of the problems are easy, as gravitation and electrostatics are analogous to each other. Just be careful
that gravitational force is always attractive, whereas electrostatic force can be attractive as well as repulsive
and make changes as necessary.
 2. Assumptions are appreciated in real cases of satellites and planetary motion.
 3. Ideas and concepts of circular motion must be strong because they are generally applied here.
 4. While dealing practical cases on Earth, be careful about Earth’s rotation on its own axis.
 5. Most questions are solved with ease by using work-energy theorem and laws of motion
1 0 . 2 0 | Gravitation

FORMULAE SHEET

S. No. Description Formulae


1 Magnitude of gravitational force between
Gm1m2
two particles of mass m1 & m2 placed at F=
a distance r is r2
G=6.67 × 10−11N − m2 / kg2
Note: It acts along the line joining two particles.

2 Acceleration due to gravity (g)


GM 2
g= SI units:- m s
R2
M is the mass of the earth and its radius R.
3 m
GMm
Gravitational force =
(R + h )
h 2

GM
′ Fm
Acceleration due to gravity= g= =
(R + h )
2

 2h 
g′ g 1 −
If h << R = 
R M  R 

4
 h
R g′ g 1 −
At a certain, Depth H, acceleration due to gravity g′ is= 
 R
g is acceleration due to gravity at surface of earth.

5 Effect of g due to axial rotation of earth g′ = g − Rω2 cos2 φ


g′ is the acceleration due to gravity on the particle on the earth
surface in latitude φ .

6 Gravitational field strength 


 F
E = SI unit is N/kg.
m
Gravitational Field Gravitational Potential

7 Point Mass GM
GM −
r2 r

8 Uniform ring at point on its axis GM


GMr −
(a )
32
2
+ r2 a2 + r 2

(towards center of ring)

9 Uniform thin spherical shell Inside θ GM


Inside −
GM a
Outside
r2 GM
Outside −
r
P hysi cs | 10.21

10 Uniform solid sphere


GMr Inside −GMr 2 a3
Inside
3
a

Outside
GM Outside −
GM
2
2a
(3a2
− r2 )
2
r
Here, a is the radius and r is the
location of point mass.
11 Gravitational potential Note: It is a scalar; SI unit is J kg .

12   ∂y ∂V ˆ ∂V ˆ 
E=−  ˆi + j+ k
 ∂x ∂y ∂z 
 
Note: It is partial derivative dV = −E.dr .

13 Gravitational potential energy


Gm1m2
U= −
r
(
System of particle m1 m2 m3 m4 )
m m m m mm mm mm mm 
−G  4 3 + 4 2 + 4 1 + 3 2 + 3 1 + 2 1 
U=
 r43 r42 r41 r32 r31 r21 

4 ( 4 − 1)
They are = 6 Pairs
2
14 For an n particle system, no. of pairs would
n (n − 1)
be Pairs
2
15 Binding Energy
GMm It is due to this energy particle is bound to earth.
m E=
R
R
M

16 Escape Velocity
v e = 2gR

17 Motion of Satellites Orbital Speed


GM
V0 vo =
r
m satellite
2πr r3
Time period: T = = 2π
r vo GM
M GMm GMm
Energy of satellite: U = − ;K =
r 2r
R
U is The potential energy Total Energy “E”
GMm
Earth K is The kinetic energy E =K +U =− =−K
2r
1 0 . 2 2 | Gravitation

18 Kepler’s Laws
1st Law:- Law of elliptical orbits

2nd Law:- Law of conservation of angular momentum
V2

A B (
3rd Law:- Harmonic law T2 ∝ r 3 )
r2 S r1
V1 v1r1 = v 2r2

v1 a (1 + e ) =
= r2 a (1 − e )

GM  1 − e 
Vmin
= V=
1  
a  1+ e 

GM  1 + e 
Vmax
= V=
2  
a  1− e 

Solved Examples

JEE Main/Boards of mass M and length l , if they are located along a


straight line at a distance a from each other.
Example 1: Two concentric shells of mass M1 and M2
are as shown. Calculate the gravitational force on m Sol: The gravitational potential energy is given by
due to M1 at points P, Q and R. Gm1m2
U= where m1 and m2 are point masses.
r
Sol: For a particle of mass m, lying at a distance r Consider the gravitational potential energy of interaction
from the center of the spherical shell of mass M, the between the point mass m and an infinitesimal element
 GMm  of the rod of mass dm. The total potential energy will
gravitational force of attraction is   . If the be the summation of energy of interaction of all the
2
 r 
small elements.
particle is lying inside the spherical shell then the force
of gravitation on it is zero. Consider small element dx of the rod whose mass
M
R dm = dx
c l
M2
M l m
Q M1
a dx
b P a
x

M 
GM1m Gm  dx 
At P, F = 0 At Q, F =
⇒ dU =
−  l 
b2 x
G(M1 + M2 )m
At R, F = GmM a+l dx GmM a +l
c2 ⇒U=∫ dU =− l ∫a x
=−
l
Inx 
a

Example 2: Find the potential energy of gravitational GmM a+l


⇒U=− loge  
interaction of a point mass m and a thin uniform rod l  a 
P hysi cs | 10.23

Example 3: If the radius of the earth contracts to half of Sol: The gravitational force acting on the particle P due
its present value without change in its mass, what will
Gm2
be the new duration of the day? to each of other particles is given by F = where
(r )
2

Sol: The angular momentum of the earth is given by r is the separation between P and the other particle.
2 2 As the force is vector quantity the resultant force on
L = I ω = MR ω since earth is considered to be sphere
5 particle P has to be found by vector addition.
of uniform mass density. As there is no external force
The force at P due to A is
is acting on the earth, the angular momentum of the
earth must remain constant after the radius of earth = Gm2 Gm2
FA = along PA. The force at P due to C is
reduces to half of its original size. The time period of
( AP ) 2d
2 2


revolution is T = . P
ω 2
Present angular momentum of earth L1 = Iω = MR ω 2
5 FA
New angular momentum because of change in radius FB
FC
2 d
2 R 
=L2 M   ω′
5 2 A d B d C
If external torque is zero then angular momentum must
Gm2 Gm2
be conserved =FC = along PC. The force at P due to B is
( CP ) 2d
2 2
L1 = L2
Gm2
2 1 2 FB = along PB
MR 2 ω= × MR 2 ω ' i.e., ω ' = 4ω d2
5 4 5
The resultant of FA , FB and FC will be along PB. Clearly
1 1
T′ = T = × 24 =6h ∠APB =
∠BPC =
45°
4 4
Gm2
Example 4: Two particles of equal mass go round a Components of FA along
= PB FA cos
= 45°
2 2d2
circle of radius R under the action of their mutual
gravitational attraction. Find the speed of each particle.
Gm2
Component of Fc along
= PB Fc cos
= 45°
Sol: As the particles go around the circle they always 2 2d2
remain diametrically opposite to each other. To
Gm2
sustain their respective circular motion the necessary Component of FB along PB =
centripetal acceleration is provided by the gravitation d2
force of attraction between them. Hence, the resultant of the three forces is
The particles will always remain diametrically opposite Gm2  1 1  Gm2  1 
so that the force on each particle will be directed along 2  + 1
+=
2 
1+  along PB.
d 2 2 2 2  d  2
the radius. Consider the motion of one of the particles.
Gm2
The force on the particle is F = . If the speed is v, Example 6: What is the fractional decrease in the value
its acceleration is v 2 / R . 4R 2
of free-fall acceleration g for a particle when it is lifted
Thus, by Newton’s law, from the surface to an elevation h? (h<<R)

Gm2 mv 2 Gm
= Or, v = Sol: The gravitational acceleration g at height h is
4R 2 R 4R
GM GM
given by g = . As here R>>h then g ≈ . The
(R + h)2
R2
Example 5: Three particles A, B and C, each of mass m, fractional decrease in g at height h above the surface of
are placed in a line with AB=BC=d. Find the gravitational ∆g
force on a fourth particle P of same mass, placed at the earth is given by .
g
a distance d from the particle B on the perpendicular
bisector of the line AC.
1 0 . 2 4 | Gravitation

GM Example 8: Find the distance of a point from the earth’s


The acceleration due to gravity is g =
R 2 center where the resultant gravitational field due to the
∆g −2GM earth and the moon is zero. The mass of the earth is
∴ = (Differentiating) 6.0 × 1024 kg and that of the moon is 7.4 × 1022 kg. The
∆R R3 distance between the earth and the moon is 4.0 × 105
dg −2GM 1 dg h km.
⇒ = ⇒ = −2  
h R 2 R g R 
Sol: If a body is placed between moon and the earth
then it is under action of gravitational force due to
Example 7: A double star is a system of two stars earth and moon simultaneously. When the gravitational
moving around the center of inertia of the system field due to earth is equal in magnitude but opposite
due to gravitation. Find the distance between the in direction to gravitational field due to moon then the
components of the double star, if its total mass equals net field is zero.
M and period of revolution is T.
The point must be on the line joining the centers of
Sol: Each star is moving in circular orbit whose center the earth and the moon and in between them. If the
is at the combined center of inertia. Find the radius distance of the point from the earth is x, the distance
of orbit of one of the stars in terms of the separation from the moon is (4.0 × 105 km – x). The magnitude of
between them and find the orbital velocity of the star the gravitational field due to the earth is
in terms of d. GMe G × 6 × 1024 kg
= E1
=
Center of inertia x2 x2
O and the magnitude of the gravitational field due to the
m1 m2
moon is
GMm G × 7.4 × 1022 kg
r (d-r) =E2 =
( ) ( )
2 2
d 4.0 × 105 km − x 4.0 × 105 km − x

The situation is shown in the above figure These fields are in opposite directions. For the resultant
field to be zero E1 = E2 ,
Here =
m1r m2 (d − r)
6 × 1024 kg 7.4 × 1022 kg
∴ (m1 + m2 ) r =
m2d Or, =
x2
( 4.0 × 10 km − x )
2
5
m2d
r=
(m1 + m2 ) Or, =
x 6 × 1024
= 9
4.0 × 105 km − x 7.4 × 1022
Also =
M (m1 + m2 )
As gravitational force provides the necessary centripetal x 3.6 × 105 km
Or, =
force for rotation, we have
Example 9: A planet of mass m1 revolves around the
m1m2 m1v12 m1v1 (m1 + m2 )
2
G = = sun of mass m2. The distance between the sun and the
d2 r m2d planet is r. Taking into consideration the motion of the
12 sun, find the total energy of the system assuming the
12
 Gm2   G  orbits to be circular.
2
=∴ v1 = m2 
  
 (m1 + m2 ) d   Md 
Sol: The gravitational pull between sun and planet
provides the necessary centripetal acceleration to
2π 2πr 2πr 2πd3 2  m2d 
make them revolve in circular orbits with same angular
Now=
T =
= =  as r = 
ω v1 m2 G Md GM  M  velocities. The center of each circular orbit will be at the
combined center of mass but their radii will be different.
2
T d3 2  T  Both the planet and the sun revolve around their center
∴ = or d = 3   GM.
of mass with same angular velocity (say ω)
2π GM  2π 
m1 r1 r2 m2
COM
P hysi cs | 10.25

r= r1 + r2  … (i)
−13.34 × 10−11 J + 0
2 2 Gm1m2
m1r1ω= m2r2 ω=  … (ii) 13.34 × 10−11 J − m 1
( )
2
r2 =
− + ( 2 kg) 10−5 m / s
d 2
Solving Eqs. (i) and (ii), we get
1
( )
2

 m 
+
2
(1 kg) 2 × 10−5 m / s
r1 = r  m22 
r1 = r  m1m + m2  Solving this, d = 0.31m.
r1 = r  m1 +2m2 
m m +m 
r2 = r 1 m11 2 
r2 = r  m1m + m2  Example 11: The gravitational field in a region is given
r2 = r  m1 +1m2    
ω22 =G (m
Gm(m1 +1 ++mm
13
2 2 )
m
2)
( )( )
by E 10Nkg−1 i + j . Find the work done by an
=
ω =G (m r + m )
ω2 =
13
r 2 external agent to slowly shift a particle of mass of 2 kg
r3 from the point (0,0) to a point (5m, 4m).
And now, total energy of the system is E= P.E. + K.E. Y B(5m,4m)
Gm1m2 1 1
or E =− + m1r12 ω2 + m2r22 ω2
r 2 2
Substituting the values of r1 , r2 and ω2 , we get
Gm1m2 O (0,0) A X
E= − .
2r Sol: As the particle is moving slowly, the kinetic energy
of the particle remains zero during its motion. The work
Example 10: Two particles A and B of masses 1 kg and done by the external agent to move the particle is given
2 kg respectively are kept 1 m apart and are released f  
to move under mutual attraction. Find the speed of A by W = −∆U =
when that of B is 3.6 cm/hour. What is the separation
∫ F.dr
i
between the particles at this instant? As the particle is slowly shifted, its kinetic energy
remains zero. The total work done on the particle is
Sol: As the particles A and B are initially at rest, the thus zero. The work done by the external agent should
system has potential energy only, but as they move be negative of the work done by the gravitational field.
towards each other the loss in potential energy is equal f  
to gain in kinetic energy. As particle is moving under The work done by the field is dW =
their mutual interaction, the linear momentum system
∫ F.dr
− dU =
i
must be conserved. Consider the figure. Suppose the particle is taken from
O to A and then from A to B. The force on the particle is
The linear momentum of the pair A+B is initially zero.
     
As only mutual attraction is taken into account – which
is internal when A+B is taken as the system – the linear
F mE
= = ( 2kg) 10Nkg−1 = ( )( )
i + j ( 20N) i + j ( )
momentum will remain zero. The particles move in
The work done by the field during the displacement
opposite directions. If the speed of A is v when the
OA is
speed of B is 3.6 cm / hour = 10−5 m / s ,
5m 5m
(1kg)v = (2kg)(10 −5 ms−1 )
−5 −1
W1 = ∫ Fx dx = ∫ ( 20N)dx = 20N × 5m = 100J.
or, v= 2 × 10 ms 0 0
GmAmB Similarly, the work done in displacement AB is
The potential energy of the pair is − with usual
symbols. Initial potential energy R
4m 4m

6.67 × 10 −11 2 2
N − m kg × 2kg × 1kg
W2
= ∫ Fy dy
= ( 20N) dy (=
∫= 20N)( 4m) 80J
= − 0 0
1m
Thus, the total work done by the field, as the particle is
−13.34 × 10−11 J.
=
shifted from O to B, is 180 J.
If the separation at the given instant is d, using The work done by the external agent is -180 J.
conservation of energy,
Note that the work is independent of the path so that
we can choose any path convenient to us from O to B.
1 0 . 2 6 | Gravitation

Example 12: A uniform solid sphere of mass M and Using conservation of angular momentum
radius ‘a’ is surrounded symmetrically by a uniform thin mv prp = mv ara
and spherical shell of equal mass and radius 2a. Find
As velocities are perpendicular to the radius, vectors at
the gravitational field at a distance
apogee and perigee, v prp = v ara
3 5
((a)
a)
2
a from the center, ((b)
b ) a from the center.
2
Using conservation of energy,

GMm 1 −GMm 1
Sol: If the particle is inside the spherical shell then the − + mv p2 = + mv 2a
rp 2 ra 2
gravitation field due to the shell is zero. The gravitational
field at distance r from the center of the sphere is given By solving, the above equations,
GM
by E = .
r2 2GMra 2GMrpra
vp
= = ; L mv
= prp m
2a
(
rp rp + ra ) rp + ra ( )
a
P1 P2
JEE Advanced/Boards
Given figure shows the situation. The point p1 is at a Example 1: The distance between the centers of two
3 5 stars is 10 a. The masses of these stars are M and 16 M
distance a from the center and p2 is at a distance
2 2 and their radii, `a’ and `2a’ respectively. A body of mass
a from the center. As p1 is inside the cavity of the thin m is fired straight from the surface of the larger star
towards the smaller star. What should be its minimum
spherical shell, the field here due to the shell is zero. initial speed to reach the surface of the smaller star?
The field due to the solid sphere is Obtain the expression in terms of G, M and a.
GM 4GM
=E = Sol: At a certain distance from the centers of the stars,
2
3  9a2
 a the gravitational fields due to the stars are equal in
2  magnitude but opposite in direction. As the body of
mass m is projected from the surface of larger star
This is also the resultant field. The direction is towards
towards the surface of smaller star, the kinetic energy
the center. The point p2 is outside the sphere as well
lost by the body is equal to gain of its potential energy
as the shell. Both may be replaced by single particles
when it reaches at the point of zero field.
of the same mass at the center. The field due to each
of them is Let O be the point along O1O2 where gravitational
intensities due to both the stars balance each other.
GM 4GM
=E′ =
2
5  25a2 M1=16M Threshold
a
 
2 
M2=M

8GM 2a O m
The resultant field is=
E 2E
=′ towards the center.
2
25a O1 O2
X
Example 13: A planet of mass m revolves in an
10a
elliptical orbit around the sun so that its maximum and
minimum distance from the sun are equal to ra and rp Let O1O = x
respectively. Find the angular momentum of this planet
relative to the sun. GM1 GM2
∴ 2
=
2
x
(10a − x )
Sol: At the apogee and perigee the radius vector is
perpendicular to the velocity vector of the plane. Use Or 16 (10 − x ) =
2
x2 or x = 8a
the law of conservation of angular momentum and
energy at these two points.
P hysi cs | 10.27

Potential energy of the body on surface of larger star,

Gm (16M) GmM 65GMm minimum and maximum distance from earth’s center
U1 =
− − =

2a 8a 8a during subsequent motion of the particle.

Potential energy at Sol: As the particle is projected from the satellite while
the satellite is still in circular motion, the net velocity of
GMm G (16M) m 5GMm the particle is sum of velocity relative to satellite and
O=
− − =
− UO
=
2a 8a 2a the velocity of the satellite. As the particle is still bound
to the gravitational attraction of the earth, the orbit
1 2
As U1 + mv min UO
= of the particle will be ellipse. The point of projection
2
is perigee. Conserve the angular momentum at the
65GMm 1 2 5GMm apogee and perigee.
∴− + mv min = −
8a 2 2a The orbital speed of satellite is
1 2 65GMm 5GMm 45 GMm GM
or mv min = − = vo = … (i)
2 8a 2a 8 a r 
2 45 GM 9 × 5 GM Where M=mass of earth
v min = ×
4 a 4 a
Absolute velocity of particle would be:
3 5GM 5
v min= × v p =v + v o = v = 1.25 v o  … (ii)
2 a 4 o
Since, v p lies between orbital velocity and escape
Example 2: Two masses m1 and m2 , at an infinite velocity, path of the particle would be an ellipse with r
distance from each other are initially at rest, start being the minimum distance.
interacting gravitationally. Find their velocity of
approach when they are at a distance r apart.

Sol: As the masses move towards each other gain in


1
r r
kinetic energy is equal to loss in gravitational potential Particle
energy. This problem is best solved in center of mass
frame where the total kinetic energy of masses depends
on the square of velocity of approach.
Let v r be their velocity of approach. From conservation Let r’ be the maximum distance and v p′ its velocity at
of energy: 5
the moment. v p = v
Increase in kinetic energy=decrease in gravitational 4 o
potential energy Then, from the conservation of angular momentum
1 Gm m and conservation of mechanical energy, we get
Or µv r2 = 1 2  ... (i)
2 r mv pr = mv p′ r ′  … (iii)
m1m2
Here, µ reduced
= = mass 1 2 GMm 1 GMm
m1 + m2 mv p − = mv p′2 −  … (iv)
2 r 2 r′
Substituting in Eq. (i), we get 5r
Solving the above Eqs. (i), (ii), (iii) and (iv), we get r ′ =
2G (m1 + m2 ) and r. 3
vr =
r 5r
Hence, the maximum and minimum distance are
and r respectively. 3
Example 3: A satellite is revolving round the earth in
a circular orbit of radius e and velocity v o . A particle
is projected from the satellite in forward direction

with relative velocity


= v ( )
5 4 − 1 v o . Calculate its
1 0 . 2 8 | Gravitation

Example 4: An earth satellite is revolving in a circular 8a ± 4a 2a


or r = or r=2a and
orbit of radius ‘a’ with velocity v o . A gun is in the 6 3
satellite and is aimed towards the earth. A bullet is Hence, the maximum and minimum distance are 2a
v
fired from the gun with muzzle velocity o . Neglecting 2a
2 and respectively.
resistance offered by cosmic dust and recoil of gun, 3
calculate maximum and minimum distance of bullet Example 5: Binary stars of comparable masses m1 and
from the center of earth during its subsequent motion. m2 rotate under the influence of each other’s gravity
with a time period T. If they are stopped suddenly in
Sol: Conserve the angular momentum and energy of their motion, find their relative velocity when they
the particle between the points, the point of projection collide with each other. The radii of the stars are R1
and at perigee. At perigee velocity is perpendicular to and R 2 respectively. G is the universal constant of
radius. gravitation.
The orbital speed of the satellite is
Sol: They rotate about center of mass, such that the
GM necessary centripetal acceleration for the rotational
vo =  … (i)
a motion is provided by the gravitational force of
attraction. As the stars start approaching each other
From conservation of angular momentum at P and Q and collide, the loss in the gravitational energy of
we have system is equal to the gain in the kinetic energy of the
mav o = mvr system. Find the initial separation in terms of the time
period.
av o
Or v =  …(ii) Both the stars rotate about their center of mass (COM).
r
From conservation of mechanical energy at P and Q. m2
we have m1
Vnet R1 COM R2
V Q V0

r1 r2
P r
V0
a 2
For the position of COM

r1 r2 r1 + r2 r
1  2 v o  GMm 1
2
GMm
= = =
m2 m1 m1 + m2 m1 + m2
(r= r1 + r2 )
m vo + − = mv 2 −
2  4  a 2 r
 
Gm m Gm  2π 
2 Also, m1r1ω2 = 1 2 or ω2 = 2  ω = 
5 2 GM v GMm r 2
r1r 2
 T 
or v − = −
8 o a 2 r
m2r
But, r1 =
Substituting values of v and v o from Eqs. (i) and (ii), m1 + m2
we get
G (m1 + m2 )
5 GM GM a2  GM  GM ∴ ω2 =
− =  − r3
8 a a r 2  2a  r
13
 G (m1 + m2 ) 
3 a 1 Or r =   …(i)
or − = − or −3r 2 = 4a2 − 8ar 
8a 2r 2 r  ω2 

or 3r 2 − 8ar + 4a2 =
0 Applying conservation of mechanical energy we have

Gm1m2 Gm1m2 1
8a ± 64a2 − 48a2 − =− + µvr2  …(ii)
or r =
6
r (R1 + R2 ) 2
P hysi cs | 10.29

m1m2 ro  2 
Here, µ reduced
= = mass and rmax
= 1 + 1 − K ( 2 − K ) sin φ 
m1 + m2 2 −K  
vr = relative velocity between the two stars. ro  2 
and r= 1 − 1 − K ( 2 − K ) sin φ 
From Eq. (ii), we find that min
2 −K  

2Gm1m2  1 1 r02 v 02
vr2  −  Here, K =
µ  R1 + R 2 r  GM

2Gm1m2  1 1 Example 7: The density inside a solid sphere of radius


 − 
m1m2  R1 + R 2 r 
`a’ is given by ρ = ρo a r , where ρo is the destiny at the
m1 + m2
surface and r denotes the distance from the center. Find
 1 1 the gravitational field due to this sphere at a distance of
=2G (m1+m2)  − 
 R1 + R 2 r  `2a’ from its center.

Sol: The given mass distribution is having spherical


Substituting the value of r from Eq. (i), we get symmetry. Any spherically symmetrical body can be
replaced by a point particle of the same mass situated
 13
at the center of the spherical body. The gravitational
 1  4 π2  
vr =2G (m1 + m2 )  −  field due to the sphere at the point 2a from the center
R1 + R 2  G (m + m ) T2  
  1 2   GM
of the sphere is given by E =
( 2a)
2

Example 6: Find the maximum and minimum distance


of the planet A from the sun S, if at a certain moment The field is required at a point outside the sphere.
of time it was a distance ro and travelling with the Dividing the sphere in concentric shells, each shell can
velocity v o with the angle between the radius vector be replaced by a point particle at its center having mass
and velocity vector being equal to φ . equal to the mass of the shell. Thus, the whole sphere
can be replaced by a point particle at its center having
mass equal to the mass of the given sphere. If the mass
of the sphere is M, the gravitational field at the given
A 
V0 point is
r0 GM GM
S =E =  … (i)
( 2a) 4a
2 2

Sol: As the planet revolves around the sun, the The mass M may be calculated as follows: Consider a
mechanical energy of the system is conserved. Conserve concentric shell of radius r and thickness dr. Its volume
the angular momentum between the given point and is
apogee. dV=( 4π ) dr and its mass is
2

At minimum and maximum distance velocity vector


 ρ  ( 4 π dr ) = 4ρ ar dr.
  a
()
2
dM =
ρdV =
v makes an angle of 90° with radius vector. Hence,  r
o o

from conservation of angular momentum, The mass of the whole sphere is


mv oro sin φ =mr v …(i) a

Here, m is the mass of the planet. M


= ∫ 4ρ0ar dr = 2πρ0 a3
0
From energy conservation law it follows that.
Thus, by (i) the gravitational field is
mv 2o GMm mv 2 GMm
− = −  …(ii)
2 ro 2 r 2π Gρ0 a3 1
E= = × Gρ0 a.
Here, M is the mass of the sun. 4a 2 2

Solving Eqs. (i) and (ii) for r, we get we values of r, one is


rmax and another is rmin .So,
1 0 . 3 0 | Gravitation

Example 8: Two satellites of same mass are launched in When there are two satellites, the total energy would
the same orbit round the earth so as to rotate opposite be
to each other. They collide solidly and stick together as  GMm   GMm   GMm 
wreckage. Obtain the total energy of the system before −  + −  = − 
 2r   2r   r 
and just after the collision. Describe the subsequent
motion of the wreckage. Let after collision, v ′ be the velocity of wreckage by the
law of conservation of momentum mv − mv = (m + m) v ′
Sol: Both the satellites are moving in the same orbit
∴ v′ =
0
so their orbital velocity will be same. As the masses
of the satellites are equal, and they are moving in the  
opposite direction their total momentum before and v v
after the collision is zero. m M m
The two satellites round the earth are shown in figure E
Potential energy of the satellite in its orbit = −GMm r
Kinetic energy of satellite in its orbit is
The wreckage of mass (2m) has no kinetic energy, but it
K = GMm 2r has only potential energy,
Where m is mass of satellite, M is the mass of the earth GM ( 2m)
and r is the orbital radius. So, energy after collision = −
r
GMm GMm GMm Now the combined mass has zero velocity just after
Total energy = − =

2r r 2r collision and therefore, the wreckage stops rotating
and falls down under gravity.

JEE Main/Boards

Exercise 1 Q.6 Where will the true weight of the body be zero?

Q.7 If the force of gravity acts on all bodies in


Q.1 Why Newton’s law of gravitation is called a universal
proportion to their masses, why does not a heavy body
law?
fall correspondingly faster than a light body.

Q.2 On earth value of


= G 6.67 × 10−11Nm2kg2 . What is
Q.8 The gravitational potential energy of a body at
its value on moon, where g is nearly one-sixth than that
a distance from the center of earth is U. What is the
of earth?
weight of the body at that point?

Q.3 An artificial satellite is revolving around the earth at


Q.9 The distance of two planets from the sun are 1011
a height 200 km from the earth’s surface. If a packet is
and 1010 meters respectively. What is the ratio of time
released from the satellite, what will happen to it? Will
periods of these two planets?
it reach the earth?

Q.10 For a satellite, escape speed is 11kms−1 . If the


Q.4 A spring balance is suspended inside an artificial
satellite is launched at an angle of 60° with the vertical,
satellite revolving around the earth. If a boy of mass
what will be the escape speed?
2 kg is suspended from it, what would be its reading?

Q.11 Prove that the value of acceleration due to gravity


Q.5 The escape velocity from earth for a piece of
at a point above the surface of the earth is inversely
10 gram is 11.2 kms−1 . What would it be for a piece of
proportional to the square of the distance of that point
mass 100 gram?
from the center of the earth.
P hysi cs | 10.31

Q.12 Gravitational force between two bodies is 1


B
newton. If the distance between them is made twice,
what will be the force?
P S S’ A
Q.13 If a person goes to a height equal to radius of
earth from its surface, what would be his weight relative C
to that on the earth?

Q.24 Two satellites of a planet have period 32 days and


Q.14 If the change in the value of g at a height h above 256 days. If the radius of the orbit of former is R, find
the surface of the earth is the same as at a depth x the orbital radius of the latter.
below it, both x and h being much smaller than the
radius of the earth, find the relation between x and h.
Q.25 If the distance of the earth from the sun were half
the present value, how many days will make one year?
Q.15 The gravitational force acting on a rocket at a Given, 1 year= 365 days.
height h from the surface of earth is 1/3 of the force
acting on a body at sea level. What is the ratio of h and
R (radius of earth)? Q.26 Estimate the mass of the sun, assuming the orbit
of earth round the sun to be a circle. The distance
between the sun and the earth is 1.49 × 1011m , and
Q.16 Does the gravitational force of attraction of the G 6.66 × 10−11Nm2kg−2 .
=
earth becomes zero at some height above the surface
of earth? Explain.
Q.27 If the mass of the sun is 2 × 1030 kg , the distance
of the earth from sun is 1.5 × 1011 m and period of
Q.17 What do you understand by gravity and revolution of the earth around sun is one year (=365.3
acceleration due to gravity. Establish a relation between days), calculate the value of gravitational constant.
g and G.

Q.28 Calculate the mass and mean density of earth


Q.18 Explain how the knowledge of g helps us to find (i) from the following data:
mass of earth and (ii) mean density of earth?
= 6.37 × 106 m, acceleration due to
Radius of earth
gravity = 9.8ms−2 and
Q.19 What do you understand by ‘Escape velocity’?
Derive an expression for it in terms of parameters of Gravitational constant
= 6.6 × 10−11 Nm2kg−2
given planet.
Q.29 If the radius of the earth shrinks by 2.5%, mass
Q.20 What do you understand by Gravitational field, remaining constant, then how would the value of
Intensity of gravitational field. Prove that gravitational acceleration due to gravity change?
intensity at a point is equal to the acceleration due to
gravity at that point. Q.30 At what altitude the acceleration due to gravity
above the earth’s surface would be half of its value on
Q.21 Explain Kepler’s laws of planetary motion and the surface of the earth? Radius of earth is 6400 km.
deduce Newton’s law of gravitation from them.
Q.31 The radius of earth is approximately 6000 km.
Q.22 Explain Newton’s law of gravitation. Define What will be your weight at 600 km above the surface
gravitational constant, and give its dimensional formula. of earth? At 12000 km above? At 18000 km above?
Give the evidences in support of the Newton’s law of Your weight on earth is 80 kg wt.
gravitation.

Q.32 At what height from the surface of earth, the


Q.23 Let the speed of the planet at the perihelion P acceleration due to gravity is the same at a depth
in the given figure be vP and the sun-planet distance 160 km below the surface of earth. Radius of earth is
SP be rP . Relate rP′ vP to the corresponding quantities 6400 km.
at the aphelion (rA , v A ) . Will the planet take equal take
equal times to traverse BAC and CPB?
1 0 . 3 2 | Gravitation

Q.33 What is the minimum energy required to launch a Q.3 If the radius of the earth be increased by a factor
satellite of mass m from the surface of earth of mass M, of 5, by what factor its density be changed to keep the
radius R in a circular orbit at an attitude 2 R. value of g the same?
(A) 1/25 (B) 1/5 (C) 1 5 (D) 5
Q.34 A rocket is launched vertically from the surface
of the earth with an initial velocity 10kms−1 . How far
Q.4 The mass and diameter of a planet are twice those
above the surface of the earth would it go? Radius of
of earth. What will be the period of oscillation of a
= 9.8ms−2 .
the earth 6400km;g
=
pendulum on this planet if it is a second’s pendulum
on earth?
Q.35 A remote sensing satellite of the earth revolves in
a circular orbit at a height of 250km above the earth’s (A) 2 second (B) 2 2 second
surface. What is the (a) orbital speed, and (b) period of
= 6.38 × 106
revolution of satellite? Radius of the earth 1 1
(C) second (D) second
m, and acceleration due to gravity at the surface of 2 2 2
earth = 9.8ms−2
Q.5 A particle of mass M is at a distance a form surface
Q.36 A satellite revolves round a planet in an of a thin spherical shell of equal mass and having
orbit just above the surface of planet. Taking radius a.

G 6.67 × 10−11Nm2kg−2 and the means density of the


=
M

= 5.51× 103 kgm−3 , find the period of satellite.


planet M

a
Q.37 Find the speed of escape at the moon given that
its radius 1.7 × 106 m and the value of g at its surface is (A) Gravitational field and potential both are zero at
1.63ms−2 . center of the shell.
(B) Gravitational field is zero not only inside the shell
Q.38 If the earth has a mass nine times and radius twice
but at a point outside the shell also.
that of the planet Mars, calculate the maximum speed
required by a rocket to pull out of the gravitational (C) Inside the shell, gravitational field alone is zero.
force of Mars. Given escape speed on the surface of (D) Neither gravitational field nor gravitational potential
earth is 11.2kms−1 is zero inside the shell.

Q.6 A spherical uniform planet is rotating about its


Exercise 2 axis. The velocity of a point on its equator is v. Due to
the rotation of planet about its axis the acceleration
Single Correct Choice Type 1
due to gravity g at equator is of g at poles. The
2
Q. 1 At what altitude will the acceleration due to gravity escape velocity of a particle on the pole of planet in
be 25% of that at the earth’s surface (given radius of terms of V.
earth is R)? (A) ve = 2v (B) ve = v
(A) R/4 (B) R (C) 3R/8 (D) R/2
(C) v e = v 2 (D) v e = 3v
Q.2 Let ω be the angular velocity of the earth’s rotation
about its axis. Assume that the acceleration due to Q.7 Two planets A and B have the same material density.
gravity on the earth’s surface has the same value at If the radius of A is twice that of B, then the ratio of the
the equator and the poles. An object weighed at the v
escape velocity A is.
equator gives the same reading as a reading taken at vB
a depth d below earth’s surface at a pole (d<<R). The
value of d is (A) 2 (B) 2 (C) 1 2 (D) 1 2
ω2R 2 ω2R 2 2ω2R 2 Rg
(A) (B) (C) (D)
g 2g g g
P hysi cs | 10.33

Q.8. The escape velocity for a planet is ve. A tunnel is Q.12 A satellite of the earth is revolving in circular
dug along a diameter of the planet and a small body is orbit with a uniform velocity V. If the gravitational force
dropped into it at the surface. When the body reaches suddenly disappears, the satellite will
the center of the planet, its speed will be
(A) Continue to move with the same velocity in the
ve ve same orbit.
(A) v e (B) (C) (D) Zero
2 2 (B) Move tangentially to the original orbit with velocity V.

Q.9 A hollow spherical shell is compressed to half its (C) Fall down with increasing velocity.
radius. The gravitational potential at the center (D) Come to a stop somewhere in its original orbit.
(A) Increases
Q.13 A satellite revolves in the geostationary orbit but
(B) Decreases
in a direction east to west. The time interval between
(C) Remains same its successive passing about a point on the equator is:
(D) During the compression increases then returns at (A) 48 hrs (B) 24 hrs
the previous value.
(C) 12 hrs (D) Never

Q.10 A (nonrotating) star collapses onto itself from an


Q.14 Two point masses of mass 4m and m respectively
initial radius R i with its mass remaining unchanged.
separated by d distance are revolving under mutual
Which curve in the following figure best gives the
force of attraction. Ratio of their kinetic energies will be:
gravitational acceleration ag on the surface of the star
as function of the radius of the star during the collapse? (A) 1:4 (B) 1:5 (C) 1:1 (D) 1:2

ag Q.15 Select the correct choice(s)

b d (A) The gravitational field inside a spherical cavity, within


a a spherical planet must be non-zero and uniform.
c (B) When a body is projected horizontally at an
R appreciable large height above the earth, with a velocity
R1
less than for a circular orbit, it will fall to the earth along
(A) a (B) b (C) c (D) d a parabolic path
(C) A body of zero total mechanical energy placed in a
Q.11 A mass is at the center of a square, with four gravitational field if it is travelling away from source of
masses at the corners as shown field will escape the field.

Rank the choices according to the magnitude of the (D) Earth’s satellite must be in equatorial plane.
gravitational force on the center mass.
Q.16 A satellite of mass m, initially at rest on the earth,
5M 3M 2M 3M
is launched into a circular orbit at a height equal to the
radius of the earth. The minimum energy required is.
(A) M (B) M

M 5M M 2M 3 1
(A) mgR (B) mgR
4 2
5M 3M 2M 3M
1 3
(C) mgR (D) mgR
(C) 2M (D) M 4 4

M 5M M 2M
Q.17 The following figure shows the variation of energy
with the orbit radius of a circular planetary motion. Find
(A) FA = FB < FC = FD (B) FA > FB < FD < FC the correct statement about the curves A, B and C

(C) FA = FB > FC = FD (D) None


1 0 . 3 4 | Gravitation

Q.22 Satellite A and B are orbiting around the in orbits


of ratio R and 4R respectively. The ratio of their aerial
A
velocities is:

Energy C
r (A) 1:2     (B) 1:4    (C) 1:8    (D) 1:16

B Q.23 In older times, people used to think that the Earth


was flat. Imagine that the earth is indeed not a sphere
of radius R, but an infinite plate of thickness H. What
(A) A shows the kinetic energy, B the total energy and C
value of H is needed to allow the same gravitational
the potential energy of the system.
acceleration to be experienced as on the surface of
(B) C shows the total energy, V the kinetic energy and A the actual Earth? (Assume that the Earth’s density is
the potential energy of the system. uniform and equal in the two models.)
(C) C and A are kinetic and potential energies 2R 4R 8R R
(A)   (B)   (C)   (D)
respectively and B is the total energy of the system. 3 3 3 3
(D) A and B are kinetic and potential energies and C is
the total energy of the system. Q.24 A planet revolves about the sun in elliptical orbit.
 dA  16 2
The aerial velocity   of the planet is 4.0 × 10 m s .
Q.18 When a satellite moves around the earth in a  dt 
certain orbit, the quantity which remains constant is: The least distance between planet and the sun is
2 × 1012 m . Then the maximum speed of the planet in
(A) Angular velocity (B) Kinetic energy
km s is:
(C) Aerial velocity (D) Potential energy
(A) 10 (B) 20 (C) 40 (D) None of these
Q.19 A satellite of mass 5 M orbits the earth in a circular
orbit. At one point in its orbit, the satellite explodes into
two pieces, one of mass M and the other of mass 4M.
After the explosion the mass M ends up travelling in Previous Years’ Questions
the same circular orbit, but in opposite direction. After
explosion the mass 4M is.
Q.1 If the radius of x the earth were to shrink by one per
(A) In a circular orbit cent, its mass remaining the same, the acceleration due
to gravity on the earth’s surface would (1981)
(B) Unbound
(A) Decrease (B) Remain unchanged
(C) Elliptical orbit
(C) Increase (D) Be zero
(D) Data is insufficient to determine the nature of the
orbit.
Q.2 If g is the acceleration due to gravity on the earth’s
surface, the gain in the potential energy of an object of
Q.20 A satellite can be in a geostationary orbit around
mass m raised from the surface of the earth to a height
earth at a distance r from the center. If the angular
equal to the radius R of the earth, is (1983)
velocity of earth about its axis doubles, a satellite can
now be in a geostationary orbit around earth if its 1
(A) mgR (B) 2mgR
distance from the center is 2
r r r r 1
(A) (B) (C) (D) (C) mgR
(D) mgR
2 4
( 4) (2)
13 13
2 2

Q.21 A planet of mass m is in an elliptical orbit around Q.3 Imagine a light planet revolving around a very
massive star in a circular orbit of radius R with a period
the sun (m << Msun ) with an orbital period T. If A be the
of revolution T. If the gravitational force of attraction
area of orbit, then its angular momentum would be: between the planet and the star is proportional to R −5 2 ,
2mA mA then (1989)
(A)   (B) mAT   (C)   (D) 2mAT
T 2T (A) T2 is proportional to R 2
P hysi cs | 10.35

(B) T2 is proportional to R 7 2 (A) 1.1 kms−1 (B) 11 kms−1

(C) T2 is proportional to R3 2 (C) 110 kms−1 (D) 0.11 kms−1

(D) T2 is proportional to R3.75


Q. 10 Statement-I: For a mass M kept at the centre of
a cube of side ‘a’, the flux of gravitational field passing
Q.4 If the distance between the earth and the sun were
through its sides is 4π GM.
half its present value, the number of days in a year
would have been  (1996) Statement-II: If the direction of a field due to a point
source is radial and its dependence on the distance ‘r’
(A) 64.5 (B) 129 (C) 182.5 (D) 730)
for the source is given as 1/r2, its flux through a closed
surface depends only on the strength of the source
Q.5 A simple pendulum has a time period T1 when on enclosed by the surface and not on the size or shape of
the earth’s surface and T2 when taken to a height R the surface (2008)
above the earth’s surface, where R is the radius of the
earth. The value of T2/T1 is (2001) (A) Statement-I is false, statement-II is true.

(A) 1 (B) 2 (C) 4 (D) 2 (B) Statement-I is true, statement-II is true; statement-II
is correct explanation for statement-I.

Q.6 A geostationary satellite orbits around the earth in a (C) Statement-I is true, statement-II is true; statement-II
circular orbit of radius 36,000km. Then, the time period is not a correct explanation for statement-I.
of a spy satellite orbiting a few hundred km above the (D) Statement-I is true, statement-II is false.
earth’s surface (R e = 6400km) will approximately be
 (2002)
Q.11 The height at which the acceleration due to gravity
(A) 1 2h (B) 1 h (C) 2 h (D) 4 h
g
becomes (where g = the acceleration due to gravity
9
Q.7 A double star system consists of two stars A and
on the surface of the earth) in terms of R, the radius of
B which have time periods TA and TB . Radius R A and
the earth is (2009)
RB and mass MA and MB . Choose the correct option.
 (2006) R R
(A) 2R (B) (C) (D) 2R
2 2
(A) TA > TB then R A > RB

(B) if TA > TB then MA > MB Q.12 Two bodies of masses m and 4 m are placed at a
distance r. The gravitational potential at a point on the
T 
2
 RA 
3 line joining them where the gravitational field is zero is:
(C)  A  =    (2011)
 TB   RB 
4Gm 6Gm
(A) − (B) −
(D) TA = TB r r
9Gm
(C) − (D) Zero
Q.8 A satellite is moving with a constant speed v in a r
circular orbit about the earth. An object of mass m is
ejected from the satellite such that it just escapes from Q.13 The mass of a spaceship is 1000 kg. It is to be
the gravitational pull of the earth. At the time of its launched from the earth’s surface out into free space.
ejection, the kinetic energy of the object is (2011) The value of ‘g’ and ‘R’ (radius of earth) are 10 m/s2 and
6400km respectively. The required energy for this work
1 2 3
(A) mv (B) mv 2 (C) mv 2 (D) 2mv 2 will be: (2012)
2 2
(A) 6.4 x 1011 Joules (B) 6.4 x 108 Joules
Q.9 A planet in a distant solar system is 10 times (C) 6.4 x 109 Joules (D) 6.4 x 1010 Joules
more massive than the earth and its radius is 10 times
smaller. Given that the escape velocity from the earth
is 11 kms−1, the escape velocity from the surface of the
planet would be (2008)
1 0 . 3 6 | Gravitation

Q.14 What is the minimum energy required to launch a −GM −GM


(A) (B)
satellite of mass m from the surface of a planet of mass 2R R
M and radius R in a circular orbit at an altitude of 2R?
−2GM −2GM
 (2013) (C) (D)
3R R
5GmM 2GmM
(A) (B)
6R 3R Q.16 A satellite is revolving in a circular orbit at a height
‘h’ from the earth’s surface (radius or earth R; h<<R).
GmM GmM
(C) (D) The minimum increase in its orbital velocity required,
2R 3R
so that the satellite could escape from the earth’s
gravitational field, is close to: (Neglect the effect of
Q.15 From a solid sphere of mass M and radius R, a atmosphere.) (2016)
spherical portion of radius R/2 is removed, as shown in
the figure. Taking gravitational potential V = 0 at r = ∞, (A) gR (B) gR / 2
the potential at the centre of the cavity thus formed is:
(G = gravitational constant) (2015) (C) gR( 2 − 1) (D) 2gR

JEE Advanced/Boards

Exercise 1 Q.4 Calculate the distance from the surface of the earth
at which above and below the surface acceleration due
to gravity is the same.
Q.1 A small mass and a thin uniform rod each of mass
‘m’ are positioned along the same straight line as
Q.5 An object is projected vertically upward from the
shown. Find the force of gravitational attraction exerted
surface of the earth of mass M with a velocity such that
by the rod on the small mass.
the maximum height reached is eight times the radius
2L R of the earth. Calculate:
L
m (i) The initial speed of projection
m
(ii) The speed at half the maximum height.
Q.2 A particle is forced vertically form the surface of the
earth with a velocity kve where ve is the escape velocity Q.6 A sphere of radius R has it center at the origin.
and k<1. Neglecting air resistance and assuming It has a uniform mass density ρ0 except that there is
earth’s radius as R e , calculate the height to which it will
rise from the surface of the earth. a spherical hole of radius r = R 2 whose center is at
x = R 2 as in the given figure. (a) Find gravitational
field at points on the axis for x>R
Q.3 A point P lies on the axis of a fixed ring of mass M
and radius a, at a distance a from its center C. A small
particle starts from P and reaches C under gravitational
attraction only. Its speed at C will be_______. O
X
P hysi cs | 10.37

(b) Show that the gravitational field inside the hole is Q.12 A thin spherical shell of total mass M and radius R
uniform. Find its magnitude and direction. is held fixed. There is a small hole in the shell. A mass m
is released from rest at a distance R from the hole along
Q.7 A small body of mass is projected with a velocity a line that passes through the hole and also through
just sufficient to make it reach from the surface of a the center of the shell. This mass subsequently moves
planet (of radius 2R and mass 3M) to the surface of under the gravitational force of the shell. How long
another planet (of radius R and mass M). The distance does the mass take to travel from the hole to the point
between the centers of the two spherical planet is 6R. diametrically opposite?
The distance of the body from the center of bigger
planet is ‘x’ at any moment. During the journey, find the Q.13 A satellite close to the earth is in orbit above the
distance x where the speed of the body is (a) maximum equator with a period of rotation of 1.5 hours. If it is
(b) minimum. Assume motion of body along the line above a point P on the equator at some time, it will be
joining centers of planets. above P again after time______.

Q.8 A man can jump over b=4m wide trench on earth. Q.14 A satellite is moving in a circular orbit around the
If mean density of an imaginary planet is twice that earth. The total energy of the satellite is E =−2 × 105 J .
of the earth, calculate its maximum possible radius so The amount of energy to be imparted to the satellite to
that he may escape from it by jumping. Given radius of transfer it to a circular orbit where its potential energy
earth=6400km. is equal to________.

Q.9 A satellite P is revolving around the earth at a height Q.15 A satellite of mass m is orbiting the earth in a
h = radius of earth (R) above equator. Another satellite circular orbit radius r. It starts losing energy due to
Q is at a height 2h revolving in opposite direction. At an small air resistance at the rate of C J/s. Then the time
instant the two are at same vertical line passing through taken for the satellite to reach the earth is_____.
center of sphere. Find the least time after which again
they are in this situation. Q.16 A satellite is orbiting the Earth of mass M in
equatorial plane in a circular orbit having radius 2R and
same sense of rotation as that of the Earth. Find duration
of time for which a man standing on the equator will be
able to see the satellite continuously. Assume that the
Earth P Q
man can see the satellite when it is above horizon. Take
M
Earth’s angular velocity = ω

Q.17 A launching pad with a spaceship is moving along


a circular orbit of the moon, whose radius R is triple that
Q.10 Two small dense stars rotate about their common of moon Rm. The ship leaves the launching pad with
center of mass as a binary system with the period 1 year a relative velocity equal to the launching pad’s initial
for each. One star is of double the mass of the other orbital velocity v 0 and the launching pad then falls to
and the mass of the lighter one is 1/3 of the mass of the moon. Determine the angle θ with the horizontal
the sun. Find the distance between the stars if distance at which the launching pad crashes into the surface if
between the earth & the sun is R. its mass is twice that of the spaceship m.

Q.11 Four masses (each of m) are placed at the vertices Q.18 A body moving radially away from a planet of mass M,
of a regular pyramid (triangular base) of side ‘a’. Find when at distance r from planet, explodes in such a way that
the work done by the system while taking them apart two of its many fragments move in mutually perpendicular
so that they form the pyramid of side ‘2a’ circular orbits around the planet. What will be
m (a) Their velocity in circular orbits
(b) Maximum distance between the two fragments
before collision and
m m
(c) Magnitude of their relative velocity just before they
a collide.
m
1 0 . 3 8 | Gravitation

Q.19 A cord of length 64 m is used to connect a 100 kg Q.25 The fastest possible rate of rotation of a planet such
astronaut to spaceship whose mass is much larger than that for which the gravitational force on material at the
that of the astronaut. Estimate the value of the tension equator barely provides the centripetal force needed
in the cord. Assume that the spaceship is orbiting near for the rotation. Show that the corresponding shortest
earth’s surface. Assume that the spaceship and the

astronaut fall on a straight line from the earth’s center. period of rotation is given by T = , where ρ is the
The radius of the earth is 6400km. Gρ
density of the planet, assumed to be homogeneous.
Q.20 Imagine a planet of mass M with a small moon of m
mass m and radius a orbiting it and keeping the same
face toward it. If the moon now approaches the planet,
there will be a critical distance from the planet’s center
at which loose material lying on the moon’s surface R
will be lifted off. Show that this distance is given by m m

re = a ( 3M m)
13
. This critical distance is called Roche’s
limit.
Exercise 2
Q.21 A hypothetical planet of mass M has three moons
each of equal mass ‘m’ revolving in the same circular Multiple Correct Choice Type
orbit of radius R. The masses are equally spaced and
thus form an equilateral triangle. Find:
Q.1 Assuming the earth to be a sphere of uniform
(i) The total P.E. of the system density the acceleration due to gravity
(ii) The orbital speed of each moon such that they (A) At a point outside the earth is inversely proportional
maintain this configuration. to the square of its distance from the center.
(B) At a point outside the earth in inversely proportional
Q.22 A remote sensing satellite is revolving in an orbit to its distance from the center.
of radius x over the equator of earth. Find the area on
earth’s surface in which satellite cannot send message. (C) At a point inside is zero.
(D) At a point inside is proportional to its distance from
Q.23 A pair of stars rotate about a common center of the center.
mass. One of the stars has a mass M which is twice as
large as the mass m of the other. Their centers are a Q.2 Two masses m1 and m2 (m1 < m2 ) are released
distance d apart, d being large compared to the size of from rest from a finite distance. They start under their
either star. mutual gravitational attraction.
(a) Derive an expression for the period of rotation of (A) Acceleration of m1 is more than that of m2.
the star about their common center of mass in terms
of d, m, G. (B) Acceleration of m1 is more than that of m1.

(b) Compare the angular momentum of the two stars (C) Center of mass of system will remain at rest in all
about their common center of mass by calculating the reference frames.
ratio Lm LM . (D) Total energy of system remains constant.
(c) Compare the kinetic energies of the two stars by
calculating the ratio Km KM . Q.3 Inside a hollow isolated spherical shell
(A) Everywhere gravitational potential is zero.
Q.24 Assume that a geosynchronous communications
(B) Everywhere gravitational field is zero.
satellite is in orbit at the longitude of Mumbai. You
are in Mumbai and want to pick up its signals. In what (C) Everywhere gravitational potential is same.
direction should you point the axis of your parabolic
(D) Everywhere gravitational field is same.
antenna? The latitude of Mumbai 30° N.
P hysi cs | 10.39

Q.4 When a satellite in a circular orbit around the earth Q.9 For a satellite to orbit around the earth, which of
centers the atmospheric region, it encounters small air the following must be true?
resistance to its motion. Then
(A) It must be above the equator at some time.
(A) Its kinetic energy increases.
(B) Its cannot pass over the poles at any time
(B) Its kinetic energy decreases.
(C) Its height above the surface cannot exceed 36,000 km
(C) Its angular momentum about the earth decreases.
(D) Its period of rotation must be > 2π R g where R is
(D) Its period of revolution around the earth increases. radius of earth

Q.5 A communications Earth satellite Q.10 Two satellites s1 & s2 of equal masses revolve
in the same sense around a heavy planet in coplanar
(A) Goes round the earth from east to west.
circular orbit of radii R & 4R.
(B) Can be in the equatorial plane only.
(A) The ratio of period of revolution s1 & s2 is 1:8
(C) Can be vertically above any place on the earth.
(B) Their velocities are in the ratio 2:1
(D) Goes round the earth from west to east.
(C) Their angular momentum about the planet are in
the ratio 2:1
Q.6 An earth satellite is moved from one stable circular
orbit to another larger and stable circular orbit. The (D) The ratio of angular velocities of s1 w.r.t. s2 when all
following quantities increase for the satellite as a result three are in same line is 9:5
of this change:-
Assertion Reasoning Type
(A) Gravitational potential energy
(A) Statement-I is true, statement-II is true and
(B) Angular velocity
statement-II is correct explanation for statement-I.
(C) Linear orbital velocity
(B) Statement-I is true, statement-II is true and
(D) Centripetal acceleration statement-II is NOT the correct the explanation for
statement-I.
Q.7 A satellite S is moving in an elliptical orbit around (C) Statement-I is true, statement-II is false.
the earth. The mass of the satellite is very small
compared to the mass of the earth. (D) Statement-I false, statement-II is true.

(A) The acceleration of S is always directed towards the


Q.11 Statement-I: Moon revolving around earth does
center of the earth.
not come despite earth’s gravitational attraction.
(B) The angular momentum of S about the center of the
Statement-II: A radially outward force balances earth’s
earth changes in direction, but its magnitude remains
force of attraction during revolution of moon.
constant.
(C) The total mechanical energy of S varies periodically
Q.12 Statement-I: Time period of simple pendulum in
with time.
an orbiting geostationary satellite in infinite.
(D) The linear momentum of S remains constant in
Statement-II: Earth’s gravitational field becomes
magnitude.
negligible at large distance from it.

Q.8 If a satellite orbits as close to the earth’s surface as


Q.13 Statement-I: Geostationary satellite may be
possible,
setup in equatorial plane in orbits of any radius more
(A) Its speed is maximum than earth’s radius.
(B) Time period of its rotation is minimum Statement-II: Geostationary satellite have period of
revolution of 24 hrs.
(C) The total energy of the ‘earth plus satellite’ system
is minimum
Q.14 Statement-I: For the calculation of gravitational
(D) The total energy of the ‘earth plus satellite’ system
force between any two uniform spherical shells, they
is maximum
1 0 . 4 0 | Gravitation

can always be replaced by particles of same mass Q.19 If U is the potential energy and K kinetic energy
placed at respective centers. then U > K at
Statement-II: Gravitational field of a uniform spherical (A) Only D (B) Only C
shell out side it is the same as that of particle of same
(C) Both D & C (D) Neither D nor C
mass placed at its center of mass.

Paragraph 3:
Q.15 Statement-I: It takes more fuel for a spacecraft to
travel from the earth to moon than for the return trip. During the formation of stars from clouds of hydrogen
gas in space, due to gravitational force of attraction,
Statement-II: Potential energy of spacecraft at moon’s
volume of gas decreases, which in turn heats the gas.
surface is greater than that at earth surface.
Specific heat capacity of gas is S, universal gravitational
constant is G and mass in a hydrogen cloud is M.
Comprehension Type
Q.20 If radius of gas cloud decreases from R to R 2 ,
Paragraph 1:
the increment in temperature of gas is (assume No loss
Two uniform spherical stars made of same material of energy outside due to radiations, and clouds are
have radii R and 2R. Mass of the smaller planet is m. spherical in shape)
They start moving from rest towards each other from
a large distance under mutual force of gravity. The GM 3GM 3GS 3GM
(A) (B) (C) (D) −
collision between the stars is inelastic with coefficient RS 5RS 5MR RS
of restitution ½.
Q.21 Assume the initial temperature of gas is 0 K and
Q.16 Kinetic energy of the system just after the collision thermonuclear reactions will start at T0 K temperature,
is: the minimum mass of gas required so that thermonuclear
reactions start when radius of cloud becomes half of
8Gm2 2Gm2
(A) (B) initial radius (R). Assume uniform temperature in entire
3R 3R volume of gas.
4Gm2
(C) (D) Cannot be determined
3R 5 SRT0 3 SRT0
(A) (B)
3 G 5 G
Q.17 The maximum separation between their centers SRT0
after their first collision (C) (D) None
G
(A) 4 R (B) 6 R (C) 8 R (D) 12 R
Paragraph 4:
Paragraph 2: In some parts of universe, it is found that acceleration
The given figure shows the orbit of a planet P round produced in a body is inversely proportional to the
the sun S, AB and CD are the minor and major axes of square of its mass and directly proportional to the net
the ellipse. F
force (F) according to equation a = c where c is
A m2
P
constant, whose magnitude is 1, if m is measured in kg,
a is measured in m/s2 and F is in Bose. Also action and
D C reaction force are equal and opposite and on different
S
interacting bodies.

B
Q.22 In the given figure shown, two blocks of mass m1
Q.18 If t1 is the time taken by the planet to travel along = 2 kg and m2 = 4 kg are attached via an ideal massless
ACB and t2 the time along BDA, then string over frictionless mass less pulley. If acceleration
due to gravity g = 5 m/s2.The tension in the string is
(A) t1 = t2 (B) t1 > t2
(C) t1 < t2 (D) Nothing can be concluded
P hysi cs | 10.41

Then
y

m x
A O B
m1 m2

(A) 6 Bose (B) 1.67 Bose


z
(C) 3 Bose (D) 32 Bose
(A) the gravitational field due to this object at the origin
is zero.
Q.23 In the given figure a block of mass m=2 kg is
placed on smooth inclined plane. The minimum value (B) the gravitational field at the point B (2, 0, 0) is zero
(
of force F needed to support the block is g = 5m s 2
) (C) the gravitational potential is the same at all points
of circle y 2 + z 2 =
36

F (D) the gravitational potential is the same at all points


on the circle y 2 + z 2 =
4

Q.3 The magnitude of the gravitational field at distance


r1 and r2 from the center of a uniform sphere of radius
R and mass M are F1 and F2 respectively. Then  (1994)
=30
o

F1 r1
(A) = if r1 < R and r2 < R
(A) Zero, Newton (B) 10 Bose F2 r2
(C) 20 Bose (D) 10 Newton
F1 r22
(B) => if r1 R and r2 > R
F2 r 2
1

Previous Years’ Questions F1 r13


(C) = if r1 < R and r2 < R
F2 r3
2
Q.1. Statement-I: An astronaut in an orbiting space
station above the earth experiences weightlessness. F1 r12
(D) = if r1 < R and r2 < R
F2 r 2
Statement-II: An object moving around the earth 2
under the influence of earth’s gravitational force is in a
state of ‘free-fall’ (2008) Q.4 Two satellites S1 and S2 revolve round a planet in
(A) If statement-I is true, statement-II is true; statement-II coplanar circular orbits in the same sense. Their periods
is the correct explanation for statement-I of revolution are 1 h and 8 h respectively. The radius
of the orbit of S1 is 10 4 km. when S2 is closed to S1 .
(B) If statement-I is true; statement-II is true; statement-II Find
is not a correct is true; statement-I
(a) the speed of S2 relative to S1 ,
(C) If statement-I is true; statement-II is false
(b) the angular speed of S2 as actually observed by an
(D) If statement-I is false; statement-II is true astronaut in S1  (1986)

Q.2 A solid sphere of uniform density and radius 4 units Q.5 Three particles, each of mass m, are situated at
is located with its center at the origin O of coordinates. the vertices of an equilateral triangle of side length a.
Two spheres of equal radii 1 unit, with their centers at A The only forces acting on the particles are their mutual
(-2, 0, 0) and B (2, 0, 0) respectively, are taken out of the gravitational forces. It is desired that each particle
solid leaving behind spherical cavities as shown in the moves in a circle while maintaining the original mutual
given figure.  (1993)
1 0 . 4 2 | Gravitation

separation a. Find the initial velocity that should be v v


given to each particle and also the time period of
circular motion  (1988) (A) (B)

Q.6 An artificial satellite is moving in circular orbit R r R r


around the earth with a speed equal to half the
magnitude of escape velocity from the earth.  (1990) v v
(a) Determine the height of the satellite above the
earth’s surface. (C) (D)

(b) If the satellite is stopped suddenly in its orbit and


allowed to fall freely onto the earth, find the speed with R r R r
which it hits surface of the earth. Q.11 Two spherical planets P and Q have the same
uniform density ρ , masses MP and MQ, an surface areas
Q.7 Distance between the centers of two stars is 10a . A and 4A, respectively. A spherical planet R also has
The masses of these stars are M and 16 M and their uniform density ρ and its mass is (MP +MQ). The escape
radii a and 2a respectively. A body of mass m is fired velocities from the planets P, Q and R, are VP, VQ and V
straight from the surface of the larger star towards the respectively. Then  (2012)
surface of the smaller star. What should be its minimum
(A) VQ > VR > VP (B) VR > VQ > VP
initial speed to reach the surface of the smaller star?
Obtain the expression in terms of G, M and a.  (1996) VR VP 1
(C) = 3 (D) =
VP VQ 2
R
Q.8 There is a crater of depth on the surface of the 1
100 Q.12 A planet of radius R = × (radius of Earth) has
10
moon (radius R). A projectile is fired vertically upward
the same mass density as Earth. Scientists dig a well of
from the crater with velocity, which is equal to the
escape velocity v from the surface of the moon. Find R
depth on it and lower a wire of the same length and
the maximum height attained by the projectile. (2003) 5
of linear mass density 10-3 kgm-1 into it. If the wire is not
touching anywhere, the force applied at the top of the
Q.9 Gravitational acceleration on the surface of a planet
wire by a person holding it in place is (take the radius of
6 Earth = 6 × 106 m and the acceleration due to gravity of
is g, where g is the gravitational acceleration on the
11 Earth is 10 ms-2) (2014)
surface of the earth. The average mass density of the (A) 96 N (B) 108 N (C) 120 N (D) 150 N
2
planet is times that of the earth. If the escape speed
3 Q.13 In an experiment to determine the acceleration
on the surface of the earth is taken to be11 km/s the due to gravity g, the formula used for the time period
escape speed on the surface of the planet in km/s will
7(R − r)
be? (2010) of a periodic motion is T = 2π = . The values
5g

Q.10 A spherically symmetric gravitational system of of R and r are measured to be (60 ± 1) mm and (10 ±
1) mm, respectively. In five successive measurements,
 for r ≤ R
ρ the time period is found to be 0.52 s, 0.56 s, 0.57 s,
particles has a mass density ρ = 0
0 for r > R 0.54 s and 0.59 s. The least count of the watch used for
where ρ0 is a constant. A test mass can undergo circular the measurement of time period is 0.01 s. Which of the
motion under the influence of the gravitational field of following statement(s) is (are) true? (2016)
particles. Its speed v as a function of distance r from the (A) The error in the measurement of r is 10%
center of the system is represented by (2008)
(B) The error in the measurement of T is 3.57%
(C) The error in the measurement of T is 2%
(D) The error in the determined value of g is 11%
P hysi cs | 10.43

PlancEssential Questions
JEE Main/Boards JEE Advanced/Boards
Exercise 1 Exercise 1
Q.23 Q.31 Q.33 Q.1 Q.5 Q.6
Q.35 Q.11 Q.16 Q.21

Exercise 2 Exercise 2
Q. 2 Q.13 Q.17 Q.1 Q.2 Q.5
Q.20 Q.6 Q.7 Q.8
Q.10 Q.11 Q.12
Q.16

Answer Key

JEE Main /Boards Q.26 1.972 × 1030 kg

Exercise 1 Q.27 6.69 × 10−11 m2kg−2

Q.3 No Q.28 6.025 × 1024 kg;5.56 × 103 kg m3

Q.4 Zero Q.29 Increase by 5%

Q.9 10 10 Q.30 2649.6 km

Q.10 11 km/s Q.31 66.12 kg wt; 8.89 kg wt; 5 kg wt.

Q.13 One-Fourth Q.32 80 km


5GmM
Q.14 x=2h Q.33
6R
Q.15 0.732 Q.34 2.56 × 10 4 km

Q.23 v A vP = rP rA ; No, time for path BAC is greater Q.35 (a) 7756.6m s ; (b) 5373 s
than time for path CPB
Q.36 5064 s
Q.24 4 R
Q.37 2.354 × 103 ms−1
Q.25 129 days
Q.38 5.28kms−1
1 0 . 4 4 | Gravitation

Exercise 2

Single Correct Choice Type

Q.1 B Q.2 A Q.3 B Q.4 B Q.5 C Q.6 A


Q.7 A Q.8 B Q.9 B Q.10 B Q.11 A Q.12 B
Q.13 C Q.14 A Q.15 C Q.16 D Q.17 D Q.18 C
Q.19 B Q.20 C Q.21 A Q.22 A Q.23 A Q.24 C

Previous Years’ Questions


Q.1 C Q.2 A Q.3 B Q.4 B Q.5 D Q.6 C
Q.7 D Q.8 B Q.9 C Q.10 B Q.11 A Q.12 C
Q.13 D Q.14 A Q.15 B Q.16 C

JEE Advanced/Boards
Q.10 R
Exercise 1

GM2 6GM2
Q.1 Q.11
2a
3L2

R ek 2 Q.12 2 × R 3 GM
Q.2
1 − k2

2GM  1  Q.13 1.6 hours if it is rotating from west to east, 24 17


Q.3 1 −  hours if it is rotating from east to west
a  2

Q.14 1× 105 J
3 −1
Q.4 h = R
2
GMm  1 1 
Q.15 t
=  − 
4 GM 2 2GM 2C  R e r 
Q.5 (i) (ii)
3 R 3 5R 2π
Q.16
 Gm 
3 – ω 
 8R3 e 
   
πGρ0R3  1 8 ˆ GM
=Q.6 (a) E  −  ,i,(b) E =
3
6
( )
2 2
 x − (R / 2 ) x  2R 2 Q.17 cos θ =
 10
Q.7 2R, 3R 3 − 3  GM 2GM
  Q.18 (a) ; (b) r 2 ; (c)
r R

Q.8 6.4 km Q.19 T= 3 × 10−2 N

Q.20 re = a ( 3M m)
13

Q.9
( )
2πR 3 2 6 6

(
GM 2 2 + 3 3 ) Q.21 (i) −
3GM  m


+ m  , (ii)

 GM (2 3 + R) 
R  3   R 2 3 

P hysi cs | 10.45

Q.22 2pR2 1 + R  −1 
Q.24 cot  3 −
32 
   to vertical
 x  105 

2πd3 2 3π
Q.23 (a) T = (b) 2 (c) 2 Q.25 T =
3GM Gρ

Exercise 2

Multiple Correct Choice Type

Q.1 A, D Q.2 A, D Q.3 B, C, D Q.4 A,C Q.5 B, D Q.6 A, C, D


Q.7 A, D Q.8 A, B, C Q.9 A, D Q.10 A, B, D

Assertion Reasoning Type

Q.11 C Q.12 B Q.13 D Q.14 D Q.15 A

Comprehension Type
Paragraph 1: Q.16 B Q.17 A
Paragraph 2: Q.18 B Q.19 C
Paragraph 3: Q.20 B Q.21 A
Paragraph 4: Q.22 D Q.23 B

Previous Years’ Questions

Q.1 A Q.2 A Q.3 B Q.4 (a) −π × 10 4 km h (b) 3 × 10−4 rad s

Gm a3
Q.5 v = , T = 2π Q.6 (a) 6400 km (b) 7.9 km/s
a 3Gm

3 5 GM
Q.7 Q.8 99.5 R Q.9 3 Q.10 C
2 a

Q.11 B, D Q.12 B Q.13 A


1 0 . 4 6 | Gravitation

Solutions

JEE Main/Boards W=
U
r
Exercise 1 Hence weight =
U
r
Sol 1: Newton’s law of gravitation is called a universal
Sol 9: T ∝ r3/2
law because it is applicable anywhere in the universe.
3 3
T1  r 2  1011  2
=  1  =   = 10 10
Sol 2: The value is same on moon. G is called universal  1010 
T2  r2   
gravitational constant, which is constant anywhere in
the universe
Sol 10: Escape speed is still 11 km s–1 because escape
speed is irrespective of angle of launch (of course not
Sol 3: The packet doesn’t reach the earth (theoretically).
towards ground). We calculate escape velocity by
Because in a satellite, centrifugal force balances the
gravitational force on it. The same will happen with 1 GMm
m V2 =
the packet, which has same initial velocity as that of 2 r
satellite. (Kinetic energy + Potential energy)
= it Irrespective of angle of each.
Sol 4: Its reading will be zero. A spring balance shows
the net force the hanging body exerts on it net force by
body = Mass × Acceleration Sol 11: Acceleration due to gravity

∴ The net acceleration is zero as the centrifugal and GM


g=
gravitational forces balance each other, it reads zero. r2
1
g∝
Sol 5: Escape velocity is always constant for a given r2
celestial body. Escape velocity of earth is 11.2 km s–1. It Hence it is inversely proportional to r2
is same irrespective of mass.
1
Sol 12: f ∝
Sol 6: Weight = Mass × Net acceleration It will be zero, r2
if net acceleration is zero like a satellite. r 
2

f2 = f1 .  1 
GMm  r2 
Sol 7: Gravitational force (f) =
r2 r1 1
∴f∝m =
r2 2
f
∴ Acceleration due to gravity g =
m f1 1
∴ f2 = = N
GM 4 4
g=
r2 ∴ Gravitational force 0.25 N
It is independent of m.
Here both fall at same time Sol 13: If he goes to a height r, his distance from center
is 2r
GMm r1 1
Sol 8: U = i.e., =
r r2 2
weight W = mg 1
w∝ (w = Weight)
 GM  1  GMm  r2
= m.  =  
 r2  r  r 
P hysi cs | 10.47

2
w2 r  GMm
=  1  Sol 16: No. gravitational force f =
r2
w1  r2 
1
2 ⇒f∝
w2  1 1 r2
⇒ =   = ’
w1 2 4 1
will never become zero.
r2
i.e., his weight quadrates
So force doesn’t become zero.
GM
Sol 14: g = for r ≥ r0 (r0 is radius of earth)
r2 Sol 17: Gravity is the force with which a body pulls
dg GM another body towards its center.
=–
dr 2r 3 Acceleration due to gravity is the acceleration which it
produces in the body due to force of gravity
GM
⇒ Dg = – .Dr GM
2r 3 g=
r2
GM
⇒ Dg = – .h GM
2r03 Sol 18: g =
r2
GM m = Mass of earth
g= r1 r1 ≤ r0
r03
r = Radius of earth
GM r 2g
Dg = (r1 – r0) m=
r03 G
r1 – r0 = – x = We can calculate g by physical means, r is known,
hence M can be calculated
GM
⇒ Dg = . (– x) M
r03 Mean density of earth =
4 3
GM GM πr
⇒ (–x) = (–2h) 3
r03 r03 ∴
M can be calculated, mean density can also be
⇒ x = 2h calculated.
2
f1 r 
Sol 15: =  2  Sol 19: Escape velocity is the velocity with which when
f2  r1  a body is projected from the surface of a celestial body,
it crosses its potential barrier and escapes into out
r1 = r0, r2 = Distance of rocket from center of earth
space for bodies to escape total energy ≥ 0
f2 1 i.e. K. E + P.E ≥ 0
=
f1 3
At escape velocity
2
r  r K.E + P.E = 0
⇒ 3 =  2  ⇒ 2 = 3
 r1  r1 1 GMm
⇒ mv2 – =0
Height of rocket n = r2 – r1 2 r

r2 2GM
= 3 ⇒v=
r1 r
r2
–1= 3–1
r1 Sol 20: Gravity field is a field in which a body produces
r2 – r1 a force on another body.
⇒ = 3 –1
r1 Intensity of gravitational force is the force which a body
h attracts a body of unit mass
⇒ = 0.732
r1
1 0 . 4 8 | Gravitation

GM force between two bodies. In simple terms it can be


Intensity = ... (i) sides the proportionality constant for Newton’s law of
2
r 
gravitation
GM
= =g m
2
r2 G = 6.67 × 10–4 N  
 kg 
Sol 21: Kepler’s law T2 ∝ r3 fL2
Dimensional formula [G] =
2πr r2 m2
T= ;⇒ ∝ r3
v v 2 GMm
f=
1 r2
⇒ v2 ∝ ...... (i)
r [G][M][m] FL2
[f] = = . m.m.L–2 = f
For a planetary motion [r]2 m2

PE + KE = 0 It supports Newton’s law empirically


1  
⇒ mv2 + PE = 0 Sol 23: The distance travelled dx in time dt is dv
2
1 m 
⇒ PE = – mv2 ; ⇒ PE ∝ – = v .dt
2 r
Area swept by radius vector
d
Gravitational force = P.E 1→ →
dr dA = r × v dt
d  m 2
⇒F∝ –  1
dr  r  dA A = r v dt and
m km 2 A A
⇒f∝ ⇒f= (k-some constant) 1
2
r r2 dAP = r v dt
Let two bodies m1, m2 exert gravity on each other. 2 p p

k 2m2 ∴ dA = Constant, (Kepler’s 2nd law)


f12 = ⇒ f12 ∝ m1  ...... (i) dt
r2
k1m1 ⇒ rAvA = rPvP
f21 = ⇒ f21 ∝ m2 ...... (ii)
2
r The time taken is different as
Where f12 is force on body 1 by body 2 similarly f21
area of SBAC ± SCPB
defined
but |f12| = |f21| newton's third law Sol 24: T2 ∝ r3
⇒ t12 ∝ m2 2 2
r1  T 3  T1  3
m1m2 ⇒ =  1  ; ⇒ r1 = r2  
⇒ f12 = k3 (k3-some constant) r2  T2   T2 
r2
m1m2 gen rL = R, T1 = 32, T2 = 256
⇒f∝
2
r ⇒ r1 = 4R
Hence newton’s law of gravity is deduced. 3
∴  r 2
Sol 25: T1 = T2 =  1  (kepler’s 3rd law)
Sol 22: Newton’s law of universal gravitation states that  r2 
every point mass in the universe attracts every other r1 1
point mass with a force that is directly proportional to =
r2 2
the product of their masses and inversely proportional 3
to the square of distance between them.  1 2
∴ T1 = 365   = 129 days
m1m2 2
f∝
r2 1 year would have 129 days.
Gravitational constant (G) is a empirical physical
constant involved in the calculation of gravitational
P hysi cs | 10.49

Sol 26: For planetary motion, ∆r ∆g


× 100 = 2.5% ; ⇒ × 100 = –2(–2.5)
r g
1 GMm 2GM
mv2 = ;⇒V=
2 r r = 5%
∴ Acceleration due to gravity increases by 5%
2πr π3
V= ; ⇒ T = 2π Note :- Try focusing on the sign convention. It you get
T 2GM
confused, use common sense which implies when body
(2π)2 .r3 gets denser, its g increases like a black hole, etc.
⇒M=
2GT2
1
2 11 3 Sol 30: g ∝
4.π .(1.49 × 10 ) r2
=
–11 2
2 × 6.66 × 10 × (365 × 24 × 3600) g1 r 
2

=  2  ; ⇒ r2 = r1 g1
= 1.972 × 1030 kg
g2  r1  g2
g1
= 2 ; ⇒ r2 = r1 2
(2π)2 r 3 (2π)2 r 3 g2
Sol 27: M = ;⇒G=
2GT2 2MT2
Height = r2 – r1 = r1 . ( 2 –1)
r = 1.5 × 1011 m
= 2649.6 km
T = 365.3 days = 365.3 × 24 × 3600 seconds
M = 2 × 1030 Kg 1
Sol 31: wt ∝
r2
2 11 3
4( π) .(1.5 × 10 ) 2
⇒G= r 
2 × 2 × 10 30
× (365.3 × 24 × 3600) 2
w2 = w1 .  1 
 r2 
= 6.69× 10–11 m2/Kg–2
r1 = 6000 km
Note: For calculation purpose, you may take p2 = 10
r2 = 6600, 18000, 2400 (r2 = r1 + h)
GM w1 = 80 kg wt.
Sol 28: g =
r2
for r2 = 6600
gr 2  6000 
2
M= w2 = 80 
G  = 66.12 kg wt
 6600 
9.8 × (6.37 × 106 )2
= = 6.025 × 1024 kg for r2 = 18000
(6.66 × 10 –11 )
2
M gr 2 1  6000 
Mean density = = . w2 = 80   = 8.89 kg wt
4 3
πr
G 4 3
πr  18000 
3 3
for r2 = 24000
3 g 3 9.8
= = × 2
4 Gr 4 6.66 × 10 × 6.37 × 106
–11  6000 
w2 = 80   = 5 kg.wt
 24000 
Mean density (e) = 5.56 × 103 kg/m3
GM
Sol 32: g at a depth x, gx = (r0 – x)
GM dg 2GM r03
Sol 29: g = ; =–
r2 dr r3 GM
g at a height h, gh =
–2GM (r0 + h)2
2GM ∆g r3 Dr gx = gh
⇒ Dg = – Dr ; =
r 3 g GM By substituting we get solution
2
r But for some intelligent manipulation
∆g  ∆r  ∆g ∆r
= – 2 ; × 100 = – 2 × 100
g  r  g r
1 0 . 5 0 | Gravitation

gm –GMx
gx = (r0 – x) ⇒ Dg = 1 1 1 v2
r03 r03 = –
r R 2 gR 2
GM
g= 1 1 1 102
r03 = – ×
r 6400 2 9.8 × 10 –3 × (6400)2
2GM
Dg = – Dr (Differentiation) r = 2.56 × 104 km
x2

∴ g is equal ⇒ Dg is equal GM
Sol 35: Orbital velocity v =
r
2GM GM x
⇒– Dr = – x ; ⇒ Dr =
r03 r03 2 GM gR 2
= =
R +H R +H
Dr = n
160 g = 9.8 ms–1
x = 160km ; ⇒ h = = 80 km
2 R = 6.38 × 10–6
⇒ It is same at a height 80 km H = 250 km = 2.5 × 105
Note: h<<r0 is assumed hence we could apply this method v = 7756.6 ms–1
of differentiation
2πr 2π(R + H)
T= =
v v
Sol 33: Energy required = Total change in energy Initial
T = 5373 s
GMm
energy = –
R
Sol 36: Let orbital velocity = v
Find energy = P. E + K. E
GM
GMm 1 ⇒v= where m = Mass, R = Radius of point
=– + mv2 R
3R 2
(r = 3R because altitude = 2R) 2πR R3
T= = 2π .... (i)
For orbital motion, centrifugal force = Gravitational for v GM 

mv 2 GMm 4 3
= M= pR ρ
r r2 3
1 GMm R3
mv2 = ⇒ T = 2π
2 2r 4
G. πR3ρ
GMm 3
= (substitute r = 3R)
6R
3
GMm GMm GMm =2π = 5064 s
⇒ Final energy = – =– 4 πGρ
6R 3R 6R

GMm  GMm 
⇒ Energy required = – –   Sol 37: For escape velocity
6R  R 
1 GMm
mv 2e =
5GMm 2 R
=
6R
2GM
ve =
Sol 34: Kinetic energy = Change in potential energy R

1 GMm  GMm  ve = 2gR


⇒ mv2 = – –  
2 r  R 
1 1 1 v2 = 2 × 1.63 × 17 × 106
⇒ = –
r R 2 GM
= 2.354×103 ms–1
P hysi cs | 10.51

rρ = constant
M
Sol 38: ve ∝
R r2 ρ1 1  ρ2 
∴ = =  = 5 given 
r1 ρ2 5  ρ1 
vm Mm Re
= × 1
ve Rm Me ∴ Radius to be changed by a factor of
5
M   D 
vm = ve  m  ×  e  
 M  R 
 e   m Sol 4: (B) T = 2π
g
Mm 1
given = =
Me 9 1 GM
T∝ ;g=
Re g r2
=2
Rm
1 r T1 r1 M2
2 2 = ; =
⇒ vm = ve =2 = 5.28 kms–1 g M T2 r2 M1
9 9

Mm = Mass of planet mass  


T2 = T1  r2 M1 
Rm = Radius of planet mass  r1 M2 
 
r2 M1 1
= 2; = ; T = 2seconds
r1 M2 2 1
Exercise 2
 1 
∴ T2 = (2×2)   = 2 2 second
 2
Single Correct Choice Type
Note: Time period of a seconds pendulum is 2
1 seconds.
Sol 1: (B) g ∝
r2
2
g1 r  r g1 Sol 5: (C) Inside the shell, the gravitational field due to
=  2  ; ⇒ 2 = = 4 sphere is zero, but there is gravity due to particle.
g2  r1  r1 g2

⇒ r2 = 2R ; ⇒ altitude h = r2 – R v2 g v2 g
Sol 6: (A) g – = ;⇒ =
h=R R 2 R 2

2GM 1 GM
Sol 2: (A) Net acceleration at equator ve = = (4R)
R 2 R2
g’ = g – Rω2
(Rω2 is radial acceleration) g v2
=   (4R) = (4R)
∴ Weight at equator mg’ = mg – mRω2 2 R

acceleration at a depth d ve = 2v
g(R– d) g
⇒ gd = =g– d
R R M
Sol 7: (A) v ∝
given mgd = mg’ R
4
mgd R 2 ω2 π.R 3ρ
⇒ mg – mRω = mg –
2
⇒d= M 3
R g = ∝ R2 ; ⇒ v ∝ R
R R
GM vA RA
Sol 3: (B) g =
r2 ⇒ = =2
vB RB
4 3 4
m= pr ρ ; ⇒ g = prr
3 3
1 0 . 5 2 | Gravitation

24
2GM ∴ Time for successful interval interval = = 12 Hrs
Sol 8: (B) ve = 2
R

GMm Sol 14: (A) Let centre of mass be at a distance  from


P.E at surface =
R 4m
3GMm ⇒ 4 m  = m(d –  )
P.E at centre of earth =
2R
d
1 GMm ⇒  =
KE = DPE = 5
2 R
d 4d
⇒ Orbital radius of 4 m = ,m=
1 1 GMm GM 5 5
mv2 = ;⇒v=
2 2 R R Both bodies have same angular velocities
ve v
= ⇒ = constant
2 r
⇒ v = Kr (K = constant)
Sol 9: (B) Potential at surface = Potential at center for kd 4kd
hollow sphere vam = = , vm =
5 5
GM 1
Potential P = – (4m)(v 4m )2 2
r KEam 2  1 1
= =4   =
r = radius KEm 1 4 4
m(v m )2
GM 2
Let P0 = –
r0
Sol 15: (C) C-options defines the information about
GM 2GM
new potential P = – =– the escape velocities
r0 r0
2 B-option it is elliptical path
P < P0
∴ Decreases Sol 16: (D) Change in potential energy
GMm GMm GMm
∆E = – =
1 R 2R 2R
Sol 10: (B) g ∝
r2 Final velocity or orbital velocity

Sol 11: (A) In A, both 5M forces, cancel each other GM


v0 =
hence net force is proportional to (3M – M) and M (at 2R
center)
Change in P.E = Change in K.E.
The same is for B. 1 2
GMm 1
∴ FA = FB = mv2 – 2 mv 0
2R 2
Similarly fC ∝ (3m–m)2m ∴ fC > fB 1 GMm 1 GM
2 mv = 2R + 4 m . R
2
same is for FD
∴ FC = fD 3 GMm
1
∴ mv2 = 4 R
FA = FB < FC = FD 2
1 3
2 mv = 4 mgR
2
Sol 12: (B) It moves tangentially as there is no
centripetal force.
3
∴ Energy required is mgR
4
Sol 13: (C) Time period of a geo stationary satellite is
24 hrs but due to the given situation, it moves twice
above same point in one day
P hysi cs | 10.53

1 1 dA
Sol 17: (D) K.E ∝ V∝ ;∴ ∝ r
r r dt
∴ A is K.E.
∴ ratio of their area velocity
Total energy > Potential energy,
r1 1
1 = = =1:2
total energy, potential energy ∝ - r2 4
r
∴ C is total energy GM
Sol 23: (A) Field due to gravity E1 =
B is potential energy R2
4 3 4
m= pR ; ∴ E1 = πGrR
Sol 18: (C) Kepler’s 2nd law, areal velocity is constant. 3 3
Field due to infinite plate E2 = 2prtG
Sol 19: (B) Let final velocity of 4 M be V1 (t =H) = 2pρHG
5MV = 4MV1 – MV E1 = E2
(Conservation of linear momentum) 4 2R
∴ πGrR = 2pρHG ; ⇒ H =
3 3 3
⇒ V1 = V
2
Sol 24: (C) Maximum speed occurs at least distance
GM
now V = dA 1
r = rmin vmax.
dt 2
2GM 1
Ve = = 2 V= V1 > 2V 4 × 1016 = × 2 × 1012 × v
r 2
v = 4 × 104 ms–1
i.e., V1 > Ve
∴ v = 40 kms–1
⇒ Body gets unbound

Sol 20: (C) T ∝ r3/2


3 Previous Years’ Questions
T2 1  r 2 1
= ;  2  =
T1 2  r1  2 GM
Sol 1: (C) g =
2
R2
 1 3 r 1
⇒ r2 = r1 = or g ∝
  1
2 R2
(4)3
g will increase if R decreases
Sol 21: (A) Consider the planet to be at one of the
mgh
vertex. Sol 2: (A) DU =
h
Let its distance from sun be r, velocity be v. 1+
R
Area covered in time dT Given, h = R
1 2dA
dA = rv. dT ; = vr mgR 1
2 dt DU = = mgR
R 2
2mdA dA A 1+
mvr = ; = R
dt dt T
2mA mv 2
∴ Angular momentum = Sol 3: (B) ∝ R–5/2∴ v ∝ R–3/4
T R
2πR
dA Now, T =
Sol 22: (A) ∝ rv v
dt
1 0 . 5 4 | Gravitation

2 Just to escape from the gravitational pull, its total


R  mechanical energy should be zero therefore, its kinetic
or T ∝  
2

v energy should be + mv2


2
 R 
or T2 ∝ 
–3/ 4  Sol 9: (C)
R 
or T2 ∝ R7/2 2GM 2G × 10M
Vesc = = = 10 × 11 = 110km / s
R R / 10
Sol 4: (B) From Kepler’s third law
T2 ∝ r3 or T ∝ (r)3/2 Sol 10: (B) g = GM/r2
3/2
T2 r 
∴ =  2  GM
Sol 11: (A) g' = , acceleration due to gravity at
T1  r1  height h (R + h)2
3/2 3/2
 1
or T2 = T1  r2  = (365)   g GM R 2  R 
2
r 
 1 2 =
⇒ = g 
9 R (R + h)
2 2
R +h
T2 ≈ 129 days 1  R 
2
R 1
=
⇒   ⇒=
9 R +h R +h 3
1 T2 g1 ⇒ 3R = R + h ⇒ 2R = h
Sol 5: (D) T ∝ i.e., =
g T1 g2
Sol 12: (C) Position of the null point from mass m,
where g1 = Acceleration due to gravity on
r r
Earth’s surface =x =
4m 3
1+
=g m
g2 = Acceleration due to gravity at a height  3 12  Gm
V=
−Gm  +  = −9
h = R from earth’s surface = g/4  r 2r  r
 
  Sol 13: (D) To launch the spaceship out into free space,
 g  T2 g
Using g(h) = =2 from energy conservation,
2 T g/4
  h  1
 1 +   −GMm
  R  +E = 0
R
GMm  GM 
Sol 6: (C) Time period of a satellite very close to = E =  = mR mgR
R  R2 
earth’s surface is 84.6 min. Time period increases as
the distance of the satellite from the surface of earth = 6.4 x 1010 J
increase. So, time period of spy satellite orbiting a few
100 km above the earth’s surface should be slightly 1 GmM 1 GM GmM GmM  1  −GMm
Sol 14: (A) Ef = mv 20 − = m − =  − 1 =
greater than 84.6 min. Therefore. The most appropriate 2 3R 2 3R 3R 3R  2  6R
option is (C) or 2 h.
1 2 GmM 1 GM GmM GmM  1  −GMm
Ef = mv 0 − = m − =  − 1 =
2 3R 2 3R 3R 3R  2  6R
Sol 7: (D) In case of binary star system angular velocity
and hence the time period of both the stars are equal. = −GMm
Ei +K
R
Sol 8: (B) In circular orbit of a satellite, potential energy Ei = Ef
5GmM
= – 2 × (kinetic energy) K=
6R
1
=–2× mv2 = – mv2
2
P hysi cs | 10.55

Sol 15: (B) Potential at point P due to complete solid JEE Advanced/Boards
sphere
Exercise 1
GM  2  R  
2

=
− 3R −  
2R 3   2   Sol 1: 
GM  2 R 
2
P
=
−  3R −  d
2R 3  4 
 consider a small strip of rod of length d  at a distance
GM  11R 
2
 =
11GM  from the small mass. Let the mass of strip be dm
=
− −
3  4  8R
2R   GM
df = dm (df= force of attraction between the strip
2
Potential at point P due to cavity part and small mass)
M
G dm = σ dl
3 8 −3GM
= − = −
2 R 8R where σ is linear density of rod
2 M
σ=
So potential due to remaining part at point P 2L
GM
−11GM  −3GM  ⇒ df = . sdl
= −  2
−11GM
8R  −3GM
8R 
= −   integrating from L to 3L
8R + 3GM
−11GM  8R −GM
=
−11GM8R+ 3GM −GM R f 3L
GM
=
8R R ∫ df = ∫ 2
σ d
0 L

32
GmM GMm 1
Sol 16: (C) = f = GMσ. –
(R + h)2 R L2
v GM
v= 2
f = GMσ.
R 3L
1 h 2 GMm
mv − 0
= M 2
2 1 R = GM . .
2GM 2L 3L
v1 =R
R
GM2
GM F=
∆V
= ( =
2 − 1) gR( 2 − 1) 3L2
R
GmM GMm Note: Try understanding the boundary conditions. It is
GmM = GMm
2 = R
(RGmM
+ h) most important aspect of physics. Here it is integrated
(R + h)2 = GMm R
from L to 3L because the rod starts from distance L till
(RGmM
+ h)GM
2 GMm
= R
v = GM distance 3L from the small mass.
2
v(R=+ h)GMR R
v1 = R GMm
2GM
= 12 R
v1 mv
mv
− GMm =
− GMm
0
0
2
1 2R R = 2GM
2 1
mv1 2GM− R = 0 Sol 2: vc =
1 2 GMm R
v21mv − R =
= 1 2GM 0
v21 = 2GM R R
v1 = R Kinetic energy = Change in potential energy
GM
2GM
R ( =
v V=
=
∆ GM 2 − 1) gR( 2 − 1)
∆1V
= RR ( =
GM 2 − 1) gR( 2 − 1) 1 GMm  GMm 
∆V
= R ( =
2 − 1) gR( 2 − 1) mv2 = – – – 
GMR ( = 2 r  R 
∆V
= 2 − 1) gR( 2 − 1)
R
1 GMm GMm
m.(k . ve)2 = –
2 R r
1 0 . 5 6 | Gravitation

1 1 1 GM 1 1 GMm GMm
⇒ – = . m . k2 . 2 . mv2 = –
R r 2 R GMm 2 a a2 + L2
1 1 k2
⇒ – =
R r R 1 GMm
mv2 = ( a2 + L2 –a)
R 2 2 2
a (a +  )
⇒r=
1– k 2
2GM
height = r – R v= ( a2 + 2 – a)
2 2
a a +
R k 2R
= –R=
1– R 2 1– k 2 Here given  = a
k 2R
Hence it will rise to a height of
1– k 2 2GM ( 2 – 1)
⇒v=
a 2
Sol 3: Consider a small path on the ring of length d  ,
which subtends an angle dθ at the center. Let its mass 2GM  1 
be dM v= 1– 
a  2

Sol 4: Let the height be h


GM
g= (above the surface)
(R + h)2

g below the surface


GM(R – h)
g=
R3
d  = a dq
GM GM(R – h)
⇒ =
dM = σ d  (R + h)2 R3
M
r = Linear density of ring = ⇒ h3 + h2R – hR2 = 0
2πa
M M h = 0 (which is an obvious solution)
⇒ dM = . a dθ = dq
2πa 2π h2 + hR – R2 = 0
Let the particle be at a distance < along the axis from
R ± 3R 2
center. h=–
2
Potential energy due to → Mass patch
3 –1 ∴
GM h= ( h > 0)
dE = .dM (m = mass of particles) 2
r
GM M
= . dq Sol 5: (i) Maximum height = 8R
r 2π
⇒ Distance from center of earth (r)
r= L2 + a2
= 8R + R
E 2π
GMm 1 =9R
∫ dE = ∫ .

dq
Kinetic energy = Change in potential energy
0 0 L2 + a2

GMm 2π GMm 1 GMm  GMm 


E= × = mv2 = – – – 
2π L2 + a2 2 9R  R 
K 2 + a2
1  8 
Kinetic energy = Change in potential energy mv2 = GMm  
2  9R 
P hysi cs | 10.57

16GM 4 GM GM2  R
v= ;v= –  x– 
9R 3 R R3  2

(ii) Half minimum height = 4R for x > R

⇒ r = 4R + R = 5R E = E1 – E2
GM GM
1 GMm  GMm  =– +
⇒ mv2 = – – –  x 2
 R
2
2 9R  5R  8 x – 
 2
 
8GM 2 2GM  
⇒v= ;v= GM  1 8 
45R 3 5R =  – 
8  R
2
x2 
 k –  
 2 
Sol 6: We use principal of superposition gravitation
field due to sphere I 4 3
M= pR ρ0
GM 3
E1 = – ; x > R,  
x2  3

πGρ0R  1 8 
⇒E=  – 
6  R
2
x2 
 x – 2  
  
for x < R
E = E1 – E2

GM GM  R
=– x+ x– 
GM 3 3  2
=– .x ; x < R R R 
R3
GM
E=
Let M2 mass of sphere II 2R 2
R If is independent of x, hence uniform
Here the center is at hence distance function is
2
R
R– . Sol 7: Potential energy due to planet, at a distance r
2
from its cents
Assume sphere is uniform
G(3M)m
4 P.E1 = – ; 2R < r < 5R
⇒ M = pR3 r
3
3
Potential due to plant 2
4 R  1 4 3 M
m2 = π   =  πR  = GMm
3 2 8 3  8 P.E2 = – ; R < r < 4R
(6R – r)
GM2
E2 = – 2
x < 0, x > R
 R
x – 
 2

GM2  R P.E = P.E1 + P.E2


– 3 
x –  0<x<R
R   2 3GMm GMm
=– –
  r 6R – r
2
GM 3 1 
=– x <0, x > R E(r) = – GMm  + 
 R
2
 r 6R –r
8 R – 
 2 Differentiating
1 0 . 5 8 | Gravitation

dE  3 1  4 πbG reρ
= – GMm – 2 – (–1) v=
dr  r (6R – r)  3

For maximum E1 dE = 0 Escape velocity Ve =


2GM
dr r
3 1
⇒– + =0
r2 (6R – r)2 2G 4 3 16πρ rP2
= . πr .2ρ =
rP 3 p 3
6 3R
⇒r=
3 +1 v= ve

For particle to reach other side it is sufficient if its 4 π bG reρ 16πρ rP2
velocity is zero at ⇒ =
3 3
6 3R
r0 = bre
3 +1 ⇒ rP =
2
i.e., K.E = 0 at this point
4 × 10 –3 × 6400
6 3R =
∴ Speed is minimum at r0 = 2
3 +1
rP = 6.4 km
= 3R(3 – 3 )
Potential energy at x = 2R Maximum radius of planet is 6.4 km

∴ PE1 + PE2
GM
Sol 9: V = where V = Orbital velocity
G.3M GMm 17GMm r
= – =–
2R 6R – R 10R
GM GM
Potential energy at x = 5 R V1 = , V2= (r = R + height)
2R 3R
⇒ PE1 + PE2
V
Angular velocity w=
G(3M)m GMm R
=– –
5R 6R – R
V GM 1 GM
8 GMm ω1 = ;ω2 =
=– PE(2r) < PE(5R) 2R 2R 3R 3R
5 R
Relative angular velocity ωR = ω1 + ω2
PE(r0) – PE(2r) > PE(r0) – PE(5R)
Hence it has maximum speed at x = 2R GM  1 1 
=  + 
3
R 2 2 3 3
v2
Sol 8: Maximum range = 2π 2π
g t= =
wR GM 2 2 + 3 3
v2
b= R3 6 6
g
3
bGM 2πR 2 (6 6)
v= bg = ∴t=
re2 GM(2 2 + 3 3)
4 3
M= pr ρ
3 Sol 10: Let d be distance between them. Distance of
centre of mass from m
bG 4 3
⇒v= . πre ρ Md 2d
r2 3
e
r1 = ; r1 =
m+M 3
P hysi cs | 10.59

GMm 2GM2 2π
F= = Sol 13: ω1 =
d2 2
d T
Angular velocity of earth
Gravitational force = Centrifugal force

mv12 ωe = (T0 = 24 Hz)
2GM2 4GM T0
= ; ⇒ v1 =
d2 2d 3d ωr = ω1 + ωe or ω1 – ωe
3 –1
2π 2π T ±T
2d 3 T= = =  0 

T=

r1 = 3 = 2 πd 2 ωr 2π 2π
±  T0 T 
v1 4GM 3GM T T0
–1 –1 –1
3d  24 ± 1.5   17  5
Ms =   =   hrs,   hrs
m= (Ms = Mass of surfs)  24 × 1.5   24  8
3
3
2πd2 Sol 14: For a satellite
∴T=
GMs 1
3 |K.E| = |P.E.| = |total energy|
2
2πd2
Time period of earth Te = E1 = – 2 × 105 J
GMs
⇒ U1 = – 4 × 10–5, K1 = 2 × 105
given T = Te ; ⇒ d = R
U2 = – 2 × 10–5
–G.MM
i j
Sol 11: Total energy = ∑ ⇒ E1 = – 1 × 10–5 J
i< j rij
∆E = 1 × 105 J
 M2  6GM2 ∴ Energy required is 105 J
= – G  × (S3) = –
 a  a
  GMm
Sol 15: Total energy = – , r = Radius
6GM2 2r
Final energy = –
2a
GMm  1 1
6GM2 6GM2 ∴ Change is energy =  – 
Change in energy = – + 
2a a
2  Re r 
t = Change in energy
6GM2
=
2a GMm  1 1
⇒t=  – 
6GM2 2C 
∴ Work done is  Re r 
2a

GM Sol 16:
Sol 12: Potential =
r
1 1 
∴ Change in potential = GMm  – 
 R 2R 
GMm
=
2R
K.E. = Charge in P.E
Angle of view = 2q
1 GMm GM
mv2 = ;v= R 2π
2 2R R = 2cos–1 2R = 2(60°) = 120° =
3
Inside sphere v is constant 2π
Angular velocity of earth ωe =
T0
2R R3
∴ Time = =2 (T0 = 24 Hrs)
v GM
1 0 . 6 0 | Gravitation

GM (b) At maximum distance, they are mutually


Angular velocity of satellite ω = perpendicular radially about the winter of planet
8R 3
 GM  ∴ Maximum distance = 2r
ω = 
 r 

2GM
Relative angular velocity ωr = ω – ωe (c) Their relative velocity = 2V=
R
GM θ 2π
= – ωe ; T = = Sol 19: Gravitational force
8R 3 ω  Gm 
3 – ωe  GMm
 8R 3  F=
 
r2
Sol 17: Let final velocity of launch pad be x GMm
⇒ ∆F = – Dr
r3
⇒ 3mv0 = m(x + v0) + 2m(x)
∴ Net force at a height r + Dr
2v 0
⇒x= ..... (i)
3  GMm  2∆r 
= 1– 
Angular momentum = mvr r2  r 
2v 0 Centrifugal force f = mrw2
=m. . (3Rm)
3
Df = mw2Dr
L = 2mv0 Rm ..... (i)
∴ Net centrifugal force – f + Df
Angular momentum is constant
 ∆r 
∴ mvx . Rm = L = mrw2 1 + r 
 
⇒ mvx . Rm = 2mv0Rm
 ∆r  GMm  2∆r 
vx = 2v0 ...... (ii) = mrw2 1 + r  – 1– 
  r2  r 
∆KE = DPt ∆r GMm 2∆r
1 1  1 1  = mrw2 r +
⇒ mv2 – mx2 = GMm  –  r2 r
2 2  Rm 3Rm 
 2  GMm
⇒ v2 = x2 + 2GM   mrw2 = =mg (∴ satellite moon)
r2
 3Rm 
Now satellite equals ∆r 64 × 10 –3
∴ T = 3Mg =3×100×10 ×
r 6400
GM GM
v= ; ⇒ v0 = T = 3 × 10–2 N
r 3Rm

GM  GMm   GMm 
⇒ = v 20 ; ⇒ v2 = x2 + 4 v 20 Sol 21: (i) P.E =  –
3Rm  3 + 3– 
 R   3R 
2v 0 2
x= ;⇒v= 10 vx 3GM  m 
3 3 =–  + m
R  3 
vx 2v 0
cosθ = ; ⇒ θ = cos–1 (ii) Centrifugal force = Force towards center
v 2 10
v0
3 mv 2 GMm  GMm 3 
3 = +  . R
3 R R 2  ( 3R)2 2 
⇒ cos θ = ⇒ θ = cos–1  
10 10
 GM (2 3 + R) 
⇒v= 
GM  R 2 3 
Sol 18: (a) Orbital velocity V = 
R
P hysi cs | 10.61

Sol 22: Surface area of earth A = 4pR2 Lm


=2
LM
1
(c) k = mv2
2
1 2
mv m 2
km 2 m  rm  1
= = .   = (2)2
kM 1 2 2m  rM  2
mv
2 m
R km
cos θ =
x kM = 2
Area covered by the satellite on surface of earth
A1 = 2pR2(1 – cos θ) Sol 24:
where θ is semi-vertical angle
∴ Area out of reach = A – A1
= 4pR2 – 2pr2 (1 – cos θ)
= 2pR2(1 + cos θ)
 R
= 2pR2 1 + x 
  Let S be satellite, M Mumbai, C center of earth.
φ = 30°
Sol 23: (a) M = 2m
let SC = r
md
Center of mass from m (r1)
m+m MC = R
2d d ⇒ r sin φ = (r cos φ – R) tan q
= ; ⇒ r2 =
3 3   2
GMm  sin φ   T GM  3
Force between them (t)= ⇒ θ = tan–1   ;r=  
d2  cos φ – R   2π 
   
2GM 2  r 
=
d2 T = 24 Hr
Let velocity of mass m be v1 GM = R g
mv12 mv12 2GM2 R = radius of earth
= (F) ; ⇒ =
r1 2d d2
Upon substitution we get
3
2π 2πr
 32 
4GM θ = cot–1  3 – 105 
⇒ v1 = ;T= =
3d w v  

2d
2π × Sol 25: Angular velocity be ω
3 = 4π 3d3 2πd3/2
= =
4GM 3 4GM 3GM mrω2 = mg
3d 2π (2π)2
ω= ; mR. = mg
vm vM T T2
(b) =
rm rM
R
2 T = 2π
Lm 1 r 
mv mrm 1 g
= –  m  = (2)2
LM 2mv mrm 2  rM  2
GM
g=
R2
1 0 . 6 2 | Gravitation

G 4 3  ∆v ∆r 
= . pR r ∆L = mvr  + 
R 2 3  v r 

R
∴ T = 2π GMm GMm
4 U=– ; Du = Dr
GR. πρ r r2
3
1
3π k= mv2
T= 2

Dk = mv Du
Du = – 2Dk

Exercise 2 GMm
Dr = – 2mvDu
r2
Multiple Correct Choice Type
GMm 1 
∆r = 2  mv 2 
Sol 1: (A, D) r 2
 2 
GM ∆r 2∆v  ∆r 
g= ; r > R =– ; ⇒ ∆L = mvr  
r 2 r v  2r 
GM Dr < 0
r; r<R
2
R ⇒ ∆L < 0

Sol 2: (A, D) f12 = f21


Sol 5: (B, D) Communication satellites are geo
⇒ m1a1 = m2a2 stationary.
⇒ if m1 < m2
Sol 6: (A, C, D) Only potential energy increase
∴ a1 > a2
GMm
Total energy is constant, by law of conservation of P.E = –
r
energy centre of mass is in motion in the reference
from of the masses. 1
v decrease; v ∝
r
Sol 3: (B, C, D) Everywhere gravitational field is zero and hence angular velocity and centripetal acceleration
which is same everywhere inside the spherical shell. decreases as r increases.
dE
Everywhere potential is same as – =0
dx Sol 7: (A, D) Acceleration is always directed towards
i.e., gravitational field is same. Potential inside sphere is centre of earth. Centripetal force.
equal to that on surface.
1
Sol 8: (A, B, C) v ∝ , T ∝ r3/2
Sol 4: (A, C) The satellite will always be in orbital r
motion at every instant ∴ Speed is maximum and time period is minimum
|u| = 2|k| Potential energy is minimum
|Du| = 2|Dk|  1
P.E ∝  – 
u = – 2k  r
⇒ Du = – 2Dk
∴ Sol 9: (A, D) Satellite has to be above equator at some
Du is –ve time
hence kinetic energy increases 2
r r
T = 2π   . ; r = radius of orbit
L = mvr R  g
P hysi cs | 10.63

Comprehension type
R
> 2π
g Paragraph 1:
2
∴ r
r > R,   > 1 Sol 16: (B) Let P.E at ∞ = 0
R 
GM1M2
Final P.E =
d
Sol 10: (A, B, D) T ∝ R3/2
final distance between center of masses
∴ S1 : S2 = 13/2 : 43/2 = 1 : 8
d = R + 2R = 3R
1
v∝ ⇒ v1 : v2 = 4 : 1 = 2 : 1
1/2 1/2
mass of small sphere is m
r
Angular momentum L ∝ r1/2 mass of smaller sphere
3 3
L1 : L2 = 1 : 41/2 = 1 : 2 R   2R 
= m  2  = m  
Let velocities be 2k, k R
  R 
Relative velocities are 3k, k =8m
i.e. v1 : v2 =3 : 1 GM.8m 8GM2
Final energy = =
Relative radii = 4R + R, 4R – R 3R 3R
Let initial velocities be v1, v2
= 5R, 3R
Let final velocities be v3, v4
i.e. R1 : R2 =4 : 3.
v since centre of mass is at rest
ω=  
R v3 = – e v1
3 1
∴ ω1 : ω2 = :
5 3 (e = Coefficient of restitution)
ω1 : ω2 = 9 : 5 1
|V3| = |V |
2 1
2
Assertion Reasoning Type 1 1 1 
⇒ mv 32 = m  v 
2 2 2 
Sol 11: (C) There is no such real radial force. It only
appears in moon’s frame of reference as centrifugal 1 1 2
=  mv 
force. 4 2 1 

1 2 1 1 2
Sol 12: (B) Statement-I is true because there is no net Similarly mv 4 =  mv 
acceleration downward in it. 2 4 2 2

1 2 1
Sol 13: (D) Geostationary satellites have fixed orbital ∴ Final energy = mv + mv 24
radius and do have 24 hours of time period of revolution. 2 3 2

1 1 2 1 2
Sol 14: (D) Statement-I is only true long distances =  mv + mv 
4 2 1 2 2
between them.

1  8Gm2 
Sol 15: (A) For travel, energy required =  
4  3R 
 
= maximum P.E – P.E at surface
2Gm2
=
3R
1 0 . 6 4 | Gravitation

Sol 17: (A) Change in P.E = kinetic energy Note: Study the self-energy of objects here is a
derivation.
2GM2  8GM2 
= P.E – –  Consider a sphere of density ρ and initial radius r initial
3R  3R 
 4 3
mass m = pr ρ
6GM2 GM(8M) 3
⇒ PE = =
3R 4R Let additional mass added
dm = 4pr2.drρ
\Maximum distance between them is 4R
GMdm
Note:- Try deriving the result Increase is energy dE =
r
→ → 4
V 3 = – e V1 used in the problem. Here centre of mass G. πr3ρ . 4 πr 2drρ
dE = 3
is at rest.
r

Paragraph 2: (4 π)2 2
dE = r . G. r4 . dr
3
Sol 18: (B) Area of ASBC > ASBD R
(4 π)2 2 r5
∴ t1 > t2
E= ∫ dE = 3
r .G.
5
0

Kepler’s 2 law
nd
M
ρ= (M is final mass)
4 3
Sol 19: (C) |u| > |k| always πR
3
R5 ( 4 π )
2
Because if |k| ≥ |u| body escapes from the suns m2
⇒E= . ×
gravitational force 2 5 3
 4π  6
  R
 3 
Paragraph 3:
3GM2
E=
Sol 20: (B) Self energy of a uniform sphere of radius R 5R
and mass M is given by
3GM2 Paragraph 4:
E=–
5R
1
∴ Change in energy Sol 22: (D) 1 Bose = Newton
c
3GM2  1 1 Let T be tension in the string.
=–  – 
5 R / 2 R 
Let a1, a2 be acceleration of m1, m2 downward
2
3GM
=  gm2 
5R  1
–T
energy a=  c c
Increase in temperature =  m2 
M.S  1 
 
3GM2 1
= . gm12
5R M.S where is downward r gravitational force on m1
3GM c
=  gm2 
5RS  2
–T
Similarly a2 =  c c
3GM  m2 
Sol 21: (A) T0 =  
5RS  
5SRT0 a1 + a2 = 0 by constrain equation
M=
3G
P hysi cs | 10.65

 gm2 – T   gm2 – T  F1 r1
 1 c +  2  =0
= for r1 < R and r2 < R
 m2   m2  F2 r2
 1   2  GM 1
And for r ≤ R, F = or F ∝
2
 m2m2  1 r r2
⇒ T = 2g  1 2 . F1 r22
 m2 + m2  c i.e., = for r1 > R and r2 > R
 1 2  F2 r12
2 × 5 × 22 × 42 1
= . Sol 4: T ∝ r3/2 v2
2 2 c
2 +4 T2
2/3
or r ∝ T
32  v1 T1
T= 2/3
c r2 T 
=  2  r1
r2
T = 32 Bose r1  T1 
Note: If you do not know bose, try guessing what it could 2/3 2/3
be. 1 newton is the force which is produced when an T  8
r2=  2  r1=   (10 4 =
) 4 × 10 4 km
object of mass 1 kg moves with an acceleration of 1 ms–2.  T1   1
Similarly define bose. This is the best assumption you can
do with the given amount of information. 2 π r1 (2 π)(10 4 )
Now, v1 = = = 2 π × 10 4 km / h
T1 1
gm2
Sol 23: (B) Force due to gravity F = =gm2 bose
c 2 π r2 (2 π)(4 × 10 4 )
v2
= = = ( π×10 4 ) km/ h
force along slope f1 = f sin q T2 8
f = f1 (a) Speed of S2 relative to S1
= f sin q
= v 2 − v1 = −π× 10 4 km / h
= gm2sin θ bose
(b) Angular speed of S2 as observed by S1
1
= 5 × (2)2.
2  5 
π×10 4 × m / s
= 10 bose | v 2 − v1 |  18 
=ωr =
| r2 − r1 | (3 × 107 m)

Previous Years’ Questions = 0.3 × 10 −3 rad / s= 3 × 10 −4 rad / s

Sol 1: (A) Force acting on astronaut is utilized in Sol 5: Centre should be at O and radius r. We can
providing necessary centripetal force, thus he fells calculate r from figure (b).
weightlessness, as he is in a state of free fall.

Sol 2: (A) The gravitational field is zero at the centre of a a


a solid sphere. The small spheres can be considered as O F
negative mass m located at A and B. The gravitational
o
30 r r
Fnet
field due to these masses at O is equal and opposite. 30o 30o
Hence, the resultant field at O is zero.
o o
a 30 30
a/2 F
(c and d) → are correct because plane of these circles
(a) (b) (c)
is y-z, i.e., perpendicular to x-axis i.e., potential at any
point on these two circles will be equal due to the
positive mass M and negative masses – m and – m. a/2 3 a
= cos=
30° ∴r =
r 2 3
GM
Sol 3: (B) For r ≤ R. F = , r or F ∝ r Further net force on any particle towards centre
R3
1 0 . 6 6 | Gravitation

=Fnet 2F cos 30° Now, the body of mass m is projected from the surface
of larger star towards the smaller one. Between C2 and
 3
P it is attracted towards 2 and between C1 and P it will
 Gm2 3 Gm2
=2   = be attracted towards 1
 a2  2  a2
  
M M
mv 2
This net force should be equal to
r C1 r1 P r2 C2
a
2a
3 Gm2 mv 2 Gm
∴ = ∴ v=
a2 a/ 3 a 1 2

Time period of circular motion Therefore, the body should be projected to just cross
point P because beyond that the particle is attracted
2 π r 2 π (a/ 3) a3 towards the smaller star itself.
=T = = 2π
v Gm / a 3 Gm 1
From conservation of mechanical energy mv 2min
2
Sol 6: (a) Orbital speed of a satellite at distance r from = Potential energy of the body at P – Potential energy
centre of earth, at the surface of larger star.

GM GM 1  GMm 16 GMm   GMm 16 GMm 


v0
= = ….(i) ∴ mv 2min =  − − − − − 
r R+h 2  r1 r2   10 a − 2a 2a 

ve 2GM / R GM
Given, v=
0 = = …(ii) C1 P C2
2 2 2R 
v=0 vmin
From Eqs. (i) and (ii), we get
h= R= 6400 km 1  48  GMm
Or mv 2min =  
2  8  a
(b) Decrease in potential energy=increase in kinetic
energy
3 5  GM 
1 2 ( ∆ U) ∴ v min =  
or mv 2 = ∆ U ∴ v = 2  a 
2 m

 mgh  Sol 8: Speed of particle at A, v A = escape velocity on


2 
 1+ h / R  the surface of earth
= = gR
m
2GM
3
=
(h = R) = 9.8 × 6400 × 10 = 7919 m / s = 7.9 km / s R
B vB=0

Sol 7: Let there are two stars 1 and 2 as shown below h


Let P is a point between C1 and C2 , where gravitational
field strength is zero or at P field strength due to star 1 R
A 100
is equal and opposite to the field strength due to star
2 Hence,

GM G (16M) r2
= or =4
r12 r22 r1

 4 
r1 + r2 =
10 a ∴ r2  =
=  (10 a) 8 a At highest point B, vB = 0
 4 +1
Applying conservation of mechanical energy, decrease
and r1 = 2 a in kinetic energy = increase in gravitational potential
energy
P hysi cs | 10.67

1 Surface area of P= A= 4 πRP2


or mv 2A = UB − UA =m (vB − v A )
2
Surface area of Q= 4A= 4 πR 2Q
v2
or A= vB − v A
2 ⇒ RQ =
2RP
  2
4 3 4 3 4 3
GM GM  GM 1.5R 2 − 0.5  R − R   Mass R is MR = MP +MQ
∴ =− − − 4 3 4 3 4 3 ρ 3 πRR =ρ 3 πRP + ρ 3 πR Q
R R + h  R3   100   ρ πRR =ρ πRP + ρ πR Q
   4
3 4
3 4
3
ρ π3RR3 = 3
ρ π3RP3 + ρ3 πR3Q ⇒ RR3 = RP3 + R3Q = 9RP3
2 ⇒3RR = RP +3R Q = 9RP3
1 1 3  1  99  1 ⇒ R 3 3 3 3
1/3RP + R Q = 9RP
= R= 91/3 RP ⇒ RR > R Q > RP
or =
− + −    . R= 9 R ⇒ R > R > R
R
R
R R + h 2R  2   100  R R P R Q P
R=
R 91/3 RP ⇒ RR > R Q > RP
Therefore VR >VQ > VP
Solving this equation , we get
VR 1/3 V 1
= 9= and P
h = 99.5 R VP VQ 2

4  Sol 12: (B) Inside planet


G  π R3  ρ
Sol 9:=
GM
g = 3  or g ∝ ρ R r 4
2 2
g=
i gs = Gπrρ
R R R 3
g Force to keep the wire at rest (F)
or R ∝
ρ = Weight of wire

Now escape velocity, v e = 2gR or v e ∝ gR R 4  4   9λ  2


= ∫4R /5 (λdr)  3 Gπ=
rρ 

 Gπρ    R
3   50 
g g2 Me
or v e ∝ g × ∝ Here, ρ = density of earth =
ρ ρ 4 2
πR
6 3 3 e
)planet (11 km s−1 )
∴ (v e = × = 3 km s−1 R
121 2 Also, R = e ; putting all values, F = 108 N
10

Sol 10: (C) For r ≤ R Sol 13: (A) Measured value of r = (10 ± 1) mm
mv 2 GmM ∆r = 1 mm
= …… (i)
r r2  ∆r 1
Relative error
= = = 100%
r 10
4 
Here, M =  πr 3  ρ0 Average value of
 3  n=5
Substituting in Eq. (i), we get ∑ Ti (0.52 + 0.56 + 0.57 + 0.54 + 0.59)
i=l
v∝r =T = s
n 5
i.e., v-r graph is a straight line passing though origin
⇒T=
0.556s  0.56s
For r > R
4  0.01
Gm  πR 3  ρ0 Relative error in time period  = 1.79%
mv 2
3  1 0.56
= or v ∝
r r 2
r Reported value of (R - r) = (50 ± 2) mm
2
The corresponding v-r graph will be as shown in option (c) Relative error in (R - r) = =4%
50

4 7(R − r) ∆g  ∆T  ∆(R − r)
2.Gρ πR 3 T=
2π ⇒ 2
= +
2GM 3 4Gρ 5g g  T  (R − r)
Sol 11: (B, D)
= Ves = = R
R R 3 ∆g
Ves ∝ R ⇒ 7.57%
=
g
2017-18 100 &
op kers
Class 11 T
By E ran culty
-JE Fa r
IIT enior emie .
S fP r es
o titut
Ins

PHYSICS
FOR JEE MAIN & ADVANCED
SECOND
EDITION

Exhaustive Theory
(Now Revised)

Formula Sheet
9000+ Problems
based on latest JEE pattern

2500 + 1000 (New) Problems


of previous 35 years of
AIEEE (JEE Main) and IIT-JEE (JEE Adv)

5000+Illustrations and Solved Examples


Detailed Solutions
of all problems available

Topic Covered Plancess Concepts


Tips & Tricks, Facts, Notes, Misconceptions,
Waves on a String Key Take Aways, Problem Solving Tactics

PlancEssential
Questions recommended for revision
11. WAV E S O N A S T R I N G

1. INTRODUCTION
We come across numerous phenomena in nature based on the properties of wave motion. This chapter describes
the equations and properties of wave motion. The study of waves on a string forms the basis of understanding the
phenomena associated with sound waves and other mechanical and non-mechanical waves. Wave transmits both
energy and momentum from one region to other. Mechanical waves require a medium to travel, whereas non-
mechanical waves don’t. Wave on a string is a mechanical wave but the properties and concepts studied here will
be useful in studying non-mechanical waves as well.

2. WAVE MOTION
A wave is a disturbance or variation traveling through space and matter. It is the undulating movement of energy
from one point to another. The medium through which the wave passes may experience some oscillations, but
the particles in the medium do not travel with the wave. The wave equation, which is a differential equation,
expresses the properties of motion in waves. Waves come in all shapes and sizes, and accordingly, the mathematical
expression of the wave equation also varies.

2.1 Types of Waves


Waves can, broadly, be classified into two types:
(a) Mechanical waves: Waves that require a medium/matter for their propagation are called mechanical waves.
These waves are generated due a disturbance in the medium (particles in the matter) and while the wave
travels through the medium, the movement of the medium (particles) is minimal. Therefore mechanical waves
propagate only energy, not matter. Both the wave and the energy propagate in the same direction. All waves
(mechanical or electromagnetic) have a certain energy. Only a medium possessing elasticity and inertia can
propagate a mechanical wave.
(b) Non-mechanical waves/Electromagnetic waves: Waves that do not require a medium/matter for their
propagation are called electromagnetic waves. These waves are formed by the coupling of electric and
magnetic fields due to acceleration of electric charge and can travel through vacuum. Depending on the
wavelength of the electromagnetic wave, they are classified as radio waves, microwaves, infrared radiation,
visible light, ultraviolet radiation, X-rays, and gamma rays.

3. WAVE PULSE ON A STRING


A wave pulse is a single, sudden, and short-duration disturbance that moves from point A to point B through a
medium, e.g., a string. We know that waves originate when a disturbance at the source point moves through one
1 1 . 2 | Waves on a String

particle to its adjacent particles from one end of the medium to the other. Now, when a disturbance-producing
source active for a short time, a wave pulse passes through the medium. Conversely, when the source remains
active for an extended time, creating a series of motions, it results in a wave train or a wave packet. Thus, a wave
train is a group of waves traveling in the same direction.
For example, if the person in figure decides to move his hand up and down 10 times and then stop, a wave train
consisting of 10 loops will move on the string.

4. EQUATION OF A TRAVELING WAVE


In the figure, let us assume that the man starts moving his hand at t = 0 and finished his job at
t = ∆t. The vertical displacement of the left end of the string, denoted as y, is a function of time. It is zero for
t < 0 and t> ∆ t . This function can be represented by f (t). Let us take the left end of the string as the source of the
wave and take the X axis along the string toward right. The function f (t) represents the displacement y of the
particle at x=0 as a function of time: y(x=0, t) =f (t).
The disturbance on the string travels towards right at a constant speed.
Thus, the displacement produced at the left end at time t reaches the point
x (a)
x at time t +   Similarly, the displacement of the particle at point x at
v (b)
time t was generated at the left end at the time t-x/. But the displacement
of the left end at time t-x/v is f (t-x/v). Hence, y (x, t) = y(x=0, t-x/v) = (c)
f(t-x/v).
The displacement of the particle at x at time t, i.e., y(x, t) is generally Figure 11.1
abbreviated as y and the wave equation is written as y = f (t-x/v). … (i)
Equation (i) represents a wave traveling in the positive direction x at a constant speed. Such a wave is called a
traveling wave or a progressive wave. The function f is dependent on the movement of the source, and therefore,
arbitrary. The time t and the position x must be represented in the wave equation in the form t-x/ only. For example,
(t − x/ v )
(t − x/ v) , and y = Ae − T
are valid wave equations.
y = A sin
T
Both these equations represent the movement of the wave in the positive direction x at constant speed v.

(x2 − v 2 t2 )
In contrast, the equation y = A sin does not represent the movement of the wave in the direction x at
L2
a constant speed . If a wave travels in the negative direction at a speed , its general equation may be written as
y = f(t + x/v)  ... (ii)
 vt − x
y f
Equation (i) can also be written as =  or y g(x − v t), 
= ....(iii)
 v 

where g is some other function having the following meaning: Let us assume that t = 0 in the wave
equation. Then, we get the displacement of various particle at t = 0, i.e., y= (x,t= 0)= g(x) . Thus, the function
g(x) represents the shape of the string at t = 0. Assuming that the displacement of the different particles at
t = 0 is represented by the function g(x), the displacement of the particle at x at time t will be
y = g(x - vt). Similarly, if the wave is traveling along the negative direction x and the displacement of a different
particle at t = 0 is g(x), the displacement of the particle at x at time t will be y = g(x + t) …(iv)

Illustration 1: The wave equation of a wave propagating on a stretched string along its length taken as the positive
  t x 2 
x axis is given as=y y 0 exp  −  −   where y0 = 4 mm, T = 1.0 s and λ = 4 cm.
 T λ 
 
P hysi cs | 11.3

(a) Find the velocity of the wave.


(b) Find the function f(t) giving the displacement of the particle at x = 0.
(c) Find the function g(x) giving the shape of the string at t = 0.
(d) Plot the shape g(x) of the sting at t = 0.
(e) Plot the shape of the string at t = 5s. (JEE MAIN)

Sol: The wave moves having natural frequency of ν and wavelength λ has velocity V = νλ .
1 λ
As the frequency is ν = the velocity of the wave is then V = .
T T
2
1 x 
− t − 
(a) The wave equation can be written as y = y 0 e T2  λ /T 

λ 4 cm
On comparison with the general equation y = f (t – x/), we can infer that, υ = = = 4 cms−1
T 1.0 s
2
(b) Putting x =0 in the given equation f(t) = y 0 e − (t/T)  … (i)
2
(c) Putting t = 0 in the given equation g(t) = y 0 e − (x/ λ )  … (ii)

(d)

x=0

(e)
x=0 x=0 cm
Figure 11.2

4.1 Sine Wave Traveling on a String


Consider the scenario where the person in the  Wave
length
Fig. 11.3 keeps moving his hand up and down y

continuously. As energy is being constantly supplied a c


by the person, the wave generated at the source e
Fixed
keeps oscillating the any part of the string through point
which it passes. Thus energy passes from the left (the x
source) to the right continuously till the person gets Hand moves b amplitude d
tired. The nature of the vibration of any particle in the up and down
Direction

string is similar to that of the left end (the source),


The rope vibrates at rights angles
to the direction of the wave.
the only difference is that there is an interval of
x/ between two motions.
When the person in the Fig 11.3 oscillates the
left end x = 0 in a simple harmonic motion, the
equation of motion of this end may be written as
f(t) = Asin ω t  … (i)
where A is the amplitude and ω is the angular
frequency. The time period of oscillation is given
by T=2π/ω and the frequency of oscillation is
Figure 11.3
1 1 . 4 | Waves on a String

v = 1/T = ω /2π. The wave produced by such an oscillation source is called a sine wave or sinusoidal wave.
The displacement of the particle at x at time t will be
y =f(t − x/ υ) or y =Asinw(t − x/ υ)  … (ii)
∂y
The velocity of the particle at x at time t is given by =A ω cos(t − x/ υ)  … (iii)
∂t

PLANCESS CONCEPTS

•• While differentiating with respect to t, we should treat x as constant – it is the same particle whose

displacement should be considered as a function of time. Therefore, the symbol is used in place
∂t
d
of .
dt
•• In the event that the waves travel along negative x direction, the direction of Vp will change.
Particle velocity is the same as wave velocity. The two are totally different. While the wave moves
on the string at a constant velocity along the x axis, the particle moves up and down with velocity

y , which changes with x and t.
∂t
P W
R Q
t

t+T/4

t+T/2

t+3T/4

t+T

x x + vT x + 2vT x + 3vT x + 4vT


Figure 11.4

Above figure shows change in shape of string with time

Vaibhav Krishan (JEE 2009 AIR 22)

4.1.1 Some Important Terms


(a) Amplitude: In a wave, the crest represents highest point the wave rises to and equilibrium represents the
default position from which a wave arises. Therefore, the distance between the crest and the equilibrium point
in a single wave cycle is referred to as the equilibrium.
P hysi cs | 11.5

(b) Wavelength: The distance between any two points with the same phase, such as between crests or troughs
is referred to as the wavelength λ. It is generally measured in meters.
(c) Wave Number: Wave number is a measurement of a certain number of wavelengths for some given distance.
In a sense, the wave number is like a spatial analogue of frequency. Typically, wave number is taken to be 2π
times the number of wavelengths per unit of distance, which is the number of radians for each unit of distance

as well. k =
λ
(d) Time Period: A period T is the time needed for one complete cycle of vibration of a wave to pass a given
point.
(e) Frequency: Frequency describes the number of waves that pass a fixed place in a given amount of time and
1
is typically measured in hertz. These are related by f =
T
(f) Angular Frequency: The angular frequency ω gives the frequency with which phase changes. It is expressed

in radians per second. It is related to the frequency or period by ω = 2πf =  ….(i)
T

Illustration 2: Consider the wave y = (5 mm) sin [(1 cm -1) x – (60s -1) t]. Find (a) the amplitude, (b) the wave number,
(c) the wavelength, (d) the frequency, (e) the time period and (f) the wave velocity.  (JEE MAIN)

Sol: Comparing the given equation =


with y A sin (kx − ωt) we get the values of wave number k, amplitude A and
angular frequency. The frequency ω =2πυ=2π/T. The velocity of wave is v = νλ and the wave number of wave is

K= .
λ
On comparing the given equation with standard equation of a traveling wave, we find

(a) Amplitude A = 5mm, (b) wave number k = 1 cm-1, (c) wavelength λ = = 2π cm
k
ω 60 30 1 π
(d) Frequency =
ν = Hz
= Hz (e) Time period T= = s
2 π 2π π ν 30

(f) Wave velocity v = νλ = 60 cms-1

4.2 Velocity of Waves on a String


The wave speed depends on the properties of the medium. For a string, the speed of a transverse wave traveling
along a vibrating string (v) is directly proportional to the square root of the tension of the string (T) over the linear
mass density (μ):

T
v=
µ
’ where the linear density μ is the mass per unit length of the string  ….(i)

4.3 Phase Difference


The amount by which two cyclical motions of the same frequency, Crest
are out of step with each other. It can be measured in degrees
from 0° to 360°, radians from 0 to 2π, or seconds of time.. If two
oscillators have the same frequency and no phase difference, they
are said to be in phase. Conversely, if they have the same frequency
and different phases, then they have a phase difference and they
Trough
are said to be out of phase with each other. If the phase difference Cycle
is 180° (π radians), then the two oscillators are said to be in anti-
phase. Figure 11.5
1 1 . 6 | Waves on a String

4.4 Crest and Trough


In a wave, the crest represents highest point the wave rises within a cycle. A trough is the opposite of a crest, hence
the minimum or lowest point in a cycle.

5. ALTERNATIVE FORMS OF WAVE EQUATION


As seen earlier, the wave equation of a wave traveling in x direction is =
y A sin ω(t − x / υ),
This can also be written in several other forms such as =
y A sin(ωt − kx),  … (i)

t x
y= A sin2π  −   … (ii)
T λ
= y A sin[k (vt − x)]  … (iii)

Please bear in mind our choice of t = 0 in writing equation (v) from which the wave equation has been derived. Also,
the point at which the left end x = 0 crosses its mean position y = 0 and goes up has been chosen as the origin of
time. For a general choice of the origin of time, a phase constant will have to be added to give the equation
y=A sin [ω (t-x/v) + φ] … (iv)
The constant ϕ will be π/2 If we choose t = 0 at an instant when the left end reaches its extreme position y=A, then
the constant ϕ will be π/2. The equation will then be
y A cos ω(t − x / v), 
= … (v)
If on the other hand, t = 0 is taken at the point when the left end is crossing the mean position from an upward to
downward direction, ϕ will be π and the equation will be

x 
y = A sin ω  − t  or y = A sin (kx - ωt) … (vi)
v 

PLANCESS CONCEPTS

Both sin(kx – ωt) and sin(ωt – kx) differ just by a phase of “π”. If a particle at t = 0, x = 0 in its mean
position is moving upwards (in first wave), then the same particle would be in mean position and the
particle would be moving down!
B Rajiv Reddy (JEE 2012, AIR 11)

Illustration 3: Fig 11.6 shows a string of linear mass density 1.0 Kg m-1 and a length of 50 cm. Find the time taken
by a wave pulse to travel through the length of the string. Take g = 10 ms-2.  (JEE MAIN)

Sol: The wave velocity on stretched string under tension F = mg is given by


F
v= where µ is mass per unit length of the string.
µ
The tension in the string is F = mg = 10N. Given that the mass per unit
F 10N 1 kg
length is μ = 1.0 Kg m-1, the wave velocity is,=
v = = 10ms−1 .
µ 0.1kgm−1 Figure 11.6

Therefore, to travel through 50 cm, the wave pulse will take 0.05 s.
P hysi cs | 11.7

Illustration 4: A rubber tube that is 12.0 m long and that has a total mass of 0.9 kg is fastened to
fixed base. At the other end of the tube, a cord is attached that passes over a pulley and supports
an object with a mass of 5.0 kg. If the tube is struck at one end, find the time required for the
transverse pulse to reach the other end. (g = 9.8 m/s2) (JEE MAIN)

Sol: For the string under the tension T =mg where m is mass of the block. When the rod is struck B

T
at lower rod, the wave thus originated travels at speed v = where µ is the mass per unit length
of the string. µ
A
Tension in the rubber tube AB, T = mg or T = (5.0) (9.8) = 49 N
0.9 Figure 11.7
Mass per unit length of rubber tube, =
µ = 0.075kg / m
12

T 49
∴ Speed of wave on the tube,=
v = = 25.56m / s
µ 0.075

AB 12
∴ The required time is=
t = = 0.47s .
v 25.56

∂2 y ∂2 y
Illustration 5: Prove that the equation y = a sin (ωt – kx) satisfies the wave equation = ν2 and find speed
∂ t2 ∂ x2
of the wave and the direction in which it is traveling. (JEE ADVANCED)

Sol: To prove the above relation, we need to take the ratio of second order time derivative of wave equation and
second order displacement derivative of wave equation.

∂2 y ∂2 y
= − ω2asin(ωt − kx) and = k 2asin(ωt − kx). We can write these two equation as,
∂ t2 ∂ x2

∂2 y ω2 ∂ 2 y ∂2 y ∂2 y
= . . Comparing this with, = ν2
∂ t2 k 2 ∂ x2 ∂ t2 ∂ x2
ω
We get, wave speed ν =
k
The negative sign between ωt and kx implies that wave is traveling along positive direction.

Illustration 6: The Fig 11.8 shows a snapshot of a sinusoidal traveling wave which Y
was taken at t = 0.3 s. The wavelength is 7.5 cm and the amplitude is 2 cm. Assuming
the crest was at x = 0 at t = 0, write the equation of traveling wave. t=0.3s
 (JEE ADVANCED) 2 cm
x
}

Sol: The equation of travelling wave


= is y A sin(kx − ωt) . The wave number is given 1.2 cm

by k = and angular frequency of wave is ω=vk.
λ
2π Figure 11.8
Given, A = 2 cm, λ = 7.5 cm ∴ =
k = 0.84 cm−1
λ
1.2
The wave has traveled a distance of 1.2 cm in 0.3 s. Hence, speed of the wave=
ν = 4 cm / s
0.3
∴ Angular frequency ω = (v) (k) = 3.36 rad/s
Since the wave is traveling along the positive direction x, and crest (maximum displacement) is at x = 0 at t = 0, we
can write the wave equation as
1 1 . 8 | Waves on a String

Y(x, t) = A cos (kx - ωt) or y(x, t) = A cos (ωt - kx) as cos(-θ) = cos θ
Therefore, the equation of the traveling wave is
y(x, t) = (2 cm) cos [(0.84 cm-1)x – (3.36 rad/s)t]

Illustration 7: The mass and length of a rope hanging from the ceiling are 0.1 kg and 2.45 m,
respectively. The rope has a uniform width.
(a) Determine the speed of transverse wave in the rope at a point 0.5 m away from the lower
end.
l
(b) Also, calculate the time taken by the wave to travel the full length of the rope.
 (JEE ADVANCED)
x
Sol: As the rope hangs under it’s own weight, the tension in string at a distance x from
x
hanging end is T = mg where  is the length of the string and m is mass of the string.

When a transverse waves are generated to travel along length of rope, they travel with speed Figure 11.9
T
v= where µ is mass per unit length of string.
µ
The tension in the string will be different at different points owing to the mass of the string and the fact that it is
suspended vertically from a ceiling.. The tension at a point which is at a distance x free end will be due to the weight
x
of the string below it. Given that m is the mass of string of length l, the mass of length x of the string will be   m

0.1 x  0.5  T
=
µ = 0.04kg / m; Tension = mg  =  mg  = 0.20N ⇒ = v = 2.236m / s
2.45 
   2.45  µ

(b) From the above equation, we see that velocity of the wave is different at different points. Therefore, if at point
dx dx
x the wave travels a distance dx in time dt, then dt
= =
v gx
1 1 dx  2.45
∴ ∫0 gx ; =t
∫0 dt = 2= 2 = 1.0s
g 9.8

Illustration 8: The mass and length of a rope hanging vertically from a rigid support are 12 m and 6 kg, respectively.
A stone of mass 2 kg is attached to the free end of the rope. The rope has a uniform width. If a transverse pulse
of wavelength 0.06 m is produced at the lower end of the rope, what will be the wavelength of the pulse when it
reaches the top of the rope?  (JEE ADVANCED)
F
Sol: The wave velocity will be V = νλ = where F is the tension in rope at a point and µ is mass
µ
per unit length of the string. As F is varying along the length of the rope so the velocity will vary 6 kg
along the length of the rope. As source frequency is constant λ will vary.
Owing to the fact that a stone is attached to the lower end of the rope, the tension in the rope will
be different at the different points. The tension at the lower end will be 20 N and at the upper end 2 kg
it will be 80 N.
Figure 11.10
F F
We have, V = νλ or, = νλ         or, =ν µ.
µ λ
P hysi cs | 11.9

The frequency of the wave pulse is affected only by the frequency of the source, and hence the wave pulse
frequency will be the same across the length of the rope as it depends only on the frequency of the source. As the
rope has a uniform width, the mass per unit length will also be consistent across the length of the rope.
F
Thus, by (i) is constant.
λ

(2kg)g (8kg)g
Hence, = where λ1 is the wavelength at the top of the rope. This gives λ1 = 0.12m
0.06m λ1

6. POWER TRANSMITTED ALONG THE STRING BY A SINE WAVE


The direction of a traveling wave on a string and the direction of the Y
energy transmitted by it is the same. Consider a sine wave traveling along
a stretched string in the direction x. The equation for the displacement in
the direction y is y = A sin ω (t -x/v) … (i) F
The Fig 11.11 the portion of the string to the left of the point x exerts a
force F on the portion of the string to the right of the point x at time t. The 
X
direction of this force is along the tangent to the string at position x. The X
∂y Figure 11.11
component of the force along the axis y is Fy = − Fsin θ ≈ − F tan θ = − F
∂x
 ∂y  ∂y
The power delivered by the force F to the string on the right of position x is, therefore, P =  − F 
 ∂x  ∂t
 ω   ω2 A 2F
By (i), it is − F  −  A cos ω ( t − x / v) [ω A cos ω=
(t − x / v)] cos2 ω (t − x / v)
 v   v

This is the rate at which energy is being transferred from left portion of the string to the right portion across the
point at x. The cos2 term oscillates between 0 and 1 during cycle and its average value is 1/2, therefore, the average
power transmitted across any point is

1 ω2 A2F
Pav = = 2π2 µ c A2 v 2  … (ii)
2 v

The power passing along the length of the string is proportional to the square of the amplitude and square of the
frequency of the wave.

Illustration 9: For a sine wave with an amplitude of 2.0 mm, the average power transmitted through a given point
on a string is 0.20 W. What will be the power that will be transmitted through this point were the amplitude to be
increased to 3.0 mm?. (JEE ADVANCED)

Sol: The power transmitted by the sine wave is P ∝ A2 where A is the amplitude of the wave.
Other things being equal, the power transmitted is proportional to the square of the amplitude.

P2 A22 P2 9
Thus, = or = 2.25
= P2 =×
2.25 0.20 W =
0.45 W
P1 A12 0.20 W 4
1 1 . 1 0 | Waves on a String

7. ENERGY IN WAVE MOTION


Every wave motion involves transfer of energy and momentum.. Waves are produced when force is applied to a
portion of the wave medium. When force is applied to a portion of the wave medium, the disturbance thus caused
in that portion of the medium generates a wave that exerts a force on the adjoining portions. This, in turn, disturbs
those portions, thereby propagating the wave further to the adjacent portions. In this way, a wave can transport
energy from one region of space to other.
The energy in wave motion is manifested in three forms, namely, energy density (u), power (P), and intensity (l). We
shall discuss them one by one.

7.1 Energy Density ( µ )


The energy density of a progressive wave is the total mechanical energy (kinetic + potential) per unit volume of
the medium through which the wave is propagated. This can be illustrated through an example. Let us imagine a
string attached to a tuning fork. When the tuning fork is struck, the vibration transmits energy to the segment of
the string attached to it, or in other words, as the vibrating fork moves through its equilibrium position, it stretches
a segment of the string, increasing its potential energy, while also imparting transverse speed to the segment,
increasing its kinetic energy. Thus, as the wave moves along the string, energy is transferred to the other segments
of the string.

7.2 Kinetic Energy Per Unit Volume


The kinetic energy of a unit volume of the string can be calculated from the wave function. Mass of unit volume is
the density ρ. Its displacement from equilibrium is the wave function
y = A sin (k x – ω t).
dy
Its speed is , where x is considered to be fixed. The kinetic energy of unit volume ΔK is then
dt
2
1 1  dy  dy
∆K = ( ∆m)v 2y = ρ   ; Using y = A sin (k x – w t), we obtain =− w A cos(kx − wt)
2 2  dt  dt
1
So the kinetic energy per unit volume is ∆K= ρ2 ω2 A2 cos 2 (kx − ωt)  … (i)
2

7.3 Potential Energy Per Unit Volume


The work done by the vibrating fork by stretching the segment of the string is the potential energy of the segment.
dy
It depends on the slope . The potential energy per unit volume of the string is related to the slope and tension
dx
T and is given by (for small slopes)
2
1  dy 
∆U= pν2    … (ii)
2  dx 
ω
where v = wave speed =
k
dy 1
Using = k A cos (k x – ω t), we obtain for the potential energy ∆U= ρω2 A2 cos2 (kx − ωt)  … (iii)
dx 2
which is the same as the kinetic energy. The total energy per unit volume is ∆E = ∆K + ∆ U = ρω2 A2 cos2 (kx − ωt) … (iv)
1
The total energy per unit volume (ΔE) varies with time. As the average value of cos2 (kx − ωt) at any point is , the
2
1
average energy per unit volume (also called the energy density μ) is µ = ρω2 A2  … (v)
2
P hysi cs | 11.11

PLANCESS CONCEPTS

In the case of a spring with mass ρ attached to it and y


oscillating in a simple harmonic wave, the energy A
density is the same as in equation (v). However, its
potential energy is maximum when the displacement is
maximum. In the case of a string segment, it is the slope x
of the spring that determines the potential energy and it B
is maximum when the slope is maximum, which is at the
equilibrium position of the segment – the same position
for which the kinetic energy is maximum.
Figure 11.12
In the Fig 11.12, the kinetic energy and potential energy
both are zero at point A, whereas at point B, both the kinetic energy and potential energy are maximum.
Aman Gour (JEE 2012, AIR 230)

7.4 Intensity (I)


The intensity of a wave is defined as the flow of energy per unit area of a cross-section of the string in unit time.

power P 1 2 2
Thus, I= = or I= ρω A V
area of cross − sec tion s 2

This is, however, the average intensity transmitted through the string. The instantaneous intensity
ρ ω2 A2 v sin2 (kx − ωt) or ρ ω2 A2 v cos2 (kx − ωt) depends on x and t.

PLANCESS CONCEPTS

•• The relation for power and intensity discussed above are for transverse waves on a string. However,
they hold good for other waves also.
•• Intensity due to a point source: Assuming that waves are propagated uniformly in all directions, the
energy at a distance r from a point source is distributed uniformly on a spherical surface of radius
r and area S= 4πr2. If P is the power per unit area that is incident perpendicular to the direction of
P P
propagation, then intensity I = or I ∝
4π r 2
r2
Since amplitude A ∝ √I, a spherical harmonic wave emanating from a point source can therefore,
A
be written as = y(r,t) sin (kr − ωt)
r
T P Varun (JEE 2012, AIR 64)

Illustration 10: An oscillator attached to stretched string with a diameter of 4 mm transmits transverse waves
through the length of the string. The amplitude and frequency of the oscillation are 10−4 m and 10 Hz, respectively.
Tension in the string is 100 N, mass density of wire is 4.2 × 103 kg/m3.
Find: (a) The wave equation along the string
(b) The energy per unit volume of the wave
1 1 . 1 2 | Waves on a String

(c) The average energy flow per unit time across any section of the string
(d) The power required to drive the oscillator. (JEE ADVANCED)

Sol: The wave equation of string= is y A sin(kx − ω t) where the wave number k = , the angular frequency
λ

ω= 2πν= . λ is the wavelength and T is the time period of wave. As the string is under tension of 100 N, the
T
T
wave velocity on string is given by V = νλ = . Use the formula for wave energy in the string .
µ
T
(a) Speed of transverse wave on the string is, V = (∵ µ = ρS)
ρS
Substituting the values, we have

100 rad rad


v= = 43.53 m / s; ω = 2πf = 20 π = 62.83
π s s
(4.2 ×103 )   (4.0 ×10−3 )2
4
 
ω
Wave number is =
k = 1.44 m−1
V
  rad  
∴ The wave equation is =y (x,t) A sin(kx= − ωt) (10 −4 m)sin (1.44m−1 )x −  62.83  t
  s  
1
(b) Energy per unit volume of the string, u = energy density = ρ ω2 A2
2
 1
Substituting the values, we have
= u   (4.2 × 103 )(62.83)2 (10−4 )2 = 8.29×10-2 J/m3
2
1 
(c) Average energy flow per unit time, P
= power=  ρω2 A2  (sv)
= (u)(sv)
 2 
π
(8.29 ×10−2 )   (4.0 × 10−3 )2 (43.53) =
Substituting the values, we have P = 4.53 ×10−5 J / s
4
 
(d) Power required to drive the oscillator is obviously 4.53 × 10-5 W.

8. INTERFERENCE
Interference is a phenomenon that occurs
when two waves superimpose while Resultant wave
traveling in the same medium. This results
in the formation of a wave of greater or
Wave 1
lower amplitude. Interference happens
with waves that emerge from the same Wave 2
source or have the similar frequencies. Constructive interference Destructive interference
Figure 11.13
8.1 Principle of Superposition
The principle of superposition of waves states that when two or more waves of same type come together at a
single point in space, the total displacement at that point is equal to the sum of the displacements of the individual
waves. Constructive interference is the meeting of two waves of equal frequency and phase, i.e., if the crest of a
wave meets a crest of another wave of the same frequency at the same point, then the total displacement is the
sum of the individual displacements. Destructive interference is the meeting of two waves of equal frequency and
opposite phase, i.e., if the crest of one wave meets a trough of another wave then the total displacement is equal
to the difference in the individual displacements.
P hysi cs | 11.13

In constructive interference, the phase difference between the waves is a multiple of 2π, whereas in a destructive
interference the difference is an odd multiple of π. If the phase difference is between these two extremes, then
the total displacement of the summed waves lies between the minimum and maximum values. If the first wave
alone were traveling, the displacement of particles may be written as y1=f1 (t – x / v). If the second wave alone were
traveling, the displacement may be written as y2=f2 (t + x / v)

If both the waves are traveling on the string, the displacement of its different particles will be given by
y = y1 + y2 = f1 (t – x / v) + f2 (t + x / v).
If the two individual displacements are in opposite directions, the magnitude of the resulting displacement may be
smaller than the magnitudes of the individual displacements. In a nutshell, when two or more waves pass through
a point at the same time, the disturbance at the point is the sum of the disturbances each wave would produce in
absence of the other wave(s).

8.2 Interference of Wave Going in Same Direction


Let us assume that two identical sources send sinusoidal waves of same angular
frequency ω in the positive direction x. It is also assumed that the wave velocity A
and consequentially, the wave number k is same for the two waves. One source
A2
may send the wave a little later than the other or the two sources may be located
at different points. Here, the phases of the two waves at the point of interference  
will be different. If we assume the amplitudes of the two waves to be A1 and A2 A1
and the phase difference of the two waves to be an angle δ, their equations may
Figure 11.14
be written as
=y1 A1 sin(kx − ωt) And
= y 2 A2 sin(kx − ωt + δ)
According to the principle of superposition, the resultant wave is represented by
y y1 + y=
= 2 A1 sin(kx − ωt ) + A2 sin(kx − ωt + δ)
= A1 sin(kx − ωt) + A2 sin(kx − ωt) cos δ + A2 cos(kx − ωt) sin δ
= sin(kx − ωt)(A1 + A2 cos δ) + cos(kx − ωt)(A2 sin δ)
We can evaluate it using the method to combine two simple harmonic motions. If we write
A1 + A2 cosδ = A cos ε  … (i)
And A2 sinδ = A sin ε  … (ii)
We get,
= y A[sin(kx − ωt)cos ε + cos(kx − ωt)sin
= ε] A sin(kx − ωt + ε )
Thus, the resultant is indeed a sine wave of amplitude A with a phase difference ε with the first wave. By (i) and (ii),
( A2 + A2cosδ ) ( A2sinδ )=
2 2
A2= A2cos2 ε + A2sin2 ε= + A12 + A22 + 2A1A2 cos δ

Or A= A12 + A22 + 2A1A2 cos δ  … (iii)

A sin ε A2 sin δ
Also =
tan ε =  … (iv)
A cos ε A 1 + A2 cos δ
These relations may be remembered by using the following geometrical model can be used to remember these
relations: draw a vector of length A1 to represent y1 = A1sin (kx - ωt) and another vector of length A2 at an angle δ
with the first one to represent y2 = A2 sin (kx – ωt + δ). The resultant vector then represents the resultant wave y=A
sin (kx – ωt + ε). The given Fig 11.14 shows the construction.

Illustration 11: The equations of two waves passing simultaneously through a string are given by y1 = A1sin k
(x – vt) and y2 = A2 sin k (x – vt + x0), where the wave number k = 6.28 cm-1 and x0 = 1.50 cm. The amplitudes for
A1 and A2 are 5.0 mm and 4.0 mm, respectively. Find the phase difference between the waves and the amplitude of
the resulting wave.  (JEE ADVANCED)
1 1 . 1 4 | Waves on a String

Sol: As there are two waves passing through the string simultaneously, the phase difference between the two

waves will be δ = kx0. And the resulting amplitude of the waves will be A = A12 + A22 + 2A1A2 cos δ .
The phase of the first wave is k (x -vt) and of the second is k (x- vt + x0).
The phase difference is, therefore, δ = kx0 = (6.28cm-1) (1.50 cm) = 2π × 1.5 = 3π
We can thus infer that this is a destructive interference. The amplitude of the resulting wave is given by |A1 – A2| =
(5.0 − 4.0) mm = 1.0mm.

9. BOUNDARY BEHAVIOUR
When a propagating wave reaches the end of the medium it encounters an obstacle or, maybe, another medium
through which it could travel. Here, the interface of the two media is referred to as the boundary and the behavior
of a wave/pulse at that boundary is described as its boundary behavior.

9.1 Fixed End Reflection


Let us consider an elastic string which is attached at one end to a pole on Fixed end reflection
a lab bench while the other end is will be held in the hand and stretched
in order to introduce pulses into the medium. The end of the elastic string
that is attached to the pole is immovable when a wave or pulse reaches
it. If a pulse is introduced at the hand-held end of the rope, it will travel
An elastic rope security to a can be used
through the string towards the fixed immovable end of the medium. This to study the behavior waves at a fixed end.
is called the incident pulse since it is incident (i.e., approaching toward) the
Figure 11.15
boundary with the pole. With the incident pulse reaches the boundary, two
things occur:
(a) Some of the energy transmitted by the pulse is reflected back towards the hand-held end of the rope. This is
known as the reflected pulse.
(b) That part of the energy that is transmitted to the pole causes the pole to vibrate.
As the vibrations of the pole are not Fixed end reflection
obvious, the energy transmitted to it is
not typically discussed. The emphasis
Incident Pulse
here will be on the reflected pulse. What
are the characteristics/properties of its
motion? Inverted reflected pulse
When seen from the fixed immovable
end, the reflected pulse is a mirror image
of incident pulse. That is, an upward
displaced pulse will be reflected and
Figure 11.16
returned as a downward displacement
pulse and vice-versa.

9.2 Free End Reflection


Continuing with the above example, let us consider the situation
where instead of being securely attached to a lab pole, the elastic
string is attached to a ring that is fixed loosely around the pole.
Since the string is no longer attached firmly to the pole, the last
particle of the rope will be able to move when a pulse reaches it. if the end of an elastic roe not fastened to the pole
then it will be free move up and down. This
Now, if a pulse is introduced at the hand-held end of the string, provides for the study of waves behavior at free ends.
it will travel through the string towards the pole at the right end
of the medium. However, the string is no longer fixed tightly to Figure 11.17
P hysi cs | 11.15

the pole and, therefore, the string and Incident Pulse Reflection Pulse
the pole will slide past each other. There
will be no interaction between the string
particle and the pole particle. In other
words, when the last particle in the string
is displaced upwards, there will be no
adjoining pole particle to pull it down. As
a result, the upward displacement of the
Figure 11.18
incident pulse is not reversed the in the
reflected pulse. Similarly, if the incident
pulse has a downward displacement the reflected pulse will also demonstrate a downward displacement. . Inversion
is not observed in free end reflection.

10. REFLECTION AND TRANSMISSION OF A PULSE ACROSS A BOUNDARY

10.1 Reflection and Transmission of a Pulse across a Boundary from Less to More
Dense
A pulse exhibits two behaviors upon reaching the boundary.
(a) A part of the energy transmitted by the incident More dense
pulse is reflected and returns towards the hand-
held end of a thin string. The pulse that returns
to the hand-held end after bouncing off the
boundary is known as the reflected pulse.
Less dense
(b) A part of the energy transmitted by the incident More dense
pulse is transmitted into the thick string. The
disturbance that continues moving to the right
is known as the transmitted pulse. Reflected pulse Transmitted pulse
In a wave traveling from a less dense to a Figure 11.19
denser medium a part of the incident pulse
will be reflected off the boundary of the less dense string while another part will be transmitted across the
boundary of the thin string into the new medium (thick string). The pulse that moves into the new medium is
the transmitted pulse and is not inverted. The pulse that is reflected off the boundary of the thinner string is
called the reflected pulse is inverted.

10.2 Reflection and Transmission of a Pulse across a Boundary from More to Less
Dense
Here, the transmitted pulse moves through the less A wave travelling from a more dense to a less dense medium
dense string/medium, while the reflected pulse travels
More dense
through the denser string/medium. The transmitted Less dense
pulse travels faster and has larger wavelength than the
reflected pulse. However, the speed and wavelength
of the reflected pulse are same as the that of the Incident pulse
incident pulse.
Here, a part of the incident pulse will be reflected Less dense
More dense
off the boundary of the denser string/medium and a
part will transmitted across the boundary of the denser
string/medium into the less dense string/medium. Reflected pulse Transmitted pulse
There is no inversion, whatsoever. Figure 11.20
1 1 . 1 6 | Waves on a String

PLANCESS CONCEPTS
•• The wave speed and the wavelength are always greatest in the least dense string/medium.
•• The wave frequency remains constant even when crosses the boundary.
•• When moving from less dense string/medium to denser string/medium, the reflected pulse gets
inverted.
•• The amplitude of the incident pulse is always greater than that of the reflected pulse.
Anand K (JEE 2011, AIR 47)

11. STANDING WAVES


Standing wave, also called a stationary wave, is 
L
combination of two waves moving in opposite directions,
each having the same amplitude and frequency. . The
manner of this interference makes it appear as if some 1
points along the medium are standing still. For this L
reason, this wave pattern is referred to as the standing
wave pattern. Let us assume that two waves of equal
amplitude and frequency propagate towards each other 2
along a string. The equation of two waves are given by
=y1 A sin(ωt − kx) and= y 2 A sin(ωt + kx + δ) .
To understand these waves, let us discuss the special
case when δ = 0. 3
The displacements of the particles of the string
consequent to the interference are given by the principle
of superposition as y = y1 + y2 = A [sin (ωt-kx) + sin
(ωt+kx)= 2A sinωt cos kx or y = (2A cos kx) sin ωt … (xix) 4
Figure 11.21

11.1 Nodes and Antinodes


As discussed earlier, the manner of interference of standing  Nodes
wave patterns is such that there are points along the medium
that appear to be stationary. These points are referred to as
nodes or points of no displacement. There are other points
along the medium that undergo v the maximum displacement
during each vibrational cycle of the standing wave. These points
along the medium are called antinodes, as they represent the
other extreme in the standing wave pattern. A standing wave
pattern always has nodes and antinodes appearing alternatively
in them
Antinodes
Figure 11.22
P hysi cs | 11.17

PLANCESS CONCEPTS

Nodes and antinodes are quite different from crests and troughs. In a traveling wave, there points of
large upward and downward displacements, referred to as the crest and trough of the wave. However, an
antinode refers to a point of the string that remains stationary or appear to be stationary.
GV Abhinav (JEE 2012, AIR 329)

11.2 Differences Between Standing Waves and Traveling Waves

Standing Wave Traveling Wave


The disturbance produced in a region appears stationary. The disturbance produced in a region is transmitted with a
definite velocity
Different particles move with different amplitudes The motion of all particles are similar in nature

The particles at node always remain at rest There is no particle which always remains at rest

All particles cross their mean positions together At no point all the particles are at mean positions together

All the particles between two successive nodes reach their ex- The phases of nearby particles are always different
treme positions together, thus moving in phase.

The energy of one region is always confined to that region Energy is transmitted from one region of space to other

Node

t=0

t=T/8
t=T/4

t=3T/8

t=T/2

t=5T/8

t=3T/4

t=7T/8

t=T

Antinode

Figure 11.23

Illustration 12: The interference of two waves with equal amplitudes and frequencies traveling in opposite
directions produces a standing wave having the equation Y = A cos kx sin ωt in which
A = 1.0 mm, k = 1.57 cm-1 and ω = 78.5 s-1
(a) Find the velocity of the component traveling waves.
(b) Find the node closest to the origin in the region x > 0.
(c) Find the antinode closest to the origin in the region x > 0.
(d) Find the amplitude of the particle at x = 2.33 cm. (JEE ADVANCED)
1 1 . 1 8 | Waves on a String

Sol: Here the two waves of same amplitude and frequency interfere with each other to form the standing waves,
ω
the velocity of the resultant wave will be V
v = where ω is the angular frequency of the wave and k is the wave
k

number. The distance of the node from the origin is given by kx = . And distance of antinode from origin is
given by kx = nπ. 2

(a) The standing wave is formed by the superposition of the waves


A A ω 78.5 s−1
y1 sin(ωt − kx) and y 2 sin(ωt + kx) . The wave velocity (magnitude) of the waves is V
v= = = 50 cms−1
2 2 k 1.57 cm−1
(b) For a node, cos kx = 0
π π
The smallest position x satisfying this relation is given by kx = or, =
x = 1cm
2 2k
(c) For an antinode, |cos kx| = 1 or cos kx = ± 1
π 3.14
Kx = 0, π,2π,........nπ⇒ x ≠ 0, x min= = = 2cm
k 1.57
(d) The amplitude of vibration of the particle at x is given by |A cos kx|

Antinode Antinode Antinode Antinode

Node Node Node Node Node

Figure (b)
11.24

For the given point, kx = (1.57cm ) (2.33cm)


−1 7
= π=π+
6
π
6

3
Thus, the amplitude will be (1.0 mm) | cos( π + =
π / 6) | = mm 0.86mm
2

11.3 Standing Waves on a String Fixed at Both Ends (Qualitative Discussion)

Let us take the example of string fixed at both the ends –- one 
end to a wall and the other end tied to a tuning fork. The tuning
fork vibrates longitudinally with a small amplitude producing sine
waves of amplitude A which travel along the string towards the
wall. The said wave then gets reflected and travels toward the fork.
Figure 11.25
This wave, being reflected from a fixed end, will be an inverted
wave.. These waves are again hit the fork back and as the fork is
heavy and vibrates longitudinally with a small amplitude, it acts like a fixed end and the waves reflected from the
fork get inverted again. Therefore, the wave produced directly by the fork initially and the twice-reflected wave
have same shape, though the twice-reflected wave has already travelled a length 2L.
Let us assume that the length of the string is 2L=λ. The wave moving from the tuning fork to the wall and the
wave reflected back from the wall to the tuning fork interfere constructively and the resultant wave that proceeds
P hysi cs | 11.19

towards the wall has an amplitude 2A. This wave of amplitude 2A is again reflected back by the wall and then again
reflected by the fork. Now, this twice-reflected wave again interfaces constructively with the new incident wave
and a wave of amplitude 3A is produced. Thus, the amplitude keeps progressing. The string gets energy from the
vibrating and the amplitude builds up. Same arguments hold if 2L is any integral multiple of λ that is L=n λ/2, where
n is an integer.
However, in the above discussion, we have not factored in any loss of energy due to air viscosity or due to the
inflexibility of the string. In the steady state, waves of invariable amplitude will be present on the string from left
to right as well as from right to left. These opposing waves will produce standing waves on the string. Nodes and
antinodes will be formed along the string and there will be large amplitudes of vibration at the antinodes. We can
then say that the string is in resonance with the fork. The condition, L=n λ/2, for such a resonance may be stated
in a different way. We have from equation (9), υ  = ν λ or λ = υ / ν
The condition for resonance is, therefore,
λ nυ nυ n
L n
= or L
= or =
ν = F / µ  … (i)
2 2ν 2L 2L
The lowest frequency with which a standing wave can be set up in a string fixed at both the ends is thus
1
=
νο F / µ  … (ii)
2L
This is called the fundamental frequency of the string. All other possible frequencies of standing waves are integral
multiples of this fundamental frequency. Equation (xx) gives the natural frequencies, normal frequencies, or
resonant frequencies.

Illustration 13: Shown in the Fig 11.26 is a wire with a length of  40 cm


50 cm and a mass of 20 g. It supports a mass of 1.6 kg. Find the
fundamental frequency of the portion of the string between the
wall and the pulley.
10 cm
Take g=10ms-2.  (JEE ADVANCED)

Sol: The string is subjected to uniform tension due to weight 1.6 kg


of the block of mass 1.6 kg. The fundamental frequency of
Figure 11.26

the string between the fixed support and pulley is given by


1 F
νo = where µ is the mass per unit length of string.
2L µ
The tension in the string is F = (1.6kg) (10ms-2) = 16N.
20g
The linear mass density=
is µ = 0.04kgm−1
50cm

1 F 1 16N
The fundamental frequency is ν ο = = ==25hz
25 Hz
2L µ 2 × (0.4m) 0.04kgm−1

11.4 Analytical Treatment of Vibration of a String Fixed at Both Ends


Let us assume a string of length L which is kept fixed at the ends x= 0 and x= L. For certain wave frequencies,
standing waves are set up in the string. Due to the repeated reflection of the wave at the ends and the damping
effects, waves going in the positive direction x interfere to give a resultant wave
= y1 A sin(kx − ωt) . Similarly, the
waves going in the negative direction x interfere to give the resultant wave
= y 2 A sin(kx + ωt + δ) . As a result, the
displacement of the particle of the string at position x and at time t is given by the principle of superposition as
y= y1 + y 2= A sin(kx − ωt) + sin(kx + ωt + δ)
1 1 . 2 0 | Waves on a String

δ δ
= 2A sin(kx − ) cos( ωt + )  … (i)
2 2
If standing waves are formed, the ends x = 0 and x = L must be nodes because they are kept fixed. Thus, we have
the boundary conditions y= 0 at x=0 for all t and y=0 at x=l for all t.
δ
The first boundary condition is satisfied by equation (i) if sin =0, or δ = 0.
2
Equation (i) then becomes
= y 2A sinkx cos ωt  … (ii)
The second boundary condition will be satisfied if

sinkL =
0 or kL =
nπ, where n =
1, 2, 3, 4, 5,.......
2π L nλ
or nπ
= or L=  … (iii)
λ 2

If the length of the string is an integral multiple of λ /2, standing waves are produced.
υ nυ n
Again writing λ = υT = , equation (xxv) becomes =
ν = F/µ
v 2L 2L
υ 1
Which is same as equation (xx). The lowest possible frequency is ν ο = = F/µ  … (iv)
2L 2L
This is the fundamental frequency of the string. The other natural frequencies with which standing wave can be
formed on the string are
Length-Wavelength
No. of
Harmonic Pattern relationship
Loops

1st 1 L = 1/2 

2nd 2 L = 2/2 

3rd 3 L = 3/2 

4rd 4 L = 4/2 

5th 5 L = 5/2 

6th 6 L = 6/2 

Figure 11.27

2
ν1 = 2 ν0 = F/µ 1st overtone, or 2rd harmonic,
2L
3
ν2 = 3 ν0 = F/µ 2nd overtone, or 3rd harmonic,
2L
4
ν3 = 4 ν 0 = F/µ 3rd overtone, or 4rd harmonic,etc.
2L

In general, any integral multiple of the fundamental frequency is a valid frequency. These higher frequencies are
called overtones. Thus, ν1 = 2 ν0 is the first overtone, ν2 = 3 ν0 is the second overtone, etc. An integral multiple of
a frequency is called its harmonic. Thus, for a string fixed at both the ends, all the overtones are harmonics of the
fundamental frequency and vice-versa.
P hysi cs | 11.21

11.5 Vibration of a String Fixed at One End


If a string is set up in such a way that one end of it remains fixed while the other end is free to move in a transverse
direction, standing waves can be produced. The free end can be created by connecting the string to a very light
thread. If the vibrations of the “correct” frequency are produced by the source, standing waves are produced.
Assuming end x=0 is fixed and x = L is free, the equation is again= given by y 2A sinkx cos ωt which is the same as
equation (xxii), with the boundary condition that x = L is an antinode. The boundary condition that x = 0 is a node
is automatically satisfied by the above equation as it is fixed. For x = L to be an antinode, Sin kL = ± 1
1
 1 2πL  1  2L v 1  1  υ n+ 2
or kL = n +  π or = n+  π or n + or v =+
= n  = F/µ  … (i)
 2 λ  2 υ 2  2  2L 2L

These are the normal frequencies of vibration. The 


fundamental frequency is obtained when n = 0,
Fundamental
i. e., ν ο =υ / 4L N
A

The overtone frequency are ν1 = =3ν ο First
4L
N N Overtone
5υ 7υ
ν2 = =5ν ο , ν3 = =7ν ο , etc A A
4L 4L
Second
It can be seen that all the harmonics of the
Overtone
fundamental frequency are not the valid frequencies N
A
N A
N
A
for the standing waves. Only the odd harmonics are
the overtones. The string shapes for some of the Figure 11.28
normal modes are shown in Fig 11.28.

Illustration 14: A string is vibrating up and down as the fifth harmonic and completes 8.2 m
21 vibrational cycles in 5 seconds. The length of the string is 8.2 meters. Determine the
frequency, period, wavelength and speed for this wave. (JEE MAIN)
number of cycles produced
Sol: The frequency of the wave is f = . The time period of
total time
1
wave T = . When string is vibrating in fifth harmonics, then 2L = 5λ. The wave velocity Figure 11.29
f
is v = f λ.

Given: L = 8.2 m and 21 cycles in 5 seconds. The frequency here refers to the number of back-and-forth movements
of a point on the string and is measured as the number of cycles per unit of time. In this case, it is f = (21 cycles)/
(5 seconds) = 4.2 Hz
The period is the reciprocal of the frequency. T = 1/ (4.2 Hz) = 0.238 s.
The wavelength of the wave is correlated to the length of the rope. For the fifth harmonic as shown in the picture,
5
the length of the rope is equivalent to five halves of a wavelength. That is, L= λ where λ is the wavelength.
Rearranging and substituting the equation gives the following results: 2

=λ (=
2 / 5 ) × L ( 2 / 5 ) × (8.2 m) =
3.28 m
The wavelength and frequency wave can be used to calculate the speed of a wave using the wave equation
V = f λ = (4.2Hz).(3.28m) = 13.8m / s
1 1 . 2 2 | Waves on a String

12. LAWS OF TRANSVERSE VIBRATIONS OF A STRING


For a string fixed at both ends, the fundamental frequency of vibration is given by equation (ix). The statements
known as “Laws of transverse vibrations of a strings” can be derived from equation (ix).

12.1 Law of Length


Tension and mass per unit length of the string remaining the same, the fundamental frequency of vibration of a
string (fixed at both ends) is inversely proportional to the length of the string.
ν ∝ 1/L if F and µ are constants.

12.2 Law of Tension


The length and the mass per unit length of the string remaining the same, the fundamental frequency of a string
is proportional to the square root of its tension. ν ∝ F ifL Lif and
F and
µµ are constants.
arecons tants.

12.3 Law of mass


The length and the tension remain the same, the fundamental frequency of a string is inversely proportional to the
square root of the linear mass density, i.e., mass per unit length.
1
ν∝ if L and F arecons tants.
µ
These above laws can be experimentally studied with an apparatus called sonometer.

12.4 Sonometer
A sonometer is an apparatus that is used to study 
the transverse vibrations of strings. It is also called A C1 C C2 B
the monochord because it often has only one
string. It consists of a rectangular wooden box
with two fixed bridges near the ends, with a pulley D1 D D2
fixed at one end. A string is fixed at one end, which
is then run over the bridges and the pulley, and
then attached to a weight holder hanging below
the pulley. Additional weights can be added to the
holder to increase the tension in the wire. A third,
movable bridge, can be placed under the string to
change the length of the vibrating section of the H
string. This device demonstrates the relationship
between the frequency of the sound produced
when a string is plucked and the tension, length, Figure 11.30
and mass per unit length of the string. These
relationships are referred to as Mersenne’s law after Marin Mersenne (1588–1648), who studied and formulated
them. For small amplitude vibration, the frequency is proportional to:
(a) The square root of the tension of the string
(b) The reciprocal of the square root of the linear density of the string,
(c) The reciprocal of the length of wire of sonometer
P hysi cs | 11.23

Illustration 15: Resonance is obtained in a sonometer experiment when the experimental wire with a length of
21 cm between the bridges is excited by a tuning fork of frequency 256 Hz. If a tuning fork of frequency 384 Hz is
used, what should be the length of the experimental wire to get the resonance? (JEE MAIN)
1 ν2
Sol: For sonometer wire the ratio of lengths of vibrating string is = .
2 ν1
ν 256
By the law of length, 1ν1 =  2 ν2 or  2 = 1 1 = × 21cm =14 cm
ν2 384

13. TRANSVERSE AND LONGITUDINAL WAVES


When there is a disturbance at the source in a string, it causes displacement of the particles of the string. The
direction of such displacements is perpendicular to the direction of the propagation of the wave. Such waves are
called transverse waves. The wave on a string is a transverse wave.. Light waves are also an example of transverse
waves. Here, the value of the electric field changes with space and time and the changes are propagated in space.
The direction of the electric field is perpendicular to the direction of propagation of light when light travels in free
space.
Sound waves are an example of non-transverse waves. The particles of the medium are carried along the direction
of propagation of sound. We shall study in some detail the mechanism of sound waves in the next chapter. If the
displacement produced by the passing wave is along the direction of the wave propagation, the wave is called a
longitudinal wave. Sound waves are longitudinal.

13.1 Compression and Rarefaction


A longitudinal wave consists of compressions and rarefactions. Those regions in a longitudinal wave where particles
are clustered together are compressions. Conversely, those regions where the particles are furthest apart are called
rarefactions.
Compressions

Figure 11.31

Illustration 16: A sonometer wire has a length of 100 cm and a fundamental frequency of 330 Hz. Find
(a) The velocity of propagation of transverse waves along the wire and
(b) The wavelength of the resulting sound in air if velocity of sound in air is 330 m/s. (JEE ADVANCED)

Sol: When sonometer wire is set to vibrate in its fundamental frequency, then wavelength is λ=2L, the wave velocity
is v = f λ where f is the frequency of oscillation.
(a) In case of transverse vibration of string for fundamental mode:
L = (λ / 2), i.e., λ = 2L = 2 × l = 2m
i.e., the wavelength of transverse wave propagation on string is 2 m. Since the frequency of the wire is given to be
330 Hz, so from υ = f λ , the velocity of transverse waves along the wire will be
Vwire= 330 × 2= 660m / s
i.e., for transverse mechanical waves propagation along the wire, Hz,m and m/s
(b) Here vibration wire will act as source and produce sound, i.e., longitudinal waves in air. Now as frequency
does not change with change in medium so Hz and as velocity in air is given to be = 330 m/s so from υ = f λ ;
1 1 . 2 4 | Waves on a String

λair (Vair=
= / f ) (330 / =
330) 1m
i. e., for sound (longitudinal mechanical waves) in air produced by vibration of wire (body),
f = 330Hz, λ=2m and v =330m/s

14. POLARIZATION OF WAVES


Let us assume that we have a cardboard 
Y
with a slit in it through which a stretched
Y
string is passed such that the card is
placed in a perpendicular position to the
string. (See Fig 11.32). If we take the string X X
as the X axis, the cardboard will be in Y-Z Z
Z
plane. Now we generate a wave along the
X axis such that the particles of the string Figure 11.32
are displaced in direction Y as the wave
passes. If the slit in the cardboard is also aligned along the Y axis, the portion of the string in the slit can vibrate
freely in the slit and the wave will pass through the slit. Now, if we turn the cardboard by 900 in its plane, the slit
will be aligned along the Z axis. As the wave reaches the slit, the portion of the string in the slit tries to move along
the Y axis but the narrow slit on the cardboard becomes an obstruction. Consequentially, the wave is not able to
pass through the slit. However, if the slit is inclined to the Y axis at a different other angle, only a part of the wave
is transmitted and in the transmitted wave the disturbance is produced parallel to the slit. The same experiment
can be conducted with two chairs as shown in the Fig 11.33. If the displacement produced is always along a fixed
direction, then the wave is said to be linearly polarized in that direction. The examples considered in this chapter
are linearly polarized in y direction. By the same token, a wave that produces a displacement along the z direction,
is a linearly polarized wave, polarized in z-direction. Its equation is given by = z A sin ω(t − x / υ) .Linearly polarized
waves are referred to as plane polarized. In the event that each particle of a string moves in a small circle when
the wave is propagated, the wave is called circularly polarized. If each particle goes in ellipse, the wave is called
polarized. If the particles are move randomly in the plane perpendicular to the direction of propagation, the wave
is called un-polarized.

Figure 11.33
P hysi cs | 11.25

PROBLEM-SOLVING TACTICS
 1. Understanding and remembering all formulae is the key to solving problems in these sections. If the relation
between the given quantities and the questions asked is known, it will be easy to solve most of the problems.
All the quantities discussed in this topic are in some sense related to each other.
 2. The concept of reflection (of waves) can be encapsulated in a single point: “Inversion- Reflected wave will
invert only when it encounters a denser medium. And transmitted wave will never invert.” If this much is clear,
one can easily identify the case in every question.
 3. Waves must always be understood in the context of transfer of energy rather than as just some function of x
and t for better understanding of physics.
 4. For questions pertaining to the derivation of the wave equation, one can begin easily with only the x part and
subsequently add or subtract vt from x depending on the direction of velocity.
 5. Most questions related to velocity and energy appear complicated due to the introduction of the usual
Newton mechanics. This should, however, be treated just as some additional information to calculate tension
in the string (e.g., Pulley systems).

FORMULAE SHEET

S. No Term Description

1 Wave It is a disturbance or variation traveling through a medium due to the repeated


undulating motion of particles of the medium through their equilibrium
position. Examples are sound waves travelling through an intervening
medium, water waves etc.

2 Mechanical waves Waves that are propagated through a material medium are called MECHANICAL
WAVES. These are governed by Newton’s Law of Motion. Sound waves are
mechanical waves propagated through the atmosphere from a source to the
listener and it requires a medium for its propagation.

3 Non mechanical Waves which are not propagated through a material medium. Eg: light waves,
waves EM waves.

4 Transverse wave These are waves in which the displacements or oscillations are perpendicular
to the direction of propagation of the wave.

5 Longitudinal wave Longitudinal wave waves in which the displacement or oscillations in medium
are parallel to the direction of propagation of wave. Example: sound waves

6 Equation of harmonic At any time t, displacement y of the particle from its equilibrium position as
wave a function of the coordinate x of the particle is y(x,=
t) A sin(ωt − kx) where,
A is the amplitude of the wave, K- is the wave number
ω is angular frequency of the wave and (ωt-kx) is the phase

7 Wave number Wavelength λ and wave number k are related by the relation k = 2 π / λ
1 1 . 2 6 | Waves on a String

8 Frequency Time period T and frequency f of the wave are related to ω by ω/2 π = f = 1/T

9 Speed of wave Speed of the wave is given by v = ω/k = λ/T = λf

10 Speed of a transverse The tension and the linear mass density of a stretched string, and not the
wave T
frequency, determines the speed of a transverse wave i.e., v=
µ
T = Tension in the string
μ = Linear mass density of the string.

11 Speed of longitudinal Speed of longitudinal waves in a medium is given by


waves B
v= ; B = bulk modulus; ρ = Density of the medium speed of longitudinal
ρ
waves in ideal gas is

γP
v= P = Pressure of the gas,
ρ
ρ = Density of the gas and γ = CP / CV

12 Principle of It states that when two or more waves of same type come together at a single
superposition point in space, the total displacement at that point is equal to the sum of the
displacements of the individual waves. It is given by y = ∑ y i (x,t)

13 Interference of waves Two sinusoidal waves traveling in the same direction interfere to produce a
resultant sinusoidal wave traveling in that direction if they have the same
amplitude and frequency, with resultant wave given by the relation
=y '(x,t) [2 A m cos(u / 2)]sin(ωt − kx + u / 2) where u is the phase difference
between two waves.
If u = 0, then interference would be fully constructive.
If u = π , then waves would be out of phase and the interference would be
destructive.

14 Reflection of waves An incident wave encountering a boundary gets reflected. If an incident wave
is represented by
= A sin(ωt − kx) then reflected wave at rigid boundary is
y i (x,t)
yr (x,t) = A sin(ωt + kx + π) =− A sin (ωt + kx)
And for reflections at open boundary, the reflected wave is given by yr (x,t)
= A sin(ωt + kx)

15 Standing waves When two identical waves moving in opposite directions meet, the interference
produces standing waves. The particle displacement in standing wave is given
by y(x,t) [2 A sin(kx)]sin(ωt) . The amplitude of standing waves is different
=
at different point i.e., at nodes amplitude is zero and at antinodes amplitude
is maximum or equal to sum of amplitudes of constituting waves.
P hysi cs | 11.27

16 Normal modes of Frequency of transverse waves in a stretched string of length L and fixed at
stretched string both the ends is given by
f = nv /2L where n = 1, 2, 3……..
The above relation gives a set of frequencies called normal modes of oscillation
of the system. Mode n=1 is called the fundamental mode with frequency
f1= v/2L. Second harmonic is the oscillation mode with n = 2 and so on.
Thus the string has infinite number of possible frequency of vibration which
are harmonics of fundamental frequency f1 such that f n =nf1.

Solved Examples

JEE Main/Boards (d)

Example 1: The length of a wave propagated on a long


stretched string is taken as the positive x axis. The wave x=0
equation is given by
2
(e)
t x
− − 
T λ
y y=
0e
  where y
0 4mm, x=0 x = 20 cm
=T 1.0 =
s and λ 4cm.
Example 2: The dimensions of a uniform rope are
(a) Find the velocity of the wave.
as follows: length 12 m, mass 6 kg. The rope hangs
(b) Find the function finding the displacement of the vertically from a rigid support with a slab of a mass of 2
particle at x = 0. kg is attached to the free end of the rope. If a transverse
pulse of wavelength 0.06 m is transmitted from the free
(c) Find the function giving the shape of the string
end of the rope, what is the wavelength of the pulse
at = 0.
when it reaches the top of the rope?
(d) Plot the shape of the string at t =0.
(e) Plot the shape of the string at t=5s.
6 kg
Sol: The wave moves having natural frequency of ν
and wavelength λ has velocity V = νλ . As the frequency
1 λ 2 kg
is ν = the velocity of the wave is then V = .
T T F
Sol: The wave velocity will be V = νλ = where F is
(a) The wave equation may be written as µ
1 t − x 
2 the tension in string at a point and µ is mass per unit
y = y0e
− 
T  λ /t

 length of the string. As F is varying along the length of
the rope so the velocity will vary along the length of the
Comparing with the general equation we see that rope. As source frequency is constant λ will vary.
λ We have, V = ν λ
=
ν = 4 cm
= 4cm / sec
1.0 s
F F
(b) Putting x = 0 in the given equation f(t) = y 0 e−(t/T)
2
Or, =νλ or =ν µ
µ λ
… (i)

(c) Putting t = 0 in the given equation g(t) = y 0 e −(x/ λ )2 Since the frequency of the wave pulse is dependent
 … (ii) only on the frequency of the source, it will be consistent
1 1 . 2 8 | Waves on a String

across the length of the rope. The mass per unit length pattern disappears but another stationary wave pattern
will also be consistent for the entire rope as the rope is reappears at a frequency of 1600 Hz. Calculate
uniform. Thus,
(i) The speed of sound in air.
F
By (i) is constant. (ii) The distance between adjacent nodes at a frequency
λ of 1600 Hz,
(2k g)g (8k g)g
Hence, = (iii) The distance between diaphragm and the closed
0.06 λ1
end
where λ1 is the wavelength at the top of the rope. This
gives λ1 = 0.12m. (iv) The next lower frequencies at which stationary wave
patterns will be obtained.

Example 3: A traveling wave pulse is given by Sol: The standing waves generated inside the tube
10 closed at one end, have the wavelength n λ =2L
y = . What is the direction, velocity and
5 + (x + 2t)2 where L is length of the tube. The velocity of the wave
amplitude of the pulse? in air is given by v = fλ, where n is the frequency of the
sound wave.
Sol: The wave equation given above is of form Since the node-to node distance is λ/2, λ / 2 = 0.08
a or λ = 0.16m
y = where ‘a’ is the amplitude of the
b + (x  υt)2
(i) c = n λ; ∴ c= 2000 × 0.16 = 320ms-1
disturbance.
(ii) 320 = 1600 × λ / 2 or λ = 0.2m
A wave pulse is a disturbance confined to only in a
small part of the medium at a given instant [see figure] ∴ Distance between nodes = 0.2/2 = 0.1 m = 10cm.
and its shape does not change during propagation. It is (iii) Since there are nodes at the ends, the distance
a between the closed end and the membrane must be
usually expressed by the form y =
b + (x  υt)2 exact integrals of λ/2.
n 5
Comparing the above with the given pulse we find that ∴ 0.4 = n λ / 2 = n’ × 0.2/2 ⇒ =
n' 4
f(x  υt) = (x + 2t)2
When n = 5, n’ = 4 l = n × 0.16/2 = 0.4m = 40cm
y (iv) For the next lower frequency n = 3, 2, 1
t=0
a ∴ 0.4 = 3 λ / 2 or λ = 0.8/3
a= V
b 320
Since c=n λ, =
n = 1200Hz
0.8 / 3
Again 0.4 = 1. λ / 2 or λ = 0.4M
x ∴ n = 320/0.4 = 800 Hz
i.e, the pulse is traveling along negative x axis with Again 0.4 = 1. λ / 2 or λ = 0.8M
velocity 2 m/s.
∴ n = 320/0.8 = 400 Hz
Further, amplitude is the maximum value of wave
function which will be when (x + 2t)2 =
0
Example 5: Consider a tuning fork of frequency 256 Hz
10 and an open organ pipe of slightly lower frequency. Both
So, =
A ymax
= = 2
5 are at 17°C temperature. When sounded together, they
produce 4 beats per second. When the temperature of
Example 4: Consider a tube that is closed at one air in the pipe is altered, the number of beats per second
end and has a vibrating diaphragm at the other first diminishes to zero and then increases again to 4.
end. The diaphragm, which may be assumed to be Determine the quantum of temperature change in the
the displacement node, produces a stationary wave pipe? Also, in what direction has the temperature of the
pattern at the frequency of 2000 Hz, in which the air in the pipe been altered?
distance between adjacent nodes is 8 cm. When the
frequency is gradually reduced, the stationary wave
P hysi cs | 11.29

Sol: In an open organ pipe the frequency of the wave is So, the required frequency are 1100, 2200, 3300, 4400,
V and 5500 Hz
n = t where Vt is the velocity of wave at temperature
λ The frequency of the nth overtone is (n +1) n0.
t and λ=2L is the wavelength of the vibrating wave. If
∴ (n +1) n0 =20000 or (n+1)100 = 20000
temperature of air inside the organ pipe changes, the
Or n = 17.18
velocity of wave also changes, since V ∝ T .
V17 The acceptable value is 17.
=n = where l =lengthof
length ofthe
thepipe
pipe;
2l
V V17 Example 7: The displacement of a particle of a string
∴ = 256 − 17 4 = or 252 carrying a traveling wave is given by
2l 2l
=
Since beats decreases first and then increases to 4, the y (3.0 cm)sin6.28(0.50x − 50t),
frequency of the pipe increases. This can happen only if where x is in centimeter and t in second. Find (a) the
the temperature increases. amplitude, (b) the wavelength, (c) the frequency and
Let t be the final temperature, in Celsius, (d) the speed of the wave.

Vt Vt
Now =256 + 4 or = 260 Sol: In a open organ pipe the frequency of the wave is
2l 2l
V
V 260 273 + t 260 n = t where Vt is the velocity of wave at temperature
Dividing t
= = or λ
V17 252 273 + 17 252 t and λ=2L is the wavelength of the vibrating wave. If
(∴ V < T ) or
= t 308.7
= – 273 35.70 C. temperature of air inside the organ pipe changes, the
velocity of wave also changes, since V ∝ T .
∴ Rise in temperature =35.7 – 17 = 18.7 0C.

Example 6: Determine the fundamental frequency and On comparing with the standard wave equation
the first four overtones of a 15 cm pipe
x t
= y A sin(kx − ωt) = A sin2π ( − )
(a) If the pipe is closed at one ends, λ T
(b) If the pipe is open at both ends
we see that, Amplitude = A = 3.0 cm,
(c) How many overtones are within the human auditory 1
range in each of the above cases? Velocity of sound in Wavelength = λ= cm = 2.0 cm, and the frequency
0.50
air = 330 ms-1.
1
=v= = 50Hz
Sol: For the organ pipe closed at one end, the T
fundamental frequency of the wave of wavelength λ The speed of the wave is V = ν λ
v
is given by, n0 = .The frequency of ith over tone is = (50 s-1) (2.0cm) = 100 cm s-1
4L
given by ni = ( i + 1) × n0 where i=1,2,3…. etc.
Example 8: The equation for a wave traveling in the
V direction x on a string is
(a) n0 =
4l
y = (3.0 cm) sin [(3.14 cm-1) x – (314 s-1) t].
Wher n0 = frequency of the fundamental node
(a) Find the maximum velocity of a particle of the string.
330
⇒=n0 = 550Hz (b) Find the acceleration of a particle at x = 6.0 cm at
4 × 0.15
time t = 0.11 s
The first four overtones are 3n0, 5n0, 7n0 and 9n0
∂y
∴ So, the required frequencies are 550, 1650, 2750, Sol: The maximum velocity is v = While the
∂t
3850, and 4950 Hz. ∂v
acceleration a =
V 330 ∂t
(b) n= = = 1100Hz ∂y
0
2l 2 × 0.15 (a) The velocity of the particle at x at time t is v =
∂t
The first overtones are 2n0, 3n0, 4n0 and 5n0
1 1 . 3 0 | Waves on a String

(3.0 cm)( −314s−1 )cos[(3.14cm−1 )x − (314s −1)t]


= Where the value of v has been put from part (a).

( −9.4ms−1 )cos[((3.14cm−1 )x − (314s−1 )t]


= (c) The displacement of the particle at x = 50 cm at time
t = 0.05 s is by equation (ii),
The maximum velocity of a particle will be v = 9.4 ms–1.
π 
(b) The acceleration of the particle at x at time t is =y (1.0cm)cos[(40π s−1 )(0.05s) −  m−1  (0.5m)]
2 
∂v
a= = − (9.4ms−1 ) (314s-1) sin [(3.14cm-1) x – (314 s-1) t]  π  1.0
∂t = (1.0cm)cos 2π − =  = 0.71cm
 4  2
= - (2952ms ) sin [(3.14 ms ) x - (3.14 s )].
-2 -1 -1

(d) The velocity of the particle at position x at time t is,


The acceleration of the particle at x = 6.0 cm at time
by equation (ii),
t = 0.11 s = 1(2952ms-2) sin [6π - 11 π] = 0.
∂y  π  
υ= = − (1.0 cm)(40π s−1 )sin (40 π s−1 )t −  m−1  x 
Example 9: One end of a long string is attached to an ∂t   2  
oscillator moving in transverse direction at a frequency
of 20 Hz. The string has a cross-section area of 0.80 Putting the values of x and t,
mm2 and a density of 12.5 g cm-3 . It is subjected to a
tension of 64 N along the X axis. At t= 0, the source is  π  40π
υ = − (40π cms−1 )sin  2π −  = cms−1 ≈ 89cms−1
at a maximum displacement y = 1.0 cm. (a) Find the  2  2
speed of the wave traveling on the string. (b) Write the
equation for the wave. (c) What is the displacement of
Example 10: The speed of a transverse wave traveling
the particle of the string at x = 50 cm at time t = 0.05
through a wire is 80 m s-1. The length of the wire is 50
s? (d) What is the velocity of this particle at this instant?
cm, the mass is 5.0 g, the area of cross-section of the
wire is 1.0 mm2, and its Young modulus is 16 ×1011 Nm-2.
Sol: As the wave is under tension F, the maximum wave
Find the extension of the wire over its natural length.
F
velocity of wave is Vv = where µ is the mass per unit
µ Sol: The maximum velocity of the wave is = vυ F / µ
where F is the tension in the string and µ is mass per
length of the string. The wave equation is y=A cos (ωt)
unit length of string. And the Young’s modulus of the
where ωis angular frequency and A is amplitude of
wave. F/A
string is Y = .
∆L / L
(a) The mass of 1 m long part of the string is
The linear mass density is
m= (0.80mm2) × (1 m) × (12.5gcm-3)
= (0.80×10-6 m3) × (12.5×103kg m-3) = 0.01 kg 5 × 10 −3 kg
=
µ = 1.0 × 10 −2 kgm−1
The linear mass density is μ 0.01 kg m-1. The wave speed 50 × 10 −2 m

F 64N The wave speed is F/µ .


is=
v = = 80ms−1 = υ
µ 0.01kgm−1
Thus, the tension is
(b) The amplitude of the source is A = 1.0cm and
the frequency is =20 Hz. The angular frequency is F = µv2 = (1.0 × 10–2 kgm–1) × 6400 m2s–2 = 64 N
ω=2πv=40 π s-1. Also at t =0, the displacement is F/A
equal to its amplitude, i.e., at t=0, x = A. The equation The Young modulus is given by Y =
∆L / L
of motion of the source is, therefore, y = (1.0cm)
cos[(40 π s-1)t] … (i) The extension is, therefore,

The equation of the wave traveling on the string along the FL (64N)(0.50m)
∆L= = = 0.02mm
position X – axis is obtained by replacing t with t – x / v in AY (1.0 ×10 6 m2 ) × (16 × 1011Nm−2 )

equation (i). It is, therefore,


  x 
=y (1.0cm)cos (40πs−1 )  t −  
  v 
 x 
= (1.0 cm)cos (40πs−1 )t −  m−1   … (ii)
 2  
P hysi cs | 11.31

JEE Advanced/Boards Example 2: A tuning fork of frequency 500 Hz is used


to generate a transverse harmonic wave of amplitude
Example 1: The interference of two traveling waves 0.01 m at one end (x = 0) of a long, horizontal string. At
of equal amplitude and frequency moving in opposite a given instant of time the displacement of the particle
directions along a string produces a standing wave at x = 0.1 m is –0.005m and that of the particle at x =
having the equation 0.2 m is +0.005m. Calculate the wavelength and wave
velocity. Assuming that the wave is traveling along
=y A coskx sin ωt in which A = 1.0 mm, k=1.57cm-1 and the positive direction x and that the end x = 0 is at
ω = 78.5s-1. equilibrium position at t = 0, obtain the equation of
(a) Find the velocity of the component traveling waves. the wave.

(b) Find s the node closest to the origin in the region Sol: The fork is the source to generate the transverse
x > 0. wave on string whose frequency is also 500 Hz. The
(c) Find the antinode closest to the origin in the region equation of this wave is given= by y A sin (kx − ωt)
x>0 where k is the wave number and x is the displacement
of particle. The wave velocity is given by V = νλ where
(d) Find the amplitude of the particle at x= 2.33 cm. υ is the frequency of source

Sol: Here the two waves of same amplitude and Since the wave is traveling along positive direction x
frequency interfere with each other to form the and the displacement of the end x = 0 is at time t = 0,
standing waves, the velocity of the resultant wave will the general equation of this wave is
ω
be V = where ω is the angular frequency of the wave  2π 
K y(x,t) A sin  ( υt − x) 
= … (i)
 λ 
and K is the wave number. At the node the waves are
90o opposite in phase, so that the amplitude of resulting Where A = 0.01 m. When x = 0.1m, Y = -0.005m
wave is zero at the node.
 2π 
(a) The standing wave is formed by the superposition ∴ −0.005 0.01sin  ( υt − x1 )
=
λ 
of the waves
A A  2π  1
y1
= sin(ωt − kt) and=
y2 sin(ωt + kt) Where x1 = 0.1m or sin  ( υt − x1 ) =−
2 2  λ  2

The wave velocity (magnitude) of either of the waves is 2π 7π


phase sin φ=
∴∴Phase 1 ( υt − x1=)  … (ii)
ω 78.5s −1 λ 6
ν= = = 50cm / s .
k 1.57 cm−1 When x = 0.2m y= + 0.005. Therefore, we have +0.005
(b) For a node, cos kx = 0
 2π 
The smallest position value of x satisfying this relation = 0.01 sin  (V t − x2 )
λ 
π
is given by kx =
2 Where x2 = 0.2 m
π 3.14 2π 2π
Or =x = = 1cm ∴=
φ2 (V t=− x1 ) … (iii)
2k 2 × 1.57cm−1 λ 6 
(c) For an antinode, |coskx| = 1 From eqs. (ii) and (iii)
The smallest positive x satisfying this relation is given ∴ ∆φ = φ1 − φ2 = π
π
by kx = π or x= = 2cm 2π
k Now, ∆φ = − ∆x thus,
(d) The amplitude of vibration of the particle at x is λ
given by |Acoskx|. For the given point, 2π 2π
π = − (x1 − x2 ) = (0.1 − 0.2) or λ = 0.2m
7 π λ λ
−1
kx = (1.57 cm )(2.33cm) = π = π +
6 6 Now, frequency n of the wave = frequency of the tuning
Thus, the amplitude will be (1.0mm)| fork = 500 Hz. Hence, wave velocity

3 υ = nλ = 500 × 0.2 = 100ms−1


cos( π + π=
/ 6) | = mm 0.86mm
2
1 1 . 3 2 | Waves on a String

Substituting for A, λ, and v in equation. (i) We get ∂y


= 0.01sin{10π (100t − x)}
y (x,t) velocity of wave is v = and the distance of nodes
∂t
This is the equation of the wave where y and x are in from any fixed end of string found using relation
meters and t in seconds. π
kx =
2
Example 3: Two tuning forks A and B sounded together Comparing given equation with equation of standing
produce 6 beats per second. With the introduction of  2π x   2π vt 
an air resonance tube closed at one end, the two forks wave, y = 2acos   sin  
 λ   λ 
give resonance when the two air columns are 24 cm
and 25 cm, respectively. Calculate the frequencies of 2π π
= ; λ 30 cm;
= = a 2cm
forks. λ 15,
2π v
Sol: Beats are produced when the two waves of similar = 96 π ⇒ v = 1440 cm / sec
λ
amplitude but different frequency are interacting with
f  π× 5  4 3

each other. For vibrating air column 1 = 2 where  ( a) x = 5 cm, ymax = 4 sin  =  = 2 3 cm
 2 f1  15  2
is length of vibrating air column and f is frequency of
(b) As λ =30 cm; nodes are at 0, 15, 30, 45, 60 cm
tuning fork.
Let the frequency of the first fork be f1 and that of ∂y  πx 
(c) =− 4 sin   sin(96πr) × 96π
second be f2. ∂t  15 
v
We then have, f 1 = and f2 = v x = constant For x = 7.5 cm, t = 0.25 sec.
4 × 24 4 × 25
We also see that f1 > f2 ∂y  π × 7.5 
= − 4 sin   sin(96π× 0.25) × 96π = 0
∂t  15 
∴ f1 - f2 = 6  … (i)  πx 
(d) Component waves y =
− 4 sin   sin(96π t)
And
f1
=
25
 … (ii)
 15 
f2 24
 πx   πx 
= 2sin  + 96πt  + 2sin  − 96πt 
 15   15 
Solving (i) and (ii), we get f1 =150 Hz and f2 =144 Hz
 πx 
⇒ Component waves are = y1 2sin  + 96 πt  ;
 15 
Example 4: The oscillation of a string of length 60  πx 
=y 2 2sin  − 96 πt 
cm fixed at both ends is represented by the equation:
 15 
 πx 
y 4 sin   cos(96π t) where x and y are in cm and t
 15  Example 5: A uniform rope hangs vertically from a
is in seconds. rigid support with a slab of mass 2 kg attached to the
(a) What is the maximum displacement of a point at x free end of the rope. The rope has a length of 12 m and
= 5 cm? a mass of 6 kg. A transverse pulse of wavelength 0.06
m is produced at the free end of the rope. Determine
(b) Where are the nodes located along the string? the wavelength of the pulse when it reaches the top of
(c) What is the velocity of a particle at x = 7.5 cm & at the rope?
t = 0.25 sec.
(d) Write down the components waves which give the 6 kg
above wave on superposition.
2 kg
Sol: The wave equation oscillating string is written
 2πx   2πvt  F
in form of y = 2acos   sin   where x is Sol: The wave velocity will be V = νλ = where F is
 λ   λ  µ
displacement and v is velocity of wave. The maximum the tension in rope at a point and µ is mass per unit
length of the string. As F is varying along the length of
P hysi cs | 11.33

the rope so the velocity will vary along the length of the −0.5cm
y =+
y1 y 2 = − 0.35cm.
=
rope. As source frequency is constant λ will vary. 2
As the rope is stretched using a slab, its tension will
be different at different points along the length of the Example 7: The vibrations of a string fixed at both ends
rope. The tension at the free end will be (2 kg) g while are described by the equation
at the upper end it will be (8kg) g. y = (5.00mm) sin[(1.57cm−1 )x]sin[(314 s−1 )t]

λ ⇒ F / µ = vλ or
We have, vυ== vvλ F / λ= v µ  … (i) (a) What is the maximum displacement of the particle
at x =5.66cm?
Since the frequency of the wave pulse is dependent (b) What are the wavelengths and the wave speeds
only on the frequency of the source, it will be consistent of the two transverse waves that combine to give the
across the length of the rope. The mass per unit length above vibration?
will also be consistent for the entire rope as the rope is
uniform. Thus, by (c) What is the velocity of the particle at x = 5.66 cm at
time t = 2.00s?
F (2kg)g (8kg)g
(i) is cons tant . Hence, = , (d) If the length of the string is 10.0 cm, locate the
λ 0.06m λ1
nodes and the antinodes. How many loops are formed
Where λ1 is the wavelength at the top of the rope, this in the vibration?
gives λ1 =0.12m.

Sol: The transverse velocity of particle of string is


Example 6: Two waves passing through a region are
∂y
represented by u= . The wave velocity is V = νλ . Comparing wave
∂t
y (1.0 cm)sin[(3.14 cm−1 )x − (157s−1 )t] equation with y=A sin kx sin ωt, we get the amplitude A
and y (1.5cm)sin[(1.57 cm−1 )x − (314 s−1 )t] and angular frequency of the wave.

Find the displacement of the particle at x = 4.5 cm at


time t = 5.0 ms. (a) The amplitude of the vibration of the particle at
position x is
Sol: As the waves are superimposed on each other, the A = |(5.00 mm) sin[(1.57cm−1 )x]
resultant displacement is Y=y1+y2. For x = 5.66 cm,
According to the principle of superposition, each π 
= A |(5.00 mm) sin  × 5.66 
wave produces its own disturbance and the resultant  2 
disturbance is equal to the vector sum of the individual
 π
disturbances. The displacements of the particle at x= = (5.00 mm) sin  2.5π + 
 3
4.5cm at time t = 5.0 due to the two waves are,
π
y1 = (1.0 cm)sin[(3.14 cm−1 )(4.5cm) = (5.00 = mm) cos 2.50 mm
3
−1 −3
− (157s )](5.0 × 10 s)
(b) From the given equation, the wave number
 π  k = 1.57 cm-1 and the angular frequency ω = 314 s-1.
= (1.0cm)sin=  4.5π −  (1.0cm)sin[4 π + π / 4] Thus, the wavelength is
 4
2π 2 × 3.14
1.0 cm =λ = = 4.00 cm
= and k 1.57 cm−1
2 ω 314s−1
and Frequency is v
= = = 50 s−1
y 2 (1.5cm)sin[(1.57 cm−1 )(4.5cm) − (314s−1 )] 2π 2 × 3.14
×(5.0 × 10−3 s) The wave speed is

 π υ = νλ = (50s−1 ) (4.00 cm) = 2.00ms−1 .


= (1.5cm)sin =2.25π −  (1.5cm)sin[2π + π / 4]
 2 (c) The velocity of the particle at position x at time t is
π 1.5cm given by
− (1.5cm)sin =
= −
4 2 ∂y
=
υ = (5.00 mm) sin [(1.57 cm−1 )x] [314s−1
∂t
The net displacement is
× cos(314s−1 )t]
1 1 . 3 4 | Waves on a String

= (157 m s−1 ) in (1.57 cm−1 ) x cos(314 s−1 )t So that ν 1L1 = ν 2L2 = ν 3L3. ... (i)
As ν 1: ν 2: ν 3 = 1: 2: 3 we have
(d) The nodes occur where the amplitude is zero, i.e.,
π  ν 2 = 2 ν 1 and ν 3 = 3 ν 1 so that by (i)
sin (1.57cm-1) x = 0 or  cm−1  x = nπ
2  ν1 L1 ν1 L1
L2
= L1
= and =
ν3 L1
= and
Where n is an integer. Thus, x = 2n cm. ν2 2 ν3 3
The nodes, therefore, occur at x = 0, 2 cm, 4 cm, 6 cm, L1+L2+L3 = 1m
8cm and 10 cm. Antinodes occur in between them, i.e.,
at x = 1 cm, 3 cm, 5cm, 7 cm and 9 cm. The string  1 1
vibrates in 5 loops. We get L1 1 + +  =
1m
 2 3
6 L1 6
Example 8: A guitar of 90 cm length has a fundamental L=
1 m Thus, L=
2 = m
frequency of 124 Hz. Where should it be pressed to 11 2 11
produce a fundamental frequency of 186 Hz? ν1 2
L=
3 = m
3 11
Sol: As wires of guitar resemble the sonometer wire,
6
thus the fundamental frequency of the guitar wire fixed One bridge should be placed at m from one end
11
1 F 2
at both ends is ν = . And for two vibrating and the other should be placed at m from other end.
2L µ 11
 ν
strings, the ratio of their vibrating lengths is 1 = 2 .
 2 ν2 Example 10: A wire having a linear mass density 5.0
×10-3 kg m-1 resonates at a frequency of 420 Hz when it
The fundamental frequency of a string fixed at both
is stretched between two rigid supports with a tension
1 F of 450 N.. The next higher frequency at which the same
ends is given by ν =
2L µ wire resonates is 490 Hz. Find the length of the wire.
As F and μ are fixed,
Sol: For vibrating string the nth harmonic of fundamental
=ν11 LL=
ν 2
2 or LL2
ν
ν11 L
= = or L1 n F
ν L
ν22 L11 2 ν
ν2 1 frequency is f = . Here L is the length of vibrating
2 2L µ
124Hz
124Hz
=
= (90
(90 cm)
cm) 60
60 cm
cm string and F is the tension in the string. The two given
186
186 Hz
Hz frequencies correspond to two consecutive values n
Thus, the string should be pressed at 60 cm from an and (n+1).
end. Suppose the wire vibrates at 420 Hz in its nth harmonic
and at 490 Hz in its (n + 1)th harmonic.
Example 9: The total length of a sonometer wire is 1 m
−1 n
between the fixed ends. Where the two bridges should 420s
= F / µ  … (i)
2L
be placed in the sonometer so that the three segments
of the wire have their fundamental frequencies in the (n + 1)
490s−1
and= F / µ  … (ii)
ratio 1:2:3? 2L
1 490 (n + 1)
Sol: For sonometer the ratio of length of wires =
is L ∝ This gives = or n 6
ν 420 n
where v is the frequency of the wave and L is length of
Putting the value in (i),
vibrating string.
Suppose the lengths of the three segments are L1, L2, 6 450N 900
=
and L3, respectively. The fundamental frequencies are 420s−1 = ms−1
2L 5.0 ×10−3 kgm−1 L
1
=
ν1 F/µ
2L1 900
Or
= L = m 2.1m
1 1 420
=
ν2 F/µ = ; ν3 F/µ
2L2 2L3
P hysi cs | 11.35

JEE Main/Boards

Exercise 1 Q.10 What will be the speed of sound in a perfect rigid


rod?

Q.1 Audible frequencies have a range 40 hertz to 30,000


Q.11 What is the distance between compression and its
hertz. Explain this range in terms of
nearest rarefaction in a longitudinal wave?
(i) Period T
(ii) Wavelength λ in air, and Q.12 What is the distance between a node and an
adjoining antinode in a stationary wave?
(iii) Angular frequency
Give velocity of sound in air is 350 ms-1
Q.13 Explain why waves on strings are always transverse.

Q.2 From a radio station, the frequency of waves is 15


Q.14 What is a wave function? Give general form of
Mega cycle/sec. Calculate their wavelength.
wave function. What is a periodic function?

Q.3 The velocity of sound in air at N.T.P is 331 ms-1. Find


Q.15 Distinguish between harmonics and overtones.
its velocity when the temperature rises to 910C and its
pressure is doubled.
Q.16 A stone is dropped into a well in which water is
78.4 m deep. After how long will the sound of splash
Q.4 A displacement wave is represented by = 0.25 ×
be heard at the top? Take velocity of sound in air
10-3 sin (500 t – 0.025 x). Deduce (i) amplitude (ii) period
= 332 ms-1
(iii) angular frequency (iv) wavelength (v) amplitude of
particle velocity (vi) amplitude of particle acceleration. ,
the t and x are in cm, sec and meter respectively. Q. 17 From a cloud at an angle of 300 to the horizontal,
we hear the thunder clap 8s after seeing the lightening
flash. What is the height of the cloud above the ground
Q.5 The length of a sonometer wire between two fixed
if the velocity of sound in air is 330 m/s?
ends is 110 cm. Where should be two bridges be placed
so as to divide the wire into three segments, whose
fundamental frequencies are in the ratio 1: 2: 3? Q.18 A steel wire 0.72m long has a mass of 5.0 × 10-3 kg.
If the wire is under a tension of 60 N, what is the speed
of transverse wave on the wire?
Q.6 Calculate the velocity of sound in a gas, in which
two wave lengths 2.04 m and 2.08 m produce 20 beats
in 6 seconds. Q.19 For a metal, bulk modulus of elasticity is
7.5 × 1010 Nm-2, and density is 2.5 × 103 m-3. Deduce the
velocity of longitudinal waves.
Q.7 A tuning fork of unknown frequency gives 6 beats
per second with a tuning fork of frequency 256. It gives
same number of beats/ sec when loaded with wax. Find Q.20 A steel wire 70 cm long has mass of 7g. If the
the unknown frequency. wire is under a tension of 100 N, what is the speed of
transverse waves in the wire?
Q.8 Is it possible to have longitudinal waves on a string?
A transverse wave in a steel rod? Q.21 Two waves of angular frequencies 50 and 5000
rad s-1 have the same displacement amplitude, 3 × 10-5
cm. Deduce the acceleration amplitude for them.
Q.9 What type of mechanical waves do you expect to
exist in (a) vacuum (b) air (c) inside the water (d) rock (e)
on the surface of water? Q.22 The equation of a wave traveling in x- direction
on a string is y = (3.0 cm) sin [(3.14cm-1) x – (314 s-1)t]
(a) Find the max. Velocity of a particle of the string.
1 1 . 3 6 | Waves on a String

(b) Find the acceleration of a particle at x = 6.0 cm and


5 cm
at time t = 00.11 s.
0.3 cm
O
Q.23 A fork of frequency 250 Hz is held over and 0.1 cm 0.5 cm
maximum sound is obtained when the column of air is
-5 cm
31 cm or 97 cm. Determine (i) velocity of sound (ii) the
end correction (iii) the radius of the tube. (A) 1.0 ms-1 (B) 1.5 ms-1
(C) 2.0 ms-1 (D) 2.5 ms-1
Q.24 In an experiment, it was found that a tuning fork
and a sonometer gave 5 beats/sec, both when length
of wire was 1 m and 1.05m. Calculate the frequency of Q.4 A block of mass 1 kg is hanging vertically from a
the fork. string of length 1 m and mass/ length = 0.001 Kg/m.
A small pulse is generated at its lower end. The pulse
reaches the top end in approximately

Exercise 2
1m
Single Correct Choice Type
(A) 0.2 sec (B) 0.1 sec
Q.1 A wave is propagating along x–axis. The
displacement of particle of the medium in z – direction (C) 0.02 sec (D) 0.01 sec
at t = 0 is given by: z = exp [- (x +2)2], where ‘x’ is in
meters. At t = 1s, the same wave disturbance is given Q. 5 A uniform rope having some mass hangs vertically
by: z = exp [-2 ( 2-x ) 2]. Then, the wave propagation from a rigid support. A transvers wave pulse is produced
velocity is at the lower end. The speed (v) of the wave pulse varies
with height (h) from the lower end as:
(A) 4 m/s in + x direction
(B) 4 m/s in - x direction v v

(C) 2 m/s in + x direction


(A) (B)
(D) 2 m/s in - x direction
h h
Q.2 The equation of a wave traveling along the positive v v
x – axis, as shown in figure at t = 0 is given by
y (C) (D)
1 h h
0 x
-0.5
-0.1 Q. 6 A wire of 10-2 kgm-1 passes over a frictionless light
pulley fixed on the top of a frictionless inclined plane
 π  π which makes an angle of 300 with the horizontal. Masses
(A) sin  kx − ωt + π  (B)sin  kx − ωt − π  m and M are tied at two ends of wire such that m rests
6
(A) sin  kx − ωt +  6
(B)sin  kx − ωt − 
6 6 on the plane and M hangs freely vertically downwards.
 π   π
(C)sin  ωt − kx + π  (D)sin  ωt − kx − π  The entire system is in equilibrium and a transverse
(C)sin  ωt − kx + 6  (D)sin  ωt − kx − 6  wave propagates along the wire with a velocity of 100
 6  6
ms-1. Then,
m 1
Q.3 In the figure shown the shape of part of a long string (A) M = 5kg (B) =
M 4
in which transverse wave are produced by attaching
one end of the string to tuning fork of frequency 250 m
Hz. What is the velocity of the waves? (C) m = 20kg (D) =4
M
P hysi cs | 11.37

Q.7 Consider a function y = 10 sin2 (100πt + 5 πz) where A transverse wave pulse: Y = (6 mm) sin (5t + 40x),
y, z are in cm and t is in second. where ‘t’ is in seconds and ‘x’ in meters, is sent along
the lighter string towards the joint. The joint is at x =
(A) The function represents a traveling, periodic wave
0. The equation of the wave pulse reflected from the
propagating in (-z) direction with speed 20m/s.
joint is
(B) The function does not represent a traveling wave.
(A) (2 mm) sin (5t – 40x)
(C) The amplitude of the wave is 5 cm.
(B) (4 mm) sin (40t – 5x)
(D) The amplitude of the wave is 10 cm.
(C) - (2 mm) sin (5t – 40x)

Q. 8 The displacement from the position of equilibrium (D) (2 mm) sin (5t – 10x)
of a point 4 cm from a source of sinusoidal oscillations
is half the amplitude at the moment t = T/ 6 (T is the Q. 13 In the previous question, the percentage of power
time period). Assume that the source was at mean transmitted to the heavier string through the joint is
position at t = 0. The wavelength of the running wave is approximately
(A) 0.96m (B) 0.48m (C) 0.24m (D) 0.12m (A) 33% (B) 89% (C) 67% (D) 75%

Q. 9 The period of oscillations of a point is 0.04 sec. Q.14 A wave pulse on a string has the dimension shown
and the velocity of propagation of oscillation is 300m/ in figure. The waves speed is V= 1 cm/s. If point O is a
sec. The difference of phases between the oscillations free end. The shape of wave at time t = 3 s is:
of two points at distance 10 and 16m respectively from
the source of oscillations is V=1cm/s
1 cm
(A) 2π (B) π/ 2 (C) π/ 4 (D) π O
1 cm 1 cm 2 cm
3
Q.10 A motion is described by y = where O
a + (x + 3t)2
2

y, x are in meter and t is in second. 1cm


(A) O (B)
(A) This represents equation of progressive wave
1cm
propagation along – x direction with 3 ms-1.
O
(B) This represents equation of progressive wave
1cm
propagation along + x direction with 3 ms-1. 2cm
(C) (D)
(C) This does not represent a progressive wave equation. 1cm
(D) Data is insufficient to arrive at any conclusion. 1cm

Q.15 A string 1 m long is drawn by a 300 Hz vibrator


Q.11 A pulse shown here is reflected from the rigid wall attached to its end. The string vibrates in 3 segments.
A and then from free end B. The shape of the string The speed of transverse waves in the string is equal to
after these 2 reflection will be
(A) 100 m/s (B) 200 m/s
(C) 300 m/s (D) 400 m/s
B A
Q.16 The resultant amplitude due to superposition
Q.12 A composition string is made up by joining two of two waves y1 = 5sin (ωt - kx) and y2 = -5 cos
strings of different masses per unit length →μ and 4μ. (ωt – kx - 1500)
The composite string is under the same tension.
(A) 5 (B) 5 3
(A)
B A
(B)
B A (C) 5 2 − 3 (D) 5 2 + 3

(C) (D)
B A B A
1 1 . 3 8 | Waves on a String

Q.17 A wave represented by the equation Q.23 A wave travels uniformly in all directions from a
= y A cos(kx − ω t) is superimposed with another wave point source in an isotropic medium. The displacement
to from a stationary wave such that the point x = 0 is a of the medium at any point at a distance r from
node. The equation of the other wave is: the source may be represented by (A is a constant
representing strength of source)
(A) − A sin (kx + ω t) (B) − A cos (kx + ω t)
(C) A sin (kx + ω t) (D) A cos (kx + ω t)
(A) [A / r ]sin(kr − ω t) (B) [A / r]sin(kr − ω t)

Q.18 A taut string at both ends vibrates in its nth (C) [Ar]sin(kr − ω t) (D) [A / r 2 ]sin(kr − ω t)
overtone. The distance between adjacent Node and
Antinode is found to be ‘d’. If the length of the string Q.24 A sinusoidal progressive wave is
is L, then generated in a string. Its equation is given by
(A)
= L 2d(n + 1) (B)=L d (n + 1) =Y (2mm)sin(2πx −100π t + π / 3) . The time when
particle at x = 4 m first passes through mean position,
(C) L = 2dn (D)
= L 2d(n − 1)
will be
1 1
Q.19 A metallic wire of length L is fixed between (A) sec (B) sec
two rigid supports. If the wire is cooled through a 150 12
temperature difference ΔT (Y = young’s modulus, ρ = 1 1
(C) sec (D) sec
density, α = coefficient of linear expansion) then the 300 100
frequency of transverse vibration is proportional to:
Q.25 A transverse wave is described by the equation
α Yα ρ ρα =Y A sin[2πx (ft − x / λ )] . The maximum particle velocity
(A) (B) (C) (D) is equal to four times the wave velocity if:
ρY ρ Yα Y
(A) λ =π A / 4 (B) λ =π A / 2
(C) λ =π A (D) λ = 2π A
 20 
Q.20 A standing wave Y = A sin  π x  cos(1000 π t) is
 3 
maintained in a taut string where y and x are expressed
in meters. The distance between the successive points Previous Years’ Questions
oscillating with the amplitude A/2 across a node is
equal to
Q. 1 A transverse wave is described by the equation
(A) 25 cm (B) 2.5 cm (C) 5 cm (D) 10 cm  x
y = y 0 sin2π  ft −  . The maximum particle velocity
 λ
Q.21 A string of length 0.4m & mass 10-2kg is tightly is equal to four time the wave velocity if  (1984)
clamped at its ends. The tension in the string is 1.6 N.
(A) λ =π (B) λ =π y 0 / 2
Identical wave pulses are produced at one end at equal
intervals of time, Δt. The minimum value of Δt which ( C ) λ = 2π ( D ) λ = 2π y 0
allows constructive interference between successive
pulses is: Q.2 A wave represented by the equation
(A) 0.05s (B) 0.10s (C) 0.20s (D) 0.40s y = acos (kx − ωt ) is superimposed with another wave
to from a stationary wave such that point x = 0 is a
node. The equation for the other wave is  (1988)
Q. 22 Fig 11.46, show a stationary wave between two
fixed points P and Q. which points (s) of 1, 2 and 3 are
in phase with the point X? Q.3 The displacement y of a particle executing periodic
1 
motion is given by y = 4 cos2  t  sin(1000 t) . This
P X 1 23 Q 2 
expression may be considered to be a result of the
(A) 1, 2 and 3 (B) 1 and 2 only superposition of ………………….. Independent harmonic
(C) 2 and 3 only (D) 3 only motions.  (1992)
(A) Two (B) Three (C) Four (D) Five
P hysi cs | 11.39

Q.4 The extension in a string, obeying Hooke’s law, is the wire. When this mass is replaced by mass M. The
x. The speed of transverse wave in the stretched string wire resonates with the same tuning fork forming three
is. If the extension in the string is increased to 1.5 x, the antinodes for the same positions of the bridges. The
speed of transverse wave will be  (1996) value of M is (2002)
(A) 1.22 (B) 0.61 (C) 1.50 (D) 0.75 (A) 25 kg (B) 5kg
(C) 12.5 kg (D) 1/25 kg
Q. 5 A traveling wave in a stretched string is described
by the equation; Y = A sin (kx – ωt)
Q.10 A massless rod BD is suspended by two identical
The maximum particle velocity is  (1997) massless strings AB and CD of equal lengths. A block
of mass m is suspended from point P such that BP is
(A) Aω (B) ω/k (C) dω/dk (D) x/ω
equal to x. If the fundamental frequency of the left wire
is twice the fundamental frequency of right wire, then
Q.6 Two vibrating strings of the same material but of the value of x is (2006)
lengths L and 2L have radii 2r and r respectively. They
are stretched under the same tension. Both the strings A C
vibrate in their fundamental modes. The one of length L
with frequency V1 and the other with frequency V2. The
ratio V1/V2 is given by  (2000)
(A) 2 (B) 4 (C) 8 (D) 1
x
P
Q.7 The ends of a stretched wire of length L are B D
fixed at x = 0 and x = L. In one experiment the
 πx  m
displacement of the= wire is y1 A sin   sin ωt and
 L  (A) l/5 (B) l/4 (C) 4l/5 (D) 3l/4
energy is E1 and in other experiment its displacement is
 2π x  Q.11 A hollow pipe of length 0.8 m is closed at one
=y 2 A sin   sin2ωt and energy is E2. Then (2011) end. At its open end, a 0.5 m long uniform string is
 L 
vibrating in its second harmonic and it resonates with
(A) E2 = E1 (B) E2 = 2E1 the fundamental frequency of the pipe. If the tension in
(C) E2 = 4E1 (D) E2 = 16E1 the wire is 50 N and the speed of sound is 320ms-1, the
mass of the string is  (2010)

Q.8 Two pulses in a stretched string, whose centers are (A) 5 kg (B) 10kg (C) 20 kg (D) 40 kg
initially 8 cm apart, are moving towards each other as
shown in the figure. The speed of each pulse is 2 cm/s. Q.12 The displacement of particles in a string stretched
After 2 s the total energy of the pulses will be  (2001) in the x – direction is represented by y. Among the
following expressions for y, those describing wave
motion is (are)  (1987)
(A) cos kx sin ωt (B) k 2 x2 − ω2 t2
8 cm (C) cos2 (kx + ωt) (
(D) cos k 2 x2 − ω2 t2 )
(A) Zero
Q.13 A wave is represented by the equation;
(B) Purely kinetic y = A sin (10 π x +15π t + π / 3 )
(C) Purely potential Where x is in meter and t is in second. The expression
(D) Partly kinetic and partly potential represents  (1990)
(A) A wave traveling in the position x – direction with a
Q.9 A sonometer wire resonates with a given tuning fork velocity 1.5 m/s
forming standing waves with five antinodes between (B) A wave traveling in the negative x – direction with a
the two bridges when a mass of 9 kg is suspended from velocity 1.5 m/s
1 1 . 4 0 | Waves on a String

(C) A wave traveling in the negative x – direction with a Q.18 In a wave motion, y = a sin(kx – wt), y can represent
wavelength 0.2 m  (1999)
(D) A wave traveling in the position x – direction with a (A) Electric field (B) Magnetic field
wavelength 0.2 m
(C) Displacement (D) Pressure

Q.14 Two identical straight wires are stretched so as to


Q.19 Standing waves can be produced  (1999)
produce 6 beats/ s when vibrating simultaneously. On
changing the tension slightly in one of them, the beat (A) On a string clamped at both ends
frequency remains unchanged. Denoting by T1, T2 the (B) On a string clamped at one end and free at the other
higher and the lower initial tension in the strings, then
it could be said that while making the above changes in (C) When incident wave gets reflected from a wall
tension (1991) (D) When two identical waves with a phase difference
(A) T2 was decreased (B) T2 was increased of π are moving in the same direction

(C) T1 was decreased (D) T1 was increased


Q.20. A wave travelling along the x-axis is described
by the equation y(x, t) = 0.005 cos (αx −βt). If the
Q. 15 A wave disturbance in a medium is described by wavelength and the time period of the wave are 0.08 m
 π and 2.0 s, respectively, then α and β in appropriate units
y ( x, t )
= 0.02cos  50 πt +  cos(10 πx) , are (2008)
 2
Where x and y are in meter and t is in second. (1995) 0.08 2.0
(A) α = 25.00 π, β = π (B) α = ,
π π
(A) A node occurs at x = 0.15m
0.04 1.0 π
(B) An antinode occurs at x = 0.3 m (C)=α =,β (D)=
α 12.50=
π, β
π π 2.0
(C) The speed of wave is 5 ms-1
(D) The wavelength of wave is 0.2 m Q.21 The equation of a wave on a string of
linear mass density 0.04 kg m–1 is given by

Q.16 The (x, y) coordinates of the corners of a square =   t x 


y 0.02 (m) sin 2π  −   . The tension
plate are (0, 0), (L, 0), (L, L) and (0, L). The edges of the   0.04(t) 0.50(m)  
plates are clamped and transverse standing waves are in the string is (2010)
set-up in it. If u ( x, y ) denotes the displacement of
the plate at the point (x, y) at some instant of time, (A) 4.0 N (B) 12.5 N
the possible expression (s) for u is (are) (a = positive
constant)  (1998) (C) 0.5 N (D) 6.25 N

(A) acos( π x / 2L)cos( π y / 2L) Q.22 The transverse displacement y (x,t) of a wave
2 +bt2 + 2 abxt)
(B) asin( π x / L)sin( π y / L) on a string is given by y(a,t) = e−(ax .This
represents a (2011)
(C) asin( π x /L)sin(2π y / L)
(D) acos(2π x /L)cos( π y / L) b
(A) Wave moving in – x direction with speed
a
Q.17 A transverse sinusoidal wave of amplitude a, (B) Standing wave of frequency b
wavelength λ and frequency is traveling on a stretched 1
string. The maximum speed of any point on the string is (C) Standing wave of frequency
b
ν/10, where is the speed of propagation of the wave. If
a = 10-3m and ν = 10m/s, then λ and f are given by a
(D) Wave moving in + x direction with
 (1998) b

(A) λ = 2π×10−2 m (B) λ = 10−3 m


10 −3
(C) f = Hz (D) f = 104Hz

P hysi cs | 11.41

Q.23 Two particles are executing simple harmonic Q.27 A pendulum made of a uniform wire of cross
motion of the same amplitude A and frequency ω along sectional area A has time period T. When an additional
the x-axis. Their mean position is separated by distance mass M is added to its bob, the time period changes to
X0 (X0 > A). If the maximum separation between them is TM. If the Young’s modulus of the material of the wire
(X0 + A), the phase difference between their motion is: 1
 (2011) is Y then is equal to : (g = gravitational acceleration)
 Y (2015)
X0+A
  T 2    T 2 
  A Mg
M
(A)   − 1 (B)  M  − 1
 T   Mg  T   A
   
X0 A
  T 2    2 
A T A
(C) 1 −  M   (D) 1 −   
  T   Mg   TM   Mg
π π π π    
(A) (B) (C) (D)
3 4 6 2
Q.28 For a simple pendulum, a graph is plotted between
Q.24 A mass M, attached to a horizontal spring, its kinetic energy (KE) and potential energy (PE)
executes S.H.M. with amplitude A1. When the mass M against its displacement d. Which one of the following
passes through its mean position then a smaller mass represents these correctly? (Graphs are schematic and
m is placed over it and both of them move together not drawn to scale) (2015)
A 
with amplitude A2. The ratio of  1  is: (2011) E E
 A2 
1/2 KE PE
M+m
(A)
M+m
(B)
(A) (B)  
M  M 
PE
d
KE
d

1/2
 M  M
(C)   (D)
M+m M+m E KE
E

PE
(D)
Q.25 A sonometer wire of length 1.5 m is made of steel.
(C) d
KE
The tension in it produces an elastic strain of 1%.What d

is the fundamental frequency of steel if density and PE

elasticity of steel are 7.7 × 103 kg/m3 and 2.2 × 1011 N/m2
Q.29 A uniform string of length 20 m is suspended
respectively? (2013)
from a rigid support. A short wave pulse is introduced
(A) 188.5 Hz (B) 178.2 Hz at its lowest end. It starts moving up the string. The
(C) 200.5 Hz (D) 770 Hz time taken to reach the support is: (take g = 10 ms–2)
 (2016)

Q.26 The period of oscillation of a simple pendulum (A) 2s (B) 2 2 s (C) 2 s (D) 2 π 2 s

L
is T = 2π . Measured value of L is 20.0 cm known Q.30 A particle performs simple harmonic motion with
g
amplitude A. Its speed is trebled at the instant that it
to 1 mm accuracy and time for 100 oscillations of the 2A
pendulum is found to be 90s using a wrist watch of 1s is at distance from equilibrium position. The new
3
resolution. The accuracy in the determination of g is:
amplitude of the motion is. (2016)
 (2015)
7A A
(A) 2% (B) 3% (C) 1% (D) 5% (A) 3A (B) A 3 (C) (D) 41
3 3
1 1 . 4 2 | Waves on a String

JEE Advanced/Boards

Exercise 1 Q.4 A steel wire has a mass of 5gm and is under


tension 450N. Find the maximum average power that
can be carried by the transverse wave in the wire if the
Q.1 A transverse wave is traveling along a string from amplitude is not to exceed 20% of the wavelength.
left to right. The figure. represents the shape of the
string (snap - shot) at a given instant. At this instant (a)
Q.5 The figure shown a triangle pulse on a rope at t =
which points have an upward velocity (b) which points
0. It is approaching a fixed end at 2 cm/s
will have downward velocity (c) which points have zero
velocity (d) which points have maximum magnitude of
2 cm/s
velocity?

C
B D 1 cm
A
E 2 cm 1 cm 1 cm
F H
(a) Draw the pulse at t = 2sec.
G
(b) The particle speed on the leading edge at the instant
Q.2 A sinusoidal wave propagates along a string. In depicted is_______.
figure (a) and (b). ‘y’ represents displacement of particle
from the mean position. ‘x’ & ‘t’ have usual meanings.
Q.6 Two strings A and B with μ =2 kg/m and μ = 8
Find:
kg/m respectively are joined in series and kept on a
(a) Wavelength, frequency and speed of the wave. horizontal table with both the ends fixed. The tension in
(b) Maximum velocity and maximum acceleration of the string is 200 N. If a pulse of amplitude 1 cm travels
the particles in A towards the junction, then find the amplitude of
reflected and transmitted pulse.
(c) The magnitude of slope of the string at x = 2 at
t = 4 sec.
Q.7 A parabolic pulse given by equation y (in cm) =
0.3 – 0.1 (x – 5t) 2 (y > 0) x in meter and t in second
y (in mm)
for x=2m traveling in a uniform string. The pulse passes through
+3 a boundary beyond which its velocity becomes 2.5 m/s.
2 4 6 What will be the amplitude of pulse in this medium
0 t (in sec.)
after transmission?
-3
(a)
Q.8 A 40 cm long wire having a mass 3.2 gm and area
y (in mm) of c.s 1 mm2 is stretched between the support 40.05
for x=2m
cm apart. In its fundamental mode, it vibrates with a
0
1 3 7 frequency 1000/64 Hz. Find the young’s modulus of the
x (in m)
wire.

(b) Q.9 A string of mass 0.2 kg/m and length


Q.3 The extension in a string, obeying Hook’s low is L = 0.6m is fixed at both ends and stretched such that
x. the speed of wave in the stretched string is v. If the is has a tension of 80 N. The string is vibrating in its
extension in the string increased to 1.5x find the new third normal mode, has an amplitude of 0.5 cm. What
the speed of wave. is the frequency of oscillation? What is the maximum
transverse velocity amplitude?
P hysi cs | 11.43

Q.10 A rope, under tension of 200N and fixed at both Q.2 A string of length 1 m and linear mass density 0.01
ends, oscillates in a second – harmonic standing wave kgm-1 is stretched to a tension of 100N. When both ends
pattern. The displacement of the rope is given by: of the string are fixed, the three lowest frequencies for
standing wave are f1, f2 and f3. When only one end of the
Y = ( 0.10m) (sin πx / 2)sin12πt string is fixed, the three lowest frequencies for standing
Where x = 0 at one end of the rope, x is in meters and t wave are n1, n2 and n3. Then
is in seconds. What are
(A) n3 = 5n1 = f3 = 125 Hz
(a) The length of the rope
(B) f3 = 5f1 = n3 = 125 Hz
(b) The speed of the progressive waves on the rope,
(C) f3 = n2 = 3f1 = 150 Hz
and
f1 + f2
(c) The mass of the rope (D)=
n2 = 75Hz
2
(d) If the rope oscillates in a third – harmonic standing
wave pattern, what will be the period of oscillation? Q.3 A chord attached to a vibrating string from divides
it into 6 loops, when its tension is 36N. the tension at
Q.11 A stretched uniform wire of a sonometer between which it will vibrate in 4 loops is
two fixed knife edges, when vibrates in its second
(A) 24N (B) 36N (C) 64N (D) 81N
harmonic gives 1 beat per second with a vibrating
tuning fork of frequency 200 Hz. Find the percentage
change in the tension of the wire to be in unison with Q.4 A wave equation is given as y = cos (500t – 70x),
the tuning fork. where y in mm and t is in sec.
(A) The wave is not a transverse propagating wave.
Q.12 A string fixed at both ends has consecutive (B) The speed of wave is 50/7 m/s
standing wave modes for which the distances between
adjacent nodes are 18 cm and 16 cm respectively. (C) The frequency of oscillation 1000π Hz

(a) What is the length of the string? (D) Two closest points which are in same phase have
separation 45 π/7 cm.
(b) If the tension is 10 N and the linear mass density is
4kg/m, what is the fundamental frequency?
Q.5 A wave pulse passing on a string with a speed of
40 cm s–1 in the negative x – direction has its maximum
Q.13 In a mixture of gases, the average number of at x = 0 at t = 0. Where will this maximum be located
degree of freedom per molecules is 6. The rms speed at t = 5s?
of the molecules of the gas is c. find the velocity of
sound in the gas. (A) 2 m (B) 3 m (C) 1 m (D) 2.5 m

Q.6 A steel wire of length 64 cm weights 5 g. If it is


stretched by a force of 8 N, what would be the speed of
Exercise 2 a transverse wave passing on it?
(A) 10 m/s (B) 38 m/s (C) 32 m/s (D) 22 m/s
Single Correct Choice Type

Q.7 Two blocks each having a mass of 3.2 kg are


Q.1 A wave is represented by the equation
connected by a wire CD and the system is suspended
Y = 10 sin2π (100t − 0.02x) + 10 sin2π (100t + 0.02x) The
from the ceiling by another wire AB. The linear mess
maximum amplitude and loop length are respectively
density of the wire AB is 10 g m-1 and that of CD is 8 gm-1.
(A) 20 units and 30 units Find the speed of a transverse wave pulse produced in
(B) 20 units and 25 units AB and in CD.

(C) 30 units and 20 units (A) 80 m/s, 63 m/s (B) 75 m/s, 54 m/s

(D) 25 units and 20 units (C) 82 m/s, 33 m/s (D) 87 m/s , 60 m/s
1 1 . 4 4 | Waves on a String

Q.8 In the arrangement shown in figure, the string


y
has a mass of 4.5 g. How much time will it take for a
transverse disturbance produced at the floor to reach Q
the pulley?
(Take g = 10 ms-2) x
P R

25 cm

2.0 cm
(A) VP is up wards (B) VQ = − VR
2 kg (C) | VP | > | VQ | =
| VR | (D) VQ = VR

(A) 0.03 s (B) 0.02 s (C) 0.01 s (D) 0.04 s Comprehension Type
The figure represents the instantaneous picture of a
Assertion Reasoning Type transverse harmonic wave traveling along the negative
X – axis. Choose the correct alternative (s) related to the
Q.9 Statement-I: In a sinusoidal traveling wave on a movement of the mine points shown in the figure.
string potential energy of deformation of string element
at extreme position is maximum y
b
Statement-II: The particle in sinusoidal traveling wave
a c
perform SHM. h
o x
d
(A) Statement-I is true, statement-II is true, statement-II
e g
is a correct explanation for statement-I
f
(B) Statement-I is true, statement-II is true, statement-II
is NOT correct explanation for statement-I Q.12 The point/s moving upward is/are
(C) Statement-I is true, statement-II is false (A) a (B) c (C) f (D) g
(D) Statement-I is false, statement-II is true
Q.13 The point/s moving downwards is/are
Q. 10 Statement-I: When a pulse on string reflects (A) o (B) b (C) d (D) h
from free end, the resultant pulse is formed in such a
way that slope of string at free end is zero.
Q.14 The stationary points is/ are
Statement-II: Zero resultant slope ensures that there is
(A) o (B) b (C) f (D) h
no force components perpendicular to string.
(A) Statement-I is true, statement-II is true, statement-II Q.15 The point/s moving with maximum velocity is/are
is a correct explanation for Statement-I
(A) b (B) c (C) d (D) h
(B) Statement-I is true, statement-II is true, statement-II
is NOT correct explanation for Statement-I
(C) Statement-I is true, statement-II is false
Previous Years’ Questions
(D) Statement-I is false, statement-II is true

Q. 1 An object of specific gravity ρ is hung from a thin


Multiple Correct Choice Type
steel wire. The fundamental frequency for transverse
standing waves in the wire is 300 Hz. The object is
Q.11 At a certain moment, the photograph of a string immersed in water, so that one half of its volume is
on which a harmonic wave is traveling to the right is submerged. The new fundamental frequency (in Hz) is
shown. Then, which of the following is true regarding  (1995)
the velocities of the points P, Q and R on the string.
P hysi cs | 11.45

1/2 1/2 Paragraph 1:


 2ρ −1   2ρ 
(A) 300   (B) 300   Two plane harmonic sound waves are expressed by the
 2ρ   2ρ −1 
equations.
 2ρ   2ρ − 1  y1 (x,t)
= A cos( π x −100 π t)
(C) 300   (D) 300  
 2ρ − 1   2ρ  and y=
2 (x,t) A cos(0.4 π x − 92 π t)
(All parameters are in MKS) (2006)
Q.2 A string of length 0.4m and mass 10-2 kg is tightly
clamped at its ends. The tension in the string is 1.6N. Q.5 How many times does an observer hear maximum
Identical wave pulses are produced at one end at equal intensity in one second?
intervals of time Δt, The minimum value of Δt, which
allows constructive interference between successive (A) 4 (B) 10 (C) 6 (D) 8
pulses, is (1998)
(A) 0.05 s (B) 0.10 s (C) 0.20 s (D) 0.40s Q.6 What is the speed of sound?
(A) 200 m/s (B) 180 m/s (C) 192 m/s (D) 96 m/s
Q. 3 A transverse sinusoidal wave moves along a string
in the positive x – direction at a speed of 10 cm/s. The Q.7 At x = 0 how many times the amplitude of y1 + y2 is
wavelength of the wave is 0.5 m and its amplitude is zero in one second?
10 cm. At a particular time t, the snap – shot of the (A) 192 (B) 48 (C) 100 (D) 96
wave is shown in figure. The velocity of point P when its
displacement is 5 cm is (2008)
Q.8 A wave equation which gives the displacement
y along the y – direction is given
= by; y 10 −4 sin(60 t + 2x) .
Where x and y are in meter and t is time in second. This
represents a wave  (1981)
P (A) Traveling with a velocity of 30 m/s in the negative
x x – direction
(B) Of wavelength π m
(C) Of frequency 30/ π Hz
3π ˆ 3π ˆ
(A) jm / s (B) − jm / s (D) Of amplitude 10-4 m
50 50

3π 3π
(C) î m / s (D) − î m / s Q.9 As a wave propagates (1999)
50 50
(A) The wave intensity remains constant for a plane wave

Q.4 A vibrating string of certain length l under a tension (B) The wave intensity decreases as the inverse of the
T resonates with a mode corresponding to the first distance from the source for a spherical wave
overtone (third harmonic) of an air column of length (C) The wave intensity decreases as the inverse square
75 cm inside a tube closed at one end. The string also of the distance from the source for a spherical wave
generates 4 beats/s when excited along with a tuning
fork of frequency n. Now when the tension of the string (D) Total intensity of the spherical wave over the
is slightly increased the number of beats reduces to 2 spherical surface centered at the source remains
per second. Assuming the velocity of sound in air to constant at all times
be 340 m/s, the frequency n of the tuning fork in Hz is
0.8
 (2008) Q.10 Y (x,t) = represents a moving pulse
[(4x + 5t)2 + 5]
(A) 344 (B) 336 (C) 117.3 (D) 109.3 where x and y are in meter and t is in second. Then,(1999)
(A) Pulse is moving in positive x – direction
(B) In 2 s it will travel a distance of 2.5 m
(C) Its maximum displacement is 0.16 m
(D) It is a symmetric pulse
1 1 . 4 6 | Waves on a String

Q.11 A copper wire is held at the two ends by rigid (C) The maximum displacement of the midpoint of the
supports. At 300C, the wire is just taut, with negligible string, from its equilibrium position is 0.01m.
tension. Find the speed of transverse waves in this wire
(D) The fundamental frequency is 100 Hz.
at 100C. (1998)
Given: Young modulus of copper = 1.3 ×1011 N/m2 Q.17 One end of a taut string of length 3m along the x
Coefficient of linear expansion of copper = 1.7 ×10-5 o C-1 axis is fixed at x = 0. The speed of the waves in the string
is 100 ms-1. The other end of the string is vibrating in
Density of copper = 9 ×103 kg/m2
the y direction so that stationary waves are set up in
the string. The possible waveform(s) of these stationary
Q.12 A string 25 cm long and having a mass of 2.5 g is waves is (are): (2014)
under tension. A pipe closed at one end is 40 cm long.
πx 50πt
When the string is set vibrating in its first overtone and (A) y(t) = A sin cos
the air in the pipe in its fundamental frequency, 8 beats/ s 6 3
are heard. It is observed that decreasing the tension in πx 100πt
(B) y(t) = A sin cos
the string decreases the beat frequency. If the speed of 3 3
sound in air is 320 m/s find the tension in the string. 5πx 250πt
 (1999) (C) y(t) = A sin cos
6 3
5πx
Q.13 A uniform rope of length 12 m and mass 6= kg (D) y(t) A sin cos250πt
6
hangs vertically from a rigid support. A block of mass
2 kg is attached to the free end of the rope. A transverse
Q.18 A metal rod AB of length 10x has its one end A
pulse of wavelength 0.06 m is produced at the lower
in ice at 0°C and the other end B in water at 100°C. If
end of the rope. What is the wavelength of the pulse
a point P on the rod is maintained at 400°C, then it is
when it reaches the top of the rope? (1984)
found that equal amounts of water and ice evaporate
and melt per unit time. The latent heat of evaporation
Q.14 A steel wire of length 1 m, mass 0.1 kg and uniform of water is 540 cal/g and latent heat of melting of ice is
cross-sectional area 10-6 m2 is rigidly fixed at both ends. 80 cal/g. If the point P is at a distance of λ x from the
The temperature of the wire is lowered by 200C. If ice end A, find the value of λ . [Neglect any heat loss to
transverse waves are set – up by plucking the string in the surrounding.] (2009)
the middle, calculate the frequency of the fundamental
mode of vibration. (2009)
Q.19. Column I shows four systems, each of the same
11
Given : Y steel = 2 ×10 N / m2
and length L, for producing standing waves. The lowest
possible natural frequency of a system is called its
αsteel = 1.21×10−5 / ο C. fundamental frequency, whose wavelength is denoted
as λ f . Match each system with statements given in
Q.15 When two progressive waves y1 = 4 sin (2x - 6t) column II describing the nature and wavelength of the
 π standing waves. (2011)
and y2 = 3 sin  2x − 6t −  are superimposed, the
 2
amplitude of the resultant wave is: (2010) Column I Column II
(A) Pipe closed at one end (p) Longitudinal waves

Q.16 A horizontal stretched string fixed at two ends, is


(A)
vibrating in its fifth harmonic according to the equation O L
y(x, t) = 0.01m sin [(62.8m-1)x] cos[(628s-1)t]. Assuming
(B) Pipe open at both ends (q) Transverse waves
π = 3.14, the correct statement(s) is (are): (2013)
(A) The number of nodes is 5. (B)
(B) The length of the string is 0.25 m. O L
P hysi cs | 11.47

Column I Column II Column I Column II


(C) Stretched wire clamped (D) Stretched wire clamped
at both ends (r) λ f =L at both ends and at mid- (s) λ f =2L
point
(C)
O L
(D)
O L/2 L

(t) λ f =4L

PlancEssential Questions
JEE Main/Boards JEE Advanced/Boards
Exercise 1 Exercise 1
Q. 5 Q. 17 Q. 22 Q. 2 Q.10 Q.12

Exercise 2 Exercise 2
Q.1 Q.4 Q.5 Q.1 Q.2 Q.4
Q.6 Q.10 Q.14 Q.9 Q.10
Q.21

Answer Key

JEE Main/Boards Q. 16 4.23 s


Q. 17 1.320 km
Exercise 1
Q. 18 92.95 ms-1
Q. 2 20 m Q. 19 5.27 ×103 ms-1
Q. 3 382.2 ms -1
Q. 20 100 ms-1
Q. 4 (i) 0.25 × 10-3 cm Q. 21 7.5 ×10-2 cms-2; 7.5 ×10 2 cms-2
(ii) π/250 sec (iii) 500 rad/sec Q. 22 (a) 9.42 m/s (b) zero
(iv) 80 π meters Q. 23 330 ms-1, 0.02 m, 0.033 m
(v) 0.125 cm/s (vi) 62.5 cm/sec 2
Q. 24 205 Hz
Q. 6 353.6 ms -1

Q. 7 250 Hz
1 1 . 4 8 | Waves on a String

Exercise 2
Single Correct Choice Type

Q.1 A Q.2 D Q. 3 A Q. 4 D Q. 5 C Q. 6 C
Q.7 C Q.8 B Q. 9 D Q. 10 A Q. 11 A Q. 12 C
Q.13 B Q.14 D Q.15 B Q.16 A Q. 17 B Q. 18 B
Q.19 B Q.20 C Q.21 B Q.22 C Q. 23 B Q.24 C
Q.25 B

Previous Years’ Questions


Q. 1 B Q. 2 C Q. 3 B Q. 4 A Q. 5 A Q. 6 D
Q. 7 C Q. 8 B Q. 9 A Q. 10 A Q. 11 B Q. 12 A, C
Q. 13 B, C Q. 14 D Q. 15 A, B, C, D Q. 16 B,C Q. 17 A Q. 18 A,B
Q. 19 A Q. 20 A Q. 21 D Q. 22 A Q. 23 D Q. 24 C
Q. 25 B Q. 26 B Q. 27 A Q. 28 B Q. 29 B Q. 30 C

JEE Advanced/Boards
Exercise 1
1
Q. 1 (a) D,E,F, (b) A,B,H, (c) C,G, (d) A,E Q. 2 (a) λ=4m, f = Hz , 1 m/s
4
3π 3π2 3π
(b) mm / s, mm / s2 , (c) Q. 3 1.22 v
2 4 2

Q. 4 106.59 kW Q. 5 (a)

(b) 2 cm/s

1 2
Q.6 A1 =
− cm, A1 = cm Q. 7 0.2 cm
3 3
Q. 8 1 ×109 Nm–2 Q. 9 50Hz, 50π cm/sec
Q. 10 4 m, 24 m/s, 25/18 kg, 1/9 sec Q. 11 1.007%
Q. 12 (a) 144 cm; (b) 17.36 Hz Q. 13 2/3c

Exercise 2
Single Correct choice type
Q. 1 B Q. 2 D Q. 3 D Q. 4 B Q. 5 A Q. 6 C
Q. 7 A Q. 8 B

Assertion Reasoning Type


Q. 9 D Q. 10 A
P hysi cs | 11.49

Multiple Correct Choice Type


Q.11 C, D

Comprehension Type
Q. 12 A, D Q.13 C Q.14 B, C Q.15 C, D

Previous Years’ Questions


Q.1 A Q.2 B Q.3 A Q.4 A Q.5 A Q.6 A
Q.7 C Q.8 A, B, C, D Q.9 A Q.10 A Q.11 70.1 m/s Q.12 27.04 N
Q.13 0.12 m Q.14 11 Hz Q.15 5 Q. 16 B, C Q. 17 A, C, D Q. 18 9
Q. 19 A → p, t; B → p, s; C → q, s; D → q, r

Solutions

JEE Main/Boards amax. = 6.25 cms–2

Exercise 1 Sol 5:
1
1 1 f∝
Sol 1: (i) s≤T≤ 
30,000 40
f → 1 : 2: 3
350 350 1 1
(ii) m/s ≤ λ ≤ m/s →1: :
30000 40 2 3
(iii) 80π rads–1 ≤ ω ≤ 60000π rads–1 →6:3:2
Bridges must be placed at 60 cm from one end and 20
Sol 2: f = 15 × 106 Hz cm from another end
V 3 × 108 V V 20  0.04  20
l= = = 20 m Sol 6: – = ;V 
f 15 × 106 =
2.04 2.08 6  2.04 × 2.05  6
V = 353. 6 ms–1
4 4
Sol 3: V’ = V= × 331 = 382.2 ms–1
3 3
Sol 7: On loading with wax frequency decreases
f – 256 = ± 6
Sol 4: (i) A = 0.25 × 10–3 cm
f = 256 ± 6 Hz
2π π
(ii) T = = s
500 250 f = 262 Hz

(iii) ω = 500 rad/s


Sol 8: No because string is not stretchable yes
2π transverse waves are possible in a steel rod.
(iv) λ = m = 80π m
0.025
(v) Vmax = 0.25 × 10–3 × 500 cm s–1 Sol 9: (a) No wave possible as there is no particle.

Vmax. = 0.125 cms–1 (b) Longitudinal waves (direction of motion of particles


parallel to direction of propagation of wave)
(vi) amax. = Vmax ω = 0.125 × 500
1 1 . 5 0 | Waves on a String

(c) Longitudinal Sol 17:


(d) Both are possible
(e) Combined longitudinal & transverse (ripples)

Sol 10: Infinite as young’s modulus of a rigid body is


infinite
 1 1 
d  –
8 
= 8 × 330 × 3 × 108
Sol 11: Half the wavelength (λ/2)  330 3 × 10 
d (3× 108 – 330)
d~ = 8 × 330
d = 264 m
height of cloud = 1320 m = 1.32 Km

5 × 10 –3 1
Sol 18: m = =
0.72 144
Sol 12:
T = 60 N

t 60
V= = = 24 15 = 92.95 ms–1
µ 1/ 144

Sol 19: B = 7.5 × 1010 N m–2


Sol 13: Strings cannot be compressed or extended
hence there won’t be regions of compression and E = 2.7 × 103 kg m–3
rarefaction. Strings have elasticity of shape. Hence
wave on strings are transverse. B 7.5 × 1010
V= = = 5270.46 ms–1
e 2.7 × 103
Sol 14: Refer theory V = 5.27 × 103 ms–1

Sol 15: A harmonic of a wave is a component frequency 7 × 10 –3


Sol 20: µ = = 10–2 kg/m
of the signal that is an integer multiple of the fundamental 0.7
frequency. If f is the fundamental frequency the harmonics
have frequency 2f, 3f, 4f ….. etc. T = 100 N

An overtone is any frequency higher than the T 100


fundamental frequency of a sound. V= = = 100 ms–1
µ –2
10

Sol 16:
Sol 21: ω, = 50 rads–1 CO2 = 5000 rads–1
a = 3 × 10–5 × (50)2 cms–2
a1 = 7.5 × 10–2 cms–2
a2 = 3 × 10–5 × (5000)2 cms–2
2 × 78.4 a2 = 750 cms–2
Time to reach water = = 45
9.8 a2 = 7.5 ms–2
78.4
Time for sound to reach top = = 0.23 s
332 Sol 22: y = (3.0 cm) sin [(3.14 cm–1)x – (314 s–1)t]
Total time = 4.23 s (a) Vmax. = 3.0 cm × 314 s–1 = 942 cm s–1
= 9.42 ms–1
P hysi cs | 11.51

(b) a = –3 ×(314)2sin(π × 6 – 100 π × 0.11) 0.4


Sol 3: (A) V = fl ⇒ V = 250 × = 1 ms–1
a = – 3 × (314)2 sin(– 5π) 100

a=0
T 10
Sol 4: (D) V = = = 100 ms–1
µ 0.001
Sol 23: f = 250 Hz
λ 1m
T= = 0.01 sec
(31+ h) = 100ms–1
2
3λ λ
(97 + h) = ⇒ 66 =
4 2 Sol 5: (C) V ∝ T1/2
λ = 132 cm T decreases linearly with height
V = fλ = 250 × 1.32 ms–1 ⇒ V = 330 ms–1 ∴ Parabolic curve
132
H= – 31 = 2 cm = 0.02 m
4
End Cross section
Radius of tube =
0.6
0.02 0.2 0.1
= = = = 0.033 m
0.6 6 3 Sol 6: (C)

V
Sol 24: –F=5
2
V
F– =5
2.1 T = V2M = 100 N; T = mg sin q

V V 1
– = 10 Mg = 100 N ; 100 = m × 10 ×
2 2.1 2
V = 420 ms–1 M = 10 kg, m = 20 kg

420 m
F=5+ = 205 Hz =2
2.1 M

Sol 7: (C) y = 5(1 – cos(200pt + 10pz))

Exercise 2 Amplitude = 5 cm

Single Correct Choice Type Sol 8: (B) y = A sin(kx + wt)

2  π A π π
Sol 1: (A) z = e–(x– vt + c) A sin  4k +  = ⇒ 4k + =
 3 2 3 6
c–V×0=2
2π π
c–V×1=–2 ⇒K= =
λ 24
V = + 4 m/s λ = 48 cm ; λ = 0.48 m

 π 2π 2π
Sol 2: (D) y = – sin  kx – ωt +  Sol 9: (D) T = 0.04 sec; ω = = = 50p
6 T 0.04

ω 50π π
 π V = 300 m/s, k = = =
⇒ y = sin  ωt – kx –  V 300 6
 6
π
Dφ = ×6=p
6
1 1 . 5 2 | Waves on a String

3 3 Sol 17: (B) y1 = A cos(kx – ωt)


Sol 10: (A) y = ; y =
2
a + (x – 3t) 2 2 2
a + (x + vt) y2 = A cos(kx + ωt + φ)

V = – 3 m/s  φ  φ
y1+y2=2A cos 20  kx +  cos  ωt + 
 2  2
Sol 11: (A) Phase change of π due to reflection from φ
rigid wall. cos =0;φ=p
2
y2 = – A cos (kx + ωt)
Sol 12: (C) V1 → speed in light string
V2 → speed in heavy string L
Sol 18: (B) (n + 1) =
2d
V1
V2 = L = (n + 1)d
2
1
V2 2
Ar = A= 6 mm = 2 mm
V1 + V2 1
1+
2
y= (2mm) sin (kx – wt)
y = (2mm) sin(40x – 5t)
Sol 19: (B)
Sol 13: (B) For a given string Power ∝ A2
T = Y αDTA
2
 1
Power reflected =   P = P/9 v v 1 T
3 f∝ ; =
  22 µ
8P
Power transmitted = 1/2
9 1 T
f∝  
∴ 89% power transmitted r2  µ 
1/2
  1/2
Sol 14: (D) By superposition 1  yα∆TA  1  yα∆T 
∝   ; ∝  
2  m  2  ρ 
 
 A 

 20 
Sol 20: (C) y = A sin  πx  cos(1000 pt)
 3 
Sol 15: (B)  = 1m f = 300 Hz  20π  1
sin  x =
3 v 2  3  2
λ = ; f = ; λ = ; v = fl
2  3
20x π 1
2 = ; x= m
v = 300 × × 1 = 200 m/s 3 6 40
3
1
Distance = 2x = m = 5 cm
20
Sol 16: (A) y1 = 5 sin (ωt – kx)
y2 = – 5 cos(ωt – kx – 150°) Sol 21: (B)  = 0.4 m, m = 10–2 kg, T = 1.6 N
y1+ y2 = 5(sin(ωt – kx) – sin(ωt – kx – 60°)) 10 –2
µ= = 2.5 × 10–2 kg/m
= 10 (sin30° cos(ωt – kx – 30°)) 0.4

= 5 cos(ωt – kx – 30°) 1 1.6 1 4


f= = × = 10 Hz
2 × 0.4 2.5 × 10 –2 0.8 0.5

T = 0.1 s
P hysi cs | 11.53

Sol 22: (C) sin φ = sin(π – φ) = sin(1001t) + sin(999t) + 2sin(1000t)


i.e., the given expression is a result of superposition
of three independent harmonic motions of angular
frequencies 999, 1000 and 1001 rad/s.

A Sol 4: (A) From Hooke’s law


Sol 23: (B) Refer theory y = sin(kr – wt)
r Tension in a string (T) ∝ extension (x) and speed of
sound in string v = T / µ or v ∝ T
Sol 24: (C) At x = 4
Therefore, v ∝ x
 π  x is increased to 1.5 times i.e., speed will increase by
⇒ y = (2mm) sin  8π + – 100πt 
 3 
1.5 times of 1.22 times. Therefore speed of sound in
π  new position will be 1.22 v.
⇒ y = (2mm) sin  – 100πt 
3 
π 1 Sol 5: (A) This is an equation of a travelling wave
= 100pt ; t = sec
3 300 in which particles of the medium are in SHM and
maximum particle velocity in SHM is Aω, where A is the
Sol 25: (B) Vpmax = 2pfA amplitude and ω the angular velocity.

Vwave =
2πf
= fl Sol 6: (D) Fundamental frequency is given by
2π / λ
1 T
V= (with both the ends fixed)
2pfA = 4fl 2 µ
πA ∴Fundamental frequency
λ=
2 1
V∝ (for same tension in both strings)
 µ
Where µ = mass per unit length of wire
Previous Years’ Questions = ρ.A (ρ = density)
coefficient of t 1
Sol 1: (B) We velocity v = =
2πf =ρ(pr2) or µ ∝r∴v∝
coefficient of x 2π / λ r
= lf V1  r   2   r  2L 
∴ =  2   =    = 1
Maximum particle velocity vpm = ωA = 2pfy0 V2 
 r1  1   2r  L 
Given, Vpm = 4V or 2pf y0 = 4lf
πy 0 Sol 7: (C) Energy E ∝ (amplitude)2 (frequency)2
\λ =
2 Amplitude (A) is same in both the cases, but frequency
2ω in the second case is two times the frequency (ω) in
Sol 2: (C) For a stationary wave to form, two indentical the first case
waves should travel in opposite direction. Further at x Therefore, E2 = 4E1
= 0, resultant y (from both the waves) should be zero
at all instant.
Sol 8: (B) After two seconds both the pulses will move
4 cm towards each other. So by their superposition,
Sol 3: (B) The given equation can be written as the resultant displacement at every point will be zero.
t Therefore total energy will be purely in the form of
y = 2(2cos2 2 )sin (1000 t)
kinetic. Half of the particles will be moving upwards
and half downwards.
y = 2 (cos t + 1)sin (1000 t)
= 2cos t sin 1000 t + 2 sin (1000 t)
1 1 . 5 4 | Waves on a String

Sol 13: (B, C) ω = 15π, k = 10 p


ω
Speed of wave, v = = 1.5 m/s
k
2π 2π
Wavelength of wave λ = = = 0.2 m
Sol 9: (A) Let f0 = frequency of tuning fork k 10π
10π x and 15pt have the same sign. Therefore, wave is
Then, f0 =
5 9g
=
3 Mg traveling in negative x-direction.
2 µ 2 µ

(µ = mass per unit length of wire) Sol 14: (D) T1 > T2

Solving this, we get M = 25 kg \v1 > v2

In the first case, frequency corresponds to fifth or f1 > f2


harmonic while in the second case it corresponds to and f1 – f2 = 6 Hz
third harmonic.
Now, if T1 is increased, f1 will increase or f1 – f2 will
increase. Therefore, (d) option is wrong.
Sol 10: (A) f ∝ v ∝ T
If T1 is decreased, f1 will decrease and it may be possible
fAB = 2fCD that now f2 – f1 become 6 Hz. Therefore, (C) option is
\TAB = 4TCD  …(i) correct. Similarly, when T2 is increased, f2 will increase
and again f2-f1 may become equal to 6 Hz. So, (B) is also
Further Stp = 0 correct. But (A) is wrong.
\TAB(x) = TCD (l – x) or 4x = l – x
(TAB = 4TCD) Sol 15: (A, B, C, D) It is given that

or x = l/5 y(x, t) = 0.02cos (50pt + π/2)cos (10px)


≅ A cos (wt + π/2) cos kx
Sol 11: (B) The fundamental mode in a pipe closed at π 3π
one end and the second harmonic in a string are shown Node occurs when kx = , etc.
2 2
in figure. It can be seen that λ=
p /4 Lp and
= λs Ls .
π 3π
For the pipe closed at one end, 10px = ,
2 2
νp νp 320 ⇒ x = 0.05m, 0.15 m option (a)
v= = = = 100 Hz
Antinode occurs when kx = π, 2π, 3π etc.
p
λp 4Lp 4(0.8)

Where νp =320m/s is the velocity of sound in the pipe 10px = π, 2π, 3π etc.

and Lp=0.8m is length of the pipe. For string of mass m, ⇒ x = 0.1m, 0.2, 0.3 m option (b)

length Ls and having tension T, velocity of the string is Speed of the wave is given by,
given by, ω 50π
v= = = 5 m/s option (c)
k 10π
νs T / (m / LS ) T 50 10
v=
s = = = = Wavelength is given by,
λs Ls mLs m(0.5 m
2π 2π  1
λ= = =   m = 0.2 m
At resonance νp = νs substitute νp and νs from first
k 10π 5
and second equation to get m= 0.01kg=10 gram.
Sol 16: (B, C) Since, the edges are clamped,
displacement of the edges u(x, y) = 0 for
Sol 12: (A, C) options satisfy the condition;
∂2 y ∂2 y
= (constant)
∂x2 ∂t2
P hysi cs | 11.55

Sol 20: (A) y = 0.005 cos (αx − βt)


Comparing the equation with the standard form,

 x t  
=y A cos  −  2π 
 λ T  
2π/λ = α and 2π/T = β
α = 2π/0.08 = 25.00 π
β=π
Line, OA i.e., y = 0; 0 ≤ x ≤ L
ω2 (2π / 0.004)2
AB i.e., x = L; 0 ≤ y ≤ L Sol 21: (D) T =µv 2 =µ =0.04 =6.25N
k2 (2π / 0.50)2
BC i.e., y = L; 0 ≤ x ≤ L
OC i.e., x = 0; 0 ≤ y ≤ L b
Sol 22: (A) y (x,t) = e− ( ax + bt)2 V =
a
The above conditions are satisfied only in alternatives
(B) and (C). Wave moving in − ve x –direction.
Note that u (x, y) = 0, for all four values eg, in alternative φ1 =0
(D), u(x, y) = 0 for y = 0, y = L but it is not zero for x = π
0 or x = L. Similarly, in option (A) u (x, y) = 0 at x = L, y Sol 23: (D) φ1 =0 ; φ2 =
2
= L but it is not zero for x = 0 or y = 0 while in option φ2 =
π
(B) and (C), u (x, y) = 0 for x = 0, y = 0, x = L and y = L. 2
Sol 24: (C) Energy of simple harmonic oscillator is
Sol 17: (A) Maximum speed of any point on the string constant.
= aω = a(2pf)
1 1
⇒ 1 Mω22 A122= 1 (m + M)ω22 A22
v 10
∴= = = 1 (Given: v = 10 m/s) ⇒ 2 Mω A1= 2 (m + M)ω A2
2 2
10 10 A122 M + m
A12 = M + m
1 A2 = M
\2paf = 1 ; f= A2 M
2πa
A1 M+m
a = 10–3 m (Given) ∴ A1 = M+m
∴ A2 = M
1 103 A2 M
\f = = Hz
2π × 10 –3 2π
v 1 T 1 T
Speed of wave v = f l Sol 25: (B)=
f = =
2 2 µ 2 Ad
 103 –1 
∴(10 m/s) =  S  λ ; λ = 2π × 10–2m T T Y∆ 1 y∆
 2π  Also,
= Y =
⇒ ⇒f
=
  A∆ A  2 d

∆
Sol 18: (A, B) In case of sound wave, y can represent  1.5m, = 0.01,d
= = 7.7 × 103 kg / m3
pressure and displacement, while in case of an 
electromagnetic wave it represents electric and y 2.2 × 1011N / m2
=
magnetic fields. After solving
Note: In general, y is general physical quantity which is
made to oscillate at one place and these oscillations are 2 1033
=f 2 × 10 Hz
propagated to other places also. =f 7 × 3 Hz
7
f ≈ 178.2Hz3
f ≈ 178.2Hz
Sol 19: (A) Standing waves can be produced only when
two similar type of waves (same frequency and speed,
∆L 0.1
but amplitude may be different) travel in oposite Sol 26: (B) Given =
L 20
directions.
1 1 . 5 6 | Waves on a String

90 1
=T 90 sec. = ∆T 1 sec. T
=T 100 sec. = ∆T 100 sec. T= λgx v= T = gx
T= λgx v= T =
λ gx
100
90 100
1 T= λgx v= λ = gx
=T∆T =901 sec. = ∆T 1 sec. 2
v 2 = gx λ
= ∆ T
TT = 100
90 1
90 sec.
= ∆T 100 1 sec. v 2 = gx
=T∆TT 100 90
1 sec.
= ∆T 100 sec. v =vdvgx g
10011  L 100 =a vdv= g
g∆TT= = 90 1 =a vdv= g
∆ T = 1 2  L2 =a = dx 2
gT= =  490 π  T dx 2
 41 π2  TL2 ∆L 1g 2 2
dx 4
gT=∆g90 1  L 2∆T = 1 g t2 ⇒= t 4= 2 2 sec

g =∆g4×1 2  2 = ∆L × 100 + 2∆T × 100
100% = 2 1g t ⇒ t
= 4g=
 2 2 sec
g=g
π 
 L
T = 22 2
2 t ⇒= t g= 2 2 sec
⇒  4×π100% T = L × 100 + T × 100
2 2
∆g g4 π2  T2 ∆LL 2∆TT 22 g
⇒ 
∆g ∆g × 100% =0.1 ∆L100× 100 + 21∆T × 100
⇒∆gg ∆×gg100% =  =0.1
× 100%
20
+ 2+ 21∆
∆LL × 100 T T100=
× 100 2.72%
⇒ ×g100% =  = L100
× 100% + 2+ 90
× 100 T 100 =
× 100 2.72%
∆gg g 20L
 0.1  90
1T  Sol 30: (C)
∆g × 100% =  0.1  100 + 2  1 100
  100
= 2.72%
∆gg × 100% =  0.120 100 + 2  90 1  = 2.72%
g × 100% =  20 100 + 2  90 100 = 2.72%
g  20   90  2
 2A 2
v ω A22
= −  2A 2
So, nearest option is 3%. v ω A2
= −  2A 3 
v= ω A −  3 
Aω  3 
v = 5 Aω
Sol 27: (A) v = 5 A3ω
v= 5 3
v new
= 3v
=3 5Aω
 v new
= 3v
= 5Aω
T = 2π  v new
= 3v
= 5Aω
T = 2π g
g So the new amplitude is given by
 + ∆ Mg
TM = 2π  + ∆ ∆ = Mg 2
TM = 2π g ∆ = AY  2A 
g AY v new =ω Anew 2 − x2 ⇒ 5Aω =
ω Anew 2 −  
 3 
7A
TM  + ∆ Anew =
= 3
T 
2
 TM 2 ∆
 T T
M 2 = 1 + ∆

 = 1 + ∆ JEE Advanced/Boards
T
 T  = 1 + 
M
 T 2 
 T 2
 TM 
 TM 2 == 1 1+
Mg
Mg
+ Mg
Exercise 1
 T M
T  = 1 + AY
AY
 
 T 2 AY
 TM 2  A
= 1  T
1
M 2 − 1 A Sol 1: (a) D, E, F
= 1
y  TTM
 − 1 Mg
 A
y
=   − 1 T   Mg
y 
 T 
 Mg (b) A, B, H
  
(c) C, G
Sol 28: (B) K.E. is maximum at mean position, whereas (d) A, E
P.E. is minimum.
At extreme position, K.E. is minimum and P.E. is
maximum.

Sol 29: (B) Let mass per unit length be λ

X= Sol 2: (a) λ = 4 m


1
f= = 0.25 Hz
X T
V = fλ = 1 ms–1 in –ve n-direction
X=0
(b) Vmax. = 0.5 π × 3 mm s–1 = 1.5π mm s–1
P hysi cs | 11.57

amax. = 0.75p2 mm s–2 1 2


Ar = – cm; AT = cm
π π  3 3
(c) y = (3 mm) sin  x – t + π 
2 2 
Sol 7: V1 = 5 m/s
π 
= (3 mm) sin  (x – t + 2)  V2 = 2.5 m/s
2 
5 2
dy  3π  π AT = A= × 0.3 = 0.2 cm
=   cos (x –t + 2) 7.5 3
dx t  2 2
Sol 8:  = 0.4 m ; m = 3.2 g ; A = 1 mm2
Slope at x = 2m & t = 4 sec
3π π 3π 3.2
m= × 10–3 = 8 × 10–3
= cos (2 – 4 + 2) = 0.4
2 2 2
100 1 T
=
Sol 3: V ∝ T , V’ = 1.5 V 64 2 × 0.4 8 × 10 –3
2
 800 
450  × 8 × 10 = T
–3
Sol 4: µ = 5 ×10–3 kg/m ; V= =300 m/s 
5 × 10 –3  64 
10 4
T = 450 N × 5 × 10–3 = T
8×8
λ T = 1.25 N
A≤
5
T 1.25 × 0.4 0.5
Y= = = × 1010 Nm–2
1 ω2 A2F A∆ –6
10 × 5 × 10 –4 5
Pavg max. =
2 V Y = 109 Nm–2
1 f 2 λ2 450
= × 4p2 × Sol 9: µ = 0.2 kg/m
2 25 300
L = 0.6 m
2π2 (300)2
= × 450 × = 106.59 kW
25 300 T = 80 N

3 80 3
f= = × 20 = 50 Hz
2 × 0.6 0.2 1.2
Sol 5: Vmax = 2pfA
= 2π × 50 × 0.5 cms–1 = 50π cms–1

Vp = – slope × Vwave = – (–1) × – 2 cm/s Sol 10: T = 200 N

Vp = – 2 cm/s π 2π 2π
k= λ= = =4m
2 k π/2
–ve sign represents particle moving down
(a) Second harmonic

Sol 6: µA = 2 kg/mµB = 8 kg/m ∴=λ=4m

T = 200 N 12π
(b) V = = 24 ms–1
π/2
200 200 T200
VA= =10m/s VB= =5m/s (c) µ = =
2 8 V 24
2 × 24
VB VA
200
mass = µ ×  = × 4 = 1.39 kg
Ar = – A; AT = A 24 × 24
VA + VB VA + VB
3
(d) f = × 24 = 9 Hz
8
1 1 . 5 8 | Waves on a String

T=
1
;T=
1
sec Exercise 2
f 9
Single Correct Choice Type
T' 200
Sol 11: f=
T 199 Sol 1: (B) y = 20 sin 2π(100t) cos(2π(0.02x))
2
T'  200  Amax. = 20 units
=  
T  199  2π
= 2π(0.02)
T’ = 1.01007 λ
DT = T’ – T = 0.01007 λ = 50 units

% change in tension = 1.007% λ 50


Maximum loop length = = = 25 units
2 2
Sol 12: l1 = 36 cm l2 = 32 cm
Sol 2: (D) M = 0.01 kg m–1
2 2
l1 = = 36; l2 = = 32
n (n + 1) 1 100
T = 100N; f1 = = 50Hz
n +1 36 9 2 0.01
⇒ = =
n 32 8 1 T
⇒ 8n + 8 = 9n f1 = ; f2 = 2f1 ; f3 = 3f1
2 µ
⇒n=8
f1 3f1 5f1
n1 = ; n2 = ; n3 =
36 × 8 2 2 2
= = 144 cm
2
 = 1.44 m n T
Sol 3: (D) ν =
2 µ
1 10
(b) f0 = ν1 = ν2
2 × 1.44 4 × 10 –3
6× 36 = 4 × T
50
⇒ f0 =
2 × 1.44 362
T= = 81 N
f0 = 17.36 Hz 42

γRT Sol 4: (B) y = cos (70x – 500 t)


Sol 13: V =
M Transverse wave as particle oscillate perpendicular to
RT c the direction of motion
=
M 3 500 50
V= = m/s
2 4 70 7
γ=1+ =
6 3 250
f = 500 =
c 4 c π
V= γ× = × 2π 20π
3 3 3 λ= m= cm
70 7
2
V= c
3
(A) y A cos (kx − ωt )
Sol 5:=
For this the maximum occurs at x = 0 and at t = 0
For getting the maximum at t = 5 sec
kx = 5ω ⇒ x = 5ω / k
x = 5 v = 200 cm = 2m
P hysi cs | 11.59

Sol 6: (C) The speed of the transverse is given as Comprehension Type

T
V=
µ

Here, T is tension and is Mass per unit length. Now


putting values in above equation, we get

1000
V= 8 × 64 × × 5 = 32 m / s
100

Sol 12: (A, D) Upward


T 6.4 × 10
Sol 7: (A)=
ν ⇒ ν= = 80ms−1
µ AB
10 × 10 −3 a, g , h

3.2 × 10 Sol 13: (C) Downward


=
νCD = 63ms−1
8 × 10−3 c, d, e

T 20 Sol 14: (B, C) Stationary


Sol 8: (B)=
ν = = 100ms−1
µ 2 × 10−3
b, f
4.5
=
µ = 2gm−1
2.25 Sol 15: (C, D) Maximum velocity
1 o, d, h
⇒ s =µt − gt2
2
⇒ 2 = 100t-5t2
⇒ t2 - 20t + 0.4 = 0 Previous Years’ Questions
+20 ± 400 − 1.6
=⇒ t = 0.02s
2 Sol 1: (A) The diagramatic representation of the
given problem is shown in figures. The expression of
Assertion Reasoning Type 1 T
fundamental frequency is V =
2 µ
Sol 9: (D) Potential energy is maximum at the extremes
and particle oscillate in SHM.

Sol 10: (A) There cannot be a perpendicular force to


a string.

Sol 11: (C, D)


In air T = mg (Vρ)g

1 Aρg
\v = …(i)
2

µ
When the object is half immersed in water

V
T’ = mg – upthrust = Vrg –   ρw g
2
1 1 . 6 0 | Waves on a String

V Sol 5: (A) In one second number of maximas is called


=   g (2ρ – rw) the beat frequency. Hence,
2
100π 92π
The new fundamental frequency is fb = f1 – f2 = – = 4Hz
2π 2π
1 T' 1 (Vg / 2)(2ρ – ρw )
V’ = × =  …(ii) Sol 6: (A) Speed of wave v =
ω
2 µ 2 µ k
 2ρ – ρ  100π 92π
v' or v = or = 200 m/s
∴ =  w  0.5π 0.46π
v  2ρ 
 
1/2 1/2 Sol 7: (C) At x = 0, y =y1 + y2
 2ρ – ρw   2ρ – 1 
or v’ = v   = 300   Hz = 2A cos 96pt cos 4pt
 2ρ   2ρ 
Frequency of cos (96 pt) function is 48 Hz and that of
cos (4pt) function is 2 Hz.
Sol 2: (B) Mass per unit length of the string.
10 –2 In one second, cos function becomes zero at 2f times,
m= = 2.5 × 10–2 kg/m where f is the frequency. Therefore, first function will
0.4
become zero at 96 times and the second at 4 times.
\Velocity of wave in the string, But second will not overlap with first. Hence, net y will
become zero 100 times in 1 s.
T 1.6
V= =
m 2.5 × 10 –2
Sol 8: (A, B, C, D) y = 10–4 sin (60t + 2x)
v = 8 m/s A = 10–4 m, ω = 60 rad/s, k = 2m–1
For constructive interference between successive pulses ω
Speed of wave, v = = 30 m/s
2 (2)(0.4) k
Dtmin = = = 0.10s
v 8 ω 30
Frequency, f = = Hz.
(After two reflections, the wave pulse is in same phase 2π π
as it was produced, since in one reflection its phase 2π
changes by π, and if at this moment next identical Wavelength λ = = pm
k
pulse is produced, then constructive interference will
Further, 60 t and 2x are of same sign. Therefore, the
be obtained.)
wave should travel in negative x-direction.

Sol 3: (A) Particle velocity vp = – v (slope of y-x graph)


Sol 9: (A) For a plane wave intesity (energy crossing
Here, v = + ve, as the wave is traveling in positive per unit area per unit time) is constant at all points.
x-direction.
Slope at P is negative.
∴ Velocity of particle is in positive y (or ˆj ) direction.

Sol 4: (A) With increase in tension, frequency of


vibrating string will increase. Since number of beats are
decreasing. Therefore, frequency of vibrating string or
third harmonic frequency of closed pipe should be less
that the frequency tuning fork by 4.
∴Frequency of tuning fork But for a spherical wave, intensity at a distance r from
= Third harmonic frequency of closed pipe +4 a point source of power P (energy transmitted per unit
time) is given by
 v   340 
= 3  + 4 = 3  + 4 = 344 Hz I=
P
or I ∝
1
4
   4 × 0.75  4 πr 2
r2
P hysi cs | 11.61

(a) (b)
Value of y is again 0.16 m, i.e., pulse has traveled a
1 distance of 1.25 m in 1 s in negative x-direction or we
Note: for a line source I ∝
r can say that the speed of pulse is 1.25 m/s and it is
P traveling in negative x-direction. Therefore, it will travel
Because, I = a distance of 2.5 m in 2 s. The above statement can be
πr
better understood from figure (b)
Sol 10: (A) The shape of pulse at x = 0 t = 0 would be
as shown, in figure(a). Sol 11: Tension due to thermal stresses,
T = YA α . Dq

T
v=
µ

Hence, µ = mass per unit length = ρA

T YAα.∆θ Yα ∆θ
\v = = =
ρA ρA ρ

Substituting the values we have,


0.8
Y (0, 0) = = 0.16m 1.3 × 1011 × 1.7 × 10 –5 × 20
5 v= =70.1 m/s
From the figure it is clear that ymax = 0.16 m 9 × 103

Pulse will be symmetric (Symmetry is checked about


ymax) if at t = 0 Sol 12: By decreasing the tension in the string beat
frequency is decreasing, it means frequency of string
y(x) = y(–x) was greater than frequency of pipe. Thus,
From the given equation First overtone frequency of string–Fundamental
0.8  frequence of closed pipe = 8
y(x) = 
2
And 16x + 5  at t = 0  v   v 
0.8 
 ∴ 2  1  –  2  = 8
y(–x) =  21   4 2 
16x2 + 5 
 v 
or y (x) = y (– x) or v1 = 1 8 + 2 
 4 2 
Therefore, pulse is symmetric.
Speed of pulse, at t = 1s, and x = – 1.25 m Substituting the value, we have
 320 
v1 = 0.25 8 +  = 52 m/s
 4 × 0.4 

T
Now, v1 =
µ
1 1 . 6 2 | Waves on a String

m  2.5 × 10 –3  Sol 16: (B, C) y = 0.01 m sin (20 π x) cos 200 π t
\T = µv12 =   v12 =   (52)2 =27. 04 N
  0.25 
 

Sol 13: v = T/µ

v top Ttop 6+2 No. of nodes is 6


= = = 2  …(i)
v bottom Tbottom 2 2π
20π =
λ
Frequency will remain unchanged. Therefore, equation 1
=
∴λ = m 0.1m
fλ top 10
(i) can be written as, =2 1
fλbottom Length of the spring = 0.5 × = 0.25
2
Or ltop = 2 (lbottom) = 2 × 0.06 = 0.12 m
Mid point is the antinode
Sol 14: The temperature stress is σ = YaDq 200π
Frequency at this mode=
is f = 100Hz
or tension in the steel wire T = σA = YAαDq 2π
100
Substituting the values, we have ∴ Fundamental frequency
= = 20Hz
5
T = (2 × 1011) (10–6) (1.21 × 10–5) (20) = 48. 4 N
T Sol 17: (A, C, D) Taking y(t) = A f(x) g(t) & Applying the
Speed of transverse wave on the wire, v = conditions:
µ
1; here x = 3m is antinode & x = 0 is node
2; possible frequencies are odd multiple of fundamental
frequency.
v 25
where, ν fundamental = = Hz
4 3
Hence, µ = mass per unit length of wire = 0.1 kg/m The correct options are A, C, D.

48.4
\v = = 22 m/s Sol 18:
0.1
x P (10 - )x
v 22
Fundamental frequency f0 = = = 11 Hz
2 2 × 1 o
0 C (ice)
o
400 C
o
100 C (steam)

Sol 15:
dmice dmvapour
=
Aeq = A12 + A22 + 2A1A2 cos φ dt dt
π 400kS 300kS
=
Aeq = 42 + 32 + 2(4)(3)cos λxLice (100 − λ )xL vapour
2
Aeq = 5 λ =9

Sol 19: A → p, t; B → p, s; C → q, s; D → q, r
2017-18 100 &
op kers
Class 11 T
By E ran culty
-JE Fa r
IIT enior emie .
S fP r es
o titut
Ins

PHYSICS
FOR JEE MAIN & ADVANCED
SECOND
EDITION

Exhaustive Theory
(Now Revised)

Formula Sheet
9000+ Problems
based on latest JEE pattern

2500 + 1000 (New) Problems


of previous 35 years of
AIEEE (JEE Main) and IIT-JEE (JEE Adv)

5000+Illustrations and Solved Examples


Detailed Solutions
of all problems available

Topic Covered Plancess Concepts


Tips & Tricks, Facts, Notes, Misconceptions,
Sound Waves Key Take Aways, Problem Solving Tactics

PlancEssential
Questions recommended for revision
12. S O U N D WAV E S

1. INTRODUCTION
This chapter discusses the nature of sound waves. We will apply concepts learned in the chapter on waves on a
string are applied to understand the phenomena related to sound waves. We will learn about what all parameters
the speed of sound in a medium depends. Reflection, transmission and interference are important phenomena
associated with sound. The study of sound waves enables us to design musical instruments and auditoriums. We
will understand the properties of sound waves in air columns and the phenomena of echo. Phenomena of beats
and doppler effect have been discussed.

2. NATURE AND PROPAGATION OF SOUND WAVES


Sound is a mechanical wave that results from the back and forth vibration of the particles of a medium through
which the sound wave is traveling. Further, if a sound wave is traveling from left to right in air, then particles in air
will be displaced in both rightward and leftward directions due to the energy of the sound wave passing through
it. However, the motion of the particles is parallel (and antiparallel) to the direction of the energy transport. This
unique property characterizes sound waves in air as longitudinal waves.
A typical case of propagation of sound waves in air p
is shown in the Fig. 12.1.
We know that as the prong vibrates in simple p0
x
harmonic motion, the pressure variations in the
layer close to the prong also change in a simple
harmonic fashion. Thus, the increase in pressure
above its normal value may, therefore, be written as p
δP =P − P0 =δP0 sin ωt ,
x
where δP0 is the maximum increase in pressure p0
above its normal value. As this disturbance, due to
the traveling of the sound wave, moves towards
right with the speed u (the above speed and not p
the particle speed), the equation for the excess
pressure at any point x at any time t is given by x
δP =δP0 sin ω ( t − x / v ) . p0
Figure 12.1
2.1 Compression and Rarefaction
Due to the phenomenon of longitudinal motion of the air particles, we observe that there are regions in the air
where the air particles are compressed together and other regions where the is air spread apart. These regions are
1 2 . 2 | Sound Waves

respectively known as compression and rarefaction. Sound is a pressure wave


The formation of these regions is due to back and
forth motion of the particles of the medium.
Compression is the result of increase in density and
pressure in a medium, such as air, due to the passage C R C R C R C R C R
of a sound wave. However, rarefaction is quite just the
opposite of compression, i.e., decrease in density and

Pressure
pressure in a medium due to the passage of a sound 0
Time
wave.

Note: "C" stands for compression and "R" stands for rarefaction

Figure 12.2
2.2 Wavelength
The wavelength of a wave is just the distance that a disturbance is carried Wavelength
along the medium when one wave cycle is completed. A longitudinal
wave typically consists of a repeating pattern of compressions and Crest Crest
rarefactions.
Hence, the wavelength is commonly measured either as the distance
from one compression point to the next adjacent compression or the
distance from one rarefaction point to the next adjacent rarefaction. Trough

2.3 Polarization of Sound Waves


Figure 12.3
It is to be noted that all directions perpendicular to the propagation
of sound waves are equivalent and therefore sound waves cannot be
polarized.

2.4 Wave Front


A wave front usually is the locus of point that is having the same phase, i.e., a line or curve in 2d, or a surface for
a wave propagating in 3d. Further, the sound observed at some point by a vibrating source travels virtually in all
directions of the medium only if the medium is extended. However, for a homogenous and isotropic medium, the
wave fronts are usually normal to the direction of propagation.

2.5 Infrasonic and Ultrasonic Sound Waves


Sound waves are audible only if the frequency of alternation of pressure is in the range of 20 Hz and 20,000Hz.
In other words, beyond this upper limit they are not audible. The waves are classified based on their frequency
range, i.e., below the audible range they are called infrasonic waves, whereas those with frequency greater than the
audible range are termed ultrasonic waves.

Illustration 1: A wave of wavelength of 0.60 cm is produced in air and it travels at a speed of 300 ms–1. Will it be
audible?  (JEE MAIN)
V
Sol: The frequency of the sound wave is given as nν =
= . The audible range is 20 Hz to 20 KHz.
λ
V
From the relation V = νλ , we calculate the frequency of the wave as nν =
=
λ
300 ms−1
= = 50000 Hz
0.60 × 10−2 m

This is clearly very much above the audible range. Therefore, it is an ultrasonic wave and hence will not be audible.
P hysi cs | 12.3

2.6 Displacement Wave and Pressure Wave


A longitudinal wave can be described either in terms of longitudinal displacement of the particle of the medium or
in terms of excess pressure generated due to phenomena of compression and rarefaction.

3. EQUATION OF SOUND WAVE


As we have already noted above, a longitudinal wave in a fluid (liquid or gas) s s+s
can be described either in terms of the longitudinal displacement suffered
by the particles of the medium or in terms of the excess pressure generated
due to the compression or rarefaction. Let us now verify how these two
representations are related to each other. x x+x
Consider a wave traveling in the x-direction in a fluid. Further, now suppose Figure 12.4
that at time t, the particle at the undisturbed position x suffers a displacement
s in the x-direction. The wave can then be described by the equation
s s0 sin ω ( t − x / v ) 
= … (i)
Now, consider the element of the material which is within x and x + ∆x (see the fig. 12.4) in the undisturbed
state. Therefore, by considering a cross-sectional area A, the volume of the element in the undisturbed
state is A∆x and its mass is r A∆x . As the wave passes through, the ends at x and x + ∆x are displaced by
amount s and s + ∆s according to Eq. (i). Thus, the increase in volume of the element at time t is given as
δs
∆V= A ∆s= A ∆x= As0 ( −ω / v ) cos ω ( t − x / v ) ∆x
δx
(where ∆s has been obtained by differentiating Eq. (i) with respect to x. The element is, therefore, under a volume
∆v −As0 ω cos ( t − x / v ) −s0 ω
strain.
= = cos ω ( t − x / v ) )
v vA∆x v
 −∆v 
However, the corresponding stress, i.e., the excess pressure developed in the element at x at time t is p = B  ,
 v 
where B is the bulk modulus of the material.
s0 ω
Thus,
= p B cos ω ( t − x / v )  … (ii)
v
Comparing with standard wave equation, we see that the amplitude p0 and the displacement amplitude s0 are

related as
= p0 = s Bks0 (where k is the wave number)
v 0
Also, we observe from (i) and (ii) that the pressure wave
differs in phase by π / 2 from the displacement wave.
Further, the pressure maxima observed is at the point where
the displacement is zero and displacement maxima occur
where the pressure is at its normal level.
The assertion here being that displacement is zero where the Figure 12.5
pressure change is maximum and vice versa, and therefore
sets the two descriptions on different footings. Naturally, the human ear or an electronic detector responds only
to the change in pressure and not to the displacement. Let us suppose that two audio speakers are driven by the
same amplifier and are placed facing each other. Further, a detector is placed midway between them. Now, it is
clear that the displacement of the air particles near the detector will be zero as the two sources drive these particles
in opposite directions. However, both the sources send compression waves and rarefaction waves together.
1 2 . 4 | Sound Waves

PLANCESS CONCEPTS

The human ear or an electronic detector responds to the pressure change and not the displacement in
a straightforward way.
Vaibhav Krishnan (JEE 2009, AIR 22)

Illustration 2: Suppose that a sound wave of wavelength 40 cm travels in air. If the difference between the maximum
and minimum pressures at a given a point is 1.0 × 10−3 Nm−2 , then find the amplitude of vibration of the particles of
the medium. The bulk modulus of air is 1.4 × 105 Nm−2 .  (JEE MAIN)
Pmax − Pmin
Sol: The amplitude of pressure at a point is given by Po = . As the bulk modulus of the air is given, the
2
P0
amplitude of the vibration is given by S0 = where k is wave number.
Bk
1.0 × 10−3 Nm−2
The pressure amplitude is P0 = = 0.5 × 1.0 × 10−3 Nm−2
2

The displacement amplitude s0 is given by P0 = Bks0

or =
s0
P0
= 0

=
(
0.5 × 1.0 × 10−3 Nm−2 × 40 × 10−2 m ) 2.2 × 10−10 m.
Bk 2πB 2 × 3.14 × 1.4 × 105 Nm−2

Illustration 3: Assume that a wave is propagating on a long stretched string along its length taken as the positive
2
t x
x-axis. The wave equation is given
= as y y 0 exp  −  where y0 = 4 mm, T =1 s, and λ =4 cm. Now, (a) Find the
velocity of the wave. T λ

(b) Find the function f ( t ) giving the displacement of the particle at x = 0.

(c) Find the function g ( x ) giving the shape of the string at t = 0.

(d) Plot the shape g ( x ) of the string at t = 0.

(e) Plot the shape of the string at t = 5 s.  (JEE MAIN)

Sol: The wave moves having natural frequency of ν and wavelength λ has velocity V = νλ . As the frequency is
1 λ
ν = the velocity of the wave is then V = .
T T
1 x  
2
(a) The wave equation may be written
= as y y 0 exp   t −  
T  λ/T 
 
λ 4cm
Comparing with the general equation, = y f ( t − x / v ) we see that ν= = / sec
T 1.0 s
2
(b) Substituting x = 0 in the given equation, we have f ( t ) = y 0 e ( ) 
− t/T
… (i)
2
(c) Substituting t = 0 in the given equation, we have g ( x ) = y 0 e ( ) 
− x/ λ
… (ii)
P hysi cs | 12.5

(d) (e)

x=0 x=0 x=20 cm


Figure 12.6

4. VELOCITY OF SOUND WAVES


We know that the sound waves travel in air or in gaseous media as longitudinal waves. Further, when these waves
travel longitudinally, then compression and rarefaction are produced in the layers of air in such a way that the
particles in layers of the air move in a to and fro fashion about their mean position in the direction exactly as that
of the direction of propagation of sound waves. Therefore, the speed v of longitudinal waves in an elastic medium
E
of modulus of elastic E and density ρ is given by v = .
ρ
For both liquids and gases, E is the bulk modulus of elasticity. For a thin solid rod, E is Young‘s modulus. However,
for large solids, E depends upon the bulk modulus and shear modulus.
Newton assumed that the changes produced due to propagation of sound in gases are isothermal; this implies that
a compressed layer of air at higher temperature loses heat immediately to the surroundings, whereas a rarefied
layer of at lower temperature gains heat from the surroundings so that temperature of air remains constant. As the
modulus of elasticity for isothermal change is equal to the pressure P according to Newton’s formula for change,
P
v= .
ρ
Laplace showed that the sound is propagated in air or gases under adiabatic change. This is because the compression
and rarefactions produced due to the propagation of sound follow each other so rapidly that there is no time
available for the compressed layer at a higher temperature and rarefied layer at a lower temperature to equalize
their temperature with the surroundings. Thus, the velocity v of sound travelling under adiabatic conditions in a
gas is given by Laplace’s formula as:

Eadiabatic γP Cp
=v = ; because Eadiabatic = γ P and γ =
ρ ρ Cv

By substituting γ = 1.41 for air, density of air = 1.293 kg/m3, atmospheric pressure = 1.013 ×105 N / m2 , the velocity
of sound in air, v = 332 m/s. However, in general, the velocity of sound in solid is greater than the velocity of sound
in liquids and the velocity of sound in liquids is greater than the velocity of sound in gases.

4.1 Sound Wave in Solids


Usually, sound waves travel in solids just like they travel in fluids. The speed of longitudinal sound waves in a solid
rod can be shown to be
= v Y/ρ ,
where Y is the Young’s modulus of the solids and ρ its density.
However, for extended solids, the speed is a more complicated function of bulk modulus and shear modulus. The
table provided hereunder gives the speed of sound in some common materials.

Medium Speed (m/s) Medium Speed (m/s)


Air (dry 0 C)
0
332 Copper 3810
Hydrogen 1330 Aluminum 5000
Water 1486 Steel 5200
1 2 . 6 | Sound Waves

4.2 Sound Wave in Fluids


A sound wave in air is a typical longitudinal wave. As a sound wave passes through air, its potential energy is usually
associated with periodic compression and expansion of small volume element of the air. The unique property
that determines the extent to which an element in the medium changes its volume as the pressure applied to it
−∆P
increases or decreases is the bulk modulus, B. B =
∆V / V
∆V
where is the fractional change in volume produced by a change in
V
pressure ∆P . Air
PA ,p

Let us now suppose that air of density ρ is filled inside a tube of cross- vt
sectional area A under a pressure P. Initially, the air is at rest.
ut (I)
At t = 0, the piston at the left end of the tube (as shown in the Fig. 12.7) is
set to motion toward the right with a speed µ . After a time interval ∆t , all
(P+P)A PA
portions of the air to the left of section 1 are moving with speed u, whereas all
portions to the right of the section are at rest. Further, the boundary between
the moving and the stationary portion travels to the right with v, the speed
of the elastic wave (or sound wave). In the time interval ∆t , the piston has Figure 12.7
moved u ∆t and the elastic disturbance has moved across a distance of v t .

B
The mass of air that has attained a velocity u in time ∆t is taken as v = P ( ∆x ) A. Therefore, now the momentum
imparted is Pv ( ∆t ) A  u and the net impulse = ( ∆PA ) .∆t . ρ

Thus, impulse = change in momentum ( ∆PA ) .∆=


t Pv ( ∆t ) A.  u or ∆=
P Pvu …(xv)

∆P  ∆V 
Since B = ∴ ∆P =B  where V= Av∆t and ∆V= Au ∆t
∆V / V  V 
∆V Au ∆t u u
∴ = = thus, ∆P =B  …(xvi)
V Av∆t v v

B
From (xv) and (xvi) v = .
P

4.3 Speed of Sound in a Gas: Newton’s Formula and Laplace Correction


The speed of sound in a gas can be expressed in terms of its pressure and density. We now summarize these
properties hereunder:
PV
(a) For a given mass of an ideal gas, the pressure, volume and the temperature are related as = constant.
T
However, if the temperature remains constant (called an isothermal process), then the pressure and the
volume of a given mass of a gas satisfy PV = constant. Here, T is the absolute temperature of the gas. This is
known as Boyle’s law.
(b) If no heat is supplied to a given mass of a gas (called an adiabatic process), then its pressure and volume
satisfy PV γ = constant where γ is a constant for the given gas. It is, in fact, the ratio Cp / CV of two specific
heat capacities of the gas.
Newton assumed that when a sound wave is propagated through a gas, the temperature variation in the layer of
compression and rarefaction is negligible. Hence, the condition here is isothermal and hence Boyle’s law will be
applicable.
∆P
PV =cons tant or, P∆V + V∆P =0 or, B =− =P  …… (i)
∆V / V
P hysi cs | 12.7

Using the above result, the speed of sound in the gas is given by
= v P/ρ .
Laplace, however, suggested that the compression or rarefaction takes place too rapidly and the gas element
being compressed or rarefied is hardly left with enough time to exchange heat with the surroundings. It is hence
an adiabatic process and therefore one should use the equation PV γ = constant. Taking logarithms, in P+in V =
constant.
∆P ∆V ∆P
Now, by taking differentials, +γ =0 or B = − = γP
P V ∆V / V
γP
Thus, the speed of sound is v = .
ρ

5. EFFECT OF PRESSURE, TEMPERATURE AND HUMIDITY AND


SPEED OF SOUND IN AIR
m γPV γRT
(a) Effect of temperature as PV = nRT and ρ = v
∴= = where Mm is mass of one mole of gas.
V m Mm

Thus, the velocity of sound is directly proportional to the square root of the absolute temperature. If vt and v0
Vt Tt 273 + t
are velocities of sound at t0C and 0 0C, respectively, then = =
V0 T0 273

where Tt and T0 are respective absolute temperatures...


(b) Effect of pressure. If the temperature of the gas remains constant, then the velocity of sound does not change
p
with the change of pressure because is a constant quantity. PM = rRT
ρ
(c) Effect of humidity. As the density of water vapor at STP, 0.8kg/m3, is lower than the density of dry air, 1.29km/
m3, the speed of sound in air increases when the humidity increases in the moist air.

6. INTENSITY OF SOUND
Normally, the intensity of sound I at any point is the quantum of energy transmitted per second across a unit area
normal to the direction of propagation of sound waves. The intensity follows the pattern of an inverse square law
of distance, i.e., I ∝ 1 2 and I is proportional to the square of amplitude. Further, the level of intensity of sound
R
as perceived by humans is called loudness. Thus, the intensity level or loudness L is quantitatively measured as
compared to a minimum intensity of sound audible to human ear. Hence, the intensity level or loudness, measured

in unit of decibel, dB, is given as L = 10 log10  I  ..


I 
0
where I0 is the minimum audible intensity which is equal to 10–12 watt/m2. Thus, the intensity of sound increases by
a factor of 10 when the intensity level or loudness increases by 10 decibels.
Now, let us consider again a sound wave travelling along the x-direction. Let the equation for the displacement of
the particles and the excess pressure developed by the wave be given as

=s s0 sin ω (t − x/ v) and=
P P0 cos ω (t − x/ v) … (i)

Bωs0
where P0 =
v
1 2 . 8 | Sound Waves

Now, consider a cross section of area ‘A’ perpendicular to x-direction. The power W, transmitted by the wave across
δs Aω2s02B
the section considered is W = (PA ) = ; W AP0 cos ω ( t − x / v ) ωs0 cos ω ( t=
− x / v) cos2 ω ( t − x / v )
δt n
2 2
1 ω s0B 2π2B 2 2 P02u
The intensity ‘I’ is=thus I = s V .; I=
2 u u 0 2B
v P02
As B = Pv −2 , the intensity can also be written
= as I = P02 .
2pv 2 2pv

Loudness: Our ear is sensitive for an extremely large range of intensity. Therefore, a logarithmic rather than a linear
scale in this regard is convenient. Accordingly, the intensity level β of t = a sound wave is defined by the equation
I
β =10log   Decibel , where I0 = 10 −12 W / m2 is the reference or threshold intensity level to which any intensity
I 
0
I is compared.

PLANCESS CONCEPTS

Intensity is directly proportional to the square of the pressure amplitude.


Nivvedan (JEE 2009, AIR 113)

Illustration 4: Assume that the pressure amplitude in a sound wave from a radio receiver is 2.0 × 10−2 Nm−2 and
the intensity at a point is 5.0 × 10−7 Wm−2 . If by turning the “volume” knob the pressure amplitude is increased to
2.5 × 10 −2 Nm−2 , then evaluate the intensity.  (JEE MAIN)

Sol: The intensity of the wave is proportional to square of the pressure amplitude of wave. If we increase the
pressure amplitude then the intensity of sound will accordingly.
The intensity is proportional to the square of the pressure amplitude.
2 2 2
I'  P'0   P'   2.5  −7 −2 −7 −2
Thus, = I'  0
or = =I   × 5.0 × 10 Wm = 7.8 × 10 Wm
I  P  P  2.0
 0   0   

7. PERCEPTION OF SOUND TO HUMAN EAR


There are three parameters which govern the perception of sound to human ear. They are listed hereunder.
(a) Pitch and frequency,
(b) Loudness, and
(c) Quality and waveform.

7.1 Pitch and Frequency


The frequency of a wave generally signifies how often the particles of the medium vibrate when a wave travels
through the medium. It is measured as the number of complete back-and-forth vibrations of a particle of the
medium per unit of time. Further, the sensation of frequency is commonly referred to as the pitch of a sound.
Therefore, a high pitch sound generally corresponds to a frequency sound wave and a low pitch sound corresponds
to a low frequency sound wave. Our ability to perceive pitch is associated with the frequency of the sound wave
that impinges upon our ear. This is because sound waves travelling through air are longitudinal waves that produce
high- and low-pressure disturbances of the particles of the air at a given frequency. Therefore, our ear has an ability
to detect such frequencies and associate them with the pitch of the sound.
P hysi cs | 12.9

7.2 Loudness
Loudness is that characteristic of a sound that is primarily a psychological correlate of physical strength (amplitude).
However, more formally it is defined as “that attribute of auditory sensation in terms of which sounds can be
ordered on a scale extending from quiet to loud.” Further, loudness is also affected by parameters other than sound
pressure, including frequency, bandwidth and duration.

7.3 Quality of Waveform


The quality of sound is typically an assessment of the accuracy, enjoyability, or intelligibility of audio output from an
electronic device. Therefore, quality of sound can be measured objectively, such as when tools are used to gauge
the accuracy with which the device reproduces an original sound; or it can be measured subjectively, such as when
we respond to the sound or gauge its perceived similarity to another sound. Thus, we differentiate between the
sound from a table and that from a mridang on the basis of their quality alone.

Illustration 5: Suppose that a source emitting sound of frequency 180 Hz is placed in front of a wall at a distance
of 2 m from it. Further, a detector is also placed in front of the wall at the same distance from it. Find the minimum
distance between the source and the detector for which the detector detects a maximum loudness. Speed of sound
in air = 360 m/s. (JEE ADVANCED)
Sol: As there is a wall at a distance of 2 m from the source, the wave will reflect from the wall and interfere with the
wave directly from the source. If constructive interference takes place between the reflected wave and original wave
then the maximum loudness is heard. The condition of constrictive interference is ∆ = nλ .
The situation is visualized in the Fig. 12.8. Now, suppose that the detector is placed at a distance of x meter from
the sources. Then, the wave received from the source after reflection from the wall has travelled a distance of
1/2   1/2 
 x2  
2 ( 2 ) + x 2 / 4 
2
is ∆ 2 ( 2 ) +  − x  m.
2
m. Therefore, the difference between the two waves =
 
  4  
However, constructive interference will take place when ∆ = λ ,2λ . Thus, the minimum distance x for a maximum
loudness corresponds to ∆ = λ  … (i)
u 360m / s −1
The wavelength is λ= = s = 2m S
ν 180
1/2
1/2  x2  x
Thus, by (i), 2 ( 2 ) + x2 / 4 
2
−x =2 or,  4 +  =1+ x
   4  2

x2 x2
Or, 4 + =1 + +x or 3 = x.
4 4
Figure 12.8
Thus, condition here is that the detector should be placed at a distance of 3 m from the source.
Note, however, that there is no abrupt phase change.

8. INTERFERENCE OF SOUND WAVES


Superposition of waves: When two or more waves travelling in the same direction act on the particles simultaneously,
then the intensity of the resultant wave is modified due to superposition of the wave according to the principle
discussed hereunder.
Superposition principle: If two or more waves arrive at a point simultaneously, then displacement at any point is
equal to the vector sum of the displacement due to individual waves:
∴ y = y1 + y 2 + ...... + yn
where y is the resultant displacement due to the superposition of displacement y1 , y 2 ……………. yn
1 2 . 1 0 | Sound Waves

Superposition can, in turn, give rise to the following phenomena:


Interference: When two waves of the same frequency and of constant phase difference travelling in the same
direction superpose, then they can effect modification in intensity in the form of alternate maximum and minimum
intensities which is called the interference phenomenon.
If the waves y1 = a1 sin ( ωt − kx ) and y 2 = a2 sin ( ωt − kx + φ ) superimpose, then by applying the principle of
superposition, y = y1 + y 2 = R sin ( ωt − kx + φ )

 a1 sin φ 
where the resultant amplitude, R = a12 + a22 + 2a1a2 cos φ and phase angle θ =tan−1  
 a1 + a2 cos φ 
When φ = 2πn where n = 0, 1, 2… it produces constructive interference which gives R = Rmax = a1+ a2 .
φ (2n + 1)π where n = 0, 1, 2…, R = Rmin = a1– a2 or amplitude is minimum due to destructive
However, when =
interference.
As intensity is proportional to the square of amplitude, the ratio of maximum intensity, Imax , to minimum intensity ,

( a1 + a2 )
2
Imax
Imin , is given by = .
Imin
( a1 − a2 )
2

8.1 Coherent and Incoherent Sources


Two sources are called coherent sources only when their phase
S1
difference remains constant in time. In case if the phase difference x
does not remain constant in time, then the sources are incoherent.
P
The Fig. 12.9 here shows two tuning forks s1 and s 2, placed side by
side, which vibrate with equal frequency and equal amplitude. The S2 x+x
point p is situated at a distance x from s1 and x + ∆x from S2.
Now, suppose that the two forks are vibrating in phase so that Figure 12.9
δ0 = 0 . Also, let p01 and p02 be the amplitudes of the wave from
s1 and s2, respectively. Now, let us examine the resultant change in pressure at a point p. The pressure change at A
due to the two waves are described by

=P1 P01 sin (kx −=


ωt ) ; P2 P02 sin k ( x + ∆=
x ) − ωt  P02 sin (kx − ωt ) + δ  ,

2πx
where δ = k∆x =  … (i)
λ
is the phase difference between two waves reaching P.

The resultant wave is thus given


= by p p0 sin (kx − ωt ) + δ  where p20 = p01
2
+ p202 + 2p01p02 cos δ

p02 sin δ
And tan ε =
p01 + p02 cos δ

The resultant amplitude is maximum when δ= 2nπ and minimum when = δ ( 2n + 1) π where n is an integer. These
are correspondingly the conditions for constructive and destructive interference:
δ= 2nπ constructive interference
=
δ ( 2n + 1) π destructive interference … (ii)

Using Eq. (i), i.e., δ = ∆x , these conditions may be written in terms of the path difference as ∆x = nλ (constructive)
λ
or ∆x = (n + 1/ 2 ) λ (destructive)  … (iii)

At constructive interference, p0 = p01 + p02 .


P hysi cs | 12.11

And at destructive interference, =


p0 p01 + p02
.
However, if the sources have an initial phase difference δ0 between them, then the wave reaching ‘p’ at time t is
represented
= t  and p p02 sin k ( x + ∆x ) − ωt + δ0 
by p p01 sin kx − ω=

2π∆x
The phase difference between these waves, therefore, is δ =δ0 + k∆x =δ0 + .
λ

Illustration 6: Two sound waves, originating from the same source, travel along different paths in air and then
meet at a point. Now, if the source vibrates at frequency of 1.0 KHz and one path is 83 cm longer than the other,
what will be the nature of interference? The speed of sound in air is 33 ms–1  (JEE ADVANCED)

Sol: The phase difference between the sound waves, is given by =
δ ∆x where λ is the wavelength of the wave
and ∆x is the path difference between the waves λ

u 332 ms−1
The wavelength of sound wave
= is λ = ; = 0.332 m
ν 1.0 × 103 Hz
The phase difference between the waves arriving at the point of observation is
2π 0.83m
δ= ∆x = 2π × = 2π × 2.5 = 5π
λ 0.332m
As this is an odd multiple of π, the waves interfere destructively.

9. REFLECTION OF SOUND
When there is discontinuity in the medium, sound waves obviously gets reflected. Therefore, when a sound wave
gets reflected from a rigid boundary, then the particles at the boundary are unable to vibrate. Thus, a reflected
wave is generated which interferes with the incoming wave to produce zero displacement at the boundary. At
these points, however, the pressure variation is maximum. Thus, a reflected pressure wave has the same phase as
the indicated wave.
Alternatively, a sound wave can also get reflected if it encounters a low pressure region. The reflected pressure
wave interferes destructively with the incoming waves in this case. Thus, there is a phase change of ∏ in this case.

10. STANDING/LONGITUDINAL WAVES


When two progressive waves of the same frequency moving in the opposite direction superpose, then stationary
waves are formed.
Let us now consider the superposition of two such waves along a stretched string having fixed ends where a

harmonic wave travels toward right as y1= asin ( vt − x ) . This wave is reflected from the second point and due
λ
to the reflection, its amplitude becomes −a due to phase change of π . Further, the reflected wave y2 = a sin

λ
( vt − x ) travels toward left and these waves superpose due to a phase change of pi, and hence give the resultant
displacement y.

2π 2π  2πvt  2πx  2πvt  where A = 2a sin  2πvt 


y=asin
λ
( vt − x ) − asin ( vt + x ) =
λ
−2acos   sin =−A cos   
 λ 

 λ  λ  λ 
The strings here apparently execute harmonic motion such that the particles of the string vibrate with the same
frequency but with different amplitudes. Such a resultant wave is called a standing or stationary wave. The portion
1 2 . 1 2 | Sound Waves

along the string where the amplitude is zero is called a node and where the amplitude is maximum is called an
antinode.

 2πvt  2πvt
For nodes: A =
2asin  =0; ⇒ nπ where n = 0, 1, 2, 3,
=
 λ  λ

nλ λ
The relation x = gives the position of the nth node and the distance between successive nodes is .
2 2

 2πvt  2πvt π
For antinodes, A = 2asin   = 2a, = ( 2n − 1) where n = 1, 2, 3…
 λ  λ 2

λ 3λ 5λ
x
= ( 2n − 1) λ4 i.e., , , ,................
4 4 4
,
Such points are called antinodes with maximum amplitude of 2a .
λ
The distance between the successive nodes and antinodes is .
4

11. MODE OF VIBRATION IN AIR COLUMNS


Longitudinal/stationary waves can be generated in both open- and closed pipes like organ pipes having both open
ends and one closed end, respectively. If a tuning fork produces a sound wave at the open end, then it is reflected
from the second end such that the incident and reflected wave superpose to generate stationary waves. Further,
the closed end is always a node and the open end is always an antinode.

11.1 Open pipe


The first three modes of vibration of an open pipe are given as follows:
L
For fundamental or first harmonic mode in the Fig. 12.10

λ1 v
(a) =l ; λ=
1 2l ; n=
1
2 2l
A N A
1
2
Figure 12.10
For the first overtone or the second harmonic in the Fig. 12.11

v
(b) l = λ2 ; n2
n2 =
2
A N A N A
2

For the second overtone or the third harmonic mode in Fig. 12.12 Figure 12.11

3λ3 21 3v pv
=l ;=
λ3 ; or λ3 = ; for path harmonic, np =
2 3 2l 2l

A N A N A N A
33
2
Figure 12.12
P hysi cs | 12.13

11.2 Closed pipe


L
The first three modes of vibration of a closed pipe are given as follows:

For fundamental or first harmonic mode in Fig. 12.13.


A N
1
λ1 v
=l ; λ=
1 4l; n=
1 4
4 4l
Figure 12.13

For the first overtone or third harmonic in Fig. 12.14


3λ2 4l 3v
=l ;=
λ2 ;=
n2
4 3 4l A N A N
33
4
For the second overtone or the fifth harmonic, Figure 12.14
5λ2 4l 5v
=l ;=
λ3 ;=
n
4 5 3 4l

For the pth overtone or (2p+1)th harmonic, n =


( 2p + 1) v .
2l A N ANA N
43
At the open end of the pipe, the antinode is formed at a small distance outside the open end. 4
Thus, the correct length of the closed pipe is l + e and that for an open pipe, it is l + 2 e and
Figure 12.15
e is equal to 0.3D where D is the internal diameter of the pipe.

12. DETERMINATION OF SPEED OF SOUND IN AIR

12.1 Resonance Column Method


Generally, systems have one or more natural
vibrating frequencies. Further, when a
system is driven at a natural frequency, then
there is a maximum energy transfer and the
vibrating amplitude steadily increases till up
to a maximum. However, when a system is
driven at a natural frequency, we say that L=3/4
L=/4
the system is in resonance (with the driven
source) and refer to the particular frequency
at which this occurs as a resonance frequency.
Moreover, from the relationship between the
frequency f, the wavelength λ , and the wave
speed v, which is λf =v , it is very obvious
that if both the frequency and wavelength
are known, then the wave speed can be Figure 12.16
easily determined. Further, if the wavelength
and speed are known, then the frequency can be determined.
We know that air column in pipes or tubes of fixed length has particular resonant frequencies. Moreover, the
interference of the waves travelling down the tube and the reflected waves traveling up the tube produces
(longitudinal) standing wave which must have a node at the closed end of the tube and an antinode at the open
end of the tube.
The resonance frequencies of a pipe or tube usually depend on its length L. As we observe from the Fig. 12.16, only
a certain number of lengths or “loops” can be “fitted” into the tube length with the node–antinode requirements.
1 2 . 1 4 | Sound Waves

However, since each loop corresponds to one-half wavelength, resonance occurs when the tube is nearly equal to
an odd number of quarter wavelength, i.e., L = λ / 4,L =
3λ / 4, L =
5λ / 4, etc
or in general, L = ( 2n + 1) λ / 4 ; =
λ 4L / ( 2n + 1) ; f=
n ( 2n + 1) v / 4L
Hence, an air column (tube) of length L has particular resonance frequencies and therefore will be in resonance with
the corresponding odd harmonic driving frequencies.
As we can observe from the above equation, the three experimental parameters involved in the resonance condition
of an air column are f, V, and L. However, to study the resonance in this experiment, the length L of an air column
will be varied for a given driven frequency. The length of the air column achieved by changing the position of the
movable piston in the tube is as seen in the Fig. 12.17.

Speaker - Connect to power supply

Resonance tube

Movable piston

Microphone - Connect to voltage sensor


Figure 12.17

Further, as the piston is removed, increasing the length of the air column, more wavelength segments will fit into
the tube, consistent with the node–antinode requirements at the ends. Thus, the difference in the tube lengths
when successive antinodes are at the open end of the tube and resonance occurs is equal to a half wavelength; for
example: ∆L = L2 − L1 = 3λ / 4 − λ / 4 = λ / 2
Further, when an antinode is at the open end of the tube, a loud resonance tone is heard. Hence, the tube length
for antinodes to be at the open end of the tube can be determined by moving the piston away from the open
end of the tube and “listening” for resonances. However, no end correction is needed for the antinode occurring
slightly above the end of the tube since in this case, difference in tube lengths for successive antinodes is equal
to λ/2. Further, if we know the frequency of the driving source, then the wavelength is determined by measuring
difference in tube length between successive antinodes, ∆L =λ / 2 or λ = 2∆L , the speed of sound in air, vs = λf .

12.2 Kundt’s Tube Method


In the Kundt’s method, a gas is filled in a long cylindrical tube
closed at both the ends, one by disk and the other by a movable
piston. A metal rod is welded with the disk and is clamped exactly
at the middle point. The length of the tube in this method can be D I
varied by moving the movable piston. Some powder is sprinkled
Figure 12.18
in the tube along its length.
The rod in the setup is set into longitudinal vibrations either electronically or by rubbing it with some cloth or
otherwise. Further, if the length of the gas column is such that one of its resonant frequency is equal to the
frequency of the longitudinal vibration of the rod, then standing waves originate in the gas. Moreover, the powder
particles at the displacement antinodes fly apart due to the inherent violent disturbance there, whereas the powder
at the displacement nodes remain undisturbed because the particles here do not vibrate. Thus, the powder which
was initially dispersed along the whole length of the tube gets collected in a heap at the displacement nodes.
By measuring the seperation ∆l between the successive heaps, we can find the wavelength of the sound in the
enclosed gas. λ = 2∆l
P hysi cs | 12.15

However, it should be noted that the length of the column is adjusted by moving the piston such that the gas
resonates and wavelength λ is obtained.
The speed of sound is given by ν = λv = 2∆l * v .
Further, if the frequency of the longitudinal vibration in the rod is not known, then the experiment is repeated with
air filled in the tube. Now, the length between the heaps of the powder, ∆l ′ is measured. The speed of sound in air
is then ν = 2∆l' v .  … (i)
ν ∆l ∆l
Now, = or ν = ν'
ν ∆l' ∆l'
By calculating the speed of ν ’ of sound in air, we can find the speed of sound in the gas.

13. BEATS
It two sources of slightly different frequencies produce sound waves in the same direction at the same point, these
waves then superpose to produce alternate loud and feeble sounds. Such variations in loudness are called beats.
The number of times such a fluctuation in loudness from maxima to minima takes place per second is called the
beat frequency.
If two waves y1 = asin(2πn2 t) of respective frequencies n1 and n2 superpose at the same place
asin(2πn1t) and y 2 =
x = 0, then y = y1 + y=
2 a[sin(2πn1t) + (sin2πn2 t)]

 2π (n1 − n2 ) t   2π (n1 + n2 ) t 
∴y 2 acos   × sin  =  2 acos 2π (n1 − n2 ) t  × sin 2π (n1 + n2 ) t 
 2   2 

y A sin  π (n1 + n2 ) t  ;=
= A 2acos  π (n1 − n2 ) t 

n +n 
The resultant wave is a harmonic wave with a frequency  1 2  but its amplitudes vary harmonically as a function
 2 
of the difference in the frequency n1 − n2 . The beat frequency nB is nB= n1 − n2 .

If n1 − n2 is small, i.e., the number of times the intensity of sound fluctuates between maxima and minima per
second is small, i.e., less than about 10 to 15, then the beats can be heard distinctly.

Illustration 7: Suppose that a string of length 25 cm and 2.5 g is under tension. A pipe closed at one end is 40 cm
long. When the string is set vibrating in its first overtone and the air in the pipe in its fundamental frequency, then
8 beats per second are heard. It is observed that decreasing the tension in the string decreases the beat frequency.
If the speed of sound in air is 320 m/s, then find the tension in the string  (JEE ADVANCED)
1 T v
Sol: The fundamental frequency of the string and the closed organ pipe are νs = and νp = . When two
2 m 4
waves of equal amplitude and slightly different frequencies superimpose with each other, phenomenon called
beats take place. Number of beats n = ∆ν where ∆ν is the difference in the frequencies of superimposed waves.

1 T 1 T
Fundamental of the string=
νs = = 20 T
2 m 2 × 0.25 10−2
v 320
The fundamental frequency of a closed pipe νp= = = 200 Hz
4 4 × 0.40

The frequency of the first overtone of the string = 2νs =40 T

Since there are 8 beat per second, 2νs − νp = 8 or 40 T − 200 =


8
1 2 . 1 6 | Sound Waves

Since on decreasing the tension, the beat frequency decreases, 2νs is definitely greater than νp
∴ 40 T − 200
= 8 or =
T 27.04N

Illustration 8: A sonometer wire of 100 cm in length has a fundamental frequency of 330 Hz. Find
(a) The velocity of propagation of transverse waves along the wire and
(b) The wavelength of the resulting sound in air if velocity of sound in air is 330 ms–1. (JEE ADVANCED)

Sol: As the wave travelling on the sonometer wire is the standing wave, the wavelength of the wire is λ =2L . And
the velocity of the wave is given by v = f λ = 2f L .
(a) In the case of transverse vibration of a string for fundamental mode: L =( λ / 2 ) ⇒ λ =2 L =2 × 1 ==2m
i.e., the wavelength of transverse wave propagating on the string is 2 m. Now, as the frequency of the wire is given
to be 330 Hz, so from v = f λ , the velocity of transverse wave along the wire will be v= 330 × 2= 660m / s
(b) Here, the vibrating wire will act as a source and produce sound, i.e., longitudinal waves in air: Now, as the
frequency does not change with change in medium so f = 330Hz, and as velocity in air is given to be = 330 m/s
v 330
so from relation v =f λ we get λair = air = =1m
f 330
i.e., for sound (longitudinal mechanical waves) in air produced by vibration of wire (body),
f = 330 s−1 , λair = 1 m and v = f × λ = 330m / s

14. DOPPLER EFFECT


We are familiar with the fact that when a source of sound or an observer or both are Velocity of sound, v
moving relative to each other, then there is an apparent change in the frequency of
sound as heard by an observer and this is called Doppler Effect. Further, the apparent Source Observer
frequency increases if the source is moving toward the observer or the observer is vs v0
moving toward the source. On the contrary, the apparent frequency decreases if either
the source is moving away from the observer or the observer is moving away from the Figure 12.19
source. This apparent change in the frequency is principally due to the basic effect of
motion of source to change the effective wavelength and the basic effect of motion of
observer is the change in the number of waves received per second by the observer.
However, if both the source and the observer are moving in the positive direction of x-axis, then sound of
frequency ‘n’ propagating in air with velocity in still air will result in an apparent frequency n′ heard by observed
 v − v0 
as n' =   n
 v − vs 
Moreover, if the direction of motion of source or observer is changed, then the signs of v0 and vs are accordingly
changed from negative to positive. Thus, the frequency n′, in still air for the different cases is obtained as follows:
(a) Both the source and observer are moving toward right when the source is approaching
vS v0
 v − v0 
a receding observer toward right n' =   n Figure 12.20
 v − vs 
 v − v0 
(b) Both the source and observer are receding from each other n' =   n vS v0
 v + vs 
Figure 12.21
 v + v0 
(c) Both the source and observer are moving toward each other n' =   n
 v − vs 
Figure 12.22
P hysi cs | 12.17

 v + v0 
(d) When the observer is approaching a receding source, n' =   n If the wind is
 v + vs  vS v0
blowing with a velocity ω in the direction of sound, then ω is added to ν and if the
Figure 12.23
wind is blowing with velocity ω opposite to direction of wind, then ω is subtracted
 ν ± ω  ν0 
from ν . The general formula for the apparent frequency n′ due to Doppler effect is, n' =   n
 ν ± ω  νs 

Illustration 9: Assume that a siren emitting a sound of frequency 2000 Hz moves away from you toward a cliff at
a speed of 8 m/s.
(a) What is the frequency of the sound you hear coming directly from the siren?
(b) What is the frequency of sound you hear reflected off the cliff? Speed of sound in air is 330 m/s.  (JEE MAIN)

Sol: As the siren being source is moving away from you the observer on cliff, the apparent frequency is given by
 v 
 v + v  . Where f0 is natural frequency of the sound wave. The intensity of the sound wave appears to be
f ' = f0 
 s 
decreasing. When sound reflects from cliff it moves towards observer (cliff) and hence the frequency of the sound
 v 
wave is f ' = fo   . When source moves towards the observer, the intensity of sound wave appears to be
 v − vs 
increasing.
(a) The frequency of sound heard directly is given by

 v   330 
=f1 f0 
v+v
; v s 8m / s; ∴
 = = f1   × 2000
 s   330 + 8 
(b) The frequency of the reflected sound is given by

 v   330  330
f2 = f1   ; ∴ f2 =   × 2000 ; f2 = × 2000 = 2050Hz.
 v − vs   330 − 8  322

Illustration 10: Let us suppose that a sound detector is placed on a railway platform. A train, approaching the
platform at a speed of 36 km h–1, sounds its horn. The detector detects 12.0 kHz as the most dominant frequency
in the horn. If the train stops at the platform and sounds the horn, what would be the most dominant frequency
detected? The speed of sound in air is 340 ms–1.  (JEE MAIN)
Sol: In the first case, when train is moving towards the stationary observer on the platform, the intensity of the
 v 
wave appears to be increasing. And the frequency is given by f ' = fo   . In the second case both the train
 v − vs 
and the observer are stationary so we hear the natural frequency f0 of the sound wave.
Here, the observer (detector) is at rest with respect to the medium (air). Suppose that dominant frequency as
emitted by the train is v0. When the train is at rest at the platform, the detector will select the dominant frequency
as v0. When this same train was approaching the observer, then frequency detected would be

ν ν − us  u 
v=' v ; or v 0= v=' 1 − s  v '
ν − us 0 ν  ν 
−1 36 × 103 m
The speed of the source
= is us 36kmh
= = 10ms−1
3600s
 10 
Thus v 0 =−
1  × 12.0kHz = 11.6kHz
 340 
1 2 . 1 8 | Sound Waves

14.1 Change in Wavelength


If a source moves with respect to the
medium, then wavelength becomes
’
different from the wavelength observed
when there is no relative motion between   ’’
the source and the medium. Thus, the 4
formula for calculation of apparent 3 4
wavelength may be derived immediately 3
2
from the relation λ = ν / v . It is given as 2
ν −u 1 1
=λ λ.
ν Figure 12.24

15. SONIC BOOM AND MACH NUMBER


A sonic boom is basically the sound associated with the shock waves created by an object travelling through air
faster than the speed of sound. This boom generates a huge amount of energy, sounding much like an apparent
explosion. The crack of a supersonic bullet passing overhead is an excellent example of a sonic boom in miniature.
Mach number is purely a dimensionless quantity representing the ratio of speed of an object moving through a
ν
fluid and the local speed of sound, M = where M is the Mach number,
νsound
ν is the velocity of the source relative to the medium, and νsound is the speed of sound in the medium.

16. MUSICAL SCALE


A musical scale is a sequence of frequencies which has a particularly pleasing effect on our ear. A widely used
musical scale, called diatonic scale, has eight frequencies covering an octave. We call each frequency as a note.

17. ACOUSTICS OF A BUILDING

Good concert halls: Good concert halls are so designed to eliminate unwanted reflection and echoes and
to optimize the quality of the sound perceived by the audience. This is accomplished by suitably engineering
the shape of the room and the walls, as well as to include sound-absorbing materials in areas that may cause
echoes.

Lecture hall: Similar consideration such as the one made in the above must be made particularly in a college
lecture hall, so that the professor can be heard by all of the students in the session. Although the sound quality
need not be as good as in a concert hall where music is being played, it still must be good enough to prevent
echoes and other things that will distort the audio quality of the speech delivered by the professor.

Work buildings: In an office building where there are cubicles with a divider in a large work area, there is often the
problem of noise from conversation and activities. However, in this case the quality of the sound is not an issue as
much as suppressing unwanted noise.

17.1 Echo
An echo (plural echoes) is a reflection of sound, arriving back at the listener particularly sometime after the direct
sound.
P hysi cs | 12.19

17.2 Reverberation and Reverberation Time


Reverberation is the persistence of sound in a particular space after the original sound is produced. A reverberation,
or reverb, is generated when sound is produced in an enclosed space causing a large number of echoes to build
up and then slowly decay as the sound is absorbed by the wall and air.
Reverberation time: The interval between the initial direct arrival of a sound wave and the last reflected wave is
called the reverberation time.

18. APPLICATION OF ULTRASONIC WAVES


Biomedical application: Ultrasound has very good therapeutic applications, which can be highly beneficial when
used with appropriate dosage precautions. Relatively high power ultrasound can eliminate stony deposits or tissue,
accelerate the effect of drugs in a targeted area, assist in the measurement of the elastic properties of tissue, and
can also be used to sort cells or small particles for research.
Ultrasonic impact treatment: Ultrasonic impact treatment (UIT) is a technique wherein ultrasound is used to
enhance the mechanical and physical properties of metals. Basically, it is a metallurgical processing technique in
which ultrasonic energy is applied to a metal object.
Ultrasonic welding: In ultrasonic welding of plastics, high frequency (15 kHz to 40 kHz) low amplitude vibration is
used to create heat by way of friction between the materials to be joined. The interface of the two parts is specially
designed so as to concentrate the energy for the maximum weld strength.
Sonochemistry: Power ultrasound in the 20–100 kHz range alone is used in chemistry. The ultrasound does
not interact directly with molecules to induce the chemical change, as its typical wavelength (in the millimeter
range) is too long compared to the molecules. Instead, the energy causes cavitation, which generates extremes of
temperature and pressure in the liquid where the reaction takes place.

19. SHOCK WAVES


A shock wave is one type of propagating disturbance. Similar to an ordinary wave, it carries energy and can
propagate through a medium (solid, liquid, gas or plasma) or in some cases even in the absence of a material
medium, through a field such as an electromagnetic field. Generally, shock waves are characterized by an abrupt,
nearly discontinuous change in the characteristics of the medium.

PROBLEM-SOLVING TACTICS
 1. Most of the questions are naturally related with the concepts of wave on a string. Therefore, one must
be thorough with the concept of that particular topic. (E.g., standing waves formed in open pipe here are
analogous to string tied at both ends. Further, many of the cases can be related in the same way.)
 2. Questions dealing with physical experiments form another set of questions. Therefore, one must be familiar
with usual as well as unusual (or specific) terminology of each experiment. Mostly, it happens that if we do not
know the term, then we are usually stuck (E.g., end correction is one term used with the resonance column
method, which is directly related with the radius of the tube.)
 3. Path difference between two sources form another set of questions and this is the only place where some
mathematical complexity can be involved. Hence, one must take care of them.
 4. Questions related to Doppler effect and beats are generally formulae specific; therefore, one must carefully
use the formulae. (It is, however, also advised that one must know about the derivation of these formulae.)
1 2 . 2 0 | Sound Waves

FORMULAE SHEET

S. No. Term Description


1. Wave It is a disturbance, which travels through the medium due to repeated periodic motion of
particles of the medium about their equilibrium position.

Examples include sound waves travelling through an intervening medium, water waves,
light waves, etc.
2. Mechanical waves Waves requiring material medium for their propagation. These are basically governed by
Newton’s laws of motion.

Sound waves are mechanical waves in the atmosphere between source and the listener
and hence require medium for their propagation.
3. Non-mechanical These waves do not require material medium for their propagation.
waves
Examples include waves associated with light or light waves, radio waves, X-rays, micro
waves, UV light, visible light and many more.
4. Transverse These are waves in which the displacements or oscillations are perpendicular to the
direction of propagation of wave.
waves
5. Longitudinal These are those waves in which displacement or oscillations in medium are parallel to the
direction of propagation of wave, for example, sound waves.
waves
6. Equation of At any time t, displacement y of the particle from its equilibrium position as a function of

harmonic wave ( )
the coordinate x of the particle is y(x,y) = A sin ωt − kx where A is the amplitude of the
wave,

k is the wave number,

ω is the angular frequency of the wave,

( )
And ωt − kx is the phase.

7. Wave number Wave length λ and wave number k are related by the relation k = 2n/λ.

8. Frequency
Wavelength λ and wave number k are related by the relation v = ω / k = λ / T = λf.

9. Speed of a wave Speed of a wave is given by v =


ω/k = λf .
λ/T=

10. Speed of a Speed of a transverse wave on a stretched


transverse wave
string depends only on tension and the linear

mass density of the string but not on frequency of the wave, i.e.,
= v T/µ .
11. Speed of a Speed of longitudinal waves in a medium is given by v =
longitudinal wave
B = bulk modulus; ρ = density of medium;

Speed of longitudinal waves in an ideal gas is v = γp / ρ P = pressure of the gas , ρ =


density of the gas and y = Cp/Cv.
12. Principle of super When two or more waves traverse through the same medium, then the displacement
position of any particle of the medium is the sum of the displacements that the individual waves
would give it, i.e., y =∑ ( )
y i x,t
.
P hysi cs | 12.21

S. No. Term Description


13. Interference of If two sinusoidal waves of the same amplitude and wavelength travel in the same direction,
waves then they interfere to produce a resultant sinusoidal wave travelling in the direction with
resultant wave given by the = ( )
relation y ' x,t ( ) (
2Am cos u / 2  sin ωt − kx + u / 2
  )
where u is the phase difference between the two waves.

If u = 0, then interference would be fully constructive.

If u = π , then waves would be out of phase and their interference would be destructive.
14. Reflection of waves When a pulse or travelling wave encounters any boundary, it gets reflected. However, if
( )
an incident wave is represented by y i x,t ( )
= A sin ωt − kx , then the reflected wave at
( )
rigid boundary is yr x,t ( ) ( )
= A sin ωt + kx + n =−A sin ωt + kx and for reflection at
( )
open boundary, reflected waves is given by yr x,t
= A sin ωt + kx .( )
15. Standing wavesThe interference of two identical waves moving in opposite directions produces
standing waves. The particle displacement in a standing wave is given by
= y x,t ( ) ( ) ( )
2A cos kx  sin ωt . In standing waves, amplitude of waves is different at
different points, i.e., at nodes, amplitude is zero and at antinodes, amplitude is maximum
which is equal to sum of amplitudes of constituting waves.

16. Normal modes of Frequency of transverse motion of stretched string of length L fixed at both the ends is
stretched string given by f = nv/2L where n = 1, 2, 3, 4. The set of frequencies given by the above relation
is called normal modes of oscillation of the system. Mode n = 1 is called the fundamental
mode with the frequency f1 = v/2L. Second harmonic is the oscillation mode with n = 2
and so on.

Thus, a string has infinite number of possible frequencies of vibration which are harmonics
of fundamental frequency f1 such that fn = nf1.
17. Beats Beats arise when two waves having slightly differing frequency V1 and V2 and comparable
amplitudes are superposed.
18. Doppler effect Doppler effect is a change in the observed frequency of the wave when the source S and
the observer O move relative to the medium.

There are three different ways where we can analyze this change in frequency as listed
hereunder.

(1) when observer is stationary and source is approaching observer

v = v0(1+Vs/V) where Vs = velocity of the source relative to the medium

v = velocity of wave relative to the medium

V = observed frequency of sound waves in terms of source frequency

V0 = source frequency

Change in the frequency when source recedes from stationary observer is


v = V0(1–VS/V)

Observer at rest measures higher frequency when source approaches it and it


measures lower frequency when source recedes from the observer.

(2) observer is moving with a velocity V0 toward a source and the source is at rest is
V = V0(1+V0/V)

(3) both the source and observer are moving, then frequency observed by observer is
V = V0 (V+V0)/(V+VS) and all the symbols have respective meanings as discussed
earlier
1 2 . 2 2 | Sound Waves

Solved Examples

JEE Main/Boards again 0.4 = 1λ / 2 or λ = 0.4m



= n 320 = / 0.4 800Hz
Example1: A tube closed at one end has a vibrating
diaphragm at the other end, which may be assumed
Example 2: A tuning fork of frequency 256 Hz and an
to be displacement node. It is found that when
open orange pipe of slightly lower frequency are at
the frequency of the diaphragm is 2000 Hz, then a
170C. When sounded together, they produce 4 beats
stationary wave pattern is set up in which the distance
per second. On altering the temperature of the air
between adjacent nodes is 8 cm. When the frequency
in the pipes, it is observed that the number of beats
is gradually reduced, then the stationary wave pattern
per second first diminishes to zero and then increases
disappears but another stationary wave pattern
again to 4. By how much and in what direction has the
reappears at a frequency of 1600 Hz. Calculate
temperature of the air in the pipe been altered?
(i) The speed of sound in air,
Sol: In a open organ pipe the frequency of the wave is
(ii) The distance between adjacent nodes at a frequency
V
of 1600 Hz, n = t where Vt is the velocity of wave at temperature
λ
(iii) The distance between the diaphragm and the t and λ=2L is the wavelength of the vibrating wave. If
closed end, and temperature of air inside the organ pipe changes, the
(iv) The next lower frequencies at which stationary wave velocity of wave also changes, since V ∝ T .
pattern will be obtained.
V17
n= where L = length of the pipe
Sol: The standing waves generated inside the tube 2l
closed at one end, have the wavelength n λ =2L where V V17
L is length of the tube. The velocity of the wave in air is − 17 4 or=
∴ 256 = 252
2L 2L
given by v = fλ, where n is the frequency of the sound
Since beats decrease first and then increase to 4, the
wave.
frequency of the pipe increases. This can happen only if
Since the node-to-node distance is the temperature increases.
or λ / 2 0.08=
= or λ 0.16m Let t be the final temperature, in Celsius,
((i)i) v = f λ ∴ v = 2000 × 0.16 = 320 m / s V V
n = t − 256 =4 or t =260
((ii)ii) 320
= 1600 × λ or = λ 0.2 m 2l 2l
V 260 273 + t 260
∴ Distance between nodes = 0.2/2 = 0.1 m = 10 cm = dividing t = or
V17 252 273 + 17 252
(iii) Since there are nodes at the ends, the distance
between the closed end and the diaphragm must be an ( )
 V < T or t = 308.7 − 273 = 35.7 − 17
integral multiple of λ/2
= 18.70 C.
∴L = nλ / 2 = n × 0.2 / 2 = n'× 0.16 / 2
∴ Rise in temperature= 35.7 − 17= 18.70 C.
n 4
⇒ = when n' = 5, n = 4
n' 5 Example 3: Find the fundamental and the first overtone
n'× 0.16 of a 15 cm pipe
= L = 0.4 = m 40 cm
2 (a) If the pipe is closed at one end,
(b) If the pipe is open at both ends,
((iv)
iv()ivFor
) Forthethenext lower
next frequency
lower n =n 3,
frequency = 3,2,12,1
∴∴ 0.40.4
= 3= λ3/λ2/ 2 oror λ =λ 0.8 / 3/ 3 (c) How many overtones may be heard by a person of
= 0.8
normal hearing in each of the above cases? Velocity of
320 sound in air = 330 ms–1
sincev =λf ,f= =1200Hz
0.8 / 3
∴ f 320
= =/ 0.4 800Hz
P hysi cs | 12.23

Sol: For the organ pipe closed at one end, the Speed of source, Vs = rω = 3 × 10 = 30 m/s
fundamental frequency of the wave of wavelength λ
Maximum velocity of detector v0 =Aω’
v
is given by, n0 = .The frequency of ith over tone is ν0 = A × 2πf ' = 6 × 2π × (5 / π ) = 60m / s
4L
given by ni = ( i + 1) × n0 where i=1,2,3…. etc. Actual frequency of source n = 340Hz
v The frequency recorded by the detector is maximum
=((a)
a) n0 =
4L
where n0 frequency of the
when both the source and detector travel along the
330 same direction.
fundamental ⇒
= n0 = 550Hz
4 × 0.15 6m 6m
(b) The first four overtones are 2n0, 3n0, 4n0 ,and 5 n0. So,

3m
the required frequencies are 1100, 2200, 3300, 4400, A B C D
and 5500 Hz.
v + v0 330 + 60
(c) the frequency of the nth overtone is (2n + 1)r nmax
= = n × 340
= 442Hz
v + vs 330 − 30
∴ ( 2n
= + 1) n0 20000;or ( 2n =
+ 1) 550 20000
The frequency recorded will be minimum when both
or n = 17.68 the source and detector are travelling in opposite
Or n = 17.18 the acceptable value is 17. directions.
v + v0 330 − 60
nmax
= = n × 340
= 255Hz
Example 4: The wavelength of the note emitted by a v + vs 330 + 30
tuning fork of frequency 512 Hz in air at 170 C is 66.6
cm. If the density of air at STP is 1.293 gram per liter,
calculate γ for air.
JEE Advanced/Boards
V 2p
Sol: The bulk modulus of gas γ is given by γ = .
Po Example 1: Two sources S1 and S2 separated by 2.0 m,
Here V is velocity of wave, and p is the pressure at a vibrate according to equation
point. And Po is the atmospheric pressure. = y1 0.03sin ( πt ) and
= y 2 0.02sin ( πt )
n= 512 Hz, = 66.6 cm  ; ν = nλ Where y1, y2 and t are in M.K.S. units. They send out
ν2p waves of velocity 1.5m/s.
= 512 × 66.6
= 340.48 m / s ; =
γ
Po Calculate the amplitude of the resultant motion of the
1.013 105 Nm−2 ; p =
Po =× 1.293 kg / m3 .; particle collinear with S1 and S2 and located at a point.
(a) To the right of s2
(330 ) × 1.293
2

=∴γ = 1.39. (b) To the left of s1 and


1.013 × 105
(c) In the middle of S1 and S2
Example 5: A source of sound is moving along orbit of
Sol: The phase difference between the two waves is
radius 3 m with an angular velocity of 10 rad/s. A source
detector located far away from the source is executing 2πx
given by φ = where x= 2.0 m is the path difference
linear simple harmonic motion along the line BD as λ
shown in the figure with an amplitude BC = CD = 6 m. between the two waves at points near to S1 or S2.
The frequency of oscillation of the detector is 5 / π per
The resultant amplitude of the superimposed wave is
second. The source is at the point A when the detector
is at the point B. If the source emits a continuous wave a= a12 + a22 + 2a1a2 cos φ .
of frequency 340 Hz, then find the maximum and the
minimum frequency recorded by the detector. Let P and R be respective points to the left of S1 and
right of S2, respectively.
Sol: Here both source and detector are performing The oscillations y1 and y2 have amplitude a1 = 0.03 m
periodic motion. When source and detector are moving and a2 = 0.02 m, respectively. These have equal period
away from each other, the detector will record the 1 1
minimum frequency and vice versa. T = 2 s and same frequency n= = = 0.5s−1 .
T 2
1 2 . 2 4 | Sound Waves

The wavelength of each vibration 2π


=
δ ∆x where wavelength is λ and ∆x is the path
v 1.5 λ
λ= = = 3.0m
n 0.5 difference. The distance between two successive node
or two successive antinode is λ/2.
(a) The path difference for point R to the
Given, n= 250 Hz, v = 30 m/s
1m 1m
given, n 250
= = Hz, v 30 m / s 
P R
S1 Q S2 v 30 3
∴ λ= = = m= 12cm
2m n 250 25
right of S2 = ∆ =(S1R –S2R) = S1S2 = 2m ( a)  ∴ Phase difference for a distance of 10 cm
(a)
2π 2π 4π 2π 2π 5
∴ Phase difference =
φ x= × 2.0
= =  × 10 = × 10 = π
λ 3 3 λ 12 3
The resultant amplitude for point R is given by
(b ) Now
(b) = a 0.03m,
= λ (3 / 25) m
a12 + a22 + 2a1a2 cos φ = 1
and = n= 250Hz
T
{(0.03) + (0.02) + 2 × 0.03 × 0.02 × cos ( 4π / 3)}
2 2
The equation of a plane progressive
t x 
Solving, we obtain a = 0.02565 m. wave is given =
by y a sin2π  − + φ 
T λ 
(b) The path difference for all point p to the left of S1 is
∆=S2P−S1P=2.0 m. ∴ where φ is initial phase ;
= y 0.03 sin2π ( 250t − 25x / 3 + φ ) ;
Hence, the resultant amplitude for all points to the left
of S1 is 0.0265 m. ( c )The
(c) thedistance
distancebetween
betweennodes
nodesininstationary
stationary
( C(c)
( C) )forfora apoint
pointQ,Q,midway
midwaybetween
betweenS1Sand SS,2 ,
1and 2 wave =
λ 12
= = 6 cm
the
thepath
pathdifference
differenceisiszero
zeroi.e., 00
i.e.,φ φ== 2 6
(d) Equation of a stationary wave is given by
Hence a = a12 + a22 + 2a1a2
2πx 2πvt
{(0.03) + (0.02) + 2 (0.03)(0.02)}
2 2 y = 2a cos sin
= λ λ
2πx
= 0.03 + 0.02 = 0.05m If there is antinode at x= 0= 2acos sin
λ
3 1
Example 2: A progressive and stationary simple As a 0.01m,
= = λ m and= 250 Hz
25 T
harmonic wave each have the same frequency of 250  50πx 
=
Hz, and the same velocity of 30 m/s. Calculate y 0.02cos   sin (500 π t ) m where x and
 3 
(a) The phase difference between two vibrating points y are n meter and t in sec
on the progressive waves which are 10 cm apart.
(b) The equation of motion of the progressive wave if Example 3: The first overtone of an open organ pipe
its amplitude is 0.03 m. beats with the first overtone of a closed organ pipe with
a beat frequency of 2.2 Hz. The fundamental frequency
(c) The distance between nodes in the stationary wave,
of the closed organ pipe is 110 Hz. Find the length of
(d) The equation of motion of the stationary wave if its the pipes.
amplitude is 0.01 m.
Sol: The difference in frequencies of the first overtones
Sol: The simple harmonic progressive waves, is of the open organ pipe and closed organ pipe is 2.2 Hz.
Write the frequencies in terms of length of the pipes
t x 
represented by=y a sin ω  − + φ  where φ is the and get the relation between the lengths of the pipes.
T λ  The fundamental frequency of the closed organ pipe is
phase constant of the wave. The phase difference is given so its length can be easily found.
P hysi cs | 12.25

The beat are produced when the wave of same amplitude ρ1 32


 V1 / V2 = 1/ 4 and = = 16
but different frequencies, resonate with each other. ρ2 2
Let the length of open and closed pipes be l1 and l2, 1 
ρ2  × 16 + 1
respectively.  4 = 4 ρ ⇒ ρ = 4 .
∴ ρ= 2
The frequency of first over tone of open organ pipe is 1  ρ2
 + 1
=n1 =
2v ν 4 
2l1 l1

The frequency of first over tone of closed organ pipe is Let V1 and V2 be the speed of sound in the mixture and
hydrogen, respectively.
3v
n2 =
4l2 γP γP v ρ1
v1 = and v 2 = ;∴ 1 =
Fundamental frequency of closed organ pipe ρ1 ρ2 v2 ρ2
v v 330 v 2 1270
n
= ;=
∴ = = 110 = 4 2 or v=
= 1 = = 635 m / sec
4l2 4l2 4l2 2 2
330
=l2 = 0.75m Example 5: The difference between the apparent
4 × 110
frequency of a source as perceived by an observer
As beat frequency = 2.2 Hz during its approaching and recession is 2% of the
natural frequency of the source. Find the velocity of the
v 3v 330 3 × 330 source. Take the velocity of sound as 350 m/s.
= − ⇒ − 2.2
=
l1 4l2 l1 4 × 0.75
Sol: By the Doppler’s method use the formula for
330 apparent frequency in terms of source velocity to
∴ l=
1 = 0.993m ;
332.2 express the difference in two frequencies of approach
3v v and recession of the source in terms of its velocity.
Beat frequency = − = 2.2
4l2 l1
For the source approaching a stationary observer,
3 × 330 330 330  v 
or − = 2.2 ; = 327.8 n' n 
4 × 0.75 l1 l1 =  ; As v >> v s ,
 v − v s 
l1 = 1.006 m. −1
 1   v 
n' n 
= =  n 1 − s 
Example 4: The speed of sound in hydrogen is 1270 m/s. 1 − ( v s / v )   v

Calculate the speed of sound in the mixture of oxygen  Vs 
∴ n'  n 1 +  .....(i) …(i)
and hydrogen in which they are mixed in 1:4 ratio. V

V1ρ1 + V2ρ2 When the source is receding, then
Sol: The density of the mixture is given as ρ =
V1 + V2
 V 
1 n" ≅ n 1 − s  ...........(ii) …(ii)
here V1:V2 =1:4. The speed of sound in gas is v ∝ .  V

ρ
From Eqs. (i) and (ii)
Let V1 and V2 be respective volume of oxygen and
hydrogen.  V   V  2nv s
n′–n″= 1 + s  − 1 − s  =
Let d1, m1 be density and mass of oxygen in the mixture  V  V v
and d2 m2 be density and mass of hydrogen in the
mixture, respectively. n'− n" 2νs
Or =
n ν
Total mass V ρ + V2ρ2  2ν 
=∴ ρ = 1 1 Percentage change in frequency =  s  × 100 =
2
total volume V1 + V2  ν 
2nνs
V2ρ2 ( V1ρ1 / V2ρ2 + 1 ) d2 ( V1ρ1 / V2ρ2 + 1 ) Or νs =3.5 m / s ; × 100 =
2
= ν
V2 ( V1 / V2 + 1 ) ( V1 / V2 + 1)
1 2 . 2 6 | Sound Waves

Example 6: A whistle emitting a sound of frequency (b) The distance from the hill at which the echo from
440 Hz is tied to a string of 1.5 m length and rotates the hill is heard by the driver and its frequency (velocity
with an angular velocity of 20 rad/s in the horizontal of sound in air 1,200 km/hr.)
plane. Calculate the range of frequency heard by an Hill
observer stationed at a large distance from the whistle.
(1-x) x
A
v
 B B’
r 1km
Sol: As train is moving towards the stationary observer
v on the hill. And the wind is in direction of the motion
B of train, the frequency of the sound waves from horn
 v' 
Sol: As the whistle is moved in the circle in horizontal heard to the observer on hill is given by n' = n  
plane, it sometimes moves away and sometimes  v '− v s 
towards the stationary observer. Thus the observer will where v’ = v+w (sum of velocities of train and train).
 v  When this sound wave reflects from the hill, and travels
hear the minimum frequency of nmin = n   when towards the moving train, the frequency heard by the
 v + vs 
v − w + v 
whistle is moving away from him. The observer will driver is n' = n  s
.
 ( v − w ) 
 v 
hear maximum frequency of nmax = n   when
 v − vs  (a) The apparent frequency is given by
the whistle is moving towards him.
 v+w 
Velocity of source = v s = rω =1.5 × 20=30 m/s n' = n  
 ( v + w ) − v s 
Frequency n= 440 Hz..
V = 1200 km/hr., w = 40 km/hr., v s = 40 km/hr. and n
And speed of sound, v = 330 m/s, the maximum = 580 Hz.
frequency nmax will correspond to a position when
source is approaching the observer  1200 + 40 
= ∴ n' 580 =  599.3Hz
 v   (1200 + 40 ) − 40 
 330 
nmax n=
=   440  
 v − v s   330 − 30  (b) As shown in the figure, let the driver hear the echo
when he is at a distance x km from the hill. Time taken
440 × 330 by the train to reach the point B′
= = 484
300
The minimum frequency nmax will correspond = to a t =
(1 − x ) 1− x
hr;
position when source is receding the observer. velocity of train 40
Time taken by the train to reach the point B′
 v   330 
=
=n n=
nmin n=
min  v  440 330
440  330 + 30  x x
 vv + vs   330 + 30  t= = hr
 + v s  velocity of sound − velocity of w ind 1200 − 40
440
440 × × 330
330
=
= = = 403Hz
403Hz 1− x x
360
360 = ; x = 0.966 km
40 1200 − 40
The range of frequency is from 403 Hz to 484 Hz. Frequency heard by driver.
1200 − 40 + 40 
Example 7: A train approaching a hill at = a speed of 40 n' 580
=   600 Hz
km/hr. sounds its horn of frequency 580 Hz when it is  (1200 − 40 ) 
at a distance of 1 km from the hill. A wind with a speed
of 40 km/hr is blowing in the direction of motion of the Example 8: A band playing music at frequency f is
train. Find moving toward a wall with velocity Vs. A motorist is
following the band with a speed of Vm. If V is speed
(a) The frequency of the horn as heard by the observer
of sound, obtain an expression for the beat frequency
on the hill,
heard by the motorist.
P hysi cs | 12.27

Sol: In this case, both the source and the observer  v+0   v 
moving with different speeds towards the wall so = fw f=  f 
the frequency of sound heard by motorist is given as  v + v s   v − v b 
 v + vm  The frequency f″ reaching the motorist is given by
f ' = f0  .
 v + v 
s 
While the sound reflected from the wall is moving  v + vm   v   v + vm 
= f '' fw =   f  
towards the motorist. Hence the frequency heard by  v − v b   v 
 v+0 
 v + vm 
the motorist will be f '' = fw   . These two waves  v + vm 
 v  =f  ∴ Beat frequency = f "− f ' =f

 v − v b 
superimpose with each other to create beats and
number of beats heard is given by n= f " − f ' .
 v + vm   v + vm 
The frequency, f, of band heard by the motorist directly = ∴ f f  −f ;
 v − v b   v + vb 
is given by
 v+v 
 v + vm  =f  ( m )( v + v b ) − ( v + v m )( v + v b ) 
f' = f 
 v + v s 

 ( )
v 2 − v b2 

 v+v 
The frequency fw reaching the wall is
= f
( m )( 2vb ) 

 ( )
v 2 − v b2 


JEE Main/Boards

Exercise 1 Q.5 What will be the speed of sound in a perfectly rigid


rod?

Q.1 The velocity of sound in air at NTP is 331 ms–1. Find


Q.6 A stone is dropped into a well in which water is
its velocity when the temperature rises to 910C and its
78.4m deep. After how long will the sound of splash be
pressure is doubled.
heard at the top? Take velocity of sound in air = 332ms-1

Q.2 A displacement wave is represented by


Q.7 From a cloud at an angle of 300 to the horizontal,
ξ 0.25 × 10 sin (500t − 0.025x )
= −3
we hear the thunder clap 8 s after seeing the lightening
flash. What is the height of the cloud above the ground
Deduce (i) amplitude (ii) period (iii) angular frequency if the velocity of sound in air is 330 m/s?
(iv) Wavelength (v) amplitude of particle velocity (vi)
amplitude of particle acceleration. ξ , t and x are in cm, Q.8 A fork of frequency 250Hz held over tube and
sec and meter respectively. maximum sound is obtained when the column of air is
31cm or 97 cm. Determine (i) velocity of sound (ii) the
Q.3 Calculate the velocity of sound in gas, in which two end correction (iii) the radius of tube.
wave lengths 2.04m and 2.08m produce 20 beats in 6
seconds. Q.9 In an experiment, it was found that a tuning fork
and a sonometer gave 5 beats/sec, both when length
Q.4 What type of mechanical wave do you expect to of wire was 1m and 1.05m. Calculate the frequency of
exist in (a) vacuum (b) air (c) inside the water (d) rock (e) the fork.
on the surface of water?
1 2 . 2 8 | Sound Waves

Exercise 2 with the next clap 10 times during every 3 seconds.


Find the velocity of sound in air.
Single Correct Choice Type (A) 420 m/s (B) 333 m/s
(C) 373 m/s (D) 555 m/s
Q.1 A firecracker exploding on the surface of lake is
heard as two sounds at a time interval t apart by a man
Q.7 Find the minimum and maximum wavelength of
on a boat close to water surface. Sound travels with a
sound in water that is in the audible range (20-20000 Hz)
speed u in water and a speed v in air. The distance from
for an average human ear. Speed of sound in water
the exploding firecracker to the boat is
=1450 ms-1.
utv t (u + v )
(A) (B) (A) 72.5 m (B) 70.5 m (C) 71.5 m (D) 70.9 m
u+ v uv
t (u − v ) utv Q.8 The sound level at a point 5.0 m away from a point
(C) (D)
uv u− v source is 40 dB. What will be the level at a point 50 m
away from the source?

Q.2 A sonometer wire has a total length of 1 m between (A) 25 lb (B) 5 lb (C) 20 db (D) 40 lb
the fixed ends. Two wooden bridges are placed below
the wire at a distance 1/7m from one end and 4/7m Q.9 A source of sound S and a detector D are placed
from the other end. The three segments of the wire at some distance from one another. A big cardboard is
have their fundamental frequencies in the ratio: placed near the detector and perpendicular to the line
(A) 1: 2: 3 (B) 4: 2: 1 SD as shown in figure. It is gradually moved away and it
is shown that the intensity change from a maximum to
(C) 1: 1/2: 1/3 (D) 1: 1: 1 a minimum as the board is moved through a distance of
20cm. What will be the frequency of the sound emitted.
Q.3 A person can hear frequencies only up to 10 kHz. Velocity of sound in air is 336 ms-1.
A steel piano wire 50 cm long of mass 5 g is stretched
with a tension of 400 N. The number of the highest
overtone of the sound produced by this piano wire that S D
the person can hear is
(A) 420 Hz (B) 422 Hz (C) 450 Hz (D) 410 Hz
(A) 4 (B) 50 (C) 49 (D) 51

Q.10 Two sources of sound, s1 and s2, emitting waves of


Q.4 How many times intense is 90 dB sound than 40dB equal wavelength 20.0 cm, are placed with a separation
sound? of 20.0 cm between them. A detector can be moved
(A) 5 (B) 50 (C) 500 (D) 105 on a line parallel to s1 s2 and at a distance of 20.0cm
from it. Initially, the detector is equidistant from the
two sources. Assuming that the waves emitted by
Q.5 At a prayer meeting, the disciples sing jai-ram jai-
the sources are in phase, find the minimum distance
ram. The sound amplified by a loudspeaker comes back
through which the detector should be shifted to detect
after reflection from a builder at a distance of 80m from
a minimum frequency of sound.
the meeting. What maximum time interval can be kept
between one jai-ram and the next jai-ram so that the (A) 12 cm (B) 24 cm (C) 36 cm (D) 48 cm
echo does not disturb a listener sitting in the meeting?
Speed of sound in air is 320 ms-1. Q.11 A cylindrical metal tube has a length of 50 cm
(A) 20 Seconds (B) 0.3 Seconds and is open at both ends. Find the frequencies between
1000 Hz and 2000 Hz at which the air column in the
(C)40 Seconds (D) 0.5 Seconds
tube can resonate. Speed of sound in air is 340 ms-1.
(A) 1020 Hz, 1360 Hz, 1700 Hz
Q.6 A man stands before a large wall at a distance of
50.0 m and claps his hands at regular intervals. Initially, (B) 1200 Hz, 1400 Hz, 1700 Hz
the interval is large. He gradually reduces the interval
(C) 1020 Hz, 1360 Hz, 2000 Hz
and fixes it at a value when the echo of a clap merges
(D) 1000 Hz, 1360 Hz, 1800 Hz
P hysi cs | 12.29

Q.12 The first overtone frequency of a closed organ overtone (third harmonic) of an air column of length
pipe p1 is equal to the fundamental frequency of an 75 cm inside a tube closed at one end. The string also
open organ pipe p2. If the length of the pipe p1 is 30cm. generates 4 beats/s when excited along with a tuning
What will be the length of p2? fork of frequency n. Now when the tension of the string
is slightly increased, the number of beats reduces to 2
(A) 12 cm (B) 24 cm (C) 20 cm (D) 38 cm
per second. Assuming the velocity of sound in air to
be 340 m/s, the frequency n of the tuning fork in Hz is
 (2008)
Previous Years’ Questions (A) 344 (B) 336
(C) 117.3 (D) 109.3
Q.1 A siren placed at a railway platform is emitting
sound of frequency 5 kHz. A passenger sitting in a Q.6 A police car with a siren of frequency 8 kHz is
moving train A records a frequency of 5.5kHz, while the moving with uniform velocity 36 km/h toward a tall
train approaches the siren. During his return journey building which reflects the sound waves. The speed of
in a different train B, he records a frequency of 6.0 kHz sound in air is 320m/s. The frequency of the siren by
while approaching the same siren. The ratio of the train the car driver is  (2011)
B to that of train A is  (2002)
(A) 8.50 kHz (B) 8.25 kHz
(A) 242/252 (B) 2
(C) 7.75 kHz (D) 7.50 kHz
(C) 5/6 (D) 11/6

Q7 Sound waves of frequency 660 Hz fall normally on


Q.2 A sonometer wire resonates with a given tuning fork a perfectly wall. The shortest distance from the wall
forming standing waves with five antinodes between at which the air particle have maximum amplitude of
the two bridges when a mass of 9 kg is suspended vibration is….....................…m. speed of sound =330m/s.
from the wire. When this mass is replaced by mass M,  (1984)
the wire resonates with the same tuning fork forming
three nodes and antinodes for the same position of the
bridges. The value of M is  (2002) Q.8 In a sonometer wire, the tension is maintained by
suspending a 50.7kg mass from the free end of the
(A) 25kg (B) 5kg wire. The suspended mass has a volume of 0.0075m3.
(C) 12.5kg (D) 1/25kg The fundamental frequency of vibration of the wire
is 260 Hz. If the suspended mass is completely
submerged in water, the fundamental frequency will
Q.3 In the experiment for the determination of the
become…....................Hz.  (1987)
speed of sound in air using the resonance column
method, the length of air column that resonates in the
fundamental mode, with a tuning fork is 0.1m. When Q.9 The ratio of the velocity of sound in hydrogen gas
this length is changed to 0.35 m, the same tuning fork  7  5
resonates with the first overtone. Calculate the end  γ =  to that in helium gas  γ =  at the same
 5  3
correction.  (2003)
temperature is 21/ 5 . State whether true or false
(A) 0.012m (B) 0.025m
(C) 0.05 m (D) 0.024  (1983)

Q.4 A source of sound of frequency 600Hz is placed Q.10 A plane wave of sound travelling in air is incident
inside water. The speed of sound in water is 1500m/s upon a plane water surface. The angle of incidence is
and in air it is 300m/s. the frequency of sound recorded 600. Assuming Snell’s law to be valid for sound waves,
by an observer who is standing in air is  (2004) it follows that the sound wave will be refracted into
water away from the normal. State whether true or false
(A) 200Hz (B) 3000Hz  (1984)
(C) 120 Hz (D) 600 Hz
Q.11 A source of sound wave with frequency 256 Hz
Q.5 A vibrating string of certain length l under a tension is moving with a velocity ν towards a wall and an
T resonates with a mode corresponding to the first observer is stationary between the source and the wall.
1 2 . 3 0 | Sound Waves

When the observer is between the source and the wall, Q15. A cylindrical tube, open at both ends, has a
he will hear beats. State whether true or false (1985) fundamental frequency, f, in air. The tube is dipped
vertically in water so that half of it is in water. The
Q.12. While measuring the speed of sound by fundamental frequency of the air-column is now
performing a resonance column experiment, a student  (2012)
gets the first resonance condition at a column length f 3f
(A) f (B) (C) (D) 2f
of 18 cm during winter. Repeating the same experiment 2 4
during summer, she measures the column length to be
x cm for the second resonance. Then (2008)
Q.16. An open glass tube is immersed in mercury in
(A) 18 > x (B) x >54 such a way that a length of 8 cm extends above the
mercury level. The open end of the tube is then closed
(C) 54 > x > 36 (D) 36 > x > 18
and sealed and the tube is raised vertically up by
additional 46 cm. What will be length of the air column
Q.13 A motor cycle starts from rest and accelerates above mercury in the tube now? (Atmospheric pressure
along a straight path at 2 m/s2. At the starting point of = 76 cm of Hg) (2014)
the motor cycle there is a stationary electric sire. How
(A) 16 cm (B) 22 cm
far has the motor cycle gone when the driver hears
the frequency of the siren at 94% of its value when the (C) 38 cm (D) 6 cm
motor cycle was at rest? (speed of sound = 330 ms-1).
 (2009)
Q.17. A train is moving on a straight track with speed
(A) 49 m (B) 98 m 20 ms–1. It is blowing its whistle at the frequency of 1000
Hz. The percentage change in the frequency heard by a
(C) 147 m (D) 196 m
person standing near the track as the train passes him
is (speed of sound = 320 ms–1) close to : (2015)
Q14. Three sound waves of equal amplitudes have
(A) 6% (B) 12% (C) 18% (D) 24%
frequencies (v – 1), v, (v + 1). They superpose to give
beats. The number of beats produced per second will
be (2009) Q.18 A pipe open at both ends has fundamental
frequency f in air. The pipe is dipped vertically in water
(A) 4 (B) 3 (C) 2 (D) 1
so that half of it is in water. The fundamental frequency
of the air column is now : (2016)
3f f
(A) (B) 2f (C) f (D)
4 2

JEE Advanced/Boards

Exercise 1 Q.3 The loudness level at a distance R from a long linear


source is found to be 40dB. At this point, the amplitude
of oscillation of air molecules is 0.01cm. Then find the
Q.1 Find the intensity of sound wave whose frequency loudness level & amplitude at a point at distance ‘10R’
is 250Hz. The displacement amplitude of particles of from the source.
the medium at this position is 1× 10−8 m. The density of
the medium is kg/m3, bulk modulus of elasticity of the
Q.4 Two identical sounds A and B reach a point in the
medium is 400N/m2.
same phase. The resultant sound is C. The loudness of
C is n dB higher than the loudness of A. Find the value
Q.2 In a mixture of gases, the average number of of n.
degrees of freedom per molecule is 6. The rms speed
of the molecules of the gas is c. find the velocity of
Q.5 Sound of wavelength λ passes through a Quincke’s
sound in the gas.
tube which is adjusted to give a maximum intensity I0.
P hysi cs | 12.31

Find the distance the sliding tube should be moved to Exercise 2


give an intensity l0/2.
Single Correct Choice Type
Q.6 The first overtone of a pipe closed at one end
resonates with the third harmonic of a string fixed at its Q.1 Two successive resonance frequencies in an open
ends. The ratio of the speed of sound to the speed of organ pipe are 1944 Hz and 2592 Hz. What will be the
transverse wave travelling on the string is 2: 1. Find the length of the tube. The speed of sound in air is 324ms-1.
ratio of the length of pipe to the length of string.
(A) 20 cm (A) 25 cm (A) 33 cm (A) 16 cm
Q.7 An open organ pipe filled with air has a fundamental
frequency 500Hz. The first harmonic of another organ Q.2 A piston is fitted in a cylindrical tube of small cross
pipe closed at one end is filled with carbon dioxide has section with the other end of the tube open. The tube
the same frequency as that the first harmonic of the resonates with a tuning fork of frequency 412 Hz. The
open organ pipe. Calculate the length of each pipe. piston is gradually pulled out of the tube and it is found
Assume that the velocity of sound in air and in carbon that a second resonance occurs when the piston is
dioxide to be 330 and 164m/s respectively. pulled out through a distance of 320.0cm. What will be
the speed of sound in the air of the tube.
Q.8 A, B and C are three tuning forks. Frequency of A (A) 328 m/s (B) 300 m/s
is 350 Hz. Beats produced by A and B are 5 per second (C) 333 m/s (D) 316 m/s
by B and C are 4 per second. When a wax is put on A,
beat frequency between A and B is 2Hz and between
A and C is 6Hz. Then, find the frequency of B and C Q.3 The fundamental frequency of a closed pipe is 293
respectively. Hz when the air in it is at a temperature of 200C. What
will be its fundamental frequency when the temperature
changes to 220C?
Q.9 Tuning fork A when sounded with fork B of
frequency 480Hz gives 5 beats per second. When the (A) 300 Hz (B) 283 Hz (C) 294 Hz (D) 262 Hz
prongs of A are loaded with wax, it gives 3 beats per
second. Find the original frequency of A. Q.4 A tuning fork produces 4 beats per second with
another tuning fork of frequency 256Hz. The first one
Q.10 A car is moving towards a huge wall with a is now loaded with a little wax and the beat frequency
speed=c/10, where c=speed of sound in still air. A is found to increase to 6 per second. What was the
wind is also blowing parallel to the velocity of the original frequency of the tuning fork?
car in the same direction and with the same speed. If (A) 252 Hz (B) 220 Hz
the car sounds a horn of frequency f, then what is the
frequency of the reflected sound of the horn headed by (C) 250 Hz (D) 222 Hz
driver of the car?
Q.5 What will be the frequency of beats produced in air
Q.11 A fixed source of sound emitting a certain when two sources of sound are activated, one emitting
frequency appears as fa when the observer is a wavelength of 32 cm and the other 32.2 cm. The
approaching the source with speed v and frequency fr speed of sound in air is 350ms-1.
when the observer recedes from the source with same (A) 11 Hz (B) 13 Hz (C) 15 Hz (D) 7 Hz
speed. Find the frequency of the source.
Q6 A traffic policeman standing on a road sounds a
Q.12 Two stationary sources A and B are sounding whistle emitting a frequency of 2.00 kHz. What could
notes of frequency 680Hz. An observer moves from A be the apparent frequency heard by a scooter-driver
to B with a constant velocity u. If the sound is 340 ms-1, approaching the policeman at a speed of 36.0 kmh-1?
what must be the value of u so that he hears 10 beats
per second? (A) 1181 Hz (B) 1183 Hz
(C) 1185 Hz (D) 1187 Hz
1 2 . 3 2 | Sound Waves

Assertion Reasoning Type Paragraph 2:


(A) Statement-I is true, statement-II is true and Two trains A and B are moving with a speed 20 m/s and
statement-II is correct explanation for statement-I 30 m/s respectively in the same direction on the same
straight track, with B ahead of A, The engines are at the
(B) Statement-I is true, statement-II is true and statement-
front ends, The engine of train A blows a long whistle.
II is not the correct explanation for statement-I
Assume that the sound of the whistle is composed of
(C) Statement-I is true, statement-II is false.
components varying in frequency from f1=800 Hz to
(D) Statement-I is false, statement-II is correct. f2=1120 Hz, as shown in the figure. The spread in the
frequency (highest frequency lowest frequency) is thus
Q.7 Statement-I: When a closed organ pipe vibrates, 320Hz. The speed of sound in air is 340m/s.  (2007)
the pressure of the gas at the closed end remains

Intensity
constant.
Statement-II: In a stationary-wave system, displacement
nodes are pressure antinodes, and displacement
f1 f2 Frequency
antinodes are pressure nodes.
Q.4 The speed of sound of the whistle is
Q.8 Statement-I: The pitch of wind instruments rises (A) 340m/s for passengers in A and 310 m/s for
and that of string instruments falls as an orchestra passengers in B
warms up.
(B) 360m/s for passengers in A and 310 m/s for
Statement-II: When temperature rises, speed of sound passengers in B
increases but speed of wave in a string fixed at both
(C) 310 m/s for passengers in A and 360 m/s for
ends decreases.
passengers in B
(D) 340 m/s for passenger in both the trains.

Previous Years’ Questions Q.5 The distribution of the sound intensity of the
whistle as observed by the passenger in train A is best
Paragraph 1: represented by
Two plane harmonic sound waves are expressed by the
equations y1 ( =
x,t ) A cos ( πx − 100πt ) ;
Intensity

Intensity

(A) (B)

(All parameters are in MKS)  (2006)


f1 f2 Frequency f1 f2 Frequency
Q.1 How many times does an observer hear maximum
intensity in one second?
Intensity

Intensity

(A) 4 (B) 10 (C) 6 (D) 8 (C) (D)

f1 f2 Frequency f1 f2 Frequency
Q.2 What is the speed of the sound?
Q.6 The spread of frequency as observed by the
(A) 200 m/s (B) 180 m/s passenger in train B is
(C) 192 m/s (D) 96m/s (A) 310 Hz (B) 330 Hz (C) 350 Hz (D) 290 Hz

Q.3 At x=0, how many times is the amplitude of y1+y2 Q.7 Velocity of sound in air is 320m/s. A pipe closed at
zero in one second? one end has a length of 1m. Neglecting end corrections,
(A) 192 (B) 48 (C) 100 (D) 96 the air column in the pipe can resonate for sound of
frequency  (1989)
(A) 80 Hz (B) 240 Hz (C) 320 Hz (D) 400 Hz
P hysi cs | 12.33

Q.8 A sound wave of frequency travels horizontally to

0.4 m
the right and is reflected from a large vertical plane

1.2 m

1.2 m

2.8 m
surface moving to left with a speed ν . The speed of
sound in medium is c.  (1995)
(A) The number of waves striking the surface per

3.2 m
(c+ v)

2.4 m
second is f

1.6 m
c

0.8 m
c(c− v)
(B) The wavelength of reflected wave is
f(c+ v)

(c+ v) Q.12 An observer standing on a railway crossing


(C) The frequency of the reflected wave is f
(c− v) receives frequency of 2.2 kHz and 1.8 kHz when the
train approaches and recedes from the observer. Find
(D) The number of beats heard by a stationary listener
the velocity of the train.  (2005)
v
to the left of the reflecting surface is f. (The speed of the sound in air is 300 m/s)
c− v

Q.9 A source of sound of frequency 256 Hz is moving Q.13 A stationary source is emitting sound at a fixed
rapidly towards a wall with a velocity of 5m/s. How frequency f0, which is reflected by two cars approaching
many beats per second will be heard by the observer the source. The difference between the frequencies of
on source itself if sound travels at a speed of 330 m/s? sound reflected from the cars is 1.2% of f0. What is the
 (1981) difference in the speed of the cars (in km per hour) to
the nearest? The cars are moving at constant speeds
much smaller than the speed of sound which is 330ms-1.
Q.10 A source of sound is moving along a circular
 (2010)
path of radius 3m with an angular velocity of 10 rad/s.
A sound detector located far away from the source is
executing linear simple harmonic motion along the Q.14 A police car with a siren of frequency 8 kHz is
line BD (see figure) with an amplitude BCD=6m. The moving with uniform velocity 36 km/hr towards a tall
frequency of an oscillation of the detector is 5/ π per building which reflects the sound waves. The speed
second. The source is at the point A when the detector of sound in air is 320 m/s. The frequency of the siren
is at the point B. If the source emits a continuous sound heard by the car driver is (2011)
wave of frequency 340 Hz, find the maximum and the (A) 8.50 kHz (B) 8.25 kHz
minimum frequencies recorded by the detector. (Speed
of sound=340 m/s)  (1990) (C) 7.75 kHz (D) 7.50 kHZ

6m 6m Q.15 A person blows into open-end of a long pipe.


As a result, a high pressure pulse of air travels down
3m

A B C D
the pipe. When this pulse reaches the other end of the
pipe,  (2012)

Q.11 A 3.6 m long pipe resonates with a frequency (A) A high-pressure pulse starts travelling up the pipe,
212.5 Hz when water level is at a certain height in the if the other end of the pipe is open.
pipe. Find the heights of water level (from the bottom (B) A low-pressure pulse starts travelling up the pipe, if
of the pipe) at which resonances occur. Neglect end the other end of the pipe is open.
correction. Now the pipe is filled to a height H(≈ 3.6
m). A small hole is drilled very close to its bottom and (C) A low-pressure pulse starts travelling up the pipe, if
water is allowed to leak. Obtain an expression for the the other end of the pipe is closed.
rate of fall of water level in the pipe as a function of H. (D) A high-pressure pulse starts travelling up the pipe,
If the radii of the pipe and the hole are 2 × 10–2 m and if the other end of the pipe is closed.
1 × 10–3 m respectively, calculate the time interval
between the occurrence of first two resonances. Speed
of sound in air is 340 m/s and g=10m/s2  (2000)
1 2 . 3 4 | Sound Waves

Q.16 A student is performing an experiment using a (C) The plot below represents schematically the
resonance column and a tuning fork of frequency 244 s-1. variation of beat frequency with time
He is told that the air in the tube has been replaced
by another gas (assume that the column remains filled v(t)
with the gas). If the minimum height at which resonance P
occurs is (0.350 ± 0.005)m, the gas in the tube is
Q
vQ
(Useful information: 167RT = 640 J mole
1/ 2
;
-1/ 2

140RT =590 J mole 1/2


. The molar masses M in
-1/ 2
R
t
10 (D) The rate of change in beat frequency is maximum
grams are given in the options. Take the values of
M when the car passes through Q.
for each gas as given there.) (2014)

 Q.18 A student performed the experiment to measure


10 7 
(A) Neon
=  M 20,
=  the speed of sound in air using resonance air-column
 20 10 
 method. Two resonances in the air-column were
obtained by lowering the water level. The resonance
 10 3  with the shorter air-column is the first resonance and
(B) Nitrogen
=  M 28,
= 
 28 5  that with the longer air column is the second resonance.

Then, (2009)
 10 9 
(C) Oxygen
=  M 32,
=  (A) The intensity of the sound heard at the first
 32 16  resonance was more than that at the second resonance

 (B) The prongs of the tuning fork were kept in a


10 17 
(D) Argon
=  M 36,
=  horizontal plane above the resonance tube
 36 32 

(C) The amplitude of vibration of the ends of the prongs
is typically around 1 cm
Q.17 Two loudspeakers M and N are located 20 m apart
and emit sound at frequencies 118 Hz and 121 Hz, (D) The length of the air-column at the first resonance
respectively. A car is initially at a point P, 1800 m away was somewhat shorter than 1/4th of the wavelength of
from the midpoint Q of the line MN and moves towards the sound in air
Q constantly at 60 km/hr along the perpendicular
bisector of MN. It crosses Q and eventually reaches a Q.19. A hollow pipe of length 0.8 m is closed at one
point R, 1800 m away from Q. Let v(t) represent the end. At its open end a 0.5 m long uniform string is
beat frequency measured by a person sitting in the car vibrating in its second harmonic and it resonates with
at time t. Let vP, vQ and vR be the beat frequencies the fundamental frequency of the pipe. If the tension in
measured at locations P, Q and R, respectively. The the wire is 50 N and the speed of sound is 320 ms−1, the
speed of sound in air is 330 m/s. Which of the following mass of the string is (2010)
statement(s) is(are) true regarding the sound heard by
(A) 5 grams (B) 10 grams
the person? (2016)
(C) 20 grams (D) 40 grams
(A) The plot below represents schematically the
variation of beat frequency with time
Q.20 A student is performing the experiment of
v(t) Resonance Column. The diameter of the column tube
is 4 cm. The distance frequency of the tuning for k is
P
512 Hz. The air temperature is 38°C in which the speed
vQ
Q of sound is resonance occurs, the reading of the water
level in the column is (2012)
R
t (A) 14.0 (B) 15.2 (C) 16.4 (D) 17.6
(B) vP + vR = 2vQ
P hysi cs | 12.35

Q.21 Two vehicles, each moving with speed u on the (C) If the wind blows from observer to the source, f2 <
same horizontal straight road, are approaching each f1.
other. Wind blows along the road with velocity w. One
(D) If the wind blows from the source to the observer
of these vehicles blows a whistle of frequency f1 . An
f2 < f1.
observer in the other vehicle hears the frequency of the
whistle to be f2 . The speed of sound in still air is V. The
correct statement(s) is (are) (2013) Q.22 Four harmonic waves of equal frequencies and
equal intensities I0 have phase angles 0, π /3, 2 π /3
(A) If the wind blows from the observer to the source, and π . When they are superposed, the intensity of the
f2 > f1. resulting wave is nI0. The value of n is (2015)
(B) If the wind blows from the source to the observer,
f2 > f1.

PlancEssential Questions
JEE Main/Boards JEE Advanced/Boards
Exercise 1 Exercise 1
Q. 6 Q.7 Q.8 Q. 1 Q.6 Q.8
Q.12

Exercise 2 Exercise 2
Q. 1 Q.2 Q.3 Q.1 Q.2 Q.3
Q.12 Q.13 Q.14 Q.8 Q.14 Q.15
Q.19 Q.21 Q.16 Q.24 Q.26

Answer Key

JEE Main/Boards
Exercise 1
Q.1 382.2 ms-1 Q.2 (i) 0.25 ×10−3 cm (ii) π / 250 s (iii)500rad (iv)80π m (v) 0.125 cm/ s (iv)62.5cm / sec2
Q.3 353.6 ms-1
Q.4 (a) No wave (b) longitudinal waves (c) longitudinal (d) transverse or longitudinal or both (separately) (e)
combined longitudinal and transverse (ripples)
Q.5 The speed of sound in a perfectly rigid rod will be infinite
1 2 . 3 6 | Sound Waves

Q.6 4.2 s Q.7 1.32 km Q.8 330 ms-1, 0.02 m; 0.033 m Q.9 205 Hz

Exercise 2

Single correct choice type


Q.1 D Q.2 B Q.3 C Q.4 D Q.5 D Q.6 B
Q.7 A Q.8 C Q.9 A Q.10 A Q.11 A Q.12 C

Previous Years’ Questions


Q.1 B Q.2 A Q.3 B Q.4 D Q.5 A Q.6 A
Q.7 0.125 Q.8 240 Q.9 False Q.10 True Q.11 False Q.12 B
Q.13 B Q.14 C Q.15 A Q.16 A Q.17 B Q.18 C

JEE Advanced/Boards
Exercise 1

π2 × 10−9
Q.1 W / m2 Q.2 2C/3 Q.3 30 dB, 10 10 µm
4
Q.4 6 Q.5 λ / 8 Q.6 1:1
Q.7 33 cm and 13.2 cm Q.8 345, 341 or 349 Hz Q.9 485 Hz
fr + fa
Q.10 11f/9 Q.11 Q.12 2.5 ms-1
2

Exercise 2

Single Correct Choice Type

Q.1 B Q.2 A Q.3 C Q.4 A Q.5 D Q.6 A

Assertion Reasoning Type

Q.7 D Q.8 A

Previous Years’ Questions


Q.1 A Q.2 A Q.3 C Q.4 B Q.5 A Q.6 A
Q.7 A, B, D Q.8 A, B, C Q.9 7.87 Hz Q.10 438.7 Hz, 257.3 Hz

Q.11 3.2 m, 2.4 m, 1.6 m, 0.8 m, −


dH
dt
= (1.11× 10 ) −2
H , 43s Q.12 vT= 30 m/s

Q.13 7 Q.14 A Q.15 B, D Q.16 D Q.17 A, B, D Q.18 A, D


Q.19 B Q.20 B Q.21 A, B Q.22 3
P hysi cs | 12.37

Solutions

JEE Main/Boards 2 × 78.4


Time to reach water = =4s
9.8
Exercise 1
78.4
Time for sound to reach top = = 0.23 s
4 4 332
Sol 1: V’ = V= × 331
3 3 Total time = 4.23 s
= 382.2 ms–1
Sol 7:
Sol 2: (i) A = 0.25 × 10–3 cm
2π π
(ii) T = = s
500 250
(iii) ω : 500 rad/s

(iv) λ = m = 80pm
0.025  1 1 
⇒d  –  =8
(v) Vmax = 0.25 × 10–3 × 500 cm s–1  330 3 × 108 
Vmax. = 0.125 cms–1 ⇒ d (3× 108 – 330)= 8 × 330 × 3 × 108
(vi) amax. = Vmax w = 0.125 × 500 ⇒d ~ = 8 × 330
amax. = 62.5 cms–2 ⇒ d = 264 m
Height of cloud = 1320 m = 1.32 km
V V 20
Sol 3: – =
2.04 2.08 6
Sol 8: f = 250 Hz
 0.04  20
V   = λ
 2.04 × 2.05  6 ⇒ (31+ h) =
2
V = 353.6 ms–1 3λ
⇒ (97 + h) =
4
Sol 4: (a) No wave possible as there is no particle. λ
⇒ 66 =
(b) Longitudinal waves (direction of motion of particles 2
parallel to direction of propagation of wave) λ = 132 cm
(c) Longitudinal V = fλ = 250 × 1.32 ms–1
(d) Both are possible V = 330 ms–1
(e) Combined longitudinal & transverse (ripples) H = 132/4-31 = 2 cm = 0.02 m
End Cross Section
Sol 5: Infinite as young’s modulus of a rigid body is Radius of tube =
0.6
infinite
0.02 0.2 0.1
= = = = 0.033 m
0.6 6 3
Sol 6:
V
Sol 9: –f=5
2

V
F– =5
2.1
1 2 . 3 8 | Sound Waves

V V So the maximum time interval will be


– = 10
2 2.1 T=5/v=160/320=0.5seconds.
V = 420 ms –1

420 Sol 6: (B) He has to clap 10 times in 3 seconds.


F=5+ = 205 Hz
2.1 So time interval between two clap =(3/10 second).
So the time taken go the wall
Exercise 2
=(3/2 x 10)=3/20 seconds =333 m/s.
Single Correct Choice Type
Sol 7: (A) For minimum wavelength n=20 KHZ
Sol 1: (D) d = u t0  1450 
⇒ v = nλ ⇒ λ =   = 7.25cm.
⇒ d = v(t0 + t)  20 × 103 
⇒ (v – u)t0 + vt = 0 (b) For maximum wavelength n should be minimum
vt
t0 = ⇒ v = nλ ⇒ λ = v / n ⇒ 1450 / 20 = 72.5m.
u– v
uvt I 
d= Sol 8: (C) Weknow that β =10log10  
u– v  I0 
IA I
Sol 2: (B) βA 10log=
= , βB 10log B
I0 I0

10(
βA /10 )
10(
βB /10 )
⇒ IA / I=
0 ⇒ IB / I=
0

λ 2l 4l 2
IA rB2  50  102
=   ⇒ 10( A B )
β β =
7λ ⇒ =
= IB rA  5 
2
2
2 βA − βB
λ= ⇒ 2 ⇒ β=
= B 40 − 20
= 20dβ
7 10
λ ratio : 1 : 2 : 4
ν ratio : 4 : 2 : 1 Sol 9 (A) According to the given data
V=336m/s,
5 × 10 –3
Sol 3: (C) µ = = 0.01 λ / 4 =distance between maximum and minimum
0.5
intensity
T = 400 N

(n + 1) 400 = (20 cm)=


⇒ λ 80 cm
ν= V 336
2 × 0.5 0.01 ⇒ n = frequency = = = 420Hz.
λ 80 × 10−2
ν = 200(n + 1) < 104 20cm
=x/4
⇒ (n + 1) < 50
D
S
⇒ n < 49

I 
Sol 4: (D) 10 log  2  = 50
 I1 
Sol 10: (A) According to the data
I2 = I1 × 105
=λ 20cm,S
= 1S2 20cm,BD
= 20cm
Sol 5: (D) Here given S=80m x 2=160m. Let the detector is shifted to left for a distance x for
hearing the minimum sound.
V=320m/s
P hysi cs | 12.39

So path difference Al=BC-AB vB = speed of train B


Solving Eqs. (i) and (ii), we get
( 20 ) + (10 + x ) ( 20 ) + (10 − x )
2 2 2 2
= −
vB
=2
So the minimum distances hearing for minimum vA

=
( 2n + 1) λ= λ 20
= = 10cm Sol 2: (A) Let f0 = frequency of tuning fork
2 2 2
5 9g
Then, f0 = (µ = mass per unit length of wire)
( 20 ) + (10 + x ) ( 20 ) + (10 − x )
2 2 2 2
⇒ − 10
= 2 µ

Solving we get x=12. 0 cm. 3 Mg


=
2 µ
Sol 11: (A) Here given that 1=50cm, v=340m/s Solving this, we get M = 25 kg
As it is an open organ pipe, the fundamental frequency In the first case, frequency corresponds to fifth
f1 =(v/21) harmonic while in the second case it corresponds to
third harmonic
340
= = 340Hz.
2 × 50 × 10−2
Sol 3: (B) Let D be the end correction.
So, the harmonies are
Given that, fundamental tone for a length 0.1 m = first
f3=3 x 340=1020 Hz overtone for the length 0.35 cm.
f5=5 x 340=1700, f6=6 x 340=2040 Hz v 3v
=
So, the possible frequencies are between 1000Hz and 4(0.1 + ∆ ) 4(0.35 + ∆ )
2000Hz are 1020, 1360, 1700.
Solving this equation, we get D = 0.025 m = 2.5 cm

Sol 12: (C) According to the questions f1 first overtone


Sol 4: (D) The frequency is a characteristic of source. It
of a closed organ pipe
is independent of the medium.
3× V
=P1 3v
= / 4I
4 × 30 Sol 5: (A) With increase in tension, frequency of
V vibrating string will increase. Since number of beats are
f2 fundamental frequency of a open organ pipe P2 =
2I2 decreasing. Therefore, frequency of vibrating string or
3V V third harmonic frequency of closed pipe should be less
Here given = ⇒ I2 = 20cm than the frequency of tuning fork by 4.
4 × 30 2I2
∴ Frequency of tuning fork
∴ Length of the pipe P2 will be 20 cm.
= Third harmonic freq1uency of closed pipe + 4

 v   340 
= 3   + 4 = 3  + 4 = 344 Hz
Previous Years’ Questions  4   4 × 0.75 

 v + v0  5
Sol 1: (B) Using the formula f’ = f   Sol 6: (A) 36 km/h = 36× = 10 m/s
 v  18
 v + vA 
we get, 5.5 = 5   ….. (i)
 v 
 v + vB 
and 6.0 = 5   ….. (ii)
 v 
Hence, v = speed of sound
vA = speed of train A
1 2 . 4 0 | Sound Waves

Apparent frequency of sound heard by car driver from a denser medium to rarer medium it refracts away
(observer) reflected from the building will be from the normal

 v + v0   320 + 10 
f’ = f   = 8   = 8.5 kHz Sol 11: For reflected wave an image of source S’ can
 v – vs   320 – 10  assumed as shown. Since, both S and S’ are approaching
towards observer, no beats will be heard
Sol 7: Wall will be a node (displacement). Therefore,
shortest distance from the wall at which air particles
have maximum amplitude of vibration (displacement
antinode) should be λ/4 S O S’
v 330
Here, λ = = = 0.5 m
f 660
0.5
∴ Desired distance is = 0.125 m
4
Wall
v T µ
Sol 8: Fundamental frequency f = =
2 2
Sol 12: (B)
or f ∝ T
1 γRT 1 γRT
f' w –F =n ⇒
= xn
= 4x M 4 M
f w ⇒x∝ T
Here, w = weight of mass and
F = upthrust Sol 13: (B) Motor cycle, u = 0, a = 2 m/s2
Observer is in motion and source is at rest.
w –F
f’ = f
w v − v0 94 330 − v 0

= n' n ⇒ = n n ⇒ 330 − v 0
Substituting the values, we have v + vs 100 330

330 × 94
(50.7)g – (0.0075)(103 )g =
f’ = 260 = 240 Hz 100
(50.7)g
94 × 33 33 × 6
⇒ v 0 = 330 − = m/s
10 10
γRT v 2 − u2 9 × 33 × 33 9 × 1089
Sol 9: vsound = =s = =  98m
M 2a 100 100

vH γH / MH (7 / 5) / 2 42
2
= 2 2
= = Sol 14: (C) Maximum number of beats = ν + 1− (ν −1)
vHe γHe / MHe (5 / 3) / 4 25 =2

v v
Sol 10: Sound wave Sol 15: (A)
= f0 = , fC
2 2

Air Denser Sol 16: (A) P + x = P0


P = (76 – x)
Water 8 × A × 76 = (76 – x) × A × (54 – x)
Rarer
x = 38
Length of air column = 54 – 38 = 16 cm.
For sound wave water is rarer medium because speed
of sound wave in water is more. When a wave travels
P hysi cs | 12.41

γRT
(54-x) Sol 2: V =
P M
54 cm
8 cm
x RT C
=
M 3
2 4
γ=1+ =
6 3

C 4 C
ν= γ× = ×
3 3 3
Sol 17: (B)
2
ν= C
 c   320  3
fbefore
= cros sing f0=   1000  
 c − vs   320 − 20  1
 c  Sol 3: For linear source, Intensity ∝
 320  R
fafter
= f0=   1000  
cros sing 1
 c + vs   320 + 20  A∝
1/2
 2cv  R
s 
∆f =f0 
 c2 − v 2  ∴ At 10R
 s 
∆f 2 × 320 × 20 I / 10
× 100% = × 100
= 12.54% ≈ 12% Loudness =10 log = 40 dB –10 dB
f 300 × 340 I0
Loudness = 30 dB
Sol 18: (C) Open organ pipe
0.01
Amplitude = cm = 10 10 mm
V 10
f=  ...(i) …(i)
2
For closed organ pipe Sol 4: I’ = 4I
V V 4I
f'
= = = f Loudness = 10 log = 10(log 4 + L0)
   2 I0
4 
2 = 20 log 2 + L0= 6.010 + L0
= 6.010 + L0 = L0 + 6.01 dB
n = 6.01 dB
JEE Advanced/Boards
Sol 5: I = I1 + I2 + 2 I I cosf
12
Exercise 1
Here I1 = I2
B I = 2I1(1+ cos φ)
Sol 1: f = 250 Hz V = = 20 ms–1A = 10–8m
ρ
I0 = 4I1
ρ = 1 kg/m 3
I0/2 = 2I1 = 2I1(1 + cos φ)
B = 400 N/m2 π
cos φ = 0 ⇒ φ =
2
B ωS 0 400 × 2π × 250 × 10 –8
p0 = = (2∆x)
V 20 ⇒ φ = 2π
λ
p0 = 3.14 × 10–4 N/m2 π
×λ
λ
p20 ⇒ Dx = 2 =
I= = 2.467 × 10–9 W/m2 2π × 2 8
2ρV
Intensity = 2.467 × 10–9 W/m2
1 2 . 4 2 | Sound Waves

Vp ∴ fA > fB
Sol 6: =2
Vs fA = 485 Hz

3 3
V = V Sol 10:
4 p p 2 p s

p Vp
⇒ = =1
s 2Vs
(C + C / 10)
⇒ lp : ls = 1 : 1 fw = × f0
(C + C / 10) – C / 10
νA
Sol 7: 500 = fw = 11 f0
λ0
10
 C C
Closed pipe: l0 =
330
= 2l1 C –  +
10  10
500 fd = fw × 
 C
l1 =
330
m = 0.33m C – 
 10 
1000
10
l1 = 0.33 m = f
9 w
264
Open pipe: 4l2 = 10 11
500 = × f
9 10 0
l2 = 0.132 m
11
⇒ fd = f
9 0
Sol 8: fA = 350 Hz
C+ν
|fA – fB| = 5 Hz Sol 11: fa = f
C
|fB – fC| = 4 Hz
fr = C – ν f
After waxing C
| fA1 – fB| = 2Hz fa + fr
⇒f=
| fA1 – fc| = 6 Hz 2
fA > fB initially as on waxing fA decreases.
 C + u  C – u 
fA – fB 5Hz ⇒ fB = 345 Hz Sol 12: f  –   = 10
 C  C 
Case-I : FB > FC FB – FC = 4 Hz ⇒ fC = 341 Hz 2fu
= 10
fA1 = 347 Hz or 343 Hz C
5C 5 × 340
fC = 341 Hz u= ⇒u=
f 680
fA' = 347 Hz
⇒ u = 2.5 ms–1
fB = 345 Hz
fC = 341 Hz
Case-II : fC > fB Exercise 2
fC = 349 Hz
Single Correct Choice Type
fA' = 343 Hz
fB = 345 Hz Sol 1: (B) Let the length of the resonating column will
be=1
Sol 9: fB = 480 Hz Here V=320 m/s
|fB – fA| decreases on waxing Then the two successive resonance frequencies are
P hysi cs | 12.43

(n+ 1) v nv
and Sol 6: (A) Here given f=
s 16 × 103 Hz
4I 4I
(n+ 1) v nv
Here given = 2592; =
λ = 1944 f ' 20 × 103 Hz (greater than that
Apparent frequency =
4I 4I value)
(n+ 1) v nv
⇒ − = 2592 − 1944 Let the velocity of the observer =v0
4I 4I
= 548cm
= 25cm. Given vs=0. So,

 330 + v 0  3
Sol 2: (A) Let, the piston resonates at length l1 and l2 20 × 10 
=  × 16 × 10
 330 + 0 
Here, l=32cm; v=?,n=512 Hz
Now ⇒ 512
= v/λ⇒=
v 512 × 0.64
= 328m / s 20 × 330 − 16 × 330 330

= v0 = m
= / s 297km / h
4 4
Sol 3: (C) We know that the frequency = f, T = temperatures
I1
f∝ T
I2
f1 T1 293 293 (I2-I1)
So = ; ⇒ =
f2 T2 f2 295

293 × 295
⇒ f2
= = 294 Hz
293 Assertion Reasoning Type

Sol 4: (A) A tuning fork produces 4 beats with a known Sol 7: (D) Closed end is displacement node. So, it must
tuning fork whose frequency =256 Hz be pressure antinode.
So the frequency of unknown tuning fork=either 256-
4=252 or 256+4=260 Hz Sol 8: (A) Statement-II explains statement-I

Now as the first one is load its mass/unit length


increases. So, its frequency decreases.
As it produces 6 beats now original frequency must be Previous Years’ Questions
252 Hz.
Sol 1: (A) In one second number of maximas is called
260 Hz is not possible as on decreasing the frequency
the beat frequency.
the beats decreases which is not allowed here.
100π 92π
Hence, fb = f1 – f2 = – = 4 Hz
Sol 5: (D) 2π 2π

ω
Group I Group II Sol 2: (A) Speed of wave v =
k
Given V=350 V=350
100π 92π
or v = or = 200 m/s
0.5π 0.46π
= 32 × 10−2 m
λ1 =32cm λ2 = = 32.2 × 10−2 m
32.2cm
Sol 3: (C) At x = 0, y = y1 + y2 = 2A cos 96pt cos 4 pt
So η2 350 / 32 × 10−2=
= −2
= η2 350 / 32.2 × 10 = 1086
Frequency of cos (96 pt) function is 45 Hz and that of
1093 Hz Hz
cos (4pt) function is 2Hz.
So beat frequency =1093-1086=7 Hz. In one second, cos function becomes zero at 2f times,
where f is the frequency. Therefore, first function will
become zero at 96 times and the second at 4 times.
But second will not overlap with first. Hence, net y will
become zero 100 times in 1 s.
1 2 . 4 4 | Sound Waves

Sol 4: (B) vSA = 340 + 20 = 360 m/s Sol 9: Frequency heard by the observer due to S’
(reflected wave)
vSB = 340 – 30 = 310 m/s
 v + v0 
f’ = f  
 v – vs 
 330 + 5 
= 256   = 263.87 Hz
 330 – 5 
Sol 5: (A) For the passengers in train A. There is no
relative motion between source and observer, as both
are moving with velocity 20 m/s. Therefore, there is no
S S’
change in observed frequencies and correspondingly 5 m/s
there is no change in their intensities. 5 m/s

Sol 6: (A) For the passengers in train B, observer is Wall


receding with velocity 30 m/s and source is approaching
with velocity 20 m/s. ∴ Beat frequency fb = f’ – f = 7.87 Hz
 340 – 30 
f1' = 800   = 775 Hz Sol 10: Angular frequency of detector
 340 – 20 
5
 340 – 30  ω = 2pf = 2π   = 10 rad/s
and f2' = 1120   = 1085 Hz π
 340 – 20 
Since, angular frequency of source of sound and of
∴ Spread of frequency = f2' – f1' = 310 Hz detector are equal, their time periods will also be equal.

 v 
Sol 7: (A, B, D) For closed pipe, f = n   ; n = 1, 3,
5 ….  4 

v 320
For n = 1, f1 = = = 80 Hz
4 4 ×1 Maximum frequency will be heard in the position
shown in figure. Since, the detector is far away from the
For n = 3, f3 = 3f1 = 240 Hz
source, we can use,
For n = 5, f5 = 5f1 = 400 Hz
 v + v0 
fmax = f  
Sol 8: (A, B, C) Moving plane is like a moving observer.  v – vs 
Therefore, number of waves encountered by moving Here, v = speed of sound = 340 m/s
plane.
(given) vs = Rω = 30 m/s
 v + v0  c+ v vo = ωA = 60 m/s
f1 = f   = f  
 v   c  (340 + 60)
∴ fmax = 340 = 438.7 Hz
Frequency of reflected wave, (340 − 30)

f2 = f1  v  = f c+ v
  
v–v
 s  c– v

Wavelength of reflected wave

v c c c– v Minimum frequency will be heard in the condition


l2 = = =  
f2 f2 f c+ v shown in figure. The minimum frequency will be:

 v − v0  (340 − 60)
fmin = f   = 340 =257.3 Hz
 v + v s  (340 + 30)
P hysi cs | 12.45

Sol 11: Speed of sound v = 340 m/s Velocity of efflux, v = 2gH


Let l0 be the length of air column corresponding to the
fundamental frequency. Then, Continuity equation at 1 and 2 gives

v  –dH 
= 212. 5 a 2gH = A  
4 0  dt 
v 340 ∴ Rate of fall of water level in the pipe,
or l0 = = = 0.4 m
4(212.5) 4(212.5)
 –dH  a
In closed pipe only odd harmonics are obtained. Now   = 2gH
 dt  A
let l1, l2, l3, 4, etc., be the lengths corresponding to the
3rd harmonic, 4th harmonic, 7th harmonic etc. Then Substituting the values, we get

–dH 3.14 × 10 –6
0.4 m

= 2 × 10 × H
dt 1.26 × 10 –3
1.2 m

1.2 m

2.8 m
dH
or – = (1.11 × 10–2) H
dt
3.2 m

Between first two resonances, the water level falls from


2.4 m

1.6 m

3.2 m to 2.4 m.
0.8 m

dH
∴ = – (1.11 × 10–2) dt
H
 v 
3   = 212. 5 ⇒ l1 = 1.2 m 2.4 1
dH
 4 1  or ∫ = – (1.11 × 10–2) ∫ dt
3.2 H 0
 v 
5   = 212. 5 ⇒ l2 = 2.0 m
 4 2  or 2[ 2.4 – 3.2 ] = – (1.11 × 10–2)t
 v 
and 7   = 212. 5 ⇒ l3 = 2.8 m or t = 43 s
 4 3 
Note: Rate of fall of level at a height h is
 v 
9   = 212. 5 ⇒ 4 = 3.6 m  –dh  a
 4 4    = 2gh ∝ h
 dt  A
or heights of water level are (3.6 – 0.4) m, (3.6 – 1.2) m, i.e., rate decreases as the height of water (or any other
(3.6 – 2.0)m and (3.6 – 2.8)m. liquid) decreases in the tank. That is why, the time
∴ Heights of water level are 3.2 m, 2.4 m, 1.6 m and required to empty the first half of the tank is less than
0.8 m the time required to empty the rest half of the tank.
Let A and a be the area of cross-sections of the pipe  v 
and hole respectively. Then Sol 12: From the relation, f’ = f   ,
 v ± vs 
A = π(2 × 10–2)2 = 1.26 × 10–3 m2
 300 
and a = π(10–3)2 = 3.14 × 10–6 m2 we have 2.2 = f   …. (i)
 300 – v T 

 300 
and 1.8 = f   ….. (ii)
 300 + v T 

Here, vT = vs = velocity of source/train
Solving Eqs. (i) and (ii), we get
vT = 30 m/s
1 2 . 4 6 | Sound Waves

Sol 13: Firstly, car will be treated as an observer which Sol 17: (A, B, D) Frequency of M received by car
is approaching the source. Then, it will be treated as a
source, which is moving in the direction of sound.  V + V0 cos θ 
f1 = 118  
 V 
 V + V0 cos θ 
f2 = 121 
 V 
 v + v1 
Hence, f1 = f0  
 v – v1  M (118 Hz)

 v + v2 
f2 = f0   v0=60 km
 v – v2 

 1.2   v + v1 v + v 2 
∴ f1 – f2 =   f0 = f0  – 
P Q R
 100   v – v1 v – v 2 
1800m 1800m

 1.2  2v(v1 – v 2 )
or   f0 = f
 100  (v – v1 )(v – v 2 ) 0
M (118 Hz)
As v1 and v2 are very very less than v.
We can write, (v – v1) or (v – v2) ≈ v
No. of beats n = ∆f=f2 − f1
 1.2  2(v1 – v 2 )
∴   f0 = f0  V + V0 cos θ 
 100  v n = 3  
 V 
v × 1.2 330 × 1.2
or (v1 – v2) = = = 1.98 ms–1  V0 
200 200 n= 3 1 + cos θ 
 V 
= 7.128 kmh–1
As θ ↑, cos θ ↓, n ↓
∴ The nearest integer is 7
dn V 
Rate of change of beat frequency = 3  0 ( − sin θ)
dθ V 
320 320 + 10
Sol 14: =
(A) f × 8 × 103 × = 8.5kHz dn
320 − 10 320 is maximum when sinθ = 1; θ=90°

i.e. car is at point Q.
So 15: (B, D) At the open end, the phase of a pressure
wave changes by π radian due to reflection. At the  V 
νp= 3 1 + 0 cos θ 
closed end, there is no change in the phase of a pressure V
 
wave due to reflection.
 V0 
νR= 3 1 − cos θ 
 V 
1 γRT
Sol 16: (D)  =
4v M At Q
No. of beats νQ =121-118 = 3
1 γRT
Calculations for for gases mentioned in ν + νR
4v M νQ =P
2
options A, B, C and D, work out to be 0.459 m, 0.363 m
0.340 m & 0.348 m respectively. As  = (0.350 ± 0.005)
m; Hence correct option is D. Sol 18: (A, D) Larger the length of air column, feebler
is the intensity.
P hysi cs | 12.47

Sol 19: (B)

T
2
vS µ
=
4LP 2 S
10 gm
µ S =

Sol 20: (B)

V
=f
4( + e)
V V
⇒  + e= ⇒ = −e
4f 4f

Here e=(0.6)r =(0.6)(2) =1.2 cm

336 × 102
So 
= − 1.2 15.2cm
=
4 × 512

Sol 21: (A, B) If wind blows from source to observer

 V + w +u
f2 = f1  
 V + w −u
When wind blows from observer towards source

 V − w +u
f2 = f1  
 V − w −u
In both cases, f2 > f1.

Sol 22: First and fourth wave interfere destructively. So


from the interference of 2nd and 3rd wave only,
 2π π 
⇒ Inet = I0 + I0 + 2 I0 I0 cos  −  = 3I0
 3 3
⇒n= 3
2017-18 100 &
op kers
Class 11 T
By E ran culty
-JE Fa r
IIT enior emie .
S fP r es
o titut
Ins

PHYSICS
FOR JEE MAIN & ADVANCED
SECOND
EDITION

Exhaustive Theory
(Now Revised)

Formula Sheet
9000+ Problems
based on latest JEE pattern

2500 + 1000 (New) Problems


of previous 35 years of
AIEEE (JEE Main) and IIT-JEE (JEE Adv)

5000+Illustrations and Solved Examples


Detailed Solutions
of all problems available

Topic Covered Plancess Concepts


Tips & Tricks, Facts, Notes, Misconceptions,
Kinetic Theory of Gases Key Take Aways, Problem Solving Tactics
and Thermodynamics
PlancEssential
Questions recommended for revision
13. KINETIC THEORY
OF GASES AND
THERMODYNAMICS

KINETIC THEORY OF GASES (KTG)

1. INTRODUCTION
The kinetic theory of gases describes a gas as a large number of small particles (atoms or molecules), all of which
are in constant, random motion. The rapidly moving particles constantly collide with each other and with the walls
of the container. Kinetic theory explains macroscopic properties of gases, such as pressure, temperature, viscosity,
thermal conductivity and volume, by considering their molecular composition and motion. The theory postulates
that gas pressure is due to the impact of molecules or atoms moving at different velocities on the walls of a
container.
But here we will only find the dependency of pressure on velocity and of velocity on temperature.

2. ASSUMPTIONS OF KTG
(a) The collisions b/w gas molecules or between a molecule and the container are completely elastic.
(b) Mass of gas molecules is negligible. So, the effect of gravity and gravitational force on each other can be
neglected.
(c) Volume of molecules is considered to be negligible.
(d) Collision b/w molecules is governed by Newton’s Law of Motion (i.e. Net force = Change in momentum per
unit time)
(e) Interaction force b/w particles is negligible. (That is, they exert no forces on one another.)
(f) Molecules are considered to be in constant, random, and rapid motion. So, average velocity of particles in all
the 3-directions will be zero i.e. <vx> = <vy>=<vz>=0.

PLANCESS CONCEPTS

These assumptions must always be kept in mind throughout the chapter.


Vaibhav Gupta (JEE 2009 AIR 54)
1 3 . 2 | Kinetic Theor y of Gases and Thermodynamics

3. PRESSURE OF AN IDEAL GAS


Applying Newton’s Laws to an ideal gas under the assumptions of kinetic theory allows the determination of the
average force on container walls. This treatment assumes that the collisions with the walls are perfectly elastic.

Force of molecular Before


Perfectly
collision with wall vx
elastic
 For N identical
Ft = p = 2mvx After collision
with wall molecules :
vx
l
vx
vx
2L
The time for a ‘’round trip’ is t =
vx L
2
2mvx mv x
So the average force is F = =
2L L l
For N molecules :
vx

Figure 13.1

An overbar indicates an average quantity. In the expression for the average force per molecule
m  v1x
2 2
+ v 2x 2
+ v 3x 2 
+  vNx
for N molecules. F =  
L

v2 + v2 + v2 +  v2 
but this can be related to the average: v 2x = 
1x 2x 3x Nx 

It is important to note that it is the average of the square of the velocity which is used, and that this is distinctly
different from the square of the average velocity. The average of the square of the velocity is referred as square of
root mean square speed.

mNv 2x
i.e. Vrms = v 2 ; and for N molecules: F =
L

And, since the molecular motion is random according to KTG, so v=


2
x
2
v=
y v 2z

PLANCESS CONCEPTS

Why are the RMS velocities the same in all the directions? Practically, if we use some pressure measuring
device and use it from all directions, we would get the same value of pressure which indicates that RMS
velocities must be same in all directions because it is directly proportional to RMS velocity.
Chinmay S Purandare (JEE 2012 AIR 698)

Thus, v 2 = v 2x + v 2y + v 2z = 3v 2x ;

mNv 2
Now, average force from N molecules can be re-written as Faverage =
3L
Favg mNv 2 mNv 2 N
Then the pressure in a container can be expressed=
as P= = = mv 2
A 3LA 3V 3V
P hysi cs | 13.3

1 2 P
P= ρv ρ: density of gas molecules in the container; Therefore,Vrms
Vrms = 3
3 ρ

2N  1 2 
Expressed in terms of average molecular kinetic energy: P =  mv 
3V  2 

4. KINETIC INTERPRETATION OF TEMPERATURE


The expression for gas pressure developed from kinetic theory relates pressure and volume to the average molecular
kinetic energy. Comparison with the ideal gas law leads to an expression for temperature sometimes referred to as
the kinetic temperature.
PV = nRT
2 1  2 N 1 2 
And we know from above that: PV = N  mv 2  ; Thus, T =  mv 
3  2  3 nR  2 
3
∴ Kinetic energy per mole of gas = RT ;
2
As R = Boltzmann’s constant x Avogadro number; R = kNA

1 2  3
Thus, Average molecular kinetic energy is:
= KEavg = mv  kT
 2  2
3RT
Therefore, Vrms = ; Where Mm is the molecular weight of the gas.
Mm

PLANCESS CONCEPTS

3RT
In Vrms = , M is the molecular weight in Kg.
M
The temperature at which molecular speed becomes zero is called absolute zero i.e. 0 kelvin
Vaibhav Krishnan (JEE 2009 AIR 22)

Illustration 1: A tank used for filling helium balloons has a volume of 0.3m³ and contains 2.0 mol of helium gas at
20.0°C. Assuming that the helium behaves like an ideal gas.
(a) What is the total translational kinetic energy of the molecules of the gas?
(b) What is the average kinetic energy per molecule? (JEE MAIN)

Sol: Formula for total kinetic energy and kinetic energy per molecule is used.
3
(a) Using (KE)Trans = nRT
2
3
With n = 2.0 mol and T = 293 K, we find that (KE)Trans = (2.0)(8.31)(293) = 7.3 x 10³ J
2
3
(b) The average kinetic energy per molecule is kT.
2
1 3 . 4 | Kinetic Theor y of Gases and Thermodynamics

1 2
or=
2
mv
1 2
=
2
mv rms
3
2
3
2
( )
kT = 1.38x10−23 (293) = 6.07x10−21 J

Illustration 2: In a circle model of a rotation diatomic molecule of chlorine (Cl2), the two Cl atoms are 2.0 x 10-10 m
apart and rotate about their centre of mass with angular speed ω = 2.0 x 1012 rad/s. What is the rotational kinetic
energy of one molecule of Cl2, which has a molar mass of 70.0 g/mol? (JEE ADVANCED)
1 2
Sol: Rotational kinetic energy of each molecule= Iω
2 m  m
Movement of inertia, I = 2 (mr²) = 2mr²
Cl Cl
70x10−3 2.0x10−10
Here m = 5.81x10−26 kg and
= = r = 1.0x10 −10 m
2x6.02x1023 2 r r
Figure 13.2
∴ I = 2(5.81) x10−26 (1.0 x10−10 )2 =1.16 x 10-45 kg-m²

1 2 1
∴ KR= Iω = x(1.16 x10−45 ) x(2.0 x1012 )2 = 2.32x10−21 J
2 2
1 1
Note: At T = 300 K, rotational K.E, should
= be equal to kT = x(1.38 x10−23 ) x(300) 2.07 x10−21 J
2 2

5. TYPES OF SPEED
There are generally three types of speed defined which are physically important:

5.1 Root Mean Square (RMS) Speed vrms

It is the square root of the average of the square of velocities v 2 i.e. v rms = v 2

As has been derived in the previous section, the value of vrms can be derived from the following formula:

3kT 3RT 3P
v rms
= = =
m M ρ

5.2 Mean/Average Speed vav


v1 + v 2 +  + vN
It is the mean of the speeds of all the particles i.e. v av =
N

8kT 8RT 8P
From the Maxwellian speed distribution law, we can show that
= v av = =
πm πm πρ

5.3 Most Probable Speed vmps


It is the mode of the speeds. That is, the speed which is possessed by most of the molecules of a gas at a given
temperature T.

2kT 2RT 2P
v mp
= = =
m M ρ
P hysi cs | 13.5

PLANCESS CONCEPTS

The above quantities are easy to derive by some basic knowledge of calculus. However, only knowing
them is also fine and observe that
1. Vmps: Vrms: Vav:: 1: 1.224: 1.28
2. Vmps =<Vrms =< Vavg
Nivvedan (JEE 2009 AIR 113)

Illustration 3: Consider an 1100 particles gas system with speeds distribution as follows:
1000 particles each with speed 100 m/s; 2000 particles each with speed 200 m/s
4000 particles each with speed 300 m/s; 3000 particles each with speed 400 m/s and 1000 particles each with
speed 500 m/s Find the average speed, and rms speed. (JEE MAIN)

 n1v1 + n2 v 2 + n3 v 3 ......  nv12 + n2 v 22 + n3 v 32 .........


Sol:  Vavg =  and Vrms
 n1 + n2 + n3 + .......  n1 + n2 + n3 + ...........

(1000)(100) + (2000)(200) + (4000)(300) + (3000)(400) + (1000)(500)


The average speed is: v av = = 309 m/s
1100

(1000)(100)2 + (2000)(200)2 + (4000)(300)2 + (3000)(400)2 + (1000)(500)2


The rms speed is: v rms = = 328 m/s
1100

v rms 3
Note: Here ≠ as values and gas molecules are arbitrarily taken.
v av 8/π

Illustration 4: Find the rms speed of hydrogen molecules at room temperature (=300 K). (JEE MAIN)
Sol: Formula based

3RT 3x8.31x300
Mass of 1 mole of hydrogen gas = 2g = 2 x 10-3kg ⇒ v rms = = =1.93x10³m/s
M 2x10 −3

Illustration 5: Find the temperature at which oxygen molecules would have the same rms speed as of hydrogen
molecules at 300 K. (JEE ADVANCED)
Sol: Compare the Vrms of oxygen and hydrogen.

3RT 3R(300)  Mo 
If T be the corresponding temperature = = ⇒ T (300)
=   4800K
Mo MH  MH 
1 3 . 6 | Kinetic Theor y of Gases and Thermodynamics

THERMODYNAMICS
1. INTRODUCTION
You would have certainly noticed the whistle of the process cooker in the kitchen. Have you ever thought of reason
for that? How small amount of water form large volume of vapours. Think of James Watt and his observation that
red to invention of steam engine !

2. GAS LAWS

2.1 Boyle’s Law


The Pressure-Volume Law (by Robert Boyle (1627-1691).
Boyle’s law or the pressure-volume law states that the volume of a given amount of gas varies inversely with the
applied pressure when the temperature and mass are kept constant.
1
V∝ ; i.e. PV = C; Therefore, P1V1 = P2 V2
P
When pressure goes up, volume goes down and vice-versa.

2.2 Charles’ Law


The Temperature-Volume Law (by Jacques Charles (1746 - 1823).
This law states that the volume of a given amount of gas held at constant pressure is directly proportional to the
Kelvin temperature.
v ∝ T ; i.e. V / T = C; Therefore, V1 / T1 = V2 / T2
As the volume goes up, the temperature also goes up, and vice-versa.

2.3 Gay-Lussac’s Law


The Pressure Temperature Law (by Joseph Gay-Lussac (1778-1850).
This law states that the pressure of a given amount of gas held at constant volume is directly proportional to the
Kelvin temperature.
P ∝ T ; i.e. P/T = C; Therefore, P1 / T1 = P2 / T2

2.4 Avogadro’s Law


The Volume Amount Law ( Amedeo Avogadro (1776-1856).
Gives the relationship between volume and amount when pressure and temperature are held constant. Remember
amount is measured in moles. Also, since volume is one of the variables, that means the container holding the gas
is flexible in some way and can expand or contract. If the amount of gas in a container is increased, the volume
increases. If the amount of gas in a container is decreased, the volume decreases.
v ∝ n ; i.e. V/n = C; Therefore, V1 / n1 = V2 / n2

The combined gas law: The volume of a given amount of gas is proportional to the ratio of its Kelvin temperature
and its pressure.
T
V∝ ; i.e. PV/T = C Therefore, P1 / V=
1 T=
1 P2 V2 / T2
P
P hysi cs | 13.7

As the pressure goes up, the temperature also goes up, and vice-versa.

The ideal gas law: On combining all the previous laws we get: PV = nRT
Where n: the number of moles and R: constant called the universal gas constant≈ 0.0821 L-atm / mol-K.

PLANCESS CONCEPTS

For solving problems, it is not advisable to individually think over each and every case. Just remember
this overall equation. PV = nRT
Nitin Chandrol (JEE 2012 AIR 134)

Illustration 6: An air bubble starts rising from the bottom of a lake. Its diameter is 3.6 mm at the bottom and 4 mm
at the surface. The depth of the lake is 250 cm and the temperature at the surface is 40°C. What is the temperature
at the bottom of the lake? Given atmospheric pressure = 76 cm of Hg and g = 980 cm/s². (JEE MAIN)
P1V1 P2 V2
Sol: Amount of air is constant, hence = applicable.
T1 T2
4 3 4
At the bottom of the lake, volume of the bubble V1 = πr = π(0.18)3 cm3
3 1 3
Pressure on the bubble; P1 = atmospheric pressure + pressure due to a column on 250 cm of water = 76 x 13.6 x
980 + 250 x 1 x 980 = (76 x 13.6 +250) 980 dyne/cm²
4 3 4
At the surface of the lake, volume of the bubble V2 = πr = π(0.2)3 cm3
3 2 3
Pressure on the bubble; P2 = atmospheric pressure = (76 x 13.6 x 980) dyne/cm²
T2 = 273 + 40°C = 313° K
4 4
(76 x13.6 + 250)980 x   π(0.18)3 (76 x13.6) x 980   π(0.2)3
PV P V
Now 1 1 = 2 2 or 3 = 3
T1 T2 T1 313

or T1 = 283.37 K; ∴ T1 = 283.37 – 273 = 10.37°C

Illustration 7: P-V diagram of n moles of an ideal gas is as shown in figure. Find the maximum temperature
between A and B. (JEE ADVANCED)

dT d2 T
Sol: Find
= 0, < 0 for Tmax in T/V relation.
dV dV 2
Procedure: For given number of moles of a gas, T ∝ PV

Although (PV)A = (PV)B or TA = TB, still it is not an isothermal process. Because in isothermal process P-V graph is a
rectangular hyperbola while it is a straight line. So, to see the behaviour of temperature first we will find either T-V
equation or T-P equation and from that equation we can judge how the temperature varies. From the graph first
we will write P-V equation, then we will convert it either in T-V equation or in T-P equation.
From the graph the P-V equation can be written as,
P  P 
−  0  V + 3P0 (y = -mx + c) or
P= −  0  V 2 + 3P0 V
PV =
V  V 
 0  0
1 3 . 8 | Kinetic Theor y of Gases and Thermodynamics

P
P  2 1  P  2
= 3P0 V −  0
or nRT  V (as PV = nRT) or
= T 3P0 V −  0  V 
V nR   V0
A
 0    2P0
This is the required T-V equation. This is quadratic in V. Hence T-V graph is a parabola.
P0 B
Now, to find maximum or minimum value of T we can substitute.
V
dT  2P  3 V0 2V0
= 0 or 3P0 −  0 0 ; or V = V0
 V =
dV V 2 Figure 13.3
 0 
d2 T 3
Further is negative at V = V
dV 2 2 0
3
Hence, T is maximum at V = V and this maximum value is,
2 0
  3V   P   3V  
2
9P V
1 
=Tmax (3P0 )  0  −  0   0   ; or Tmax = 0 0
nR  2 V  2  4nR
    0    Tmax

Thus, T-V graph is as shown in figure A B


TA-TB
2P0 V0 9P0 V0 P0 V0
T=
A T=
B
and Tmax = = 2.25 V
nR 4nR nR V0 2V0
Figure 13.4

3. DEGREE OF FREEDOM (f )
Each independent mode by which a system can absorb energy (Kinetic + Potential) is called a degree of freedom.
It is the number of independent motions possible or number of independent coordinates to specify the dynamic
position of a body. A molecule can have translational K.E, rotational K.E and vibrational energy (potential, kinetic).
(a) Monatomic Gas: It has 3 degrees of freedom, since it can have translational motion in any direction in space.
It can rotate but due to its small moment of inertia, rotational energy is neglected. f = 3
(b) Diatomic gas and linear polyatomic gas: It has 5 degrees of freedom (3 translational + 2 Rotational). Since,
the moment of inertia about the axis joining the atoms is negligible, thus, it only has two rotational degrees
of freedom. In all, f = 5.
At high temperature, its degree of freedom increases by 2 due to the vibrational energy (1 kinetic + 1 potential
vibrational energy). Thus f = 7 at high temperature.
(c) Non-linear Polyatomic gas: It has 6 degrees of freedom (3 translational + 3 rotational). The one rotational
degree of freedom which was neglected above, can’t be neglected over here. Since, the moment of inertia
about any of the three axes is not tending to zero i.e. f = 6.
At high temperature, its degree of freedom increases by 2 due to Vibrational energy
(1 kinetic + 1 potential) Thus, f = 8.
(d) Solid: Solids don’t have any translational or rotational degrees of freedom. But at high temperature, it can
vibrate along 3 axes. Thus it has 2 degrees of freedom per axis (i.e. potential + kinetic vibrational energy).
Thus, at high temperature, solids have 6 degrees of freedom. i.e. f = 6
P hysi cs | 13.9

PLANCESS CONCEPTS

fmix = (n1f1 + n2f2+……………) / (n1+n2+……………) is the equation for effective ‘f’ for a mixture of gases.
•• Degrees of freedom of a diatomic and polyatomic gas depend on temperature and there is no
clear cut demarcation line above which vibrational energy becomes significant. Moreover, this
temperature varies from gas to gas. On the other hand for a monatomic gas, there is no such
confusion. Degree of freedom here is 3 at all temperatures. Unless and until stated in the question
you can take f = 3 for a monatomic gas, f = 5 for a diatomic gas and f = 6 for a non-linear
polyatomic gas.
•• When a diatomic or polyatomic gas dissociates into atoms it behaves as a monatomic gas, whose
degrees of freedom are changed accordingly.
B Rajiv Reddy (JEE 2012 AIR 11)

4. INTERNAL ENERGY (U)


(a) In thermodynamics, the internal energy is the total energy contained by a thermodynamic system. It is the
energy needed to create the system but excludes any energy due to external force fields (e.g. Internal energy
does not include the energy due to the motion of the system as a whole. It further excludes any kinetic
or potential energy the body may have because of its location in external gravitational, electrostatic, or
electromagnetic fields.).
(b) Internal energy has two major components, kinetic energy (Ukin) and potential energy(Upot). The kinetic energy
is due to the motion of the system’s particles (translations, rotations, vibrations), and the potential energy is
associated with the static rest mass energy of the constituents of matter. U = Upot + Ukin
(c) The internal energy of a system can be changed by heating the system or by doing work on it.
(d) It is an extensive quantity. The internal energy is a state function of a system, because its value depends only
on the current state of the system and not on the path taken or process undergone to arrive at this state
(e) The SI unit of energy is the joule (J).

5. LAW OF EQUIPARTITION OF ENERGY


According to the law of equipartition of energy, the total energy of a molecule is equally distributed on the average
1
among all the degrees of freedom. For an ideal gas at absolute temperature T, the energy is: kT per molecule per
2
1
degree of freedom. k = Boltzmann’s constant ; RT per mole, R = gas constant.
2
f
Thus, if f be the number of degrees of freedom, the internal energy of n moles of the gas will be nRT . Thus,
2
f
U = nRT
2

6. MOLAR HEAT CAPACITIES (C)


(a) The amount of heat needed to raise the temperature of one mole of a substance by one degree Celsius. i.e.
∆Q
C= ⇒ ∆Q = nC∆T
n∆T
(i) S.I. unit is J/mol K
(ii) The value of C depends on the process through which its temperature is raised.
1 3 . 1 0 | Kinetic Theor y of Gases and Thermodynamics

(b) Molar heat Capacity at constant Pressure CP : The amount of heat needed to raise the temperature of one
mole of a substance by one degree Celsius at constant pressure.
(c) Molar heat Capacity at constant Volume CV : The amount of heat needed to raise the temperature of one
mole of a substance by one degree Celsius at constant volume.
CP
=(i)   γ Ratio
= of CP to C v ; γ is also referred to as the adiabatic constant.
Cv

PLANCESS CONCEPTS

You Can Write:


CVmix
= (n1Cv1 + n2Cv 2 + ) / (n1 + n2 + )
CPmix
= (n1CP1 + n2CP2 + ) / (n1 + n2 + )
fmix = (n1f1 + n2 f2 + ) / (n1 + n2 + )
But γmix ≠ (n1γ1 + n2 γ 2 + ) / (n1 + n2 + )
1 2 / fmix (you will see it later that γ = 1 + 2/f) = CPmix / C vmix
γmix =+
Anand K (JEE 2011 AIR 47)

7. ZEROTH LAW OF THERMODYNAMICS


If two bodies A and B are in thermal equilibrium and A and C are also in thermal equilibrium, then B and C are also
in thermal equilibrium.

8. FIRST LAW OF THERMODYNAMICS


It is based on the law of conservation of energy. It states that
Heat supplied to the gas = Increment in internal energy + work done by the gas.
∆Q = ∆U + ∆W
In differential form, dQ = dU + dW
We will be using the following sign convention:
(a) For heat transfer: ∆Q is +ve for heat supplied;
∆Q is -ve for heat rejected
(b) For Work Done: ∆W=+ve for work done by gas (in expansion of gas)
∆W=-ve for work done on the gas (in contraction of gas)
We know:
n
Internal energy: Earlier we have seen that U= fRT
2
n
Thus, change in internal energy ∆U= fR∆T
2
n
or in differential from, dU = fRdT
2
Note: Change in Internal Energy is path independent whereas work done or heat energy released are path
dependent. Internal Energy depends only on initial and final state of the system.
P hysi cs | 13.11

Illustration 8: Calculate the change in internal energy of 3.0 mol of helium gas when its temperature is increased
by 2.0 K. (JEE MAIN)

Sol: Formulas based : f degree of freedom for monoatomic gas is 3.


Helium is a monatomic gas. Internal energy of n moles of the gas is,
3 3
U = nRT ; U
∴ ∆= nR( ∆ T)
2 2
3
Substituting the
= values, ∆U  =  (3)(8.31)(2.0) 74.8 J
2
Heat transfer: From molar heat capacity, we saw that ∆Q = nC∆T or in differential form, dQ = nCdT
Work done: dW = PdV
For a finite change in volume from Vi to Vf, this equation is then integrated between Vi to Vf to find the net work.
v
W=
∫ dw =
∫ v f PdV
i
Now, there are two methods of finding work done by a gas.
P
Method 1: This is used when the P-V equation is known to us. Suppose P as a function of V
1
v
is known to us. P = f(V) then work done can be found by, W = ∫ v f f(V)dV
i
A B
Method 2: Since, work done is the integration of pressure w.r.t volume. So, it is clear that
work represents the area under P-V graph. But, always take care of the sign of the work done.
2
If the volume increases or in case of cyclic process if it is clockwise then +ve work is done by
the gas or vice versa.
Figure 13.5

Illustration 9: A certain amount of an ideal gas passes from state A to B first by means of process 1, then by means
of process 2. In which of the process is the amount of heat absorbed by the gas greater?  (JEE MAIN)

Sol: ∆V is state function, remains constant. Hence work done directly P P


proportional to heat obserbed.
1
Q=
1 W1 + ∆U1 and Q=
2 W2 + ∆U2
A B A B
U is a state function. Hence, ∆U depends only on the initial and final W1
W2
positions. Therefore, ∆U1 = ∆U2 . But W1 > W2 as the area under 1 is
v v
greater than area under 2. Hence, Q1 > Q 2
Figure 13.6

9. REVERSIBLE AND IRREVERSIBLE PROCESS

9.1 Reversible Process


A process in which the system and surroundings can be restored to the initial state from the final state without
producing any change in the thermodynamic properties of the universe.
E.g. an infinitesimal compression of a gas in a cylinder assuming frictionless surfaces.

9.2 Irreversible Process


In irreversible processes, the system is not in equilibrium throughout the process. The initial state can’t be obtained
from the final state without producing changes in the universe.
E.g. heat transfer through a finite temperature difference.
1 3 . 1 2 | Kinetic Theor y of Gases and Thermodynamics

10. DIFFERENT THERMODYNAMIC PROCESSES

10.1 Isochoric Process (V = Constant)


CV is the molar heat capacity for constant volume.
Since ∆V =0 . Therefore, ∆W =
0;
P
f f
= n R∆T and
We know that, nC v ∆T = n R∆T
2 2
By 1st Law of Thermodynamics: ∆Q = ∆U + ∆W ; ∆Q =∆U + 0 ;
V=constant
f f
= n R∆T ; Therefore, ∆Q =
Thus nC v ∆T ∆U ⇒ C v = R;
2 2
So in isochoric process we have ∆Q =∆U =nC v ∆T ; ∆W =
0 T(in K)
Note: Since, ∆U is an extrinsic property(property that is not inherent). So, Figure 13.7
∆U= nC v ∆T can be used for any process.

10.2 Isobaric Process (P = Constant)


Here ∆P =0 ; CP is the molar heat capacity for this process.
Thus, dQ = mCP dT V
P=constant
Since, dU is an extrinsic property, Thus, dU = nC v dT
We know PV = nRT ; ⇒ PdV + VdP = nRdT ⇒ PdV = nRdT [∵ dP = 0]
And dW = PdV = nRdT
So, by using the 1st law of thermodynamics.
T(in K)
nRdT; ⇒ CP = CV + R
We get , dQ= dU + dW; ⇒ nCP dT = nCV dT + nRdt
Thus, for the process, we have, ∆W= P(Vf − Vi )= nR(Tf − Ti ) ; ∆Q = nCP ∆T ; ∆U= nC v ∆T Figure 13.8

Note: We have seen, from above two process that,


f
CV = R and C=
P CV + R  … (i)
2
f 
Thus, C=P  + 1 R  …by (i)
 2 
f 
 + 1 R
CP 2  ; ⇒ γ = 1 + 2 Thus, C= R / ( γ − 1) And C = γR / ( γ − 1)
We know, γ = ; ⇒ γ = V P
CV f f
 R
2
Thus, for different degrees of freedom (i.e. for different gases) we have:

Types of Gas Degrees of freedom Total Energy CP CV γ

3 5 3
Monatomic 3 RT R R 1.67
2 2 2

Diatomic 5
5 7 5 1.4
RT R R
2 2 2
Polyatomic( ≥ 3 ) 6 3RT 4R 3R 1.33
P hysi cs | 13.13

Illustration 10: A cylindrical vessel of 28cm diameter contains 20g of nitrogen compressed by a piston supporting
a weight of 75kg. The temperature of the gas is 17°C. What work will the gas do, if it is heated to a temperature of
250°C? What amount of heat should be supplied? To what distance will the weight be raised? The process should
be assumed to be isobaric; the heating of the vessel and the external pressure are negligible. (JEE ADVANCED)

Sol: Use isobaric process equations .


m R∆T m 20
∆W (work done) = ∫ PdV = P∆V = P × = R∆T ∴ ∆W
= × 8.3(250 − 17)
= 1381J
M P M 28
7 20
Q n Cp ∆=
∆= T nR ∆T = 3.5 × × 8.3 × (250 − 17) = 4834.7J
2 28

∆V nR∆T 1
h
= = ×
A P A

 mg 
P  P0 +
= 
 A 

20 × 8.3 × (250 − 17)


⇒ h= ⇒ h 0.2
= = m 20 cm
5 22 2 −4
28 × 75 × 9.8 + (10 × × 14 × 10 )
7

10.3 Isothermal Process (T = Constant)

∆T =0 ; ∆U= nCV ∆T= 0 ; ⇒ ∆W =∆Q ;


nRT P
We know, PV = nRT; ⇒ P = ;
V
Vf
nRT Vf Vf
∴∆W
= ∫= V
dv nRTlnV
=  nRTln
Vi
T=constant
V Vi
i

Slope of P-V graph: PV=nRT; ⇒ (dP)V+P(dV)= nRdT


V
Since, dT =0;
Figure 13.9
Thus, (dP)V+P(dV)=0 ; ⇒ dP/dV=- P/V
So, in an isothermal process we have: ∆U =0
Vf Pi P
∆Q =∆W =nRTln =nRTln ; Slope (isothermal) = −
Vi Pf V
And C (molar heat capacity) tends to infinity since even when temperature change is zero, heat transfer is non-zero.

Illustration 11: Three moles of an ideal gas being initially at a temperature T0 = 273K were isothermally expanded
η =5.0 time its initial volume and then isochorically heated so that the pressure in the final state became equal to
that in the initial state. The total amount of heat transferred to the gas during the process equals Q = 80 kJ. Find
the ratio γ =CP / CV for this gas. (JEE ADVANCED)

Sol: Try draw the process on P/V graph, during isothermal process, the change in internal energy is zero while in
R
isochoric process work dne is zero. Take C v = and proceed to find y.
y −1
Procedure:
(1) Find heat expression (Q1) given in isothermal process
(2) Find heat expression (Q2) given in isochoric process
1 3 . 1 4 | Kinetic Theor y of Gases and Thermodynamics

(3) Find total heat expression (Q=Q1+Q2)


(4) Put the value of Q given in question in the expression obtained above
In Isothermal process, the heat transferred to the gas is given by

2 / V1 ) nRT0 ln η 
Q1 nRT0 ln(V=
= … (i)
=∴ η (V
=2 / V1 ) (P1 / P2 )
In isochoric process, Q2 =∆ U (W=0)
∴ Q=
2 T n{R / ( γ − 1)}∆T 
nC v ∆= … (ii)

P2 T0 P 
Now = or T = T0  1  = ηT0  … (iii)
P1 T  P2 

∴ ∆ T = η T0 − T0 = (η − l)T0
Substituting the value of ∆T from equation (iii) in equation (ii), we get

 R   R 
=Q2 n  (η − 1) T0 ; ∴
= Q nRT0 ln η + n   (n − 1) T0
 γ − 1  γ − 1

Q  η − 1 η −1 η −1
or − ln η =  ; or γ − 1 = ∴ γ=1 +
nRT0  γ −1 Q Q
− ln η − ln η
nRT0 nRT0

(5 − 1)
Substituting given values, we get γ = 1 + ; Solving, we get γ =1.4
80x102
− ln5
3x8.3x273

10.4 Adiabatic Process (∆Q = 0)


For adiabatic process: ∆Q =
0 ; ∆U = −∆W
1monotomic
nCV dT = −Pdv  … (i) P 2diatomic
By equation of gas: PV = nRT ; ⇒ Pdv + Vdp =
nRdT ; 3polytomic
Pdv + Vdp
∴ ndT = 3 g=1.33
R 2
1 g=1.4
 Pdv + Vdp 
∴ Substituting in (i) we get: C v   = −Pdv g=1.67
 R 
V
C d −dp
⇒ (C v + R)Pdv = ; ⇒ P V
− C v Vdp = CV + R CP  ;
= Adiabatic expansion of mono, dia
CV V P
and polyatomic gases
dV dP
⇒γ =− Figure 13.10
V P
On integrating both sides we get,

dv P dp V P
⇒ γ∫ ∫ = −Pf ∫ ; ⇒ γ ln f =ln i
V i P Vi Pf

γ
Pi  Vf 
⇒ =   ; (say)
Pf  Vi 

∴ PV γ =
Const
P hysi cs | 13.15

The relation found above can be written in many different ways using ideal gas equation (PV=nRT)
TV γ−1 = constant ; T γP1−γ = constant
Vf Vf
k k
; ∴= ∫=
Pdv ∫ dv
γ
PV = Const = K ⇒ P = ∆W
V γ
Vi Vγ Vi

KVf1−γ − KVi1−γ (Pf Vf γ ) Vf1−γ − (PV γ


i i ) Vi
1−γ
= =
1− γ 1− γ

Pf Vf − PV
i i nR
∆W
⇒= = (T − T )
1− γ 1− γ f i

Slope of P-V graph:


We know, PV γ = constant
P
dp d  cons tant  P P
=   = −γ   ; Thus, (Slope)adiabatic = −γ  
dV dV  V γ
 V
  V Isothermal
Because γ >1, the isothermal curve is not as steep as that for the
adiabatic expansion.
Adiabatic
So, in adiabatic process we have: ∆Q =
0
V
Pf Vf − PV nR Adiabatic and isothermal expansion
=W = i i
(T − T ) ; ∆U = −∆W
1− γ 1− γ f i of an idea gas

P Figure 13.11


(Slope)adiabatic = −γ  
V
And C (molar heat capacity) is zero, since even when temperature change is not zero, heat transfer is zero.

Illustration 12: An ideal monatomic gas at 300K expands adiabatically to twice its volume. What is the final
temperature? (JEE MAIN)

Sol: Adiabatic process PV γ = constant or TV γ−1 = constant.


5
For an ideal monatomic gas, γ =
3
In an adiabatic process, TV γ−1 = cons tant ; ∴ Tf Vf γ−1 =
Ti Viγ−1
γ−1 5
−1
V   1 3
or = Ti  i 
Tf = (300)
=   189K
 Vf  2

Illustration 13: An ideal gas having initial pressure P, volume V and temperature T is allowed to expand adiabatically
until its volume becomes 5.66 V while its temperature falls to T/2.
(a) How many degrees of freedom do the gas molecules have?
(b) Obtain the work done by the gas during the expansion as a function of the initial pressure P and volume V.
 (JEE ADVANCED)
γ−1
T1 V 
Sol: Apply T1V1γ−1 = T2 V2 γ−1 and hence =  2 
T2  V1 

nR  T  nRT nRT PV
=ω  = − T = =
1− γ  2  2( γ − 1) 2 × 0.4 0.8
1 3 . 1 6 | Kinetic Theor y of Gases and Thermodynamics

(a) For adiabatic change,


γ−1 1
TV = T'(V')γ−
= ; T' T=
/ 2, V' 5.66V
γ−1 γ−1 γ−1
TV = (T/ 2) x(5.66 V) ; (5.66) = 2
It shows that the gas is diatomic for which the gas molecules have five degrees of freedom.
(b) Work done by the gas during adiabatic expansion, W, is given by,

PV − P'V' P'V' PV PVT'



= W ; As
= = , P'
γ −1 T T V'T
1 1 P 1  P  PV
P'
= x xP
= ;∴ W=  PV − x5.66V  = = 1.25PV.
2 5.66 11.32 0.4  11.32  0.8

10.5 Cyclic Process


In a cyclic process, the thermodynamic process ends at the initial state. A
Since, initial and final states are same, there, P
B
∴ ∆Q = ∆W ; ∴ ∆Q = ∆W = Area enclosed by the curve.

Sign of ∆W is ‘+’ if process is clockwise and ‘-‘ if process is anti-clockwise. V


Figure 13.12
10.6 Polytropic Process
For a process: PVx = constant where x ≠ 1, also called a polytropic process.

R R R
∆W = (Pf Vf − PV
i i ) / 1 − x = nR ∆ T/ 1 − x ; C =Cv + = +
1− x γ − 1 1− x
dP P
Slope of P-V diagram (also known as Indicator diagram) at any point is = −x
dV V

11. FREE EXPANSION


In a free expansion, the gas is allowed to expand into a vacuum. This happens quickly, so there is no heat transferred.
No work is done, because the gas does not displace anything. According to the First Law, this means that: ∆U = 0
There is no change in internal energy, so the temperature stays the same.
On a PV diagram all you can do is plot the end-points. The process follows a path on the diagram that is not well-
defined because the temperature is constant. The connection between the initial and final states is: PiVi = PfVf
Thus, ∆W=0; ∆Q=0; ∆U=0; T=const. ; thus, PV = constant

Illustration 14: An insulated container is divided into two equal portions. One portion contains an ideal gas at
pressure P and temperature T, while the other portion is a perfect vacuum. If a hole is opened between the two
portions, find the change in internal energy and temperature of the gas?  (JEE ADVANCED)

Sol: No opposing force, hence work done is zero. As the system is thermally insulated, ∆Q =
0
Further as here the gas is expanding against vacuum (surroundings) the process is called free expansion and for it,
∆W= ∫ Pdv = 0 [as for vacuum P = 0]
So in accordance with first law of thermodynamics, i.e. ∆Q = ∆U + ∆W , we have 0 =∆U + 0,
i.e. ∆U =0 or U = constant
P hysi cs | 13.17

So in this problem internal energy of the gas remains constant, i.e. ∆U = 0 . Now as for an ideal gas=
U 3 / 2 µRT ,
i.e. U ∝ T ; so temperature of the gas will also remain constant, i.e. ∆T =0

PLANCESS CONCEPTS
(a) A non-conducting container with no external source of heat energy will always undergo any change
adiabatically.
(b) A conducting (finitely conducting) container with no external source of heat energy will undergo
change:
(i) Isothermally if the process is slow
(ii) Adiabatically if the process is fast
(c) An infinitely conducting container with no external source of heat energy will always undergo any
change isothermally.
Yashwanth Sandupatla (JEE 2012 AIR 821)

1
Illustration 15: A gas undergoes a process such that P ∝ If the molar heat capacity for this process is
T
C= 33.24 J/mol-K, find the degree of freedom of the molecules of the gas.  (JEE ADVANCED)
Q
Sol: Heat capacity = amount of heat required for per unit change of temperature. C =
∆T
1
As P ∝ or PT = constant .… (i)
T
We have for one mole an ideal gas PV=RT .… (ii)
From Eqs. (i) and (iii)
P²V = constant or PV1/2 = K (say)  … (iii)
From first law of thermodynamics, ∆Q =∆U + W or C∆T = CV ∆T + W
W
or =
C CV +  … (iv)
∆T
Vf
1/2 Pf Vf − PV R(Tf − Ti ) RT W
Here, ∆W
= ∫ PdV
= K ∫ V −= dv ; = i i
= ;∴ 2R
=
Vf
1 − (1/ 2) 1/ 2 1/ 2 ∆T

R
Substituting in Eq. (iv), we have C = C v + 2R = + 2R
γ −1

 1   1 
Substituting the values, 33.24
= R 2  8.31
+= + 2
 γ − 1   γ − 1 
2 2 2
Solving this we get γ =1.5 ; Now, γ = 1 + or degree of freedom =F = = 4
F γ − 1 1.5 − 1

R R
Alternate Solution: In process PVx =constant, molar heat capacity is given by=C +
γ − 1 1− x

1 R R R
The given process is PV1/2 = constant or x = ; ∴ C = + = + 2R
2 γ −1 1 γ −1
1−
2
Now, we may proceed in the similar manner.
1 3 . 1 8 | Kinetic Theor y of Gases and Thermodynamics

Illustration 16: An ideal gas expands isothermally along AB and does 700 J of work. P
A
(a) How much heat does the gas exchange along AB?
B
(b) The gas then expands adiabatically along BC and does 400 J of work. When the gas
returns to A along CA, it exhausts 100J of heat to its surroundings. How much work is done C
on the gas along this path? (JEE MAIN)
V
Sol: Apply Q =∆U + W for the forces AB, BC & then CA separately. Remember ∆U is a state Figure 13.13
function.
(a) AB is an isothermal process. Hence, 0 and
∆UAB = Q
= AB W
=AB 700J
(b) BC is an adiabatic process. Hence, QBC = 0 ; WBC = 400J (given)
∴ ∆UBC = − WBC =
−400J
ABC is a cyclic process and internal energy is a state function. Therefore, ( ∆U)whole cycle = 0 = ∆UAB + ∆UBC + ∆UCA
and from first law of thermodynamics, Q AB + QBC + Q CA = WAB + WBC + WCA
Substituting the values, 700 + 0 − 100
= 700 + 400 + ∆WCA
∴ ∆WCA =
−500J
Negative sign implies that work is done on the gas.

12. HEAT ENGINE & EFFICIENCY


Heat Engine is a device which converts the input W
heat energy into mechanical energy by using a
cyclic process.
T1 T2
It absorbs a quantity of heat Q1 from a source Hot Q1 Working Cold
and performs an amount of work W and returns (Source) substance (Source)
to the initial state after rejecting some heat Q2
to a sink. The working substance which is gas
or liquid undergoes a cyclic thermodynamic Figure 13.14
process. The source is at a higher temperature
than the sink.
The efficiency η , of a heat engine is given by

 Work done by the working subs tance 


 
=(an ideal gas in our case) during a cycle  W
η
 Heat supplied to the gas during the cycle  Q
1
 
 
From conservation of energy. Q=
1 W + Q2

W Q1 − Q 2 Q Heat released
η= = = 1 − 2 ; Thus, ∴ Efficiency(η) = 1 −
Q1 Q1 Q1 Heat absorbed

Illustration 17: The P-V diagram of 0.2 mol of a diatomic ideal gas is shown in figure. Process BC is adiabatic. The
value of γ for this gas is 1.4. (JEE ADVANCED)
(a) Find the pressure and volume at points A, B and C.
(b) Calculate ∆Q , ∆W and ∆U for each of the three processes.
P hysi cs | 13.19

(c) Find the thermal efficiency of the cycle. P


Take 1 atm = 1.0x105 N/m². B
600 K

Sol: Try to start from A where PA and TA are known.


(a) PA = PC = 1 atm = 1.01 x 105 N/m²
A
1.0 atm C
Process AB is an isochoric process. 300 K
V
PB TB
∴ P ∝ T or = ;
PA TA Figure 13.15

 TB   600 
∴ PB =
=  PA   (1atm) 2atm = 2.02 x 10 N/m²
5
T
 A  300 
nRT
From ideal gas equation V =
P
nRTA (0.2)(8.31)(300)
∴ V=
=
A V=
B ≈ 5.0x10−3 m3 ; = 5 litre
PA (1.01x105 )

nRTC (0.2)(8.31)(455)
VC
= = = 7.6x10−3 m3 ≈ 7.6 litre
PC 5
(1.01x10 )

A 1 atm 5 lt
B 2 atm 5 lt
C 1 atm 7.6 lt

(b) Process AB is an isochoric process. Hence, ∆WAB =


0
5  5
∆Q AB =∆UAB =nC v ∆T =n  2=
R  (TB − TA ) (0.2)   (8.31)(600 − 300) ≈ 1246 J
  2
Process BC is an adiabatic process. Hence, ∆QBC =
0
∴ ∆WBC = −∆UBC
5  5
∆UBC= nC v ∆T==
nCV (TC − TB ) (0.2)  R=
 (455 − 600) (0.2)   (8.31)( −145) J ≈ −602J
2  2
∴ ∆WBC =
−UBC =
602J

Process CA is an isobaric process. Hence,


7  7
∆Q CA= nCP ∆T==
n  R  (TA − TC ) (0.2)   (8.31)(300 − 455)
2  2

12.1 Types of Heat Engine


(a) External combustion engine: In which heat is produced by burning the fuel in a chamber outside the
main body (working substance) of the engine. Steam engine is an external combustion engine. The thermal
efficiency of a steam engine varies from 10 to 20%.
(b) Internal combustion engine: In which heat is produced by burning the fuel inside the main body of the
engine. Petrol engine and diesel engines are internal combustion engine. Here, we will just discuss about only
one heat engine i.e. Carnot engine.
1 3 . 2 0 | Kinetic Theor y of Gases and Thermodynamics

12.2 Carnot Engine


(T1)
Carnot Cycle is the most efficient heat engine which undergoes a cycle of A Q1


two isothermal and two adiabatic processes. It absorbs heat Q1 by expanding
isothermally at T1 along AB and then expands adiabatically along BC to Isothermal (T )
1
temperature T2. It is compressed isothermally at T2 rejecting heat Q2 along B

adiabatic
Pressure
CD and is compressed adiabatically along DA. Its efficiency η is given by:
T2 adiabatic
η = 1−
T1
(T2) D
C (T2)
Isothermal
Q2


Volume
Derivation of efficiency in Carnot engine: Figure 13.16

qin

I isothermal
P IV adiabatic T1=T2=T3 (High temperature)

II adiabatic

III isothermal
T3=T4=T1 (Low temperature)
qout

V
Figure 13.17: P-V diagram of the carnot cycle

PLANCESS CONCEPTS

Try to prove that carnot engine is the most efficient one among all engines working in the same
temperature range. You will come across a nice use of the second law of thermodynamics!
GV Abhinav (JEE 2012 AIR 329)

Work, heat, ∆ U. and, ∆ H(ncp∆T) in the P-V diagram of the Carnot Cycle.

Process w q ∆U ∆H

V  V 
I −nRTh ln  2  nRTh ln  2  0 0
 V1   V1 

II −nC v (Tl − Th ) 0 nC v (Tl − Th ) nCp (Tl − Th )

V  V 
III −nRTl ln  4  nRTl ln  4  0 0
V V
 3   3 
P hysi cs | 13.21

Process w q ∆U ∆H

IV −nC v (Th − Tl ) 0 nC v (Th − Tl ) nCp (Th − Tl )

V  V  V  V 
Cycle −nRTh ln  2  − nRTl ln  4  nRTh ln  2  + nRTl ln  4  0 0
V V
 V1   3   V1   3 

The efficiency of the Carnot engine is defined as the ratio of the energy output to the energy input.

V  V 
nRTh ln  2  + nRTl  4 
V  V 
Net work done by heat engine − Wsys  1   3 
Efficiency = = =
Heat absorbed by heat engine qh  V2 
nRTh ln  
 V1 
C /R C /R
 T2  V V3 T  V V4
Since processes II (2-3) and IV (4-1) are adiabatic.
=   = ,and  1 
 T3  V2  T4  V1

V3 V2
and since T1 = T2 and T3 = T4. =
V4 V1

V   V2 
nRThln  2  − nRTln
l 
 V 
 V1   1; Th − Tl
Therefore Efficiency = Efficiency= = 1 − Tl / Th
 V2  Th
nRThln  
 V1 

PLANCESS CONCEPTS

•• By the second law of thermodynamics we can say that, it is impossible to construct an engine,
operating in a cycle, which will extract heat from a source and convert the whole of it into work. It
implies that it is impossible to make heat flow from a colder body to a hotter body without the help
of an external agency or an engine from outside.
•• That efficiency of Carnot engine is maximum (not 100%) for given temperatures T1 and T2. But still
Carnot is not a practical engine because many ideal situations have been assumed while designing
this engine which can practically not be obtained.
Anurag Saraf (JEE 2011 AIR 226)

Illustration 18: Carnot’s engine takes in a thousand kilo calories of heat from a reservoir at 827°C and exhausts it
to a sink at 27°C. How much work does it perform? What is the efficiency of the engine? (JEE MAIN)

Q1 − Q 2 Q2 T2 Q2 T2
Sol: η = = 1− and for cannot cycle η = 1 − , hence =
Q1 Q1 T1 Q1 T1

Given, Q1 = 106 Cal T1 = (827 + 273) = 1100K and T2 =(27 + 273) =300K

Q2 T2 T2  300  6
as, = Q2
∴ = = Q   (10 ) = 2.72 x 10 cal
5
Q1 T1 T1 1  1100 
1 3 . 2 2 | Kinetic Theor y of Gases and Thermodynamics

 T   300 
Efficiency of the cycle, η= 1 − 2  x100 or η= 1 −  x100 = 72.72%
 T1   1100 

PROBLEM-SOLVING TACTICS

(a) Most of the problems of Tmax, Pmax and Vmax are solved by differentiating. Sometimes the graph will be given
and sometimes, the equation will be given. For problems of finding Pmax , you will require either the P-V or
P-T equation.
(b) From the sections of thermodynamic, graphical questions are usually asked so one must have a thorough
understanding of PV diagrams at least. It is generally advised to convert other graphs namely TV and PT graph
to PV graph before solving to visualize it easily. Also in many cases we can directly get asked quantities like
work done because it involves integration of area under the PV graph.
(c) Silly mistakes must be avoided because there is not much variety in problems and not so much to think.
Remember the following points:-
(i) Be very careful in signs of Q and W while solving questions of thermodynamics.
(ii) Be careful in noting the type of gas used in question whether it is monatomic or diatomic.
(iii) Be careful with units of given quantities and asked quantities.
(iv) You can list more from your personal experience.
(d) It would be helpful to note keywords in a question (like noting down slow and fast changes because slow
changes imply reversible change whereas fast change imply irreversible change).
(e) It is always helpful to make a separate list of known quantities and asked quantities and then think of a way,
a link between them.
(f) Questions are mostly based on various reversible or irreversible processes which have their own set of
formulae. And no more processes can be found other than these. Hence, these concepts must be strong and
you should know the algorithm of each one thoroughly.

FORMULAE SHEET

Kinetic Theory of Gases

1
=PV = nMm Vrms2 nRT
3
1 3
=
Kinetic energy per mole of gas = E = Mm Vrms2 RT
2 2
3kT 3RT 3P
Vrms
= = =
m M ρ
P hysi cs | 13.23

8kT 8RT 8P
Vav
= = = M is the molar mass in kilogram per mole.
πm πM πρ

2kT 2RT 2P
Vmp
= = =
m M ρ

8
Vrms : Vav : Vmps :: 3 : : 2 ; Vmps : Vrms : Vav ::1 : 1.224 : 1.28
π
The mean free path of a gas molecule is the average distance between two successive collisions. It is represented by λ.

kT σ =Diameter of themolecule
λ= Here,
2πd ρ 2 k = Boltzmann'scons tant

Thermodynamics
(a) Boyle’s law: According to this law, for a given mas of a gas the volume P
of a gas at constant temperature (called isothermal process) is inversely
proportional to its pressure, i.e., T=constant
1
V∝ (T= constant) or PV = constant
P
V

Figure 13.18

(b) Charle’s law: According to this law, for a given mass of a gas, the volume of V
a gas at constant pressure (called isobaric process) is directly proportional P=constant
to its absolute temperature, i.e.,
V
V ∝ T (P= constant) or = cons tant
T
T(in K)

Figure 13.19

(c) Gay Lussac’s law or Pressure law: According to this law, for a given P
mass of a gas the pressure of a gas at constant volume (called isochoric
process) is directly proportional to its absolute temperature, i.e.,
P V=constant
P∝T (V= constant) or = cons tant
T
T(in K)

Figure 13.20

(d) Avogadro’s law: According to this law, at same temperature and pressure
equal volumes of all gases contain equal number of molecules.

Ideal gas equation


PM
PV = nRT OR ρ=
RT
Degree of freedom of monoatomic f = 3 (all translational)
gas
1 3 . 2 4 | Kinetic Theor y of Gases and Thermodynamics

Degree of freedom of a diatomic f = 5 (3 translational + 2 rotational) at room temperatures and f = 7


and linear polyatomic gas
(3 translational + 2 rotational + 2 vibrational) at high temperatures.
Degree of freedom of nonlinear f = 6 (3 translational + 3 rotational) at room temperatures and f = 8
polyatomic gas
(3 translational + 3 rotational + 2 vibrational) at high temperatures.
Degree of freedom of solid An atom in a solid has no degrees of freedom for translational and rotational
motion. At high temperatures due to vibration along 3 axes it has 3 x 2 = 6 degrees
of freedom. f = 6 (all vibrational) at high temperatures.

∆Q
C= or ∆Q = nC∆T
n∆T

For a gas the value of C depends on the process through which its temperature is raised.

Types of gas Degrees of freedom Total energy CP CV γ

3 5 3
Monoatomic 3 RT R R 1.67
2 2 2

5 7 5
Diatomic 5 RT R R 1.4
2 2 2

Polyatomic( ≥ 3 ) 6 3RT 4R 3R 1.33

mic 5/3 = 1.67


o n oato
CP f+2 m
Ratio of specific heat :-  = diatomic 7/5 = 1.4
=
CV f polya
tomi
c 4/3 = 1.33

Note: C of a gas depends on the process of that gas, which can be infinite in types.
2 R γR
and f= ; Cv = ; CP =
γ −1 γ −1 γ −1

Often the first law must be used in its differential form, which is dU = dQ – dW
This can also be written as dQ = dU + dW
∆ Q is +ve for heat supplied
∆ Q is -ve for heat rejected

1. ∆ W = +ve for work done by the gas (in expansion of gas)


∆ W = -ve for work done on the gas (in contraction of gas)
f
2. For change in internal energy of the gas, apply ∆U= nC v ∆T or in differential form, dU = nCvdt = nRdT
2
3. For heat transfer, apply Q
= nC∆T
or in differential form dQ = nCdT
dW = PdV
Vf
W= ∫ dW = ∫ PdV = Area under P-V curve
Vi
P hysi cs | 13.25

Isothermal Q=W 0 V   Pi 
nRT ln  f  = nRT ln  
 Vi   Pf 

PV
i i − Pf Vf nR∆T
Adiabatic 0 nCV ∆T = −∆U=
γ −1 γ −1

Isobaric nCP ∆T nCV ∆T P(Vf − Vi )

Q = ∆U
Isochoric nCV ∆T 0
= nCV ∆T

P P
(Slope)isothermal = − (Slope)adiabatic = −γ  
V V

 Work done by the working subs tance 


   
(an ideal gas in our case) during a cycle  WTotal Q − ve
η=  × 100= × 100= 1 −  × 100
 Heat supplied to the gas during the cycle  Q + ve Q + ve
   
 
T
For Carnot Engine: η = 1 − 2
T1

Solved Examples

JEE Main/Boards 10 −3 × 250


=
N× 300
760 × 22400 23
6 ×10 × 273
Example 1: An electric bulb of volume 250cm³ was
sealed off during manufacture at the pressure of
10−3 × 250 × 6 × 1023 × 273
10-3mm of Hg at 27°C. Find the number of air molecules N= = 8.02 x1015 mole
in the bulb. 760 × 22400 × 300

N R
Sol:=
PV = RT N= T NkT Example 2: One gram-mole of oxygen at 27°C and one
A A
atmospheric pressure is enclosed in a vessel.
Let N be the number of air molecules in the bulb.
(a) Assuming the molecules to be moving with Vrms , find
V1 =250cm³, P1 = 10-3mm of Hg,
the number of collisions per second which molecules
T1 = 273 + 27 = 300°K make with one square meter area of the vessel wall.
As P1V1 = Nk T where k is constant, then (b) The vessel is next thermally insulated and moved
with a constant speed ν0 . It is then suddenly stopped.
10-3x 250 = N.k.300  … (i)
The process results in a rise of the temperature of the
At N.T.P., one mole of air occupies a volume of 22.4 litre,
gas by 1°C. Calculate the speed ν0 .
=V0 22400
= cm3 ,P0 = 760mmof
760 mm ofHg,
Hg,
P 3NAkT
T = 273° K and N0 = 6 × 1023 molecules Sol: Formula based: n = & Vrms = . Recall
kT Mn
∴760 × 22400 = 6x1023 × k × 273  … (ii)
the assumption of KTG. Kinetic energy changed to
Dividing equation (i) by equation (ii), we get internal energy.
1 3 . 2 6 | Kinetic Theor y of Gases and Thermodynamics

(a) n = P/kT f
+1 2
where k = Boltzmann constant = Sol: (a) y = 2 or y = 1 +
f f
1.38 x 10-23 J/mol/K 2
nR
P = 1 atmosphere = 1.01 x 105 n/m², (b)
= W (T − T )
1− γ f i
T = 27°C = 300°K
(a) For adiabatic process
1.01x105
n = 2.44x1025 m−3 TV γ−1 = T1V1γ−1 ; V1 5.66
= = V and T1 T / 2
1.38x10−23 x300
T
The root mean square velocity Vrms is given by ∴ TV γ−1 = x (5.66 V)γ−1 ; (5.66)γ−1 = 2
2
 3RT   3NAkT  ( γ − 1)log5.66 =log2
Vrms
= 
=   = (R NAk)
M   Mm 
 m log2 0.3010
γ −1 = ; γ − 1 = 0.4
=
NA= Avogadro number = 6.02 × 1023 molecule/mole log5.66 0.7528

Mm = 32gm = 32 × 10-3 kg. ∴ γ =1.4 (diatomic gas)

 36.02x1023 x1.38x10-23 x300  The degrees of freedom of gas molecules = 5


Vrms =  =483.4m/s. (b) Work done during adiabatic change, W, is given by
 32x10 -3 
 
1
= W [P V − PV]
Since each molecule may be moving at a given instant, 1− γ 1 1
along any of the six directions (i.e. +X, +Y and +Z) only
For an ideal gas equation,
(1/6)th of the total molecules contained in the volume
along positive and negative x, y, z directions, move PV P1V1 T1V 1 1 P
= = or P1 = P ; P1 = x P
towards the wall. The number of collisions per second T T1 TV1 2 5.66 11.32
with one square meter area of the vessel wall will be
1  P 
1 1 = W  x5.66V − PV 
= xn × Vrms = × 2.44 × 1025 × 483.4 = 1.97 × 1027. 1 − 1.4 11.32 
6 6
1
(b) K.E. = mv 02 PV  5.66  PV
2 = 1 − = = 1.25 PV.
0.4  1132  0.8
Heat energy gained = C v =
∆T C v =
x1 C v
5
R Example 4: Two moles of helium gas with γ equal to
R or C v =
Cp − C v = 3
γ −1 are initially at temperature 27°C and occupy a volume of
1 R 20 litres. The gas is first expanded at constant pressure
∴ γ =1.41 ; 2
mv 0= C=
2 v
γ −1 until the volume is doubled. Then it undergoes an
adiabatic change until the temperature returns to its
 2R   2x8.31x1  initial value.
=ν0 =   =  35.6m / sec.
 m( γ − 1)  −3
 32x10 x141 − 1  (a) Sketch the process on P-V diagram.
(b) Find the final value of volume and pressure of the
Example 3: An ideal gas having initial pressure P, gas.
volume V and temperature T is allowed to expand (c) Find the work done by the gas under isobaric and
adiabatically until its volume be-comes 5.66 V while its adiabatic processes and total work done.
temperature fall to T/2.
(a) How many degrees of freedom do the gas molecules Sol: Relate P1 V1 and T1 with P2, V2, T2 and P3, V3, T3.
have? Calculate the work alone of process AB, BC separately.

(b) Obtain the work done by the gas during the (a) The sketch of P-V diagram is shown in the figure.
expansion as a function of the initial pressure P and
volume V.
P hysi cs | 13.27

2.5 x 10
5 P1V1T1 P2V2T2 Example 5: Two moles of helium gas undergo a cyclic
A B process as shown in the figure.
Assuming the gas to be ideal, calculate the following
C quantities in this process.
-2
P(Nm ) P3V3T3 (a) The net change in the heat energy

20 x 10
-3
40 x 10
-3 (b) The net work done
(c) The net change in internal energy
-3
V(m)

(b) At A, = = 20 x10−3 m3 T1 = 27°C = 300K ; P1 = ?


V1 20l Sol: Calculate for isobaric and isothermal process only.

For an ideal gas, PV = nRT. (a) AB is isobaric process. The work done during this
process from A to B:
nRT1 2x8.31x300
=P1 = = 2.5 x105 Nm−2 WAB = P(V2 − V1 ) = nR (T2 − T1 )
V1 20x10 −3
or=
WAB 2 x 8.3x(400 =
− 300) 1160 joule
At B, P2= P1= 2.5 x105 Nm−2 , V2 = 40 x10−3 m3
Work done during isothermal process from B to C:
For isobaric process,
P
A B
T2 V2 40 x10−3 2 atm
= = T2 2=
= 2= T1 2 x300
= 600K
T1 V1 20 x10−3

The gas now undergoes adiabatic expansion from B to C. 1 atm


D C
T
T3 V3γ−1 = T2 V2 γ−1 300 K 400 K
1
1/(5/3−1) =WAB nRT
= C loge (V2 / V1 ) nRTC loge (P2 / P1 )
V3  T2  γ−1  600  3
=  =   = 2=
2 2=
2 2.83
V2 = 2 x 8.3x 400 x 2.303log10 2
 T3   300 
= 2 x 8.3 x 400 x 2.303 x 0.3010 = 4602.9 joule
Final Volume =
−3
Work done during isobaric process from C to D:
=V3 V=
2 x 2.82 40 x10 = x 2.83 113 x10−3 m3 V
WCD= nR(TD − TC )= 2 x 8.3x(300 − 400) = − 1660 joule
Final pressure P3 is given by P3 V3γ = P2 V2 γ
Work done during isothermal process from D to A:
γ 5/3
V   40 x10 −3 
= x  2  2.5 x105 x 
P3 P2=  WDA = mRTD loge (PD / PA ) = nRTD loge 2
V   113 x10 −3 
 3   = 2 x8.03 x 300 x 2.0303 x 0.3010 = −3452.2 Joule
2.5
= x105 x (0.353)5/3 2.5 x105 x 0.176 Net workdone = WAB + WBC + WCD + WDA
5 −2
= 4.410 N.m = 1660 + 4602.9 – 1660 – 3452.2
(c) Work done during isobaric process along AB = P = 1150.7 joule
5 −3
(V2 − V1 ) 2.5 x10
= = x 20 x10 5000J (b) First law of thermodynamics gives

Work done during adiabatic process along BC = ∆Q = ∆U + ∆W


Work done during adiabatic process along BC =
nR(T2 − T3 ) 2x831x (600 − 300) As ∆U =
0 , in cyclic process
nR(T2=− T3 ) 2x831x (600 − 300)
BC =
γ −1 = 5
∴ ∆Q =∆W =1150.7 joule
γ −1 −1 5
3 −1
3 The heat given to the system = 1150.7 joule
Total work done = 5000 + 7479 = 11479 J. (c) As the gas returns to its original state, there is no
change in internal energy.
1 3 . 2 8 | Kinetic Theor y of Gases and Thermodynamics

Example 6: An ideal gas is taken a cyclic thermodynamic (b) Is there any way of telling afterwards which sample
process through four steps. The amount of heat involved of Helium went through the process ABC and which
in these steps are Q1 =
5960 J,Q 2 = − 5585 J,Q 3 =
− 2980 J went through the process ADC? Write Yes or No.
and Q 4 = 3645 J respectively. The corresponding works (c) How much is the heat involved in each of the
involved are W1 = 2200 J, W2 = −825 J, respectively. processes ABC and ADC?
(a) Find the value of W4
Sol: Work = Area under P–V curve hence, work done
(b) What is the efficiency of the cycle? in ABC is more than in ADC so is the heat (Q). At C,
system’s thermodynamic states are same, it can’t be
Sol: QTotal =WTotal as ∆U =0 in cyclic process, determined how they are achieved.
WT
η= only 2 x103
(a) n = 500
( Q absorbed) =
4
As the process is cyclic, ∆U =0 At A, PA VA = nRTA or TA = (PA VA / nR)
Net heat absorbed by the system (5x10 4 )x10
∴ T=
A = 120.33K
Q = Q1 + Q 2 + Q 3 + Q 4 500x831
= 5960 – 5585 – 2980 + 3645 = 1040 J (10x10 4 )x10
Similarly,
= TB = 240.66K
Net work performed 500x831

W = W1 + W2 + W3 + W4 (10x10 4 )x20
=TC = 481.32K
= 2200 – 825 – 1100 + W4 = 275 + W4 500x831

According to the first law of thermodynamics (5x10 4 )x20


=TD = 240.66K
Q =∆U + W ; 1040 = 0 + 275 + W4 500x831

∴ W4 = 1040 − 275 = 765 joule. (b) No


(c) For process ABC:
Work done(W)
Efficiency η= Change in internal energy ∆U= nC v ∆T
Heat absorbed(Q1 + Q 4 )
275 + 765 1040 3 
= = = 0.1082 ( ∆ U)ABC
= n  R  ∆T
5960 + 3645 9605
2 
Percentage efficiency = 10.82%
3 
= 500x  x8.3  x[481.32 − 120.33] = 2.25 x106 J
2 
Example 7: A sample of 2 kg of monoatomic Helium
Work done ( ∆W)ABC =10 x (10 x 104)
(assumed ideal) is taken through the process ABC and
another sample of 2 kg of the same gas is taken through = 106 J ( ∆Q)
= ABC (dU)ABC + ( ∆ W)ABC
the process ADC as shown in the figure.
= 2.25 x106 += 106 3.25 x106 J
4 2
P(10 N/m ) For process ADC:
B C ( ∆ W)ADC = 5 x 104 [20-10] = 0.5 x 106 J
10
( ∆U)ADC = 2.25 x 106
∴ ( ∆ Q)ADC = (2.25 x 106) + (0.5 x 106)

5 = 2.75 x 106 J
D
A
Example 8: Pressure versus temperature graph of an
0 10 20 3
V(m ) ideal gas is as shown in figure. Density of the gas at
point A is ρ0 . Density at B would be
Given molecular mass of Helium = 4
(a) What is the temperature of Helium in each of the Sol: Use relation between density and temperature,
states A, B, C and D? given below.
P hysi cs | 13.29

PM P the gases A and B are 4 and 32 respectively. The gases


=
ρ ∝ A and B do not react with each other and are assumed
RT T
to be ideal. The gaseous mixture follows the equation
Pressure PV19/13 = constant, in adiabatic processes.

P   P0  (a) Find the number of gram moles of the gas B in the


  = 2   gaseous mixture.
 T A  T0 
(b) Compute the speed of sound in the gaseous mixture
at T=300K.
P
(c) If T is raised by 1 K from 300 K, find the percentage
3P0 B change in the speed of sound in the gaseous mixture.
(d) The mixture is combined adiabatically to 1/5th of
P0 A its initial volume V. Find the change in its adiabatic
compressibility in terms of the given quantities.
T
T0 2T0 Sol: γ of the mixture is known from equation of process.
Therefore CV can be known. Compare CV (mixture)
P  3  P0  3  P  nA CV + nB (CV )B
and =
  =     = A
 T B 2  T0  4  T A (nA + nB )
3 3 (a) As the gaseous mixture follows the equation PV19/13
∴ ρ= ρA = ρ
4 4 0 = constant, then for the mixture of the gas γ =19 / 13.
CP CV + R R 19
γ= = = 1+ =
Example 9: The root mean square (rms) speed of CV CV C v 13
hydrogen molecules at a certain temperature is
300 m/s. If the temperature is doubled and hydrogen R 6 13R
= , ∴ CV =
gas dissociates into atomic hydrogen the rms speed CV 13 6
will become 19
CP = CV + R = R
6
Sol: Formula based Vrms ∝ T .
For gas A, γ A =5 / 3 ; For gas B, γB =7 / 5.
3RT 3 5
Vrms = (CV )A
= =R and(CV )B R
M 2 2
T is doubled and M is halved. Therefore, rms speed will 5 7
= (CP )A = R and(CP )B R
become two times or 600 m/s. 2 2
Let nA and nB be the number of kg moles in gas A and
Example 10: The changes in temperature of an ideal gas B respectively.
gas, when its volume changes from V to 2V in the
nA =1gm mole = 10−3 kg.mole
process P = aV, is (Here a is a positive constant)
As the gases have fixed volume, no work is done by the
Sol: Use relation between P and V. gas and vessel system. In the adiabatic process, no heat
is exchanged with the surroundings, the internal energy
P ∝ V (P=aV)
of the system will remain constant.
Therefore, pressure and volume both are doubled or ∴ (nA + nB )C=
V dT nA (C V )A dT + nB (C V )B dT
temperature becomes four times ( T ∝ PV )
(nA + nB )CV = nA (CV )A + nB (CV )B

JEE Advanced/Boards  13  3 5 
(1x10−3 + nB )  R  = (1x10−3 ) R + nB  R 
6  2 6 
Example 1: A gaseous mixture enclosed in a vessel of 13x10−3 + 13nB= 9x10−3 + nB (15)
volume V consists of one gram mole of a gas A with
5 / 3 and another has B with γ =7 / 5 at a
γ(CP / CV ) = 2nB = 4x10−3
certain temperature T. The gram molecular weights of
1 3 . 3 0 | Kinetic Theor y of Gases and Thermodynamics

4x10−3 (a) For adiabatic expansion


∴ nB= = 2x10−3 kgmole= 2gm mole
2 T1V1γ−1 = T2 V2 γ−1 ; 300 x (V)γ−1 = T2 (2 V)γ−1
(b) The speed of sound in gaseous mixture is given by
1 300
=T2 300
= x = (γ 5 / 3)
 γRT  (2) γ−1
(2)2/3
ν=  
 M 
∴ T2 =
188.99K
where R is gas constant and M is equivalent gram
molecular weight of gaseous mixture. Let MA and (b) Change in thermal energy is given by
MB be the gram molecular weights of gases A and B ∆ U nCV (T2 −=
= T1 ) 2 x(3R/ 2)(188.99 − 300)
respectively, then
nAMA + nBMB =
(nA + nB )M  3 x 8.3 
= 2x  x ( −111.09)
= 2767.5 J
nAMA + nBMB (1x 4) + (2 x 32)  2 
or M
= = The negative sign indicates that there is a decrease in
nAnB 1+ 2
the internal energy.
68 68
= = gm x10−3 kg (c) For adiabatic process,
3 3
∆W + ∆U =
0 or ∆W =
− ∆U
 19  8.3 x 300 x 3  The workdone by the gas is given by
∴ν   x
=  400.7m / s
 13  68 x10
−3
 nR(T2 − T1 ) 2x8.3x(188.99 − 300)
∆W =− =−
γ −1 (5 / 3 − 1)
 γRT 
(c) At temperature T, ν =   = 2764.2 J
 M 
Example 3: Two moles of an ideal monoatomic gas,
 γRT'  initially at pressure p1 and volume V1 undergo an
T' (300 + 1)K,=
= ν'  
 M  adiabatic compression until its volume is V2. Then the
gas is given heat Q at constant volume V2.
ν'  T'   301  ν '   301  
∴= =    or = −1    − 1 (a) Sketch the complete process on a P-V diagram.
ν T  300  ν   300  
(b) Find the total workdone by the gas, the total change
ν '− ν in its internal energy and the final temperature of the
The percentage change in speed of sound = x100
ν gas.
 301 
= − 1 x100 = 0.17% [Give your answers in terms of p1 , V1 , V2 ,Q and R].
 300  nR∆T
Sol: Use formula TV γ−1 = constant and W =
(d) Adiabatic compressibility = 1− γ
1  dv  1 (a) P-V diagram is shown in the figure where AB is
βmt =−   = −
V  dp mt γP adiabatic compression and BC is isobaric heating

1  1  13 V  1  −3
P
=
∆β 1 − =  1 − =  2.487 x10 .
γP  (5)  3x19 RT  (5)γ 
γ
P3 C
P2 AB=Adiabatic compression
B
Example 2: At 27°C, two moles of an ideal monoatomic BC= Isobaric heating
gas occupy a volume V. The gas expands adiabatically
to a volume 2V. Calculate
P1 A
(a) The final temperature of the gas
V
(b) Change in its internal erergy V2 V1
...(l)
(c) The workdone by the gas during this process
(b) Let T1 and T2 be the initial temperature and the
Sol: Formula TV = constant, ∆=
γ
U n C v ∆T W = −∆U temperature after adiabatic compression respectively.
used.
P hysi cs | 13.31

For adiabatic compression The pressure and temperature at A, B etc., are denoted
V1γ−1T1 = V2γ−1T2 by PA , TA ;PB , TB etc. respectively.
p1V1 5
T1
= = ,γ for monoatomic gas Given
= TA 1000K,P
= B (2 / 3)PA & PC = (1/ 3)PA . Calculate
2R 3
(a) The work done by the gas in the process A → B
p V
  p V 5/3 
∴ V12/3  1 1
 V
=2
2/3
T2 ∴ T  1 1  
2
….(i) (b) The heat lost by the gas in the process B → C and
2R 2/3 
   2RV2 
(c) Temperature TD given (2/3) = 0.85 and R-8.31 J/mol K.
For isochoric process at temperature T3, heat supplied
is Q. Sol: Use the relation for the respective processes. Such
as T/P relation in adiabatic process.
∴Q= V dT ; Q 2.3 / 2R(T3 − T2 )
nC=
(a) As for adiabatic change
Q Q Q p1V15/3 PV γ = constant
= T3 − T2 ;∴ T3 = + T2 = + …. (ii)
3R 3R 3R 2RV 2/3
2   nRT 
γ

The total work done by the gas is equal to the work i.e. P   = cons tant [as PV = nRT ]
 P 
done in adiabatic process plus the work done in
isochoric process when WBC = 0 Tγ
i.e. = cons tant
∴ WTotal = WAB + WBC = WAB P γ−1

nR(T1 − T2 ) 5
WTotal
= = where γ =
γ −1 3
1
  V 2/3  1− 2/5
2R  p1V1 p1v1  3
5/3
2 γ 2
=  −  = p V 1 −  1   i.e. TB T=
= A  1000
=   850K
2 / 3  2R 2RV 2/3  2 1 1   V2   3 3
 2   
nR[Ti − Tf ] 1x8.3[1000 − 850]
Change in internal energy =
∆ U nCV (T3 − T1 ) so
= WAB =
γ −1 [(5 / 3) − 1]
3  Q p V pV
5/3
∴ ∆U 2x  R   + 1 1 − 1 1 
= i.e. WAB (3
= = / 2)x 8.31x150 1869.75J
 2   3R 2RV22/3 2R 

(b) For B → C, V = constant so ∆ W =
0
5/3  V 2/3 
3 p1V1 3 3 so from first law of thermodynamics
Q+
= Q + p1V1  1  − 1
− p1V1 =
2 V 2/3 2 2  V2  
2   ∆ Q =∆U + ∆ W =µCV ∆T + 0

Example 4: One mole of monoatomic ideal gas is taken 3  3


or =
∆ Q 1x  R  (TC − 850) as CV = R
through the cycle shown in figure. 2  2

Now along path BC, V = constant; P ∝ T


A
B
PC TC (1/ 3)PA TB 850
i.e. = , T=
C xT=
B = = 425K ….(ii)
PB TB (2 / 3)PA 2 2
P
3
D C So, ∆Q =
1x x8.31(425 − 850) =− 5297.625 J
2
V [Negative heat means heat is lost by the sys.]
(c) D → A process is isochoric
A →B Adiabatic expansion
PD TD TD
B→C Cooling at constant volume = , i.e. PD = PA
PA TA TA
C →D Adiabatic compression
But C and D are on the same adiabatic
D →A Heating at constant volume
1 3 . 3 2 | Kinetic Theor y of Gases and Thermodynamics

γ γ−1 γ−1
 TD   PD   PA TD   η − 1
=
  =    V=  V0  … (ii)
 TC   PC   PC TA   η + 1

1 The work done by the agent is given by


1−
 P  γ
or (TD )1/ γ = TC  A  , V V
2P0 V0 V
 PC TA  W = ∫ (P2 − P1 )dV =∫ dV
2/5 0 0 V02 − V 2
 T  PA 
i.e. TC3/5 =  B    − P0 V0 [ln(V02 − V 2 )V ]0v =
= − P0 V0 [ln(V02 − V 2 ) − lnV02 ]
 2   (1/ 3)PA 1000 

   η − 1  2 
2 
 1  2 2/3   3  2/5 
i.e. −P0 V0 ln V02 − 
=  V0  − lnV0 
2
TD3/5 =    x1000       η + 1 
2  3   1000     
 
i.e. TD = 500K  (η + 1)2 
= −P0 V0 [ln{4η / (η + 1)2 }] = P0 V0 ln  .
 4η 
Example 5: A piston can freely move inside a
horizontal cylinder closed from both ends. Initially, the
Example 6: An ideal gas has a density of 1.78 kg / m³ is
piston separates the inside space of the cylinder into
contained in a volume of 44.8 x 10-3 m³. The temperature
two equal part of volume V0, in which an ideal gas is
of the gas is 273 K. The pressure of the gas is
contained under the same pressure P0 and at the same
0.01 x 105 Pa. The gas constant R = 8.31 J-K-1 mole -1.
temperature.
(a) What is the root mean square velocity of the air
x molecules?
(b) How many moles of gas are present?
P1A
(c) What is the gas?
P2A
Fagent
(d) What is the internal energy of the gas?
What work has to be performed in order to increase
Sol: Use relation Vrms and P given. m = ρ V = nM also.
isothermally the volume of one of gas η times
compared to that of the other by slowly moving piston? 1 2
(a) P = ρV ⇒
3
Sol: Apply isothermal condition on both compartments. 1/2 1/2
 3P   (3)(1.01x105 N/ m2 ) 
Then, proceed to find Vf (left)/Vf (right). =Vrms  =  
  
 ρ   1.78kg / m3 
Let the agent move as shown.
In equilibrium position, P1A + Fagent = = 4.13 x 10² m/s
P2 A
F = (P − P )A (b) PV nmRT ⇒
=
agent 2 1

Elementary work done by the agent PV (1.01x105 Nm−2 )(44.8x10−3 m3 )


n=
m =
RT (8.31JK −1mole−1 )(273K)
Fagent dx = (P2 − P1 )dV 
(P2 − P1 ) Axdx = … (i)
= 2.0 moles
Applying PV = constant for two parts, we have
ρV (1.78kg / m3 )(44.8 x10−3 m3
P=
1(V0 + Ax) P0 V0 and P=
2 (V0 − Ax) P0 V0 Mmolar
= =
nm (2.0moles)
P0 V0 P0 V0
P1 = and P2 = = 340.0 x10−3 kg / mole
(V0 + Ax) (V0 − Ax)
(c) This ideal gas is Argon.
P0 V0 (2 Ax) 2P0 V0 V
P2 − P1
∴ = = (d) Internal energy of monoatomic gas=3/2 nRT.
V02 − A2 x2 V02 − V 2

When the volume of the left end is η times the volume


of right end, we have (V0 + V) =η(V0 − V)
P hysi cs | 13.33

Example 7: Plot P-V, V-T and ρ -T graph corresponding P-V graph: As P is constant. Therefore, P-V graph is a
to the P-T graph for an ideal gas shown in the figure. straight line parallel to V-axis with VC > VB (because V
∝ T in an isobaric process)
P
V-T graph: In an isobaric process V ∝ T, i.e., V-T graph
B C is a straight line passing through the origin, with TC> TB
and VC> VB.

A D ρ -T graph: ρ ∝ 1 (when P = constant), i.e., ρ -T graph


T T
is a hyperbola with TC > TB and ρC > ρB .
Sol: Look for parameter which is constant in the each
process. There is no need to discuss C-D and D-A processes
as they are opposite to AB and BC respectively. The
Process AB is an isothermal process with T = constant
corresponding three graphs are shown above.
and PB> PA.
D
1 V
P-V graph: P ∝ i.e., P-V graph is a hyperbola with P P
V B A B
PB > PA and VB > VA . C
C C
D
V-T graph: T= constant. Therefore, V-T graph is a A B A
straight line parallel to V-axis with VB > VA . V D
T T
PM D
ρ − T graph
= P :ρ or ρ ∝ P . V
P
RT
B A
As T is constant. Therefore,
C ρ − T graph is a straight line B
parallel to ρ -axis with ρB > ρA as PB > PA . C C
A D
Process BC is isobaric process with P = constant
B and A
TC > TB . V T D
T

JEE Main/Boards

Exercise 1 Q.5 State the postulates of Kinetic Theory of gases.


Explain the pressure exerted by an ideal gas.

Q.1 Although the r.m.s. speed of gas molecules is of the


Q.6 Find an expression for the pressure exerted by a
order of the speed of sound in that gas yet on opening
gas and establish its relation with kinetic energy of the
a bottle of ammonia in one corner of a room, its smell
gas.
takes time in reaching the other corner. Explain Why?

Q.7 From Kinetic Theory of gases, explain kinetic


Q.2 The pressure of a gas at – 173°C temperature is
interpretation of temperature and absolute zero.
1 atmosphere, keeping the volume constant, to what
temperature should the gas be heated so that its
pressure becomes 2 atmosphere. Q.8 Explain the concept of mean free path.

Q.3 Explain (i) Boyle’s law (ii) Charle’s law. Why they are Q.9 Explain what is meant by Brownian Motion?
not applicable to real gases at all states?
Q.10 The density of water is 1000kg/m³. The density of
Q.4 State and explain (i) Guy Isac’s law and (ii) Gas water vapour at 100°C and 2 atmospheric pressure is
equation. Distinguish clearly between R and r for a gas. 0.6kg m³. The volume of a molecule multiplied by the
1 3 . 3 4 | Kinetic Theor y of Gases and Thermodynamics

total number gives what is called, molecular volume. Q.23 State the sign conventions used in all
Estimate the ratio (or fraction) of the molecular volume thermodynamic processes.
to the total volume occupied by the water vapour under
the above conditions of temperature and pressure. Q.24 What do you learn by applying first law of
thermodynamics to isothermal and adiabatic processes?
Q.11 A 3000cm³ tank contains oxygen at 20°C and a
gauge pressure of 2.5 x 106Pa. Find the mass of the Q.25 Explain what is meant by isothermal operations.
oxygen in the tank. Take 1 atm = 105 Pa. Give some examples.

Q.12 Calculate the r.m.s. velocity of air molecules at Q.26 What are adiabatic operations? Enumerate
N.T.P. Given that 22400 c.c. of gas at N.T.P. weight 64 gm. some examples. State equations representing these
operations.
Q.13 How many collisions per second does each
molecule of a gas make, when average speed of the Q.27 Obtain an expression for work done by a gas in
molecule is 500ms-1 and mean free path is 2.66 x 10-7 m? isothermal expansion.

Q.14 Calculate the mean free path of gas molecules, if Q.28 Derive an expression for work done in an adiabatic
number of molecules per cm³ is 3 x 1019 and diameter process.
of each molecule is 2Å.

Q.29 What are cyclic and non cyclic processes? Calculate


Q.15 The diameter of a gas molecules is 2.4 x 10-10m. work done in such processes.
Calculate the mean free path at N.T.P. Given Boltzmann
constant k = 1.38 x 10-23 J molecule-1 K-1.
Q.30 What are reversible and irreversible processes?
Give some examples of each.
Q.16 Which molecules, ice at 0°C or water 0°C have
greater potential energy and why?
Q.31 What is a heat engine? Obtain an expression for
its efficiency.
Q.17 An ideal gas is compressed at a constant
temperature, will its internal energy increases of
decrease? Q.32 A tyre pumped to a pressure of 3 atmosphere
suddenly bursts. Calculate the fall in temperature due
to adiabatic expansion. The temperature of air before
Q.18 Which type of motion of the molecules is expansion is 27°C and value of γ = 1.4.
responsible for internal energy of a monoatomic gas?

Q.33 A quantity of air at 27°C and atmospheric pressure


Q.19 The volume of an ideal gas is V at a pressure P. On is suddenly compressed to half its original volume. Find
increasing the pressure by ∆ P, the change in volume of the final (i) pressure and (ii) temperature. Given γ for air
the gas is ( ∆ V1) under isothermal conditions and ( ∆ V2 ) = 1.42.
under adiabatic conditions, Is ∆V1 > ∆V2 or vice-versa
and why?
Q.34 A Cylinder containing one gram mole of gas was
put on boiling water bath and compressed adiabatically
Q.20 200 joule of work is done on a gas to reduce till its temperature rose by 70°C. Calculate the work
its volume by coming it. If this change is done under done and increase in energy of the gas, γ = 1.5, R = 2
adiabatic conditions, find out the change in internal cal. mole-1 K-1.
energy of the gas and also the amount of heat absorbed
by the gas?
Q.35 One gram mole of an ideal gas at S.T.P. is subjected
to reversible adiabatic expansion to double its volume.
Q.21 Give briefly the concept of internal energy. Find the change in internal energy in the process. Take
γ =1.4.
Q.22 Define the four thermodynamic processes. What
is meant by indicator diagram?
P hysi cs | 13.35

Q.36 If 1 gram oxygen at 760mm pressure and 0°C has Q.7 A barometer tube, containing mercury, is lowered
it volume doubled in an adiabatic change, calculate in a vessel containing mercury until only 50 cm of the
the change in internal energy. Take R=2 cal. mole-1 K-1, tube is above the level of mercury in the vessel. If the
J=4.2 J cal-1 and γ =1.4. atmospheric pressure is 75 cm of mercury, what is the
pressure at the top of the tube?
(A) 33.3 kPa (B) 66.7 kPa
Exercise 2
(C) 3.33 MPa (D) 6.67 MPa
Single Correct Choice Type
Q.8 A vessel contains 1 mole of O2 gas (molar mass
Q.1 Find the approx. number of molecules contained in
32) at a temperature T. The pressure of the gas is P. An
a vessel of volume 7 litres at 0°C at 1.3 x 105Pascal
identical vessel containing one mole of He gas (molar
(A) 2.4 x 1023 (B) 3 x 1023 mass 4) at a temperature 3T has a pressure of
(C) 6 x 1023 (D) 4.8 x 1023 (A) P/8 (B) P (C) 2P (D) 8P

Q.2 An ideal gas mixture filled inside a balloon Q.9 The ratio of average translational kinetic energy
expands according to the relation PV2/3 = constant. The to rotational kinetic energy of a diatomic molecule
temperature inside the balloon is temperature T is
(A) Increasing (B) Decreasing (A) 3 (B) 7/5 (C) 5/3 (D) 3/2
(C) Constant (D) Can’t be said
Q.10 One mole of an ideal gas at STP is heated in an
Q.3 A rigid tank contains 35 kg of nitrogen at 6 atm. insulated closed container until the average speed of
Sufficient quality of oxygen is supplied to increase its molecules is doubled. Its pressure would therefore
the pressure to 9 atm, while the temperature remains increase by factor.
constant. Amount of oxygen supplied to the tank is:
(A) 1.5 (B) 2 (C) 2 (D) 4
(A) 5 kg (B) 10 kg (C) 20 kg (D) 40 kg

Q.11 One mole of an ideal gas is contained within


Q.4 At temperature T K, the pressure of 4.0g argon in
a cylinder by a frictionless piston and is initially at
bulb is p. The bulb is put in a bath having temperature
temperature T. The pressure of the gas is kept constant
higher by 50K than the first one. 0.8g of argon gas had
while it is heated and its volume doubles. If R is molar
to be removed to maintained original pressure. The
gas constant, the work done by the gas in increasing its
temperature T is equal to
volume is?
(A) 510 K (B) 200 K (C) 100 K (D) 73 K
(A) RT ln2 (B) 1/2RT (C) RT (D) 3/2 RT

Q.5 When 2 gms of a gas are introduced into an


Q.12 A polyatomic gas with six degrees of freedom
evacuated flask kept at 25°C the pressure is found to be
does 25J work when it is expanded at constant pressure.
one atmosphere. If 3 gms of another gas added to the
The heat given to the gas is?
same flask the pressure becomes 1.5 atmosphere. The
ratio of the molecular weights of these gases will be (A) 100J (B) 150J (C) 200J (D) 250J
(A) 1: 3 (B) 3: 1 (C) 2: 3 (D) 3: 2
Q.13 In thermodynamic process pressure of a fixed
mass of gas is changed in such a manner that the gas
Q.6 During an experiment, an ideal gas obeys an
release 30 joule of heat and 18 joule of work was done
addition equation of state P2V = constant. The
on the gas. It the initial internal energy of the gas was
initial temperature and pressure of gas are T and V
60 joule, then, the final internal energy will be?
respectively. When it expands to volume 2 V, then its
temperature will be? (A) 32 joule (B) 48 joule
(C) 72 joule (D) 96 joule
(A) T   (B) 2 T    (C) 2 T    (D) 2 2 T
1 3 . 3 6 | Kinetic Theor y of Gases and Thermodynamics

Q.14 An ideal gas undergoes an adiabatic process Q.3 70 cal of heat are required to raise the temperature
obeying the relation PV4/3 = constant. If its initial of 2 moles of an ideal diatomic gas at constant pressure
temperature is 300 K and then its pressure is increased from 30°C to 35°C. The amount of heat required
upto four times its initial value, then the final (in calorie) to raise the temperature of the same gas
temperature is (in Kelvin)? through the same range (30°C to 35°C) at constant
volume is  (1985)
(A) 300 2 (B) 300 3 2
(A) 30 (B) 50 (C) 70 (D) 90
(C) 600 (D) 1200
Q.15 1 kg of a gas does 20kJ of work and receives 16kJ
Q.4 If one mole of a monatomic gas ( γ =5 / 3) is mixed
of heat when it its expanded between two states. A
with one mole of a diatomic gas ( γ =7/5), the value of
second kind of expansion can be found between the
γ for the mixture is  (1988)
initial and final state which requires a heat input of 9 kj.
The work done by the gas in the second expansion is: (A) 1.40 (B) 150 (C) 1.53 (D) 3.07
(A) 32 kJ (B) 5 kJ
Q.5 The temperature of an ideal gas is increased from
(C) -4 kJ (D) 13 kJ 120 K to 480 K. If at 120 K the root mean square velocity
of the gas molecules is v, at 480 K it becomes  (1996)
Q.16 A mixture of ideal gases 7 kg of nitrogen and 11
(A) 4 v (B) 2 v (C) v/2 (D) v/4
kg of CO2. Then (Take γ for nitrogen and CO2 as 1.4 and
1.3 respectively)
Q.6 The average translational energy and the rms
(A) Equivalent molecular weight of the mixture is 36. speeds of molecules in a sample of oxygen gas at
(B) Equivalent molecular weight of the mixture is 18. 300 K are 6.21 x 10-21J and 484 m/s respectively. The
corresponding values at 600 K are nearly (assuming
(C) γ for the mixture is 5/2
ideal gas behaviour) (1997)
(D) γ for the mixture is 4/3
(A) 12.42x10-21 J, 968 m/s
(B) 8.78x10-21 J, 684 m/s
(C) 6.21x10-21 J, 968 m/s
Previous Years’ Questions
(D) 12.42x10-21 J, 684 m/s

Q.1 An ideal mono-atomic gas is taken round the cycle


ABCD as sown in the P-V diagram (see figure). The work Q.7 A vessel contains 1 mole of O2 gas (molar mass
done during the cycle is (1983) 32) at a temperature T. The pressure of the gas is p. An
identical vessel containing one mole of the gas (molar
P 2p, V 2p, 2V mass 4) at a temperature 2T has a pressure of  (1997)
B C
(A) p/8 (B) p (C) 2 p (D) 8 p

A D Q.8 Two identical containers A and B with frictionless


p, V p, 2V pistons contain the same ideal gas at the same
temperature and the same volume V. The mass of
the gas in A is mA and that in B is mB. The gas in each
(A) PV (B) 2 PV
cylinder is now allowed to expand isothermally to the
1 same final volume 2V. The changes in the pressure in
(C) pV (D) Zero
2 A and B are found to be ∆ P and 1.5 ∆ p respectively.
Then  (1998)
Q.2 At room temperature, the rms speed of the (A) 4mA = 9mB (B) 2mA = 3mB
molecules of a certain diatomic gas is found to be
1930 m/s. The gas is  (1984) (C) 3mA = 2mB (D) 9mA = 4mB

(A) H2 (B) F2 (C) O2 (D) Cl2


P hysi cs | 13.37

Q.9 A mixture consists of 2 moles of oxygen and 4 moles P


of argon at temperature T. Neglecting all vibrational B A
modes, the total internal energy of the system is 3P
 (1999)
(A) 4 RT (B) 15 RT (C) 9 RT (D) 11RT
1P
C D
Q.10 A monoatomic ideal gas, initially at temperature 0 1V 3V V
9V
T1, is enclosed in a cylinder fitted with a frictionless
piston. The gas is allowed to expand adiabatically to Column I Column II
a temperature T2 by releasing the piston suddenly.
If L1 and L2 are the lengths of the gas column before (A) Process A → B (p) Internal energy decreases
and after expansion respectively, thenT1/T2 is given by
 (2000) (B) Process B → C (q) Internal energy increases

(A) (L1 / L2 )2/3 (B) (L1 / L2 ) (r) Heat is lost


(C) Process C → D
(C) L2 / L1 (D) (L2 / L1 )2/3
(D) Process D → A (s) Heat is gained

Q.11 An ideal gas is expanding such that pT² = (t) Work is done on the gas
constant. The coefficient of volume expansion of the
gas is (2008)
Q.14 For an ideal gas
1 2 3 4
(A) (B) (C) (D) (A) The change in internal energy in a constant
T T T T
pressure process from temperature T1 to T2 is equal to
nCV (T2 − T1 ) , where CV is the molar heat capacity at
Q.12 Match the following for the given process (2006)
constant volume and n the number of moles of the gas
P
(B) The change in internal energy of the gas and the
30
J work done by the gas are equal in magnitude in an
M adiabatic process
20
(C) The internal energy does not change in an isothermal
10 K L process
3 (D) No heat is added or removed in an adiabatic
10 20 V (m )
process. (1989)
Column I Column II
Q.15 One mole an ideal gas in initial state A undergoes
(p) Q>0
(A) Process J → K a cyclic process ABCA, as shown in figure. Its pressure at
A is P0. Choose the correct option(s) from the following
(q) W<0
(B) Process K → L  (2010)
V
(r) W>0 B
(C) Process L → K 4V0

(s) Q<0
(D) Process M → J
V0 C A

Q.13 One mole of a monatomic ideal gas is taken T0 T


through a cycle ABCDA as shown in the P-V diagram.
(A) Internal energies at A and B are the same
column II gives the characteristics involved in the
cycle. Match them with each of the processes given in (B) Work done by the gas in process AB is P0V0 In 4
column I. (2011) P0
(C) Pressure at C is
4
T0
(D) Temperature at C is
4
1 3 . 3 8 | Kinetic Theor y of Gases and Thermodynamics

Q.16 The speed of sound in oxygen (O2) at a certain Q.23 The potential energy function for the force
temperature is 460 ms−1. The speed of sound in helium between two atoms in a diatomic molecule is
(He) at the same temperature will be (assumed both a b
gases to be ideal) (2008) approximately given by U(x) = − , where a and b
12
x x6
(A) 460 ms−1 (B) 500 ms−1 are constants and x is the distance between the atoms.
If the dissociation energy of the molecule is D = [U(x =
(C) 650 ms−1 (D) 1420 ms−1
∞) – Uat equilibrium], D is (2010)

Q.17 An insulated container of gas has two chambers b2 b2 b2 b2


(A) (B) (C) (D)
separated by an insulating partition. One of the chambers 2a 12a 4a 6a
has volume V1 and contains ideal gas at pressure P1 and
temperature T1. The other chamber has volume V2 and Q.24 Carnot engine operating between temperatures
contains ideal gas at pressure P2 and temperature T2. If
1
the partition is removed without doing any work on the T1 and T2 has efficiency . When T2 is lowered by 62
6
gas, the final equilibrium temperature of the gas in the 1
K, its efficiency increases to . Then T1 and T2 are,
container will be (2008) 3
T1T2 (P1V1 + P2 V2 ) P1V1T1 + P2 V2 T2 respectively: (2011)
(A) (B)
P1V1T2 + P2 V2 T1 P1V1 + P2 V2 (A) 372 K and 330 K (B) 330 K and 268 K
P1V1T2 + P2 V2 T1 T1T2 (P1V1 + P2 V2 ) (C) 310 K and 248 K (D) 372 K and 310 K
(C) (D)
P1V1 + P2 V2 P1V1T1 + P2 V2 T2
Q.25 Helium gas goes through a cycle ABCDA
Q.18 One kg of a diatomic gas is at a pressure of (consisting of two isochoric and two isobaric lines)
8 × 104 N/m2. The density of the gas is 4 kg m-3. What as shown in figure. Efficiency of this cycle is nearly:
is the energy of the gas due to its thermal motion? (Assume the gas to be close to ideal gas) (2012)
 (2009)
(A) 3 × 104 J (B) 5 × 104 J 2P0
B C

(C) 6 × 104 J (D) 7 × 104 J


P0 D
A
Q.19 Assuming the gas to be ideal the work done on
the gas in taking it from A to B is (2009)
V0 2V0
(A) 200 R (B) 300 R (C) 400 R (D) 500 R
(A) 15.4% (B) 9.1% (C) 10.5% (D) 12.5%
Q.20 The work done on the gas in taking it from D to A
is (2009)
Q.26 A Carnot engine, whose efficiency is 40%, takes
(A) – 414 R (B) + 414 R in heat from a source maintained at a temperature of
500 K It is desired to have an engine of efficiency 60%.
(C) – 690 R (D) + 690 R
Then, the intake temperature for the same exhaust
(sink) temperature must be (2012)
Q.21 The net work done on the gas in the cycle ABCDA
is (2009) (A) Efficiency of Carnot engine cannot be made larger
than 50%
(A) Zero (B) 276 R
(B) 1200 K
(C) 1076 R (D) 1904 R
(C) 750 K

Q.22 A diatomic ideal gas is used in a Car engine as the (D) 600 K
working substance. If during the adiabatic expansion
part of the cycle, volume of the gas increases from V to Q.27 An ideal gas enclosed in a vertical cylindrical
32V the efficiency of the engine is (2010) container supports a freely moving piston of mass M.
(A) 0.5 (B) 0.75 (C) 0.99 (D) 0.25 The piston and the cylinder have equal cross sectional
area A. When the piston is in equilibrium, the volume of
P hysi cs | 13.39

the gas is V0 and its pressure is P0. The piston is slightly (i) Sequentially keeping in contact with 2 reservoirs
displaced from the equilibrium position and released. such that each reservoir supplies same amount of heat.
Assuming that the system is completely isolated from
(ii) Sequentially keeping in contact with 8 reservoirs
its surrounding, the piston executes a simple harmonic
such that each reservoir supplies same amount of heat.
motion with frequency: (2013)
In both the cases body is brought from initial
1 AγP0 1 V0MP0
(A) (B) temperature 100°C to final temperature 200°C. Entropy
2π V0M 2π A 2 γ changes of the body in the two cases respectively is
2  (2015)
1 A γP0 1 MV0
(C) (D) (A) ln 2, 4 ln2 (B) ln 2, ln 2
2π MV0 2π AγP0
(C) ln 2, 2 ln 2 (D) 2 ln 2, 8 ln 2

Q.28 One mole of diatomic ideal gas undergoes a


Q.31 Consider an ideal gas confined in an isolated
cyclic process ABC as shown in figure. The process
closed chamber. As the gas undegoes an adiabatic
BC is adiabatic. The temperatures at A, B and C are
expansion, the average time of collision between
400 K, 800 K and 600 K respectively. Choose the correct
molecules increases as Vq, where V is the volume of the
statement (2014)
B  C 
800 K gas. The value of q is :  γ = P  (2015)
 Cv 

P
3γ + 5 3γ − 5
600 K (A) (B)
A C 6 6
400 K
γ +1 γ −1
V (C) (D)
2 2
(A) The change in internal energy in whole cyclic
process is 250R Q.32 ‘n’ moles of an ideal gas undergoes a process
(B) The change in internal energy in the process CA is A →B as shown in the figure. The maximum temperature
700R of the gas during the process will be: (2016)

(C) The change in internal energy in the process AB is P


–350R
2P0 A
(D) The change in internal energy in the process BC is
– 500R
P0 B

Q.29 Consider a spherical shell of radius R at


V
temperature T. The black body radiation inside it can V0 2V0
be considered as an ideal gas of photons with internal
U 3P0 V0 9P0 V0 9P0 V0 9P0 V0
energy per unit volume u = ∝ T 4 and pressure (A) (B) (C) (D)
V 2nR 2nR nR 4nR
1 U 
P =   . If the shell now undergoes an adiabatic
3 V  Q.33 An ideal gas undergoes a quasi static, reversible
expansion the relation between T and R is (2015) process in which its molar heat capacity C remains
constant. If during this process the relation of pressure
(A) T ∝ e−R (B) T ∝ e−3R P and volume V is given by PVn = constant, then n is
given by (Here Cp and CV are molar specific heat at
1 1
(C) T ∝ (D) T ∝ constant pressure and constant volume, respectively):
R R3  (2016)
C − Cp Cp − C
Q.30 A solid body of constant heat capacity 1 J/°C is (A) n = (B) n =
C − Cv C − Cv
being heated by keeping it in contact with reservoirs
in two ways: Cp
C − Cv
(C) n = (D) n =
C − Cp Cv
1 3 . 4 0 | Kinetic Theor y of Gases and Thermodynamics

JEE Advanced/Boards

Exercise 1 Q.9 An ideal gas has a molar heat capacity CV at


constant volume. Find the molar heat capacity of this
Q.1 A closed vessel of volume V0 contains oxygen at gas as a function of volume, if the gas undergoes the
a pressureP0 and temperatureT0. Another closed vessel process: T = T0eαV .
of the same volume V0 contains helium at a pressure V
P0 and temperatureT0/2. Find ratio of the masses of
oxygen to the helium.

Q.2 V-T curve for 2 moles of a gas is straight line as 1


shown in the graph here. Find the pressure of gas at A. T

V(lit.) B Q.10 One mole of an ideal monoatomic gas undergoes


a process as shown in the figure. Find the molar specific
A heat of the gas in the process.
o
53
T(K) Q.11 The pressure of an ideal gas changes with volumes
as P = aV where ‘a’ is a constant. One mole of this gas is
Q.3 A gas is undergoing an adiabatic process. At a
expanded to 3 times its original volume V0 . Find
certain stage A, the values of volume and temperature
= (V0 , T0 ) and the magnitude of the slope of V-T curve (i) The heat transferred in the process.
is m. Find the value of CP and CV .
(ii) The heat capacity of the gas

Q.4 Find the molecular mass of a gas if the specific


Q.12 In a cycle ABCA consisting of isothermal expansion
heats of the gas are CP = 0.2 cal/gm°C and CV =0.15
AB, isobaric compression BC and adiabatic compression
cal/gm°C. [Take R = 2 cal/mole°C]
CA, find the efficiency of cycle.
A
A
Q.5 The average degrees of freedom per molecules P
for a gas is 6. The gas performs 25 J of work when it
expands at constant pressure. Find the heat absorbed B
C
by the gas. >V
V0 2V0

Q.6 A mixture of 4gm helium and 28 gm of nitrogen in


(Given: TA= TB= 400K,=
γ 1.5 )
enclosed in a vessel of constant volume 300 K. Find the
quantity of heat absorbed by the mixture to double the
room mean velocity of its molecules. (R = Universal gas Q.13 A highly conduction solid cylinder of radius a
constant) and length  is surrounded by co-axial layer of a
material having thermal conductivity K and negligible
heat capacity. Temperature of surrounding space (out
Q.7 One mole of an ideal gas is compressed from
side the layer) is T0, which is higher than temperature
0.5 lit to 0.25 lit. During the compression, 23.04 x 10² J
of the cylinder. If heat capacity per unit volume of
of work is done on the gas and heat is removed to keep
cylinder material is s and outer radius of the layer is b,
the temperature of the gas constant at all times. Find
calculate time required to increase temperature of the
the temperature of the gas. (Take universal gas constant
cylinder from T1 to T2. Assume and faces to be thermally
R = 8.31 J mol-1K-1)
insulated.

Q.8 Ideal diatomic gas is taken through a process


Q.14 A vertical brick duct (tube) is filled with cast iron.
∆Q =2∆U . Find the molar heat capacity for the process
The lower end of the duct is maintained at a temperature
(where ∆ Q is the heat supplied and ∆ U is change in
T1 which is greater than the melting point Tm of cast iron
internal energy)
P hysi cs | 13.41

and the upper end at a temperature T2 which is less than Q.19 At a temperature of T0 = 273°K, two moles of
the temperature of the melting point of cast iron. It is an ideal gas undergoes a process as shown. The total
given that the conductivity of liquid cast iron is equal to amount of heat imparted to the gas equals Q = 27.7 kJ.
k times the conductivity solid cast iron. Determine the Determine the ratio of molar specific heat capacities.
fraction of the duct filled with molten metal.
T
C

k A
273 K B

V 4V
h

Q.20 A cylinder containing a gas is closed by a


movable piston. The cylinder is submerged in an ice-
water mixture. The piston is quickly pushed down from
Q.15 A lagged stick of cross section area 1cm² and position 1 to position 2. The piston is held at position
length 1 m is initially at a temperature of 10°C. It is then 2 until the gas is again at 0°C and then slowly raised
kept between 2 reservoirs of temperature 100°C and back to position 1. Represent the whole process on P-V
0°C. Specific heat capacity is 10 J/kg°C and linear mass diagram. If m = 100 gm of ice are melted during the
density is 2 kg/m. Find cycle, how much work is done on the gas, Latent heat
of ice = 80 cal/gm.
o o
100 C 0C
x 1

(a) Temperature gradient along the rod in steady state. 2

(b) Total heat absorbed by the rod to reach steady state.

Q.16 A cylindrical block of length 0.4 m and area of


cross-section 0.04m² is placed coaxially on a thin metal Q.21 A parallel beam of particles of mass m moving with
disc of mass 0.4 kg and of the same cross-section. The velocities v impinges on a wall at an angle θ to its normal.
upper face of the cylinder is maintained at a constant The number of particles per unit volume in the beam is
temperature of 400K and the initial temperature of the n. If the collision of particles with the wall is elastic, then
disc is 300K. If the thermal conductivity of the material of find the pressure exerted by this beam on the wall.
the cylinder is 10 watt/m-K and the specific heat of the
material of the disc in 600 J/kg-K, how long will it take for Q.22 For the thermodynamic process shown in the figure.
the temperature of the disc to increase to 350K? Assume, 5
for purposes of calculation, the thermal conductivity= of PA 1x10 = Pa;PB 0.3 x105 Pa
the disc to be very high and the system to be thermally= PD 0.6x10
= 5
Pa; VA 0.20litre VD = 1.30litre
insulated expect for the upper face of the cylinder.
P

Q.17 A liquid takes 5 minutes to cool from 80°C to PA A


50°C. How much time will it take to cool from 60°C
to 30°C? The temperature of surrounding is 20°C. Use PD D
exact method. PD B C

V
Q.18 An ideal gas at NTP is enclosed in an adiabatic VA VC VD
vertical cylinder having area of cross section A = 27 cm²,
(a) Find the work performed by the system along path AD.
between two light movable pistons as shown in the
figure. Spring with force constant k = 3700 N/m is in (b) If the total work done by the system along the path
a relaxed state initially. Now the lower piston is moved ADC is 85J find the volume at point C.
upwards a height h/2, h being the initial length of gas
(c) How much work is performed by the system along
column. It is observed that the upper piston moves up
the path CDA?
by a distance h/16. Find h taking γ for the gas to be 1.5.
Also find the final temperature of the gas.
1 3 . 4 2 | Kinetic Theor y of Gases and Thermodynamics

Exercise 2 mean square velocity of molecules of gas B is v 2 and


mean square of x component of velocity of molecules
Single Correct Choice Type of gas A is w2 . The ratio of w2 / v 2 is?
(A) 1 (B) 2 (C) 1/3 (D) 2/3
Q.1 A perfect gas of a given mass is heated first in
small vessel and then in a large vessel, such that their Q.7 A reversible adiabatic path on a P-V diagram for an
volumes remain unchanged. The P-T curves are ideal gas passes through state A where P=0.7x105 N/m-2
(A) Parabolic with same curvature and v = 0.0049m³. The ratio of specific heat of the gas
is 1.4. The slope of path at A is?
(B) Parabolic with different curvature
(A) 2.0 x107 Nm−5 (B) 1.0 x107 Nm−5
(C) Linear with same slopes
(C) −2.0 x107 Nm−5 (D) −1.0 x107 Nm−5
(D) Linear with different slopes
Q.8 A cylinder made of perfectly non conducting
Q.2 An open and wide glass tube is immersed vertically material closed at both ends is divided into two equal
in mercury in such a way that length 0.05 m extends parts by a heat proof piston. Both parts of the cylinder
above mercury level. The open end of the tube is closed contain the same masses of a gas at a temperature
and the tube is raised further by 0.43 m. The length of t0 = 27°C and pressure P0 =1 atm. Now if the gas in
air column above mercury level in the tube will be? Take one of the parts is slowly heated to t = 57°C while
Patm = 76cm of mercury. the temperature of first part is maintained at t0 the
(A) 0.215 m (B) 0.2 m (C) 0.1 m (D) 0.4 m distance moved by the piston from the middle of the
cylinder will be (length of the cylinder = 84cm)

Q.3 A container X has volume double that of container (A) 3cm (B) 5cm (C) 2cm (D) 1cm
Y and both are connected by a thin tube. Both contains
same ideal gas. The temperature of X is 200K and that Q.9 A vessel contains an ideal monoatomic gas which
of Y is 400 K. If mass of gas in X is m then Y it will be: expands at constant pressure, when heat Q is given to
(A) m/8 (B) m/6 (C) m/4 (D) m/2 it. Then the work done in expansion is:
3 2 2
(A) Q (B) Q (C) Q (D) Q
Q.4 An ideal gas of Molar mass M is contained in a 5 5 3
vertical tube of height H, closed at both ends. The tube
is accelerating vertically upwards with acceleration g. Multiple Correct Choice Type
Then, the ratio of pressure at the bottom and the mid
point of the tube will be Q.10 Two bodies A and B have thermal emissivities of
0.81 respectively. The outer surface areas of the two
(A) exp[2MgH/RT] (B) exp[-2MgH/RT]
bodies are the same. The two bodies radiate energy at
(C) exp[MgH/RT] (D) MgH/RT the same rate. The wavelength λB , corresponding to
the maximum spectral radiancy in the radiation from
Q.5 Two monoatomic ideal gas at temperature T1 and B, is shifted from the wavelength corresponding to the
T2 are mixed. There is no loss of energy. If the masses maximum spectral radiancy in the radiation from A by
of molecules of the two gases are m1 and m2 and 1.00 µm . If the temperature of A is 5802 K,
number of their molecules are n1 and n2 respectively. (A) The temperature of B is 1934K
The temperature of the mixture will be?
(B) λB= 1.5 µm
T1 + T2 T1 T2
(A) n + n (B) s + (C) The temperature of B is 11604 K
1 2 n1 n2
(D) The temperature of B is 2901 K
n2 T1 + n1T2 n1T1 + n2 T2
(C) (D)
n1 + n2 n1 + n2 Q.11 During an experiment, an ideal gas is found to
obey a condition VP² = constant. The gas is initially
Q.6 At temperature T, N molecules of gas A each having at a temperature T, pressure P and volume V. The gas
mass m and at the same temperature 2N molecules of expands to volume 4V.
gas B each having mass 2m are filled in a container. The
P hysi cs | 13.43

P Q.15 The figure shows a radiant energy spectrum graph


(A) The pressure of gas changes to
2 for a black body at a temperature T. Choose the correct
statement(s)
(B) The temperature of gas changes to 4T
dE
(C) The graph of above process on the P-T diagram is T
d
parabola
(D) The graph of above process on the P-T diagram is
hyperbola. O m 
(A) The radiant energy is not equally distributed among
Q.12 The total kinetic energy of translatory motion all the possible wavelengths
of all the molecules of 5 litres of nitrogen exerting a
(B) For a particular wavelength the spectral intensity is
pressure P is 3000 J.
maximum
(A) The total K.E. of 10 litres of N2 at a pressure of 2P is
(C) The area under the curve is equal to the total rate at
3000 J
which heat is radiated by the body at that temperature
(B) The total K.E. of 10 litres of He at a pressure of 2P
(D) None of these
is 3000 J
(C) The total K.E. of 10 litres of Q2 at a pressure of 2P is Q.16 Two metallic sphere A and B are mode of same
20000 J material and have got identical surface finish. The mass
(D) The total K.E. of 10 litres of Ne at a pressure of 2P of sphere A is four times that of B. Both the spheres are
is 12000 J heated to the same temperature and placed in a room
having lower temperature but thermally insulated from
each other.
Q.13 A container holds 1026 molecules/m³, each of mass
3 x 10-27 kg. Assume that 1/6 of the molecules move (A) The ratio of heat loss of A to that of B is 24/3.
with velocity 2000 m/s directly towards one wall of the
(B) The ratio of heat loss of A to that of B 22/3.
container while the remaining 5/6 of the molecules
move either away from the wall or in perpendicular (C) The ratio of the initial rate of cooling of A to that of
direction, and all collisions of the molecules with the B is 2-2/3.
wall are elastic (D) The ratio of the initial rate of cooling of A to that of
(A) Number of molecules hitting 1 m² of the wall every B is 2-4/3.
second is 3.33 x 1028.
(B) Number of molecules hitting 1 m² of the wall every Q.17 50 gm ice at = 10°C is mixed with 20 gm steam at
second is 2 x 1029. 100°C. When the mixture finally reaches its steady state
inside a calorimeter of water equivalent 1.5 gm then:
(C) Pressure exerted on the wall by molecules is [Assume calorimeter was initially at 0°C, take latent
24 x 105 Pa. heat of vaporization of water = 1 cal/gm-°C, specific
(D) Pressure exerted on the wall by molecules is heat capacity of ice = 0.5 cal/gm°C]
4 x 105 Pa. (A) Mass of water remaining is: 67.4 gm
(B) Mass of water remaining is: 67.87 gm
Q.14 Two gases have the same initial pressure, volume
and temperature. They expand to the same final volume, (C) Mass of steam remaining is: 2.6 gm
one adiabatically and the other isothermally (D) Mass of steam remaining is: 2.13 gm
(A) The final temperature is greater for the isothermal
process Q.18 A gas expands such that it is initial and final
(B) The final pressure is greater for the isothermal temperature are equal. Also, the process followed by
process the gas traces a straight line on the P-V diagram:

(C) The work done by the gas is greater for the (A) The temperature of the gas remains constant
isothermal process. throughout.

(D) All the above options are incorrect (B) The temperature of the gas first increases and then
decreases.
1 3 . 4 4 | Kinetic Theor y of Gases and Thermodynamics

(C) The temperature of the gas first decreases and then Q.21 If the temperature of the body is raised to higher
increases. temperature T’, then choose the correct statement(s)
(D) The straight line has a negative slopes (A) The intensity of radiation for every wavelength
increases
Q.19 A cyclic process ABCD is shown in the P-V (B) The maximum intensity occurs at a shorter
diagram. Which of the following curves represents the wavelength
same process if BC & DA are isothermal processes.
(C) The area under the graph increases
A B
(D) The area under the graph is proportional to the
fourth power of temperature
P
C
Paragraph 2:
D
V Two rods A and B of same cross-sectional area A
and length / connected in series between a source
A B D C (T1 =100°C) and a sink (T2 = 0°C) as shown in figure. The
rod is laterally insulated
(A) (B)
P C V T1 T2
B
D A o 3K K o
100 C 0C
T T  

B A B Q.22 The ratio of the thermal resistance of the rod is


A RA 1 R
(C) (D) (A) = (B) A = 3
P V RB 3 RB
C
D D C RA 3 4
(C) = (D)
T T RB 4 3

Q.23 If TA and TB are the temperature drops across the


Comprehension Type
rod A and B, then
Paragraph 1: TA T
3 1
(A) = (B) A =
TB 1 TB 3
dE T
TA 3 T 4
d (C) = (D) A =
TB 4 TB 3

O m 
Q.24 If GA and GB are the temperature gradients across
the rod A and B, then
Q.20 The figure shows a radiant energy spectrum graph
for a black body at a temperature T. GA 3 G 1
(A) = (B) A =
Choose the correct statement (s) GB 1 GB 3

(A) The radiant energy is not equally distributed among GA 3 G 4


(C) = (D) A =
all the possible wave lengths GB 4 GB 3
(B) For a particular wavelength the spectral intensity is
maximum Paragraph 3:
(C) The area under the curve is equal to the total rate at In fluids heat transfer tales place and molecules of
which heat is radiated by the body at that temperature. the medium takes very active part. The molecules
(D) None of these take energy from high temperature zone and move
towards low temperature zone. This method is known
P hysi cs | 13.45

as convection, when we require heat transfer with fast Q.29 The work done in the complete cycle ABCA is
phase, we use some mechanism to make the flow of fluid
on the body fast. The rate of loss of heat is proportional (A) 90 J (B) 60 J (C) 0 J (D) 30 J
to velocity of fluid (v), and temperature difference ( ∆ T)
between the body any fluid, of course more the surface
area of body more the rate of loss of heat. We can write Paragraph 5:
dQ Five moles of helium are mixed with two moles of
the rate of loss of heat as = KAv∆T
dt hydrogen to form a mixture. Take molar mass of helium
M1 = 4g and that of hydrogen M2 = 2g
Where K is positive constant.
Now answer the following questions
Q.30 The equivalent molar mass of the mixture is
13g
Q.25 A body is being cooled with fluid. When we (A) 6g (B)
increase the velocity of fluid 4 times and decrease the 7
temperature difference 1/2 time, the rate of loss of heat 18g
(C) (D) None
increases. 7
(A) Four times (B) Two times
Q.31 The equivalent degree of freedom f of the mixture
(C) Six times (D) No change is
(A) 3.57 (B) 1.14 (C) 4.4 (D) None
Q.26 In the above question if mass of the body
increased two times, without change in any of the other
parameters, the rate of cooling Q.32 The equivalent value of γ is

(A) Decreases (A) 1.59 (B) 1.53 (C) 1.56 (D) None

(B) Increases
Q.33 If the internal energy of He sample of 100J and
(C) No effect of change of mass that of the hydrogen sample is 200J, then the internal
(D) None of these energy of the mixture is
(A) 900J (B) 128.5J
Paragraph 4: (C) 171.4J (D) 300J
When a thermo-dynamic process is shown on P-V
diagram, area under curve represents work done Match the Column
during process. During cyclic process work done is
area enclosed. The P-V graph for a thermodynamically Q.34 An ideal gas at pressure P and volume V is
system is shown in figure. expanded to volume 2V. Column I represents the
thermodynamic processes used during expansion.
20 A column II represents the work during these processes
Pressure P (in Nm )
-2

in the random order.

10
B Column I Column II
C

(p) isobaric PV(1 − 21−γ )


(x)
6 12 γ −1
3
Volume V (in m )
(q) isothermal (y) PV
Q.27 The work done by the system in the process A to
B is (r) adiabatic (z) PV/n 2

(A) 90 J (B) 60 J (C) 0 J (D) 30J The correct matching of column I and column II is given
by:
Q.28 The work done in the process B to C is
(A) p-y, q-z, r-x (B) p-y, q-x, r-z
(A) -90 J (B) -60 J (C) 0 J (D) -30J
(C) p-x, q-y, r-z (D) p-z, q-y, r-x
1 3 . 4 6 | Kinetic Theor y of Gases and Thermodynamics

Previous Years’ Questions  


 
Q.1 When the ideal diatomic gas is heated at constant
pressure the fraction of the heat energy supplied which
increases the internal energy of the gas is (1990)
2 3 3 5 T T
(A) (B) (C) (D) T T+T T T+T
5 5 7 7 T T
 T T+T  T T+T

Q.2 A vessel contains a mixture of one mole of oxygen  


and two moles of nitrogen at 300 K. The ratio of the
average rotational kinetic energy per O2 molecule to
per N2 molecule is (1998)
T T
(A) 1: 1 T T+T T T+T
T T
(B) 1: 2 T T+T T T+T
(C) 2: 1
(D) Depends on the moment of inertia of the two Q.7 P-V plots for two gases during adiabatic processes
molecules are shown in the figure. Plots 1 and 2 should correspond
respectively to  (2001)
Q.3 Two cylinders A and B fitted with pistons contain P
equal amounts of an ideal diatomic gas at 300 K. The
piston of A is free to move, while that of B is held fixed.
The same amount of heat is given to the gas in each
1
cylinder. If the rise in temperature of the gas in A is 30 K,
then the rise in temperature of the gas in B is  (1998) 2
V
(A) 30 K (B) 18 K (C) 50 K (D) 42 K
(A) He and O2 (B) O2and He
Q.4 The ratio of the speed of sound in nitrogen gas to
(C) He and Ar (D) O2and N2
that in helium gas, at 300 K is (1999)

(A) (2 / 7) (B) (1/ 7) Q.8 An ideal gas is taken through the cycle
A → B → C → A, as shown in the figure. If the net heat
(C) ( 3) / 5 (D) ( 6) / 5 supplied to the gas in the cycle is 5J, the work done by
the gas in the process C → A is (2002)
Q.5 Starting with the same initial conditions, and ideal 3
V(m )
gas expands from volume V1 to V2 in three different
ways. The work done by the gas is W1 if the process 2 C B
is purely isothermal, W2 if purely isobaric and W3 if
purely adiabatic, then (2000)
(A) W2 > W1 > W3 ; (B) W2 > W3 > W1 1 A

(C) W1 > W2 > W3 ; (D) W1 > W3 > W2


10 p(N/m )
2

Q. 6 An ideal gas is initially at temperature T and volume


V. Its volume is increased by ∆V due to an increase (A) -5J (B) -10 J (C) -15 J (D) -20 J
in temperature ∆T , pressure remaining constant. The
quantity= δ V / V∆T varies with temperature as ( 2000)
Q.9 Liquid oxygen at 50 K is heated to 300 K at constant
pressure of 1 atm. The rate of heating is constant.
Which of the following graphs represent the variation
of temperature with time? (2004)
P hysi cs | 13.47

Temp
Temp Temp
Temp Q.12 The piston is now pulled out slowly and held at a
distance 2L from the top. The pressure in the cylinder
between its top and the piston will then be (2007)
(A) (B)
(A) (B)
Time P0
Time
Time Time (A) P0 (B)
2
P0 Mg P0 Mg
Temp Temp (C) + (D) −
Temp Temp 2 πR 2 2 πR 2

(C) (D) Q.13 Whilethe piston is at a distance 2L from the top,


(C) (D)
Time Time
Time the hole at the top is sealed. The piston is then released,
Time
to a position where it can stay in equilibrium. In this
condition, the distance of the piston from the top is
 (2007)
Q.10 An ideal gas expands isothermally from a volume  2P πR 2   P πR 2 − Mg 
V1 to V2 and then compressed to original volume V1 (A)  0  (2L) (B)  0  (2L)
 πR 2P + Mg   πR 2P 
adiabatically. Initial pressure is P1 and final pressure is  0   0 
P3 . The total work done is W. Then, (2004)
 P πR 2 + Mg   P πR 2 
(A) P3 > P1 , W > 0 (B=
) P3 P1 , W > 0 (C)  0  (2L) (D)  0  (2L)
 πR 2P   πR 2P − Mg 
(C) P3 > P1 , W < 0 (D=
) P3 P1=
,W 0  0   0 

Q.14 The piston is take completely out of the cylinder.


Q.11 Statement-I The total translational kinetic energy The hole at the top is sealed. A water tank is brought
of all the molecules of a given mass of an ideal gas is below the cylinder and put in a position so that the
1.5 times the product of its pressure and its volume water surface in the tank is at the same level as the top
Statement-II The molecules of a gas collide with each of the cylinder as shown in the figure. The density of
other and the velocities of the molecules change due to the water is ρ . In equilibrium, the height H of the water
the collision. (2007) column in the cylinder satisfies  (2007)

(A) If statement-I is true, statement-II is true;


statement-II is the correct explanation for statement-I
(B) If statement-I is true, statement-II is true;
statement-II is not a correct explanation for statement-I
(C) If Statement-I is true; statement true is false L0
(D) If Statement-I is false; statement-II is true
H
Paragraph 1:
A fixed thermally conducting cylinder has a radius R
and height L0 . The cylinder is open at its bottom and (A) ρg(L0 − H)2 + p0 (L0 − H) + L0 p0 =
0
has a small hole at its top. A piston of mass M is held at
(B) ρg(L0 − H)2 − p0 (L0 − H) − L0 p0 =
0
a distance L from the top surface as shown in the figure.
The atmospheric pressure is P0 . (C) ρg(L0 − H)2 + p0 (L0 − H) − L0 p0 =
0
2R
(D) ρg(L0 − H)2 − p0 (L0 − H) + L0 p0 =
0
L

Q.15 C v and Cp denote the molar specific heat


L0
capacities of a gas at constant volume and constant
pressure, respectively. Then,  (2009)
(A) CP − CV is larger for a diatomic ideal gas than for a
Piston monoatomic ideal gas
1 3 . 4 8 | Kinetic Theor y of Gases and Thermodynamics

(B) CP + CV is larger for a diatomic ideal gas than for a


Column I Column II
monoatomic ideal gas
C (B) An ideal monoatomic gas (p) The temperature
(C) P is larger for a diatomic ideal gas than for a expands to twice its original of the gas increases
CV
volume such that its pressure or remains constant
monoatomic ideal gas
1
(D) CP *CV is larger for a diatomic ideal gas than for a p∝ , where V is the volume
V2
monoatomic ideal gas
of the gas.
(C) An ideal monoatomic gas (q) The gas loses
Q.16 The figure shows the P-V plot an ideal gas taken
expands to twice its original heat
through a cycle ABCDA. The part ABC is a semi-circle
volume such that its pressure
and CDA is half of an ellipse. Then,  (2009)
1
p∝ , where V is its volume.
4/3
P V
A
3
(D) An ideal monoatomic gas (r) The gas gains
2 D expands such that its pressure heat
B
p and volume V follows the
1
C behaviour shown in the graph.

0 1 2 3 V P

(A) The process during the path A → B is isothermal


(B) Heat flows out of the gas during the path B → C → D
(C) Work done during the path A → B → C is zero
V
V1 2V1
(D) Positive work is done by the gas in the cycle ABCDA

Match the Columns


Q.18 A real gas behaves like an ideal gas if its (2010)
Q.17 Column I contains a list of processes involving (A) Pressure and temperature are both high
expansion of an ideal gas. Match this with column II
(B) Pressure and temperature are both low
describing the thermodynamic change during this
process. Indicate your answer by darkening the (C) Pressure is high and temperature is low
appropriate bubbles of the 4 x 4 matrix given in the (D) Pressure is low and temperature is high
ORS.  (2008)

Q.19 One mole of an ideal gas in initial state A


Column I Column II
undergoes a cyclic process ABCA, as shown in the figure.
(A) An insulated container has two (p) The temperature Its pressure at A is P0. Choose the correct option(s) from
chambers separated by a valve. of the gas decreases the following: (2010)
Chamber I contains an ideal gas V
and the Chamber II has vacuum.
The valve is opened.
4V0 B

I II A
Ideal gas Vacuum V0 C
T
T0

(A) Internal energies at Aand B are the same


(B) Work done by the gas in process AB is P0V0 ln 4
P0
(C) Pressure at C is
4
T0
(D) Temperature at C is
4
P hysi cs | 13.49

Q.20 5.6 liter of helium gas at STP is adiabatically Q.25 A water cooler of storage capacity 120 litres can
compressed to 0.7 liter. Taking the initial temperature cool water at a constant rate of P watts. In a closed
to be T1, the work done in the process is (2011) circulation system (as shown schematically in the
figure), the water from the cooler is used to cool an
9 3 15 9
(A) RT (B) RT (C) RT (D) RT external device that generates constantly 3 kW of heat
8 1 2 1 8 1 2 1 (thermal load). The temperature of water fed into the
device cannot exceed 30°C and the entire stored 120
Q.21 A mixture of 2 moles of helium gas (atomic litres of water is initially cooled to 10°C. The entire
mass = 4 amu) and 1 mole of argon gas (atomic mass system is thermally insulated. The minimum value of
= 40 amu) is kept at 300 K in a container. The ratio of P (in watts) for which the device can be operated for 3
 V (helium)  hours is
the rms speeds  rms  is (2012) Cooler Device
 Vrms (argon)  Hot

(A) 0.32 (B) 0.45 (C) 2.24 (D) 3.16

Q.22 Two non-reactive monoatomic ideal gases have their Cold


atomic masses in the ratio 2 : 3. The ratio of their partial
pressures, when enclosed in a vessel kept at a constant (Specific heat of water is 4.2 kJ kg-1 K-1 and the density
temperature, is 4 : 3. The ratio of their densities is (2013) of water is 1000 kg m-3) (2016)
(A) 1 : 4 (B) 1 : 2 (C) 6 : 9 (D) 8 : 9 (A) 1600 (B) 2067 (C) 2533 (D) 3933

Q.23 A thermodynamic system is taken from an initial Q.26 A cylindrical vessel of height 500 mm has an orifice
state i with internal energy Ui = 100 J to the final state (small hole) at its bottom. The orifice is initially closed
and water is filled in it upto height H. Now the top is
f along two different paths iaf and ibf, as schematically
completely sealed with a cap and the orifice at the bottom
shown in the figure. The work done by the system along is opened. Some water comes out from the orifice and
the paths af, ib and bf are Waf = 200 J, Wib = 50 J and Wbf = the water level in the vessel becomes steady with height
100 J respectively. The heat supplied to the system along of water column being 200 mm. Find the fall in height (in
the path iaf, ib and bf are Qiaf, Qib and Qbf respectively. mm) of water level due to opening of the orifice.
If the internal energy of the system in the state b is [Take atmospheric pressure = 1.0 × 105 N/m2, density of
Ub = 200 J and Qiaf = 500 J, the ratio Qbf / Qib is (2014) water = 1000 kg/m3 and g = 10 m/s2. Neglect any effect
of surface tension.]  (2009)
a
f
Q.27 A diatomic ideal gas is compressed adiabatically
P
1
to of its initial volume. If the initial temperature of
32
i b the gas is Ti (in Kelvin) and the final temperature is aTi,
v the value of a is (2010)

Q.24 A container of fixed volume has a mixture of Q.28 One mole of a monatomic gas is taken through
one mole of hydrogen and one mole of helium in a cycle ABCDA as shown in the P-V diagram. column
equilibrium at temperature T. Assuming the gases are II give the characteristics involved in the cycle. Match
ideal, the correct statement(s) is(are) (2015)
them with each of the processes given in column I.
(A) The average energy per mole of the gas mixture is 2RT.  (2011)
P
(B) The ratio of speed of sound in the gas mixture to B A
3P
that in helium gas is.
(C) The ratio of the rms speed of helium atoms to that
of hydrogen molecules is 1/2. 1P C D
(D) The ratio of the rms speed of helium atoms to that
of hydrogen molecules is. 0 1V 3V 9V V
1 3 . 5 0 | Kinetic Theor y of Gases and Thermodynamics

Q.30 If the piston is pushed at a speed of 5 mms–1, the


Column I Column II
air comes out of the nozzle with a speed of
(A) Process A → B (p) Internal energy decreases
(A) 0.1 ms–1 (B) 1 ms–1 (C) 2 ms–1 (D) 8 ms–1
(B) Process B → C (q) Internal energy increases.
Q.31 If the density of air is ρa and that of the liquid ρ ,
(C) Process C → D (r) Heat is lost
then for a given piston speed the rate (volume per unit
(s) Heat is gained time) at which the liquid is sprayed will be proportional
(D) Process D → A
to
(t) Work is done on the gas
ρa ρ
(A) (B) ρaρ (C) (D) ρ
ρ ρa
Q.29 The figure shows the variation of specific heat
capacity (C) of a solid as a function of temperature Q.32 An ideal monoatomic gas is confined in a horizontal
(T). The temperature is increased continuously from cylinder by a spring loaded piston (as shown in the
0 to 500 K at a constant rate. Ignoring any volume figure). Initially the gas is at temperature T1, pressure P1
change, the following statement(s) is (are) correct to a and volume V1 and the spring is in its relaxed state.
reasonable approximation. (2013)

C
The gas is then heated very slowly to temperature
T2, pressure P2 and volume V2. During this process
the piston moves out by a distance x. Ignoring the
friction between the piston and the cylinder, the correct
100 200 300 400 500 statement(s) is(are)  (2015)
T(K)
(A) The rate at which heat is absorbed in the range (A) If V2 = 2V1 and T2 = 3T1, then the energy stored in
0-100 K varies linearly with temperature T. 1
the spring is P1 V1
(B) Heat absorbed in increasing the temperature from 4
0-100 K is less than the heat required for increasing the (B) If V2 = 2V1 and T2 = 3T1, then the change in internal
temperature from 400 – 500 K. energy is 3P1V1
(C) There is no change in the rate of heat absorption in
(C) If V2 = 3V1 and T2 = 4T1, then the work done by the
range 400 – 500 K.
7
(D) The rate of heat absorption increases in the range gas is P V
3 1 1
200 – 300 K.
(D) If V2 = 3V1 and T2 = 4T1, then the heat supplied to
17
Paragraph 1: the gas is P V
6 1 1
A spray gun is shown in the figure where a piston
pushes air out of a nozzle. A thin tube of uniform cross Q.33 A gas is enclosed in a cylinder with a movable
section is connected to the nozzle. The other end of the frictionless piston. Its initial thermodynamic state at
tube is in a small liquid container. As the piston pushes pressure Pi = 105 Pa and volume Vi = 10-3 m3 changes
air through the nozzle, the liquid from the container to a final state at Pf = (1/32) × 105 Pa and Vf = 8 × 10-3
rises into the nozzle and is sprayed out. For the spray m3 in an adiabatic quasi-static process, such that P3V5
gun shown, the radii of the piston and the nozzle are = constant. Consider another thermodynamic process
20 mm and 1 mm respectively. The upper end of the that brings the system from the same initial state to the
container is open to the atmosphere. (2014) same final state in two steps: an isobaric expansion at
Pi followed by an isochoric (isovolumetric) process at
volume Vf. The amount of heat supplied to the system
in the two-step process is approximately (2016)
(A) 112 J (B) 294 J (C) 588 J (D) 813 J
P hysi cs | 13.51

PlancEssential Questions
JEE Main/Boards JEE Advanced/Boards
Exercise 1 Exercise 1
Q. 11 Q.20 Q.32 Q.12 Q.15 Q.18
Q.33 Q.34 Q.35 Q.19 Q.22
Q.36

Exercise 2 Exercise 2
Q.4 Q.5 Q.15 Q.8 Q.10 Q.11
Q.16 Q.17 Q.20
Q.28 Q.29 Q.30

Answer Key

JEE Main/Boards
Exercise 1

2x(273 − 173)
Q.2 P1 / T1 P2=
= / T2 or T2 P2 T1 / P1 = 200K =
= − 73°C
1
Q.10 6 x 10-4 Q.11 0.103 kg Q.13 1.88 x 109 s-1

Q.14 1.87 x 10-7 m Q.15 1.47 x 10-7 m

Q.17 No, because internal energy of an ideal gas depends only on temperature of the gas
∆V1
Q.18 Translational motion of molecules. Q.19 =γ. As γ > 1, ∴( ∆ V1 ) > ( ∆ V2 )
∆V2
Q.20 Internal energy increasing by 200 J. Heat absorbed is zero.

Q.32 80.8°C Q.33 (i) 2.675 atm (ii) 128.3°C

Q.34 -1176 joule, -280 cal Q.35 – 1374 J Q.36 – 43.38 J


1 3 . 5 2 | Kinetic Theor y of Gases and Thermodynamics

Exercise 2
Single Correct Choice Type
Q.1 A Q.2 A Q.3 C Q.4 B Q.5 A Q.6 B
Q.7 A Q.8 C Q.9 D Q.10 D Q.11 C Q.12 A
Q.13 B Q.14 A Q.15 D Q.16 A

Previous Years’ Questions


Q.1 A Q.2 A Q.3 B Q4 B Q.5 B Q.6 D

Q.7 C Q.8 C Q.9 D Q.10 D Q.11 C

Q.12 A → s; B → r, p; C → p; D → s Q.13 A → p; r, t; B → p, r; C → q, s; D → r, t Q.14 A, B, C, D

Q.15 A, B Q.16 D Q.17 A Q.18 B Q.19 C Q.20 A

Q.21 B Q.22 B Q. 23 C Q.24 D Q.25 A Q.26 C

Q.27 C Q.28 D Q.29 C Q.30 B Q.31 C Q.32 D

Q.33 A

JEE Advanced/Boards
Exercise 1

Q.1 4:1 Q.2 1.25 × 104 Pa

mRT0  T0m 
Q.3 1 + R Q.4 The molar mass of the gas is 40 gm,
V0  V0 

Q.5 100J Q.6 3600 R
Q.7 400 K Q.8 5R
R R
Q.9 CV + Q.10
αV 2
P  1 
Isobatic

3 1 − 1/3 
 γ + 1  γ + 1 R 2 
Q.12 1 − 
Adiabatic 2
Q.11 (i)   4aV0 , (ii)  
 γ − 1   γ − 1 2
 ln2
Isotherm
V

a2s b  T −T  l1 k(T1 − Tm )


Q.13 loge   loge  0 1  Q.14 =
2K T −T 
a  0 2
l k(T1 − Tm ) + (Tm − T2 )

Q.15 (a) -100°C/m, (b) 1000 J Q.16 166.3 sec

Q.17 10 minutes Q.18 1.6 m, 364 K

Q.19 1.63 Q.20 8000 cal.

Q.21 2mv2 cos2 θ (a) WAD 88


Q.22= = J, (b) VC 1.23litre,
(c) WCDA = − 85 J
P hysi cs | 13.53

Exercise 2

Single Correct Choice Type

Q.1 D Q.2 C Q.3 C Q.4 C Q.5 D Q.6 D


Q.7 C Q.8 C Q.9 C

Multiple Correct Choice Type

Q.10 A, B Q.11 A, D Q.12 C, D Q.13 A, D Q.14 A, B, C Q.15 A, B


Q.16 A, C Q.17 A, C Q.18 B, D Q.19 A, B

Comprehension Type

Paragraph 1: Q.20 A, B Q.21 A, B, C, D Paragraph 2: Q.22 A Q.23 B Q.24 B

Paragraph 3: Q.25 B Q.26 A Paragraph 4: Q.27 A Q.28 B Q.29 D

Paragraph 5: Q.30 D Q.31 A Q.32 C Q.33 D

Match the Column

Q.34 A

Previous Years’ Questions


Q.1 D Q.2 A Q.3 D Q.4 C Q.5 A Q.6 C
Q.7 B Q.8 A Q.9 C Q.10 C Q.11 B Q.12 A
Q.13 D Q.14 C Q.15 B, D Q.16 B, D
Q.17 A → q; B → p, r; C → p, s; D → q, s Q.18 D Q.19 A, B Q.20 A
Q.21 D Q.22 D Q.23 2 Q.24 A, B, D Q.25 B Q.26 6
Q.27 4 Q.28 A → p, r, t; B → p, r; C → q, s; D → r, t Q.29 A, B, C, D Q.30 C
Q.31 A Q.32 B or A, B, C Q.33 C

Solutions

JEE Main/Boards aren’t. So, the molecules are not going in straight line.
Thus, it takes time for the smell to spread in the room.
Exercise 1
Sol 2: Initial
Sol 1: The speed of molecules follows directly from Pi = 1 atm
measuring the pressure and density-you don’t need to
know the size of molecules. In standard kinetic theory, T0 = -173°C = 100 k
collisions with other molecules have always been Vi = V
ignored, because the molecules were tiny. Though, they
1 3 . 5 4 | Kinetic Theor y of Gases and Thermodynamics

Finally 3. Attraction or repulsion forces between any two


particles are negligible
Pf = 2 atm
4. The collisions between gas molecules or molecule
Tf = ?
and wall of container are completely elastic meaning
Vf = V = const. no energy is gained or lost from collisions.
By ideal gas law 5. The time it takes to collide is negligible
PV = nRT 6. All gases at a given temperature have same Kinetic
T Energy.
⇒V= = constant
P 7. Motion of particles is random
Ti Tf 8. Effect of gravity on gas molecule is neglected
∴ =
Pi Pf 9. Average momentum of gas molecule is zero.
Ti × Pf 100 × 2 Expression for the pressure and vrms of a gas-
⇒ Tf = =
Pi 1
v 2 = v 2x + v 2y + v 2z
= 200 K = -73°C
By postulates of KTG we know
<vx2> = <vy2> = <vz2>
Sol 3: Boyle’s law states that absolute pressure exerted
by a given mass of an ideal gas is inversely proportional {<> means average}
to the volume it occupies if the temperature and amount
∴ <v2> = 3 <vx2>
of gas remains unchanged within a closed system.
Since collision is elastic
1
i.e. P ∝
V
[Keeping temp. and amt. of gas fixed]
Charles’s law m:molecular Vy
mass
Law of volumes: - V ∝ T Vz Vx
VX :velocity of
[Given pressure and amount of gas remains constant] molecule in
These laws are not applicable to real gases since in real x direction
gases size of particles is not negligible, collisions of 
particles are not elastic and there are attractive forces
between particles. Change in momentum in one collision
(DP) = 2m vx
Sol 4: (i) Guy Isaac’s law: - The pressure of a gas of fixed 2
mass and fixed volume is directly proportional to the Time interval between collision (Dt) =
vx
gas absolute temperature.
i.e. P ∝ T. ∆P mv x 2
∴ Force by one molecule = =
∆t 
[Keeping volume and amount of gas fixed]
P m  2
⇒ = constant ∴ Total force = v + v 2x2 + ......v 2xn 
T   x1 

(ii) Ideal gas equation: - PV = nRT [Assuming n molecules in the vessel]


R → Gas constant
 2 2 2 
mn  v x1 + v x2 + ......v xn  M
r → Radius of gas molecule = = < v12 >
 n 

Sol 5: Postulates of Kinetic Theory Gases (KTG): M = mn : - Total mass


1. Gases consist of particles in random motion. M
⇒ Total force = <v2>
2. Volume of molecule is negligible. 3
P hysi cs | 13.55

Force Sol 8: Mean free path is the average distance travelled


Pressure (P) =
Area by the molecules between successive collisions.

Assume vessel to be a cube.


Sol 9: Brownian motion is the random motion of
1M particles in a fluid. Resulting from their collision with
< v2 >
3  1 M the other atoms or molecules in the fluid.
∴P= = <v2>
× 3 3
m
1 Sol 10: Volume of m kg of molecules =
⇒ P = ρv2 rms 1000
3
m
[Since volume = l3 ∴ (M/l3) = ρ and vrms = < v2 > Volume occupied by water vapour =
0.6
m
Sol 6: Expression for pressure has been found in
∴ Required ratio = 1000 = 6 × 10-4
previous question. m
1 0.6
i.e. P = ρv2rms
3 Note: Here water is assumed to be completely compact.
Therefore, by using density of water we calculate
1 M 2 molecular volume.
P=   v
3  V  rms

1 Sol 11: V = 3000 cm3 = 3000 × 10-6 m3


⇒ PV = n Mm v2rms
3 T = 20° C = 293 k
[Mm: molecular weight] P = 2.5 × 106 Pa + 105 Pa = 2.6 × 106 Pa
Using gas equation we get PV 2.6 × 106 × 3000 × 10−6
n= =
1 RT 8.31× 293
PV = n Mm Vrms2
3 _ 3.20 c/moles

1 3
⇒ Mm v2rms = PVm ; Vm is molar volume ∴ Mass of O2 in tank
2 2
= 3.204 × 32 _ 102.5 gm = _ 0.103 kg
L.H.S. is the expression for kinetic energy per mole of
gas.
3RT 3 × 8.314 × 273
Sol 12: Vrms = =
Mm 64 × 10−3
Sol 7: We know that molar kinetic energy
_ 326.18 m/s
3 3
= PVm = RT [From previous question]
2 2
1 3 Sol 13: Average time per collision
∴ Kinetic energy per molecule = [ RT]
NA 2 2.66 × 10−7 m
= = 5.32 × 10-10 sec
1 3 3 500m sec −1
= × KNA T = KT
NA 2 2
∴ No. of collision per second
{NA: Avogadro’s number
1
K: Boltzmann constant} = _ 1.88 × 109 sec
5.32 × 10−10 
1 3
⇒ m v2rms = kT [m: molecular mass] 1
2 2 Sol 14: Mean free path estimate =
2πd2nv
mv rms2 2
∴T= = × kinetic energy per molecule.
3k 3k d: diameter of molecule, nv molecules per unit volume
∴ The average energy of the molecule is proportional to nV = 3 × 1019 cm-3 = 3 × 1025 m-3
absolute temperature. Absolute zero is the temperature
d = 2 × 10-10 m
at which the kinetic energy of the molecules becomes
zero. i.e. they stop.
1 3 . 5 6 | Kinetic Theor y of Gases and Thermodynamics

∴ Mean free path ∆V2 1  −V 


⇒ =+  …….(ii)
1 ∆P γ  P 
=
1.414 × 3.14 × (2 × 10−10 )2 × 3 × 1025 By (i) and (ii) we get

= 0.0187 × 10-5 m ∆V2 1  ∆V1 


=  
_ 1.87 × 10-7 m ∆P γ  ∆P 
∆V1
1 ⇒ =γ
Sol 15: Mean free path (x) = ∆V2
2
2πd nv
nNA As r > 1
nv =
V ∴ DV1 > DV2
n: number of moles in volume V
NA = Avogadro’s number = 6.02 × 1023 Sol 20: ∆W = -200 J
n P DQ = 0 [adiabatic conditions]
By gas equation: - =
V RT By 1st law of thermodynamics:-
PNA
∴ nv = DQ = DU + DW
RT
⇒ DU = + 200 J
RT
∴ x =
2πd2PNA
Sol 21: The internal energy is the total energy contained
8.314 × 273
= by a thermodynamics system. It is the energy needed
1.414 × 3.14 × (2.4 × 10−10 )2 × 105 × 6.02 × 1023 to create the system but excludes the energy due to
_ 1.47 × 10-7 m external force fields. E.g. Kinetic, energy if the sample
is moving or potential energy if the sample is at a
height from the ground. Internal energy has two major
Sol 16: Water has higher potential energy. As we already components i.e. kinetic and internal potential energy.
know that kinetic energy depends on temperature. So,
as we cool water to ice its K.E. remains constant at 0°C.
Therefore, the heat energy removed from water, is Sol 22: Four thermodynamics process are:-
accounted as decrease in potential energy of ice. (i) Isothermal process: Temperature remains constant.
i.e. PV = constant [∵ DT = 0]
Sol 17: We know that internal energy is a state function ∴ DU = nCV DT = 0
and depends on temperature.
(ii) Adiabatic process:- No flow of heat either into or out
∴ DU = 0 → No change from the system.
i.e. DQ = 0 = DU + W
Sol 18: Translational motion of molecules.
∴ DU = -W
Sol 19: For isothermal (iii) Isochoric process:- Volume of the system remains
constant.
PV = constant
i.e. V = constant
⇒ PDV1 + VDP = 0
⇒ DV = 0
∆V1−V
⇒ =–  …...(i) ∴ W = ∫ PdV = 0
∆P P
Thus, Dθ = DU
For adiabatic process
(iv) Isobaric process:- Pressure remains constant
PV γ = constant
i.e. P = constant
⇒ V γ ∆P + PγV γ−1∆V2 = 0
DP = 0
W = P (Vf – Vi)
P hysi cs | 13.57

Indicator diagram: are graphical representation of cyclic nRT


We know, PV = nRT ⇒ P =
variations of pressure and volume within a heat engine. V
Vf
nRTdV V V
Sol 23: Sign Convention ∴ DW = ∫ = nRT ln V f = nRT ln f
Vi
V Vi Vi
DQ → +ve for heat supplied to gas
Vf Pi
→ -ve for heat rejected by gas ⇒ DW = nRT ln = nRT ln
Vi Pf
DW→+ve for work done by gas
→ -ve for work done on gas Sol 28: For adiabatic process
DQ = 0
Sol 24: Isothermal process ∴ DU = -DW
DT = 0 ∴ nCVdT = -PdV  …(i)
∴ DU = nCv DT = 0 By equation of gas:- PV = nRT
By 1 law of thermodynamics:-
st
⇒ PdV + VdP = nRdT
∴ DQ = DU + DW ⇒ DQ = DW PdV + VdP
∴ ndT =
No change in internal energy R
Work done by system = heat supplied to the system. ∴ Substituting in (i) we get :-

Adiabatic Process:-  PdV + VdP 


CV   = -PdV
DQ = 0  R 
So, DQ = DU + DW ⇒ (CV+R) PdV = -CV VdP
⇒ DU = -DW
CP dV −dP
⇒ = [∵ CV + R = CP]
No heat is supplied to the system CV V P
Change in internal energy = Work done on gas dV −dP
⇒γ =
V P
Sol 25: Isothermal Operation: Operations/ process in
On integrating both sides, we get
which temperature stays constant.
Vf f P
Example:- System immersed in a large constant dV dP V P
γ ∫ = −∫ ⇒ γ ln f = ln i
temperature ice-water bath. Vi
V P
P Vi Pf
i
γ
Sol 26: Adiabatic operations are those operations in Pi V  γ
⇒ =  f  ⇒ PV
i i = Pf Vfγ = k (say)
which neither heat enters or leaves a system. Pf  Vi 
Example:- A system having a perfectly non-conducting k
boundary. ∴ PV γ = const. = k ⇒ P =

Q = DU + W Vf Vf
k
∴W= ∫ PdV = ∫ Vγ dV
∵Q=0
Vi Vi
∴ DU + W = 0 ⇒ DU = -W
Vf
V −γ+1 kVf1−γ − KVi1−γ
⇒W=k =
Sol 27: DT = 0 −γ + 1 1− γ
Vi
DU = nCVDT = 0
∴ DW = DQ - DU ⇒ DW = DQ (Pf Vf γ )Vf1−γ − (PV γ
i i )Vi
1−γ
=
1− γ
Vf

DW = ∫ PdV Pf Vf − PV
i i nR
Vi
⇒W= = (T – Ti)
1− γ 1− γ f
1 3 . 5 8 | Kinetic Theor y of Gases and Thermodynamics

Sol 29: Cyclic processes:- In cyclic process, thermo- w


dynamic process start from the same state and end at
the initial state. T1 T2

Hot Q1 Q2 Cold
A Working
P (source) substance (sink)

Here, Q1 is the heat given to the substance whereas Q2


B is the heat released.

V ∴ Qgiven = Q1

Since, initial and final states are same, W Q − Q2 Q


∴ η = = 1 =1- 2
Q1 Q⊥ Q1
Thus DU = 0,
∴ DQ = DW Heat released
∴ Efficiency ( η ) = 1 -
Heat absorbed
∴ |DQ| = |DW| = Area enclosed by the curve.
Sign of DW is ‘+’ if process is clockwise and ‘–‘ if process Two types of heat engines:-
in anti-clockwise.
(i) Internal combustion engines
Non-cyclic process: If initial and final states are not
(ii) External combustion engines
same.
A
P Sol 32: For adiabatic process
PV γ = constant
B
nRT
We know V =
P
V γ
 nRT 
B ∴P   = const.
DW = ∫ PdV = Area enclosed by P-V curve  P 
A
⇒ P1−γ T γ = const.
Sol 30: Reversible process: - The process in which the Pi = 3 atm; Pf = 1 atm
system and surroundings can be restored to the initial
state from the final state without producing any change Ti = 300 K; Tf = ?
in the thermodynamic properties of the universe.
Pi1−γ Tiγ
Ex: - An infinitesimal compression of a gas in a cylinder. ∴ Tfγ =
Pf1−γ
Irreversible process:- In irreversible process system is
1−1.4
not in equilibrium throughout the process. Initial state 3
⇒ Tfγ =   (300)1.4
can’t be obtained from final state without producing  1
changes in the universe.
−0.4
Example:- (i) Processes having friction. ⇒ Tf = 3 1.4 × 300 _ 219.2 K
(ii) Heat transfer through a finite temperature difference.
∴ Fall in temperature = Ti – Tf = 80.8° C

Sol 31: Heat engine is a device which convert heat


energy into mechanical energy by using a cyclic process. Sol 33: Ti = 27°C = 300 K; Tf = ?

W = Q1 - Q2 Pi = 1 atm; Pf = ?

W Vi = V; Vf = V/2
Efficiency: η =
Q given (i) PV γ = constant
γ
V 
∴ Pf =  i  Pi = (2)1.42 × 1 _ 2.675 atm
 Vf 
P hysi cs | 13.59

(ii) PV γ = constant  1
1.4 −1
=   × 273 = 206.89 K
nRT γ 2
⇒ V = constant
V
nR
⇒ TV γ−1 = constant ∴ DU = (T – Ti)
1− γ f
γ−1
V 
∴ Tf =  i  Ti  1 
−   × (2 × 4.2)
 Vf  32
=   (206.89 - 273)
= (2)0.42 × 300 1 − 1.4

⇒ T ≈ 128.38 °C  −43.38 Joule

Sol 34: n = 1
Ti = T; Tf = (T + 70) K
Exercise 2
nR 1× 2
W= [T – Ti] = [70] = -280 Cal
1− γ f 1 − 1.5
Sol 1: (A) V = 7L = 7 × 10-3 m3
∴ Work done = -1176 joules
T = 273 K
DQ = DU + DW ⇒ DU = 280 Cal
P = 1.3 × 105 Pa
Work done on gas can be seen by increase in
temperature of the gas, which accounts for 280 calories PV 1.3 × 105 × 7 × 10−3
∴m= = _ 0.4 moles
of energy. RT 8.314 × 273
∴ No. of molecules = 0.4 × 6.02 × 1023
Sol 35: n = 1
= 2.4 × 1023 molecules
Vi = V; DQ = 0
−nR Sol 2: (A) PV2/3 =const.
Vf = 2Vi; DU = -DW = [T – Ti]
1− γ f nRT 2/3
⇒ V = const.
Ti = 273K V
TV γ−1 = constant ⇒ TV-1/3 = const.

V 
γ−1 ⇒ T = conts. × V1/3
∴ Tf =  i  × Ti
∴ On increasing volume, temperature increase
 Vf 
⇒ Tf = (0.5)0.4 × 273 = 206.89 K
Sol 3: (C) T = constant, V = constant,
1× 8.314
∴ DU = - [206.89-273] 35
1 − 1.4 ni = = 1.25 Kmoles
28
⇒ DU  −1374 Joule
P RT
∴ = = const.
Sol 36: Vi = V; Vf = 2V n V
Pi = 760 mm of Hg= 1 atm _ 105 Pascal Pf Pi
∴ =
1 nf ni
Ti = 273 K, n =
32 Pf 9 35
⇒ nf = × ni = × ≈ 1.875 kmoles
∵ DQ = 0 Pi 6 28
For adiabatic process:- ∴ Moles of O2 supplied
γ−1
TV = constant = 1.875 – 1.25 = 0.625 kmoles

V 
γ−1
∴ Amount of oxygen = 20 kg
∴ Tf =  i  Ti
 Vf 
1 3 . 6 0 | Kinetic Theor y of Gases and Thermodynamics

4 ∴ PV = 1×RT \P’V=1×R×(2T)
Sol 4: (B) PV = RT  .....(i)
M ⇒ PV = RT ⇒ P’ = 2P
(4 − 0.8) 3.2R
PV = R (T + 50) = (T + 50) .....(ii)
M M Sol 9: (D) For diatomic molecule
By (i) and (ii) we get, Translational degree of freedoms = 3

4 3.2R Rotational degree of freedoms = 2


RT = (T + 50) ⇒ 4T = 3.2T + 3.2 × 50
M M 3KT 3
Ratio = =
2KT 2
⇒ T = 200 K
Sol 10: (D) We know
Sol 5: (A) For P = 1 atm, m1 = 2gms, T=298 K
vrms = 1.085 vavg
2R
∴ 1× V= × 298 ….(i)
M1 3P
⇒ = 1.085 vavg ; ρ is density
ρ
For P = 1.5 atm, m1=2 gm, m2=3 gm, T=298 K.
In this case ρ is constant
2 3 
∴ 1.5× V=  +  R × 298  ….(ii) ∴ P ∝ (vavg)2
 M1 M2 
2
Using (ii)/(i) we get, Pf  (v avg )f 
∴ =   =4
Pi (v )
 avg i 
 2 2 
 + 
 M1 M2   3M1  M1 1
1.5 = ⇒ 3 = 2 +  ⇒ = Sol 11: (C) Frictionless piston means, no generation of
2  M2  M2 3 heat due to the motion of piston.
M1
n=1

nRT Ti = T
Sol 6: (B) P =
V P = const.
2 2
 nRT  T Vi = V Vf = 2V
∴   V = constant ⇒ = constant
 V  V Initially:- PV = RT
1
V 2 Work done = PDV = P(2V – V)=PV=RT
∴ Tf =  f  × Ti = 2T
 Vi 
Sol 12: (A) f = 6
Sol 7: (A) Pat top of tube + Pdue to 50 cm of Hg= Patmospheric DW = 25 ⇒ PDV = 25 ⇒ nRDT = 25 J
[By using gas equation]
⇒ Pat top of tube = (75 – 50) cm of Hg
∴ P = constant
= 25 cm of Hg
DQ = DU + DW [By 1st law of thermodynamics]
105
= 25 × Pa = 33.3 k Pa nf 6
75 and DU = RDT = × 25 = 75 J
2 2

Sol 8: (C) ∴ DQ = 100 J

For O2 For H Alternative Method


n = 1 n=1 P∆V
At constant pressure, DQ = nCP DT = CP
T 2T R
[⇒ PV = nRT
P P’
PDV = nRDT at const. pressure]
V V
P hysi cs | 13.61

DW = PDV = 25 J 11kg
nCO = = 0.25 k moles
 f
2 44
f = 6; CP = 1 +  R = 4R 2 2 20
 2 fCO = = =
2 1.3 − 1 0.3 3
 25 
∴ DQ = (4R)   = 100 J
R  ∴ Equivalent molecular weight
n1M1 + n2M2 m1 + m2
= =
Sol 13: (B) DQ = -30 J n1 + n2 n1 + n2
DW = -18 J 7 + 11
= = 36 gm
By 1st law of thermodynamics 0.25 + 0.25
DQ = DU + DW n1γ1 + n2 γ 2 + .....
γmix ≠
⇒ DU = -30 – (-18) = -12 J n1 + n2 + .....
⇒ UB – Ui = -12 n1f1 + n2 f2 + .....
fmix =
∴ UB = -12 + 60 = 48 Joules n1 + n2
 20 
Sol 14: (A) Ti = 300 K Tf = ?  0.25 × 5 + 0.25 × 
3  35
=  _
Pi = P Pf = 4P 0.25 + 0.25 6
2 2 47
nRT ∴ γmix = 1 + =1+ = = 1.34
By gas equation we know: - V = fmix 35 / 6 35
P
∴ PV4/3 = constant
4 4
 nRT  3
⇒P 
T3
 =constant ⇒ 1 = constant
Previous Years’ Questions
 P 
P3
1 4
÷ 1 Sol 1: (A) Work done in a cyclic process = area between
P 3 3  4P  4 the cycle
∴ Tf =  f  × T i =   × 300 K
 Pi   P  = AB × BC = (2P – P) × (2V – V) = PV
⇒ Tf = 300 2 K Note if cycle is clockwise (p on y-axis and V on x-axis)
work done is positive and if it is anticlockwise work
Sol 15: (D) For first kind of expansion:- done is negative.

DW = 20 KJ
3RT
DQ = 16 KJ Sol 2: (A) vrms =
M
∴ DU = DQ – DW = -4 KJ
Room temperature T = 300 K
Since, U is a state function. Therefore, value of DU in
both expansions remain same. 3 × 8.31× 103 × 300
∴ 1930 =
Thus, for second expansion:- M
DU = -4KJ, DQ = 9 KJ ∴ M = 2.0 g/mol or the gas is H2.
∴ By first law of the thermodynamics:-
Q2 CV 1
DW = DQ – DU =13 KJ Sol 3: (B) Q1 = nCP DT, Q2 = nCv DT, = =
Q1 CP γ
7kg Q1 70
Sol 16: (A) nN = = 0.25 k moles or Q2 = = = 50 cal
2 28 γ 1.4
2 2
fN = = =5
2 γ −1 1.4 − 1
1 3 . 6 2 | Kinetic Theor y of Gases and Thermodynamics

5 3 nART nART
Sol 4: (B) g1 = means gas is monatomic or CV = R – DP = (pA)i – (pA)f = –
3 1 2 V 2V
nART
7 5 = .…. (i)
g2 = means gas is diatomic or CV = R 2V 
5 2 2
In chamber B →
CV (of the mixture) nBRT nBRT
– 1.5DP = (pB)i - (pB)f = –
3  5 V 2V
n1CV + n2CV (1)  R  + (1)   R
2   2  = 2R nBRT
= 1 2
=  =  ……. (ii)
n1 + n2 1+ 1 2V

CP (of the mixture) = CV + R = 3R From Equation (i) and (ii)


nA 1 2 m /M 2 m 2
CP 3R = = or A = or A = or 3mA
∴ gmixture = = = 1.5 nB 1.5 3 mB / M 3 mB 3
CV 2R
= 2mB

3RT
Sol 5: (B) vrms = Sol 9: (D) Internal energy of n moles of an ideal gas at
M
temperature T is given by
i.e., Vrms ∝ T f 
U = n  RT 
When temperature is increased from 120 K to 480 K (i.e., 2 
four times), the root mean square speed will become where, f = degrees of freedom

4 or 2 times i.e., 2v. = 5 for O2 and 3 for Ar


Hence, U = UO + UAr
3 2
Sol 6: (D) The average translational KE = kT which is
2 5  3 
= 2  RT  + 4  RT  = 11 RT
directly proportional to T, while rms speed of molecules 2  2 
is given by

3RT Sol 10: (D) During adiabatic expansion, we know


vrms = i.e., vrms ∝ T
M TVγ–1 = constant
When temperature of gas is increased from 300 K to or T1 V γ –1 = T2 V2γ –1
600 K (i.e., 2 times), the average translational KE will 1
5
increase to 2 times and rms speed to 2 or 1.414 times. For a monoatomic gas, γ =
3
∴ Average translational KE = 2 × 6.21 × 10–21 J
5
γ –1 –1
= 12.42 ×10–21 J T1 V   AL  3
∴ =  2  =  2 
And vrms = (1.414)(484)m/s ≈ 684 m/s T2  V1   AL1 
(A = Area of cross-section of piston)
nRT
Sol 7: (C) PV = nRT or P = or P ∝ T L 
2/3
V =  2 
If V and n are same. Therefore, if T is doubled pressure  L1 
also becomes two times i.e., 2p.
Sol 11: (C) pT2 = constant
Sol 8: (C) Process is isothermal. Therefore
 nRT  2
 T = constant or T V = constant
3 –1
 1 ∴ 
T = constant.  p ∝   V 
 V 
Volume is increasing, therefore, pressure will decrease. Differentiating the equation, we get

In chamber A → 3T2 T3
.dT – dV = 0
V V2
P hysi cs | 13.63

T Sol 14: (A) DU = nCV DT = nCV (T2 – T1) in all processes.


or 3dT = .dV  ……. (i)
V (B) In adiabatic process DQ = 0
From the equation, dV = Vγ dT
∴ DU = – DW or |DU| = |DW|
dV
γ = coefficient of volume expansion of gas = (C) In isothermal process DT = 0
V.dT
dV 3 ∴ DU = 0 (as DU = nCVDT)
From Equation (i) γ = =
V.dT T
(D) In adiabatic process DQ = 0
Sol 12: A → s; B → r, p; C → p; D → s
Sol 15: (A, B) TA = TB ∴ UA = UB
In process J → K V is constant whereas p is decreasing.
Therefore, T should also decrease. V   4V0 
WAB = (1)(R)T0 ln  f  = RT0 ln   = p0V0 ln(4)
\ W = 0, DU = – ve and Q < 0  Vi   V0 
In process K → L p is constant while V is increasing. Information regarding p and T at C cannot be obtained
Therefore, temperature should also increase. from the given graph. Unless it is mentioned that line
W > 0, DU > 0 and Q > 0 BC passes through origin or not.
In process L → M This is inverse of process J → K.
Sol 16: (D) No option is correct
∴ W = 0, DU > 0 and Q > 0
In process M → J γRT
v=
M
V is decreasing. Therefore, W < 0
7
(PV)J < (PV)M v1 γ1M2 ×4
= = 5
\ Tf < TM or DU < 0 v2 γ 2M1 5
× 32
3
Therefore, Q < 0
460 21 460 × 5 × 2 2
= v2
⇒= 1420 ms
= 1420 m/s
v2 25 × 8 21
Sol 13: A → p, r, t; B → p, r; C → q, s; D → r, t
Internal energy ∝ T ∝ PV
nf f Sol 17: (A) U=U1 + U2
This is because U = RT = (PV)
2 2 (P1V1 + P2 V2 )T1T2
T=
Here, n = number of moles, f = degree of freedom. (P1V1T2 + P2 V2 T1 )
∴ If the product PV increases then internal energy will
increase and if product decreases the internal energy
Sol 18: (B) Thermal energy corresponds to internal
will decrease.
energy
Further, work is done on the gas, if volume of gas
decreases. For heat exchange, Mass = 1 kg

Q = W + DU Density = 8 kg/m3
Work done is area under p–V graph. If volume increases mass 1 3
⇒ Volume
= = m
work done by gas is positive and if volume decrease density 8
work done by gas is negative. Further DU is positive
if product of PV is increasing and DU is negative if Pressure = 8 × 104 N/m2
product of PV is decreasing. 5
∴ Internal Energy = P × V =5 × 10 4 J
If heat is taken by the gas, Q is positive and if heat is 2
lost by the gas, Q is negative.
Keeping the above points in mind the answer to this Sol 19: (C) WAB = ΔQ - ΔU = nCpdT – nCvdT (at constant
question is as under. pressure)

(A) → (p, r, t); (B) → (p, r);


(C) → (q, s); (D) → (r, t)
1 3 . 6 4 | Kinetic Theor y of Gases and Thermodynamics

P a b
5 A B Sol 23: (C) U(x)
= −
12
2 x 10 Pa x x6

n=2, =1.67 U(x = ∞) =0

dU 12a 6b 
as, F =
− =
− + 
5
1 x 10 Pa
D C dx x
13
x7 
At equilibrium, F = 0
300 K 500 K T
2a
= n(Cp – Cv)dt ∴ x6 =
b
= nRdT = 2 × R × (500 – 300) = 400 R a b −b2
∴ Uat equilibrium = − =
 2a 
2  2a  4a
Sol 20: (A) At constant temperature (isothermal process)    
b  b 
P   105  b2
WDA= nRTln  1 = 2.303 × 2R × 300log   ∴ D = [U( x = ∞ ) – Uat equilibrium ] =
5 
 P2   2 × 10  4a
 1
= 2.303 × 600log   T − T2 1
2 Sol=
24: T η− T2 1 =1
η1 (D)1 =
= 1
T 6
0.693 600lR =
=× −414R T1 61
T − (T2 − 62) 1
= T1η− (T2 −1 62)
= 1 T −T 62 1
and η2
= =2
T1 ⇒ 13 2 + =
T1 3 T1 T1 3
Sol 21: (B) Net work done in a cycle = WAB + WBC + WCB
+ WBA 1 62 1 62 1
+ =⇒ =
= 400 R + 2 × 2.303 × 500 R ln 2 – 400R – 414 R
6 T1 3 T1T−1 T2 6 1
=
T1T1 − T26 1
= 1000R x ln 2 – 600R x ln 2 = 400R x ln 2 = 276R T
∴T1 = −
1 62×T 2 61 =
= = 372KT T1 1 6
T1 61− 2 =
T1 − T2T 1 T T6 1
Sol 22: (B) The efficiency of cycle is 2= 1 ⇒ 11− 2 =
1 −
T1 T 6 6 T2= T5 6
T2 1 = 1
η = 1− TT2 1 372 T2 6 5
T1 1 − 2= 5 =
⇒ T =6 ⇒ T3722 = 310K
6
372 6
1
for adiabatic process T2⇒ T5 =310K
⇒ T2 = 310K
=2
TV γ−1 = cons tant 372 6
Sol 25: (A) Work done in complete cycle = Area under
7 ⇒ T2 = 310K
For diatomic gas γ = P–V graph = P0V0
5
From A to B, heat given to the gas
T1 V1γ−1 = T2 V2γ−1
3 3 3
= n C v ∆T= n R∆T= V0 ∆P= PV
V 
γ−1
2 2 2 0 0
T1 = T2  2 
From B to C, heat given to the system
 V1 
7 −1 5  5
T= T2 (32)5 = T2 (25 )2/5
= T2 × 4 = n C v ∆T= n  R  ∆T= ( 2P ) ∆V= 5P0 V0
2 0
1
2 
T1 = 4T2
From C to D and D to A, heat is rejected.
 1 3
η=  1 − = = 0.75 Work done by gas
 4 4 Efficiency, η
= × 100
Heat given to the gas
P0 V0
=η = 15.4%
3
P0 V0 + 5P0 V0
2
P hysi cs | 13.65

40 500 − TS 1
Sol 26:
= (C) = , TS 300K Sol 31: (C) Since τ =
100 500 nπ 2v rms d2

600 T − 300 1
= ⇒ T 750K
= n∝ and v rms ∝ T
100 T V
V
⇒τ∝
Sol 27: (C) T
Mg =n C1V −1 =
< v > C2 T1/2
P=
0 P0 V0γ PV γ
A γ+1
Mg P0 A..............(1)
= (i) P0=AX 0γ PA(x0 − x)γ Since TV γ−1 = constant ⇒ τ ∝ V 2

P0 Ax0γ
Let piston is displaced by x P= P0 P − P = P
(x0 − x)γ Sol
P − P32:=(D)
− P (V − 02V −) 0 (V − 2V0 )
P − P00 = − V00 (V − 2V00 ) V0
 P xγ  V0 
 P= x0γ  Mg = 0 0  A Frestoring P0= P0
=Mg = 0  A Frestoring
 γ  =P 3P0 − P0 V P 3P − V ....(1) ....(1) …(i)
 (x − γx)γ   0 − x) 
(x
=P 3P0 − V0 V
0
V0 ....(1)
 P 0 x  V0
=Mg = P00 x00γγ γ  A Frestoring
 x0γ  nRT P
=Mg  = (x0 −x0x)γ A P0FArestoring
=1−  F [x0 − x ≈nRT
x= P0 0
P0 A = 1−  F  (x − x)[xγ 0 − xrestoring
≈ x0 ] nRT0 ] 3P0 − P = V 3P0 − V
V V
  (x(x0 −−γx)x)γ restoring
0
 0  V= 3P0 − V0 V 0
 0 x0  V V0
P0 A = 1− x0γ γ  Frestoring
γP Ax [x0 − x ≈ x0 ] P0 23P V − P0 V 2
P0 A =γP Ax = 0
10− (x0 − x)  FFrestoring [x0 − x ≈ x0 ] = 3P0 V −nRT
nRT = P V 0
F =  (x − x)γ  x0 = 3P0 V − V00 V 2
nRT V0
γPx0Ax 0  V0
F = γP00 Ax 1 γP0 A
F =f =x10 γP0 A ∴f =
∴ x20π x M 2π x0M Differentiate w.r.t. Volume
1 γP00 A
∴ f =1 γP20 A 1 γP0 A
2
∴ f 1=2π γP0 Ax0M = 2P0 3V0
= 2π x M 2π MV0 3P0 − V =0 ⇒ V =
2π MV020 V0 2
1 γP0 A2
= 1 γP0 A
= 2π MV0 5R
Sol2π28: MV (D)0 ∆U = n CV ∆T = 1× ∆T Putting in equ (i)
2
P0  3V0  3P0
For BC, ΔT = –200 K P=
3P0 −   =
V0  2  2
⇒ ΔU = –500R
Now, PV = xRT 9P0 V0 = nRT
4
1U
Sol 29: (C) p = 9P0 V0 9 P0 V0
3V = nRT ⇒ T =
4 4 xR
nRT 1 4 9 P0 V0
∝ T T=
V 3 4 xR R R
Sol
C 33:
= CV +(A) = C CV +
VT3 = const 1− n 1− n
CP =
−CC−vC CP −CC v− C CP − C v
C − CV
= = ; 1v− n
; V1 − n 1 − nP =
4 3 3 C − Cv
πR T =
const 1− n C − Cv
3 CP − C v C − CP
C − Cnv = 1C− − C =
1 n= 1− P = P
TR
= const ⇒ T ∝ C − Cv C −CC−v C v C − Cv
R

Sol 30: (B) Since entropy is a state function, therefore


change in entropy in both the processes should be
same.
1 3 . 6 6 | Kinetic Theor y of Gases and Thermodynamics

JEE Advanced/Boards Sol 4: CP’ and Cv’ be molar specific heats.


∴ CP’ = CP × M = 0.2 M cal/mole °C
Exercise 1 CV’ = CV × M = 0.15 M cal/mole °C
and CP’ – CV’ = R
Sol 1: For O2 For He
⇒ 0.2 M - 0.15 M = 2
P = P0 P = P0
2
V = V0 V = V0 ⇒M= = 40 gms
0.05
T = T0 T = T0/2
M = 32 gm M = 4gm Sol 5: f = 6

PV = n RT At constant pressure

m PV DQ = nCP DT
⇒ T= = constant
M R P∆V
= CP ×
mO mHe R
∴ 2
TO = THe
MO 2 MHe [PV = nRT ⇒ P DV = nR DT at constant pressure]
2
CP
mO mHe mO = × DW
⇒ 2
× T0 = × T0/2 ⇒ 2
=4 R
32 4 mHe
 f
CP = 1 +  R = 4R
 nR   2
Sol 2: We have, V =   T and we know that V – T
 P  And DW = 25 J
curve is a straight line. 4R
DQ = × 25 = 100 J
nR R
∴ Pressure is constant and its slope = = tan 53o
P
2 × 8.314 × 103 Pa L / moleK 4 Sol 6: He N2
⇒ =
P 3 m = 4 gm mN = 28 gm
2
⇒ P ≈ 1.25 × 10 Pa 4
4 28
∴ nHe= =1 mole nN2= =1 mole
4 28
Sol 3: PV γ = constant
⇒ TV γ−1 = constant f = 3 f=5

⇒ V γ−1 dT + (γ-1) V γ−2 T dV = 0 3


∴ CV = ∴ CV = 5/2 R
2
dV −V dV V nHeCV + nN CV
⇒ = ⇒ = He 2 N2
dt ( γ − 1) T dT ( γ − 1) T ∴ CV mix =
nHe + nN
2
+ V0 1 mT0
⇒m= ⇒ =+ 3 5
( γ − 1) T0 γ −1 V0 R+ R
∴ CV mix = 2 2 = 2R
mT0R 1+ 1
R
∴ CV = =+
γ −1 V0 3RT
We know vrms =
M
And Cp = CV + R
∴ To double the vrms temperature must be made 4
(V0 + mT0 )R mRT0  T0m  times the original temperature.
= = 1 + R
V0 V0  V0  ∴ Tf = 4 Ti = 1200 K
∴ DT = 900 K
P hysi cs | 13.67

∴ DQ = n CV DT [At constant volume] Sol 10: From graph, it is clear that V and 1/T have a
linear relationship
= (nHe + nN )CV ∆T
2 mix
m
= 2 × 2R × 900 = 3600 R ∴V=
T
m is the slope of straight line
Sol 7: T = constant
⇒ VT = constant
∴ DU = 0
⇒ PV2 = constant = k (say)
Thus DW = –23.04 × 102 J
Vf Vf
Vf DW = −2
⇒ nRT ln
Vi
= –23.04 × 10 2
∫ PdV = ∫ kV dV
Vi Vi

0.25 Vf
⇒ 8.31 × T ln = –23.04 × 102 = −kV −1 = – [k Vf-1 – kVi-1]
0.50 Vi

+23.04 × 102 = – [(PfVf2) vf-1 – (PiVi2) Vi-1] = – [PfVf – PiVi]


∴T= ≈ 400 k
8.31 × ln2 = – nRDT.
Thus DQ = DU + DW
Sol 8: Let C be the molar heat capacity of this gas.
⇒ nC DT = nCV DT – nR DT
∵ DQ = nCDT
⇒ C = CV – R
And we know DU = nCV DT
3 R
⇒C= R–R=
∴ ∆Q =2∆U 2 2
∴ nC DT = 2nCV DT
Vf Vf
5  Sol 11: (i) DW = =
⇒ C = 2CV = 2  R  = 5R ∫ PdV ∫ aVdV
2  Vi Vi

 Pf  2  Pi  2
Sol 9: C → Molar heat capacity of the process V   Vf −   Vi
2 f 2
aV aVf − aVi2 V  Vi 
By 1st law of thermodynamics:- = = =  f 
2 2 2
Vi
dQ = dU + dW
⇒ nC dT = nCv dT + dW  …..(i) Pf Pi
(a = = )
Vf Vi
dW = PdV
Te αV Pf Vf − PV nR∆T
nRT ⇒ DW = i i
=
= dV = nR 0 dV 2 2
V V
nR
T = T0 eaV DU = nCV DT = DT
γ −1
∴ dT = aT0 eaV dV
DQ = DW + DU
dT
⇒ dV =
αT0 eαV nR∆T nR 1 1 
= + DT = nRDT  + 
nR
2 γ −1  2 γ − 1
⇒ dW = dT
αV
nR∆T  γ + 1
Substituting value of dW in (i) we get =   ....(i)
2  γ − 1
nR
nCdT = nCV dT + dT We know P = aV
αV
R nRT aV 2
∴ C = CV + ⇒ = aV ⇒ T =
αV V nR
aV02 9aV02
∴ Ti = ad Tf =
R R
1 3 . 6 8 | Kinetic Theor y of Gases and Thermodynamics

8aV02 DW = - 400 n R [1-2-1/3]


∴ DT = .....(ii)
R For process AC
Substituting (ii) in (i) DQ = 0

R 8aV02  γ + 1   γ + 1  DU = nCVDT = 2 ×400 nR [1 – 2-1/3]


Thus DQ = ×   =  4aV0
2
2 R  γ − 1  γ − 1 DW= -DU = -2 × 400 nR [1-2-1/3]

(ii) C → Heat capacity of gas Q released


Efficiency (e) =1 -
So DQ = nCDT Q absorbed
Using (i) we get
3 × 400nR[1 − 2−1/3 ] 3(1 − 2−1/3 )
nR∆T  γ + 1  =1– =1–
nCDT =   400nR ln2 ln2
2  γ − 1
 γ + 1 R
⇒C=   dQ −kAdT −k[2π(b − x) × ]dT
 γ − 1 2 Sol 13: = =
dt dx dx
Sol 12: At A x

VA = V0 , TA = 400 K d
b
400nR
∴ PA =
V0
At B
400nR
VB = 2V0 , TB = 400 K, PB = T0
2V0
At C
400nR
VC = ?, TC = ?, PC =
2V0
∵ AC is adiabatic process
∴ PV γ = constant
1 On integrating with respect to dx we get
P γ 1
 VA = ( 2 )1.5 V0 ⇒ VC =
22/3 V0 b −a b −a
⇒ VC =  A dQ 1 dT
P
 C  ⇒ ∫ ×
dt 2π(b − x)
dx= ∫ −k
dx
× dx
0 a
PC VC
⇒ TC = =400 × (2)-1/3 dQ − ln
b −a
nR ⇒ × (b − x) =- kDT
dt 2π 0
For process AB
dQ b
DU = 0 .ln =−2πkl(T − T0 )
dt a
 2V 
DQ = DW = nR × (400) × ln  0 
 V0  ( )
 2 dT   b 
 πa l 3  ln   =−2πkl(T − T0 )
 dx   a 
= 400 nR ln 2
For process BC ⇒ On integrating w.r.t. t we get
nR b
DU = nCV DT = × [400 (2)-1/3 – 400] −πa2 ls ln   T2
γ −1 t
a dT
= -2 × 400 n R [1-2-1/3]

2πkl ∫ T − T0
= ∫ dt
T⊥ 0
nγR
DQ = nCPDT = - × 400 [1-2-1/3]
γ −1 a2s  b   T −T 
⇒ ln   ln  0 1  = t
= -3 × 400 n R [1-2 ]
-1/3
2K a  T0 − T2 
P hysi cs | 13.69

Sol 14: kS → Conductivity of solid cast iron. At the Area of cross-setion(A)=0.04m


2

point of junction of solid cast iron and liquid cast iron,


temperature would be equal to melting point of iron. T1=400k

T2 k=10/m-k
=0.4cm

Solid cast
iron (ks)
 Tm
Liquid cast
This metalic
x iron (kks) T0=300K
disk s=60 J/kRk
T1 m=0.1kg

 dQ   dQ  On integrating we get
  =  
 dt solid cast iron  dt liquid cast iron 350 t
ms dT
⇒-
kA ∫ T − 400 = ∫ dt
−k s A[T2 − Tm ] −kk s [Tm − T1 ] 300 0
⇒ =
−x x
−0.4 × 600 × 0.4 350 − 400
⇒t= ln
x k[Tm − T1 ] 10 × 0.04 300 − 400
⇒ =
−x T2 − Tm
≈ 166.3 sec
x k[Tm − T1 ] k(T1 − Tm )
⇒ = = Sol 17: (T – T0) = (T1 – T0) e-kt
 (T2 − Tm ) + k(Tm − T1 ) k(T1 − Tm ) + (Tm − T2 )
1 T −T 
⇒t= ln  1 0 
dT k  T − T0 
Sol 15: (a) Temperature gradient =
dx
T0 : Surrounding temp
(0 − 100) o C
= = -100°C/m T1 : Initial temp of object
(1 − 0)m
T : Final temp of object
dT T − 100
(b) = = -100°C/m
dx x−0 1  80 − 20  1
∴5= ln   = ln (2)
k  50 − 20  R
⇒ T = 100 (1 - x) °C
Total heat absorbed 1  60 − 20  1 1
=and t =ln   ln(2)2 = 2 × ln2
k  30 − 20  k k
= ∫ mass × specific heat × change of temp
∴ t = 10 min
1 1
= ∫ (2dx)(10)(T − 0) = 20 ∫ 100(1 − x)dx Sol 18: A = 27 cm2 = 27 × 10-4 m2
0 0
k = 3700 N/m
1
 x 2 
= 2000  x −  = 1000 J Pi = 1 atm = 105 Pa
 2  0
Ti = 273 K
Vi = Ah.
dQ −kA(T − 400)
Sol 16: = Initial contraction in spring
dt 
dT −kA(T − 400) Pi × A 105 × 27 × 10−4 27
⇒ ms = = = = m
dt  k 3700 370

ms dT Pf = ?
⇒ = -dt
kA T − 400
1 3 . 7 0 | Kinetic Theor y of Gases and Thermodynamics

Tf = ? ⇒ TC = 4TA
9hA DQAB = DU + DW
Vf =
16 = nRTA ln 4
Since, spring contract by h/16 length
DQBC = DU + DW
 h 27 
∴ Force exerted by spring = k  +  = nCV [TC – TA] = 3n CV TA
 16 370 
Avg. pressure due to spring (P0) ∴DQAB + DQBC = 27.7× 103 J

k  h 27  ⇒ nRTA ln 4 + 3 nCV TA = 27700


=  + 
A  16 370  27700
⇒ R ln 4 + 3CV = ≈ 50.73
2 × 273
Since process is adiabatic
γ ∴ CV ≈ 13.068 J/mol-K
V 
∴ Pf =  i  Pi CP CV + R R
 Vf  = =1+ ≈ 1.63
CV CV CV
3/2
 16  4 × 16
  105 = × 105
 09  27 Sol 20: Initially
Ti = 00C= 273K = T1
Pf must be equal to P0 for equilibrium
Pi = P1
k h 27  4 × 16
Thus,  +  = × 105 V i = V1
A 16 370  27
From (1) to (2)
5 −4
h 27 4 × 16 × 10 × 27 × 10
⇒ + = Process is adiabatic
16 370 27 × 3700
DQ = 0
4 × (16)2 27 × 16
⇒h= - DU = -DW
370 370
P = P2
⇒ h = 1.6 m
T = T2
In adiabatic process TV γ−1 = constant
γ−1
V = V 2 < V1
V 
⇒ Tf =  i  Ti At (2)
 Vf 
Process is isochoric
1/2
 16  ∴ V = constant
=   × 273
 9  ∴ V = V2
⇒ Tf = 364 K ∴ P = P3 < P1
T =273 K = T1 < T2
Sol 19: Process AC is isobaric
From (2) to (1)
Process is isothermal
d
T = 273 K = T1

273k A V = V1
B
∴ P = P1

4 4V

VC 4V
∴ TC = TA = × TA
VA V
P hysi cs | 13.71

Exercise 2
P
P2
Single Correct Choice Type

Isobaric
Adiabatic Sol 1: (D)
P3

P1
Isothemal
n moles
n moles
V1 V1
V
Small vessel
DQprocess = 100 × 80 = 8000 cal P
DUprocess =0 cal Large vessel

∴ DW + DU = DQ T
∴ DW =8000 cal V = constant
and PV = nRT
Sol 21: Number of molecules hitting 1m2 of wall per
molecules  nR 
second (N)== nv cos θ ⇒P=   T
m2 sec  V 
∴ P = mT where m is a constant
∴ P-T curve for both vessels will be linear but with
different slopes, since the constant volumes have

 different values in both cases.

Sol 2: (C) Initially


Pi = 76 cm of Hg
Change in momentum of 1 molecule
Vi = 5A
= m (2v cos θ) =2mv cos θ
∴ Pressure exerted = N ×change of momentum of 1 5cm
molecule
= nv cos θ × 2 mv cos θ = 2mv2 cos2 θ

Sol 22: (a) Work along path A-D


Finally
= Area under curve AD
Pf = (76) – (48 – x) = (28 + x) cm of Hg
1
= [P + PD] × [VD - VA]
2 A Vf = x A

1
= × 1.6 × 105 × 1.1 × 10-3 = 88 J
2
x
(b) DWADC = DWAD + DWDC ⇒ DWDC = -3J

1
∴ - [ [PD + PB] × [VD – VC]] = -3
2 48-x
⇒ 0.9 × 10 (1.3 – Vc) × 10 = 6
5 -3

⇒ VC ≈ 1.23 L
(c) WCDA = –WADC = –85 J
1 3 . 7 2 | Kinetic Theor y of Gases and Thermodynamics

Since, temperature remains constant PB


dP
H/2
2Mg
∴ PiVi = PfVf ∫ P
= ∫ RT
dh
Pm 0
⇒ 76 × 5A = (28 + x) x A
P  MgH
⇒ x2 + 28 x – 380 = 0 ln  B  =
 Pm  RT
⇒ (x + 38) (x – 10) = 0
PB
⇒ x = 10 cm or x = -38 cm = exp (MgH/RT)
Pm
x = -38 rejected since x can’t be -ve
Sol 5: (D) Since there is no loss of energy
Sol 3: (C) ∴ Sum of change of internal energies must be zero.
Container X Container Y i.e. DU1 + DU2=0
2V
2V V ⇒ n1 Cv [T-T1]+ n2 Cv [T – T2] = 0
x Y ⇒ (n1 + n2) T = n1 T1 + n2 T2
n1T1 + n2 T2
Process (P) in both vessels will be same. ⇒T=
n1 + n2
Thus V ∝ n T
V 3RT 1
∴n∝ Sol 6: (D) vrms = ⇒ vrms2 ∝
T M M
2V
∴ nX = k [k is some constant] (v rms2 )B MA m × NA 1
200 ∴ = = =  ....(i)
2
(v rms )A MB 2m × NA 2
kV
nY =
400 Since by the postulate of KTG that, molecules move in
nX 4 random motions.
∴ =
nY 1 ∴ <vx2> = <vY2> = <vX2> and v2 = vx2+ vy2 + vz2
Since gas in both container is same ∴ <v2> = 3 <vx2>
mX nX Thus (vrms2)A = 3w2 & (vrms2)B = v2
∴ = =4
mY nY
v2 1 w2 2
m mX Thus, by (i) we get: - = ⇒ =
∴ mY = = 3w2 2 v 2 3
4 4

Sol 4: (C) Pressure gradient will develop due to the Sol 7: (C) P = 0.7 × 105 N/m2
upward acceleration so V = 0.0049 m3
dP
= 2ρg CP
dh γ= = 1.4 = ratio of specific heats
CV
dP PM CV
= 2g ≠ 1.4 since, we know CP > CV
dh RT CP

g In reversible adiabatic process


PV γ = constant
H/2
∴ V γ DP + Pγ V γ−1 DV = 0
Pm ∆P −γP 1.4 × 0.7 × 105
h ⇒ = =–
∆V V 0.0049
dP
⇒ = -2 × 107 Nm-5
PB dV
P hysi cs | 13.73

Sol 8: (C) Equal mass of same gas ⇒ Equal moles Multiple Correct Choice Type
Initially
Sol 10: (A, B) eA = 0.01 and eB = 0.81
X n moles n moles Y AA = AB
T = 300 k T = 300 k
P = 1 atm P = 1 atm EA = EB
V = 42 A V = 42 A ⇒ eAσ AA TA4 = eB σ AB TB4
42cm 42cm ⇒ 0.01 TA4 = 0.81 TB4
Area of 1
cross-sextion = A ⇒ TB = × TA
3
Using gas equation we get 1
⇒ TB = × 5802 = 1934 K
3
PV = nRT
By Wien’s displacement law
⇒ 42 A = nR 300
lm T = constant = 2.93 × 10-3 mK
nR 42
⇒ =  ……(i) ∴ λm = 0.5 mm
A 300 A

Finally Since, it is given in the question that


λm = 1 mm + λm
X n moles n moles B A
T = 300 k T = 300 k ∴ λm = 1.5 mm
B
Pf = P P = 1 atm
V = (42+x) A V = 42 A Sol 11: (A, D) P2 V = const.
(42+x)cm 42-x 1/2 1/2
V   V 
Using gas equation we get ∴ Pf =  i  × Pi =   P = P/2
 Vf   4V 
P (42 + x) A = n R 330 …..(ii)
 nRT 
P (42 – x) A = n R 300  …..(iii) P2   = constant
 P 
(ii)/(iii) gives
⇒ PT = constant
42 + x 330
= ∴ P – T curve is hyperbola
42 − x 300
P 
2x 30 and Tf =  i  Ti = 2T
⇒ =
42 − x 300  Pf 

⇒ 20x = 42 – x Sol 12: (C, D) P = 3000 J


⇒ x = 2 cm V=5

Sol 9: (C) At constant pressure 3RT


vrms =
Mm
W = PDV
5 1 3 3
For monoatomic: - CP = R ⇒ (nMm) vrms2 = nRT = PV
2 2 2 2
and Q = nCP DT 3
Translational K.E. of all molecules = PV
2
5R P∆V 5
⇒Q=n× × = W 3
2 nR 2 ∴ P × 5  = 3000 J
2
2
∴W= Q
5 ∴ P = 400 J/l
1 3 . 7 4 | Kinetic Theor y of Gases and Thermodynamics

Total K.E. of 10  of gas at Pressure 2P for a monoatomic Alternative method:


3
gas = × 800 × 10 = 12000 J Adiabatic process
2
5
Total K.E. of 10 , 2P pressure for a diatomic gas = PV P Isothermal process
2
Initial stage
5 Final stage
= × 800 ×10 = 2000 Joules
2 Pf isothermal
Note: In options its asked total K.E. not translational K.E. Pf adiabatic
V
Sol 13: (A, D) No. of molecules moving towards wall Vf

1 molecules From graph, it is very clear, that


per unit volume = × 1026
6 m3
Pf isothermal > Pf adiabatic
∴ No. of molecules hitting 1m2 of the wall every sec (n)
and for same volume and moles of gas
1 
=  × 1026  × (2000) T ∝ P [PV = nRT ]
 6 
molecules Thus, Tf isothermal >Tf adiabatic
= 3.33 × 1028
m2 sec Vf

Pressure exerted = (change in momentum) × no. of Work done = ∫ P dV = area under P-V diag
molecules putting wall per unit area per sec Vi

= (3 × 10-27 kg/molecules) × (2 × 2000 m/s) × 3.33 × Thus, (work done)isothermal > (work done)adiabatic
1028 molecules/m2 sec)
≈ 4 × 105 Pa Sol 15: (A, B) Area under the curve gives the rate at
which heat per unit surface is radiated by the body.
Sol 14: (A, B, C) Isothermal process Adiabatic process i.e. total rate of heat radiation = (Area under the curve)
× (Surface area of the body)
PV = constant PV γ = constant
T m
⇒ = constant ⇒ TVr-1 =constant Sol 16: (A, C) ρ =
V V
V V 
γ−1 4 3
⇒ρ× pr = m
∴ Tf = f × Ti ⇒ Tf =  i  Ti 3
Vi  Vf 
⇒ r ∝ (m)1/3
(Since there is expansion)
Vf Vi And area of sphere (A) ∝ r2
∴ > 1 ⇒ <1
Vi Vf ∴ A ∝ (m)2/3
γ−1
V  AA
Thus  i  <1 ∴ = (4)2/3 = (2)4/3
 Vf  AB
eA A σ(T − T0 )4
Since γ-1 is +ve ∴ Ratio of heat loss =
eAB σ(T − T0 )4
(∴ Isothermal temp. > adiabatic temp.)
AA
γ = = (2)4/3
Vi V  AB
Pf = × Pi Pf =  i  × Pi
Vf  Vf  By Newton’s law of cooling:
Vi dQ
(Since γ > 1 and < 1) = ms(COOH)2 = -k (T – T0)
Vf dt
γ dT −k
Vi V  ⇒ = (T – T0)
∴ >  i  dt ms
Vf  Vf 
∴ Isothermal pressure > Adiabatic pressure
P hysi cs | 13.75

Where k = 4e A σ T03 nR
∴ V is a linear function of T with slope and passing
dT A through origin in V-T curve. P
∴ ∝
dt m Secondly for process CD: P-T will be a linear curve
 dT  AA passing through origin.
 
dt
 A m (2)4/3
∴ = A = = 2-2/3
 dT  AB 4 Comprehension Type
 
 dt B mB
Paragraph 1:

Sol 17: (A, C) Sol 20: (A, B) Area under the curve gives the rate at
o o o o
which heat per unit surface is radiated by the body.
-10 C 0C 0C 100 C
50 gm 250cal 50 gm 4000cal 50 gm 5000cal 50 gm i.e. total rate of heat radiation = (Area under the curve)
ice ice water water × (Surface area of the body)
o o
0C 100 C
Calorimeter 150cal 1.5 gm
Sol 21: (A, B, C, D) lm T = constant [By Wien’s
1.5 gm
water
Displacement Law]
water
T1
dE
∴ Heat absorbed by ice and calorimeter to reach 100° T2
T1>T2>T3
d
C water
= 250 + 4000 + 5000 + 150 = 9400 cal T3

∴ Amount of steam converted into water


m1 m2 m3 x
9400
= = 17.4 gm
540 Area under graph = Eλ = e σ T4 ∝ T4
∴ Amount of water remaining = 50 + 17.40 = 67.4 gm
∴ Amount of steam remaining = 20 – 17.4 = 2.6 gm Paragraph 2:

Sol 18: (B, D) DT = 0 Sol 22: (A) AA = A AB = A


The slope of straight line can’t be +ve. Since, T ∝ PV  A = l B = l
and if slope is +ve, then both P and V are increasing.
kA = 3k kB = k
Therefore, temperature will always increase.
T1 < T2 < T3 < T4 A   
∴ RA = = RB = B =
k A AA 3kA kB AB k A
P T1T2T3 T4
is thermal curve RA 1
∴ =
i.e. PV = const. RB 3

St. line Sol 23: (B) Rate at which heat flows from A
= Rate at which heat flows from B
Thus, from graph it can be seen, that temperature first
increases and then decrease.  dQ   dQ 
⇒   =  
 dt A  dt B
Sol 19: (A, B) Process AB is isochoric: - i.e. P = constant
TA TB TA RA 1
PV = nRT ⇒ = ⇒ = =
RA RB TB RB 3
 nR 
⇒V=  T
 P 
1 3 . 7 6 | Kinetic Theor y of Gases and Thermodynamics

TA TA TB TB Sol 31: (A) fH = 5 and fHe = 3


Sol 24: (B) GA = = and GB = = 2
A  B 
nH fH + nHe fHe
G T 1 ∴ fmix = 2 2
∴ = A = nH + nHe
GB TB 3 2

2×5 + 5×3 25
= = = 3.57
Paragraph 3: 2+5 7

 dQ  2 2
Sol 25: (B)   = kAV DT Sol 32: (C)∴ rmix = 1 + = 1+ = 1.56
 dt initially fmix 25 / 7

 dQ   ∆T   dQ  Sol 33: (D) Since internal energy is an extensive


  =kA(4V)   =2  
 dt finally  2   dt initially property
∴ Umix = UHe + UH = 200+100=300 J
2
Sol 26: (A) If all the parameters are kept constant then
dQ dT Match the Columns
= ms = kA v DT
dt dt
dT kAv∆T Sol 34: (A) Vi = V Vf = 2V
∴ =
dt ms
Pi = P
(P) → (y) isobaric process
Paragraph 4: W = PDV = PV
(q) → (z) isothermal
Sol 27: (A) wab= Area below curve AB
Vf
1 W = nRT ln = PV ln 2
= [10 + 20] × (12 – 6) Vi
2
1 (r) → (x) Adiabatic
= × 30 × 6 = 90 J γ
2 V  γ
Pf =  i  Pi = 2 P
 Vf 
Sol 28: (B) wBC = -Area below curve BC
= – 10 [12 – 6] = – 60 J Pf Vf − PV
i i (2−γ P)(2V) − PV
W= =
Note:- wBC is –ve, since volume is decreasing 1− γ 1− γ

PV(1 − 21−γ )
Sol 29: (D) wCA = 0 since volume is constant ⇒W=
γ −1
∴ wABCA = wAB + wBC + wCA = (90) + (-60) + 0 = 30 J

Paragraph 5: Previous Years’ Questions


nHe = 5 moles nH = 2 moles
2
Sol 1: (D) The desired fraction is
MHe = 4g MH = 2 g
2

∆U nCV ∆T C 1
nHeMHe + nH MH f= = = V =
∆Q nCp ∆T Cp γ
Sol 30: (D) Equivalent molar mass = 2 2
nHe + nH
2
5  7
5× 4 + 2× 2 24 or f =  as γ = 
= = gm 7  5
5+2 7
P hysi cs | 13.77

Sol 2: (A) Average kinetic energy per molecule per Area under the graph gives the work done by the gas
1
degree of freedom = kT. Since, both the gases are
2 P
diatomic and at same temperature (300 K), both will A 2
have the same number of rotational degree of freedom
i.e., two. Therefore, both the gases will have the same
average rotational kinetic energy per molecule 1

1 3
=2× kT or kT
2
V
Thus, ratio will be 1: 1. V1 V2

Sol 3: (D) A is free to move, therefore, heat will be (Area)2 > (Area) > (Area)3
supplied at constant pressure
∴ W2 > W1 > W3
∴ dQA = nCP dTA  …… (i)
B is held fixed, therefore, heat will be supplied at Sol 6: (C) For an ideal gas : PV = nRT
constant volume. For P = constant
∴ dQB = nCVdTB  …… (ii) PDV = nRDT
But dQA = dQB (given)
∆V nR nR V
∴ = = =
 Cp  ∆T p nRT T
nCpDTA = nCVDTB ∴ dTB =   dTA
V
 CV 
  ∆V 1 1
∴ = or δ =
= γ(dTA) [γ = 1.4 (diatomic)] V∆T T T

(dTA = 30 K)

= (1.4)(30 K)
∴ dTB = 42 K

Sol 4: (C) Speed of sound in an ideal gas is given by

γRT T
v=
M
Therefore, δ is inversely proportional to temperature T.
γ i.e., when T increases, δ decreases and vice-versa.
∴v∝ (T is same for both the gases)
M
Hence, δ-T graph will be a rectangular hyperbola as
vN γN shown in the above figure.
MHe 7 /5 4 
∴ 2
= 2
. =   = 3 /5
vHe γHe MN 5 / 3  28 
2 Sol 7: (B) In adiabatic process
7 dP P
γN = (Diatomic) Slope of P-V graph, =–γ
2 5 dV V
5 Slope ∝ γ (with negative sign)
γHe = (Monoatomic)
3 From the given graph,
(slope)2 > (slope)1
Sol 5: (A) The corresponding p–V graphs (also called
indicator diagram) in three different processes will be ∴ g2 > g1
as shown:
Therefore, 1 should correspond to O2 (γ = 1.4) and 2
should correspond to He (γ = 1.67).
1 3 . 7 8 | Kinetic Theor y of Gases and Thermodynamics

Sol 8: (A) DWAB = PDV = (10)(2 – 1) = 10 J Sol 12: (A) Since it is open from top. Pressure will be P0.
DWBC = 0 (as V = constant)
Sol 13: (D) Let p be the pressure in equilibrium.
From first law of thermodynamics PA
DQ = DW + DU
DU = 0 (process ABCA is cyclic)
∴ DQ = DWAB + DWBC + DWCA Mg P0A
∴ DWCA = DQ – DWAB – DWBC
Then, PA = P0A – Mg
   = 5 – 10 – 0 = – 5 J
Mg Mg
P = P0 – = p0 –
Sol 9: (C) Temperature of liquid oxygen will first increase A πR 2
in the same phase. Then, phase change (liquid to gas)
Applying, P1V1 = P2V2
will take place. During which temperature will remain
constant. After that temperature of oxygen in gaseous ∴ P0 (2AL) = (P)(AL’)
state will further increase.
 
2P0L  P0 
Sol 10: (C) Slope of adiabatic process at a given state ∴ L’ = =   (2L)
(P, V, T) is more than the slope of isothermal process. P  Mg 
The corresponding P-V graph for the two processes is  P0 − 2 
 πR 
as shown in figure.
 P πR 2 
=  0  (2L)
 πR 2P − Mg 
P  0 
C
P3

P1 Sol 14: (C)


B


p
V
V1 V2 1 2

In the graph, AB is isothermal and BC is adiabatic.


WAB = positive (as volume is increasing) p1 = p2
and WBC = negative (as volume is decreasing) plus, P0 + rg (L0 – H) = P  ……..(i)
|WBC| > |WAB|, as area under p–V graph gives the work Now, applying P1V1 = P2V2 for the air inside the cylinder,
done. we have
Hence, WAB + WBC = W < 0 P0(L0) = P(L0 – H)
From the graph itself, it is clear that P3 > P1 P0L0
∴p=
Note: At point B, slope of adiabatic (process BC) is L0 − H
greater than the slope of isothermal (process AB)
Substituting in Equation (i), we have

Sol 11: (B) Total translational kinetic energy P0L0


P0 + ρg(L0 – H) =
L0 − H
3 3
= nRT = PV = 1.5 PV
2 2 or ρg(L0 – H)2 + P0(L0 – H) – P0L0 = 0
P hysi cs | 13.79

Sol 15: (B, D) In case of free expansion under adiabatic Sol 16: (B, D) For monoatomic gas
conditions, change in internal energy DU = 0 3 5
Cv
= =R, Cp R
∴ Internal energy and temperature will remain constant. 2 2
5 7
1 For diatomic gas,
= Cv =R, Cp R
(B) P ∝ 2 5
V2
∴ PV2 = constant  ……… (i) Sol 17: A → q; B → p, r; C → p, s; D → q, s
 nRT  2
or   V = constant (A) P-V graph is not rectangular hyperbola. Therefore,
 V  process A – B is not isothermal.
1
∴ T ∝  ……… (ii) (B) In process BCD, product of PV (therefore temperature
V
and internal energy) is decreasing. Further, volume is
If volume is doubled, temperature will decrease as per decreasing. Hence, work done is also negative. Hence,
equation (ii). Q will be negative or heat will flow out of the gas.
Further, molar heat capacity in process PVx = constant is (C) WABC = positive
R (D) For clockwise cycle on p- v diagram with P on y-axis,
C = Cv +
1– x net work done is positive.
From equation (i) , x = 2
Sol 18: (D) At low pressure and high temperature inter
3 R R
∴C= R+ =+ molecular forces become ineffective. So a real gas
2 1– 2 2 behaves like an ideal gas.
Since, molar heat capacity is positive, according to
Q = nCDT, Q will be negative if DT is negative. Or gas
Sol 19: (A, B) V
loses heat if temperature is decreasing. 4V0 B
1
(C) P ∝
4/3
V V0 A
C
PV4/3 = constant
T0 T
 nRT  4/3 f
∴   V = constant  U = nRT
 V  2
1
∴T∝ Where f, n, R are constants. Also temperature T is same
1/3
V at A and B.
Further, with increase in volume temperature will
decrease. ∴ UA =UB

4 Also,
Here, x =
3  Vf  4V0
Also, ∆W=
AB nRT0 ln  =  nRT0 ln = nRT0 ln4
= P0 V0 ln4
3 R  Vi  V0
∴C= R+ = – 1.5 R
2 4
1–
3 Sol 20: (A) TV λ -1 = C
As molar heat capacity is negative, Q will be positive T1(5.6)2/3 = T2 (0.7)2/3 ⇒ T2 = T1(8)2/3 = 4T1
if DT is negative. Or gas gains heat with decrease in
nR∆T 9
temperature. ∴∆w(work done on the system)
= = RT
γ −1 8 1
(D) T ∝ PV
3RT
In expansion from V1 to 2V1, product of PV is increasing. Sol 21: (D) Vrms =
M
Therefore, temperature will increase. Or DU = +ve.
Further, in expansion work done is also positive.
MAr 40
Hence, Q = W + DU = +ve or, gas gains heat. Required ratio
= = = 10 3.16
=
MHe 4
1 3 . 8 0 | Kinetic Theor y of Gases and Thermodynamics

m m (3kW − P) × 3 × 3600
Sol 22: (D) PV
PV = nRT = =RTnRT = M RT 30 − 10 =
M 120 × 4.2 × 103
⇒ PM = ρRT⇒ PM = ρRT
20 × 120 × 42 2800
ρ1 P1M1 ρ1P1= P1M 1M1=  P14 × 2 M18 = 4 × 2 = 8 3kW − P
= =
3 × 36 3
= =ρ P×M
 =  ×  M = 
ρ2 P2M2 2P2  2  M 2  P23  3 2 9 3 3 9
2 

Here ρ1 and ρ2 are the densities of gases in the vessel P = 3000 - 933 = 2067 W
containing the mixture.
Sol 26: (6) P = P0 − ρgh = 98 × 103 N/m2
Sol 23: (2) Ub = 200 J, Ui =100 J
Process iaf 500 cm
Process W (in Joule) ∆U (in Joule) Q (in Joule)
500 mm
ia 0 H
af 200 200 mm
Net 300 200 500

⇒ Uf =400 Joule P0V0 = PV


Process ibf 105[A(500 − H)] = 98 × 103[A(500 − 200)]
Process W (in Joule) ∆U (in Joule) Q (in Joule) H = 206 mm
ia 100 50 150
af 200 100 300
Level fall = 206 − 200 = 6 mm
Net 300 150 450
Sol
TV γ−27:
1 (4) TV γ−1 = constant
= constant
Q bf 300 7/5−1
⇒ = =2 7/5−1
Q ib 150 7/5−1  v7/5−1 = aT  v 
TV
TV = aT    
 32   32 
∴a =4 ∴a = 4
Sol
U 24: (A, B, D)
= U n C V1 T + n CV T
= n CV T + n CV T
1 2
2

5 3 5 3
Sol 28: A → p, r, t; B → p, r; C → q, s; D → r, t
= 1× RT + 1× RT==1×4RT
2
RT + 1× RT = 4RT
2
2 2
⇒ 2C T = 4RT Process A → B → Isobaric compression
⇒ 2C T= 4RT
Vmix
V mix
Process B → C → Isochoric process
Average energy per mole =2RT ⇒ CV =2R Process C → D → Isobaric expansion
mix

Process D → A → Polytropic with TA = TD


Cmix  λmix   MHe  3 3 4 6
=     = × × =
CHe  λHe   Mmix  2 5 3 5
Sol 29: (A, B, C, D) Option (A) is correct because the
VrmsHe MH 1 graph between (0 – 100 K) appears to be a straight line
2
= = upto a reasonable approximation.
VrmsH Mhe 2
2
Option (B) is correct because area under the curve in
dQ the temperature range (0 - 100 K) is less than in range
Sol 25: (B) Rate of heat generated ==3KW
3kW (400 - 500 K.)
dt
Let at any time ‘t’, temperature of cooler = T P A C
Rate of cooling:
dT
ms= 3kW − P
dt B
30 3 V
(3kW − P)
∫ dT = ms ∫ dt
10 0
P hysi cs | 13.81

Option (C) is correct because the graph of C versus T is For process CD


constant in the temperature range (400 - 500 K)
3  3
∆Q 2= n C v ∆T=n  R  ∆T= V( ∆P)
Option (D) is correct because in the temperature range 2  2
(200 – 300 K) specific heat capacity increases with 3  1 
temperature. ∆Q 2 = × 8 × 10 −3 ×  −1 +  × 10
5
2  32 
−93
Sol 30: (C) By A1V1 = A2V2 ∆Q 2 = × 102 = −11.625 × 102
8
⇒ π(20)2 × 5 = π(1)2 V2 ⇒ V2 = 2m / s2 ∆Q net = 1750 − 1162 = 588 J

1 1
Sol 31: (A) ρ V2 = ρ V2
2 a a 2  
For given Va

ρa
V ∝
ρ

kx
Sol 32: (B or A, B, C) P (pressure of gas)= P1 +
A
kx2 (P − P )(V − V1 )
W= ∫ PdV = P1(V2 − V1 ) +
2
= P1(V2 − V1 ) + 2 1 2
2
3
∆U= nCV ∆T= (P V − P V )
2 2 2 1 1
Q= W + ∆U

5P1V1 17P1V1 P1V1


Case=
I: ∆U 3P1V
=1, W = ,Q ,U=
spring
4 4 4

9P1V1 7P1V1 41P1V1 P1V1


Case II:
= ∆U = ,W = ,Q ,U=
spring
2 3 6 3

Note: A and C will be true after assuming pressure to


the right of piston has constant value P1.

Sol 33: (C) For adiabatic process


P3V5 = constant
5
PV 3 = constant
5
γ = gas is monoatomic
3
For process AC
5  5
∆Q1= n Cp ∆T= n  R  ∆T= P.∆V
 2  2
5
∆Q1 = × 105 × (8 − 1) × 10 −3
2
∆Q1 = 17.5 × 102 J = 1750J
2017-18 100 &
op kers
Class 11 T
By E ran culty
-JE Fa r
IIT enior emie .
S fP r es
o titut
Ins

PHYSICS
FOR JEE MAIN & ADVANCED
SECOND
EDITION

Exhaustive Theory
(Now Revised)

Formula Sheet
9000+ Problems
based on latest JEE pattern

2500 + 1000 (New) Problems


of previous 35 years of
AIEEE (JEE Main) and IIT-JEE (JEE Adv)

5000+Illustrations and Solved Examples


Detailed Solutions
of all problems available

Topic Covered Plancess Concepts


Tips & Tricks, Facts, Notes, Misconceptions,
Heat Transfer Key Take Aways, Problem Solving Tactics

PlancEssential
Questions recommended for revision
14. H E AT T R A N S F E R

1. INTRODUCTION
Heat can be transformed from one place to another place by the three processes - conduction, convection and
radiation. In conduction, the heat flows from a place of higher temperature to a place of lower temperature through
a stationary medium. The molecules of the medium oscillate about their equilibrium positions more violently at
a place of higher temperature and collide with the molecules of adjacent position, thus transferring a part of
their energy to these molecules which now vibrate more violently. Thus heat can be transmitted by collision of
molecules. In metals, the conduction of heat takes place by the movement of free electrons. In the cases of liquids
and gases, the heat is transferred not only by collision but also by motion of heated molecules which carry the heat
in such media. This process is called convection. When a liquid in a vessel is heated, the lighter molecules present
in the lower layer of the liquid get heated which rise to the surface of the liquid and cold molecules at the surface
go towards the bottom of vessel. These are convection currents and are the major means of heat transport in fluids.
Radiation is mode of transfer of heat in which the heat travels directly from one place to another without the role
of any intervening medium. The heat from the sun propagates mostly through vacuum to reach the earth by the
process of radiation.

2. CONDUCTION
The figure shows a rod whose ends are in thermal contact with a hot reservoir at T1 > T2
temperature T1 and a cold reservoir at temperatureT2 . The sides of the rod are insulated, T1 T2
hence heat transfer is only along the rod and not through its sides. The molecules at the 0
hot reservoir have greater vibrational energy. This energy is transferred by collisions to
the atoms at the end face of the rod. These atoms in turn transfer the energy to their
neighbors further along the rod. Such transfer of heat through a substance in which (Hot) (Cold)
heat is transported without direct mass transport is called conduction.
Figure 14.1
The quantity of heat conducted Q in time t across a slab of length L, area of cross-
section A and steady state temperature θ1 and θ2 at respective hot and cold ends is
kA(θ1 − θ1 )t
given by Q = , where k is the coefficient of thermal conductivity which is equal to the quantity of heat
L
flowing per unit time through unit area of cross-section of a material per unit length along the direction of flow of

heat.
Units of k are kilocalorie/meter second degree centigrade or J.m-1sec-1 K-1. In C.G.S. units,
k is expressed in calcm-1 (℃)-1 sec-1 1 2

dθ L
The temperature Gradient ⁄ (unit distance) = −
dx Figure 14.2
1 4 . 2 | Heat Transfer

 dθ  ∆Q dT
∴ Q = −kA   t ; = − kA
dx
  ∆ t dx
The quantity dT/dx is called the temperature gradient. The minus sign indicates that dT/dx is negative along the
direction of the heat flow, i.e., heat flows from a higher temperature to a lower one.
dT ∆t ∆T
= H= =
dx L / kA R
L
Here ∆T = temperature difference (TD) and R= = Thermal resistance of the rod.
kA

PLANCESS CONCEPTS

This relation is mathematically equivalent to Ohm’s Law and can be used very effectively in solving
problems effectively by considering temperature analogous to potential and heat transferred per unit
time as current.
Nivvedan (JEE 2009, AIR 113)

Heat flow through a conducting rod Current flow through a resistance

dQ dq
Heat current H =
=Rate of heat flow = Rate of charge flow
Electric current i =
dt dt
∆T T(temp diff) ∆V PD(potential diff) i
H =
= =i = ; R=
R R R R σA
L
where R= and k = Thermal conductivity σ = Electrical conductivity.
kA

3. GROWTH OF ICE ON PONDS


When temperature of the atmosphere falls below 0°C, the water in the pond starts freezing. Let at time t thickness
of ice in the pond is y and atmospheric temperature is -T°C. The temperature of water in contact with the lower
surface of ice will be 0°C.

dQ  dm  TD d
Using = L  ; = L {Aρy} (A = Area of pond)
dt  dt  R dt

00−−((−−TT)) dy
dy dy kT 1
∴
∴  =
LA
=LAρρ.. ∴− = . where L -> Latent heat of fusion
((yy kA
kA)) dt
dt dt ρ Ly

ρL y 1 ρL 2
And hence time taken by ice to grow a thickness y t =
kT ∫0
ydy or t = y
2 kT
Time does not depend on the area of pond.

PLANCESS CONCEPTS

Time taken by ice to grow on ponds is independent of area of the pond and it is only dependent only
the thickness of ice sheet.
Vaibhav Krishnan (JEE 2009, AIR 22)
P hysi cs | 14.3

4. SERIES AND PARALLEL CONNECTION OF RODS

4.1 Series Connection


Consider two rods of thermal resistances R1 R2
R1 and R2 joined one after the other as T1 A B T2
shown in figure. The free ends are kept T
at temperatures T1 and T2 with T1>T2. In
Figure 14.3
steady state, any heat that goes through
the first rod also goes through the second
rod. Thus, the same heat current passes through the two rods. Such a connection of rods is called a series connection.
Suppose, the temperature of the junction is T, the heat current through the first rod is,
∆Q T1 − T
=i = or T1 − T =
R1i  ... (i)
∆t R1
∆Q T − T2
and that through the second rod =
is i = or T − T2 =
R 2i  … (ii)
∆t R2
T −T
Adding (i) and (ii) T1 − T2 = (R1 + R 2 ) i or i = 1 2
R1 + R 2
Thus, the two rods together is equivalent to a single rod of thermal resistance R1+R2.
If more than two rods are joined in series, the equivalent thermal resistance is given by, R = R1 + R 2 + R 3 + ...

4.2 Parallel Connection


Now, suppose the two rods are joined at their ends as shown in figure. R1
The left end of both the rods are kept at temperature T1 and the right A
ends are kept at temperature T2.
T1 T2
So the same temperature difference is maintained between the ends of
B
each rod. Such a connection of rods is called a parallel connection. The
∆Q1 T1 − T2 R2
heat current going through the first rod =
is i1 =
∆t R1 Figure 14.4
∆Q 2 T1 − T2
and that through the second rod =is i2 =
∆t R2
 1 1 
The total heat current going through the left end is i = i1 + i2 = (T1 − T2 )  + 
 R1 R 2 
T −T
or i = 1 2
R
1 1 1
Where = +  … (i)
R R1 R 2

5. RADIAL FLOW OF HEAT THROUGH A CYLINDRICAL TUBE


Consider a cylindrical tube of length l and respective inner and outer radii as r1 and r2. If the heat flows radially i.e.,
perpendicular to the axis of the tube from the steady state temperatures θ1 at the inner surface to the temperature
θ2 at the outer surface, then the rate of heat flowing through an element of shell lying between radius r and r+ dr

is given by ∆Q = −k ( 2πr ) where dθ is temperature difference across the shell.
dr
It can be integrated for total heat flow per second.
1 4 . 4 | Heat Transfer

2

r
r1 r+dr
1

2

Figure 14.5
∴ Total heat flowing per second,

2πk ( θ1 − θ2 ) 2πk ( θ1 − θ2 )
Q= ; Q =
r2 r 
dr ln  2 
∫ r  r1 
r1

PLANCESS CONCEPTS

No mass movement of matter occurs in conduction. Solids are better


conductors than liquids, liquids are better conductors than gases. Q1 Q2

Consider a section ab of a rod as shown in figure. Suppose Q1 heat a b


enters into the section at ‘a’ and Q2 leaves at ‘b’, then Q2 < Q1. Figure 14.6
Part of the energy Q2 - Q1 is utilized in raising the temperature of
section ab and the remaining is lost to the atmosphere through ab. If heat is continuously supplied from
the left end of the rod, a stage comes when temperature of the section becomes constant. In that case
Q1=Q2 if rod is insulated from the surroundings (or loss through ab is zero). This is called the steady state
condition. Thus, in steady state temperature of different sections of the rod becomes constant (but not
same).
Nitin Chandrol (JEE 2012, AIR 134)

Illustration 1: One face of a copper cube of edge 10 cm is maintained at 100℃ and the opposite face is maintained
at 0℃. All other surfaces are covered with an insulating material. Find the amount of heat flowing per second
through the cube. Thermal conductivity of copper is 385 Wm-1 ℃-1.  (JEE MAIN)

Sol: Always consider the A which perpendicular to the flow of heat.


The heat flows from the hotter face towards the colder face. The area of cross section perpendicular to the heat
flow is A = (10cm)
2

T − T2 (100°C − 0°C )
The amount of heat flowing per second is
∆Q
∆t
= KA 1 =
X
(385Wm−1
)
°C−1 × ( 0.1m) ×
2

0.1m
= 3850W.

Illustration 2: A cylindrical block of length 0.4 m and area of cross-section 0.04m2 is placed coaxially on a thin
metallic disc of mass 0.4 kg and of the same cross-section. The upper face of the cylinder is maintained at the
constant temperature of 400 K and initial temperature of the disc is 300 K. If the thermal conductivity of the
material of the cylinder is 100 watt/m-K and the specific heat of the material of the disc is 600 J/kg-K, how long
will it take for the temperature of the disc to increase to 350 K? Assume, for the purpose of calculation, the thermal
P hysi cs | 14.5

conductivity of the disc to be very high and the system to be thermally insulated except for the upper face of the
cylinder. 
 (JEE ADVANCED)
400 K

0.4n

300 K

Figure 14.7

Sol: Write the equation rate of heat transfer at any temperature ‘T’ for the disc. Rate of heat transfer proportional
to rate of change in temperature.
As heat is conducted from the cylinder to the disc, the temperature of the disc increases. If the temperature of the
dQ KA ( 400 − T )
disc at some instant is T, then rate of flow of heat through the cylinder at that instant is =  … (i)
dt L
If dT is the further increase in the temperature of the disc in the infinitesimal time interval dt,
dQ dT
then = ms  … (ii)
dt dt
Where m is the mass of the disc and c is its specific heat.
From equations (i) and (ii)

KA ( 400 − T ) dT msL  dT 
= ms ; dt =  
L dt KA  400 − T 

350
msL dT msL  400 − 300 
Integrating we get, =t ∫ = × 2.303log10  
KA 300 400 − T KA  400 − 350 

0.4 × 600 × 0.4



= × 2.303 × =
0.3010 166s.
10 × 0.04

6. CONVECTION
In this process, actual motion of heated material results in transfer of heat from one place to another. For example,
in a hot air blower, air is heated by a heating element and is blown by a fan. The air carries the heat wherever it
goes. When water is kept in a vessel and heated on a stove, the water at the bottom gets heated due to conduction
through the vessel’s bottom. Its density decreases and consequently it rises. Thus, the heat is carried from bottom
to the top by the actual movement of the parts of the water. If the heated material is forced to move, say by
a blower or by a pump, the process of heat transfer is called forced convection. If the material moves due to
difference in density, it is called natural or free convection.
1 4 . 6 | Heat Transfer

PLANCESS CONCEPTS

The convection currents created in a room by a radiator means that the warm air is circulated around and
the warming is more uniform than just being the air around the radiator. When heating water on a stove,
the convection currents created by the rising hot water means that all the water gets heated instead of
just the water at the very bottom of the pan. Some rainfall is also caused by moist air being heated and
rising, then cooling quickly and allowing the water vapor to condense into rain.
Anand K (JEE 2011, AIR 47)

7. RADIATION
The third means of energy transfer is radiation which does not require a medium. The best known example of this
process is the radiation from Sun. All objects radiate energy continuously in the form of electromagnetic waves.
The rate at which an object radiates energy is proportional to the fourth power of its absolute temperature. This is
known as the Stefan’s law and is expressed in equation form as P = σAeT 4
Here P is the power in watts(J/s) radiated by object, A is the surface area in m2 ,e lies between 0 and 1 and
is called emissivity of the object and σ is universal constant called Stefan’s constant, which has the value,
=σ 5.67 × 10−8 W / m2 − K 4 .

8. PERFECTLY BLACK BODY


A body that absorbs all the radiation incident upon it and has as emissivity equal
to 1 is called a perfectly black body. A black body is also an ideal radiator. It implies
that if a black body and an identical another body is kept at the same temperature,
then the black body will radiate maximum power as is obvious from equation
P = σAeT 4
This is also because e=1 for a perfectly black body while for any other body,
e<1.
Figure 14.8

PLANCESS CONCEPTS

Always remember that black body is a perfect absorber and emitter of light. At temperatures higher than
the surrounding, it is the most shining thing and at lower temperatures it is the darkest thing.
There is no perfect black body. Materials like black velvet or lamp black come close to being ideal black
bodies, but the best practical realization of an ideal black body is a small hole leading into a cavity, as
this absorbs 98% of the radiation incident on them.
GV Abhinav (JEE 2012, AIR 329)

Illustration 3: A solid copper sphere of density ρ, specific heat c and radius r is at temperature T1. It is suspended
inside a chamber whose walls are at temperature 0K. What is the time required for the temperature of sphere to
drop to T2? Take the emissivity of the sphere to be equal to e. (JEE MAIN)

Sol: Heat lost by radiation cause temperature to fall.


P hysi cs | 14.7

dT dT
The rate of loss of energy due to radiation, P = σAeT 4 . This rate must be equal to mc Hence, −mc σAeT 4
=
dt dt
Negative sign is used as temperature decreases with time. In this equation,
T
ρcr  1 1 
1
4 3 2 dT 3eσ 4 ρcr 2 dT
m =  πr  ρ and A= 4 πr ∴ − = T or − ∫ dt = ∫= ; t  − 
3  dt ρcr 3eσ T T 4 9eσ  T3 T3 
0 1  2 1 

9. ABSORPTIVE POWER ‘a’


“It is defined as the ratio of the radiant energy absorbed by a body in a given time to the total radiant energy
incident on it in the same interval of time.”
Energy absorbed
a=
Energy incident

As a perfectly black body absorbs all radiations incident on it, the absorptive power of perfectly black body is
maximum and unity.

10. SPECTRAL ABSORPTIVE ‘aλ’


This absorptive power ‘a’ refers to radiations of all wavelengths (or the total energy) while the spectral absorptive
power is the ratio of radiant energy absorbed by a surface to the radiant energy incident on it for a particular
wavelength λ. It may have different values for different wavelengths for a given surface. Let us take an example,
suppose a = 0.6, aλ = 0.4 for 1000 Å and aλ = 0.7 for 2000 Å for a given surface. Then it means that this surface
will absorb only 60% of the total radiant energy incident on it. Similarly it absorbs 40% of the energy incident
on it corresponding to 1000 Å and 70% corresponding to 2000 Å. The spectral absorptive power aλ is related to

absorptive power a through the relation
= a ∫0 aλ dλ

11. EMISSIVE POWER ‘e’


(Don’t confuse it with the emissivity e which is different from it, although both have the same symbols e).
“For a given surface it is defined as the radiant energy emitted per second per unit area of the surface.” It has the
units of W / m2 or J/s-m2 , for a black body e = σT 4
Note: Absorptive power is dimensionless quantity where emissive power is not.

12. SPECTRAL EMISSIVE POWER


Similar to the definition of the spectral absorptive power, it is emissive power for a particular wavelength λ.

Thus,
= e ∫0 eλ dλ

13. KIRCHHOFF’S LAW


The ratio of emissive power to absorptive power is the same for all bodies at a given temperature and is equal to
the emissive power E of a blackbody at that temperature. Thus,
E(body)
= E(blackbody)
a(body)
1 4 . 8 | Heat Transfer

Kirchhoff’s law tells that if a body has high emissive power, it should also have high absorptive power to have the
ratio e/a same. Similarly, a body having low emissive power should have low absorptive power. Kirchhoff’s law may
be easily proved by a simple argument as described below.
Consider two bodies A and B of similar geometrical shapes placed in an enclosure. Suppose A is any random body
and B is a blackbody. In thermal equilibrium, both the bodies will have the same temperature as the temperature
of the enclosure. Suppose an amount ∆U of radiation falls on the body A in a given time ∆t. As A and B have the
same geometrical shapes, the radiation falling on the blackbody B is also ∆U. The blackbody absorbs all of this ∆U.
As the temperature of the blackbody remains constant, it also emits an amount ∆U of radiation in that time. If the
emissive power of the blackbody is e0, we have ∆U ∝ E0 or ∆U = kE0  ... (i)
where k is constant.
Let the absorptive power of A be a. Thus, it absorbs an energy of a ∆U of the radiation falling on it in time ∆t. As
its temperature remains constant, it must also emit the same energy a ∆U in that time. If the emissive power of the
body A is e, we have a ∆U=ke ... (ii)
The same proportionality constant k is used in (i) and (ii) because the two bodies have identical geometrical shapes
and radiation emitted in the same time ∆t is considered.
From ( i ) and ( ii ),
E E E(body)
a= or = E0 or = E(blackbody)
E0 a a(body)

PLANCESS CONCEPTS

It can be thought like, good absorber is a good emitter because at some point of time, it might have
stored energy because it is a good absorber. Now as soon as the temperature of the surrounding
becomes low than that of the body, this energy starts decreasing until the steady state is reached. Hence,
it must be a good emitter too.
Good absorbers for a particular wavelength are also good emitters of the same wavelength.
Anurag Saraf (JEE 2011, AIR 226)

14. STEFANS-BOLTZMANN LAW


The energy of thermal radiation emitted per unit time by a blackbody of surface area A is given by u = σAT 4  … (i)
Where is a universal constant known as Stefan Boltzmann constant and T is its temperature on absolute scale.
The measured value of σ is 5.67×1-8 Wm-2 K-4. Equation (i) itself is called the Stefan-Boltzmann law. Stefan had
suggested this law based on his experimental data on radiation and Boltzmann derived it from thermo dynamical
analysis. The law is also quoted as Stefan’s law and the constant σ as Stefan constant.

A body which is not a blackbody, emits less radiation than given by equation (i). It is, however, proportional to T 4 .
The energy emitted by such a body per unit time is written as u= eσAT 4  … (ii)
Where e is a constant for the given surface having a value between 0 and 1. This constant is called the emissivity
of the surface. It is zero for completely reflecting surface and is unity for a blackbody.
E(body)
Using Kirchhoff’s law =a  … (i)
E(blackbody)
Where a is the absorptive power of the body. The emissive power E is proportional to the energy radiated per unit
eσAT 4
time, that is, proportional to u. Using above equations, = a or e=a.
σAT 4
P hysi cs | 14.9

Thus, emissivity and absorptive power have the same value.


Consider a body of emissivity e kept in thermal equilibrium in a room at temperature T0 .
The energy of radiation absorbed by it per unit time should be equal to the energy emitted by it per unit time. This
is because the temperature remains constant. Thus, the energy of the radiation absorbed per unit time is u= eσAT04 .
Now suppose the temperature of the body is changed to T but room temperature remains T0 . The energy of the
thermal radiation emitted by the body per unit time is u= eσAT 4 .

The energy absorbed per unit time by the body is u0 = eσAT04 .


Thus, the net loss of thermal energy per unit time is ∆u = u − u0 = eσA(T 4 − T0 4 )  … (iii)

Illustration 4: A blackbody of surface area 10cm2 is heated to 127℃ and is suspended in a room at temperature
27℃. Calculate the initial rate of loss of heat from the body to the room.  (JEE MAIN)

Sol: Heat lost by radiation and gained by absorption.


For a blackbody at temperature T, the rate of emission is u = σAT 4 . When it is kept in a room at temperature T0 ,
the rate of absorption is u0 = σAT04 .
σA(T 4 − T0 4 )
The net rate of loss of heat is u − u0 =

= 10 × 10−4 m2 T = 400K T0 = 300K


Here A

Thus, u − u0= (5.67 × 10−8 Wm−2K −4 )(10 × 10−4 m2 )(400 4 − 300 4 )K 4 = 0.99W

Illustration 5: Energy falling on 1.0 area placed at right angles to a sun beam just outside the earth’s atmosphere
is 1.35 K joule in one second. Find sun’s surface temperature. Mean distance of earth from sun is 1.50 × 108 km ,
mean distance of sun= 1.39 × 106 km and Stefan’s constant= 5.67 × 10−8 watt m−2K −4 .  (JEE MAIN)

Sol: σ Asun T 4 =
S × Aearth
The temperature of the sun is given by
2
4 S R 
T =  
σ r 

S 1.35kJ m2 − sec 135 × 103 watt m2


= = = 2.38 × 1010 K 4
8 2 2
σ 5.67 × 10 watt m − K

5.67 × 10 watt m − K 4
−8 2

R 1.50 × 108 km
= = 215.8
r 0.695 × 106 km

∴ T 4= (2.38 × 1010 K 4 )(215.8)


= 2
1108 × 1012 K 4
=T 5.770 × 103 K or T = 5770K

15. NEWTON’S LAW OF COOLING


The rate of cooling of a body is directly proportional to the difference of temperature of the body over its
surroundings.
If a body at temperature θ1 is placed in surroundings at lower temperature θ2, the rate of cooling is given by
dQ
∝ ( θ1 − θ2 ) where dQ is the quantity of heat lost in time dt.
dt
1 4 . 1 0 | Heat Transfer

dQ
Newton’s law of cooling gives = −k ( θ1 − θ2 ) where k is constant.
dt
dQ dθ
If a body of mass m and specific heat s loses a temperature dθ in time dt, then = ms = −k(θ1 − θ2 )
dt dt

Illustration 6: A liquid cools from 70℃ to 60℃ in 5 minutes. Calculate the time taken by the liquid to cool from
60℃ to 50℃, if the temperature of the surrounding is constant at 30℃.  (JEE MAIN)

Sol: Use newton’s law cooling and taking temperature of the body is average of initial and final value.
70°C + 60°C
The average temperature of the liquid in the first case is θ1 = = 65°C
2
The average temperature difference from the surrounding is θ1 − θ0= 65°C − 30°C= 35°C .
dθ1 70°C − 60°C
The rate of fall of temperature is − = 2°Cmin−1 .
=
dt 5mins
2
From Newton’s law of cooling, 2°Cmin−1 =bA(35°C) Or bA =  … (i)
35min
60°C + 50°C
In the second case, the average temperature of the liquid is θ2 = = 55°C
2
So that, θ2 − θ0= 55°C − 30°C= 25°C
dθ 60°C − 50°C 10°C
If it takes a time t to cool down from 60℃ to 50℃, the rate of fall in temperature is =
− 2 = .
dt t t
10°C 2
From Newton’s law of cooling and (i), = × 25°C Or t = 7min.
t 35min

Illustration 7: At midnight, with the temperature inside your house at 70℉ and the temperature outside at 20℉,
your furnace breaks down. Two hours later, the temperature in your house has fallen to 50℉. Assume that the
outside temperature remains constant at 20℉. At what time will the inside temperature of your house reach 40℉?
 (JEE ADVANCED)

Sol: Newton’s law of cooling, follow logarithm curve in cooling.


The boundary value problem that models this situation is
dT T(0) = 70
= k(20 − T)
dt T(2) = 50
t2
3
Where time 0 is midnight. The solution of this boundary value problem is=
T 20 + 50  
5
This is obtained by solving above differential equation.
Note (for the purpose of a reasonableness check) that this formula given us
02 22
3 3
T(0) =
20 + 50   70. and T(2) =
= 20 + 50   50.
=
5 5
t 2
3
To find when the temperature in the house will reach 40℉, we must solve equation 20 + 50   40
=
5
 ln(2 5) 
The solution of this equation
= is t 2   ≈ 3.6
 ln(3 5) 
Thus, the temperature in the house will reach 40℉ a little after 3.30 a.m.
P hysi cs | 14.11

PLANCESS CONCEPTS

Newton’s law of cooling can also be thought in the context of Stefan-Boltzmann law by considering
the temperature difference between the body and the surroundings very close to zero, i.e. it can be
considered as a special case of the latter.
Vijay Senapathi (JEE 2011, AIR 71)

16. WIEN’S DISPLACEMENT LAW


At ordinary temperatures (below about 600℃), the thermal radiation emitted by bodies is invisible, most of them
lie in wavelengths longer than visible light. The figure shows how the energy of a black body radiation varies
with temperature and wavelength. As the temperature of the black body increases, two different behaviors are
observed. The first effect is that the peak of the distribution shifts to shorter wavelengths. This shift is found to
satisfy the following relationship called Wien’s displacement law.

λmaxT=b . Here b is a constant called Wien’s constant. The value of this constant in SI unit is 2.898 × 10−3 m-K. Thus,
λmax α1/ T
Here λmax is the wavelength corresponding to the maximum spectral emissive power eλ.
The second effect is that the total amount of energy the black body emits per unit area per unit time (=σT4)
increases with fourth power of absolute temperature T.
This is also known as emissive power. We know

=e ∫0 e=
λ dλ Area under graph, eλ Vs λ = σT 4
Area ∝ T 4 =
A2 (2)
= 4
16A1
Thus, if the temperature of the black body is made two fold, λmax remains half while the area becomes 16 times.

PLANCESS CONCEPTS

Have you ever wondered how do scientists calculate the temperature of sun and other stars? It is through
this law.
Ankit Rathore (JEE Advanced 2013, AIR 158)

Illustration 8: The light from the sun is found to have a maximum intensity near the wavelength of 470 nm.
Assuming that the surface of the sun emits as a blackbody, calculate the temperature of the surface of the sun.
 (JEE MAIN)
Sol: Formula of Wien’s displacement law.
0.288 cmK
For a blackbody, λm T=0.288 cmK. =
Thus, T = 6130K
470nm

Illustration 9: What is the wavelength of the brightest part of the light from our next closest star, Proxima Centauri?
Proxima Centauri is a red dwarf star about 4.2 light years away from us with an average surface temperature of
3,042 Kelvin?  (JEE MAIN)

Sol: λmax T =
b
1 4 . 1 2 | Heat Transfer

We don’t really need the distance to solve this. All we need is the surface temperature to plug into our Wien’s law
equation
0.0029meters − K
Wavelength λmax in meters = which is 0.000000953 meters.
3.042K
We can convert this to nanometers and we get a peak wavelength of 953 nm.

Illustration 10: Two bodies A and B have thermal emissivity of 0.1 and 0.81 respectively. The outer surface areas
of the two bodies are identical. These two bodies emit total radiative power at the same rate. The wavelength λB
corresponding to the maximum spectral radiancy in the radiation from B is 1.0 µm larger than the wavelength λ A
corresponding to the maximum spectral radiancy in the radiation from A. If the temperature of body A is 5802 K,
find (a) temperature of (B) and (b) λB .  (JEE MAIN)

e e

A2
T 2T
A1
 
m m
2
Figure 14.9

Sol: By equating their emissive power, ratio of temperatures (a) could be calculated.
(a) Power radiated from A= PA= EA A= eA σTA4 A
= PB= EA A= eB σTB4 A
Power radiated from B 
Where A is surface area of both the bodies as P1 = P2 , eA TA4 = eB TB4
4
T   0.01  1  TB  1  1 1
∴ 0.01TA4 = 0.81TB4 ∴ B  =   =  ; =   or TB = × TA = × 5802 =1934K
 TA   0.81  81 TA  3  3 3

(b) λm T = constant as per Wien’s law


λB TA λ λB 2λB
∴ λ A TA = λB TB or = = 3 ; λ A =B ; λB − λ A =1µm, λB − = =1µm
λ A TB 3 3 3
1× 3
∴ λB= = 1.5µm
2

17. SOLAR CONSTANT AND TEMPERATURE OF SUN


Solar constant is defined as the amount of radiation received from the sun at the earth per minute per cm2 of a
surface placed at right angle to the solar radiation at a mean distance of the earth from the sun. Assuming that the
absorption of solar radiation by the atmosphere near the earth is negligible, the value of solar constant, S, is equal
to 1.94 cal.cm−2 min−1 .
2
S R 
The temperature of the sun, T, is given as follows T 4 =  
σ r 
Where S is solar constant, σ is Stefan’s constant, R is mean distance of earth from sun and r is radius of sun.
P hysi cs | 14.13

PROBLEM-SOLVING TACTICS
 1. Problems of conduction can be easily solved by making analogy with current electricity (Problems like
calculation of net conductance of series and parallel connection. Actually, the way in which steady state is
achieved in heat transfer and current electricity is very similar. At steady state considering a cylindrical rod,
potential at each point becomes constant in current electricity and so does temperature in heat transfer.
The amount of charge transferred per unit time is related in same way to potential as that of heat energy
transferred relates to temperature difference and the constant of proportionality have similar properties.)
 2. Most of the problems involve concepts of integration, so be careful with infinitesimal elements. Basically, try
to be physically involved in the problem and understand it event by event so that you learn more. Toughness
in most of the questions is involved only in its mathematical analysis.
 3. Problems from radiation and law of cooling also generally involve integration which becomes necessary to
do at times. However an approximate approach is also available in case of law of cooling useful in solving
problems without involving integration.
 4. Laws must be carefully known because many questions directly focus on understanding of laws rather than
involving calculations (Example - If temperature of a body is doubled, find the ratio of maximum wavelength
for final and initial state.)
 5. Noting down the known and asked quantities and thinking of a link between them will always prove to be a
good way.
 6. Questions from this topic usually come in a hybrid involving concepts of other topics like thermodynamics,
gaseous state and calorimetry. So one must be strong in their concepts too!!

FORMULAE SHEET

S. No. Term Descriptions


1. Conduction Due to vibration and collision of medium particles.
2. Steady state In this state heat absorption stops and temperature gradient throughout the rod becomes
dT
constant i.e. = const ant.
dx
3. Before steady state Temp of rod at any point changes.

Note: If specific heat of any substance is zero, it can be considered always to be in steady
state.
4. Ohm’s law for thermal Let the two ends of rod of length L is maintained T1 T2
Conduction in Steady
state (
At temp T1 and T2 T1 > T2 ) L
dQ T1 − T2 L
Thermal Current = . Where R Th =
dT R Th KA
(L is length of material, K is coefficient of thermal conductivity, A is area of cross- section)
5. Differential form of T T-dT
dQ dT
Ohm’s law = KA
dT dx
dT
= Temperature gradient
dx dx
6. Convection Heat transfer due to movement of medium particles.
1 4 . 1 4 | Heat Transfer

7. Radiation Every body radiates electromagnetic radiation of all possible wavelength at all temp>0 K

8. Stefan’s Law
Rate of heat emitted by a body at temp T K from per unit area E = σT 4 J / sec/ m2
dQ
Radiation power = P = σAT 4 watt
dT dQ
If body is placed in a surrounding of temperature Ts σA(T 4 − Ts 4 ) valid only for
=
black body
dT
heat from general body
Emissivity or emmisive power e =
heat from black body
If temp of body falls by dT in time dt

=
dT eAσ 4
dt ms
( )
T − Ts 4 (dT/dt=Rate of cooling)

9. Newton’s law of If temp difference of body with surrounding is small i.e.


cooling
T = Ts Then,
=
dT 4eAσ 3
dt
T T − Ts
ms s
( ) So dT
dt
(
∝ T − Ts )
10. Average form of
If a body cools from T1 to T2 in time δt
Newton’s law of
cooling T1 − T2 K  T1 + T2  dT K
=  − TS  (Used generally in objective questions)= (T − TS )
δt mS  2  dt mS
(For better results use this generally in subjective )
11. Wien’s black body At every temperature (>0K) a body radiates energy radiations of all wavelengths.
radiation According to Wien’s displacement law if the wavelength I
corresponding to maximum energy is λm then λm T=b T3
> >
T2 T3 T2 T1
where b= is a constant( Wien’s Constant )

T=Temperature of body T2

m3
m2
m3

Solved Examples

JEE Main/Boards Sol: Recall the formula of heat transfer.


(a) Thermal resistance
Example 1: A copper rod 2 m long has a circular
1 1 2
cross section of radius 1 cm. One end is kept at 100℃ R
= == or R = 15.9K / W
and other at 0℃, and the surface is insulated so that kA k( π r )
2
(401)( π)(10−2 )2
negligible heat is lost through the surface. Find ∆T ∆θ 100
(b) Thermal current,=
H = = or
(a) The thermal resistance of bar R R 15.9
(b) The thermal current H H = 6.3W
dT (c) Temperature gradient
(c) The temperature gradient
dx
(d) The temperature 25 cm from hot end. Thermal 0 − 100
= =−50K / m =−50°C / m
conductivity of copper is 401 W/m-K 2
P hysi cs | 14.15

(d) Let be θ°C the temperature at 25 cm from hot end conductivity of copper is 401 W/m-K and that of
then aluminium is 237 W/m-K
o
100 C oC 0oC
o o
100 C 20 C
0.25 m
2.0 m
(θ – 100)=(Temperature gradient) × (Distance)
θ − 100 = ( −50)(0.25) Sol: This is parallel combination and thermal current
θ 87.5°C
= would be sum of both cubes.
(a) Thermal resistance of aluminum cube
Example 2: In a murder investigation, a corpse was 1 (3 × 10 −2 )
found by a detective at exactly 8 P.M. Being alert, the = R1 = or R1 = 0.14K / W
kA (237)(3 × 10−2 )2
detective also measured the body temperature and
found it to be 70℉. Two hours later, the detective and Thermal resistance of aluminum cube
measured the body temperature again and it found to
be 60℉. If the room temperature is 50℉, and assuming (3 × 10 −2 )
= R2 = 0.08K / W
that the body temperature of the person before death (401)(3 × 10 −2 )2
was 98.6℉, at what time did the murder occur?
As these two resistances are in parallel, their equivalent
resistance will be
Sol: Newton’s law of cooling is used.
With time 0 taken to be 8 P.M., we have the boundary= R1R 2 (0.14)(0.08)
R = = 0.05K / W
value problem R1 + R 2 (0.14) + (0.08)
Temperature difference
T(0) = 70 Thermal Current H =
dT Thermal resistance
= k(50 − T);
dt T(2) = 60
(100 − 20)
t2 = = 1.6 × 103 W
 1 0.05
Whose solution is = T 50 + 20  
2 (b) In parallel thermal current distributes in the inverse
ratio of resistance.
We would like to find the value of t for which T(t)=98.6.
Solving the equation H R Al R1 0.14
Hence, Cu = = = = 1.75
t 2 HAl R Cu R 2 0.08
 1
50 + 20   = 98.6
2 Example 4: One end of a copper rod of length 1 m
 ln(48.6 20)  and area of cross section 4.0 × 10−4 m2 is maintained at
Given us t 2 
=  ≈ −2.56. 100℃. At the end of rod ice is kept at 0℃. Neglecting
 ln(1 2) 
the loss of heat from the surroundings, find the mass
It appears that this person was murdered at about 530 of ice melted in 1 h. Given= k cu 401W m − K and
P.M. or so. 5
L f 3.35 × 10 J kg .
=
t2
 1
From the function =
T 50 + 20   Sol: Find total heat transfer in 1 hr time through rod
2
and hence, melted ice can be found.
Over the time interval −2.56. ≤ t ≤ 2.56.
Thermal resistance of the rod,
Example 3: Two metal cubes with 3 cm edges of copper o
100 C 0oC
and aluminium are arranged as shown in fig. find H
(a) The total thermal current from one reservoir to the 1 1.0
R
= = = 6.23K W
other kA (401)(4 × 10−4 )
(b) The ratio of the thermal current carried by the Temperature difference
copper cube to that of the aluminium cube. Thermal Heat Current H =
Thermal resistance
1 4 . 1 6 | Heat Transfer

(100 − 0) Substituting the values


= = 16W
6.23 1
 0.01  4
=TB =  (5802) 1934K
Heat transferred in 1 h,  0.81 
 Q
= 57600 J  H = 
Q = Ht (16)(3600)
= (b) According to Wein’s displacement law,
 T
λ A TA =
λB TB
Now, let m mass of ice melts in 1 h, then
 5802 
Q ∴ λB =   λ A or λB = 3λ A
m= (Q=mL)  1934 
L
 1
57600 Also, λB − λ A = 1µm or λB −   λB =1µm
= = 0.172kg or 172g
3.35 × 105 3
Or λB = 1.5µm
Example 5: A body cools in 10 minutes from 60℃
to 40℃. What will be its temperature after next 10
Example 7: Two plates each of area A, thickness L1
minutes? The temperature of the surrounding is 10℃
and L2 thermal conductivities K1 and K2 respectively are
Sol: Think of Newton’s law of cooling. joined to form a single plate of thickness L1 + L2. If the
temperatures of the free surfaces are T1 and T2, calculate
According to Newton’s law of cooling

 θ1 − θ2   θ1 + θ2  
  = α   − θ0 
 t   2   Heat
Flow
For the given conditions, K1 K2 T2
T1
60 − 40  60 + 40 
=
α − 10   … (i) L1 L2
10  2 
(a) Rate of flow of heat
Let be the temperature after next 10 minutes.
40 − θ  40 + θ  (b) Temperature of interface
Then =
α − 10   … (ii)
10  2  (c) Equivalent thermal conductivity
Solving Eqs. ( i ) and ( ii ), we get θ= 28°C Sol: Consider as thermal current where thermal resistors
in series.
Example 6: Two bodies A and B have thermal emissivity
(a) If the thermal resistances of the two plates are R1
of 0.01 and 0.81 respectively. The outer surface areas
and R2 respectively then as plates are in series.
of the two bodies are same. The two bodies emit
total radiant power at the same rate. The wavelength L1 L2
RS = R1 + R 2 = +
corresponding to maximum spectral radiancy from AK1 AK 2
B is shifted from the wavelength corresponding to L
maximum spectral radiancy in the radiation from A by As R = and so
AK
1.0 µm. If the temperature of A is 5802 K. calculate (a)
The temperature of B, dQ ∆Q (T1 − T2 ) A(T1 − T2 )
H
= = = =
(b) Wavelength λB dt R (R1 + R 2 )  L1 L2 
 + 
 K1 K 2 
Sol: Compare the emissive power of both and then
temperature and λm of B can be calculated, Use (b) If T is the common temperature of interface then
λB − λ A = 1µm . as in series, rate of flow of heat remains same. i.e.
H H=
= 1 ( H2 )
(a) PA = PB ∴ eA σA A TA4 = eB σAB TB4
T1 − T2 T1 − T T1R 2 + T2R1
1 = i.e. T =
 eA 4 R1 + R 2 R1 (R1 + R 2 )
∴ TB =
  TA … (A A = AB )
 eB 
P hysi cs | 14.17

 L1 L2  Example 9: The lower surface of a slab of stone of


 T2 + T1  face-area 3600 cm and thickness 10 cm is exposed to
K1 K2 
or T= steam at 100℃. A block of ice at 0℃ rests on the upper
 L1 L2 
 +  surface of slab. 4.8 g of ice melts in one hour. Calculate
 K1 K 2  the thermal conductivity of the stone. Latent heat of
fusion of ice = 3.36 × 105 Jkg−1 .
(c) If K is the equivalent conductivity of composite slab
i.e. slab of thickness L1 + L2 and cross sectional area A, Sol: Amount of heat transfer per second would be used
then as in series to melt the mass of ice per second.

R= R1 + R 2 or
(L1 + L2=) R1 + R 2 The amount of heat transferred through the slab to the
S
AK eq ice in one hour is

=K eq =
(L1 + L2 ) L1 + L2 As R = L Q =(4.8 × 10−3 kg) × (3.36 × 105 Jkg−1 )
A(R1 + R 2 )  L1 L2  AK = 4.8 × 336J.
 + 
 K1 K 2  Using the equation Q =
KA(θ1 − θ2 )t
x
Example 8: One end of a rod of length 20cm is 2
K(3600 cm) (100° C)(3600 s)
inserted in a furnace at 800K. The sides of the rod are 4.8 × 336J =
10cm
covered with an insulating material and the other end
emits radiation like a black body. The temperature of 1.24 × 10−3 Wm−1° C−1
or K =
this end is 750K in the steady state. The temperature
of the surrounding air is 300K. Assuming radiation is
Example 10: An icebox made of 1.5 cm thick
the only important mode of energy transfer between
Styrofoam has dimensions 60cm × 60cm × 30cm . It
the surrounding and the open end of the rod. Find the
contains ice at 0℃ and kept in a room at 40℃. Find
thermal conductivity of the rod. Stefan constant
the rate at which ice is melting. Latent heat of fusion of
= 6.0 × 10−3 W / m2 − K 4
σ ice = 3.36 × 105 Jkg−1 and thermal conductivity of
Styrofoam = 0.04Wm−1°C−1 .
750 K
Furnace
air temp Sol: Heat transfer through Styrofoam will melt the ice.
800 K
20 cm 300 K The total surface area of the walls
= 2(60cm × 60cm + 60cm × 30cm + 60cm × 30cm)
Sol: Rate of heat through radiation would be equal to = 1.44m2
rate of heat transfer through rod. The rate of heat flow into the box is
Quantity of heat flowing though the rod per second in ∆Q KA(θ1 − θ2 )
steady state =
∆t x
dQ K.A.dθ
=  … (i) (0.04 Wm−1 ° C−1 )(1.44m2 )(40° C)
dt x = = 154W
0.015m
Quantity of heat radiated from the end of the rod per
The rate at which the ice melts is
second in steady state
154W
dQ = = 0.46gs−1
=Aσ(T 4 − T04 )  … (ii) 3.36 × 105 Jkg−1
dt
From Eqs. (i) and (ii),
K.dθ
=σ(T 4 − T04 )
x JEE Advanced/Boards
K × 50
=6.0 × 10−8 (7.5)4 − (3)4  × 108
0.2   Example 1: Three rods of the material x and three rods
K = 74 W mK of material y are connected as shown in figure. All the
rods are of identical length and cross sectional area. If
the end A is maintained at 60℃ and the junction E is
1 4 . 1 8 | Heat Transfer

at 10℃ Calculate temperature of junction B, C, D. the (30 − T)


Or 15 = (30 − T) +
thermal conductivity of x is 0.92 cal/cm-s℃ and that of 2
y is 0.46 cal/cm-s℃. Solving this we get T
= 20°C or TC= TD= 20°C
C
x x Example 2: A cylinder of radius R made of a thermal
o
y 10oC
60 C
x E conductivity K1 is surrounded by cylindrical shell of inner
B radius R and another radius 2R made of a material of
y y thermal conductivity K2. The two ends of the combined
D system are maintained at two different temperatures.
There is no loss of heat across the cylindrical surface
Sol: Think of temperature drop across BCE and across
and system is in steady state. What is the effective
BDE, temperature of C and D would be same as similar
thermal conductivity of system?
drop across BC and CE,
Sol: Assume this to parallel combination of thermal
K2
resistors. As both have same temperature across their
Heat flow R 2R ends.

K1 In this situation a rod of length L and area of cross


section πR 2 and another of same length L and area
of cross-section π (2R)2 − R 2  =
3πR 2 will conduct heat
1 2  
simultaneously so total heat flowing per second will be,
1
also across BD and DE.Thermal resistance R = dQ dQ1 dQ 2
kA = +
Rx ky dt dt dt
∴ = (as lx = ly and A x = A y )
Ry kx K1πR 2 (θ1 − θ 2 ) K 2 3πR 2 (θ1 − θ 2 )
= +  ...(i)
Rx L L
0.46 1
∴ = = Now, if the equivalent conductivity is K then,
R y 0.92 2

So, if R x = R then R y = 2R dQ 4 πR 2 (θ1 − θ 2 )


=K [ As A = π(2R)2 ]…(ii)
dt L
CEDB forms a balanced Wheatstone bridge i.e.
TC = TD and no heat flows through CD So, from Eqs. (i) and (ii), we have

1 1 1 4 (K1 + 3K 2 )

∴ = + or RBE = R = K1 + 3K 2 i.e. K =
4K
RBE R + R 2R + 2R 3 4

The total resistance between A and E will be, Example 3: A point source of heat of power P is
4 10 placed at the center of a spherical shell of mean radius
R AE =R AB + RBE =2R + R= R R. the material of the shell has thermal conductivity
3 3
k. calculate the thickness of the shell if temperature
∴ Heat current between A and E is difference between the outer and inner surfaces of the
shell in steady state is T.
( ∆ T) (60 − 10) 15
=H = =
R AE (10 3)R R

Now, if TB is the temperature at B, d

15 60 − TB
( ∆ T)AB r1 P
HAB = or
= or TB= 30°C
R AB R 2R
d
15 30 − Tc 30 − TD
Further, H=AB or
HBC + HBD = + r
R R 2R
r2
(Say T=
C T=
D T)
P hysi cs | 14.19

dr  l  1 7×10−2 m dr 1 7
Sol: Total thermal resistance ∫ k 4 π r2  . Power
2πk ∫5×10 m r
= = −2 = ln  
 KA  2πk  5 
of source equal to rate of heat transfer at steady state.
1
= ln (1.4 ) = 0.765K / W
Consider a concentric spherical shell of radius r and 2π(0.07)
thickness dr as shown in figure. In steady state, the rate
Temperature difference
of heat flow (heat current) through this shell will be, Heat current H =
Thermal resis tance
∆T ( − d θ)  1 
H
= = R =  (100 − 20)
R dr  kA  = = 104.6W
0.765
(k)(4 π r 2 )
∴ Heat lost in one hour =
Heat current × time

or H =−(4 π kr 2 ) =
= (104.6)(3600) J 3.76 × 105 J
dr
Here, negative sign is used because with increase in r,
decreases. Example 5: A closed cubical box is made of perfectly
insulating material and the only way for heat to enter
r2 dr 4 πk θ2
∴ ∫r1 = − ∫θ1 dθ or leave the box is through two solid cylindrical metal
r 2 H plugs, each of cross sectional area 12 cm2 and length
4 πkr1r2 (θ1 − θ2 ) 8 cm fixed in the opposite walls of the box. The outer
This equation gives, H =
(r2 − r1 ) surface of the plug is kept at a temperature of 100℃
while the outer surface of the other plug is maintained
In steady state,
= H P,r
= 1r2 R2 and θ2 T
θ1 −= at a temperature of 4℃. The thermal conductivity of the
4 πkR 2 T material of the plug is 2.0Wm-1 ℃-1. A source of energy
∴Thickness of shell, r2 − r1 = generating 13 W is enclosed inside the box. Find the
P
equilibrium temperature of the inner surface of the box
assuming that it is the same at all points on the inner
Example 4: A steam pipe of radius 5cm carries steam
surface.
at 100℃. The pipe is covered by a jacket of insulating
material 2cm thick having a thermal conductivity 0.07
Sol: At steady state, rate of heat transfer through both
W/m-K. If the temperature at the outer wall of the pipe
plugs would be same.
jacket is 20℃, how much heat is lost through the jacket
per meter length in an hour?

1= 100oC 2= 4oC


Sol: Heat lost through curved surface of the pipe.
S
dr
R thermal = ∫ for pipe of length l.
K 2 π rl

Thermal resistance per meter length of an element at The situation is shown in figure. Let the temperature
distance r of thickness dr is inside the box be θ. The rate at which heat enters the
∆Q1 KA(θ1 − θ)
box through the left plug is =
k=0.07W/m-k ∆t x
The rate of heat generation in the box=13 W. The rate
at which heat flows out of the box through the right
7cm plug is
r dr
∆Q 2 KA(θ − θ2 )
=
5cm ∆t x
∆Q1 ∆Q 2
In the steady state + 13W =
∆t ∆t
dr 1 KA KA
dR = (R = ) or, (θ − θ) + 13 =
W (θ − θ2 )
k(2π r) kA x 1 x

2 r =7cm

∴ Total resistance R = dR
r1=5cm
1 4 . 2 0 | Heat Transfer

KA KA 2 cm. calculate the time required for the thickness to be


or, 2 KA=θ KA (θ1 + θ2 ) + 13 W doubled. Thermal conductivity ice = 0.004cal/cm/s/℃,
or, 2 x =θ x (θ + θ2 ) + 13 W
x θ + θx 1 density of ice = 0.92 g/cm3 and latent heat of ice
(13 W) x
or,
= θ θ11 + θ22 + (13 W) x = 80cal/g.
or,
= θ 2 + 2KA
2 2KA
Sol: Amount of Heat transfer through ice at any time
100°C + 4°C (13 W) × 0.08m would result in freezing the water of lake. Proceed with
= +
2 2 × (2.0 Wm−1 °C−1 )(12 × 10−4 m2 ) assuming Area of lake = A, eventually it will cancel out.
As the temperature of air is below 0℃, water begins to
= 52°C + 216.67°C = 269°C freeze to form a layer of ice. The thickness of the layer
gradually increases.
Example 6: Two thin metallic spherical shells of radii r1 Consider that a layer of thickness x has already been
and r2 (r1 < r2) are placed with their centres coinciding. formed on a lake at 0℃. Let A be the area of the layer,
A material of thermal conductivity K is filled in the L the latent heat of ice and ρ its density. The amount
space between the shells. The inner shell is maintained of heat required when the thickness of ice increases by
at temperature and the outer shell at temperature θ1 dx is
(θ1< θ2). Calculate the rate at which heat flows radially -
through the material.
x
2 x+dx

o
0C
1 X
Q mL
= = (Adx ρ)L
r1
This quantity of heat is conducted upwards through the
r2 layer in time dt when the temperature of air is −θ .
KA(0 − ( −θ)) dx Kθ ρLxdx
∴ A ρLdx = dt; = ; dt =
Sol: Heat flowing radially outward through spherical x dt ρLx Kθ
shells. Both connected in series. Time taken t for the thickness to increase from x1 and x2
to is obtained by integrating
Let us draw two spherical shells of radii x and x+dx
concentric with the given system. Let the temperatures t x
ρL 2
at these shells be θ and θ + dθ respectively. The amount =t ∫=
dt
Kθ x∫
xdx Or
of heat flowing radially inward through the material 0 1
between x and x+dx is ρL 0.92 × 80
= t (x22 − x12 ) ∴ t
= (42 − 22 )
2Kθ 2 × 0.004 × 10
∆Q K4 πx2 = 11040
= s 3.07hr
= . dQ
∆t dx

Thus, Example 8: A liquid placed in a container open to


θ r atmosphere takes 5 minutes to cool from 80℃ to 50℃.
2
∆Q 2 dx How much time will it take to cool from 60℃ to 30℃?
K4 π ∫ dθ =
θ
∆t r∫ x2 The temperature of the surroundings is 20℃.
1 1

∆Q  1 1  Sol: Newton’s law of cooling.


− θ1 ) ∆Q  1 − 1 
or, K4 π(θ2=
or, K4 π(θ2=
− θ1 ) ∆t  r − r 
∆t  r11 r22  The rate of cooling of a body at temperature T is given
∆Q K4 πr r (θ − θ ) dT
or, ∆Q = K4 πr11r22 (θ22 − θ11 ) by Newton’s law of cooling as =−K(T − T0 )
or, ∆t = r2 − r1 dt
∆t r2 − r1
Where K is a constant for the body and T0 is the
temperature of the surroundings.
Example 7: The temperature of air above a lake is
-10℃. At some instant, the thickness of ice in the lake is T − T0
= −Kdt
dT
P hysi cs | 14.21

The negative sign indicates that the temperature is  dT  2.8  6 4 − 34 


falling.   =  =  9.72°C / minute
T2 t
 dt 327°C 2  4 4 − 34 
dT
Integrating, we get ∫ T − T0
= −K ∫ dt
T1 0 Example 10: A 2m long wire of resistance 4 ohm and
 T − T0  diameter 0.64 mm is coated with plastic insulation of
loge  2  = −Kt thickness 0.06 mm. When a current of 5 ampere flows
 T −T
 1 0 
through the wire, find the temperature difference
As t =5, T1 =80°C, T2 =50°C, T0 =20°C across insulation in steady state if

1  80 − 20  K 0.16 × 10−2 cal / cm − °Cs 


=
∴ 5 = loge    
K  50 − 20 
Sol: Tricky one! Rate of heat generation in the wire due
or 5K = loge (2)  … (i) to flow of current must be same as rate of heat transfer
If t is time taken when through plastic insulation.
Considering a concentric cylindrical shell of radius r and
T1 =
60°CandT2 =
30°C
thickness dr as shown in figure. The radial rate of flow
 60 − 20  of heat through this shell in steady state will be
Kt = loge    … (ii)
2
 30 − 20 
dr
or Kt = loge (4) 1
r a b
Dividing equation (ii) by equation (i)
t loge 4 1.386
= = = 2 or t = 10 minutes
5 loge 2 0.693
dQ dθ
Example 9: A solid copper sphere cools at the rate of H= = −KA
dt dr
2.8℃ per minute, when its temperature is 127℃. Find
the rate at which another copper sphere of twice the Negative sign is used as with increase in r, θ decrases
radius will lose its temperature at 327℃, if in both the
Now as for cylindrical shell A = 2πrL
cases, the room temperature is maintained at 27℃.

Sol: Get the rate of heat loss through radiations. H =−2πrLK
dr
dQ dT b θ
The rate of loss of heat
= = ms dr −2πrLK 2
dt dt or ∫r = −
H θ∫

a 1
dT σA 4
σA(T 4 − T04 ) or
= = (T − T04 )
dt ms Which on integration and simplification gives
4 3
If r is radius of sphere is r, then m= πr × ρ
3
dQ 2πLK(θ1 − θ2 )
Where ρ is density and s is specific heat H= … (i)
= − .......(i)
dt ln( b )
dT σ × 4 πr 2 4 4 3σ a
= (T − T=
0 ) (T 4 − T04 )
dt 4 3 rρ× s I2R (5)2 × 4 cal
πr ρ× s Here,=
H = = 24
3 4.2 4.2 s
 dT  3σ
  = 2.8 = (400 4 − 300 4 )......(i)
 …(i) =
L 2m= 200cm
 dt 127°C rρ× s
= r1 (0.64
= 2) 0.032cm
For the second sphere of radius 2r and R 2 = r1 + d = 0.032 + 0.006 = 0.038

 dT 
=
 

(600 4 − 300 4 )......(ii)
…(ii) So (θ1 − θ2 ) = 32 (
24 × ln 38 )
 dt 327°C (2r)ρ× s
2 × 2.3026 log10 38 − log10 32
Dividing equation (ii) by equation (i), we get
1 4 . 2 2 | Heat Transfer

24 × 2.3026 log10 38 − log10 32


=
3.14 × 0.64

55 × 1.57 − 1.50  L
1
or (θ1 − θ2 ) = = 2°C. 0
2

Example 11: A rod CD of thermal resistance 5.0KW-1 is Sol: Rate of heat transfer is variable as temperature of
joined at the middle of an identical rod AB as shown in small vessel will be changing.
figure. The ends A, B and D are maintained at 100℃, Suppose, the temperature of the water in the smaller
0℃, and 25℃ respectively. Find the heat current in CD. vessel is at time t. In the next time interval dt, a heat dQ
is transferred to it where
A o o
B
100 C C 0C KA
Q
∆= (θ − θ)dt. … (i)
L 0 
25 C
o This heat increases the temperature of the water of
D
mass m to θ + dθ where
Sol: At point C, total thermal current inflow equal to
∆ Q= msdθ  … (ii)
total thermal current out flow.
From (i) and (ii),
The thermal resistance of AC is equal to that of CB and
is equal to 2.5KW-1. Suppose, the temperature at C is θ. KA
(θ − θ)dt
= msdθ
The heat current through AC, CB, and CD are L 0
T θ
Lms dθ Lms 2 dθ
∆Q1
=
100°C − θ
;
or, dt =
KA θ0 − θ
or, ∫ dt =
KA θ∫ θ0 − θ
0
∆t 2.5KW −1 1

Where T is the time required for the temperature of the


∆Q 2 θ − 0°C ∆Q 3 θ − 25°C water to become.
= = and
∆t 2.5KW −1 ∆t 5.0KW −1
Lms θ0 − θ1
Thus, T = ln
We also have KA θ0 − θ2

∆Q1 ∆Q 2 ∆Q 3
= + Example 13: The earth receives solar radiation at a rate
∆t ∆t ∆t of. 8.2 J cm−2 min−1 . Assuming that the sun radiates
like a blackbody, calculate surface temperature of the
100°C − θ θ − 0°C θ − 25°C
or, = + sun. The angle subtended by the sun on the earth is
2.5 2.5 5.0 0.530 and Stefan constant=σ 5.67 × 10−8 Wm−3K −4 .
or, 225°C = 5θ
Sol: Think of intensity of thermal heat out a distance R
or, θ= 45°C from the source.
∆Q 3 45°C − 25°C 20K Let the diameter of the sun be D and its distance from
Thus,
= = = 4.0W. the earth be R. From the question,
∆t 5.0KW −1
5.0KW −1
D π
= 0.53 × = 9.25 × 10−3 ...(i)
R 180
Example 12: Figure shows a large tank of water at a
constant temperature θ0 and a small vessel containing The radiation emitted by the surface of the sun per unit
a mass m of water at an initial temperature θ1 (<θ0). A time is
metal rod of length L, area cross section A and thermal
conductivity K connects the two vessels. Find the time
taken for the temperature of the water in the smaller D
vessel become θ2 (θ1<θ2 <θ0). Specific heat capacity of
water is s and all other heat capacities are negligible.
R
Sun

2
D 
4 π   σT 4 =πD2 σT 4
2
P hysi cs | 14.23

At distance R, this radiation falls on an area of 4πR2 in Example 15: A thermometer is taken from a room that
unit time. The radiation received at the earth’s surface is 20℃ to the outdoors where the temperature is 5℃.
per unit time per unit area is, therefore, After one minute, the thermometer reads 12℃. Use
2
Newton’s law of cooling to answer following questions.
πD2 σT 4 σT 4  D  (a) What will the reading on the thermometer be after
=  
4 πR 2 4 R  one more minute?
2 (b)When will the thermometer read 6℃.?
σT 4  D  −2 −1
Thus,   = 8.2Jcm min
4 R  Sol: Get the ‘k’ for Newton’s law of cooling by given
1 condition, then the all desired value.
or, 1 × (5.67 × 10−−88 Wm−−22K −−44 ) T 44 ×
or, 4 × (5.67 × 10 Wm K ) T × If T is the thermometer temperature, then Newton’s law
4
(9.25 × 10−−33 )22 x T 44 of cooling tells us that
(9.25 × 10 ) x T
dT
= k(5 − T) ; T(0) = 20.
8.2 −2 dt
= Wm
10−4 × 60 The solution of this initial value problem is
or, =T 5794K ≈ 5800K T= 5 + 15e−kt .
We still need to find the value of k. We can do this by
Example 14: On a cold winter day, the atmospheric using the given information that T(1)=12. In fact, let us
temperature is θ (on Celsius scale) which is below 0℃. pause here to consider the general problem of finding
A cylindrical drum of height h made of a bad conductor the value of k. We will obtain some facts that can be
is completely filled with water at 0℃ and is kept outside used in the rest of the problems involving Newton’s law
without any lid. Calculate the time taken for the whole of cooling.
mass of water to freeze. Thermal conductivity of ice is K -
and its latent heat of fusion is L. Neglect expansion of
water on freezing. x

Sol: Rate of heat transfer would be dependent on h o


dx
thickness of layer of ice. Write equation of heat transfer 0C
at any time ‘t’ when thickness of ice is ‘x’.
Suppose, the ice starts forming at time t=0 and a
thickness x is formed at time t. The amount of heat Suppose that we have the model
flown from the water to the surrounding in the time
interval t to t+dt is dT T(0) = T0
= k(Ts − T) ;
KAθ dt T(t1 ) = T1
∆Q = dt.
x Where t1 is some time other than O. then the first two
The mass of the ice formed due to the loss of this equations in the model, we obtain T =Ts + (T0 − Ts )e−kt
amount of heat is and from the third equation we obtain
−kt1
∆Q KAθ Ts + (T0 − Ts )e T1
=
dm
= = dt.
L xL −kt1
The thickness dx of ice formed in time dt is Thus, (T0 − Ts )e T1 − Ts
=

dm KAθ ρL which gives us


dx
= = dt or, dt = xdx.
Aρ ρxL Kθ −kt T1 − Ts kt T0 − Ts
= e 1 = or e 1
T0 − Ts T1 − Ts
Thus, the time T taken for the whole mass of water to
freeze is given by 1  T −T 
or k = ln  0 s 
t1  T1 − Ts 
T h
ρL ρLh2
∫ dt = Kθ ∫ xdx or, T=
2Kθ The latter equations give us the value of k. However,
0 0
note that, in most problems that we deal with, it is not
really necessary to find the value of k. Since the term
1 4 . 2 4 | Heat Transfer

e−kt that appears in the solution of Newton’s Law of To find what the thermometer will read two minutes
−kt − kt t t after being taken outside, we compute
cooling can be written as e = (e 1 ) 1
2
We really just need (in most situations) to know the  7 
T(2) =
5 + 15   ≈ 8.3.
− kt
value of e 1 , and this value has been obtained in the  15 
work done above. In particular, the solution of Newton’s This tells us that the thermometer will read about 8.3℃
Law of Cooling, two minutes after being taken outside.
T = Ts + (T0 − TS )e−kt , Finally, to determine when the thermometer will read
6℃, we solve the equation
Can be written as t
− kt1 t t1  7 
T = Ts + (T0 − TS )(e ) 5 + 15   = 6
 15 
t t1
T −T  The step-by-step solution of this equation is
or as T = Ts + (T0 − TS )  1 s 
T −T
 0 s  t t
 7   7  1
t t1 15   = 1   =
T −T   15   15  15
T = Ts + (T0 − TS )  1 s 
T −T
 0 s    7 t   1 7  1
ln     = ln   ; tln   = ln  
Returning now to the problem at hand (with the   15    15   15   15 
thermometer), we see that temperature function for  
t ln(1 15)
 7  = t ≈ 3.5.
the thermometer is T= 5 + 15   . ln(7 15)
 15 
Note that this makes sense because this formula gives Thus, the thermometer will reach 6℃ after being
0 outside for about 3.5 minutes.
 7 
us T(0) =5 + 15   = 20.
 15 
1
 7 
And T(1) =
5 + 15   = 12.
 15 

JEE Main/Boards

Exercise 1 Q.6 What are the thermal radiations? From where do


you obtain them? How do they transfer from one place
to another?
Q.1 Which metal is the best conductor of heat?

Q.7 Discuss the variation of temperature of the hot


Q.2 Which mode of transfer of heat is quickest? body with time during cooling process. What do you
conclude from this?
Q.3 What is temperature gradient?
Q.8 What is meant by thermal conductivity and its
Q.4 How can heat be transferred from one place to coefficient? What are its SI units and CGS units?
other?
Q.9 Explain Newton’s law of cooling and discuss its
Q.5 What are the basic differences between conduction, experimental verification.
convection and radiation?
Q.10 Thickness of ice on a lake is 5 cm. and the
temperature of air is -20℃. If the rate of cooling of
P hysi cs | 14.25

water inside the lake be 20000 cal min-1 through each the temperature and distance are doubled, the power
square meter surface, find K of ice? received by the foil will be
(A) 16 P (B) 4 P (C) 2 P (D) P
Q.11 A metal plate 4 mm thick has a temp difference of
32℃ between its faces. It transmits 200kcal h-1 through
Q.4 The rate of emission of radiation of a black body at
an area of 5 cm2. Calculate thermal conductivity of the
273℃ is E, then the rate of emission of radiation of this
material of the plate.
body at 0℃ will be
E E E
Q.12 Estimate the rate at which ice would melt in a (A) (B) (C) (D)0
wooden box 2.5 cm thick and of inside measurements 16 4 8
100×60×40 cm, assuming that the external temperature
is 32℃ and coefficient of thermal conductivity of wood Q.5 The power radiated by a black body is P and it
is 0.168 Wm-1 K-1. Given L=80cal/g. radiates maximum energy around the wavelength λ0 .
If the temperature of the black body is now changed
so that it radiates maximum energy around wavelength
Q.13 A pan filled with hot food cools from 94℃ to
3/4 λ0 , the power radiated by it will increase by a factor
86℃ in 2 minutes when the room temperature is at
of
20℃. How long will it take to cool from 71℃ to 69℃?
Here cooling takes place according to Newton’s law of (A) 4/3 (B) 16/9 (C) 64/27 (D) 256/81
cooling.
Q.6 Star S1 emits maximum radiation of wavelength
Q.14 A liquid initially at 70℃ cools to 55℃ in 5 minutes 420 nm and the star S2 emits maximum radiation of
and 45℃ in 10 minutes. What is the temperature of the wavelength 560 nm, what is the ratio of the temperature
surroundings? of S1 and S2
(B) ( 4 3) (D) ( 3 4 )
1/ 4 1/2
(A) 4 3 (C)3 4

Exercise 2
Q.7 Spheres P and Q are uniformly constructed from
the same material which is a good conductor of heat
Single Correct Choice Type
and the radius of Q is thrice the radius of P. the rate of
fall of temperature of P is x times that of Q when both
Q.1 Four rods of same material with different radii r and are at the same surface temperature. The value of x is
length l are used to connect two reservoirs of heat at
different temperatures. Which one will conduct most (A) 1/4 (B) 1/3 (C) 3 (D) 4
heat?
Q.8 A black body calorimeter filled with hot water cools
(A) r=2cm, l=0.5 cm (B) r=2cm, l=2m
from 60℃ to 50℃ in 4 min and 40℃ to 30℃ in 8 min.
(C) r=0.5cm,l=0.5m (D)r=1cm, l=1m The approximate temperature of surrounding is
(A) 10℃ (B) 15℃ (C) 20℃ (D) 25℃
Q.2 A wall has two layers A and B each made of different
materials, both the layers have same thickness. The
Q.9 A system S receives heat continuously from an
thermal conductivity of the material A is twice of that
electrical heater of power 10W. The temperature of
of B. Under thermal equilibrium, the temperature
S becomes constant at 50℃. When the surrounding
difference across the wall B is 36℃. The temperature
temperature is 20℃. After the heater is switched off,
difference across wall A is
S cools from 35.1℃ to 34.9℃ in 1 minute. The heat
(A) 6℃ (B) 12℃ (C) 18℃ (D) 72℃ capacity of S is
(A) 100 J/℃ (B) 300 J/℃
Q.3 A black metal foil is warmed by radiation from a
small sphere at temperature ‘T’ and at a distance‘d’. It (C) 750 J/℃ (D) 1500 J/℃
is found that the power received by the foil is P. If both
1 4 . 2 6 | Heat Transfer

Previous Years’ Questions outer surfaces. The wavelengths corresponding to


maximum intensity are 300 nm, 400 nm and 500 nm
respectively. The power radiated by them are QA, QB and
Q.1 A cylinder of radius R made of a material of thermal QC respectively. Which is the maximum power radiated?
conductivity K1 is surrounded by a cylindrical shell of  (2004)
inner radius R and outer radius 2R made of a material of
thermal conductivity K2. The two ends of the combined
Q.8 A long metallic bar is carrying heat from one of its
system are maintained at two different temperatures.
ends to the other end under steady-state. The variation
There is loss of heat across the cylindrical surface and
of temperature θ along the length x of the bar from
the system is in steady state. The effective thermal
its hot end is best described by which of the following
conductivity of the system is_________. (1988)
figure. (2009)

 
Q. 2 Two metallic spheres S1 and S2 are made of the
same material and have got identical surface finish. The (A) (B)
mass of S1 is thrice that of S2 . Both the spheres are
heated to the same high temperature and placed in the x x
same room having lower temperature but are thermally
insulated from each other. The ratio of the initial rate of  
cooling of S1 to that S2 is________. (1995)
(C) (D)

Q.3 The intensity of radiation emitted by the sun has x x


its maximum value at a wavelength of 510 nm and that
emitted by the North Star has the maximum value at
Q.9. 100g of water is heated from 30°C to 50°C.
350 nm. If these stars behave like blackbodies, then
Ignoring the slight expansion of the water, the change
the ratio of the surface temperature of the North Star
in its internal energy is (specific heat of water is 4148 J/
is______. (1997)
kg/K): (2011)

Q. 4 A spherical black body with radius of 12 cm radiates


(A) 8.4 kJ (B) 84 kJ
450 W power at 500 K. If the radius were halved and
the temperature doubled, the power radiated would (C) 2.1 kJ (D) 4.2 kJ
be____________. (1997)
Q.10. A liquid in a beaker has temperature θ(t) at time t
Q.5 A black body is at temperature of 2880 K. The energy and θ0 is temperature of surroundings, then according
of radiation emitted by this body with wavelength to Newton’s law of cooling the correct graph between
between 499 nm and 500 nm is U1, between 999 nm loge (θ – θ0) and t is (2012)
and 1000 nm is U2 and between 1499 nm and 1500
loge (-0)

loge (-0)

nm is U3. The Wien constant, b = 2.88 × 106 nm − K. Then,


what can be inferred about the relation between the
energies? (1998) (A) (B)

Q.6 Two identical conducting rods are first connected 0 t 0 t


independently to two vessels, one containing water at
100℃ and the other containing ice at 0℃. In the second
case, rods are joined end to end and connected to the
loge (-0)

loge (-0)

same vessels. Let q1 and q2 gram per second be the


rate of melting of ice in the two cases respectively. The
(C) (D)
q
ratio 1 is ______.  (2004)
q2
0 t 0 t
Q.7 Three discs, A, B and C having radii 2m, 4m and
6m respectively are coated with carbon black on their
(A) (B)
o

O t O t
P hysi cs | 14.27

Q.11. If a piece of metal is heated to temperature θ and


then allowed to cool in a room which is at temperature
θ0, the graph between the temperature T of the metal T T
(C) (D)
and time t will be closest to: (2013) o o

O t O t

T T
(A) (B)
o

O t O t

(C)
T
(D)
JEE Advanced/Boards
T
o o

Exercise
O
1 t O t
B
L
C

Q.1 A thin walled metal tank of surface area 5 m2 is L L


filled with water and contains an immersion heater
dissipating 1kW. The tank is covered with 4 cm thick A D
layer of insulation whose thermal conductivity is 0.2 L
W/m/K. The outer face of the insulation is 25℃. Find
Q.5 One end of copper rod of uniform cross-section
the temperature of the tank in the steady state.
and of length 1.5 meters is in contact with melting ice
and the other end with boiling water. At what point
Q.2 The figure shows the face and interface temperature along the length should a temperature of 200℃ be
of a composite slab containing of four layers of two maintained, so that in steady state, the mass of ice
materials having identical thickness. Under stady state melting is equal to that of steam produced in the same
condition, find the value of temperature θ interval of time? Assume that the whole system is
insulated from the surroundings.
20 C 10 C  -5 C -10 C
o o o o

k 2k k 2k
Q.6 An empty pressure cooker of  Whistle
volume 10 liters contains air at
k=thermal conductivity atmospheric pressure 105 Pa and
temperature of 27℃. It contains a
Q.3 Three conducting rods of same material and cross- whistle which has area of 0.1 cm2 and weight of 100 gm.
section are shown in figure. Temperature of A,D and C What should be temperature of air inside so that the
are maintained at 20℃, 90℃ and 0℃. Find the ratio of whistle is just lifted up?
length BD and BC if there is no heat flow in AB
A B C
o
20 C
o
0C Exercise 2
o
90 C D Multiple Correct Choice Type

Q.4 In the square frame of side L of metallic rods, the Q.1 Two metallic spheres A and B are made of same
corners A and C are maintained at T1 and T2 respectively. material and have got identical surface finish. The mass
The rate of heat flow from A to C is W. If A and D are of sphere A is four times that of B. Both the spheres are
instead maintained at T1 and T2 respectively, find the heated to the same temperature and placed in a room
total rate of heat flow.
1 4 . 2 8 | Heat Transfer

having lower temperature but thermally insulated from (B) The maximum intensity occurs at a shorter
each other. wavelength
(A) The ratio of heat loss of A to that of B is 24/3 (C) The area under the graph increases
2/3
(B) The ratio of heat loss of A to that of B is 2 (D) The area under the graph is proportional to the
fourth power of temperature
(C) The ratio of the initial rate of cooling of A to that of
B is 2−2/3
Paragraph 2:
(D) The ratio of the initial rate of cooling of A to that of
B is 2−4/3 Two rods A and B of same cross-sectional area A
and length l connected in series between a source
Q.2 Two bodies A and B have thermal emissivity of 0.01
(T1 = 100 °C) and a sink (T2 = 0 °C) as shown in figure.
and 0.81 respectively. The outer surface areas of the two
bodies are the same. The two bodies radiate energy at The rod is laterally insulated
the same rate. The wavelength λB , corresponding to
the maximum special radiancy in the radiation from B,
is shifted from the wavelength corresponding to the T1
3K K
T2
maximum spectral radiancy in the radiation from A by o
100 C
o
0C
1.00 µm. If the temperature of A is 5802 K,
L L
(A) The temperature of B is 1934 K
(B) λB= 1.5 µm Q.5 The ratio of the thermal resistance of the rod is
(C) The temperature of B is 11604 K R 1 R R 3 4
(A) A
= = (B) A 3=
(C) A (D)
(D) The temperature of B is 2901 K RB 3 RB RB 4 3

Q.6 If TA and TB are the temperature drops across the


Comprehension Type
rod A and B, then
Paragraph 1: T 3 T 1 T 3 T 4
(A) A =
= (B) A = (C) A = (D) A
TB 1 TB 3 TB 4 TB 3
dE T
d Q.7 If GA and GB are the temperature gradients across
the rod A and B, then
G 3 G 1 G 3 G 4
O m  (A) A
= = (B) A = (C) A = (D) A
GB 1 GB 3 GB 4 GB 3
Q.3 The figure shows a radiant energy spectrum graph
for a black body at a temperature T.
Paragraph 3:
Choose the correct statement(s)
In fluids heat transfer takes place and molecules of
(A) The radiant energy is not equally distributed among the medium take very active part. The molecules
all the possible wavelengths take energy from high temperature zone and move
(B) For a particular wavelength the spectral intensity is towards low temperature zone. This method is known
maximum as convection, when we require heat transfer with fast
phase, we use some mechanism to make the flow
(C) The area under the curve is equal to the rate at of fluid on the body fast. The rate of loss of heat is
which heat is radiated by the body at that temperature proportional to velocity of fluid (v), and temperature
(D) None of these difference (∆T) between the body and fluid, of course
more surface area of body , more rate of loss of heat.
We can write the rate of loss of heat as
Q.4 If the temperature of the body is raised to higher
temperature T’, then choose the correct statement(s) dQ
= KAv∆T where K is Positive constant.
dt
(A) The intensity of radiation for every wavelength
increases Now answer the following questions:-
P hysi cs | 14.29

Q.8 A body is being cooled with fluid. When we Q.4 A double-pane window used for insulating a room
increase the velocity of fluid 4 times and decrease the thermally from outside consists of two glass sheets
temperature difference ½ time, the rate of loss of heat each of area 1 m2 and thickness 0.01 m separated
increases by a 0.05 m thick stagnant air space. In the steady
state, the room glass interface and glass-outdoor
(A) Four times (B) Two times
interface are at constant temperatures of 27℃ and 0℃
(C) Six times (D) No change respectively. Calculate the rate of heat of flow through
window pane. Also find the temperatures of other
Q.9 In the above question, if mass of the body interfaces. Given thermal conductivities of glass and air
increased two times, without change in any of the other as 0.8 and 0.08 Wm−1K −1 respectively.  (1997)
parameters, the rate of cooling
(A) Decreases Q.5 A solid body X of heat capacity C is kept in an
(B) Increases atmosphere whose temperature is TA = 300 K. At
(C) No effect of change of mass time t=0, the temperature of X is T0 = 400 K. It cools
(D) None of these according to Newton’s law of cooling. At time t1 its
temperature is found to be 350 K.
At this time (t1), the body X is connected to a large body
Previous Years’ Questions Y at atmospheric temperature TA through a conducting
rod of length L, cross-sectional area A and thermal
conductivity K. The heat capacity of Y is so large that
Q.1 A solid sphere of copper of radius R and a hollow
any variation in its temperature may be neglected. The
sphere of the same material of inner radius r and outer
cross-sectional area A of the connecting rod is small
radius R are heated to the same temperature and
compared to the surface area of X. Find the temperature
allowed to cool in the same environment. Which of
of X at time t = 3t1 .  (1998)
them cools faster? (1982)

Q.6 The top of an insulated cylindrical container is


covered by a disc having emissivity 0.6 and conductivity
Q.2 An electric heater is used in a room of total wall
0.167 W/Km and thickness 1 cm. The temperature is
area 137 m2 to maintain a temperature of +20℃ inside
maintained by circulating oil as shown
it, when the outside temperature is -10℃. The walls
have three different layers. The innermost layer is of
wood of thickness 2.5 cm, the middle layer is of cement
of thickness 1.0 cm and the outer most layer is of brick Oil out
of thickness 25.0 cm. Find the power of the electrical
heater. Assume that there is no heat loss through the
floor and the ceiling. The thermal conductivities of
wood, cement and brick are 0.125, 1.5 and 1.0 W/m/℃
respectively.  (1986)
Oil in
Q.3 A cylindrical block of length 0.4 m and area of
cross-section 0.04 m2 is placed coaxially on a thin metal
disc of mass 0.4 kg and of the same cross-section. The
upper face of the cylinder is maintained at a constant (a) Find the radiation loss to the surroundings in W/m2
temperature of 400K and initial temperature of the disc if temperature of the upper surface of the disc is 127℃
is 300K. If the thermal conductivity of the material of and temperature of surroundings is 27℃.
the cylinder is 10W/mK and specific heat capacity of
(b) Also find the temperature of the circulating oil.
the material of the disc is 600 J/kg-K, how long will
Neglect the heat loss due to convection (2003)
it take for the temperature of the disc to increase to
350 K? Assume, for purpose of calculation, the thermal
conductivity of the disc to be very high and the system Q.7 One end of a rod of length L and cross-sectional
to be thermally insulated except for the upper face of area A is kept in a furnace of temperature T1. The other
the cylinder. (1992) end of the rod is kept at temperature T2. The thermal
1 4 . 3 0 | Heat Transfer

conductivity of the material of the rod is K and emissivity of the middle (i.e. second) plate under steady state
of the rod is e. condition is (2012)

TS 1/ 4 1/ 4
 65   97 
(A)   T (B)   T
Insulated  2   4 
Furnace 1/ 4
 97 
(D) ( 97 )
Rod 1/ 4
T1 T2 (C)   T T
L  2 
Insulated
Q.11 Two rectangular blocks, having identical
dimensions, can be arranged either in configuration I
It is given that T2= TS + ∆T , where ∆T<< TS , TS being or in configuration II as shown in the figure. One of the
the temperature of the surroundings. If ∆T ∝ ( T1 − TS ) , blocks has thermal conductivity κ and the other 2 κ .
find he proportionality constant. Consider that heat is The temperature difference between the ends along the
lost only by radiation at the end where the temperature x-axis is the same in both the configurations. It takes 9
of the rod is T2 . (2004)
s to transport a certain amount of heat from the hot
end to the cold end in the configuration I. The time to
Q.8 Two spherical bodies A (radius 6 cm) and B (radius transport the same amount of heat in the configuration
18 cm) are at temperatures T1 and T2 , respectively. The II is (2013)
maximum intensity in the emission spectrum of A is at
Configuration II
500 nm and in that of B is at 1500 nm. Considering Configuration I
them to be black bodies, what will be the ratio of the
rate of total energy radiated by A to that of B?  (2010) 2

 2 
Q.9 A composite block is made of slabs A, B, C, D and x
E of different thermal conductivities (given in terms of
a constant K) and sizes (given in terms of length, L) as (A) 2.0 s (B) 3.0 s (C) 4.5 s (D) 6.0
shown in the figure. All slabs are of same width. Heat
‘Q’ flows only from left to right through the blocks. Q.12 Two spherical stars A and B emit blackbody
Then in steady state radiation. The radius of A is 400 times that of B and A
emits 104 times the power emitted from B. The ratio
0 1L 5L 6L
(λ A / λB ) of their wavelengths λ A and λB at which
Heat A B 3K E the peaks occur in their respective radiation curves is
1L  (2015)
6K
2K C 4K Q.13. A metal is heated in a furnace where a sensor
3L is kept above the metal surface to read the power
radiated (P) by the metal. The sensor has scale that
D 5K displays log2(P/P0), whre P0 is a constant. When the
4L
metal surface is at a temperature of 487°C, the sensor
(A) Heat flow through A and E slabs are same. shows a value 1. Assume that the emissivity of the
metallic surface remains constant. What is the value
(B) Heat flow through slab E is maximum. displayed by the sensor when the temperature of the
(C) Temperature difference across slab E is smallest. metal surface is raised to 2767°C? (2016)

(D) Heat flow through C = heat flow through B + heat


flow through D. (2011) Q.14 Two moles of ideal helium gas are in a rubber
balloon at 30°C. The balloon is fully expandable and can
be assumed to require no energy in its expansion. The
Q.10 Three very large plates of same area are kept temperature of the gas in the balloon is slowly changed
parallel and close to each other. They are considered to 35°C. The amount of heat required in raising the
as ideal black surfaces and have very high thermal temperature is nearly (take R =8.31 J/mol.K) (2012)
conductivity. The first and third plates are maintained at
temperatures 2T and 3T respectively. The temperature (A) 62 J (B) 104 J (C) 124 J (D) 208 J
P hysi cs | 14.31

Q.15 One mole of mono-atomic ideal gas is taken allowing no heat transfer between outside and inside
along two cyclic processes E → F → G → E and E → F → the container. The container is divided into two
H → E as shown in the PV diagram. compartments by a rigid partition made of a thermally
conducting material that allows slow transfer of heat.
The processes involved are purely isochoric, isobaric,
The lower compartment of the container is filled with
isothermal or adiabatic.
2 moles of an ideal monatomic gas at 700 K and the
Match the paths in list I with the magnitudes of the upper compartment is filled with 2 moles of an ideal
work done in list II and select the correct answer using diatomic gas at 400 K. The heat capacities per mole
the codes given below the lists. (2013) 3 5
of an ideal monatomic gas are= CV =R,CP R , and
2 2
List I List II 5 7
those for an ideal diatomic gas are
= CV = R,CP R.
2 2
P. G→ E 1. 160 P0V0 ln2

Q. G→ H 2. 36P0 V0

R. F→ H 3. 24P0 V0

S. F→ G 4. 31P0 V0

Codes:
Q.16 Consider the partition to be rigidly fixed so that it
P Q R S
does not move. When equilibrium is achieved, the final
(A) 4 3 2 1 temperature of the gases will be (2014)
(B) 4 3 1 2
(A) 550 K (B) 525 K (C) 513K (D) 490 K
(C) 3 1 2 4
(D) 1 3 2 4
Q.17 Now consider the partition to be free to move
without friction so that the pressure of gases in both
Paragraph 1: compartments is the same. Then total work done by
In the figure a container is shown to have a movable the gases till the time they achieve equilibrium will be
(without friction) piston on top. The container and the  (2014)
piston are all made of perfectly insulating material (A) 250 R (B) 200 R (C) 100 R (D) –100 R

PlancEssential Questions
JEE Main/Boards JEE Advanced/Boards
Exercise 1 Exercise 1
Q.10 Q.11 Q.12 Q.13 Q.2 Q.3 Q.4 Q.6

Exercise 2 Exercise 2
Q.1 Q.2 Q.5 Q.6 Q. 1 Q.2 Q.5 Q.6
Q.9 Q.7 Q.12 Q.15
1 4 . 3 2 | Heat Transfer

Answer Key

JEE Main/Boards
Exercise 1
Q.1 Silver is the best conductor of heat

Q.2 Radiation is the quickest mode of transfer of heat.

Q.3 The fall in temperature in a body per unit distance is called temperature gradient.

Q.10 3.5 Wm−1°C−1 Q.11 58.33 Wm−1°C−1



Q.12 1.587 gms−1 Q.13 42 s

Q.14 25℃

Exercise 2
Single Correct Choice Type

Q.1 A Q.2 C Q.3 B Q.4 A Q.5 D Q.6 A


Q.7 C Q.8 B Q.9 D

Previous Years’ Questions


Q.2 (1/3)1/3 Q.3 0.69 Q.4 1800 W Q.5 U2>U1 Q. 6 4/1 Q.7 QB
Q.8 B Q.9 A Q.10 A Q.11 C

JEE Advanced/Boards
Exercise 1
Q.1 65℃ Q.2 5℃ Q.3 7/2 Q.4 (4/3) W Q.5 10.34 cm Q.6 327 oC

Exercise 2
Multiple Correct Choice Type

Q.1 A, C Q.2 A, B

Comprehension Type

Paragraph 1: Q.3 A, B Q.4 A, B, C, D

Paragraph 2: Q.5 A Q.6 B Q.7 B

Paragraph 3: Q.8 B Q.9 A


P hysi cs | 14.33

Previous Years’ Questions


Q.1 Hollow Sphere Q.2 9091 W Q.3 T=166.32 s
 −2AKt1 
Q.4 41.6 W, 26.48 °C, 0.52 °C T2  300 + 12.5e
Q.5= CL  Q.6 (a) 595 W/m2, (b) 162.6℃
 
 
K
Q.7 Q.8 9 Q 9: A, C, D or A, B, C, D
4eσLTS3 + K

Q.10 C Q.11 A Q.12 2


Q.13 9 Q.14 D Q.15 A
Q.16 D Q.17 D

Solutions

JEE Main/Boards Convection: When a fluid in a vessel is heated, lighter


molecules present in the lower layer of the fluid get
heated, which rise and cold molecules go to the bottom
Exercise 1 of the vessel. i.e. by movement of the molecules of fluid.
E.g. A gas vessel filled with fluid being heated from
Sol 1: Silver is the best conductor of heat
bottom.

Sol 2: Radiation Heated


fluid
Sol 3: Temperature gradient → Fall in temperature rises
in a body per unit distance is called the temperature
gradient.
Cold fluid
replaces
Sol 4: Three Methods:
Radiation:- Heat travels directly from one place to
(i) Conduction
another without any intervening medium.
(ii) Convection E.g. Heat from the sun to the earth.
(iii) Radiation – fastest one because heat travels without
any intervening medium. Sol 6: Thermal radiations are electromagnetic waves
which are invisible. These are radiated from a heated
Sol 5: Conduction:- Heat flows from a place of higher surface in all directions. These travel with velocity
temperature to a place of lower temperature with the of light in a straight line and does not require an
medium remaining stationary. intervening medium to carry it.
Eg. A metal rod heated from one end
Sol 7: If a body at temperature q1 is placed in
surroundings at lower temperature q2, then it is
observed that magnitude of temperature gradient
decreases with time

i.e. – ∝ + (θ – q2) [Newton’s law of cooling]
dt

⇒ = – k (θ – q2) k → a constant
dt
1 4 . 3 4 | Heat Transfer

Sol 11:  = 4mm = 4 × 10-3 m


 DT = 32° C
dQ
= 200 k cal h -1 _ 233.33 J/sec
dt
t 4mm
Sol 8: Thermal conductivity is the property of a material
to conduct heat.
T2
Coefficient of thermal conductivity (k) is the measure T1
of thermal conductivity which is equal to the quantity
of heat flowing per unit time through area of cross-
section of a material per unit length along the direction Area = 5cm
2

of flow of heat.
KA∆T
S.I. Units: - J.m-1 sec-1 K-1 ⇒ = 233.33 J/sec

C.G.S. Units: - cal. cm-1 (°C)-1 233.33 × 4 × 10−3
⇒K= = 58.33 J/m °C sec
32 × 5 × 10 −4
Sol 9: The rate of cooling of a body is directly
proportional to the difference of temperature of the Sol 12: Area of surface perpendicular to direction of
body over its surrounding. flow of heat _ surface area of inner rectangle
Body temperature at any time ’t’ → θ Text=32 C
o

Body initial temperature → q1


Surrounding temperature → q2 T=0 C
o 2.5cm
-1 -1
−dQ inner k=0.168Wm K
∴ Rate of cooling i.e. ∝ (θ – q2)
dt
dQ
∴ = -k (θ – q2)
dt _ [2 [100 × 60]+2 [60 × 40]+2 [100×40]×10-4
dθ = 2.48 m2
⇒ ms = – k (θ – q2)
dt
dQ −kA
dθ  k  ∴ = (Text – Tinner)
⇒ =–   (θ – q2) dt 
dt  ms 
0.168 × 2.48 × 32
=- = -533.29 J/sec
dQ 2.5 × 10−2
Sol 10: = 20000 cal min-1
dt Therefore, rate at which ice melts
20000 × 4.2 533.29
= J sec-1 = gm/sec =1.587 gm/sec
60 80 × 4.2
Tair=-20oC
Sol 13: T = Tsurrounding + (Tinitial-Tsurrounding) e-kt
5cm Ice o
T=0 C
Water  T − Tsurrounding 
⇒ ln   = -kt
 Tinitial − Tsurrounding 
Ice-water  
juctionm
Let when t = 0, Tinitial = 94oC, Tsurrounding = 20oC
−K∆θ 20000 × 4.2
⇒ =
∆x 60  86 − 20 
∴ -k × 2 = ln  
−K[( −20) − (0)]  94 − 20 
⇒ = 1400 ⇒ K = 3.5 Wm-1 °C-1
5 / 100  66  −0.114
⇒ -2k = ln   ⇒ -k =
74
  2
P hysi cs | 14.35

and let t = 0 when, Tinitial = 71 oC A


(D) =π
L
 69 − 20   49 
∴ -kt = ln   = ln   Therefore (A) will conduct more heat.
 71 − 20   51 
⇒ t _ 0.70 min = 42 sec
 dQ   dQ 
Sol 2: (C)   =  
 dt acr os s A  dt acrossB
Sol 14: Ti = 70°C
Tf = 55°C → t = 5 min −(2k) × A −(k) × A
⇒ DTA = × 36
 
T’f = 45° C → t’ = 10 min
⇒ DTA = 18°C
T0 → Temperature of surrounding
∴ From Newton’s law A B
T – T0 = (Ti – T0) e -kt
2k k
We have following equation.  
⇒ 55 – T0 = (70 – T0) e -5k
 …(i)
And 45 – T0 = (70 – T0) e-10k  …(ii) Sol 3: (B) P = Area of foil× Intensity (Afoil)
Dividing equations (ii)/(i)
45 − T0
⇒ = e-5k
55 − T0 d
T
Substitute value of e-5k in (i)
Area A
 45 − T0 
⇒ (55 – T0) = (70 – T0)  
 55 − T0 
Power emitted eAσT 4
Intensity = =
55 − T0 45 − T0 Area of sphere 4 πd2
⇒ -1 = -1
70 − T0 55 − T0 eAσT 4
∴ P = Afoil ×
−15 −10 4 πd2
⇒ =
70 − T0 55 − T0 When temperature and distance are double

⇒ T0 = 25° C eAσ(2T)4
∴ P’ = Afoil × = 4P
4 π(2d)2

Sol 4: (A) At T = 273°C = (273 + 273) K


Exercise 2
E = eAσ (273 + 273)4 = 16eAσ (273)4
Single Correct Choice Type E
At T = 273 K; E’ = eAσ (273)4 =
16
dQ kA
Sol 1: (A) =- DT E
dt L ∴ E’ =
16
A
The greater the value of , more the heat will be
conducted. L
Sol 5: (D) lm T = const. = 2.93 × 10-3 mK
A π(2)2 P = eA σ T4
(A) = = 8π
L 0.5 Given: l0 T= const. = c (say)
A 3 3
(B) = 2π When lm = l0 then l0 × T’ = c
L 4 4
A π 4
(C) = ⇒ T’ = T
L 2 3
1 4 . 3 6 | Heat Transfer

4 
4 On substituting value of e-4k from (i) into (iii) we get:
∴ P’ = eAσ  T 
3  30 − Ta 40 − Ta  50 − Ta 
= ×  
256
⇒ P’ = P 50 − Ta 60 − Ta  60 − Ta 
81
⇒ (30 – Ta) (60 – Ta)2 = (40 – Ta) (50 – Ta)2
Sol 6: (A) By Wien’s displacement law:- ⇒ (Ta - 60) (Ta – 60) (Ta – 30)
lm T = constant = C = (Ta – 50) (Ta – 50) (Ta – 40)
∴ lm1 T1 =lm2 T2
⇒ Ta3 – [60 + 60 + 30] Ta2 + [60 × 60 + 60 × 30 + 60 ×
⇒ 420 T1 = 560 T2 30] Ta - 60× 60× 30 = Ta3 – [50 + 50 +40] Ta2+ [50 × 50

T1 560 4
+ 50 × 40 + 50 × 40]Ta –50 × 50 × 40
⇒ = =
T2 420 3 ⇒ – 10T a2+700 Ta – 8000 = 0
⇒ Ta2 – 70 Ta + 800 = 0
Sol 7: (C) rθ = 3rp
⇒ Ta = 55.61 or 14.38
P = eAσ T4
dT dQ dθ
⇒ mc = eAsT4 Sol 9: (D) P = = S× = k(θ1 − θ0 )
dt dt dt

(c: specific heat, m: mass of sphere) 10W


10 W =
k(50 − 20) k=
30 o C
dT −eAσT 4 −eAσT 4 −e4 πr 2 σT 4
⇒ = = = dθ
dt mc (ρV)c 4 3 S× = k(θ − θ0 )
πr c dt
3
(V: volume of sphere)
∆θ 35.1 + 34.9
⇒ S = k(θ − θ0 ) θ= = 35
dT 1  −3eσT  4 ∆t 2
⇒ = ×  
dt r  c   0.2  10
S =  (35 − 20)
[Quantity in parenthesis is Constant for both spheres]  60 sec  30
 dT  1 S = 1500 J / o C
 
 dt P r
∴ = P =3=x
 dT  1
  rQ
 dt Q
Previous Years’ Questions
Sol 8: (B) By Newton’s law of cooling:- Sol 1: Let R1 and R2 be the thermal resistances of inner
(T – Ta) = (T0 – Ta) e -kt and outer portions. Since, temperature difference at
both ends is same, the resistances are in parallel. Hence,
Ta : Surrounding temperature
1 1 1
T0 : Initial temperature = +
R R1 R2
When T0 = 60° C, T = 50° C, t = 4 min
K(4 πR 2 ) K ( πR 2 ) K (3πR 2 )
∵ = 1 + 2
∴ (50 – Ta) = (60- Ta) e -4k
 ....(i)   
When T0 = 40, T = 30 then t = 8 min 3K 2 + K1
∴K=
∴ (30 – Ta) = (40 – Ta) e-8k  ......(ii) 4

Dividing (ii)/(i) gives


30 − Ta 40 − Ta
= e-4k ......(iii) R 2R
50 − Ta 60 − Ta
P hysi cs | 14.37

Sol 2: The rate at which energy radiates from the object is Energy distribution with wavelength will be as follows:
∆Q
= eσAT4 E
∆t
Since, DQ = mcDT, we get

∆T eσAT 4
=
∆t mc U1 U2 U3
(nm)
4

499
500

999
1000

1499
1500
Also, since m = pr3ρ for a sphere, we get
3
2/3 m=1000
 3m 
A = 4pr = 4π 
2
 From the graph it is clear that
 4 πρ 
U2 > U1 (In fact U2 is maximum)
eσT 4   3m  
2/3 1/3
∆T 1
Hence, = 4π   = K 
∆t mc   4 πρ   m
  dm 1
Sol 6: q = ∝
For the given two bodies dt Thermal Resis tance

1/3 1/3 In the first case rods are in parallel and thermal
( ∆T / ∆t)1 m   1
=  2  =   R
resistance is while in second case rods are in series
( ∆T / ∆t)2  m1  3 2
and thermal resistance is 2R.
Sol 3: From Wien’s displacement law q1 2R 4
= =
lmT = constant q2 R/2 1
1
or T=
λm Sol 7: Q ∝ AT4 and lmT = constant.
Tsun (λm )north star 350 A r2
∴ = = ≈ 0.69 Hence, Q ∝ or Q ∝
Tnorthstar (λm )sun 510 ( λm ) 4 ( λm ) 4

(2)2 (4)2 (6)2


Sol 4: Power radiated ∝ (surface area)(T) . The radius 4 QA: QB: QC = : :
(3)4 (4)4 (5)4
1
is halved, hence, surface area will become times.
4 4 1 36
= : :
Temperature is doubled, therefore, T4 becomes 16 81 16 625
times.
= 0.05 : 0.0625 : 0.0576
 1
New power = (450)   (16) = 1800 W. i.e., QB is maximum.
4

Sol 5: Wien’s displacement law is dQ dθ


So 8: (B) We know that = kA
dt dx
lmT = b (b = Wien’s constant)
In steady state flow of heat
b 2.88 × 106 nm – K
∴ lm = = dQ 1
T 2880 K dθ = . dx
dt kA
∴ λ = 1000 nm
⇒ θH - θ = k x ′ ⇒ θ = θ H - k x ′
Equation θ = θH - k′ x represents a straight line.

Sol 9: (A) ΔQ = ΔU+ ΔW (ignoring expansion)


ΔU = msΔT = 0.1× 4.184× 20 = 8.368kJ
1 4 . 3 8 | Heat Transfer

Sol 10: (A) According to Newtons law of cooling. kA(TB − TD ) −kA (TC − TB )
⇒– =
DB BC

∝ (θ − θ0 )
dx 
(20 − 90) (0 − 20) −70
⇒ = ; ⇒ BD = = 3.5
dθ DB BC BC −20
⇒ dθ = −k(θ − θ0 )
⇒d dtθ = −k(θ − θ0 )
⇒ dtd=θ −k(θ − θ0 )
∫∫dtθ dd−θθθ == ∫∫ −−kdt
kdt Sol 4:
θ − θ00 = −kdt

⇒ ln( ∫
θ θ− −θ θ ) = −kt + c T2
⇒ ln(θ − 0θ00 ) = −kt + c B C
⇒ ln(θ − θ0 ) = −kt + c 2L
T1 B T2
Hence the plot of ln(θ – θ0) vs t should be a straight line A C
with negative slope. 2L
T1 A D
Sol 11: (C) According to Newtons cooling law, option C
is the correct option.  dQ   dQ   dQ 
∴   =   +  
 dt AC  dt ABC  dt ADC

JEE Advanced/Boards ⇒ W=
−kA(T2 − T1 )
+
−kA(T2 − T1 )
2L 2L
Exercise 1 −kA(T2 − T1 )
⇒ W=
L
Sol 1: Continuously 1 kW of heat is being dissipated
from 25°C tank. B C
3L
dQ −KA[25 − T]
∴ = 103 = B C
dt  T2
T1 D
A L
dQ −0.2 × 5 × [25 − T] A D
⇒ = 103 =
dt 4 × 10−2
T1 T2
⇒ 25 – T = –40 ; ⇒ T = 65°C
 dQ  −kA(T2 − T1 ) −kA(T2 − T1 )
∴   = +
Sol 2: For 1st layer  dt AD 3L L

dQ −KA∆T −KA(10 − 20) +KA × 10 −4kA(T2 − T1 )


= = = 4
dt    = = W
L 3
For 2nd layer
dQ −(2k) A (θ − 10) −2AK Sol 5:
= = [θ–10]
dt   T2=200 C
o

o
Rate for both layers must be equal
o
T1=0 C I II T=100 C

kA × 10 −2kA x 1.5x
∴ = (θ – 10) ; ⇒ θ = 5°C
  1.5m

 dQ  Mass of ice melting per second = mass of steam


Sol 3: If   =0
 dt AB produced per sec

Then rate of heat flow from D to B must be equal to rate −kA(0 − 200) −kA(100 − 200)
of heat flow from B to C. ⇒ x = 1.5 − x
80 540
 dQ   dQ 
i.e.   =  
 dt DB  dt BC
P hysi cs | 14.39

1.5 − x 1 80 and Area of sphere (A) ∝ r2


⇒ = ×
x 2 540 ∴ A ∝ (m)2/3
27 × 1.5 AA
⇒x= _ 1.3966 m ∴ = (4)2/3 = (2)4/3
29 AB

∴ 1.5 – x = 0.1034 m = 10.34 cm eA A σ(T − T0 )4 AA


∴ Ratio of heat loss = = = (2)4/3
eAB σ(T − T0 ) 4 AB
Sol 6: Mass of whistle = 100 gm = 0.1 kg By Newton’s law of cooling:
∴ Weight of whistle = 1 N dQ dT
= ms = -k (T – T0)
To just lift the whistle, pressure in pressure cooker must dt dt
be equal to = Atmospheric pressure + Pressure due to dT −k
weight of whistle ⇒ = (T – T0)
dt ms
1N where k = 4e A σ T03
= 105 + = 2 × 105 Pa
0.1 × 10−4 m2 dT A
∴ ∝
Free-body diagram of whistle Patm A dt m
 dT  AA
 
dt
 A m (2)4/3
Weight ∴ = A = = 2-2/3
Pcooker xA  dT  AB 4
of  
 dt B mB
whistle

Sol 2: (A, B) eA = 0.01 and eB = 0.81


By Force-balance
AA = AB
Patm × A + weight of whistle = Pcooker×A
EA = EB
weight of whistle ⇒ eAσ AA TA4 = eB σ AB TB4
⇒ Pcooker = Patm +
A
⇒ 0.01 TA4 = 0.81 TB4
Initially, it is given P = 10 Pa, V = 10 L, T = 300K
5
1
⇒ TB = ×T
10 × 10 5
3 A
∴ P.V. = n RT ; ⇒ nR = Pa.L/K  ….(i)
300 1
⇒ TB = × 5802 = 1934 K
Finally, we require P = 2 × 10 Pa, V = 10 L ,T =?
5 3

∴ By gas equation:- PV = nRT By Wien’s displacement law


lm T = const. = 2.93 × 10-3 mK
10 × 105
⇒ 2×10 ×10=
5
× T [using (i)] ∴ λm = 0.5 mm
300 A
Since, it is given in the question that
⇒ T = 600 K = 327 °C
λm = 1 mm + λm
B A
∴ λm = 1.5 mm
B

Exercise 2
Comprehension Type
Multiple Correct Choice Type
Paragraph 1
m
Sol 1: (A, C) ρ =
V Sol 3: (A, B) Area under the curve gives the rate at
4 3 which heat per unit surface is radiated by the body i.e.
⇒ρ× pr = m ; ⇒ r ∝ (m)1/3 total rate of heat radiation = (Area under the curve) ×
3
(surface area of the body)
1 4 . 4 0 | Heat Transfer

Sol 4: (A, B, C, D) lm T = const. [By Wien’s displacement Sol 9: (A) If all the parameters are kept constant then
law]
dQ dT
= ms = kA v DT
Area under graph = Eλ = e σ T ∝ T 4 4
dt dt
dT kAv∆T
T1 ∴ =
dt ms
dE T1 > T2 > T3
T2
d

T3
Previous Years’ Questions
m1 m m x
2 3  dQ 
Sol 1: Net rate of heat radiation   will be same in
 dt 
both the cases, as temperature and area are same.
Paragraph 2 Therefore, from equation
 dθ  dQ dθ 1
Sol 5: (A) AA = A ; AB = A ms  – = or – ∝
 dt  dt dt m
 A = l ; B = l The hollow sphere will cool faster as its mass is less.
kA = 3k ; kB = k
A    Sol 2: Let R1, R2 and R3 be the thermal resistances of
∴ RA = = ; RB= B = wood, cement and brick. All the resistances are in
k A AA 3kA kB AB k A
series. Hence,
RA 1
∴ =
RB 3
o
20 C
o
R1 R2 R3 -10 C
Sol 6: (B) Rate at which heat flows from A
= Rate at which heat flows from B R = R 1 + R2 + R3
 dQ   dQ  2.5 × 10 –2 1.0 × 10 –2 25 × 10 –2
⇒   =   = + +
 dt A  dt B 0.125 × 137 1.5 × 137 1.0 × 137
TA TB TA RA 1   
⇒ = ⇒ = = = 0.33 × 10–2 °C/W  as R = 
RA RB TB RB 3  KA 
∴ Rate of heat transfer,
TA TA TB TB
Sol 7: (B) GA = = and GB = = dQ Temperature difference 30
LA L LB L = =
dt thermal resis tance 0.33 × 10 –2
G T 1
∴ = A = ≈ 9091 W
GB TB 3
∴ Power of heater should be 9091 W.

Paragraph 3 Sol 3: Let at any time temperature of the disc be θ.


At this moment rate of heat flow,
 dQ  dQ KA( ∆θ) KA
Sol 8: (B)   = KAv DT = = (q – θ)  ….. (i)
 dt initially dt   0
This heat is utilised in increasing the temperature of the
 dQ   ∆T   dQ  disc.
  =KA(4v)   =2  
 dt finally  2   dt initially
dQ dθ
Hence, = ms  ….. (ii)
dt dt
P hysi cs | 14.41

Equating Eqs. (i) and (ii), we have  dQ 


Now the rate of heat flow  dt  will be equal from all
dθ KA  
ms = (q – θ)
dt  0 the three sections and since rate of heat flow is given
by
dθ KA
Therefore, = dt
θ0 – θ ms dQ Temperature difference
=
dt Thermal resis tance
350K dθ KA t
or ∫300K θ = ∫0 dt
0 – θ ms  dQ   dQ   dQ 
and   =   =  
 dt 1  dt 2  dt 3
350K KA
or  – ln(θ0 – θ) = t
300K ms
27 – θ1 θ1 – θ2 θ2 – θ
Therefore, = =
0.0125 0.625 0.0125
dQ Solving this equation, we get
dt
 = 0.4m q1 = 26.48°C
and q2 = 0.52°C

dQ 27 – θ1
 and =
dt 0.0125
ms  θ0 – 300 
∴t= ln   dQ (27 – 26.48)
KA = = 41.6 W
 θ0 – 350  dt 0.0125
Substituting the values, we have
Sol 5: In the first part of the question (t ≤ t1)
(0.4)(600)(0.4)  400 – 300 
T= ln   At t = 0, TX = T0 = 400 K and at t = t1,
(10)(0.04)  400 – 350 
TX = T1 = 350 K
T = 166.32 s
Temperature of atmosphere,

Sol 4: Let q1 and q2 be the temperatures of the two TA = 300 K (constant)


interfaces as shown in figure. This cools down according to Newton’s law of cooling.
o
1 2
Therefore,
27 C
rate of cooling ∝ temperature difference.
     
dQ
dt 1
dQ
dt 2
dQ
dt 3 TA = 300K
X

R1 R2 R3
Room Outdoor
atmosphere

0.01 m 0.05 m 0.01 m


 dT 
∴ −  = k(T – TA)
  dt 
Thermal resistance, R =
KA dT
⇒ = – k dt
(0.01) T – TA
∴ R1 = R3 = = 0.0125 K/W or °C/W
(0.8)(1)
T1 1 t
dT
(0.05) ⇒ ∫ = – k ∫ dt
and R2 = = 0.625 °C/W T – TA
(0.08)(1) T0 0
1 4 . 4 2 | Heat Transfer

T –T  Sol 6: (a) Rate of heat loss per unit area due to radiation
⇒ ln  1 A  = – kt1
 T0 – TA  I = eσ(T4 – T04 )
Here, T = 127 + 273 = 400 K
 350 – 300 
⇒ kt1 = – ln   and T0 = 27 + 273 = 300 K
 400 – 300 
17
⇒ kt1 = ln (2) ∴ I = 0.6 × × 10–8[(400)4 – (300)4]
3
In the second part (t > t1), body X cools by radiation = 595 W/m2
(according to Newton’s law) as well as by conduction.
(b) Let θ be the temperature of the oil. Then, rate of
T = TA heat flow through conduction = rate of heat loss due
TA
to radiation
Y
X temperature difference
∴ = (595)A
thermal resis tance

(θ – 127)
= (595)A
Therefore, rate of cooling   
 
= (cooling by radiation) + (cooling by conduction)  KA 
Here, A = area of disc; K = Thermal conductivity and
 dT  KA
∴ –  = k(T – TA) + (T – TA)  …. (ii)  = thickness (or length) of disc
 dt  CL
K
KA(T – TA ) ∴ (θ – 127) = 595
dQ  dT  
In conduction, = = C– 
dt L  dt  
∴ θ = 595   + 127
 dT  KA K 
∴–  = (T– TA)
 dt  LC
595 × 10 –2
= + 127 = 162.6°C
where, C = heat capacity of body X 0.167

 dT   KA 
–  = k +  (T – TA) ….. (iii) Sol 7: Rate of heat conduction through rod
 dt   CL 
= rate of the heat lost from right end of the rod.
Let at t = 3t1 temperature of X becomes T2 KA(T1 – T2 )
∴ = eAσ( T24 – Ts4 )  …… (i)
Then from eq. (iii) L
T2 3t1 Given that T2 = Ts + DT
dT  KA 
∫ T – T = –  k + LC  ∫ dt  ∆T 
4
T1 A tl \ T24 = (Ts + DT) =4
Ts4 1 + 
 Ts 
 T – TA   KA 
ln  2  = – k +  (2t ) Using binomial expansion, we have
 T1 – TA   LC  1

 2KA   ∆T 
= –  2kt1 + t  T24 = Ts4 1 + 4  (as DT << Ts)
 LC 1   Ts 

 T – 300  2KAt1 ∴ T24 – Ts4 = 4(DT)( Ts3 )


or ln  2  = – 2ln(2) – ;
 350 – 300  LC
Substituting in Eq. (i), we have
kt1 = ln (2) from Eq. (i)
K(T1 – Ts – ∆T)
This gives equation :- = 4eσ Ts3 .DT
L
 –
2KAt1 
T2 =  300 + 12.5e CL  K K(T1 – Ts )  K
  or =  4eσTs3 +  DT
  L  L
P hysi cs | 14.43

K(Tl – Ts ) Sol 10: (C)


\ DT =
(4eσLTs3 + K) σA(2T)4 + σA(3T)4 = σ2A(T)4 2T 3T
Comparing with the given relation, 16T 4 + 81T 4 =
2(T')4
K 97T 4 = 2(T')4
proportionality constant =
4eσLTs3 +K 97 4
(T')4 = T
2
1
Sol 8: lm ∝  97 
1/ 4
T ∴ T' = T
 
 2 
λA TB 500 1
∴ = = =
λB TA 1500 3
L L 3L
Sol 11: LL (A) LLR1 =κA 3L + =
E ∝ T A 4
R 3L 2κA 2κA
R11 ==κLA + + 2κ L A
=
=23L κ A
R1 =κLLA + 2κ 1
LLA = 23Lκ1A + 1 =3κA
(Where A = surface area = 4pR2) 1 1 = 1 3L
R =
R111 = = κ A + 2 κ
1+ κ+RA2 =A = 1 κLA
2 κ = 33κκA A L  L
\ E ∝ T4R2 R1 =κκA AL1 2 2κ+A L12 2 κA
A=  3 κLLA2κA 
R122 = L1  +  L1 κA= 3
1  κ1A  +  2κ1A  = 3κκA A 
4 2
 6 
2 R2 = = κLLA  +  2κ LAL = L
EA T   RA  R
R 22  L  R  =LL  L
L
=  A    = (3)4  18  = 9  LLκA  2 2κ3AκA
  R = κ A 2 κ A
EB  TB   RB  R 22 = 3κLκAA   2κA 
R 2 = 3κLLA ∆Q1 = ∆Q 2
R

R Q
∆2Q1 ==
= =
3 κ∆ AQ
∆Q 2 ∆T
∴ Answer is 9. 2 1
3 2 ∆T
∆ Q
T 3κ
= κ∆AA
∆QT t1 = t2
∆ T
Q
∆Q1t1=
1
t = = ∆ ∆Q
∆Q 2 T 2
t R R
1 = ∆T2t2
1 2 1 2
Sol 9: (A, C, D) or (A, B, C, D) ∆R
RT 1 t = R R

∆ T
T1 1 ∆T
∆ T2 t
2 2 R
R1 tt11 = = R tt2⇒ t2= 2 t1= 2 sec
⇒R
R11 tt2= RR2222 t=2 2 secR1
⇒ 2= R 22 t1= 2 sec
1
2 ⇒ t2= R R
1 t= 2 sec
1 1
2
⇒ tt22=
⇒ = R R
2 t= 2 sec
1 t1= 2 sec
R11 1
1 3 5
Sol 12: (2)  dQ 
4  dQ 
 dQ  4  dQ = 10  
4 dt
 dQ  = 104  dQ A  dt B
 dt A = 10(400R)
 dt 2T
B4 4 2 4
 dt A2 4  dt BA = 10 (R TB )
(400R)2 TA4 = 10 44 (R 22 TB44 )
R2
(400R) TA =So,2T10 (R T ) λ A TB
R1 λA A BTBTBand λ= T= 2
=
R3 R5 So,2T= A T and λ= T= 2 B A
So,2T= A TBB and λ= A
B T=B
A 2
λB TA
R4

Let width of each rod is d Sol 13: (9) At (T1 = 487+ 273 = 760K) P1 ∝ (760)4

1 1 i.e. P1 = c(760)4 where c = constant


=R1 = 1 ,R 2 1
=R1 8kd
=1 ,R 2
3kd
1 P1 P1
=R1 8kd
=1 ,R 2 3kd
1 log2 =1 ⇒ P1 =2P0 ⇒ P0
R 3 8kd P0 2
= =1 ,R 4 1 ,
3kd
=R 3 2kd
=1 ,R 4
5kd
1 ,
=R 3 2kd
= 1 ,R 4 5kd , at (T2 = 2767 + 273 = 3040)
R5 = 2kd
1 5kd
24kd
R5 = 1 P2 = c(3040)4
R5 = 24kd P 
24kd Reading of the sensor at T2 = log2  2 
 P0 
 P2    3040  
4
= log  log2 2=   log  1 8 9
= 2 2.  = 2 2 .2 
P
 0   760 
   
∴ Reading of T2 = 9
1 4 . 4 4 | Heat Transfer

Sol 17: (D) Heat given by lower compartment


Sol 14: (D) ∆Q = n CP ∆T
5
=2 × R × (700 − T)  ...(i) … (i)
f  3  2
= 2  R + R  ∆=
T 2  + R + R × 5
2  2  Heat obtained by upper compartment
5
= 2 × × 8.31× 5 = 208 J 7
2 =2 × R × (T − 400)  ...(ii) … (ii)
2
By equating (i) and (ii)
Sol 15: (A) P. → 4 ; Q. → 3; R. → 2; S. → 1
5(700 - T) = 7(T - 400)
Apply PV1+2/3 = constant for F to H.
3000 – 5T = 7T– 2800
(32P0 )V05/3
= P0 VH5/3 ⇒ V= 8V0
H 6300 = 12 T
For path FG PV = constant T = 525K
⇒ (32P0 )V0 = P0 VG ⇒ VG = 32V0 ∴ Work done by lower gas = nR∆T = – 350 R

Work done in GE = 31 P0V0 Work done by upper gas = nR∆T = +250 R

Work done in GH = 24 P0V0 Net work done - 100 R

F n=1
32P0 f=3

Isothermal

Adiabatic
P0
E H G

V0 8V0 32V0 V

PHVH − PF VF
Work done
= in FH = 36P0 V0
( −2 / f)
 VG 
Work done
= in FG RTln
=   160P0 V0 ln2
 VF 

Sol 16: (D) 13. Heat given by lower compartment


3
=2 × R × (700 − T)  ...(i) … (i)
2
Heat obtained by upper compartment
7
=2 × R × (T − 400)  ...(ii) … (ii)
2
equating (i) and (ii)
3 (700 – T) = 7 (T – 400)
2100 – 3T = 7 T – 2800
4900 = 10 T ⇒ T = 490 K
2017-18 100 &
op kers
Class 11 T
By E ran culty
-JE Fa r
IIT enior emie .
S fP r es
o titut
Ins

PHYSICS
FOR JEE MAIN & ADVANCED
SECOND
EDITION

Exhaustive Theory
(Now Revised)

Formula Sheet
9000+ Problems
based on latest JEE pattern

2500 + 1000 (New) Problems


of previous 35 years of
AIEEE (JEE Main) and IIT-JEE (JEE Adv)

5000+Illustrations and Solved Examples


Detailed Solutions
of all problems available

Topic Covered Plancess Concepts


Tips & Tricks, Facts, Notes, Misconceptions,
Calorimetry and Thermal Key Take Aways, Problem Solving Tactics
Expansion
PlancEssential
Questions recommended for revision
15. CALORIMETRY
AND THERMAL
E X PA N S I O N

1. INTRODUCTION
Have not been we dealing with the temperature and thermal energy in our daily life? Such as, we store our
perishable food in refrigerator, switch on the heater of the car if we ever feel cold, and always handle hot utensils
with thermal glove. To make a cup of cold coffee, ice cubes are used by our mother and how can of coke kept out
of refrigerate comes to the room temperature.

2. DEFINITION OF HEAT
Heat is energy in transient. Heat energy flows from one body to another body due to their temperature difference.
It is measured in units of calories. The SI unit is Joule. 1 calorie = 4.2J

Illustration 1: What is the difference between heat and temperature? (JEE MAIN)
Sol: Temperature is associated with kinetic energy of atoms/molecule while heat is energy in transit. Temperature
is a measure of the motion of the molecules or atoms within a substance; more specifically, it is the measure of
the average kinetic energy of the molecules or atoms in a substance. Heat is the flow of energy from one body to
another as a result of a temperature difference. It is important to point out that matter does not contain heat; it
contains molecular kinetic energy and not heat. Heat flows and it is the energy that is being transferred. Once heat
has been transferred to an object, it ceases to be heat. It becomes internal energy.

3. DEFINITION OF CALORIE
The amount of heat needed to increase the temperature of 1 g of water from 14.5°C to 15.5°C at a pressure of
1 atm is called 1 calorie.
1 kilo calorie =103 calories; 1 calorie = 4.186 Joule
If the temperature of a body a mass m is raised through a temperature ∆ T, then the heat, ∆ Q, given to the body is
∆ Q = m.s. ∆ T where s is the specific heat of the body which is defined as the amount of the heat required to raise
the temperature of a unit mass of the body through 1°C. Its unit is cal/gm/°C or J/kg/K.
1 5 . 2 | Calorimetr y and Thermal Expansion

Thermal capacity of a body is the quantity of heat required to raise its temperature through 1°C and is equal to
T2
the product of mass and specific heat of the body. Q=m ∫ sdt (be careful about unit of temperature, use units
T1
according to the given units of s)

PLANCESS CONCEPTS

Historically, first calorie was defined and hence such a weird unit conversion is used between calorie and
Joule.
Chinmay Spurandare (JEE 2012, AIR 698)

4. PRINCIPLE OF CALORIMETRY
When two bodies at different temperatures are mixed, heat will pass from the body at a higher temperature to the
body at a lower temperature until the temperature of the mixture becomes constant. The principle of calorimetry
implies that heat lost by the body at a higher temperature is equal to the heat gained by the other body at a lower
temperature assuming that there is no loss of heat in the surroundings.

5. TEMPERATURE SCALES

5.1 Kelvin Temperature Scale


Kelvin is a temperature scale designed such that zero K is defined as absolute zero (at absolute zero, a hypothetical
temperature, all molecular movement stops- all actual temperatures are above absolute zero) and the size of one
unit is the same as the size of one degree Celsius. Water freezes at 273.15K; water boils at 373. 15K. [K=C+273.15°,
F = (9/5) C+32°] . For calculation purposes, we take 0oC = 273K.

5.2 Celsius Temperature Scale


Celsius Temperature Scale - Temperature Scale according to which the temperature difference between the
reference temperature of the freezing and boiling of water is divided into 100 degrees. The freezing point is taken
as zero degree Celsius and the boiling point as 100 degrees Celsius. The Celsius scale is widely known as the
centigrade scale because it is divided into 100 degrees.

5.3 Fahrenheit Scale


Fahrenheit temperature scale is a scale based on 32 for the freezing point of water and 212 for the boiling point of
water, the interval between the two being divided into 180 parts.
The 18th –century German physicist Daniel Gabriel Fahrenheit originally took as the zero of his scale the temperature
of an ice- salt mixture and selected the value of 30 and 90 for the point of water and normal body temperature,
respectively; these later were revised to 32 and 96, but the final scale required an adjustment to 98.6 for the latter
value

Kelvin Celsius Fahrenheit


Water boils 373.16K 1000C 2120F
Water freezes 273.16K 00 320F
Absolute zero 0k -273.160C -459.70F
P hysi cs | 15.3

o o
K C F
Water boils 373.15 100 212
Body temperature 31015 37 98.6
Water freezes 373.15 0 32

Absolute zero 0 -273.15 -459.60

Figure 15.1

PLANCESS CONCEPTS

For easy conversion of temperature units, remember the following equation


 C−0   F − 32   K − 273 
=
  =
   
 100 − 0   212 − 32   373 − 273 
Where C, F and K are respectively temperatures in Celsius, Fahrenheit and Kelvin scale. Note the values
used in denominator, are actually the boiling and melting points of water in respective scales, so quite
easy to remember.
Nitin Chandrol (JEE 2012, AIR 134)

Illustration 2: Express a temperature of 60° F in degree Celsius and in Kelvin.  (JEE MAIN)
9 9
TC = TC = T − 273.15
T(i)− 273.15 .......(i); (ii).......(i);
TF = 32 + TF T=32 + TC ..........(ii) ..........(ii)
5 C 5
Sol: (Using above formulas) Find the temp in Celsius first, then in Kelvin as kelvin and Celsius have more simple
5 5
relation. Substituting TF =60°C in Eq. (ii); T
=C
9
( TF − 32
= ) 9
( 60°C − 32°C=) 15.55°C
From Eq. (i) T=Tc +273.15=15.55°C+273.15=288.7K

Illustration 3: Calculate the temperature which has the same value on (i) the Celsius and Fahrenheit (ii) Fahrenheit
and Kelvin scales. (JEE ADVANCED)
Sol: The value of temp which shows same reading on the Celsius as well as on the Fahrenheit (i part) and on the
Kelvin and Fahrenheit (ii part).
9
(i) Let the required temperature be x°, now=
TF T + 32
5 C
160
or 5T
= F 9TC + 160 or 5X=9X+160 ∴ X = =−40° ⇒ -40°C =-40°F
−4
TF − 32 Tk − 273.15
(ii) Let the required temperature be x° =
180 100
X − 32 Tk − 273.15
∴ =
180 100
On solving we get, X = 574.6
1 5 . 4 | Calorimetr y and Thermal Expansion

5.4 Triple Point of Water


The triple point of water is that unique temperature and pressure at which water can coexist in equilibrium between
the solid, liquid and gaseous states. The pressure at the triple point of water is 4.58 mm of Hg and the temperature
is 273.16K (or 0.01°C). The absolute or Kelvin temperature T at any point is then defined, using a constant volume
p
gas thermometer for an ideal gas= as: T 273.16 × [ideal gas; constant volume]
ptp
In this relation, ptp is the pressure in the thermometer at the triple point temperature of water and P is the pressure
in the thermometer when it is at the point where T is being measured. Note that if we let P=Ptp in this relation,
T=273.16 K as it must.

Illustration 4: When in thermal equilibrium at the triple point of water, the pressure of Hg in a constant volume
gas thermometer is 1020 Pa. The pressure of He is 288 pa when the thermometer is in thermal equilibrium with
liquid nitrogen at its normal boiling point. What is the normal boiling point of nitrogen as measured using this
thermometer? (JEE MAIN)

Sol: As we consider volume of the fluid to be constant, and hence T/P ratio remains constant, Normal boiling point
p
of nitrogen
= is T 273.16 × ; Here P=288 Pa; ptp =1020 Pa
ptp
288
T 273.16 ×
∴= = 77.1 K
1020

6. HEAT CAPACITY
The heat capacity of a body is defined as the amount of heat required to raise its temperature by 1°C. It is also
known as the thermal capacity of the body. Suppose a body has mass m and specific heat c. Heat capacity = Heat
required to raise the temperature of the body by 1°C = mc × 1 =mc
∴ Heat capacity =mc
Hence heat capacity of a body (solid or liquid) is equal to the product of its mass and specific heat. Clearly, the SI
unit of heat capacity is J/°C or J/K. The greater the mass of a body, the greater is its heat capacity.

7. SPECIFIC HEAT CAPACITY


When we supply heat to a solid substance (or liquid), its temperature increases. It is found that the amount of heat
Q absorbed by the solid substance (or liquid), is
(i) Directly proportional to the mass (m) of the substance i.e., Q ∝ m
(ii) Directly proportional to the rise in temperature ( ∆T ) i.e.. Q∝ ∆T
Combining the two factors, we have, Q ∝ m ∆T  ……. (i)
or Q = cm ∆T
Where C is constant of proportionality and is called specific heat capacity or simply specific heat of the substance.
Q
From eq. (i), we have c =  ……. (ii)
m∆T
If m = 1 kg and ∆T = 1°C, then c = Q.
Hence the specific heat of a solid (or liquid) may be defined as the amount of heat required to raise the temperature
of 1kg of solid (or liquid) through 1°C (or 1K). It is clear from eq. (ii) that SI unit of specific heat is J kg -1 °C-1 or J kg -1 K-1.
P hysi cs | 15.5

PLANCESS CONCEPTS

Don’t get confused here with the terminology of heat capacity and specific heat capacity. Always
remember that Specific heat capacity is the property of material and heat capacity is property of a given
body.
B Rajiv Reddy (JEE 2012, AIR 11)

Illustration 5: A geyser heats water flowing at the rate of 3.0 liters per minute from 27°C to 77°C. If the geyser
operates on a gas burner, what is the rate of combustion of the fuel if its heat of combustion is 4.0 × 104 J g-1?
 (JEE ADVANCED)
Sol: The total heat required to increase the temperature of the water is equal to the heat supplied by the combustion
of gas per minute. Mass of 3 liters of water =3kg ∴ Mass of water flowing per minute, m= 3 kg=3000 g min -1
Rise of temperature, Δ θ=77-27=50°C; Heat absorbed by water per minute= mc ∆ θ =3000 × 1 × 50cal
= 3000 × 1 × 50 × 4.2 J min -1 = 630000 J min-1
∴ Heat supply by gas burner= 630000 J min-1 and heat of combustion of fuel = 4.0 × 104 J g-1

630000
∴ Rate of combustion of fuel = = 15.75 g min-1
4.0 × 10 4

Illustration 6: A copper block of mass 60 g is heated till its temperature is increased by 20°C. Find the heat
supplied to the block. Specific heat capacity of copper= 0.09 cal g-1 °C -1. (JEE MAIN)

Sol: Here the heat is utilized to increase the temperature of the block only.
The heat supplied is Q=ms ∆ θ = (60 g) (0.09 cal g-1 ° C-1) (20°C) = 108 cal.
The quantity ms is called the heat capacity of the body. Its unit is J K-1. The mass of water having the same heat
capacity as given body is called the water equivalent of the body.

8. MOLAR SPECIFIC HEAT CAPACITY FOR SOLIDS OR LIQUIDS


The molar specific heat of a solid (or liquid) is defined as the amount of heat required to raise the temperature of
1 mole of the solid (or liquid) through 1°C (or 1K). It is denoted by the symbol C. Therefore, the amount of heat Q
required to raise the temperature of n moles of a solid (or liquid) through a temperature change ∆T is given by;
Q n C∆T
=
It is clear that SI unit of C is J mol-1 K-1. For any material of mass m and molecular mass M, the number of moles
m m
n=m/M. ∴ Q = C∆T also ∴ Q = m C∆T ∴ C∆T= mc ∆T or C = Mc  ...(i)
M M
Eq. (i) gives the relation between molar specific heat C and the ordinary specific heat.

9. MOLAR SPECIFIC HEAT CAPACITY FOR THE GASES


The amount of heat required to increase the temperature of 1 mole of a gas through 1°C is called molar heat
capacity.
Mass of the gas
The number of moles, n, in mass m of the gas is given by n =
Molecular weight
1 5 . 6 | Calorimetr y and Thermal Expansion

9.1 Molar Specific Heat at Constant Volume, Cv:


If (∆Q)V is the heat required to raise the temperature of mass m gm or n moles of gas of molecular weight M at

constant volume through temperature ∆ T, ( ∆Q )V= mc v ∆=


T nMc v ∆T
= nC v ∆T

Where C v molar specific is heat at constant volume and is equal to McV .

9.2 Molar specific heat at Constant Pressure, Cp:


If (∆Q)P is the heat required to raise the temperature of mass m gm or n moles of gas of molecular weight M at
constant pressure through temperature ∆ T, ( ∆Q )p= mcp ∆=
T nMcp ∆T= nCp ∆T

Where Cp molar specific is heat at constant volume and is equal to Mcp .


5R 3R Cp 5 7R 5R Cp 7
For monatomic gases, Cp = ,CV = ; γ= =
= = 1.67 ; for diatomic gases, Cp = ,CV γ= = = 1.4 ;
2 2 Cv 3 2 2 Cv 5
R γR
R ; where
Mayer’s relation gives, Cp − CV = = CV = ,C
γ −1 p γ −1

Illustration 7: How much heat is required to raise the temperature of an ideal monoatomic gas by 10 K if the gas
is maintained at constant pressure?  (JEE MAIN)

Sol: The process is at constant pressure here. Formula for heat capacity of gas at constant pressure is used.

The heat required is given by = Q n Cp ∆T Here n=1 & ∆T =10 K;


5 5 5
Cp= R= × 8.3 J mol−1 K −1 ; ∴ Q =1× × 8.3 × 10 = 207.5 J
2 2 2

PLANCESS CONCEPTS

Without calculation, one can tell that Cp is always greater that Cv. Think of a situation in which we need
to raise the temperature of same amount of gas in constant pressure conditions and constant volume
conditions. It is quite obvious that in constant volume conditions all the heat will be used up to raise
internal energy of gas. We see that the rise in internal energy of gas is same in both cases as increase in
temperature is same. However, we see that for constant pressure conditions, more heat is required as
some of it will also be used to expand the volume. This condition requires that Cp must be greater than Cv.
Anand K (JEE 2011, AIR 47)

Illustration 8: Calculate the amount of heat necessary to raise the temperature of 2 moles of He gas from 20°C to
50°C using (i) constant –volume process and (ii) constant-pressure process.
For He CV =1.5 R and Cp = 2.49 R.  (JEE ADVANCED)

Sol: Heat capacity at constant volume and constant pressure are applicable here.
(i) The amount of heat required for constant –volume process is Q =
v C v ∆T ; Here n=2 moles;
C=
V R 1.5 × 8.314J mol−1 0 C−1 ; ∆T =50-20=30°C; Q v =2 × (1.5 × 8.314) × 30 =748 (ii) The amount of heat
1.5 =
required for constant –pressure process is Q=
p nCp ∆T
Here n=2 moles; =
Cp 2.49
= R 2.49 × 8.314J mol−1 0 C−1 ; ∆T = 30 °C
∴ Q p= ( 2.49 × 8.314 ) × 30= 1242J
P hysi cs | 15.7

Since the temperature rise is the same in the two cases, the change in internal energy is same i.e, 748 J. however, in
constant-pressure excess heat supplied =1242-748=494 J. This extra heat of 494 J went into the work of expansion
of the gas.

10. LATENT HEAT


The amount of heat required to change a unit mass of a substance completely from one state to another at
constant temperature is called the latent heat of the substance.
If a substance of mass m required heat Q to change completely from one state to another at constant temperature,
Q
then, the latent heat L = . The SI unit of latent heat of a substance is J kg-1. There are two types of latent heats
m
viz. latent heat of fusion and latent heat of vaporization.
(a) Latent heat of fusion. We know that a solid changes into liquid at a constant temperature which is called the
melting point.
The amount of heat required to change the unit mass of solid mass into its liquid state at constant temperature
is called the latent heat of fusion of the solid.
For example, the latent heat of fusion of ice is 334 J/kg. It means to change 1 kg of ice at 0°C into liquid water
at 0°C, we must supply 334 KJ of heat.

(b) Latent heat of vaporization. We know that a liquid changes into gaseous state at a constant temperature
which called the boiling point. The amount of heat required to change the unit mass of a liquid into its
gaseous state at constant temperature is called latent heat of vaporization of the liquid.

Illustration 9: A piece of ice of mass 100 g and at temperature 0°C is put in 200 g of water at 25°C. How much
ice will melt as the temperature of the water reaches 0°C? The specific heat capacity of water =4200 JK -1 and the
specific latent heat of fusion of ice= 3.4 × 105 JK -1.  (JEE MAIN)

Sol: Total heat lost by the water equal to the total heat gained by the ice.

The heat released as the water cools down from 25°C to 0°C is
Q
= ms∆θ
= ( 0.2 kg) ( 4200 Jk −1 K −1 ) ( 25K=) 21000 J.
Q 21000 J
The amount of ice melted by this much heat is given by m
= = = 62g
L 3.4 × 105 Jkg−1

11. WATER EQUIVALENT


The water equivalent of a body is defined as the mass of water that will absorb or lose the same amount of heat as
the body for the same rise or fall in temperature. The water equivalent of a body is measured in kg in SI unit and in
g in C.G.S. units. Suppose the water equivalent of a body is 10 kg. It means that if the body is heated through, say
10°C, it will absorb the same amount of heat as absorbed by 10 Kg of water when heated through 10°C. Consider
a body of mass m and specific heat required to raise the temperature of the body through ∆T is = Q cm ∆T  ...(i)
Suppose w is water equivalent of this body. Then, by definition, Q is given by:
Q= w ∆T From eqs. (ii) And (ii), we have, w ∆T = cm ∆T or w= m c
Thus the water equivalent of a body is numerically equal to the product of the mass of the body and its specific
heat. Note that mc is the heat capacity of the body. Therefore, we may conclude that water equivalent and heat
capacity of a body are numerically equal.
1 5 . 8 | Calorimetr y and Thermal Expansion

Illustration 10: A calorimeter of water equivalent 15g contains 165 g of water at 25°C. Steam at 100° is passed
through the water for some time. The temperature is increased to 30°C and the mass of calorimeter and its contents
are increased by 1.5 g. Calculate the specific latent heat of vaporization of water. Specific heat capacity of water is
1 cal g -1 °C-1.  (JEE ADVANCED)

Sol: The change in mass of the content of calorimeter is due to formation of more water from condensation of
steam and all comes to the same temperature.
let L be the specific latent heat of vaporization of water. The mass of the steam condensed is 1.5 g. Heat lost in
condensation of steam is Q1 = (1.5g) L . The condensed water cools from 100°C to 30 °C. Heat lost in the process is

Q2 (1.5g) (=
1 calg−1 0 C−1 ) ( 70°C ) 105cal.
Heat supplied to the calorimeter and to the cold water during the rise in temperature from 25°C to 30°C is
Q3= (15g + 165g) (1 calg −1 0
)
C −1 (5°C=
) 900cal.
If no heat is lost to the surrounding.
−1
(1.5g) L + 105cal =
900 cal or L= 530 cal g

Illustration 11: The water equivalent of a body is 10 kg. What does it mean?  (JEE MAIN)

Sol: It means that if a body is heated through say 5°C, it will absorb the same amount of heat as absorbed by 10
kg of water when heated through 5°C.

12. MECHANICAL EQUIVALENT OF HEAT


In early days, heat was not recognized as a form of energy. Heat was supposed to be something needed to raise
the temperature of a body or to change its phase. Calorie was defined as the unit of heat. A number of experiments
were performed to show that the temperature may also be increased by doing mechanical work on the system.
These experiments established that heat is equivalent to mechanical energy and measured how much mechanical
energy is equivalent to a calorie. If mechanical work W produces the same temperature change as heat H, we write,
W=JH. Where J is called mechanical equivalent of heat. It is clear that if W and H are both measured in the same
unit then J=1. If W is measured in joule (work done by a force of 1 N in displacing an object by 1 m in its direction)
and H in calorie (heat required to raise the temperature of 1 g of water by 1°C) then J is expressed in joule per
calorie. The value of J gives how many joules of mechanical work is needed to raise the temperature of 1 g of water
by 1°C.

Illustration 12: Assuming that the density of air at N.T.P=0.0013 g/cc, CP = 0.239 cal g-1K-1 and the ratio
CP/CV = 1.40, calculate the mechanical equivalent of heat. (JEE MAIN)
Sol: Compare the value of Gas Constant (R) by calculating in different unit (Calorie and Joule). =
R Cp − CV . And
R=PV/T, then find the ratio (in joule/in calorie).
Cp 0.239
Now, Cp = 0.239 cal g-1 K −1 ; Cp / CV =1.40; ∴ CV = = = 0.171cal g−1 K-1
1.40 1.40
1 10−6
Volume of 1 g of air at N.T.P. = cc = m3
0.0013 0.0013
10−6 10−3
Volume of 1 kg ( = 1000g) of air at N.T.P., V= × 1000 m3
0.0013 0.0013

Normal pressure, p = hρg = 0.76 × 13600 × 9.8 = 101292.8 Nm-2


P hysi cs | 15.9

Normal temperature, T = 273 K


PV 10−3 1
Gas constant r for 1 kg of air is given by; R = =101292.8 × × =285.4 Jkg−1 K −1
T 0.0013 273
R Cp − CV . It means that if 1 kg of air is heated through 1°C (or 1K) first at constant pressure and then
Note that =
at constant volume, then extra heat needed for constant-pressure process to do this work of 285.4 J i.e., W = 285.4 J
Heat supplied to do work is =
Q 1000g × Cp × 1K − 1000g × CV × 1K
= 1000g × 0.239 × 1K − 1000g × 0.171× 1K
W 285
= 239-171=68 cal; Now W=JQ ; J
∴= = = 4.2J / cal
Q 68

13. LAW OF HEAT EXCHANGE


When a hot body is mixed or kept in contact with a cold body, the hot body loses heat and its temperature falls.
On the other hand, the cold body gains heat and its temperature rises. The final temperature of the mixture will lie
between the original temperatures of the hot body and the cold body. If a system is completely isolated, no energy
can flow into and out of the system. Therefore according to the law of conservation of energy, the heat lost by one
body is equal to the heat gained by other body i.e. Heat lost = heat gained
This is known as law of heat exchange.

THERMAL EXPANSION
1. DEFINITION OF THERMAL EXPANSION
It is the expansion due to increase in temperature. Most substances expand
when they are heated. Thermal expansion is a consequence of the change
in average separation between the constituent atoms of an object. Atoms of
an object can be imagined to be connected to one another by stiff springs
as shown in figure. At ordinary temperatures, the atoms in a solid oscillate
about their equilibrium positions with an amplitude of approximately 10-11 m.
The average spacing between the atoms is about 10-1°m. As the temperature
of the solid increases, the atoms oscillate with greater amplitudes, as a result
the average separation between them increases, and consequently the object Figure 15.2
expands. More precisely, thermal expansion arises from the asymmetrical
nature of the potential energy curve.

2. THERMAL EXPANSION OF SOLIDS

2.1 Linear Expansion


When a solid substance is heated, most of them generally expand. If a solid has a length L0 and has a very small
area of cross-section, at a temperature T0, its length increases to LT when its temperature is increased by ∆T . The
increase in length, ∆L , is then given by,

∆L = L T − L0 = L0 × α × ∆T Where α is the coefficient of linear expansion which is given by


L T − L0
=
α ; =
L T L0 (1 + α ∆T)
L0 ∆T
The coefficient of linear expansion is equal to the increase in length per unit length per degree rise of temperature.
1 5 . 1 0 | Calorimetr y and Thermal Expansion

The SI unit of α is /°C or /K. Its value is different for different solid materials. For example α for aluminum is
2.4 × 10−5 / 0 C whereas for brass, its value is 2.0 × 10 -5/oC. Note that the change in temperature ∆T will be the same
whether it is measured in Celsius scale or on the Kelvin scale: ∆T 0 C = ∆T K.

2.2 Superficial Expansion


If a solid plate of area A0 and of very small thickness is heated through a temperature ∆T so that its area increases
to A T, then the increase in area ∆A is given by
A T − A0
∆A = A T − A0 = βA0 ∆T or β=
A0 ∆T
Where β is called the coefficient of superficial expansion. β = 2α
Hence the coefficient of superficial expansion of a solid may be defined as the fractional change in surface area
( ∆S / S ) per degree change in temperature. Its SI unit is also / 0 C or /K.
Note that exchange in temperature ∆T will be the same whether it is measured on the Celsius scale or on the Kelvin
scale.

2.3 Volume Expansion


If a solid of initial volume V0 at any temperature is heated so that its volume is increased to VT with increase of
temperature ∆T, the increase in volume, ∆V, is given by
VT − V0 ∆V
∆ V= VT − V0 =γV0= ∆T ; γ =
V0 ∆T V0 ∆T
Where γ is called the coefficient of volume or cubical expansion. γ = 3 α
As the temperature of solid increases, the amplitude of oscillation of atoms increases which results in an increase
of average distance between atoms with increase of temperature due to which the volume increases. If ρ0 is the
density of a solid at 0°C and ρT is its density T°C, then for a constant mass m of the solid,
m m
ρ0 = And ρT = where V0 and VT are its respective volume at 0°C & T°C
V0 VT
ρ V V0 (1 + γT ) ρ
∴ 0 = T = = 1 + γT ∴ ρT = 0
ρT V0 V0 1 + γT
Hence coefficient of cubical expansion of a solid may be defined as the fractional change in volume ( ∆V / V ) per
degree change in temperature. Its SI unit is / °C or /K.

PLANCESS CONCEPTS

For anisotropic solids β = α1 + α2 and γ = α1 + α2 + α3 .


Here α1 , α2 and α3 are coefficients of linear expansion in X, Y and Z directions.
For solid value of are generally small so we can write density, d=d0 (1 − γ∆T ) (Using binomial expansion).
γ is not always positive. It can have a negative value.
E.g. For water, density increases from 0 to 4°C so γ is -ve (0 to 4°C) and for 4°C to higher temperature γ
is –ve. At 4°C density is maximum. Coefficients of thermal expansion are generally not independent of
temperature. But for JEE purpose you are supposed to assume it as a constant if not mentioned.
If α is not constant
1
(i) ( α varies with distance) Let α = ax +b; Total expansion= ∫ expansion of length dx = ∫ ( ax + b ) dx∆t
0
P hysi cs | 15.11

PLANCESS CONCEPTS

T2

(ii) (α Varies with temperature) ; Let α =f(T) ; ∆ = ∫ α 0dT


T1
Caution: If α is in°C,
x
Then put T1 and T2 in °C. Similarly if α is in K then put T1 and T2 dx
in K.
If you have a difficulty in remembering the definition of different
capacity then just look at the units given heat capacity and figure Figure 15.3
out whether it’s per unit mass/mole/ or for entire mass.
Yashwanth Sandupatla (JEE 2012, AIR 821)

3. PRACTICAL APPLICATION OF THERMAL EXPANSION OF SOLIDS


There are a large number of important practical applications of thermal expansion of solids. However, we shall brief
only a few of them by way of illustration.
(a) While laying the railway tracks, a small gap is left between the successive lengths of the rails. This gap is
provided to allow for the expansion of the rails during summer. If no gap is left, these expansions cause the
rails to buckle.
(b) When the iron tyre of a wheel to be put on the wheel, the tyre is made slightly smaller in diameter than that
of wheel. The iron tyre is first heated uniformly till its diameter becomes more than that of the wheel and is
then slipped over the wheel. On cooling the tyre contracts and makes a tight fit on the wheel.
(c) In bridges, one end is rigidly fastened to its abutment while the other rests on rollers. This provision allows
the expansion and contraction to take place during changes in temperature.
(d) The fact that a solid expands on heating and contracts on cooling is utilized in riveting e.g., riveting two
metal plates together, joining steel girders etc. For joining two steel plates, holes are drilled between them.
The rivets (small rods) are made red hot and inserted in the holes in the plates. The ends of the rivets are
hammered into the head. After some time, the rivets contract on being and hold the plates very tightly.
(e) The concrete roads and floor are always made in sections and enough space is provided between the sections.
This provision allows expansion and contraction to take place due to change in temperature.

PLANCESS CONCEPTS

•• If a solid object has a hole in it, what happens to size of the hole, when
the temperature of the object increases? A common misconception is that
if the object expands, the hole will shrink because material expands into
aa
the hole. But the, truth is that if the Object expands the hole will expand Ti
too, because every linear dimension of an object change in the same way
when the temperature changes.
b
•• Effect of temperature on the time period of a pendulum:
The time Period of a simple pendulum is given by. Figure 15.4


T=
2π ; or T∝ 
g
1 5 . 1 2 | Calorimetr y and Thermal Expansion

PLANCESS CONCEPTS

As the temperature is increased length of the pendulum and hence,


time period gets increased or a pendulum clock becomes slow and a + a
it’s loses the time. Time lost in time t(by a pendulum clock whose a
Ti + T

actual time period is T and changed time period at some higher


b + b
 ∆T 
temperature is T’) is ∆t = t .
 T'  Figure 15.5
Similarly, if the temperature is decreased the length and hence, the
time period gets decreased. A pendulum clock on this case runs fast
 ∆T 
and it gains the time. Time gained in time t is the same, i.e., ∆t = t .
 T' 
Gv Abhinav (JEE 2012, AIR 329)

Illustration 13: A steel ruler exactly 20 cm long is graduated to give correct measurements at 20°C.
(a) What happens to the reading if the temperature decreases below 20°C?
(b) What is the actual length of the ruler at 40°C? (JEE MAIN)

Sol: Lowering the temperature, shorten the scale from 1 m of original length. It’ll show length of 1m lengthier than
its length. And hence will show 1m to be more than 1m. It will now measure more. And reverse, in case of increasing
the temperature.
(a) If the temperature decreases, the length of the ruler also decreases through thermal contraction. Below 20°C,
each centimeter division is actually somewhat shorter than 1.0 cm, so the steel ruler gives reading that are too long.
(b) At 40°C, the increase in length of the ruler is

∆ = α∆T = ( 20 ) (1.2 × 10−5 )( 400 − 200 ) =0.48 × 10−2 cm


∴ The actual length of the ruler is,  ' =  + ∆ = 20.0048 cm

Illustration 14: A second pendulum clock has a steel wire. The clock is calibrated at 20°C. How much time does the
clock lose or gain in one week when the temperature is increased to 30° C?

( C)
−1
αsteel =1.2 × 10−5 0
 (JEE ADVANCED)

Sol: Increment in length increase the time period of oscillation.


The time period of second’s pendulum is 2 seconds. As the temperature increases, length and hence, time
1
period increases, clock becomes slow and it loses the time. The change in time period is ∆= T Tα∆θ =
2
 1
(
  ( 2 ) 1.2 × 10
2
−5
)( )
300 − 200 = 1.2 × 10−4 s ∴ New time period is, T’ = T + ∆T = (2 + 1.2 × 10-4) = 2.00012s

 ∆T   1.2 × 10−4 
∴ Time lost in one week= ∆t  =  t   ( 7 × 24 × 3600
= ) 36.28s
 T'  
 2.0012 
P hysi cs | 15.13

4. THERMAL EXPANSION OF LIQUIDS


As a liquid in a vessel acquires the shape of the vessel, its heating increases the volume of the vessel initially due
to expansion of the vessel which decreases the level of the liquid initially. When the temperature of the liquid is
increased further, it increased the volume of the liquid. Thus the observed or apparent expansion of the liquid is
lesser than the real level of the liquid when the temperature increases. Thus the apparent expansion of the liquid is
lesser than the real expansion of the liquid which gives a value of coefficient of real expansion more than that for
the coefficient of apparent expansion.
The coefficient of real expansion, gr, of a liquid is defined as the real increase in volume per degree rise of temperature
per unit original volume of the liquid.
Real increase in volume
gr =
Orignal volume × rise in temperature

The coefficient of apparent expansion, γ a of a liquid is defined as the ratio of the observed increase in volume of
the liquid with respect to the original level before heating per degree rise of temperature to the original volume
of the liquid.
Observed increased in volume
γa =
Orignal volume × rise in temperature

If γ g is the coefficient of cubical expansion of the material of the vessel, then γr = γ a + γ g ; γ g = 3α


ρ
Then density of the liquid ρT at temperature T is related to density ρo at °C as ρT = 0
1 + γT

Where γ is the coefficient of real expansion of the liquid and T is the increase in temperature.
It is clear that γr > γ a and both are measured unit °C-1. It can be shown that: γr = γ a + γ g
Where γ g is the coefficient of cubical expansion of glass (or material of the container).

Illustration 15: Find the coefficient of volume expansion for an ideal gas at constant pressure.  (JEE MAIN)

Sol: Recall the formula for coefficient of volume expansion for ideal gas.
For an ideal gas PV = nRT
dV nR 1 dV nR nR 1 1
As P is constant, we have P.dV= nRdt ∴ = or γ = . = = = ∴ γ=
dT P v dT PV nRT T T

5. THERMAL EXPANSION OF GASES


The molecules in an ideal gas have only kinetic energy due to their motion but do not possess any potential energy.
The thermodynamic state of any gas is defined in terms of its pressure, volume and temperature denoted as P, V
and T respectively. A change in one of these quantities produces a corresponding change in the other quantities
depending upon the condition under which the transformation take place. Such changes are governed by the
following gas laws:

5.1 Boyle’s Law


The pressure of given mass of a gas is inversely proportional to its volume if temperature T remains constant
1
P∝ or PV = cons tant ;
V
If the pressure P1 and volume V1 changes to the respective values P2, V2 when the temperature remains
Constant, then P1 V1 = P2 V2 .
1 5 . 1 4 | Calorimetr y and Thermal Expansion

5.2 Charles’s Law of Volume


The volume V of a given mass of a gas is directly proportional to its absolute temperature, T, when its pressure
V
remains constant. V ∝ T = cons tant
T
V V
If the volume V1 and temperature T1 are respectively changed to V2 , T2 at constant pressure, then 1 = 2 .
T1 T2
Where temperatures T1 and T2 are in Kelvin scale.

If V0 and Vt are volume of the gas at 0°C and t°C respectively,

Vt V0  t 
= ; Vt= V0 1 +  V0= 1 + α v t 
273 + t 273  273 
Where α V is the volume coefficient of a gas and is equal to 1/273.

5.3. Gay Lussac’s Law of Pressure


The pressure of a given mass of a gas is directly proportional to its absolute temperature provided the volume of
P
the gas is kept constant. PαT or cons tant
T
If the pressure and temperature P1 , T1 is change respectively to P2 , T2 at constant volume, then
P1 P2
= = constant.
T1 T2
 t 
If Pt and P0 are pressure of the gas at t °C & 00 respectively, then Pt = P0 1 + α=
pt (
P0 1 +
 273 
 )
Where αp is equal to the pressures coefficient of the gas which also equal to 1/273.

5.4 Gas Equation


PV P1V1 P2 V2
If the above mentioned three laws are combined, then =constant; = = cons tant .
T T1 T2
The value of the constant depends on the mass of the gas.
If the gas has n moles, PV=nRT which is called the equation of the state of an ideal gas. R is called the universal or
molar gas constant and its value in S.I. units is 8.314J. mol-1 K-1.

6. RELATION BETWEEN COEFFICIENTS OF EXPANSION


We shall now show that for solid, the approximate relations between α , β and γ are:
β = 2α and γ = 3α ;

(a) Relation between β and α . Consider a square plate of side  0 at °C and 1 at t °C.
1 =  0 (1 + αt ) ;

Area of plate at 0°C, A0= 20 ;

Area of plate at t°C, A1 = 12 = 20 (1 + αt ) = A0 (1 + αt )


2 2

Also Area of plate at t°C, A1 = A0 (1 + β t )

∴ A0 (1 + αt ) = A0 (1 + βt ) or ∴ 1 + α2 t2 + 2αt = 1 + βt
2

Since the value of α is small, the term α2 t2 may be neglected. ∴ β = 2α


P hysi cs | 15.15

The result is altogether general because any flat surface can be regarded as a collection of small squares.
(b) Relation between γ and α . Consider a cube of side  0 at °C and 1 at t °C.

∴ 1 =  0 (1 + αt ) ; Volume of cube at 0C, V0 = 30 ;


13= 30 (1 + αt ) = V0 (1 + αt )
3
Volume of cube at t °C, V=
1

Volume of cube at t °C, V1 = V0 (1 + γt ) ; ∴ V0 (1 + αt ) = V0 (1 + γt )


3
Also
Or 1 + 3αt + 3α2 t2 + α3 t3 = 1 + γt
Since the value of α is small, we can neglect the higher power of α .

∴ 3αt = γt or γ = 3α
Again, result is general because any solid can be regarded as a collection of small cubes.

7. VARIATION OF DENSITY WITH TEMPERATURE


Variation of Density with temperature: Most substances expand when they are heated, i.e.. Volume of a given mass
1
of a substance increases on heating, so the density should decrease (as ρ ∝ )
V
m 1
Let us see how the density (ρ) varies with increase in temperature. ρ = or ρ ∝ (for a given mass)
v V
ρ' V V 1 ρ
∴ = = = ; ∴ ρ' =
ρ V' V + ∆V 1 + γ∆T 1 + γ∆T

This expression can also be written as, ρ ' = ρ (1 + γ∆T )


−1

As γ is small. (1 + γ∆T ) ≠ 1 − γ∆T ∴ ρ '  ρ (1 − γ∆T )


−1

Illustration 16: A glass flask of volume 200 cm3 is just filled with mercury at 20° C. How much mercury will over
flow when the temperature of the system is raised to 100°C? The coefficient of volume expansion of glass is 1.2 ×
10-5/°C and that of mercury is 18 × 10-5/°C.  (JEE MAIN)
Sol: Increase in temperature, increase the volume of both, mercury as well as flask but mercury expands more than
flask because the coefficient of volume expansion of mercury is more than of flask.

( )
The increase in the volume of the flask is ∆V =γR V∆T = 1.2 × 10−5 × ( 200 ) × (100 − 20 ) =0.19cm3

( )
The increase in the volume of the mercury is ∆V' =γm V∆T = 18 × 10−5 × ( 200 ) × (100 − 20 ) =2.88cm3

∴ The volume of the mercury that will overflow ∆V'− = = 2.69cm3


∆V 2.88 − 0.19

Illustration 17: A sheet of brass is 40 cm long and 8 cm broad at 0 °C. If the surface area at 100°C is 320.1 cm2, find
the coefficient of linear expansion of brass. (JEE MAIN)
Sol: Calculate the coefficient of area expansion, coefficient of linear exp. Equal to half of coeff. of area expansion.
Surface area of sheet at 0°C, A0 = 40 × 8 =320cm2
Surface area of sheet at 100°C, A100=320.1cm2
Rise in temperature, ∆T= 100 − 0= 100°C
Increase in surface are ∆A
= A100 − A=
0 = 0.1 cm2
320.1 − 320

Coefficient of surface expansion β is given by; β= ∆A 0.1


= = 31 × 10 −7 / °C
A0 × ∆T 320 × 100
1 5 . 1 6 | Calorimetr y and Thermal Expansion

β 31 × 10 −7
∴ Coefficient of linear expansion, α= = = 15.5 × 10 −7 / °C
2 2

8. THERMAL STRESS
If the ends of rods of length L0 are rigidly fixed and it is heated, its length L0 tends to increase due to increase in
temperature ∆T , but it is prevented from expansion. It results in setting up compressive or tensile stress in the rod
which is called the thermal stress.
Stress Y∆L YαL0 ∆T
As Y= , Stress=Y × Strain = = = Yα∆T The force, F, on rigid support is given by.
Strain L0 L0
Where A is area of cross-section of the rod.
If ∆T represent a decrease in temperature, then F/A and F are tensile stress and tensile force respectively.
Note: When the temperature of a gas enclosed in a vessel of rigid material is increased, then thermal stress is equal
to the increase in pressure ( ∆P ) and is given by: ∆P = Kγ∆T
Where K= bulk modulus of gas; γ =coefficient of cubical expansion; ∆T =increase in temperature
V∆P V∆P
V V (1 + γ∆T ) or V − V= Vγ∆T
Proof.= or ∆V= Vγ∆T now K = = ∴ ∆P =γK∆T
∆V γV∆T

Illustration 18: A steel wire of 2.0mm cross-section is held straight (but under no tension) by attaching it firmly to
two rigid walls at a distance 1.50 m apart, at 30o C. If the temperature now decreases to -10oC, and if the end points
remain fixed, what will be the tension in the wire? For steel, Y = 200 000M Pa  (JEE MAIN)

Sol: Here the concept of strain is applicable with linear expansion. Decreased temp. tends to decrease the length
of wire but strain keep it intact.
Conceptualize: If free to do so, the wire would contract but since we have tied its ends, it will not contract and
maintain its original length.
Classify: Until now we have seen when the length of a wire is changed, it produces strain and hence stress. This
situation is different as strain will be produced because of wire maintaining its length. At a lower temperature
the wire would have an unstrained length smaller than the original length. However since its ends are tied, it will
maintain its length but develop strain. Or in other words it has longer length than what it would have had at this
temperature if not tied at its ends.
Compute: If free to do so, the wire would contract a distance ∆L as it cooled, where

( ) ( )
∆L =αL, ∆T = 1.2 × 10−5 0 C−1 (1.5m) 400 C =7.2 × 10−4 m
But the ends are fixed. As a result, forces at the ends must, in effect, stretch the wire this same length, ∆L . Therefore,

from Y= (F / A ) / ( ∆L / L ) , we have tension F=


YA∆L
=
( )( )(
2 × 1011 N / m2 2 × 10−6 m2 7.2 × 10−4 m )
= 192 N
L 1.5m
(
Conclude: Strictly, we should have substituted 1.5 ± 7.2 × 10−4 ) m for L in the expression of tension. However. The
error incurred in not doing so, is negligible.

PROBLEM-SOLVING TACTICS
While solving a problem of heat transfer in these cases, do look for state changes because that’s where students
generally make a mistake. State changes cause some of the energy to be used up as latent heat and hence must
be taken care of always.
P hysi cs | 15.17

FORMULAE SHEET
 1. Type of thermal expansion
Coefficient of expansion For temperature change ∆t change in
1 ∆
(i) Linear α =Lim Length ∆=  0 α∆t
∆t →0  0 ∆t

1 ∆A
(ii) Superficial β =Lim Area ∆A= A0β∆t
∆t →0 A 0 ∆t

1 ∆V
(iii) Volume γ =Lim Volume ∆V= V0 γ∆t
∆t →0 V0 ∆t

• For isotropic solids α1 = α2 = α3 = α(let) so β = 2α and γ = 3α


• For anisotropic solids β = α1 + α2 and γ = α1 + α2 + α3 Here α1 , α2 and α3 are coefficient of linear
expansion in X, Y, and Z directions.

Variation in density: With increase of temperature volume increases so density decreases and vice-versa.
ρ0

= ρ ≈ ρ0 (1 − r ∆ T)
(1 + γ∆t )
Thermal Stress: A rod of length  0 is clamped between two fixed walls with distance  0 .
F
If temperature is changed by amount ∆t then stress= (area assumed to be constant)
A
∆ F/A F 0 F
Strain= ; so,=Y = − or F=YA α∆t
0 ∆ /  0 A∆ Aα∆t F F
• ∆Q = mc∆T where c: Specific heat capacity l0
• ∆Q = nC∆T C: Molar heat capacity Figure 15.6
• mL L: latent heat of substance
Heat transfer in phase change : ∆Q =
• 1 Calorie= 4.18 joules of mechanical work
• Law of Calorimetry: heat released by one of the substances = Heat absorbed by other substances.

Solved Examples

JEE Main/Boards Heat required to take the ice from -10 °C to

Example 1: Calculate the amount of heat required to ( )


0 0 C = (1kg) 2100 JKg−1 K −1 (10K ) = 21000 J.
convert 1.00kg of ice at -10°C into steam at 100°C at
normal pressure. Specific heat capacity of ice = 2100 Heat required to melt the ice at 0 °C to water =
Jk-1 K-1, latent heat of fusion of ice=3.36 ×105 JKg−1 K −1 , (1kg) (3.36 ×105 )
336000 J .
JKg−1 =
specific heat capacity of water= 4200 JKg−1 K −1 and
Heat required to take 1 kg of water from 0 °C
latent heat of vaporization of water =2.25 ×106 JKg−1 .
( )
to 100 = (1kg) 4200JKg−1K −1 (100K ) = 420000J.
Sol: Here the temperature of ice and water changes
along with change in phases. i. e. ice to water and then Heat required to convert 1kg of water at 100°C
water to steam.
(
 into steam = (1kg) 2.25 ×106 JKg−1 = )
2.25 ×106 J.
1 5 . 1 8 | Calorimetr y and Thermal Expansion

Total heat required = 3.03 ×10


6
J. Example 4: A lead bullet penetrates into a solid object
and melts. Assuming that 50% of its kinetic energy
was used to heat it, calculate the initial speed of the
Example 2: A 5 g piece of ice at-20°C is put into 10g of water bullet. The initial temperature of the bullet is 27°C and
at 30°C. Assuming that heat is exchanged only between the its melting point is 327°C. Latent heat of fusion of lead
ice and the water, find the final temperature of the mixture.
= 2.25 ×10 4 JKg‒1 and specific heat capacity of lead =
Specific heat capacity of ice. = 2100 JKg‒1 °C-1 specific heat
capacity of water = 4200 Jkg-1 oC-1and latent heat of fusion 125JKg‒1 K-1
of ice = 3.36 × 105 JKg‒1.
Sol: Kinetic energy of bullet spatially converted into
Sol: Always proceed in similar questions assuming heat and melt it.
the final temperature to be the temperature of phase Let the mass of bullet = m.
change (i.e. 0 here)
Heat required to take the bullet from 27°C to 327°C =
The heat given by the water when it cools down from
30°C to 0°C is ( )
m × 125JKg−1 K −1 ( 300K )

( 0.0kg) ( 4200JKg−1 °
C−1 ) (30°C ) =
1260 J
(
= m × 3.75 × 10 4 JKg−1 )
Heat required to melt the bullet
The heat required to bring the ice to 0°C is

( 0.005kg) ( 2100 JKg−1 °


C−1 )( 20 C ) = 210 J.
° (
m 2.10 ×106 JKg−1 .
=× )
1
The heat required to melt 5 g of ice is If the initial speed be v, the kinetic energy is mv 2 and
2
( 0.005kg) (3.36 ×105 1680 J .
JKg−1 °C−1 = ) hence the heat developed is
1 1 2 1 2
 mv  = mv . thus,
22  4
We see that whole of the ice cannot be melted as the 1
required amount of heat is not provided by the water.
2
mv= m ( 3.75 + 2.5) × 10 4 JKg−1 or v= 500 ms-1
4
Also, the heat is enough to bring the ice to 0°C. Thus
the final temperature of the mixture is 0°C with some of Example 5: An aluminum vessel of mass 0.5 kg
the ice is melted. contains 0.2 kg of water at 20°C. A block iron of mass
0.2 kg at 100°C is gently put onto the water. Find the
Example 3: A thermally isolated vessel contains 100g equilibrium temperature of the mixture. Specific heat
of water at 0°C. When air above the water is pumped capacities of aluminum, iron and water are 910 JKg‒1 K‒1,
out, some of the water freezes and some evaporates at 470 JKg‒1 K‒1 and 4200 JKg‒1 K‒1 respectively.
0°C itself. Calculate the mass of ice formed if no water
is left in the vessel. Latent heat of vaporization of water Sol: Heat lost by the iron block increase the temperature
at 0°C= 2.10 ×106 JKg−1 and latent heat of fusion= of vessel and water.
3.36 ×105 JKg−1 . Mass aluminum = 0.5kg,

Sol: Some water evaporates and Heat of vaporization Mass of water = 0.2kg;
comes from water itself and hence remaining water Mass of iron = 0.2kg
freezes by giving the heat for vaporization.
Temp. of aluminum and water = 20°C = 293K
Total mass of water=M=100g. Latent heat of vaporization
Temperature of iron = 100°C = 373K
of water at 0°C=L1=21.0×105Jkg-1 latent heat of fusion
of ice= L2= 3.36 ×105 JKg−1 . Suppose, the mass of the Specific heat aluminum = 910J/kg-K
ice formed = m. Then the mass of water evaporated Specific heat of iron = 470J/kg-K
= M – m. Heat taken by the water to evaporate = (M
– m)L1 and heat given by the water in freezing=mL2. Specific heat of water = 4200J/kg-K
Thus, mL2= (M-m) L1 Heat gain = Heat lost;
ML1 (100g) ( 2.10 ×10 6
JKg −1
) = 86g. ⇒ (T − 293)(0.5 × 910 + 0.2 × 4200)
or, m = =
L1 + L2 ( 21.0 + 3.36 )105 −1
JKg =0.2 × 470 × (373 − T)

⇒ ( T − 293 )( 455 + 8400


= ) 49 (373 − T ) ;
P hysi cs | 15.19

 1295  In accordance with the principle of calorimetry when


⇒ ( T − 293)  = (373 − T ) ; A& B are mixed
 94 
⇒ ( T − 293) × 14 = 373 − T MCA (16 − 12 ) =
MCB (19 − 16 )
3
4475 ⇒ M=CA 4 MCB 3 ⇒ MCA = M …….. ( i) …(i)
T
⇒= = 298K ∴ T = 298 − 273 = 250 C 4 CB
15 
And when B and C are mixed;
The final temp= 25°C .
MCB ( 23 − 19
= ) MCC ( 28 − 23)
Example 6: A Piece of iron of mass 100 g is kept inside 4
⇒ 4MCB =
5MCC ⇒ MCC = M …….. ( ii)    …(ii)
a furnace for long time and then put in a calorimeter of 5 CB 
water equivalent 10 g containing 240 g of water 20°C.
The mixture attains an equilibrium temperature of 60°C. When A& C are mixed, if T is the common temperature
of mixture
Find the temperature of furnace. Specific heat capacity
3
of iron=470JKg‒1 °C‒1. MCA ( T − 12
= ) MCC =( 28 − T )  4  CB ( T − 12 )
 
Sol: This can be calculated in reverse manner, Heat lost 4
⇒   MCB ( 28 − T )
by the iron piece is equal to heat required to increase 4
the temperature of water and calorimeter. 628
= 15T − 180 = 488 − 16T ⇒ T= = 20.258= 20.3°C
Mass of iron = 100g 31
Water Eq of calorimeter = 10g;
Example 8: A glass cylinder can contain m0 =100g of
Mass of water = 240g mercury at a temperature of T0=0°C. When T1=20°C,
Let the Temp. of surface = 0°C the cylinder can contain m1 =99.7g of mercury. In both
cases the temperature of the mercury is assumed to
Siron = 470J/kg °C
be equal to that of the cylinder. Use this data of find
Total heat gained = Total heat lost. the coefficient of linear expansion of glass α , bearing
in mind that the coefficient of volume expansion of
100 250
So, × 470 × ( θ −=
60 ) × 4200 × ( 60 − 20 ) mercury γ1 = 18 × 10−5 deg−1
1000 1000

⇒ 470 − 47 × 60 = 25 × 42 × 40 Sol: Get the γ of glass with the information of


mercury. Find the relation between the densities at
2820 44820 different temperature, and then get coefficient of linear
⇒ θ 4200 + =
= = 953.610 C
47 47 expansion of glass cylinder.
When the cylinder is heated, its volume increases
Example 7: The temperature of equal masses of three according to the same Law as that of the glass:
different liquids A, B and C are 120°C, 19°C and 280°C
=V1 V0 (1 + γT1 ) where γ is the coefficient of volume
respectively. The temperature when A and B are mixed
is 160°C, and when B and C are mixed, it is 23°C what expansion of glass. If the densities of mercury at the
will be the temperature when A and C are mixed? temperature T0 and T1 are denoted by ρ0 and ρ1. We can
ρ0
write that m0=V0ρ0 and m1 = V1ρ1 , where ρ1 = .
Sol: All liquids have same mass. The heat lost by one 1 + γT1
equals to heat gain by other, so we can try to solve for This system of equations will give the following
the ratio of their heat capacities. expression for γ ;
The temp. of A = 12° C
m1 (1 + γ1T1 )
The temp. of B = 19 °C =γ ≈ 3 × 10 −5 deg−1
m0 T0
The temp. of C = 28 °C γ
The coefficient of linear expansion, α= = 10−5 deg−1
The temp of A + B = 16° C 3
The temp. of B + C = 23° C
1 5 . 2 0 | Calorimetr y and Thermal Expansion

JEE Advanced/Boards Sol: Here the amount air remains while P, V and T all
parameters changes. Hence PV/T =constant.
Example 1: An open glass tube is immersed in mercury Volume of the bubble of lake
in such a way that a length of 8cm extends above the
4 3 4
πr1 = π ( 0.18 ) cm3
3
mercury level. The open end of the tube is then closed = V1 =
and raised further by 44 cm. What will be the length of 3 3
the air column above mercury in the tube? Atmospheric Pressure on the bubble P1
pressure= 76 cm of mercury. = Atmospheric pressure + Pressure due to a column of
250 cm of water
Sol: Air column will get trapped and follow PV=constant.
= 76 × 13.6 × 980 + 250 × 1× 980
Let A be the area of cross section of the tube.
= ( 76 × 13.6 + 250 ) 980dyne / cm2 ; T1 = ?
x cm Volume of the bubble at the surface of lake
4 3 4
πr2 = π ( 0.2 ) cm3
3
52cm V2 =
3 3
8 cm (52-x) Pressure on the bubble P2
cm =Atmospheric pressure = 76 × 13.6 × 980 dyne / cm2

T2= 273+40°C = 313°K As

( 76 × 13.6 + 250 ) 980 × 4π ( 0.18 )3


3
P1V1 P2 V2
P1V1 =P2 V2 or ( 76 × 13.6 + 250 ) 980 × 4 π ( 0.18 )
T1 = T2 or T1 × 3
T1 T2 T13 × 3
Initial Atmospheric pressure of air in the tube outside ( 76 × 13.6 × 980 ) 4π ( 0.2)3
= ( 76 × 13.6 × 980 ) 4 π ( 0.2 ) ;
the mercury surface=P1=76 cm of Hg 313 × 3
= ;
313 × 3
Initial volume of air, V1=8A
New pressure of air in the tube 1283 × (0.18)3 1033.6(0.2)3
or =
P2 = 76 − (52 − x ) = ( 42 − x ) cm of Hg T1 313

1283 × ( 0.18 ) × 313


3
New volume of air, V = xA
=T1 = 1283.350 K
As P1V1 = P2 V2 ; 76 × 8A = ( 42 + x ) xA (1033.6 )( 0.2)
3

= x2 + 24x
or 608 ∴ T1 = 1283.35 -273=10.35°C

or x2 + 24x − 608
Example 3: A mixture of 250 gm of water and 200
gm of ice at 0°C is kept in a calorimeter which has a
( 24 )
2
−24 ± − 4 × 608
= 0,=
x water equivalent of 50 gm. If 200 gm of a steam at
2 100°C is passed through this mixture, calculate the
∴x=15.2cm or x =−39.4 cm final temperature and weight of the content of the
calorimeter. Latent heat of fusion of ice=80 Cal/gm.
Which is negative
latent heat of vaporization of water of steam=540cal/
∴ The length of air column =15.4cm gm., Specific heat of water=1cal/gm./°C.

Example 2: An air bubble starts rising from bottom Sol: Latent heat of vaporization of water is approx. 7
of a lake. Its diameter is 3.6 mm at the bottom and 4 times of latent heat of fusion. So 1g steam can melt
mm at the surface. The depth of the lake is 250 cm about 7g of ice. The mass of steam equals to mass of
and the temperature at the surface is 40°C. What is ice, so part of steam is condensed to melt the ice.
the temperature at the bottom of the lake? Given Heat lost by 200 gm. of steam before it is condensed to
atmospheric pressure=76 cm of Hg and g=980cm/sec2. water at 100°C
P hysi cs | 15.21

the calorimeter originally contains x gm of ice and


= 200 × 540 =
108000cal  … (i)
(200-x) gm. of water.
Heat gained by 200 gm. of ice at 0°C
Heat gained by calorimeter
= mL + m × s × ∆T = 200 × 80 × 1× (100 − 0 )
100
= × 0.42 × 103 × (50 − 0 ) =
2100J
= 36000 cal 1000
Heat gained by 250gm of water and 50 gm of
Heat gained by ice
water equivalent of calorimeter at 100°C to 0°C
=200 × 80 × (100 − 0 ) + 50 × (100 − 0 ) =
x
( )
× 3.36 × 105 + 4.2 × 103 × 50 
1000  
= 300 × 100 = 30000cal
= x [336 + 210] = x × 546 J
Total heat gained
Heat gained by water
66000cal …(ii)
30000cal + 36000 = .......(ii)
 200 − x    42000 − 210xJ
Amount of heat lost by the system (i) is greater than = 3
 4.2 × 10 × 50=
 1000   
heat gained by ice. This shows that only a part of the
steam will condense to water at 100°C which will be Heat lost by steam
sufficient for melting ice.
 330 − 200 −100  
 22.5 × 10 × 4.2 × 10 × 50 
3 3
Let M be mass of steam which will be sufficient for 
melting ice,  1000 
∴ Mass M of steam required is given by. 30 2250 + 210  =
= 30 × 2460 =
73800J
1100
Or M 66000
= = / 540 = 122.2gm Heat gained = heat lost;
9
2100+546x+42000-210x = 73800;
Final temperature of system= 100°C
336x = 73800-44100 = 29700
Weight contents
29700
= Weight of ice +Water+ Steam condensed Mass of ice
= = x = 88.39gm
336
=250+200+122.2=572.2gm Mass of water = 111.61 gm
Ratio of ice to water = 88.39:111.6=1:1.263 ≈ 0.79
Example 4: A copper calorimeter of mass 100 gm
contains 200g of a mixture of ice and water; Steam
at 100°C under normal pressure is passed into the Example 5: A one liter flask contains some mercury.
calorimeter and the temperature of the mixture is It is found that at different temperatures the volume
allowed to rise to 50°C. If the mass of the calorimeter of air inside the flask remains the same. What is the
and its contents is now 330gm, what was the ratio of volume of mercury in the flask? Given coefficient of
ice and water in the beginning? Neglect heat losses. linear expansion of glass= 3 × 10−6 per degree Celsius.
Given that: Coefficient of volume expansion of Hg
3
Specific heat of copper= 0.42 × 10 J / kg K. = 1.8 × 10−4 per degree Celsius.

Specific heat of water = 4.2 × 103 J / kg K.


Sol: Volume of air in the flask is independent of
Latent heat of fusion of ice = 3.36 × 105 J / kg K. temperature.
Latent heat of condensation of steam Let x be the volume of mercury in the flask
5
= 22.5 × 10 J / kg K. Volume of air = Volume of flask – Volume of Hg.

Sol: Total amount of heat lost by the steam will bring = 1000 cm3 – x cm3
the water and calorimeter to 50 degree temp. remaining At any Temperature ‘T’ –
heat would have been used to melt the ice.
Volume of flask = 1000 + 1000 x 3 αg ∆ T. …(i)
Heat is lost by steam in getting condensed and heat
is gained by the water, ice and the calorimeter. Let and Volume of Hg = x + x × γm × ∆T  …(ii)
1 5 . 2 2 | Calorimetr y and Thermal Expansion

Hence volume of air = Volume of flask – Volume of Hg


1 15.5 1.24  1 1
Or γ= =  × −1= ×
= 1000 − x + (1000 × 3αg − x × γm ) × ∆T 15 1.20 16  15 960
Given: Volume of air remains constant at all temperatures 1
∴=
α = 2.31× 10−5 / 0 C
Hence, coefficient of ∆T 3 × 15 × 960
i.e. ( 1000 × 3αg − x × γm ) =0
Example 7: A composite rod is made by joining a copper
3 × 1000 × αg 9 × 1000 cm3 × 10−6 / o C rod end to end with a second rod of different material
=⇒x = but of the same cross-section. At 25°C, the composite
γm 1.8 × 10 −4 / o C
rod is 1 m in length of which of the copper rod is 30
= 50 cm3 cm. At 125°C, the length of the composite rod increases
by 1.91 mm. When the composite rod is not allowed
Example 6: A piece of metal weights 46 gm in air. When to expand by holding it between two rigid walls, it is
it is immersed in a liquid of specific gravity 1.24 at 37°C, found that the lengths of the two constituents do not
it weighs 30 gm. When the temperature of the liquid is change with the rise of temperature. Find the Young’s
raised to 42°C, the metal piece weights 30.5 gm. The modulus and the coefficient of the linear expansion
specific gravity of the liquid at 42°C, is 1.20. Calculate of the second rod. Given Young’s modulus copper
the coefficient of linear expansion of the metal. =1.3 ×1011 N/ m2 , coefficient of the linear expansion of
copper=. 1.7 × 10−5 / 0 C
Sol: Applying Archimedes’ principle, i.e. lose in wt= wt
of liquid displaced. We can get volume of metal at two Sol: First part, α2 can be calculated and then same
temp. so we can have coefficient of volume expansion. compressive force applied by the wall, this will give the
Weight of the piece of metal in air=46gm. weight of the Young’s modulus of the material.
piece of metal in liquid at 27°C =30gm
Length of copper rod at 25°C, l1=30m

∴ Loss of the weight of the piece of metal in liquid = Length of second rod at 25°C, l2=70cm
46-30 = 16gm = Weight of liquid displaced If α1 and α2 are respective linear expansion coefficients,
Volume of liquid Displaced the total expansion of the composite rod when the

You might also like